Sunteți pe pagina 1din 986

USMLE Step 2 Practice Test Block 1 Name: Instructions: Answer the questions below to the best of your ability.

When you finish the test, click the Check button at the bottom to view the results.

1.A family physician cares for a family consisting of a 4 !year ol" h#s$an"% 4&!year!ol" 'ife an" a 12!year!ol" "a#ghter. The family reports that recently the ((!year!ol" maternal gran"mother 'ho li)e" 'ith them "ie" after a prolonge" respiratory infection. A#topsy s#$se*#ently confirms that she ha" acti)e p#lmonary t#$erc#losis at the time of "eath. The organism teste" sensiti)e to all anti!t#$erc#losis "r#gs. +n respon"ing to the gran"mother,s illness% 'hich of the follo'ing is the most appropriate step in managementA) Obtain leukocyte counts on all family members )Obtain s!utum cultures for aci" fast bacilli C)Obtain chest com!uteri#e" tomo$rams on all members %)&lace !rotein !urifie" "erivative '&&%) test on all members ())che"ule bronchosco!y lava$e for the a"ults 2. A (!year!ol" man comes to the emergency "epartment $eca#se of e.cr#ciating pain in his right $ig toe. /e "escri$es the pain as so se)ere that it 'oke him from a "eep sleep. /e has no chronic me"ical con"itions% "oes not take any me"ications% an" "enies any similar episo"es in the past. /e a"mits to a fe' 0"rinking $inges0 o)er the past 2 'eeks. /is temperat#re is &1.1 2 3144. 56% $loo" press#re is 1&4784 mm /g% an" p#lse is 147min. E.amination sho's an erythemato#s% 'arm% s'ollen% an" e.*#isitely ten"er right great toe. The skin o)erlying the first metatarsophalangeal 9oint is "ark re"% tense% an" shiny. Syno)ial fl#i" analysis re)eals negati)ely $irefringent% nee"le! shape" crystals 'ithin polymorphon#clear le#kocytes 3PM:s6. La$oratory st#"ies sho'; Ser#m Le#kocytes........1<%4447mm& Uric aci"...........1 mg7"L 2alci#m.............8 mg7"L

=hich of the follo'ing is the most appropriate pharmacotherapyA Allo!urinol Ceftria*one C In"omethacin % &robeneci" ( e)ulfin!yra#one &. A & !year!ol" 'oman arri)es on the floor after an #ne)entf#l hysteroscopy to e)al#ate her long history of #terine fi$roi"s. A$o#t &4 min#tes after her arri)al% she $egins to complain of na#sea an" has t'o episo"es of )omiting. The physician a"ministers 4.<2 mg of "roperi"ol an" 444 mg of acetaminophen $y mo#th. >n follo'!#p e)al#ation% the patient,s neck is in)ol#ntarily fle.e" to one si"e. She is alert% oriente"% an" con)ersant an" has an other'ise normal ne#rologic e.amination. =hich of the follo'ing is the most likely "iagnosisA) Cerebral vascular acci"ent )Conversion "isor"er C) %ystonic reaction to "ro!eri"ol %) +unchausen syn"rome () )ei#ure 4. A 4!year!ol" n#rse cons#lts a physician $eca#se of a rash a$o)e $oth her ankles. Physical e.amination "emonstrates marke" ankle e"ema 'ith erythema% mil" scaling% an" $ro'n "iscoloration of the o)erlying skin of the "istal lo'er legs. ?aricose )eins are also note". =hich of the follo'ing is the most likely "iagnosisA) Ato!ic "ermatitis ) Cellulitis C),ichen sim!le* chronicus %)Nummular "ermatitis ())tasis "ermatitis . An A+@S patient "e)elops symptoms s#ggesti)e of a se)ere% persistent pne#monia 'ith co#gh% fe)er% chills% chest pain% 'eakness% an" 'eight loss. The patient "oes not respon" to penicillin therapy% $#t goes on to "e)elop )ery se)ere hea"aches. The presence of focal ne#rologic a$normalities lea"s the clinician to or"er a 2T scan of the hea". This "emonstrates se)eral metastatic $rain a$scesses. Biopsy of one of these lesions "emonstrates $ea"e"% $ranching% filamento#s gram!positi)e $acteria that are 'eakly aci" fast. =hich of the follo'ing is the most likely ca#sati)e organism-

A. Actinomyces . As!er$illus C. urkhol"eria %. -rancisella (. Nocar"ia <. A 21!year!ol" patient 'ith en"!stage renal "isease 3ESA@6 on contin#o#s am$#latory peritoneal "ialysis 32AP@6 for t'o months presents 'ith fe)er% a$"ominal pain an" clo#"y "ialysis fl#i". There is no "iarrhea or )omiting an" the pain has $een present for a$o#t 12 ho#rs. The patient has ESA@ secon"ary to chronic glomer#lonephritis% there is no history of "ia$etes% #rinary infections or anti$iotic #se. E.amination re)eals a temperat#re of &1.8 2 3142 56% an" $loo" press#re of 1147(4 mm /g. The throat is clear% as are the l#ngs. 2ar"iac e.amination re)eals a gra"e 27< systolic m#rm#r. A$"ominal e.amination re)eals "ecrease" $o'el so#n"s 'ith "iff#se ten"erness. There is mil" re$o#n". There is no e"ema or skin rash. A complete $loo" co#nt sho's a le#kocyte co#nt of 14%2447mm&% hemoglo$in is 12. g7"L. Peritoneal fl#i" is clo#"y 'ith 1%444 'hite $loo" cells% 1 B of 'hich are polymorphon#clear le#kocytes. Cram,s stain of fl#i" is negati)e. 2#lt#res of $loo" an" peritoneal "ialysis fl#i" are taken. =hich of the follo'ing is the most appropriate initial step in managementA. -lucona#ole . Imme"iate removal of "ialysis catheter. C. Intravenous vancomycin %.Intravenous $entamicin (.Oral ci!roflo*acin (. A 4!year!ol" man presents for a perio"ic health e.amination. /is family history is significant for his mother 'ho "ie" of a cere$ro)asc#lar acci"ent at age (2% his father 'ho "ie" of a myocar"ial infarction at age <1% an" a $rother 'ho "e)elope" sigmoi" cancer at age <4. The patient is on no me"ications e.cept for aspirin% 11 mg "aily. /is physical e.amination is #nremarka$le. The patient asks for a recommen"ation regar"ing c#rrent cancer screening. =hich of the follo'ing is the most appropriate screening test for this patientA. Annual "i$ital rectal e*amination an" fecal occult bloo" testin$ . -le*ible si$moi"osco!y C. -le*ible si$moi"osco!y an" barium enema %. Colonosco!y (. .enetic testin$ for the !/0 $ene

1. An A+@S patient #n"er treatment 'ith a n#cleosi"e analog an" a protease inhi$itor comes to me"ical attention 'ith complaints of leg 'eakness an" incontinence. /is )ital signs are 'ithin normal limits. Physical e.amination re)eals re"#ce" strength in the lo'er e.tremities 'ith accompanying mil" spasticity. There is also "iminishe" sensation in the feet an" legs $ilaterally. L#m$ar p#nct#re sho's; >pening press#re.....144 mm /24 2ell co#nt................ lymphocytes7mm& Cl#cose...................41 mg7"L Proteins% total..........&& mg7"L Camma glo$#lin.......1B total protein A""itional la$oratory in)estigations sho' normal hematologic parameters% )itamin B12 'ithin normal )al#es% an" negati)e serology for syphilis. MA+ of the hea" fails to re)eal any focal a$normality. =hich of the follo'ing is the most likely "iagnosisA. AI%) "ementia com!le* . C+1 !olyra"iculo!athy C. Cry!tococcal menin$oence!halitis %. 1acuolar '2I1) myelo!athy (. 3i"ovu"ine4relate" to*icity 8. A 4 !year!ol" man is in)ol)e" in an a#tomo$ile acci"ent an" s#stains se)ere in9#ries 'ith consi"era$le $loo" loss an" hypotension. /e is transferre" from the emergency "epartment to an intensi)e care #nit% 'here he "e)elops m#ltiorgan fail#re. @#ring the first 2 "ays in the intensi)e care #nit% his plasma phosphate 'as 'ithin normal limits. S#$se*#ently% it $egan to rise% e)ent#ally reaching <.4 mg7"L. 5ail#re of 'hich of the follo'ing organs 'o#l" most likely ha)e this effectA. 2eart . 5i"neys C. ,iver %. ,un$s (. &ancreas 14. A 21!year!ol" 'oman is "iagnose" 'ith l#p#s nephritis% =orl" /ealth >rganiDation 3=/>6 type +?. She has a malar rash% "iff#se arthritis% an" e"ema. /er $loo" press#re is 1847114 mm /g. /er creatinine is 2.1 mg7"L 'ith a $loo" #rea nitrogen of 21 mg7"L. /er #rine re)eals 2 re" $loo" cells per hpf% an" &E protein. >ne re" $loo" cell cast is seen. A 24!ho#r #rine collection re)eals a protein of 11 grams 'ith a creatinine of 1 gram. =hich of the follo'ing 'o#l" $e the most appropriate management-

A. Oral a#athio!rine . Oral cyclo!hos!hami"e C. Oral $ol" %. Oral !re"nisone (. &ulse I1 cyclo!hos!hami"e 11. A 41!year!ol" 'oman is seen $y a clinician. She has a $right re"% sharply "emarcate"% ooDing an" cr#sting rash in)ol)ing one $reast in the areola area. She has ha" this lesion for si. months an" states that it is slo'ly gro'ing. The lesion "oes not respon" to anti$iotic ointment% antif#ngal ointment% or steroi" ointment. This lesion is most likely relate" to 'hich of the follo'ing con"itionsA. reast cancer . Crohn "isease C. .astric cancer %. 6heumatoi" arthritis (. )ystemic lu!us erythematosus 12. A recent article in a prominent me"ical 9o#rnal e.plore" the "isparity of reso#rce #tiliDation $et'een men an" 'omen. More men than 'omen ha)e ma9or car"iac proce"#res% incl#"ing catheteriDation% performe". This is tho#ght to $e $eca#se of 'hich of the follo'ing reasonsA. -ewer outcome stu"ies have stu"ie" women with these "isease . 7he inci"ence of car"iovascular "isease is lower in women C. +en have better health insurance %. +en receive too many car"iovascular !roce"ures (. &rovi"er attitu"es has le" to this situation 1&. A 4&!year!ol" man presents 'ith a 4!year history of 9oint pain. The "istri$#tion is asymmetric% in)ol)ing the pro.imal an" "istal small 9oints of the right han"% the left knee% the ankle% an" right el$o'. Pain an" morning stiffness are mo"erate. Physical e.amination re)eals mil" nail pitting% an" the "istal thir" interphalangeal 9oint is partially s#$l#.ate". F!rays of the han"s sho' resorption of the "istal en" of the phalan.. The erythrocyte se"imentation rate 3ESA6 is ele)ate" to 4< mm7hr% an" rhe#matoi" factor is negati)e. =hich of the follo'ing is the most likely "iagnosisA. &rimary $enerali#e" osteoarthritis . &seu"o$out C. &soriatic arthritis %. 6heumatoi" arthritis

14. A 4 !year!ol" man presents to a physician 'ith complaints of 'eakness% fatig#e% an" feeling near fainting 'hen he stan"s #p *#ickly. Screening chemistry st#"ies "emonstrate so"i#m% 121 mE*7LG potassi#m% .2 mE*7LG $icar$onate% 1( mE*7LG an" #rea nitrogen% 4 mg7"L. The physician is consi"ering A""ison "isease in his "ifferential "iagnosis. =hich of the follo'ing feat#res on physical e.amination 'o#l" $e most s#ggesti)e of this "iagnosisA. lack freckles on the shoul"ers . ,ar$e, furrowe" ton$ue C. +any s!i"er an$iomas %. &rotru"in$ eyeballs (. )mall $listenin$ bum!s on the li!s 1 . A (1!year!ol" man presents to the emergency "epartment 'ith fe)er an" co#gh. /e has kno'n hypercholesterolemia an" is stat#s post a right hemicolectomy for colon cancer. The patient states that he has ha" & "ays of fe)er to 142 5% co#gh pro"#cti)e of green sp#t#m% as 'ell as general malaise an" 'eakness. /is physical e.amination is remarka$le for "ecrease" $reath so#n"s at the left $ase% left $asilar egophony an" "#llness to perc#ssion. A complete $loo" co#nt re)eals a le#kocyte co#nt of 1 4447mm&. A chest ra"iograph re)eals a left lo'er lo$e infiltrate. =hich of the follo'ing is the most important part of the history to ascertain prior to initiating therapyA. Influen#a immuni#ation status . &lasma li!i" !rofile C. )ta$e of the colon cancer %. )ocial su!!ort structure for home thera!y (. 6ecently hos!itali#ations 1<. A 48!year!ol" 'oman presents to the office $eca#se of complaints of fatig#e. She has ha" progressi)e e.ercise intolerance o)er the prior < months. >n physical e.amination% she is pale an" afe$rile. /er $loo" press#re is 1127<1 mm /g% an" her p#lse is 117min. /eart an" l#ng e.aminations are normal e.cept for a +7?+ systolic flo' m#rm#r at the left sternal $or"er. Ao#tine la$oratory res#lts re)eal hemoglo$in of 1.& g7"L% a mean corp#sc#lar )ol#me of 111 HL7m&% an" a B12 of 12 pg7mL 3normal I214 pg7mL6. She #n"ergoes a Schilling test% 'hich re)eals mala$sorption of ra"iola$ele" B12. +ntrinsic factor is a"ministere" an" the ra"iola$ele" B12 is s#$se*#ently a$sor$e". =hich of the follo'ing is the "iagnosis-

A. Atro!hic $astritis . acterial over$rowth C. Chronic !ancreatitis %. Crohn "isease (. .astric ulcer 1(. A < !year!ol" 'oman cons#lts a physician $eca#se of a &!month history of 'eight loss% $#rning sensation of the tong#e% fatig#e% anore.ia% an" poorly localiDe" a$"ominal pain. The 'oman appears pale to the physician. +ntraoffice hematocrit is & B 'ith peripheral smear sho'ing large erythrocytes an" hypersegmente" ne#trophils. Ser#m folate is 2.4 ng7mL 3normal greater that 1.8 ng7mL6 an" ser#m )itamin B12 is 144 pg7mL 3normal 244!144 pg7mL6. Stomach $iopsy "emonstrates chronic gastritis. A#toanti$o"ies to 'hich of the follo'ing are most likely in)ol)e" in this patient,s con"itionA. asement membrane . Insulin rece!tor C. Intrinsic factor %. ))4 (. 7)2 rece!tor 11. A pre)io#sly healthy 4!year!ol" 'oman comes to the physician $eca#se of "o#$le )ision for three "ays. /er temperat#re is &( 2 381.< 56. The patient "enies na#sea or )omiting. E.amination re)eals ptosis an" slight "i)ergence of the right eye. E.traoc#lar mo)ements are limite" in all "irections% e.cept laterally. The right p#pil is larger than the left an" poorly reacti)e to light. E.amination of the f#n"#s fails to re)eal papille"ema. =hich of the follo'ing is the most likely #n"erlying con"itionA. Aneurysm of the !osterior communicatin$ artery . Carcinoma of the ri$ht !ulmonary a!e* C. %iabetes mellitus %. .iant cell arteritis (. )y!hilis -. )ystemic hy!ertension

18. A patient 'ith a history of chronic $acterial sin#sitis presents to the emergency "epartment 'ith a )ery se)ere hea"ache. =hile 'aiting to $e seen% he "e)elops a generaliDe" gran" mal seiD#re. Physical e.amination% after the seiD#re is o)er% "emonstrates high fe)er% e.ophthalmos% papille"ema% an" ner)e palsies of the ?+ an" +++ cranial ner)es on one si"e. =hich of the follo'ing is the most appropriate ne.t stepA. A"mit to the me"ical floor for monitorin$ of !ro$ression of sym!toms . (mer$ency C7 scan C. (mer$ency e*!loratory sur$ery %. (mer$ency ultrasoun" (. 5ee! in emer$ency "e!artment for monitorin$ of !ro$ression of sym!toms 24. A 41!year!ol" man presents 'ith complaints of mil" intermittent heart$#rn after meals for the past < months. /e has trie" )ario#s o)er! the!co#nter antaci"s an" /2 receptor antagonists 'ith only minimal relief. /e "enies any "ysphagia or o"ynophagia% an" is other'ise in goo" health. /e is concerne" a$o#t the risk of "e)eloping cancer% $eca#se his father "ie" of gastric cancer at age 48. /is physical e.amination is #nremarka$le. =hich of the follo'ing 'o#l" $e the most appropriate ne.t step in managementA. Avoi"ance of a hi$h4!rotein "iet . Avoi"ance of as!irin C. Avoi"ance of acetamino!hen %. (levation of the hea" of his be" (. Increase" consum!tion of carbohy"rates 21. A 44!year!ol" 'oman is $ro#ght to the emergency "epartment follo'ing a s#ici"e attempt 'ith imipramine. /er fiancee fo#n" her #nresponsi)e% 'ith an empty $ottle of the imipramine at her si"e. The imipramine ha" $een his% an" the prescription ha" $een fille" that morning. /er past me"ical history is significant for hypertension% atrial fi$rillation% "ia$etes% an" asthma. /er me"ications incl#"e f#rosemi"e% procainami"e% gly$#ri"e% pre"nisone% an" al$#terol. She has no kno'n "r#g allergies. She is afe$rile% has a $loo" press#re of 1447<4 mm /g% p#lse of <27min% an" respirations of 227min. A gastric la)age yiel"s m#ltiple pill fragments. She is conf#se" an" somnolent% an" has shallo' respirations. /er physical e.amination is other'ise #nremarka$le. >n an E2C% 'hich of the follo'ing a$normalities 'o#l" most likely reflect possi$le car"iac to.icity-

A. ,eft "eviation of the 86) a*is . &rolon$ation of the 86) interval C. )hortenin$ of the 87 interval %. )7 se$ment "e!ression (. 7 wave inversion 22. A 4&!year!ol" $#s "ri)er presents to his gastroenterologist 'ith complaints of "iffic#lty s'allo'ing soli" foo"s. The e)al#ation "emonstrates a smooth% tapere" strict#re of the "istal esophag#s% an" $iopsies re)eal changes consistent 'ith chronic esophagitis an" fi$rosis. The strict#re is "ilate" 'ith an en"oscopic $alloon "ilator% an" the patient,s symptoms resol)e. /e reports that altho#gh he has ha" "ysphagia for the past 2 months prior to the en"oscopy% he rarely has heart$#rn an" #ses an o)er!the!co#nter antaci" only occasionally. =hich of the follo'ing is the most appropriate f#t#re management of this patientA. -amoti"ine . ,anso!ra#ole C. +a$nesium hy"ro*i"e %. +etoclo!rami"e (. No me"ication is necessary Normal ,abs 2&. A 2&!year!ol" professional $asket$all player presents to the team physician & ho#rs $efore game time complaining of a$"ominal pain. The symptoms $egan appro.imately 1 ho#rs earlier in a "iff#se fashion. T'o ho#rs later% he $egan feeling na#seate" an" )omite" t'ice. >)er the past 4 ho#rs% the a$"ominal pain has $ecome more se)ere an" 'ell localiDe" in the right lo'er *#a"rant. /is e.amination no' re)eals 'ell! localiDe" pain in the right lo'er *#a"rant inferolateral to the #m$ilic#s. =hich of the follo'ing is the most likely "iagnosisA. Acute obstruction of the a!!en"iceal lumen by a fecalith . Acute onset of ileocolitis C. Acute onset of ischemic colitis %. Acute 9ersinia infection (. Obstruction of the ileocecal valve by a mass

24. A &1!year!ol" homeless 'oman is $ro#ght to the emergency "epartment after $eing fo#n" face "o'n on the street. The 'oman has a long history of a"missions to the hospital for alcohol!relate" iss#es% incl#"ing seiD#res% 'ith"ra'al% an" hall#cinations. To"ay% she 'as seen to fall in the street% ha)e 'hat 'ere "escri$e" as 0con)#lsions0 an" then )omit. She remaine" face "o'n in the street #ntil the parame"ics arri)e". >n physical e.amination% she has "ry m#co#s mem$ranes% a 9#g#lar )eno#s press#re of less than cm% an" "iff#se ecchymoses on her face% $o"y% an" $reasts. =hich of the follo'ing )itamins sho#l" $e a"ministere" prior to )ol#me res#scitation 'ith +? fl#i"s containing gl#coseA. 1itamin : '7hiamin) . 1itamin 0 'Niacin) C. 1itamin :; 'Cobalamin) %. 1itamin C (. 1itamin 5 2 . A &<!year!ol" man "e)elops rapi" mental stat#s "eterioration t'o "ays after s#staining a femoral fract#re in a skiing acci"ent. Physical e.amination sho's m#ltiple petechiae in the anterior chest an" a$"omen. >n the thir" "ay% the patient lapses into coma an" "ies. Postmortem e.amination of the $rain re)eals n#mero#s petechial hemorrhages in the corp#s callos#m an" centr#m semio)ale. =hich of the follo'ing is the most likely "iagnosisA. %iffuse a*onal in<ury . -at embolism C. )e!tic embolism %. )ystemic thromboembolism (. Watershe" infarction 2<. A 4 !year!ol" man cons#lts a physician $eca#se of "ys#ria. The patient is treate" 'ith anti$iotics% $#t symptoms rec#r one 'eek after anti$iotic therapy is stoppe". A "ifferent anti$iotic is trie"% $#t symptoms again rec#r after cessation of the anti$iotic. Aectal e.amination "emonstrates an enlarge" prostate 'ith areas of ten"erness an" fl#ct#ance. =hich of the follo'ing is the most likely "iagnosisA. eni$n !rostatic hy!er!lasia . Chronic nonbacterial !rostatitis C. &rosta"ynia %. &rostatic abscess (. &rostatic carcinoma

2(. A &1!year!ol" 'oman presents at the hospital for a pre!employment physical e.amination prior to $eginning her year as a me"ical intern. She is se.#ally inacti)e an" "enies alcohol #se. She ha" infectio#s monon#cleosis 'hile in college an" recei)e" the recom$inant hepatitis B )accine $efore starting me"ical school. =hich of the follo'ing 'o#l" "escri$e her hepatitis B serologic profileA. 2e!atitis surface anti$en !ositive, core antibo"y !ositive, an" surface antibo"y ne$ative . 2e!atitis surface anti$en ne$ative, core antibo"y !ositive, an" surface antibo"y !ositive C. 2e!atitis surface anti$en !ositive, core antibo"y ne$ative, an" surface antibo"y ne$ative %. 2e!atitis surface anti$en ne$ative, core antibo"y ne$ative, an" surface antibo"y !ositive (. 2e!atitis surface anti$en ne$ative, core antibo"y ne$ative, an" surface antibo"y ne$ative. 21. A (!year!ol" man presents to his physician for a preoperati)e e)al#ation. /e has $een a long!time patient in this office an" has $een treate" for hypertension an" gastritis. /e has $een sche"#le" for an electi)e open cholecystectomy in 2 "ays. /e c#rrently takes omepraDole for his gastritis an" thiaDi"e for his hypertension "aily. /e smokes t'o packs of cigarettes per "ay. /is home $loo" press#re log sho's that his systolic press#res range from 1 4 to 184 mm /g% an" his "iastolic press#res range from 14 to 14 mm /g% in"icating that his $loo" press#re may $e not a"e*#ately controlle" for the s#rgical proce"#re. =hich of the follo'ing me"ications is most appropriate in the perioperati)e perio" for a""e" $loo" press#re controlA. Ca!to!ril . Cloni"ine C. +eto!rolol %. Nife"i!ine (. &ra#osin

28. A <<!year!ol" man presents to the clinic complaining of progressi)ely 'orsening shortness of $reath an" nonpro"#cti)e co#gh o)er the past 2 years. /e retire" 1 year ago% after 'orking as a rock miner for more than &4 years. /e has no other significant past me"ical history. >n physical e.amination% he is a thin man 'ho appears tachypneic at rest. /is l#ngs ha)e re"#ce" chest e.pansion an" "ry inspiratory rales in the #pper lo$es $ilaterally. The remain"er of his e.amination is normal. A chest .!ray film re)eals m#ltiple ro#n" opacities in the #pper lo$es accompanie" $y hilar lympha"enopathy 'ith lymph no"e calcification. =hich of the follo'ing is the most likely "iagnosisA. Asbestosis . As!er$illosis C. Cystic fibrosis %. )ilicosis (. 7uberculosis &4. A !year!ol" 'oman 'ith a long!stan"ing history of atrial fi$rillation secon"ary to mitral reg#rgitation presents to the emergency "epartment 'ith a painf#l right foot. The patient reports that% o)er the past fe' ho#rs% her foot has $ecome more painf#l an" no' is nearly insensate. She "escri$es the pain as $#rning an" states that it is not relie)e" $y any inter)ention. She takes co#ma"in% atenolol% "igo.in% an" aspirin. >n physical e.amination% her p#lse is irreg#larly irreg#lar. /er l#ngs are clear% an" she has a lo#" holosystolic m#rm#r hear" $est at the ape.. /er right foot is gray an" cool to the to#ch an" has poor capillary refill. @orsalis pe"is an" posterior ti$ial p#lses are a$sent on the right. /er prothrom$in time is 14.4 secon"s 3+:A 1.46. =hich of the follo'ing is the most appropriate co#rse of actionA. Arran$e for her to be seen by a vascular sur$eon in the emer$ency "e!artment now . Arran$e for her to be seen by a neurolo$ist within the ne*t few "ays C. Arran$e for her to un"er$o an +6I of the hea" that "ay %. Ask her to make an a!!ointment to be in seen in your office within : week (. Instruct her to soak her le$ in warm water an" to !lace a fitte" stockin$ on her affecte" le$

&1. A 4!year!ol" man is $ro#ght to the emergency "epartment complaining of light!hea"e"ness. /e has a history of l#ng cancer% 'hich 'as "iagnose" a month ago an" fo#n" to $e 'i"ely metastatic to the $one an" pericar"i#m. >n physical e.amination% his $loo" press#re is (4744 mm /g% an" his p#lse is 1447min. /is heart so#n"s are "istant an" soft. /is E2C "emonstrates lo' )oltage% an" electrical alternans is present. A chest .!ray film sho's that the car"iac silho#ette has a 0'ater $ottle0 appearance.=hich of the follo'ing is the most appropriate inter)ention in this patientA. eta4blockers . Nonsteroi"al anti4inflammatory "ru$s C. )teroi"s %. &ericar"iocentesis (. Car"iac catheteri#ation &2. A <2!year!ol" man 'ith a 114 pack!year history of smoking presents 'ith chest pain. /e states that for the past fe' months% he has $een getting chest 0press#re0 localiDe" to the s#$sternal region% ra"iating to the left arm on occasion. The pain occ#rs 'ith mil" e.ertion% $#t ne)er at rest. /e f#rther states that 'hen he gets the pain% it #s#ally last a$o#t min#tes $#t goes a'ay 'ith rest. /e reports that his e.ercise tolerance is mo"erate% an" he gets "yspnea on e.ertion after a fe' $locks of 'alking. >n physical e.amination% he has no chest 'all ten"erness to palpation% $#t a caroti" $r#it is hear"% an" his "orsalis pe"is p#lses are "ecrease". /e has no history of coronary "isease $#t his family history is significant for his father ha)ing a myocar"ial infarction at age <. /e "enies chest pain at this time. +n a""ition to ascertaining his other coronary risk factors% 'hich of the follo'ing is the most appropriate "iagnostic inter)entionA. Obtain a restin$ electrocar"io$ram . )che"ule the !atient for a car"iac echocar"io$ram C. )che"ule the !atient for an e*ercise trea"mill test %. )che"ule the !atient for non4ur$ent coronary an$io$ra!hy (. )che"ule the !atient for imme"iate coronary an$io$ra!hy

&&. A & !year!ol" man comes to the physician for a health maintenance e.amination. /e recei)e" $loo" transf#sions for hypo)olemic shock follo'ing a g#nshot 'o#n" 14 years earlier. /e is c#rrently in goo" health% an" physical e.amination is #nremarka$le. A ser#m chemistry panel sho's; ALT 2 4 U7L AST 144 U7L Alkaline phosphatase (4 U7L Serologic e)al#ation for )iral hepatitis re)eals positi)e anti$o"ies to hepatitis 2 )ir#s 3/2?6. A perc#taneo#s li)er $iopsy sho's marke" portal inflammatory infiltrate "isr#pting the limiting plate of hepatic lo$#les. =hich of the follo'ing is the inci"ence rate of this complication follo'ing /2? infectionA. /= . :>= C. ;>= %. ?>= (. @>= &4. A (1!year!ol" 'oman is a"mitte" to the hospital for pne#monia. The patient presente" to the hospital 2 "ays ago for co#gh an" fe)er. She reporte" temperat#res to &1.8 2 3142 56 an" a co#gh pro"#cti)e of green% copio#s sp#t#m. She also reporte" ple#ritic chest pain 'ith "eep inspiration. The initial e.amination re)eale" "iminishe" $reath so#n"s in the left lo'er lo$e 'ith "#llness to perc#ssion% an" a chest ra"iograph re)eale" a "ense left lo'er lo$e infiltrate. =hich of the follo'ing organisms is most likely responsi$le for her pne#moniaA. or"etella !ertussis . 5lebsiella !neumoniae C. +yco!lasma !neumoniae %. &neumococcus (. )ta!hylococcus aureus

& . A <8!year!ol" 'oman presents to her physician of & years 'ith progressi)e shortness of $reath. The "yspnea 'as initially limite" to e.ertion $#t has progresse" to shortness of $reath at rest. She has ha" intermittent co#gh $#t no fe)er. /er past me"ical history is significant for mil" hypertension an" seropositi)e rhe#matoi" arthritis. =hich of the follo'ing aspect of her social history is the most important consi"eration to re)ie' at this pointA. Alcohol history . %ru$s of abuse history C. +arital status %. Occu!ation (. 7obacco history &<. A 42!year!ol" man presents for his ann#al physical e.amination. /e 'as last seen 2 years ago for a perio"ic health e.amination an" 'as in goo" health. /e is on no me"ications. /is past me"ical history is significant for a cholecystectomy 2 years ago an" rhe#matic fe)er at age 1 . /e has $een smoking appro.imately ten cigarettes "aily for the past 2& years. >n physical e.amination% his $loo" press#re is 1 47 < mm /g% p#lse is <17min% an" respirations are 147min. /e is afe$rile. A hea" an" neck e.amination is normal. /is l#ngs are clear. /e has a reg#lar heart rhythm% 'ith a ++7+? $lo'ing "ecrescen"o "iastolic m#rm#r hear" at the aortic area. /is a$"ominal an" rectal e.aminations are normal. 2omplete $loo" co#nt% electrolytes% an" thyroi" f#nction tests are normal. =hich of the follo'ing is the most appropriate a")ice for this man regar"ing f#t#re pre)enti)e health maintenanceA. Antibiotic !ro!hyla*is before "ental work . Annual chest *4ray film C. Annual echocar"io$ram %. Annual fle*ible si$moi"osco!y (. Annual !rostate s!ecific anti$en testin$

&(. A &4!year!ol" man cons#lts a physician at his 'ife,s insistence $eca#se 0his eyes are a little yello' all the time no'0. Screening chemistry st#"ies sho' mo"est ele)ations of li)er transaminases an" total $ilir#$in 2.4 mg7"L% almost all of 'hich is con9#gate". The patient "enies e)er #sing alcohol. ?iral hepatitis st#"ies are negati)e. Li)er $iopsy sho's hepatic fi$rosis 'ith normal iron le)els an" no e)i"ence for alpha!1!antitrypsin "eficiency. >n f#rther *#estioning a$o#t his general health% the patient re)eals that he has ha" an #n#s#ally large n#m$er of $acterial pne#monias in his life. /e has e)en ha" Pne#mocystis pne#monia at one point. /+? testing at that time an" repeate" t'ice since has al'ays $een negati)e. A "octor at the time ha" commente" that he seeme" to ha)e some tro#$le making ne#trophils. T an" B cell n#m$ers are 'ithin normal limits. Anti$o"y st#"ies re)eal the follo'ing; +gC total 244 mg7"L Jnormal (2&!1<1 mg7"LK +gA 44 mg7"L Jnormal 11!4<& mg7"LK +gM 4 4 mg7"L Jnormal 41!2(1 mg7"LK =hich of the follo'ing is the most likely "iagnosisA. A"enosine "eaminase "eficiency . rutonAs a$amma$lobulinemia C. I$. subclass "eficiency %. 2y!er I$+ immuno"eficiency (. )elective I$A "eficiency &1. A 42!year!ol" man cons#lts a physician $eca#se he has a 0l#mp0 on his forearm. E.amination of the arm "emonstrates a &!cm "iameter no"#le protr#"ing a$o)e the forearm s#rface. The lesion is co)ere" 'ith apparently normal skin an" is soft an" freely mo)a$le. +t location appears to $e s#$c#taneo#s. The lesion has $een slo'ly gro'ing o)er the past 2 years% an" the patient has e.perience" no "iscomfort. /e has cons#lte" a physician at this time $eca#se his 'ife keeps pestering him to get something "one a$o#t it. =hich of the follo'ing is the most likely "iagnosisA. Ca!illary heman$ioma . %ermatofibroma C. Intra"ermal nevus %. ,i!oma (. )eborrheic keratosis

&8. A 4 !year!ol" man #n"ergoes a ro#tine e.amination. =hile the physical e.amination is #nre)ealing% a hematocrit performe" in the physician,s office gi)es a )al#e of 2 B. Ae)ie' of the peripheral smear re)eals smaller!than!normal erythrocytes. The cells )ary in siDe% an" some ha)e a$normal shapes. The cells "o not appear paler than normal. Aetic#locytes are "ecrease". Ass#ming that this patient has only a single ca#se for his anemia% 'hich of the follo'ing is most likely to $e seen on f#rther e)al#ationA. ,ow iron . ,ow iron bin"in$ ca!acity C. ,ow folate %. ,ow mean cor!uscular hemo$lobin concentration '+C2C) (. ,ow vitamin :; 44. A 2&!year!ol" African American man 'ith A+@S is sent for 'ork #p of the nephrotic syn"rome. /is $loo" press#re is 144712 mm /g. /e has &E e"ema in $oth legs. /is risk factor for A+@S is +? heroin #se. /is creatinine is 2.4 mg7"L% an" his #rine re)eals E& protein% no $loo". A ki"ney $iopsy 'o#l" most likely re)eal 'hich of the follo'ingA. %iabetic ne!hro!athy . -ocal $lomerular sclerosis C. I$A ne!hro!athy %. +embranous ne!hro!athy (. Nil "isease 41. A (!year!ol" 'oman presents to her physician for follo'!#p of a fasting ser#m cholesterol le)el of 2&< mg7"L. She is post!menopa#sal since age 2% an" has $een not $een on hormone replacement therapy. She has a positi)e family history for coronary artery "isease an" she has smoke" one!half pack of cigarettes per "ay for the past 24 years. @#ring her last physical e.amination% a lipi" profile 'as or"ere"% an" she presents to"ay for e)al#ation of those res#lts. =hich of the follo'ing lipi" panels 'o#l" most strongly s#ggest the nee" for pharmacologic therapy in this patientA. 7otal cholesterol :@> m$B",, ,%, cholesterol :?> m$B", . 7otal cholesterol :@? m$B",, ,%, cholesterol :>> m$B", C. 7otal cholesterol ;0> m$B",, ,%, cholesterol :>> m$B", %. 7otal cholesterol ;?/ m$B",, ,%, cholesterol :C/ m$B", (. 7otal cholesterol ;@/ m$B",, ,%, cholesterol :>> m$B",

42. A 4(!year!ol" man presents for follo' #p of his pre)io#s )isit 2 'eeks ago% 'hen he 'as seen for e)al#ation of his "#o"enal #lcer. At that time% a test for /elico$acter pylori 'as performe". The patient 'as other'ise 'ell $#t ha" $een complaining of epigastric pain that 'as e.acer$ate" $y eating. An esophageal!gastro"#o"enoscopy re)eale" the presence of a "#o"enal #lcer% an" $iopsies 'ere taken at that time. +n a""ition% the patient 'as tol" that he nee"e" to mo"ify his "iet% s#ch as "ecreasing his coffee intake% an" c#tting his to$acco #se. The patient ret#rns to"ay to "isc#ss his test res#lts% 'hich 'ere positi)e for the /. pylori organism. =hich of the follo'ing is the most appropriate therapy at this timeA. Amo*icillin orally . ismuth, metroni"a#ole, tetracycline, an" ome!ra#ole orally C. +etroni"a#ole orally %. Ome!ra#ole orally (. )ucralfate orally 4&. A &&!year!ol" 'oman comes to the physician $eca#se of palpitations% restlessness% s'eating% 'eight loss% an" a tremor for the past & 'eeks. She "oes not "rink coffee% tea% so"a% or alcohol% an" she "oes not smoke cigarettes. /er temperat#re is &( 2 381.< 56% $loo" press#re is 1&4714 mm /g% an" p#lse is 847min. E.amination sho's a fine tremor% li" lag an" stare% an" preti$ial my.e"ema. The thyroi" glan" is "iff#sely enlarge"% asymmetric% an" lo$#lar. A $r#it is present o)er the glan". La$oratory st#"ies sho' an #n"etecta$le le)el of thyroi"!stim#lating hormone% an increase" le)el of thyroi" hormones% an" an increase" ra"ioacti)e io"ine #ptake 3AA+U6. The "iagnosis of Cra)es, "isease is ma"e an" the treatment options are "isc#sse". The patient selects ra"ioacti)e io"ine therapy. This patient is at greatest risk for 'hich of the follo'ing con"itionsA. Cholestasis . .ranulocyto!enia C. 2y!othyroi"ism %. 6ecurrent laryn$eal nerve "ama$e (. 7hyroi" carcinoma

44. A < !year!ol" man complains of increasing #rinary fre*#ency an" "ri$$ling at night. /e has no past me"ical history an" is on no me"ications. >n physical e.amination% a "igital rectal e.am re)eals a normal!siDe" prostate. The prostate!specific antigen 3PSA6 le)el is ele)ate" at 1 .4 ng7mL. Ultrasonography re)eals a small hypoechoic area on the prostate meas#ring < L 1 mm in the right lo$e. =hich of the follo'ing is the most appropriate ne.t stepA. A"minister leu!roli"e . io!sy !rostate lesion C. &erform bone scan %. 6e!eat &)A in 0 months (. )can !elvis an" retro!eritoneum 4 . The "ay after h#nting an" skinning 'il" ra$$its% a h#nter "e)elops an inflame" pap#le on one finger. The pap#le rapi"ly enlarges an" then $#rsts% releasing p#s an" forming a clean #lcer ca)ity pro"#cti)e of thin% colorless e.#"ate. Se)eral "ays later% the patient "e)elops se)ere illness 'ith atypical pne#monia an" "eliri#m. +t is at this point that the patient seeks me"ical care. The regional lymph no"es of the a.illa of the affecte" arm are enlarge". Ae"#ce" $reath so#n"s an" occasional rales are hear". Splenomegaly is note". Bloo" st#"ies sho' a mil" le#kocytosis. =hich of the follo'ing is the most likely "iagnosisA. Actinomycosis . rucellosis C. +elioi"osis %. &la$ue (. 7ularemia

4<. A (2!year!ol" man 'ith a 2 !year history of emphysema presents to his physician after he "e)elops the ac#te onset of fe)ers% rigors% an" a co#gh pro"#cti)e of green sp#t#m. The symptoms gra"#ally 'orsen o)er &< ho#rs an" he presents to the emergency "epartment. /e has $een taking a $eclomethasone inhaler t'ice "aily% an al$#terol ne$#liDer treatment at home fo#r times "aily% an" has $een taking erythromycin for a recent $ronchitis. >n physical e.amination he is 11& cm 3< feet6 tall an" 'eighs 1 kg. /is temperat#re is &1.& 2 3144.8 56% $loo" press#re is 1<2782 mm /g% p#lse is 847min% an" respirations are &27min. /is l#ng e.amination re)eals "iff#se $ilateral coarse rhonchi. /e #ses his sternoclei"omastoi" m#scles 'ith each inspiration. An arterial $loo" gas re)eals a p/ of (.24% a p2>2 of <4 mm /g% an" a p>2 of 2 mm /g. >)er the ne.t 2 ho#rs% he $ecomes increasingly tachypneic% an" his p2>2 rises to (4 mm /g. The "ecision is ma"e to int#$ate him at that point. =hich of the follo'ing settings 'o#l" $e most appropriate for his ti"al )ol#me on the respiratorA. />> m,Bbreath . C>> m,Bbreath C. D>> m,Bbreath %. @/> m,Bbreath (. :>>> m,Bbreath Normal ,abs

4(. A 41!year!ol" man presents to his physician for a ro#tine physical e.amination. /e is a ne' to this office an" $rings his pre)io#s me"ical recor" 'ith him. /e has no significant past me"ical history $#t he "oes ha)e a strong family history of cancer an" heart "isease. /is father an" his $rother $oth ha" myocar"ial infarctions $efore age of % an" his sister% mother% an" a#nt ha" $reast cancer. /e e.ercises reg#larly an" eats 'ell% 'ith most of his "iet $eing lo' in sat#rate" fat an" cholesterol. /e smokes one pack of cigarettes per 'eek. /is re)ie' of systems is #nremarka$le. /e is )ery an.io#s an" 'o#l" like only minimal inter)entions "one $eca#se of his goo" health. =hich of the follo'ing is an age!appropriate screening test in this patientA. -astin$ li!i" !rofile . Non4fastin$ total cholesterol level C. Oral $lucose tolerance test %. &rostate e*amination (. )i$moi"osco!y 41. T'o 'eeks after recei)ing an allogeneic $one marro' transplant for treatment of ac#te myelogeno#s le#kemia% a 4 !year!ol" man "e)elops fe)er% intracta$le "iarrhea% generaliDe" rash% an" non!pro"#cti)e co#gh. 2hest .!ray films sho' $ilateral interstitial infiltrates in the l#ng. The patient "ies of o)er'helming sepsis an" m#ltiorgan fail#re. A#topsy in)estigations re)eal cytomegalo)ir#s pne#monia% an" e.tensi)e single cell necrosis in the intestinal epitheli#m an" skin. This complication of $one marro' transplantation is principally me"iate" $y 'hich of the follo'ing cellsA. 4lym!hocytes of bone marrow $raft . ,eukemic cells C. Natural killer cells of reci!ient %. 74lym!hocytes of bone marrow $raft (. 74lym!hocytes of reci!ient

48. A 2 !year!ol" 'oman cons#lts a "ermatologist $eca#se of scaling skin since chil"hoo". Physical e.amination "emonstrates fine scaling of the $ack an" e.tensor s#rfaces of the e.tremities. +n)ol)e" areas also sho' horny pl#gs in the orifices of hair follicles. The fle.or s#rfaces are #nin)ol)e". 2racking of the skin is prominent on the palms an" soles. The patient also has a history of atopy. =hich of the follo'ing is the most likely "iagnosisA. (!i"ermolytic hy!erkeratosis . Ichthyosis vul$aris C. ,amellar ichthyosis %. E4linke" ichthyosis (. Eero"erma 4. A &2!year!ol" 'oman has ha" a 1 !year history of heart$#rn. >)er the past 4 months% she has ha" "iffic#lty s'allo'ing large $ites of soli" foo". She has no "iffic#lty 'ith soft foo"s or li*#i"s% an" she has not lost 'eight. =hich of the follo'ing is the most likely e.planation for her symptomsA. A"enocarcinoma in the lower thir" of the eso!ha$us . arrettAs eso!ha$us in the "istal eso!ha$us C. -ibrosis an" narrowin$ at the "istal eso!ha$us %. )chat#ki rin$ in the "istal eso!ha$us (. )quamous carcinoma in the mi"4thir" of the eso!ha$us
Note: Check your own answers before hittin$ the Check button below. When you click the Check button, a browser win"ow will a!!ear that contains a summary of your results. (*!lanations

Block 1 E.planations

:) 7he correct answer is @. 7he imme"iate ste! is to screen the family for 7 e*!osure. 7he most effective manner in which to accom!lish this is by !lacin$ &&%s on all members an" workin$ u! those with a !ositive test. 7he white cell count may be elevate" for a variety of reasons an" woul" not necessarily hel! in "ia$nosis or mana$ement 'choice A). )!utum cultures will take C months to $row an" may be too cumbersome to obtain 'choice ). Chest C7 scans may show the tuberculosis lesion but a more effective metho" woul" be to !lace the &&% an" !erha!s then scan those with a !ositive test 'choice C). A bronchosco!y woul" be too invasive an o!tion at this !oint 'choice (). ;)7he correct answer is C. 7his !atient has the classic !resentation of a !atient with acute $outy arthritis with the su""en onset of severe !ain 'ty!ically in the mi""le of the ni$ht), swellin$, erythema an" warmth of a sin$le <oint. ,ow4$ra"e fever an" leukocytosis may be seen. It is more common in men an" it is associate" with hy!eruricemia, usually "ue to "ecrease" renal e*cretion of uric aci". Common causes are thia#i"es an" alcohol. %ia$nosis is ma"e by e*amination of <oint flui" un"er !olari#in$ li$ht. Ne$atively birefrin$ent, nee"le4sha!e" crystals within !olymor!honuclear leukocytes, hy!eruricemia, an" acute monoarticular arthritis make the "efinitive "ia$nosis of $out. In"omethacin or colchicine is the treatment "urin$ an acute attack. Allo!urinol, !robeneci", an" sulfin!yra#one are use" for !ro!hyla*is a$ainst further attacks. Allo!urinol 'choice A) is a *anthine o*i"ase inhibitor that is use" as an antihy!eruricemic a$ent by in"ivi"uals with recurrent $outy attacks. Common si"e effects inclu"e rash, hea"ache, an" $astrointestinal u!set. Ceftria*one 'choice ) is the treatment of acute $onococcal arthritis. It has no role in the treatment of $out. &robeneci" 'choice %) is a uricosuric a$ent that increases the rate of urate e*cretion. It is use" to !revent $outy attacks. It may !reci!itate ne!hrolithiasis. )ulfin!yra#one 'choice () is another uricosuric a$ent that increases urate e*cretion. It is use" to !revent $outy attacks. It, too, may !reci!itate ne!hrolithiasis.

0)7he correct answer is C. 7he most likely "ia$nosis is a "ystonic reaction to the "ro!eri"ol. %ro!eri"ol causes its antiemetic effect by anta$oni#in$ "o!aminer$ic rece!tors in the vomitin$ center 'central chemorece!tor #one) of the brain. 7his anti"o!aminer$ic action can !ro"uce torticollis or other "ystonias. A cerebral vascular acci"ent 'choice A) is unlikely $iven that the !atient is alert an" oriente", has no "etectable lan$ua$e "eficit, an" has an otherwise nonfocal neurolo$ic e*amination. A conversion "isor"er 'choice ) is unlikely since the !atient has no !rior history of a !sychiatric "isor"er an" has a viable me"ical reason '"ystonia from "ro!eri"ol) for her neuromuscular "eficit. +unchausen syn"rome 'choice %) is also unlikely since the !atient ha" vali" me"ical reasons for her initial a"mission an" your current visit. We are also not informe" of any !rior history of hos!itali#ations or seekin$ of me"ical attention without a!!ro!riate cause. A sei#ure 'choice () is similarly unlikely since the !atient has no history of a sei#ure "isor"er an" is alert, oriente", an" conversant. ?) (*!lanation: 7he correct answer is (. 7his is stasis "ermatitis, which is a !ersistent inflammation of the skin of the lower le$s. 7he con"ition is often relate" to varicose veins, althou$h it has been !ostulate" that the true cause may instea" be !erivascular fibrin "e!osition an" abnormal small vessel vasoconstrictive refle*es. 7he !resentation illustrate" is ty!ical. +ost !atients are relatively asym!tomatic an" may not seek me"ical attention until the e"ema becomes severe or the lesions become com!licate" by secon"ary bacterial infection or ulceration. It is im!ortant to increase the venous return to the heart by elevatin$ the ankles while restin$ an" use of !ro!erly fitte" su!!ort hose. ,ocal to!ical ta! water com!resses can be hel!ful. &urulent lesions can be treate" with hy"rocolloi" "ressin$s. Flcers are treate" with com!resses an" blan" "ressin$s, such as #inc o*i"e !aste. Ato!ic "ermatitis 'choice A) ty!ically involves the antecubital an" !o!liteal fossas, eyeli"s, neck, an" wrists. Cellulitis 'choice ) is a bacterial infection of the subcutaneous tissues, an" causes local erythema, ten"erness, an" often lym!han$itis. ,ichen sim!le* chronicus 'choice C) is a skin rash cause" by chronic scratchin$ characteri#e" by "ry, scalin$, well4"emarcate", hy!er!i$mente" !laques. Nummular "ermatitis 'choice %) causes wi"es!rea" coin4sha!e", cruste" skin lesions.

/) (*!lanation: 7he correct answer is (.Nocar"ia asteroi"es is an aerobic soil sa!ro!hyte that can cause acute or chronic infectious "isease often characteri#e" by $ranulomatous4su!!urative lesions that may become wi"ely "isseminate". +any, but not all, !atients have un"erlyin$ causes for immuno"eficiency, inclu"in$ a"vance" a$e, lym!horeticular mali$nancies, or$an trans!lantation, hi$h "ose corticosteroi" thera!y, or 'increasin$ly commonly) AI%). %isseminate" nocar"iosis usually starts as a !ulmonary infection that can resemble either a severe !neumonia or tuberculosis. Once "issemination occurs, metastatic brain abscesses are !articularly common, occurrin$ in as many as :B0 of !atients with nocar"iosis. Nocar"iosis is treate" with sulfa "ru$s, such as sulfa"ia#ine or trimetho!rim4sulfametho*a#ole, for !erio"s of months. Actinomyces 'choice A) is very similar to Nocar"ia, but is not aci"4fast. As!er$illus'choice ) is a fun$us. urkhol"eria'choice C)!seu"omallei is a $ram4ne$ative bacillus that causes melioi"osis, which is characteri#e" by lun$ involvement or "isseminate" infection. -rancisella'choice %)tularensis causes tularemia, which is usually acquire" by contact with infecte" wil" rabbits. C) (*!lanation: 7he correct answer is C. &eritonitis in a !atient on CA&% is usually "ue to $ram4!ositive !atho$ens such as )ta!hylococcus aureus or e!i"ermi"is. It is usually characteri#e" by ab"ominal !ain an" over :>> white bloo" cells 'ty!ically !olymor!honuclear leukocytes) in a sam!le of !eritoneal "ialysis flui". Intravenous vancomycin woul" be a reasonable treatment to cover $ram4!ositive !atho$ens. -lucona#ole 'choice A) woul" be in"icate" for a fun$al infection. -un$al !eritonitis is not usually seen until !atients have been treate" with multi!le antibiotics or are further immunosu!!resse". Imme"iate removal of the "ialysis catheter 'choice ) is usually not nee"e" unless the !atient has a !eritonitis that has not im!rove" with a trial of antibiotics. Intravenous $entamicin 'choice %) has $oo" $ram4ne$ative covera$e but woul" not be an i"eal "ru$ to cover )ta!hylococcus. Ci!roflo*acin 'choice () woul" be a very broa" s!ectrum antibiotic that woul" not be a first choice as a sin$le antibiotic to treat sta!hylococcal !eritonitis. -urther, the oral route may not be a"equate as !atients with !eritonitis may have nausea an" vomitin$.

D) (*!lanation: 7he correct answer is %. Any !atient with a first4"e$ree relative who has "evelo!e" an a"enoma or colorectal cancer shoul" un"er$o colonosco!y for screenin$ at a$e />, or :> years before the relative "evelo!e" the a"enoma or carcinoma, whichever comes first. 7his !atient has a brother who has a colon cancer at a$e C>G therefore, a full colonosco!y is warrante". Althou$h there are various o!inions re$ar"in$ a!!ro!riate screenin$ in the Havera$e risk in"ivi"ual,H there is a consensus that full colonosco!y is require" in !atients who have an increase" risk, e.$., first4"e$ree relative with a !ositive history. Annual "i$ital rectal e*amination an" fecal occult bloo" testin$ 'choice A) are no lon$er consi"ere" a reliable metho" of screenin$ for colon cancer, since a shift in the "emo$ra!hics of colon cancer has lea" to more than half bein$ i"entifie" in the first half of the colon. %i$ital rectal e*amination also often fails to i"entify !remali$nant colonic !oly!s. -le*ible si$moi"osco!y 'choice ) is a $oo" initial screenin$ technique for !atients ol"er than /> with no s!ecific known risk factors. If !oly!s are i"entifie", they can be bio!sie", their ty!e establishe", an" subsequent com!lete colonosco!y !erforme" if a"enomas were i"entifie" microsco!ically. -le*ible si$moi"osco!y an" barium enema 'choice C) offers an alternative way of screenin$ the entire colon in !atients in whom a com!lete colonosco!y cannot be !erforme". .enetic testin$ for the !/0 $ene 'choice () is not currently use" for colon cancer screenin$. @) (*!lanation: 7he correct answer is %. 7his is one of the most common neurolo$ic com!lications of AI%). Its !atholo$ic substrate is "e$eneration of the s!inal tracts in the !osterior an" lateral columns, which have a vacuolate" microsco!ic a!!earance. Althou$h the mor!holo$ic chan$es an" clinical manifestations are similar to those associate" with vitamin :; "eficiency, the !atho$enetic mechanism is !robably not relate" to "ietary "eficiencies. )ince there is no s!ecific clinical or laboratory test available for the "ia$nosis of this syn"rome, vacuolar myelo!athy in AI%) !atients remains a "ia$nosis of e*clusion. 7his im!lies that other 2I14relate" neurolo$ic com!lications must be rule" out 'see below). AI%) "ementia com!le* 'choice A) manifests with !ro$ressive memory loss, alterations in fine motor control, urinary incontinence, an" altere" mental status. C+1 !olyra"iculo!athy 'choice ) may simulate 2I1 myelo!athy an" is a relatively frequent com!lication of AI%). It can be e*clu"e" by the results of C)- analysis. C+1 infection lea"s to neutro!hilic !leocytosis in the C)-.

Cry!tococcal menin$oence!halitis 'choice C) woul" lea" to si$ns an" sym!toms of menin$itis. 7he C)- woul" show the fun$al or$anism, which can be "etecte" by s!ecial stains an" culture stu"ies. 3i"ovu"ine4relate" to*icity 'choice () woul" lea" to !ro*imal muscle weakness an" ten"erness "ue mainly to a myo!athic !rocess. I) (*!lanation: 7he correct answer is . 7he usual cause of hy!er!hos!hatemia is a"vance" renal insufficiency that "estroys the ki"neysA ability to e*crete !hos!hate, thereby lea"in$ to hy!er!hos!hatemia. Other causes inclu"e hy!o!arathyroi"ism, !seu"ohy!o!arathyroi"ism, an" e*cessive oral !hos!hate a"ministration. Acute transcellular shifts of !hos!hate into the e*tracellular s!ace can also occur in the settin$s of "iabetic ketoaci"osis, crush in<uries, rhab"omyolysis, systemic infections, an" tumor lysis syn"rome. -ailure of the heart 'choice A), liver 'choice C), or lun$s 'choice %) has no "irect effect on !hos!hate metabolism. &ancreatic failure 'choice () has no "irect effect on !hos!hate metabolism. %iabetic ketoaci"osis can cause a transcellular shift of !hos!hate into the e*tracellular s!ace. :>) (*!lanation: 7he correct answer is (. 7he most effective treatment for a$$ressive systemic lu!us erythematosus with ne!hritis is !ulse cyclo!hos!hami"e. 7his has been shown to be the best a$ent to treat ty!e I1 lu!us ne!hritis. Oral cyclo!hos!hami"e 'choice ) is effective but not as effective as the !ulse I1 form. Oral $ol" 'choice C) is use" to treat rheumatoi" arthritis. Oral !re"nisone 'choice %) an" oral a#athio!rine 'choice A) are other commonly use" a$ents but are not as effective as cyclo!hos!hami"e.

::) (*!lanation: 7he correct answer is A. 7his is &a$et "isease of the breast, which actually corres!on"s microsco!ically to the !resence of in"ivi"ual a"enocarcinoma cells in the e!i"ermis. When it involves the ni!!le area, it usually overlies an area of breast cancer. '&a$et "isease can also involve the vulva an" va$ina, where it may be unrelate" to bulk cancer.) It is im!ortant to reco$ni#e the !resentation, since focusin$ on treatin$ the skin lesion em!irically can lea" to a "elay in reco$ni#in$ the cancer. 7he clinical "escri!tion $iven in the question stem is ty!ical. +astectomy is a common form of treatment in these cases because of the ni!!le involvement. Crohn "isease 'choice ) can cause erythema no"osum '"ee! subcutaneous no"ules on lower le$s). .astric cancer 'choice C) can cause su!raclavicular no"e enlar$ement "ue to metastasis. 6heumatoi" arthritis 'choice %) can cause subcutaneous rheumatoi" no"ules. )ystemic lu!us erythematosus 'choice () can cause a rash, most commonly seen on the face. :;) (*!lanation: 7he correct answer is (. Attitu"es of !hysicians has been shown to have a si$nificant effect on the "elivery of health care. 2ealth !rovi"ers believe men to be more likely to have car"iac "isease than women an" this has $ui"e" !ractice, lea"in$ to a lar$er amount of car"iac !roce"ures for men, com!are" with women. -ewer outcome stu"ies have stu"ie" women because of the belief that car"iac "isease is more common in men 'choice A). 7he inci"ence of car"iovascular "isease in women is similar to that in men 'choice ) across the entire life4s!an. 7here is a si$nificant "is!arity between men an" women in the use of car"iovascular interventions in the absence of financial 'i.e., insurance) barriers 'choice C). One stu"y has re!orte" that women receive more a!!ro!riate car"iac services than men an" that ma<or interventions in men are over4utili#e" 'choice %), but this "oes not e*!lain the overall "is!arity.

:0) (*!lanation: 7he correct answer is C. Nail !ittin$ an" "ystro!hy associate" with "istal "estructive asymmetric arthritis are virtually "ia$nostic of !soriatic arthritis. )kin "isease may or may not be severe an" obvious. Nail chan$es occur in @/= of those with !soriatic arthritis, an" in only ;>= of those with uncom!licate" !soriasis. 7he clinical cate$ories of !soriatic arthritis inclu"e "istal inter!halan$eal, asymmetric, symmetric, mutilatin$, an" s!inal. Antimalarials shoul" be avoi"e" if "isease4mo"ifyin$ thera!y is in"icate", as they can e*acerbate !soriasis. &rimary $enerali#e" osteoarthritis 'choice A) can involve the "istal inter!halan$eal <ointsG <oint erosions "o not occur, an" osteo!hytes are seen ra"iolo$ically. Nail "ystro!hy "oes not occur. A variant of !seu"o$out 'choice ) can closely mimic rheumatoi" arthritis or a mutilatin$ arthro!athy, but nail chan$es are absent, an" there is ra"iolo$ic evi"ence of chon"rocalcinosis 'calcification of articular cartila$e). 6heumatoi" arthritis 'choice %) "oes not cause "istal erosive "isease an" is $enerally 'but not invariably) symmetric. :?) (*!lanation: 7he correct answer is A. While there is some variation in the usa$e of the term, A""ison "isease is usually taken to mean a"renocortical insufficiency relate" to "isease that "estroys the a"renal $lan". +ost authors se!arate out secon"ary a"renocortical insufficiency "ue to !ituitary failure an" recent or current e*o$enous steroi" thera!y. 7rue A""ison "isease, which is not relate" to ina"equate !ituitary secretion of AC72, frequently has sti$mata of hy!er!i$mentation relatin$ to a melanocyte4stimulatin$ hormone '+)2) effect seen with hi$h AC72 levels. 7he biochemical basis of this is a homolo$y between !art of the AC72 molecule an" the +)2 molecule. 7y!ical hy!er!i$mentation features inclu"e black freckles of the shoul"ers, hea", an" neckG bluish4black "iscoloration of areolas an" mucous membranes 'both oral an" ano$enital)G an" "iffuse tannin$, s!ecifically inclu"in$ non4sun4e*!ose" skin. 7he !attern of laboratory screenin$ stu"ies illustrate" in the question stem is also very su$$estive, with very low serum so"ium, hi$h !otassium, low bicarbonate, an" hi$h serum urea nitro$en. A lar$e, furrowe" ton$ue 'choice ) su$$ests acrome$aly. +any s!i"er an$iomas 'choice C) su$$est chronic liver "isease. &rotru"in$ eyeballs 'choice %) su$$est .raveAs "isease. )mall $listenin$ bum!s on the li!s 'choice () su$$ests the mucosal neuromas of +(N IIb.

:/) (*!lanation: 7he correct answer is (. 7his !atient clearly has !neumonia. 7he absolute requirement to "ia$nose !neumonia is an infiltrate on chest ra"io$ra!h, cou!le" with clinical fin"in$s su$$estive of a !neumonia, which this !atient has. 7he ne*t relevant issue is, what is the likely or$anismJ If this man were livin$ at home, the most common or$anism is the !neumococcus. 2owever, $iven his recent hemicolectomy, the !ossibility of a more virulent or$anism emer$es. 7his is the most critical factor in "ictatin$ his course of thera!y. Whether the !atient receive" the annual influen#a vaccine 'choice A) is im!ortant $iven his a$e an" the morbi"ity associate" with an influen#a infection, however, it has no bearin$ on the course of this !neumonia or its thera!y. 2is li!i" status 'choice ) is not relevant to thera!y of his !neumonia. 7he sta$e of his colon cancer 'choice C) is im!ortant, but not for the treatment of his !neumonia. 7here is no clinical relationshi! between !neumonia severity an" colon cancer !ro$ression until late sta$e metastases occur an" there is a !ossibility of !ost4 obstructive !neumonia "evelo!in$. 7he !atients social su!!ort structure for home thera!y 'choice %) is an issue only at the time of "ischar$e when the issue of continue" home thera!y is im!ortant. At this time his ability to have assistance with !ossible intravenous me"ications is an issue. :C) (*!lanation: 7he correct answer is A. 7his !atient has !ernicious anemia, as "emonstrate" by correction of her "eficiency in intrinsic factor !ro"uction by her !arietal cells. 7his is an autoimmune "isease "irecte" a$ainst the !arietal cells of the stomach, which are the normal !ro"ucers of the intrinsic factor nee"e" for absor!tion of vitamin :;. 7he lack of :; then causes "evelo!ment of a me$aloblastic 'with hi$h mean cor!uscular volume) anemia. io!sy of the $astric mucosa in these cases reveals atro!hic $astritis. 7he )chillin$ test e*amines the absor!tion of ra"ioactively labele" vitamin :; before an" after a"ministration of intrinsic factor. A result of !oor absor!tion of :; before a"ministration of intrinsic factor an" $oo" absor!tion after stron$ly su$$ests !ernicious anemia as the "ia$nosis. &atients with !ernicious anemia require lon$4term '!robably life4lon$) !arenteral re!lacement of vitamin :; an" may also have other si$nificant autoantibo"ies, notably those "irecte" a$ainst thyroi" anti$ens. Choices , C, an" % may all !ro"uce a :; "eficiency, but they "o not correct with intrinsic factor. In !atients with bacterial over$rowth 'choice ), the e*cess bacteria will !referentially absorb intraluminal :;. Chronic !ancreatitis 'choice C) may !re"is!ose to a :; "eficiency by failin$ to secrete the en#ymes that are necessary to cleave the salivary 6 factor from the :;, makin$ it unavailable for bin"in$ to intrinsic factor.

Crohn "isease 'choice %) may cause a :; "eficiency if the terminal ileum is severely inflame" or has been resecte". .astric ulcer 'choice () can cause anemia secon"ary to blee"in$, but it woul" be a normocytic anemia 'unless there ha" been enou$h bloo" loss to cause iron "eficiency, in which case it woul" microcytic) an" woul" not be e*!ecte" to res!on" to :;. :D)(*!lanation: 7he correct answer is C. 7his !atient has !ernicious anemia, in which autoimmune $astritis causes a lack of the intrinsic factor nee"e" to absorb vitamin :;. Autoantibo"ies that are often !resent inclu"e those "irecte" a$ainst the microsomal fraction of !arietal cells an" those ca!able of neutrali#in$ intrinsic factor. 7he result is that vitamin :; can no lon$er be absorbe" by the terminal ileum. )ince some vitamin :; is store" in the liver, "eficiency ten"s to "evelo! slowly. 1itamin :; "eficiency can cause me$aloblastic anemiaG neurolo$ic abnormalities that ten" to be$in with loss of !osition an" vibration senseG an" .I manifestations inclu"in$ anore*ia, intermittent consti!ation or "iarrhea, an" ab"ominal !ain. Antibo"ies to basement membrane 'choice A) are associate" with .oo"!asture syn"rome. Antibo"ies to insulin rece!tors 'choice ) are associate" with insulin resistance. Antibo"ies to ))4 'choice %) occur in association with )<K$ren syn"rome. Antibo"ies to 7)2 rece!tor 'choice () are associate" with .raves "isease. :@) (*!lanation: 7he correct answer is A. 7his !atient "is!lays si$ns of oculomotor !alsy, with restriction of the eye movements in all "irections 'e*ce!t laterally, "ue to !reservation of the si*th cranial nerve, the ab"ucens), an" !tosis. %ilatation of the !u!il, which fails to react to li$ht, is a si$n of intracranial com!ression of the thir", or oculomotor cranial nerve. 7his shoul" !rom!t search for an un"erlyin$ sur$ical cause of oculomotor !alsy. Fncal herniation an" aneurysm of the !osterior communicatin$ artery are the two most common sur$ical con"itions lea"in$ to oculomotor !alsy. In the absence of clinical evi"ence of increase" intracranial !ressure, it may be assume" that the !atient has an aneurysm of the !osterior communicatin$ artery until !roven otherwise. Cerebral an$io$ra!hy is the investi$ation of choice to confirm the "ia$nosis. All of the most common me"ical causes of oculomotor nerve !alsy result in !aresis of e*traocular movements an" !tosis, but the !u!illary li$ht refle* is !reserve". 7hese con"itions inclu"e "iabetes mellitus 'choice C), $iant cell arteritis 'choice %), sy!hilis 'choice () an" systemic hy!ertension 'choice -).

Carcinoma of the ri$ht !ulmonary a!e* 'choice ) may result in 2orner syn"rome 'miosis, !tosis, eno!hthalmos, an" loss of sweatin$ on the affecte" hemiface) "ue to infiltration of the cervical autonomic $an$lia.

:I) (*!lanation: 7he correct answer is . 7his is the way that cavernous sinus thrombosis !resents. 7his con"ition is "ue to a se!tic thrombosis that can com!licate chronic bacterial sinusitis. +enin$itis is another si$nificant !ossibility. ,umbar !uncture is "an$erous in a !atient with increase" intracranial !ressure, as in"icate" by the !a!ille"ema. (mer$ency C7 scan of the cavernous sinus, air sinuses, orbit, an" brain is warrante". A""itionally, cultures of bloo" an" any nasal "ischar$e are warrante"G .ramAs stain of the nasal "ischar$e may $ive a !reliminary in"ication of the causative or$anism. 2i$h "ose intravenous antibiotics are starte", an" then altere", if necessary, when culture results are re!orte". Cavernous sinus thrombosis has a 0>= mortality rate, even when !rom!t, a!!ro!riate me"ical care is $iven. )im!ly monitorin$ 'choices A an" () a !atient like this woul" be very "an$erous. Fltrasoun" 'choice %) woul" !robably not a"equately visuali#e the com!le* structures of the sinuses, orbits, an" brain. )ur$ery 'choice C) is not in"icate" in this settin$. ;>) (*!lanation: 7he correct answer is %. efore initiatin$ !harmacolo$ic thera!y, it is worthwhile to consi"er lifestyle mo"ifications that may re"uce sym!toms in !atients with $astroeso!ha$eal reflu* "isease '.(6%). In this re$ar", elevatin$ the hea" of his be" is an im!ortant ste!, since it will re"uce the "e$ree of nocturnal aci" reflu* while the !atient is in the su!ine !osition. Other non!harmacolo$ic measures that may be hel!ful inclu"e avoi"ance of stron$ stimulants of aci" secretion 'coffee, alcohol), avoi"ance of certain "ru$s 'anticholiner$ics) an" foo"s 'fats, chocolates), an" cessation of smokin$. Althou$h fatty foo"s may e*acerbate .(6% by re"ucin$ the !ressure on the lower eso!ha$eal s!hincter ',()), hi$h4!rotein "iets 'choice A) an" carbohy"rates 'choice () have no !articular effect on the mechanism or sym!toms or .(6%. Althou$h as!irin 'choice ) may be in<urious to the $astric an" "uo"enal mucosa, it is not im!licate" in e*acerbations of .(6%. Acetamino!hen 'choice C) has no effect on the sym!toms or cause of .(6%.

;:) (*!lanation: 7he correct answer is . A !rolon$ation of the 86) interval is hi$hly !re"ictive of both car"iac an" CN) to*icities from tricyclic anti"e!ressant in$estion. ,eft "eviation of the 86) a*is 'choice A), which can be seen with con"itions such as left ventricular hy!ertro!hy an" left bun"le branch block, is not ty!ically associate" with tricyclic car"iac to*icity. Conversely, ri$ht "eviation of the 86) a*is '$reater than :;> "e$rees) is very !re"ictive of car"iac to*icity from tricyclics. )hortenin$ of the 87 interval 'choice C) is not seen with tricyclic to*icity, but can be seen with metabolic "eran$ements such as hy!ercalcemia. Neither )7 se$ment "e!ression 'choice %) nor 7 wave inversion 'choice () is "irectly associate" with car"iac to*icity from tricyclic over"ose. 7hese chan$es may be seen, however, in con<unction with the more classic (C. manifestations of tricyclic to*icity '!rolon$e" 86) interval, ri$ht a*is "eviation) if the resultin$ car"iac to*icity lea"s to "iminishe" coronary !erfusion an" ischemia. ;;) (*!lanation: 7he correct answer is . Althou$h this !atient has rarely been aware of sym!toms of $astroeso!ha$eal reflu* "isease '.(6%), the "evelo!ment of a !e!tic stricture clearly in"icates lon$stan"in$ aci" reflu* into the "istal eso!ha$us. 7his will be a !ersistent !rocess an", if not treate", will lea" to recurrent strictures. 2e therefore requires chronic treatment with a !roton !um! inhibitor to su!!ress aci" secretions. -amoti"ine an" antaci"s, such as ma$nesium hy"ro*i"e 'choices A an" C), are a"<uncts to the mainstay of thera!y, which is !roton !um! inhibition. (ven thou$h the !atient is not sym!tomatic, he "oes require continue" aci" su!!ression. +etoclo!rami"e 'choice %) re"uces the lower eso!ha$eal s!hincter !ressure an" is an a"<unct to aci" su!!ression in the mana$ement of !atients with reflu*. It is not use" as first4line thera!y, however. It is nowhere near as effective as !roton !um! inhibitors an" frequently lea"s to si"e effects of se"ation because of its ability to cross the bloo"4brain barrier an" inhibit "o!amine, !ro"ucin$ &arkinson4like sym!toms. As state" above, this !atient will have recurrent strictures if he "oes not receive treatment. 7herefore, choice ( is incorrect.

;0) (*!lanation: 7he correct answer is A. Acute a!!en"icitis is the secon" most common cause in the F.)., behin" hernia, of severe acute ab"ominal !ain that requires ab"ominal o!eration. Althou$h it can occur at all a$es, many !atients, like this man, are teena$ers or youn$ a"ults. 7his !atientAs !resentation is ty!ical for acute a!!en"icitis, with initially !oorly locali#e" !ain that is followe" by nausea an" vomitin$. In classic a!!en"icitis, the !ain shifts to the ri$ht lower qua"rant, where it becomes more locali#e". In most !atients, acute obstruction of the a!!en"iceal orifice by a fecalith initiates the acute a!!en"icitis. 7he acute onset of ileocolitis 'choice ) will !ro"uce "iarrhea or bloo"y stools. 7here is no evi"ence to su$$est an etiolo$y for ischemic colitis 'choice C), which will ty!ically !resent with bloo"y "iarrhea an" often with left4si"e" ab"ominal !ain. Acute 9ersinia infection 'choice %) will !ro"uce acute ri$ht lower qua"rant fin"in$s similar to those of acute a!!en"icitis. 2owever, it is accom!anie" by "iarrhea, which is not "escribe" in this case. 7here is no reason to sus!ect obstruction of the ileocecal area by any mass 'choice () in a ;04year4ol" man. )uch an obstruction, shoul" it occur, woul" ty!ically !resent with ab"ominal "istention as a result of bowel obstruction. ;?) (*!lanation: 7he correct answer is A. A"ministerin$ $lucose to a !atient who is "eficient in thiamin may !reci!itate Wernicke45orsakoff syn"rome, which is a combination of confusion, ata*ia, o!hthalmo!le$ia, antero$ra"e an" retro$ra"e amnesia, an" confabulation. It is therefore im!erative to a"minister I1 thiamin !rior to $lucose4containin$ I1 flui"s. Niacin 'choice ) is an essential com!onent of the coen#ymes involve" in o*i"ation4 re"uction reactions. &rofoun" "eficiency in niacin causes the classic tria" of !ella$ra: "ermatitis, "iarrhea, an" "ementia. 1itamin :; "eficiency 'choice C) may lea" to me$aloblastic anemia, neurolo$ic com!lications, an" "ementia. 1itamin C "eficiency 'choice %) may lea" to "ifficulty with woun" healin$ an" scurvy. 1itamin 5 'choice () is essential for the !ro"uction of selecte" clottin$ factors. Althou$h alcoholics may be "eficient in all the vitamins mentione" in the answer choices, only "eficits in thiamin are associate" with harmful effects if $lucose is a"ministere" without re!lenishment.

;/) (*!lanation: 7he correct answer is . 7he clinical manifestations are consistent with fat embolism. 7his com!lication is frequent, followin$ fractures of lon$ bones, but is usually asym!tomatic. -at embolism mainly affects the lun$s an" the brain, an" the clinical !icture consists of "ys!nea, tachycar"ia, an" mental status chan$es. Only rarely, "oes this con"ition lea" to "eath. In the lun$s, fat emboli can be visuali#e" histolo$ically. In the brain, multifocal !etechiae in the white matter re!resent the most common !atholo$ic chan$e. %iffuse a*onal in<ury 'choice A) is one of the most common forms of traumatic brain in<ury. It involves the central white matter, es!ecially the cor!us callosum an" cerebral !e"uncles. It is sometimes associate" with small !etechiae in these areas. 7he !atient may "evelo! coma a few hours to "ays after hea" trauma. )e!tic embolism 'choice C) results from se!tic emboli lo"$in$ in the terminal intra!arenchymal arteries of the brain. It lea"s to multi!le cortical infarcts, usually of the hemorrha$ic ty!e. 7he white matter is s!are". )ystemic thromboembolism 'choice %) is usually of car"iac ori$inLfor e*am!le, in !atients with car"iac arrhythmias with thrombi in the ri$ht atrium or ventricle. 7hromboemboli in the brain cause hemorrha$ic infarction in the corte*. Watershe" infarction 'choice () is often seen in !atients sufferin$ from acute hy!otensive e!iso"es, es!ecially if the circle of Willis is alrea"y com!romise" by atherosclerotic chan$e. 7he cortical re$ions at the bor"er #one between "ifferent vascular territories 'e.$., between the "istribution of the anterior an" mi""le cerebral arteries) un"er$o ischemic necrosis. ;C) (*!lanation:7he correct answer is %. 7he !atient has a !rostatic abscess. 7he ty!ical a$e is ?> to C> years, an" is consequently somewhat youn$er than the a$es at which beni$n !rostatic hy!er!lasia an" !rostate cancer become ma<or !roblems. Infectin$ or$anisms inclu"e aerobic $ram4ne$ative bacilli an" )ta!hylococcus aureus. &rostatic abscess shoul" be sus!ecte" when a man "evelo!s re!eate" urinary tract infections that seem to $et better with antibiotic thera!y, only to recur later. 7he most im!ortant "ia$nostic clue, if "etectable, is the !resence of a fluctuant mass in the !rostate on rectal e*am. )ome !atients have only !rostatic enlar$ement, or even no !ositive fin"in$s on !hysical e*amination. &atients may have normal urine, althou$h it is more usual for an or$anism to be culture" at some !oint. &rostatic ultrasoun" may be hel!ful if the "ia$nosis is sus!ecte". A few cases are even !icke" u! at the time of !rostatic resection for beni$n !rostatic hy!er!lasia or other "isease. 7reatment is with evacuation of the abscess by a transurethral or !erineal route followe" by a!!ro!riate antibiotics. eni$n !rostatic hy!er!lasia 'choice A) can cause urinary obstruction !re"is!osin$ for bla""er infection, but the !rostate woul" not be fluctuant.

Chronic nonbacterial !rostatitis 'choice ) can cause sym!toms resemblin$ urinary tract infection, but woul" not cause a fluctuant !rostate. &rosta"ynia 'choice C) is a noninfectious, noninflammatory con"ition of youn$er men that can mimic !rostatitis, but woul" not cause a fluctuant !rostate. &rostatic carcinoma 'choice () is usually asym!tomatic, an" can cause a firm !rostatic mass. ;D) (*!lanation: 7he correct answer is %. &atients who receive the he!atitis vaccine will "evelo! only surface antibo"ies, since the vaccine contains only e!ito!es of the surface anti$en an" NO7 of the intact viral !article, which contains the core antibo"y. 7hese !atients will therefore have a ne$ative surface anti$en, ne$ative core antibo"y, an" !ositive surface antibo"y. Choice A "escribes a !atient who has recent he!atitis "evelo!e" surface antibo"y . infection an" has not yet

Choice "escribes a !atient that has ha" a !ast he!atitis immunity.

infection an" has "evelo!e" an" has not yet

Choice C refers to a !atient who has "evelo!e" acute he!atitis "evelo!e" immunity.

Choice ( "escribes a !atient who has never been e*!ose" to surface anti$en an" has no immunity. ;@) (*!lanation: 7he correct answer is C. 7here is an e*tensive bo"y of literature in"icatin$ that beta blockers $iven to non4car"iac sur$ical !atients who are at risk of car"iac events are associate" with a more favorable outcome in terms of !osto!erative car"iovascular morbi"ity an" mortality. 7his !atient has somewhat !oorly controlle" hy!ertension, as well as at least three car"iovascular risk factors 'hy!ertension, tobacco, a$e). I"eally, one woul" like to have better control of the bloo" !ressure an" to re"uce any risk for a"verse !erio!erative events. eta blockers can achieve both of these en"!oints. Ca!to!ril 'choice A) is an AC( inhibitor that has $oo" efficacy in the treatment of hy!ertension. 7his class of "ru$s has also been shown to !rolon$ survival in !atients with con$estive heart failure. Cloni"ine 'choice ) is a central al!ha4; rece!tor a$onist that works to attenuate sym!athetic outflow an" thus lower bloo" !ressure. Althou$h it is a reasonably

efficacious "ru$, it is associate" with reboun" hy!ertension if abru!tly "iscontinue". It has no role in the !erio!erative mana$ement of bloo" !ressure. Nife"i!ine 'choice %) is a calcium channel blocker that has reasonable efficacy in treatin$ hy!ertension. 7here is no benefit to $ivin$ this a$ent in the !erio!erative !erio". &ra#osin 'choice () is a al!ha4: rece!tor anta$onist that is very efficacious in the treatment of hy!ertension. 7his class of "ru$s is also useful in the treatment of beni$n !rostatic hy!ertro!hy ' &2). ;I) (*!lanation: 7he correct answer is %. 7his !atientAs occu!ational history of workin$ in the minin$ in"ustry shoul" always !rom!t the consi"eration of the "ia$nosis of silicosis. 7here is usually bilateral u!!er lobe involvement associate" with hilar lym!ha"eno!athy an" He$$shellH calcification of the visuali#e" lym!h no"es. 7here is no "escri!tion of asbestos e*!osure 'choice A), makin$ this "ia$nosis unlikely. roncho!ulmonary as!er$illosis 'choice ) is often seen in farm workers who have been workin$ in silos. Cystic fibrosis 'choice C) !resents "urin$ chil"hoo", an" these !atients "o not survive to this a$e. 7here is no evi"ence of tuberculosis e*!osure 'choice () by the !atientAs history. 0>) (*!lanation: 7he correct answer is A. 7he sym!toms an" si$ns that she is "escribin$, !articularly in the conte*t of atrial fibrillation 'A-), su$$est !eri!heral emboli#ation, which is a sur$ical emer$ency. 7he treatment of choice involves imme"iate embolectomy, which is usually !erforme" by a vascular sur$eon, followe" by anticoa$ulation. 2er subthera!eutic !rothrombin time an" !ersistent A- on e*amination are su!!ortive of this "ia$nosis. All the other choices 'choices throu$h () re!resent actions that woul" "elay sur$ical care an" likely lea" to the loss of her limb. (ven if the !hysician has not seen this !atient before, it is incumbent on him to "irect her to an" facilitate imme"iate sur$ical intervention.

0:) (*!lanation: 7he correct answer is %. 7his !atient is in !ericar"ial tam!ona"e, most !robably as a result of his mali$nancy. ,un$ cancer is !articularly likely to cause !ericar"ial effusions. -urthermore, since this !atient has metastases to the !ericar"ium, he mi$ht be blee"in$ into the !ericar"ial s!ace. 7his tam!ona"e may be the cause of his si$nificant hy!otension an" the soft car"iac soun"s. (lectrical alternans, a !henomenon in which the 86) chan$es a*is, is in"icative of !ericar"ial effusion, since the heart is movin$ freely in the flui", causin$ a chan$e in a*is note" on the (C.. (mer$ently, this !atient nee"s "ecom!ression of the !ericar"ial s!ace with the ai" of !ericar"iocentesis, whereby a catheter "irectly "rains the flui" in the !ericar"ial sac. eta4blockers 'choice A) woul" be of no benefit in treatin$ car"iac tam!ona"e. Nonsteroi"al anti4inflammatory "ru$s 'N)AI%s) 'choice ) can be useful in treatin$ !ericar"itis, which may cause !ericar"ial effusions. 2owever, this is a lon$er term o!tion an" will have little utility emer$ently. )teroi"s 'choice C) may similarly be use" in !ericar"itis, after N)AI%s have faile". 2owever, this is an o!tion to be e*!lore" after the !ericar"ial flui" has been "raine". Car"iac catheteri#ation is often use" to confirm the "ia$nosis of tam!ona"e 'choice (). 7y!ically the !ressure equali#es across the ri$ht atrium an" ventricle. 2owever, emer$ently, this !atient shoul" have !ericar"iocentesis. 0;) (*!lanation: 7he correct answer is C. 7his is a !atient who has 0 clear risk factors for coronary artery "isease 'tobacco, family history an" a$e) an" base" on his !hysical e*amination, likely has severe !eri!heral vascular "isease. 2e has, by "efinition, ty!ical chest !ain, so calle" Hnew onset an$inaH. 2e is a !rime !atient to have si$nificant coronary "isease, an" thus we sus!ect ischemia as a cause for his !ain. As a surro$ate for coronary an$io$ra!hy, which actually shows anatomy, an e*ercise trea"mill test allows us to "etect (C. chan$es of ischemia with activity an" thus stratify this !atient as requirin$ intervention 'such as !ercutaneous transluminal coronary an$io!lasty, or coronary artery by!ass $rafts), or !erha!s an$io$ra!hy to better evaluate his anatomy. A restin$ (C. 'choice A) is a!!ro!riate, but not the most a!!ro!riate, $iven that he is !ain4free at !resent an" one woul" not e*!ect to see any (C. chan$es associate" with ischemia. A car"iac echocar"io$ram 'choice ) will likely be !erforme", $iven his "ys!nea on e*ertion, but is not an a!!ro!riate test in the tria$in$ of sus!ecte" ischemic chest !ain. In

some centers a Hstress4echoH, s!ecifically a "obutamine echocar"io$ram, is use" to evaluate ischemic !otential. A non4ur$ent coronary an$io$ra!hy 'choice %) is also ina!!ro!riate since an$io$ra!hy is an invasive !roce"ure reserve" for !eo!le that have ha" equivocal results from less invasive "ia$nostic !roce"ures, or are havin$ si$ns of crescen"o an$ina. 7his !atient has new an$ina, but it is Hty!icalH an$ina in that it is e*ertional. An imme"iate coronary an$io$ram 'choice () is clearly not in"icate" as the !atient is not havin$ active ischemia or a myocar"ial infarction requirin$ re!erfusion. 00) (*!lanation: 7he correct answer is (. 7he acute infection "ue to he!atitis C virus '2C1) is most commonly asym!tomatic, but @>= of these cases !ro$ress to chronic he!atitis. Of the @>=, ;>= will eventually evolve to cirrhosis. 7he source of infection remains unknown in a substantial number of cases, but />= are relate" to I1 "ru$ abuse an" ?= are attributable to bloo" transfusion. 2C1, on the other han", is now the most common cause of transfusion4associate" he!atitis. 7he mo"e of !resentation of chronic he!atitis C is often insi"ious, an" !atients mi$ht well be in $oo" health when elevate" aminotransferases are "iscovere". 7his laboratory fin"in$ !rom!ts a""itional investi$ations, usually inclu"in$ a !ercutaneous liver bio!sy. 7his will "emonstrate the ty!ical histolo$ic chan$es of chronic he!atitis, namely chronic !ortal inflammation ero"in$, to varyin$ e*tents, into the he!atic lobule. 7he "e$ree of lobular HinvasionH by the !ortal inflammatory infiltrate is the main in"icator of the !ro!ensity for evolution to cirrhosis. +ale se*, infection after a$e ?>, an" alcohol consum!tion are risk factors for evolution of chronic he!atitis C to cirrhosis. Nowa"ays, 2C1 is consi"ere" the most common cause of chronic he!atitis an" one of the most common causes of cirrhosis in in"ustriali#e" countries.

0?) (*!lanation: 7he correct answer is %. 7he etiolo$y of !neumonia is relate" to both the a$e of the !atient an" the !articular risk factors that he or she may e*hibit. -or !atients with no s!ecific risk factors, !neumonia is referre" to as community4acquire" !neumonia 'CA&). CA& has a variable etiolo$y "e!en"in$ on the a$e of the !atient. In !atients a$e" ;I4//, the !neumococcus ')tre!tococcus !neumoniae), a $ram4!ositive or$anism, is the most frequent a$ent causin$ so4calle" ty!ical or bacterial !neumonia.

or"etella !ertussis'choice A) causes whoo!in$ cou$h in chil"ren. +ost a"ults in the F. ). have been vaccinate" a$ainst this or$anism. 2owever, ;> years after the last booster, immunity be$ins to fa"e, an" it is reasonably common to see !atients a$e" // an" ol"er !resentin$ with u!!er an" lower res!iratory tract infections cause" by this or$anism. 5lebsiella !neumoniae'choice ) is a reasonably frequent source of !neumonia in both hos!itali#e" !atients an" those with chronic as!iration !roblems, such as !ost4stroke !atients. +yco!lasma !neumoniae'choice C) is the !rimary a$ent res!onsible for so4calle" aty!ical !neumonia in the same a$e bracket. )ta!hylococcus aureus'choice () is a $ram4!ositive or$anism that causes severe cavitatin$ !neumonia. It is most often res!onsible for !neumonia in "iabetic !atients. 0/) (*!lanation: 7he correct answer is (. In the F.)., lun$ "ama$e from smokin$ is by far the most im!ortant contributor to lun$ "isease, from an e!i"emiolo$ic stan"!oint. In a""ition, smokin$ can si$nificantly e*acerbate the clinical course of other "iseases that affect the lun$s, such as asthma or cystic fibrosis. -ortunately, smokin$ behavior can be !otentially altere" by new !harmacolo$ic a!!roaches. Althou$h the !hysician may have aske" this !atient about smokin$ before, it is now time to review her smokin$ history in "etail. Askin$ about alcohol 'choice A) an" "ru$s of abuse 'choice ) is always im!ortant, even in the el"erly, but reviewin$ the smokin$ history shoul" take !rece"ence here. Alcohol use ty!ically "oes not "irectly lea" to lun$ !atholo$y or cause "ys!nea. )ome "ru$s of abuse, such as mari<uana, crack cocaine, an" heroin, have "eleterious effects when intro"uce" into the lun$s. 7hat sai", the a$e of this !atient makes it much more likely that she woul" be usin$ tobacco. Althou$h e*!lorin$ the marital status 'choice C) an" key relationshi!s in a !atientAs life is very im!ortant in terms of the !atientAs overall health, this !rocess has little a""itional role "urin$ this visit. 6eviewin$ the occu!ational history 'choice %) is also an im!ortant !art in the evaluation of "ys!nea. Althou$h smokin$ usually oversha"ows occu!ational4relate" lun$ in<uries, the occu!ational history shoul" take a close secon" to the smokin$ history. Occu!ational e*!osures can e*acerbate "iseases such as asthma an" can cause "iseases such asbestosis an" silicosis. If the !atient ha" been a />4year4ol" brake mechanic or shi!yar" worker, the occu!ational history may have been a more im!ortant com!onent of the social history on which to concentrate.

0C) (*!lanation: 7he correct answer is A. 7his !atient shoul" have antibiotic !ro!hyla*is before un"er$oin$ "ental work. 7he !atientAs !hysical e*amination is consistent with asym!tomatic aortic insufficiency, as in"icate" by his lack of sym!toms combine" with a characteristic "iastolic murmur. 7his has occurre" as a result of his chil"hoo" rheumatic fever. &atients with any si$nificant car"iac valvular "isease shoul" be instructe" to have antibiotic !ro!hyla*is before "ental work to re"uce the risk of subacute bacterial en"ocar"itis. Althou$h this man is at increase" risk for lun$ cancer $iven his lon$ history of smokin$, chest *4ray films 'choice ) have never been !roven effective as early "etection. Althou$h he "oes have un"erlyin$ valvular heart "isease, there is no in"ication for an annual echocar"io$ram 'choice C) unless s!ecific sym!toms "evelo! an" warrant evaluation. A si$moi"osco!y 'choice %) is one of several choices that are a!!ro!riate colorectal cancer screenin$ e*aminations be$innin$ at a$e />. &rostate s!ecific anti$en testin$ 'choice () remains controversial in asym!tomatic a"ults an" is certainly not recommen"e" in asym!tomatic men youn$er than />. 0D) (*!lanation: 7he correct answer is %. 2y!er I$+ immuno"eficiency is a con$enital, often E4linke", form of immuno"eficiency which is characteri#e" by low I$. an" I$A an" com!ensatory hi$h I$+. 7he immuno"eficiency causes increase" susce!tibility to ma<or $ram4!ositive !atho$ens an" o!!ortunistic infections 'such as the !atientAs &neumocystis infection). 7he biochemical basis of the con"ition a!!ears to be a "efect in a rece!tor on the 7 cell membrane that hel!s to tri$$er cell switchin$ from I$+ to I$A, I$., an" I$ (. Cases, such as in this question, in which the !roblem is not !icke" u! because of the immuno"eficiency may come to me"ical attention with other features of the syn"rome, inclu"in$ lym!ha"eno!athy, autoimmunity 'notably Coombs !ositive hemolytic anemia), or chronic liver "isease. A"enosine "eaminase "eficiency 'choice A) is a cause of a form of severe combine" immuno"eficiency that usually !resents 'often with thrush) in the first three months of life. rutonAs a$amma$lobulinemia 'choice ), also known as *4linke" a$amma$lobulinemia, is characteri#e" by marke"ly "ecrease" cell numbers, an" low values of all of the immuno$lobulins, !articularly I$..

I$. subclass "eficiency 'choice C) is characteri#e" by marke"ly "ecrease" levels of a sin$le I$. subclass in the settin$ of normal total I$. levels. )elective I$A "eficiency 'choice () is very mil", an" is usually clinically si$nificant only because of a ten"ency to ana!hyla*is if $iven I$A4containin$ bloo" !ro"ucts. 0@) (*!lanation: 7he correct answer is %. 7his is !robably a li!oma, which is a beni$n mass lesion com!ose" of mature a"i!ose tissue boun" by a limitin$ membrane. 'Another !ossibility is an e!i"ermoi" cyst, which can be in"istin$uishable clinically from li!oma.) ,i!omas are very common, an" !atients may have more than one li!oma. Common sites inclu"e the trunk, na!e of the neck, an" forearms. 7he lesions are only rarely mali$nant, althou$h a ra!i"ly $rowin$ lesion shoul" be bio!sie" to make sure of the "ia$nosis. 7hey are usually asym!tomaticG a small !ercenta$e are !ainful. 7hey can be treate" with sur$ical e*cision or li!osuction. Ca!illary heman$ioma 'choice A), also known as strawberry mark, is a bri$ht re", vascular lesion that usually "evelo!s shortly after birth an" then often involutes by late chil"hoo". %ermatofibroma 'choice ) causes a firm, re" to brown, small !a!ule or no"ule that is most frequently foun" on the le$s. Intra"ermal nevus 'choice C) causes a flesh colore" to black, elevate", lesion that is usually 0 to C mm in si#e. )eborrheic keratosis 'choice () causes a !i$mente", su!erficial, usually warty, e!ithelial lesion. 0I) (*!lanation: 7he correct choice is A. 7his !atient has a microcytic, normochromic anemia an" is not obviously ill on !hysical e*amination. 7he overwhelmin$ly most likely "ia$nosis is iron4"eficiency anemia, which is, in turn, almost always "ue to blee"in$ in a"ults on a ty!ical American "iet. Occult .I blee"in$ is a common source in both men an" women. +enstrual "isor"ers are also im!ortant causes of iron "eficiency in women. ,ow iron bin"in$ ca!acity 'choice ) is seen in the anemia of chronic "isease. ,ow folate 'choice C) !ro"uces a me$aloblastic anemia.

,ow mean cor!uscular hemo$lobin concentration '+C2CG choice %) is seen in hy!ochromic anemias. ,ow vitamin :; 'choice () !ro"uces a me$aloblastic anemia. ?>) 7he correct answer is . -ocal $lomerular sclerosis is the ty!e of ne!hro!athy most commonly seen in African American I1 "ru$ users with AI%). It is likely to lea" to a very ra!i" loss of renal function. 7here is no clinical evi"ence to in"icate that this !erson has "iabetes, makin$ "iabetic ne!hro!athy 'choice A) unlikely. Nil "isease 'choice (), I$A ne!hro!athy 'choice C) an" membranous ne!hro!athy 'choice %) are only very rarely associate" with AI%). ?:) 7he correct answer is %. -or those !atients in whom a fastin$ !anel has been obtaine", a ste!wise a!!roach to intervention base" on the !atientAs ,%, an" risk factors may be use". A !atient with ;M risk factors 'this !atient) an" an ,%, of $reater than :C> m$B", warrants me"ical thera!y. A total cholesterol of :@> m$B",, ,%, cholesterol of :?> m$B", 'choice A) or a total cholesterol of :@? m$B", with an ,%, cholesterol :>> m$B", 'choice ) in this !atient coul" be mana$e" with a trial of "ietary mo"ification an" e"ucation. -or mar$inally hi$h total cholesterol: total cholesterol ;0> m$B",, ,%, cholesterol :>> m$B", 'choice C), there is no in"ication for "ru$ thera!y because the ,%, is still not above :0>. A total cholesterol of ;@/ m$B", with an ,%, cholesterol of :>> m$B", 'choice (), althou$h "isconcertin$, "oes not require "ru$ thera!y. 7he total cholesterol is elevate", but the ,%, is not, su$$estin$ either increase" tri$lyceri"es or an equally hi$h 2%, level. ?;) 7he correct answer is .2elicobacter !ylori !lays a ma<or role in the !atho$enesis of !e!tic ulcer "isease. 7he or$anism is !resent in I/= to :>>= of !atients with "uo"enal ulcers an" in D/= to @/= of those with $astric ulcers. (ra"icatin$ the or$anism $enerally results in a cure for the

"isease. 7hera!y varies, but one of the more common re$imens consists of antibiotics an" a !roton4!um! inhibitor. Oral amo*icillin 'choice A) an" oral metroni"a#ole 'choice C) are !ossible antibiotics use" in combination thera!y. 7hey are not efficacious when $iven without the other a$ents in the combination. 7he same is true for oral ome!ra#ole 'choice %). 7his !roton4!um! inhibitor is not efficacious in era"icatin$ the or$anism when it is $iven without antibiotics. Oral sucralfate 'choice () has no role in thera!y of 2. !ylori infection. 7his "ru$ coats !re4e*istin$ $astric erosions to !revent worsenin$ of ulcers, not to !revent aci" secretion. ?0) (*!lanation: 7he correct answer is C. 2y!othyroi"ism is the main com!lication of ra"ioactive io"ine thera!y, affectin$ u! to D>= of !atients in :> years. 6a"ioactive io"ine thera!y is a safe an" effective treatment for .ravesA "isease because it can !rovi"e the same ablative effects of sur$ery without the sur$ical com!lications. 7here is no evi"ence that this treatment increases the risk for carcinoma 'choice (). Cholestasis 'choice A) an" $ranulocyto!enia 'choice ) are si"e effects of lon$4term antithyroi" thera!y '!ro!ylthiouracil). 6ecurrent laryn$eal nerve "ama$e 'choice %) is a com!lication of subtotal thyroi"ectomy. )ubtotal thyroi"ectomies !rovi"e ra!i" control of the "isease but can lea" to nerve "ama$e, hemorrha$e, hy!othyroi"ism, an" hy!o!arathyroi"ism. ??) (*!lanation: 7he correct answer is . +easurement of serum levels of !rostate4s!ecific anti$en '&)A) can be use" to screen for !rostate cancer. 2owever, this substance can also be elevate" in !rostate hy!ertro!hy. A transrectal ultrasoun" can i"entify lesions not !al!able on rectal e*amination, an" the area can be bio!sie" un"er ultrasoun" $ui"ance. +etastatic !rostate cancer can be treate" with leu!roli"e 'choice A). 7his is a luteini#in$ hormone releasin$ hormone ',2462) a$onist that su!!resses testicular testosterone !ro"uction. 7his is equivalent to orchiectomy or estro$en thera!y. &atients with !rostate cancer will un"er$o a metastatic work4u! as well, inclu"in$ a bone scan 'choice C) to rule out bony metastases. 2owever, the "ia$nosis of cancer must first be ma"e.

6e!eat &)A analysis woul" be hel!ful in followin$ res!onse to thera!y 'choice %). 2owever, the &)A is hi$h enou$h to warrant imme"iate work4u!. Ima$in$ of the !elvis an" retro!eritoneum 'choice () woul" be !art of the stan"ar" metastatic work4u! as well. ?/) (*!lanation: 7he correct answer is (. 7his is tularemia, the causative or$anism of which is -rancisella tularensis. 7he classic clue in test questions is e*!osure to wil" rabbits, althou$h wil" ro"ents an" their arthro!o" vectors may also carry the "isease. 9ou shoul" also be aware that this hi$hly infectious or$anism shoul" not be isolate" e*ce!t in s!ecial !rotective hoo"s. 7he "escri!tion in the question stem is ty!ical of the ulcero$lan"ular formG less common forms inclu"e "isease resemblin$ ty!hoi" fever, an oculo$lan"ular form secon"ary to eye inoculation, an" a $lan"ular form in which the initial site of infection is not obvious. 1ery severe cases may "evelo! "isseminate" necrotic lesions of various si#es throu$hout the bo"y. A$$lutination tests can confirm the "ia$nosis after about the :>th "ay of illness. Fntreate" cases ten" to last 0 to ? weeks before resolvin$. )tre!tomycin is the antibiotic of choiceG $entamicin an" chloram!henicol can alternatively be use". %eaths occur in about C= of untreate" cases an" are very rare in treate" cases. Actinomycosis 'choice A) causes multi!le "rainin$ sinuses. rucellosis 'choice ) causes recurrent fevers. +elioi"osis 'choice C) causes lun$ an" "isseminate" infection, usually followin$ contamination of woun"s by infecte" soil or water. &la$ue 'choice %) causes lym!h no"e an" lun$ involvement after e*!osure to infecte" ro"ents an" their !arasites. ?C) (*!lanation: 7he correct answer is %. 7he ti"al volume for a !atient is $enerally estimate" as :> m,Bk$ of wei$ht, which for this !atient woul" be @/> m,Bbreath. .ivin$ a lower ti"al volume will yiel" hy!oventilation an" be insufficient to eliminate !CO;. &rovi"in$ a ti"al volume $reater than :> m,Bk$ increases the risk of !neumothora*, !articularly in a !atient with lon$stan"in$ em!hysema who may have thin4walle" alveoli. A low ti"al volume with risk of hy!oventilation woul" be !ro"uce" by choice A '/>> m,Bbreath), choice 'C>> m,Bbreath), an" choice C 'D>> m,Bbreath).

A hi$h ti"al volume with risk of !neumothora* woul" be !ro"uce" by choice % ':>>> m,Bbreath). ?D) (*!lanation: 7he correct answer is . 7he current recommen"ations for routine, a$e4a!!ro!riate screenin$ are base", in some measure, on "ata from clinical trials. %e!en"in$ on the source of the recommen"ations, there is consi"erable variability in these recommen"ations. One of the more a$ree" on recommen"ations is that, at least every / years, a ran"om cholesterol level shoul" be checke". A fastin$ li!i" !rofile 'choice A) is usually obtaine" only after a screenin$ cholesterol is shown to be $reater than ;?> m$B",. An oral $lucose tolerance test 'O.77) 'choice C) is $iven to !re$nant women to screen for $estational "iabetes. 7here is no current recommen"ation for usin$ O.77 in routine screenin$ !ractice in any a$e $rou!. 7he inci"ence of !rostate cancer is a$e4relate" an" becomes reasonably !revalent after a$e />. 7herefore, !rostate e*aminations 'choice %) are recommen"e" annually after a$e />. ,ike !rostate cancer, colon cancer is also a$e4relate" an" be$ins to have si$nificant inci"ence after the /th "eca"e. )i$moi"osco!y 'choice () is in"icate" every 04/ years after a$e /> to monitor for lesions u! to the s!lenic fle*ure. Colonosco!y is necessary to survey the entire colon. ?@) (*!lanation: 7he correct answer is %. Allo$eneic bone marrow trans!lantation has become a frequent thera!eutic a!!roach to a variety of con"itions, inclu"in$ leukemic "iseases. 7he !atient un"er$oin$ bone marrow trans!lantation is !rofoun"ly immunosu!!resse" an" !rone to "evelo!in$ o!!ortunistic infections. 7he clinical !icture "escribe" in this case is consistent with $raft versus host "isease '.12%), in which 7 cells 'both hel!er an" su!!ressor cells) of the en$rafte" marrow react a$ainst the reci!ientAs anti$ens, thus tri$$erin$ inflammation an" in<ury to the host tissues. 7he most severely affecte" or$ans inclu"e the immune system, $astrointestinal tract, liver, skin, an" lun$s. 7his com!lication may be acute 'this case) or chronic. C+1 !neumonia is a frequent fatal com!lication in the acute sta$e. 7he chronic sta$e is characteri#e" by !ro$ressive fibrosis of affecte" or$ans. 4lym!hocytes of a bone marrow $raft 'choice A) "o not !lay a si$nificant role in .12 %.

,eukemic cells 'choice ) may $ive rise to recurrence of the ori$inal "isease, which must be "istin$uishe" from .12 %. 7he combination of skin rash an" o!!ortunistic infections stron$ly favor .12 %. In a""ition, sin$le cell necrosis in the e!ithelia of skin, .I tract, an" liver is hi$hly characteristic of .12 %. Natural killer cells of the reci!ient 'choice C) an" 74lym!hocytes of the reci!ient 'choice () !lay a crucial role in me"iatin$ re<ection of allo$eneic marrow trans!lants. ?I) 7he correct answer is . 7his is ichthyosis vul$aris, which is the most frequent form of inherite" ichthyosis. Ichthyosis vul$aris has autosomal "ominant inheritance with a frequency of ::0>> in the $eneral !o!ulation. 7he con"ition usually be$ins in chil"hoo" an" has the features illustrate" in the question stem. )kin care shoul" involve minimi#in$ bathin$ with use of soa!s only in the intertri$inous areas. athin$ limite" to :>4minute !erio"s 'to hy"rate the stratum corneum), followe" by imme"iate a!!lication of an emollient such as !etrolatum, can hel! to control the scalin$. In a""ition, />= !ro!ylene $lycol in water un"er occlusion by thin !lastic film or ba$s "urin$ the ni$ht is hel!ful in a"ults, but is not usually use" in chil"ren. (!i"ermolytic hy!erkeratosis 'choice A) is a rare, autosomal "ominant form of ichthyosis that is !resent from birth an" is characteri#e" by thick, warty skin all over the bo"y, most !rominently in fle*ural creases. ,amellar ichthyosis 'choice C) is a rare, autosomal recessive form of ichthyosis that is !resent from birth an" causes lar$e, coarse scale over most of the bo"y. E4linke" ichthyosis 'choice %) is a relatively common form of ichthyosis that can !resent at birth or in chil"hoo" an" usually causes lar$e, "ark scales with a !re"ilection for the neck an" trunkG the !alms an" soles are normal. Eero"erma 'choice () is a mil", acquire" form of "ry skin, sometimes with crackin$ or mil" scalin$.

/>.7he correct answer is C. 7his !atient has classic sym!toms of mechanical "ys!ha$ia, as she has "ifficulty with lar$e soli" foo" but not softer foo"s or liqui"s. +echanical "ys!ha$ia frequently follows many years of heartburn an" is often in"icative of a !e!tic stricture that has "evelo!e" as a result of fibrosis after a lon$ !erio" of chronic inflammation "ue to $astroeso!ha$eal reflu* "isease '.(6%). 7hese beni$n strictures can usually be "ilate" en"osco!ically. An intensive re$imen of !roton4!um! inhibitors shoul" then be institute" to re"uce the frequency of recurrence. Althou$h chronic aci" reflu* can !re"is!ose for arrettAs eso!ha$us 'choice ) an" then subsequently a"enocarcinoma 'choice A), arrettAs eso!ha$us is a mucosal chan$e only that woul" not cause lumenal narrowin$. -urthermore, a"enocarcinoma woul" be very unusual in a !atient this youn$. )chat#ki rin$ 'choice %) is unlikely, since it ty!ically !ro"uces e!iso"ic mechanical "ys!ha$ia rather than the !ro$ressive mechanical "ys!ha$ia "escribe" in this question. )quamous carcinoma 'choice () in the mi"4thir" of the eso!ha$us can !ro"uce mechanical "ys!ha$ia. 2owever, this !atient is far youn$er than the usual !atient with squamous carcinoma, an" she has no risk factors, such as smokin$, "rinkin$, lye in$estion, or u!!er eso!ha$eal web '&lummer41inson syn"rome).

Block 2

1.A &1!year!ol" homose.#al man presents complaining of pain 'ith "efecation. /e "enies any symptoms of "iarrhea% a$"ominal pain% or fe)ers. Si. months earlier% he "e)elope" tra)eler,s "iarrhea 'hile )acationing in Me.ico. >n physical e.amination% he is afe$rile an" has an #nremarka$le a$"ominal e.amination. >n e.amination of the perianal area% there is a gro#p of fi)e cl#stere" #lcers a"9acent to the anal orifice an" e.ten"ing into the anal canal. A sigmoi"oscopy re)eals normal rectosigmoi" m#cosa. =hich of the follo'ing is the most likely "iagnosisA)Cytome$alovirus infection )2er!es infection C)Neisseria $onorrhea %))hi$ella "ysenteriae ()Flcerative colitis 2. An el"erly "ia$etic cons#lts a physician $eca#se of se)ere an" persistent earache. >toscopic e.amination "emonstrates fo#l!smelling p#r#lent otorrhea an" a re" mass lesion of the e.ternal ear canal. Biopsy of the mass "emonstrates gran#lation tiss#e rather than t#mor. =hich of the follo'ing is the most likely ca#sati)e organisma)(scherichia coli b)2aemo!hilus influen#ae c)&roteus vul$aris ")&seu"omonas aeru$inosa e))ta!hylococcus aureus

0.A (4!year!ol" 'oman presents complaining of )ery se)ere a$"ominal pain% 'hich $egan a$r#ptly 1 ho#rs ago. She "escri$es the pain as 0the 'orst +,)e e)er ha".0 >n *#estioning% she is #na$le to gi)e a precise location $#t in"icates that her entire mi"!a$"omen is e.tremely painf#l. She has $een follo'e" for the past 14 years for symptoms of congesti)e heart fail#re after she ha" an anterior 'all myocar"ial infarction. She has remaine" relati)ely 'ell controlle" 'ith only occasional "yspnea on e.ertion. /er me"ications incl#"e captopril% f#rosemi"e% "igo.in% isosor$i"e "initrate% an" aspirin. She has not ha" any prior s#rgery. >n physical e.amination% she appears e.tremely #ncomforta$le. /er temperat#re is &1.8 2 3141.8 56% $loo" press#re is 1(47142 mm /g% an" p#lse is 1117min an" irreg#larly irreg#lar. >n car"iac e.amination% there is a reg#lar heart rhythm 'ith a ++7?+ holosystolic m#rm#r hear" $est at the ape. an" ra"iating to the a.illa. She has an irreg#larly irreg#lar S1 an" S2% an" scattere" $i$asilar rales. An a$"ominal e.amination re)eals mil" "istention an" no hepatosplenomegaly. The a$"omen is "iff#sely soft $#t )ery ten"er to palpation. A rectal e.amination re)eals $ro'n% g#aiac!positi)e stool. She has no a#"i$le $o'el so#n"s. =hich of the follo'ing is the most likely "iagnosisa) %iverticulitis b)Ischemic colitis c) +esenteric ischemia ")&ancreatitis e))mall bowel obstruction Normal ,abs 4. A 22!year!ol" man comes to the emergency "epartment $eca#se of "yspnea% palpitations% an" a hea"ache. These symptoms came on soon after he took trimethoprim!s#lfametho.aDole for a #rinary tract infection. La$oratory st#"ies sho' a normochromic% normocytic anemia. A peripheral $loo" smear re)eals /einD $o"ies. =hich of the follo'ing is the most likely ca#se of this patient,s anemiaa6 Lea" poisoning $65olate "eficiency c6Cl#cose!<!phosphate "ehy"rogenase "eficiency "6/ere"itary spherocytosis

e6 >cc#lt $loo" loss :ormal La$s .A 4!year!ol" o$ese man presents for a ro#tine physical e.amination. /e 'as "iagnose" 'ith type 2 "ia$etes 1 year earlier. /e has $een mo"erately compliant 'ith "ietary preca#tions an" his morning gl#cose has $een persistently $et'een 1 4 an" 244 mg7"L. /e is therefore starte" on glipiDi"e. >ne month later% metformin is a""e" $eca#se of contin#e" poor control. /is other me"ications are propranolol an" nife"ipine for hypertension% an" napro.en% 'hich he $egan appro.imately 2 'eeks ago for se)ere knee pain "#e to osteoarthritis. >n physical e.amination his $loo" press#re is 1 4782 mm /g% an" he has a soft fo#rth heart so#n". The remain"er of the physical e.amination is normal. /is ro#tine electrolytes are checke" an" re)eal a BU: of 28 mg7"L an" a creatinine of 1.1 mg7"LG $oth ha" $een normal 1 year earlier. =hich of his me"ications is most likely responsi$le for the increase in BU: an" creatininea6 ClipiDi"e $6Metformin c6:apro.en "6:ife"ipine e6 Propranolol :ormal La$s <.A pre)io#sly healthy &(!year!ol" 'oman comes to the physician $eca#se of rec#rrent episo"es of "o#$le )ision an" "rooping of her eyeli"s for the last month. S#ch episo"es occ#r 'itho#t apparent reason% last for ho#rs% an" resol)e spontaneo#sly. She also reports occasional hoarseness an" "iffic#lty in s'allo'ing% 'hich also come an" go. ?ital signs an" physical e.amination are normal. =hich of the follo'ing is the most appropriate ne.t step in "iagnosisa6 Bloo"% #rine% an" 2S5 analysis $6MA+ of the hea" c6EEC recor"ing "6Electromyography #n"er repetiti)e stim#lation e6 M#scle $iopsy

:ormal La$s (. @#ring the month of @ecem$er% a mi""le!age" patient 'ith chronically "ry skin "e)elops 'i"esprea" coin!shape" lesions. The lesions $egin as itchy patches of )esicles an" pap#les. These later ooDe ser#m an" cr#st o)er. The lesions are most n#mero#s on the e.tensor s#rfaces of the e.tremities an" on the $#ttocks. The patient says that some of the lesions ha)e appeare" to heal an" then reappear at the same sites. =hich of the follo'ing is the most likely "iagnosisa6 :#mm#lar "ermatitis $6Pompholy. c6Psoriasis "6Se$orrheic "ermatitis e6Stasis "ermatitis :ormal La$s 1.=hich of the follo'ing me"ical inter)entions is an e.ample of a primary pre)enti)e meas#rea6+solation of "isease contacts $6Mammography c6Ao#tine imm#niDation "6Screening for )is#al ac#ity e6 Testing of stool for occ#lt $loo" :ormal La$s 8. A 1(!year!ol" $oy presents 'ith chronic lo' $ack pain for the past 1 months. /e 'as the most promising mem$er of the high school s'im team $#t 'as force" to *#it $eca#se of his $ack pain. The pain $egins fre*#ently at night% ra"iates "o'n the thighs% an" is accompanie" $y prono#nce" stiffness of the l#m$ar spine. /e "enies any gastrointestinal or genital infections. /is temperat#re is &(.4 2 381.< 56. E.amination re)eals mo"erate limitation of $ack motion an" ten"erness of the lo'er spine. A "iastolic m#rm#r along the left sternal $or"er is hear" on chest e.amination. La$oratory in)estigation sho's an ele)ate" erythrocyte se"imentation rate 3ESA6 an" negati)e rhe#matoi" factor. F!ray films of the )erte$ral col#mn an" pel)ic region sho' flattening of the l#m$ar c#r)e an" s#$chon"ral $one erosion in)ol)ing the sacroiliac 9oints. =hich of the follo'ing is the most likely "iagnosisa6 Ankylosing spon"ylitis

$6@egenerati)e 9oin "isease c6 Aeiter syn"rome "6Seronegati)e rhe#matoi" arthritis e6 Still "isease :ormal La$s 14. A <!year!ol" man has $een a"mitte" to the me"ical intensi)e care #nit in respiratory "istress. An en"otracheal t#$e is place" for mechanical )entilation at a ti"al )ol#me of 844 mL% a rate of 12 $reaths7min% an" a fraction of inspire" o.ygen of 4B. The positi)e en" e.piratory press#re is 14 cm of 'ater. Me"ications incl#"e s#$c#taneo#s heparin an" aspirin. /e no' "e)elops tachycar"ia an" a $loo" press#re of (47palpation mm /g. 2ar"iac e.amination re)eals m#ltiple premat#re contractions. /is arterial $loo" gas re)eals a P>2 of 44 mm /g. =hich of the follo'ing is the most likely ca#se of this con"itiona6 2ar"iac arrhythmia $6Bronchial secretions c6Myocar"ial infarction "6Pne#mothora. e6 P#lmonary em$ol#s :ormal La$s 11. A 4 !year!ol" alcoholic man 'ith cirrhosis is transferre" to the intensi)e care #nit after "e)eloping esophageal )arices complicate" $y shock. A screening $attery of tests is or"ere"% re)ealing a total thyro.ine 3T46 of &.1 mg7"L. Physical e.amination of the thyroi" glan" is #nremarka$le. 5ollo'!#p st#"ies sho'e" a total triio"othyronine 3T&6 of &4 ng7"L an" TS/ 3thir"!generation test6 of 4.( m+U7mL. =hich of the follo'ing is the most likely "iagnosis in this patienta6 E#thyroi" sick syn"rome $6Cra)es "isease c6 /ashimoto "isease "6Me"#llary carcinoma of the thyroi" e6 Silent lymphocytic thyroi"itis :ormal La$s

12.A 28!year!ol" man presents to the clinic complaining of generaliDe" fatig#e. The patient is ne' to the clinic an" reports that% o)er the past fe' 'eeks% he has $een feeling m#ch more tire" than #s#al. /e also reports that this happens to him 9#st a$o#t e)ery year% an" that other physicians ha)e tol" him that he is 0o)er'orke".0 /is re)ie' of symptoms is nota$le for fre*#ent sneeDing% post!nasal "rip% eye 'atering% an" a itch of his posterior pharyn.. These symptoms ten" to $e 'orse in the spring an" s#mmer an" ha)e $een $othering him since mi"!April% a$o#t 1 month ago. /is past me"ical history is remarka$le only for mil" asthma in"#ce" $y $eing o#t"oors. /e takes no reg#lar me"ications $#t "oes take "iphenhy"ramine on occasion. /e "enies to$acco% ethanol% or illicit s#$stance #se. =hich of the follo'ing is the most appropriate "iagnostic test at this timea6 Bloo" ra"ioallergosor$ent test $6:one% the "iagnosis is $ase" solely on the history an" physical e.amination c6+ntra"ermal testing "6Ser#m protein electrophoresis e6Skin!prick testing :ormal La$s 1&.A 1!year!ol" man 'ith kno'n hepatitis 2 an" cirrhosis complains of 'orsening fatig#e an" conf#sion o)er the past "ays. /e has $een a"mitte" three times in the past 4 months for )ariceal $lee"ing an" has ha" ascites that has $een refractory to high!"ose oral "i#retic #se. /e also reports that o)er the past 41 ho#rs he has ha" a "eclining #rinary o#tp#t. >n physical e.amination% he is ga#nt an" 9a#n"ice". /e has tense ascites an" a li)er span of ( cm in the mi"cla)ic#lar line. La$oratory res#lts re)eal a 'hite $loo" cell co#nt of 4<447mm&% a hemoglo$in of 8.4 g7"L% an" a hematocrit of 28B. /is electrolytes re)eal a BU: of &4 mg7"L an" a creatinine of &.1 mg7"L. A #rinary so"i#m is less than 14 mE*7L. =hich of the follo'ing is the most appropriate treatment for his ele)ate" BU: an" creatininea6Large )ol#me paracentesis $6/emo"ialysis c6Mesoca)al sh#nt "6Mi"ney transplantation e6Li)er transplantation :ormal La$s

14. A 4!year!ol"% malno#rishe" man is a"mitte" for e)al#ation of 9a#n"ice% ascites% an" ten"erness in the right #pper a$"omen. /is temperat#re is &1 2 3144.4 56. Physical e.amination re)eals mil" hepatomegaly an" splenomegaly% as 'ell as generaliDe" m#scle 'asting. Ultraso#n" e.amination confirms the presence of an enlarge" li)er an" a small amo#nt of ascitic fl#i". La$oratory st#"ies sho'; Ser#m al$#min...........................2. g7"L Clo$#lin.....................................&.1 g7"L Bilir#$in; Total........................................&. mg7"L @irect......................................1.( mg7"L AST.........................................&44 U7L ALT.........................................124 U7L Amylase...................................144 U7L Alkaline phosphatase................144 U7L /ematologic hemoglo$in...........14 g7"L Mean corp#sc#lar )ol#me.........144 Hm Le#kocyte co#nt.......................44447mm& Segmente" ne#trophils..............<<B Ban"s......................................(B Prothrom$in time......................11 sec =hich of the follo'ing is the most likely "iagnosisa6 Ac#te pancreatitis $6Alcoholic hepatitis c62holecystitis "6@#o"enal peptic #lcer e6?iral hepatitis :ormal La$s 1 . A 21!year!ol" man 'ho recently emigrate" from +taly ret#rns to the physician,s office for a follo' #p )isit to e)al#ate his anemia. 5o#r "ays ago% he presente" 'ith fatig#e an" "yspnea on e.ertion% an" 'as fo#n" to ha)e a hematocrit of 22B. At that time he 'as a"mitte" to the local hospital for e)al#ation. La$oratory analysis sho's; /ematocrit........................................2&B Mean corp#sc#lar )ol#me 3M2?6...... 8 Hm& Aetic#locyte co#nt.............................4.&B

Ser#m iron........................................1<4 Hg7"L Total iron $in"ing capacity 3T+B26.....2&4 Hg7mL Ser#m ferritin...................................14 ng7mL /emoglo$in electrophoresis;..............A$sent $eta $an"s =hich of the follo'ing is the most likely "iagnosisa6 Alpha!thalassemia $6Beta!thalassemia c6+ron "eficiency anemia "6Megalo$lastic anemia e6Sickle cell "isease :ormal La$s 1<. A 2 !year!ol" man has ha" type 1 "ia$etes mellit#s for years. /is physician is concerne" a$o#t the possi$ility of permanent renal "amage. =hich of the follo'ing is the $est early in"icator for "ia$etic nephropathya6Al$#min#ria $6/ypertension c6Aising $loo" #rea nitrogen "6Aising creatinine e6 Urinary tract infection :ormal La$s 1(. A 44!year!ol" 'oman presents 'ith complaints of $#rning an" tingling sensations in the left han" for se)eral months. She relates that she has $een fre*#ently a'akene" at night $y aching pain in the same han". She is other'ise in goo" health. E.amination fails to "etect any impairment in sensation% $#t pain is elicite" $y e.treme "orsifle.ion of the 'rist. The patient is #na$le to correctly i"entify "ifferent clothes $y r#$$ing $et'een the left th#m$ an" in"e. finger. =hich of the follo'ing is the most likely "iagnosisa6Angina pectoris $62arpal t#nnel syn"rome c6@#p#ytren contract#re "65i$rositis e6Aefle. sympathetic "ystrophy :ormal La$s

11.A 4!year!ol" man "e)elops a mass on the $ack of his han". The lesion some'hat resem$les a 0)olcano0 an" consists of a ro#n"% firm% flesh colore"% 1!cm no"#le 'ith sharply rising e"ges an" a central crater. Meratotic "e$ris can $e e.presse" from the central crater. The lesion has "e)elope" )ery rapi"ly o)er a$o#t a three!month perio". =hich of the follo'ing is the most likely "iagnosisa6Meratoacanthoma $6Lipoma c6Malignant melanoma "6Pyogenic gran#loma e6 Se$orrheic keratosis :ormal La$s 18. A physician is calle" to see a <8!year!ol" 'oman 'ho #n"er'ent car"iac catheteriDation )ia the right femoral artery earlier in the morning. She is no' complaining of a cool right foot. Upon e.amination she has a p#lsatile mass o)er her right groin 'ith loss of her "istal p#lses% an" a#sc#ltation re)eals a $r#it o)er the point at 'hich the right femoral artery 'as entere". =hich of the follo'ing is the most likely "iagnosisa62holesterol em$oli syn"rome $65emoral ane#rysm c65emoral hernia "65emoral pse#"oane#rysm e6Aetroperitoneal hematoma :ormal La$s 24. A patient complains to a physician of chronic pain an" tingling of the $#ttocks. The pain is e.acer$ate" 'hen the $#ttocks are compresse" $y sitting on a toilet seat or chair for long perio"s. :o l#m$ar pain is note". Pain is elicite" 'hen the physician performs 5rei$erg,s mane#)er% in 'hich there is a forcef#l internal rotation of the e.ten"e" thigh. =hich of the follo'ing is the most likely "iagnosisa6@isk compression of the sciatic ner)e $65i$romyalgia c6Piriformis syn"rome "6Poplite#s ten"initis

e6Posterior femoral m#scle strain :ormal La$s 21. A (4!year!ol" 'oman% 'ho has $een follo'e" for the past 2 years for chronic o$str#cti)e p#lmonary "isease 32>P@6 presents complaining of 41 ho#rs of temperat#res to &1.< 2 3141.4 56 an" 'orsening shortness of $reath. She has a chronic pro"#cti)e co#gh% 'hich has $ecome more copio#s. >n physical e.amination% she has rhonchi an" increase" fremit#s in the posterior mi"!l#ng fiel". A Cram,s stain re)eals many epithelial cells an" m#ltiple gram!positi)e an" gram!negati)e organismsG no ne#trophils are seen. =hich of the follo'ing is the most likely organism ca#sing the symptomsa6Escherichia coli $6/aemophil#s infl#enDae c6Mle$siella pne#moniae "6Myco$acteri#m t#$erc#losis e6Mycoplasma pne#monia :ormal La$s 22. A &!year!ol" man comes to the physician $eca#se of progressi)e 'eakness an" 'eight loss o)er the past 2 months. /e says that he also $egan noticing areas of his skin getting "arker e)en tho#gh it is 'inter an" he is ne)er in the s#n. /e takes no me"ications an" has no other me"ical con"itions. Physical e.amination sho's no a$normalities e.cept for orthostatic hypotension an" hyperpigmentation of his skin. La$oratory st#"ies sho'; So"i#m........................1&4 mE*7L 2hlori"e ......................8 mE*7L Potassi#m...................<. mE*7L Bicar$onate................24 mE*7L Le#kocyte co#nt.............. 4447mm& Segmente" ne#trophils......44B Ban" forms ......................4B Lymphocytes...................44B Monocytes .....................<B Eosinophils......................8. B Basophils........................4. B =hich of the follo'ing is the most likely "iagnosis-

a6A""ison,s "isease $6 2onn,s syn"rome c62#shing,s "isease "62#shing,s syn"rome e6 Syn"rome of inappropriate anti"i#retic hormone secretion :ormal La$s 2&. A & !year!ol" /+?!positi)e man comes to me"ical attention 'ith a <! month history of progressi)e memory loss an" incontinence. /e is taking Di"o)#"ine an" a protease inhi$itor. /e first notice" "iffic#lties 'ith han"'riting. :e#rologic e.amination "emonstrates "eficits in cogniti)e an" fine motor control f#nctions. La$oratory in)estigations sho' a 2@4 cell co#nt of 2 7mm&. MA+ st#"ies re)eal mo"erate $rain atrophy $#t no focal lesions. A l#m$ar p#nct#re sho's no 2S5 a$normalities. =hich of the follo'ing is the most likely "iagnosisa6 2M? encephalitis $62ryptococcal meningoencephalitis c6/+? encephalitis "6 /+? myelopathy e6Primary $rain lymphoma f6Progressi)e m#ltifocal le#koencephalopathy g6To.oplasmosis :ormal La$s 24. A (!year!ol" 'oman presents 'ith progressi)e shortness of $reath o)er the past 2 "ays. The 'oman 'as a"mitte" to the me"ical ser)ice < "ays ago after a fall an" has $een on $e" rest for a non"isplace" p#$ic ram#s fract#re. She has $een on "eep )ein throm$osis prophyla.is 'ith s#$c#taneo#s heparin. /er past me"ical history is significant for type 2 "ia$etes an" "ialysis!"epen"ent renal fail#re secon"ary to "ia$etic nephropathy. She makes no #rine at $aseline. /er last "ialysis r#n 'as 4 "ays ago% tho#gh she #s#ally #n"ergoes "ialysis & times per 'eek. She has no chest pain. >n physical e.amination% she appears an.io#s. /er $loo" press#re is 1<4714 mm /g% p#lse is 1147min% an" respirations are 227min. /er o.ygen sat#ration is 14B on room air% an" she appears cyanotic. She has a 9#g#lar )eno#s press#re of 14 cm an" inspiratory crackles half 'ay #p from the $ases on a#sc#ltation of the l#ngs. An E2C re)eals a rate!relate" right $#n"le $ranch $lock $#t no ischemic changes. A chest .!ray film o$taine" yester"ay re)eale" interstitial

e"ema an" )asc#lar re"istri$#tion to the apices. =hich of the follo'ing is the most appropriate initial therapya6 An anti!hypertensi)e agent to "ecrease her $loo" press#re to normal $6A $eta $locker to $etter control her p#lse c6+? morphine to "ecrease her respirations to normal "6>.ygen $y en"otracheal int#$ation to ma.imiDe the o.ygen concentration e6 >.ygen $y face mask to increase her o.ygen sat#ration :ormal La$s 2 . A &(!year!ol" acco#ntant presents to ask for a")ice regar"ing the f#t#re management of his #lcerati)e colitis. /e has ha" pancolitis for the past 18 years an" has $een tol" that he is at an increase" risk for "e)eloping colorectal cancer. /e asks for the physician,s recommen"ation regar"ing appropriate s#r)eillance. =hich of the follo'ing is the most appropriate responsea6Ann#al stool g#aiac testing $6Bari#m enema c62olonoscopy "62olonoscopy an" m#ltiple $iopsies e65le.i$le sigmoi"oscopy 'ith m#ltiple $iopsies :ormal La$s 2<. A (4!year!ol" 'oman has $een in long!stan"ing poor health% 'ith se)ere "ia$etes mellit#s an" rhe#matoi" arthritis. /er physician notes that she appears pale an" or"ers a hematocrit% 'hich sho's a res#lt of & B. E.amination of the $loo" smear re)eals a microcytic anemia. The physician is consi"ering a "ifferential "iagnosis of iron "eficiency anemia )ers#s anemia of chronic "isease. =hich of the follo'ing la$oratory "eterminations 'o#l" $e most helpf#l in "isting#ishing these con"itionsa6Erythrocyte;gran#locyte ratio in $one marro' $6 Presence or a$sence of polychromatophilic target cells c6Presence or a$sence of stipple" erythrocytes "6Ser#m ferritin e6Ser#m iron :ormal La$s

2(. A 4 !year!ol" patient on hemo"ialysis for one 'eek has note" that his $loo" press#re is more "iffic#lt to control. /e reports goo" compliance 'ith his me"ications% 'hich incl#"e erythropoietin% ferro#s s#lfate% )ancomycin% an" )itamin @. /is $loo" press#re is 114788 mm /g. =hich of the follo'ing is the most likely ca#se for the 'orsening control of his $loo" press#rea6Erythropoietin $65erro#s s#lfate c6?ancomycin "6?itamin @ e6 Uremia :ormal La$s 21. A 44!year!ol" man is $ro#ght to the emergency room $y his frien"s. Apparently% he has ingeste" some #nkno'n me"ication in a s#ici"e attempt. The patient is "isoriente" to time. /is temperat#re is &8.& 2 314& 56% $loo" press#re is 12471 mm /g% p#lse is 1447min an" irreg#lar% an" respirations are 227min. The skin is fl#she" an" "ry. @ilate" p#pils an" m#scle t'itching are also note" on physical e.amination. E2C re)eals prolonge" NAS comple.es. /epatic transaminases are normal% an" $loo" gas analysis sho's a normal p/. These fin"ings are most likely "#e to into.ication $y 'hich of the follo'ing s#$stancesa6Acetaminophen $6Alcohol c6BenDo"iaDepines "62loni"ine e6 Monoamine o.i"ase 3MA>6 inhi$itors f6Tricyclic anti"epressants :ormal La$s 28.A (2!year!ol" man comes to the physician $eca#se of a &!"ay history of right!si"e" chest pain. /e "enies any shortness of $reath% na#sea or )omiting. Physical e.amination sho's a #nilateral% erythemato#s% mac#lopap#lar rash e.ten"ing from the anterior chest 'all aro#n" to the $ack in a "ermatomal pattern. The remain"er of the e.amination is normal. +n con)ersation% he states that is he is going to )isit his

gran"chil"ren ne.t 'eek an" that their mother 0"oesn,t $elie)e in imm#niDations0. /is gran"chil"ren are at increase" risk for 'hich of the follo'ing rashesa6 @iscrete mac#lopap#lar lesions that $ecome confl#ent as they sprea" from 0hea" to toe0 $6@ome!shape" pap#les 'ith central #m$ilication c6E.pan"ing ann#lar lesion 'ith central clearing "6 0Slappe"!cheek0 appearance an" a lacy retic#lar rash e6 ?esicles at )ario#s stages of e)ol#tion :ormal La$s &4. A 4!year!ol" man cons#lts a physician $eca#se he has $een ha)ing transient perio"s of rapi" heart $eat accompanie" $y s'eating% fl#shing% an" a sense of impen"ing "oom. Physical e.amination is #nre)ealing% 'ith no e)i"ence of arrhythmia at the time of the e.am. /o'e)er% the man,s 'ife is a n#rse% so the physician asks that she take )ital signs the ne.t time one of the episo"es occ#rs. She "oes% an" "emonstrates a $loo" press#re of 18 7144 mm /g 'ith heart rate 1<47min "#ring the episo"e. She promptly takes her h#s$an" to the emergency room% $#t the spell is o)er $y the time that he is seen. Urinary meas#rement of 'hich of the follo'ing 'o#l" most likely $e "iagnostic in this casea6 @ehy"roepian"rosterone 3@/EA6 $6/#man chorionic gona"otropin 3h2C6 c61(!ketosteroi"s "6?anillylman"elic aci" ?MA e6 Oinc protoporphyrin :ormal La$s &1. A & !year!ol" 'oman cons#lts an ophthalmologist $eca#se of "o#$le )ision an" "roopy eyeli"s. She also has complaints of generaliDe" m#scle 'eakness. +? in9ection of e"rophoni#m "ramatically% $#t only $riefly% re)erses her symptoms. This patient,s pro$a$le "isease has a pathophysiologic $asis that is closest to that of 'hich of the follo'ing con"itionsa6 B#llo#s pemphigoi" $6@ia$etes mellit#s type 1 3some cases6 c6+"iopathic A""ison "isease

"6+ns#lin resistance e6Systemic l#p#s erythematos#s :ormal La$s &2.A &4!year!ol" 'oman 'ho is healthy 'itho#t #n"erlying me"ical pro$lems presents to clinic 'ith complaints of temperat#re #p to 141 5 an" co#gh 'ith greenish sp#t#m pro"#ction for 2 "ays 'itho#t any "yspnea. /er heart rate is 117min% an" her respiratory rate is 117min. There is no accessory m#scle #se or con)ersational "yspnea% nor are there 'heeDes% $ronchial $reath so#n"s% rales% or egophony o)er the right lo'er l#ng fiel"s. 2hest .!ray film re)eals a right lo'er lo$e consoli"ation. A 2B2 sho's a le#kocyte co#nt of 1&%4447mm&. =hich of the follo'ing is the most appropriate pharmacotherapya6 Amo.icillin $6Ampicillin!s#l$actam c62eftria.one "6Erythromycin e6 Erythromycin pl#s ceftria.one :ormal La$s &&. A 2&!year!ol" type 1 "ia$etic is $ro#ght to the emergency "epartment after $eing fo#n" in a coma. The scent of acetone is present on the patient,s $reath. Urinary catheteriDation 'ith s#$se*#ent "ipstick analysis "emonstrates marke" positi)ity for gl#cose an" ketones. Stat $loo" chemistries 'o#l" most likely sho' 'hich of the follo'ing )al#es for the anion gapa6 < mE*7L $611 mE*7L c61& mE*7L "61 mE*7L e6 24 mE*7L :ormal La$s &4. A (4!year!ol" man presents to the emergency "epartment 'ith a &! "ay history of right temporal hea"ache% fe)er% an" profo#n" malaise.

/e appears ac#tely ill. /is temperat#re is &8. 2 314&.1 56% $loo" press#re is 1&4714 mm /g% p#lse is 817min% an" respirations are 247min. Ten"erness o)er the right temporal region is appreciate" on palpation. The right temporal artery is ten"er an" slightly no"#lar. :e#rologic e.amination is normal% incl#"ing f#n"#scopic e.amination. /o'e)er% )is#al ac#ity is re"#ce". La$oratory st#"ies sho'; /ematocrit &8.4B /emoglo$in 14.8 g7"L Le#kocytes 11447HL 3ne#trophils <1B6 Erythrocyte se"imentation rate 14 mm7hr =hich of the follo'ing is the most appropriate ne.t step in managementa6Meas#rement of intraoc#lar press#re $6?is#al fiel" assessment c6Lo'!"ose 314 mg7"ay6 pre"nisone treatment "6/igh!"ose 3<4 mg7"ay6 pre"nisone treatment e6 Temporal artery $iopsy :ormal La$s & .A (4!year!ol" 'oman presents to her physician for a postoperati)e me"ical )isit. Three "ays ago% she #n"er'ent a left total knee replacement for se)ere osteoarthritis. She has a past me"ical history significant for type 1 "ia$etes mellit#s an" gla#coma. /er hospital co#rse 'as #ne)entf#l. She contin#es to take "aily :P/ ins#lin an" has goo" control of her $loo" gl#cose. She also takes o.yco"one% 'hich 'as gi)en to her in the hospital for pain. She is in)ol)e" in a physical therapy reha$ilitation program at the local hospital. >n re)ie' of her me"ications% 'hich of the follo'ing is most ac#tely in"icate" at this timea6 An A2E inhi$itor $6A nonsteroi"al anti!inflammatory agent c6>ral aspirin "6>ral 2o#ma"in e6 S#$c#taneo#s #nfractionate" heparin :ormal La$s

&<. A &(!year!ol" 'oman presents 'ith complaints of se)ere heart$#rn 'ith or 'itho#t meals. She has a history of hypertension% 'hich has $een treate" 'ith captopril. She also has a history of Aayna#" "isease% m#ltiple facial telangiectasias% an" )ery ta#t skin on the "ors#m of $oth han"s. She has faile" to o$tain relief for her heart$#rn 'ith large "oses of antaci"s% raniti"ine% or omepraDole. Esophageal manometry is or"ere". =hich of the follo'ing 'o#l" $e the most likely res#lts of this testa6 @ecrease" esophageal peristalsis an" "ecrease" LES press#re $6@ecrease" esophageal peristalsis an" increase" LES press#re c6+ncrease" esophageal peristalsis an" "ecrease" LES press#re "6+ncrease" esophageal peristalsis an" increase" LES press#re e6 :ormal esophageal peristalsis an" normal LES press#re :ormal La$s &(. A <4!year!ol" 'oman cons#lts a physician $eca#se of 'eakness% hea"aches% "iDDiness% an" tingling in her han"s an" feet. Physical e.amination "emonstrates m#ltiple areas of $r#ising on the $ack of her forearms an" shins. >n specific *#estioning% she reports ha)ing ha" fi)e nose$lee"s in the past t'o months% 'hich she ha" attri$#te" to 0"ry air0. Bloo" st#"ies are "ra'n 'hich sho' a platelet co#nt of 1.2 L 14<7HL% a re" cell co#nt of .1 L14<7HL% an" a 'hite co#nt of 14% 447HL 'ith a normal "ifferential co#nt. Ae)ie' of the peripheral smear "emonstrates many a$normally large platelets% platelet aggregates% an" megakaryocyte fragments. :o a$normal re" or 'hite $loo" cells are seen. Phila"elphia chromosome st#"ies are negati)e. =hich of the follo'ing is the most likely "iagnosisa62hronic myelogeno#s le#kemia $6Myelofi$rosis c6Polycythemia )era "6Primary throm$ocythemia e6 Secon"ary throm$ocythemia :ormal La$s &1. A 2&!year ol"!"ancer presents 'ith a chief complaint of 'eakness. She "enies any other symptoms% incl#"ing na#sea or )omiting. She "enies "iarrhea. /er $loo" press#re is 14744 mm /g. There is no e"ema an" the l#ngs are clear. La$oratory analysis of ser#m sho's;

So"i#m 12< mE*7L Potassi#m 2.2 mE*7L Bicar$onate 28 mE*7L Magnesi#m 2.4 mg7"L 2alci#m 8.4 mg7"L The most likely ca#se of the patient,s 'eakness is an a$normality in 'hich of the follo'inga6 Bicar$onate $62alci#m c6Magnesi#m "6 Potassi#m e6 So"i#m :ormal La$s

&8. A 4&!year!ol" 'oman is a"mitte" for ne'!onset of seiD#res in the setting of hyponatremia. At $aseline% she is 'ell e"#cate" an" 'orks as a comp#ter marketer. /er me"ical history is remarka$le for a long history of "epression an" alcoholism% 'ith m#ltiple )isits to the Emergency @epartment for tra#ma. She 'as initially fo#n" in her hot apartment $y parame"ics. At that time% she 'as postictal% incontinent of #rine% an" oriente" only to name. She 'as last seen at 'ork & "ays ago. +n the Emergency @epartment her systolic $loo" press#re is (4 mm /g an" her p#lse is 1&47min. Upon physical e.amination% she has "ry m#co#s mem$ranes% a 9#g#lar )eno#s press#re of less than cm% an" "iff#se ecchymoses on her face% $o"y% an" $reasts. She procee"s to ha)e t'o a""ition seiD#res in the Emergency @epartment that are controlle" 'ith intra)eno#s loraDepam. La$oratory st#"ies re)eal a ser#m so"i#m of 11 mE*7L% potassi#m of 2.1 mE*7L% an" $icar$onate of &2 mE*7L. =hich of the follo'ing is the most appropriate ne.t test to o$taina6Electroencephalogram 3EEC6 $6Magnetic resonance imaging 3MA+6 of the hea" c6 :on!contrast comp#te" tomography 32T6 of the hea" "6F!ray films of the sk#ll e6L#m$ar p#nct#re 3LP6 :ormal La$s

44.A < !year!ol" =est Te.as farmer of S'e"ish ancestry has an in"olent% pale% raise"% 'a.y% 1.2!cm skin mass o)er the $ri"ge of the nose. The mass has $een slo'ly gro'ing o)er the past & years. There are no enlarge" lymph no"es in the neck. >ther than a 0'eather!$eaten0 appearance for the rest of his e.pose" skin% the remain"er of the physical e.amination is #nremarka$le. =hich of the follo'ing is the most likely "iagnosisa6Basal cell carcinoma $6+n)asi)e melanoma c6Meratoacanthoma "6Pyogenic gran#loma e6S*#amo#s cell carcinoma :ormal La$s 41.A 22!year!ol" 'oman goes to the emergency "epartment $eca#se she feels )ery 'eak an" is ha)ing m#scle cramping an" fascic#lations. Bloo" chemistry st#"ies "emonstrate a plasma potassi#m of 1. mE*7L. >n *#estioning% she a"mits to chronic #se of la.ati)es an" "i#retics to control her 'eight. =hich of the follo'ing E2C changes 'o#l" $e most characteristic of changes relate" to her ME le)ela6+ncrease" U 'a)e amplit#"e $6Prolongation of the P 'a)e c6Shortening of the NT inter)al "6Tall% symmetric% peake" T 'a)es e6=i"ening of the NAS comple. :ormal La$s 42. A &4!year!ol" man presents 'ith a s'ollen left knee of 2 "ays, "#ration. /e "enies any kno'n tra#ma to that region an" has no prior history of any m#sc#loskeletal complaints. /e is in other'ise e.cellent health. /e is homose.#al an" practices safe se. 'ith a single partner. >n physical e.amination% his knee is s'ollen% ten"er to palpation% an" erythemato#s an" has a limite" range of motion. An arthrocentesis is performe". =hich of the follo'ing is most s#ggesti)e of a septic arthritis in this patienta6A complete $loo" cell co#nt 'ith 14%&44 'hite $loo" cells per mL

$6A 9oint fl#i" aspirate 'ith a 'hite $loo" cell co#nt of 21%444 per mL c6A 9oint fl#i" aspirate 'ith a 'hite $loo" cell co#nt of &<%444 per mL "6A 9oint fl#i" aspirate 'ith a 'hite $loo" cell co#nt of 41%444 per mL e6 A 9oint fl#i" aspirate 'ith a 'hite $loo" cell co#nt of 8&%444 per mL :ormal La$s 4&. A 8!year!ol" man presents to the hospital complaining of co#gh. The patient "escri$es a co#gh that has progressi)ely 'orsene" o)er the past & "ays% $ecoming more pro"#cti)e of yello'ish sp#t#m. /e also reports one episo"e of shaking chills 2 "ays ago. /is past me"ical history is remarka$le for rhe#matoi" arthritis% for 'hich he takes a nonsteroi"al agent for pain control. >n e.amination% his $loo" press#re is 144784 mm /g% an" his p#lse is 1447min. /e has coarse $reath so#n"s o)er his right $ase an" a normal car"iac e.amination. =hich of the follo'ing is the most appropriate "iagnostic test for this patienta6/igh!resol#tion chest 2T $6Positron emission tomography 3PET6 scan of the l#ngs c6MA+ of the chest "6?entilation!perf#sion scan e6F!ray films of the chest% posterior!anterior 3PA6 an" lateral )ie's :ormal La$s 44. A 2!year!ol" 'oman is seen $y in clinic for a")ice on osteoporosis. She has $een a patient there for a n#m$er of years. She has a past me"ical history significant for hypertension an" "iet!controlle" "ia$etes mellit#s. She smokes 1 pack of cigarettes per "ay. She 'as "oc#mente" $y pre)io#s L/ an" 5S/ le)els to $e in menopa#se 'ithin the last year. She is concerne" a$o#t 0$reaking her hip 'hen +,m ol"0 an" she is seeking a")ice on osteoporosis pre)ention. She sho#l" $e tol" that in"epen"ent of si"e effects% the $est therapy c#rrently a)aila$le for pre)ention is 'hich of the follo'inga6Bisphosphonates $6 2alcitonin c62alci#m an" )itamin @ "6 2on9#gate" estrogens e6So"i#m fl#ori"e :ormal La$s

4 .A 22!year!ol" man comes to the emergency "epartment 'ith a &!"ay history of fe)er% chills% a co#gh% ple#ritic chest pain% an" lo'!$ack pain. /e says that the symptoms came on 0o#t of the $l#e0. /e is the son of a 'ealthy local $#siness'oman an" still li)es at home% 'hich he says 0is cool $eca#se my parents are ne)er aro#n"0. /is temperat#re is &8 2 3142.2 56% $loo" press#re is 124714 mm /g% p#lse is (47min% an" respirations are 1<7min. Physical e.amination sho's o)al% retinal hemorrhages 'ith a clear% pale center an" pinpoint lesions $et'een his toes. Bloo" c#lt#res are "ra'n. A chest .!ray film sho's m#ltiple patchy infiltrates. La$oratory st#"ies sho'; /emoglo$in...................................11 g7"L /ematocrit....................................&8B Erythrocyte se"imentation rate.......&8 mm7hr =hich of the follo'ing is the most likely pathogena62an"i"a al$icans $6Pse#"omonas aer#ginosa c6Serratia marcescens "6Staphylococc#s a#re#s e6Streptococc#s )iri"ans :ormal La$s 4<. A (!year!ol" man comes to his physician for his semi!ann#al )isit. /e has a me"ical history significant for long!stan"ing chronic o$str#cti)e p#lmonary "isease 32>P@6. /e has ha" a t'o to three pack per "ay smoking history for the past 44 years. /e also has hypertension an" "iet!controlle" type 2 "ia$etes mellit#s. /is me"ications incl#"e lisinopril once "aily an" thiaDi"e. /e has no allergies. /e reports that he contin#es to smoke one to t'o packs of cigarettes per "ay an" "rinks one glass of 'hisky each night. /e seems to $e compliant 'ith his me"ications. /e 'alks one half mile per "ay at a fairly $risk pace $#t is limite" $y fatig#e an" shortness of $reath. /is home $loo" gl#cose log sho's a range of )al#es from 141 to 241 mg7"L. >n physical e.amination% he is a fairly o$ese man 'ith a large $arrel chest. /e is $reathing comforta$ly. /is $loo" press#re is 1 2711 mm /g% an" p#lse is 127min an" reg#lar.

/is l#ngs are hyperresonant to perc#ssion 'ith scant $i$asilar crackles. /e has an S4 gallop an" a gra"e one systolic e9ection m#rm#r ra"iating to the caroti"s $ilaterally. /is e.tremities are 'itho#t e"ema or cl#$$ing. =hich of the follo'ing is the most appropriate pre)entati)e meas#re in this patienta6A"" an oral gl#cose control agent $6Enco#rage a""itional e.ercise c6Enco#rage rapi" cessation of alcohol #se "6Enco#rage rapi" cessation of to$acco #se e6+ncrease his "ose of thiaDi"e :ormal La$s 4(. A &!year!ol" 'oman presents complaining of fatig#e o)er the past < months. @#ring this time% she has also "e)elope" pr#rit#s an" lost 4 po#n"s. She is not se.#ally acti)e% an" her past me"ical history is significant only for S9Pgren syn"rome. >n physical e.amination% she is afe$rile an" has mil"ly icteric sclera. There are e.coriations note" on all fo#r e.tremities an" tr#nk an" $ack. The li)er e"ge is smooth an" non!ten"er an" meas#res 8 cm at the mi"cla)ic#lar line. There is no ascites% splenomegaly% or peripheral e"ema. La$oratory res#lts re)eal a normal complete $loo" co#nt% normal electrolytes% an" li)er f#nction tests 'ith an alkaline phosphatase of 2<4 U7L 3normal% Q114 U7L6% total $ilir#$in of &.1 mg7"L% an" normal transaminase le)els. =hich of the follo'ing is the most likely "iagnosisa6 Ac#te cholecystitis $6Ac#te hepatitis A infection c6Bacterial cholangitis "6Primary $iliary cirrhosis e6Primary sclerosing cholangitis :ormal La$s 41. An el"erly 'oman cons#lts a physician $eca#se she is 0feeling so tire" all the time0. +ntraoffice hematocrit is & B. Peripheral $loo" smear sho's many macrocytic re" cells. >n *#estioning% the 'oman%

'hose finances are limite"% a"mits that has $een li)ing on a 0tea an" toast0 type "iet. She has $een "rinking a po'"ere" orange 9#ice s#$stit#te 3Tang6. She has not $een taking )itamin pills $eca#se she feels she can,t affor" them. A n#tritional "eficiency of 'hich of the follo'ing is the most likely ca#se of this patient,s anemiaa6 5olate $6+ron c6?itamin B12 "6?itamin 2 e6?itamin M :ormal La$s 48. A < !year!ol" 'oman is a"mitte" to the hospital for constant% se)ere a$"ominal pain that has 'orsene" o)er the prior 'eek. She has no other associate" symptoms% s#ch as na#sea or )omiting% $#t has notice" that her "aily #rine o#tp#t has sharply "ecrease". She has ha" a constant "esire to #rinate% $#t% 'hen she tries% only a small amo#nt of $loo"y #rine is "ischarge". The patient is a long!time smoker% ha)ing smoke" three packs per "ay for more than 4 years% altho#gh she claims to ha)e *#it 2 "ays ago. A $la""er #ltraso#n" in the emergency "epartment re)eals a mass consistent 'ith $la""er cancer% as 'ell as significant #rinary retention. =hich of the follo'ing is most likely to $e "etecte" #pon imaging the patient,s genito#rinary systema6Bilateral hy"ronephrosis $6Bla""er "ilation c6Bla""er "yskinesis "6Unilateral hy"ronephrosis e6Ureteral "ilation :ormal La$s 4. A 28!year!ol" man is $ro#ght to the emergency "epartment in a comatose state a fe' ho#rs after complaining of s#""en onset of e.cr#ciating hea"ache. :e#rologic e.amination re)eals "ilate" p#pils poorly responsi)e to light. A 2T scan of the hea" 'itho#t contrast "emonstrates hyper"ensity 'ithin the s#prasellar cistern% 'hile MA+ is

#nremarka$le. L#m$ar p#nct#re sho's hemorrhagic cere$rospinal fl#i". =hich of the follo'ing is the most likely "iagnosisa6Amyloi" angiopathy!relate" hemorrhage $62a)erno#s sin#s throm$osis c6/emorrhagic infarction "6Pit#itary apople.y e6A#pt#re" $erry ane#rysm
Normal ,abs Note: Check your own answers before hittin$ the Check button below. When you click the Check button, a browser win"ow will a!!ear that contains a summary of your results. (*!lanations lock ; (*!lanations

:) (*!lanation: 7he correct answer is . 7his !atient is com!lainin$ of !ain with "efecation without any associate" ab"ominal or bowel sym!toms. 7he reference to travelerAs "iarrhea is a re" herrin$. 7he $rou!e" ulcers are characteristic of a her!etic infection. 7he ulcers be$in as vesicular lesions an" then !ainfully ulcerate. 7he !erineal re$ion is frequently involve", an" the lesions may s!rea" into the anal canal but "o not usually cause any evi"ence of !roctosi$moi"itis. 7hese sym!toms are often accom!anie" by neuro!athic sym!toms, as the her!es resi"es in the "orsal $an$lia. Cytome$alovirus infection 'choice A) may involve the colon in a severely immunocom!romise" 2I1 !atient whose C%? count is less than />. 7his man has no evi"ence of 2I1 an" furthermore has no colitic sym!toms. In a""ition, cytome$alovirus will not cause ulceration on the e*terior !erianal skin. Neisseria $onorrhea'choice C) may be the cause of a se*ually transmitte" !roctitis but will !resent with a muco!urulent "ischar$e an" !erha!s sym!toms of mil" !roctitis but without ulceration. )hi$ella "ysenteriae'choice %) will !resent as an invasive ty!e of "iarrhea with bloo"y, mucoi" stools an" may cause ulceration in the colon or small bowel but "oes not cause ulcerations in the anal canal or !erianal re$ion. Flcerative colitis 'choice () woul" have an abnormal si$moi"osco!ic a!!earance an" !resent with bloo"y "iarrhea. Flcerations of the !erineal re$ion are not characteristic of ulcerative colitis.

;) (*!lanation: 7he correct answer is %. (*ternal otitis, or infection of the e*ternal ear canal, can be cause" by a variety of or$anisms, notably inclu"in$ (scherichia coli, &seu"omonas aeru$inosa, &roteus vul$aris, an" )ta!hylococcus aureus. 7here is, however, a severe subty!e of e*ternal otitis, mali$nant e*ternal otitis, of which you shoul" be aware. 7his form is s!ecifically cause" by &seu"omonas aeru$inosa, an" ten"s to affect el"erly "iabetics an" AI%) !atients, causin$ the fin"in$s illustrate" in the question stem. It is !articularly worrisome both because the &seu"omonas or$anism is so tissue "estructive an" because it is often hi$hly resistant to most intravenous antibiotics. 'Consult your local microbiolo$y or !harmacolo$y "e!artments for a"vice about local sensitivities if you encounter the con"ition.) Com!lications can be "evastatin$, inclu"in$ "eafness, facial nerve !aralysis, osteomyelitis, an" menin$itis. (scherichia coli'choice A) can cause both e*ternal otitis an" acute otitis me"ia, but "oes not usually cause mali$nant e*ternal otitis. 2aemo!hilus influen#ae'choice ) is an im!ortant cause of acute otitis me"ia. &roteus vul$aris'choice C) can cause e*ternal otitis, but "oes not usually cause the mali$nant form. )ta!hylococcus aureus'choice () can cause e*ternal otitis, but "oes not usually cause the mali$nant form. 0) (*!lanation: 7he correct answer is C. 7his !atient has sym!toms of con$estive heart failure an" !ossible atrial fibrillation, as "emonstrate" by her irre$ularly irre$ular heartbeat. In a""ition, she is on "i$o*in an" is at hi$h risk for the "evelo!ment of an embolic occlusion of the su!erior mesenteric artery. 7hese !atients will !resent with severe !ain out of !ro!ortion to their ob<ective !hysical fin"in$s. 7he "ia$nosis shoul" be sus!ecte" clinically, an" imme"iate su!erior mesenteric arterio$ram shoul" be !erforme". If evi"ence of ischemia is confirme", the !atient shoul" !rocee" to e*!loratory la!arotomy to evaluate for intestinal ischemia an" !ossible $an$renous bowel. %iverticulitis 'choice A) may !resent with severe ab"ominal !ain but is $enerally lower ab"ominal an" is often locali#e" in the left lower qua"rant, the site of si$moi" "iverticulitis. &atients will often $ive a history of chronic cram!y, !ost!ran"ial !ain in the left lower qua"rant.

Ischemic colitis 'choice ) will usually !resent as "iarrhea, often bloo"y, in el"erly !atients with known atherosclerotic heart "isease. Althou$h !ancreatitis 'choice %) may "evelo! abru!tly, !articularly with $allstone !ancreatitis, the sym!toms are usually locali#e" to the e!i$astric lesion, with ra"iation to the back an" associate" nausea an" vomitin$. -urthermore, chronic !ancreatitis "oes not cause heme4!ositive stools, as in this !atient. A small bowel obstruction 'choice () is unlikely in the absence of !rior ab"ominal sur$ery, an" associate" a"hesions an" will $enerally !resent with ab"ominal "istension in association with hi$h4!itche" hy!eractive bowel soun"s, as well as nausea an" vomitin$. ?) (*!lanation: 7he correct answer is C. 7his !atient has $lucose4C4!hos!hate "ehy"ro$enase '.C&%) "eficiency, which is an E4linke" "isor"er that lea"s to hemolytic crises within hours of e*!osure to o*i"ant stress. 7he most common stressors are viral an" bacterial infections, sulfa "ru$s, quinines, an" fava beans. %urin$ an acute hemolytic crisis, hemo$lobin becomes "enature" an" lea"s to the formation of 2ein# bo"ies. 7he "ia$nosis is ma"e by the "emonstration of 2ein# bo"ies "urin$ an acute crisis, an" low levels of .C&% "urin$ normal times. 7he treatment inclu"es maintainin$ a"equate urine out!ut an" the !revention of future e!iso"es. ,ea" !oisonin$ 'choice A) lea"s to a normochromic, normocytic anemia with baso!hilic sti!!lin$. 7he clinical features inclu"e ab"ominal !ain, hea"ache, irritability, an" !eri!heral motor neuro!athy. 7reatment inclu"es the use of chelatin$ a$ents. -olate "eficiency 'choice ) lea"s to me$aloblastic anemia an" is most common in alcoholics. 2ere"itary s!herocytosis 'choice %) is an inherite" membrane "isor"er that lea"s to hemolytic anemia an" re" bloo" cell swellin$. )mall, roun", hy!erchromatic re" cells without a central area of !allor are seen on bloo" smears. Occult bloo" loss 'choice () lea"s to chronic iron loss an" microcytic anemia. 7he sym!toms inclu"e a $ra"ual onset of weakness an" fati$ue. +ana$ement inclu"es the "ia$nosis an" control of the un"erlyin$ "isor"er. /) (*!lanation: 7he correct answer is C. 7he use of nonsteroi"al anti4inflammatory "ru$s 'N)AI%s), such as na!ro*en, may cause a usually mil" renal insufficiency, !ossibly relate" to a mil" interstitial ne!hritis or

$lomerulone!hritis. 6isk of N)AI%4in"uce" renal "ama$e is increase" in the el"erly an" in !atients with un"erlyin$ renal "isease. .li!i#i"e 'choice A), a secon"4$eneration sulfonylurea, may !re"is!ose !atients to hy!o$lycemia but is not associate" with renal to*icity. +etformin 'choice ) "oes not in"uce renal "ama$e but shoul" be use" cautiously in !atients with un"erlyin$ renal "ama$e because of the !ossibility of "evelo!in$ lactic aci"osis. Neither nife"i!ine 'choice %) nor !ro!ranolol 'choice () has a ten"ency to a"versely affect the ki"neys. C) (*!lanation: 7he correct answer is %. 7he clinical manifestations are hi$hly characteristic of myasthenia $ravis. 7his "isor"er has three $eneral features: the fluctuatin$ nature of muscle weakness, !re"ominant involvement of ocular muscles 'with "i!lo!ia an" !tosis), an" !ositive clinical res!onse to a"ministration of choliner$ic a$ents. Crisis of weakness involvin$ res!iratory muscles was the most frequent cause of "eath before the a"vent of !ositive !ressure res!irators. 7he "isease is autoimmune4me"iate" an" results from autoantibo"ies to the muscular nicotinic rece!tors. esi"es the !harmacolo$ic test, a !ro$ressive "ecrease in the am!litu"e of muscle !otential is the "ia$nostic feature of myasthenia $ravis. (lectromyo$ra!hy is therefore very useful in the "ia$nosis of this con"ition. loo", urine, an" C)- analysis 'choice A) are entirely within normal limits in myasthenia $ravis, althou$h they are in"ee" frequently !erforme" in the initial screenin$. +6I of the hea" 'choice ) an" ((. recor"in$ 'choice C) woul" be entirely useless in this settin$. )ince the "isor"er is "ue to im!aire" choliner$ic transmission at the neuromuscular <unction, skeletal muscle bio!sy 'choice () is within normal limits at the li$ht microsco!ic level. Occasionally muscle bio!sy is !erforme" to rule out other causes of muscle weakness, such as myo!athic !rocesses. D) (*!lanation: 7he correct answer is A. 7he !atient most likely has nummular "ermatitis, which is a chronic inflammation of the skin, the etiolo$y of which is still unknown. 7he !resentation illustrate" in the question stem is ty!ical. 7his con"ition shoul" be in your "ifferential "ia$nosis whenever the terms Hcoin4sha!e"H or H"iscoi"H are use" to "escribe a !atientAs skin lesions in a question. +icrosco!ically, the "ominant feature is a locali#e" s!on$iosis 'corres!on"in$

to e"ema) of the e!i"ermis, which may also contain minute flui"4fille" holes that corres!on" to the tiny vesicles seen clinically in early lesions. 7reatment of these !atients is !roblematic, an" numerous re$imens involvin$ corticosteroi"s or antibiotics have been recommen"e", each of which a!!ears to work with some but not all !atients. &om!holy* 'choice ) !ro"uces "ee!4seate" !ruritic vesicles on the !alms, fin$ers, an" soles. &soriasis 'choice C) can !ro"uce coin4sha!e" lesions, but they are covere" with silvery scale. )eborrheic "ermatitis 'choice %) !ro"uces hy!erkeratosis on the scal! an" face. )tasis "ermatitis 'choice () can !ro"uce "iscoloration an" ulceration of the lower le$s near the ankles. @) (*!lanation: 7he correct answer is C. &rimary !revention requires im!lementation of a !roce"ure when the "isease is not !resent even in a !resym!tomatic !hase. 6outine immuni#ation of in"ivi"uals at risk, whether chil"ren or a"ults, is !rimary !revention. Isolation of "isease contacts is tertiary !revention 'choice A). +ammo$ra!hy is a form of secon"ary !revention 'choice ) where the early or !resym!tomatic reco$nition of "isease is involve". 1isual screenin$ is a screenin$ test an" a form of secon"ary !revention 'choice %). Occult bloo" testin$ in stool is a form of secon"ary !revention as well 'choice ().

I) (*!lanation: 7he correct answer is A. 7he !atientAs youn$ a$e, occurrence of !ain at ni$ht, ne$ativity of rheumatoi" factor, an" es!ecially, bilateral involvement of sacroiliac <oints are consistent with ankylosin$ s!on"ylitis. 7his is one of the serone$ative s!on"yloarthro!athies, characteri#e" by onset before ?> years of a$e, absence of circulatin$ autoantibo"ies, frequent association with 2,A4 ;D histocom!atibility anti$en, an" common involvement of the s!inal column. Ankylosin$ s!on"ylitis shoul" be sus!ecte" in any youn$ !erson com!lainin$ of chronic lower back !ain an" confirme" by ra"io$ra!hs or C7 scans of sacroiliac <oints. 7he "isease usually !ro$resses to involve the whole vertebral column, !ro"ucin$ ankylosis an" res!iratory failure secon"ary to restrictive lun$ "isease. Fveitis an" aortic insufficiency are a""itional manifestations.

%e$enerative <oint "isease 'choice ) woul" be e*ce!tional at such a youn$ a$e, unless !re"is!osin$ con"itions were !resent. %e$enerative <oint "isease is not associate" with systemic si$ns an" sym!toms. 6a"io$ra!hs of affecte" <oints show narrowe" interarticular s!aces, osteo!hytes, an" increase" "ensity of subchon"ral bone. )acroiliac <oints are not involve". 6eiter syn"rome 'choice C) is one of the serone$ative s!on"yloarthro!athies. It "evelo!s as a sequela of $astrointestinal infections "ue to )almonella, )hi$ella, or Cam!ylobacter, or after se*ually transmitte" infection cause" by Chlamy"ia or Frea!lasma. Arthritis of lar$e <oints 'knee an" ankle), con<unctivitis, an" skin vesicular eru!tion are the hallmarks of this con"ition. )erone$ative rheumatoi" arthritis 'choice %) refers to those cases in which a ty!ical !icture of rheumatoi" arthritis is associate" with ne$ative rheumatoi" factor. 6heumatoi" arthritis involves small <oints, es!ecially those of the han"s. )till "isease 'choice () is a rare systemic form of arthritis with onset before a$e :D. It manifests with s!ikin$ fever an" systemic sym!toms that usually ante"ate arthritis. Associate" manifestations inclu"e a morbilliform rash, he!atos!lenome$aly, serositis, anemia, an" leukocytosis. :>) (*!lanation: 7he correct answer is %. 7he su""en onset of tachycar"ia an" hy!otension in"icates an acute !rocess. )ince the !atient is bein$ mechanically ventilate" with !ositive !ressure, he is at increase" risk of a bullous ru!ture from barotrauma, lea"in$ to a !neumothora*. Car"iac arrhythmia 'choice A) coul" lea" to tachycar"ia an" hy!otension. 1entricular tachycar"ia an" atrial fibrillation with a ra!i" ventricular res!onse may cause this from "ecrease" ventricular fillin$. An (C. woul" ai" in this "ia$nosis. In the settin$ of mechanical ventilation, however, a !neumothora* must be e*clu"e" first. ronchial secretions 'choice ) usually have a !ro$ressively worsenin$ !resentation. -urthermore, the !atient woul" e*hibit "esaturation, but not necessarily hy!otension. +yocar"ial infarction 'choice C) may lea" to car"io$enic shock from failure. 2owever, this woul" most likely be a bit more !ro$ressive an" less acute. Infarction must remain hi$h on the "ifferential "ia$nosis, an" the !atient may require vaso!ressors because of the shock. In the imme"iate settin$, the !neumothora* is more likely, $iven the acuity of onset. &ulmonary embolus 'choice () is on the "ifferential "ia$nosis of electromechanical "issociation. 7his !atientAs risk of an embolus is increase" because of !rolon$e"

immobili#ation. 2owever, the subcutaneous he!arin shoul" be a"equate !ro!hyla*is a$ainst an embolism. ::) (*!lanation: 7he correct answer is A. 7his !atient is seriously ill, with low 7? an" low 70, but normal 7)2. 7his is ty!ical for euthyroi" sick syn"rome, which occurs in many seriously ill !atients who "o not have clinical hy!othyroi"ism. It can be enou$h of a "ia$nostic !roblem that some references su$$est that thyroi" hormones not be measure" in !atients in the intensive care unit unless they are stron$ly clinically sus!ecte" of havin$ thyroi" "isease. 7he 7)2 level is usually most hel!ful in "istin$uishin$ euthyroi" sick syn"rome from true hy!othyroi"ism, as it often above 0> mFBm, in true hy!othyroi"ism an" may be below normal, normal, or minimally elevate" in euthyroi" sick syn"rome. %is!ro!ortionately "ecrease" 70 is also ty!ical of euthyroi" sick syn"rome, an" 7? may be normal or "ecrease". .raves "isease 'choice ) !ro"uces hy!erthyroi"ism. 2ashimoto "isease 'choice C) ty!ically !ro"uces a rubbery $oiter. +e"ullary carcinoma of the thyroi" 'choice %) is rare an" woul" !robably not affect the serum thyroi" hormone levels unless it was very e*tensive. )ilent lym!hocytic thyroi"itis 'choice () can cause hy!othyroi"ism that ty!ically occurs in the !ost!artum !erio". :;) (*!lanation: 7he correct answer is . 7his !atient most likely has seasonal aller$ic rhinitis. It has been shown quite convincin$ly that the "ia$nosis is base" on the history an" !hysical alone an" that further work4u! is not cost4effective an" shoul" be reserve" for refractory cases. 7he bloo" ra"ioaller$osorbent test 'choice A) reliably "etects aller$en4s!ecific I$( antibo"ies in the serum an" quantifies their concentrations. Intra"ermal testin$ 'choice C) involves the intro"uction of a measurable amount of aller$en into the "ermal layer of the skin. )erum !rotein electro!horesis 'choice %) is not routinely use" to ai" in the "ia$nosis of rhinitis. It is commonly use" in the "ia$nosis of sickle cell "isease an" multi!le myeloma.

)kin4!rick testin$ 'choice () involves the a!!lication of a small amount of concentrate" aller$en to the skinG it is then H!ricke"H throu$h to the e!i"ermal layer. 7his ty!e of testin$ is less sensitive than intra"ermal testin$. :0) (*!lanation: 7he correct answer is (. 7his !atient with well a"vance" cirrhosis an" !ortal hy!ertension has "evelo!e" the onset of renal insufficiency consistent with he!atorenal syn"rome. 7his occurs "urin$ the en" sta$es of cirrhosis an" is characteri#e" by "iminishe" urine out!ut an" low urinary so"ium. In the settin$ of en"4sta$e liver "isease, renal vasoconstriction occurs, an" the "istal convolute" tubule res!on"s by conservin$ so"ium. Fnless the renal function is allowe" to "eteriorate further, liver trans!lantation will reverse this vasoconstriction an" ki"ney function will return to normal. A lar$e volume !aracentesis 'choice A) may relieve the ascites but will have no si$nificant benefit on the im!aire" renal function. 7here are no in"ications in this question to su$$est that the !atient requires acute hemo"ialysis 'choice ). A mesocaval shunt 'choice C) is a sur$ical !roce"ure that may "ecom!ress the !ortal !ressure but will not have any benefit on renal function. 6enal trans!lantation 'choice %) is of no value in this !atient since the un"erlyin$ lesion is in the liverG the ki"neys will return to normal function if there is im!rovement in he!atic function. :?) (*!lanation: 7he correct answer is . 7his !atientAs sym!tomatolo$y is consistent with acute he!atitis. Althou$h alcoholic he!atitis may be in"istin$uishable from other forms of acute he!atitis, an alcoholic etiolo$y is favore" by !revalent elevation of A)7 'more than two times) com!are" with A,7. All the remainin$ laboratory fin"in$s, e.$., neutro!enia with relative $ranulocytosis, hy!oalbuminemia with hy!er$amma$lobulinemia, an" !rolon$e" &7, may be !resent in many other forms of acute he!atitis, inclu"in$ viral he!atitis 'choice (). (vi"ence of malnourishment is also consistent with alcoholism. +acrocytic anemia is a frequent coe*istin$ fin"in$ "ue to vitamin :; an" folic aci" "eficiency. In short, only the !resence of A)7 elevation $reater than ; times that of A,7 is hi$hly su$$estive of alcoholic he!atitis. None of the remainin$ sym!toms or laboratory chan$es is !atho$nomonic of this con"ition. Alcoholic in<ury is, however, the most likely etiolo$y in consi"eration of the whole clinical !icture.

Acute !ancreatitis 'choice A) most frequently "evelo!s in !atients with a history of alcoholism or cholelithiasis. It !resents with a "ramatic !icture of e*tremely severe, "ee! ab"ominal !ain often ra"iatin$ to the back. 7he !atient is restless an" "ia!horetic. )erum levels of amylase an" li!ase are usually marke"ly elevate". +il" elevations of the latter en#ymes are often seen in alcoholic he!atitis. Cholecystitis 'choice C) is usually secon"ary to a stone im!acte" in the cystic "uct, resultin$ in "istention of the $allbla""er an" colicky !ain. -ever an" mil" <aun"ice may be !resent, but usually A)7 an" A,7 are normal or sli$htly elevate". %uo"enal !e!tic ulcer 'choice %) is accom!anie" by e!i$astric !ain or "iscomfort. If !erforation occurs, the !ain be$ins su""enly an" is associate" with ab"ominal $uar"in$. Naun"ice, laboratory evi"ence of liver "ama$e, an" ascites are absent. :/) (*!lanation: 7he correct answer is . 7his is a !atient of +e"iterranean "escent who has a microcytic anemia with an a!!ro!riate reticulocyte count, normal iron stu"ies an" a hemo$lobin electro!horesis with an increase" al!ha chain com!onent of hemo$lobin. 7he thalassemias are a hetero$eneous $rou! of inherite" "isor"ers characteri#e" by the un"er!ro"uction of either the al!ha or the beta chains of the hemo$lobin molecule. In beta thalassemia, a re"uce" !ro"uction of beta chains occurs with normal amounts of al!ha !ro"uction. Al!ha4thalassemia 'choice A) is not su!!orte" by the !resence of al!ha chains on the electro!horetic !attern. Iron "eficiency anemia 'choice C) althou$h a microcytic anemia, is not su!!orte" by the normal iron stu"ies. +e$aloblastic anemia 'choice %) is not su!!orte" by the low +C1. )ickle cell "isease 'choice () is not su!!orte" by any of the "ata. It is usually "ia$nose" by sickle cells on the !eri!heral smear an" the !resence of hemo$lobin ) on electro!horesis. :C) (*!lanation: 7he correct answer is A. &ermanent renal "ama$e can "evelo! in as little as / years after "iabetes mellitus. %iabetic ne!hro!athy com!licates about one thir" of cases of ty!e ; "iabetes mellitus, an" a smaller !ro!ortion of ty!e ; cases. One of the !roblems is that "iabetic ne!hro!athy ten"s to be asym!tomatic until en"4sta$e "isease "evelo!s, so there has been consi"erable interest in i"entifyin$ early markers for si$nificant renal "isease. 7he s!illin$ of albumin

into the urine an", more s!ecifically, the s!illin$ of very small levels of albumin 'HmicroalbuminuriaH) are the best markers to "ate for si$nificant "iabetic ne!hro!athy. 2y!ertension 'choice ) often coe*ists with, an" a!!arently favors the "evelo!ment of, microalbuminuria, but is not a s!ecific marker for renal "ama$e. 6isin$ bloo" urea nitro$en 'choice C) an" risin$ creatinine 'choice %) occur a little later, when there is alrea"y a very si$nificant "ecrease in $lomerular filtration rate. Frinary tract infection 'choice () can !ermanently "ama$e the ki"neys but "oes not always "o so. :D) (*!lanation: 7he correct answer is . 7he sym!tomatolo$y is classic for car!al tunnel syn"rome, which is a form of neuro!athy resultin$ from anatomic com!ression of the me"ian nerve. &ain, tin$lin$ sensations, an" hy!oesthesia in the "istribution of the me"ian nerve are the car"inal manifestations. 7hese often un"er$o e*acerbations at ni$httime. A shock4like !ain u!on !ercussion on the volar as!ect of the wrist '7inel si$n) is an a""itional characteristic si$n. Car!al tunnel syn"rome is most often i"io!athic, but may re!resent a manifestation of un"erlyin$ "isor"ers such as rheumatoi" arthritis, sarcoi"osis, amyloi"osis, acrome$aly, an" leukemia. Car!al tunnel syn"rome may be confuse" with an$ina !ectoris 'choice A) when locate" on the left si"e. 2owever, an$ina !ectoris ty!ically manifests with !hysical or emotional stress an" very rarely results in !ain limite" to the han". %u!uytren contracture 'choice C) is a relatively common "isor"er characteri#e" by fibrous thickenin$ of the !almar fascia. Contracture an" no"ule formation ensue. 7his con"ition is most common in Caucasian men over />. -ibrositis 'choice %), also known as fibromyal$ia, refers to a !oorly un"erstoo" syn"rome of wi"es!rea" musculoskeletal !ain associate" with ten"erness in multi!le tri$$er !oints. -ati$ue, hea"ache, an" numbness are also common. Women between ;> an" /> years of a$e are most commonly affecte". Neck, shoul"ers, low back an" hi!s are usually involve". 6efle* sym!athetic "ystro!hy 'choice () "escribes a syn"rome of !ain an" swellin$ of one e*tremity 'most commonly a han"), associate" with skin atro!hy. It is thou$ht to be secon"ary to vasomotor instability. )ometimes, it follows in<uries to the shoul"er 'shoul"er4han" variant). :@) (*!lanation:

7he correct answer is A. 7his is a keratoacanthoma. 7hese lesions have the "istinctive a!!earance "escribe" in the question stem, an" ten" to occur on sun4e*!ose" areas inclu"in$ the face, the forearm, an" the "orsum of the han". 7hese lesions microsco!ically closely resemble squamous cell carcinoma, but are now consi"ere" beni$n. 7hey may resolve s!ontaneously, an" some authors su$$est that they may not nee" resection. 2owever, they are usually resecte" since both !atient an" !hysician often feel uncomfortable with ne$lectin$ them without havin$ a "efinitive "ia$nosis an" bein$ absolutely certain they will s!ontaneously re$ress. ,i!oma 'choice ) causes a soft subcutaneous mass. +ali$nant melanoma 'choice C) usually causes a !i$mente" lesion, often with feathery e"$es su$$estin$ invasion. &yo$enic $ranuloma 'choice %) causes a re" mass, often at a site of !revious in<ury. )eborrheic keratosis 'choice () causes a warty lesion most often foun" on the tem!le or trunk. :I) (*!lanation: 7he correct answer is %. -emoral !seu"oaneurysms re!resent an im!ortant vascular com!lication of car"iac catheteri#ation. 7he combination of a !ulsatile mass, femoral bruit, an" com!romise" "istal !ulses make this "ia$nosis likely. 7he "ia$nosis can be confirme" by ultrasoun" of the $roin. Cholesterol emboli syn"rome 'choice A) is also an im!ortant com!lication to reco$ni#e in the !ost4catheteri#ation !atient. It usually !resents, however, with skin fin"in$s in the "istal e*tremities of live"o reticularis, ischemic ulcerations, cyanosis, $an$rene, or subcutaneous no"ules. -emoral aneurysms 'choice ), like the more common aortic an" !o!liteal aneurysms, are true aneurysms that re!resent a "ilation of the arterial wall itself, often associate" with an un"erlyin$ connective tissue "isor"er or atherosclerotic "isease. While true aneurysms may also !resent as !ulsatile masses in the $roin an" may be associate" with "istal emboli#ation of clots, the !ro*imity to car"iac catheteri#ation makes !seu"oaneurysm likelier. -emoral hernias 'choice C) occur when ab"ominal contents !ass throu$h the femoral canal, with the hernia sac lyin$ below &ou!artAs li$ament. While it may !resent as a mass in the $roin, a femoral hernia woul" unlikely be associate" with a bruit or arterial vascular com!romise.

Another im!ortant com!lication of car"iac catheteri#ation via the femoral artery is a retro!eritoneal blee" 'choice (). 7his com!lication !resents, however, as either new back !ain, an une*!laine" "ro! in the hematocrit, or !ur!ura over the flanks. ;>) (*!lanation: 7he correct answer is C. 7his is !iriformis syn"rome. As you may recall from your anatomy, the !iriformis is the small muscle that crosses the $reater sciatic foramen, cuttin$ it into two s!aces as the muscle !asses from the e"$e of the sacrum to the $reater trochanter. 7he sciatic nerve comes out of the $reater sciatic foramen below the !iriformis, an" is liable to com!ression by the muscle. )ym!toms are as "escribe" aboveG bicycle ri"in$ an" runnin$ may also set off the sym!toms, which may take the form of chronic na$$in$ ache, !ain, tin$lin$, or numbness. 7reatment is usually to teach the !atient to avoi" maneuvers that set off the sym!toms. )ome !atients have been hel!e" by corticosteroi" in<ection near the site where the !iriformis muscle crosses the sciatic nerveG this thera!y is thou$ht to work by re"ucin$ the fat aroun" the muscle an" thereby increasin$ the available s!ace in the area. %isk com!ression of the sciatic nerve 'choice A) can also !ro"uce sciatic !ain, but there is almost always lumbar as well as buttock !ain. -ibromyal$ia 'choice ) causes achy !ain, ten"erness, an" stiffness of involve" sites, inclu"in$ muscles, ten"on insertions, an" nearby soft tissues. )ites commonly involve" inclu"e hea", neck, shoul"ers, thora*, low back, an" thi$hs. &o!liteus ten"initis 'choice %) is inflammation of the ten"on of the !o!liteus muscle of the knee. &osterior femoral muscle strain 'choice () !ro"uces !ain in the !osterior thi$h on <um!in$. ;:) (*!lanation: 7he correct answer is . 7his !atient, with a lon$ history of chronic obstructive !ulmonary "isease 'CO&%), has evi"ence of a community4acquire" !neumonia. 7he common or$anisms causin$ !neumonias in !atients with CO&% are )tre!tococcus !neumoniae, 2aemo!hilus influen#ae, an" +ora*ella catarrhalis. 7his !atient has no other history su$$estive of (scherichia coli'choice A) infection elsewhere 'such as in the urinary tract), an" !rimary (. coli !neumonia is rare.

5lebsiella !neumoniae'choice C) is ty!ically foun" in alcoholic !atients an" it may cavitate. 7here is no evi"ence of tuberculosis 'choice %) by history. 7uberculosis usually !resents with a more chronic !resentation. -urthermore, it woul" $enerally be foun" as an u!!er lobe infiltrate, consistent with reactivation tuberculosis. +uch less commonly, tuberculosis may !resent as a !rimary infection, but this is $enerally seen in !atients with an un"erlyin$ immunocom!romise" state. In the settin$ of !rimary tuberculosis, a lower lun$ fiel" !neumonia is in fact !ossible. +yco!lasma !neumonia'choice () "oes not !resent with a lobar consoli"ation an" is $enerally a "isease of youn$er !eo!le who !resent with fever, malaise of at least several "ays "uration, an" a non!ro"uctive cou$h. 7he chest *4ray film in a !atient with +yco!lasma !neumonia woul" classically reveal faint bilateral interstitial infiltrates. ;;) (*!lanation: 7he correct answer is A. 7his !atient most likely has A""isonAs "isease, which is !rimary a"renocortical "eficiency. It is a rare "isease that is cause" by a !ro$ressive "estruction of the a"renal $lan"s, usually "ue to i"io!athic atro!hy, sur$ery, infection, or hemorrha$e. 7he clinical sym!toms inclu"e weakness, wei$ht loss, hy!er!i$mentation, nausea an" vomitin$, an" hy!otension. ,aboratory fin"in$s inclu"e hy!onatremia '"ue to al"osterone "eficiency), hy!erkalemia, an" normocytic anemia with eosino!hilia an" lym!hocytosis. 7he "ia$nosis is ma"e with the AC72 stimulation test. Cortisol an" al"osterone levels "o not increase when the AC72 is $iven. 7he treatment is $lucocorticoi" an" mineralocorticoi" re!lacement. ConnAs syn"rome 'choice ) is !rimary al"osteronism "ue to an al"osterone4!ro"ucin$ a"enoma. )ym!toms inclu"e hea"aches an" hy!ertension. ,aboratory fin"in$s inclu"e hy!okalemia an" hy!ernatremia. +etabolic alkalosis is common. Cushin$As "isease 'choice C) is cause" by an AC724!ro"ucin$ !ituitary tumor that !ro"uces e*cess al"osterone an" a"renal an"ro$en secretion. 7he sym!toms inclu"e weakness an" fati$ue, increase" wei$ht, osteo!orosis, hy!ertension, striae, amenorrhea, e"ema, an" im!aire" $lucose tolerance. A hi$h4"ose "e*amethasone su!!ression test, C7 scans an" +6Is are use" to make the "ia$nosis. 7reatment is sur$ery. Cushin$As syn"rome 'choice %) is similar to Cushin$As "isease e*ce!t that it is "ue to a"renal hy!er!lasia, an a"renal neo!lasm or e*o$enous, iatro$enic causes. )yn"rome of ina!!ro!riate anti"iuretic hormone secretion 'choice () is characteri#e" by hy!onatremia, anore*ia, nausea an" concentrate" urine. It is cause" by hea" trauma, a tumor or infections. 7he treatment is flui" restriction.

;0) (*!lanation: 7he correct answer is C. AI%) may lea" to various com!lications affectin$ the CN). Amon$ these, 2I1 ence!halitis, clinically known as AI%) "ementia com!le*, is the most common. 7he !atholo$ic substrate is a subacute inflammatory infiltration of the brain cause" by "irect s!rea" of 2I1 to the CN). &resence of the 2I1 $enome can be "emonstrate" by in situ hybri"i#ation in micro$lia an" histiocytes. 7he "ia$nosis of 2I1 ence!halitis 'or AI%) "ementia com!le*) must be reache" by e*clusion of other infective an" neo!lastic con"itions associate" with AI%). AI%) "ementia com!le* is characteri#e" by co$nitive im!airment, incontinence, im!airment of motor skills, an" confusion. +6I stu"ies an" C)- analysis are useful in e*clu"in$ other CN) "iseases 'see below). C+1 ence!halitis 'choice A) usually affects the !eriventricular re$ions of the brain an" the retina. C+1 ence!halitis is usually associate" with "isseminate" infection. C+1 can be isolate" in the C) -. +6I may also "emonstrate !eriventricular white matter abnormalities. Cry!tococcal menin$oence!halitis 'choice ) is an acute life4threatenin$ "isease manifestin$ with si$ns an" sym!toms of increase" intracranial !ressure an" fever. 7he C)- woul" show numerous cry!tococcal or$anisms. 2I1 myelo!athy 'choice %) manifests mainly with s!astic !ara!aresis. It is a com!lication similar in !atholo$ic substrate to vitamin :; "eficiency, i.e., vacuolar "e$eneration of the !osterior an" lateral columns of the s!inal cor". Its !atho$enesis is still unclear, but a "irect viral effect is sus!ecte". &rimary brain lym!homa 'choice () is a frequent manifestation of AI%). 7he +6I woul" show a rin$4enhancin$ mass, which is not the ty!ical ra"iolo$ic !resentation of brain lym!homas in immunocom!etent hosts. &ro$ressive multifocal leukoence!halo!athy 'choice -) consists of multifocal areas of myelin "estruction. 7hese chan$es woul" be visible on +6I. 7his com!lication is "ue to NC virus, a !a!ovavirus that causes asym!tomatic infections in immunocom!etent in"ivi"uals. 7o*o!lasmosis 'choice .) manifests on +6I in a manner similar to lym!homa, i.e., a rin$4enhancin$ mass. 7his o!!ortunistic infection is e*tremely frequent in AI%) !atients. ;?) (*!lanation: 7he correct answer is (. Althou$h the !atient clearly has abnormal vital si$ns that are worrisome, the fin"in$ most likely to !lace the !atient in imme"iate "an$er is her hy!o*ia. 7he !atientAs o*y$en

saturation of @>= !laces her at si$nificant risk for "elirium, car"iac arrhythmias, an" car"io!ulmonary arrest. O*y$en shoul" first be a"ministere" noninvasively in this case, startin$ with a non4rebreather face mask until the clinical !icture can be stabili#e". An anti4hy!ertensive a$ent 'choice A) woul" be an im!ortant thera!y if her con$estive heart failure an" hy!o*ia were the result of "iastolic ventricular "ysfunction from a hy!ertensive emer$ency. (ven if this were the case, however, the initial thera!y woul" still be to initially treat her hy!o*ia while a"ministerin$ an anti4hy!ertensive a$ent. eta blocka"e 'choice ) is effective at controllin$ heart rate throu$h the "ru$As actions on no"al con"uction in the heart. 2owever, there is rarely an in"ication to treat sinus tachycar"ia, as the un"erlyin$ cause 'in this case likely an*iety an" the increase" work of breathin$ a$ainst non4com!liant lun$s) shoul" be a""resse" first. A$ain, the hy!o*ia takes !rece"ence. I1 mor!hine 'choice C) woul" have the effect here of "iminishin$ an*iety an" "ecreasin$ venous return to the heart. Althou$h these effects may be attractive, mor!hine is also a res!iratory "e!ressant, an" the !atient may require her com!lete res!iratory "rive to maintain her bloo" o*y$en levels. Of all of the ways to a"minister o*y$en, intubation, either en"otracheal 'choice %) or nasotracheal, is the only means to ensure :>>= o*y$en "elivery to the lun$s. 7his !atient may ultimately require intubation, but, $iven its risks, it is a!!ro!riate to attem!t noninvasive o*y$en "elivery first. If the question ha" su$$este" that the !atient was unable to maintain her airway or that noninvasive metho"s ha" faile" to correct the hy!o*ia, this choice woul" have been correct. ;/) (*!lanation: 7he correct answer is %. &atients with lon$stan"in$ e*tensive ulcerative colitis for at least :> yearsA "uration are at increase" colon cancer risk. A!!ro!riate surveillance involves annual or biannual colonosco!y with multi!le bio!sies at re$ular intervals, even of normal a!!earin$ mucosa, to check for "ys!lasia. None of the other choices allows sam!lin$ of the entire colonic mucosa for histolo$ic e*amination for the !recancerous lesion of low4 or hi$h4$ra"e "ys!lasia. ;C) (*!lanation: 7he correct answer is %. 7his is a common clinical scenario in real life. )erum ferritin is marke"ly "ecrease" in iron "eficiency anemia an" is normal to mo"estly elevate" in anemia of chronic "isease. 7his "ifference makes this test very useful in this settin$. 9ou shoul" be aware that serum

ferritin may behave like an acute !hase reactant. 7herefore, in an acute inflammatory situation, erythrocyte ferritin may be more reliable if serum ferritin is equivocal. Also, "o not for$et that iron "eficiency anemia can com!licate the anemia of chronic "isease 'e.$., $astrointestinal blee"in$ can com!licate $astrointestinal cancers)G in this situation, most !hysicians treat the a!!arent iron "eficiency anemia an" see to what "e$ree the anemia corrects. 7he erythrocyte:$ranulocyte ratio in bone marrow 'choice A) is sli$htly "ecrease" in both iron "eficiency anemia an" anemia of chronic "isease, but may be marke"ly increase" in si"eroblastic anemias. &olychromato!hilic tar$et cells 'choice ) an" sti!!le" erythrocytes 'choice C) are absent in both iron "eficiency anemia an" anemia of chronic "isease, but may be !resent in si"eroblastic anemias an" other iron4utili#ation anemias. )erum iron 'choice () is "ecrease" in both iron "eficiency anemia an" anemia of chronic "isease, but may be marke"ly increase" in si"eroblastic anemia. ;D) (*!lanation: 7he correct answer is A. 7he !atient most likely has a worsenin$ of his bloo" !ressure "ue to erythro!oietin. 7his is seen in about 00= of "ialysis !atients. 1itamin % 'choice %), iron 'choice ) an" vancomycin 'choice C) $enerally "o not raise bloo" !ressure. 7he !atient is now on "ialysis an" shoul" not be uremic 'choice (). ;@) (*!lanation: 7he correct answer is -. 7his !atientAs clinical !icture is consistent with into*ication with tricyclic anti"e!ressants such as amitri!tyline an" imi!ramine. 7o*ic effects are me"iate" by !eri!heral anticholiner$ic activity an" Hquini"ine4likeH action. 7he anticholiner$ic effects inclu"e my"riasis, tachycar"ia, im!aire" sweatin$ with flushe" skin, "ry mouth, consti!ation, an" muscle twitchin$. 8uini"ine4like effects '"ue to block of so"ium channels in the heart) result in car"iac arrhythmias, es!ecially ventricular tachyarrhythmias. In this settin$, !rolon$ation of the 86) com!le* is !articularly im!ortant in the "ia$nosis. 86) wi"th is, in fact, an even more faithful !arameter of "ru$ to*icity than serum "ru$ levels. In severe into*ication, !atients will "evelo! sei#ures, severe hy!otension, an" coma. Acetamino!hen 'choice A) results in liver to*icity. ,iver en#ymes woul" be elevate".

Alcohol into*ication 'choice ) manifests with res!iratory "e!ression, hy!othermia, an" coma. 7he manifestations of ben#o"ia#e!ine into*ication 'choice C) are similar to alcohol inasmuch as central nervous system "e!ression is common to both "ru$s. 7hus, acute ben#o"ia#e!ine into*ication !ro"uces stu!or, coma, an" res!iratory "e!ression. 7he sym!atholytic !ro!erties of cloni"ine 'choice %) e*!lain the clinical sym!toms of into*ication. Cloni"ine over"ose causes bra"ycar"ia, hy!otension, miosis, an" res!iratory "e!ression. +onoamine o*i"ase '+AO) inhibitors 'choice () re!resent a secon"4line treatment for ma<or "e!ression. Over"ose in"uces ata*ia, e*citement, hy!ertension, an" tachycar"ia. )uch reactions can be !reci!itate" by concomitant in$estion of tyramine4containin$ foo"s 'a$e" cheese an" re" wine, for e*am!le). ;I) (*!lanation: 7he correct answer is (. 7his !atient has her!es #oster, which is a reactivation of the varicella4#oster virus '131) that was "ormant in the "orsal root $an$lion. 3oster 'shin$les) affects in"ivi"uals in the Cth to @th "eca"es. It is characteri#e" by !ain, fever, an" a "ermatomal erythematous J vesicular rash. 7reatment inclu"es antiviral thera!y. In"ivi"uals with her!es #oster are conta$ious an" can s!rea" the 131 virus. Chicken!o* 'varicella) is characteri#e" by a vesicular rash at various sta$es of evolution. 7he varicella vaccination is recommen"e" at :; months, however if the $ran"chil"ren have not been immuni#e", they may "evelo! chicken!o*. %iscrete maculo!a!ular lesions that become confluent as they s!rea" from Hhea" to toeH 'choice A) is the ty!ical !resentation of measles, which is cause" by a !aramy*ovirus. It is associate" with cou$h, con<unctivitis, cory#a, an" 5o!likAs s!ots. %ome4sha!e" !a!ules with central umbilication 'choice ) is the "escri!tion of molluscum conta$iosum, cause" by a !o*virus. An e*!an"in$ annular lesion with central clearin$ 'choice C), also known as erythema chronicum mi$rans, is the rash of ,yme "isease, cause" by orrelia bur$"orferi 'after a tick bite). 7he rash be$ins as an erythematous macule that e*!an"s with central clearin$, lea"in$ to the ty!ical HbullAs eyeH lesion AH sla!!e"4cheekH a!!earance an" a lacy reticular rash 'choice () is the "escri!tion of the rash of erythema infectiosum '-ifth "isease), which is cause" by !arvovirus :C.

0>) (*!lanation: 7he correct answer is %. 7his !atientAs history su$$ests !heochromocytoma. 7his rare 'but often consi"ere" "ia$nostically) tumor is most often foun" in the a"renal me"ulla, althou$h it can also be foun" in other tissues "erive" from neural crest cells. 7he tumor cells secrete catecholamine hormones or their !recursors, which can cause either !aro*ysmal 'as in this case) or !ersistent hy!ertension. Frinary metabolites of e!ine!hrine an" nore!ine!hrine are vanillylman"elic aci" '1+A) an" homovanillic aci", so screenin$ ;? hour urine collections for these substances can be hel!ful in establishin$ or e*clu"in$ these "ia$noses even in cases in which a !hysician "oes not observe one of the !aro*ysms an" thus bloo" cannot be "rawn for serum catecholamine levels at that time. %2(A 'choice A) is the a"renal an"ro$en "ehy"roe!ian"rosterone 'ma"e by the a"renal corte* rather than the a"renal me"ulla), an" is measure" in serum in cases where a"renal virilism is sus!ecte". hC. 'choice ) is human chorionic $ona"otro!in, an" both serum an" urine levels can increase in !re$nancy or tro!hoblastic "isease. :D4ketosteroi"s 'choice C) are measure" in urine "urin$ evaluation of con$enital a"renal hy!er!lasia 'a "isor"er of the a"renal corte* rather than me"ulla). 3inc !roto!or!hyrin 'choice () is measure" in bloo" when evaluatin$ !ossible !or!hyria. 0:) (*!lanation: 7he correct answer is %. 7his !atient has myasthenia $ravis, which was sus!ecte" base" on the womanAs clinical !resentation an" confirme" with the res!onse to the short4actin$ anticholinesterase "ru$ e"ro!honium. +yasthenia $ravis is an autoimmune "isease in which antibo"ies "irecte" a$ainst the acetylcholine rece!tor of the muscle si"e of the neuromuscular <unction block the ability of the rece!tor to bin" to acetylcholine. Of the "iseases liste" above, only insulin resistance is !ro"uce" by a similar mechanism, i.e. antibo"ies to insulin rece!tors block the rece!torsA ability to bin" to insulin. ullous !em!hi$oi" 'choice A) is cause" by antibo"ies "irecte" a$ainst the basement membrane of the skin, which "ama$e the basement membrane an" cause blister formation. )ome cases of "iabetes mellitus ty!e : 'choice ) an" i"io!athic A""ison "isease 'choice C) are characteri#e" by humoral, an" !robably cell4me"iate", reactions a$ainst the cells in the en"ocrine tissue.

)ystemic lu!us erythematosus 'choice () has circulatin$ an" locally $enerate" immune com!le*es me"iatin$ the !atho!hysiolo$y. 0;) (*!lanation: 7he correct answer is %. 7he first ste! in the a!!roach to this !atient with a community4acquire" !neumonia is to cate$ori#e her con"ition accor"in$ to the American 7horacic )ociety $ui"elines ':II0), which are base" on severity of illness, a$e, comorbi"ities, an" the nee" for hos!itali#ation. 7his !atient "oes not meet the criteria for hos!itali#ation 'one of the followin$ is nee"e": res!iratory rate O 0> breathsBmin, room air &aO;P C> mm 2$, O; saturation less than I>= on room air, or bilateral or multi!le lobes involve"), an" she is youn$er than C> years without any comorbi"ities. 7he most common or$anisms are )tre!tococcus !neumoniae, +yco!lasma !neumoniae, Chlamy"ia !neumoniae, 2aemo!hilus influen#ae, ,e$ionella, an" res!iratory viruses. 6ecommen"e" treatment is with erythromycin or a relate" macroli"e, such as a#ithromycin or clarithromycin. Amo*icillin 'choice A) "oes not have broa" enou$h covera$e to inclu"e the or$anisms liste" above. Ceftria*one 'choice C) an" am!icillin4sulbactam 'choice ) are use" for hos!itali#e" !atients with community4acquire" !neumonia. (rythromycin !lus ceftria*one 'choice () is reserve" for !atients who are severely ill an" hos!itali#e". 00) (*!lanation: 7he correct answer is (. 7he !atient is in "iabetic ketoaci"osis, as in"icate" by the acetone scent to the breath an" the $lucose an" ketones in the urine. %iabetic ketoaci"osis !ro"uces an increase" anion $a!, since the anion for the aci" that is !ro"uce" 'acetoacetate) is not one of the usually measure" ions. 7he anion $a! is usually estimate" by subtractin$ the sum of the Cl4 an" 2CO04 concentrations from the !lasma NaM concentrationG the normal value for the anion $a! is :; Q ? m(qB,. Causes of increase" anion $a! inclu"e con"itions that !ro"uce ketoaci"osis '"iabetes mellitus, alcoholism, starvation), renal failure with retaine" sulfate an" !hos!hate, "ru$s or metabolites 'salicylate or ethylene $lycol !oisonin$), alkalosis with increase" ne$ative char$e of !rotein anions, an" "ehy"ration 'hemoconcentration). C m(qB, 'choice A) is below the lower limit of normal. 7he anion $a! may be "ecrease" because of a "ecrease of ne$atively char$e" serum !roteins 'e$, in hy!oalbuminemia), an

increase in !roteins carryin$ few ne$ative char$es 'e$, hy!er$amma$lobulinemia), or an increase in unmeasure" cations 'e.$., ma$nesium, calcium, or lithium). 7he other values 'choice , C, an" %) are within normal limits. 0?) (*!lanation: 7he correct answer is %. 7he !atient nee"s ur$ent treatment with hi$h4"ose !re"nisone for $iant cell arteritis 'i.e., tem!oral arteritis). 7his systemic "isease overla!s with !olymyal$ia rheumatica in a!!ro*imately ?>= of cases. It affects el"erly !ersons who !resent with fever, malaise, tem!oral hea"ache, an" scal! ten"erness. .iant cell arteritis is a frequent cause of fever of unknown ori$in in the el"erly. 7he leukocyte count may be entirely normal, while the erythrocyte se"imentation rate '()6) is marke"ly elevate". 7his con"ition may involve arteries other than the su!erficial tem!oral artery, inclu"in$ the aortic branches. 7he most im!ortant reason to start !re"nisone thera!y is to !revent blin"ness secon"ary to e*tension of the !rocess to the o!hthalmic artery. &re"nisone shoul" be a"ministere" in hi$h "oses 'usually C> m$B"ay). +easurement of intraocular !ressure 'choice A) is a!!ro!riate to confirm a "ia$nosis of acute 'narrow4an$le) $laucoma. 7his $ives rise to a characteristic acute sym!tomatolo$y of !ainful, re" eyes, blurre" vision, an" halos aroun" li$hts. 1isual fiel" assessment 'choice ) is not in"icate" in this case. 2i$h4"ose !re"nisone may be slowly ta!ere" to low4"ose ':> m$B"ay) !re"nisone treatment 'choice C) over a !erio" of ; months, once the acute !hase has resolve". ,ow4 "ose !re"nisone is use" for !olymyal$ia rheumatica. 7em!oral artery bio!sy 'choice () is !erforme" routinely in any !atient with clinical si$ns an" sym!toms of $iant cell arteritis. It is !ositive in u! to @>= of cases. It shows the characteristic $iant cell4rich $ranulomatous reaction in the me"ia, with "estruction of the elastic lamina. 7he bio!sy shoul" be !erforme" after startin$ corticosteroi" treatment. 0/) (*!lanation: 7he correct answer is %. 7his !atient is !ost a total knee re!lacement an" is currently not on any anticoa$ulation thera!y. 7he risk of "ee! venous thrombosis '%17) an" subsequent !ulmonary embolism is very hi$h in this !o!ulation, an" it is the stan"ar" of care to initiate Couma"in or low4molecular4wei$ht he!arin !osto!eratively for a !erio" of C weeks to C months.

An AC( inhibitor 'choice A) is, in the lon$ term, an e*cellent "ru$ for this !atient $iven her "iabetes. In the !ost4sur$ical !erio", however, the $reatest consi"eration shoul" be $iven to the most !ressin$ issue. A nonsteroi"al anti4inflammatory a$ent 'choice ) "oes not a!!ear to be in"icate" at this time, as the !atient a!!ears to have reasonable !ain control with her o!iate. Oral as!irin 'choice C) is an anti4!latelet a$ent that has no role in the !revention of %17. )ubcutaneous unfractionate" he!arin 'choice () is use" for !revention of %17 in immobile !atients or in hos!itali#e" !atients unable to ambulate. 2owever, after ortho!e"ic sur$ery, es!ecially after <oint !roce"ures, its efficacy is very !oor, $iven the increase" venous stasis. 0C) (*!lanation: 7he correct answer is A. 7his !atient has the classic !resentation of $astroeso!ha$eal reflu* "isease '.(6%) in association with sclero"erma. 7hese !atients have the "e!osition of colla$en in the bo"y of the eso!ha$us, as well as the lower eso!ha$eal s!hincter ',()). 7his results in the ty!ical !attern of "ecrease" eso!ha$eal !eristalsis an" the re"uce" ability of the ,() to maintain its hi$h !ressures between swallowin$. 7hese !atients are therefore at risk for severe .(6% an" subsequent com!lications of !e!tic stricture an" arrettAs eso!ha$us. Althou$h there is no corrective thera!y to im!rove eso!ha$eal motility or increase ,() !ressure, a$$ressive treatment is $enerally aime" at re"ucin$ aci" !ro"uction with the use of hi$h "oses of !roton !um! inhibitors. Nevertheless, many of these !atients "evelo! the lon$4term consequences of .(6 %. Choice su$$ests achalasia, in which im!aire" eso!ha$eal !eristalsis is often accom!anie" by a lack of lower eso!ha$eal s!hincter rela*ation. Choice C "oesnAt "escribe any of the more common eso!ha$eal motor "isor"ers. Choice % su$$ests sym!tomatic "iffuse eso!ha$eal s!asm, !articularly if the !eristaltic waves were !oorly or$ani#e". Choice ( woul" be seen in !atients without eso!ha$eal motor "isease. 0D) (*!lanation: 7he correct answer is %. 7he most likely "ia$nosis is !rimary 'essential) thrombocythemia. 7he con"ition is "ue to a clonal abnormality of a multi!otent hemato!oietic cell that !ro"uces

me$akaryocytic hy!er!lasia with resultant increase" !latelet count. )ince the !latelets are often abnormal, either a thrombotic or a hemorrha$ic ten"ency may be seen. 7he !latelet count may be as low as />>,>>>BR, or $reater than :,>>>,>>>BR,. 7he clinical !resentation an" laboratory fin"in$s illustrate" in the question stem are ty!ical. 7he other choices liste" commonly must be e*clu"e" before a "ia$nosis of !rimary thrombocythemia is confirme". Chronic myelo$enous leukemia 'choice A) can be a cause of increase" !latelet count, but the absence of either a &hila"el!hia chromosome or a marke"ly increase" white count ar$ues a$ainst this !ossibility. +yelofibrosis 'choice ) can also cause thrombocythemia, but woul" likely show some abnormally sha!e" 'often tear "ro!s) re" cells. &olycythemia vera 'choice C) can also cause thrombocythemia, but woul" be associate" with an increase" re" cell mass. )econ"ary thrombocythemia 'choice () is a reactive !rocess that may occur in a variety of settin$s inclu"in$ chronic inflammatory "isor"ers, acute infection, hemorrha$e or hemolysis, tumors, iron "eficiency, or s!lenectomy. Abnormal !latelet forms are not usually seen on smears from these !atients an" !latelet function tests are usually normal. 0@) (*!lanation: 7he correct answer is %. 7he low !otassium is the most likely cause of the weakness. 7he !atient may be a "iuretic abuser or may vomit as !art of an eatin$ "isor"er. 7he hi$h bicarbonate 'choice A) may reflect an alkalosis, but shoul" not cause sym!toms like this. 7he ma$nesium 'choice C) an" calcium 'choice ) values are close to the normal ran$e. 7he low so"ium 'choice () is not likely to cause weakness. 0I) (*!lanation: 7he correct answer is C. 7he test of choice to rule out acute intracerebral hemorrha$e 'IC2) is the hea" C7 without contrast, as it is wi"ely available, relatively ine*!ensive, an" has hi$h levels of sensitivity an" s!ecificity for acute blee"in$.

While ((. 'choice A), may hel! locali#e a sei#ure focus, it has relatively !oor s!ecificity in establishin$ the etiolo$ic basis of the sei#ure an" may be "ifficult to obtain on an ur$ent basis. 2ea" +6I 'choice ), is more e*!ensive, less wi"ely available, an" more time4 consumin$ than hea" C7. It also has less sensitivity for IC2 within the first ?@ hours. E4ray films of the skull 'choice %), while im!ortant in "ia$nosin$ skull fractures an" such con"itions as &a$etAs "isease, have no role in the "ia$nosis of IC2. ,umbar !uncture 'choice (), is one of the mo"alities use" to "ia$nose subarachnoi" hemorrha$e ')A2). It is relatively contrain"icate" in IC2 because it may !reci!itate a herniation syn"rome in !atients with lar$e hematomas, an" has less sensitivity for intracerebral blee"in$ than C7. ?>) (*!lanation: 7he correct answer is A. asal cell carcinoma affects sun4e*!ose" areas, !articularly the mi" an" u!!er face, in !atients lackin$ !rotective !i$mentation. One of its mor!holo$ic forms is that of a raise", wa*y, !ale lesion that $rows very slowly an" "oesnAt metastasi#e to lym!h no"es. +elanoma 'choice ) woul" have !resente" as a !i$mente" lesion, with asymmetry, irre$ular bor"ers, "ifferent colors, an" a "iameter of more than >.C cm. It woul" not a!!ear as a raise", wa*y, !ale no"ule. 5eratoacanthoma 'choice C) $rows very ra!i"ly in a matter of weeks an" has a scaly, rou$h a!!earance, with a core of keratin. If untreate", it eventually slou$hs off. &yo$enic $ranuloma 'choice %) also $rows very ra!i"ly. It has the a!!earance of wet $ranulation tissue, with visible yellowish !us. )quamous cell carcinoma 'choice () is usually an ulcer, rather than a no"ule. In the face, it favors the lower li!. If !resent for several years, lym!h no"e metastasis can sometimes occur. ?:) (*!lanation: 7he correct answer is A. oth chronic la*ative use an" chronic "iuretic use can !ro"uce hy!okalemia. )evere hy!okalemia, with !lasma !otassium P0 m(qB,, can marke"ly affect skeletal, smooth, an" car"iac muscles. )keletal muscle effects can inclu"e weakness, cram!in$, fasciculations, !aralysis 'with risk of res!iratory failure), tetany, an" rhab"omyolysis. )mooth muscle effects inclu"e hy!otension an" !aralytic ileus. Car"iac muscle effects

inclu"e !remature ventricular an" atrial contractions, tachyarrhythmias, an" A1 block. A""itional (C. chan$es can inclu"e )7 se$ment "e!ression, increase" F wave am!litu"e, an" 7 wave am!litu"e less than F wave. 7he chan$es illustrate" in choices , C, %, an" ( are characteristic of hy!erkalemia. ?;) (*!lanation: 7he correct answer is (. )e!tic arthritis will !ro"uce the hi$hest <oint flui" white bloo" cell counts, ty!ically with counts of $reater than D/,>>> !er cc. Non4inflammatory arthritis, such as osteoarthritis, will ty!ically !ro"uce <oint as!irate counts of less than :>,>>>. If se!tic arthritis is sus!ecte" base" u!on the as!irate white bloo" cell count, then a!!ro!riate antibiotics shoul" be starte", while awaitin$ cultures. -ailure to initiate a!!ro!riate antibiotic thera!y until final i"entification of the or$anism woul" !otentially lea" to irreversible <oint "estruction. An elevate" !eri!heral white bloo" cell count of :?,0>> 'choice A) may be seen in either a crystalline or se!tic arthritis an" will not "istin$uish between the two. Inflammatory arthritis, i.e. crystalline arthritis, will ty!ically !ro"uce a <oint flui" as!irate with a white bloo" cell count of a!!ro*imately ;/4/>,>>> 'choices , C, an" %). )ince the as!irate" flui" is also routinely e*amine" for crystals, "ifferentiation of any cases of se!tic versus crystalline arthritis with bor"erline values for white bloo" cell count can usually be ma"e on that basis. ?0) (*!lanation: 7he correct answer is (. Althou$h there has been consi"erable !ro$ress in ima$in$ mo"alities available to the clinician for e*amination of the lun$, the !osterior4anterior '&A) an" lateral chest *4ray films remain the screenin$ test of choice, base" on its wi"e availability, low cost, safety, an" clinical usefulness. 7he chest C7 'choice A) has assume" an im!ortant role in ima$in$ of the lun$s, as it is able to better "efine the interstitium an" airs!aces of the lun$s. As such, this !atient may ultimately require a chest C7, but it shoul" not be the initial ima$in$ stu"y !erforme" here, as it is more costly, less wi"ely available an" e*!oses the !atient to si$nificantly more ra"iation than the !lain *4ray film. &ositron emission tomo$ra!hy '&(7) scannin$ 'choice ) currently has limite" clinical utility in the evaluation at han". Its use in other areas of me"icine an" research is e*!an"in$, however, an" it may !lay a lar$er role in !ulmonary me"icine in the future.

+6I 'choice C) has !laye" a revolutionary role in the evaluation of the brain an" connective tissues, but its clinical role in ima$in$ the chest is limite", es!ecially in li$ht of its cost. A ventilation4!erfusion scan 'choice %) is the ima$in$ test of choice for the screenin$ of sus!ecte" !ulmonary embolism. ??) (*!lanation: 7he correct answer is %. (arly intervention can !revent osteo!orosis. ,ater intervention can halt its !ro$ression, but it is not currently !ossible to reverse establishe" "isease. All current thera!ies for osteo!orosis are "irecte" at inhibitin$ bone resor!tion. one loss is $reatest within the first year of meno!ause so these a$ents are likely to be most efficacious if starte" at this time. (stro$en re"uces the rate of bone loss an" im!roves "ensity. 7he beneficial effects of estro$en re!lacement are well4"ocumente", an" it is estimate" that less than ;>= of women are takin$ estro$ens, "es!ite their enormous benefit. A""itionally, estro$en re!lacement has been shown to be im!ortant for !revention of coronary "isease in this a$e $rou!. is!hos!honates 'choice A) are a$ents that inhibit osteoclastic bone resor!tion. 7hese a$ents are efficacious for both !revention an" treatment of "isease. In some trials, their effect on au$mentin$ bone "ensity is similar to that of estro$ens. 2owever, $iven the si"e effects of these a$ents 'eso!ha$itis) as well as the other beneficial effects of estro$ens 'effects on 2%,, ,%, an" coronary "isease), estro$en is still the a$ent of choice for !revention. Calcitonin 'choice ) is -%A4a!!rove" only for establishe" "isease, but stu"ies have shown conflictin$ results re$ar"in$ its efficacy. Calcium an" vitamin % 'choice C) are critical com!onents of !revention an" on$oin$ treatment, but the effects of $ivin$ calcium an" vitamin % su!!lements are !ro!ortional to the "uration of thera!y. +ost !ostmeno!ausal women receive less than the recommen"e" calcium intake ':/>> m$B"ay) an" have hy!ovitaminosis % 'P:/ n$Bm, ;/4hy"ro*y vitamin % levels). Calcium an" vitamin % is the cornerstone of $oo" thera!y whether hormone re!lacement is utili#e" or not, but the effects are $enerally not si$nificant unless be$un early in life. )o"ium fluori"e 'choice () has been su!!lante" by newer thera!ies an" is use" !rimarily in (uro!e for treatment of establishe" "isease. ?/) (*!lanation: 7he correct answer is

%. 7his !atient has acute bacterial en"ocar"itis, most likely "ue to )ta!hylococcus aureus, the most common or$anism causin$ en"ocar"itis in intravenous "ru$ abusers. 7he H!in!oint lesionsH between his toes are si$ns of in<ection "ru$ abuse. Acute en"ocar"itis in "ru$ abusers ty!ically !resents with a hi$h fever, !leuritic chest !ain, an" a cou$h. 7he tricus!i" valve is commonly affecte" in these !atients. A murmur may not be !resent in early acute en"ocar"itis or in in<ection "ru$ abusers with tricus!i" valve "isease. 7he retinal lesions are calle" 6oth s!ots. Other fin"in$s inclu"e anemia an" an elevate" erythrocyte se"imentation rate. %ia$nosis is with bloo" cultures, which are ty!ically !ositive for ). aureus, an" with echocar"io$ra!hy. 7reatment is with antibiotics. Can"i"a albicans'choice A), &seu"omonas aeru$inosa'choice ), an" )erratia marcescens'choice C) are infrequent causes of en"ocar"itis. )tre!tococcus viri"ans'choice () is a common cause of en"ocar"itis in in"ivi"uals who are not in<ection "ru$ abusers. 7he onset of sym!toms is usually more $ra"ual, !atients !resent with a low4$ra"e fever, new car"iac murmur, s!lenome$aly, hematuria, !roteinuria, an" an elevate" erythrocyte se"imentation rate. ?C) (*!lanation: 7he correct answer is %. 7here is much "iscussion amon$ !rimary care !hysicians, an" the !ublic in $eneral, about !reventive measures for health maintenance. A little4"iscusse" relate" issue, however, is the ma<or im!act that mo"ification of lifestyle can have on e*istin$ con"itions. In this case, cessation of smokin$ is almost imme"iately associate" with tremen"ous benefit in !atients with chronic obstructive !ulmonary "isease 'CO&%). 7his $oal shoul" be "iscusse" at every meetin$ the !hysician has with this !atient. 7his !atientAs bloo" $lucose is not o!timal an", after sto!!in$ smokin$, a""in$ an oral $lucose control a$ent 'choice A) woul" be most a!!ro!riate. Fnless this !atient arrests the "ecline in his lun$ function, however, the en"4or$an "ama$e from "iabetes may never have the o!!ortunity to cause morbi"ity. (ncoura$in$ a""itional e*ercise 'choice ) may be useful, but this !atient is limite" by "ys!nea, which is likely "ue to his CO& %. Whatever mar$inal benefit may come from a few hun"re" e*tra yar"s walke" is nothin$ in com!arison to the benefit to be $ain from cessation of tobacco use. Althou$h o!inions "iffer as to what amount of alcohol is hel!ful, it is certain that not many !hysicians woul" "iscoura$e a !atient from "rinkin$ one $lass of whiskey !er "ay 'choice C). Clearly, this !atientAs bloo" !ressure is not o!timal, an" increasin$ his "ose of thia#i"e 'choice () may certainly be beneficial in the lon$ term, but woul" not be nearly as beneficial as the cessation of tobacco use woul" have on his "eclinin$ lun$ function.

?D) (*!lanation: 7he correct answer is %. 7his woman has a classic !resentation of !rimary biliary cirrhosis. It ty!ically affects mi""le4a$e" women an" will !ro$ress $ra"ually to the !oint of en"4sta$e liver "isease over a number of years. 7he "isease is "ue to an autoimmune "estruction of intrahe!atic bile "uctules, an" the "ia$nosis is ma"e by liver bio!sy. 7he serolo$y that shoul" be checke" is the antimitochon"rial antibo"y. &rimary biliary cirrhosis is often seen in in"ivi"uals with other autoimmune "iseases, such as )<K$ren syn"rome, !ernicious anemia, an" 2ashimoto thyroi"itis. Acute cholecystitis 'choice A) !resents acutely with ri$ht u!!er qua"rant !ain an" fever an" not with chronic fati$ue an" !ruritus. Acute he!atitis A 'choice ) may cause <aun"ice an" fati$ue, but it is a self4limitin$ infection an" "oes not last C months. Cholan$itis 'choice C) is "ue to acute obstruction of the common bile "uct an" !resents ur$ently with fever, ri$ht u!!er qua"rant !ain, an" <aun"ice 'CharcotAs tria"). &rimary sclerosin$ cholan$itis 'choice () is a sclerosin$ !rocess of the e*tra4 an" intrahe!atic "ucts, which usually !resents in youn$ males with un"erlyin$ inflammatory bowel "isease. ?@) (*!lanation: 7he correct answer is A. In real life, you woul" evaluate this woman for "eficiencies in iron, folate, an" 1itamin :;, since a woman with a "iet this !oor may very well have multi!le !roblems. 2owever, for the !ur!ose of this ty!e of test question, you shoul" reason as follows: both vitamin :; an" folate "eficiency can cause me$aloblastic anemia. 7he !atientAs Htea an" toastH "iet is much more su$$estive of folate "eficiency than :; "eficiency. -olate is wi"ely foun" in !lant an" animal tissues, but is easily "estroye" by over4cookin$. &articularly vulnerable !o!ulations inclu"e the el"erly, alcoholics, chronic liver "isease !atients, !re$nant women, tro!ical s!rue !atients, chronic hemolytic anemia !atients, an" !atients bein$ treate" 'usually chronically) with certain me"ications 'anti4convulsants, oral contrace!tives, antimetabolites, an" antibiotics that interru!t folate metabolism). Iron "eficiency 'choice ) causes a microcytic anemia. 1itamin :; "eficiency 'choice C) is an im!ortant cause of me$aloblastic anemia, but is more likely to be relate" to chronic $astritis with "estruction of intrinsic factor4secretin$ !arietal cells '!ernicious anemia), fish ta!eworm infestation, or malabsor!tion.

1itamin C "eficiency 'choice %) is an occasional cause of me$aloblastic anemia 'often in con<unction with mil" folate "eficiency), but this !atient is "rinkin$ a vitamin C4 containin$ oran$e <uice substitute. 1itamin 5 "eficiency 'choice () is usually relate" to either malabsor!tion or intestinal !arasitic infection, an" causes a blee"in$ ten"ency 'because it is nee"e" for synthesis of many clottin$ factors) rather than anemia. ?I) (*!lanation: 7he correct answer is A. Frinary retention is most often cause" by an anatomic obstruction to urine outflow. In men, this is often "ue to beni$n !rostatic hy!ertro!hy ' &2). Women have a variety of causes. 7he common manifestation of !rolon$e" urinary retention is bilateral hy"rone!hrosis "ue to urinary retention an" !ressure increases in the urinary system. 7he bla""er is a very muscular or$an. Increases in !ressure "o not cause bla""er "ilation 'choice ), but rather, hy!ertro!hy. la""er "yskinesis 'choice C), like ventricular "yskinesis, woul" most likely be seen in an area of focal bla""er in<ury. 7his most often results from e*ternal im!in$ement on the bla""er wall. Fnilateral hy"rone!hrosis 'choice %) is most often encountere" in cases of ureteral obstruction in which only one ki"ney suffers from the increase" !ressure. Freteral "ilation 'choice () woul" not be seen in urinary retention until very late in the course. It is an uncommon fin"in$ because most !atients !resent to a !hysician !rior to this late sta$e. />) (*!lanation: 7he correct answer is (. 2ea"ache of su""en onset 'Hthun"ercla!H hea"ache), ra!i" "eterioration of mental status an" bloo" in the C)- are virtually "ia$nostic of ru!ture" berry aneurysms. Note the characteristic hy!er"ensity on C7 of the su!rasellar cistern, in"icatin$ bloo" in the subarachnoi" s!ace. 6u!ture of a berry aneurysm is the most common cause of subarachnoi" blee"in$. erry aneurysms "evelo! as a result of con$enital weakness at branchin$ !oints of the arteries in the circle of Willis. 7hese out!ouchin$s ten" to e*!an" !ro$ressively, but in most cases they remain asym!tomatic. 2y!ertension facilitates "evelo!ment an" ru!ture of berry aneurysm. One thir" of !atients recover, one thir" "ie, an" one thir" "evelo! re4blee"in$. 6a!i" onset of coma is an ominous si$n.

Amyloi" an$io!athy4relate" hemorrha$e 'choice A) woul" manifest as a cortical4base" hematoma in a lobar "istribution. It is "ue to accumulation of AS amyloi" in bloo" vessel walls. Cavernous sinus thrombosis 'choice ) is a rare com!lication of con"itions lea"in$ to coa$ulation abnormalities, such as se!sis, anti!hos!holi!i" antibo"y syn"rome, an" leukemias. It lea"s to hemorrha$ic infarction of lar$e areas of hemis!heric $ray an" white matter. 2emorrha$ic infarction 'choice C) usually "evelo!s as a result of embolic occlusion of an intra!arenchymal artery. It $ives rise to a hy!er"ense we"$e4sha!e" area in a cortical fiel" corres!on"in$ to a s!ecific vascular territory. &ituitary a!o!le*y 'choice %) refers to hemorrha$e in the !ituitary $lan". It may occur in the settin$ of a lar$e !ituitary a"enoma or in !re$nancy. It manifests with ra!i" onset of !anhy!o!ituitarism.

USMLE Step 2 Practice Test Block & Name: Instructions: Answer the questions below to the best of your ability. When you finish the test, click the Check button at the bottom to view the results.

1. A 1!year!ol" 'el"er presents complaining of se)ere fatig#e an" the onset of 9a#n"ice. /e has a kno'n history of hepatitis 2% 'hich he ac*#ire" after +? "r#g #se 24 years earlier. >)er the past < months% he has "e)elope" ascites an" has ha" t'o a"missions to the hospital for esophageal )ariceal $lee"ing. >n physical e.amination% he is icteric 'ith $itemporal 'asting an" m#ltiple stigmata of chronic li)er "isease. >n a$"ominal e.amination% his li)er is ( cm in the mi"cla)ic#lar line% an" splenomegaly is present. There is near!tense ascites an" mo"erately se)ere lo'er e.tremity e"ema% 'hich e.ten"s to the mi"!calf. La$oratory res#lts re)eal an al$#min of 2.1 g7"L% total $ilir#$in of 12.1 mg7"L% an" a prothrom$in time of 18 secon"s. =hich of the follo'ing is the most appropriate therapy for this patientA6 +nterferon B6Ai$a)irin 26+nterferon pl#s ri$a)irin @6Mesoca)al sh#nt

E6 E)al#ation for li)er transplantation :ormal La$s 2.A 2 !year!ol" man of Eastern E#ropean Re'ish "escent is $eing e)al#ate" for a right hip fract#re an" splenomegaly. /e reports rec#rrent ac#te episo"es of $one pains for the past years. 2omplete $loo" co#nt sho's; Erythrocyte co#nt..............2%444%4447mm& Le#kocyte co#nt................&%&447mm& Platelet co#nt....................(4%4447mm& F!ray films "emonstrate m#ltiple osteolytic lesions in the )erte$ral col#mn an" fem#rs. A $one marro' aspirate re)eals cl#sters of histiocytes sho'ing fi$rillary cytoplasm 'ith a typical 0cr#mple" tiss#e paper0 appearance. The fi$rillary material in the cytoplasm is PAS! positi)e. =hich of the follo'ing is the most effecti)e treatment a)aila$le for this con"itiona6A"ministration of pyri"o.ine an" folate $6A"ministration of hematin c6Allogeneic $one marro' transplantation "62ytoto.ic "r#gs s#ch as hy"ro.y#rea e6/igh!car$ohy"rate "iet Aeplacement therapy 'ith algl#cerase g6Splenectomy :ormal La$s &. A 4!year!ol" 'oman complains of se)ere lo'er a$"ominal pain an" "istention. The symptoms $egan appro.imately 24 ho#rs ago% 'hen her a$"omen $ecame )isi$ly s'ollen an" she "e)elope" na#sea an" )omiting. She has not mo)e" her $o'els o)er the past 24 ho#rs. >)er the past 4 months% she has lost 14 po#n"s an" has note" progressi)e symptoms of constipation. >n se)eral occasions% she has note" $loo" mi.e" in 'ith her $o'el mo)ements% 'hich ha)e $ecome thinner in cali$er. She "enies any recent tra)el% #se of anti$iotics% or fe)ers. >n physical e.amination% she appears ac#tely #ncomforta$le an" has a temperat#re of &1.& 2 3144.8 56. /er a$"omen is "iff#sely "isten"e" an" ten"er to palpation in the left lo'er *#a"rant. There are hyperacti)e r#shing $o'el so#n"s. >n rectal e.amination% her stool is $ro'n an" g#aiac positi)e. An o$str#cti)e series re)eals m#ltiple small $o'el air

fl#i" le)els an" a "ilate" colon pro.imal to the sigmoi" colon. =hich of the follo'ing is the most likely "iagnosisa6Ame$ic a$scess $62olon polyp c6@i)ertic#litis "6@i)ertic#losis e6Sigmoi" carcinoma :ormal La$s 4.A pre)io#sly healthy 4 !year!ol" 'oman is in a motor )ehicle acci"ent. She s#ffers m#ltiple internal an" e.ternal in9#ries from 'hich she is still acti)ely $lee"ing 'hen reache" $y an am$#lance. /er $loo" press#re is 4724 mm /g. =hich of the follo'ing 'o#l" $e the most likely res#lt of a complete $loo" co#nt performe" in the emergency room 1 min#tes latera6Ae" cell co#nt 1. microliter $6Ae" cell co#nt 1. microliter c6Ae" cell co#nt 2. microliter "6Ae" cell co#nt 2. microliter e6Ae" cell co#nt 4. microliter :ormal La$s million per microliterG 'hite cell co#nt 2444 per million per microliterG 'hite cell co#nt <444 per million per microliterG 'hite cell co#nt &444 per million per microliterG 'hite cell co#nt <444 per million per microliterG 'hite cell co#nt <444 per

.A 24!year!ol" male 'ith a history of nephrotic syn"rome is seen in clinic after a renal $iopsy. /e presente" a 'eek ago 'ith anasarca% an" foamy #rine 'itho#t hemat#ria. /is $loo" press#re is 1447<4 mm /g. E.amination re)eals perior$ital e"ema% 'ith clear l#ngs% a normal car"iac e.amination an" 4E e"ema an" anasarca. /is 24 !ho#r #rine contains 1 grams of protein an" his ser#m creatinine. is 4.1 mg7"L. /is #rinalysis sho's 4E protein% $#t no re" cells or casts. /is renal $iopsy is

rea" as minimal change "isease. =hich of the follo'ing is the most appropriate ne.t step in managementa6+ntra)eno#s p#lse cyclophosphami"e $6>ral pre"nisone c6>$ser)ation "6>ral 5M 4< 3Prograft6 e6Aenal transplant
:ormal La$s

<. A 41!year!ol" man presents 'ith recent onset of comple. partial seiD#res. 2T scan an" MA+ of the hea" 3see pict#res6 sho' a 'ell! circ#mscri$e"% "#ral!$ase" mass in the right mi""le cranial fossa. The t#mor "isplaces posteriorly the tip of the temporal lo$e% 'itho#t e)i"ence of intraparenchymal infiltration. The a"9acent cal)arial $one is mil"ly thickene". =hich of the follo'ing is the most likely "iagnosisa62raniopharyngioma $6Clio$lastoma m#ltiforme c6Meningioma "6Paget "isease e6Sch'annoma :ormal La$s (. A !year!ol" 'oman 'ith long history of s#n$athing "e)elops an erythemato#s% slightly raise"% 2!cm patch of skin on her cheek. The in)ol)e" area has a ro#gh% )ery a"herent% yello'!$ro'n scale. The lesion is treate" 'ith t'ice "aily application of !fl#oro#racil cream for a total of 4 one!'eek cycles alternate" 'ith no treatment 'eeks. @#ring this treatment% the lesion an" s#rro#n"ing skin initially $ecome marke"ly erythemato#s $#t then e)ent#ally heal 'itho#t scarring. =hich of the follo'ing is the most likely "iagnosisa6Actinic keratosis $6Basal cell carcinoma c6Bo'enoi" pap#losis "6Meratoacanthoma e6 Malignant melanoma :ormal La$s

1. A &4!year!ol" me"ical resi"ent presents to her physician complaining of e.cessi)e fatig#e. She is concerne" $eca#se se)eral 'eeks )acation has faile" to alle)iate her fatig#e. She fin"s that she is some'hat tire" thro#gho#t the "ay an" goes to sleep almost imme"iately after eating "inner. She "i" not ha)e this pro$lem "#ring her first 2 years of resi"ency an" is concerne" a$o#t 'hy% in her last year% she sho#l" $e so e.ha#ste". /er re)ie' of systems is other'ise #nremarka$le. She has reg#lar menses an" she "enies pregnancy. She has ha" no fe)ers or s'eats% an" she has not tra)ele" recently. /er physical e.amination re)eals normal )ital signs an" is 'itho#t any significant fin"ings. A re)ie' of her la$oratory "ata re)eals a hematocrit of &1B% 'ith a mean corp#sc#lar )ol#me of <8 fL. =hich of the follo'ing is the most likely ca#se of her anemiaa6 5olate "eficiency $6/emolysis c6+ron "eficiency "6Aenal "isease e6 ?itamin B12 "eficiency :ormal La$s 8.A 2(!year!ol" man )isits his primary care physician for a ro#tine check!#p. /e has no me"ical history an" has starte" 'orking in a hospital < months ago. As a health screen% he nee"s a p#rifie" protein "eri)ati)e 3PP@6 test an" an /+? test. Three "ays later% the PP@ site is in"#rate". /e has ne)er $een gi)en the Bacille 2almette!C#erin 3B2C6 )accine. =hich of the follo'ing feat#res 'o#l" ten" to arg#e against chemoprophyla.is 'ith isoniaDi"a6E.pos#re to a patient 'ith acti)e t#$erc#losis $6/+? positi)e% in"#ration of mm c6/+? negati)e% in"#ration of 4 mm "6:egati)e PP@ 1 year earlier e6 Positi)e PP@ 1 year earlier :ormal La$s 14. A (8!year!ol" man% 'eighing ( kg% 'ith emphysema is int#$ate" in the intensi)e care #nit $eca#se of respiratory fail#re after "e)eloping a"#lt respiratory "istress syn"rome secon"ary to an Escherichia coli

$acteremia after an #ntreate" #rinary tract infection. /is )entilator is set to a respiratory rate of 247min% a ti"al )ol#me of ( 4 mL7$reath% an" a p>2 of 144B. +f these settings are contin#e" for the ne.t (2 ho#rs% the patient is at most increase" risk for the "e)elopment of 'hich of the follo'ing complicationsa62ongesti)e heart fail#re $6R#g#lar )eno#s "istention c6P#lmonary em$ol#s "6P#lmonary fi$rosis e6 Tension pne#mothora. :ormal La$s 11.A <4!year!ol" 'oman comes to the emergency "epartment $eca#se of se)ere 'rist pain after a fall. She says that she fell on an o#tstretche" han" 2 "ays ago% an" has $een e.periencing se)ere pain e)er since. She has ha" & episo"es of nephrolithiasis% chronic m#scle 'eakness% an" )ag#e a$"ominal complaints o)er the past fe' months. Physical e.amination is normal. A .!ray film of her 'rist sho's a fract#re of the "istal ra"i#s an" osteopenia. La$oratory st#"ies sho'; So"i#m.............1&8 mE*7L 2hlori"e ...........84 mE*7L Potassi#m........4. mE*7L Bicar$onate.....2& mE*7L 2alci#m ..........1& mE*7L Uric aci"......... mg7"L A splint is place" on her 'rist an" a follo'!#p )isit is sche"#le". =hich of the follo'ing is the most likely e.planation for these fin"ingsa6@ecrease" e.cretion of #ric aci" $6@e)elopment of thyroi"!stim#lating hormone receptor anti$o"ies c6Ele)ation of circ#lating al"osterone "6Ele)ation of circ#lating cortisol e6Ele)ation of circ#lating parathyroi" hormone :ormal La$s 12. An 11!year!ol" man is taken to the emergency room $y his family 'hen he "e)elops )ery se)ere hea"ache accompanie" $y high fe)er. >n physical e.amination% the patient is incoherent an" "emonstrates n#chal rigi"ity. 2S5 sho's gram!negati)e "iplococci. =hen talking to the

family% the physician learns that the patient has ha" episo"es of meningococcal meningitis in the past% the earliest $eing at age <. +mm#no"eficiency relate" to impaire" f#nction of 'hich of the follo'ing sho#l" $e s#specte"a6B cells $62omplement factors c6Eosinophils "6:e#trophils e6 T cells :ormal La$s 1&. A !year!ol" man presents 'ith se)ere lo'er a$"ominal pain an" s#prap#$ic press#re. Earlier in the "ay% 'hile #n"ergoing coronary catheteriDation for #nsta$le angina% he ha" transient hypotension an" $ra"ycar"ia that 'as s#ccessf#lly treate" 'ith atropine an" epinephrine. /e has not #rinate" since the proce"#re. The n#rse notes that he is tachycar"ic% 'ith a reg#lar p#lse of 1147min. A re)ie' of systems taken on a"mission re)eals noct#ria an" a 'eak #rinary stream. 5rom a historical conte.t% 'hich of the follo'ing is the most likely etiology of his a$"ominal paina62holesterol em$oli syn"rome $62ontrast!in"#ce" ac#te renal fail#re c6:ephrolithiasis "6Pyelonephritis e6Urinary retention :ormal La$s 14. A 4&!year!ol" 'hite 'oman presents to the emergency "epartment 'ith 1 "ay of increasingly se)ere pain localiDe" to the right #pper *#a"rant an" ra"iating to the right lo'er scap#la. She has also $een e.periencing na#sea an" )omiting. The 'oman has ha" similar% $#t mil"er% episo"es of pain in the past% 'hich ha" resol)e" spontaneo#sly in a fe' "ays. Physical e.amination "emonstrates in)ol#ntary g#ar"ing of a$"ominal m#scles on the right. The gall$la""er is palpa$le. =hich of the follo'ing is the most appropriate ne.t step in "iagnosisa6 2T scan $6En"oscopic retrogra"e cholangiography c6Esophagogastro"#o"enoscopy "6MA+ scan

e6 Ultraso#n" :ormal La$s 1 . A & !year!ol" 'oman complains of increasing 'eakness an" shortness of $reath. A complete $loo" co#nt 'ith "ifferential in"icates a megalo$lastic anemia. /er )itamin B12 le)el is fo#n" to $e lo'. She is also fo#n" to ha)e hypothyroi"ism an" "ia$etes an" is gi)en the "iagnosis of chronic type A gastritis. =hich of the follo'ing is associate" 'ith this illnessa6 Antral in)ol)ement $6@ecrease" ser#m gastrin le)el c6/elico$acter pylori infection "6:on!steroi"al anti!inflammatory "r#gs 3:SA+@s6 e6Parietal cell anti$o"y :ormal La$s 1<. A 4 !year!ol" 'oman is a"mitte" to the hospital after $eing "iagnose" 'ith post!streptococcal glomer#lonephritis. The patient 'as 'ell #ntil & 'eeks ago% 'hen she notice" se)ere $o"y s'elling an" $loo" in her #rine. T'o 'eeks prior to that% she ha" a se)ere #pper respiratory infection% 'ith a sore throat an" high fe)ers. She so#ght no treatment at that time. She finally presente" to the emergency "epartment $eca#se her shoes no longer fit. >n a"mission% her creatinine le)el 'as <.< mg7"L% an" she 'as note" to ha)e 1 g of protein in her #rine per "ay. The patient ha" a "ialysis catheter place"% an" hemo"ialysis 'as starte" on hospital "ay & for hyperkalemia. =hich of the follo'ing "ietary s#pplements or regimens 'ill most likely $enefit this patienta62alci#m s#pplementation $6/igh!protein "iet c6Lo'!fat "iet "6?itamin 2 s#pplementation e6?itamin E s#pplementation :ormal La$s

1(.A 44!year!ol" man is $ro#ght #nconscio#s to the emergency "epartment ho#rs after a motor )ehicle acci"ent% in 'hich he 'as e9ecte" o#t of the car an" hit his hea" on the pa)ement. /e lost conscio#sness & ho#rs follo'ing the tra#ma. >n a"mission% the patient is #nresponsi)e to )er$al or painf#l stim#li% an" his left p#pil is fi.e" an" "ilate". F!ray films of his hea" sho' a close" left cal)arial fract#re. 2T scan "emonstrates e)i"ence of intracranial $lee"ing. +n 'hich of the follo'ing compartments is the $lee"ing most likely to ha)e "e)elope"a6 Epi"#ral space $6. S#$"#ral space c6S#$arachnoi" space "6+ntracere$ral e6+ntra)entric#lar :ormal La$s 11. A 44!year!ol" 'oman is e)al#ate" $y a "ermatologist $eca#se she has many pigmente" lesions on her $o"y. E.amination of the skin an" scalp "emonstrates o)er 144 in"i)i"#al lesions% most of 'hich )ary in siDe from to 12 mm. They are fo#n" all o)er her $o"y% $#t most commonly on s#n!e.pose" skin. They are pre"ominately ro#n" in color% $#t some ha)e s#$tly notche" $or"ers or are slightly asymmetrical. The 'oman,s entire skin s#rface is photographe"% an" 'hen the photographs are repeate" si. months later% no change in appearance of any of the lesions is note". =hich of the follo'ing is the most likely "iagnosisa62ompo#n" ne)i $6@ysplastic ne)i c6/alo ne)i "6Lentigos e6Malignant melanomas :ormal La$s 18. A 48!year!ol" man 'ith a history of hypertension s#staine" an anterior 'all myocar"ial infarction 2 'eeks ago. The patient #n"er'ent s#ccessf#l throm$olysis 'ith complete resol#tion of symptoms. Echocar"iography sho's an e9ection fraction of 2B. A pre!"ischarge stress test sho'e" no symptoms or E2C e)i"ence of ischemia 'ith

ma.imal effort. =hich of the follo'ing therapies has $een sho'n to increase s#r)i)al in this sit#ationa6Angiotensin!con)erting enDyme inhi$itor $6Beta $locker c6@igo.in "6Loop "i#retic e6=arfarin :ormal La$s 24. A 2(!year!ol" actor presents 'ith a s'ollen left knee. The pain $egan &< ho#rs earlier an" has limite" his a$ility to perform in his c#rrent play. /e has ha" a fe)er an" shaking chills o)er the past 24 ho#rs. Physical e.amination re)eals a temperat#re of &1.8 2 3141.8 56 an" a p#lse of 1447min. The remain"er of the physical e.amination is #nremarka$le e.cept for a s'ollen% erythemato#s knee 'ith an o$)io#s eff#sion. There is limite" range of motion. An arthrocentesis re)eals 84%4447mm& 'hite $loo" cells an" 12B ne#trophils. A Cram,s stain re)eals many ne#trophils% an" no organisms are seen. PolariDing microscopy re)eals no crystals. =hich of the follo'ing is the most appropriate initial step in therapya6 2eftria.one $62olchicine c6+n"omethicin "6:afcillin e6:afcillin an" ceftria.one :ormal La$s 21. A 41!year!ol" man is $ro#ght to the emergency "epartment $y parame"ics after $eing fo#n" conscio#s at the scene of a motor )ehicle acci"ent. /e ha" $een the passenger in a high!spee" collision an" 'as thro'n a "istance of 24 feet thro#gh the front 'in"shiel". An e)al#ation re)eals fract#res of $oth fem#rs% the pel)is% the left ti$ia% an" the left h#mer#s. The patient is a"mitte" to the intensi)e care #nit for o$ser)ation. T'enty!fo#r ho#rs later he $ecomes conf#se" an" marke"ly "yspneicG 144B o.ygen is a"ministere" )ia face mask% $#t he remains 'ith a p>2 of 4 mm /g. :#mero#s petechiae also ha)e "e)elope" "iff#sely on all fo#r e.tremities an" on his tr#nk. =hich of the follo'ing is the most likely ca#se of his clinical con"ition-

a6Aspiration pne#monia $62ar"iogenic p#lmonary e"ema c65at em$olism "6P#lmonary em$olism e6 Tension pne#mothora. :ormal La$s 22. A !year!ol" man cons#lts a physician $eca#se of episo"ic 'eakness an" paresthesias. >n one occasion% he e.perience" transient paralysis. The patient has also $een e.periencing poly#ria an" poly"ipsia. ?ital signs "emonstrate a $loo" press#re of 1447144 mm /g 'ith normal temperat#re% p#lse% an" respirations. The remain"er of the physical e.amination is #nremarka$le an" specifically sho's no significant a$normal ne#rologic fin"ings an" no peripheral e"ema. Ao#tine screening chemistry st#"ies are remarka$le only for a ser#m potassi#m of 2.1 mE*7L. 2T scan "emonstrates a small a"renal mass. =hich of the follo'ing is the most appropriate pharmacotherapy to treat this patient,s hypertensiona62aptopril $65#rosemi"e c6/y"rochlorothiaDi"e "6Propanolol e6 Spironolactone :ormal La$s 2&.A 1!year!ol" man presents to the emergency "epartment 'ith a$"ominal pain. /e 'as 'ell #ntil 2 "ays ago% 'hen he $egan to e.perience se)ere right #pper *#a"rant pain% ra"iating to the epigastric region. /e reports temperat#res to &1.& 2 3141 56 an" some na#sea an" )omiting. /is temperat#re is no' &8.1 2 3142.& 56% $loo" press#re is 1&47(4 mm /g% an" p#lse is 847min. Physical e.amination re)eals ten"erness in his right #pper *#a"rant% 'ith a$r#pt cessation of inspiration on "eep palpation of his right #pper *#a"rant. =hich of the follo'ing is the most appropriate management for this patienta6+? fl#i"s an" o$ser)ation $6+? anti$iotics an" o$ser)ation c6A"mission to a s#rgical ser)ice for ne.t "ay s#rgery "6Urgent s#rgical e)al#ation for imme"iate s#rgery e6Urgent perc#taneo#s "rainage :ormal La$s

24. A &!year!ol" man is a"mitte" to the hospital for fe)er an" a$"ominal pain. /e has a history of cirrhosis an" is kno'n to $e hepatitis 2 positi)e. /e 'as "iagnose" 'ith cirrhosis fo#r years ago. /e "enies any alcohol or to$acco #se. /is only me"ications are spironolactone an" propranolol. /e reports that fi)e "ays ago% he ha" fe)er to 142 "egrees 5 an" the gra"#al onset of "iff#se a$"ominal pain. >n e.amination% his temperat#re is &1.& 2 3141 56% his $loo" press#re is 1447 4 mm /g% an" his p#lse is 1147min an" reg#lar. /is l#ngs are clear% he has n#mero#s spi"er angiomata on his thora. an" $ack% an" his a$"omen is massi)ely "isten"e"% 'ith shifting "#llness $y perc#ssion. La$oratory st#"ies sho'; Le#kocytes..............1&%2447mm& /ematocrit..............&&B Prothrom$in time.....1 .2 secon"s Al$#min..................4.1 g7"L So"i#m...................1& mE*7L Potassi#m...............4.( mE*7L =hich of the follo'ing is the most appropriate ne.t step in "iagnosisa6A$"ominal #ltraso#n" $6A$"ominal 2T scan c6Electrocar"iogram "6A$"ominal paracentesis e6L#m$ar p#nct#re :ormal La$s 2 . A &4!year!ol" 'oman complains of "iffic#lty s'allo'ing $oth li*#i"s $oth soli"s for the past < months. She has a history of hypertension an" Aayna#" phenomenon. A physical e.amination re)eals tight skin on her face an" on the "orsal s#rfaces of $oth han"s. =hich of the follo'ing manometric fin"ings 'o#l" most likely $e fo#n" in this patientPeristalsis in $o"y Aesting lo'er of esophag#s press#re esophageal sphincter 3LES6 A . @ecrease" B. @ecrease" 2. +ncrease" :ormal +ncrease" :ormal

@. @ecrease" E. +ncrease" a6 $6 c6 "6 e6 :ormal La$s

:ormal +ncrease"

2<. A 18!year!ol" man cons#lts a physician a$o#t a 2 cm neck mass. The patient has no systemic symptoms at that time. The mass is resecte" an" pro)es to $e a lymph no"e sho'ing replacement of normal follicles $y sheets of a mi.e" pop#lation of cells incl#"ing histiocytes% lymphocytes% moncytes% plasma cells% an" eosinophils. Scattere" large $in#cleate cells are also seen. These cells are positi)e for 2@1 an" 2@&4 on imm#nohistochemical staining. =hich of the follo'ing is the most likely "iagnosisa6 B#rkitt lymphoma $6/airy cell le#kemia c6/o"gkin "isease "6Mycosis f#ngoi"es e6=al"enstrom macroglo$#linemia :ormal La$s 2(. A (4!year!ol" man presents to his physician 'ith an episo"e of apparently $loo"y #rine. /e "enies prior episo"es an" has $een pre)io#sly healthy. /e is not on any me"ications. +n the office% a #rinalysis confirms gross hemat#ria 'itho#t protein#ria or casts. The patient "enies any pain. The physical e.amination is normal. =hich of the follo'ing is the most appropriate ne.t stepa6Pel)ic 2T scan $6Trimethoprim!s#lfametho.aDole c62ystoscopy "6Aenal angiogram e6 Transrectal prostatic $iopsy :ormal La$s

21. A & !year!ol" man 'ho 'orks as an in)estment cons#ltant in a $rokerage firm comes to the physician $eca#se of "aily hea"aches o)er the past 2 'eeks. /e reports that the hea"aches $egin in the morning an" last for the 'hole "ay. The pain feels like a s*#eeDing tightness aro#n" his hea" an" is partic#larly intense in the $ack of the neck. /e "oes not "rink alcohol or smoke% $#t he "oes "rink many c#ps of coffee "#ring the "ay. /e a"mits that recently he has $een #n"er tremen"o#s stress $eca#se of the )olatility of the stock market an" pro$lems 'ith his clients. Physical e.amination is normal. =hich of the follo'ing is the most appropriate ne.t step in managementa6:onsteroi"al anti!inflammatory "r#g trial $6S#matriptan trial c6Psychiatric referral "6L#m$ar p#nct#re e62T of the hea" MA+ of the hea" :ormal La$s 28.A mi""le age" man "e)elops a sclero"erma!like illness 'ith thickening of the skin. The thickene" skin is most marke" o)er the anterior s#rfaces of the e.tremities an" has a characteristic 0orange peel0 config#ration. The man has not e.perience" Aayna#",s phenomenon. :o telangiectasias an" no calcinosis is seen. These skin changes sho#l" s#ggest 'hich of the follo'ing "iagnosesa62AEST syn"rome $6@ermatomyositis c6Eosinophilic fasciitis "6Polymyalgia rhe#matica e6Polymyositis :ormal La$s &4. A 48!year!ol" man presents to his physician for follo'!#p of a pre)io#s fasting total ser#m cholesterol le)el of 288 mg7"L. The patient is other'ise 'ell% 'ith only mil" hypertension that he is attempting to control 'ith e.ercise an" a lo'!fat% lo' cholesterol "iet. /e has a family history of ischemic heart "isease on his father,s si"e% an" he smokes a pack of cigarettes "aily. The patient is a plant manager an" is happy

'ith his 9o$ an" reports no home!life iss#es 'ith his 'ife or three sons. A lipi" profile 'as "ra'n at his last )isit% an" he presents to"ay for re)ie' of those res#lts% 'hich are as follo's; Total cholesterol 2&4 mg7"L /@L 4 mg7"L L@L 144 mg7"L =hich of the follo'ing is the most appropriate inter)ention at this timea6:o inter)ention is in"icate" $6E"#cate the patient a$o#t "iet an" e.ercise c6E"#cate the patient a$o#t "iet an" e.ercise an" repeat the tests 'ithin a year "6+nitiate a man"atory lo'!fat "iet for the patient e6+nitiate "r#g therapy for control of his hyperlipi"emia :ormal La$s &1.A 48!year!ol" 'aitress presents 'ith painf#l s'elling of the left knee. The symptoms $egan o)er the past 41 ho#rs an" ha)e limite" her a$ility to 'ork. She has "e)elope" a fe)er o)er the past 24 ho#rs $#t "enies rigors% co#gh% rash% or hea"aches. >n physical e.amination% her temperat#re is &1.& 2 3144.8 56% $loo" press#re is 11<7(2 mm /g% p#lse is 8<7min% an" respirations are 1<7min. There is a palpa$le eff#sion aro#n" the left knee caps#le in association 'ith erythema an" 'armth. =hich of the follo'ing 'o#l" $e the most appropriate ne.t step in managementa62omplete $loo" co#nt $6Left knee .!ray film c6Left knee arthrocentesis "6Left knee MA+ e6Left knee arthroscopy :ormal La$s &2A !year!ol" man complains of shortness of $reath an" 'heeDing. /e has no' "e)elope" 'orsening "yspnea on e.ertion. /e reports a past history of smoking more than three packs of cigarette a "ay for the past 44 years. /e has ne)er $een int#$ate" $#t has $een a"mitte" in the past for treatment 'ith $roncho"ilators. >n physical e.amination% he has poor air mo)ement in his l#ngs. A chest .!ray film sho's flattening of

the "iaphragm an" a $arrel chest. /e #n"ergoes p#lmonary f#nction testing. =hich of the follo'ing patterns 'ill $e seen on his p#lmonary f#nction testsa6 @ecrease" resi"#al )ol#me7total l#ng capacity 3A?7TL26 $6@ecrease" A? c6 :ormal force" e.piratory )ol#me in 1 secon"7force" )ital capacity 35E?175?26 "6Ae"#ce" 5E?175?2 e6Ae"#ce" ?2 an" TL2 :ormal La$s &&. A 11!year!ol" 'oman is $ro#ght to the clinic $y her parents% 'ho report that she has $een increasingly 'eak an" fatig#e" for the past < months. She has no prior me"ical history an" "enies any me"ication #se. >n physical e.amination% she has a $loo" press#re of 1 7<4 mm /g an" is tachycar"ic 'ith a p#lse of 1147min. La$oratory st#"ies are nota$le for a normal so"i#m% potassi#m of 2.2 mE*7L% an" a plasma $icar$onate of 44 mE*7L. =hich of the follo'ing is the most likely ca#se of her hypokalemic alkalosisa62hronic "iarrhea $62#shing syn"rome c6Licorice ingestion "6Primary al"osteronism e6S#rreptitio#s )omiting :ormal La$s &4.A (1!year!ol" man presents to the hospital 'ith an episo"e of $right re" $loo" per rect#m. The patient reports that% a fe' ho#rs ago% he passe" a grossly $loo"y $o'el mo)ement. The passage 'as associate" 'ith some cramping% lo'er a$"ominal pain. The patient,s past me"ical history is significant for coronary artery "isease an" a myocar"ial infarction & years ago. /e is poorly compliant 'ith his $eta $locker an" "i#retic therapy% an" his $loo" press#res ha)e r#n aro#n" 14471 mm /g. =hile the patient is on the floor% he has another episo"e of large! )ol#me% $right re" $loo" per rect#m. /is $loo" press#re is 1447<4 mm /g 'hile s#pine% an" his p#lse is 1247min. /is hematocrit is 21B. /e then $egins to complain of s#$sternal chest tightness ra"iating to his left sho#l"er. An E2C sho's ne' T 'a)e in)ersions in the anterior lea"s.

=hich of the follo'ing is the most appropriate inter)ention at this 9#nct#rea6A"ministration of aspirin $y mo#th $6A"ministration of a $eta $locking agent c6A"ministration of nitroglycerin s#$ling#ally "6A"ministration of nitroglycerin topically e6Bloo" transf#sion :ormal La$s & . A &1!year!ol" 'oman presents complaining of a $#rning "iscomfort in the s#$sternal region. The symptoms are 'orse follo'ing any meal an" after reclining for sleep at night. She has a history of Aayna#" phenomenon an" mil" hypertension. >n physical e.amination% her $loo" press#re is 1<2784 mm /g% p#lse is (17min% an" respirations are 1<7min. She is afe$rile. There are m#ltiple facial telangiectasias on $oth cheeks% an" she has ta#t skin on $oth han"s. The remain"er of her m#sc#loskeletal e.amination is #nremarka$le. =hich of the follo'ing is most likely responsi$le for her chest "iscomforta62oronary )asospasm $62ostochon"ritis c6Esophageal hypomotility "6/ypertension of the lo'er esophageal sphincter e6 P#lmonary fi$rosis :ormal La$s &<. A &4!year!ol" 'aiter 'ith a kno'n history of asthma presents 'ith complaints of se)ere shortness of $reath% co#gh% an" 'heeDing. /e arri)es at the emergency "epartment an" appears e.tremely "yspneic. /is $loo" press#re is 1 4714 mm /g% p#lse is 1447min% an" respirations are &27min. =ith inspiration% his $loo" press#re falls to 1127(4 mm /g. >n l#ng e.amination% there are lo#"% high!pitche" 'heeDes an" a prolonge" e.piratory phase. =hich of the follo'ing physical e.amination fin"ings is of most significance in e)al#ating this patienta6/ypertension $6Lo#" 'heeDing c6Prolonge" e.piratory phase "6P#ls#s para"o.#s e6 Tachycar"ia

:ormal La$s &(.An 11!year!ol" army recr#it goes on a 24!ho#r force" march. =hen he #rinates t'o ho#rs follo'ing the en" of the march% his #rine is re"! tinge". /e goes to the infirmary 'here a "ipstick test of #rine is positi)e for hemoglo$in. =hen the $loo" is sp#n in the small hematocrit machine a)aila$le in the infirmary% the o)erlying ser#m has a re" color an" the hematocrit is & B. Peripheral smear sho's a fe' o""!shape" AB2 fragments. This patient,s anemia is most likely "#e to 'hich of the follo'inga6Ac#te le#kemia $6A#toimm#ne attack on re" cells c6+na"e*#ate hemoglo$in synthesis "6+ron o)erloa" e6Mechanical in9#ry to re" cells :ormal La$s &1. A (4!year!ol" man complains of lo'er a$"ominal pain. /e has $ecome a'are of a peri#m$ilical mass "#ring that time. /is past me"ical history is significant for hypertension an" type 2 "ia$etes. /e has a history of "i)ertic#litis 1 year ago. >n physical e.amination% he is afe$rile an" appears mil"ly #ncomforta$le. >n a$"ominal e.amination% there is a palpa$le mass e.ten"ing from the p#$ic ram#s to the #m$ilic#s 'ith a smooth conto#r. La$oratory res#lts re)eal a BU: of 14 mg7"L an" a creatinine of 1.1 mg7"L. =hich of the follo'ing is the most appropriate ne.t step in the management of this patienta6 Mast#r$ation $6Ureteral catheteriDation c6A$"ominal sonogram "6A$"ominal an" pel)ic 2T scan e6Perc#taneo#s nephrostomy :ormal La$s &8. A < !year!ol" man comes to the physician $eca#se of increasing 'eakness an" fatig#e for < months. /e "enies 'eight loss. /is temperat#re is &( 2 381.< 56% $loo" press#re is 124714% p#lse is (47min% an" respirations are 147min. The patient speaks 'ith a soft an"

monotono#s tone of )oice. /e 'alks 'ith a rigi" post#re an" limite" arm s'ing. Physical e.amination re)eals a fine tremor of the fingers% 'hich manifests at rest an" "isappears 'ith mo)ement. +ncrease" m#scle tone is appreciate" 'hen his arms are passi)ely fle.e". Sensation an" m#scle strength are other'ise intact. =hich of the follo'ing "r#gs is most likely to pro"#ce a marke" impro)ement in this patient,s symptomsa62loDapine $62orticosteroi"s c6/aloperi"ol "6Le)o"opa e6Tacrine :ormal La$s 44.A & !year!ol" 'oman cons#lts a physician a$o#t a changing mole on her neck. The lesion is a mostly flat% 2!cm "iameter patch 'ith )arying colors from re" to $ro'n to $lack. +n se)eral areas% small no"#les are seen 'ithin the patch. M#ch of the peripheral circ#mference of the lesion has a poorly "efine"% 0feathery0 e"ge. Biopsy "emonstrates a malignant t#mor of melanocytes. The lesion is resecte" 'ith a 'i"e resection margin an" then e.amine" pathologically. =hich of the follo'ing is the most significant prognostic in"icator in this lesiona6Area of in)ol)e" skin $6@egree of atypia of the t#mor cells c6@egree of color )ariation "6@epth of in)asion e6Percentage of circ#mference in)ol)e" 'ith 0feathering0 :ormal La$s 41.A (1!year!ol" man presents to his physician for follo'!#p of a recent emergency "epartment )isit. The patient has a 2!year history of mil" congesti)e heart fail#re in the setting of long!stan"ing hypertension. /e reports that yester"ay he so#ght care at the local emergency "epartment for palpitations an" shortness of $reath. /e 'as tol" that his heart 'as 0fi$rillating0% $#t later% the fi$rillation ha" 0stoppe" on its o'n.0 /is me"ications incl#"e a thiaDi"e "i#retic an" an A2E inhi$itor. >n physical e.amination% he appears 'ell an" in no "istress. /is $loo" press#re is 1&4714 mm /g% an" his p#lse is 1447min an"

reg#lar. /is l#ngs ha)e scant $i$asilar rales% an" no gallops are appreciate". /e has a gra"e 2 holosystolic m#rm#r hear" $est at the ape.. /is 9#g#lar )eno#s press#re 3R?P6 is 14 cm at &4 "egrees. An E2C taken in the office re)eals atrial fi$rillation at a rate of 847min 'ith normal ST segments. =hich of the follo'ing is the most appropriate ne.t step in managementa6 @iscontin#e the A2E inhi$itor $6+nitiate amio"arone therapy c6+nitiate $eta $locker therapy "6+nitiate "igo.in therapy e6+nitiate f#rosemi"e therapy :ormal La$s 42. A <8!year!ol" retire" police officer presents 'ith complaints of e.cr#ciating right foot pain. The pain $egan appro.imately < ho#rs ago% imme"iately on a'aking. >ne night earlier% he ha" gone o#t for a steak "inner an" "rank half of a $ottle of 'ine to cele$rate his recent retirement. /is past me"ical history is significant for hypertension% for 'hich he takes hy"rochlorothiaDi"e. >n physical e.amination% he is afe$rile $#t appears ac#tely #ncomforta$le. There is s'elling an" ten"erness in the right ankle an" in the first right toe. La$oratory tests re)eal a 'hite $loo" cell co#nt of 12%4447mm& an" a hematocrit of &8B. Ser#m electrolytes an" li)er f#nction tests are normal. Uric aci" is 4.1 mg7"L 3normal 2. ! . mg7"L6. =hich of the follo'ing 'ill most likely $e seen on e.amination of the 9oint arthrocentesisa6Cram!negati)e ro"s $6Cram!positi)e cocci pairs an" chains c6:egati)ely $irefringent nee"le!shape" crystals "6:egati)ely $irefringent rhom$oi" crystals e6Positi)ely $irefringent nee"le!shape" crystals :ormal La$s 4&.A <(!year!ol" retire" li$rarian presents to her physician $eca#se of 41 ho#rs of persistent fe)ers% shaking chills% an" a pro"#cti)e co#gh. She is a former smoker 'ho *#it 2 years ago. She is in other'ise goo" health% e.cept for a history of a transient ischemic attack 2 yeas ago. >n physical e.amination% there are coarse rhonchi at the left posterior l#ng $ase 'ith increase" fremit#s an" hyperresonance to perc#ssion. A chest

.!ray film re)eals a lo$ar consoli"ation in the left lo'er posterior l#ng segment. =hich of the follo'ing 'o#l" $e most likely to appear on a sp#t#m Cram,s staina6 Cram!negati)e cocci in chains $6Cram!negati)e ro"s c6Cram!positi)e cocci in chains "6Cram!positi)e cocci in cl#sters e6 Cram!positi)e ro"s :ormal La$s 44. A 4 !year!ol" 'oman is taken to an emergency "epartment after an all "ay car trip% "#ring 'hich she "i" not ha)e a"e*#ate access to fl#i"s. She ha" re*#ire" many $athroom stops% incl#"ing three times 'hen she ha" #rinate" $ehin" a tree $eca#se no $athroom 'as imme"iately a)aila$le. +n the emergency "epartment% she is fo#n" to ha)e se)ere post#ral hypotension. Urine "ipstick is negati)e for gl#cose. She contin#es to pro"#ce large )ol#mes of #rine% an" #rine osmolarity is lo'. >n *#estioning% she has not note" any #n#s#al "rinking pattern% $#t her h#s$an" states that she 0#s#ally goes thro#gh fo#r si.!packs of "iet soft "rink0 e)ery "ay. The patient,s con"ition impro)es 'ith +? fl#i" replacement. ?asopressin in9ection increases her #rine osmolarity $y more than 4B. =hich of the follo'ing is the most likely "iagnosisa62entral "ia$etes insipi"#s $62omp#lsi)e 'ater "rinking c6@ia$etes mellit#s type + "6@ia$etes mellit#s type ++ e6 :ephrogenic "ia$etes insipi"#s :ormal La$s 4 .A (4!year!ol" 'oman presents 'ith a co#gh% "yspnea% chest pain% an" ankle s'elling. She is ill appearing an" cachectic. /er $loo" press#re is 1447(4 mm /g% 'ith no inspiratory "ecrease% an" her p#lse is 1447min. She has a$sent rales on e.amination. She has 9#g#lar )eno#s "istention% 'ith a "ecline "#ring inspiration. /er apical car"iac imp#lse is "ecrease". She has an early thir" heart so#n". /er li)er is enlarge". An E2C re)eals lo' NAS )oltage. A chest .!ray film is clear. The

car"iac silho#ette is enlarge". =hich of the follo'ing fin"ings 'ill most likely appear on an echocar"iograma62ollapse" right )entricle in "iastole $6Large right )entricle c6Pericar"ial eff#sion "6Thick myocar"i#m e6 Thick pericar"i#m :ormal La$s 4<. A &<!year!ol" man 'ith a history of alcoholism is a"mitte" to the hospital after an o)er"ose of acetaminophen. /is 'ife fo#n" him #nresponsi)e on the sofa% an" he 'as $ro#ght to the emergency "epartment. At that time% he 'as fo#n" to ha)e to.ic le)els of acetaminophen in his $loo". /is trachea 'as int#$ate" at that time% an" he 'as a"mitte" to the intensi)e care #nit. The patient s#r)i)es the first 41 ho#rs of hospitaliDation 'ith s#pporti)e care an" is s#ccessf#lly e.t#$ate". /e is no' a'ake an" clinically m#ch impro)e"% e.cept for marke"ly ele)ate" hepatic enDyme le)els an" a rising prothrom$in time. A "epletion in 'hich meta$olic s#$stance is most likely ca#sing these ne' la$oratory fin"ingsa6Alcohol "ehy"rogenase $6 2atalase c6Cl#tathione "6Clycogen e6?itamin M :ormal La$s 4(. A (4!year!ol" man presents 'ith t'o episo"es of passing large amo#nts of $right re" $loo" per rect#m. /e has a long history of chronic constipation 'itho#t a recent change in his $o'el ha$its. A fle.i$le sigmoi"oscopy re)eals m#ltiple large "i)ertic#la in the sigmoi" colon 'ith copio#s amo#nts of fresh $loo" in this region. :o other a$normalities are seen on e.amination of the splenic fle.#re% 'here forme" $ro'n stool is enco#ntere". =hich of the follo'ing is the most likely e.planation for his $lee"inga6@iff#se "escen"ing colon ischemia $6/emorrhage from a single "i)ertic#lar arteriole c6>oDing from an inferior mesenteric )ein

"6Throm$osis of a $ranch of the inferior mesenteric artery e6Throm$osis of internal hemorrhoi"s :ormal La$s 41. A ne' techni*#e for screening patients for lymphomas has $een "e)elope". This in)ol)es taking $loo" from can"i"ates% attaching anti$o"ies to a #ni*#e antigen 32@&&!test6% an" analyDing the $loo" thro#gh flo' cytometry. Those sho'ing positi)ity on the test 'ith the presence of the antigen 2@&&!test are tho#ght to ha)e a lymphoma. This is f#rther corro$orate" 'ith a $one marro' $iopsy. After the appropriate h#man s#$9ect ethics committee has appro)e" this st#"y% the st#"y is #n"ertaken in 144 patients. The res#lts are as follo's; 2@&&!Test Aes#lts Biopsy Positi)e :egati)e Positi)e 2 3a6 443$6 :egati)e 1 3c6 243"6 =hich of the follo'ing is the ne' test,s sensiti)itya6&&.&B $6&1. B c6 (.1B "6<2. B e6 144B :ormal La$s 48. A 4!year!ol" man 'ith chronic renal fail#re is seen in the renal clinic. /e has renal fail#re "#e to chronic glomer#lar "isease. /is $loo" press#re is 'ell controlle"% an" he has no complaints an" no e"ema. /is e.amination is generally normal. >f note% la$oratory analysis re)eals a calci#m of (.8 mg7"L% al$#min of &.( g7"L% phosphate of 1.2 mg7"L 3normal & S4. mg7"L6% parathyroi" hormone of 12 pg7mL% #rea nitrogen of 14 mg7"L% creatinine of (.2 mg7"L. =hich of the follo'ing is the most appropriate initial step in management-

a6+mme"iately start oral ?itamin @ $6Lo'er his phosphate 'ith oral calci#m gi)en 'ith meals c6Lo'er his phosphate 'ith oral al#min#m hy"ro.i"e $in"ers "6Start #rgent "ialysis e6 Urge him to ha)e parathyroi" s#rgery 'ith resection of &. of his glan"s. :ormal La$s 4. A & !year!ol" man is taken to an emergency room $y a frien" after the man starts ha)ing se)ere tonic spasms that ca#se his arms an" legs to flail. The frien" thinks that he is ha)ing seiD#res% $#t there is no loss of conscio#sness an" the man is a$le to "escri$e his me"ical con"ition. The patient,s "iffic#lties ha" $eg#n a "ay an" a half earlier 'hen he notice" 9a' stiffness. This has since progresse" to "iffic#lty s'allo'ing% sore throat% stiff neck% fe)er% chills% an" tonic spasms of his arms an" legs. The patient,s speech is "iffic#lt to #n"erstan" $eca#se he has tro#$le 0making his 9a's 'ork right.0 Mental stat#s is intact. =hich of the follo'ing is the most likely "iagnosisa6 Bot#lism $6@iphtheria c6Lyme "isease "6Pert#ssis e6 Tetan#s
:ormal La$s

Note: Check your own answers before hittin$ the Check button below. When you click the Check button, a browser win"ow will a!!ear that contains a summary of your results. (*!lanations lock 0 (*!lanations

:) (*!lanation: 7he correct answer is (. 7his !atient has known he!atitis C. -eatures in"icatin$ that he has a"vance" si$ns of cirrhosis an" !ortal hy!ertension inclu"e ascites, he!atos!lenome$aly, <aun"ice with elevate" bilirubin, hy!oalbuminemia with le$ e"ema, increase" !rothrombin time !robably secon"ary to ina"equate synthesis of clottin$ factors by the liver, an" eso!ha$eal variceal blee"in$. In the settin$ of "isease this a"vance", antiviral treatment for he!atitis C is without value an" the !atient shoul" un"er$o evaluation for a liver trans!lant. As "iscusse" above, all anti4viral thera!y 'choices A, , an" C) woul" be futile $iven the a"vance" sta$e of his cirrhosis at this !oint. A mesocaval shunt 'choice %) is a sur$ical !roce"ure whereby !ortal flow is "iverte" from the su!erior mesenteric vein into the inferior vena cava to re"uce !ortal !ressures. It is often com!licate" by ence!halo!athy an" "oes not im!rove the un"erlyin$ liver "ysfunction.

;) (*!lanation: 7he correct answer is -. Clinical manifestations an" bio!sy results are consistent with .aucher "isease, cause" by "eficiency of the en#yme $lucocerebrosi"ase, which results in !ro$ressive accumulation of $lucocerebrosi"e within lysosomes of histiocytes. In the most common a"ult variant 'ty!e I), the most severely affecte" or$ans are the bone marrow, liver, an" s!leen. one marrow involvement accounts for !ro$ressive !ancyto!enia an" bone fractures. )!lenome$aly contributes to thrombocyto!enia an" anemia. 7he "ia$nosis is establishe" by "etermination of $lucocerebrosi"ase levels in circulatin$ leukocytes. 2owever, bone marrow bio!sies are frequently !erforme" to "efine the e*tent of marrow involvement. 7he most characteristic mor!holo$ic fin"in$s are .aucher cells, lar$e histiocytes with their cyto!lasm en$or$e" with $lycoli!i". In the F), the "isease is most common amon$ Ashkena#i News 'of (astern (uro!ean ori$in). In the !ast, the treatment of this con"ition was limite" to s!lenectomy 'choice .), which ameliorates thrombocyto!enia an" anemia, but "oes not affect the other clinical manifestations, in !articular bone fractures an" he!atic "ysfunction. A recent a"vance in thera!y is a commercially available mo"ifie" $lucocerebrosi"ase name" al$lucerase. 7his a$ent is effective an" safe, but e*tremely e*!ensive. One4year thera!y with al$lucerase at the currently recommen"e" re$imen costs about T0/>,>>>. 6ecent stu"ies in"icate that lower "osa$es may be equally effective an" lower the annual cost to T:>>,>>>. A"ministration of !yri"o*ine an" folate 'choice A) is the most effective treatment to !revent mental retar"ation an" recurrent thrombosis in those cases of homocystinuria that are "ue to "eficiency of cystathionine S4synthase 'a!!ro*imately />= of cases). A"ministration of hematin 'choice ) has been recently intro"uce" in the treatment of acute intermittent !or!hyria, which manifests with recurrent attacks of ab"ominal !ain an" neuro!sychiatric abnormalities. An effective an" more !ractical thera!eutic a!!roach is a hi$h4carbohy"rate "iet 'choice (). Allo$eneic bone marrow trans!lantation 'choice C) woul" not !rovi"e any benefit in .aucher "isease, since the un"erlyin$ $enetic "efect is !resent in all cells of the or$anism. Cytoto*ic "ru$s such as hy"ro*yurea 'choice %) are not useful for .aucher "isease. 2y"ro*yurea is frequently use" in the thera!y of chronic myelo$enous leukemia, essential thrombocytosis, !olycythemia vera, an" sickle cell "isease. 0) (*!lanation: 7he correct answer is (. 7his !atient has sym!toms of a chronic $astrointestinal !rocess as "emonstrate" by her wei$ht loss, chan$e in bowel habits, an" thinner caliber stools with blee"in$. 7he thinner

caliber stools s!ecifically su$$est that a mass lesion or luminal narrowin$ is !resent. Of the lesions liste", only colonic cancer woul" be likely to !ro"uce this !attern. In other settin$s, Crohn "isease an" tuberculosis of the colon coul" !ro"uce similar clinical !atterns. 7his !atientAs chan$e in sym!toms over the !ast ;? hours su$$ests that she may have "evelo!e" an acute lar$e bowel obstruction. An amebic abscess 'choice A) woul" be e*!ecte" to $ive more focal fin"in$s an" sym!toms of hi$h s!ikin$ fevers. -urthermore, there woul" usually be a history of travel to su$$est this "ia$nosis. A colon !oly! 'choice ) will very rarely !ro"uce obstructive sym!toms an" "oes not !ro"uce the sym!toms of wei$ht loss an" !ersistent thin caliber stool. %iverticulitis 'choice C) is a result of a micro4 or macro!erforation of a "iverticulum an" results with an acute onset of local inflammatory si$ns, usually in the left lower qua"rant, corres!on"in$ to the si$moi" colon "iverticulosis. %iverticulosis 'choice %) may $ive her cram!y lower ab"ominal !ain an", on rare occasion, may !ro"uce obstructive sym!toms. 2owever, this !atientAs sym!toms of wei$ht loss an" bloo"y bowel movements with thinner caliber stools are more su$$estive of colon carcinoma. ?) (*!lanation: 7he correct answer is (. 7hese are the only values within normal limits, an" are the correct choice. 7his is an im!ortant clinical !oint 'an" thus likely to be teste" on the F)+,(): 7he com!lete bloo" count 'or alternatively the hematocrit or hemo$lobin) cannot be use" to assess the si#e of an acute bloo" loss. 7he reason is that the hematocrit or re" cell count will "ro! only as flui" from other bo"y sources 'mostly e*tracellular flui", with a smaller shift of intracellular flui" to e*tracellular) enters the ca!illary be"s in res!onse to a "ecrease" intraca!illary bloo" !ressure. 7his takes at least a few hours to ha!!en. ,ow or "ro!!in$ bloo" !ressure is consequently a much better initial in"icator of si$nificant hemorrha$e. When the re" cell count "oes eventually "ro!, the white cell count usually "ro!s as well 'choice A an" C), rather than remainin$ hi$h 'choices an" %), since the hemo"ilution will affect all the cells in the bloo". /) (*!lanation: 7he correct answer is . +inimal chan$e "isease is a cause of ne!hrotic syn"rome. Althou$h it is more common in chil"ren, it can still be seen in a"ults. 7hese !atients will usually have a

!aucity of hematuria or a#otemia. +ost !atients will have a $oo" res!onse to oral steroi"s, which are consi"ere" to be the initial treatment of choice. Intravenous !ulse cyclo!hos!hami"e 'choice A) is the "ru$ of choice for !roliferative lu!us ne!hritis. It may occasionally be use" for other $lomerular "iseases that fail steroi" thera!y. Observation 'choice C) woul" not be a reasonable choice in a !atient with a treatable cause of ne!hrotic syn"rome. -5/>C 'choice %) is an anti4re<ection me"ication use" to treat !atients with or$an trans!lants. It is only rarely use" as a "ru$ to treat $lomerular "iseases. 6enal trans!lant 'choice () is of value for !atients with a"vance" renal failure. 7his is not the case in this !atient. C) (*!lanation: 7he correct answer is C. 7he C7 an" +6I scans clearly show an e*traa*ial mass 'i.e., locate" outsi"e the brain !arenchyma). 7he mass !ushes an" "is!laces the un"erlyin$ brain but "oes not infiltrate it. It is "ural4base", i.e., it "evelo!s from the "ura. 7hese features are characteristic of menin$iomas, which are the most common beni$n intracranial neo!lasms. 7hese tumors ori$inate from menin$othelial cells an" may $row to a lar$e si#e before !ro"ucin$ clinical sym!tomatolo$y, es!ecially when locate" over the cerebral conve*ities. On the other han", if menin$iomas "evelo! from the menin$es of the skull base, brain com!ression or im!in$ement on nerves an" vessels results in clinical sym!toms at early sta$es of $rowth. +enin$iomas may in"uce a reactive bone thickenin$ in the a"<acent calvarium. Cranio!haryn$ioma 'choice A) "evelo!s in the su!rasellar re$ion close to the !ituitary $lan", hy!othalamus, an" o!tic chiasm. 7hus, the most common initial manifestations are "iabetes insi!i"us, visual "eficits, an" hy!o!ituitarism. 7his tumor ty!ically occurs in chil"ren. It is often heavily calcifie", which facilitates the ra"iolo$ic "ia$nosis. .lioblastoma multiforme '. +) 'choice ) is the most frequent !rimary mali$nant brain tumor. It "erives from astrocytes an" is therefore intraa*ial 'i.e., $rows within the brain). .rossly, it is characteri#e" by areas of hemorrha$e an" necrosis. Central necrosis within the tumor results in a ty!ical +6IBC7 a!!earance, with a !eri!heral rim of contrast enhancement 'rin$4enhancin$ mass) surroun"in$ the central "ark re$ion. &a$et "isease 'choice %) is a common bone "isease that involves the skull, !ro"ucin$ abnormal an" irre$ular thickenin$ of the bone. 7he !atient may !resent to me"ical attention after noticin$ an increase in hea" si#e.

)chwannoma 'choice () is a beni$n tumor of )chwann cells. 7he ei$hth cranial nerve is its most common nerve of ori$in, an" the cerebello!ontine an$le is thus the most common location. &atients with schwannomas !resent with !ro$ressive hearin$ loss. If bilateral, acoustic schwannomas are ty!ical of neurofibromatosis ty!e ;. D) (*!lanation: 7he correct answer is A. 7he lesion "escribe" coul" be either an actinic 'solar) keratosis or squamous cell carcinoma in situ. '7he lesions are actually very closely relate", as some authors consi"ere" the !remali$nant actinic keratoses to re!resent very early squamous cell carcinoma in situ.) While e*!erience" "ermatolo$ists can reliably reco$ni#e many lesions as actinic keratoses, as o!!ose" to squamous cell carcinomas or other mali$nant skin lesions, relatively ine*!erience" internists or other clinicians shoul" !robably have the lesions bio!sie" 'a shave bio!sy may be all that is nee"e") before treatment. Actinic keratoses occur on sun e*!ose" skin areas, with favorite tar$et sites inclu"in$ face, back of han"s an" arms, neck, bal" hea"s, an" back an" chests of men who work without shirts. 7reatment o!tions inclu"e /4fluorouracil '!articularly $oo" for lar$e lesions or multi!le lesionsG "oes not cause scarrin$), cryothera!y '$oo" for a few lesionsG "oes not cause scarrin$), an" sur$ery 'scarrin$, but obtains tissue for microsco!ic e*amination). asal cell carcinoma 'choice ) ten"s to have raise" e"$es, an" only !artially res!on"s to to!ical /4fluorouracil, leavin$ cancer s!rea"in$ below the skin surface because the cancer cells !ro<ect "ee! into the "ermis. owenoi" !a!ulosis 'choice C) can cause multi!le carcinomas in situ involvin$ $enital areas. 5eratoacanthoma 'choice %) cause a volcano4sha!e" lesion with a central keratotic center. +ali$nant melanoma 'choice () will show !i$mentary color variations an" irre$ular mar$ins, an" will not necessarily res!on" to /4fluorouracil, as it can e*ten" "ee! into the "ermis. @) (*!lanation: 7he correct answer is C. Iron "eficiency usually results in a microcytic, hy!ochromic anemia. In women of chil"4bearin$ a$e, iron "eficiency usually results from monthly menses. In ol"er women, colon cancer is the most common etiolo$y. 7his is a classic !resentation for iron "eficiency anemia in women of this a$e an" activity level.

-olate "eficiency 'choice A) results in a macrocytic anemia an" has been associate" with neural tube "efects. -olate is containe" in $reen, leafy ve$etables, but startin$ in the mi" :II>s, many $rain !ro"ucts were su!!lemente" with folate. 2emolysis 'choice ) can cause a normochromic, normocytic anemia. %isease states that are associate" with hemolysis are often obvious, an" the !eri!heral manifestations of hemolysis '<aun"ice, scleral icterus, tea4colore" urine) are also often !resent. 7here is no evi"ence in this !atientAs history that she has hemolysis. 6enal "isease 'choice %) is a fairly common cause of anemia, es!ecially if the !atient is "ialysis "e!en"ent. 7he ki"ney, in a""ition to its role in !lasma filtration an" in bloo" !ressure control, also controls the hematocrit via secretion of erythro!oietin, which acts to stimulate bone marrow !ro"uction of re" bloo" cells. When erythro!oietin !ro"uction "eclines, a normochromic, normocytic anemia often ensues. 1itamin :; "eficiency 'choice () is often associate" with a me$aloblastic or macrocytic anemia. :; "eficiency is e*cee"in$ly rare, since most !eo!le have at least a 04year su!!ly. I) (*!lanation: 7he correct answer is C. 7he !arameters for a !ositive &&% test "e!en" on the 2I1 status of the !atient. If the !atient is immunocom!etent an" can mount a "elaye" ty!e hy!ersensitivity res!onse to &&%, then an in"uration of ? mm woul" ar$ue a$ainst chemo!ro!hyla*is with isonia#i". Fsually, in"uration of at least :> mm is require" to call a test !ositive. If the !atient was recently e*!ose" to active !ulmonary tuberculosis, 'choice A), then he must have the &&% !lace" an", if !ositive, be initiate" on isonia#i". ,esser "e$rees of in"uration may re!resent cross4reactions to other mycobacteria or fa"in$ skin test reactivity, an" &&% testin$ may be re!eate" in a cou!le of weeks. If the !atient is 2I1 !ositive, he may not be able to mount a robust res!onse to the &&% challen$e. In immunocom!romise" !atients, a &&% in"uration of / mm woul" be consi"ere" !ositive, an" the !atient woul" be starte" on isonia#i" 'choice ). A ne$ative &&% a year a$o "oes not necessarily mean that the !atient is not !ositive now if he ha" tuberculosis e*!osure in the hos!ital recently 'choice %). If his new &&% is !ositive, he may nee" to be treate". All a"ults youn$er than 0/ years with !ositive &&% test results shoul" have isonia#i" !ro!hyla*is 'choice () unless they cannot tolerate the me"ication, have alrea"y receive" a full course, or are known to have been e*!ose" to isonia#i"4resistant tuberculosis.

:>) (*!lanation: 7he correct answer is %. 2i$h concentrations of ins!ire" o*y$en "elivere" throu$h a ventilator may lea" to !ulmonary fibrosis, which becomes irreversible. In the settin$ of a"ult res!iratory "istress syn"rome 'A6%)), if the ins!ire" fraction of o*y$en cannot be lowere" without !ro"ucin$ hy!o*ia, the a""ition of !ositive4en" e*!iratory !ressure '&((&) is in"icate". Althou$h &((& "oes increase the risk of both barotrauma an" hy!otension by im!airin$ ri$ht4si"e" heart fillin$, it is in"icate" to !revent the "evelo!ment of o*y$en to*icity, which may result in irreversible !ulmonary fibrosis. Con$estive heart failure 'choice A) can occur as a com!lication of the !atientAs lon$stan"in$ un"erlyin$ !ulmonary "isease, but the inci"ence woul" not likely be increase" because of his ventilator settin$s. Nu$ular venous "istension 'choice ) can be a marker for either ri$ht heart con$estive failure 'see choice A "iscussion) or tension !neumothora* 'see choice ( "iscussion), but woul" not be a likely com!lication of a hi$h !O;. &ulmonary embolus 'choice C) woul" more likely be relate" to !rolon$e" be" rest with resultant venous thrombosis. 7ension !neumothora* 'choice () woul" be more likely to occur if the ti"al volume were si$nificantly $reater than D/> m,Bbreath 'corres!on"in$ to the o!timal flow rate of :> m,B k$). ::) (*!lanation: 7he correct answer is (. 7his !atient has hy!er!arathyroi"ism, which is characteri#e" by an elevation of circulatin$ !arathyroi" hormone '&72), most likely "ue to a beni$n a"enoma. An elevation of &72 lea"s to hi$h calcium an" low !hos!hate levels. 7hese !atients often have unbalance" bone resor!tion an" formation lea"in$ to osteo!enia an" fractures, recurrent ne!hrolithiasis, va$ue ab"ominal sym!toms, weakness, an" easy fati$ability. H ones, stones, ab"ominal $roans, an" moansH is a mnemonic use" to remember the sym!toms. 7he treatment is ty!ically sur$ical. %ecrease" e*cretion of uric aci" 'choice A) is the most common cause of hy!eruricemia an" $out. It can be cause" by increase" tubular absor!tion, "ecrease" tubular secretion, or "ecrease" $lomerular filtration. 7hese in"ivi"uals ty!ically have a uric aci" level $reater than D m$B",. Com!lications inclu"e $outy arthritis, ne!hrolithiasis, renal insufficiency, an" renal failure. 7he "evelo!ment of thyroi"4stimulatin$ hormone rece!tor antibo"ies 'choice ) is the un"erlyin$ mechanism of .raves "isease, which is one of the most common causes of

hy!erthyroi"ism. 7he si$ns an" sym!toms inclu"e a "iffuse, to*ic $oiter, tachycar"ia, tremor, nervousness, an" e*o!hthalmos. 7his !atient "oes not fit this "escri!tion. An elevation of circulatin$ al"osterone 'choice C) is a si$n of Conn "isease, which is cause" by a"renal hy!er!lasia or an a"enoma. 7hese !atients ty!ically have hy!ertension an" hy!erkalemia. 7his !atient has neither of these fin"in$s. An elevation of circulatin$ cortisol 'choice %) is a si$n of Cushin$ syn"rome, which may be cause" by a !ituitary a"enoma 'Cushin$ "isease), an a"renal cortical tumor, ecto!ic AC72 !ro"uction 'a lun$ tumor), or $lucocorticoi" thera!y. )i$ns an" sym!toms inclu"e central obesity, moon facies, muscle wastin$, osteo!orosis, striae, bruisin$, hy!ertension, menstrual irre$ularities, hirsutism, an" acne. 7his !atient has osteo!enia, but none of the other si$ns or sym!toms. :;) (*!lanation: 7he correct answer is . A wi"e variety of "efects relate" to com!lement can occur an" can !ro"uce immuno"eficiency. 7he !attern shown in the question stem of recurrent Neisseria menin$itis is ty!ical of "eficiency of com!lement factors CC, CD, or C@. Other com!lement "eficiencies !ro"uce the followin$ !atterns. %eficiency of C:q !ro"uces a combine" immuno"eficiency with an ),(4like syn"rome. %eficiencies of C:rs, C:s, C?, or C; !ro"uce an ),(4like syn"rome an" $lomerulone!hritis. %eficiencies of C0 or C/ !ro"uce a ten"ency to !yo$enic infections. No "isease has yet been associate" with CI "eficiency. cell "eficiencies 'choice A) occur in a number of con"itions an" ten", in $eneral, to be associate" with an increase" susce!tibility to !yo$enic infections. (osino!hil "eficiency 'choice C) is not a usual cause of immuno"eficiency. Neutro!hil abnormalities 'choice %) can cause a variety of con"itions that may !resent with either !yo$enic infections or $ranulomatous "isease. 7 cell "eficiencies 'choice () may !resent with infections with o!!ortunistic or$anisms such as Can"i"a, &neumocystis, or cytome$alovirus. :0) (*!lanation: 7he correct answer is (. One of the most embarrassin$ fin"in$s on ur$ent ab"ominal C7 scans is an enlar$e" bla""er from urinary retention. 7he fact that the !atient ha" sym!toms of an enlar$e" !rostate !rior to a"mission, combine" with the atro!ine that he receive" "urin$ his catheteri#ation, !lace" him at risk for urinary retention. In a""ition to anticholiner$ic

"ru$s like atro!ine, other me"ications that have anticholiner$ic effects 'e.$., tricyclic anti"e!ressants) may also !lace !atients at risk for urinary retention. 7he cholesterol emboli syn"rome 'choice A) may !resent after invasive vascular !roce"ures such as coronary an$io$ra!hy. Althou$h intestinal ischemia, with atten"ant ab"ominal !ain, may be a sequela of this syn"rome with !rotean manifestations, it classically !resents as !ur!ura seen in the "istal e*tremities in association with live"o reticularis, $an$rene, ischemic ulcerations, or renal failure. Contrast4in"uce" acute renal failure 'choice ), !ostulate" to be cause" by intense vasoconstriction, "oes not ty!ically !resent with any !ain. )tones in the urinary tract 'choice C) may cause a !atient to have urinary retention if a stone obstructs the bla""er outlet. .iven the historical conte*t "escribe" in this case, a "ru$ effect in the settin$ of !rostatism is more likely. &yelone!hritis 'choice %) classically !resents with flank !ain in the settin$ of fever an" chills. :?) (*!lanation: 7he correct answer is (. 7he !resentation is ty!ical for acute cholecystitis, which occurs most frequently in the settin$ of cholelithiasis '$allstones). Other common features inclu"e an initially low4 $ra"e fever with neutro!hilia an" !ainful s!lintin$ "urin$ "ee! breathin$. )erum amylase is ty!ically elevate" in $allstone !ancreatitis. )eriously ill !atients with hi$h fever, ri$ors, or si$nificant reboun" ten"erness may require ur$ent sur$ical interventionG in less seriously ill !atients, it is feasible to establish the "ia$nosis an" "efer sur$ical intervention until after the acute e!iso"e has resolve". In most hos!itals, ultrasoun" is or"ere" first, since this relatively ine*!ensive, fast, an" noninvasive stu"y can usually establish the !resence of $allstones. In aty!ical cases, when acute cholecystitis without stones is !resent, cholescinti$ra!hy usin$ ra"ioactive technetium IIm may be use" to sequentially visuali#e the liver, e*trahe!atic bile "ucts, $allbla""er, an" "uo"enum. C7 'choice A) an" +6I 'choice %) scans are e*!ensive an" are usually not require" for ty!ical acute cholecystitis. (n"osco!ic retro$ra"e cholan$io$ra!hy 'choice ) can be hel!ful in "efinin$ a small stone in the e*trahe!atic bile "uct system, but it is not usually use" as an initial test. (so!ha$o$astro"uo"enosco!y 'choice C) woul" not be hel!ful in classic $allstone "isease, but mi$ht "emonstrate a "uo"enal cancer com!ressin$ the am!ulla of 1ater if a !atient with what a!!eare" clinically to be $allstone "isease ha" a ne$ative ultrasoun".

:/) (*!lanation: 7he correct answer is (. Chronic ty!e A $astritis is immunolo$ically me"iate" an" is associate" with an elevate" level of !arietal cell antibo"y. 7hus, aci" secretion is re"uce". 7he $astritis !rimarily involves the stomach bo"y as well as the antrum 'choice A). %ue to the re"uce" aci" level 'choice ), the $astrin level woul" be hi$h. 7y!e C). $astritis is more commonly associate" with 2elicobacter !ylori infection 'choice $astritis as well, involvin$ the antrum

Chronic N)AI% use 'choice %) will lea" to ty!e of the stomach.

:C) (*!lanation: 7he correct answer is A. 6enal failure wreaks havoc on a number of metabolic functions. One of these is calcium homeostasis. 6enal failure !atients waste calcium an" "evelo! secon"ary hy!er!arathyroi"ism in an attem!t to correct the hy!ocalcemia. 7his lea"s to bone calcium "eficits an" !atholo$ic bone lesions. Calcium su!!lementation slows this !rocess. In this !atient, a hi$h4!rotein "iet 'choice ) woul" worsen her renal con"ition. 7here is evi"ence that hi$h4!rotein "iets are associate" with worsenin$ renal function in !atients with "iabetes or ne!hrotic syn"rome. Althou$h the benefits of a low4fat "iet 'choice C) are well known in terms of car"iovascular benefit, there is little a""itional benefit that is e*ten"e" to this s!ecific !atient. )u!!lementation with vitamin C 'choice %) or vitamin ( 'choice () is often recommen"e" to !atients in the conte*t of im!rovin$ nutritional status "urin$ a chronic illness. %es!ite some anec"otal evi"ence, there is little substantial evi"ence that there is any benefit to such su!!lementation. :D) (*!lanation: 7he correct answer is A. 7he clinical history an" ima$in$ fin"in$s are consistent with blee"in$ within the e!i"ural s!ace. (!i"ural hemorrha$e is traumatic in ori$in in most cases an" usually associate" with fractures of the calvarial wall. 7his results in tearin$ of one of the

e!i"ural arteries 'most commonly the mi""le menin$eal artery), with ra!i" accumulation of bloo" between the calvarial bone an" the un"erlyin$ "ura. rief loss of consciousness is often followe" by a luci" !erio", which may last for a few hours. 7he !atient rela!ses into coma because of the enlar$in$ hematoma that "is!laces the brain an" causes uncal herniation. A fi*e" "ilate" !u!il is the result of the herniate" uncus com!ressin$ the i!silateral oculomotor nerve. 2emorrha$e within the sub"ural s!ace 'choice ) is usually "ue to tearin$ of the so4 calle" bri"$in$ veins. It is usually of traumatic ori$in, but cerebral atro!hy is an im!ortant !re"is!osin$ con"ition. 7hus, this form of blee"in$ "evelo!s most commonly in el"erly !atients, often after minimal trauma. lee"in$ in the subarachnoi" s!ace 'choice C) manifests with hea"ache of su""en onset 'thun"ercla! hea"ache) an" nuchal ri$i"ity. 6u!ture of berry aneurysms in the circle of Willis is the most common cause. Intracerebral hemorrha$e 'choice %) has a wi"e ran$e of etiolo$ies, hy!ertension bein$ one of the most common. Other causes inclu"e trauma, infections, vascular malformations, blee"in$ "iathesis, neo!lasms, an" amyloi" an$io!athy. Clinical manifestations vary, "e!en"in$ on the location an" e*tent of the blee"in$. Intraventricular blee"in$ 'choice () is usually "ue to an e*tension from subarachnoi" or intracerebral blee"in$ an" is thus "ue to the same un"erlyin$ con"itions as the ones mentione" above. :@) (*!lanation: 7he correct answer is . &i$mente", mole4like lesions are very common in the $eneral !o!ulation, an" virtually every !hysician nee"s to "evelo! some skill in "istin$uishin$ obviously beni$n lesions from !otentially mali$nant ones. %ys!lastic nevi are an interme"iate cate$ory between obviously beni$n nevi an" mali$nant melanoma. While not consi"ere" cancerous themselves, they "o have an increase" rate of !ro$ression to melanoma. Isolate" "ys!lastic nevi are often e*cise" to remove the melanoma risk. +ore !roblematic are cases like this one, in which lar$e numbers of "ys!lastic nevi are !resent. In this situation, careful monitorin$ with serial !hoto$ra!hs can i"entify any chan$in$ lesions which may be un"er$oin$ mali$nant transformation. Com!oun" nevi 'choice A) are usually "ark, ty!ically elevate", 0 to C mm lesions with a very re$ular sha!eG most !eo!le have about :> of these lesions. 2alo nevi 'choice C) are flesh4colore" or "ark no"ules, usually 0 to / mm, surroun"e" by a rin$ of "e!i$mente" skin.

,enti$os 'choice %) are flat, shar!ly mar$inate", uniformly !i$mente", ; to ? mm "iameter skin lesions. +ali$nant melanomas 'choice () are often quite asymmetrical in sha!e an" the color varies more wi"ely than in "ys!lastic nevi, !otentially showin$ tan, brown, black, blue, re", or clear areas. :I) (*!lanation: 7he correct answer is . eta blockers have been shown in multi!le stu"ies to im!rove survival after myocar"ial infarction '+I) by "ecreasin$ both o*y$en "eman" an" the inci"ence of ventricular arrhythmia. An$iotensin4convertin$ en#yme 'AC() inhibitors 'choice A), such as enala!ril, have been shown to im!rove survival in !ost4+I !atients who have e<ection fractions less than ?>=. %i$o*in 'choice C) woul" be relatively contrain"icate" in this instance in that it woul" increase myocar"ial o*y$en "eman" by increasin$ inotro!y. 7his !atient "oes not nee" !ositive inotro!ic su!!ort, as there is only minimal "ecrease in e<ection fraction 'normal //= to DD=). ,oo! "iuretics 'choice %) may hel! control this !atientAs hy!ertension, but they have not been shown to im!rove survival. Warfarin 'choice () is beneficial in lar$e anterior wall infarcts when there is a severely low in<ection fraction. 7he "ru$ "ecreases the risk of thromboembolic !henomena. ;>) (*!lanation: 7he correct answer is (. 7his !atient has a monoarticular arthritis with a very hi$h white cell count in the arthrocentesis stron$ly su$$estive of a se!tic arthritis. )ince the $ram stain is unrevealin$ in this case 'which you shoul" be aware can ha!!en) as to the s!ecific or$anism, broa"4 s!ectrum antibiotics to cover the two most likely or$anisms is a!!ro!riate. 7his inclu"es thera!y for $onococcus 'ceftria*one) as well as nafcillin for )ta!hylococcus aureus se!tic arthritis. (ither ceftria*one or nafcillin alone 'choices A an" %) woul" be insufficient. )ince the !olari#in$ microsco!y is ne$ative, there is no in"ication for colchicine 'choice ).

In"omethicin 'choice C) may be an a"<unctive thera!y but is not the a!!ro!riate thera!y in a !atient with this many white bloo" cells in the <oint flui", consistent with a se!tic arthritis. ;:) (*!lanation: 7he correct answer is C. 7he fin"in$s of !etechiae, alteration in mental status, an" refractory hy!o*emia in a !atient with multi!le bone trauma shoul" su$$est the "ia$nosis of fat embolism. 7his is usually seen :40 "ays after the initial trauma. -at $lobules will be release" from the fracture" bones an" obstruct !ulmonary vessels. An altere" sensorium an" !etechiae are also ty!ical of fat embolism. Althou$h an as!iration !neumonia 'choice A) can "evelo! in a !atient who has an altere" mental status, this !atient was alert an" oriente" until the su""en an" acute "eterioration. 7here is no evi"ence here for any "ecom!ensation of left ventricular function 'choice ). Althou$h a !ulmonary embolus 'choice %) is !ossible in a !atient who is immobili#e", it woul" not e*!lain his !etechiae. -urthermore, the !atient has been immobili#e" only for the !ast ;? hours. A tension !neumothora* 'choice () mi$ht cause hy!o*ia but woul" not cause !etechiae. ;;) (*!lanation: 7he correct answer is (. 2y!ertension with an a"renal mass shoul" brin$ to min" two "istinct con"itions: an e!ine!hrineBnore!ine!hrine4secretin$ !heochromocytoma, or an al"osterone4secretin$ a"renocortical tumor 'which can !ro"uce Conn syn"rome). Althou$h both of these can !ro"uce hy!ertension, other as!ects of the clinical !resentation are quite "ifferent. &heochromocytoma can !ro"uce either e!iso"ic severe hy!ertension or continuous hy!ertension. In either situation, electrolyte stu"ies are usually normal. In contrast, Conn syn"rome 'which this !atient has) often shows !rominent sym!toms relate" to hy!okalemia, inclu"in$ weakness, !aresthesias, an" transient !aralysis. 7he treatment of hy!ertension relate" to hi$h al"osterone levels is to block the al"osterone activity with the !otassium4s!arin$ "iuretic 'an" anti4al"osterone a$ent) s!ironolactone. Ca!to!ril 'choice A) is an an$iotensin convertin$ en#yme 'AC() inhibitor that works well to re"uce bloo" !ressure in many !atients by blockin$ the renin4an$iotensin systemG however, it is less effective when there is alrea"y an e*cess of al"osterone !resent. -urosemi"e 'choice ), a loo! "iuretic, an" hy"rochlorothia#i"e 'choice C), a thia#i"e ty!e "iuretic, woul" ten" to make the !atientAs hy!okalemia worse.

&ro!anolol 'choice %) is a beta blocker that woul" re"uce bloo" !ressure but woul" not s!ecifically counter the cause of this !atientAs hy!ertension. ;0) (*!lanation: 7he correct answer is C. 7his !atient has sym!tomatic cholecystitis. 7his com!lication is most commonly associate" with lon$4stan"in$ $allstones an" less frequently with severe illness 'so4calle" acalculous cholecystitis). 7he thera!y for such !atients is usually !rom!t sur$ical removal of the inflame" $allbla""er. If left in !lace, there is an increase" risk of infection, abscess formation, or se!sis. All such !atients shoul" receive I1 flui"s, resuscitation, an", if very ill a!!earin$, covera$e with broa"4s!ectrum antibiotics. I1 flui"s an" observation 'choice A) an" I1 antibiotics an" observation 'choice ) are ina!!ro!riate since the !atient requires sur$ery. -ailure to reco$ni#e this fact will result in serious morbi"ity. Once the !atient is a!!ro!riately referre", then flui"s an" !ossibly antibiotics are crucial com!onents of thera!y. Fr$ent sur$ical evaluation for imme"iate sur$ery 'choice %) is not a!!ro!riate since the !atient a!!ears stable, althou$h febrile an" uncomfortable. Fr$ent !ercutaneous "raina$e 'choice () is an interventional ra"iolo$y !roce"ure that allows for "raina$e of the $allbla""er. 7hese !roce"ures are usually reserve" for very ill !atients who coul" not tolerate sur$ery an" $eneral anesthesia. ;?) (*!lanation: 7he correct answer is %. 7he !revalence of infection of ascites flui", so calle" s!ontaneous bacterial !eritonitis ') &), is estimate" at />= in hos!itali#e" cirrhotics. A,, febrile !atients a"mitte" with ascites must have an ab"ominal !aracentesis !erforme" to both "etermine the cause for the ascites an" rule out infection of the ascites. 7he "ia$nosis of ) & is ma"e when there is an elevate" ascitic flui" absolute neutro!hil count 'O ;/> cellsBmm0) without an evi"ent intra4ab"ominal or sur$ically treatable cause for the infection. ) & most often occurs with !ortal hy!ertensive ascites. 7he most sensitive marker available for such ascites is a serumBascites albumin $ra"ient O :.: $B",. An ab"ominal ultrasoun" 'choice A) is in"icate" only if ascites is sus!ecte" but "ifficult to confirm by !aracentesis or !hysical e*amination. Ab"ominal ultrasoun" can "etect as little as :>>cc of ascitic flui" whereas !hysical e*amination requires the !resence of at least :>>>cc of flui" to be a sensitive in"icator for the !resence of ascites.

An ab"ominal C7 scan 'choice ) is only in"icate" after an ab"ominal !aracentesis is !erforme" an" ) & is e*clu"e". An electrocar"io$ram 'choice C) is not the most a!!ro!riate initial ste! $iven that there is no clinical or laboratory in"ication for it at !resent. A lumbar !uncture 'choice () woul" be in"icate" if the !atient ha" any chan$e in his mental status or neurolo$ical e*amination cou!le" with laboratory evi"ence that more common causes for infection have been e*clu"e". ;/) (*!lanation: 7he correct answer is %. )clero"erma '!ro$ressive systemic sclerosis) is an im!ortant "isease in which a variety of bo"y tissues can un"er$o fibrosis. 7he most obvious of these is the skin, which !ro"uces the ti$ht thick skin seen on this !atientAs han"s an" face. Internally, the eso!ha$us is !articularly susce!tible to this fibrotic !rocess, which will "estroy the ability of the eso!ha$us to un"er$o !eristalsis, because of both the loss of muscle an" the stiffenin$ by fibrosis. 7y!ical manometric fin"in$s in these !atients are "iminishe" or absent !eristalsis in the bo"y of the eso!ha$us with a $reatly re"uce" restin$ lower eso!ha$eal s!hincter ',()) !ressure. )clero"erma !atients are at si$nificant risk for severe $astroeso!ha$eal reflu* "isease '.(6%) an" its com!lications. Choices A an" C are not ty!ical of any con"ition you nee" to remember. Associate the fin"in$s illustrate" in Choice with achalasia.

Associate the fin"in$s illustrate" in Choice ( with sym!tomatic "iffuse eso!ha$eal s!asm. ;C) (*!lanation: 7he correct answer is C. 7his !atient has 2o"$kin "isease. 7he lar$e binucleate cells are 6ee"4)ternber$ cells. Immunohistochemical confirmation of these cells is im!ortant, because, "es!ite the a!!arently "istinctive a!!earance of 6ee"4)ternber$ cells in te*tbooks, Hlook4alikeH cells with very similar a!!earance can be seen in a variety of situations. 2o"$kin "isease has a bimo"al a$e "istribution with one !eak in late a"olescence4youn$ a"ulthoo" an" a secon" !eak in ol"er in"ivi"uals. )ystemic sym!toms are usually not initially !rominent, an" these !atients ten" to be i"entifie" either when a neck mass is evaluate" or when a chest *4ray !erforme" for other reasons "emonstrates me"iastinal lym!ha"eno!athy. urkitt lym!homa 'choice A) ten"s to affect the <aw, ovary, or ab"ominal cavity.

2airy cell leukemia 'choice ) often shows small numbers of leukemic cells with characteristic HhairyH !ro<ections in bloo", bone marrow, or s!leen. +ycosis fun$oi"es 'choice %) is a 7 cell lym!homa with a !re"ilection for affectin$ the skin, an" less commonly, the internal or$ans. Wal"enstrom macro$lobulinemia 'choice () is a !lasma cell "yscrasia with clonal !ro"uction of I$+. ;D) (*!lanation: 7he correct answer is C. 2ematuria may be cause" by "isor"ers such as tumors, urethritis, cystitis or !rostatitis. 7he best way to narrow the "ifferential is by "irect visuali#ation of the bla""er an" urethra with cystosco!y. 7his is relatively non4invasive an" !ainless an" is the fastest o!tion. A C7 scan 'choice A) may be hel!ful for "etectin$ a renal tumor, !elvic abscess or mass. ut the chances of missin$ a small lesion in the urolo$ical system are hi$h an" a cystosco!y must be com!lete" first. A course of trimetho!rim4sulfametho*a#ole 'choice ) is use" to treat urinary tract infections an" !rostatitis. Frinary tract infections are $enerally not !ainless. A renal an$io$ram 'choice %) woul" be nee"e" to evaluate hy!ertension or worsenin$ renal function in or"er to "etect ailments such as renal artery stenosis. A !rostatic bio!sy 'choice () may be nee"e" if !rostate cancer is sus!ecte". Certainly the !atient has the risk factor of increase" a$e. 2owever, with no history of urinary retention, this can be "eferre" until the cystosco!y. ;@) (*!lanation: 7he correct answer is A. 7he vise4like quality of this hea"ache is consistent with tension hea"ache, which is often tri$$ere" or worsene" by stressful situations, an*iety, an" fati$ue, for e*am!le. Fsually, this form of hea"ache "oes not require further clinical investi$ations an" res!on"s in most cases to nonsteroi"al anti4inflammatory "ru$s 'N)AI%s). Althou$h tension hea"ache is sai" to be e*tremely !revalent in the $eneral !o!ulation, !atients rarely come to the $eneral !ractitioner.

)umatri!tan 'choice ) is the "ru$ of choice for treatment of mi$raine "urin$ acute attacks. It is a centrally actin$ serotonin4rece!tor a$onist. It is not use" for tension hea"ache, unless this is associate" with mi$raine. Althou$h !sycho"ynamic analysis may be beneficial in "isclosin$ the un"erlyin$ causes of an*iety, !sychiatric referral 'choice C) is usually unnecessary. On the other han", e*ercise an" rela*ation techniques are useful non!harmacolo$ic a!!roaches. ,umbar !uncture 'choice %), C7 of the hea" 'choice (), an" +6I of the hea" 'choice -) shoul" be consi"ere" when there is a reasonable sus!icion of or$anic con"itions, such as intracranial blee"in$, tumor, ischemic brain "isease, an" infections. ;I) (*!lanation: 7he correct answer is C. Oran$e !eel skin on the anterior as!ects of the e*tremities is a "istinctive feature of eosino!hilic fasciitis, which is a sclero"erma4like "isor"er involvin$ the arms, le$s, an" sometimes face an" trunk, but not usually the han"s an" feet. +i""le a$e" men are most commonly affecte". )ym!toms usually "evelo! insi"iously, an" lea" to eventual restriction of arm an" le$ motion. 7he restriction of movement is relate" to inflammation an" fibrosis of fascia an" sometimes ten"ons, synovial membranes, an" muscle. io!sy of the skin or fascia shows cellular infiltrates with histiocytes, !lasma cells, lym!hocytes, an" 'in only some cases, "es!ite the name) eosino!hils. 7reatment is usually with initially hi$h "ose !re"nisone followe" by ta!erin$ an" maintenance for ; to / years on low "ose !re"nisone. C6()7 syn"rome 'choice A) is a variant ofsclero"erma with !rominent involvement of the han"s 'sclero"actyly), calcinosis, 6aynau"As !henomenon, telan$iectasias, an" eso!ha$eal "ysfunction. %ermatomyositis 'choice ) !ro"uces inflammatory an" "e$enerative chan$es in muscles an" skin. )kin manifestations inclu"e a "usky, erythematous rash, which may show "esquamation with s!littin$ of the skin, !articularly of the fin$ers. &olymyal$ia rheumatica 'choice %) "oes not cause skin manifestations. &olymyositis 'choice () by itself "oes not cause skin manifestationsG if it is accom!anie" by "usky rash, it is calle" "ermatomyositis. 0>) (*!lanation: 7he correct answer is

C. 7his !atient has two risk factors 'family history an" tobacco) but has an ,%, less than :0> m$B", an" a total cholesterol less than ;0I m$B",. 2e can therefore be mana$e" by "iet an" e*ercise with a follow u! !rofile bein$ "rawn in : year. No intervention is in"icate" 'choice A) is incorrect. 7he !atient "oes have mar$inally elevate" cholesterol an" a subo!timal ,%, com!onent. ("ucatin$ the !atient about "iet an" e*ercise 'choice ) is !artially correct, but he must have follow u!. )ince the !atient alrea"y a"heres to a low4fat, low4cholesterol "iet, initiatin$ a man"atory low4fat "iet 'choice %) woul" be of little hel!. .iven the combination of this !atientAs risk factors an" cholesterol levels, there is no reason to initiate "ru$ thera!y to control his hy!erli!i"emia 'choice (). 2is ,%, woul" nee" to be $reater than :0> m$B", or his total cholesterol $reater than ;0I m$B", to initiate "ru$ thera!y. 0:) (*!lanation: 7he correct answer is C. 7his !atient has monoarticular arthritis. 7he most likely "ia$noses in this settin$ are crystal arthritis '$out or !seu"o$out) or bacterial arthritis '$onorrheal or sta!hylococcal). (stablishin$ the "ia$nosis is best accom!lishe" with arthrocentesis for e*amination of the e*tracte" synovial flui" for crystals an"Bor bacteria. In both situations, lar$e numbers of neutro!hils may also be seen in the e*tracte" flui". A com!lete bloo" count 'choice A) will not be hel!ful in "istin$uishin$ crystal from bacterial arthritis, since an elevate" white count is common in both con"itions. Ima$in$ stu"ies of the knee with *4ray or +6I 'choices since they will not yiel" a "efinitive "ia$nosis. an" %) are not a!!ro!riate,

Arthrosco!y 'choice () has no role in the mana$ement of acute monoarticular arthritisG !atients who "o have a <oint effusion may benefit from <oint "raina$e after a!!ro!riate "ia$nosis an" thera!y. 0;) (*!lanation: 7he correct answer is %. 7he force" e*!iratory volume in : secon"Bforce" vital ca!acity '-(1:B-1C) is a ratio that is effort "e!en"ent. It increases with increasin$ e*!iratory effort. -(1: is the volume of air forcefully e*!ire" "urin$ the first secon" after a "ee! breath. -1C is the ma*imal volume of air e*!elle" with "ee! e*!iration. 6esi"ual volume '61) is the

volume of air remainin$ in the lun$s after a ma*imal e*!iratory effort. 1ital ca!acity '1C) is the ma*imal volume of air that may be e*!elle" from the lun$s followin$ a ma*imal ins!iration. A re"uce" -(1:B-1C is in"icative of obstructive airway "isease. 7he re"uce" ratio im!lies "ecrease" airway em!tyin$ because of air tra!!in$. 7his !atient !robably has chronic obstructive !ulmonary "isease 'CO&%) from smokin$. CO&% woul" cause an increase in 61B7,C, not a "ecrease 'choice A) as a result of air tra!!in$ since the terminal airways close before all the air is e*!elle". 61 'choice ) will be increase" because of air tra!!in$. 7he obstructive !attern shoul" cause a "ecrease in -(1:B-1C 'choice C). 7he flow loo! woul" show characteristic scoo!in$ because of this !henomenon. A re"uction in 1C an" 7,C 'choice (), which is cause" by lun$ stiffness an" re"uce" lun$ com!liance, is seen in restrictive lun$ "isease. It is the most useful in"icator of this ty!e of lun$ "isease. 00) (*!lanation: 7he correct answer is (. 2y!okalemia can occur in each of the above con"itions. 7he combination of alkalosis an" hy!okalemia su$$ests surre!titious vomitin$. y vomitin$, the !atient loses !rotons in the form of aci" in the stomach, effectively makin$ her alkalotic. 7o correct for the alkalosis, the !roton4!otassium anti!ort system !um!s out !rotons an" increases the intracellular concentration of !otassium, thus "ecreasin$ the serum !otassium level. 7he !atient is hy!otensive from the flui" loss. Chronic "iarrhea 'choice A) is characteri#e" by aci"osis, rather than alkalosis, since basic salts are lost throu$h the lower $astrointestinal system. &otassium is still lost, an" the !atient will be hy!okalemic. .iven the flui" loss, she woul" be hy!otensive as well. Cushin$ syn"rome 'choice ) is cause" by e*cessive amounts of cortisol. Clinical features inclu"e hy!ertension, central obesity, an" muscle weakness. Cortisol e*cess will also cause hy!okalemia an" alkalosis, however, the !atient woul" not be hy!otensive. ,icorice 'choice C), or anise, contains a mineralocorticoi"4like substance calle" $lycyrrhetinic aci". In$estion may lea" to hy!okalemia, hy!ertension, an" metabolic alkalosis. &rimary al"osteronism 'choice %) can be the result of an a"renal a"enoma that !ro"uces e*cessive amounts of al"osterone. 7he al"osterone increases the reabsor!tion of so"ium an" the e*cretion of !otassium an" hy"ro$en ions in the "istal renal tubules, !ro"ucin$

hy!okalemia, alkalosis, an" hy!ertension. 7he so"ium retention lea"s to increase" intravascular volume. 0?) (*!lanation: 7he correct answer is (. Accor"in$ to the available "ata, the !atient is !resentin$ with myocar"ial ischemia in the !resence of anemia. 7he a!!ro!riate treatment, therefore, is a bloo" transfusion. A"ministration of as!irin 'choice A) woul" not be a!!ro!riate in this man who is !assin$ bri$ht re" bloo" !er rectum. 7his !atientAs tachycar"ia is an a!!ro!riate res!onse to the anemia an" hy!ovolemia 'an effort to maintain o*y$en "elivery). 7he a"ministration of a beta blockin$ a$ent 'choice ), therefore, woul" be ina!!ro!riate. 7he !atientAs bloo" !ressure 'com!are it with his !revious !ressures) while su!ine stron$ly su$$ests hy!ovolemia. 7he a"ministration of nitro$lycerin, either sublin$ually 'choice C) or to!ically 'choice %), therefore, becomes ina!!ro!riate. 7he nitro$lycerin will further re"uce !reloa", which in turn, coul" further com!romise the !atientAs car"iac out!ut an" worsen his myocar"ial ischemia. 0/) (*!lanation: 7he correct answer is C. 7his !atient has ty!ical features of !ro$ressive systemic sclerosis 'systemic sclero"erma), as "emonstrate" by her 6aynau" !henomenon, facial telan$iectasias, taut skin, an" hy!ertension. 'If she later "evelo!s calcinosis, she will have e*hibite" the characteristic features of the C6()7 variant of !ro$ressive systemic sclerosis.) 7he sym!toms she "escribes are ty!ical of $astroeso!ha$eal reflu* "isease, which is seen in these !atients as a result of eso!ha$eal hy!omotility, as well as fibrosis of the lower eso!ha$eal s!hincter ',()), which causes re"uce" lower eso!ha$eal s!hincter !ressures. 7he un"erlyin$ "isease !rocess in !ro$ressive systemic sclerosis is a small vessel obliteration that lea"s to secon"ary "iffuse fibrosis. When this affects the eso!ha$us, the eso!ha$eal musculature no lon$er contracts effectively. No s!ecific thera!y is available for the eso!ha$eal "ysfunction of !ro$ressive systemic sclerosis, an" !atients are treate" with anti4eso!ha$eal reflu* re$imens 'e.$., antaci"s, 2; blockers, frequent small fee"in$s, an" elevate" hea" of be") an" !erio"ic 'mechanical) "ilation of any eso!ha$eal strictures that "evelo!. 6emember also that the lon$ history of reflu* eso!ha$itis will !re"is!ose for "evelo!ment of arrettAs eso!ha$us with the risk of !ro$ression to eso!ha$eal cancer. )clero"erma !atients are at risk for coronary artery "isease 'choice A), but her sym!toms '!articularly the tie to in$estion of foo") are not ty!ical of coronary ischemia.

Costochon"ritis 'choice ) is $enerally "ia$nose" with ten"erness over !al!ation of the costochon"ral <oints an" is not associate" with sclero"erma. As state" above, these !atients "evelo! re"uce" !ressures in the ,() an" not hy!ertension of the ,() 'choice %). &ulmonary fibrosis 'choice () can be seen in these !atients, but her sym!toms '!articularly the tie to in$estion of foo") are of $astroeso!ha$eal reflu* an" not of !ulmonary "isease. 0C) (*!lanation: 7he correct answer is %. 7he si$nificant fall in systolic bloo" !ressure with ins!iration "escribes a !atient with a !ulsus !ara"o*us in the settin$ of asthma. 7his is a very im!ortant !hysical fin"in$ in that it in"icates this !atient is in the mi"st of a severe asthmatic attack. 7he other !hysical fin"in$ that in"icates a severe asthmatic attack is the use of the accessory muscles of res!iration, i.e., the internal intercostal muscles an" the sternoclei"omastoi" muscles. 2is hy!ertension 'choice A), lou" whee#in$ 'choice ), !rolon$e" e*!iratory !hase 'choice C), an" tachy!nea 'choice () may all be seen in a !atient "urin$ an asthmatic attack but are not !ro$nosticators of outcome. 0D) (*!lanation: 7he correct answer is (. +any me"ical stu"ents an" resi"ents for$et to think of traumatic hemolytic anemias when consi"erin$ the "ifferential "ia$nosis of anemia. In this case, the !rolon$e" force" march has re!eate"ly com!resse" tiny bloo" vessels, causin$ fra$mentation of some re" cells. '7his force" march may also !re"is!ose for rhab"omyolysis with release of myo$lobin, which can also cause !ink urine, !ositive Hhemo$lobinH on urine "i!stick, an" !ink serum. 6hab"omyolysis coul" be "ifferentiate" from traumatic hemolysis by a very hi$h creatine kinase en#yme level in the bloo".) 7y!ical re" cell fin"in$s in traumatic hemolytic anemia inclu"e trian$le an" helmet sha!es. 7he +C1 may be low an" the 6%W hi$h, reflectin$ a mi*ture of normal an" fra$mente" erythrocytes. Acute leukemia 'choice A) woul" not cause fra$mente" re" cells. Autoimmune attack on re" cells 'choice ) is unlikely because the !atient has no !re"is!osin$ factors, such as other autoimmune "iseases or recent me"ication use. Ina"equate hemo$lobin synthesis 'choice C) woul" cause chronic "isease.

Iron overloa" 'choice %) "oes not cause anemia. 0@) (*!lanation: 7he correct answer is . 7his el"erly man has an acute onset of a Hlower ab"ominal massH an" "iscomfort. Its location an" contour su$$est that it may be a "isten"e" bla""er. 7he mo"est increase in FN an" creatinine is !robably secon"ary to the im!airment to urine !ro"uction !ro"uce" by bla""er obstruction, with resultin$ back !ressure on the ki"ney tubules. 7his !robably occurre" as a result of beni$n !rostatic hy!ertro!hy. 7he first or"er of business woul" be a ureteral catheteri#ation, which woul" !robably allow the "isa!!earance of the Hab"ominal mass,H which is in fact his "isten"e" bla""er. 7his shoul" be !erforme" before any ima$in$ such as sono$ram 'choice C) or C7 scan 'choice %). +asturbation 'choice A) woul" not "ecom!ress the bla""er. &ercutaneous ne!hrostomies 'choice () are invasive !roce"ures that are "one if there is a unilateral or bilateral hy"rone!hrosis. 7his woul" never take !rece"ence over the initial attem!t at "ecom!ression with a urethral catheter. 0I) (*!lanation: 7he correct answer is %. 7he clinical !icture is consistent with &arkinson "isease '&%), a neuro"e$enerative con"ition cause" by "e$eneration of the "o!aminer$ic neurons in the substantia ni$ra that normally !ro<ect to the striatum. 6estin$ tremor, ri$i"ity, an" akinesia are the !rinci!al manifestations. ,evo"o!a is a !recursor of "o!amine an" the most effective "ru$ for the sym!tomatic treatment of & %. Anticholiner$ic "ru$s 'e.$., ben#tro!ine, !rocycli"ine), +AO4 inhibitors 'sele$iline), "o!amine releasers 'amanta"ine), "o!amine a$onists 'e.$., bromocri!tine, !er$oli"e), an" muscle rela*ants '"ia#e!am) are a""itional o!tions frequently use" instea" of or in association with levo"o!a. Clo#a!ine 'choice A) is an anti!sychotic "ru$ that blocks %? "o!aminer$ic rece!tors. It is sometimes use" to anta$oni#e the a"verse effects of confusion, hallucinations, an" "elusions resultin$ from levo"o!a treatment. 7hese sym!toms are !robably relate" to enhancement of "o!aminer$ic transmission in !athways other than the ni$rostriatal !athway. Clo#a!ine "oes not anta$oni#e levo"o!a thera!eutic effects. Corticosteroi"s 'choice ) have no known thera!eutic effects on & %. 2alo!eri"ol 'choice C) is an anti!sychotic that blocks "o!aminer$ic rece!tors. &arkinsonism is one of the main a"verse effects.

7acrine 'choice () is cholinesterase inhibitor that enhances central choliner$ic neurotransmission. &atients with mil" forms of Al#heimer "isease may benefit from treatment with tacrine an" similar "ru$s. ?>) (*!lanation: 7he correct answer is %. 7he !atient has a mali$nant melanoma. +elanomas that have not yet metastasi#e" can be sim!ly treate" with wi"e4sur$ical e*cision, while melanomas that have metastasi#e" are ty!ically fatal because our thera!eutic mo"alities are usually unsuccessful. 9ou shoul" be aware 'it is a favorite !oint amon$ test question writers) that the sin$le most im!ortant !ro$nostic factor for melanoma is the "e!th of the invasion into the "ermis. 7his is because of a feature of the mor!holo$y of the skin: the very most su!erficial layer of the "ermis is relatively free of lym!hatics, while a very rich lym!hatic be" is foun" sli$htly "ee!er. +elanomas $reater than : mm in "e!th are very likely to have !enetrate" this lym!hatic be" an" !otentially metastasi#e". Area of involve" skin 'choice A), "e$ree of aty!ia of the tumor cells 'choice ), "e$ree of color variation 'choice C), an" !ercenta$e of circumference involve" with Hfeatherin$H 'choice () are only in"irectly im!ortant as they may contribute to the local a$$ressiveness of the tumor an" thus increase the chance of "ee!er tumor !enetration. ?:) (*!lanation: 7he correct answer is %. An im!ortant conce!t to reco$ni#e in the treatment of me"ical con"itions is that certain me"ications overla! syn"romes an" are efficacious in many areas. 7his Hco4 treatmentH o!tion ma*imi#es the benefits of each "ru$ in a re$imen an" often a""resses two or more issues simultaneously. In this case, AC( inhibitors have been shown to be very beneficial in !rolon$in$ the survival of !atients with con$estive heart failure 'C2-). 7hey are also useful antihy!ertensive a$ents. .iven this, "iscontinuin$ his AC( inhibitor 'choice A) is clearly incorrect. 7his !atient requires rate control for his atrial fibrillation, that, even at mo"erately elevate" rates, causes car"iovascular embarrassment an" !ulmonary e"ema. )hort of restorin$ this !atientAs atrial contractions, rate control is the best metho" to ensure a"equate mana$ement of atrial fibrillation. %i$o*in, with or without a no"al a$ent such as a beta blocker, has been shown to be reasonably effective at rate control. Amio"arone thera!y 'choice ) is a !harmacolo$ic metho" to convert atrial fibrillation to normal sinus rhythm. It has about the same efficacy as electrical car"ioversion. It "oes nothin$, however, in the short term, to control the rate.

eta blocker thera!y 'choice C) alone is not as efficacious as "i$o*in alone. In a""ition, althou$h some beta blockers 'carve"ilol) are bein$ use" clinically in C2-, not all beta blockers have been shown to be safe for use with this con"ition. 7herefore, althou$h "i$o*in !lus a beta blockin$ a$ent woul" be !referre", there is $oo" reason to initiate "i$o*in thera!y alone for this !atient. -urosemi"e thera!y 'choice () is !artially correct. Althou$h furosemi"e will hel! clinically with the failure, the un"erlyin$ cause for the !ulmonary e"ema will not be a""resse" solely by $ivin$ a "iuretic. 7he more a!!ro!riate thera!y is to control the rate, then the e"ema will resolve. ?;) (*!lanation: 7he correct answer is C. 7his !atient has the acute onset of !o"a$ra an" ankle involvement as well, secon"ary to $out. 7hese attacks often follow the in$estion of re" meats an"Bor alcohol since both of these lea" to transient hy!eruricemia. Note that this !atient has a normal serum uric aci", which may occur "urin$ an acute attackG however, all !atients with $out have some history of hy!eruricemia. 7he a!!ro!riate evaluation of an e"ematous <oint is an arthrocentesis. (*amination of the flui" un"er !olari#in$ li$ht microsco!y in a case such as this will reveal the ty!ical uric aci" crystals, which a!!ear as ne$atively birefrin$ent nee"le4sha!e" crystals 'com!are with choice () un"er !olari#in$ li$ht microsco!y. Althou$h it is always im!ortant to consi"er the !ossibility of an infectious arthritis, this !atient has no risk factors for either $ram4ne$ative bacteremias or arthritis 'choice A). 7he $ram4!ositive cocci in !airs an" clusters 'choice ) are "escribin$ )tre!tococcus, but this or$anism "oes not cause <oint involvement. 6homboi"al crystals 'choice %), when !ositively birefrin$ent, are "ue to calcium !yro!hos!hate '!seu"o$out crystals), an" the ty!ical history of recent in$estion of alcohol or re" meat woul" not usually be elicite". ?0) (*!lanation: 7he correct answer is C. 7he most likely or$anism in an otherwise healthy !atient resi"in$ at home 'Hcommunity4acquire" !neumoniaH) is )tre!tococcus !neumoniae, otherwise known as the H!neumococcus.H 7his or$anism a!!ears as a $ram4!ositive, lancet4sha!e" "i!loccus in short chains on a .ramAs stain of s!utum. It is usually !resent in very lar$e numbers in clinical cases of !neumococcal !neumonia. ). !neumoniae accounts for about two thir"s of community acquire" !neumonias an" occurs most frequently in the very youn$ an" the el"erly. +ore than @> "istinct seroty!es e*ist, which is why the !neumococcal vaccine

'"irecte" a$ainst ;0 "ifferent !olysacchari"e anti$ens), althou$h very useful, "oes not !rotect !erfectly a$ainst !neumonia cause" by ). !neumoniae. .ram4ne$ative cocci in chains 'choice A) is consistent with $onococcus, which rarely causes a bacterial !neumoniaLan" almost always in !eo!le who have $onorrhea of the $enital or$an systems. 7he !atient has no un"erlyin$ "iseases to su$$est that there mi$ht be a !re"is!osition to a $ram4ne$ative !neumonia 'choice ), such as "ue to &seu"omonas or (scherichia coli. (. coli !neumonia is seen in !atients with un"erlyin$ (. coli infection elsewhere, notably in the urinary tract. &seu"omonas !neumonia is common in !atients with un"erlyin$ cystic fibrosis, neutro!enia, AI%), bronchiectasis, or other or$an system "isease serious enou$h to require thera!y in an intensive care unit. .ram4!ositive cocci in clusters 'choice %) su$$est )ta!hylococcus aureus. Althou$h ). aureus can occur ';= of the time) in community4acquire" !neumonias an" may affect the el"erly, it is more commonly seen as a nosocomial infection, !articularly in !atients who have tracheostomy, en"otracheal intubation, immunosu!!ression, or recent sur$ery. Other !articularly vulnerable !atients inclu"e those with un"erlyin$ cystic fibrosis, $ranulomatous lun$ "isease, I1 "ru$ abuse with sta!hylococcal en"ocar"itis, or bacterial su!erinfection followin$ viral !neumonia. -inally, there is no history to su$$est an anaerobic infection with Clostri"ium or ,isteria monocyto$enes, which are $ram4!ositive ro"s 'choice (). Clostri"ium infection can com!licate woun"sG ,isteria infection can be acquire" in utero or throu$h in$estion of contaminate" milk !ro"ucts. ??) (*!lanation: 7he correct answer is A. 7his !atient has central "iabetes insi!i"us. &eo!le can slowly "evelo! "ietary or "rinkin$ habits that seem !erfectly normal to them, but a!!ear strikin$ly unusual to other !eo!le. 7his frequently ha!!ens with con"itions relate" to flui" balance, an" the !atient may be co!in$ with a si$nificant un"erlyin$ !roblem without even reali#in$ it. It may only be when somethin$ unusual ha!!ens, such as this womanAs car tri!, that the !atient comes to me"ical attention. In this case, the !atient essentially un"erwent a water "e!rivation test, but without the close me"ical su!ervision always offere" when such a test is !erforme" in a hos!ital settin$. 7he facts that she continue" to !ro"uce "ilute urine an" that the urine osmolarity coul" be increase" with vaso!ressin in"icate that the "iabetes insi!i"us !robably has a central ori$in. A hea" C7 woul" likely confirm the "ia$nosis by showin$ a lar$e !ituitary a"enoma, which coul" com!romise anti"iuretic hormone flow "own the !ituitary stalk. Com!ulsive water "rinkin$ 'choice ) is more likely to mimic ne!hro$enic "iabetes insi!i"us than central "iabetes insi!i"us.

%iabetes mellitus 'choices C an" %) can !ro"uce hi$h urine flow rates, but only secon"ary to an osmotic "iuresis seen when urine $lucose is hi$h. Ne!hro$enic "iabetes insi!i"us 'choice () "oes not res!on" to vaso!ressin. ?/) (*!lanation: 7he correct answer is (. 7his !atient has constrictive !ericar"itis. -illin$ is re"uce" abru!tly when the elastic limit of the !ericar"ium is reache". )uch !atients often a!!ear to have a chronic illness. 7hey have a !ositive 5ussmaulAs si$n in which venous !ressure "eclines in ins!iration. 7he a!ical !ulse is re"uce". A !ericar"ial knock is hear". 86) is low in volta$e. 7he con"ition occurs because of the healin$ of a former acute !ericar"itis or a chronic !ericar"ial effusion with obliteration of the !ericar"ial cavity. Colla!se" ri$ht ventricle in "iastole 'choice A) is seen in acute !ericar"ial tam!ona"e because of an obstruction to car"iac fillin$ an" concomitant elevate" ri$ht4si"e" !ressure. 7he accumulation of flui" in the !ericar"ium is sufficient to cause a si$nificant obstruction to inflow of bloo" to the ventricles. A lar$e ri$ht ventricle 'choice ) is seen ty!ically in con"itions such as a ri$ht ventricular infarct. 7he heart will a!!ear lar$e on a !lain chest *4ray film. +ana$ement of a ri$ht ventricular infarct involves a$$ressive flui" resuscitation. A !ericar"ial effusion 'choice C) may result from accumulation of flui" in the !ericar"ium. 7his may lea" to car"iac tam!ona"e. 7he effusion may be seen in !atients with cancer or rheumatoi" arthritis, or in those with con"itions lea"in$ to blee"in$ into the !ericar"ial s!ace. A thick myocar"ium 'choice %) is seen in restrictive car"iomyo!athy. Abnormalities of the !ericar"ium are not seen. 7his car"iomyo!athy may result from amyloi"osis, hemochromatosis, sarcoi"osis, or sclero"erma. It is characteri#e" by the !atient havin$ a well4"efine" a!ical beat, left ventricular failure, )0, an" bun"le branch block. ?C) (*!lanation: 7he correct answer is C. 7he !atient most likely is e*!eriencin$ the effects of acetamino!hen over"ose. After a lar$e over"ose of acetamino!hen, the metabolism of the "ru$ results in the formation of merca!turic aci", a metabolite that can be e*crete" safely if con<u$ate" to $lutathione. When $lutathione levels are "e!lete", reactive metabolites are instea" forme" that bin" covalently to he!atocytes an" cause cell lysis, a !rocess that may lea" to fulminant he!atic failure.

Alcohol "ehy"ro$enase 'choice A), the en#yme res!onsible for the metabolism of alcohol, may in fact be elevate" in this man, who has a known history of alcohol abuse. Catalase 'choice ) is an antio*i"ant that reacts with !ero*i"e molecules an" shoul" not necessarily be affecte" in this case. .lyco$en 'choice %) an" vitamin 5 'choice (), althou$h !ossibly "e!lete" in this case, are not the !ro*imate or ultimate cause of the elevate" he!atic en#ymes an" risin$ !rothrombin time. ?D) (*!lanation: 7he correct answer is . One of the im!ortant com!lications of "iverticular "isease is blee"in$ that can be massive enou$h, as in this !atient, to be !otentially life4threatenin$. 7he mechanism involve" in !ro"ucin$ blee"in$ in "iverticular "isease is usually the "isru!tion of an often sin$le arteriole or small artery in a "iverticulum. 7he "iverticulum involve" is often, but not always, !articularly lar$e. 7he small vessel can be sufficiently stretche" by the "iverticulum that it cannot un"er$o contraction, which normally sto!s bloo" flow throu$h this si#e vessel after "ama$e. In many cases, the blee"in$ may sto! s!ontaneously. If it recurs within a few "ays, sur$ical resection of the involve" se$ment of bowel 'if it can be i"entifie", which can be !roblematic) may be require". 7he !atient "oes not have any evi"ence of a "iffuse "escen"in$ colon ischemia 'choice A), which woul" have !ro"uce" an obviously e"ematous bowel mucosa, often with many focal hemorrha$es. Inferior mesenteric veins course !rimarily in the serosal fat of the bowel, an" consequently "o not oo#e into the bowel lumen as a cause of blee"in$ 'choice C). 7hrombosis of a branch of the inferior mesenteric artery 'choice %) woul" cause an ischemic colitis with features as "escribe" in the "iscussion of choice A, which are not "escribe" in this !atient. 7hrombosis of internal hemorrhoi"s 'choice () is very common but "oes not cause blee"in$ !er se. ?@) (*!lanation: 7he correct answer is %. 7he sensitivity of a test is the !robability that results will be !ositive in !atients who actually have the "isease. It reflects the tests ability to accurately "ia$nose all cases of the "isease. When "ata are arran$e" as above, the sensitivity will be aB'aMc) U C;./=.

7he s!ecificity of a test is the !robability that the test result will be ne$ative in those without the "isease. If the !atient has the "isease but has a ne$ative test result the result is terme" a false ne$ative. It is "B'bM") U 00.0= 'choice A). 7he !ositive !re"ictive value of a test is the !robability that an in"ivi"ual who $ets a !ositive test result actually has the "isease. It is aB'aMb) U 0@./= 'choice ). 7he ne$ative !re"ictive value 'choice C) of a test is the !robability that an in"ivi"ual who $ets a ne$ative test result "oes not have the "isease. It is "B'cM") U /D.:=. 7he sensitivity woul" be :>>= 'choice () if the screenin$ test were as accurate as the $ol" stan"ar" 'bio!sy), an" woul" become the new $ol" stan"ar". ?I) (*!lanation: 7he correct answer is . 7his !atient has secon"ary hy!er!arathyroi"ism with a hi$h !arathyroi" hormone '&72) an" a calcium * !hos!hate of about C?. 9ou must first lower the !hos!hate quickly to avoi" metastatic calcification. Oral calcium $iven with meals will bin" the !hos!hates in the foo", an" lower the !hos!hate. 1itamin % $iven orally 'choice A) will lower the &72 but may only be $iven after the level of !hos!hate is lowere". Otherwise, the vitamin % coul" raise the calcium an" !hos!hate. Aluminum hy"ro*i"e 'choice C) will "o the same thin$, but will risk aluminum to*icity an" shoul" be avoi"e". 7here is usually no nee" to start ur$ent "ialysis 'choice %) to lower the !hos!hate. It is not use" as a first4line treatment in hy!er!hos!hatemia. )ur$ery 'choice () is ke!t as a last o!tion if all else fails. />) (*!lanation: 7he correct answer is (. 7his is tetanus, cause" by a reaction to the to*in !ro"uce" by Clostri"ium tetani. While the classic history involves a contaminate" !uncture woun", you shoul" be aware that no such history may be eluci"ate" in many !atients, !ossibly because the initial in<ury was so trivial as to have since been for$otten. Chil"hoo" !rimary immuni#ation is common, but a"ult !atients who have never receive" booster shots may have sufficiently low antibo"y titers to "evelo! clinical "isease. 7he incubation !erio" followin$ e*!osure ran$es from ; to /> "ays, with ty!ical incubation times bein$ / to :> "ays. Worl"wi"e,

tetanus has a />= mortality rate, with !articularly hi$h mortality amon$ babies, the el"erly, an" "ru$ abusers. 7he !resentation illustrate" in the question stem is ty!ical. As the "isease !ro$resses, the ri$i"ity an" s!asm of muscles becomes wi"es!rea", an" may even leave the !atient in o!isthotonus with an arche" back. 7reatment of establishe" tetanus is com!le*. 7he !atient shoul" be ke!t in a quiet room, an" a"equate "oses of human immune serum $lobulin an" anti4to*in be use". Woun" "ebri"ement shoul" be use" at the site of in<uryG antibiotics '!enicillin . or tetracycline) are use" a"<unctively. It is critical that the airways be maintaine", if necessary with mechanical ventilation throu$h a tracheostomy cou!le" with neuromuscular blocka"e. I1 hy!eralimentation may re"uce the risk of as!iration. Chest !hysiothera!y is nee"e" to !revent !neumonia. otulism 'choice A) causes a flacci" !aralysis. %i!htheria 'choice ) causes severe res!iratory "isease with !seu"omembrane formation on the res!iratory mucosa. ,yme "isease 'choice C) causes a rash followe" by a flu4like illness. &ertussis 'choice %) causes Hwhoo!in$ cou$h.H

USMLE Step 2 Practice Test Block 4 Name: Instructions: Answer the questions below to the best of your ability. When you finish the test, click the Check button at the bottom to view the results.

1.>ne year follo'ing a "iffic#lt $irth% 'hich 'as accompanie" $y consi"era$le $loo" loss% a &4!year!ol" 'oman comes to the physician complaining of profo#n" fatig#e% loss of appetite% col" intolerance% an" constipation. She "enies feelings of "epression% $#t she feels constantly apathetic an" tire" an" has no se.#al "ri)e. /er menses ha)e not

res#me" after "eli)ery. /er $loo" press#re is 1447< mm /g. E.amination re)eals "ry an" pale skin% 'ith thin an" $rittle hair. =hich of the follo'ing is the most likely "iagnosisA6Postpart#m "epression B6Primary a"renal ins#fficiency 26Primary hypothyroi"ism @6Sheehan syn"rome :ormal La$s 2.A <1!year!ol" 'oman chooses a physician as a primary care pro)i"er after rea"ing his $iography in the health maintenance organiDation literat#re. The physician "isco)ers that she 'as recently a"mitte" to the hospital for palpitations an" fo#n" to ha)e ne'ly "iagnose" atrial fi$rillation 3A56. She 'as place" on "igo.in an" )erapamil for rate control an" loa"e" 'ith 2o#ma"in for anticoag#lation. After se)eral trips to the anticoag#lation clinic she "eci"e" to stop taking her 2o#ma"in% as it 'as "iffic#lt for her to lea)e 'ork "#ring the "ay. She no' calls the office complaining of a col"% pale% an" painf#l right leg starting an ho#r ago. =hich of the follo'ing is the most likely e.planation for her symptomsa6 2ere$ro)asc#lar acci"ent 32?A6 $6@eep )ein throm$osis 3@?T6 c6M#ltiple sclerosis 3MS6 "6Aayna#" syn"rome e6Throm$oem$olism :ormal La$s &.A 2 !year!ol" patient 'ith asthma arri)es at the emergency "epartment complaining of e.treme shortness of $reath. /e has $een compliant 'ith his me"ications% 'hich are metaproterenol an" inhale" $etamethasone. >)er the past < ho#rs% his 'heeDing an" shortness of $reath ha)e $ecome increasingly se)ere. At present% his $loo" press#re is 1&<714 mm /g an" 'ith "eep inspiration falls to 11<7 < mm /g. /is p#lse is 12<7min% an" respirations are &27min an" la$ore". There is retraction of the sternoclei"omastoi" m#scles 'ith inspiration. There are soft 'heeDes an" poor air mo)ement "iff#sely thro#gho#t $oth l#ng fiel"s. /is peak e.piratory flo' rate is <4 L7min% an" he is imme"iately starte" on an al$#terol ne$#liDer an" gi)en +? hy"rocortisone. /is

arterial $loo" gas re)eale" a p/ of (. % p2>2 of 21 mm /g% an" a p>2 of <4 mm /g. T'enty min#tes later% a repeat peak flo' rate is still <4 L7min. A repeat arterial $loo" gas re)eals that his p/ is no' (.4<% his p2>2 is no' &4 mm /g% an" his p>2 is <4 mm /g. =hich of the follo'ing is the most appropriate ne.t step in managementa6A"minister +? epinephrine $6A"minister 144B o.ygen )ia a tight fitting face mask c6A"minister s#$c#taneo#s epinephrine "6Prepare for int#$ation e6 Place a right thoracostomy t#$e :ormal La$s 4.A 4!year!ol" man presents to his physician for a cons#ltation. The patient is concerne" that he may ha)e gotten some 0"isease0 from $loo". /e has rea" in the ne'spapers an" seen on T? that people 'ho get $loo" transf#sions can get 0"iseases%0 an" he recei)e" a transf#sion in 1812. The patient has $een a patient in this office for a fe' years% an" his only other past me"ical history is hypercholesterolemia an" hypertension% for 'hich he is $eing treate" 'ith pra)astatin an" hy"rochlorothiaDi"e. =hich of the follo'ing is the most appropriate screening testa6/epatitis B $6/epatitis 2 c6/+? "6PP@ skin testing e6Aapi" plasma reagin testing :ormal La$s .A & !year!ol" male 'ith en"!stage renal "isease presents 'ith syncope. The patient "enies chest pain or "yspnea. Physical e.amination is remarka$le for a left e.tremity A? fist#la. /is creatinine is 14 mg7"L% #rea nitrogen is 11 mg7"L an" potassi#m is 1.1 mE*7L. E2C re)eals sin#s $ra"ycar"ia 'ith a sine 'a)e pattern. =hich of the follo'ing is the most appropriate initial step in managementa6+? calci#m gl#conate $6+? "e.trose

c6+? f#rosemi"e "6+? so"i#m $icar$onate e6Aectal Maye.alate :ormal La$s <.A 4!year!ol" man presents 'ith a history of rec#rrent attacks of hea"ache% 'hich manifest 'ith a pec#liar rhythmic pattern. Each attack $egins on a'akening% appro.imately at the same time. /e reports se)ere #nilateral pain localiDe" in the right eye an" temple% an" accompanie" $y r#nning nose% lacrimation% an" s'eating on the same si"e. The attack spontaneo#sly resol)es after 2 ho#rs. :either his si$lings nor his parents ha" similar hea"aches. /is temperat#re is &( 2 381.< 56% $loo" press#re is 1&4714 mm /g% an" p#lse is 147min. /is p#pils are e*#al an" normally reacti)e to light. 5#n"#scopic e.amination is #nremarka$le. =hich of the follo'ing is the most likely "iagnosisa6Ac#te gla#coma $62l#ster hea"ache c6Ciant cell arteritis "6Clossopharyngeal ne#ralgia e6Migraine f6Trigeminal ne#ralgia :ormal La$s (. A & !year!ol" 'oman cons#lts a physician a$o#t a lesion on her finger that "e)elope" after a minor in9#ry. The lesion is a scarlet% apparently )asc#lar% raise" no"#le that $lee"s easily an" "oes not $lanch on press#re. :o inflammatory changes are seen in the s#rro#n"ing skin an" the 'oman reports that the lesion is not #ncomforta$le% altho#gh its cosmetic appearance "oes $other her. =hich of the follo'ing is the most likely "iagnosisa6 @ermatofi$roma $6Meloi" c6Meratoacanthoma "6Pyogenic gran#loma e6 =art :ormal La$s

1.A !year!ol" man 'ith no kno'n car"iac history presents to the emergency "epartment complaining of cr#shing s#$sternal chest press#re that $egan 24 min#tes prior to his arri)al. /e took sil"enafil 3?iagra6 earlier in the e)ening prior to se.#al interco#rse. /is past me"ical history is significant for $ilateral ing#inal hernia repairs < 'eeks ago% remote peptic #lcer "isease% an" prior cocaine an" heroin a""iction. >n e.amination he is "iaphoretic an" appears an.io#s. /is $loo" press#re is 1 47( mm /g 'ith a p#lse of 1447min. An electrocar"iogram 3E2C6 o$taine" 'hile he ha" se)ere chest pain re)eale" tall positi)e T 'a)es 3hyperac#te T 'a)es6 an" 1!mm ele)ation of the ST segments in lea"s ?2 thro#gh ? . A ser#m creatine kinase 32M6 "ra'n on presentation ret#rne" at 1 U7L 3normal Q 2 4U7L6.=hich of the follo'ing is the most likely "iagnosisa6 Aortic "issection $6Myocar"ial infarction 3M+6 c6P#lmonary em$ol#s 3PE6 "6Spontaneo#s pne#mothora. e6Sta$le angina :ormal La$s 8.A (4!year!ol" man presents 'ith a complaint of intense pain from a re" an" s'ollen right toe. /e has ha" no prior e)ents s#ch as this. /e cannot recall tra#ma to the toe. An .!ray film of the right foot is negati)e for a fract#re. /e is taking no me"ications% an" "enies a history of alcohol a$#se. /e is gi)en a "iagnosis of go#t an" his ser#m is "ra'n for analysis. =hich of the follo'ing fin"ings 'o#l" $e most helpf#l in making the "iagnosis of go#t in this patienta6 Associate" right ankle eff#sion $6Painless el$o' no"#le c6Aesponse to colchicine "6Ser#m #ric aci" ele)ation e6F!ray film sho'ing first metatarsophalangeal 9oint erosion :ormal La$s 14.A 2&!year!ol" man is a"mitte" to the me"ical ser)ices 'ith a se)ere asthma attack. /e is also na#seo#s an" has )omite" t'ice to"ay. The patient has a long history of se)ere asthma 'ith m#ltiple

hospitaliDations an" one int#$ation & years ago. T'o "ays prior to a"mission% he 'as e.pose" to "#st 'hile mo)ing a file ca$inet in his $asement. Since that time% has ha" progressi)ely 'orsening shortness of $reath. /e ha" trie" home al$#terol an" ipratropi#m ne$#liDers% as 'ell as his stan"ar" cromolyn therapy% $#t none of these inter)entions relie)e" his symptoms. +n the hospital% the man,s peak flo' rates are "ecrease" $y nearly 4B from $aseline. =hich of the follo'ing agents sho#l" most likely $e a""e" to the patient,s therapy to alle)iate his c#rrent symptomsa6 Beclomethasone $6@iso"i#m cromoglycate c6/y"rocortisone "6Pre"nisone e6 Theophylline :ormal La$s 11. A 42!year!ol" man presents 'ith a 1!"ay history of increasing na#sea% )omiting% an" lethargy. /e has an e.tensi)e smoking history an" 'as recently "iagnose" 'ith l#ng cancer. /e is not taking any me"ications an" has not yet initiate" chemotherapy. >n physical e.amination% he is afe$rile an" somnolent. /is l#ngs are clear to a#sc#ltation% an" his heart is reg#lar in rate an" rhythm. /is skin sho's ero"ing calci#m "eposits. La$oratory res#lts in"icate a ser#m calci#m le)el of 1&.4 mg7"L. =hich of the follo'ing is the most appropriate initial step in managementa62alcitonin $6Eti"ronate c6/y"rochlorothiaDi"e "6+? saline e6 Pre"nisone :ormal La$s 12. A < !year!ol" retire" acco#ntant 'ith a <4!pack!year smoking history presents for his ann#al physical e.amination. /e 'as last seen & years ago% 'hen he presente" 'ith an episo"e of ac#te $ronchitis. /e reports that he has $een co#ghing o)er the past se)eral months 'ith pro"#cti)e green sp#t#m. /e "enies any "yspnea or limitation in his "aily acti)ities. >n physical e.amination% his $loo" press#re is 12<7<2

mm /g% p#lse is 147min% an" respirations are 247min. /e has "iff#se $ilateral e.piratory rhonchi 'ith a marke"ly prolonge" e.piratory phase. /is car"iac e.amination re)eals a secon" heart so#n" that increases in intensity 'ith inspiration. The li)er e"ge is 14 cm into the mi"cla)ic#lar line an" mil"ly ten"er to palpation. There is $ilateral lo'er e.tremity e"ema to the knees. A chest .!ray film re)eals hyperinflation of $oth l#ng fiel"s% an" p#lmonary f#nction tests re)eal a "iminishe" force" e.piratory )ol#me in 1 secon" 35E?16 to force" )ital capacity 35?26 ratio. Ao#tine la$oratories are sent. =hich of the follo'ing 'o#l" most likely $e e.pecte"a6A hemoglo$in of 8 g7"L $6A hemoglo$in of 1( g7"L c6A platelet co#nt of 14%444 "6A platelet co#nt of 142%444 e6A 'hite $loo" cell co#nt of &444 :ormal La$s 1&. A 4 !year!ol" S'e"ish man presents for follo'!#p after "ischarge from the hospital for an esophageal )ariceal $lee". /e ha" #n"ergone esophageal sclerotherapy to control the $lee"ing. >n "ischarge% he 'as prescri$e" atenolol. /e has no history of alcohol #se or me"ication #se. /e has no risk factors for hepatitis. /e kno's of t'o co#sins 'ho 0"ie" of li)er "isease.0 /is sister "ie" of melanoma% so he takes preca#tions to a)oi" s#n e.pos#re. >n physical e.amination% his skin has a "ark% $ronDe" appearance. /e is anicteric. /is li)er an" spleen are normal in siDe. There is no ascites or peripheral e"ema. =hich of the follo'ing are the most appropriate la$oratory tests to or"era6Alpha1 antitrypsin genotype $6Alpha1 antitrypsin phenotype c6Ser#m copper "6Ser#m ferritin e6 Ser#m nickel :ormal La$s 14.A 48!year!ol" man 'ith a long history of alcohol a$#se is $ro#ght to the physician $y his 'ife $eca#se of gra"#ally increasing conf#sion. /e ha" $een 'orking as an apartment $#il"ing s#perinten"ent #ntil appro.imately 2 'eeks ago% 'hen he $egan feeling "ro'sy thro#gho#t the "ay an" ha" "iffic#lty sleeping at night. Since then% he has $ecome

conf#se" an" occasionally "isoriente" as to time an" "ay. /e can recogniDe his 'ife an" neigh$ors% $#t cannot maintain cas#al con)ersations. /e "enies any fe)er% chills% or a$"ominal pain. Si. months ago% he 'as a"mitte" to the hospital 'ith an #pper gastrointestinal $lee"% 'hich 'as "#e to $lee"ing esophageal )arices. /is me"ications incl#"e raniti"ine% spironolactone% f#rosemi"e% an" propranolol. >n physical e.amination% he is lethargic an" "ishe)ele". /is temperat#re is &<.8 2 381.4 56% $loo" press#re is 1127<4 mm /g% p#lse is <27min% an" respirations are 117min. /e has mil"ly icteric sclera an" $itemporal 'asting. /is oral m#co#s mem$ranes are "ry. There is no 9#g#lo)eno#s "istension. /is l#ngs are clear% an" he has a reg#lar heart rhythm. /is a$"omen re)eals a firm li)er e"ge 'ith a li)er span of ( cm in the mi"cla)ic#lar line. There is no shifting "#llness% an" a spleen tip is not palpa$le. /e has no peripheral e"ema. >n a mental stat#s e.amination he recogniDes the physician $#t cannot name the "ate% the reason for his )isit% or his home a""ress. :e#rologic e.amination is nonfocal% an" asteri.is is present. =hich of the follo'ing la$oratory a$normalities 'ill most likely $e fo#n"a6BU: of & mg7"L $6Cl#cose of 182 mg7"L c6Potassi#m of .( mE*7L "6Ser#m $icar$onate of 1< mE*7L e6 Ser#m calci#m of 14.2 mE*7L :ormal La$s 1 .A 4!year!ol" 'oman is $eing treate" for long!stan"ing se)ere rhe#matoi" arthritis. Screening $loo" st#"ies "emonstrate a mo"erate anemia 'ith a hemoglo$in le)el of 14 g7"L. The patient,s n#tritional stat#s is goo"% an" the clinician s#spects she has anemia of chronic "isease. =hich of the follo'ing erythrocyte fin"ings 'o#l" $e most likely to $e seen on peripheral $loo" smeara6All normal morphology $6Macrocytes c6:#mero#s spherocytes "6Target cells e6Tear "rop forms :ormal La$s

1<.A &1!year!ol" man comes to his physician,s office t'o 'eeks after $eing seen in the emergency room for ac#te nephrolithiasis. /e passe" his stone in the emergency room an" he reports that it 'as ma"e of calci#m o.alate. /e is concerne" a$o#t rec#rrences% as the pain 'as )ery se)ere. =hich of the follo'ing is the most appropriate ne.t step in managementa6+nitiation of thiaDi"e "i#retic therapy $6+nitiation of loop "i#retic therapy c6+nitiation of allop#rinol therapy "6Maintenance of an alkaline #rine e6 Maintenance of large #rine )ol#mes )ia copio#s 'ater cons#mption :ormal La$s 1(. A & !year!ol" man comes to the physician $eca#se of "aily se)ere% perior$ital% right!si"e" hea"aches o)er the past < 'eeks. The pain often a'akens him from sleep an" is so e.cr#ciating that he 'ants to $ang his hea" against the 'all. /e reports nasal 0st#ffiness0 an" na#sea. /e has ha" similar episo"es on 2 pre)io#s occasions. Physical e.amination sho's right!si"e" ptosis% lacrimation% an" re""ening of the right eye. La$oratory st#"ies sho'; /emoglo$in....................................1< g7"L /ematocrit.....................................48B Le#kocyte co#nt.............................<4447mm& Erythrocyte se"imentation rate......... mm7hr =hich of the follo'ing is the most likely "iagnosisa62lassic migraine $62l#ster hea"ache c6 2ommon migraine "6S#$arachnoi" hemorrhage e6Temporal arteritis f6Tension hea"ache :ormal La$s 11.A & !year!ol" 'oman has a pigmente" lesion remo)e" from her palm for cosmetic reasons. The lesion is light $ro'n in color% consisting of a 2 mm )ery slightly ele)ate" mac#le 'ith 'ell "emarcate" e"ges. The a"9acent skin has normal color. Microscopic e.amination of the lesion

"emonstrates cl#stering of pigmente" melanocytes at the epi"ermo"ermal 9#nction. :o cl#sters of melanocytes are seen in the "ermis. =hich of the follo'ing "iagnoses is correcta62ompo#n" ne)#s $6R#nctional ne)#s c6/alo ne)#s "6+ntra"ermal ne)#s e6Lentigo :ormal La$s 18. A <1!year!ol" man is $ro#ght to the emergency "epartment for chest pain. The patient has a long!history of coronary artery "isease an" is stat#s!post a coronary $ypass proce"#re < years ago. The patient has chronic sta$le angina that is #s#ally precipitate" $y acti)ity an" relie)e" $y rest. A$o#t & 'eeks ago% his physician prescri$e" sil"enafil 3?iagra6% an" he has $een #sing the "r#g 'ith s#ccess. This morning% he "e)elope" ac#te onset of s#$sternal chest pain% ra"iating to his left arm. This pain is not relie)e" $y rest. The patient last took a sil"enafil the night $efore. =hich of the follo'ing treatments is a$sol#tely contrain"icate" in this sit#ationa6Aspirin 3ASA6 $62aptopril c6Metoprolol "6Morphine e6 :itroglycerin f6Tiss#e plasminogen acti)ator 3tPA6 :ormal La$s 24.A &4!year!ol" 'oman presents 'ith lo' $ack pain% an" a s'ollen an" painf#l right ankle an" left knee for 1 'eek. She also reports temperat#res to &8.4 2 3142.2 56. She recalls that a fe' 'eeks ago% she ha" "iarrhea% a$"ominal cramps% an" fe)er% 'hich laste" for a fe' "ays. She "oes not ha)e pre)io#s history of 9oint "iseases or se.#ally transmitte" "iseases. E.amination re)eals skin p#st#les an" cr#ste" )esicles on her palms an" soles% re" con9#ncti)ae% an" arthritis of the left knee an" right ankle. There is no hepatomegaly% splenomegaly% or lympha"enopathy. La$oratory analysis sho' an ele)ate" erythrocyte

se"imentation rate 3ESA6% $#t no anemia. =hich of the follo'ing is the most likely "iagnosisa6 Psoriatic arthritis $6Aeiter syn"rome c6Ahe#matic fe)er "6Ahe#matoi" arthritis e6Syphilitic infection :ormal La$s 21. A (4!year!ol" man presents to the clinic for e)al#ation of shortness of $reath an" chest pain. The pain has $een gra"#ally progressi)e o)er the past < months an" increases 'ith inspiration% 'hich he "escri$es as 0sharp an" sta$$ing.0 /e has ha" a nonpro"#cti)e co#gh "#ring the same time an" "enies any smoking history. /e retire" ( years ago after 'orking as a ship $#il"er for more than 44 years. >n physical e.amination% his $loo" press#re is 1<4782 mm /g% p#lse is 117min% an" respirations are 247min. There is mil" cl#$$ing of the fingernail $e"s% an" he appears mil"ly cyanotic. >n a l#ng e.amination% there are en" inspiratory "ry rales $ilaterally 'ith re"#ce" $reath so#n"s. P#lmonary f#nction tests re)eal a "ecrease" force" e.piratory )ol#me in 1 secon" 35E?16 an" a "ecrease" force" )ital capacity 35?26% $#t the ratio 'as normal. Total l#ng capacity is re"#ce". A chest .!ray film re)eals "iff#se $ilateral interstitial fi$rosis an" calcifications at $oth ple#ral $ases. =hich of the follo'ing is the most likely "iagnosisa6As$estosis $6+"iopathic interstitial fi$rosis c6Legionella pne#monia "6Pne#mocystis carinii pne#monia e6T#$erc#losis :ormal La$s 22.>ne month follo'ing "eli)ery of her $a$y% a ne' mother complains to her physician of feeling )ery tire" an" col" all of the time% e)en 'hen she gets a"e*#ate sleep. Physical e.amination is nota$le for marginal thyroi" enlargement 'itho#t ten"erness. Thyroi" st#"ies sho' a total T4 of &. mg7"L% 'ith thir"!generation st#"ies sho'ing a thyroi"! stim#lating hormone 3TS/6 of (. m+U7mL. =hich of the follo'ing is the most likely "iagnosis-

a6E#thyroi" sick syn"rome $6 Cra)es "isease c6+o"ine "eficiency "6Silent lymphocytic thyroi"itis e6 S#$ac#te thyroi"itis :ormal La$s 2&.A <&!year!ol" man comes to the physician $eca#se of a lesion on the si"e of his forehea". /e has ha" the lesion for 0many years0. /e is in the office to"ay $eca#se% he says 0 my "a#ghter,s 'e""ing is in 2 months an" my 'ife 'ants me to ha)e this #gly thing remo)e"0. >n physical e.amination% the lesion is a 1. cm $ro'n pla*#e 'ith an a"herent greasy scale. +t appears to $e 0st#ck!on0. =hich of the follo'ing is the most likely "iagnosisa6Actinic keratosis $6Basal cell carcinoma c6@ermatofi$roma "6Melanoma e6Psoriasis f6Se$orrheic keratosis g6S*#amo#s cell carcinoma :ormal La$s 24. A <&!year!ol" retire" airline pilot comes to the office for a first time )isit. /e is s'itching primary care "octors $eca#se he is )ery #nhappy 'ith the care that he is getting else'here. /e $rings his me"ical recor"% 'hich "oc#ments long!stan"ing cirrhosis an" portal hypertension "#e to pre)io#s alcohol a$#se. /e has no history of gastrointestinal $lee"ing. /e has a )ery long me"ication list an" he reports that his pharmacist tol" him that his me"ications 0'ere 'rong0 an" that his "octor 'as prescri$ing "r#gs improperly. >n re)ie' of his me"ication list% a n#m$er of me"ication incompati$ilities are fo#n". 2oa"ministration of 'hich of the follo'ing is likely to ca#se this patient the greatest harm if not correcte"a6Lact#lose an" oral potassi#m $6Pre"nisone an" inhale" al$#terol c6Propranolol an" isosor$i"e "initrate "6Spironolactone an" oral potassi#m e6Spironolactone an" hy"rochlorothiaDi"e

:ormal La$s 2 . A &1!year!ol" professional $o"y$#il"er presents complaining of & 'eeks of 'orsening fatig#e% lo'!gra"e fe)ers% an" myalgias% as 'ell as the gra"#al onset of 9a#n"ice o)er the past 'eek. /e a"mits to #nprotecte" anal se. & months earlier after a championship $o"y$#il"ing e)ent. /e "enies any other homose.#al acti)ities. /e has no other me"ical pro$lems an" is on no me"ications. >n physical e.amination% he appears ac#tely ill% 'ith a temperat#re of &8.& 2 3142.( 56% a $loo" press#re of 11<7<4 mm /g% an" a p#lse of 1147min. /e is "eeply icteric an" has m#ltiple e.coriations o)er his entire $o"y. /is li)er e"ge is smooth an" )ery ten"er an" meas#res 14 cm in the mi"cla)ic#lar line. There is no shifting "#llness% an" a spleen tip is palpa$le 4 cm $elo' the left costal margin. Stool is $ro'n an" g#aiac negati)e% an" there is no peripheral e"ema. =hich of the follo'ing la$oratory fin"ings 'o#l" most likely in"icate the 'orst prognosisa6Prothrom$in time of 18.< secon"s $6Aspartate aminotransferase 3AST6 of 81& U7L c6Alanine aminotransferase 3ALT6 of 1&%424 U7L "6Total $ilir#$in of 2(.4 mg7"L e6=hite $loo" cell co#nt of 11%4447mm& :ormal La$s 2<.The 11!year!ol" son of an Ara$ Sheik is flo'n to a hospital in the Unite" States after his local "octor "isco)ers an a$"ominal mass. The yo#ng man ha" a t'o year history of 'eight loss accompanie" $y pro"#ction of malo"oro#s stool 'ith e.cessi)e flat#lence. @#o"enal $iopsy 'ith histopathologic e.amination "emonstrates )illo#s atrophy an" massi)e infiltration of the lamina propria $y lymphocytes% plasma cells% an" imm#no$lasts that "o not morphologically appear o)ertly malignant. Biopsy of a mesenteric lymph no"e sho's no"e replacement $y a similar infiltrate. =hich of the follo'ing is the most likely "iagnosisa6/ea)y chain "isease $6Macroglo$#linemia c6Monoclonal gammopathy of #n"etermine" significance "6M#ltiple myeloma e6Myelo"ysplastic syn"rome

:ormal La$s 2(.A 4(!year!ol" la'yer presents to his physician for e)al#ation of "iffic#lty )oi"ing for the past fe' years. /e "escri$es a "elay 'hen he 0attempts to go0 an" that he often 'ets his #n"er'ear 'ith a""itional #rine after he has finishe". /e is marrie" an" has one teenage chil". /is past me"ical history is remarka$le for essential hypertension that he claims is "iet controlle". /is $loo" press#re is 1(4782 mm /g% an" his p#lse is 127min. /is physical e.amination is remarka$le for a prominent apical imp#lse% laterally "isplace"% an" a faint a$"ominal $r#it. /is prostate is non!ten"er an" appears large 'ith no palpa$le no"#les or irreg#larities. =hich of the follo'ing is the most appropriate antihypertensi)e therapy for this patienta6Lisinopril $6Metoprolol c6:ife"ipine "6TeraDosin e6 ?erapamil :ormal La$s 21. A &4!year!ol" man 'ith a long history of seiD#res has $een me"ically manage" 'ith anticon)#lsants. /e no' presents 'ith repetiti)e generaliDe" motor con)#lsions that ha)e persiste" for the past 1 ho#r. After a"ministration of intra)eno#s $enDo"iaDepines at a high "ose% his seiD#res stoppe". =hich of the follo'ing is the ne.t co#rse of actiona6Electrocon)#lsi)e therapy $6+ntra)eno#s pento$ar$ital c6+ntra)eno#s phenytoin "6>ral car$amaDepine e6>ral ethos#.imi"e :ormal La$s 28.A <4!year!ol" man comes to the office $eca#se of a lesion on his lo'er lip that has $een gro'ing for the past 8 months. +t is painless an" he is no' here $eca#se his 'ife ma"e him come. /e has not seen a physician for the past 4 years. /e 'orks as a farmer an" is e.pose" to the s#n. >n physical e.amination% the lo'er lip is fi.e" to the lo'er aspect of the man"i$le. =hich of the follo'ing is the correct "iagnosis-

a6Basal cell carcinoma $6Erythema no"os#m c6Le#koplakia "6Melanoma e6S*#amo#s cell carcinoma :ormal La$s &4. A 1!year!ol" man presents to his physician for e)al#ation of his risk for heart "isease. The patient is )ery concerne" a$o#t his chance for a heart attack $eca#se his father "ie" at age & from a massi)e infarction. The patient reports that he eats 'ell% e.ercises reg#larly% an" has $een tol" $y his 'orkplace screening program that he has 0normal0 cholesterol le)els. >n *#estioning% he reports that he ha" pre)io#sly smoke" one pack per "ay for a$o#t 24 years $#t *#it 2 years ago. /o' many years m#st this patient ha)e stoppe" smoking $efore his to$acco #se no longer co#nts as a risk factora6 1 year $6 years c614 years "61 years e6 24 years :ormal La$s &1. A 1!year!ol" "ental hygienist presents complaining of s'elling in the right knee. >)er the past 24 ho#rs% her right knee s#""enly $ecame s'ollen an" painf#l to 'eight $earing% 'ith a limite" range of motion. She "oes not recall any history of arthritis or tra#ma in that region. She is not a'are of any nee"le!stick in9#ries 'hile at 'ork. Until the onset of knee pain% she ha" $een e.ercising "aily on a trea"mill. A physical e.amination re)eals ten"erness% s'elling% an" erythema in the right knee. There is painless fle.ion an" limitation in e.tension. =hich a""itional information 'o#l" $e most rele)ant in this patient,s historya65amily history of rhe#matoi" arthritis $6/istory of $acterial gastroenteritis c6/istory of hepatitis B )accination "6/istory of a tra#matic right ankle in9#ry e6Se.#al history :ormal La$s

&2.A 24!year!ol" gra"#ate st#"ent presents to the #ni)ersity health clinic $eca#se of complaints of intermittent nonpro"#cti)e co#gh an" 'heeDing. She #s#ally has these symptoms aro#n" the time of her menses an" has no other p#lmonary symptoms $et'een episo"es. The symptoms #s#ally last & or 4 "ays an" gra"#ally resol)e spontaneo#sly. She has no other past me"ical history an" smokes only at "inner parties% appro.imately once e)ery fe' months. She is on no me"ications% e.cept for i$#profen for se)ere premenstr#al cramping. >n physical e.amination% she appears comforta$le $#t is mil"ly tachypneic. >n l#ng e.amination% there are high!pitche" e.piratory 'heeDes in $oth l#ng fiel"s an" a mo"erately impaire" air mo)ementG the flo' is meas#re" at 84 L7min 3normal% I2 4 L7min6. A chest .!ray film is normal. =hich of the follo'ing mechanisms $est e.plains this patient,s $ronchospasma6@ecrease" le#kotriene pro"#ction $6+ncrease" cycloo.ygenase e.pression c6+ncrease" le#kotriene pro"#ction "6+ncrease" pro"#ction of prostaglan"ins e6 +ncrease" prostacyclin pro"#ction :ormal La$s &&.A <1!year!ol" 'oman is $ro#ght to the hospital $y am$#lance after $eing fo#n" #nconscio#s. /er Me"ic!Alert $racelet states that she has hypothyroi"ism. All that is kno'n of her history is that she 'as fo#n" $y her family on her kitchen floor an" 'as nonresponsi)e an" col" to the to#ch. +t is not clear 'hether she has $een taking her thyro.ine therapy. >n e.amination% her skin is thin% cool% an" moist% 'ith nonpitting "iff#se e"ema of her #pper an" lo'er e.tremities. /er $loo" press#re is (47non"etecta$le mm /g% p#lse is 447min% an" respirations are 17min 'ith an o.ygen sat#ration of 12B on room air. =hich of the follo'ing is the most appropriate ne.t step in her managementa6+? hypertonic saline $6+? pressor s#pport c6+? thyro.ine "6En"otracheal int#$ation an" mechanical )entilation e62ar"iac "efi$rillation :ormal La$s

&4.A 44!year!ol" 'oman is referre" for ne#rologic cons#ltation $eca#se of 'eakness an" spasticity of the #pper lim$s for the past three years. She has a long history of psychiatric pro$lems% characteriDe" $y "epression an" poorly "efine" somatic complaints s#ch as a rec#rrent sensation of electric shocks "o'n the spine. 5o#r years ago% the patient e.perience" transient loss of )is#al ac#ity% 'hich 'as interprete" at that time as a psychosomatic manifestation. >n e.amination% she appears mo"erately an.io#s% $#t 'ith no a$normalities in orientation% memory% or 9#"gment. :e#rologic e.amination re)eals spastic paraparesis an" mil" loss of propriocepti)e sensation in $oth arms. The ne#rologist arranges for a l#m$ar p#nct#re. This sho's increase" protein 'ith oligoclonal $an"s in the 2S5. =hich of the follo'ing is the most likely "iagnosisa6 Ac#te "isseminate" encephalomyelitis $6Lyme "isease c6M#ltiple sclerosis "6Paraneoplastic syn"rome e6 Spinocere$ellar syn"rome :ormal La$s & .An a"#lt "e)elops insi"io#s onset of a se)ere infectio#s "isease. The con"ition is characteriDe" initially $y high fe)er% hea"ache% pharyngitis% an" arthralgias. The patient then goes on to "e)elop intestinal complaints of constipation% anore.ia% an" a$"ominal pain an" ten"erness. @#ring the secon" 'eek of the illness% he has a rash 'ith "iscrete pink% $lanching lesions 3rose spots6 on the chest an" a$"omen. The rash resol)es a$o#t three "ays later. By the thir" 'eek of the "isease% the patient appears )ery ill an" has "e)elope" a flori" "iarrhea that is positi)e for occ#lt $loo". @#ring this same perio"% the man also "e)elops secon"ary pne#mococcal pne#monia. At the height of the patient,s illness% he 'as st#poro#s an" ha" short perio"s of "eliri#m. The spleen 'as palpa$le "#ring this perio". Bloo" st#"ies "emonstrate le#kopenia% anemia% li)er f#nction a$normalities% an" a mil" cons#mption coag#lopathy. =hich of the follo'ing is the most likely "iagnosisa6 Br#cellosis $62holera c6Melioi"osis "6Plag#e

e6Typhoi" fe)er :ormal La$s &<.A 24!year!ol" tire salesman complains of a lo'!gra"e fe)er an" a nonpro"#cti)e co#gh for 1 'eek. /is girlfrien" ha" similar symptoms 1 'eek earlier. /e "enies any shortness of $reath or ple#ritic chest pain. >n physical e.amination% his temperat#re is &(.8 2 3144.2 56. A l#ng e.amination re)eals coarse $ilateral inspiratory an" e.piratory crackles. /is la$oratory res#lts re)eal a 'hite $loo" cell co#nt of <&447mm&% a hemoglo$in of 14.2 g7"L% an" a hematocrit of &2B. A col" aggl#tinin assay of his re" $loo" cells is positi)e. =hich of the follo'ing is the most appropriate treatment for this patienta6Ampicillin $62ephale.in c6Erythromycin "6Tetracycline e6 Trimethoprim7s#lfametho.aDole :ormal La$s &(.A 4!year!ol" 'oman is seen $y her family physician $eca#se she is feeling poorly. The 'oman has a kno'n history of se)ere chronic gastritis. Physical e.amination "emonstrates pallor of skin an" m#cosal mem$ranesG no other positi)e fin"ings are note". A hematocrit of &&B is "emonstrate" in the physician,s office. Bloo" smear performe" an" re)ie'e" in the office sho's enlarge" erythrocytes that ha)e enlarge" central pale areas. :e#trophils an" other 'hite cells are present in normal n#m$ers. Some of the ne#trophils ha)e hypersegmente" n#clei. =hich of the follo'ing is the most likely "iagnosisa6Beta thalassemia trait $65olate "eficiency anemia c6+ron "eficiency anemia "6Sickle cell anemia e6 ?itamin B12 "eficiency anemia :ormal La$s &1. A <2!year!ol" man presents 'ith stat#s epileptic#s. /e has a history of small cell l#ng cancer. /e has no history of hypertension% "ia$etes%

thyroi" "isease or congesti)e heart fail#re. E.amination re)eals a male in a postictal state% 'ith a $loo" press#re of 1&4784 mm /g. A f#n"#scopic e.amination re)eals no papille"ema% his neck is s#pple. L#ngs are clear% his heart e.amination is normal 'itho#t 9#g#lar )eno#s "istention. There is no e"ema or cl#$$ing. La$oratory st#"ies sho'; Ser#m So"i#m 112 mE*7L Potassi#m 4.4 mE*7L Urea nitrogen 14 mg7"L 2reatinine 4.1 mg7"L Urine >smolality <12 m>smol7kg Urine so"i#m 1 mE*7L A hea" 2T is normal. A chest .!ray film re)eals a right perihilar mass #nchange" from a film one month ago. =hich of the follo'ing is the most appropriate initial step in managementa6@emeclocycline $65l#i" restriction c6/y"rocortisone "6/ypertonic saline e6Thyro.ine :ormal La$s &8.A pre)io#sly healthy !year!ol" man is referre" for ne#rologic cons#ltation $eca#se of persistent hea"ache an" change in personality o)er the past 2 months. /e has smoke" one pack of cigarettes "aily for &4 years. :e#rologic e.amination re)eals "ecrease" sensation in the right #pper e.tremity. :e#roimaging st#"ies sho' a !cm% ill!"efine" mass in the left parietal 'hite matter. A ring!like Done of contrast enhancement is seen on T1!'eighte" MA+ images% s#rro#n"e" $y e.tensi)e e"ema appreciate" on T2!'eighte" images. La$oratory in)estigations are 'ithin normal limits% incl#"ing negati)e anti$o"ies to /+?. =hich of the follo'ing is the most likely "iagnosisa6A$scess $6@emyelinating "isease c6Clio$lastoma m#ltiforme "6+nfarct e6Metastasis

:ormal La$s 44.An el"erly patient complains to a physician of sores in the mo#th an" on the skin. These lesions ha)e "e)elope" o)er a$o#t a month. Physical e.amination "emonstrates m#ltiple painf#l erosions on the oral m#cosa an" tong#e. Aa' areas 'ith cr#sting on the skin is seen on the face an" tr#nk. The patient states that the skin lesions ha" starte" as $listers that ha" *#ickly $roken. =hile the in)ol)e" areas 'ere painf#l% no itchiness ha" $een e.perience". 2aref#l e.amination of the e"ge of the skin lesions "emonstrates a fe' flacci" $#llae. A#$$ing of the skin near an affecte" area easily "etaches the s#perficial part of the epi"ermis from the #n"erlying skin 3:ikolsky,s sign6. :o target!like lesions are seen. =hich of the follo'ing is the most likely "iagnosisa6 B#llo#s pemphigoi" $6@ermatitis herpetiformis c6Pemphig#s )#lgaris "6Ste)ens!Rohnson syn"rome e6To.ic epi"ermal necrolysis :ormal La$s 41.A (!year!ol" 'oman presents to the emergency "epartment complaining of a painf#l left foot. The patient "escri$es the ac#te onset of foot tingling an" n#m$ness & "ays ago. Since that time% the symptoms ha)e progresse" to constant pain an" ina$ility to $ear 'eight on that foot. The patient,s other me"ical history is significant for atrial fi$rillation secon"ary to amyloi"osis. She has $een in atrial fi$rillation intermittently for the past & years an" has ha" attempte" chemical an" electrical car"io)ersion% $#t 'itho#t long!term s#ccess. She c#rrently takes no me"ications. >n e.amination% she has an irreg#larly irreg#lar p#lse at 847min. An E2C sho's atrial fi$rillation. =hich of the follo'ing a""itional fin"ings 'o#l" most likely s#pport the s#specte" "iagnosisa6A$sent "orsalis pe"is p#lse on the left foot $6Lactate "ehy"rogenase le)el of 44 U7L c6Loss of light to#ch sensation in her affecte" lim$ "6Prothrom$in time of 14 secon"s e6Aeg#lar heart rhythm

:ormal La$s 42. A 44!year!ol" 'oman has an 1!month history of "iff#se 9oint pain an" s'elling that in)ol)es $oth han"s an" knees. She states that she cannot get any 'ork "one in the morning "#e to the pain% $#t it #s#ally s#$si"es as the "ay progresses. She tires easily an" constantly feels 0fe)erish0. /er temperat#re is &(.1 2 3144 56% $loo" press#re is 1147(4 mm /g% p#lse is <47min% an" respirations are 117min. Physical e.amination sho's ten"er% s'ollen% an" 0$oggy0 han"s an" knees. La$oratory st#"ies sho'; /ematocrit............&4B /emoglo$in...........14 g7"LA ra"iograph of this patient,s knee is most likely to sho' 'hich of the follo'inga6Bone erosions $6>steophyte formation c6S#$chon"ral cyst formation "6S#$chon"ral sclerosis e6 S#$chon"ral tophi :ormal La$s 4&.A &&!year!ol" man 'ith a history of +? "r#g #se presents to the emergency "epartment complaining of fe)ers% chills% an" a pro"#cti)e co#gh. /e has a prior history of cell#litis an" tric#spi" )al)e en"ocar"itis. A recent /+? test at his metha"one clinic 'as negati)e. >n physical e.amination% his temperat#re is &8.1 2 3142.& 56% $loo" press#re is 1127<2 mm /g% p#lse is 817min% an" respirations are 217min. L#ng e.amination re)eals rhonchi in the right mi" posterior l#ng fiel". There is a soft% holosystolic m#rm#r hear" at the left lo'er sternal $or"er that increases 'ith inspiration. A chest .!ray film sho's a lo$ar consoli"ation in the right mi""le lo$e 'ith an air fl#i" le)el. =hich of the follo'ing is the most likely ca#se of his pne#moniaa6Escherichia coli $6/aemophil#s infl#enDae c6Staphylococc#s a#re#s "6Staphylococc#s epi"ermis e6 Streptococc#s pne#monia :ormal La$s

44. A <4!year!ol" 'oman complains of a 1!year history of $one pains% anore.ia% 'eight loss% an" constipation. Ser#m calci#m is ele)ate" an" she is "iagnose" 'ith primary hyperparathyroi"ism. She is treate" s#rgically% 'ith remo)al of her parathyroi" a"enoma. She then has a long perio" of hypocalcemia an" re*#ires contin#o#s treatment 'ith )itamin @ an" calci#m. S#$se*#ently% after < months she $ecomes normocalcemic an" "oes not nee" therapy. =hich of the follo'ing con"itions $est e.plains these e)entsa6/er remaining & parathyroi" glan"s 'ere "estroye" $6She has #nrecogniDe" pse#"ohypoparathyroi"ism c6She ha" se)ere $one "isease "6She has pancreatitis from hyperparathyroi"ism e6 The 'rong parathyroi" glan" 'as remo)e" :ormal La$s 4 .A 1!year!ol" man 'ith chronic li)er "isease presents 'ith recent onset of ascites an" rapi" 'eight loss. E.amination re)eals a palpa$le ten"er li)er mass. Paracentesis re)eals hemorrhagic ascitic fl#i". 2T scan of the li)er sho's a large intrahepatic m#ltifocal t#mor. Perc#taneo#s fine nee"le aspiration is positi)e for hepatocell#lar carcinoma. The patient "eteriorates rapi"ly an" "ies & months follo'ing presentation. =hich of the follo'ing is the most important pre"isposing con"ition to this cancer in the U.S.a6Aflato.in e.pos#re $6Ana$olic steroi"s c62irrhosis "6/emochromatosis e6 >ral contracepti)es f6Tyrosinemia :ormal La$s 4<. A <(!year!ol" man is a"mitte" to the hospital for right foot pain. The patient reports that% o)er the past 2 "ays% his right foot has $ecome progressi)ely more painf#l an"% a fe' ho#rs ago% $egan to t#rn gray. The patient reports significant pain an" some tingling in his foot% $#t no other symptoms. /is past me"ical history is nota$le for hypertension% hypercholesterolemia% an" peripheral )asc#lar "isease. The patient ha"

#n"ergone $ilateral caroti" en"arterectomies in the past 2 years. Physical e.amination re)eals a col"% $l#e!gray right foot 'ith no palpa$le "orsalis pe"is or posterior ti$ialis p#lse. =hich of the follo'ing is the most appropriate "iagnostic test at this timea62T scan of the a$"omen $62T scan of the hea" an" neck c6Echocar"iogram "6Lateral c!spine ra"iograph e6 Lo'er e.tremity angiography :ormal La$s 4(.A 41!year!ol" 'oman presents complaining of fatig#e an" malaise. She has $een pre)io#sly healthy% e.cept for a motor )ehicle acci"ent that re*#ire" an emergency splenectomy 22 years earlier. @#ring that operation% she re*#ire" the transf#sion of m#ltiple #nits of re" cells. +n her c#rrent e)al#ation% hepatitis serologies are checke" an" re)eal the follo'ing; hepatitis A +gC anti$o"y positi)e% hepatitis B s#rface anti$o"y positi)e% an" hepatitis 2 anti$o"y positi)e. A li)er $iopsy is consistent 'ith chronic hepatitis 'ith mo"erate fi$rosis. =hich of the follo'ing is the most appropriate treatmenta6 Alpha interferon $6Alpha interferon an" ri$a)irin c6ADathioprine "6Pre"nisone e6 Ai$a)irin :ormal La$s 41. A 4&!year!ol" man cons#lts a physician $eca#se of progressi)ely se)ere fatig#e "e)eloping o)er a perio" of a$o#t & months. >n physical e.amination% his skin an" m#co#s mem$ranes sho' a 'a.y pallor. +ntraoffice hematocrit is 11B. +ntraoffice peripheral $loo" smear sho's cells of normal morphology in all cell lines% $#t $an" form ne#trophils an" retic#locytes are nearly a$sent. >n specific *#estioning a$o#t possi$le to.ic e.pos#res% the man reports #sing $enDene fre*#ently to clean his han"s after 'orking on the ol" ho#se he recently $o#ght an" is remo"eling. Marro' $iopsy 'o#l" most likely sho' 'hich of the follo'inga6A$#n"ant megakaryocytes

$6/ypersegmente" ne#trophils c6Many ringe" si"ero$lasts "6 Marro' fi$rosis e6Minimal hematologic elements :ormal La$s 48. A <!year!ol" "ia$etic 'oman presents 'ith right flank pain an" high spiking fe)er. She also notes "ys#ria an" occasional hemat#ria. She has rigors% s'eats% an" na#sea. She has no history of #n"erlying renal "isease $#t "oes ha)e a history of hypertension. /er temperat#re is &8.4 2 314& 56% an" her $loo" press#re is 847 4 mm /g.The patient appears fl#she"% an" has e.*#isite right costo)erte$ral angle ten"erness./er li)er is not palpa$le. There is no e"ema or cyanosis. La$oratory analysis sho's; Bloo" /emoglo$in.............12.2 g7"L Le#kocyte co#nt......22%4447mm& Ser#m Urea nitrogen..........11 mg7"L 2reatinine...............1.& mg7"L So"i#m...................1&2 mE*7L Potassi#m...............&.1 mE*7L Urine; 124 'hite $loo" cells7hpf 144 re" $loo" cells7hpf Many $acteria seen 2!& 'hite $loo" cell casts A 2T of a$"omen sho's air in the right renal parenchyma. =hich of the follo'ing is the most likely "iagnosisa6Anaero$ic #rosepsis $6Emphysemato#s pyelonephritis c6>$str#cti)e #ropathy "6Pyelonephritis e6 T#$erc#losis of the ki"ney :ormal La$s

4. A &4!year!ol" man comes to the physician $eca#se of progressi)e m#scle 'eakness. /e 'as 'ell #ntil & "ays ago% 'hen he $egan to notice increasing 'eakness in his lo'er legs an" "iffic#lty clim$ing stairs. /e has $een al'ays in goo" health. T'o 'eeks prior to these symptoms% he ha" an #pper respiratory infection. Physical e.amination sho's symmetric 'eakness% 'hich is se)ere in the lo'er e.tremities an" mil" in the arms. Temperat#re is &(. 2 388. 56% $loo" press#re is 1&27(( mm /g% p#lse is 147min% an" respirations are 217min. Sensation is intact% $#t "eep ten"on refle.es are a$sent in the lo'er e.tremities an" "ecrease" in the #pper e.tremities. L#m$ar p#nct#re re)eals the follo'ing 2S5 )al#es; >pening press#re........:ormal 2ells.......................... 4 cells7mm& 3mostly lymphocytes6 Protein.......................14 mg7"L Cl#cose...................... 4 mg7"L =hich of the follo'ing is the most likely "iagnosisa6 Bot#lism $6C#illain!BarrT c6Myasthenia gra)is "6Poliomyelitis e6Polymyositis :ormal La$s
Note: Check your own answers before hittin$ the Check button below. When you click the Check button, a browser win"ow will a!!ear that contains a summary of your results. (*!lanations lock ? (*!lanations

:) (*!lanation: 7he correct answer is %. )heehan syn"rome is a rare cause of !anhy!o!ituitarism, cause" by necrosis of the !ituitary $lan" occurrin$ "urin$ "eliveries com!licate" by severe hy!otension. )i$ns an" sym!toms of !anhy!o!ituitarism may become a!!arent many months or even years after !arturition. 7he "ia$nosis is establishe" by measurin$ !lasma levels of $rowth hormone an" cortisol after an insulin challen$e, as well as by evaluatin$ thyroi" hormones an" 7)2 levels. It is treate" with a"ministration of hy"rocortisone an" thyroi" hormones.

&ost!artum "e!ression 'choice A) "evelo!s within C months !ost!artum. Fsually, sym!toms of "e!ression are mil %. In :>= of these cases, however, "e!ression may be so severe as to require hos!itali#ation an" intensive anti"e!ressant treatment. Althou$h "e!ression may mimic sym!toms of several en"ocrine illnesses an" vice versa, the si$ns "etecte" on !hysical e*amination, alon$ with the lon$ interval followin$ "elivery in this case, are more consistent with hy!o!ituitarism. &rimary a"renal insufficiency 'choice ) woul" e*!lain only !art of the syn"rome in this case, such as fati$ue an" hy!otension, but not amenorrhea, col" intolerance, an" skin chan$es. In a""ition, !rimary hy!ocortisolism 'i.e, "ue to "estruction of a"renal $lan"s) woul" be associate" with skin hy!er!i$mentation. &rimary hy!othyroi"ism 'choice C) may !ro"uce a similar clinical !icture, but the history of "ifficult !arturition with abun"ant bloo" loss an" the wi"e4ran$in$ sym!tomatolo$y su$$estin$ multi!le en"ocrine "eficits make !anhy!o!ituitarism more likely.

;) (*!lanation: 7he correct answer is (. Overall, atrial fibrillation confers about a := annual risk of a thromboembolic event, which, amon$ other thin$s, may cause stroke, intestinal ischemia, renal infarcts, or a threatene" limb. )!ecific etiolo$ies of A- such as valvular "isease are associate" with hi$her rates of embolic events. A C1A 'choice A) is the most "rea"e" com!lication of A-, but there is no in"ication of its !resence here, as a thromboembolic C1A woul" not cause a !ainful or col" le$, but instea" woul" !resent as loss of motor, sensation, or co$nitive function. %17 'choice ) is a common !roblem in $eneral me"icine that causes !ain an" swellin$ in the affecte" limb. %es!ite her A-, this !atient woul" be at no $reater risk than the $eneral !o!ulation to acquire a %17. %17s usually "o not !resent !reci!itously. 7hey also are associate" with inflammation of the vessels, conferrin$ warmth, not col", to the affecte" le$. +) 'choice C), a "emyelinatin$ inflammatory "isor"er of the central nervous system, usually affects !eo!le for the first time between the a$es of ;> an" ?> with remittin$ e!iso"es of unilateral loss of vision, ata*ia, verti$o, !aralysis, !aresthesias, or "i!lo!ia. 7he e!iso"es are ty!ically !ainless. 6aynau" syn"rome 'choice %) is cause" by vasos!asm of the "i$ital vessels !reci!itate" by col" an" relieve" by heat. It woul" not lea" to ischemia of an entire limb.

0) (*!lanation: 7he correct answer is %. 7his !atient is havin$ a very severe asthmatic attack, as "emonstrate" by his severe shortness of breath, use of accessory muscles of res!iration, an" variation in bloo" !ressure with breathin$ '"ue to establishin$ a very ne$ative intrathoracic !ressure with force" attem!ts at ins!iration). 7he initial !O; is marke"ly "ecrease" at C> 'com!are" to the normal :>>) mm 2$. 2is !CO; is also "ecrease" at ;: mm 2$ initially, "ue to Hblowin$ off CO;H with his increase" res!irations while tryin$ to $et enou$h o*y$en. 7his causes his res!iratory alkalosis with !2 D.//. 2e is then treate", but the treatment im!roves his !O; only sli$htly. %es!ite this, his !CO; rises, an" his !2 returns towar" normal. 7his "oes not mean he is $ettin$ better. Instea", we are seein$ worsenin$ res!iratory failure as his ability to ventilate "ecreases, causin$ the ina!!ro!riate rise in !CO; even thou$h he is still very ba"ly hy!o*i C. 7his !atient shoul" therefore be !re!are" for !ossible intubation if there is not imme"iate im!rovement. (!ine!hrine 'choices A an" C) no lon$er is use" in the mana$ement !rotocols of acute asthma because of its hi$h rate of to*icity. A"ministerin$ :>>= o*y$en 'choice ) may be an a"<unctive thera!y, but at this !oint the !atient is !robably saturatin$ above I>= anyway. 7here is no in"ication of a !neumothora* in this !atient, so there is no nee" for a chest tube 'choice (). ?) (*!lanation: 7he correct answer is C. It is $enerally acce!te" that the bloo" su!!ly from :ID@ to :I@/ was likely to be tainte" with 2I1 !ositive bloo". -or this reason, as well as the in"olent nature of 2I1 infection, !atients with a history of bloo" transfusions "urin$ these years, even if currently asym!tomatic, shoul" be screene %. 7estin$ for he!atitis 'choice A) or he!atitis C 'choice ) woul" not be incorrect, but there are no formal $ui"elines recommen"in$ this. 7he !rimary reason is that such "iseases $enerally manifest in some way after a ;>4year !erio", an" the !atient woul" not be asym!tomati C. &&% skin testin$ 'choice %) is reserve" for health care workers, alcoholics, I1 "ru$ abusers, "iabetics, an" en"4sta$e renal !atients. 7his "oes not chan$e with a history of bloo" transfusion.

6&6, or ra!i" !lasma rea$in testin$ 'choice (), is use" to test for sy!hilis. It is recommen"e" if the !atient is !re$nant, has a history of se*ual transmitte" "iseases, or is a !rostitute. /) (*!lanation: 7he correct answer is A. I1 calcium will work very ra!i"ly to counteract the effect of the hi$h !otassium on the heart an" muscle an" shoul" be $iven first. 7he calcium will not actually lower the level of !otassium, however. All of the other a$ents are $oo" at lowerin$ !otassium, but "o not have the same imme"iate effect to s!are the muscle that calcium has. 7herefore, these a$ents shoul" be $iven after I1 calcium when there are severe (C. chan$es. C) (*!lanation: 7he correct answer is . 7his is the classic !resentation of cluster hea"ache, which is an infrequent con"ition 'inci"ence a!!ro*imately C>4D>B:>>,>>> a"ults) that most frequently affects mi""le4a$e" men. Nocturnal attacks are characteristic. 7hey awaken the !atient or occur imme"iately after awakenin$, consistently at the same time of the ni$ht. (ach attack lasts between 0> minutes an" ;40 hours. %rinkin$ alcohol may tri$$er the e!iso" (. 7he "esi$nation refers to the fact that the "aily attacks occur in clusters lastin$ for several weeks 'usually no more than @ weeks), an" then cease for a lon$ !erio" ': year on avera$e). 6ecent observations su$$est that the un"erlyin$ abnormality involves hy!othalamic nuclei that re$ulate circa"ian rhythm, but the !ain an" accom!anyin$ vasomotor sym!toms are !robably me"iate" by serotoniner$ic !athways. O*y$en a"ministration, sumatri!tan, an" er$otamine me"ications are useful in treatin$ acute attacks. Acute $laucoma 'choice A) manifests with e*tremely !ainful, re" eyes an" blurre" vision. %ia$nostic confirmation is obtaine" by tonometric measurement of intraocular !ressure, which is marke"ly elevate %. .iant cell arteritis 'choice C) is a form of arteritis that most commonly involves the tem!oral artery an" affects el"erly !atients. )cal! ten"erness over the affecte" su!erficial tem!oral artery an" systemic si$ns an" sym!toms 'e.$., fever, elevate" erythrocyte se"imentation rate, malaise) are ty!ical. 7his serious con"ition nee"s ur$ent corticosteroi" treatment because of the risk of blin"ness "ue to o!hthalmic artery involvement. .losso!haryn$eal neural$ia 'choice %) an" tri$eminal neural$ia 'choice -) are syn"romes characteri#e" by !aro*ysmal shootin$ !ain alon$ the "istribution of

$losso!haryn$eal or, res!ectively, tri$eminal branches. 7ri$eminal neural$ia 'tic "ouloureu*) is more frequent. In this case, lancinatin$ !ain arises near the mouth an" ra"iates towar" the i!silateral eye or nostril. ,i$ht touch or talkin$ may tri$$er the event. .losso!haryn$eal neural$ia 'rare) manifests with shootin$ !ain in the throat or "ee! ear, which is usually tri$$ere" by swallowin$, chewin$, or talkin$. +i$raine 'choice () be$ins usually in early a"ulthoo" an" manifests as e!iso"ic unilateral throbbin$ hea"ache, often associate" with nausea, !hoto!hobia, an" visual sym!toms. +i$raine "oes not have the characteristic rhythmic recurrence of cluster hea"ache, but it may be tri$$ere" by relatively constant stimuli, such as me"ications, noise, an" stress. D) (*!lanation: 7he correct answer is %. All of the skin lesions liste" are common minor skin !roblems that you will nee" to "istin$uish clinically. 7his woman has a !yo$enic $ranulom A. 7his lesion most commonly "evelo!s at a site of !revious in<ury, an" is best consi"ere" a variation of $ranulation tissue formation. 7he name is a misnomer, since the lesion is neither relate" to bacterial infection nor a true $ranulom A. +icrosco!ically, !roliferatin$ bloo" vessels in an immature fibrous stroma are seen. 7he lesions can occasionally be confuse" with "ys!lastic nevi or mali$nant melanoma, but are com!letely beni$n. 7reatment is with sur$ical e*cision, curetta$e, or electro"esiccation. A "ermatofibroma 'choice A) is a re" to brown, firm, small !a!ule or no"ule, often foun" on the le$s. A keloi" 'choice ) is a shiny, har", smooth, often "ome4sha!e" over$rowth of a scar. A keratoacanthoma 'choice C) is a roun", firm, flesh4colore" no"ule with a central crater. A wart 'choice () is a rou$h4surface", shar!ly "emarcate", roun", firm, no"ule whose color may be $ray, yellow, brown, or $ray4black. @) (*!lanation: 7he correct answer is . 7he combination of substernal chest !ressure with (C. fin"in$s of hy!eracute 7 waves an" )7 elevation in this clinical settin$ su$$ests early anterior myocar"ial infarction. 7he myocar"ial "ama$e causes time4"e!en"ent effects on the electrical !ro!erties of the myocar"ial cells. 7he earliest electrical evi"ence of myocar"ial in<ury is often "evelo!ment of hy!eracute 7 waves, followe" by elevation of the )7 se$ments, inversion of the 7 wave, return of the )7 se$ments to normal, an", finally, the

"evelo!ment of 8 waves. It "oes not matter that the C5, a marker of myocar"ial in<ury, is not elevate", as it takes several hours for the C5 to a!!reciably rise after myocar"ial "ama$e has occurre". In this case, his !rior cocaine use may have contribute" to !remature coronary artery "isease. Aortic "issection 'choice A) is often a misse" "ia$nosis an" therefore always im!ortant to think about, even in relatively youn$ !atients. 7he fact that the !atient is e*!eriencin$ chest !ressure, in "istinction to !ain, an" has (C. chan$es reflectin$ myocar"ial in<ury makes an +I more likely. 2avin$ sai" that, aortic "issection can cause +I by "issectin$ the coronary arteries 'usually the ri$ht coronary artery). 7he !atient shoul" have his bloo" !ressure checke" in both arms lookin$ for a "iscre!ancy an" $et a chest *4ray lookin$ for wi"ene" me"iastinum, two classical fin"in$s of acute aortic "issection. &( 'choice C) shoul" always be a consi"eration of !atients with chest !ain, as it too is often a misse" "ia$nosis. Fsually, however, there will be some com!laint of "ys!nea with &( an" the (C. fin"in$s are of ri$ht heart strain, not anterior myocar"ial in<ury. Classic (5. fin"in$s of &( inclu"e new ri$ht bun"le branch block, sinus tachycar"ia an" ri$ht a*is "eviation. &atients with s!ontaneous !neumothora* 'choice %), similar to those with &(, will often have !leuritic chest !ain, an" !neumothora* is ty!ically not associate" with hy!eracute 74waves. %ifferentiatin$ unstable an$ina from +I may be "ifficult at times, an" sometimes the "istinction cannot be ma"e until the car"iac en#ymes are measure" over a !erio" of ;? hours. 2owever, acute chest !ressure with )7 elevations su$$ests an acute coronary syn"rome that is "istinct from the chronic an$ina 'choice () that some car"iac !atients e*!erience with e*ertion. )7 elevation is a si$n of acute, active myocar"ial in<ury an" necrosis. I) (*!lanation: 7he correct answer is C. A $oo" way to "ia$nose $out is to $ive oral colchicine hourly until the !atient "evelo!s im!rovement in <oint !ain an" inflammation. 7he most s!ecific metho" of "ia$nosis, however, is <oint as!iration an" i"entification of ne$atively birefrin$ent nee"le sha!e" crystals. A ri$ht ankle effusion 'choice A) may be relate" to $out, or coul" be "ue to another rheumatolo$ic !roblem. (lbow no"ules 'choice ) are a non4s!ecific "ia$nostic feature. Fric aci" levels 'choice %) may be elevate" in $out, as well as "isor"ers such as tumor lysis syn"rom

(. 7he results will be too non4s!ecific. 7he toe <oint erosion 'choice () may be relate" to trauma, "e$enerative <oint "isease or arthritis. :>) (*!lanation: 7he correct answer is C. It woul" be most a!!ro!riate to a"" an I1 steroi"al a$ent, such as hy"rocortisone, to au$ment the action of broncho"ilators by re"ucin$ inflammation surroun"in$ the airways. eclomethasone 'choice A), a surface4actin$ steroi" "is!ense" in aerosoli#e" form, is use" when the si"e effects of systemic steroi"s nee" to be avoi"e". )uch a$ents, however, also have minimal utility in the acute treatment of bronchos!asm. %iso"ium cromo$lycate 'choice ) is a mast cell stabili#er that is use" only for the !revention of bronchos!asm "ue to asthma. Once bronchos!asm is establishe", this ty!e of a$ent has little utility. An I1 steroi" is !referre" over an oral steroi", such as !re"nisone 'choice %), since this !atient has a history of recent nausea an" vomitin$ an", therefore, may have im!aire" absor!tion of oral me"ications. In a""ition, the time course of onset of an oral a$ent is too lon$ to have any effect in an acute situation. 7heo!hylline 'choice (), a methyl*anthine !hos!ho"iesterase inhibitor, is useful for the chronic control of asthma, but it has lesser utility in the acute treatment of bronchos!asm. ::) (*!lanation: 7he correct answer is %. 2y!ercalcemia can be treate" with I1 saline an" furosemi"e. -lui" re!lacement with I1 saline an" force" "iuresis with saline an" a loo! "iuretic, such as furosemi"e, is a ra!i" an" safe way to lower serum calcium an" shoul" be the initial a!!roach to thera!y. 7his !atient has lun$ cancer an" is !robably e*hibitin$ a !araneo!lastic secretion of !arathyroi"4relate" hormone, which is makin$ him hy!ercalcemic. Calcitonin 'choice A), a hormone secrete" by the !arafollicular cells of the thyroi", inhibits osteoclast activity an" "ecreases the rate of bone loss an" fractures in osteo!orosis. It may be a""e" after I1 hy"ration an" "iuresis with furosemi"e. (ti"ronate 'choice ) is a bis!hos!honate that !revents bone resor!tion by inhibitin$ osteolytic activity. It is often use" in !reventive thera!y for osteo!orosis. 2owever, it woul" be of limite" value in this acute settin$.

2y"rochlorothia#i"e 'choice C) is a thia#i"e "iuretic that "ecreases urinary calcium e*cretion an" can cause hy!ercalcemi A. &re"nisone 'choice () is a $lucocorticoi" that is effective in treatin$ hy!ercalcemia cause" by vitamin % e*cess, sarcoi"osis, an" some hematolo$ic mali$nancies. 2owever, it "oes not lower calcium in most cases of hy!ercalcemia associate" with soli" tumors. :;) (*!lanation: 7he correct answer is . A hemo$lobin of :D $B", woul" be e*!ecte". 7he question "escribes a !atient with chronic obstructive !ulmonary "isease 'CO&%) confirme" by his !hysical e*amination, which also su$$ests ri$ht4si"e" heart failure, lou" &;, an" he!atic con$estion an" !eri!heral e"ema "ue to ri$ht4si"e" heart failure. 7he !ulmonary function tests are consistent with the "ia$nosis of obstructive airway "iseas (. .iven the lon$ history of smokin$, this !atient most likely has CO&%. ecause of chronic hy!o*ia, these !atients also "evelo! erythrocytosis in an effort to increase the o*y$en carryin$ ca!acity. Choice A woul" be seen in anemi A. 7here is no reason for this !atient to "evelo! thrombocyto!enia 'choices C an" %) or leuko!enia 'choice ().

:0) (*!lanation: 7he correct answer is %. 7his !atient has bron#e" skin an" a family history of liver "isease, an" "oes not use alcohol or "ru$sLthis combination shoul" su$$est hemochromatosis. A serum ferritin woul" be the a!!ro!riate screenin$ test to evaluate for iron overloa %. 7echnically, hemochromatosis is "ia$nose" when the total bo"y iron loa" is estimate" to be $reater than / $G hemosi"erosis is the term use" for mil"er iron overloa" con"itions. 2emochromatosis can be either !rimary '$enetic) or secon"ary to a wi"e variety of con"itions, inclu"in$ bloo" "isor"ers 'hemolytic anemias, thalassemias), !arenteral iron overloa" 'many transfusions), an" increase" iron in$estion. If the term Hsecon"aryH is "elete", the $enetic form of hemochromatosis is usually assume %. %es!ite massive amounts of research 'which have even i"entifie" mutations in an 2,A4A like molecule), the un"erlyin$ !atho$enesis is still unclear. 7he !roblem "oes

a!!ear to be !rimarily an overabsor!tion of iron throu$h the $ut, which cannot be correcte" because the bo"yAs mechanisms for e*cretin$ iron are limite" ' (.$., bloo" loss or its equivalent of menstruation). &atients usually "o not become sym!tomatic until mi""le a$e, when si$nificant tissue in<ury has occurre %. 7he skin "iscoloration in non sun4e*!ose" areas is a very hel!ful clinical clu (. Im!ortant com!lications inclu"e he!atic cirrhosis 'which may itself be com!licate" by he!atocellular carcinoma), "iabetes mellitus secon"ary to !ancreatic "ama$e, car"iome$aly or arrhythmias, !ituitary failure, an" arthritis. &hlebotomy is the !referre" metho" of iron removal in !atients with establishe" $enetic hemochromatosis. 7he chelatin$ "ru$ "efero*amine has sometimes been use" to increase urinary iron secretion in asym!tomatic relatives with the un"erlyin$ $enetic !roblem. Al!ha: antitry!sin "eficiency 'choices A an" ) also !resents with familial liver "isease, often associate" with em!hysema in nonsmokers. 2owever, it "oes not "emonstrate the ty!ical skin !i$mentation seen in hemochromatosis. (levate" serum co!!er levels 'choice C) are associate" with Wilson "isease, which, like hemochromatosis, has a stron$ family history but $enerally !resents at a youn$er a$e an" without the skin hy!er!i$mentation. Nickel 'choice (), like co!!er, is use" in coins, but "oes not cause liver "iseas (. :?) (*!lanation: 7he correct answer is A. 7his !atient with alcoholic cirrhosis an" !ortal hy!ertension has si$ns an" sym!toms consistent with he!atic ence!halo!athy. 7he one !ossible e*!lanation in the absence of any obvious infection or $astrointestinal blee"in$ is "ehy"ration relate" to his "iuretic us (. Althou$h the "iuretics may have been a!!ro!riately !rescribe" for ascites, at !resent he has multi!le !hysical fin"in$s su$$estin$ that he is "ehy"rate": "ry mucous membranes an" absence of <u$ulovenous "istention, ascites, an" !eri!heral e"em A. 2is !ulse "oes not "emonstrate a refle* tachycar"ia because of the !ro!ranolol, which is bein$ use" to re"uce his !ortal !ressur (. In the settin$ of "ehy"ration, the !atientAs FN woul" be e*!ecte" to be elevate" an" is a frequent e*acerbatin$ factor for he!atic ence!halo!athy. 7here is no history of "iabetes in this !atient, so a $lucose of :I; m$B", woul" not be e*!ecte" 'choice ), an" this "e$ree of relatively mil" hy!er$lycemia woul" not e*!lain his mental status. Althou$h the !atient is on s!ironolactone, he is also on furosemi"e, so it woul" not be likely that he woul" be hy!erkalemic 'choice C).

7here is nothin$ to su$$est that the !atient has a metabolic aci"osis so it is unlikely that he woul" have a low serum bicarbonate 'choice %). 7here is nothin$ in the history to su$$est the !ossibility of "evelo!in$ hy!ercalcemi A. -urthermore, the !atientAs furosemi"e woul" ten" to "ecrease, not increase, serum calcium 'choice (). :/) (*!lanation: 7he correct answer is A. 7he anemia of chronic "isease is seen in !atients with un"erlyin$ severe chronic "isor"ers such as infections, inflammatory "iseases 'such as this !atientAs rheumatoi" arthritis), an" cancers. 7he anemia is usually initially normocytic but may, with time, become microcyti C. -actors contributin$ to the anemia can inclu"e shortenin$ of the erythrocyte life s!an, ina"equate erythro!oietin !ro"uction or im!aire" res!onse to erythro!oietin, an" alterations in iron metabolism. If iron "eficiency has not yet also "evelo!e", serum ferritin levels are usually hi$h in anemia of chronic "iseas (. 7his can be hel!ful in "istin$uishin$ the microcytic form of anemia of chronic "isease from iron "eficiency anemi A. +acrocytes 'choice ) su$$est vitamin :; or folate "eficiency. )!herocytes 'choice C) su$$est here"itary s!herocytosis. 7ar$et cells 'choice %) su$$est thalassemi A. 7ear "ro! forms 'choice () su$$est myelo!hthisic anemi A. :C) (*!lanation: 7he correct answer is A. 7he com!osition of the stone is the most im!ortant com!onent in "esi$nin$ thera!y to !revent future events of ne!hrolithiasis. Calcium stones account for a!!ro*imately @>= of all stones. 2y!ercalciuria is usually !resent an" may be i"io!athic or secon"ary to some other caus (. Calcium stones may also !reci!itate aroun" a uric aci" ni"us, even in !atients in whom hy!ercalciuria is not !resent. If "ietary measures are not effective in "ecreasin$ calciuria, then a thia#i"e "iuretic may be utili#e" to increase calcium reabsor!tion by the ki"ney. Initiation of loo! "iuretic thera!y 'choice ) is contrain"icate", as loo! "iuretics !romote hy!ercalciuri A.

Initiation of allo!urinol thera!y 'choice C) is in"icate" in the treatment of uric aci" stones or when hy!eruricosuria is !resent with calcium stones. +aintenance of an alkaline urine 'choice %) is only useful if the !atient also has hy!eruricosuri A. +aintenance of lar$e urine volumes via co!ious water consum!tion 'choice () is recommen"e" to all acute stone !atients but has limite" efficacy in !reventin$ recurrence of "iseas (. :D) (*!lanation: 7he correct answer is . 7his !atient has cluster hea"aches, which are "aily, unilateral, severe hea"aches that occur over a !erio" of :4; months. 7hey are associate" with nasal stuffiness, !tosis, an" lacrimation. +i""le4a$e" males are most commonly affecte %. 7reatment "urin$ the acute attack is o*y$en. &ro!hyla*is inclu"es !re"nisone, lithium, er$otamine, an" vera!amil. Classic mi$raine 'choice A) is a severe, throbbin$ hea"ache that is associate" with nausea, vomitin$, an" !hoto!hobi A. It is !rece"e" by an aura 'focal neurolo$ical "isturbances). 7reatment is with sumatri!tan an" er$otamin (. Common mi$raine 'choice C) is similar to classic mi$raine e*ce!t that it is not !rece"e" by an aur A. A subarachnoi" hemorrha$e 'choice %) is often cause" by a ru!ture" saccular aneurysm, an" is "escribe" as the Hworst hea"ache of my lifeH. It may be associate" with a loss of consciousness an" vomitin$. A C7 scan followe" by a lumbar !uncture confirms the "ia$nosis. &rom!t microsur$ical cli!!in$ of the aneurysm is the treatment. 7em!oral arteritis 'choice () !ro"uces a unilateral, throbbin$ hea"ache with tem!oral artery ten"erness. Associate" fin"in$s inclu"e anemia an" an increase" erythrocyte se"imentation rat (. %ia$nosis is ma"e by tem!oral artery bio!sy. 7reatment is with corticosteroi"s, which shoul" be $iven imme"iately to !revent blin"ness. 7ension hea"ache 'choice -) is a bilateral occi!ital hea"ache that is "escribe" as Hban"4 likeH or Hvise4likeH. 7hera!y consists of anal$esics.

:@) (*!lanation: 7he correct answer is . 7he !atient has a <unctional nevus, which is one ty!e of mol (. Nunctional nevi may be li$ht brown to nearly black, an" ran$e in si#e from : to :> mm. 7hey may be either flat or very sli$htly raise %. It is thou$ht that almost all melanocytic nevi may be$in as <unctional nevi. In a"ulthoo", the lesions are seen most frequently on the !alms, soles, an" $enitali A. Clusterin$ of melanocytes at the "ermal4e!i"ermal <unction is characteristic microsco!ically. 7hese lesions are very common, an" removal is usually un"ertaken only when the mole has been e*hibitin$ color chan$esG is irritate"G or is cosmetically a !roblem. Com!oun" nevus 'choice A) is characteri#e" by clusters of melanocytes both at the e!i"ermo"ermal <unction an" within the "ermis. A halo nevus 'choice C) is a com!oun" or intra"ermal nevus surroun"e" by an area of intense inflammation. 7he inflammation "estroys the melanocytes of the imme"iately a"<acent skin, !ro"ucin$ a !ale halo aroun" the nevus. Intra"ermal nevus 'choice %) is characteri#e" by clusters of melanocytes with the "ermis but not at the <unction. ,enti$o 'choice () clinically closely resembles a <unctional nevus, but microsco!ically there is an increase" number of melanocytes occurrin$ as isolate" cells, rather than in nests, in the lower e!i"ermis. :I) (*!lanation: 7he correct answer is (. Althou$h nitrate thera!y is ty!ically one of the cornerstones of treatment for car"iac ischemia, the co4a"ministration of nitrates within ;? hours after takin$ sil"enafil is absolutely contrain"icate %. 7he vaso"ilatory effects of nitrates are !rofoun"ly am!lifie" when a"ministere" in the !resence of sil"enafil '1ia$ra), which can lea" to refractory an" life4threatenin$ hy!otension. 7herefore, !atients usin$ sil"enafil shoul" be instructe" to re!ort their use on !resentation to any emer$ency "e!artment an" to never take nitrates while usin$ the "ru$. Of all of the a"<unctive treatments for myocar"ial infarction, as!irin 'A)A) 'choice A) likely has the lar$est mortality benefit. All !atients with sus!ecte" car"iac ischemia who "o not have a history of ana!hyla*is to A)A shoul" be $iven as!irin !rom!tly. A history

of !e!tic ulcer "isease or u!!er $astrointestinal blee"in$ is not an absolute contrain"ication in this settin$, as the !otential benefits woul" likely outwei$h the risks. An$iotensin4convertin$ en#yme inhibitors, such as ca!to!ril 'choice ), may !lay a role in myocar"ial remo"elin$ "urin$ myocar"ial infarctions an" may actually have a mortality benefit if $iven in the !eri4infarction !erio %. eta4blockers, such as meto!rolol 'choice C), re"uce car"iac "eman" an" "ecrease the frequency of ventricular arrhythmias after myocar"ial infarction, with well4establishe" survival benefit. 7reatin$ !ain an" an*iety "urin$ a myocar"ial infarction with narcotics such as mor!hine 'choice %) may re"uce car"iac "eman %. 7hese "ru$s have classically !laye" an im!ortant role in the treatment of myocar"ial infarction. 7he use of thrombolytics, such as tissue !lasmino$en activator 't&A) 'choice -), has evolve" as one of the most im!ortant "evelo!ments the treatment of acute myocar"ial infarction, as it can re"uce in4hos!ital mortality by as much as />= when a"ministere" within the first hour of the onset of sym!toms. Clear contrain"ications to the use of t&A inclu"e a history of stroke, a history of an invasive sur$ical !roce"ure within the !rece"in$ ; weeks, marke" hy!ertension, or active !e!tic ulcer "iseas (. 7hus, this !atient has no contrain"ications to receivin$ t&A or other thrombolytics. ;>) A. &soriatic arthritis . 6eiter syn"rome C. 6heumatic fever %. 6heumatoi" arthritis (. )y!hilitic infection (*!lanation: 7he correct answer is . 6eiter syn"rome "evelo!s as a com!lication of two ty!es of infections: enteric infections "ue to )hi$ella, )almonella, 9ersinia, or Cam!ylobacter, an" se*ually transmitte" "iseases cause" by Chlamy"ia or Frea!lasm A. (i$hty !ercent of cases occur in 2,A4 ;D4!ositive in"ivi"uals. A male !re"ominance of I:: is characteristic of cases followin$ se*ually transmitte" infections. Nonsteroi"al anti4inflammatory "ru$s 'N)AI%s) are use" for treatmentG antibiotics are use" to re"uce the likelihoo" of "evelo!in$ 6eiter syn"rome after se*ually transmitte" "iseases, but not after enteric infections. &soriatic arthritis 'choice A) is similar to rheumatoi" arthritis with re$ar" to the !attern of <oint involvement, but is not associate" with a !ositive rheumatoi" factor. Fsually,

!soriasis ante"ates the onset of arthritis, but sometimes the o!!osite occurs. Involvement of sacroiliac <oints is common in this con"ition. Arthritis associate" with rheumatic fever 'choice C) is characteri#e" by a mi$ratory course an" is associate" with si$ns an" sym!toms of car"iac involvement. )kin lesions are known as erythema mar$inatum. 6heumatoi" arthritis 'choice %) characteristically affects small <oints 'most commonly in the han"s) in a symmetric fashion. +ost cases show circulatin$ rheumatoi" factor. 7he manifestations of sy!hilitic infection 'choice () are classically "ivi"e" into !rimary, secon"ary, an" tertiary sta$es. 7he !rimary sta$e is characteri#e" by a !ainless ulcer at the site of entry, ie, a chancr (. 7he secon"ary sta$e "evelo!s u! to C months followin$ the !rimary sta$e an" is associate" with a "iffuse maculo!a!ular eru!tion involvin$ skin an" mucous membranes, $enerali#e" lym!ha"eno!athy, fever, he!atitis, iritis, osteitis, ase!tic menin$itis, an" arthritis. 7he tertiary sta$e manifests after lon$ !erio"s of latency an" involves the ascen"in$ aorta, bones, liver, skin, an" CN). ;:) (*!lanation: 7he correct answer is A. 7he occu!ational history of workin$ in shi!yar"s always raises the sus!icion of asbestosis. Asbestosis ty!ically !ro$resses $ra"ually an" is characteri#e" by a "iffuse an" bilateral interstitial fibrosis. 7he fin"in$ of !leural calcifications is virtually !atho$nomonic for asbestosis, which causes restrictive lun$ "iseas (. Asbestosis is a risk factor for mali$nant mesothelioma, as well as for lun$ a"enocarcinoma, which is, in fact, more common in these !atients than mesotheliom A. Asbestosis an" ci$arette smokin$ are syner$istic risk factors for lun$ carcinom A. I"io!athic intersititial fibrosis 'choice ) woul" be very unlikely to cause calcifications at the base of the lun$s. ,e$ionella !neumonia 'choice C) causes lun$ infiltrates an" often !leural effusions. &atients with &neumocystis carinii !neumonia 'choice %) may have a normal chest *4ray film or !erihilar infiltrates. 7uberculosis 'choice () woul" cause a mass lesion. ;;) (*!lanation: 7he correct answer is

%. 7his !atient is hy!othyroi %. )ilent lym!hocytic thyroi"itis is a sur!risin$ly common "isor"er of !ost!artum women, occurrin$ in about /= to :>= of these !atients. It a!!ears to be an autoimmune reaction to the thyroi" $lan" '!ossibly somewhat relate" to 2ashimoto thyroi"itis) that can !ro"uce transient hy!erthyroi"ism 'relate" to follicle "estruction) followe" by hy!othyroi"ism 'that may be either !ermanent or resolve within : year). It is thou$ht that many cases are not "ia$nose %. (uthyroi" sick syn"rome 'choice A) is ty!ically seen in intensive care !atients who "o not have true clinical hy!othyroi"ism. .raves "isease 'choice ) is a cause of hy!erthyroi"ism. Io"ine "eficiency 'choice C) is now a rare cause of $oiter an" hy!othyroi"ism in the F.). )ubacute thyroi"itis 'choice () characteristically !ro"uces a ten"er thyroi" $lan %. ;0) A. Actinic keratosis . asal cell carcinoma C. %ermatofibroma %. +elanoma (. &soriasis -. )eborrheic keratosis .. )quamous cell carcinoma (*!lanation: 7he correct answer is -. 7his !atient has a seborrheic keratosis. 7his lesion is characteri#e" by li$ht brown to black !a!ules or !laques with an a"herent wa*y, $reasy scal (. 7he Hstuck4onH a!!earance is very characteristi C. It is most often foun" on the face an" trunk. Actinic keratosis 'choice A) is characteri#e" by flesh4colore" or re"4to4brown macules or !a!ules with a rou$h scal (. It is most common on sun4e*!ose" skin. asal cell carcinoma 'choice ) is characteri#e" by a !a!ule with a central ulceration an" a !early, raise" bor"er. It is most common on sun4"ama$e" skin. %ermatofibroma 'choice C) is characteri#e" by a firm, re"4to4brown no"ule with a "im!lin$ of the skin. It can occur anywher (.

+elanoma 'choice %) is characteri#e" by an asymmetric !i$mente" lesion with an irre$ular bor"er. Any chan$e in an e*istin$ skin lesion shoul" be evaluate" for melanom A. &soriasis 'choice () is characteri#e" by erythematous !laques an" !a!ules covere" with silvery scales. 7he most common sites are elbows, knees, scal!, an" back. &ittin$ of the nails may be !resent. )quamous cell carcinoma 'choice .) is characteri#e" by a hy!erkeratotic, ulcerate" or crusty lesion. 7he face, es!ecially the lower li!, is the ty!ical site of involvement. ;?) (*!lanation: 7he correct answer is choice %. &harmacolo$y is one of the most im!ortant a!!lie" basic science to!ics in clinical me"icin (. 7he !ossibility of "ru$ interactions is so im!ortant to reco$ni#e that entire or$ani#ations '!harmacies) an" com!uter cross4checkin$ '+icro+e"e*) have been create" to a""ress this issu (. 7hese conveniences, however, in no way absolve a !hysician from knowin$ the consequences an" !otential interactions of each an" every me"icine that he or she !rescribes. )!ironolactone is a !otassium4s!arin$ "iureti C. 7he mistake of callin$ this "ru$ a H"iureticH an" cuein$ your brain to think Hre!lenish electrolytesH coul" cause this !atient to "ie from hy!erkalemic car"iac arrest. ,actulose an" oral !otassium 'choice A) is a useful combination for the treatment of he!atic ence!halo!athy. ,actulose is a non4metaboli#able su$ar that aci"ifies the stool an" thus tra!s ammonia in the .I lumen for e*cretion. &re"nisone an" inhale" albuterol 'choice ) is the combination of a $lucocorticoi" an" an inhale" beta a"rener$ic a$onist for asthma. &ro!ranolol an" isosorbi"e "initrate 'choice C) is a combination of a non4s!ecific beta4 blocker an" a non4s!ecific veno"ilator. 7his combination is useful in the treatment of an$ina since the beta4blocker !revents refle* tachycar"ia secon"ary to the isosorbi"e "initrate4in"uce" "ro! in !reloa %. )!ironolactone an" hy"rochlorothia#i"e 'choice () is a combination of a 5 s!arin$ "iuretic an" a thia#i"e "iuretic that is very effective in the treatment of cirrhotic e"ema an" ascites. ;/) (*!lanation:

7he correct answer is A. 7his !atient has !robable acute he!atitis. -eatures s!ecifically su$$estin$ acute he!atitis inclu"e his <aun"ice, itchiness lea"in$ to multi!le e*coriations, ten"er enlar$e liver, an" !al!able s!leen ti!. 2e re!orts no risk factors for he!atitis A infection, such as "rinkin$ water in a forei$n country with !erio"ic e!i"emics of he!atitis A. 2e "oes have a risk factor for he!atitis , which can be transmitte" throu$h bloo" !ro"ucts 'now rare because of bloo" screenin$), contaminate" syrin$es amon$ "ru$ users, an" se*ual contact '!articularly when involvin$ the rectum). 7he fin"in$s of a coa$ulo!athy or of an ence!halo!athy confer the worst !ro$nosis in !atients with an acute viral he!atitis. +ost !atients with he!atitis , even with severe laboratory abnormalities, may be followe" conservatively so lon$ as they "o not "evelo! a coa$ulo!athy or e*hibit si$ns of ence!halo!athy. 7hese fin"in$s, in fact, su$$est the !ossibility of fulminant he!atic failur (. Althou$h the transaminases may rise to very hi$h levels 'choices of !ro$nostic value in viral he!atitis. an" C), they are not

)imilarly, severe <aun"ice 'choice %) is of limite" !ro$nostic value, as is leukocytosis 'choice (). ;C) (*!lanation: 7he correct answer is A. 7his is a ty!ical !resentation of I$A ty!e heavy chain "iseas (. 7he "isease is centere" in the +i""le (ast, an" affecte" in"ivi"uals are usually between the a$es of :> an" 0>. 7hey !resent with ab"ominal mass an" malabsor!tion. While the lesion behaves like an ab"ominal lym!homa, the mor!holo$y of the involve" cells in the lym!hocyte4!lasma cell line may or may not be obviously mali$nant. 7he lesion is usually confine" to the $ut an" mesenteric lym!h no"esG other sites such as !eri!heral no"es, s!leen, liver, an" bone marrow are usually unaffecte %. )erum !rotein electro!horesis ty!ically shows either a "ecrease" $amma fraction or a broa" ban" in the al!ha ; an" beta re$ions. +onoclonal s!ikes are less common. 7he area of the broa" ban" ty!ically reacts with anti4I$A but "oes not react with anti4li$ht chain, in"icatin$ that it is com!ose" of 'usually hetero$eneous) heavy chain I$A material. It has been s!eculate" that this con"ition may re!resent an aberrant reaction to some sort of bacterial infection, since !rolon$e" remissions have sometimes been re!orte" after treatment with corticosteroi"s, cytoto*ic "ru$s, an" broa"4s!ectrum antibiotics. +acro$lobulinemia 'choice ) is a mali$nant !lasma cell "yscrasia of cells that normally secrete I$+. It usually !resents with hy!erviscosity sym!toms, inclu"in$ hea"aches an" visual "isturbances.

+onoclonal $ammo!athy of un"etermine" si$nificance 'choice C) refers to the !resence of a usually small + !rotein in urine or serum, ty!ically from an a!!arently healthy ol"er a"ult. +ulti!le myeloma 'choice %) usually !resents with skeletal !ain secon"ary to lytic bone lesions, renal failure, or recurrent bacterial infections. +yelo"ys!lastic syn"rome 'choice () is a cause of !ro$ressive bone marrow failure an" can be a !recursor of acute myelo$enous leukemi A. ;D) (*!lanation: 7he correct answer is %. An im!ortant conce!t to reco$ni#e in the treatment of me"ical con"itions is that certain me"ications overla! syn"romes an" are efficacious in many areas. 7his Hco4 treatmentH o!tion ma*imi#es the benefits of each "ru$ in a re$imen an" often a""resses two or more issues simultaneously. 7his "ru$ is an al!ha4: rece!tor anta$onist that is very efficacious in the treatment of hy!ertension. 7his class of "ru$s is also useful in the treatment of beni$n !rostatic hy!ertro!hy ' &2). ,isino!ril 'choice A) is an AC( inhibitor. 7hese "ru$s have e*cellent efficacy in treatin$ bloo" !ressur (. In a""ition, they are very useful in "elayin$ the !ro$ression of renal "isease in "iabetics '"iabetic ne!hro!athy). +eto!rolol 'choice ) is a beta blocker. 7his class of "ru$s is recommen"e" as first4line monothera!y for hy!ertension. Fse of these a$ents has been associate" with im!rove" survival in a number of clinical trials. Nife"i!ine 'choice C) is a calcium channel blocker that has reasonable efficacy in treatin$ hy!ertension. It has no known !rotective benefit in !atients with &2. 1era!amil 'choice () is also a calcium channel blocker. In a""ition to its use as a thera!y for hy!ertension, is also use" as a rate control a$ent in !atients with atrial fibrillation. ;@) (*!lanation: 7he correct answer is C. A"ministration of intravenous !henytoin shoul" follow intravenous ben#o"ia#e!ines. 7his is the usual !rotocol in the mana$ement of status e!ile!ticus. -ast actin$ ben#o"ia#e!ines have a "uration of action of a few minutes. Fnless there is a contrain"ication, the !atient shoul" receive a loa"in$ "ose of !henytoin intravenously.

(lectroconvulsive thera!y 'choice A) is use" in the mana$ement of severe "e!ression. Intravenous !entobarbital 'choice ) woul" be use" if he were still sei#in$. 2owever, at this time, in"ucin$ a !entobarbital coma is not nee"e %. Carbama#e!ine 'choice %) is an effective anticonvulsant, but cannot be $iven intravenously or intramuscularly. (thosu*imi"e 'choice () is in"icate" for the treatment of absence sei#ures. ;I) (*!lanation: 7he correct answer is A. asal cell carcinoma is the most common form of skin cancer an" risk factors inclu"e sun e*!osure an" ultraviolet ra"iation. -eatures inclu"e an ulcerate" center with !early, hea!e" u! e"$es. (*cision may be require %. (rythema no"osum 'choice ) is a vascular "isor"er whose skin manifestations inclu"e erythematous an" no"ular lesions, es!ecially on the anterior as!ect of the tibi A. ,euko!lakia 'choice C) is an (!stein4 arr virus4relate" lesion that woul" be foun" on the lateral as!ects of the ton$ue an" is white in a!!earanc (. +elanoma 'choice %) has the ma*imum metastatic !otential an" may be no"ular or ra"ial. It may have irre$ular bor"ers, may be of varie$ate" coloration, an" may be on any as!ect of the bo"y. )quamous cell carcinoma 'choice () is the secon" most common form of skin cancer after basal cell carcinoma an" woul" be most common on sun4e*!ose" surfaces as well. 0>) (*!lanation: 7he correct answer is %. 7he ma<or car"iac risk factors are family history, a$e, tobacco use, hy!ertension, "iabetes, an" low 2%,. 7obacco use counts, even if it is no lon$er current, for :/ years. 0:) (*!lanation: 7he correct answer is

(. 7his !atient has monoarticular arthritis of the ri$ht kne (. 7he ma<or "ifferential "ia$nosis in these !atients is of a crystalline arthritis '$out) versus an infectious arthritis ')ta!hylococcus vs. .onococcus). In this re$ar", knowle"$e of her se*ual history an" any hi$h4risk se*ual behaviors that woul" !ut her at risk for $onococcal infection woul" be relevant. acterial $astroenteritis 'choice ) may be associate" with 6eiter syn"rome, but this involves multi!le <oints in association with con<unctivitis an" urethritis an" "oes not !resent as monoarticular arthritis. 2e!atitis vaccination 'choice C) is a!!ro!riate in occu!ations such as "ental hy$iene, which are e*!ose" to bo"ily flui"s. 2owever, he!atitis is not associate" with a monoarticular arthritis, althou$h !olyarthral$ias may occur in !atients with he!atitis who "evelo! a serum4sickness like illness. A traumatic <oint in<ury 'choice %) is always of relevance in the settin$ of monoarticular arthritis. 2owever, since the arthritis is in the ri$ht knee, trauma to the ri$ht ankle woul" be unlikely to be causativ (. 0;) (*!lanation: 7he correct answer is C. 7he only risk factor for reversible bronchos!asm "escribe" in this !atient is the use of a nonsteroi"al anti4inflammatory "ru$ 'N)AI%) for !remenstrual cram!s. In fact, her sym!toms "o occur at the time of her !erio", relatin$ it tem!orally to the use of N)AI%s. As!irin or N)AI%s can !rovoke bronchos!asm by inhibitin$ cycloo*y$enase e*!ression, which results in $reater shuntin$ of arachi"onic aci" metabolism throu$h the li!o*y$enase !athway. 7his results in the !ro"uction of leukotrienes, which are !otent bronchoconstrictors. 7he inhibition of cycloo*y$enase lea"s to re"uce" !ro"uction of !rosta$lan"ins an" !rostacyclins. Choice A is wron$ because the leukotriene !ro"uction woul" be increase %. Choice %. is wron$ because cycloo*y$enase e*!ression is "ecrease

Choice % is wron$ because the !rosta$lan"in synthesis woul" be "ecrease %. Choice ( is wron$ because the !rostacyclin !ro"uction woul" be "ecrease %.

00) (*!lanation: 7he correct answer is %. Althou$h this !atient is likely in my*e"ema coma, the basics of me"ical mana$ement A,WA9) take !rece"enc (. Airway, breathin$, an" circulation evaluation are a !riority in this !atient with a res!iratory rate less than :> an" clearly abnormal o*y$en saturation of her bloo %. In this !atient, securin$ an airway an" then a""ressin$ her remainin$ issues are of !aramount im!ortanc (. I1 hy!ertonic saline 'choice A) woul" be incorrect, for two reasons. -irst, the A Cs of me"ical mana$ement take !riority. )econ", althou$h many severely hy!othyroi" !atients are hy!onatremic, this has yet to be "emonstrate" in this !atient. In this settin$, !ressors 'choice ) nee" to be $iven via central venous line, an" this access has not yet been establishe" in this !atient. I1 thyro*ine 'choice C) woul" be the a!!ro!riate treatment A-7(6 thyroi" function tests were verifie" an" the !atient was stabili#e %. Car"iac "efibrillation 'choice () is incorrect because it is not yet known whether this !erson is e*!eriencin$ a "ysrhythmia that woul" res!on" to this intervention. 7he two most common ty!es of such rhythms seen in this ty!e of emer$ency settin$ are ventricular fibrillation an" ventricular tachycar"i A. 0?) (*!lanation: 7he correct answer is C. 7he clinical sym!tomatolo$y is hi$hly su$$estive of multi!le sclerosis '+)). Often, the manifestations are mistaken for !sychiatric or !sychosomatic "isor"ers. Clues to the "ia$nosis inclu"e a !ast e!iso"e of transient blurre" vision 'in"icatin$ a bout of o!tic neuritis), s!astic !ara!aresis 'su$$estin$ involvement of the corticos!inal tracts), an" oli$oclonal I$. ban"s in the C) -. +6I investi$ations woul" !robably reveal well4"emarcate" areas of hy!erintensity on 7;4wei$hte" ima$es, which corres!on" to re$ions of "emyelination. 7he sensation of electric shocks "own the s!ine is known as ,hermitteAs si$n. Acute "isseminate" ence!halomyelitis 'A%(+) 'choice A) is a rare acute form of "emyelination that follows immuni#ation or viral infections. 7he first e!iso"e of +) may be "ifficult to "ifferentiate from A%(+.

,yme "isease 'choice ), an infection cause" by orrelia bur$"orferi, can also cause s!astic !ara!aresis, as well as cranial nerve !alsies an" cerebellar ata*ia, thus mimickin$ +). 2owever, ,yme "isease is associate" with a characteristic rash an" a s!ecific $eo$ra!hic "istribution. &araneo!lastic syn"romes 'choice %) may be consi"ere" in the ol"er a$e $rou!. )everal !araneo!lastic neurolo$ic syn"romes have been "escribe", inclu"in$ cerebellar "e$eneration, ence!halomyelitis, sensory an" autonomic neuro!athy, myasthenic syn"rome, an" visual syn"romes. 2owever, the lon$4stan"in$ course of this case an" youn$ a$e ar$ue a$ainst this !ossibility. )!inocerebellar syn"rome 'choice () can be e*clu"e %. -rie"reich ata*ia is familial an" associate" with !es cavus an" scoliosis. 2ere"itary s!astic !ara!aresis may be confuse" with +), but the C)- is normal. 0/) (*!lanation: 7he correct answer is (. 7his is ty!hoi" fever, which, "es!ite wi"es!rea" immuni#ation in many !arts of the country, still has an inci"ence of ?>> to />> cases !er year in the Fnite" )tates. 7he or$anism is an enteric or$anism s!rea" most frequently by a fecal4oral route 'inclu"in$ contamination of foo" or water su!!lies). +any of the estimate" ;>>> carriers of the "isease in the Fnite" )tates are el"erly women with biliary tract "iseas (. 7he "isease can be "ifficult to "ia$nose, often because it is not sus!ecte %. It shoul" be consi"ere" in !atients who a!!ear much more ill than a sim!le listin$ of their com!laints woul" su$$est. 7he rose s!ot rash "escribe" in the question stem is a classic "ia$nostic clue that will !robably show u! in test questions about the "isease, but you shoul" be aware that it is only seen in about :>= of cases. CN) sym!toms an" su!erinfections such as the !neumococcal !neumonia are relatively common in severe cases. 7he bloo" stu"y results note" in the question stem can be another hel!ful clue to the !ossibility of ty!hoi" fever, an" are unusual in other .I con"itions. 7he or$anism can be culture" from bloo" or bone marrow in the first two weeks of illness, an" from stool in the thir" to fifth week of illness. Antibiotic thera!y with ce!halos!orins or quinolones may "ecrease the severity or "uration of the illnessG rela!ses may occur, which also usually res!on" to antibiotic thera!y. )everely ill !atients may benefit from su!!ortive care inclu"in$ nutritional su!!ort an" sometimes $lucocorticoi" thera!y. rucellosis 'choice A) causes a recurrent fever after e*!osure to contaminate" milk !ro"ucts. Cholera 'choice ) causes a !rofoun" secretory "iarrhea with rice water stools an" has fewer systemic manifestations 'other than those "ue to "ehy"ration an" electrolyte imbalance) than ty!hoi" fever. +elioi"osis 'choice C) causes !neumonia an" "isseminate" infection.

&la$ue 'choice %) causes massive lym!h no"e enlar$ement an" !neumonia after e*!osure to infecte" ro"ents an" their !arasites. 0C) (*!lanation: 7he correct answer is C. 7his !atient has the ty!ical fin"in$s of an aty!ical !neumonia, with low4$ra"e fevers an" a bilateral infiltrate without a !ro"uctive s!utum. 7he col" a$$lutinin assay is !ositive, in"icatin$ that the un"erlyin$ infection is +yco!lasma !neumoni A. 7his is also the e*!lanation for his anemia, since he is a$$lutinatin$ re" cells at col" tem!eratures. 7he a!!ro!riate antibiotic for +yco!lasma is oral erythromycin. Althou$h the other antibiotics, am!icillin 'choice A), ce!hale*in 'choice ), tetracycline 'choice %), an" trimetho!rimBsulfametho*a#ole 'choice (), are a!!ro!riate broa"4broa" s!ectrum antibiotics for community4acquire" !neumonias, they are not effective a$ainst the aty!ical !neumonias.

0D) (*!lanation: 7he correct answer is (. While either folate "eficiency or vitamin :; "eficiency coul" !ro"uce a me$aloblastic anemia such as that seen in this !atient, the s!ecific hint of chronic $astritis shoul" su$$est insufficient synthesis of the intrinsic factor nee"e" for 1itamin :; absor!tion by the small intestin (. -or classic !ernicious anemia, autoantibo"ies "irecte" a$ainst !arietal cells an" intrinsic factor are "ocumente", but some !hysicians use the term !ernicious anemia to also inclu"e vitamin :;4"eficient anemic states in which another !rocess, such as chronic $astritis 'as in this case), has "estroye" lar$e amounts of $astric mucos A. 1itamin :; "eficiency anemia can also be seen as a result of ina"equate "iet '!articularly amon$ ve$etarians) an" small intestinal "iseas (. A""itionally, ina"equate utili#ation of :; can occur in many severely ill !atients. 7he "ia$nosis of vitamin :; "eficiency can be confirme" with a serum vitamin :; assay. )u!!lemental vitamin :; is $iven as an I+ in<ection. :; su!!lementation must be $iven for life unless the un"erlyin$ abnormality !ro"ucin$ the "eficiency is correcte %. eta thalassemia trait 'choice A) mi$ht !ro"uce some !ale cells, but they woul" not be enlar$e %.

-olate "eficiency anemia 'choice ) woul" a!!ear similar to :; "eficiency, but the s!ecific hint of chronic $astritis shoul" su$$est :; "eficiency instea %. Iron "eficiency anemia 'choice C) !ro"uces small erythrocytes. )ickle cell anemia 'choice %) is associate" with abnormally sha!e" erythrocytes. 0@) (*!lanation: 7he correct answer is %. 2y!ertonic saline woul" be the most a"visable treatment. 7he !atient has hy!onatremia an" sei#ures, which woul" su$$est an acute "ro! in so"ium. 7here is no mention of a brain lesion to e*!lain the sei#ures. 7he !atient most likely has a syn"rome of ina!!ro!riate "iuretic hormone secretion ')IA%2) "ue to his lun$ cancer. -urosemi"e may be $iven with the hy!ertonic saline to !romote water loss. %emeclocycline 'choice A) is a tetracycline antibiotic that causes a ne!hro$enic "iabetes insi!i"us. It can be use" to treat )IA%2 on a chronic basis. It will not work imme"iately an" is not a"vise" to acutely raise so"ium -lui" restriction 'choice ) woul" be a $oo" lon$4term treatment for )IA%2 but woul" be too slow to treat a !atient with sei#ures. 2y!onatremia may be cause" by A""ison "iseas (. Only in this case will steroi"s 'choice C) be of any valu (. 2y!othyroi"ism can cause hy!onatremi A. 2owever, there is no evi"ence that this !atient has hy!othyroi"ism, an" $ivin$ em!iric thyro*ine 'choice () has no role in treatin$ hy!onatremi A. 0I) (*!lanation: 7he correct answer is C. 7his is the most frequent clinical !resentation of $lioblastoma multiforme '. +). 7his hi$hly a$$ressive tumor is the most common !rimary mali$nant neo!lasm of the CN). It is com!ose" of !oorly "ifferentiate" astrocytes with areas of necrosis an" microvascular !roliferation. In the W2O $ra"in$ system of brain tumors, . + corres!on"s to $ra"e I1 astrocytoma 'i. (., the most ana!lastic form). Fsually, !atients !resent with a clinical sym!tomatolo$y of less than 0 monthsA "uration. +6I an" C7 scans show the !icture outline" in the questions stem. Of $reat im!ortance in the "ifferential "ia$nosis with other cerebral

lesions are the !oorly "efine" bor"ers, rin$4enhancin$ a!!earance, an" location in the white matter. +ost !atients "ie within : year of "ia$nosis, even with the most a$$ressive sur$ical an" ra"iation treatments. Abscess 'choice A) is an im!ortant "ifferential consi"eration in rin$4enhancin$ brain masses. 7he clinical history is usually hel!ful, since brain abscesses "evelo! in the !resence of some !re"is!osin$ con"ition, such as otitis, sinusitis, or se!sis. -urthermore, abscesses ten" to be well4"emarcate", with a thin uniform rim of contrast enhancement. . + has a more irre$ular an" "iffuse outlin (. %emyelinatin$ "isease 'choice ) woul" manifest with white matter chan$es i"entifiable as 7; hy!erintensity on +6I. In multi!le sclerosis, !laques of "emyelination are shar!ly circumscribe" an" more frequently locate" in the !eriventricular re$ions. Infarction 'choice %) involves the $ray matter in a s!ecific vascular "istribution corres!on"in$ to the affecte" artery. It !resents with an acute sym!tomatolo$y. +etastasis 'choice () shoul" also be consi"ere" in this cas (. -actors a$ainst a metastatic lesion inclu"e !oor "emarcation an" location within the white matter. +etastases are usually shar!ly "emarcate", multi!le, an" situate" at the $ray4white matter <unction. ?>) (*!lanation: 7he correct answer is C. 7he !atient !robably has !em!hi$us vul$aris, which is an uncommon autoimmune skin "isor"er characteri#e" by blisterin$ an" erosions involvin$ the mucous membranes an" skin. 7he autoimmune attack is on the <unctions between e!ithelial cells in the e!i"ermis. 7he blisters occur hi$h in the e!ithelium an" can ru!ture easily, !ro"ucin$ !ainful erosions. NikolskyAs si$n, in which rubbin$ of a!!arently unaffecte" skin causes a se!aration of the outer layers, is a hel!ful "ia$nostic clu (. &em!hi$us ten"s to be$in in the mouth, where ra!i" ru!ture of the blisters may lea" to the im!ression that the initial lesion is an ulcer rather than a blister. io!sy with immunofluorescence stu"ies can "emonstrate blister formation hi$h in the e!ithelium that is accom!anie" by I$. "e!osition on e!ithelial cell surfaces. &em!hi$us vul$aris is a serious, chronic con"ition that can be life4threatenin$ as a result of flui"Belectrolyte imbalance, secon"ary infection, or com!lications of the hi$h4"ose corticosteroi" thera!y that may be necessary to brin$ the con"ition un"er control. ullous !em!hi$oi" 'choice A) causes tense bullae that "o not ru!ture easily. %ermatitis her!etiformis 'choice ) causes clusters of intensely !ruritic vesicles, !a!ules, an" urticarial lesions.

)tevens4Nohnson syn"rome 'choice %) is a severe, blisterin$, form of erythema multiforme that characteristically shows at least a few tar$et lesions. 7o*ic e!i"ermal necrolysis 'choice () can cause wi"es!rea" flacci" blisters an" NikolskyAs si$n may be !ositive, but this "isor"er ten"s to "evelo! much more ra!i"ly than !em!hi$us vul$aris. ?:) (*!lanation: 7he correct answer is A. An emboli#ation that woul" cause the sym!toms "escribe" by the !atient woul" most likely lea" to an absence of "istal !ulses in the affecte" le$, unless the embolism un"erwent autolysis. 7he cell "eath an" subsequent lysis cause" by a thromboembolic event woul" likely be reflecte" by increases in the lactate "ehy"ro$enase level 'choice ). ,oss of sensation 'choice C) is one of the classic si$ns of an ischemic lim . 7he other si$ns inclu"e !ain, !allor, !aresthesias, loss of function, an" coolness. A normal level of the !rothrombin time 'choice %) reflects the fact that she is not currently anticoa$ulate", which, in the settin$ of atrial fibrillation 'A-), !laces her at si$nificant risk for a thromboembolic event. -in"in$ that the !atient has a re$ular rhythm 'choice () may, at first blush, make the !hysician question the "ia$nosis. On further reflection, however, the !atient may have !aro*ysmal atrial fibrillation '&A-). 7he s!ontaneous conversion to normal sinus rhythm may have !reci!itate" thrombi from the atrium to be transmitte" to the "istal arteries. 7he !ossible occurrence of this ty!e of event is !recisely why !atients who are chronically in A- or &A- are ty!ically anticoa$ulate" for several months !rior to car"ioversion. ?;) (*!lanation: 7he correct answer is A. 7his !atient most likely has rheumatoi" arthritis, a chronic inflammatory "isor"er characteri#e" by symmetrical <oint involvement an" e*traarticular manifestations. It ty!ically affects mi""le4a$e" women. )ym!toms inclu"e <oint !ain an" swellin$, low4 $ra"e fever, malaise, fati$ue, vasculitis, !ericar"itis, rheumatic no"ules, e!iscleritis, an" scleritis. ,aboratory fin"in$s inclu"e normochromic, normocytic anemia, increase" erythrocyte se"imentation rate, an" serum rheumatoi" factors. 7he !ro*imal inter!halan$eal <oints '&I&), metacar!o!halan$eal <oints '+C&), an" the wrist <oints are the most commonly involve" <oints. %istal inter!halan$eal <oints '%I&) are usually s!are %. 6a"io$ra!hic fin"in$s inclu"e soft tissue swellin$, <oint effusions, <u*taarticular osteo!enia, loss of articular cartila$e, <oint s!ace narrowin$, an" bone erosions. 7he

treatment inclu"es rest, !hysical thera!y, N)AI%s, as!irin, corticosteroi"s, $ol", methotre*ate, !enicillamine, cyclos!orine, sulfasala#ine, an" hy"ro*ychloroquin(. Osteo!hyte formation 'choice ), subchon"ral cyst formation 'choice C), an" subchon"ral sclerosis 'choice %) are the ra"io$ra!hic fin"in$s of osteoarthritis. Osteoarthritis is a noninflammatory <oint "isease characteri#e" by loss of articular cartila$e an" the aforementione" fin"in$s. 7he sym!toms inclu"e a "ee!, achin$ <oint !ain that is a$$ravate" by use an" relieve" by rest. &hysical e*amination shows <oint ten"erness, bony cre!itus, warmth an" "eformities '2eber"enAs no"es4%I& an" ouchar"As no"es4&I&). A$e, wear an" tear, obesity, trauma, an" certain chronic con"itions are factors that increase the risk of osteoarthritis. 7he treatment inclu"es wei$ht loss, !hysical thera!y, acetamino!hen, an" sur$ery. )ubchon"ral to!hi 'choice () are foun" in $outy arthritis. 7hey ty!ically a!!ear as H!unche" outH lesions of the subchon"ral bone. A to!hus is a collection of urate crystals, inflammatory cells, an" fibrosis. 7o!hi can lea" to cartila$e an" bone "estruction.

?0) (*!lanation: 7he correct answer is C. 7his !atient is an I1 "ru$ abuser with a history that "escribes two !rior infections !robably relate" to "ru$ use an" )ta!hylococcus infection, i. (., cellulitis an" tricus!i" valve en"ocar"itis. 7he lobar consoli"ation with an air4flui" level is consistent with )ta!hylococcus aureus !neumonia with cavitation. None of the other infections note" cause a cavitatin$ !neumonia. Associate (scherichia coli'choice A) with !neumonia secon"ary to "issemination of a urinary tract or other (. coli infection. Associate 2aemo!hilus influen#ae'choice ) with community acquire" !neumonia, !articularly in chil"ren an" in the el"erly with !re4e*istin$ lun$ "iseas (. Associate )ta!hylococcus e!i"ermis'choice %) with con"itions causin$ a skin break. 6emember: the resultin$ !neumonia will not cause cavitation, since this or$anism "oes not have the ran$e of lytic en#ymes that )ta!hylococcus aureus "oes. Associate )tre!tococcus !neumonia'choice () with the most common form of community4acquire" !neumoni A.

??) (*!lanation: 7he correct answer is C. 7he !atient initially ha" osteitis fibrosa cystica as a result of the !rimary hy!er!arathyroi"ism. When the a"enoma was remove" an" &72 level "ecrease", the skeleton un"erwent ra!i" reminerali#ation. 7his create" an increase in calcium requirement. Once the re!air was com!lete, the calcium "eman" "ecrease". If the remainin$ $lan"s were "estroye" 'choice A) she woul" have been !ermanently hy!ocalcemi C. In !seu"ohy!o!arathyroi"ism 'choice ), there is en" or$an resistance to &72, resultin$ in ki"neys an" bones bein$ unres!onsiv (. &atients have hy!ocalcemia an" hy!er!hos!hatemia. )evere !ancreatitis woul" cause sa!onification an" hy!ocalcemia 'choice %). )he woul" also have sym!toms of e!i$astric !ain. If the wron$ $lan" were remove", she woul" be still hy!ercalcemic 'choice (). ?/) (*!lanation: 7he correct answer is C. +ost cases of he!atocellular carcinoma in the F.). are associate" with cirrhosis, re$ar"less of the un"erlyin$ caus (. Alcohol4relate" cirrhosis, as well as cirrhosis "evelo!in$ in the conte*t of chronic viral he!atitis 'either or C), is the most common !re"is!osin$ con"ition. %evelo!ment of liver cancer is thus often maske" by the un"erlyin$ "iseas (. 2owever, this cancer shoul" be sus!ecte" when a !atient with cirrhosis e*hibits ra!i" "eterioration an" hemorrha$ic ascites. Aflato*in e*!osure 'choice A) is a ma<or cause of he!atocellular carcinoma in some re$ions of Africa an" Chin A. Aflato*ins are !ro"uce" by fun$i 'As!er$illus flavus) that may contaminate foo %. 7hese to*ins are metaboli#e" by the liver into muta$enic interme"iates that act by "irect interaction with the %N A. 7he !/0 tumor su!!ressor $ene is !articularly susce!tible to such muta$enic action. Anabolic steroi"s 'choice ) an" oral contrace!tives 'choice () are associate" with an increase" risk of "evelo!in$ liver a"enoma an" !eliosis he!atis. 7he former is a beni$n he!atocellular neo!lasm, whereas the latter is a vascular lesion consistin$ of "ilate" vascular channels. oth are usually clinically silent, but liver a"enoma may occasionally manifest with catastro!hic intra!eritoneal hemorrha$ (.

2emochromatosis 'choice %) is associate" with a $reatly increase" risk of he!atocellular carcinoma 'u! to ;>= of affecte" in"ivi"uals). 7yrosinemia 'choice -) is a rare $enetic metabolic "isor"er associate" with increase" risk of neonatal he!atitis an" he!atocellular carcinom A. F! to 0/= of !atients with this inborn error of metabolism will eventually "evelo! he!atocellular carcinom A. ?C) (*!lanation: 7he correct answer is (. In the case of a col" foot, the key issues are "ocumentin$ a neurolo$ic e*amination as it !ertains to the area, as well as "ocumentin$ any com!romise of circulation. -or this !atient, the lack of !ulses an" the likely insensate an" weak foot necessitate an$io$ra!hic "ocumentation of any vascular issues that may be re!arable or relieve" by fasciotomy. A C7 scan of the ab"omen 'choice A) is not in"icate" as a routine screenin$ test in the absence of blunt or !enetratin$ ab"ominal traum A. 7he same hol"s true for a C7 scan of the hea" an" neck 'choice ). In !ractice, however, many trauma !atients who have any history of hea" in<ury often $et a hea" C7 to ensure that there is no un"erlyin$ brain trauma or blee"in$. An echocar"io$ram 'choice C), is not in"icate" in this !atient. In trauma !atients, this ima$in$ mo"ality is often use" to evaluate for car"iac contusions or wall motion abnormalities after blunt chest traum A. )ince this !atient was not un"er the influence of any substances an" was oriente" an" alert, a clinical e*amination is a"equate to clear his cervical s!ine from in<ury, an" a lateral c4s!ine ra"io$ra!h 'choice %) is not nee"e %. ?D) (*!lanation: 7he correct answer is . 7his !atientAs chronic he!atitis is !robably relate" to her history of viral infections by he!atitis viruses. 2e!atitis A "oes not cause chronic "isease, an" can thus be e*clu"e" from consi"eration as a cause of her chronic he!atitis. oth he!atitis an" he!atitis C are ma<or causes of chronic he!atitis. Althou$h the !atient has antibo"ies to he!atitis surface anti$en, the question stem "oes not in"icate that she still has the anti$en. 7his

makes it less likely that he!atitis is the cause of her current !roblems. On the other han", !ositivity for he!atitis C antibo"y, !articularly when cou!le" with bio!sy evi"ence of chronic he!atitis, is consi"ere" "ia$nostic for current he!atitis C infection. 'We currently lack "irect markers for viral anti$ens.) 7he combination of al!ha4interferon an" ribavirin antiviral me"ications is the current state4of4the4art treatment for he!atitis C. -ormerly, al!ha4interferon 'choice A) was use" as monothera!y but ha" only a mo"erate short4term success rate an" a very hi$h rela!se rat (. 7he a""ition of ribavirin si$nificantly increase" the short4term an" lon$4term remission rates. &re"nisone 'choice %) an" a#athio!rine 'choice C) are immunosu!!ressant "ru$s that are contrain"icate" in !atients with active he!atitis C infection. 6ibavirin as monothera!y 'choice () was foun" to be unsuccessful in multi!le !lacebo control trials. ?@) (*!lanation: 7he correct answer is (. 7his !atient has a!lastic anemi A. 7his con"ition is characteri#e" by !anhy!o!lasia of the hematolo$ic elements of the marrow with resultant severe anemia, leuko!enia, an" thrombocyto!eni A. While a!!ro*imately half of cases of a!lastic anemia are i"io!athic, incitin$ causes can be i"entifie" 'an" shoul" be remove" if !ossible) in the remain"er. Amon$ the reco$ni#e" contributors are e*!osures to chemicals 'notably ben#ene an" inor$anic arsenic), ra"iation, an" some "ru$s 'notably antineo!lastic a$ents, antibiotics, N)AI%s, an" anticonvulsants). It is thou$ht that these a$ents may sometimes tri$$er a hy!ersensitivity reaction in susce!tible in"ivi"uals. Abun"ant me$akaryocytes 'choice A) su$$est !rimary thrombocythemia an" other myelo!roliferative "iseases. 2y!erse$mente" neutro!hils 'choice ) su$$est me$aloblastic anemi A. +any rin$e" si"eroblasts 'choice C) su$$ests si"eroblastic anemi A. +arrow fibrosis 'choice %) su$$ests myelofibrosis. 7he marrow in a!lastic anemia is fatty, rather than fibroti C. ?I)

(*!lanation: 7he correct answer is

. (m!hysematous !yelone!hritis is a com!licate" !yelone!hritis seen in "iabetic women. 7hey have air in the renal !arenchyma, usually "ue to a $ram4ne$ative or$anism such as (scherichia coli. Without ra!i" treatment, often requirin$ sur$ical "raina$e, em!hysematous !yelone!hritis is a fatal "iseas (. (arly reco$nition is a critical as!ect in a !atient like this, so that treatment with antibiotics an" sur$ery can be quickly be$un. Anaerobic urose!sis 'choice A) is very rare, an" is cause" by anaerobic bacteri A. Fncom!licate" !yelone!hritis 'choice %) is unlikely, $iven the evi"ence of air in the renal !arenchym A. 7here is no reason to sus!ect $enitourinary tuberculosis 'choice () or obstruction 'choice C) here if the ima$in$ "oes not reveal hy"rone!hrosis.

/>) (*!lanation: 7he correct answer is . .uillain4 arrV syn"rome is a "emyelinatin$ inflammatory !olyneuro!athy often tri$$ere" by an u!!er res!iratory infection. 7he ty!ical clinical manifestations inclu"e motor weakness that manifests with an Hascen"in$H !attern 'involvin$ the lower an" then the u!!er limbs) an" elevate" !rotein with a normal cell count or mil" lym!hocytosis in the C) -. 7his con"ition may be fatal when res!iratory muscles are involve %. It is !robably of autoimmune ori$in. otulism 'choice A) manifests with su""en "evelo!ment of "i!lo!ia, "ys!ha$ia, an" nasal s!eech, followe" by !aralysis of res!iratory muscles an" limbs.

+yasthenia $ravis 'choice C) is a "isor"er cause" by a ty!e II hy!ersensitivity reaction, in which autoantibo"ies are !ro"uce" a$ainst muscular nicotinic rece!tors. 7his lea"s to interference with choliner$ic neurotransmission an" re"uce" motor res!onse followin$ nerve stimulation. Characteristically, the "isease be$ins with fati$ability of e*ternal ocular muscles an" !ro$ressive involvement of other skeletal muscles. &oliomyelitis 'choice %) has become a rare "iseas (. It manifests with flacci" !aralysis, hea"ache, stiff neck, an" fever. C)fin"in$s inclu"e mil" lym!hocytosis 'rarely above />> cellsBmm0) an" normal or sli$htly elevate" !rotein. .uillain4 arrV may simulate !oliomyelitis. Increase" !rotein in the C)- an" the characteristic ascen"in$ !aralysis hel! "istin$uish .uillain4 arrV from !oliomyelitis. &olymyositis 'choice () is a chronic inflammatory "isease of skeletal muscles an" !resents with !ro*imal muscle weakness an" !ain. 7he erythrocyte se"imentation rate an" creatine kinase are increase %. 7he "isease is thou$ht to arise from an abnormal 74lym!hocyte4me"iate" res!onse tar$etin$ myofiber anti$ens.

USMLE Step 2 Practice Test Block :ame; +nstr#ctions; Ans'er the *#estions $elo' to the $est of yo#r a$ility. =hen yo# finish the test% click the 2heck $#tton at the $ottom to )ie' the res#lts.

1.A 22!year!ol" sociologist ret#rns from < months in Ramaica an" notes a <!po#n" 'eight loss "#ring her 1st month $ack home. She reports that her appetite is normal. She has "e)elope" symptoms of mil" a$"ominal cramping an" $loating after meals an" fre*#ent greasy $o'el mo)ements. A physical e.amination is consistent 'ith e)i"ence of recent

'eight loss% an" her a$"ominal e.amination is normal. A stool specimen is g#aiac negati)e. There are scattere" ecchymoses on all fo#r e.tremities. La$oratory res#lts re)eal an al$#min of 2.8 g7"L 3normal I&.1 g7"L6% an +:A of 1.8% an" normal li)er f#nction tests. A trial of a gl#ten!free "iet is attempte". There is no change in symptoms o)er the s#$se*#ent & 'eeks% an" she has an a""itional 4!po#n" 'eight loss. Stool c#lt#res for enteric pathogens an" o)a an" parasites are negati)e on three occasions. =hich of the follo'ing is the most likely "iagnosisa62eliac spr#e $6Enterohemorrhagic Escherichia coli c6Enteroto.igenic Escherichia coli "6Pancreatic ins#fficiency e6 Tropical spr#e :ormal La$s 2.A 2(!year!ol" office manager 'ith a history of generaliDe" an.iety has ha" m#ltiple recent )isits to the physician for a se)eral month history of fatig#e. E)al#ation th#s far has re)eale" a normal physical e.amination as 'ell as screening la$oratories that are 'ithin normal limits. The ser#m thyroi"! stim#lating hormone le)el is 2.& HU7mL. +n*#iring a$o#t specific symptoms of "epression on the last )isit yiel"e" the "iagnosis of "epression% an" treatment options 'ere "isc#sse" 'ith her. >n this )isit she e.presses fr#stration that a me"ical etiology of her fatig#e has not $een i"entifie" an" she "eman"s to see the clinic notes from her last se)eral )isits. =hich of the follo'ing 'o#l" $e the most appropriate response the physician co#l" makea6Agree to sho' her the recor"s only after she #n"ergoes treatment for her "epression $6Attempt to arrange to re)ie' her clinic recor"s 'ith her as soon as possi$le c6@isc#ss her case 'ith a la'yer prior to sho'ing her the recor"s "6E.plain to her that physician!patient confi"entiality prohi$its her from seeing her me"ical recor" e6+mme"iately make copies of her me"ical recor" for her to re)ie' at her con)enience :ormal La$s &.A 24!year!ol" 'oman presents to the emergency "epartment complaining of shortness of $reath an" 'heeDing. She reports a history

of mil" episo"ic asthma $#t has not ha" an attack in more than & years. The c#rrent symptoms $egan appro.imately ho#rs ago an" ha)e $ecome gra"#ally more se)ere. She is on no me"ications. >n physical e.amination% she is tachypneic 'ith respirations of 217min. /er l#ngs ha)e lo#" e.piratory 'heeDing $ilaterally. The e.piratory phase is mil"ly prolonge"% an" air mo)ement is fair. A peak e.piratory flo' rate is 124 L7min. /er o.ygen sat#ration at room air is 84B. =hich of the follo'ing is the most appropriate ne.t step in managementa6 Al$#terol ne$#liDer $6Atropine ne$#liDer c6+? hy"rocortisone "6>ral pre"nisone e6S#$c#taneo#s epinephrine :ormal La$s 4.A ( !year!ol" man has 'i"ely "isseminate" prostate cancer. /e is $eing follo'e" at home $eca#se he has ref#se" any f#rther specific anti! cancer therapy. /is hematocrit "rops from a pre)io#sly sta$le 42B to 2 B o)er a t'o month perio". Ae)ie' of the peripheral $loo" smear "emonstrates normochromia% anisocytosis% poikilocytosis% an" n#cleate" re" $loo" cells. Aare immat#re myeloi" cells are also seen in the smear. This patient,s anemia is $est classifie" as 'hich of the follo'inga6 A#toimm#ne anemia $6/ere"itary hemoglo$in synthesis a$normality c6Myelophthisic anemia "6:#tritional "eficiency anemia e6Ae" cell mem$rane "efect anemia :ormal La$s .A 24!year!ol" college st#"ent comes to the emergency "epartment $eca#se of the s#""en onset of e.cr#ciating left!si"e" flank pain ra"iating to the la$ia. She states that the flank pain is associate" 'ith tenesm#s% rectal pain% na#sea% an" )omiting. She ha" 2 episo"es of hemat#ria. She has no chronic me"ical con"itions an" has ne)er ha" any similar episo"es in the past. She has recently starte" taking

0mega"oses0 of )itamin A% )itamin 2% )itamin E% pyri"o.ine% an" methylpheni"ate 3her roommate,s6 to 0help 'ith final e.aminations0. /er temperat#re is &(.1 2 3144 56% $loo" press#re is 1&4714 mm /g% p#lse is 147min% an" respirations are 2<7min. E.amination sho's se)ere left costo)erte$ral angle ten"erness. An .!ray film of the ki"neys% #reter% an" $la""er 3MUB6 sho's a ra"io!opacity in the left #reter. =hich of the follo'ing is the most likely e.planation for these fin"ingsa6?itamin A e.cess $6?itamin 2 e.cess c6?itamin E e.cess "6Methylpheni"ate e6Pyri"o.ine e.cess :ormal La$s <. An 14!year!ol" 'oman e.periences the s#""en onset of lo'er e.tremity 'eakness an" collapse on getting o#t of $e". She has $een pre)io#sly healthy an" has not taken any me"ications. She has no history of $ack pain. >n e.amination% she is alert an" oriente" to time% place an" "ate. /er #pper e.tremity sensation an" strength are intact. /er legs are 'eak $ilaterally% 'ith loss of pain an" temperat#re sensation an" arefle.ia. /er $la""er is "isten"e". =hich of the follo'ing is the most likely "iagnosisa6Anterior cere$ral artery occl#sion $6Anterior spinal artery occl#sion c62a#"a e*#ina syn"rome "6C#illain!BarrT syn"rome e6Thoracic spinal cor" compression :ormal La$s (. A &(!year!ol" man cons#lts a physician a$o#t m#ltiple pap#les an" pla*#es on his $o"y. The lesions are 'i"ely "isseminate" across the $o"y% 'ith increase" n#m$ers on the tr#nk an" arms. M#cosal s#rfaces are also in)ol)e". The lesions are $arely ele)ate" an" )ary in color from p#rple to pink to $ro'n. Some of the lesions are irritate" an" ha)e $le" prof#sely. The lesions ha)e $een "e)eloping o)er the pre)io#s year. The patient thinks he may ha)e also ha" $loo" in his #rine an" in his stool.

This patient,s con"ition is most likely to $e relate" to 'hich of the follo'ing con"itionsa6 A+@S $62ontact allergy c6/emophilia A "6Melanoma e6 Streptococcal infection :ormal La$s 1.A <2!year!ol" man 'ith hypertension an" angina has se)ere retrosternal chest pain ra"iating to the left sho#l"er on a'akening% accompanie" $y "iaphoresis an" na#sea. The pain is not relie)e" $y s#$ling#al nitroglycerin. An E2C is consistent 'ith an e)ol)ing anterior 'all myocar"ial infarction. T'o "ays later% the patient "e)elops se)ere shortness of $reath. /is temperat#re is &<.( 2 381 56% $loo" press#re is 1447<4 mm /g% p#lse is 14 7min% an" respirations are 287min. Bilateral 'et crackles are note"% an" a holosystolic m#rm#r is hear" at the ape. ra"iating to the left a.illa. :o m#rm#r 'as hear" "#ring an e.amination 4 months earlier. =hich of the follo'ing complications has most likely occ#rre"a6 Aortic "issection $62ar"iac r#pt#re c6Papillary m#scle r#pt#re "6Septal perforation e6 ?entric#lar ane#rysm formation :ormal La$s 8.An 11!year!ol" college st#"ent comes to the st#"ent health clinic $eca#se of a 2!'eek history of fe)er% chills% an" a sore throat. /is temperat#re is &1.& 2 3141 56% $loo" press#re is 1147(4% p#lse is (47min% an" respirations are 117min. Physical e.amination sho's marke" pharyngeal hyperemia% tonsillar e.#"ates% cer)ical lympha"enopathy% an" splenomegaly. Ampicillin therapy is $eg#n an" the patient is sent $ack to his "ormitory. T'o "ays later he ret#rns $eca#se of a mac#lopap#lar rash. =hich of the follo'ing is most likely to confirm the "iagnosisa62hest .!ray $6/eterophile anti$o"y test c6Lateral .!ray film of the neck "6Aapi" Streptococc#s test

e6?aricella )ir#s anti$o"y imm#nofl#orescence :ormal La$s 14. A 4!year!ol" man presents to his physician 'ith 2 "ays of co#gh an" fe)er. /is past me"ical history is significant for mil" asthma an" peripheral )asc#lar "isease 3P?@6. /e takes al$#terol metere" "ose inhalers as nee"e" an" has ne)er $een int#$ate" for his asthma. /is P?@ manifests as calf cla#"ication an" has $een sta$le o)er the past fe' years. /is other me"ications incl#"e atenolol% lisinopril% an" *#inine. /e has no "r#g allergies. /e "enies rigors% chills% na#sea% )omiting% or any ple#ritic chest pain. >n physical e.amination% he appears 'ell 'ith an occasional co#gh. /is temperat#re is &1.4 2 3144.4 56% $loo" press#re is 1 4714 mm /g% p#lse is 847min an" reg#lar% an" respirations are 227min an" some'hat la$ore". /is l#ngs ha)e $i$asilar crackles an" a *#estiona$le area of increase" "#llness near the right $ase. The rest of the e.amination is #nremarka$le. =hich of the follo'ing is the most appropriate inter)ention at this timea6:o inter)ention is in"icate" $6Prescri$e penicillin an" sen" the patient home c6>$tain a chest ra"iograph "6>$tain an arterial $loo" gas e6Aefer the patient to the hospital for a"mission :ormal La$s 11. A 4 !year!ol" 'oman presents to clinic 'ith a 4!month history of hea"aches an" changes in her )ision. She has $een pre)io#sly healthy an" has not $een on any me"ications. >n e.amination% she is fo#n" to ha)e a small fiel" "efect in $oth eyes. The "iagnosis of a pit#itary microa"enoma is entertaine". =hich of the follo'ing tests 'o#l" $e the most sensiti)e in "iagnosis of s#ch an entitya62omp#teriDe" tomography 32T6 scan $6+ns#lin!tolerance test c6Magnetic resonance imaging 3MA+6 "6Ser#m prolactin meas#rement e6?is#al fiel" e.amination :ormal La$s

12. A 4&!year!ol" man reports that he has ha" a 8!po#n" 'eight loss o)er the past 8 months. The symptoms ha)e $een accompanie" $y "iffic#lty s'allo'ing $oth soli"s an" li*#i"s "#ring that time. /e has 'oken on se)eral occasions at appro.imately 4 AM an" reg#rgitate" partially "igeste" "inner contents. An #pper gastrointestinal series is performe" an" re)eals a 'i"ely "ilate" esophag#s 'ith a smoothly tapering "istal esophag#s. There appears to $e partially "igeste" foo" present in the esophag#s. =hich of the follo'ing 'o#l" most likely $e seen on esophageal manometrya6/igh resting LES press#re $6/igh resting #pper esophageal sphincter press#re c6+ncrease" peristalsis in mi" esophag#s "6Lo' resting LES press#re e6Lo' resting #pper esophageal sphincter press#re :ormal La$s 1&.A 1(!year!ol" $oy is $ro#ght to the hospital $y parame"ics $eca#se of possi$le antifreeDe ingestion. The father reports that his son has recently $een "rinking alcohol e.cessi)ely. >)er the past fe' "ays% the son ha" seeme" )ery "epresse" an" ha" $een ca#ght sneaking alcohol into the ho#se on t'o occasions. The father trie" confining the son to his $e"room% $#t later fo#n" him face "o'n on the garage floor near a $ottle of antifreeDe. The patient,s girlfrien" confirme" that the patient "i" cons#me some li*#i" that she $elie)e" to $e antifreeDe. To rapi"ly assess this possi$ility% 'hich of the follo'ing is the most appropriate initial step in managementa6 E.amine the patient,s $reath for a fr#ity o"or $6E)al#ate for hyperemia of his optic "iscs c6>$tain an E2C "6E)al#ate his #rine #n"er a =oo",s lamp e6 Ae*#est a ser#m osmolarity meas#rement :ormal La$s 14.An A+@S patient #n"ergoes en"oscopy for chronic s#$sternal pain that is e.acer$ate" 'hen he s'allo's. The st#"ies "emonstrate inflammation an" s#perficial #lceration of the "istal esophag#s. Biopsies taken from the area sho' inflammation an" a fe' cells% partic#larly in the en"otheli#m of small $loo" )essels% 'ith marke"ly

enlarge"% sm#"gy% eosinophilic n#clei. =hich of the follo'ing is the most likely ca#se of the patient,s esophagitisa6 Aci" refl#. $62an"i"a c62ytomegalo)ir#s "6/erpes simple. e6 /erpes Doster :ormal La$s 1 .A 2 !year!ol" man has a se)eral year history of occasionally noting pink "iscoloration in his morning #rine. /e presents to the emergency room three "ays after "e)eloping a se)ere col" 'ith symptoms of a$"ominal an" l#m$ar pain. A #rine sample appears grossly $loo"y. :o cells are seen 'hen a #rine smear is ma"e% $#t hemosi"erin crystals are note". The hematocrit is 2 B% an" the o)erlying ser#m is "ark pink in color. The re" cell morphology on peripheral smear is normal. =hich of the follo'ing tests is most likely to confirm the "iagnosisa6 /am,s test $6/emoglo$in electrophoresis c6Phila"elphia chromosome "6Sickle cell test e6 Tartrate resistant aci" phosphatase :ormal La$s 1<. A 2(!year!ol" 2hinese male presents 'ith gross hemat#ria. The patient has a long history of rec#rrent hemat#ria% ma"e 'orse $y pharyngitis. /e "enies arthralgia% fe)er% chills or skin rash. There is no family history of ki"ney "isease or hemat#ria. /is temperat#re is &( 2 381.< 56 an" his $loo" press#re is 1247(4 mm /g. /is throat an" l#ngs are clear% his a$"omen is soft an" nonten"er. There is no e"ema% rash or arthritis. /is $loo" #rea nitrogen is 14 mg7"L% an" his creatinine is 4.1 mg7"L. A #rine specimen sho's 1 !24 "ysmorphic re" $loo" cells7 hpf% there is E2 protein an" 1 AB2 cast. =hich of the follo'ing is the most likely "iagnosisa6Alport syn"rome $6Coo"past#re syn"rome c6+gA nephropathy "6:il "isease

e6=egener gran#lomatosis :ormal La$s 1(.A pre)io#sly healthy &&!year!ol" la'yer is $ro#ght to the emergency room $eca#se of the ac#te onset of hea"ache an" conf#sion. @#ring e.amination% he "e)elops gran" mal seiD#res. Promptly treate"% he is a"mitte" to the ne#rology 'ar"% 'here a 2T scan of the hea" re)eals $ilateral hemorrhage in the temporal lo$es. Papille"ema is a$sent% an" 2S5 analysis re)eal mil" pleocytosis 3mostly lymphocytes6 'itho#t changes in gl#cose an" protein concentration. =hich of the follo'ing is the most likely "iagnosisa6Ar$o)ir#s encephalitis $6Brain to.oplasmosis c6Echo)ir#s encephalitis "6/erpetic encephalitis e6Metastatic melanoma :ormal La$s 11. A 44!year!ol" 'oman cons#lts a physician a$o#t lesions on her neck that she fin"s cosmetically #nattracti)e. >n e.amination of the neck% m#ltiple lesions that seem to $e hanging off the skin are seen. Each lesion is small% soft% an" pe"#nc#late". The largest lesion is a$o#t 4 mm in "iameter. The color of "ifferent lesions )aries from flesh colore" to slightly hyperpigmente". =hich of the follo'ing is the most likely "iagnosisa6 Acrochor"ons $6 Lentigos c6Lipomas "6Se$orrheic keratoses e6Spi"er angiomas :ormal La$s 18.A (2!year!ol" 'oman 'ith no prior me"ical history presents in the emergency "epartment 'ith a &!ho#r episo"e of cr#shing s#$sternal chest pain. The pain ra"iates to her arm an" neck. An E2C re)eals ST segment ele)ation in lea"s ++% +++ an" a?5. The patient has no o$)io#s

contrain"ication to anticoag#lation. =hich of the follo'ing is the most optimal treatment at this timea6A)oi"ance of throm$olytic treatment gi)en the patient,s age $6A"ministration of +? fl#i"s c6A"ministration of aspirin an" heparin only "6A"ministration of throm$olytic therapy% heparin% an" aspirin e62ar"iac s#rgery to $ypass the occl#"e" )essel :ormal La$s 24. A 2 !year!ol" 'oman comes to the physician $eca#se of a facial rash% fatiga$ility% 9oint an" m#scle pains% an" temperat#res #p to &1. 2 3141 56 for t'o 'eeks. /er temperat#re is &( 2 381.< 56% $loo" press#re is 12714 mm /g% p#lse is 147min% an" respirations are 247min. She "oes not take any me"ications. Physical e.amination re)eals a $ilateral malar rash% ten"erness to palpation of the knees an" 'rists% an" a ple#ritic r#$$ing so#n" on chest a#sc#ltation. La$oratory in)estigations sho'; /emoglo$in............................1. g7"L Le#kocyte co#nt.....................&%1447 mm& Platelets.................................114%4447 mm& Urinalysis...............................negati)e for gl#cose an" protein Antin#clear anti$o"y titer........ele)ate" Antiphospholipi" anti$o"ies......positi)e =hich of the follo'ing is the most appropriate ne.t step in "iagnosisa6Assay for anticentromere anti$o"ies $6Assay for anti!"o#$le stran"e" @:A an" anti!Smith anti$o"ies c6Assay for anti!ne#trophil cytoplasmic anti$o"ies 3A:2As6 "6Assay for rhe#matoi" factor e6F!ray st#"ies of affecte" 9oints f6Skin $iopsy :ormal La$s 21. A (<!year!ol" man presents to the clinic for his semiann#al e.amination. The patient is 'ell kno'n to the clinic an" has $een ha)ing semiann#al e.aminations for the past & years to follo' #p his chronic o$str#cti)e p#lmonary "isease 32>P@6. The patient is a long! time smoker% 'ith a &44E pack year history. /e is stat#s post a right

#pper 'e"ge resection 2 years ago for a"enocarcinoma% an" at that time ha" se)ere o$str#cti)e "isease. A recent chest 2T sho'e" apical $#llae an" se)ere emphysemato#s changes. The patient has mo"erate "yspnea on e.ertion an" often has shortness of $reath 'ith minimal acti)ity. >n this )isit% the patient relates that he is e)en more short of $reath at rest an" is almost #na$le to perform any physical acti)ity as a res#lt. =hich of the follo'ing 'o#l" most strongly s#ggest the nee" to initiate home o.ygen therapya6E.ercise!in"#ce" o.ygen "esat#rations to less than 82B $6Aesting arterial Pa>2 sho'ing an al)eolar arterial gra"ient of more than 12 mm /g c6Aesting arterial Pa>2 of less than mm /g "6Aesting Pa>2 of greater than 44 mm /g e6Aoom air o.ygen sat#ration of less than 82B :ormal La$s 22.A 41!year!ol" man comes to the physician $eca#se of the recent onset of o$esity an" easy $r#isa$ility. /is $loo" press#re is 1< 78 mm /g. @ermal striae are fo#n" on physical e.amination. A "ipstick e.amination of #rine re)eals glycos#ria. =hich of the follo'ing is the most appropriate ne.t step in "iagnosisa6 Baseline plasma A2T/ meas#rement $62T scans of the chest an" a$"omen c6@e.amethasone s#ppression test "6Meas#rement of mi"night ser#m cortisol le)el e6 Meas#rement of 24!ho#r #rine cortisol an" creatinine f6MA+ scans of the hea" :ormal La$s 2&.A <4!year!ol" man is referre" for ne#rologic cons#ltation $eca#se of rapi"ly progressi)e memory loss an" 9erking mo)ements of the #pper an" lo'er e.tremities. :e#rologic e.amination confirms the presence of se)ere cogniti)e "eficits an" memory "ysf#nction. An electroencephalogram sho's generaliDe" perio"ic sharp 'a)es. :e#roimaging st#"ies sho' mo"erately a")ance" cere$ral atrophy. A cortical $iopsy is o$taine"% 'hich sho's "iff#se )ac#olar changes of the gray matter 'ith reacti)e astrocytosis $#t no significant inflammatory

infiltration. 2S5 analysis is #nremarka$le. =hich of the follo'ing is the most likely "iagnosisa6AlDheimer "ementia $62re#tDfel"t!Rako$ "isease c6Clioma "6Pse#"ot#mor cere$ri e6 ?iral encephalitis :ormal La$s 24.A (!year!ol" man comes to the physician $eca#se of 0erectile pro$lems0. /e says that he has not $een a$le to ha)e an erection in months. /e has seen so many tele)ision commercials lately that he e.pects a c#re in the form of a 0little $l#e pill0. /e is marrie"% has & chil"ren% 'orks as a tra"er on the stock e.change% an" is an a)i" cyclist. /e takes no me"ications% "oes not "rink alcohol% an" has ha" no serio#s me"ical con"itions. Physical e.amination is normal. =hich of the follo'ing *#estions is most likely to help "etermine the ca#se of this patient,s con"itiona60/o' often "o yo# ri"e yo#r $icycle-0 $60Are yo# se.#ally attracte" to yo#r 'ife-0 c60Are yo# feeling #n#s#ally an.io#s lately-0 "60@o yo# ha)e noct#rnal or early morning erections-0 e60@o yo# lo)e yo#r 'ife-0 :ormal La$s 2 .An 12!year!ol" 'oman is $ro#ght to the emergency "epartment from the local n#rsing home $eca#se of complaints of se)ere constipation. She has not mo)e" her $o'els in the past "ays an" to"ay $egan complaining of lo'er a$"ominal "iscomfort an" "istention. She takes hy"rochlorothiaDi"e an" acetaminophen 'ith co"eine for se)ere arthritic pain in $oth hips. >)er the past 'eek she has $een $e"ri""en $eca#se of the se)ere pain. She is afe$rile. An a$"ominal e.amination sho's mi"! an" lo'er!a$"ominal "istention 'ith mil" ten"erness. Bo'el so#n"s are normoacti)e. A rectal e.amination "emonstrates that the rectal )a#lt is fille" 'ith har" stool. =hich of the follo'ing is the most appropriate ne.t step in managementa6Bari#m enema $62olonoscopy c65le.i$le sigmoi"oscopy

"6Man#al "isimpaction e6Passage of a nasogastric t#$e :ormal La$s 2<. A 1!year!ol" 'oman is hospitaliDe" $eca#se of fatig#e% night s'eats% lo'!gra"e fe)er% an" splenomegaly. La$oratory st#"ies come $ack sho'ing ac#te myelocytic le#kemia 3AML6 'ith A#er ro"s. The physician taking care of this patient is picking #p l#nch in the cafeteria an" sees her eating 'ith her relati)es. /e 'alks $y her ta$le an" she asks 'hat the res#lts of the st#"ies sho'. =hich of the follo'ing is the most appropriate response on the part of the physiciana6Aemain stan"ing an" tell the patient that there are A#er ro"s in her cells $6Sit "o'n an" tell the patient that there are A#er ro"s in her cells c6Sit "o'n an" tell the patient that she has ac#te myelocytic le#kemia "6Tell her that the res#lts are $ack% $#t it 'o#l" $e $etter to go some'here *#iet an" pri)ate e6 Tell her that the she has ac#te myeloi" le#kemia% $#t that she sho#l" ask *#estions in pri)ate :ormal La$s 2(.A 21!year!ol" man 'ith chronic nephritis comes to the clinic for a check#p. /e has a creatine of <.( mg7"L% #rea nitrogen of (4 mg7"L% an" a hemoglo$in of 1.4 g7"L. /is platelet co#nt is 244%4447mm&. /e feels 'ell e.cept for some mil" fatig#e 'ith e.ercise. /is $loo" press#re is 144714 mm /g% his l#ngs are clear. /is heart is reg#lar 'ith a soft systolic m#rm#r. /is a$"omen is soft. /e is g#aiac negati)e for occ#lt $loo". /is ferritin le)el is 244 ng7mL an" is iron sat#ration is 2 B. /is mean corp#sc#lar )ol#me is 1 Hm&=hich of the follo'ing 'o#l" $e the most appropriate treatment for his anemiaa6 :o treatment% follo' )al#es $6Erythropoietin c65olic aci" "6Trial of iron therapy e6Bloo" transf#sion :ormal La$s

21. A < !year!ol" 'oman presents 'ith slo'ly progressi)e "ementia an" )is#al hall#cinations. Mental stat#s e.amination re)eals se)ere "eficits in attention% )is#ospatial skills% an" )er$al fl#ency% 'hile short!term memory is only mil"ly impaire". Physical e.amination sho's rigi"ity of arms an" Parkinson!like e.trapyrami"al signs. An MA+ of the hea" "emonstrates mil" "iff#se cortical atrophy. =hich of the follo'ing is the most likely "iagnosisa6AlDheimer!type "ementia $62re#tDfel"t!Raco$ "isease c6@ementia 'ith Le'y $o"ies "6@ementia associate" 'ith motor ne#ron "isease e6Pick "ementia :ormal La$s 28.A & !year!ol" 'oman presents to a clinician $eca#se of an itchy a patch of skin near her knee. < months pre)io#sly% she ha" ha" se)eral mos*#ito $ites in the relati)ely small area. The mos*#ito $ites e)ent#ally resol)e"% $#t the area ha" contin#e" to $e itchy% an" she ha" contin#e" to scratch it perio"ically. The 'ell!"efine" <!cm "iameter patch of skin is no' "ry% scaling% hyperpigmente"% an" thickene". A ring of "iscrete $ro'nish pap#les can $e seen at the periphery of the lesion. =hich of the follo'ing is the most likely "iagnosisa6Lichen simple. chronic#s $6Pompholy. c6Psoriasis "6Se$orrheic "ermatitis e6 Stasis "ermatitis :ormal La$s &4.A 2(!year!ol" man presents 'ith increasing "yspnea on e.ertion. >)er the past & 'eeks% he has note" increasing shortness of $reath 'hile 'alking to the $#s stop% three $locks from his home. /e has also fo#n" it "iffic#lt to sleep comforta$ly 'hile lying flat an" no' sleeps on three pillo's. /e has a'oken se)eral times in the past month 'ith shortness of $reath. /e "enies any history of chest pain. /e "oes not smoke% an" his cholesterol tests 'ere normal 1 year earlier. >n physical e.amination% he appears comforta$le at rest. /is $loo" press#re is 1 17 < mm /g% p#lse is 1<7min an" reg#lar% an" respirations are

1<7min. /e has 9#g#lo)eno#s "istention 'hile $eing e.amine" at &4 "egrees. /is l#ngs ha)e $i$asilar rales. >n car"iac e.amination% there is a reg#lar S1 an" S2% 'ith a $lo'ing "iastolic m#rm#r hear" lo#"est at the left sternal $or"er. A +7?+ systolic m#rm#r is hear" thro#gho#t the precor"i#m. An e.tra heart so#n" imme"iately follo'ing the S2 is hear". There is mil" lo'er e.tremity pe"al e"ema $ilaterally. =hich of the follo'ing is the most likely ca#se of his symptomsa6Aortic ins#fficiency $6Aortic stenosis c6Mitral reg#rgitation "6Mitral stenosis e6Tric#spi" reg#rgitation :ormal La$s &1. A 48!year!ol" man 'ith a history of t'o episo"es of painf#l s'elling in his right toe presents 'ith his thir" episo"e in the past 2 months. Arthrocentesis re)eals m#ltiple polymorphon#clear cells an" se)eral negati)ely $irefringent nee"le!shape" crystals. /e has #se" acetaminophen at home 'itho#t relief. =hich of the follo'ing is the most appropriate ne.t step in therapya6 A'ait c#lt#re res#lts of the 9oint aspirate $6Start high!"ose oral i$#profen c6Start high!"ose oral pre"nisone "6+nitiate treatment 'ith +? ceftria.one e6 >r"er a ra"ion#clear $one scan :ormal La$s &2.A <2!year!ol" man 'ith a !pack!year smoking history presents 'ith a 'orsening of his chronic shortness of $reath. /e has notice" a change in his chronic "aily co#gh. /e has also ha" se)eral episo"es of $loo" tinge" sp#t#m o)er the past fe' 'eeks. /is 'ife states that o)er the past 2 'eeks he has seeme" more hoarse than #s#al. >n physical e.amination% he has "ense rhonchi in the right posterior mi"!l#ng fiel". There are also soft scattere" rhonchi in $oth l#ng fiel"s 'ith a prolonge" e.piratory phase. A chest .!ray film re)eals a (!cm% irreg#larly shape" mass in the right mi""le lo$e 'ith associate" lo$ar consoli"ation. =hich of the follo'ing 'o#l" most s#ggest that the patient has s#rgically inc#ra$le l#ng cancer-

a6The change in his pattern of co#gh $6+ncreasing hoarseness c6/emoptysis "6A posto$str#cti)e pne#monia e6=orsening "yspnea :ormal La$s &&. A < !year!ol" man "e)elops tr#ncal o$esity an" a $#ffalo h#mp. /e is not on any me"ications. Early morning cortisol is & micrograms7"L 3normal to 2 micrograms7"L6. Late e)ening cortisol is 24 micrograms7"L 3normal Q 14 micrograms7"L6. 2ortisol pro"#ction "oes not s#ppress 'ith either lo' or high "e.amethasone. 2T scan of the hea" "emonstrates a normal sella t#rcica an" pit#itary glan". An ectopic so#rce of A2T/ pro"#ction is s#specte". A malignancy of 'hich of the follo'ing organs is most likely to $e the so#rce of the A2T/a62olon $6L#ng c6Pancreas "6Stomach e6 Testis :ormal La$s &4.A patient is seen $y a specialist $eca#se of chronic% intracta$le% sin#sitis. The "ecision is ma"e to treat the patient s#rgically% 'ith e)ac#ation of sin#s contents an" "ilation of the sin#s ostia. The material remo)e" is sent ro#tinely for pathologic e.amination. An #ne.pecte" fin"ing is the presence of f#ngi 'ith $roa"% nonseptate% irreg#larly shape" hyphae. S#$se*#ent re)ie' of the patient,s chart re)eals a long history of poorly controlle" "ia$etes mellit#s. =hich of the follo'ing is the most likely ca#sati)e organisma6Aspergill#s $6Blastomyces c62an"i"a "6AhiDop#s e6 Sporothri. :ormal La$s

& . A 24!year!ol" man is $ro#ght to the emergency room & ho#rs after ingesting appro.imately fifty &2 !mg acetaminophen ta$lets in a s#ici"e attempt. /e has na#sea an" )omiting% $#t no other a$normalities on physical e.amination. /is temperat#re is &( 2 381.< 56% $loo" press#re is 1& 714 mm /g% p#lse is 1447min% an" respiratory rate is 247min. /is ser#m acetaminophen concentration is 'ithin the range of pro$a$le to.icity% $#t ser#m transaminases an" other hepatic markers are normal. =hich of the follo'ing is the most appropriate ne.t step in managementa6Acti)ate" charcoal treatment $6Acetylcysteine treatment c6Penicillamine treatment "6Acti)ate" charcoal an" acetylcysteine treatment e6Castric la)age :ormal La$s &<.A 44!year!ol" 'oman 'ith a history of rec#rrent $iliary colic presents 'ith 11 ho#rs of )ery se)ere right #pper *#a"rant pain% fe)er% an" 9a#n"ice. An a$"ominal #ltraso#n" re)eals a marke"ly "ilate" common $ile "#ct. M#ltiple gallstones are seen in the gall$la""er. =hich of the follo'ing 'o#l" $est "etermine 'hether there is cystic "#ct o$str#ctiona6A$"ominal 2T scan $6A$"ominal MA+ c6A$"ominal #ltraso#n" 'ith @oppler flo' st#"ies "6En"oscopic retrogra"e cholangiopancreatograph 3EA2P6 e6 /+@A scan :ormal La$s &(.A patient 'ho has $een in hemo"ialysis for 11 years has persistent tingling of his left han". /e has $een "iagnose" 'ith carpal t#nnel syn"rome. /e is 'ell "ialyDe" an" takes his me"ications as "irecte". /is $loo" press#re is 124714 mm /g. E.amination re)eals ten"erness o)er the left me"ian ner)e at a left #pper arm A? graft. Thyroi" stim#lating hormone is 'ithin normal limits.=hich of the follo'ing is the most likely ca#se of his carpal t#nnel syn"romea6@ialysis!associate" amyloi" $6>cc#lt hypothyroi"ism

c6Press#re on the me"ian ner)e from the A? graft "6Aec#rrent tra#ma at "ialysis e6 Aepetiti)e motion in9#ry :ormal La$s &1.A (4!year!ol" 'oman is $ro#ght to the physician $y her "a#ghter $eca#se of personality change an" forgetf#lness for the past & months. /er last me"ical check!#p 'as 1 year earlier% 'hen she appeare" in goo" spirits an" e.cellent health. Si. months ago% her h#s$an" "ie". @#ring the e.amination% the patient ans'ers the physician,s *#estions 'ith 0Ues%0 0:o%0 or 0+ "on,t kno'%0 an" appears apathetic an" in"ifferent. =hen aske" *#estions testing short!term memory% she often replies 0+ "on,t kno'.0 Physical e.amination is #nremarka$le. Bloo" st#"ies are normal% an" TS/ is 'ithin normal range. The physician arranges for an MA+ e.amination of the $rain. The T1!'eighte" image is sho'n $elo'. =hich of the follo'ing is the most likely "iagnosisa6AlDheimer "ementia $6Brain t#mor c6:ormal press#re hy"rocephal#s "6Pick "ementia e6 Pse#"o"ementia :ormal La$s &8. A 1!year!ol" "entist complains of chest pain 'ith e.ertion. /e has $een in e.cellent health his 'hole life an" "oes not smoke or "rink. +n a""ition% he 9ogs 2 miles "aily. Both of his parents li)e" into their 84,s an" "ie" of 0nat#ral ca#ses%0 an" his three si$lings are ali)e an" 'ell. >n physical e.amination% his $loo" press#re is 1<4784 mm /g% p#lse is (47min% an" respirations are 1<7min. A hea" an" neck e.amination is normal% as is a l#ng e.amination. A car"iac e.amination re)eals a reg#lar S1 an" S2% an" an S4 is hear". There is a systolic crescen"o! "ecrescen"o m#rm#r at the right #pper sternal $or"er ra"iating to the caroti" arteries. An E2C re)eals left )entric#lar hypertrophy. =hich of the follo'ing "iagnostic tests 'o#l" confirm the "iagnosis in this patient-

a6E.ercise stress test $6Echocar"iogram c6Thalli#m stress test "62ar"iac catheteriDation e6 P#lmonary artery capillary 'e"ge press#re meas#rement :ormal La$s 44.A 4 !year!ol" 'oman comes to the physician $eca#se of progressi)e 'eakness for se)eral months. At first% she $ecame a'are of increasing "iffic#lty in clim$ing stairs% then lifting o$9ects a$o)e hea" le)el. +n the past 'eek% the m#scles of the sho#l"er gir"le ha)e $ecome sore. She has ha" fe)er #p to &1. 2 3141.& 56. >n e.amination% pro.imal m#scles of the #pper an" lo'er e.tremities are some'hat ten"er to palpation $#t not atrophic. There is perior$ital e"ema associate" 'ith a p#rple "iscoloration of the #pper eyeli"s. Scaly erythemato#s patches are seen on the "orsal aspect of han"s an" fingers. La$oratory st#"ies sho'; /emoglo$in.............................................1&.2 g7"L Le#kocyte co#nt......................................8&447mm& 2reatine kinase.......................................2 4 U7L Erythrocyte se"imentation rate 3ESA6......22 mm7hr Antin#clear anti$o"y 3A:A6 titer............Positi)e at 1;1214 Ahe#matoi" factor..................................Positi)e at 1; 12 A $iopsy of the "eltoi" m#scle sho's atrophy of m#scle fi$ers an" interstitial lymphocytic infiltration. =hich of the follo'ing is the most appropriate treatment at this timea6 ADathioprine $6Methotre.ate c6:onsteroi"al anti!inflammatory "r#gs 3:SA+@s6 "6Plasma e.change e6Pre"nisone :ormal La$s 41.A 22!year!ol" college senior presents to the #ni)ersity health clinic complaining of shortness of $reath an" co#gh for the past 2 'eeks. /e has 9#st transferre" to this #ni)ersity in 2hicago from his hometo'n in AriDona% 'here he ha" st#"ie" for the past & years. Each e)ening% on ret#rning from classes to his "ormitory room% he "e)elops a "ry nonpro"#cti)e co#gh that typically lasts for se)eral ho#rs $efore he falls

asleep. /e is a'akene" from sleep 'ith shortness of $reath% co#gh% an" 'heeDing. /e "enies cigarette smoking% $#t "rinks se)eral $eers on 'eeken"s an" has occasionally smoke" mari9#ana. Symptoms often occ#r on 'eeken"s% 'hen he spen"s time st#"ying in his "ormitory room. >n physical e.amination% he "enies any respiratory symptoms% an" his l#ng e.amination is normal. =hich of the follo'ing is the most likely mechanism for his symptomsa6 Bronchospasm $6Laryngeal spasm c6Ple#ral eff#sion "6P#lmonary congestion e6?entilation!perf#sion mismatch :ormal La$s 42. A 11!year!ol" man presents for a health maintenance e.amination. /e has a family history of Tangier "isease% an" a n#m$er of a"#lts in his family ha)e either hepatosplenomegaly% rec#rrent polyne#ropathy% or $oth. =hich of the follo'ing 'o#l" $e the strongest fin"ing on physical e.amination to s#ggest the presence of this "iseasea6Angiokeratomas $6Crey!$ro'n pigmentation of the forehea"% han"s% an" pre!ti$ial region c6+rreg#lar $lack "eposits of cl#mpe" pigment in the peripheral retina "6>range!yello' tonsillar hyperplasia e6Ping#ec#lae :ormal La$s 4&.A 28!year!ol" 'oman is seen in the emergency "epartment for a $roken ankle. She "escri$es a fall "o'n her stairs at home earlier that "ay. She "enies any alcohol or "r#g #se at the time. She has no significant past me"ical history% $#t she has $een a"mitte" to the hospital many times for pre)io#s fract#res an" she has $een seen in the emergency "epartment on m#ltiple occasions for )ario#s lacerations an" minor in9#ries. The patient e.plains that all these )isits are "#e to her 0acti)e lifestyle0 an" her fre*#ent home repair pro9ects. The patient has $een marrie"% she claims happily% for 2 years. She is employe" as a manager at a local resta#rantG her h#s$an" is #nemploye" an" on "isa$ility pay for $ack pro$lems. The patient is

a"mitte" to the hospital an" #n"ergoes s#ccessf#l open re"#ction an" internal fi.ation for her ankle. She is a$le to gi)e a more "etaile" history a$o#t her social sit#ation. She "escri$es feeling )ery "epresse" lately an" ha)ing ha" an arg#ment 'ith her h#s$an" the morning of her in9#ry. @omestic a$#se is s#specte". =hich of the follo'ing is the most appropriate initial step in a""ressing this concerna6A""ress the concerns 'ith her h#s$an" "irectly $6Ask her to offer more "etails a$o#t the nat#re of her relationship 'ith her h#s$an" c6E.plain to her that her marriage is o$)io#sly not ha)ing a positi)e impact on her life "6Aefer her case to the "epartment of social ser)ices as o$ligate" e6Aefer her to a psychiatrist 'ho specialiDes in "omestic a$#se :ormal La$s 44. A (!year!ol" 'oman 'ith kno'n congesti)e heart fail#re is a"mitte" to the hospital 'ith increase" shortness of $reath. The patient reports that% o)er the past fe' "ays% she has ha" increasing "iffic#lty $reathing% $oth at rest an" 'ith e.ertion. She is on a strict lo'!so"i#m "iet $#t he states that she has not $een compliant. She reports no associate" pain 'ith her shortness of $reath. /er 'eight has increase" o)er the past month from a sta$le $aseline of 1<4 po#n"s to nearly 1(& po#n"s. /er physical e.amination re)eals scattere" crackles on a#sc#ltation of the l#ngs an" mil" pitting e"ema of his lo'er e.tremities. A ro#tine la$oratory e.amination re)eals a ser#m potassi#m of .4 mE*7L% a ser#m #rea nitrogen of 24 mg7"L% an" a creatinine of 4.1 mg7"L. =hich of the follo'ing is the most likely pharmacologic ca#se of the electrolyte a$normalitya6Acetaminophen $62lomiphene c6/y"ro.ychloro*#ine "6+$#profen e6Spironolactone :ormal La$s 4 .A &4!year!ol" homose.#al man 'ith a 4!year history of /+? presents complaining of 2 'eeks of "iarrhea% 'hich has $ecome $loo"y o)er the past 'eek. /e has appro.imately si. $o'el mo)ements per "ay% 'ith

$loo" in almost all $o'el mo)ements% an" has se)ere fecal #rgency. /e has ha" t'o episo"es of fecal incontinence o)er the past 'eek. /e has a history of a Pne#mocystis carinii infection & years earlier% an" his last 2@4 co#nt 'as less than 4. A fle.i$le sigmoi"oscopy re)eals a proctosigmoi"itis 'ith "eep #lcers. =hich of the follo'ing is the most likely "iagnosisa6 2rohn "isease $62ytomegalo)ir#s colitis c6Entamoe$a histolytica infection "6Salmonella enteriti"is infection e6Ulcerati)e colitis :ormal La$s 4<.A 4 !year!ol" 'oman reports that she has $een ha)ing increase" in)ol#ntary loss of #rine% 'hich is especially prono#nce" 'hen she is feeling ner)o#s or 'hile sitting at her "esk. Rogging "oes not 'orsen the incontinence. She has not ha" these symptoms in the past an" is other'ise healthy. She is not taking any me"ications an" has ne)er $een pregnant. >n physical e.amination% she is afe$rile% 'ith sta$le )ital signs. /er a$"omen is $enign% an" )aginal e.amination re)eals no prolapse. Sensation in all e.tremities is intact% 'ith goo" motor strength. /er gait is normal% an" refle.es are intact. /er 'ork!#p re)eals a negati)e #rinalysis% an" $loo" c#lt#res an" #rine c#lt#res are all negati)e. Ser#m gl#cose le)el an" glycosylate" hemoglo$in le)el is normal. =hich of the follo'ing tests 'o#l" $e the most appropriate ne.t step in "iagnosisa6+ntra)eno#s pyelogram 3+?P6 $6Stress testing c6N!tip test "62ystometry e6 2ystoscopy :ormal La$s 4(. A (1!year!ol" man is $ro#ght to his family physician $eca#se of progressi)e memory loss an" occasional episo"es of "isorientation for the past < months. Aecently% he 'as fo#n" $y the police 'an"ering in the streets #na$le to recollect his 'ay home. A mental stat#s e.amination re)eals se)ere "eficits in short!term memory% $#t the

patient retains the a$ility to follo' a three!stage comman". Motor an" sensory f#nctions are intact% an" no tremor is o$ser)e". The patient is other'ise in goo" physical health an" "oes not smoke or take me"ications. Bloo" an" thyroi" f#nction tests are 'ithin normal limits. MA+ st#"ies sho' "iff#se cortical atrophy an" )entric#lar "ilatation. =hich of the follo'ing is the most likely "iagnosisa6 2hronic s#$"#ral hematoma $6@ementia of AlDheimer type c6:ormal!press#re hy"rocephal#s "6Parkinson "isease e6Pick "ementia :ormal La$s 41. A 4 !year!ol" man comes to his physician for e)al#ation of high $loo" press#re. /e has ha" t'o pre)io#s rea"ings% 'ith )al#es of systolic press#re 'ithin 144!1 4 mm /g% an" "iastolic press#re 'ithin 84!8 mm /g. At this time% his $loo" press#re is 14171( mm /g. /is height is 114 cm 3(1 in6% an" 'eight is ( kg 31< l$6. /is family history is negati)e for car"io)asc#lar "isease% stroke% "ia$etes% or hyperlipi"emia. The patient has not ha" any significant illness. /e "enies episo"es of palpitations% s'eating% or tremor. /e e.ercises reg#larly% "oes not smoke% an" takes no me"ications. /e has 4! alcoholic "rinks "aily on 'eek"ays an" #p to 14 "rinks on 'eeken"s. Physical e.amination re)eals no a$normalities. =hich of the follo'ing is the most appropriate ne.t step in managementa6 Aestriction of alcohol cons#mption $6Meas#rement of #rinary catecholamines c6Antihypertensi)e treatment 'ith an A2E inhi$itor "6Antihypertensi)e treatment 'ith a "i#retic e6 Antihypertensi)e treatment 'ith a $eta!$locker :ormal La$s 48.A 2!year!ol" carpenter complains of s'elling in his right knee% 'hich $egan 2 "ays earlier. /e "enies any history of arthritis or tra#ma in that region. Until the s'elling $egan% he ha" $een 9ogging appro.imately 2 miles "aily. >)er the past 41 ho#rs% his knee has $ecome s'ollen an" painf#l to 'eight $earing. /is temperat#re &1.2 2

3144.( 56% an" the knee has a ten"er eff#sion% 'hich is erythemato#s an" 'arm. There is a limite" range of motion. =hich of the follo'ing 'o#l" $e the most rele)ant in this patient,s historya65amily history of rhe#matoi" arthritis $6/istory of a $acterial gastroenteritis c6/istory of hepatitis B e.pos#re "6/istory of a recent #pper respiratory tract infection e6 Unprotecte" se. 'ith a prostit#te :ormal La$s 4. A 4 !year!ol" man presents 'ith a &!"ay history of hemoptysis. /e has ha" a co#gh an" has also $een complaining of sin#s congestion an" "rainage. 5#rthermore% he has note" $loo" in the #rine. /e has no other me"ical pro$lems an" takes no me"ications. An arteriole $iopsy in"icates necrotiDing inflammation. Urine se"iment sho's re" cell casts an" "ysmorphic re" cells. =egener gran#lomatosis is s#specte"% an" steroi"s are starte". Presence of 'hich of the follo'ing entities 'ill confirm the "iagnosisa6 An ele)ate" angiotensin con)erting enDyme 3A2E6 $6Antiglomer#lar $asement mem$rane anti$o"ies 3anti!CBM6 c6Antine#trophilic cytoplasmic anti$o"ies 3A:2A6 "6Eosinophilia e6F!$o"ies
Normal ,abs Note: Check your own answers before hittin$ the Check button below. When you click the Check button, a browser win"ow will a!!ear that contains a summary of your results. (*!lanations lock / (*!lanations

:) (*!lanation: 7he correct answer is (. 7his !atient !resents with si$ns of malabsor!tion as "escribe" by her loss of wei$ht, frequent loose stools, an" evi"ence of multi!le vitamin "eficiency. )he is also

hy!oalbuminemic an" has <ust returne" from a tro!ical re$ion. 7here are no risk factors or evi"ence, by history, for liver "isease. 7he hy!oalbuminemia, ecchymosis, an" IN6 are e*!laine" by the malabsor!tive !rocess instea", which is consistent with a vitamin 5 "eficiency. 7he fin"in$s are consistent with a tro!ical s!rue. 7hese !atients often "evelo! a :; "eficiency, as the terminal ileum is also affecte" more severely in the !ro*imal small bowel. 7he refractory res!onse to a $luten4free "iet an" the abru!t onset makes the "ia$nosis of celiac s!rue 'choice A) less likely. (nterohemorrha$ic strains of (scherichia coli'choice ) can cause a hemorrha$ic colitis, which woul" have $uaiac4!ositive stool. (nteroto*i$enic strains of (. coli'choice C) can cause travelerAs "iarrhea an" cholera4like "isease with !rofuse watery "iarrhea. 7here is no evi"ence here for chronic !ancreatitis 'choice %), which usually will occur in !atients with a lon$ history of chronic alcoholism. ;) (*!lanation: 7he correct answer is . 7he !atientAs me"ical recor" is technically their !ro!erty, an", as such, they may ask to review it at any time. +uch of the "ocumentation in the chart may be unclear to !atients, however, an" may be !otentially misconstrue". -or this reason, it is always best to first offer to review the !atientAs chart with them. Whereas the !atientAs <u"$ement about the content of the me"ical recor" may be affecte" by her "e!ression, there is no <ustification for waitin$ until she be$ins treatment to show her the recor" 'choice A). ,awyers may be hel!ful at times to mana$e risk in the clinical settin$, but there is no in"ication in this case for consultin$ a lawyer 'choice C). As state" above, it is the !atientAs le$al ri$ht to see her me"ical chart. -urthermore, tellin$ the !atient that she must await a lawyerAs consultation before seein$ the recor" may in<ure an alrea"y tenuous relationshi!. &hysician4!atient confi"entiality laws 'choice %) !rotect the !atient from thir" !arties havin$ access to their recor" an" "o not limit the !atientAs access to their own recor". +akin$ co!ies of the me"ical recor" without offerin$ to review it with her is 'choice () a subo!timal choice because of the lost chance to fully e*!lain the clinical reasonin$ an" the !otential to worry the !atient or cause her un"ue emotional stress.

0) (*!lanation: 7he correct answer is A. 7his !atient is !resentin$ with sym!toms of a mo"erate asthmatic attack. )he has not been treate" !rior to her !resentation to the emer$ency "e!artment. 7he first a!!ro!riate ste! woul" be the a"ministration of a beta4; a$onist, such as albuterol, as a broncho"ilator. A"ministerin$ it in a nebuli#er allows "istal "elivery of the me"ication to the reactive airways an" is not "e!en"ent on the !atientAs technique in the use of an inhaler, which is often erratic. eta4; a$onist thera!y will stimulate broncho"ilation but "o nothin$ for any un"erlyin$ inflammatory com!onent of this asthmatic attack. 2owever, it is the first4line thera!y for sym!tomatic relief. Atro!ine nebuli#ers 'choice ) are occasionally use" as a"<unctive thera!y to beta4; a$onist nebuli#er treatment. 2owever, because of their systemic anticholiner$ic effects, they are not use" as first4line thera!y. I!ratro!ium, another anticholiner$ic broncho"ilator, is of value because it has far fewer systemic si"e effects. I1 hy"rocortisone 'choice C) may be a!!ro!riate in the emer$ency "e!artment settin$ in a !atient with a severe attack but "oes not have an effect for several hours. )imilarly, oral !re"nisone 'choice %) may be a!!ro!riate in severe asthma but is not the first4line thera!y. )ubcutaneous e!ine!hrine 'choice () is no lon$er consi"ere" as !art of the !rotocol for the mana$ement of asthma in the emer$ency "e!artment because of its hi$h risk of tachycar"ias an" ventricular arrhythmias. ?) (*!lanation: 7he correct answer is C. +yelo!hthisic anemia is anemia cause" by re!lacement of marrow by other constituents, either abnormal hemato!oietic cells or nonhemato!oietic cells. 7hese, in effect, H!ush outH the normal marrow constituents lea"in$ to the anemia. 7he fra$mente" an" "istorte" cells seen in anemias "ue to this ty!e of !rocess are the result of trauma to erythrocytes $oin$ throu$h "istorte" an" com!resse" ca!illary be"s. If the un"erlyin$ "isease !rocess is 'not very common) myelofibrosis or a 'rare) here"itary stora$e "isease, s!lenome$aly 'sometimes massive) relate" to either e*trame"ullary hemato!oiesis or un"erlyin$ "isease !rocess may be !resent. When the un"erlyin$ "isease !rocess is an en"4sta$e cancer 'common) or $ranulomatous "isease 'less common), s!lenome$aly is usually not seen because little e*trame"ullary hemato!oiesis occurs. 7reatment involves mana$ement of the un"erlyin$ "isease, if !ossible, with transfusions in"icate" if the anemia !ro"uces car"iovascular sym!toms. (rythro!oietin is often trie" in these settin$s, but ty!ically !ro"uces only a mo"est res!onse since the un"erlyin$ !roblem is a lack of marrow room rather than sta$nant erythrocyte "evelo!ment.

Autoimmune anemias 'choice A) ten" to be hemolytic an" ten" to have normal a!!earin$ erythrocytes on !eri!heral smearG autoa$$lutination of re" cells may be seen in some ty!es. 2ere"itary hemo$lobin synthesis !roblems 'choice ) inclu"e "iseases such as sickle cell anemia, thalassemia, an" hemo$lobin C "isease, an" have usually been a!!arent throu$h most of the !atientAs life. Nutritional "eficiency anemias 'choice %) can be either me$aloblastic 'folate, vitamin :;, or less commonly vitamin C "eficiency) or microcytic 'iron "eficiency). +il"er forms can sometimes be normocytic. -ra$mente" an" mechanically "istorte" erythrocytes are not seen in these !rocesses. Anemias "ue to re" cell membrane "efects 'choice () inclu"e here"itary s!herocytosis an" here"itary elli!tocytosis. 7hese "isor"ers !ro"uce, res!ectively, re$ular s!herocytic an" re$ular elli!tical erythrocytes, but woul" not !ro"uce fra$mente" an" mechanically "istorte" erythrocytes. /) (*!lanation: 7he correct answer is . 7his !atient has a ki"ney stone, most likely "ue to e*cess su!!lementation with vitamin C. )he has renal colic 'flank !ain ra"iatin$ to the $roin), hematuria, an" ra"io$ra!hic evi"ence of a stone. )ome in"ivi"uals believe that vitamin C can !revent or shorten the "uration of a col", however stu"ies have faile" to "emonstrate any "ifferences in occurrence, "uration or severity of a col" with vitamin C a"ministration. (*cess vitamin C su!!lementation can lea" to uricosuria an" the "evelo!ment of calcium o*alate stones. 1itamin A e*cess 'choice A) lea"s to $astrointestinal "isturbances, "i##iness, hair loss, "ry skin, an" bone !ain. 1itamin ( e*cess 'choice C) lea"s to $astrointestinal "isturbances an" coa$ulo!athies. +ethyl!heni"ate 'choice %) is use" to treat attention4"eficit hy!eractivity "isor"er but is sometimes use" by other !eo!le 'es!ecially teena$ers an" youn$ a"ults) as a "ru$ of abuse because of its ability to increase concentration an" "ecrease the nee" for slee! "urin$ stu"yin$ for e*aminations. Common si"e effects inclu"e wei$ht loss, tachycar"ia, an" "ia!horesis. &yri"o*ine e*cess 'choice () can lea" to severe !eri!heral neuro!athies, not ne!hrolithiasis.

C) (*!lanation: 7he correct answer is . 7he !erfusion territory of the anterior s!inal artery inclu"es the anterior horn cells an" !art of the !ain an" tem!erature !athways. 7hrombosis of this artery causes flacci" !aralysis, loss of bowel an" bla""er function an" loss of !ain an" tem!erature sensation. Anterior cerebral artery occlusion 'choice A) can cause le$ weakness but there shoul" be no sensory loss. F!!er motor neuron si$ns are !resent in the le$. Cau"a equina syn"rome 'choice C) can cause flacci" !ara!le$ia but somatic sensation shoul" be com!romise" since sensory nerve roots are involve" in the "isease !rocess. .uillain4 arrV syn"rome 'choice %) rarely "evelo!s this acutely. If loss of !ain an" tem!erature sensation occurs, it !resents with concurrent loss of !osition an" vibration sense. 7horacic cor" com!ression 'choice () is associate" with low back !ain an" !ro"uces u!!er motor neuron "ysfunction in the le$s. D) (*!lanation: 7he correct answer is A. 7his is 5a!osi sarcoma, which may be the !resentin$ manifestation of AI%). 7he lesions, which behave like a mali$nant vascular sarcoma, are cause" by infection with a member of the her!es virus family 'her!esvirus ty!e @). As you may have notice", AI%) !atients are $enerally very vulnerable to her!es virus infections, which also inclu"e her!es sim!le* I, her!es sim!le* II, her!es #oster4varicella, cytome$alovirus, an" (!stein arr virus. In 5a!osi sarcoma, the virus infects en"othelial cells in bloo" vessels, an" causes formation of tumors that have s!in"le cells alternatin$ with slit4like vascular s!aces. 7he slit4like vascular s!aces lack the normal mechanisms of vasoconstriction in res!onse to in<ury, an" consequently may blee" !rofusely after minor trauma. While the most obvious lesions are on the skin, similar lesions may be foun" in internal or$ans an" may $ive rise to internal blee"in$. 5a!osi sarcoma has been treate" with a variety of chemothera!iesG isolate" su!erficial lesions may alternatively res!on" to electrocoa$ulation, cryothera!y, an" electron beam ra"iothera!y. +ost !atients "ie of other infections secon"ary to the immunosu!!ression of the AI%), rather than of the 5a!osi sarcoma. An in"olent form of 5a!osi sarcoma is also rarely seen in ol"er men of Italian or Newish ancestry who "o not have AI%). Contact aller$y 'choice ) can cause erythema or blisterin$, but woul" not usually cause blee"in$. 2emo!hilia A 'choice C) can cause blee"in$ but woul" not cause !ersistent skin lesions other than '!ossibly) !ur!ura.

+elanoma 'choice %) ten"s to !ro"uce lesions that may vary in color an" may metastasi#e to the skin, but woul" not be e*!ecte" to be s!ecifically associate" with marke" blee"in$ from the lesions. )tre!tococcal infection 'choice () can cause a "isseminate" skin rash, but this woul" not !ersist for a year an" woul" not be a source of blee"in$. @) (*!lanation: 7he correct answer is C. &a!illary muscle ru!ture occurs in less than /= of !atients with acute myocar"ial infarction. 7he !atient usually !resents with acute onset of con$estive heart failure a!!ro*imately :4D "ays after infarction. 6u!ture of an entire !a!illary muscle is usually ra!i"ly fatal secon"ary to massive mitral re$ur$itation. 2owever, ru!ture of one hea" of a !a!illary muscle may be tolerate" for a !erio" of time. A lou" systolic murmur is hear" at the left sternal bor"er, an" echocar"io$ram shows a flail mitral valve. )ur$ical re!air or re!lacement of the valve can be effective if the !atient tolerates the !roce"ure. Aortic "issection 'choice A) is associate" with the su""en onset of severe chest !ain ra"iatin$ to the ab"omen an" back. &ain may ra"iate as the "issection !ro$resses. Auscultation is si$nificant for murmur of aortic insufficiency. &ulses may be absent or asymmetric. Car"iac ru!ture 'choice ) is usually ra!i"ly fatal. 6u!ture usually occurs a!!ro*imately / "ays after transmural infarction. 7he !atientAs initial course may be uneventful, followe" by abru!t occurrence of car"io$enic shock an" ra!i" "eath. )e!tal !erforation 'choice %) of the ventricle occurs in less than := of !atients followin$ myocar"ial infarction an" results in marke" biventricular failure. It is associate" with a holosystolic murmur alon$ the left sternal bor"er, an" a thrill is commonly hear" on auscultation. It is "ifficult to "ifferentiate from acute mitral insufficiency. 1entricular aneurysm formation 'choice () is a locali#e" area of thin scarre" myocar"ium that !rotru"es beyon" an" "istorts the ventricular cavity. It may "evelo! "ays followin$ myocar"ial infarction an" may $ra"ually enlar$e over several weeks. &atients are $enerally asym!tomatic but may "evelo! con$estive heart failure, arrhythmias, or emboli with enlar$ement of the aneurysm. I) (*!lanation: 7he correct answer is . 7his !atient most likely has infectious mononucleosis, which is cause" by the (!stein4 arr virus. 7he clinical features inclu"e sore throat, hea"ache, fever, malaise,

lym!ha"eno!athy, !haryn$itis an" tonsillitis, he!atos!lenome$aly, !eriorbital e"ema, rash, an" a !alatal enanthem. 7he "ia$nosis is ma"e by hetero!hile antibo"y testin$, ( 1 antibo"y titers, an" lym!hocytosis with aty!ical lym!hocytes. 7reatment inclu"es rest, flui"s, an" anal$esics. Am!icillin thera!y lea"s to a maculo!a!ular rash. An antibiotic shoul" not be $iven because this is a viral infection. A chest *4ray film 'choice A) is useful for a res!iratory or car"iac !rocess. Infectious mononucleosis cannot be "ia$nose" by any ra"io$ra!hic stu"y. A lateral *4ray of the neck 'choice C) is useful in acute e!i$lottitis. It woul" show e!i$lottic swellin$. 7he sym!toms of e!i$lottitis inclu"e e*treme sore throat, "roolin$, an" "ifficulty swallowin$. A ra!i" stre!tococcus test 'choice %) is use" to "ia$nose stre!tococcal !haryn$itis. )tre!tococcal !haryn$itis is characteri#e" by fever, sore throat, tonsillar e*u"ates, an" cervical lym!ha"eno!athy. A scarlatiniform rash may follow the !haryn$itis. 7he rash is a "iffuse erythema that later "esquamates. It is cause" by an erythroto*in. Antibo"ies to the varicella virus 'choice () can be "etecte" by immunofluorescence. 7he sym!toms of varicella inclu"e a vesicular rash in various sta$es of evolution, fever, an" malaise. 7he treatment is bathin$, soaks, an" anti!ruritic to!ical me"ication. 7reatment with as!irin has been associate" with 6eye syn"rome an" shoul" therefore be avoi"e". :>) (*!lanation: 7he correct answer is C. A !atient such as this, who !resents with si$ns an" sym!toms of a res!iratory tract infection, requires only one ma<or initial thera!eutic "ecision: are antibiotics require"J Antibiotics are in"icate" for cases of !neumonia. +any !atients like this one actually have bronchitis. 7herefore, to "etermine whether antibiotics are require", a chest ra"io$ra!h is nee"e" to "etect any infiltrates. %eci"in$ that no intervention is in"icate" 'choice A) is hasty since there are currently no "ata in his history an" !hysical e*amination that allow a $oo" "ecision to be ma"e as to what this !atient actually has. &rescribin$ !enicillin an" sen"in$ the !atient home 'choice ) assumes that this !atient has !neumonia. 7his has not yet been "emonstrate". An arterial bloo" $as 'choice %) is not in"icate" for this !atient. -irst, his !hysical e*amination "oes not su$$est that severe res!iratory com!romise, an" therefore a ma<or aci"4base abnormality, woul" be !resent. )econ", this test in no way hel!s with mana$ement.

6eferrin$ the !atient to the hos!ital for a"mission 'choice () is much too !remature a "ecision. 7here is no in"ication that this !atient fits the "escri!tion for a"mission. ::) (*!lanation: 7he correct answer is C. +6I with $a"olinium is consi"ere" the most sensitive test for "etectin$ microa"enoma. 7he test can reveal microa"enomas in ;>= of normal women. C7 scans can be quite sensitive in the "etection of microa"enomas, but +6Is are even more so 'choice A). Insulin4resistance as a result of im!aire" $rowth hormone res!onse to insulin4in"uce" hy!o$lycemia is not as common 'choice ). 7he serum !rolactin level is elevate" "ue to hy!ersecretion in 0>4/>= of !atients 'choice %). -or the o!tic chiasm to be com!resse" in or"er to cause visual fiel" chan$es, the microa"enoma woul" have to be quite lar$e, an" a screenin$ test base" on this fin"in$ woul" be ineffective 'choice (). :;) (*!lanation: 7he correct answer is A. 7his question "escribes a ty!ical !resentation of achalasia. Achalasia is a neuro$enic eso!ha$eal "isor"er thou$ht to be cause" by a malfunction of the myenteric !le*us of the eso!ha$us. 7he result is "enervation of the "istal eso!ha$eal muscle with resultin$ im!aire" eso!ha$eal !eristalsis. 7he characteristic fin"in$s on eso!ha$eal manometry are "iminishe" or absent !eristalsis in the bo"y of the eso!ha$us an" a hi$h restin$ lower eso!ha$eal !ressure. 7he latter fin"in$ is relate" to a failure of the usual rela*ation of the s!hincter as the bolus of foo" reaches it. Choices an" ( are wron$, because the u!!er eso!ha$eal s!hincter is usually unaffecte" in this con"ition. Choice C is wron$ because !eristalsis is "ecrease" or absent in the mi"4eso!ha$us in achalasia. Choice % is wron$ because restin$ lower eso!ha$eal !ressure is characteristically hi$h in achalasia. :0)

(*!lanation: 7he correct answer is %. 7o ra!i"ly assess for the !ossibility of antifree#e in$estion, the !hysician can evaluate the !atientAs urine un"er a Woo"As lam! for fluorescence. +anufacturers of ethylene $lycol4containin$ antifree#es ty!ically a"" fluorescein to the mi*, which will fluoresce un"er a Woo"As lam!. (*aminin$ a !atientAs breath for a fruity o"or 'choice A) is ty!ically reserve" for instances in which ketosis, as with "iabetic ketoaci"osis, is sus!ecte". (levate" ketones ty!ically cause this fruity o"or. (valuatin$ the o!tic "iscs for hy!eremia 'choice ) is ty!ically reserve" for sus!ecte" methanol to*icity, since methanol is metaboli#e" to formic aci", which can cause irritation of the o!tic nerve. Obtainin$ an (C. 'choice C) will be of minimal utility in the "ia$nosis, since ethylene $lycol into*ication is not associate" with a !atho$nomonic electrocar"io$ra!hic fin"in$. Obtainin$ a serum osmolarity 'choice ()an" assessin$ the osmolar $a! will also be of limite" use since an elevate" osmolar $a! 'measure" osmolarity 4 calculate" osmolarity) will be seen with in$estions of many osmotically active com!oun"s, inclu"in$ ethanol. 7his measure is therefore not s!ecific for ethylene $lycol. :?) (*!lanation: 7he correct answer is C. AI%) an" other !rofoun"ly immunosu!!resse" !atients are vulnerable to infectious eso!ha$itis. (n"osco!y with bio!sy is usually use" to i"entify the causative a$ents, which are ty!ically Can"i"a, 2er!es sim!le*, or cytome$alovirus. Careful review of the bio!sy material is warrante", since these !atients may actually be infecte" by more than one a$ent. In the case of cytome$alovirus infection, the "istinctive histolo$ical fin"in$ is the !resence of small numbers of cells with marke"ly enlar$e" nuclei, which on careful observation may show both cyto!lasmic an" nuclear viral inclusions. Cytome$alovirus infection can be treate" with $anciclovir. Aci" reflu* 'choice A) coul" !ro"uce inflammation, but woul" not alter nuclear mor!holo$y. Can"i"a 'choice ) woul" have hy!hal an" yeast forms. 2er!es sim!le* 'choice %) causes multinucleate" cells with nuclear viral inclusions. 2er!es #oster 'choice () infection woul" resemble her!es sim!le* infection, but it much less commonly involves the eso!ha$us.

:/) (*!lanation: 7he correct answer is A. 7his !atientAs !robable "isease is !aro*ysmal nocturnal hemo$lobinuria, also known as +archiafava4+icheli syn"rome. 7his rare "isor"er a!!ears to have a $enetic !re"is!osition relate" to the &I.4A $ene on the E4chromosome that makes a $lycosyl4 !hos!hati"yl4inositol HanchorH use" by a variety of membrane !roteins. &aro*ysmal nocturnal hemo$lobinuria behaves like an acquire" hemolytic anemia with marke" sensitivity to serum com!lement factor C0. Affecte" !atients are most often men in their ;>As, but both se*es an" all a$es can "evelo! the con"ition. etween crises, intermittent hemo$lobinuria "evelo!s at ni$ht. 2emolytic crises can be tri$$ere" by infection 'as in this case), iron use '!rescribe" to treat the anemia), vaccination, or menstruation. In a""ition, these !atients are !re"is!ose" for thrombotic "isease, inclu"in$ u""4Chiari )yn"rome. 7he classic 'but somewhat non4s!ecific) "ia$nostic test for !aro*ysmal nocturnal hemo$lobinuria is the 2am test, in which aci" incubation causes re" cell lysisG flow cytometric analysis usin$ C% // an" C% /I is consi"ere" the most "efinitive way to confirm the "ia$nosis. +any !atients survive for "eca"es an" a few have been successfully treate" with allo$enic bone marrow trans!lant. 2emo$lobin electro!horesis 'choice ) is use" for sickle cell "isease an" other hemo$lobino!athies. &hila"el!hia chromosome 'choice C) is !ositive in many cases of chronic myelo$enous leukemia. )ickle cell test 'choice %) is use" for sickle cell anemia, sickle cell trait, an" )4C "isease. 7artrate4resistant aci" !hos!hatase 'choice () is use" to confirm hairy4cell leukemia. :C) (*!lanation: 7he correct answer is C. 7he !atient most likely has I$A ne!hro!athy. 7his is the most common i"io!athic ne!hro!athy in the worl". It will usually !resent with hematuria followin$ a sore throat. &roteinuria is frequent, but only rarely is severe. I$A ne!hro!athy is much more common in Asia an" )outh America. Al!ort syn"rome 'choice A) is a here"itary ne!hritis that causes hematuria, renal failure an" "eafness. A family history of hematuria is usually obtaine". .oo"!asture syn"rome 'choice ) is an acute $lomerulone!hritis that causes a ra!i" loss of renal function. 7hese !atients will often have hemo!tysis as well. Nil "isease 'choice %) is common in chil"ren an" !resents with heavy !roteinuria without hematuria.

We$ener $ranulomatosis 'choice () is a systemic "isease that causes a ra!i" loss of renal function. :D) (*!lanation: 7he correct answer is %. 2er!es sim!le* virus '2)1) is the most common etiolo$ic a$ent of s!ora"ic viral ence!halitis. 7he clinical !resentation is variable but usually abru!t, an" the ra!i" onset of confusion an" sei#ures are frequent manifestations. 7he most characteristic !atholo$ic lesions, which are "emonstrate" by C7 stu"ies, consist of hemorrha$ic necrosis of the tem!oral lobes. 2er!es ence!halitis is almost always "ue to 2)1 ty!e ; an" affects !reviously healthy in"ivi"uals. F! to />= of newborns "elivere" va$inally from mothers with active 2)1 infection "evelo! a severe $enerali#e" form of her!etic ence!halitis. Arbovirus ence!halitis 'choice A) is the most im!ortant cause of e!i"emic viral ence!halitis. +any "ifferent s!ecies may be involve", each with a s!ecific re$ional $eo$ra!hic "istribution. 7he clinical course is mil"er an" !ro$nosis is better than her!etic ence!halitis. rain to*o!lasmosis 'choice ) affects severely immunocom!romise" !atients, most frequently those with AI%). It lea"s to roun", rin$4enhancin$ intracerebral masses that may "evelo! anywhere in the brain. (chovirus ence!halitis 'choice C) is one of the most common etiolo$ic a$ents of the so4 calle" lym!hocytic menin$oence!halitis, also known 'ina!!ro!riately) as ase!tic menin$itis. )ym!toms are mil", often limite" to hea"ache an" malaise. C)- is usually normal or shows mil" lym!hocytosis. +etastatic melanoma 'choice () in the brain may "evelo! hemorrha$e. 2owever, metastases a!!ear on neuroima$in$ as s!ace occu!yin$ masses, usually multi!le an" locate" at the corte*4white matter <unction. :@) (*!lanation: 7he correct answer is A. 7hese lesions are skin ta$s, more formally known as acrochor"ons. 7hey are very common beni$n lesions that can occur at any skin site, but have a !re"ilection for the neck, a*illa, an" $roin. +ulti!le lesions are common, an" the lesions ten" to increase in number with a$e. 7hey are usually asym!tomatic but can be irritatin$. +icrosco!ically, an acrochor"on consists of a fibrovascular core, sometimes also with fat cells, covere" by an unremarkable e!i"ermis. Asym!tomatic skin ta$s are usually not treate". +any metho"s can be use" to remove "isfi$urin$ or irritate" skin ta$s, inclu"in$ free#in$ with liqui" nitro$en, li$ht electro"esiccation, or e*cision with scal!el or scissors.

,enti$os 'choice ) are flat macules characteri#e" microsco!ically by an increase" number of melanocytes occurrin$ sin$ly rather than in clusters in the lower !art of the e!i"ermis. ,i!omas 'choice C) !ro"uce soft, movable, subcutaneous no"ules com!ose" of mature a"i!ose tissue. )eborrheic keratoses 'choice %) are !i$mente" su!erficial e!ithelial lesions that are usually warty in consistency. )!i"er an$iomas 'choice () are bri$ht re", faintly !ulsatile, vascular lesions of the skin. :I) (*!lanation: 7he correct answer is %. 7he !atient is havin$ an acute myocar"ial infarction. 7he infarct occurs as a result of an atherosclerotic !laque with thrombus formation, lea"in$ to coronary artery obstruction. ,ysis with a thrombolytic a$ent has been shown to "ecrease mortality from early !ostmyocar"ial infarction. As!irin !revents both !latelet a$$re$ation an" reocclusion of the re!erfuse" vessels. In association with as!irin an" lytic thera!y, he!arin re"uces mortality to a $reater "e$ree. 7hrombolytic thera!y is in"icate" in !atients u! to D/ years of a$e 'choice A). Absolute contrain"ications inclu"e a blee"in$ "iathesis, ma<or sur$ery or trauma within C months, $astrointestinal blee"in$, or the !resence of aortic "issection or a known intracranial tumor. If the !atient is hy!otensive, I1 flui"s may be nee"e" 'choice ). 7he hy!otension may result from car"io$enic shock as a result of heart failure from the acute myocar"ial infarction. At this time, the !atient is hemo"ynamically stable, an" the treatment in choice % is more a!!ro!riate. As!irin is a !latelet a$$re$ation inhibitor an" has been shown to re"uce mortality in myocar"ial infarction an" ischemia 'choice C). .iven alone, however, it will not !revent mortality in this !atient unless the thrombus is imme"iately lyse". 2e!arin forms a com!le* with antithrombin III, an" !revents action by thrombin. It is more effective when $iven in association with a lytic a$ent. If me"ical treatment fails, the !atient may require car"iac sur$ery for a coronary artery by!ass $raft 'choice (). In the imme"iate !erio", the thrombus must be lyse" an" the !atient stabili#e". )tent !lacement an" balloon an$io!lasty shoul" be e*!lore" as o!tions. ;>)

(*!lanation: 7he correct answer is . 7he manifestations are hi$hly su$$estive of systemic lu!us erythematosus '),(). 7his immune4relate" connective tissue "isease has a !re"ilection for women of chil"bearin$ a$e. Noint !ains, low4$ra"e fever, wei$ht loss, fati$ability, an" rash over sun4e*!ose" areas are amon$ the most frequent early si$ns. Often, !hysical e*amination reveals evi"ence of fibrinous !leuritis or !ericar"itis. Antinuclear antibo"ies 'ANAs) are hi$hly sensitive but not s!ecific, an" thus elevate" levels of ANA confirm the clinical sus!icion of ),( but "o not establish a "efinitive "ia$nosis. Anti4"ouble stran"e" %NA an" anti4 )mith antibo"ies are most s!ecific of ),( an" are thus e*tremely useful to confirm the "ia$nosis. Anti4"ouble stran"e" %NA antibo"ies also reflect "isease activity, while anti4 )m antibo"ies "o not. Note also that anti!hos!holi!i" antibo"ies are !ositive, which is often the case with ), (. +il"4to4mo"erate anemia, low leukocyte count, an" low !latelet counts are frequently foun" in ),( !atients. Occasionally, anemia is "ue to autoantibo"ies to re" bloo" cells. Anticentromere antibo"ies 'choice A) are virtually !atho$nomonic of the C6()7 variant of sclero"erma, manifestin$ with calcinosis, 6aynau" !henomenon, eso!ha$eal "ysmotility, sclero"actyly, an" telan$iectasia. Assay for anti4neutro!hil cyto!lasmic antibo"ies 'ANCAs) 'choice C) woul" ai" in the investi$ation of the !ossibility of certain forms of vasculitis syn"romes, such as We$ener $ranulomatosis, Chur$4)trauss syn"rome, an" microsco!ic !olyan$iitis. Assay for rheumatoi" factor 'choice %) is not useful in this situation. 6heumatoi" factor is often !ositive not only in rheumatoi" arthritis, but also in other colla$en vascular con"itions, inclu"in$ ), (. E4ray stu"ies of affecte" <oints 'choice () woul" yiel" nons!ecific results. Arthritis in ),( is usually not associate" with "estructive or "eformin$ chan$es of the <oints. )kin bio!sy 'choice -) may be hel!ful in su!!ortin$ a "ia$nosis of ),( in uncertain cases, es!ecially when "ermatolo$ic manifestations are the !re"ominant si$ns. ;:) (*!lanation: 7he correct answer is C. 7he requirements for utili#ation of home o*y$en thera!y have been "etermine" on the basis of !lacebo4controlle", ran"omi#e" trials. In !atients on a stable me"ical re$imen, an arterial o*y$en !ressure of less than // mm 2$ has been "etermine" as the cutoff below which ma*imal benefit is obtaine". (*ercise4in"uce" o*y$en "esaturations to less than I;= 'choice A) are common in !atients with chronic obstructive !ulmonary "isease 'CO&%).

A restin$ arterial &aO; showin$ an alveolar arterial $ra"ient of more than :; mm 2$ 'choice ) is abnormal, but is not s!ecific for CO&%, an" is not $enerally an in"ication for su!!lemental o*y$en thera!y. +ost CO&% !atients late in the "isease have a restin$ &aO; of $reater than ?> mm 2$ 'choice %). 7his is one of !arameters that benefits from su!!lemental o*y$en, but it is not use" in the "ecision tree to initiate thera!y. 6oom air o*y$en saturation of less than I;= 'choice () certainly in"icates some element of 1B8 mismatchin$, but not enou$h to merit continue" su!!lemental o*y$en. ;;) (*!lanation: 7he correct answer is C. In the !resence of hy!ertension of recent onset, alon$ with "ermal striae, easy bruisability, an" evi"ence of $lucose intolerance '$lycosuria), hy!ercortisolism shoul" be sus!ecte". 7he first test to !erform is the "e*amethasone su!!ression, which consists of a"ministerin$ : m$ of "e*amethasone at :: &+, an" then measurin$ serum cortisol levels in a bloo" sam!le "rawn at @ A+ ne*t "ay. Abnormally hi$h cortisol levels after this test confirms hy!ercortisolism. 7he ne*t ste! is to fin" the source of e*cessive cortisol or AC72. Other manifestations of hy!ercortisolism inclu"e osteo!orosis, muscle wastin$, !sycholo$ic alterations, hirsutism, an" $ranulocytic leukocytosis with lym!ho!enia. aseline !lasma AC72 measurement 'choice A) shoul" be !erforme" after a "ia$nosis of hy!ercortisolism has been confirme" by "e*amethasone su!!ression test. 7his test is use" to "etermine whether e*cessive cortisol !ro"uction is secon"ary to increase" AC72 levels. C7 scans of the chest an" ab"omen 'choice ) are !erforme" to look for ecto!ic sources of AC72. 7hese ra"iolo$ic stu"ies are es!ecially useful in fin"in$ neo!lasms that may manifest with ina!!ro!riate AC72 secretion, the most frequent of which bein$ small cell carcinoma of the lun$s. +easurement of mi"ni$ht serum cortisol level 'choice %) hel!s "istin$uish Cushin$ syn"rome from other causes of !seu"o4Cushin$ states, such as alcoholism, "e!ression, an" anore*ia nervosa. 7his test requires the !atient to remain in the same time #one for 0 consecutive "ays, be without foo" for at least 0 hours, an" have an in"wellin$ catheter rea"y for the bloo" "raw. +easurement of ;?4hour urine cortisol an" creatinine 'choice () may become necessary for some !atients in whom the "e*amethasone su!!ression test $ives equivocal results. 7he above mentione" !seu"o4Cushin$ states, however, may show abnormally hi$h levels of free urine cortisol.

+6I of the hea" 'choice -) is the metho" of choice to stu"y the !ituitary $lan" an" look for a"enomas. AC724!ro"ucin$ a"enoma is the cause of Cushin$ "isease, which accounts for D>= of cases of hy!ercortisolism not "ue to e*o$enous corticosteroi"s. ;0) (*!lanation: 7he correct answer is . Creut#fel"t4Nakob "isease 'CN%) is the most im!ortant of the !rion4relate" s!on$iform ence!halo!athies. It manifests with a ra!i"ly !ro$ressive "ementin$ !icture, which is further "istin$uishe" by the !resence of myoclonic <erkin$ movements an" characteristic electroence!halo$ra!hic chan$es. 7he "isease is rare, with an inci"ence of : case !er : million in the $eneral !o!ulation. A!!ro*imately @/= of such cases are s!ora"ic an" !robably relate" to acquire" mutations in the $ene co"in$ for !rion !rotein '&r&C). A minority of cases are familial an" cause" by inherite" mutations of the &r&C $ene. 6are cases have occurre" followin$ im!lants of ca"averic tissues 'e.$., cornea, "ura mater) from infecte" "onors, or intranasal a"ministration of $rowth hormone of ca"averic ori$in. A new variant 'bovine s!on$iform ence!halo!athy) is !robably transmitte" to humans from infecte" cattle an" results in a clinical sym!tomatolo$y similar to CN %. 1acuolar 's!on$iform) chan$es in the $ray matter are the "efinin$ mor!holo$ic chan$e of CN %. Al#heimer "ementia 'choice A) has a !rotracte" course, usually over a !erio" of many years. 7he clinical !icture is characteri#e" by !ro$ressive "eterioration of memory functions without any s!ecific motor or sensory "eficits. A cortical bio!sy woul" reveal senile !laques an" neurofibrillary tan$les. .lioma 'choice C) is the $eneric "esi$nation of any !rimary brain neo!lasm of $lial ori$in, inclu"in$ astrocytomas an" oli$o"en"ro$liomas. A $lioma woul" !ro"uce white matter chan$es "etectable on +6I as a s!ace4occu!yin$ lesion. &seu"otumor cerebri 'choice %) is a rare "isor"er "ue to increase" intracranial !ressure without any a!!arent cause. 7his i"io!athic con"ition manifests acutely with hea"ache an" other si$ns of raise" intracranial !ressure. It occurs most commonly in obese women in association with other !re"is!osin$ con"itions, such as renal failure, en"ocrine "iseases, an" "ru$ 'tetracycline) or vitamin a"ministration. 1iral ence!halitis 'choice () is characteri#e" by !erivascular chronic inflammatory infiltration, scattere" micro$lial no"ules, neuronal loss, an" $liosis. ;?) (*!lanation: 7he correct answer is

%. 7he main issue in evaluatin$ im!otence is "istin$uishin$ between !sycholo$ical causes an" or$anic causes. 7he !resence of nocturnal or early mornin$ erections basically eliminates the or$anic causes an" lea"s to a "ia$nosis of !sycholo$ical im!otence. 7he most common !sycholo$ical causes inclu"e an*iety an" "e!ression. Nocturnal !enile tumescence, which occurs "urin$ 6(+ slee!, can be assesse" in a slee! lab. If nocturnal erections are not !resent, the im!otence is most likely "ue to an or$anic cause. 7he most likely causes are testicular failure, hy!er!rolactinemia, me"ications, alcohol, o!ioi"s, nicotine, trauma, !ria!ism, "iabetes, vascular "isease, an" neurolo$ic "iseases, such as "iseases of the s!inal cor" an" loss of sensory in!ut. &hysical e*amination shoul" inclu"e a "etaile" $enital e*amination, evaluation for si$ns of femini#ation, an" neurolo$ic an" vascular e*ams. H2ow often "o you ri"e your bicycleJH 'choice A) may be relevant because of neurolo$ic an" vascular com!romise cause" by the seat, however it will not "istin$uish between or$anic an" !sycholo$ic causes. HAre you se*ually attracte" to your wifeJH 'choice ) is a relevant question if the !atient is havin$ nocturnal erections an" is not "e!resse" or an*ious. HAre you feelin$ unusually an*ious latelyJH 'choice C) is a $oo" question to follow the question about nocturnal erections 'if he is havin$ them). H%o you love your wifeJH 'choice () is a question that may be aske" if he is havin$ nocturnal erections an" is an*ious an" "e!resse". It may be a little blunt an" can !robably be aske" in a subtler manner. ;/) (*!lanation: 7he correct answer is %. 7his el"erly woman who has been be"ri""en an" on a narcotic anal$esic has become severely obsti!ate". 7his multifactorial con"ition is cause" by her immobili#ation, bowel hy!omotility secon"ary to the co"eine, an" !ossible mil" "ehy"ration from her "iuretic. 7he treatment is manual "isim!action, which will !rom!tly Ho!en the floo" $atesH an" yiel" imme"iate relief for the !atient. No other ima$in$ stu"ies, i.e., barium enema 'choice A), colonosco!y 'choice ), or fle*ible si$moi"osco!y 'choice C) are necessary at !resent. 7he !atient shoul" be "isim!acte" to !rom!tly resolve her ab"ominal "istention an" "iscomfort. &assa$e of a naso$astric 'choice () tube woul" be in"icate" if the !atient showe" si$ns of u!!er $astrointestinal blee"in$, but woul" not be a!!ro!riate for this woman with obvious consti!ation. ;C)

(*!lanation: 7he correct answer is %. 7his question a""resses the issue of Hbreakin$ ba" newsH. 7here is a si*4ste! !rotocol that is recommen"e". 7he first ste! is H$ettin$ starte"H an" $ettin$ the !hysical conte*t ri$ht. 7he meetin$ shoul" i"eally be in a quiet, !rivate area, either alone or with !eo!le that she chooses to be !resent. 7he !hysician shoul" be seate". 7he meetin$ shoul" start with $eneral questions or statements that $ive the !atient the i"ea that the !hysician is intereste" in her con"ition an" that this is a two4way conversation. 7his makes all the other choices incorrect. Ne*t, it is im!ortant to fin" out how much the !atient knows. It is $oo" to ask the !atient what she is thinkin$ about the sym!toms an" illness so far. 7hen it is critical to fin" out how much the !atient wants to know. )ome !atients "o not want to know the full "etails of their "ia$nosis, while others nee" to know every sin$le thin$. 7he fourth ste! is sharin$ the information an" e"ucatin$ the !atient. .ive the information in small !ieces an" fire a Hwarnin$ shotH. It is im!ortant to use (n$lish an" not Hme"s!eakH 'me"ical <ar$on) when talkin$ to a !atient. A !atient has no i"ea what HAuer ro"sH are 'choice A an" ) an" may not know if that is a $oo" or ba" thin$. It is also necessary to ask the !atient if they are followin$ the conversation. Fse "ia$rams, $ive written material an" listen to the !atientAs concerns. 7he fifth ste! is to res!on" to the !atientAs feelin$s. 7he si*th ste! is to summari#e the conversation an" make a !lan for the future. 6emainin$ stan"in$ an" tellin$ the !atient that there are Auer ro"s in her cells 'choice A) is incorrect because the !hysician is in the cafeteria 'not !rivate), stan"in$, an" usin$ Hme"s!eakH. )ittin$ "own an" tellin$ the !atient that there are Auer ro"s in her cells 'choice ) is incorrect because the !hysician is in the cafeteria an" usin$ Hme"s!eakH. )ittin$ "own an" tellin$ the !atient that she has acute myelocytic leukemia 'choice C) is incorrect because the !hysician is in the cafeteria. H,eukemiaH is an acce!table term, however the ste!s before this were i$nore". 7ellin$ her that the she has acute myeloi" leukemia but that she shoul" ask questions in !rivate 'choice () is incorrect because the "ia$nosis was $iven in the cafeteria an" ste!s ; an" 0 'fin"in$ out how much the !atient knows an" wants to know) were i$nore". ;D) (*!lanation: 7he correct answer is . 7he !atient has an anemia of a"vance" chronic renal failure. It shoul" not be sim!ly watche" 'choice A), as a hemo$lobin this low will cause a hi$h out!ut state with car"iac hy!ertro!hy. 7he !atient shoul" be starte" on erythro!oietin by the subcutaneous route. -olic aci" 'choice C) shoul" be $iven as an a""e" vitamin to assure $oo" marrow function but it is very unlikely to treat the anemia alone. 7his is es!ecially so $iven the normal +C1.

2e is alrea"y iron4loa"e" with a ferritin of over :>> n$Bm, an" an iron saturation of over ;>=. .ivin$ more iron 'choice %) will not hel! further. loo" transfusion 'choice () is not necessary in this !atient. ;@) (*!lanation: 7he correct answer is C. 7he association of "ementia with visual hallucinations an" e*tra!yrami"al si$ns is hi$hly characteristic of a ty!e of "ementin$ "isor"er variably referre" to as "iffuse ,ewy bo"y "isease or "ementia with ,ewy bo"ies. 7his con"ition has been i"entifie" as a well4 "efine" entity in recent times 'early @>s). At first consi"ere" a rare con"ition, it is now reco$ni#e" as one of the most frequent "ementin$ "isor"ers, secon" in inci"ence only to Al#heimer "isease in most hos!ital4base" e!i"emiolo$ical stu"ies. 7he mor!holo$ic substrate consists of intraneuronal inclusions i"entical to the ,ewy bo"ies in the substantia ni$ra of &arkinson !atients. %ementia with ,ewy bo"ies affects the substantia ni$ra 'thus e*!lainin$ the e*tra!yrami"al sym!toms), various neocortical fiel"s 'es!ecially the cin$ulate $yrus), an" subcortical nuclei, inclu"in$ the basal nucleus of +eynert an" amy$"ala. Another feature of this con"ition is its fluctuatin$ clinical course, with alternatin$ !erio"s of im!rovement an" "eterioration. Al#heimer4ty!e "ementia 'choice A) is characteri#e" by slowly !ro$ressive intellectual loss that !re"ominantly affects short4term memory. Neither visual hallucinations nor e*tra!yrami"al manifestations are !art of the clinical !icture of Al#heimer "isease. Creut#fel"t4Nacob "isease 'CN%) 'choice ) is cause" by an aberrant form of !rion !rotein '&r&). %ementia, mental status chan$es an" myoclonus are characteristic of CN%, as well as a ra!i"ly !ro$ressive course lea"in$ to "eath within C4:; months. %ementia associate" with motor neuron "isease 'choice %) is infrequent. %ementia "evelo!s in one tenth of !atients with motor neuron "isease. +emory "eficits are similar to those seen in Al#heimer "isease. &ick "ementia 'choice () is a rare form of "ementia, !re"ominantly affectin$ !resenile 'i.e., youn$er than C/) !atients. It is characteri#e" by strikin$ atro!hy of the frontal an" anterior tem!oral lobes. Clinically, "isinhibition an" !ersonality chan$es constitute the most !eculiar manifestations. ;I) (*!lanation: 7he correct answer is A. 7his is lichen sim!le* chronicus, also known as locali#e" scratch "ermatitis an" neuro"ermatitis. What basically ha!!ens is that the !atient $ets into a vicious cycle of

itchin$ causin$ scratchin$, which causes more itchin$. 7he initiatin$ cause may or may not be known or remembere", but the !rocess !ersists lon$ after the incitin$ !rocess has resolve". )tress an" tension may e*acerbate the !rocess. 7he a!!earance "escribe" in the question stem is ty!ical. 7reatment involves breakin$ u! the itch4scratch cycle, which may be "ifficult. 7o!ical steroi"s may be hel!ful, as may coverin$ the area with a "ressin$ to make scratchin$ more "ifficult. &om!holy* 'choice ) is a cause of marke" erythema of the !alms an" soles. &soriasis 'choice C) causes salmon4colore" !laques with silvery scale. )eborrheic "ermatitis 'choice %) causes "an"ruff an" cra"le ca!. )tasis "ermatitis 'choice () is seen at the ankles an" is relate" to varicose veins. 0>) (*!lanation: 7he correct answer is A. 7his !atient has the ty!ical sym!toms of con$estive heart failure with left ventricular failure. 7he !hysical e*amination has the characteristic fin"in$s for aortic insufficiency, inclu"in$ wi"e !ulse !ressure, blowin$ "iastolic murmur, an" a )0 consistent with left ventricular "ilatation. 7he soft systolic murmur is "ue to the volume overloa" an" increase" flow in the left ventricle an" "oes not im!ly an a""itional valvular "isor"er. Aortic stenosis 'choice ) !resents with a crescen"o4"ecrescen"o systolic murmur hear" at the aortic area an" often ra"iates to the caroti"s. It will ty!ically cause a hy!ertro!hic, !ressure4overloa"e" ventricle. +itral re$ur$itation 'choice C) !resents with a holosystolic blowin$ murmur hear" best at the a!e* an" ra"iatin$ to the a*illa. +itral stenosis 'choice %) !resents with a "iastolic rumblin$ murmur hear" at the a!e* of the heart. It is often also associate" with an o!enin$ sna!, which occurs after );. +itral stenosis is almost always the result of !revious rheumatic fever an" !resents with !ulmonary con$estion, while the left ventricle remains normal. 7ricus!i" re$ur$itation 'choice () is the lesion most ty!ically seen in !atients with acute bacterial en"ocar"itis after infection with )ta!hylococcus aureus. It is commonly seen in I1 "ru$ abusers. 0:) (*!lanation: 7he correct answer is . 7his !atient has fin"in$s that are !atho$nomonic for a $outy <oint, as "emonstrate" by the ne$atively birefrin$ent, nee"le4sha!e" crystals. A!!ro!riate thera!y is to start either

hi$h "oses of oral nonsteroi"al anti4inflammatory "ru$s 'N)AI%s) or colchicine. 2owever, colchicine in hi$h "oses is usually not well tolerate", so N)AI%s 'such as oral ibu!rofen) have emer$e" as the first4line thera!y for !rom!t sym!tomatic relief. 7here is no evi"ence of infection, so awaitin$ cultures 'choice A) or initiatin$ ceftria*one thera!y 'choice %) is incorrect. Althou$h corticosteroi"s are sometimes in<ecte" into $outy <oints, oral use of hi$h4"ose !re"nisone 'choice C) woul" not be in"icate". 7here is nothin$ by history to su$$est an osteomyelitis, so a ra"ionuclear bone scan 'choice () is not warrante". 0;) (*!lanation: 7he correct answer is . A !atient with chronic obstructive !ulmonary "isease 'CO&%) an" sus!ecte" lun$ carcinoma who becomes hoarse likely has metastatic "isease to the recurrent laryn$eal nerve. 7his in"icates e*tra!ulmonic s!rea" an" incurability by sur$ical means. 7he other sym!toms "escribe" can all be seen with an e*acerbation of his CO&% an" "o not necessarily im!ly any factors that woul" result in an incurable lun$ lesion. 7he only chance for a cure of a lun$ carcinoma is sur$ical resection. 'Chemothera!y an" ra"iation thera!y in !atients with oat cell carcinoma of the lun$ can cause remission, but these thear!ies are not curative.) &atients who have mali$nant !leural effusions, evi"ence of contralateral lym!ha"eno!athy or lun$ involvement, or any e*tra!ulmonic s!rea" are not consi"ere" sur$ically curable. A chan$e in the !attern of the !atientAs cou$h 'choice A), hemo!tysis 'choice ), !ostobstructive !neumonia 'choice %), an" worsenin$ "ys!nea 'choice () all su$$est cancer, but "o not s!eak to the question of whether the tumor is resectable. 00) (*!lanation: 7he correct answer is . Cushin$ syn"rome is a cluster of clinical abnormalities seen in the settin$ of chronic e*!osure to hi$h cortisol levels. 7runcal obesity, buffalo hum!, increase" susce!tibility to infection, hy!ertension, an" $lucose intolerance are common manifestations. Cushin$ syn"rome can be "ue to a"ministration of e*o$enous corticosteroi"s, a"renal hy!er!lasia or tumor, !ituitary a"enoma, or ecto!ic AC72 !ro"uction by another mali$nancy. 7he later is most commonly a small cell carcinoma of the lun$. ,un$ cancers are also notorious for a wi"e variety of other !araneo!lastic syn"romes, inclu"in$ hy!ercalcemia, hy!o!hos!hatemia, somatostatinoma syn"rome 'with !rominent .I com!laints), syn"rome of ina!!ro!riate anti"iuretic hormone secretion ')IA%2), (aton4,ambert syn"rome, !olymyositis, cerebellar "e$eneration, !eri!heral neuro!athy, osteoarthro!athy, marantic en"ocar"itis, mi$ratory thrombo!hlebitis, an" "isseminate" intravascular coa$ulation.

Cancers of the colon 'choice A) can cause .I sym!toms an" anemia relate" to bloo" loss, but are not a !rominent cause of en"ocrine4relate" !araneo!lastic syn"romes. Islet cell cancers of the !ancreas 'choice C) can secrete clinically si$nificant amounts of insulin, $astrin, $luca$on, an" vasoactive intestinal !e!ti"e, but "o not usually secrete AC72 or corticosteroi"s. Cancers of the stomach 'choice %) can cause sym!toms such as <aun"ice an" ascites relate" to "irect e*tension into !eri$astric sites, but are not a source of en"ocrine4relate" !araneo!lastic syn"romes. While some testicular cancers 'choice () can secrete tiny amounts of hormones such as human chorionic $ona"otro!in 'hC.), clinically si$nificant en"ocrine "isease "oes not usually occur. 0?) (*!lanation: 7he correct answer is %. 7he !atient has rhinocerebral mucormycosis, which can be cause" by fun$al s!ecies inclu"in$ 6hi#o!us, 6hi#omucor, Absi"ia, an" asi"iobolus. &re"is!osin$ con"itions inclu"e immunosu!!ression, uncontrolle" "iabetes mellitus, an" !atients usin$ the iron4 chelatin$ "ru$ "esferrio*amine. While these infections occasionally a!!ear more or less inci"entally, as in this case, they are very im!ortant to "ia$nose because they have a ten"ency to become fulminant. 7he lesions ten" to be very locally "estructive an" can ero"e into the eye, !alate, an" central nervous system, often from an initial site in the sinuses. -ulminant infections are frequently fatal. &ulmonary infections can also occur. 7he or$anism can be "ifficult to culture. 7he a!!ro!riate antibiotic is I1 am!hotericin , but sur$ical "ebri"ement shoul" also be stron$ly consi"ere", since !enetration of antibiotic into necrotic tissues may be !oor. As!er$illus'choice A) can also cause sinusitis, but has narrow hy!hae. lastomyces'choice ) usually involves the lun$ an" occurs as a yeast form in the bo"y. Can"i"a'choice C) can infect sinuses, but has narrow hy!hae an" yeast forms. )!orothri*'choice () usually infects the skin an" subcutaneous tissues an" occurs as a yeast. 0/) (*!lanation: 7he correct answer is %. Acetamino!hen over"ose may result in massive he!atocellular necrosis an" fulminant he!atic failure. ,iver metabolism of acetamino!hen by &4?/> cytochrome !ro"uces a hi$hly to*ic interme"iate, which "e!letes $lutathione stores, resultin$ in accumulation of

free ra"icals. In the first few hours after in$estion, the !atient may have nausea an" vomitin$, but no si$ns of liver failure. iochemical evi"ence of he!atic "ama$e, namely elevation of transaminases, be$ins ;?4?@ hours after in$estion. 7hera!y, therefore, shoul" be starte" before he!atic transaminases be$in to increase. 7he likelihoo" of he!atic to*icity is base" on a combination of serum acetamino!hen levels an" time ela!se" from in$estion. Nomo$rams have been "evelo!e" to "etermine the likelihoo" that a s!ecific level of acetamino!hen at a $iven time followin$ in$estion will !ro"uce he!atoto*icity. Within ? hours followin$ in$estion, the initial treatment shoul" inclu"e activate" charcoal 'to a"sorb resi"ual "ru$ in the intestine) an" N4acetylcysteine. 7reatment with N4 acetylcysteine is most effective if starte" within :> hours after in$estion. 7he -%A4 a!!rove" recommen"ation is to continue oral treatment for D; hours. Activate" charcoal treatment 'choice A) alone is not sufficient to !revent the to*ic effects of acetamino!hen that has been alrea"y absorbe". Charcoal a"sorbs almost all "ru$s an" !oisons in the $ut. It is effective in the first few hours after acetamino!hen in$estion an" "oes not interfere with a"ministration of acetylcysteine. Acetylcysteine treatment 'choice ) is use" to counteract he!atic to*icity of acetamino!hen. In the first few hours after in$estion, it shoul" be combine" with a"ministration of activate" charcoal to !revent further intestinal absor!tion of the "ru$. &enicillamine 'choice C) is a chelator use" for treatment of acute arsenic !oisonin$. &enicillamine shoul" be a"ministere" after "imerca!rol ' A,). .astric lava$e 'choice () is effective within a!!ro*imately : hour of in$estion of !oisons or "ru$s. It is in"icate" when in"uce" emesis is refuse", unsuccessful, or contrain"icate". 0C) (*!lanation: 7he correct answer is (. 7his !atient has evi"ence of a !rolon$e" e!iso"e of biliary colic, which may !ro$ress to acute cholecystitis. An ab"ominal ultrasoun" has reveale" a marke"ly "ilate" common bile "uct, as well as $allstones within the $allbla""er. Acute cholecystitis is the result of cystic "uct obstruction, an" this woul" be best "emonstrate" by a 2I%A scan. A 2I%A scan measures flow of imino"iacetic aci", which follows the flow of bile, i.e., from the bloo" stream into the liver, into the $allbla""er, an" then throu$h the cystic "uct into the common bile "uct. If there is cystic "uct obstruction, the common bile "uct is not visuali#e" on the 2I%A scan. An ab"ominal C7 scan 'choice A) may reveal $allstones an" !ericholecystic chan$es but "oes not "emonstrate $oo" visuali#ation of the cystic "uct. Ab"ominal +6I 'choice ) is not effective in the "emonstration of cystic "uct obstructions.

An ab"ominal ultrasoun" 'choice C) is not as effective in revealin$ cystic "uct obstructions, an" %o!!ler flow stu"ies are useful for bloo" flow !atterns but woul" not assist in the "ia$nosis of acute cholecystitis. An en"osco!ic retro$ra"e cholan$io!ancreato$ra!h '(6C&) 'choice %) is more effective in "emonstratin$ the intra4 an" e*trahe!atic biliary trees but is not as effective in "emonstratin$ the cystic "uct. 0D) (*!lanation: 7he correct answer is A. Car!al tunnel syn"rome in "ialysis !atients is usually "ue to the "e!osit of beta4; micro$lobulin, which causes "ialysis4associate" amyloi". 7his can cause osteoarterial amyloi". 7here is no evi"ence to su!!ort any of the other choices here as a cause of the car!al tunnel syn"rome. Occult hy!othyroi"ism 'choice ) coul" cause car!al tunnel syn"rome but the normal 7)2 makes this "ia$nosis unlikely. 0@) (*!lanation: 7he correct answer is (. 7he !atientAs +6I scan "oes not reveal any si$nificant cortical atro!hy. Cortical atro!hy manifests with narrowin$ of $yri an" wi"enin$ of sulci. As a secon"ary effect, enlar$ement of the ventricles takes !lace, a feature known as hy"roce!halus e* vacuo. 7he ventricles are normal in this case. 7he !atientAs behavior, lack of interest, an" verbal re!lies su$$est a "ia$nosis of "e!ression. (l"erly !eo!le with this con"ition often !resent with co$nitive "eficits, which may constitute the !re"ominant clinical manifestation. %e!resse" !atients ten" to re!ly HI "onAt knowH to questions aime" at testin$ memory skills. A "ia$nosis of !seu"o"ementia is a!!licable whenever severe co$nitive "eficits are not e*!laine" by ob<ective fin"in$s by clinical, ra"iolo$ic, or !atholo$ic investi$ations. %e!ression is certainly an im!ortant cause to consi"er in the "ifferential "ia$nosis of "ementin$ "isor"ers. Al#heimer "isease 'choice A) an" &ick "ementia 'choice %) are e*clu"e" not only by the clinical !icture but also by the lack of cortical atro!hy on +6I. Whereas Al#heimer "ementia is characteri#e" by "iffuse cortical atro!hy, &ick "isease has a s!ecific !re"ilection for the frontal an" tem!oral lobes. rain tumor 'choice ) may be rule" out by the neuroima$in$ stu"ies. Althou$h brain tumors may manifest with slowly !ro$ressive memory "eficits mimickin$ "ementia 'e.$., menin$iomas), they are usually associate" with focal neurolo$ic si$ns an" sym!toms. Normal !ressure hy"roce!halus 'choice C) is a rare form of treatable "ementin$ illness "ue to recurrent transient increases in C)- !ressure, with resultant !ro$ressive

enlar$ement of ventricles. 7he ventricular cavities a!!ear entirely within normal limits in this case. 0I) (*!lanation: 7he correct answer is . 7he !atientAs systolic crescen"o4"ecrescen"o murmur su$$ests either aortic or !ulmonary valve stenosis. ,ocali#ation of the murmur to the ri$ht u!!er sternal bor"er, cou!le" with the !atientAs sym!toms of an$ina, s!ecifically su$$ests that the affecte" valve is the aortic rather than the !ulmonary. 7his !atient has sym!toms of an$ina althou$h his coronary arteries may be entirely normal. 7his is because of the increase" "eman" !lace" on the heart owin$ to the increase" afterloa" an" hy!ertro!hie" left ventricle seen in !atients with aortic stenosis. In this a$e $rou!, the most common etiolo$y is a calcific aortic valve. An echocar"io$ram is the most effective test at "emonstratin$ the valvular lesion, cross4sectional aortic valve "iameter, an" severity of left ventricular hy!ertro!hy. An e*ercise stress test 'choice A) may be a!!ro!riate if the !atient ha" coronary artery risk factors an" may be a!!ro!riate in this !atient after the echocar"io$ram. With the fin"in$ of severe aortic stenosis, the first assessment shoul" involve characteri#ation of aortic valve area. A thallium stress test 'choice C) is a nuclear me"icine ima$in$ stu"y that evaluates for Hhy!o!erfuse" areasH "urin$ e*ercise. Its chief value is in !atients who have an abnormal baseline (C. that interferes with the inter!retation of (C. stress testin$ 'e.$., un"erlyin$ left ventricular hy!ertro!hy, !rior myocar"ial infarction, or left bun"le branch block). Car"iac catheteri#ation 'choice %) is a!!ro!riate in the evaluation of aortic stenosis if sur$ery is bein$ contem!late". It allows !recise quantification of the aortic valve area, left ventricular function, an" any co4e*istin$ coronary artery "isease. A !ulmonary artery ca!illary we"$e !ressure tracin$ 'choice (), i.e., )wan4.an# catheteri#ation, is of value in the measurement of left ventricular en" "iastolic fillin$ !ressures, as well as the car"iac out!ut. 2owever, is not in"icate" in the out!atient assessment in an in"ivi"ual sus!ecte" of havin$ aortic stenosis. ?>) (*!lanation: 7he correct answer is (. 7he clinical !icture is "ia$nostic of "ermatomyositis. Women are more frequently affecte" than men. +uscle weakness an" !ain, alon$ with the characteristic !eriorbital e"ema an" !ur!le "iscoloration of the u!!er eyeli"s, su$$est the "ia$nosis, which is confirme" by muscle bio!sy. 7he !atches on the "orsal as!ects of inter!halan$eal an"

metacar!o!halan$eal <oints are referre" to as .ottron si$n. Anti4nuclear antibo"ies 'ANAs) are foun" in the ma<ority of !atients, an" rheumatoi" factor is !ositive in a minority. Corticosteroi"s !rovi"e ra!i" relief in most !atients. &re"nisone is a"ministere" at hi$h "oses '?>4C> m$ "aily). A#athio!rine 'choice A) an" methotre*ate 'choice ) are both effective, but they may cause serious a"verse effects. 7hus, they shoul" be use" only in !atients intolerant or unres!onsive to corticosteroi"s. N)AI%s 'choice C) are not sufficient to control the "isease. Often, !atients with "ermatomyositis e*!erience e*acerbations when corticosteroi"s are with"rawn. &lasma e*chan$e 'choice %), as well as leuka!heresis, is not effective in treatin$ "ermatomyositis. ?:) (*!lanation: 7he correct answer is A. 7his !atient !resents with the classic tria" of shortness of breath, non!ro"uctive cou$h, an" whee#in$, all of which is consistent with bronchos!asm. 7he mechanisms involve" in an asthmatic attack are bronchos!asm an" airway obstruction "ue to reversible inflammation. 7he other answer choices !ro"uce res!iratory !roblems but vary in their sym!tomatolo$y. ,aryn$eal s!asm 'choice ) is usually an aller$ic !henomenon an" !resents with stri"or an" lou" au"ible whee#in$. A !leural effusion 'choice C) ty!ically !resents with !leuritic chest !ain, usually locali#e" to one hemithora*. &ulmonary con$estion 'choice %) !resents with shortness of breath but $enerally has a cou$h that is !ro"uctive of s!utum an" is often !ink an" frothy. A ventilation4!erfusion mismatch 'choice () may occur for various reasons, e.$., !ulmonary embolus, !neumonia, an" con$estive heart failure. It !ro"uces shortness of breath but "oes not !resent with the !articular !resentation an" tria" of sym!toms that this !atient "escribes. ?;) (*!lanation: 7he correct answer is %. 7an$ier "isease is a rare familial "isor"er characteri#e" by al!ha4li!o!rotein "eficiency, which lea"s to very low hi$h4"ensity li!o!rotein '2%,), recurrent !olyneuro!athy, lym!ha"eno!athy, an" he!atos!lenome$aly "ue to stora$e of cholesterol

esters in reticuloen"othelial cells. Althou$h you may never see this "isease, the association of oran$e4yellow tonsillar hy!er!lasia '"ue to the cholesterol ester "e!osition there as well) with 7an$ier "isease is a sufficiently "istinctive clue in !hysical "ia$nosis to be worth rememberin$. 'One rule of thumb in me"icine is that althou$h you will !robably never see most of the very rare "iseases, you will almost certainly see some of them.) 7he !resence of multi!le an$iokeratomas 'choice A) on the lower half of the bo"y su$$ests -abry "isease. .rey4brown !i$mentation of the forehea", han"s, an" !re4tibial re$ion 'choice ) su$$ests .aucher "isease. Irre$ular black "e!osits of clum!e" !i$ment in the !eri!heral retina 'choice C) are characteristic of retinitis !i$mentosa, which may occur in association with abetali!o!roteinemia an" 6efsum "isease. &in$ueculae 'choice () coul" su$$est .aucher "isease, but this coul" also occur in normal a"ults. ?0) (*!lanation: 7he correct answer is . 7he concern for !otential abuse must be a""resse" as any other new !iece of clinical sus!icion. 7he best initial ste! is to try to $ather more information in a non4threatenin$ way that is comfortable for the !atient. Althou$h the !hysician will ultimately want to ask her "irectly if she is bein$ abuse", it is often better to let that "isclosure Hfall outH from a "iscussion about the !articulars of the relationshi!. Althou$h !hysicians often feel the "esire to Htake thin$s into their own han"sH an" a""ress alle$e" abusers "irectly 'choice A), this a!!roach can be e*tremely "an$erous for the abuse" !arty. Abusers obviously "rea" bein$ "iscovere" an" are likely to seek retribution a$ainst their victim if they are confronte" by a "octor. (*!lainin$ that her marria$e is not havin$ a !ositive im!act on her life 'choice C) is !resum!tuous without first e*!lorin$ all of the !articulars of the relationshi!. &hysicians in most states are obli$ate" to re!ort 'choice %) !otential abuse only in cases involvin$ chil"ren youn$er than :@ an" a"ults ol"er than C/. 7here is no obli$atory re!ortin$ for "omestic abuse not fallin$ within these !arameters. Althou$h the !atient may res!on" very well to treatment from a !sychiatrist who s!eciali#es in "omestic abuse 'choice (), this referral, like any other, shoul" be ma"e after an earnest attem!t to $ather the clinical an" social "etails from the !atient by the referrin$ !hysician.

??) (*!lanation: 7he correct answer is (. 7he most likely !harmacolo$ic a$ent res!onsible for the !atientAs hy!erkalemia is s!ironolactone. )!ironolactone has an anti4al"osterone effect an" therefore !re"is!oses a !atient to hy!erkalemia. '6emember that hy!erkalemia is an im!ortant stimulus for the secretion of al"osterone.) Acetamino!hen 'choice A) can !re"is!ose susce!tible !atients to renal failure throu$h tubular in<ury 'acetamino!hen is a metabolite of !henacetin, a known tubular to*in), but it is not ty!ically associate" with an increase" risk for hy!erkalemia. Clomi!hene 'choice ), a !artial estro$en anta$onist use" in the treatment of the anovulatory com!onent of !olycystic ovarian "isease, is not ty!ically associate" with an elevate" risk for hy!erkalemia. 2y"ro*ychloroquine 'choice C), an antimalarial a$ent use" in the treatment of rheumatoi" arthritis, is not ty!ically associate" with an elevate" risk for hy!erkalemia. Ibu!rofen 'choice %), alon$ with other nonsteroi"al anti4inflammatory "ru$s 'N)AI%s), !re"is!oses !atients to $astritis an" $astric ulcer "isease by re"ucin$ !rosta$lan"in formation 'com!oun"s that are !rotective of the $astric mucosa throu$h a variety of mechanisms) throu$h the inhibition of the cycloo*y$enase !athway. ?/) (*!lanation: 7he correct answer is . Cytome$alovirus may "evelo! in 2I1 !atients, ty!ically when the C%? count falls below />. It will !resent as an acute colitic !rocess with left lower qua"rant !ain, bloo"y "iarrhea, an" !rominent sym!toms of ur$ency an" tenesmus. 7he si$moi"osco!y "escribe" here is characteristic but not s!ecific for this infection. Crohn "isease 'choice A) rarely !resents in 2I1 !atients because it is an autoimmune "isease an" will !resent in a subacute or chronic fashion. In a""ition, it will most likely !resent with ri$ht lower qua"rant sym!toms, su$$estive of ileitis. Althou$h (ntamoeba histolytica'choice C) an" )almonella enteriti"is'choice %) infections are more common in men havin$ se* with men, the !roctosi$moi"itis an" !rominent sym!toms of tenesmus an" ur$ency are more su$$estive of cytome$alovirus infection. Flcerative colitis 'choice () rarely !resents in 2I1 !atients, as it is an autoimmune "isease. -urthermore, it "oes not !resent as an acute illness as "escribe" here.

?C) (*!lanation: 7he correct answer is %. 7he !atient most likely has "etrusor instability or "yssyner$ia. )he "oes not seem to have a neurolo$ic, infectious, or systemic 'e. $. , "iabetes) "isor"er. )ince she can tolerate <o$$in$ without a !roblem, she !robably "oes not have stress4in"uce" incontinence ')FI). 7hus, the "ia$nosis will !robably be ma"e by cystometry to assess her uro"ynamics. 7his test is also use" to evaluate bla""er ca!acity an" tone. An I1& 'choice A) woul" be use" in a !atient with renal failure. It woul" hel! "elineate the cause of re"uce" urinary out!ut. (ssentially, a "ye is in<ecte", an" films are taken to track the u!take in the ki"neys. 7his woul" not hel! in the evaluation of incontinence. )tress testin$ 'choice ) reveals leaka$e of urine coinci"ent with increase" intra4 ab"ominal !ressure, which is "ia$nostic of stress urinary incontinence. 7he 84ti! test 'choice C) is an in"irect measure of the urethral a*is 'an$le of inclination). A 84ti! is inserte" into the urethra with the !atient in the lithotomy !osition. If the 84ti! moves more than 0> "e$rees from the hori#ontal, there is abnormal urethral mobility. 7his woul" be hel!ful in the evaluation of stress urinary incontinence, since it woul" hel! in"icate a "istortion between the urethra an" the bla""er. Cystosco!y 'choice () is !erforme" to e*amine the bla""er an" urethral mucosa for "iverticula or if neo!lasia is sus!ecte". 7his woul" be nee"e" to evaluate for inflammation if the !atient were com!lainin$ of increase" urinary frequency an" ur$ency or hematuria. ?D) (*!lanation: 7he correct answer is . 7he clinical history, neurolo$ic evaluation, an" +6I fin"in$s are consistent with "ementia of the Al#heimer ty!e WAl#heimer "isease 'A%)X. 7he "ia$nosis of A% is mostly base" on clinical !arameters, i.e., history an" careful neurolo$ic evaluation, inclu"in$ mental status. Neuroima$in$ stu"ies are usually useful in the "ifferential "ia$nosis of "ementia. 1entricular "ilatation in A% is "ue to hy"roce!halus e* vacuo, which is secon"ary to atro!hy of the brain. 7here is no available laboratory test to "efinitely confirm or e*clu"e this con"ition. In uncertain cases, a brain bio!sy may be !erforme", which may "emonstrate senile !laques an"Bor neurofibrillary tan$les in the corte*. )hort4term memory "eficits an" "isorientation are early features of A %. 7hese !ro$ressively worsen until the !atient is unable to reco$ni#e the most familiar faces an" ob<ects or !erform the sim!lest "aily tasks. &atients with this con"ition eventually "ie be"ri""en of infectious com!lications, such as !neumonia an" se!sis.

Chronic sub"ural hematoma 'choice A) may in"ee" manifest with a "ementin$ !icture, but is "etectable on C7B+6I as a sub"ural mass. 7his con"ition most frequently affects el"erly !atients, in whom cerebral atro!hy may result in the stretchin$ of Hbri"$in$H veins. 2ea" trauma, sometimes of minimal severity, may lea" to ru!ture of these veins. Normal4!ressure hy"roce!halus 'choice C) is characteri#e" by marke" ventricular "ilatation in the absence of si$nificant cortical atro!hy. 7his is an infrequent yet im!ortant cause of "ementia. 7he sym!tomatolo$y of normal !ressure hy"roce!halus, in fact, may resolve with C)- shuntin$ 'e.$., ventriculo!eritoneal shuntin$) in the early sta$es. &arkinson "isease 'choice %) is cause" by "e$eneration of the "o!aminer$ic neurons in the substantia ni$ra. It manifests with the ty!ical tria" of restin$ tremor, akinesia, an" ri$i"ity. It res!on"s to treatment with ,4%O& A. &ick "ementia 'choice () is characteri#e" clinically by flat emotional affect an" lan$ua$e "isturbances. +6I shows atro!hy of the frontal an" anterior tem!oral corte*, with s!arin$ of the remainin$ lobes. A bio!sy of the frontal corte* will show neuron loss with intracyto!lasmic ar$yro!hilic inclusions. ?@) (*!lanation: 7he correct answer is A. 7his !atient has sta$e I hy!ertension, "efine" as a systolic !ressure of :?>4:/I mm 2$, an"Bor a "iastolic !ressure of I>4II mm 2$ on at least two se!arate rea"in$s after the initial screenin$. A!!ro*imately I>= to I/= of hy!ertension cases are i"io!athic. 6ecent stu"ies have shown that e*cessive alcohol consum!tion mi$ht be the most common, an" often un"erestimate", cause of secon"ary hy!ertension. 7herefore, in evaluatin$ hy!ertensive !atients, the !hysician shoul" always inquire about alcohol consum!tion. Althou$h mo"est amounts of alcohol 'two "rinksB"ay) have been shown to lower the risk of car"iovascular "isease, alcohol in$estion in e*cess of two "rinks "aily are !ositively correlate" with increase" risk of hy!ertension an" atherosclerotic "isease. (*cessive alcohol in$estion may also e*!lain the a!!arent refractoriness of some hy!ertensive !atients to stan"ar" thera!ies. +easurement of urinary catecholamines 'choice ) is an a!!ro!riate investi$ation in the !resence of si$ns an" sym!toms su$$estive of !heochromocytoma. 7his accounts for a very small !ercenta$e of cases of hy!ertension seen in !rimary care '!robably only : in :>,>>>). 7he absence of e!iso"es of !al!itations, sweatin$, an" tremor ar$ues a$ainst this hy!othesis. Antihy!ertensive treatment with an AC( inhibitor 'choice C), "iuretic 'choice %), or beta4blocker 'choice () may be consi"ere" once i"entifiable causes of secon"ary

hy!ertension have been e*clu"e" an" lifestyle mo"ifications have !rove" insufficient to control bloo" !ressure. ?I) (*!lanation: 7he correct answer is (. A monoarticular arthritis with an acute onset, such as this !atient has, shoul" raise the question of an acute infectious arthritis. In this re$ar", the ma<or "ifferential is between )ta!hylococcus an" .onococcus. 7he .onococcus, Neisseria $onorrhea, is usually obtaine" throu$h se*ual contact an" is now the most common cause bacterial arthritis in a"ults. 7he or$anism s!rea"s from infecte" mucosal surfaces to <oints, with the small <oints of the han"s, wrists, knees, an" ankles the most commonly affecte". Involvement of the a*ial skeleton is uncommon. &atients with $onococcal arthritis may "emonstrate features of a "isseminate" $onococcal infection with a /4D "ay history of fever an" shakin$ chills. A variety of skin lesions may also be !resent, inclu"in$ !etechiae, !a!ules, !ustules, hemorrha$ic bullae, or necrotic lesions. A history of mi$ratory arthral$ias an" tenosynovitis may also !rece"e the "evelo!ment of !ersistent inflammatory arthritis in one or a few <oints. 6heumatoi" arthritis 'choice A) rarely !resents with a monoarticular arthritis an" is uncommon in men. Althou$h a bacterial $astroenteritis 'choice ) "oes not cause a monoarticular arthritis, it may cause 6eiter syn"rome, with "iffuse arthral$ias an" con<unctivitis. 2e!atitis 'choice C) may be associate" with a serum sickness4like illness, which affects multi!le <oints with arthral$ias. F!!er res!iratory tract infections 'choice %) are not associate" with monoarticular arthritis. />) (*!lanation: 7he correct answer is C. We$ener $ranulomatosis is i"entifie" by the classic clinical tria" of u!!er an" lower res!iratory involvement, su!!orte" by a !ositive antineutro!hilic cyto!lasmic antibo"y 'ANCA) test. 7he !atholo$ic lesion is an an$iitis of small vessels with characteristic tissue necrosis surroun"e" by mononuclear inflammatory cells, formin$ noncaseatin$ $ranulomas. In the lun$, this !rocess results in e*cavation an" "estruction of the lun$ !arenchyma. 7he renal lesion is a focal $lomerulone!hritis that can !ro$ress to renal failure. )arcoi"osis is a multisystem "isease that involves the lun$ in I>= of cases. Cou$h is non!ro"uctive, an" hemo!tysis is rare. An elevate" an$iotensin convertin$ en#yme 'AC() level 'choice A) has been associate" with this illness.

.oo"!asture syn"rome is a !ro$ressive autoimmune "isease of the lun$s an" ki"neys that !ro"uces intra4alveolar hemorrha$e an" $lomerulone!hritis. It is cause" by an anti$lomerular basement membrane 'anti4. +) antibo"y 'choice ), usually I$., that reacts with $lomerular an" alveolar basement membranes. )ystemic illnesses such as sclero"erma are associate" with eosino!hilia 'choice %). 7his may lea" to lun$ involvement an" ultimately cause !ulmonary hy!ertension. 2istiocytosis E is a $eneric term for a $rou! of systemic "isor"ers characteri#e" by various "e$rees of fibrosis with focal infiltrations of tissue by nonmali$nant histiocytes an" eosino!hils. (osino!hilic $ranuloma of the lun$ is locali#e" an" may cause "ys!nea an" a cou$h. &atholo$y shows cyto!lasmic inclusions, the so4calle" E bo"ies 'choice ().

USMLE Step 2 Practice Test Block < :ame; 1. @#ring a camping trip% a 12!year!ol" $oy sco#t is $itten $y a rattlesnake on his right ankle. Pain an" s'elling of the $itten area $ecome rapi"ly intense. =hile the $oy is transporte" to the closest me"ical facility% 'hich of the follo'ing is the most appropriate initial step in managementa6Apply ice to the $itten part $6Apply a to#rni*#et c6Ci)e alcohol to the patient "6+mmo$iliDe the $itten part in horiDontal position e6Perform incision an" s#ction on the $ite :ormal La$s 2.A 2!year!ol"% o$ese African American 'oman is a"mitte" to the hospital for an electi)e gastric $ypass operation for 'eight re"#ction. >n post!op "ay 1% the patient ac#tely "e)elops tachypnea% tachycar"ia% an" an o.ygen re*#irement of 4 L. She complains of left!si"e" ple#ritic chest pain. =hich of the follo'ing is the most appropriate "iagnostic st#"ya62hest .!ray

$62T scan c6E2C 'ith rhythm strip "6MA+ e6 ?entilation!perf#sion scan :ormal La$s &. A <2!year!ol" social 'orker complains of increasing "iffic#lty 'ith s'allo'ing. She has ha" tro#$le 'ith soli" foo"s an" senses a "iscomfort in the mi"!lo'er chest after eating meats or "ry $rea". She has not lost any 'eight an" "enies any other me"ical pro$lems. There is no family history of gastrointestinal malignancy. The physical e.amination is #nremarka$le. =hich of the follo'ing 'o#l" $e the most appropriate ne.t step in the e)al#ation of this patient,s symptomsa6Bari#m esophagram $62hest .!ray c62T scan of the chest "6Esophagoscopy e6 24!ho#r p/ monitor :ormal La$s 4. A &4!year!ol" man is seen for e)al#ation of nephrotic syn"rome. /e is not "ia$etic or hypertensi)e. /e is a former #ser of in9ecta$le "r#gs 3 0skin popper06. /e "enies arthralgia% rash% or hemat#ria. /is $loo" press#re is 1447<4 mm /g. >n e.amination% the patient has anasarca% an" a large% palpa$le li)er. M#ltiple ol" skin #lcers are note" on $oth lo'er e.tremities.La$oratory analysis sho's; 2reatinine 1.< mg7"L Potassi#m &.( mE*7L Urea nitrogen 24 mg7"L A 24!ho#r #rine contains 12 grams of protein 'ith no casts. A renal #ltraso#n" re)eals echogenic ki"neys% each meas#ring a$o#t 1& cm. An /+? test is negati)e. =hich of the follo'ing is the most likely "iagnosisa6 A+@S nephropathy $6Amyloi" nephropathy c6/eroin nephropathy "6Minimal change "isease

e6 Poststreptococcal glomer#lonephritis :ormal La$s

.A pre)io#sly healthy < !year!ol" man comes to me"ical attention $eca#se of increasingly se)ere memory "ist#r$ances% loss of $alance% an" #rinary incontinence for 14 months. /is )ital signs are normal. Mini!mental stat#s e.amination sho's mil"!to!mo"erate short!term memory "eficits. La$oratory screening tests are 'ithin normal limits. Papille"ema is a$sent on f#n"#scopic e.amination. Magnetic resonance imaging re)eals "ilate" )entric#lar spaces% 'hile the cortical mantle is normal% 'itho#t 'i"ening of s#lci or narro'ing of gyri. The 'hite matter appears #nremarka$le% 'ith no e)i"ence of "emyelination. A l#m$ar p#nct#re yiel"s the follo'ing )al#es; 2S5 press#re............124 mm /24 2ell co#nt.................& lymphocytes7mm& Cl#cose.................... 4 mg7"L Proteins% total............28 mg7"L =hich of the follo'ing is the most appropriate ne.t step in managementa6 2#lt#re of 2S5 $6Trial 'ith cholinesterase inhi$itors c6Treatment 'ith anti"epressant "r#gs "6Treatment 'ith le)o"opa e62S5 sh#nting proce"#re :ormal La$s <.A 21!year!ol" professional "ancer complains of se)eral episo"es of near loss of conscio#sness "#ring a performance. She has $een in e.cellent health an" is a principal "ancer in the :e' Uork 2ity Ballet 2orps. She has no family history of coronary artery "isease. She "oes not smoke% an" a recent cholesterol profile 'as normal. >n physical e.amination% her $loo" press#re is 142711 mm /g% an" her p#lse is 147min an" reg#lar. She has a $risk caroti" #pstroke 'ith a "o#$le imp#lse palpa$le. She has a lo#" S4 an" a harsh systolic m#rm#r hear" along the left sternal $or"er. The m#rm#r is accent#ate" "#ring the ?alsal)a mane#)er. An E2C re)eals se)ere left )entric#lar

hypertrophy. =hich of the follo'ing is the most appropriate me"ication in the management of this patienta6 2aptopril $6@igo.in c6@iltiaDem "65#rosemi"e e6Propranolol :ormal La$s (. A 18!year!ol" 'oman cons#lts an allergist a$o#t constant nasal st#ffiness she e.periences. She has a )ariety of pets in her ho#se% incl#"ing fish% "ogs% an" a cat. Screening tests "emonstrate $loo" ele)ations of eosinophils an" +gE. The patient #n"ergoes e.tensi)e skin testing% 'hich "emonstrates marke" sensiti)ity to cat "an"r#ff. =hich of the follo'ing is the preferre" treatment for her cat allergya6>ral chlorpheniramine maleate $62romolyn nasal spray c6+M "iphenhy"ramine /2L "6Allergen imm#notherapy e6Cet ri" of the cat :ormal La$s 1. A 82!year!ol" man 'ith a 4 !year history of chronic o$str#cti)e p#lmonary "isease is int#$ate" in the intensi)e care #nit $eca#se of a $o#t of )iral pne#monia that fails to impro)e after (2 ho#rs of anti$iotics. Altho#gh the inspire" fraction of o.ygen is 144B% the patient,s p>2 remains at ( mm /g. Positi)e!en" e.piratory press#re 3PEEP6 is a""e" to allo' the inspire" fraction of o.ygen. T'el)e ho#rs after the intro"#ction of PEEP the patient s#""enly $ecomes hypotensi)e. At the same time% his o.ygen sat#ration "rops from 82B to <1B. >n physical e.amination% his $loo" press#re is 147 4 mm /g% an" his p#lse is 1247min. /e ha" "isten"e" neck )eins an" "istant heart so#n"s. =hich of the follo'ing 'o#l" also most likely $e seen on this patient,s physical e.aminationa6 A$sence of $reath so#n"s in the right hemithora. $6/igh amplit#"e caroti" artery #pstroke c6 A ple#ral friction r#$ "6P#ls#s alternans

e6Splenomegaly :ormal La$s

8.A 2 !year!ol" 'oman comes to the physician 4 'eeks after an #ne)entf#l "eli)ery $eca#se of increasing irrita$ility% an.iety% an" heat intolerance. /er temperat#re is &(.2 2 388 56% $loo" press#re is 1 47< mm /g% p#lse is 1447min% an" respirations are 147min. Palpation of the neck re)eals no thyroi" enlargement or ten"erness. /er skin is 'arm an" moist% an" a fine han" tremor is note". =hich of the follo'ing is the most likely "iagnosisa6Cra)es "isease $6/ashimoto thyroi"itis c6Postpart#m thyroi"itis "6Aie"el thyroi"itis e6 Sheehan syn"rome f6S#$ac#te 3"e N#er)ain6 thyroi"itis :ormal La$s 14.A &1!year!ol" 'oman complains of se)ere lo' $ack pain after playing tennis for 4 ho#rs. She $egins taking i$#profen e)ery & ho#rs $#t fin"s little relief. She is prescri$e" co"eine for her pain relief. /er physician also prescri$es misoprostol% since the patient has a history of a $lee"ing gastric #lcer 1 year earlier after taking i$#profen. She has also $een taking s#matriptan for fre*#ent migraine hea"aches. =hich of the follo'ing is the most appropriate a")ice for this patienta6 A)oi" "ietary potassi#m $6+ncrease "ietary ro#ghage c6Use an effecti)e metho" of contraception "6+ncrease her "aily folate intake e6+ncrease her "aily aero$ic e.ercise acti)ity :ormal La$s

11. A 2(!year!ol" man is a"mitte" to the hospital for lethargy an" conf#sion. The patient has a !year history of se)ere hypertension that has $een refractory to me"ical therapy. >)er the past fe' "ays% the man has $ecome increasingly lethargic an" conf#se". >n the "ay of a"mission% he 'as fo#n" in his home% lying on the floor an" non! responsi)e. /is roommate calle" EMS% an" the patient 'as $ro#ght emergently to the hospital. La$oratory )al#es are as follo's; Ser#m so"i#m...........112 mE*7L Ser#m potassi#m....... .1 mE*7L Ser#m osmolality......2&4 m>smol7kg /24 Urine so"i#m............4 mE*7L =hich of the follo'ing s#$stances is most likely me"iating the hyponatremiaa6Al"osterone $6Anti"i#retic hormone 3A@/6 c6 2ortisol "6Somatostatin e6 Thyroi" stim#lating hormone 3TS/6 :ormal La$s 12. A 21!year ol" college senior presents 'ith a 2!month history of fre*#ent episo"es of loose stool% prece"e" $y lo'er a$"ominal cramping. >)er the past 4 'eeks% the stools ha)e $ecome increasingly $loo"y. >n a n#m$er of occasions he has the sensation of rectal f#llness $#t is #na$le to pass any fecal matter. A sigmoi"oscopy is performe" an" re)eals inflammation in a circ#mferential pattern from the anal )erge to the mi"!sigmoi" colon% 'here a transition to normal m#cosa is seen. =hich of the follo'ing is the most appropriate treatment for this patienta6 +? hy"rocortisone $6+? infli.ima$ c6>ral aDathioprine "6>ral pre"nisone e6Topical mesalamine

:ormal La$s

1&.A (2!year!ol" "ia$etic female patient is seen for hypertension. >n physical e.amination% she has clear l#ngs an" a soft a$"omen. There is E1 pe"al e"ema. /er $loo" press#re is 1117144 mm /g. /er creatinine is 1.< mg7"L an" her $loo" #rea nitrogen is 22 mg7"L. /er #rine re)eals E& protein. =hich of the follo'ing 'o#l" $e the most appropriate pharmacotherapya6A2E inhi$itor $6Alpha $locker c6Amlo"ipine "6Beta $locker e6 ThiaDi"e "i#retic :ormal La$s 14. A 4!year!ol" 'oman presents to the emergency "epartment complaining of the 0'orst hea"ache of her life.0 +t $egan a$r#ptly an" has persiste" for <4 min#tes. She is na#seo#s an" lethargic. >n physical e.amination% her $loo" press#re is 24471&4 mm /g% an" p#lse is <27min. She has n#chal rigi"ity $#t no localiDing ne#rologic signs. =hich of the follo'ing is the most appropriate ne.t step in managementa62ontrol her $loo" press#re 'ith a nitropr#ssi"e "rip $62ontrol her $loo" press#re 'ith s#$ling#al nife"ipine c6>$tain a 2T scan of the $rain 'ith contrast "6>$tain a 2T scan of the $rain 'itho#t contrast e6Perform a l#m$ar p#nct#re :ormal La$s 1 . A <&!year!ol" 'oman presents to the emergency "epartment complaining of chest pain. The patient states that the pain $egan "#ring her morning 'alk. +t starte" as a "#ll press#re o)er her $reast$one an" then ra"iate" to her left arm. >)er the ne.t fe' min#tes% it escalate" in intensity an" 'as not relie)e" $y rest. She calle" 811 an" 'as $ro#ght to the emergency "epartment. /er other past me"ical history is significant for hypertension an" hyperlipi"emia. /er me"ications incl#"e atenolol an" sim)astatin "aily. >n physical e.amination% her $loo" press#re is 1847144 mm /g% an" her p#lse is <47min. =hich of the follo'ing is the most appropriate agent to lo'er her $loo" press#rea6+? norepinephrine $6+? nitroglycerin

c6>ral hy"rochlorothiaDi"e "6 >ral metoprolol e6S#$ling#al nitroglycerin 1<.An 11!year!ol" man is e)al#ate" for possi$le imm#no"eficiency "isease $eca#se of a life!long history of chronic l#ng infections% rec#rrent otitis me"ia% an" m#ltiple episo"es of $acterial meningitis. =hile total +gC is normal% the patient is fo#n" to ha)e a selecti)e "eficiency of +gC2. An associate" "eficiency of 'hich of the follo'ing other s#$stances may pro"#ce anaphyla.is 'hen $loo" pro"#cts are gi)en to s#ch in"i)i"#alsa6 2& $624 c6+gA "6+gE e6+gM :ormal La$s 1(. A 18!year!ol" college freshman presents at the #ni)ersity health clinic $eca#se of rec#rrent episo"es of 'heeDing "#ring $asket$all practices. /e has $een a starting shooting g#ar" for this team an" has #s#ally $een hampere" $y shortness of $reath shortly after $eginning practice an" "#ring games. The symptoms are accompanie" $y a nonpro"#cti)e co#gh an" chest tightness. /e "enies any symptoms at rest. The symptoms occ#r 'hether the practices are in"oors or o#t"oors. >n physical e.amination% he is comforta$le an" "enies any symptoms. /is physical e.amination is #nremarka$le. =hich of the follo'ing cells are most likely to me"iate his symptomsa6Eosinophils $6Lymphocytes c6Mast cells "6Monocytes e6:e#trophils :ormal La$s

11.A & !year!ol" male has $een complaining of hea"aches for the past < months an" has $een reporting changes in his )ision. >n e.amination he has loss of )is#al fiel"s $ilaterally an" gets a magnetic resonance image 3MA+6 of his $rain. This sho's a pit#itary a"enoma. =hich of the follo'ing hormones is most likely to $e ele)ate" in this patienta6A"renocorticotropic hormone 3A2T/6 $6Cro'th hormone 3C/6 c6Prolactin "6Somatome"in 2 e6 Thyroi"!stim#lating hormone 3TS/6 :ormal La$s 18. A <4!year!ol" man 'ith alcoholic cirrhosis presents 'ith a 1!"ay history of "iff#se a$"ominal pain. /e "enies na#sea or )omiting. /e ne)er ha" an episo"e $efore% $#t has ha" a history of esophageal )arices. /is reg#lar me"ications incl#"e thiamine% m#lti)itamins% an" folate. >n physical e.amination% he is fe$rile an" in mil" "istress. /is a$"omen is "isten"e" 'ith a clear fl#i" 'a)e. There is "iff#se ten"erness on a$"ominal palpation. Paracentesis re)eals a clear fl#i" 'ith 1444 le#kocytes7HL% 'ith a pre"ominance of ne#trophils 314B6. Cram,s stain sho's no $acteria. =hich of the follo'ing is the most likely "iagnosisa62holecystitis $6Li)er a$scess c6Pancreatitis "6Peptic #lcer "isease e6Primary peritonitis :ormal La$s

24.A 21!year!ol" African American 'oman comes to the physician $eca#se of chest pain for 2 "ays. She has $een other'ise in goo" health. She "oes not "rink alcohol% smoke% or take me"ications. There is no family history of car"io)asc#lar "isease. /er $loo" press#re is 1247(4 mm /g. /er pain is constant an" is e.acer$ate" $y "eep inspiration. Physical e.amination is normal e.cept for ten"erness on palpation of the thir" left costochon"ral 9#nction. =hich of the follo'ing is the most appropriate ne.t step in managementa6 2hest .!ray films $6E2C an" echocar"iographic st#"ies c6La$oratory st#"ies to assess risk factors for atherosclerosis "6Aeferral for psychiatric e)al#ation e6 Treatment 'ith nonsteroi"al anti!inflammatory "r#gs an" rest :ormal La$s 21.A pre)io#sly healthy% 22!year!ol" 'oman comes to me"ical attention $eca#se of mil" 9a#n"ice% lo'!gra"e fe)er% arthralgia% malaise% an" amenorrhea for & months. She "oes not "rink alcohol or smoke. She "oes not take any me"ications other than oral contracepti)es. Ser#m chemistry st#"ies sho' ele)ate" AST an" ALT le)els% hypergammaglo$#linemia% an" high titers of circ#lating antin#clear an" anti!smooth m#scle a#toanti$o"ies. Serologic st#"ies of anti$o"ies to hepatitis )ir#ses are as follo's; /A? :egati)e /B? :egati)e /2? Positi)e% $y enDyme imm#noassay /2? :egati)e% $y recom$inant imm#no$lot assay 3A+BA6 A li)er $iopsy "emonstrates lymphocytic portal inflammation 'ith early $ri"ging necrosis. =hich of the follo'ing is the most likely "iagnosisa6A#toimm#ne hepatitis $62hronic hepatitis 2 c6/epatic a"enoma "6:onalcoholic steatohepatitis e6Primary $iliary cirrhosis :ormal La$s

22.A hospitalist on "#ty for the 'eeken" is calle" to cons#lt on a case in)ol)ing a &!year!ol" 'oman in the me"ical intensi)e care #nit. The patient has $een hospitaliDe" for 'eeks. /er initial in9#ry 'as a massi)e s#$arachnoi" $lee" complicate" $y an ischemic infarct of her $rain & "ays later. Since that time% she has $een persistently )egetati)e. She re*#ires mechanical )entilation an" e.ternal fee"ing to maintain her )ital f#nctions. +t is "isco)ere" that% "#ring a pre)io#s a"mission to the hospital% the patient clearly state" that she 'o#l" 'ant to $e maintaine" on life s#pport only if she 'ere likely to regain a meaningf#l *#ality of life. The me"ical team $elie)es that she "oes not ha)e a significant chance of regaining an accepta$le le)el of f#nction. The chil"ren maintain that the sit#ation is re)ersi$le an" 'ant to contin#e care% 'hereas the h#s$an" 'ants to 'ith"ra' care in f#lfillment of his 'ife,s 'ishes. =hich of the follo'ing is the most appropriate ne.t stepa6 Arrange a family meeting% hoping to resol)e the patient,s pre)io#sly e.presse" 'ishes 'ith those of the chil"ren an" h#s$an" $6Begin the 'ith"ra'al of care% "espite the reser)ations of the chil"ren% after "isc#ssion 'ith the hospital la'yer c62ons#lt a psychiatrist to speak to the chil"ren "6:otify the "epartment of social ser)ices for the *#estion of el"er a$#se e6Aefer the case to the ethics committee for re)ie' :ormal La$s 2&. A &4!year!ol" 'oman presents to the emergency "epartment after a syncopal episo"e. & months ago she ha" a fe)er% chills% an" generaliDe" 'eakness "#ring a )isit to 2ape 2o". She #ses oral contracepti)es. She #se" cocaine 4 months ago. /er $loo" press#re is 1147< mm /g% an" p#lse is 4&7min. 2hemistry res#lts an" a chest film are normal. An E2C re)eals complete heart $lock% $#t there is no e)i"ence of ischemia or prior infarct. =hich of the follo'ing is the most likely ca#se of this patient,s complete heart $locka6+nfection $y Borrelia $#rg"orferi $6+nfection $y /+? c6+nfection $y +.o"es "ammini "6Myocar"ial infarction from cocaine #se e6Myocar"ial infarction from a coronary throm$#s :ormal La$s

24. A <4!year!ol" 'oman complains of increasing 'eakness an" fatig#e o)er the past se)eral months. She reports that any physical acti)ity lea"s to fatig#e% an" he is #na$le to fin" any comforta$le position at rest. She has lost po#n"s o)er the past & months. Prior to this% she has al'ays $een healthy an" is not on any me"ications. Physical e.amination is #nremarka$le. =hich of the follo'ing is the most likely "iagnosisa6 @epression $6/ypokalemia c6Polymyalgia rhe#matica 3PMA6 "6Polymyositis e6 Temporal arteritis :ormal La$s 2 .A <4!year!ol" 'oman complains of increasing 'eakness an" fatig#e o)er the past se)eral months. She reports that any physical acti)ity lea"s to fatig#e% an" he is #na$le to fin" any comforta$le position at rest. She has lost po#n"s o)er the past & months. Prior to this% she has al'ays $een healthy an" is not on any me"ications. Physical e.amination is #nremarka$le. =hich of the follo'ing is the most likely "iagnosisa6@epression $6/ypokalemia c6Polymyalgia rhe#matica 3PMA6 "6Polymyositis e6Temporal arteritis :ormal La$s

2<.A <1!year!ol" man presents for follo'!#p. T'o 'eeks earlier% he 'as "iagnose" 'ith chronic o$str#cti)e p#lmonary "isease 32>P@6 an" #n"er'ent p#lmonary f#nction tests 3P5Ts6 to assess re)ersi$ility of his o$str#ction. /e has smoke" three packs per "ay for 4 years an" has hypercholesterolemia. +n the 2& years that he has $een a patient in this practice% he has ha" terri$le compliance an" "oes not see a physician #nless he is )ery ill. /is P5Ts "oc#ment mo"erate o$str#cti)e l#ng "efects. /is *#ality of life is c#rrently goo"% an" he has not ha" to c#rtail his "aily acti)ities significantly. =hich of the follo'ing imm#niDations is most important for this patienta6/aemophil#s infl#enDae type B )accine $6+nfl#enDa )accine c6Measles $ooster "6Pert#ssis $ooster e6?aricella )accine :ormal La$s 2(. A (&!year!ol" man presents to the emergency "epartment complaining of a$"ominal pain. /e "escri$es a "#ll% aching% constant pain in his mi"!#m$ilical region. The pain has persiste" o)er the past fe' "ays 'ith increasing intensity% an" it is not relie)e" $y changes in position or eating. The patient has a past me"ical history significant for hypertension an" coronary artery "isease. /e ha" a heart attack & years ago. The man has mo"erate peripheral )asc#lar "isease 'ith a prior femoral!popliteal $ypass graft on the left. >n physical e.amination% his $loo" press#re is 1147144 mm /g% an" his p#lse is 1<7min. /e has a lo#" S4% a p#lsatile% mi"line a$"ominal mass an" )eno#s stasis changes $ilaterally on his lo'er e.tremities. =hich of the follo'ing is the most appropriate "iagnostic test at this timea6A$"ominal #ltraso#n" 3U7S6 $6L#m$osacral 3L7S6 spine films c62T of the spine "62T of the a$"omen e6Spinal MA+ :ormal La$s

21. A patient 'ith ac#te hepatitis B presents to yo#r office complaining of se)ere fatig#e% lo'!gra"e fe)ers an" 'eight loss. /e 'as "iagnose" 2 'eeks earlier% an" hepatitis B s#rface antigen 'as positi)e. A physical e.amination re)eals 9a#n"ice 'ith "iff#se skin e.coriations. /is li)er an" spleen are $oth marke"ly enlarge" an" ten"er. Peripheral e"ema is present. =hich of the follo'ing 'o#l" $e the 'orst prognostic signa6 Al$#min of &.1 g7"L $6Bilir#$in of 8.4 mg7"L c6Prothrom$in time of 18 secon"s 'ith an +:A of 2.1 "6SC>T 3AST6 of 2 U7L e6SC>T 3AST6 of 1244 U7L :ormal La$s 28.An other'ise healthy 2&!year!ol" 'oman comes to the physician $eca#se of episo"ic hea"aches. She "escri$es them as thro$$ing or po#n"ing in character% affecting only one si"e an" #s#ally associate" 'ith na#sea an" )omiting. The episo"es are sometimes triggere" $y noise or stress% $#t sometimes $egin 'itho#t an apparent ca#se. Tingling of the face often prece"es each attack. These episo"es last for many ho#rs. The patient has trie" common analgesic me"ications% s#ch as aspirin% acetaminophen% an" i$#profen% $#t 'ith limite" $enefit. Physical e.amination is normal. =hich of the follo'ing is the most likely "iagnosisa6 2l#ster hea"ache $6/ea"ache secon"ary to $rain t#mor c6Migraine "6Temporal arteritis e6Tension hea"ache :ormal La$s

&4. A 21!year!ol" English professor complains of chest pain 'ith e.ertion. The symptoms $egan appro.imately 4 months ago an" ha)e $een accompanie" $y "yspnea. >n se)eral occasions% she $ecame lighthea"e" an" felt faint 'hile e.ercising. She has no prior me"ical history. She "oes not smoke or "rink. >n physical e.amination% her $loo" press#re is 1 4782 mm /g% p#lse is 127min% an" respirations are 1<7min. She is afe$rile. A hea" an" neck e.amination is normal. There is no 9#g#lo)eno#s "istention. /er l#ngs are clear. /er caroti" #pstrokes are "elaye" an" "iminishe" in amplit#"e. >n car"iac e.amination% there is a forcef#l apical imp#lse an" a soft S2. There is a harsh% late!peaking% crescen"o!"ecrescen"o systolic m#rm#r that is hear" $est at the right secon" intercostal space an" ra"iates to the caroti"s. The remain"er of the physical e.amination is #nremarka$le. An E2C re)eals a normal sin#s rhythm an" left )entric#lar hypertrophy $y )oltage criteria. =hich of the follo'ing is the most likely "iagnosisa6 Aortic ins#fficiency $6Aortic stenosis c6+schemia of the left anterior "escen"ing artery "6+schemia of the left main coronary artery e6 Mitral reg#rgitation :ormal La$s &1. A &4!year!ol" 'oman cons#lts a physician $eca#se of a painless% pea!like lesion on the $ack of her 'rist. She fin"s the lesion annoying an" "isfig#ring% an" so has it remo)e" $y a s#rgeon. Crossly% the lesion is 'hite an" transl#cent% an" ooDes gelatino#s material 'hen c#t. =hich of the follo'ing is the most likely "iagnosisa6Ane#rysmal $one cyst $65elon c6Canglion cyst "6/erpetic 'hitlo' e6 >steoi" osteoma :ormal La$s

&2. A & !year!ol" man presents to the emergency "epartment 'ith shortness of $reath% 'hich has 'orsene" o)er the past & "ays an" is associate" 'ith a co#gh pro"#cti)e of yello' sp#t#m. /is past me"ical history is significant for asthma% an" his me"ications incl#"e an al$#terol inhaler. /e has a 24!pack!year history of to$acco #se% an" c#rrently smokes t'o packs per "ay. >n physical e.amination% his temperat#re is &(.1 2 314456% $loo" press#re is 1<471( mm /g% p#lse is <87min% an" respirations are 247min. /is l#ng e.amination is significant for "iminishe" $reath so#n"s% 'ith "iff#se 'heeDing $#t no e)i"ence of consoli"ation. /is chest ra"iograph is significant for hypere.pansion. P#lse o.imetry sho's 84B o.ygen sat#ration on room air. =hich of the follo'ing is the most appropriate ne.t step in therapya6Al$#terol ne$#liDer $6+pratropi#m ne$#liDer c6/eli#m!o.ygen mi.t#re "6+? $eta a"renergic $locker e6 Aacemic epinephrine :ormal La$s &&.A 2(!year!ol" homeless man presents to the emergency "epartment complaining of fe)er% rigors% an" a pro"#cti)e co#gh for the past 24 ho#rs. /e a"mits to ha)ing last #se" +? heroin 24 ho#rs earlier. /e has rhonchi in $oth lo'er l#ng fiel"s posteriorly. A chest .!ray film re)eals $ilateral lo$ar consoli"ations% 'ith an air!fl#i" le)el in the left lo'er lo$e. A sp#t#m Cram,s stain re)eals gram!positi)e cocci in cl#sters in association 'ith many polymorphon#clear le#kocytes. =hich of the follo'ing 'o#l" $e the most appropriate ne.t step in managementa6Begin therapy 'ith oral "iclo.acillin $6Begin therapy 'ith ampicillin fo#r times "aily c6+nitiate therapy 'ith oral erythromycin "6+nitiate therapy 'ith +? erythromycin e6+nitiate therapy 'ith +? nafcillin :ormal La$s

&4.A !year!ol" man is $ro#ght to the emergency "epartment $eca#se of increasing incoherence o)er the past 24 ho#rs. /e has a history of hypertension an" "ia$etes. /e is "isoriente". >n physical e.amination% he is afe$rile. /is $loo" press#re is 2&471&4 mm /g% p#lse is 1247min% an" respirations are 247min. /e has an S4 on car"iac e.amination. The patient is place" on a car"iac monitor% an" +? an" intraarterial lines are place". A hea" 2T scan sho's no mass or $lee". =hich of the follo'ing is the most appropriate ne.t step in managementa6>$ser)e in a *#iet room $62heck chemistry for an anion gap c6A"minister a $eta $locker "6A"minister so"i#m nitropr#ssi"e e6Perform a l#m$ar p#nct#re :ormal La$s & .A &8!year!ol" 'oman presents 'ith complaints of "iffic#lty s'allo'ing. She has a history of sclero"erma for the past 1 years% "#ring 'hich time she has re*#ire" a )ariety of me"ications to re"#ce the symptoms of heart$#rn. >)er the past < months% she has also note" "iffic#lty s'allo'ing foo"% s#ch as steak% an" has felt as if foo" 0sticks0 in her lo'er chest. She is a$le to tolerate li*#i"s 'itho#t "iffic#lty. She "enies any 'eight loss. =hich of the follo'ing has most likely occ#rre"a6 @e)elopment of a s*#amo#s carcinoma in the #pper thir" of the esophag#s $6Aecent ret#rn of peristaltic acti)ity in the $o"y of the esophag#s c6Ae)erse peristalsis in the $o"y of the esophag#s "6Ae)erse peristalsis of the lo'er esophag#s e6Scarring at the lo'er esophag#s $eca#se of chronic aci" refl#. :ormal La$s

&<.A 18!year!ol" college )arsity s'immer an" "i)er "e)elops a hea"ache% "iDDiness% left!si"e" arm cl#msiness an" leg 'eakness. /e also "e)elops loss of pain an" temperat#re sensation in the left facial region an" right $o"y areas after a practice session. /e "enies prior illness or #se of me"ications. There is no family history of any "iseases. =hich of the follo'ing is the most likely "iagnosisa6Astrocytoma $6Benign positional )ertigo 3BP?6 c6La$yrinthitis "6M#ltiple sclerosis e6 ?erte$ral artery "issection :ormal La$s &(. A <1!year!ol" man is a"mitte" to the me"ical ser)ice for chest pain. The patient has an 14 pack!year smoking history an" is kno'n to ha)e an ele)ate" total cholesterol $#t #nkno'n L@L an" /@L components. /e is a kno'n ins#lin!"epen"ent "ia$etic 'ith a recent hemoglo$in A1c fraction of 1.&B. /e has a history of chronic% sta$le angina precipitate" $y e.ertion an" relie)e" $y rest. @#ring the e.amination% he is free from chest pain. /is $loo" press#re is 1<4784 mm /g% p#lse is 847min% an" respirations are 227min. /e is mil"ly "iaphoretic. >n physical e.amination% he has an S& gallop% $i$asilar co#rse rales% an" an a$"ominal $r#it. A chest ra"iograph sho's mil" p#lmonary e"ema. >n E2C o$taine" on arri)al to the floor sho's ST segment "epressions in lea"s ?&% ?4% ? an" ?<. =hich of the follo'ing is the most likely "iagnosisa62ostochon"ritis $6P#lmonary em$ol#s c6M#sc#loskeletal chest pain syn"rome "6Myocar"ial ischemia e6 Myocar"ial infarction :ormal La$s

&1.A 44!year!ol" 'oman presents to a physician 'ith a marke"ly painf#l% s'ollen% 'arm knee. Aspiration of the knee "emonstrates milky fl#i" 'ith a )ery high le#kocyte co#nt. :o crystals are seen. Base" on this information% 'hich of the follo'ing is the most likely pathogena6Entero$acter $6:eisseria gonorrhoeae c6Pse#"omonas aer#ginosa "6Staphylococc#s a#re#s e6Streptococci :ormal La$s

&8.A <4!year!ol" man presents to the physican,s office complaining of fe)ers for the past 2 "ays. >)er the past 24 ho#rs% he has "e)elope" a pro"#cti)e co#gh. /e also reports that he has fre*#ent chills% an" has $een 'aking for the past 2 nights 'ith "renching s'eats. /is past me"ical history is remarka$le only for mil" e.ertional angina. >n physical e.amination% he "oes not appear chronically ill $#t appears mo"erately "yspneic. /is temperat#re is &1.< 2 3141.4 56% $loo" press#re is 1&<784 mm /g% an" respirations are 2<7min. There is no 9#g#lar )eno#s "istention. The l#ngs ha)e coarse rhonchi at the right l#ng $ase 'ith increase" fremit#s in the same area. /e has a reg#lar heart rhythm% 'ith a 17< systolic m#rm#r at the left sternal $or"er. The remain"er of the physical e.amination is #nremarka$le. =hich of the follo'ing is the most appropriate ne.t step in "iagnosisa62hest .!ray film $6Sp#t#m Cram,s stain c62hest 2T scan "6P#lmonary f#nction test e6 Peak e.piratory flo' rate meas#rement :ormal La$s

44.5oot e.amination is performe" on a 4 !year!ol" man 'ho ha" $een "iagnose" 'ith Type 1 "ia$etes mellit#s at age 1 . =hile the man has no complaints of foot pain% $oth feet appear clearly "eforme"% 'ith loss of the normal arches an" a$normal orientation of the toes. :o inflammation is note". F!ray films sho' "eforme" $ones 'ith ne' $one formation a"9acent to the $one corte.. Se)eral large% $iDarrely shape" osteophytes are seen at the 9oint margins. =hich of the follo'ing is the most likely "iagnosisa6 A)asc#lar necrosis $6Co#t c6+nfectio#s arthritis "6:e#rogenic arthropathy e6Ahe#matoi" arthritis :ormal La$s 41.An 14!year!ol" 'oman complains of a 4 month history of 'orsening gait an" lo' $ack pain that is 'orse on 'alking. She "enies any tra#ma an" is not incontinent. She has $een fairly healthy an" only takes iron s#pplements. >n e.amination% she has hypoacti)e m#scle stretch refle.es in the legs. The plain .!rays of the l#m$osacral region sho' "egenerati)e changes that seem age!appropriate. =hich of the follo'ing is the most likely "iagnosisa6 Ac#te l#m$ar "isc herniation $62er)ical stenosis c6L#m$ar stenosis "6Myopathy e6:ormal press#re hy"rocephal#s 3:P/6 :ormal La$s

42. A 4(!year!ol" man 'ho a"mits to "aily alcohol #se for the past & 'eeks% presents to the emergency "epartment complaining of se)ere mi"epigastric pain ra"iating to his $ack an" accompanie" $y rec#rrent )omiting. /e has a nasogastric t#$e place"% is kept :P>% an" is gi)en +? hy"ration. >)er the ne.t (2 ho#rs% ho'e)er% his clinical con"ition contin#es to "eteriorate. =hich of the follo'ing complications 'o#l" $e most likely to occ#ra62a)itating pne#monia $6/ypercalcemia c6:ephrotic syn"rome "6>ral aphtho#s #lcers e6Ple#ral eff#sion :ormal La$s 4&.>ne 'eek after an #pper respiratory infection% an a"#lt "e)elops con9#ncti)al hyperemia% 'atery "ischarge% an" oc#lar irritation. =hile $oth eyes are in)ol)e" $y the time a physician 'as cons#lte"% the symptoms $egan in one eye. >n the morning of the "octor,s )isit% the patient ha" "iffic#lty opening his eyeli"s on a'akening as they 'ere 0gl#e" sh#t0. Physical e.amination "emonstrates hyperemic $#l$ar an" tarsal con9#ncti)ae. :o p#r#lent fl#i" is seen. The prea#ric#lar lymph no"e on one si"e is enlarge". =hich of the follo'ing pathogens 'o#l" $e most likely to ca#se these symptomsa6A"eno)ir#s $6/erpes simple. + c6/erpes simple. ++ "6:eisseria gonorrhoeae e6Staphylococc#s a#re#s :ormal La$s

44. A (1!year!ol" retire" physician complains of increasing calf pain 'hen 'alking #phill. The symptoms ha)e gra"#ally increase" o)er the past 2 months. /is past me"ical history is significant for a history of an #ncomplicate" myocar"ial infarction 2 years earlier an" a transient ischemic attack < months ago. >)er the past month% his $loo" press#re has 'orsene" "espite pre)io#s control 'ith "iltiaDem% hy"rochlorothiaDi"e% an" propranolol. /is other me"ications are isosor$i"e "initrate% hy"rochlorothiaDi"e% an" aspirin. >n physical e.amination% his $loo" press#re is 1 2784 mm /g% an" his p#lse is <<7min. There is a right caroti" $r#it. /is lo'er e.tremities are mil"ly cool an" ha)e "iminishe" p#lses at the "orsalis pe"is. =hich of the follo'ing is the most appropriate in the management of his leg paina6@ecrease "iltiaDem "ose $6@ecrease hy"rochlorothiaDi"e "ose c6@ecrease propranolol "ose "6+ncrease aspirin "ose e6 +ncrease isosor$i"e "initrate "ose :ormal La$s 4 . A &(!year!ol" 'oman presents 'ith a s'ollen an" ten"er left 'rist. The symptoms $egan appro.imately 1 "ay earlier an" ha)e $ecome increasingly se)ere "#ring that time. She has note" a fe)er an" ha" shaking chills last night. She has no past me"ical history an" is on no me"ications% e.cept for i$#profen for occasional premenstr#al cramps. >n physical e.amination% her 'rist has a "ecrease" range of motion an" is ten"er to palpation. The 9oint space has an o$)io#s eff#sion an" is ten"er% erythemato#s% an" 'arm. =hich of the follo'ing is the most appropriate ne.t step in the managementa6 +nitiating colchicine $6+nitiating in"omethacin therapy c6+nitiating +? anti$iotics "6>$taining a 'rist .!ray film e6Arthrocentesis :ormal La$s

4<. A healthy 2(!year!ol" man has a co#gh 'ith sp#t#m pro"#ction for 2 "ays. The co#gh has $een keeping him #p at night an" it is affecting his 9o$ performance. /e has no history of respiratory "isease. /is temperat#re is &( 2381.< 56% $loo" press#re is 1&4714% p#lse is (47min% an" respirations are 117min. Physical e.amination is normal. =hich of the follo'ing is the most appropriate ne.t step in managementa6A"mit to the hospital for me"ical management $6Perform a chest .!ray film c6Perform a sp#t#m c#lt#re "6Sen" home 'ith anti$iotic therapy e6Sen" home 'ith no therapy :ormal La$s 4(. A <8!year!ol" 'oman presents to her physician,s office complaining of 1 'eek of crampy lo'er a$"ominal pain an" $loo"y "iarrhea. She ha" pre)io#sly $een follo'e" for symptoms of sta$le e.ertional angina an" hypertension. She ha" an #ncomplicate" myocar"ial infarction & years earlier. /er symptoms $egan 1 'eek ago 'ith mil" postpran"ial a$"ominal cramping follo'e" $y "iarrhea% 'hich $ecame $loo"y after 2 "ays. She has not tra)ele" recently an" is a retire" li$rarian. /er temperat#re is &1.< 2 3141.4 56% $loo" press#re is 124714 mm /g% an" p#lse is 8<7min. She has mo"erate ten"erness to palpation of the left lo'er *#a"rant. A rectal e.amination re)eals $loo"y stool an" no masses. =hich of the follo'ing is the most likely "iagnosisa6Arterio)eno#s malformation $6@i)ertic#litis c6 @i)ertic#losis "6+schemic colitis e6Ulcerati)e colitis :ormal La$s

41. An 11!year!ol" girl comes to the physician for a 0check!#p0 $efore going off to college. She has no complaints. She states that she is )ery e.cite" to finally $e on her o'n. She e.ercises 0reg#larly0% gets goo" gra"es in school% an" is se.#ally acti)e. /er 'eight is normal for her height. Physical e.amination sho's many "ental caries% perio"ontal "isease% pharyngeal a$rasions% nail changes% an" m#ltiple% linear lacerations on her forearms in )ario#s stages of healing. La$oratory st#"ies sho' hypokalemia an" meta$olic alkalosis. =hich of the follo'ing 'o#l" most likely esta$lish a "iagnosisa6 0@o yo# e)er feel g#ilty a$o#t "rinking alcohol-0 $60@o yo# typically restrict yo#r "iet to #n"er 144 calories a "ay-0 c60/a)e yo# e)er taken la.ati)es as a 'ay to lose 'eight-0 "60/a)e yo# e)er cons#me" large *#antities of foo" an" then reg#rgitate" it to pre)ent 'eight gain-0 e6 0Please "escri$e yo#r e.ercise ro#tine.0 :ormal La$s 48. A &8!year!ol" chronic alcoholic presents to the emergency "epartment 'ith complaints of se)ere epigastric pain% na#sea% an" )omiting. The patient a"mits to acti)ely "rinking an" reports that the pain is se)ere an" ra"iates to his $ack. /e reports that he ha" three similar episo"es in the past year that re*#ire" hospitaliDation an" resol)e" after se)eral "ays of not eating. /is temperat#re is &1.4 2 3141 56. /is ser#m amylase an" lipase le)els are ele)ate". =hich of the follo'ing may occ#r as a se*#ela of this episo"ea6 A"#lt respiratory "istress syn"rome "#e to circ#lating phospholipase $6@e)elopment of a carcinoma as a res#lt of pancreatic "#ct in9#ry c6/ypercalcemia as a res#lt of stim#lation of parathyroi" secretion "6/ypertension "#e to "ecrease" intra)asc#lar )ol#me e6/ypoglycemia "#e to impaire" ins#lin pro"#ction :ormal La$s

4. A patient 'ith A+@S an" a 2@4 cell co#nt of 1<7mm& comes to me"ical attention $eca#se of the recent onset of motor an" sensory ne#rologic "eficits an" mental stat#s changes. 2S5 e.amination re)eals mil" lymphocytosis. :e#roimaging st#"ies sho' m#ltiple ill!"efine" areas of T2 changes% interprete" as e)i"ence of "emyelination. These areas are locate" in the 'hite matter of $oth cere$ral hemispheres an" cere$ell#m. =hich of the follo'ing is the most likely "iagnosisa6A+@S!"ementia comple. $62ere$ral to.oplasmosis c62M? encephalitis "6Progressi)e m#ltifocal le#koencephalopathy e6M#ltiple sclerosis :ormal La$s

Note: Check your own answers before hittin$ the Check button below. When you click the Check button, a browser win"ow will a!!ear that contains a summary of your results. (*!lanations lock C (*!lanations

:) (*!lanation: 7he correct answer is %. 7he most crucial measure is to trans!ort the victim of a snakebite to the nearest me"ical facility for intravenous a"ministration of antivenin horse serum. 7he most current recommen"ations for Hfiel"H first ai" inclu"e immobili#ation of the !atient an" the bitten !art in a hori#ontal !osition. )ystemic s!rea" of the venom can be retar"e" if the victim remains as inactive as !ossible. 7he bitten limb can be s!linte" an" ke!t at heart level if feasible. 7he clinical manifestations "e!en" on the ty!e of venom, whether !re"ominantly cytolytic 'rattlesnake an" other !it vi!ers) or neuroto*ic 'coral snake). &re"ominantly cytoto*ic venom causes local !ain, re"ness, swellin$, followe" by !erioral !aresthesias an" metallic tasteG sometimes, shock, an" coa$ulo!athy may "evelo!. Neuroto*ic envenomation 'e.$. coral snake) causes si$ns an" sym!toms similar to botulism, i.e. "i!lo!ia, "ys!ha$ia, !tosis, an" res!iratory arrest. Any mani!ulation of the bitten area, such as a!!lication of ice 'choice A), tourniquet 'choice ), or incision an" suction 'choice () is stron$ly "iscoura$e". 7hese measures are either ineffective in !reventin$ the venom from enterin$ circulation or result in unnecessary local trauma an" ischemia. Commercially available mechanical "evices for local suction may be hel!ful if use" within / minutes, but mouth suction results in insi$nificant venom e*traction an" may lea" to contamination of the woun" by oral bacteria. .ivin$ alcohol 'choice C) or stimulants to victims of snakebites is also useless or harmful. ;) (*!lanation: 7he correct answer is (. 7he most likely "ia$nosis in this case is !ulmonary embolus '&(). 7he risk factors for this !atient are obesity an" sur$ery. &leuritic chest !ain can "evelo! in !atients with a & (. 7achycar"ia, tachy!nea, an" increase" o*y$en requirements are hi$hly su$$estive of the "ia$nosis. 7he most a!!ro!riate test to confirm the clinical sus!icion is a ventilation4 !erfusion scan.

A chest *4ray 'choice A) can su$$est a &( by showin$ a we"$e4sha!e" area if there is an infarctG however, this is not the most sensitive or s!ecific test to confirm the "ia$nosis. Chest *4ray films are most often normal. A C7 scan 'choice ), like a chest *4ray, may reveal an area of infarct, but is not the most a!!ro!riate test in this scenario. An (C. with rhythm stri! 'choice C) mi$ht show tachycar"ia, !ossible ri$ht a*is "eviation "ue to !ulmonary hy!ertension, an" ri$ht ventricular strain with )747 wave chan$es. ut, the most common fin"in$ is a normal (C.. +6I 'choice %) is use" to show soft tissue abnormalities an" is most often use" in neurolo$ic an" ortho!e"ic "ia$noses. It woul" not be the test of choice for & (. 0) (*!lanation: 7he correct answer is %. 7his !atient is !resentin$ with sym!toms of a !e!tic stricture after many years of $astroeso!ha$eal reflu* "isease '.(6%) sym!toms. An en"osco!y not only will allow evaluation of the stricture but will also allow bio!sy to ensure that it is of a beni$n nature. -urthermore, it woul" allow "ilatation of the stricture usin$ an en"osco!ic4$ui"e" balloon to relieve the !atientAs sym!toms. A barium eso!ha$ram 'choice A) woul" "emonstrate the ty!ical smooth ta!erin$ nature of a !e!tic stricture, "istin$uishin$ it from the irre$ular, ulcerate", mass4like stricture seen in !atients with eso!ha$eal carcinoma. 2owever, it woul" not conclusively make this "istinction an" woul" require a follow4u! en"osco!y with bio!sy anyway. A chest *4ray 'choice ) is not of s!ecific "ia$nostic value in !atients who are un"er$oin$ evaluation for "ys!ha$ia. A C7 scan of the chest 'choice C) is not very useful in "eterminin$ intraluminal eso!ha$eal lesions. C7 is use" for !atients with sus!ecte" mali$nant strictures to evaluate the !araeso!ha$eal re$ions an" assess for the !ossibility of local s!rea". A ;?4hour !2 monitor 'choice () is useful to "emonstrate that there is aci" reflu*, which is not in question with this !atient with the lon$stan"in$ history of ty!ical sym!toms. ?) (*!lanation: 7he correct answer is

. Amyloi"osis is a syn"rome "escribe" in "ru$ users who Hskin !o!H their "ru$s an" have recurrent infections. 7hese !atients will !resent with a ne!hritic syn"rome with a blan" urine. 7hey usually have lar$e ki"neys. AI%) ne!hro!athy 'choice A) is usually seen in !atients who are 2I14!ositive. It may take si* months to make antibo"ies to the AI%) virus, but AI%) ne!hro!athy is usually not seen so early on. 7hese !atients ra!i"ly $o onto "ialysis. 2eroin ne!hro!athy 'choice C) is seen in "ru$ users who use heroin I1. 7hey usually have small ki"neys an" hy!ertension. +inimal chan$e "isease 'choice %) is usually seen in youn$ !atients. Althou$h still !ossible, it woul" not be the most likely "ia$nosis. &oststre!tococcal $lomerulone!hritis 'choice () will !resent with hematuria an" hy!ertension, which this !atient "i" not have. It is usually seen about :? "ays after a stre!tococcal sore throat an" causes an acute ne!hritis. /) (*!lanation: 7he correct answer is (. 7he "iscre!ancy between ventricular "ilatation an" absence of cortical atro!hy is the most im!ortant clue to the "ia$nosis of normal !ressure hy"roce!halus 'N&2). 7he most characteristic manifestations of N&2 inclu"e "ementia, ata*ia, an" urinary incontinence. 7his form of "ementia, althou$h less frequent than Al#heimer "isease an" vascular "ementia, has !articular relevance since a shuntin$ !roce"ure in the early sta$es may lea" to "ramatic amelioration of the clinical sym!toms. ,on$4stan"in$ N&2 results in irreversible cortical atro!hy. Culture of C)- 'choice A) is necessary in the !resence of si$ns or sym!toms of menin$itis. 7he C)- values in this case rule out menin$itis. 7rial with cholinesterase inhibitors 'choice ) is warrante" when a clinical "ia$nosis of Al#heimer "isease 'A%) is ma"e. In A%, cortical atro!hy is usually severe an" rea"ily a!!reciate" in C7B+6I scans. 1entricular "ilatation in A% is secon"ary to cerebral atro!hy 'i.e., hy"roce!halus Ae* vacuoA). 7reatment with anti"e!ressant "ru$s 'choice C) shoul" be consi"ere" if the !atientAs intellectual "eterioration is "ue to "e!ression. %e!ression may occasionally 'es!ecially in the el"erly) manifest with a !icture known as !seu"o"ementia. 7reatment with levo"o!a 'choice %) is the !rimary !harmacolo$ical intervention for &arkinson "isease '&%). &% manifests with restin$ tremor, akinesia, an" ri$i"ity. Ata*ia is not !art of &%As clinical !icture.

C) (*!lanation: 7he correct answer is (. 7his !atient has the !resentation of i"io!athic hy!ertro!hic subaortic stenosis, which is a frequent cause of synco!e or near synco!e in youn$ !atients. )he has a characteristic murmur, which can be "istin$uishe" from other systolic murmurs by its increase with the 1alsalva maneuver. Any maneuver that acts to "ecrease left ventricular si#e will increase the murmur of i"io!athic hy!ertro!hic subaortic stenosis because the obstructive com!onent increases as the left ventricular cavity shrinks. eta4blockers, such as !ro!anolol, hel! rela* the left ventricular smooth muscle an" re"uce ventricular outflow obstruction. 7he an$iotensin convertin$ en#yme inhibitor ca!to!ril 'choice A), the car"iotro!ic a$ent "i$o*in 'choice ), an" the "iuretic furosemi"e 'choice %) are variably effective in !atients with "ilate" car"iomyo!athy. 2owever, they are !otentially harmful in a !atient such as this, who alrea"y has a hy!ertro!hic ventricle an" increase" e<ection fraction. %iltia#em 'choice C), a calcium channel blocker, has lesser effects than !ro!ranolol on the rela*ation of ventricular smooth muscle. D) (*!lanation: 7he correct answer is (. Fnfortunately for many !atients who are attache" to their towns, houses, <obs, an" !ets, the !referre" an" most effective treatment for aller$ies is avoi"ance of the aller$en. In the case of !et aller$ies, the most effective metho" of avoi"ance is to $ive the family !et away. Chlor!heniramine maleate 'choice A) is an alkylamine ty!e 2: blocker that is commonly use" for relief of aller$y sym!toms. Cromolyn nasal s!ray 'choice ) acts by inhibitin$ mast cell $ranule release an" is use" !ro!hylactically for control of aller$y sym!toms. I+ "i!henhy"ramine 2C, 'choice C) is usually reserve" for !otential ana!hylactic reactions, such as "urin$ a bloo" transfusion reaction. Aller$en immunothera!y 'choice %), commonly calle" Haller$y shotsH, is sometimes 'but not always) hel!ful in "esensiti#in$ in"ivi"uals to !articular anti$ens. @) (*!lanation: 7he correct answer is

A. 7his !atient will also likely have absent breath soun"s in the ri$ht hemithora*. 7he question clearly "escribes the su""en hemo"ynamic colla!se in a !atient who has "evelo!e" a tension !neumothora* while bein$ $iven !ositive4en" e*!iratory !ressure '&((&) on a res!irator. 7he su""en hy!otension an" "ecrease" o*y$enation is consistent with a tension !neumothora*, which is com!ressin$ venous return to the ri$ht si"e of the heart an" thus !ro"ucin$ <u$ular venous "istention. 7he treatment woul" be the imme"iate !lacement of either a nee"le or tube thoracostomy to allow ra!i" re4e*!ansion of the ri$ht lun$. A hi$h am!litu"e car"iac u!stroke !ulse 'choice ) is usually seen in con"itions such as aortic re$ur$itation, in which e*tra bloo" must be !ushe" into the aorta to com!ensate for backflow into the heart "urin$ "iastole throu$h an incom!etent valve. A !leural friction rub 'choice C) su$$ests inflammation of the !leura, which can be seen either when an un"erlyin$ lun$ !rocess involves the lun$ tissue a"<acent to the !leura or when infection or irritatin$ substances are !resent in the !leural s!ace. It woul" not be e*!ecte" in the settin$ of a !neumothora*, in which the visceral !leura becomes se!arate" from the !arietal !leura by a lar$e volume of air. &ulsus alternans 'choice %), also known as alternatin$ !ulse, is a !ulse in which the beat is re$ular but alternate beats are weaker or stron$er. It usually in"icates serious myocar"ial "isease. )!lenome$aly 'choice () is not usually a feature of tension !neumothora* because the venous return from the ab"omen enters the heart throu$h the inferior vena cava, which is much less affecte" by the !neumothora* than is the su!erior vena cava, which has a lon$er course throu$h the chest. I) (*!lanation: 7he correct answer is C. 7he clinical manifestations are consistent with hy!erthyroi"ism. (s!ecially su$$estive are heat intolerance an" systolic hy!ertension with a low "iastolic !ressure. An*iety, han" tremor, an" warm skin are a""itional characteristic sym!toms. &ost!artum thyroi"itis is a frequent, but usually self4limite", !ost!artum com!lication. It "evelo!s a few weeks followin$ "elivery an" a!!ears to result from an autoimmune res!onse. A minority of cases are followe" by hy!othyroi"ism. 7he $lan" is not swollen or finely no"ular as occurs in 2ashimoto thyroi"itis 'choice ), which is also "ue to autoantibo"ies a$ainst thyroi" anti$ens. .raves "isease 'choice A) refers to thyroto*icosis "evelo!in$ as a result of an autoimmune mechanism involvin$ stimulatory autoantibo"ies "irecte" a$ainst 7)2 rece!tors. 7he thyroi" is usually enlar$e" an" warm. O!hthalmo!athy 'e*o!hthalmos) is usually !resent.

6ie"el thyroi"itis 'choice %), also known as chronic fibrous thyroi"itis, is an uncommon form of chronic thyroi"itis lea"in$ to sclerosis of the thyroi" $lan" an" a"<acent structures. It manifests in mi""le4a$e" or el"erly women with slowly "evelo!in$ hy!othyroi"ism an" is characteri#e" by a stony har" thyroi" on !al!ation. )heehan syn"rome 'choice () is a rare syn"rome "ue to com!lete ischemic necrosis of the anterior !ituitary $lan", secon"ary to hy!otension "urin$ com!licate" "eliveries. It manifests with !anhy!o!ituitarism 'inclu"in$ secon"ary hy!othyroi"ism). )ubacute '"e 8uervain) thyroi"itis 'choice -) is a relatively common "isor"er characteri#e" by !ainful swellin$ of the thyroi" an" often associate" with "ys!ha$ia. It may !ersist for months but usually subsi"es s!ontaneously. :>) (*!lanation: 7he correct answer is C. 7his !atient has been a!!ro!riately !rescribe" miso!rostol for the !revention of N)AI%4in"uce" ulcers, $iven the history that she has ble" once !reviously with the use of N)AI%s. )ince miso!rostol is a !rosta$lan"in analo$ue, it will counter the effect of N)AI%s on the $astric mucosa. 2owever, as a !rosta$lan"in analo$ue, it will cause smooth muscle contractions, inclu"in$ within the uterus, which can lea" to a s!ontaneous abortion in a !re$nant woman. )he shoul" therefore be a"vise" to maintain a"equate birth control measures while usin$ miso!rostol. None of the other choices 'choices A, , %, or () have !articular relevance to this !atientAs health concerns. ::) (*!lanation: 7he correct answer is . 2y!onatremia is the most common electrolyte abnormality encountere" in the hos!ital. 7he combination of hy!otension, "ry mucous membranes, low <u$ular venous !ressure, an" low urine so"ium su$$ests that the !atient is marke"ly volume "e!lete". In states in which the bo"y is volume "e!lete", the "rive to re!lenish intravascular volume is $reater than the nee" to maintain osmolality. As such, secretion of anti"iuretic hormone 'A%2) from the !ituitary increases, an" free water is retaine" at the level of the collectin$ "ucts in the ki"ney. 7his !rocess can lea" to !rofoun" hy!onatremia, as occurre" in this case. Al"osterone 'choice A), also referre" to as the Hsalt4savin$ hormone,H is also increase" in hy!ovolemia. 7his !rocess occurs in"irectly throu$h the secretion of renin by the <u*ta$lomerular cells of the ki"ney, which lea"s to the conversion of an$iotensin I to an$iotensin II, the substance that ultimately causes al"osterone secretion. 2i$h levels of al"osterone woul" !resent as hy!okalemia in the absence of hy!onatremia. Althou$h it is

likely to be increase" in this case, al"osterone is not the !rime !layer me"iatin$ the hy!onatremia. A "eficiency, not an elevation, of cortisol 'choice C), as in A""isonAs "isease, may cause hy!onatremia secon"ary to a hy!ersecretion of A%2. 7his hy!ersecretion of A%2 occurs "irectly throu$h co4secretion with corticotro!in4releasin$ factor an" in"irectly throu$h volume "e!letion. )omatostatin 'choice %) is a hy!othalamic hormone, as well as a neurotransmitter. )omatostatin is also !resent in the $astrointestinal tract an" hel!s re$ulate insulin an" $luca$on secretion. %es!ite these functions, it !lays little role here in me"iatin$ the hy!onatremia. 7hyroi" stimulatin$ hormone '7)2) levels 'choice () are elevate" in the !resence of hy!othyroi"ism, a con"ition that can contribute to elevate" levels of A%2 an" hy!onatremia in"irectly throu$h "ecrease" car"iac out!ut. In this case, however, elevate" A%2 levels from volume "e!letion a!!ear to be the central mechanism me"iatin$ the hy!onatremia. :;) (*!lanation: 7he correct answer is (. 7he !atient "escribe" here has ulcerative colitis confine" to the "istal colon, also known as ulcerative !roctosi$moi"itis. )ince the "isease is limite" to the "istal colon, to!ical a$ents such as mesalamine 'or alternatively hy"rocortisone) woul" be effective in re"ucin$ inflammation. +esalamine is an anti4inflammatory "ru$ use" !rinci!ally to control ulcerative colitis. Its active in$re"ient is also known as /4aminosalicylic aci", which is available in the forms of rectal sus!ension, su!!ositories, "elaye" release oral tablets, an" controlle" release oral ca!sules. 7he mo"e of action is unknown, but is thou$ht to involve to!ical 'since mesalamine is !oorly absorbe"), rather than systemic, mo"ulation of arachi"onic aci" metabolites, inclu"in$ !rosta$lan"ins, leukotrienes, an" hy"ro*yeicosatetraenoic aci"s. It is usually well tolerate", but it can cause si$nificant aller$ic reactions relate" to sulfite sensitivity. I1 hy"rocortisone 'choice A) is reserve" for !atients who "o not res!on" to hi$h "oses of oral !re"nisone. I1 infli*imab 'choice ) is use" for !atients with severe refractory Crohn "isease. Oral a#athio!rine 'choice C) is use" in Crohn "isease an" ulcerative colitis in !atients alrea"y refractory or "e!en"ent on steroi"s to control sym!toms or maintain remission. Oral !re"nisone 'choice %) is not warrante" in !atients who have not been treate" !reviously with safer me"ications, such as to!ical mesalamine or hy"rocortisone or oral mesalamine or sulfasala#ine.

:0) (*!lanation: 7he correct answer is A. AC( inhibitors are the !referre" a$ents in the "iabetic !atient, as they lower both systemic bloo" !ressure as well as intra$lomerular !ressure. 7his will lessen the amount of !roteinuria an" sclerosis. All of the other a$ents will lower bloo" !ressure, but "o not s!ecifically lower intra$lomerular !ressure. :?) (*!lanation: 7he correct answer is A. 7his !atient may have a subarachnoi" hemorrha$e. Classically, these !atients !resent with Hthe worst hea"ache of their life.H Controllin$ bloo" !ressure is the treatment of choice, but the bloo" !ressure shoul" not be lowere" too far. 7he systolic !ressure shoul" be in the ran$e of :C>4:D> mm 2$. It shoul" not be lowere" more than this because some of the elevate" !ressure may re!resent a com!ensatory mechanism to maintain cerebral !erfusion !ressure in the face of increase" intracranial !ressure or cerebral arterial narrowin$. I1 nitro!russi"e is a $oo" a$ent to use because it can be titrate" with the bloo" !ressure. If the !ressure "ro!s too low, the I1 can be turne" off. )ublin$ual nife"i!ine 'choice ) may be "an$erous in this !atient, because it can si$nificantly "ro! the bloo" !ressure very quickly. A hea" C7 with contrast 'choice C) may be "one only after one without contrast is "one. After im!rovin$ her bloo" !ressure, this !atient shoul" then have a hea" C7 without contrast 'choice %) to evaluate for bloo". ,umbar !uncture 'choice () is an a!!ro!riate ne*t ste! after lowerin$ the !atientAs bloo" !ressure. (vi"ence for *anthochromia or menin$itis is sou$ht. :/) (*!lanation: 7he correct answer is . Nitro$lycerin is a !otent vaso"ilator that acts !re"ominantly on venous com!liance to re"uce !reloa" to the heart. When a"ministere" intravenously, it can be ra!i"ly titrate" to !ro"uce o!timi#e" bloo" !ressure control. Nore!ine!hrine 'choice A) is an a$onist at catecholamine rece!tors an" therefore woul" likely increase bloo" !ressure. Althou$h not a!!ro!riate here, nore!ine!hrine !lays a role in !atients who are in shock, as it can both raise car"iac out!ut an" increase !eri!heral resistance.

Oral forms of hy"rochlorothia#i"e 'choice C) an" meto!rolol 'choice %) are e*cellent first4line antihy!ertensives with !roven mortality benefit. 7he onset of their antihy!ertensive activity is relatively slow an" therefore not a!!ro!riate for this case, in which bloo" !ressure control must be achieve" quickly. )ublin$ual nitro$lycerin 'choice () has a quick onset of activity an" e*cellent vaso"ilatory effects, makin$ it a mainstay of thera!y for treatment of acute coronary ischemia. It is not a!!ro!riate in this case, however, because it is "ifficult to titrate its antihy!ertensive effect via the sublin$ual route. :C) (*!lanation: 7he correct answer is C. I$. subclass "eficiency is one of the more minor forms of immuno"eficiency "isease, an" most !atients with this con"ition who are o!timally mana$e" can e*!ect a normal life s!an. 7he ty!ical !resentation is as illustrate" in the question stem. 7he "eficiency may involve either or both I$.; an" I$.0 with or without I$.? "eficiency. 'I$.: is the ma<or form, an" its "eficiency lea"s to a "eficiency of total I$., an" so, by "efinition, it is not consi"ere" a HsubclassH "eficiency.) A !otentially clinically im!ortant !oint is that some !atients with I$.; "eficiency also have I$A "eficiency an" may "evelo! ana!hyla*is if $iven I$A4containin$ bloo" !ro"ucts. C0 "eficiency 'choice A) ten"s to !ro"uce !yo$enic infections. C? "eficiency 'choice ) ten"s to !ro"uce an ),(4like syn"rome. I$( "eficiency 'choice %) can be seen in some incom!lete antibo"y "eficiency syn"romes, but is usually not clinically si$nificant. I$+ "eficiency 'choice () occurs as !art of severe combine" immuno"eficiency. :D) (*!lanation: 7he correct answer is C. 7his !atient has sym!toms of e*ercise4in"uce" asthma. Fnsur!risin$ly, he a!!ears normal while in the office. 7he sym!toms of e*ercise4in"uce" asthma are "ue to mast cell release of histamines, which "e$ranulate with the initiation of e*ercise. 7hese sym!toms can be !revente" with the !re4e*ercise use of inhale" cromolyn, which will act to stabili#e the mast cells. It is less effective once e*ercise has be$un. Once bronchoconstriction has occurre", sym!tomatic thera!y can be !rovi"e" with a beta4 a$onist inhaler. (osino!hils 'choice A) are involve" in aller$en4in"uce" asthma.

,ym!hocytes 'choice ), monocytes 'choice %), an" neutro!hils 'choice () are involve" with inflammation, but "o not me"iate the !rocess of asthma. :@) (*!lanation: 7he correct answer is C. />= of !ituitary a"enomas have been foun" to secrete !rolactin. ,evels of this hormone must be measure" in a !atient sus!ecte" of havin$ a !ituitary tumor. Cushin$As "isease "ue to an e*cess of a"renocorticotro!ic hormone 'choice A) is consi"erably less common. Acrome$aly "ue to $rowth hormone e*cess is an infrequent com!lication as well 'choice ). )omatome"in C, which is insulin4like $rowth factor I, is increase" in acrome$aly but not in !atients with !ituitary tumors 'choice %). Over!ro"uction of thyroi" stimulatin$ hormone in these situations is rare 'choice (). :I) (*!lanation: 7he correct answer is (. 7he !atient is e*hibitin$ sym!toms consistent with !rimary !eritonitis. It can be "ifficult to "istin$uish between !rimary 's!ontaneous) !eritonitis an" ru!ture of a hollow viscus resultin$ in !eritoneal soilin$. 7he !resence of fever an" leukocytes in the ascitic flui" su$$ests a !eritoneal infection. Cholecystitis 'choice A) is associate" with ab"ominal !ain in the ri$ht u!!er qua"rant, elevate" "irect an" in"irect bilirubin, an" nausea an" vomitin$. ,iver abscess 'choice ) !resents as a subacute illness without !rominent !eritoneal sym!toms. %ia$nosis is often ma"e by ab"ominal ima$in$, such as C7 or ultrasoun". &ancreatitis 'choice C) is characteri#e" by severe locali#e" 'mi"e!i$astric) !ain, which ra"iates to the back. Amylase an" li!ase levels may be elevate". Calcium levels may be "iminishe". &e!tic ulcer "isease 'choice %) is !ossible in this !atient with a history of alcohol abuse. &atients will ty!ically be afebrile, an" ascitic flui" shoul" not reveal leukocytosis. A com!lication of ulcer "isease woul" be !erforation.

;>) (*!lanation: 7he correct answer is (. 7he !atientAs sym!tomatolo$y an" !hysical fin"in$s are consistent with costochon"ritis 'A5A 7iet#e syn"rome), an inflammatory "isease of the costochon"ral <unction that seems to be most !revalent amon$ African American women. 7his con"ition is often associate" with recent overstrain of chest muscles an" res!on"s to rest an" anti4inflammatory a$ents. 7he most common sites of involvement are the thir" an" fourth costochon"ral <unctions. Im!ortant clues to "ia$nosis inclu"e ten"erness on !al!ation of the involve" <oint an" accentuation of !ain on ins!iration. Note: in this a$e $rou!, the $reat ma<ority of cases of chest !ain are "ue to muscular !ain, costochon"ritis, an" $astroeso!ha$eal reflu*. Chest *4ray films 'choice A) "o not a"" any useful information on classic cases of costochon"ritis. (C. an" echocar"io$ra!hic stu"ies 'choice ) woul" be a!!ro!riate if history an" !hysical e*amination su$$este" a car"iac ori$in of chest !ain. ,aboratory stu"ies to assess risk factors for atherosclerosis 'choice C) are not necessary. 7he clinical an" family history is a$ainst coronary artery "isease. 6eferral for !sychiatric evaluation 'choice %) is a!!ro!riate if there is evi"ence of un"erlyin$ !sycho!atholo$ic causes. An*iety an" !anic attacks often $ive rise to somatic sym!toms that may mimic car"iac "isease 'e.$., !al!itation, chest !ain) or res!iratory "isease 'e.$., tachy!nea, sense of suffocation). ;:) (*!lanation: 7he correct answer is A. Clinical sym!tomatolo$y an" laboratory fin"in$s are consistent with chronic HautoimmuneH he!atitis 'A2). 7he bio!sy fin"in$s su!!ort this "ia$nosis, "emonstratin$ !ortal inflammation with lobular "ama$e resultin$ in bri"$in$ necrosis. 7he youn$ a$e of the !atient an" !resence of hy!er$amma$lobulinemia are also common in this con"ition. 7here are two ma<or ty!es of A2. 7y!e I, or the classic ty!e, is the most frequentG it is associate" with antinuclear an" anti4smooth muscle autoantibo"ies. 7y!e II, which is more common in women of Western (uro!ean "escent, is associate" with autoantibo"ies to circulatin$ liver4ki"ney microsomes. 7he onset is usually insi"ious, an" a history of amenorrhea is frequently !resent. Chronic he!atitis C 'choice ) "evelo!s in @>= of cases of he!atitis C virus infection. It may manifest years after the often4asym!tomatic acute infection with si$ns an" sym!toms of chronic he!atitis or liver failure. 7he "ia$nosis is confirme" by !ositive en#yme immunoassay test for 2C14antibo"ies. 2owever, this test may be falsely !ositive

in situations with hy!er$amma$lobulinemia. In such cases, !ositivity shoul" be confirme" by a more s!ecific 6I A. In this case, 6I A was ne$ative, rulin$ out he!atitis C. 2e!atic a"enoma 'choice C) is a beni$n he!atocellular neo!lasm associate" with use of oral contrace!tives or anabolic an"ro$ens. It is usually clinically silent but may occasionally manifest with life4threatenin$ !eritoneal blee"in$. Nonalcoholic steatohe!atitis 'choice %) mimics alcoholic he!atitis both clinically an" histo!atholo$ically. It is characteri#e" by fatty chan$e of the liver with associate" laboratory si$ns of he!atocellular in<ury. Obesity an" "iabetes are the most common !re"is!osin$ con"itions. &rimary biliary cirrhosis 'choice () has a !re"ilection for mi""le4a$e" women. It manifests with !ro$ressive si$ns of cholestasis, in which !ruritus an" *anthomas are often the earliest si$ns. Antimitochon"rial autoantibo"ies are foun" in I>= to I/= of these !atients. ;;) (*!lanation: 7he correct answer is A. Althou$h it is clear that the !hysician is ethically boun" to follow the !atientAs wishes, which in this case a!!ears to be the with"rawal of care, it is always better to "o so with the resolve of the family. Oftentimes, feelin$s such as $uilt will "rive family members to insist on seemin$ly unreasonable or ina!!ro!riate action. A thou$htful "iscussion, whereby the family members are allowe" to e*!ress their reservations in a su!!ortive settin$, often will !ro"uce a resolution amon$ !revious "is!arate views. Althou$h with"rawin$ care 'choice ) a!!ears to be what ultimately is the most a!!ro!riate action, it is worth tryin$ first to have all of the intereste" !arties Hon the same !a$eH before !rece"in$ if this resolution can be achieve" in a timely manner without causin$ sufferin$ on the !atientAs !art. Whereas consult services such as !sychiatry 'choice C) may be hel!ful on selecte" occasions, the first attem!ts to resolve conflict shoul" fall on the !rimary me"ical team an" !rimary care !hysicians. Althou$h !hysicians have the !ositive "uty to re!ort sus!ecte" el"er abuse, there is no in"ication from the information !resent that the chil"ren have abuse" their mother 'choice %). (thics committees 'choice () may be hel!ful in resolvin$ conflicts, but, a$ain, the !rimary me"ical !rovi"ers shoul" first attem!t to resolve conflicts in or"er to res!ect the !atientAs wishes in a thou$htful an" timely manner.

;0) (*!lanation: 7he correct answer is A. 7his !atient is e*hibitin$ manifestations of secon"ary ,yme "isease, cause" by infection with the s!irochete, orrelia bur$"orferi. )he !robably contracte" the infection in Ca!e Co". 7he three sta$es of ,yme "isease are a skin rash 'erythema chronica mi$rans) with flu like sym!toms, car"iac an" neurolo$ic sym!toms, an" finally an arthritis. 7he car"iac sym!toms arise from A1 no"al con"uction "efects. Antibiotics are nee"e", but a tem!orary !acemaker may also be require". 2I1 car"itis 'choice ) is ty!ically not manifeste" by com!lete heart block. +ore often, it !resents as a car"iomyo!athy associate" with con$estive heart failure, $enerally occurrin$ as a late com!lication of 2I1 infection. &ericar"ial "isease may be "ue to 5a!osi sarcoma, !resumably "ue to acute hemorrha$e. I*o"es "ammini'choice C) is the "eer tick, whose bite transmits the s!irochete. 7he vector itself "oes not cause ,yme "isease. 7he vector is the si#e of a !encil eraser, an" close e*amination is nee"e" to "etect the tick on the bo"y. Cocaine use 'choice %) may lea" to coronary atherosclerosis an" !re"is!ose the !atient to a myocar"ial infarct. 2owever, the timin$ is ty!ically more acute, an" the timin$ in this !atient woul" be incorrect. An acute coronary thrombus 'choice () woul" !resent with substernal chest !ain, "ia!horesis, left arm !ain, "ys!nea, an" )7 elevation on the (C.. 7he !atientAs !resentation is inconsistent with this "ia$nosis. ;?) (*!lanation: 7he correct answer is C. &olymyal$ia rheumatica '&+6) is characteri#e" by muscle "iscomfort an" !atients often have va$ue sym!toms. 7he neuromuscular e*amination is normal. %e!ression 'choice A) can cause fati$ue an" "iminishe" activity but "iscomfort is less frequently a com!laint. 2y!okalemia 'choice ) can cause muscle weakness but "iscomfort is not a ty!ical feature. &olymyositis 'choice %) can cause muscle "iscomfort but there shoul" be accom!anyin$ weakness on e*amination. 7he serum creatine kinase 'C5) is often elevate".

7em!oral arteritis 'choice () can occur with &+6, but this !atient is not com!lainin$ of hea"ache or vision chan$e. ;/) (*!lanation: 7he correct answer is C. &olymyal$ia rheumatica '&+6) is characteri#e" by muscle "iscomfort an" !atients often have va$ue sym!toms. 7he neuromuscular e*amination is normal. %e!ression 'choice A) can cause fati$ue an" "iminishe" activity but "iscomfort is less frequently a com!laint. 2y!okalemia 'choice ) can cause muscle weakness but "iscomfort is not a ty!ical feature. &olymyositis 'choice %) can cause muscle "iscomfort but there shoul" be accom!anyin$ weakness on e*amination. 7he serum creatine kinase 'C5) is often elevate". 7em!oral arteritis 'choice () can occur with &+6, but this !atient is not com!lainin$ of hea"ache or vision chan$e. ;C) (*!lanation: 7he correct answer is . 7he morbi"ity an" mortality rate from annual flu e!i"emics is very hi$h. (ven in otherwise healthy el"erly !atients, influen#a can result in a hi$h risk of serious illness. In !atients with chronic obstructive !ulmonary "isease 'CO&%), this risk is even hi$her. (very CO&% !atient shoul" be $iven an annual flu vaccine. 2aemo!hilus influen#ae ty!e vaccine 'choice A) is $iven to chil"ren, not a"ults. As a result, the inci"ence of infections cause" by this or$anism have "ecrease" almost to trivial levels. 7he measles booster 'choice C) is not as im!ortant since the boosters are $iven every :> years. In a""ition, the acuity of this illness, if one were to contract it, is not as $rave as the flu. 7he !ertussis booster 'choice %), !art of the %!7 booster series, is usually $iven every :> years. Althou$h a small number of a"ults "o have !ertussis !neumonia, this illness is $enerally not as serious as the flu. 7herefore, the recommen"ations are only for all a"ults to make certain that their vaccinations are u! to "ate. )ince most a"ults in the F. ). have ha" chicken!o* as chil"ren, the varicella vaccine 'choice () is $enerally not nee"e".

;D) (*!lanation: 7he correct answer is A. Ab"ominal ultrasoun" 'FB)) is the most cost4effective screenin$ test for a sus!ecte" ab"ominal aortic aneurysm 'AAA). A C7 of the ab"omen 'choice %) with I1 contrast is about twice as e*!ensive as FB), a""s little if any benefit, an" e*!oses the !atient to unnecessary ra"iation. 7he other choices, lumbosacral ',B)) s!ine film 'choice ), C7 of the s!ine 'choice C), an" s!inal +6I 'choice () are ima$in$ stu"ies "irecte" at evaluatin$ s!inal !atholo$y, which is not sus!ecte" here. +6I !rovi"es the hi$hest resolution an" is useful for "etectin$ abscesses or cor" com!ression. C7 is e*cellent for "isk !atholo$y an" even bone !atholo$y. &lain ra"io$ra!hs are useful for very $ross visuali#ation of the bony "ensity an" inte$rity. ;@) (*!lanation: 7he correct answer is C. +any liver function test abnormalities can be seen in a !atient with acute he!atitis , but the si$ns of he!atic failure, inclu"in$ coa$ulo!athy or ence!halo!athy, are the worst !ro$nostic si$ns. 7hese !atients may require emer$ent liver trans!lantation, as more conservative measures may not be lifesavin$. A minimally re"uce" albumin of 0.: 'choice A) is not an im!ortant !ro$nostic factor, althou$h it may in"icate lon$stan"in$ he!atic synthetic "ysfunction if no other etiolo$y of hy!oalbuminemia is !resent. An elevate" bilirubin 'choice ) is not a !oor !ro$nostic si$n. In fact, !atients with he!atitis may become very "ee!ly <aun"ice", with bilirubin elevation as hi$h as 0>, an" still return to normal. A low ).O7 'choice %) has no !ro$nostic im!ortance. An elevate" ).O7 'choice () may be seen in these !atients but is not a s!ecific !ro$nostic si$n. ;I) (*!lanation: 7he correct answer is C. 2ea"aches affect u! to I>= of the $eneral !o!ulation. 7he classification of this common clinical manifestation is com!le*. 7he International 2ea"ache )ociety 'I2)) classification lists :0 "ifferent cate$ories of hea"ache, with several subty!es. Of these, mi$raine an" tension hea"aches are the most common forms. +i$raine has a ty!ical !resentation: it is e!iso"ic an" recurrent, !rece"e" by visual or cutaneous auras in one thir" of cases, often unilateral, an" associate" with nausea an" vomitin$. 7he !atient may fin" relief lyin$ in "arkness, away from noises. Noise, stress, s!ecific foo"s or "rinks,

an" li$ht may act as tri$$erin$ factors. &ositive familial history is often elicite". When the !atient !resents to the !hysician between attacks, neurolo$ic an" !hysical e*amination is entirely normal. 7he con"ition is !robably "ue to abnormalities in serotoniner$ic !athways, with secon"ary alterations in vasomotor control of cerebral vessels. Cluster hea"ache 'choice A) is rare but hi$hly characteristic in its !resentation. &atients e*!erience "aily hea"ache attacks that be$in usually at ni$ht an" last from 0> minutes to ;40 hours. Alcohol consum!tion is a well4reco$ni#e" tri$$er. 2ea"ache e!iso"es recur re$ularly each ni$ht for u! to ; months, after which this HclusterH is followe" by a !ain4 free interval that may last several months or years. 2ea"ache secon"ary to brain tumor 'choice ) is a rare but im!ortant cause of hea"ache. It shoul" be consi"ere" when the clinical manifestations are aty!ical an"Bor in the !resence of accom!anyin$ focal neurolo$ic "eficits. 7em!oral arteritis 'choice %) is an inflammatory "isease of me"ium4si#e" arteries with a !re"ilection for the su!erficial tem!oral artery an" its branches. It usually affects mi""le4 a$e" or el"erly !atients. 7he hea"ache is constant, locali#e" to the tem!oral re$ion, an" associate" with fever an" malaise. 7he erythrocyte se"imentation rate is elevate". io!sy of the affecte" artery often "iscloses the ty!ical $iant4cell reaction in the me"ia 'hence the alternative name of $iant cell arteritis). 7ension hea"ache 'choice () is usually relate" to stress an" an*iety. It is worse in the nuchal re$ion an" has a vise4like, squee#in$ character. 7he !ain may last for many hours or "ays. 0>) (*!lanation: 7he correct answer is . 7his !atientAs sym!toms are su$$estive of coronary artery "isease. 2owever, she is !remeno!ausal, makin$ this "ia$nosis unlikely in the absence of a very stron$ family history of early coronary artery "isease or of a familial hy!erli!i"emia. -urthermore, her (C. "oes not reveal chan$es consistent with coronary artery ischemia. )he has the characteristic fin"in$s of aortic stenosis with the classic murmur "escribe". In this !atientAs a$e $rou!, aortic stenosis is most likely to result from a con$enital bicus!i" aortic valve or, less likely, as a sequelae of rheumatic valvular "isease. 7he an$inal ty!e sym!toms "evelo! in the !resence of normal coronary arteries because there is increase" "eman" by the hy!ertro!hie" left ventricle, which e*cee"s the coronary arteriesA ability to meet "eman"s. 7he murmur of aortic insufficiency is a blowin$ "iastolic murmur usually associate" with a wi"e !ulse !ressure 'choice A).

As "escribe" above, coronary artery "isease is very unlikely in this !atientAs a$e ran$e, so choices C an" % are wron$. +itral insufficiency !resents with a blowin$ holosystolic murmur, hear" best at the car"iac a!e* an" ra"iatin$ to the a*illa 'choice (). 0:) (*!lanation: 7he correct answer is C. 7his is a $an$lion cyst. 7hese cystic swellin$s occur on the han"s, !articularly on the "orsal as!ect of the wrists, an" are usually near or attache" to ten"on sheaths an" <oint ca!sules. About C/= of them arise near the sca!holunate <oint on the back of the wristG other common sites inclu"e the "istal volar as!ect of the ra"ius an" the fle*or ten"on sheath. 7he lesions a!!ear to be "e$enerative rather than tumorous in character, an" the $elatinous material foun" in the center has a hi$h hyaluronic aci" content. +any $an$lion cysts re$ress s!ontaneously or after nee"le as!iration of the contents. 6ecurrent $an$lia or $an$lia that are cosmetically unacce!table to the !atient can be sur$ically e*cise", but may recur after e*cision. Aneurysmal bone cyst 'choice A) is a cystic lesion usually locate" in a lon$ bone. It is most commonly i"entifie" before a$e ;>. -elon 'choice ) is an infection of the !ul! s!ace of a fin$er. 2er!etic whitlow 'choice %) is a viral infection of the "istal fin$er. Osteoi" osteoma 'choice () is a beni$n bone tumor that is most common in lon$ bones. 0;) (*!lanation: 7he correct answer is A. 7he !atientAs hy!o*ia 'on room air) is most likely a result of bronchos!asm in the settin$ of !re4e*istin$ reactive airways "isease. 7he a!!ro!riate treatment, therefore, is a broncho"ilator, an" the most a!!ro!riate broncho"ilator woul" be albuterol, since it "oes not have the unwante" associate" si"e effect of !romotin$ the "ryin$ of res!iratory secretions as with an antimuscarinic a$ent. An i!ratro!ium nebuli#er 'choice ) woul" be less !referre" com!are" with the albuterol nebuli#er since it is associate" with the risk of "ryin$ mucous secretions an" worsenin$ !ossible airway !lu$$in$. 7his is a "efinite concern in this !atient since his bronchos!asm is most likely occurrin$ in the settin$ of bronchitis '!ro"uctive cou$h without evi"ence of !neumonia to e*amination or chest ra"io$ra!h).

A mi*ture of helium an" o*y$en 'choice C) can be consi"ere" when o*y$enation is im!aire" "es!ite ma*imal thera!y with broncho"ilatory a$ents. It is not use" as a first4 line a$ent for bronchos!asm. 7he low "ensity of this $as makes flow throu$h hi$h4 resistance airways fasterG thus, it can be use" to "eliver o*y$en to !atients with a severe bronchos!astic attack. I1 beta a"rener$ic blocka"e 'choice %) will worsen the bronchos!asm by inhibitin$ beta4 ; a"rener$ic rece!tor me"iate" broncho"ilation. 7herefore, it has no !lace in the treatment of bronchos!asm. 6acemic e!ine!hrine 'choice () is reserve" for refractory bronchos!asm or for the treatment of airway e"ema associate" with ana!hyla*is. 00) (*!lanation: 7he correct answer is (. 7his I1 "ru$ user has a classic "escri!tion of a )ta!hylococcus aureus !neumonia with cavitation an" a "ia$nostic .ramAs stain. A!!ro!riate thera!y woul" be to a"mit the !atient for I1 nafcillin. 2is s!utum shoul" also be culture" to "etermine sensitivity. If the !atient fails to im!rove with I1 nafcillin, or his s!utum culture "emonstrates resistance to nafcillin, the a!!ro!riate ne*t thera!y woul" be I1 vancomycin. Oral antibiotics, such as "iclo*acillin 'choice A), am!icillin 'choice ), an" erythromycin 'choice C) are not a!!ro!riate in this !atient, who has a "evastatin$, !otentially fatal, !neumonia 'with a 0>= to ?>= mortality rate) that is actively "estroyin$ his lun$s. )ta!hylococcus aureus is one of the notoriously "ru$4resistant or$anisms, an" I1 erythromycin 'choice %) is not consistently active a$ainst this or$anism. If a !enicillinase4resistant !enicillin, such as nafcillin, cannot be use" because of reason of !atient aller$y, su$$este" alternative "ru$s inclu"e a first or secon" 'but not thir") $eneration ce!halos!orin or clin"amycin. +ethicillin4resistant )ta!hylococcus aureus can be treate" with I1 vancomycin. 0?) (*!lanation: 7he correct answer is %. 7his !atient is havin$ a hy!ertensive emer$ency, $iven the !resence of en"4or$an "ama$e in the settin$ of hy!ertension. 2y!ertensive ence!halo!athy counts as an emer$ency. Imme"iate thera!y before the laboratory results are known is nee"e". 7his requires imme"iate, but not !reci!itous, lowerin$ of the bloo" !ressure over a !erio" of minutes to hours. )o"ium nitro!russi"e is use" since it allows titratable bloo" !ressure re"uction.

Observation 'choice A) is not a!!ro!riate for this !atient, since he has an emer$ent me"ical con"ition that nee"s to be a""resse". %iabetic ketoaci"osis or se!sis 'choice ) may !resent with mental status chan$es an" an anion $a!. 2owever, the !atient most likely has hy!ertensive ence!halo!athy, an" this nee"s to be a""resse" first. A beta blocker 'choice C) may re"uce the !ulse but will not cause much vaso"ilation or arterial "ilation, which is nee"e" to imme"iately lower the bloo" !ressure. It may use" as an a"<unct thera!y. I1 labetalol may !rove to be an acce!table alternative, however. 7he !atient has mental status chan$es, an" menin$itis is on the "ifferential. 2owever, mana$ement of the bloo" !ressure shoul" not be "elaye" to !erform a lumbar !uncture 'choice (). -urthermore, he is afebrile, !lacin$ menin$itis lower on the "ifferential. 0/) (*!lanation: 7he correct answer is (. 7his !atient has a history of sclero"erma, which can cause eso!ha$eal "ysfunction secon"ary to fibrosis. &atients with sclero"erma are very vulnerable to aci" reflu* because neither !eristalsis nor the lower eso!ha$eal s!hincter function normally after much of the muscle tissue has been re!lace" by fibrosis. 7he situation is com!licate" by the fact that !ersistent aci" reflu* will eventually itself in"uce scarrin$ with formation of a !e!tic stricture in the "istal eso!ha$us. Care shoul" be taken to carefully mana$e even mil" reflu* sym!toms in !atients with sclero"erma to !revent 'or at least to slow) the "evelo!ment of this troublin$ com!lication. Althou$h chronic aci" reflu* can !re"is!ose for arrettAs eso!ha$us, with the risk of !ro$ression to a"enocarcinoma of the "istal eso!ha$us, squamous carcinoma of the u!!er thir" of the eso!ha$us 'choice A) woul" not be an e*!ecte" com!lication. Once !eristalsis is lost, there is almost never a return of !eristaltic activity 'choice ). 6everse" !eristalsis 'choices C an" %) is not usually seen clinically. %isor"ere" !eristalsis can be seen in sym!tomatic "iffuse eso!ha$eal s!asm. 0C) (*!lanation: 7he correct answer is (. 7his !atient has fin"in$s consistent with a lateral me"ullary syn"rome. .iven that this !atient has sub<ecte" himself to strenuous e*ercise the most likely e*!lanation is a vertebral artery "issection.

An astrocytoma may cause "i##iness an" unstea"iness alon$ with a hea"ache. Onset is insi"ious with !ro$ressive sym!toms 'choice A). eni$n !ositional verti$o 'choice ) causes su""en e!iso"es of "i##iness, ty!ically with !osition chan$e, an" with no associate" neurolo$ic sym!toms other than nysta$mus. ,abyrinthitis 'choice C) causes severe verti$o. &atients fin" it "ifficult to move, !referrin$ to remain still. +ulti!le sclerosis 'choice %) can cause "i##iness an" clumsiness, but the su""en onset an" constellation of fin"in$s makes this "ia$nosis unlikely. 0D) (*!lanation: 7he correct answer is %. 7his is a classic !atient !resentin$ with a very common "isease. 7he first ste! in correctly answerin$ this question is to i"entify this !atient as almost certainly havin$ coronary artery "isease. 2is a$e an" risk factors are im!ressive. )econ"ly, one must reali#e that "iabetics often have silent ischemia. 2is clinical !icture !rior to even obtainin$ an (C. shoul" alert you to the likelihoo" that he is havin$ ischemia. 7he (C. confirms ischemia in the anterior !ortion of the heart, the left ventricle. Once this is un"erstoo", his )0 an" !ulmonary e"ema are e*!laine". 7his is a me"ical emer$ency an" shoul" be "ealt with as such. 7reatment requires a$$ressive lowerin$ of myocar"ial o*y$en "eman", i.e., bloo" !ressure an" heart rate. Costochon"ritis 'choice A) or musculoskeletal chest !ain syn"rome 'choice C) are in no way su!!orte" by the !hysical e*amination. 7here is no ten"erness on !al!ation. A !ulmonary embolus 'choice ) is unlikely $iven the non!leuritic nature of his !ain an" his (C. chan$es that are hi$hly su$$estive of ischemia. +yocar"ial infarction 'choice () is not the "ia$nosis, as this is a clinical one that requires either characteristic )7 se$ment elevations on the (C. O6 elevations in serum markers for car"iac in<ury. 0@) (*!lanation: 7he correct answer is . 7he most common bacterial cause of acute infectious arthritis in a"ults is Neisseria $onorrhoeae, the causative or$anism of the venereal "isease $onorrhea. 7he initial sites of infection are mucosal surfaces such as cervi*, rectum, an" !haryn*. -rom these sites, the or$anism s!rea"s hemato$enously to involve <oints that can inclu"e the small <oints of the han"s, wrists, an" ankles, an" the lar$er <oints of the elbows an" knees. &atients !resent with <oint swellin$, !ain, warmth, an" ten"ernessG the onset may be ra!i", e.$.,

over a few hours. .onococcal arthritis may also 'an" commonly) occur as !art of a "ermatitis4!olyarthritis4tenosynovitis syn"rome. (nterobacter'choice A) is a relatively uncommon cause of se!tic arthritis. &seu"omonas aeru$inosa'choice C) accounts for about ?>= of the infectious arthritis not "ue to Neisseria $onorrhoeae. )ta!hylococcus aureus'choice %) accounts for about ?/= of the infectious arthritis not "ue to Neisseria $onorrhoeae. )tre!tococci 'choice () account for about I= of the infectious arthritis not "ue to Neisseria $onorrhea. 0I) (*!lanation: 7he correct answer is . 7his !atient has a classic !resentation of an acute community acquire" bacterial !neumonia, as "emonstrate" by the fin"in$s of acute onset of fevers, ri$ors, an" a !ro"uctive s!utum. 2is !hysical e*amination is consistent with a ri$ht lower lobe !neumonia an" consoli"ation in this re$ion. A s!utum .ramAs stain may "emonstrate the or$anism res!onsible for this !atientAs !neumonia. 7he ne*t ste! woul" then be a chest *4 ray to confirm the !resence of a !neumonia an" i"entify its si#e an" any associate" !ara!neumonic effusions. A chest *4ray film 'choice A) is also in"icate", but may take an hour or more to actually be obtaine". -or this reason, collectin$ s!utum for a .ramAs stain, before the *4ray, may shorten the time to "efinitive thera!y. Chest C7 scan 'choice C) is usually reserve" for evaluation of sus!ecte" masses. &ulmonary function tests 'choice %) an" !eak e*!iratory flow rate measurement 'choice () are usually reserve" for evaluation of chronic lun$ "iseases, such as em!hysema, chronic bronchitis, !ulmonary fibrosis, an" asthma. ?>) (*!lanation: 7he correct answer is %. 7his !atient most likely has neuro$enic arthro!athy. 7he "isor"er "evelo!s in a settin$ of im!aire" !ain !erce!tion an" !osition sense, an" can cause a ra!i"ly "estructive osteoarthritis4like arthro!athy. 7he !roblems a!!ear to be "ue to unfelt minor in<ury, without the normal res!onse of !ain, causin$ restin$ of, an" natural s!lintin$ 'by muscle contraction or shiftin$ !osition) of the affecte" <oints. 7he most commonly encountere" settin$ is "iabetic foot "isease. 9ou shoul" be aware that it can also occur in a wi"e

variety of other con"itions, inclu"in$ tabes "orsalis 'sy!hilis), syrin$omyelia, Arnol"4 Chiari malformation, menin$omyelocele, le!rosy, tumors of !eri!heral nerve or s!inal cor", vertebral "isease with "ama$e to !eri!heral nerves, amyloi"osis, an" familial here"itary neuro!athies. 7he <oints affecte" are those that have lost !ain innervation, !ossibly in association with !alsies of nearby muscles. Avascular necrosis 'choice A) usually involves a sin$le <oint rather than multi!le <oints. .out 'choice ) can involve the feet with crystal "e!osition, but "oes not usually cause osteo!hyte formation. 7he $reat toe is the most likely site of involvement in the foot. Infectious arthritis 'choice C) can com!licate "iabetes mellitus, but usually !ro"uces an obviously inflame" <oint. 6heumatoi" arthritis 'choice () usually !ro"uces obviously inflame" <oints an" more frequently involves the han"s or lar$e <oints of the feet. ?:) (*!lanation: 7he correct answer is C. ,umbar stenosis is cause" by "e$enerative chan$es in the lumbosacral s!ine. 7he history is that of va$ue low back !ain with subtle !hysical e*amination fin"in$s referable to im!in$ement on motor an" sensory roots. Acute "isc herniation 'choice A) is characteri#e" by low back "iscomfort an" !ain e*ten"in$ in a ra"icular fashion. (*amination is consistent with im!in$ement on a sin$le sensory or motor root. Cervical stenosis 'choice ) can cause a myelo!athy an" resultant $ait !roblem, but this !atient has "e$enerative chan$es in the lumbosacral re$ion. +yo!athy 'choice %) can cause an im!aire" $ait, low back "iscomfort an" hy!oactive muscle refle*es, ty!ically at the knee. Weakness ten"s to be symmetric an" !ro*imal. N&2 'choice () causes an a!ra*ic $ait, "ementia an" urinary incontinence. ?;) (*!lanation: 7he correct answer is (. )evere mi"e!i$astric !ain ra"iatin$ to the back in a !atient with history of recent heavy alcohol use shoul" be !resume" to be !ancreatitis. 7he "ia$nosis can be confirme" by measurin$ serum amylase an" li!ase levels. )evere acute !ancreatitis, such as this !atient has, can "estroy lar$e !arts of both the en"ocrine an" e*ocrine !ancreas. %estruction of the e*ocrine !ancreas can release many en#ymatically or !hysiolo$ically

active substances that can have local or "istant effects. &ulmonary com!lications inclu"e !leural effusion secon"ary to chemical irritation of the "ia!hra$m, increase" alveolar arterial $ra"ient, an" a"ult res!iratory "istress syn"rome 'A6%)) secon"ary in !art to the hy!otension that can be a !rominent feature of !ancreatitis. Cavitatin$ !neumonias 'choice A) "o not occur as a com!lication of acute !ancreatitis since the con"ition is not "ue to an infection. 2y!ercalcemia 'choice ) may be a cause of acute !ancreatitis but is not a com!lication of acute !ancreatitis. On the other han", hy!ocalcemia can result from severe acute !ancreatitis. Ne!hrotic syn"rome 'choice C) "oes not occur with acute !ancreatitis althou$h severe renal insufficiency "ue to hy!ovolemia may occur. An oral a!hthous ulcer 'choice %) is a common con"ition unrelate" to !ancreatitis. It is associate" with Crohn "isease, althou$h the vast ma<ority of !atients with a!hthous ulcers "o not have un"erlyin$ systemic "isease. ?0) (*!lanation: 7he correct answer is A. 7his is viral con<unctivitis, which is an acute con<unctival inflammation most often cause" by a"enovirus. +ost of the !atients with this con"ition have either been recently e*!ose" to someone with viral con<unctivitis or have recently ha" a viral u!!er res!iratory infection. 7he !resentation illustrate" in the question stem is ty!ical. )evere cases also a""itionally have !hoto!hobia an" a forei$n bo"y sensation. &seu"omembranes of fibrin an" inflammatory cells on the con<unctival surfaces can occur. Corneal involvement can leave resi"ual scarrin$ visible by slit lam! for u! to ; years after a severe case. Cases are self4limite" but ten" to be e*tremely conta$ious, so care shoul" be taken to have both the !atient an" the !hysician wash their han"s very thorou$hly after touchin$ the face. 2er!es virus 'choices an" C) can cause corneal ulceration, hy!erkeratosis, or scarrin$.

Neisseria $onorrhoeae'choice %) can cause con<unctivitis in both a"ults an" neonates, an" !ro"uces a !urulent, rather than a watery "ischar$e. )ta!hylococcus aureus'choice () is a common cause of bacterial con<unctivitis, an" !ro"uces a !urulent, rather than a watery "ischar$e. ??) (*!lanation: 7he correct answer is

C. &ro!ranolol is consi"ere" to be relatively contrain"icate" in !atients with !eri!heral clau"ication. Althou$h beta4blockers are e*cellent me"ications in !atients with hy!ertension an" a history of myocar"ial infarction, the beta blocka"e may !re"is!ose to arterial vasoconstriction !eri!herally. 7he "ose of beta4blockers in this !atient shoul" therefore be re"uce" or $ra"ually eliminate". %ecreasin$ "iltia#em 'choice A) woul" not be of value since calcium channel blockers, such as "iltia#em, ten" to cause arterial vaso"ilation. 7he "ecrease" use of hy"rochlorothia#i"e 'choice ) woul" have no effect on arterial vaso"ilation. Increasin$ as!irin 'choice %) "oes not re"uce the sym!toms of clau"ication. Increasin$ isosorbi"e "initrate 'choice () woul" increase veno"ilation but woul" not have an influence on the arterial vascular be". ?/) (*!lanation: 7he correct answer is (. A monoarticular arthritis shoul" raise the question of a se!tic versus a crystalline arthritis. )e!tic arthritis an" crystalline arthritis can be notoriously similar in their clinical !resentations, so clearly establishin$ the "ia$nosis, !articularly in a !atient who "oes not have a known history of !rior crystalline "isease, is of $reat a"vanta$e before initiatin$ "efinitive thera!y. An arthrocentesis is the most a!!ro!riate choice an" allows e*amination of flui" for white bloo" cells, crystals, an" bacteria. It woul" be ina!!ro!riate to start em!iric thera!y with nonsteroi"al anti4inflammatory "ru$s 'N)AI%s) 'choice ), colchicine 'choice A), or antibiotics 'choice C) before the arthrocentesis. An *4ray film 'choice %) woul" not yiel" a "efinitive "ia$nosis in this situation. ?C) (*!lanation: 7he correct answer is (. 7his !atient most likely has acute bronchitis. Acute bronchitis in a healthy !atient with no other me"ical con"itions is often "ue to a viral infection that is usually self4limite". .iven that this !atient has only ha" ; "ays of sym!toms, an antibiotic is not necessary an" is ina!!ro!riate. If the sym!toms !ersist for lon$er than : week, a macroli"e antibiotic may be $iven. A chest *4ray film an" a s!utum culture are not in"icate". A"mission to the hos!ital for me"ical mana$ement 'choice A) is ina!!ro!riate for a healthy !atient with acute bronchitis.

A chest *4ray film 'choice ) has no role in the "ia$nosis of acute bronchitis in a healthy !atient. A s!utum culture 'choice C) is use" to i"entify or$anisms, but shoul" only be use" in el"erly !atients with chronic "isease that fail antibiotic thera!y. )en"in$ the !atient home with antibiotic thera!y 'choice %) is a!!ro!riate mana$ement for acute bronchitis in an el"erly !atient with chronic "isease. A macroli"e is the treatment of choice. ?D) (*!lanation: 7he correct answer is %. 7his el"erly woman with a history of atherosclerotic vascular "isease as "emonstrate" by a history of a myocar"ial infarction an" e*ertional an$ina has "evelo!e" colitic sym!toms, as "emonstrate" by the left lower qua"rant !ain an" bloo"y "iarrhea. 7his is ty!ically "ue to ischemia of small branches of the inferior mesenteric artery. 7he "ia$nosis is sus!ecte" clinically an" $enerally confirme" with a fle*ible si$moi"osco!y, since many cases involve the rectosi$moi" re$ion. Arteriovenous malformation 'choice A) !ro"uces !ainless blee"in$ in the el"erly, most commonly locate" in the cecum an" often, but not always, associate" with aortic stenosis. %iverticulitis 'choice ) "oes !resent with !ost!ran"ial cram!y left lower qua"rant ab"ominal !ain an" fever, but "oes not !ro"uce bloo"y "iarrhea. Often, these !atients have sym!toms of consti!ation. %iverticulosis 'choice C) also !resents with !ost!ran"ial cram!y left lower qua"rant !ain or may be asym!tomatic, but "oes not !ro"uce a bloo" "iarrheal illness. Flcerative colitis 'choice () ty!ically !resents in a subacute or chronic fashion in a youn$er !atient !o!ulation. Althou$h it can occur in this !atientAs a$e $rou!, it rarely !resents as an acute illness. ?@) (*!lanation: 7he correct answer is %. 7his !atient most likely has bulimia nervosa. It is a con"ition characteri#e" by recurrent e!iso"es of bin$e eatin$ followe" by a com!ensatory behavior to !revent wei$ht $ain 'vomitin$, e*ercise, la*ative abuse). Other features inclu"e stealin$ 'foo"), alcohol an" "ru$ abuse, self4mutilation, an" "e!ression. 7he in"ivi"uals are usually at or sli$htly over the normal wei$ht for their hei$ht, se*ual activity is normal or increase", an" they continue to menstruate. Clinical fin"in$s that are cause" by recurrent vomitin$

inclu"e "ental caries, !erio"ontal "isease, !haryn$eal lacerations an" nail chan$es. +etabolic alkalosis an" hy!okalemia are also !resent. Com!lications inclu"e as!iration an" ru!ture of the eso!ha$us or stomach. H%o you ever feel $uilty about "rinkin$ alcoholJH 'choice A) is a question that woul" be aske" to an alcoholic. While alcohol abuse has been associate" with bulimia nervosa, it "oes not seem that this !atient is an alcoholic. H%o you ty!ically restrict your "iet to un"er @>> calories a "ayJH 'choice ) woul" be an in"ication of anore*ia nervosa, which is characteri#e" by the refusal to maintain a normal, healthy bo"y wei$ht an" the "isturbance of bo"y ima$e. 7hese in"ivi"uals have an intense fear of $ainin$ wei$ht an" therefore restrict foo" intake to "an$erously low levels. Women become amenorrheic, have "ecrease" se*ual "esire, rituali#e" e*ercise routines, an" chan$es in their skin an" hair. Com!lications inclu"e ventricular tachyarrhythmias. H2ave you ever taken la*atives as a way to lose wei$htJH 'choice C) an" H&lease "escribe your e*ercise routine.H 'choice () are questions that shoul" be aske" to both anore*ics an" bulimics. ,a*ative abuse an" e*ercise rituals are common in both "iseases, but la*ative abuse is !robably more common in bulimia, while rituali#e" e*ercise is more common in anore*ia. 7his !atient has the clinical fin"in$s associate" with vomitin$ an" therefore the question about bin$ein$ an" !ur$in$ woul" more likely establish a "ia$nosis. ?I) (*!lanation: 7he correct answer is A. Acute !ancreatitis is su$$este" by this !atientAs severe e!i$astric !ain, which ra"iates to his back an" is accom!anie" by nausea an" vomitin$. 7he "ia$nosis is confirme" by "emonstratin$ serum elevation of the !ancreatic en#ymes amylase an" li!ase. +ore than @>= of the hos!ital a"missions for acute a!!en"icitis are relate" to either biliary tract "isease 'with ty!ically a stone lo"$in$ in the "uct system below the entry of the !ancreatic "uct) or alcoholism, an" this !atient a"mits to recent active "rinkin$. Acute !ancreatitis is a very "an$erous "isease, not only because it !otentially "estroys lar$e areas of !ancreatic tissue, but also because the "estruction of the e*ocrine $lan" tissues !otentially releases a $reat many en#ymatically or !hysiolo$ically active substances into the bloo" stream. Amon$ the release" substances is !hos!holi!ase, which circulates throu$h the bloo"stream an" "ama$es the alveolar ca!illary membranes in the lun$s, !re"is!osin$ for a"ult res!iratory "istress syn"rome 'A6%)). &ancreatic cancer 'choice ) "oes not "evelo! as a sequela of acute !ancreatitis. 2y!ercalcemia 'choice C) "oes not occur as a result of !ancreatitis, althou$h it may be an infrequent cause of !ancreatitis. On the other han", severe acute !ancreatitis can lea" to the sa!onification of calcium by the unabsorbe" fats in the small bowel an" lea" to hy!ocalcemia.

%ecrease" intravascular volume 'choice %) "oes occur, but it lea"s to hy!otension an" not hy!ertension. In !atients with severe !ancreatitis, insulin !ro"uction an" release is im!aire" 'choice (), lea"in$ to hy!er$lycemia rather than hy!o$lycemia. />) (*!lanation: 7he correct answer is %. 7he history of severe 2I14relate" immune com!romise an" evi"ence of "estruction of myelin at multi!le sites in the CN) !oints to a "ia$nosis of !ro$ressive multifocal leukoence!halo!athy '&+,). &+, is cause" by NC virus, a !a!ovavirus that !ro"uces asym!tomatic infections in immunocom!etent hosts. Oli$o"en"rocytes in active lesions contain characteristic intranuclear inclusions. Fsually, bio!sy of the lesions is not necessary, as a !resum!tive "ia$nosis of &+, can be ma"e on clinical $roun"s. &+, is also seen in !atients with lym!homas, or those receivin$ or$an trans!lants. In AI%)4"ementia com!le* 'choice A), there is no focal brain lesion. )ometimes, "iffuse, but usually mil", cerebral atro!hy can be i"entifie" by +6I. &atients !resent with !ro$ressive "ementia, often associate" with incontinence an" "isorientation. Cerebral to*o!lasmosis 'choice ) ty!ically manifests with a roun", well4circumscribe" lesion that shows a !eri!heral rim of contrast enhancement. C+1 ence!halitis 'choice C) has a !re"ilection for the !eriventricular $ray matter an" e!en"yma, as well as the retina. +ulti!le sclerosis '+)) 'choice () "oes not "evelo! in the conte*t of immune im!airment. %emyelinatin$ !laques of +) are ty!ically well4"emarcate" an" most commonly locate" in the !eriventricular re$ions.

USMLE Step 2 Practice Test Block ( :ame; +nstr#ctions; Ans'er the *#estions $elo' to the $est of yo#r a$ility. =hen yo# finish the test% click the 2heck $#tton at the $ottom to )ie' the res#lts.

1.An asymptomatic <4!year!ol" 'oman comes to the physician for a health maintenance e.amination. She has smoke" 2 packs of cigarettes "aily for 2( years. She ha" $een se.#ally acti)e #ntil 2 years ago 'hen her se.#al partner "ie". /er $loo" press#re is 1147(4 mm /g% p#lse is <47min% an" respirations are 1<7min. =hich of the follo'ing is the most appropriate screening testa6 2hest .!ray film to screen for l#ng cancer $6Electrocar"iogram 3E2C6 to screen for asymptomatic coronary artery "isease 32A@6 c6 E.amination of the #terine a"ne.a to screen for o)arian cancer "6Pap smear to screen for cer)ical cancer e6Sp#t#m cytology to screen for l#ng cancer :ormal La$s 2.A 44!year!ol" man is $ro#ght to the emergency "epartment after cons#ming a large *#antity of acetaminophen in a s#ici"e attempt. /e "escri$es taking at least 4 pills $#t "enies other ingestions. /e has a history of se)ere "epression $#t is other'ise healthy. /e is afe$rile an" has sta$le )ital signs. /is physical e.amination is #nremarka$le. The time of ingestion is 2 ho#rs prior to his present a"mission. Ci)en the history s#ggesti)e of a se)ere o)er"ose% 'hich of the follo'ing is the most appropriate treatmenta6Amyl nitrite $6Methylene $l#e

c6 :alo.one "6:!acetylcysteine e6Penicillamine :ormal La$s &.A (2!year!ol" 'oman presents complaining of se)ere left lo'er *#a"rant pain for 24 ho#rs. She has a long history of constipation an" fre*#ently has left lo'er *#a"rant cramping after meals. >)er the past 24 ho#rs% she has ha" increasing "iscomfort in the left lo'er *#a"rant an" has ha" a temperat#re to &8.4 2 3142.1 56. >n e.amination% there is ten"erness to palpation an" g#ar"ing in the left lo'er *#a"rant. A 2T scan re)eals an a$scess in)ol)ing an" contig#o#s to the sigmoi" colon. =hich of the follo'ing is the most likely ca#se of this patient,s con"itiona6Blee"ing at the site of the a$scess $6Micro! or macroperforation of a "i)ertic#l#m c6M#cosal inflammation "6M#cosal ischemia e6Pancreatitis :ormal La$s 4.A 1&!year!ol" girl is $ro#ght to the emergency "epartment $eca#se of generaliDe" seiD#res lasting for 14 min#tes. She has ha" p#r#lent otitis for the last 'eek% 'hich has $een poorly responsi)e to anti$iotic treatment. /er temperat#re is &8. 2 314& 56% $loo" press#re is 121714 mm /g% p#lse is 1<7min% an" respirations are 117min. She appears oriente" an" cooperati)e "#ring physical e.amination. 5#n"#scopy re)eals papille"ema. =hich of the follo'ing is the most appropriate ne.t step in managementa6Bloo" c#lt#res $6EEC st#"ies c62T7MA+ of the hea" "6L#m$ar p#nct#re :ormal La$s

. A 4&!year!ol" 'oman presents 'ith a se)eral month history of "yspnea on e.ertion. She "enies chest pain or a family history of coronary artery "isease. She has a history of rhe#matoi" arthritis. /er chest .!ray film is remarka$le for calcification of the heart $or"er on the lateral film. She appears to ha)e constricti)e pericar"itis. =hich of the follo'ing physical e.amination fin"ings 'o#l" most likely $e e.pecte" in this patienta6 2entral cyanosis $6>pening snap an" "iastolic r#m$le c6Pericar"ial knock "6Systolic e9ection m#rm#r e6 =i"ely split S2 :ormal La$s <. A 8!year!ol" man presents to his primary care physician 'ith shortness of $reath. The patient has $een coming to this office for many years for treatment of osteoarthritis. /e has a long smoking history of greater than 1 4 pack years. /e ro#tinely takes only a nonsteroi"al anti!inflammatory agent for pain. /e presents 'ith "ays of fe)er an" chills% as 'ell as a pro"#cti)e co#gh. /e has not $een hospitaliDe" recently an" li)es at home 'ith his 'ife an" no sick contacts. The patient reports a slo'ly 'orsening an" progressi)e shortness of $reath e)en 'ith minimal e.ertion. There is no associate" chest pain or press#re. The patient,s p#lmonary stat#s is e)al#ate" 'ith p#lmonary f#nction testing. =hich of the follo'ing are the most representati)e res#lts for this patient 'ith mo"erate chronic o$str#cti)e p#lmonary "iseasea65E?1 2.1 L% 5E?175?2 4.8% resi"#al )ol#me 2.4 L% total l#ng capacity (L $65E?1 4.8 L% 5E?175?2 4.1% resi"#al )ol#me &. L% total l#ng capacity 8L c65E?1 2.< L% 5E?175?2 4.1% resi"#al )ol#me 1.& L% total l#ng capacity L "65E?1 1.& L% 5E?175?2 4.8% resi"#al )ol#me 4.8 L% total l#ng capacity 8L e65E?1 4.8 L% 5E?175?2 4.8% resi"#al )ol#me 1.4 L% total l#ng capacity 12 L :ormal La$s

(. A (<!year!ol" man is $ro#ght to the emergency "epartment 3E@6 $y am$#lance for altere" mental stat#s. The patient has a past me"ical history significant for m#ltiple myeloma. /e 'as last seen $y his primary care physician 4 'eeks ago an" 'as reporte" $y family mem$ers no' present in the E@ to $e 'ell at that time. T'o "ays ago% the family notice" that the patient ha" increase" conf#sion an" o)er the ne.t 41 ho#rs% slo'ly $ecame o$t#n"e" an" co#l" no longer follo' simple comman"s. >n physical e.amination% his $loo" press#re is 847 4 mm /g% his p#lse is 1147min% his m#co#s mem$ranes are "ry% there is no 9#g#lar )eno#s p#lse e)i"ent. /is l#ngs are clear an" he is alert only to person an" cannot follo' comman"s. /is "eep ten"on refle.es are $risk. Ser#m chemistries an" to.icologies are sent. =hich of the follo'ing is the most appropriate initial step in managementa6A"minister +? salmon calcitonin $6A"minister +? pami"ronate c6A"minister +? $icar$onate "6Begin aggressi)e "i#resis 'ith f#rosemi"e e6 Begin aggressi)e intra)eno#s hy"ration an" "i#resis :ormal La$s 1.A <4!year!ol" 'oman 'ith a long history of hypertension presents 'ith a complaint of increasing an" rec#rrent shortness of $reath 'ith minimal e.ertion. She is note" to ha)e a prominent precor"ial imp#lse% an" a chest .!ray film re)eals a prominent left )entric#lar sha"o'. A stress test is negati)e for ischemia. She is fo#n" to ha)e left )entric#lar hypertrophy on an echocar"iogram. 2atheteriDation re)eals normal )entric#lar e9ection fractions. =hich of the follo'ing is the most likely "iagnosisa6 2hronic o$str#cti)e p#lmonary "isease 32>P@6 $6@iastolic "ysf#nction c6Myocar"ial ischemiaMyocar"ial ischemia "6Aeacti)e air'ays "isease e6Systolic congesti)e heart fail#re :ormal La$s 8. A <8!year!ol" man in the car"iothoracic intensi)e care #nit is postoperati)e "ay < for a coronary artery $ypass graft. /e 'as initially e.t#$ate" on postoperati)e "ay 1% $#t 'as then re!int#$ate" the

follo'ing "ay for aspiration pne#monia% 'hich ha" 'orsene" "espite a"ministration of +? penicillin an" gentamicin. /e is also recei)ing "aily f#rosemi"e% aspirin% an" s#$c#taneo#s heparin 444 #nits. >ne ho#r ago% he passe" a large )ol#me of $lack tarry stool. /is temperat#re is &1.8 2 3142 56% $loo" press#re is 114784 mm /g% an" p#lse is 1427min. Lo#" rhonchi are hear" $ilaterally. /e has a reg#lar% rapi" heart rhythm 'ith a ++7?+ systolic m#rm#r. /is a$"omen is soft 'ith mil" epigastric ten"erness. :o masses are palpa$le. Aectal e.amination re)eals no masses% an" melena is present. La$oratory st#"ies sho' a le#kocyte co#nt of 14%444% a hemoglo$in of 8.1 g7"L% an" a hematocrit of 28B. =hich of the follo'ing is the most likely so#rce of this patient,s $lee"inga6 2ecal arterial )eno#s malformations $6@iff#se gastritis c6Erosi)e esophagitis "6Esophageal )arices e6 Ciant gastric #lcer :ormal La$s 14. A <4!year!ol" pe"iatrician "e)elops hea"ache of s#""en onset% sl#rring of speech% an" conf#sion. /e is $ro#ght to the emergency "epartment% 'here he arri)es 'ith a Clasgo' coma score of 1. An emergency 2T scan of the hea" re)eals 'ell!"emarcate" lo$ar $lee"ing in)ol)ing the left frontal lo$e% centere" in the corte. an" e.ten"ing into the #n"erlying 'hite matter. The patient,s history is negati)e for hypertension or )asc#lar risk factors. =hich of the follo'ing is the most pro$a$le #n"erlying ca#se of the patient,s $lee"inga6Berry ane#rysm r#pt#re $6Bri"ging )ein tears c62ere$ral amyloi" angiopathy "6Mi""le meningeal artery laceration e6Un"iagnose" hypertension :ormal La$s 11.A !year!ol" man 'ith recent syncopal episo"e is a"mitte" to the hospital in congesti)e heart fail#re. /is $loo" press#re is 1<47144 mm /g% an" p#lse is 847min. /e has a gra"e 27< harsh systolic e9ection

m#rm#r. An echocar"iogram re)eals a thickene" )entric#lar sept#m an" systolic anterior motion of the mitral )al)e. =hich of the follo'ing 'ill most likely $e fo#n" in this patienta6@ecrease" m#rm#r 'ith han" grip $6@ecrease" m#rm#r 'ith ?alsal)a c6@elaye" caroti" #pstroke "6+ncrease" m#rm#r 'ith s*#atting e6 M#rm#r ra"iating to caroti" arteries :ormal La$s 12.A 1!year!ol" 'oman is postoperati)e "ay 4 after a colon cancer resection. She "e)elops the s#""en onset of sharp left!si"e" chest pain% 'hich increases 'ith inspiration. She is note" to $e hypo.emic% an" an arterial $loo" gas re)eale" a p/ of (. 4% a p2>2 of 24 mm /g% an" a p>2 of & mm /g. A )entilation!perf#sion scan is consistent 'ith a massi)e left!si"e" p#lmonary em$ol#s to the left mainstem p#lmonary artery. @espite 144B o.ygen a"ministere" )ia a tight fitting face mask% she is #na$le to raise her o.ygen sat#ration a$o)e 84B. She is int#$ate"% an" positi)e!en" e.piratory press#re 3PEEP6 is a""e". T'o ho#rs later% she s#""enly $ecomes hypotensi)e% an" her o.ygenation is f#rther impaire". Breath so#n"s are a$sent in the right hemithora.% an" there are "isten"e" neck )eins as 'ell as tracheal "e)iation to the left. =hich 'o#l" $e the most appropriate ne.t step in managementa6>$tain a repeat )entilation!perf#sion scan $6>$tain p#lmonary angiography c6+nitiate therapy 'ith +? streptokinase "6Perform a nee"le thoracostomy in the right hemithora. e6Begin $roa"!spectr#m anti$iotics :ormal La$s 1&.A 22!year!ol" gra"#ate st#"ent ret#rns from hiking in the Aocky Mo#ntains in 2olora"o. Beginning on the last "ay of his trip an" contin#ing o)er the past 'eek% he has "e)elope" #pper a$"ominal cramping 'ith gaseo#s "istention% fre*#ent er#ctations% an" loose% non! $loo"y stools. >n physical e.amination% he is afe$rile an" on a$"ominal e.amination he has mil" ten"erness in the mi" epigastric area. /is stool is g#aiac negati)e. =hich of the follo'ing is the most likely infection ca#sing his symptomsa6Entamoe$a histolytica

$6Enteroin)asi)e Escherichia coli c6Enteroto.igenic E. coli "6Ciar"ia lam$lia e6Shigella "ysenteriae :ormal La$s 14.A 22!year!ol" constr#ction 'orker is $ro#ght to the emergency "epartment after $eing pinne" $y a steel $eam% 'hich ha" fallen on his legs an" mi"!torso. The patient 'as $ro#ght to the hospital $y the EMS team 'ho ha" e.tricate" him. At the scene% his Clasgo' score 'as 1 71 % an" he 'as alert an" oriente". >n arri)al% the patient appears $loo"y% an" in a significant amo#nt of pain% $#t still alert. /is e.amination re)eals $ilaterally cr#she" lo'er e.tremities 'ith a )isi$ly p#lsating $lee". Emergency la$oratory "ata are as follo's; So"i#m.............................................14& mE*7L Potassi#m......................................... .2 mE*7L Bicar$onate......................................24 mE*7L 2hlori"e .........................................81 mE*7L Urea nitrogen...................................&( mg7"L 2reatinine........................................1.1 mg7"L 2reatine phosphokinase....................1 %&44 U7L Aspartate aminotransferase..............112 +U7L Alanine aminotransferase.................88 +U7L /ematocrit.......................................&1B Arterial $loo" gas on 44B o.ygen.....P>2 S 4 mm /g% P2>2 S &( mm /g% p/ S (.&& =hich of the follo'ing is the most appropriate inter)ention at this timea612!lea" E2C $6Echocar"iogram c6+? crystalloi" an" $icar$onate "65ractionate" creatine kinase le)els e6 Aight #pper *#a"rant #ltraso#n" :ormal La$s 1 . A &2!year!ol" man has ha" asthma for the past 8 years. The symptoms are fre*#ently e.acer$ate" $y changes in the 'eather an" ho#sehol" allergens. /e has ha" t'o emergency "epartment )isits o)er the past year $#t has not re*#ire" hospitaliDation. /e #ses a

metaproterenol inhaler spora"ically for symptom relief. =hich of the follo'ing 'o#l" $e the most appropriate therapy to maintain remission $et'een his asthmatic attacksa6Aminophylline $6Beclomethasone inhaler c62romolyn nasal spray "6Metaproterenol inhaler e6 >ral pre"nisone :ormal La$s 1<. A (4!year!ol" 'oman 'ith long history of "ia$etes mellit#s presents 'ith s#""en onset of 'eakness in the right arm. Physical e.amination confirms loss of strength in the right #pper e.tremity an" f#rther "emonstrates s#$tle $ilateral ne#rologic "eficits% $oth sensory an" motor. A 2T scan of the hea" sho's small p#nche"!o#t hypo"ense areas in the $asal ganglia an" anterior lim$ of the left internal caps#le. =hich of the follo'ing is the most likely "iagnosisa6Berry ane#rysms $62ere$ral hemorrhage c6Em$olic infarcts "6Lac#nar infarcts e6 Metastatic lesions :ormal La$s 1(.A ((!year!ol" man presents 'ith "ecreasing e.ercise tolerance. R#st 1 year earlier% he 'as a$le to play "o#$les tennis for 2 ho#rs. >)er the past fe' months% ho'e)er% he has ha" progressi)e "yspnea on e.ertion an" no' can 'alk only t'o $locks on le)el gro#n" $efore $ecoming short!'in"e". /e has also $een a'aking from sleep 'ith shortness of $reath an" re*#ires three pillo's to sleep comforta$ly. /e has a history of rhe#matic fe)er as a teenager. >n physical e.amination% his $loo" press#re is 1<17<4 mm /g% p#lse is 827min% an" respirations are 117min. /e is afe$rile. /e has 9#g#lo)eno#s "istention lying s#pine. /e has $i$asilar rales e.ten"ing 174 #p $oth posterior l#ng fiel"s. /e has a reg#lar S1 an" S2% 'ith a $lo'ing "iastolic m#rm#r hear" at the aortic area% 'hich is gra"e ++7+?. An S& is a#"i$le. The li)er e"ge is mil"ly ten"er% an" there is mo"erate lo'er e.tremity e"ema e.ten"ing to $oth

knees. =hich of the follo'ing me"ications 'ill most likely $e effecti)e in the management of his car"iac "isor"era6 2aptopril $62o#ma"in c6@igo.in "65#rosemi"e e6+sosor$i"e "initrate :ormal La$s 11. A <8!year!ol" man presents to the hospital for a follo'!#p physical e.amination an" chest 2T scan. /e has kno'n long!stan"ing l#ng "isease secon"ary to amio"arone therapy for atrial fi$rillation an" re*#ires ann#al e.aminations an" 2T scans to "oc#ment progression of his "isease. /e first took amio"arone < years ago an" has contin#e" to #se the "r#g to control his atrial fi$rillation. /is last p#lmonary f#nction tests sho'e" a mo"erate restricti)e "efect. =hich fin"ing on physical e.amination is most consistent 'ith a")ance" restricti)e l#ng "iseasea6+ncrease" anterior!posterior 3AP6 "iameter on inspection of the chest $6@iff#se e.piratory 'heeDes on a#sc#ltation of the chest c6@#llness to perc#ssion at the $ases of the posterior thora. "6 5ine inspiratory crackles at the $ases on a#sc#ltation of the chest e6 /yperresonance to perc#ssion of the chest :ormal La$s 18. A 2 !year!ol" ski instr#ctor presents to the local clinic complaining of fatig#e o)er the past 'eek. /e ha" pre)io#sly $een in e.cellent physical health an" ha" $een a$le to 'ork 12 ho#rs "aily. /e complains of "iff#se myalgias an" temperat#res as high as &8.2 2 3142. 56. /e has also $een complaining of "iffic#lty s'allo'ing $eca#se of a sore throat. /e "rinks appro.imately (2 o#nces of $eer on 'eeken"s an" smokes one pack of cigarettes per "ay. >n e.amination% his temperat#re is &1.( 2 3141.< 56% an" his posterior pharyn. is in9ecte". /e has $ilateral anterior an" posterior cer)ical a"enopathy% 'ith lymph no"es meas#ring as large as 1 cm. They are firm% mo$ile% an" mil"ly ten"er. >n a$"ominal e.amination% his li)er span is 14 cm in the mi"cla)ic#lar line% an" a spleen tip is palpa$le. There is no ascites or peripheral e"ema. =hich of the follo'ing is the most likely ca#se of his symptoms-

a6Ac#te hepatitis B infection $6+nfectio#s monon#cleosis c6:on!/o"gkin lymphoma "6Sarcoi"osis e6Streptococcal pharyngitis :ormal La$s 24. A 1(!year!ol" girl is $ro#ght to the emergency "epartment after $eing fo#n" #nresponsi)e $y her parents. /er father reports that she has $een )ery "epresse" o)er the past fe' 'eeks o)er a $reak!#p 'ith a $oy. The father "escri$es the "a#ghter as $eing isolate" an" spen"ing m#ch of her time alone in her room. =hen she faile" to get #p for school% the father 'ent into her locke" $e"room an" fo#n" her face "o'n an" $arely $reathing on her $e". An EMS team ga)e her a narcotic antagonist% +? "e.trose sol#tion% an" +? fl#i"s. +n a""ition% her trachea 'as int#$ate" for air'ay protection $efore she 'as $ro#ght to the emergency "epartment. >n physical e.amination% her p#pils are 1 mm $ilaterally an" minimally reacti)e. /er a$"omen is marke"ly "isten"e". An #rgent #rine an" ser#m to.icology screen re)eals the follo'ing "ata; Urine BenDo"iaDepine Positi)e >piates Positi)e Ser#m @iphenhy"ramine (84 Hg7L Aef. range; not "efine"G to.ic; I244 Hg7L Ethanol 281( Hg7L Aef. range; not "efine"G to.ic; I1444 Hg7L >lanDapine Present =hich of the follo'ing ingeste" s#$stances 'ill most likely threaten the patient,s life 'ithin the ne.t 24 ho#rsa6 BenDo"iaDepines $6@iphenhy"ramine c6Ethanol "6>lanDapine e6 >piates :ormal La$s

21.A 24!year!ol" 'hite 'oman presents complaining of < months of crampy a$"ominal pain. The pain has $een localiDe" to the right lo'er *#a"rant an" is ma"e 'orse $y eating. She has also note" an increase in the n#m$er of her $o'el mo)ements to appro.imately fo#r per "ay% an" the stools ha)e $ecome semi!forme". She "enies any fe)ers% chills% or night s'eats "#ring this perio". She has lost 1 po#n"s from her $aseline 'eight of 121 po#n"s o)er the past < months. She has also note" aching in her knees an" ankles "#ring this inter)al. >n physical e.amination% she is slightly pale an" has t'o oral aphtho#s #lcers on the inner lo'er lip. The a$"omen is soft $#t ten"er in the right lo'er *#a"rant. :o masses are palpa$le% an" there is no hepatosplenomegaly. A rectal e.amination re)eals $ro'n stool% 'hich is g#aiac positi)e. =hich of the follo'ing "iagnostic tests 'o#l" $e most appropriate for this patienta6A$"ominal 2T scan $6Bari#m enema c6Sigmoi"oscopy "6A$"ominal sonogram scan e6Upper gastrointestinal an" small $o'el $ari#m .!ray films :ormal La$s 22. A 4!year!ol" man is $ro#ght to his ne' primary care physician $y his family% 'ho report that he has "e)elope" personality changes% impaire" memory% an" "iffic#lty 'ith speech. /is me"ical history is nota$le for mental retar"ation. /is me"ical chart in"icates that% at $irth% he 'as "iagnose" 'ith trisomy 21 $y genetic karyotyping. Physical e.amination re)eals epicanthal fol"s% a trans)erse palmar crease% an" Br#shfiel" spots on the iris. /e is slo' to respon" to *#estions an" can recall only one o#t of three o$9ects after min#tes. =hich of the follo'ing is the most likely ca#se of these ne' ne#rologic symptomsa6AlDheimer "ementia $6 /y"rocephal#s c6/ypothyroi"ism "6 M#ltiple strokes e6Prion infection :ormal La$s

2&. A 4!year!ol" man is a"mitte" secon"ary to respiratory fail#re an" tachycar"ia. /is temperat#re is &1. 8 2 3142 56% $loo" press#re is 1147<4 mm /g% an" respirations are &47min. /is E2C sho's P 'a)es prece"ing the NAS comple.. :o t'o P 'a)es ha)e the same morphology. =hich of the follo'ing is the most appropriate ne.t step in managementa6A"ministration of "igitalis $6A"ministration of 'arfarin c6Electrical car"io)ersion "6Mechanical )entilation e6 Placement of a "efi$rillator :ormal La$s 24.A <&!year!ol" man is a"mitte" to the hospital for fe)er an" a pro"#cti)e co#gh. The patient reports that% o)er the past fe' "ays% he has ha" a 'orsening co#gh that has $ecome pro"#cti)e of greenish! crimson sp#t#m. The patient reports temperat#res to &8. 2 314& 56 o)er the past 24 ho#rs. The patient has ha" nothing to eat or "rink for the past &< ho#rs. >n f#rther *#estioning% the man "escri$es a pro"romal perio" ( "ays prior to the onset of the co#gh that 'as remarka$le for rhinorrhea an" general malaise. >n physical e.amination% the patient appears ac#tely ill. /is $loo" press#re is 1&4714 mm /g% an" his p#lse is 1147min an" reg#lar. E.amination is remarka$le for "iminishe" $reath so#n"s on the right l#ng!$ase 'ith 0a to e0 egophony an" 'hispere" pectorilo*#y. =hich of the follo'ing is re*#ire" for the "iagnosis of pne#moniaa6/ypo.emia on p#lse o.imetry $6+nfiltrates present on chest ra"iograph c6Sp#t#m Cram,s stain sho'ing gram!positi)e "iplococci "6Sp#t#m Cram,s stain sho'ing ne#trophils e6Temperat#re to &1.< 3141.4 56 :ormal La$s 2 .A 4(!year!ol" homeless 'oman presents to the emergency "epartment complaining of mi"epigastric pain o)er the past 24 ho#rs. The pain $egan gra"#ally an" has $een $#il"ing in intensity. +t no' ra"iates to her $ack an" is associate" 'ith )omiting of $ilio#s material. She reports that she has ha" three prior )isits to the emergency "epartment o)er the past year for similar symptoms an" generally gets $etter after 0they p#t a t#$e in my nose an" "on,t let me eat for a fe'

"ays.0 >n physical e.amination% her temperat#re is &1.& 2 3144.8 56% $loo" press#re is 1447<2 mm /g% an" p#lse is 1427min. >n a$"ominal e.amination% there is mo"erate ten"erness to mil" palpation in the mi"! epigastri#m. There is no costo)erte$ral angle ten"erness an" no shifting "#llness. A rectal e.amination re)eals $ro'n% g#aiac!negati)e stool. La$oratory st#"ies re)eal a =B2 of 12%&447mm&% an amylase of 412 U7L 3normal Q1246% an" a lipase of < m+U7mL 3normal Q146. =hich of the follo'ing is the most likely "iagnosisa6Ac#te $acterial cholangitis $6Ac#te cholecystitis c6Ac#te pancreatitis "6/epatitis A e6/epatitis B :ormal La$s 2<. An 11!year!ol" 'oman is in the hospital for terminal $reast cancer. She 'as a"mitte" to the ser)ice & "ays ago after $eing $ro#ght to the hospital $y her family for 'orsening mental stat#s. The patient 'as "iagnose" 'ith $reast cancer 2 years ago% an" the "isease is no' 'i"ely metastatic to her spine an" $rain. /er family is concerne" that she is not $eing gi)en a"e*#ate se"ation an" analgesia. She has long!stan"ing $one pain an" ha" $een #sing a fentanyl patch an" hy"romorphone orally at home. She is c#rrently on a morphine "rip. The family 'o#l" like the rate of the "rip increase". E)al#ation of the patient re)eals a cachectic 'oman lying comforta$ly in $e". /er $loo" press#re is 1447<4 mm /g% p#lse is (<7min% an" respirations are 147min. She appears to $e sleeping. The family "eman"s that she $e 0ma"e to stop s#ffering0 an" asks that her *#antity of morphine $e increase". =hich of the follo'ing is the most appropriate co#rse of actiona6 @ecrease the morphine "rip rate as the patient appears se"ate" $6+gnore the family,s 'ishes as the patient appears comforta$le c6+ncrease the morphine "rip rate "6+nform the family that the patient appears comforta$le an" to not make re*#ests of the me"ical team e6 +nform the family that a physician cannot participate in killing a patient :ormal La$s

2(.A &2!year!ol" man has a 1 !year history of celiac spr#e. /e a"mits to $eing noncompliant 'ith the "iet prescri$e" o)er the past < months an" he has lost 1 po#n"s "#ring that time in association 'ith fre*#ent "iarrhea. =hich of the follo'ing 'ill $e most likely to occ#r in this patienta6+ron "eficiency 'ill pro"#ce a microcytic anemia $6?itamin A "eficiency 'ill pro"#ce nystagm#s c6?itamin B12 "eficiency 'ill pro"#ce a megalo$lastic anemia "6 ?itamin @ "eficiency 'ill pro"#ce hypercalcemia e6?itamin M "eficiency 'ill pro"#ce hypokalemia :ormal La$s 21. A (&!year!ol" o$ese 'oman comes to the emergency "epartment $eca#se of a painf#l right eye. The pain came on s#""enly as she 'as preparing "inner% an" 'as accompanie" $y se)erely $l#rre" )ision% na#sea% an" one episo"e of )omiting. E.amination sho's an e.tremely ten"er eye 'ith a haDy cornea an" a partially "ilate" an" fi.e" p#pil. The left eye is normal. =hich of the follo'ing is the most likely "iagnosisa6 Ac#te con9#ncti)itis $6Ac#te gla#coma c6>ptic ne#ritis "6Pse#"ot#mor cere$ri e6 Aetinal "etachment :ormal La$s 28.A <1 year!ol" Latin female is a"mitte" to the me"ical ser)ice for e)al#ation of her "iffic#lt!to!control hypertension. She has $een treate" for hypertension for three years $y her primary care physician. She has a "oc#mente" intolerance to A2E inhi$itors manifeste" $y a rapi" "ecline in her renal f#nction. She also has ha" t'o episo"es of ac#te p#lmonary e"ema in the past. She has cease" smoking% has altere" her "iet an" claims she is compliant 'ith her me"ications. At her last office )isit% a mil"ly ele)ate" creatinine of 1.2 mg7"L 'as note"% as 'as microscopic hemat#ria. Physical e.amination is remarka$le for a $loo" press#re of 1147144 mm /g% a prominent apical imp#lse% an a$"ominal $r#it to the right of mi"line an" 2E "orsalis pe"is an" ra"ial p#lses. La$oratory st#"ies are as follo's;

/ematocrit .....................&1B Le#kocyte co#nt.............. 1447mm& So"i#m............................144 mE*7L Potassi#m........................&.< mE*7L Bicar$onate.....................21 mE*7L Urea nitrogen 3BU:6.......22 mg7"L 2reatinine .......................1.& mg7"L =hich of the follo'ing is the most likely ca#se of this patient,s hypertensiona6Al"osterone secreting t#mor $62oarctation of the aorta c6Essential hypertension "6Pheochromocytoma e6Aenal artery stenosis :ormal La$s &4.A 1!year!ol" man presents complaining of shortness of $reath. The patient "escri$es a slo'ly progressi)e ina$ility to perform physical acti)ities o)er the past 2 years. /e reports $eing a$le to start an acti)ity% s#ch as golf or tennis% $#t 'ithin a fe' min#tes he feels like 0he has lost his $reath.0 There is no associate" chest pain% press#re% or "iscomfort. More recently% he has ha" significant shortness of $reath e)en at rest. The clinical pict#re% 'ith the a""ition of a restricti)e pattern on p#lmonary f#nction tests an" interstitial infiltrates on the chest .!ray film% s#ggests the "iagnosis of interstitial l#ng "isease. =hich of the follo'ing is the most appropriate ne.t step in managementa65ollo'!#p in the clinic in 2 months% as most interstitial l#ng "iseases are #ntreata$le $6Aeferral to a p#lmonologist for f#rther e)al#ation an" possi$le trans$ronchial $iopsy c6An empiric co#rse of pre"nisone for < 'eeks "6+mm#nos#ppressi)e therapy 'ith aDathioprine e6 Aeferral to a thoracic s#rgeon for f#rther e)al#ation an" possi$le open l#ng $iopsy :ormal La$s &1. A &4!year!ol" man is $ro#ght to the emergency "epartment $eca#se of fe)er% hea"ache% an" seiD#res of a$r#pt onset. /is temperat#re is &8.4 2 314& 56. The patient is "isoriente" as to place an" time. Physical

e.amination re)eals mil" n#chal rigi"ity. >n a"mission% la$oratory st#"ies sho' 14%244 ne#trophils7mm& in the peripheral $loo"% 'hile a l#m$ar p#nct#re is significant for mo"erately increase" 2S5 press#re% lymphocytosis 3 447mm&6% an" presence of re" $loo" cells. Electroencephalographic st#"ies "emonstrate $itemporal perio"ic comple.es on a slo' $ackgro#n". :e#roimaging sho's m#ltifocal hemorrhagic lesions in the temporal lo$es. =hich of the follo'ing is the most appropriate ne.t step in managementa6S#pporti)e treatment #ntil 2S5 c#lt#re res#lts are a)aila$le $62ere$ral angiographic st#"ies c6Treatment 'ith acyclo)ir "6Treatment 'ith antimycotic agent e6Treatment 'ith antimicro$ial agents :ormal La$s &2.A &&!year!ol" female 'ith $ipolar "isor"er an" history of alcohol an" "r#g a$#se presents to the emergency "epartment after $eing fo#n" "o'n at home. She 'as fo#n" #nresponsi)e to )oice an" to#ch% 'ith 1< empty packets of me"ication ne.t to her% each containing 4 mg of "iphenhy"ramine. There 'as also an empty $ottle of acetaminophen an" a half!empty 'hiskey $ottle near her $e". A s#ici"e note 'as present on a near$y ta$le. +n the fiel"% her temperat#re 'as &< 2 38<.1 56% $loo" press#re 'as 847 mm /g% her p#lse 'as 144 an" reg#lar% an" her respirations 'ere 17min. A""itional "ata o$taine" in the fiel" incl#"e" a finger stick gl#cose of 24 mg7"L an" an o.ygen sat#ration of 1(B on room air. >f the "ata a)aila$le in the fiel"% 'hich of the follo'ing fin"ings is most imme"iately threatening to her o#tcomea6Bloo" press#re of 847 $65inger stick gl#cose of 24 mg7"L c6>.ygen sat#ration of 1(B on room air "6P#lse of 144 e6 Aespirations of 1 :ormal La$s &&. A &&!year!ol" female 'ith $ipolar "isor"er an" history of alcohol an" "r#g a$#se presents to the emergency "epartment after $eing fo#n" "o'n at home. She 'as fo#n" #nresponsi)e to )oice an" to#ch% 'ith 1< empty packets of me"ication ne.t to her% each containing 4 mg of "iphenhy"ramine. There 'as also an empty $ottle of acetaminophen

an" a half!empty 'hiskey $ottle near her $e". A s#ici"e note 'as present on a near$y ta$le. +n the fiel"% her temperat#re 'as &< 2 38<.1 56% $loo" press#re 'as 847 mm /g% her p#lse 'as 144 an" reg#lar% an" her respirations 'ere 17min. A""itional "ata o$taine" in the fiel" incl#"e" a finger stick gl#cose of 24 mg7"L an" an o.ygen sat#ration of 1(B on room air. >f the "ata a)aila$le in the fiel"% 'hich of the follo'ing fin"ings is most imme"iately threatening to her o#tcomea6 Bloo" press#re of 847 $65inger stick gl#cose of 24 mg7"L c6>.ygen sat#ration of 1(B on room air "6P#lse of 144 e6Aespirations of 1 :ormal La$s &4.A pre)io#sly healthy% & !year!ol" 'oman comes to me"ical attention $eca#se of an #nstea"y gait. /er temperat#re is &( 2 381.< 56% $loo" press#re is 1&47(4 /g% p#lse is 147min% an" respirations are 247min. :e#rologic e.amination re)eals spasticity an" "ecrease" )i$ratory sensation in her right lo'er e.tremity% an" "ecrease" strength in her left arm. MA+ st#"ies sho' 'ell!"emarcate" peri)entric#lar areas of T2 hyperintensity. A l#m$ar p#nct#re sho's mil"ly increase" protein concentration 'ith oligoclonal +gC $an"s. =hich of the follo'ing is the most likely "iagnosisa62ytomegalo)ir#s encephalitis $6M#ltiple sclerosis c6>ligo"en"roglioma "6Progressi)e m#ltifocal le#koencephalopathy e6Lymphoma :ormal La$s & .A <2!year!ol" man presents 'ith symptoms of 'orsening congesti)e heart fail#re. /e has a history of rhe#matic heart "isease as a chil". >)er the past & years% he has ha" progressi)e symptoms of "yspnea on e.ertion% paro.ysmal noct#rnal "yspnea% an" orthopnea. /e has $een maintaine" on "igo.in% f#rosemi"e% an" enalapril for symptoms of his congesti)e heart fail#re. /is car"iac e.amination re)eals a lo#" $lo'ing "ecrescen"o "iastolic m#rm#r. /e has $o#n"ing peripheral p#lses.

=hich of the follo'ing a""itional fin"ings 'o#l" most likely $e fo#n" on physical e.aminationa6Bra"ycar"ia $6+solate" systolic hypertension c6P#ls#s para"o.#s "6Tachycar"ia e6=i"e p#lse press#re :ormal La$s &<.A (1!year!ol" man 'ith a (4!pack!year smoking history presents after he notes that his right eye has a lagging li". The physician has $een seeing this patient for more than 14 years for management of his symptoms of chronic o$str#cti)e p#lmonary "isease 32>P@6. >n physical e.amination% he has ptosis of the right eye 'ith a constricte" right p#pil. The remain"er of the ophthalmologic e.amination is normal. 2ranial ner)e f#nction is other'ise normal. =hich of the follo'ing 'o#l" most likely $e e.pecte" on a chest .!ray filma6A normal chest .!ray film $6An irreg#larly shape" mass at the ape. of the right l#ng c6A calcifie" gran#loma in the left mi"!l#ng fiel" "6A left si"e" ple#ral eff#sion e6A right #pper lo$e pne#monia :ormal La$s &(.A 4&!year!ol" man reports that he ha" a 8!l$. 'eight loss o)er the past 8 months. The symptoms are accompanie" $y "iffic#lty s'allo'ing $oth soli"s an" li*#i"s "#ring that time. /e has 'oken on se)eral occasions at appro.imately 4 AM an" reg#rgitate" partially "igeste" "inner contents. An #pper gastrointestinal series is performe" an" re)eals a 'i"ely "ilate" esophag#s 'ith a smoothly tapering "istal esophag#s. There appears to $e partially "igeste" foo" present in the esophag#s. =hich of the follo'ing is the most likely ca#se of this patient,s symptomsa6 Achalasia $6@iff#se esophageal spasm c6Esophageal s*#amo#s cancer "6Peptic strict#re

e6 Sclero"erma :ormal La$s &1. A 24 year!ol" $lack man comes to the emergency room complaining of a fe' ho#rs history of $ilateral knee pain an" a painf#l erection. /e ha" a similar episo"e t'o years ago. /e has no na#sea% )omiting% or fe)er% an" his )ital signs are 'ithin normal limits. After recei)ing treatment 'ith o.ygen an" intra)eno#s fl#i"s% his symptoms resol)e an" he is "ischarge" home. =hat is the most likely "iagnosisa6@ehy"ration $6Conococcal infection c6Le#kemia "6Sickle cell "isease e6Tra#matic in9#ry :ormal La$s

&8.A 18!year!ol" college freshman #n"ergoes colonoscopy $eca#se of a family history of m#ltiple polyps in his yo#ng si$lings. /is $rother #n"er'ent total proctocolectomy at age 2&% an" his sister #n"er'ent a total proctocolectomy at age 28% after $oth 'ere fo#n" to ha)e h#n"re"s of colonic a"enomas on colonoscopy. Both si$lings are ali)e an" 'ell years later an" 'itho#t any other fin"ings of neoplasms. The patient #n"ergoes sigmoi"oscopy an" is fo#n" to ha)e se)eral "oDen small colonic polyps 'ithin the rectosigmoi". 5i)e of these are $iopsie" an" are all $enign a"enomas. =hich of the follo'ing is the most appropriate ne.t step in managementa6Sche"#le a repeat sigmoi"oscopy in 1 year $6Sche"#le a colonoscopy in 1 year c6E)al#ate the more pro.imal colon 'ith a $ari#m enema "6Sche"#le a f#ll colonoscopy e6Sche"#le a total proctocolectomy :ormal La$s 44. MA+ st#"ies re)eal the follo'ing congenital malformations in the 2:S of a < month!ol" $a$y presenting 'ith intracta$le )omiting; small posterior fossa% "o'n'ar" "isplacement of the cere$ellar )ermis an"

me"#lla thro#gh the foramen magn#m% syringomyelia% an" myelomeningocele. =hich of the follo'ing is the most likely "iagnosisa6Anencephaly $6Arnol"!2hiari type 1 malformation c6Arnol"!2hiari type 2 malformation "6@an"y!=alker malformation e6/oloprosencephaly :ormal La$s 41. A 4&!year!ol" 'oman comes to the office for e)al#ation of high $loo" press#re. She 'as informe" that she ha" high $loo" press#re 1 'eek ago "#ring a ro#tine screening at a health fair in a local shopping mall. She has $een pre)io#sly healthy an" is on no me"ications. >n e.amination to"ay her $loo" press#re is 14 78 mm /g. =hich of the follo'ing is the most appropriate step in managementa6A")ise her to monitor her $loo" press#re t'ice a "ay an" ret#rn in < months $6Ask her to ret#rn for ree.amination after her ne.t menstr#al cycle c6Ask her to ret#rn for ree.amination in 2 'eeks "6Ask her to ret#rn for ree.amination in 4 months e6Meas#re her $loo" press#re after she e.ercises for min#tes :ormal La$s 42.A 4!year!ol" hea)y smoker comes to the emergency "epartment $eca#se of a mil" co#gh% chest pain% "iarrhea% fatig#e% hea"ache% an" fe)er for & "ays. /e has a scant amo#nt of nonp#r#lent sp#t#m. Se)eral of his co'orkers are e.periencing similar symptoms. /is temperat#re is &8.1 2 314&.< 56% $loo" press#re is 124714 mm /g% an" p#lse is 47min. Aales are hear" on a#sc#ltation. @iff#se a$"ominal ten"erness is present. A chest .!ray film re)eals $ilateral infiltrates. A Cram,s stain of his sp#t#m sho's n#mero#s ne#trophils% $#t no organisms. A sp#t#m c#lt#re on $#ffere" charcoal yeast e.tract 3B2UE6 agar gro's gram! negati)e $acilli. =hich of the follo'ing is the most likely pathogena62hlamy"ia trachomatis $6Legionella pne#mophila c6Mora.ella catarrhalis "6Mycoplasma pne#moniae e6Pne#mocystis carinii

:ormal La$s 4&. A <4!year!ol" man smokes one or t'o cigarettes a "ay spora"ically on 'eeken"s an" he has $een "iagnose" 'ith se)ere emphysema. /is p#lmonologist% on e.amining his ro#tine $loo" 'ork% fin"s ele)ate" ser#m transaminases. /epatitis serologies re)eal no e)i"ence of )iral hepatitis A% B% or 2. A yo#nger $rother "ie" of emphysema at age 4 an" ha" no smoking history. =hich of the follo'ing "iseases sho#l" most likely $e consi"ere" to e.plain this patient,s li)er a$normalities A:@ his l#ng "iseasea6Alpha!1!antitrypsin "eficiency $6Primary hemochromatosis c6 Primary sclerosing cholangitis "6Secon"ary hemochromatosis e6 =ilson "isease :ormal La$s 44. A & year ol"!man is $ro#ght to the emergency "epartment $y a frien". The frien" reports that the patient has 0li)er "isease0 an" has $een "rinking hea)ily lately an" has not taken his me"ications. /e has gotten progressi)ely more conf#se" o)er the past fe' "ays. >n e.amination% the man is afe$rile% his $loo" press#re is 1247(4 mm /g an" his heart rate is 1447min. /e has no o$)io#s signs of tra#ma $#t has some ol"% 'ell!heale" lacerations on his forehea". /e has "eep scleral icter#s an" his skin is 9a#n"ice". /is l#ngs are clear. an" car"iac e.amination is normal% $#t he has a "isten"e" a$"omen 'ith shifting "#llness. /e is alert to person only an" his ne#rological e.amination is remarka$le for the ina$ility to perform finger!to!nose to#ching an" heel!to!shin mane#)ers. Asteri.is is present. La$oratory st#"ies sho'; So"i#m.....................................12 mE*7L Potassi#m.................................&.1 mE*7L Bicar$onate..............................11 mE*7L Urea nitrogen............................2 mg7"L 2reatinine.................................1.2 mg7"L Aspartate aminotransferase.......2&4 U7L Alanine aminotransferase..........&14 U7L Prothrom$in time......................14.1 secon"s Alkaline phosphatase................( U7mL

Le#kocyte co#nt.......................<%4447mm& /ematocrit...............................& B Bloo" alcohol le)el....................2144 mgB =hich of the follo'ing is the most likely "iagnosisa6 Ac#te hyponatremia $6Ascen"ing cholangitis c6/epatic encephalopathy "6Meta$olic aci"osis e6 S#$"#ral hematoma :ormal La$s 4 .A 41!year!ol" 'oman 'ith a long history of hepatitis 2 is a"mitte" for hematemesis 3)omiting $loo"6. She is sta$iliDe" in the emergency "epartment% an" an #pper en"oscopy re)eals $lee"ing esophageal )arices. >n e.amination% she is also note" to ha)e mo"erate amo#nt of ascites. =hich of the follo'ing is the most appropriate ne.t step in the management of her $lee"inga6Alpha interferon $6?asopressin c6En"oscopic )ariceal $an"ing "6T+PS 3trans9#g#lar intrahepatic portosystemic sh#nt6 e6Li)er transplant :ormal La$s 4<. A 2 !year!ol" 'oman is referre" for ne#rologic e)al#ation of slo'ly progressi)e ata.ia. :e#roimaging st#"ies re)eal a 'ell!circ#mscri$e" cystic mass 'ith a m#ral no"#le in the right cere$ellar hemisphere. Significant hematologic )al#es incl#"e; /ematocrit............ B Ae" $loo" cells......< L 14<7mm& The patient #n"ergoes s#rgical e.cision of the cere$ellar t#mor. The histologic "iagnosis is 0hemangio$lastoma.0 5#rther clinical in)estigations lea" to "isco)ery of a )asc#lar t#mor in the right retina. =hich of the follo'ing syn"romes is most likely responsi$le for this clinical presentationa6Li!5ra#meni syn"rome $6:e#rofi$romatosis type 1 c6:e#rofi$romatosis type 2

"6T#$ero#s sclerosis e6)on /ippel!Lin"a# syn"rome :ormal La$s 4(.A <&!year!ol" 'oman presents to the emergency "epartment complaining of se)ere mi"a$"ominal pain. The patient reports that the pain has increase" in intensity o)er the past fe' "ays. There has $een no associate" na#sea or )omiting% no change in $o'el ha$its an" no relief affor"e" $y position changes. The 'oman is postmenopa#sal an" "oes not take hormone replacement therapy. She has a &4!year history of hypertension% an" has $een noncompliant 'ith her calci#m channel $locker an" thiaDi"e "i#retic therapy. >n e.amination% her a$"omen is o$ese% $#t there is a s#ggestion of a non!ten"er% p#lsatile mass in the epigastric region. The remain"er of the physical e.amination is normal. +maging st#"ies are "one% an" imme"iate s#rgical e)al#ation is in"icate". =hich of the follo'ing res#lts 'ere most likely o$taine" from the imaging st#"iesa6An a$"ominal aortic segment spanning ( cm in "iameter $6A li)er span of more than 8 cm c6M#ltiple cysts 'ithin the li)er "6Posterior herniation of a l#m$ar inter)erte$ral "isk e6Stones in the renal pel)is :ormal La$s 41. A &(!year!ol" 'oman 'ith /+?% last 2@4 co#nt &847mm& an" last )iral A:A copy n#m$er 124%444 copies7mL% presents to the me"ical 'alk!in clinic 'ith co#gh% fe)er an" shortness of $reath. The patient has $een 'ell an" has $een compliant 'ith her triple "r#g therapy. She reports that & 'eeks ago% she $egan to e.perience increasing shortness of $reath an" "e)elope" a non!pro"#cti)e co#gh. >)er the past fe' 'eeks% $oth of these complaints ha)e gra"#ally 'orsene". She also reports fe)ers to 14& "egrees 5 o)er the past fe' "ays. >n e.amination% her $loo" press#re is 1&47(4 mm /g% p#lse is 847min% an" o.ygen sat#ration on room air is (1B. /er l#ngs ha)e "iff#se crackles% no egophony an" no "#llness to perc#ssion. The rest of her e.amination is #nremarka$le. A chest ra"iograph sho's "iff#se interstitial an" al)eolar infiltrates 'ith hilar pre"ominance $ilaterally. =hich of the follo'ing is the most appropriate co#rse of therapy for this patient-

a6 oral isoniaDi"% rifampin% pyraDinami"e an" etham$#tol $6intra)eno#s trimethoprim!s#lfametho.aDole c6intra)eno#s aDithromycin "6intra)eno#s trimethoprim!s#lfametho.aDole an" pre"nisone e6intra)eno#s amphotericin B :ormal La$s 48.A <<!year!ol" 'oman complains of non$loo"y% nonm#co#s 'atery stools an" "iff#se crampy a$"ominal pain for & "ays. She recently complete" a 14!"ay co#rse of cef#ro.ime for a comm#nity!ac*#ire" pne#monia. There is no history of tra)el. >n e.amination% her temperat#re is 144 5% p#lse is 847min% $loo" press#re is 1&4714 mm /g% an" respiratory rate is 187min. /er l#ngs are clear% an" her a$"omen is mil"ly "isten"e" $#t nonten"er. Acti)e $o'el so#n"s are present% an" rectal e.amination is normal. =hich of the follo'ing is the most appropriate "iagnostic testa62lostri"i#m "ifficile to.in assay $6 Stool collection for fecal fat *#antification c6Stool c#lt#re "62olonoscopy :ormal La$s 4. A &1!year!ol" man "escri$es a $#rning sensation in the s#$sternal area after eating chocolates% caffeine% or alcohol. The symptoms are e.acer$ate" at night% an" he has 'oken on se)eral occasions from sleep $eca#se of co#ghing. /e has trie" o)er!the!co#nter antaci"s an" /2 receptor antagonists 'ith little relief. /e often takes antaci"s $efore "inner. =hich of the follo'ing is the most likely e.planation for his symptoms of co#ghinga6Aci"!in"#ce" $ronchoconstriction $6Aspiration of antaci"s taken $efore $e"time c6Aspiration of soli" foo" ingeste" 'ith "inner "6 /2 receptor antagonist!in"#ce" $ronchoconstriction e6Tracheoesophageal fist#la :ormal La$s

Note: Check your own answers before hittin$ the Check button below. When you click the Check button, a browser win"ow will a!!ear that contains a summary of your results. (*!lanations lock D (*!lanations

:) (*!lanation: 7he correct answer is %. 7his !atient is an asym!tomatic C?4year4ol" smoker. 7he only screenin$ test liste" that is recommen"e" by the F.). &reventive )ervices 7ask -orce is the &a! smear. 6e$ular 'every :40 years) &a! smears shoul" be $iven to all women who are or who have been se*ually active. 7hey may be "iscontinue" at a$e C/ if consistently normal. Other screenin$ tests recommen"e" for this woman woul" be a mammo$ram, clinical breast e*amination, a si$moi"osco!y 'every ;40 years) an"B or an annual fecal occult bloo" test. Counselin$ to en$a$e in !hysical activity, !romote a healthy "iet an" to wear seatbelts is recommen"e". 7obacco cessation counselin$ an" !ostmeno!ausal estro$en re!lacement counselin$ is recommen"e". 7" to*oi" boosters shoul" be $iven every :> years. &neumococcal vaccine shoul" be a"ministere" at least once an" an influen#a vaccination a"ministere" yearly after C/. +easurement of total serum cholesterol may be !ru"ent. &erio"ic measurement of bloo" !ressure, hei$ht, an" wei$ht is recommen"e". Chest *4ray films 'choice A) an" s!utum cytolo$y 'choice () are not recommen"e" to screen asym!tomatic !ersons for lun$ cancer. 6outine screenin$ of asym!tomatic smokers is not recommen"e". An (C. 'choice ) is not recommen"e" to screen for asym!tomatic CA %. It may be !ru"ent in certain hi$h4risk $rou!s, but there is insufficient evi"ence to recommen" for or a$ainst its routine use. (*amination of the uterine a"ne*a 'choice C) to screen for ovarian cancer is not recommen"e". 6outine screenin$ for ovarian cancer by any metho" is not recommen"e". ;) (*!lanation: 7he correct answer is

%. 7he a!!ro!riate anti"ote for acetamino!hen to*icity is N4acetylcysteine, a com!oun" that will increase he!atic $lutathione stores. 7he re!letion of he!atic $lutathione is crucial in this instance, since acetamino!hen 'when in$este" in lar$e amounts) becomes metaboli#e" by he!atic microsomal en#ymes releasin$ to*ic metabolites that will be re"uce" an" inactivate" by he!atic $lutathione 'unless when "e!lete"). Amyl nitrite 'choice A) is use" in the treatment of cyani"e to*icity. +ethylene blue 'choice ) is use" as an anti"ote for the treatment of methemo$lobinemia. Nalo*one 'choice C) is use" as a treatment for o!ioi"Bo!iate over"ose. &enicillamine 'choice () is a co!!er chelator use" in con"itions with elevate" serum co!!er, such as Wilson "isease. 0) (*!lanation: 7he correct answer is . 7his question illustrates the classic !resentation of acute "iverticulitis. It is im!ortant to "istin$uish "iverticulosis, the !resence of "iverticula in the colon, an" acute "iverticulitis, which im!lies acute inflammation of a "iverticulum. %iverticulosis is !rimarily a "isease of the el"erly, who may "evelo! hun"re"s of "iverticula. +ost of the "iverticula ten" to "evelo! in the very "istal colon 'where the stool ten"s to be the most "ehy"rate" an" har"est to move by !eristalsis), althou$h in severe cases even the ascen"in$ colon may be involve". 7his !re"ilection for "istal colonic involvement ten"s to clinically !ro"uce chronic consti!ation an" left lower cram!in$ after meals, as seen in this !atient. Acute "iverticulitis shoul" be sus!ecte" in cases like this one. +ost cases of acute "iverticulitis occur as the result of a micro4 or macro!erforation of a "iverticulum, which allows $ut bacteria to esca!e from the bowel lumen. In this case, a transmural !erforation has create" an abscess in the re$ion of the si$moi" colon. &atients with acute "iverticulitis "o not "evelo! blee"in$ from the inflame" area of bowel 'choice A). )uch blee"in$ can be seen in ulcerative colitis an" other severe mucosal con"itions. %iverticulitis is not a consequence of mucosal inflammation 'choice C). Althou$h mucosal ischemia 'choice %) can affect this re$ion of the colon, it ty!ically !resents with "iarrheal blee"in$ an" "oes not cause an abscess. 7here is no association between !ancreatitis 'choice () an" the sym!toms "escribe" in this !atient. &ancreatitis will usually !resent with u!!er ab"ominal !ain that often ra"iates to the back.

?) (*!lanation: 7he correct answer is C. One of the most serious com!lications of sinusitis an" otitis is the "evelo!ment of cerebral abscesses. 7his often manifests with !ersistent fever, variable neurolo$ic "eficits, hea"ache, an" sei#ures. ecause of the accom!anyin$ e"ema aroun" the abscess, intracranial !ressure may "evelo!, which e*!lains !a!ille"ema in this case. Cerebral abscesses are life4threatenin$ con"itions that often require sur$ical evacuation. loo" cultures 'choice A) are often ne$ative in the case of an isolate" cerebral abscess. On the other han", the sur$eon shoul" submit sam!les for culture in case of sur$ical intervention. ((. stu"ies 'choice ) may, at best, show non4s!ecific 'an" thus non4"ia$nostic) focal chan$es in the tem!oral re$ion. ,umbar !uncture 'choice %), as in all situations in which si$ns of increase" intracranial !ressure are "etecte", shoul" be avoi"e". A lumbar ta! may !reci!itate fatal cerebellar tonsillar herniation. /) (*!lanation: 7he correct answer is C. In constrictive !ericar"itis, the lateral view of the chest may "emonstrate calcification of the anterior !ericar"ium. 7his may be seen in />= of !atients with lon$4stan"in$ constriction. 7he !ericar"ial thickenin$ may be seen on an echocar"io$ram. A !ericar"ial knock is hear" >. >C4>. :; secon"s after the aortic valve closes. 7his corres!on"s to the su""en cessation of ventricular fillin$. Central cyanosis 'choice A) is cause" by "ecrease" arterial o*y$en saturation in the arterial bloo". 7his may result from a "ecrease" atmos!heric !ressure, im!aire" !ulmonary function, anatomic shunts, or hemo$lobin abnormalities. 7he o!enin$ sna! 'choice ) is hear" in mitral stenosis. 7his is most rea"ily hear" in e*!iration an" follows the soun" of the aortic valve closure. It follows &;. 7he time interval between A; closure an" the o!enin$ sna! varies inversely with the severity of mitral stenosis. 7he o!enin$ sna! is followe" by a low4!itche", rumblin$, "iastolic murmur, an" its "uration correlates with severity of stenosis. A systolic e<ection murmur 'choice %) is associate" with mitral re$ur$itation an" may be accentuate" by isometric e*ercises. Constrictive !ericar"itis is not associate" with any murmurs.

A wi"ely s!lit ); 'choice () is hear" in con"itions associate" with ri$ht ventricular volume overloa" an" a "istensible !ulmonary vascular be". It may also be hear" in con"itions such as !ulmonary hy!ertension. C) (*!lanation: 7he correct answer is . Chronic obstructive !ulmonary "isease 'CO&%) is the !rototy!ical obstructive lun$ "isease. 7y!ical intrathoracic obstruction causes a "ecrease" amount of flow on e*halation Wforce" e*!iration in : secon" '-(1:)X, an increase" amount of air left in the lun$s at the en" of a ma*imal e*!iration 'resi"ual volume), an" an increase" total lun$ ca!acity '7,C). 7he ratio of -(1: to force" vital ca!acity '-1C) stays relatively unchan$e" until late in the CO&% course, when -1C "ro!s !reci!itously. -(1: ;.: ,, -(1:B-1C >.I, resi"ual volume ;.> ,, total lun$ ca!acity D , 'choice A) is re!resentative of normal &-7s. -(1: ;.C ,, -(1:B-1C >.@, resi"ual volume :.0 ,, total lun$ ca!acity / , 'choice C) is re!resentative of restrictive lun$ "isease. -(1: :.0 ,, -(1:B-1C >.I, resi"ual volume >.I ,, total lun$ ca!acity I , 'choice %) is consistent with restrictive lun$ "isease, e*ce!t for the increase" 7, C. -(1: >.I ,, -(1:B-1C >.I, resi"ual volume :.> ,, total lun$ ca!acity :; , 'choice () is consistent with obstructive lun$ "isease, e*ce!t for the low resi"ual volume, which is more reflective of severe restrictive !hysiolo$y. D) (*!lanation: 7he correct answer is (. 7his !atient has hy!ercalcemia secon"ary to his multi!le myeloma. 7he issue here is the acute mana$ement of sym!tomatic hy!ercalcemia. 7hese !atients are very "ehy"rate" from the solute4in"uce" "iuresis that also elevates the serum concentrations secon"ary to contraction of the !lasma volume. 7he only way to effectively eliminate calcium from the bo"y quickly is in the urine. 7herefore, "iuresis with a$$ressive intravenous flui" su!!ort to avoi" further "ehy"ration is the fun"amental thera!y. I1 salmon calcitonin 'choice A) is not in"icate" emer$ently, as its efficacy in lowerin$ calcium is !oor an" its kinetics are slow. 7his a$ent can be use" alon$ with flui"s, "iuresis, an" bis!hos!honates in severe cases.

I1 !ami"ronate 'choice ) is the treatment of choice once the initial crisis !hase has been resolve". All !atients with cancerous hy!ercalcemia will be treate" with a bis!hos!honate of some sort to avoi" this !resentation. I1 bicarbonate 'choice C) woul" be $iven in an attem!t to alkalini#e the serum an" shift calcium to bone in e*chan$e for !rotons. 7his thera!y has not been shown to be efficacious, however. A$$ressive "iuresis with furosemi"e 'choice %) without concomitant rehy"ration woul" likely result in the !atientAs "eath, as the state of "ehy"ration woul" only be e*acerbate". @) (*!lanation: 7he correct answer is . Increase" resistance to fillin$ of one or more car"iac ventricles has been terme" "iastolic heart failure an" can !ro"uce increase" !ulmonary ca!illary we"$e !ressures an" res!iratory com!laints. In myocar"ial hy!ertro!hy, im!aire" "iastolic rela*ation occurs. 2y!ertro!hic heart "isease is the best reco$ni#e" cause of "iastolic "ysfunction. Chronic obstructive !ulmonary "isease 'CO&%) 'choice A) may lea" to !ulmonary hy!ertension an" cause elevate" ri$ht4si"e" !ressures, which may cause shortness of breath. 7he we"$e !ressure will be "ecrease", an" the ri$ht ventricle will be "ilate". 7he left ventricle woul" not be affecte". In the settin$ of a normal stress test, myocar"ial ischemia 'choice C) is an unlikely source of this !atientAs sym!toms. 7y!ically, she may have chest !ain an" some ventricular "yskinesis on echocar"io$ram, alon$ with the "ys!nea. 6eactive airways "isease 'choice %), in a manner similar to CO&%, may lea" to !ulmonary hy!ertension. Acutely, bronchoconstriction may lea" to hy!o*ia an" sym!toms of "ys!nea, chest !ain, an" whee#in$. 2owever, the left ventricle shoul" be s!are" if there is no hy!ertension. )ystolic failure 'choice () is unlikely in the settin$ of no !ulmonary e"ema, normal neck veins, lack of e"ema in e*tremities, or a he!ato<u$ular refle*. 7he ventricular function woul" be im!aire" in this case. I"io!athic "ilate" car"iomyo!athy an" ischemic car"iomyo!athy may cause this con"ition. I) (*!lanation: 7he correct answer is . 7his scenario is the classic situation for the "evelo!ment of "iffuse stress $astritis, which has resulte" in a lar$e volume, black tarry stool. -urthermore, the !atient is on as!irin an" he!arin, which !re"is!ose to blee"in$ from stress $astritis.

Cecal arterial venous malformations 'choice A) are a common cause of blee"in$ in the el"erly, but $enerally !resent with lower $astrointestinal blee"in$ with either hematoche#ia or maroon stool. Althou$h erosive eso!ha$itis 'choice C) may also occur in the critically ill !atient, it is not as common as "iffuse stress $astritis. 7here is no evi"ence that the !atient has eso!ha$eal varices 'choice %), an" !atients in this settin$ are not at !articular risk for a sin$le $iant $astric ulcer 'choice (). :>) (*!lanation: 7he correct answer is C. Cerebral amyloi" an$io!athy 'CAA) is "ue to "e!osition of AS amyloi" 'the same form as that associate" with Al#heimer "isease) in the small arteries an" arterioles of the le!tomenin$es an" brain. Intracerebral blee"in$ relate" to CAA occurs in the neocorte* in a HlobarH "istribution 'i.e. frontal, !arietal, tem!oral, or occi!ital lobe). 7he hematoma is rea"ily i"entifiable with C7 scans an" is centere" in the cerebral corte*. +ost !atients are over C>, an" bloo" is usually absent from the C) -. &atients sufferin$ from CAA4relate" blee"in$ may not have Al#heimer "isease. erry aneurysm ru!ture 'choice A) is the most common cause of subarachnoi" blee"in$. erry aneurysms "evelo! in the circle of Willis, but most cases re!resent inci"ental auto!sy fin"in$s. Increase" risk of ru!ture is "irectly relate" to their si#e. 2y!ertension is the most im!ortant risk factor for ru!ture. ri"$in$ vein tears 'choice ) cause sub"ural hemorrha$es, usually resultin$ from hea" traumas in !atients with cerebral atro!hy. Cerebral atro!hy lea"s to stretchin$ of the bri"$in$ veins, which are more likely to ru!ture even after sli$ht traumatic events. +i""le menin$eal artery laceration 'choice %) is the most common cause of e!i"ural hemorrha$e, which is virtually always traumatic in ori$in. C)- is free of bloo". In fact, a !atient with sus!icion of e!i"ural hematoma shoul" not un"er$o s!inal ta! because of increase" risk of fatal cerebral herniation. 2y!ertension 'choice () !rimarily affects intra!arenchymal arterioles an" small arteries. 2y!ertensive blee"in$, therefore, is intra!arenchymal. asal $an$lia, cerebellum, an" !ons are the most commonly affecte" sites. Cortical blee"in$ is usually not relate" to hy!ertension. ::) (*!lanation: 7he correct answer is

A. 7he thickene" ventricular se!tum an" the systolic anterior motion of the mitral valve su$$est i"io!athic hy!ertro!hic subaortic stenosis 'I2))). 7he murmur is harsh an" systolic an" "ecreases when ventricular volume increases as a result of isometric e*ercise, as seen with han" $ri! e*ercise. A "ecrease in len$th an" intensity is seen with 1alsalva 'choice ) with most systolic murmurs 'but not the murmur of mitral valve !rola!se). &rolon$e" e*!iratory !ressure a$ainst a close" $lottis re"uces the intensity of most murmurs by "ecreasin$ both ri$ht an" left ventricular fillin$. With the re"uce" chamber si#e, however, the I2)) murmur increases. A "elaye" caroti" u!stroke is seen in aortic stenosis 'choice C). In I2)), the caroti" u!stroke is brisk an" often e*hibits the bisferiens !ulse, in which two fairly ra!i" caroti" !eaks are felt as a result of a brief "ecline in !ressure followin$ the su""en "ecrease in the rate of left ventricular e<ection "urin$ mi"4systole from the "evelo!ment of obstruction. )quattin$ increases both venous return an" chamber si#e. It also increases systemic arterial resistance an" increases most murmurs, e*ce!t those cause" by I2)) 'choice %). 7he increase in the chamber si#e causes a re"uction in the murmur. Aortic stenosis lea"s to a systolic murmur that may ra"iate to the neck 'choice (). 7he ty!ical harsh systolic murmur of I2)) "oes not usually ra"iate to the caroti" arteries. :;) (*!lanation: 7he correct answer is %. 7his question is "escribin$ someone who "evelo!e" a tension !neumothora* while on !ositive4en" e*!iratory !ressure '&((&), as "emonstrate" by <u$ular venous "istention, tracheal "eviation to the left, an" the absence of breath soun"s in the ri$ht hemithora*. .iven these classic !hysical fin"in$s an" the !atientAs acute car"iores!iratory colla!se, a nee"le thoracostomy to allow ra!i" ri$ht lun$ e*!ansion is essential, an", in a situation such as this, shoul" be "one even before a chest *4ray film is obtaine". 6e!eate" ventilation4!erfusion scan 'choice A), !ulmonary an$io$ra!hy 'choice ), an" thera!y with I1 stre!tokinase 'choice C) woul" all be a!!ro!riate if the !atientAs !roblem was a secon" !ulmonary embolus. 2owever, a secon"ary !ulmonary embolus woul" not cause absences of breath soun"s in the ri$ht hemithora*, "isten"e" neck veins, or tracheal "eviation. roa"4s!ectrum antibiotics 'choice () woul" be a!!ro!riate if the !atient ha" "evelo!e" !neumonia, but this woul" be unlikely to cause the very su""en colla!se in her status "escribe" in the question stem.

:0) (*!lanation: 7he correct answer is %..iar"ia lamblia is often acquire" "urin$ hikin$ if fresh stream water is in$este" before it is !urifie". .iar"ia will !rimarily infect the !rimary small bowel, i.e., the "uo"enum an" !ro*imal <e<unum, !ro"ucin$ u!!er $astrointestinal sym!toms with frequent eructation, bloatin$, "istention, flatus, an" loose stools. (ntamoeba histolytica'choice A) is often acquire" "urin$ travel an" can !ro"uce an ileocolitis an"Bor a he!atic abscess. 7hese !atients ten" to a!!ear systemically ill. (nteroinvasive (scherichia coli'choice ) can be acquire" "urin$ travel or as a foo" !oisonin$ an" will !ro"uce an invasive4ty!e "iarrhea, i.e., bloo"y stools with muco!us, fevers, an" leukocytosis. (nteroto*i$enic (. coli'choice C) is most frequently acquire" as a travelerAs "iarrhea an" !ro"uces a watery "iarrheal syn"rome, which is usually self4limitin$. )hi$ella "ysenteriae'choice () !ro"uces the ty!ical "ysenteric !icture with a to*ic a!!earin$ !atient with fever, leukocytosis, ab"ominal ten"erness, an" bloo"y stools with muco!urulence. :?) (*!lanation: 7he correct answer is C. Crush in<uries, such as the one sustaine" by this !atient, often result in a massive release of muscle contents. Creatine kinase 'C5) is an en#yme release" by "ea" or "ama$e" muscle into the bloo". y itself it is harmless, but it is a marker for myo$lobin, which is "irectly ne!hroto*ic. +yo$lobin, which is not measure" by conventional assay, is release" after "ama$e to muscle. +uch "ata e*ist showin$ early intervention with co!ious alkalini#e" I1 crystalloi" can !revent renal "ama$e. A :;4lea" (C. 'choice A) may be useful, but $iven the nature of the in<uries, an" the fact that there are no clinical si$ns or sym!toms su$$estin$ myocar"ial ischemia, this intervention is not the most useful. 7he same hol"s true for a echocar"io$ram 'choice ). In the case of blunt chest wall trauma, this test is in"icate". 2owever, in the case of a lower e*tremity crush in<ury, it is not imme"iately in"icate". ecause there is no reason to sus!ect that the heart is the source of the elevate" C5, fractionate" C5 levels 'choice %) woul" not be useful. A concern in trauma !atients is always for liver "ama$e in the form of contusion or avulsion.

7his !atient has relatively normal liver function tests an" hematocrit. 7herefore, a ri$ht u!!er qua"rant ultrasoun" 'choice () woul" not be very useful for a sus!ecte" intra4 ab"ominal !rocess. :/) (*!lanation: 7he correct answer is . Inhale" steroi"s shoul" be the mainstay of thera!y to maintain remission between asthmatic e!iso"es. 7hey effectively re"uce airway inflammation, whereas the beta4; a$onist inhalers are more effective for short4term relief of sym!toms. 7he most likely si"e effect you woul" fin" on the boar"s from steroi" inhalers is the "evelo!ment of oral can"i"iasis, which can be !revente" by !rom!t mouth rinsin$ after inhaler use. Amino!hylline 'choice A) has no role in the maintenance thera!y of asthma an" has very limite" use "urin$ an acute attack as well. Cromolyn 'choice C) is not effective in maintainin$ remissions. 2owever, it is useful as a !ro!hylactic measure before initiatin$ e*ercise in those !atients whose tri$$er for bronchoconstriction is e*ercise. A meta!roterenol inhaler 'choice %) is of value in re"ucin$ sym!toms "urin$ an acute attack but is not effective in !reventin$ rela!ses. Oral !re"nisone 'choice () is effective in !reventin$ rela!ses but is reserve" for !atients who have ha" severe refractory asthmatic attacks an" shoul" not be use" for !atients with a mil"er course. :C) (*!lanation: 7he correct answer is %. ,acunar infarcts are small 'less than >./ cm) focal areas of !arenchymal loss foun" in the basal $an$lia, anterior limb of the internal ca!sule, !ons, an" occasionally cerebral white matter. 7hey are !resume" to be of ischemic ori$in, but their !atho$enetic mechanism has not yet been eluci"ate". It is a fact that lacunae are !articularly frequent in !ersons sufferin$ from hy!ertension or "iabetes mellitus, both con"itions bein$ associate" with systemic arteriolosclerosis. Arteriolosclerosis lea"s to ischemic "ama$e to several or$ans, !articularly the brain an" ki"neys. ,acunar infarcts manifest with focal neurolo$ic "eficits that ten" to recover s!ontaneously. erry aneurysms 'choice A) are either asym!tomatic or manifest with su""en subarachnoi" blee"in$. Only rarely "o they manifest by com!ression of cranial nerves. Aneurysms of the circle of Willis are not visible on re$ular C7 scans.

Cerebral hemorrha$e 'choice ) is i"entifie" on C7 scans as intra!arenchymal hy!er"ense areas. 2y!ertension is the most common !re"is!osin$ con"ition. 7he !atient usually !resents with su""en onset of hea"ache an" ra!i" "eterioration of mental status. (mbolic infarcts 'choice C) are most commonly hemorrha$ic in ori$in. 7hey usually involve the cerebral corte* in a we"$e4sha!e" "istribution. +etastatic lesions 'choice () manifest as shar!ly "emarcate" no"ules usually locate" at the <unction between $ray an" white matter :D) (*!lanation: 7he correct answer is A. 7his !atient has the !hysical fin"in$s of aortic insufficiency an" the "evelo!ment of con$estive heart failure with biventricular failure. ,eft ventricular sym!toms "escribe" here are the fin"in$s of !ulmonary con$estion, an" ri$ht4si"e" heart failure is "emonstrate" by the <u$ulovenous "istention, con$este" liver, an" !eri!heral e"ema. )ince the Hbackwar"H failure sym!toms are "ue to re$ur$itant flow across the incom!etent aortic valve, the most useful thera!y woul" be afterloa" re"uction with an AC( inhibitor, such as ca!to!ril. Couma"in 'choice ) is sometimes use" in !atients with a "ilate" left ventricle, to re"uce the risk of left ventricular thrombus an" thromboembolic !henomena. It will not, however, re"uce sym!toms of biventricular failure. Althou$h "i$o*in 'choice C) woul" be hel!ful in increasin$ the left ventricular contractility, it is less effective than ca!to!ril in treatin$ the un"erlyin$ !atho!hysiolo$y. ,ikewise, furosemi"e 'choice %) woul" be useful in alleviatin$ the sym!toms of !ulmonary con$estion but woul" not a""ress the un"erlyin$ !atho!hysiolo$ic !rocess. Isosorbi"e "initrate 'choice (), like other nitrates, is a veno"ilator, which will re"uce !reloa" an" re"uce !ulmonary con$estion but will not a""ress the un"erlyin$ re$ur$itant flow that is causin$ the left ventricular failure. :@) (*!lanation: 7he correct answer is %. Interstitial lun$ "iseases often "ecrease lun$ com!liance an" au$ment elastic recoil of the lun$s, a !rocess that may lea" to the fin"in$ of late ins!iratory crackles hear" on auscultation of the chest. Other "isease !rocesses !ro"ucin$ this fin"in$ inclu"e con$estive heart failure an" bilateral basilar air s!ace "isease.

An increase" anterior4!osterior 'A&) "iameter on ins!ection of the chest 'choice A) an" hy!erresonance to !ercussion of the chest 'choice () are im!ortant e*amination fin"in$s of em!hysema, but not interstitial lun$ "isease, when !resent alone. %iffuse e*!iratory whee#es 'choice ) are a classic fin"in$ of "iseases affectin$ the airways, such as asthma or chronic bronchitis. %ullness to !ercussion at the bases 'choice C) may be a!!reciate" when !leural effusions are !resent, but is not classically associate" with interstitial lun$ "iseases. :I) (*!lanation: 7he correct answer is . 7he !atient is !resentin$ with the classic clinical features of infectious mononucleosis, with fever, fati$ue, sym!toms of !haryn$itis, cervical lym!ha"eno!athy, an" he!atos!lenome$aly. A hetero!hile a$$lutinin test '+onos!ot) shoul" be !ositive at this !oint into the clinical illness. A com!lete bloo" count will most likely reveal a lym!hocytosis with aty!ical lym!hocytes. Acute he!atitis infection 'choice A) can !resent with fevers, myal$ias, fati$ue, an" he!atos!lenome$aly, but "oes not have the !haryn$eal sym!toms or cervical a"eno!athy seen in this !atient. Non42o"$kin lym!homa 'choice C) can !resent with the cervical lym!ha"eno!athy, fevers, an" he!atos!lenome$aly, but "oes not $enerally !resent this acutely an" "oes not !resent with the !haryn$eal sym!toms seen here. )arcoi"osis 'choice %) may !resent with cervical a"eno!athy an" he!atos!lenome$aly with fevers, but will not !resent with the !haryn$eal sym!toms an" will almost always have !rominent !ulmonary fin"in$s. )tre!tococcal !haryn$itis 'choice () may !ro"uce the fever, !haryn$eal sym!toms, an" cervical lym!ha"eno!athy, but woul" not e*!lain the systemic sym!toms or myal$ia, an" he!atos!lenome$aly. ;>) (*!lanation: 7he correct answer is . An in$estion of to*ic quantities of the antihistamine "i!henhy"ramine is "an$erous because of the anticholiner$ic si"e effects, which inclu"e "ry mouth, my"riasis, ab"ominal "istension, tachycar"ia, e*citation, an" coma. 7he imme"iate life4threatenin$ "an$er of to*ic levels of anticholiner$ic me"ications inclu"es the vulnerability to "evelo!in$ mali$nant heart rhythms.

en#o"ia#e!ines 'choice A) are usually not lethal unless combine" with ethanol 'choice C) or o!iates 'choice (), in which case their res!iratory su!!ressive effects are enhance". As the !atient is currently intubate" an" su!!orte" with mechanical ventilation, there is no current "an$er that the res!iratory su!!ression effect of either alcohol or ben#o"ia#e!ines will lea" to !roblems. Olan#a!ine 'choice %) is an aty!ical anti!sychotic with a relatively beni$n to*icity !rofile an" relatively few anticholiner$ic si"e effects. ;:) (*!lanation: 7he correct answer is (. 7his !atient has the classic !resentation of Crohn "isease. 7his $enerally !resents in youn$ a"ults with subacute or chronic sym!toms, ty!ically of ri$ht lower qua"rant !ain, "iarrhea, an" wei$ht loss. )he also has e*traintestinal manifestations of a!hthous ulcers an" arthral$ias. 7he "escri!tion of this !atientAs !ain su$$ests that it is locate" in the terminal ileum. 7his area is best seen with an u!!er $astrointestinal an" small bowel barium stu"y. An ab"ominal C7 scan 'choice A) is sometimes useful in !atients with Crohn "isease with a sus!ecte" abscess or fistula. 2owever, this is not su$$este" by the history or !hysical in this case. An ab"ominal sono$ram 'choice ) woul" not !rovi"e sufficient infromation for the "ia$nosis of Crohn "isease. arium stu"ies are much better for visuali#ation of the ty!ical features of inflammatory bowel "isease. A barium enema 'choice C) is not as effective a test as a small bowel series at visuali#in$ the terminal ileum. 7he barium enema woul" "emonstrate evi"ence of colonic Crohn "isease but the history an" !hysical here su$$est ileal "isease is more likely. A si$moi"osco!y 'choice %) woul" not be of value in assessin$ the terminal ileum. ;;) (*!lanation: 7he correct answer is A. 7he !atient has the clinical an" $enetic features of %own syn"rome. )uch !atients are at a hi$h risk of "evelo!in$ senile "ementia of the Al#heimer ty!e ')%A7) in the ?th an" /th "eca"es of life. Neuro!atholo$y often reveals senile !laques an" neurofibrillary tan$les. 2y"roce!halus 'choice ) can cause neurolo$ic "eterioration. It may occur in %own syn"rome !atients, but no association has been "escribe". %iseases such as normal !ressure hy"roce!halus may cause ata*ia, incontinence, an" "ementia.

2y!othyroi"ism 'choice C) may cause mental status chan$es, such as lethar$y an" obtun"ation, but are not !articularly associate" with %own syn"rome. 7he ailment may be reversible with thyroi" hormone re!lacement. +ulti4infarct "ementia "ue to multi!le strokes 'choice %) can be a cause of "ementia in !atients !rone to embolic strokes, such as those with vasculo!athies. 2owever, !atients are usually ol"er. &rion "iseases 'choice (), such as Creut#fel"t4Nakob "isease, cause "ementia. )ome ty!es have been foun" to be transmitte" via infecte" cattle. ;0) (*!lanation: 7he correct answer is %. 7he rhythm "escribe" on the (C. is multifocal atrial tachycar"ia. 7his is characteri#e" by variable & wave mor!holo$y an" &6 an" 66 intervals. 7he control of this tachycar"ia comes with im!rove" ventilation an" o*y$enation. 7his con"ition is associate" with severe !ulmonary "isease. %i$italis 'choice A) enhances myocar"ial contractility !rimarily by inhibitin$ membrane so"ium4!otassium A7&ase. It is useful in the mana$ement of heart failure an" rate control in atrial fibrillation. Warfarin 'choice ) is use" in the mana$ement of chronic atrial fibrillation to re"uce the risk of an embolic stroke. In the acute settin$, however, it has little utility in rate control or anticoa$ulation. (lectrical car"ioversion 'choice C) is of little benefit in rhythm or rate control in multifocal atrial tachycar"ia. It is in"icate" in the conversion of atrial fibrillation to a normal sinus rhythm. In those situations, the !atient must first be a"equately anticoa$ulate". %efibrillation 'choice () is in"icate" in !atients with arrhythmia who are hemo"ynamically unstable. 7y!ically, it is in"icate" in ventricular fibrillation an" tachycar"ia. If a !atient has synco!e as a result of such tachyarrhythmias, then an internal "efibrillator must be installe". ;?) (*!lanation: 7he correct answer is . 7he "ia$nosis of !neumonia absolutely requires that an infiltrate of some sort 'whether interstitial or !arenchymal, lobar or "iffuse) be !resent on a chest ra"io$ra!h.

2y!o*emia on !ulse o*imetry 'choice A) is one !ossible !hysical manifestation of severe !neumonia but is certainly not require" for the "ia$nosis. A s!utum .ramAs stain showin$ $ram4!ositive "i!lococci 'choice C) is su$$estive of a !neumonia, but it coul" also easily be e*!laine" by !haryn$eal coloni#ation with this or$anism. A s!utum .ramAs stain showin$ neutro!hils 'choice %) is also hi$hly su$$estive of some !ulmonary inflammatory !rocess, althou$h not necessarily !neumonia. A tem!erature to 0@.C ':>:.? -) 'choice () is clearly a nons!ecific si$n an" can be associate" with a multitu"e of !ossible etiolo$ies. ;/) (*!lanation: 7he correct answer is C. 7his !atient has ha" multi!le a"missions for similar sym!toms an" is !resentin$ to the emer$ency "e!artment with vomitin$ an" mi"e!i$astric !ain ra"iatin$ to the back. 7his !attern of !ain shoul" always su$$est the !ossibility of !ancreatitis. Note that acute !ancreatitis can mimic infection with mil" fever, tachycar"ia, an" mo"estly increase" white count. 2owever, amylase an" li!ase levels shoul" be or"ere" in this clinical settin$ as elevation of these !ancreatic en#ymes can confirm the "ia$nosis. In this homeless !atient, who has ha" several e!iso"es of !ancreatitis, either alcohol !ancreatitis or $allstone !ancreatitis may be res!onsible. 2owever, a!!ro!riate care of $allstone !ancreatitis woul" have resulte" in a cholecystectomy, which shoul" have !revente" future e!iso"es, leavin$ it most !robable that her !ancreatitis is relate" to alcohol use. Acute bacterial cholan$itis 'choice A) ty!ically !resents with CharcotAs tria", consistin$ of ri$ht u!!er qua"rant !ain, fever, an" <aun"ice. Acute cholecystitis 'choice ) $enerally !resents as ri$ht u!!er qua"rant !ain, which may ra"iate to the intersca!ular area. 7he elevate" amylase an" li!ase levels are far more su$$estive of !ancreatitis than acute cholecystitis. 1iral he!atitis "ue to he!atitis A or 'choices % an" () may cause e!i$astric !ain, but more commonly !resents with ri$ht u!!er qua"rant !ain an" is unlikely to "evelo! quite so abru!tly. -urthermore, the recurrent nature of these e!iso"es is aty!ical for viral he!atitis for he!atitis an" "oes not occur with he!atitis A. ;C) (*!lanation: 7he correct answer is

. 7hese situations are all too common in the in!atient settin$. &hysicians walk a very fine ethical line, with one si"e bein$ active hastenin$ of "eath while the other is ina"equate su!!ort of terminal illness. One of the more common ways in which !hysicians ensure that this line is obeye" is by titratin$ !ain me"ications in terminal !atients to Hcomfort. H 7his !atient has ob<ective evi"ence 'vital si$ns, bloo" !ressure, slee!in$) that she is comfortable. 7here is therefore no me"ical in"ication to increase !alliation via a""itional !ain relief. ,ikewise, once a !atient is comfortable, there is $enerally no reason to "ecrease the mor!hine "ri! rate 'choice A). As "iscusse" above, there is no ob<ective evi"ence to su$$est that increasin$ the mor!hine "ri! rate 'choice C) is in"icate". (ven if the !atient "oes a!!ear comfortable, as is the case in this vi$nette, informin$ the family that the !atient a!!ears comfortable an" not to make requests of the me"ical team 'choice %) is ina!!ro!riate, since the family will have multi!le requests, issues an" concerns they ri$htfully will want to "iscuss with the care$ivers. Informin$ the family that you will not !artici!ate in killin$ the !atient 'choice () woul" be an ina!!ro!riate way in which to brin$ such sentiments to the forefront. ;D) (*!lanation: 7he correct answer is A. Celiac s!rue will !ro"uce si$ns an" sym!toms of malabsor!tion of the !ro*imal small bowel. Iron 'alon$ with folate an" calcium) is !referentially absorbe" in the !ro*imal small bowel an" is not well com!ensate" for in the "istal small intestine when there is !ro*imal small bowel malabsor!tion. 1itamin A, %, an" 5 "eficiencies will occur but "o not !ro"uce the "escribe" sym!toms. 1itamin A "eficiency 'choice ) will !ro"uce ni$ht visual "isturbances rather than nysta$mus. 1itamin :; "eficiency 'choice C) rarely occurs in celiac s!rue since the malabsor!tive !rocess is $enerally more severe in the !ro*imal small bowel whereas :; absor!tion occurs in the terminal ileum. 1itamin % "eficiency 'choice %) will !ro"uce hy!ocalcemia rather than hy!ercalcemia. 1itamin 5 "eficiency 'choice () !ro"uces a coa$ulo!athy rather than hy!okalemia. ;@) (*!lanation:

7he correct answer is . 7his !atient has acute $laucoma, which is an emer$ency brou$ht on by increase" intraocular !ressure. It is characteri#e" by a ra!i" onset of severe !ain an" blurry vision in one eye. H2alosH are often seen aroun" li$hts, an" nausea an" vomitin$ may be !resent. A HsteamyH4a!!earin$ cornea an" a "ilate" an" nonreactive !u!il are seen. 7he eye is ten"er an" Hhar"H when !al!ate". (mer$ency treatment inclu"es carbonic anhy"rase inhibitors or mannitol in con<unction with to!ical beta blockers to re"uce intraocular !ressure. ,aser iri"ectomy is !erforme" when intraocular !ressure is un"er control. 7he worst com!lication of $laucoma is blin"ness. Acute con<unctivitis 'choice A) is cause" by bacteria, viruses or aller$ens. acterial infections cause a muco!urulent "ischar$e. 1iruses an" aller$ens cause a watery, clear "ischar$e. 7here is mil" "iscomfort with a minimal chan$e in visual acuity. 7he con<unctiva is in<ecte" an" the !u!il is reactive. O!tic neuritis 'choice C) is an inflammatory "isease of the o!tic nerve. 7he ty!ical !atient is a youn$ woman with a ra!i" onset of !ainful loss of vision. &a!ille"ema is !resent. +any !atients later "evelo! multi!le sclerosis. &seu"otumor cerebri 'choice %) is characteri#e" by !a!ille"ema cause" by i"io!athic intracranial hy!ertension. +ost !atients are youn$, obese females with severe hea"aches an" vomitin$. 7he treatment is carbonic anhy"rase inhibitors. 7he worst com!lication is blin"ness. 6etinal "etachment 'choice () is a characteri#e" by a unilateral, !ainless loss of vision. It is often "escribe" as a HcurtainH comin$ "own over one eye. -un"usco!ic e*amination reveals a $ray retina Hfloatin$H within the vitreous humor. 7his is "escribe" as a $ray clou". ;I) (*!lanation: 7he correct answer is (. Althou$h essential hy!ertension is the most common cause of hy!ertension, accountin$ for a!!ro*imately I:= of all hy!ertensive "ia$noses, in this case, there are si$ns an" sym!toms su$$estive of another "ia$nosis. -or this !atient, her sensitivity to AC( inhibitors, her elevate" creatinine an" her con$estive heart failure all stron$ly su$$est some "e$ree of renal artery stenosis. An al"osterone secretin$ tumor 'choice A) is incorrect, since for this to be "ia$nose", serum so"ium woul" be quite elevate" an" serum !otassium quite low. Coarctation of the aorta 'choice ) is not likely since !ulses are equal an" symmetrical, an" because she is C@ years ol". +ost of these !atients "ie at a youn$ a$e if the "efect is not correcte" in chil"hoo".

(ssential hy!ertension 'choice C) is the most common cause for hy!ertension in the Fnite" )tates. As reflecte" by its name, the etiolo$y of the hy!ertension is not yet clear. Althou$h by "efinition this is a "ia$nosis of e*clusion, most !atients are assume" to have essential hy!ertension, an" secon"ary causes seriously investi$ate" only if the & is very "ifficult to control. A !heochromocytoma 'choice %) is a very uncommon cause for hy!ertension. 7he classic !resentation is with H!aro*ysmsH of autonomic activity. 7his !atient has no evi"ence for this. 0>) (*!lanation: 7he correct answer is . A !ulmonolo$ist may be able to hel! confirm your "ia$nosis an" is best equi!!e" to recommen" confirmatory tests, such as transbronchial bio!sy, an" hel! "irect treatment o!tions. )im!ly arran$in$ for follow u! 'choice A) without confirmin$ the "ia$nosis is ina"equate. It is true that many of the interstitial lun$ "iseases have "isa!!ointin$ treatment res!onses, but there are im!ortant new a"vances all the time, an" a tissue "ia$nosis may be in"icate". Overall, !atients will be hel!e" by referral to a s!ecialist at this !oint. &re"nisone 'choice C) an" a#athio!rine 'choice %) are immunosu!!ressive a$ents that are sometimes use" to treat the active inflammation seen in some interstitial lun$ !rocesses. ecause of their severe, systemic si"e effects, however, they shoul" $enerally be use" in coor"ination with a lun$ s!ecialist. O!en lun$ bio!sy !erforme" by a thoracic sur$eon 'choice () may ultimately hel! confirm the "ia$nosis, but referral to a !ulmonolo$ist is more a!!ro!riate initially. &ulmonolo$ists are more likely to !lay a role in the confirmatory evaluation an" hel! "irect treatment. 7hey may then "eci"e to !erform a bio!sy via bronchosco!y or consult a thoracic sur$eon if in"icate". 0:) (*!lanation: 7he correct answer is C. 7he clinical !resentation outline" is consistent with her!es sim!le* ence!halitis, which is usually "ue to her!es sim!le* virus : '2)1 :). 2)1 is thou$ht to cause ence!halitis followin$ trans!ort to the brain alon$ the tri$eminal nerve. 7his usually occurs in !ersons harborin$ the latent viral form in the .asserian $an$lion. item!oral hemorrha$ic necrosis is characteristic of her!es ence!halitis. +ost !atients "evelo! sym!tomatolo$y of abru!t onset, with fever, hea"ache, nuchal ri$i"ity, an" confusion. +otor an" sensory "eficits are often observe" on !hysical e*amination. 7reatment shoul"

be !rom!tly starte" whenever there is reasonable sus!icion of her!es ence!halitis. In fact, acyclovir an" similar "ru$s are hi$hly effective a$ainst this form of ence!halitis. 2er!es ence!halitis is usually fatal without treatment. In this case, note the ty!ical C)- chan$es associate" with viral ence!halitis. )u!!ortive treatment until C)- culture results are available 'choice A) is not a"equate, since often culture results are ne$ative. -urthermore, treatment is effective only if starte" in the earliest sta$es. Cerebral an$io$ra!hic stu"ies 'choice ) are not useful in this case. 7reatment with an antimycotic a$ent 'choice %) is a"equate in cases of fun$al menin$oence!halitis, which usually occurs in immunocom!romise" !atients. Cry!tococcal menin$oence!halitis, for e*am!le, is frequent in AI%) !atients. 7reatment with antimicrobial a$ents 'choice () shoul" be imme"iately starte" whenever there is clinical an"Bor laboratory evi"ence of bacterial menin$itis. 7he C)- in !urulent 'bacterial) menin$itis shows marke" neutro!hilia, increase" !rotein concentration, an" re"uce" $lucose. 0;) (*!lanation: 7he correct answer is . When the bloo" $lucose level "ro!s below a!!ro*imately ;/ m$B",, as in this case, cerebral $lucose reserves are quickly "e!lete". In res!onse, !rotein an" li!i" com!onents of neurons are metaboli#e", which can lea" to irreversible brain "ama$e. 7herefore, !rom!t treatment with intravenous $lucose is critical in this !atientAs initial mana$ement. While the bloo" !ressure of I>B// is relatively low 'choice A), it shoul" be well tolerate" by the youn$ woman in this case. 2owever, ol"er !atients with si$nificant vascular "isease who are accustome" to hi$her bloo" !ressures may not tolerate this bloo" !ressure. (ven thou$h the o*yhemo$lobin "esaturation curve is relatively flat at the u!!er en" of the saturation scale, an o*y$en saturation of @D= 'choice C), while not o!timal, is consistent with life an" shoul" not !ose any "ama$e to vital tissues in the short term. A !ulse of :?> 'choice %) is imme"iately threatenin$ to life only if the heart rhythm is at risk to "e$enerate into a "an$erous rhythm or if the !atient is unable to maintain a reasonable bloo" !ressure with the tachycar"ic !ulse. 7his is rarely the case with a sinus tachycar"ia 'as evi"ence" by the fact that most !eo!le e*ercisin$ are not unstable "ue to tachycar"ia).

7he relatively slow res!iration rate of @ 'choice () likely reflects the metabolic an" to*ic state of the !atient, an" is threatenin$ to the !atient only if it limits o*y$enation, ventilation, or aci"Bbase balance. 00) (*!lanation: 7he correct answer is . When the bloo" $lucose level "ro!s below a!!ro*imately ;/ m$B",, as in this case, cerebral $lucose reserves are quickly "e!lete". In res!onse, !rotein an" li!i" com!onents of neurons are metaboli#e", which can lea" to irreversible brain "ama$e. 7herefore, !rom!t treatment with intravenous $lucose is critical in this !atientAs initial mana$ement. While the bloo" !ressure of I>B// is relatively low 'choice A), it shoul" be well tolerate" by the youn$ woman in this case. 2owever, ol"er !atients with si$nificant vascular "isease who are accustome" to hi$her bloo" !ressures may not tolerate this bloo" !ressure. (ven thou$h the o*yhemo$lobin "esaturation curve is relatively flat at the u!!er en" of the saturation scale, an o*y$en saturation of @D= 'choice C), while not o!timal, is consistent with life an" shoul" not !ose any "ama$e to vital tissues in the short term. A !ulse of :?> 'choice %) is imme"iately threatenin$ to life only if the heart rhythm is at risk to "e$enerate into a "an$erous rhythm or if the !atient is unable to maintain a reasonable bloo" !ressure with the tachycar"ic !ulse. 7his is rarely the case with a sinus tachycar"ia 'as evi"ence" by the fact that most !eo!le e*ercisin$ are not unstable "ue to tachycar"ia). 7he relatively slow res!iration rate of @ 'choice () likely reflects the metabolic an" to*ic state of the !atient, an" is threatenin$ to the !atient only if it limits o*y$enation, ventilation, or aci"Bbase balance. 0?) (*!lanation: 7he correct answer is . Clinical sym!tomatolo$y, +6I fin"in$s, an" I$. oli$oclonal ban"s are hi$hly characteristic of multi!le sclerosis '+)). It shoul" be em!hasi#e", however, that a "ia$nosis of "efinite +) entails i"entification of multi!le e!iso"es of white matter "emyelination se!arate" in s!ace an" time. 7hus, a sin$le e!iso"e of "emyelination4 relate" neurolo$ic "eficits is not sufficient to make a "ia$nosis of +). 2owever, the characteristic !eriventricular "istribution of white matter chan$es on 7;4wei$hte" ima$es an" the C)- chan$es are ty!ical of +). 7he latter is thou$ht to be "ue to activation of s!ecific lym!hocytic clones a$ainst white matter anti$ens.

Cytome$alovirus 'C+1) ence!halitis 'choice A) affects severely immunocom!romise" !atients, es!ecially those with AI%). C+1 has a s!ecific tro!ism for e!en"ymal cells an" causes !eriventricular lesions. Oli$o"en"ro$lioma 'choice C) is a !rimary brain tumor of oli$o"en"ro$lial ori$in. It accounts for a!!ro*imately /= of all !rimary brain neo!lasms. It is usually locate" in the white matter of the centrum semiovale close to the corte*. +il" lym!hocytosis may be seen in the C)- as a nons!ecific res!onse known as nei$hborhoo" reaction. &ro$ressive multifocal leukoence!halo!athy 'choice %) also occurs in immunocom!romise" !atients. It is "ue to NC virus 'a !a!ovavirus), an" is characteri#e" by multi!le foci of white matter "estruction scattere" throu$hout the cerebral hemis!heres an" cerebellum. ,ym!homa 'choice () is !articularly frequent in immunocom!romise" in"ivi"uals, such as those with AI%) an" those receivin$ immunosu!!ressant thera!y followin$ trans!lantation. In AI%) !atients, brain lym!homa manifests as a s!ace4occu!yin$ mass that a!!ears as a rin$4enhancin$ lesion on +6IBC7. In immunocom!etent in"ivi"uals, !rimary brain lym!homa involves the !eriventricular re$ions an" may be confuse" with +). Aty!ical lym!hocytes may be i"entifie" in the C) -. 0/) (*!lanation: 7he correct answer is (. 7his !atient has the characteristic !hysical fin"in$s of an aortic insufficiency, inclu"in$ the lou" blowin$ "ecrescen"o murmur an" the boun"in$ !eri!heral !ulses. 7hese !atients ty!ically will have a wi"e !ulse !ressure, i.e., elevate" systolic bloo" !ressure relate" to a lar$e left ventricular volume bein$ e<ecte" 'until left ventricular failure su!ervenes). 7he low "iastolic !ressure is a result of the ra!i" run4off from the aorta "ue to the re$ur$itant flow across the aortic valve into the left ventricle, as well as the forwar" flow to the aorta. ra"ycar"ia 'choice A) is not a feature of either aortic insufficiency or left ventricular "ilatation. Isolate" systolic hy!ertension 'choice ) may be foun" in the el"erly an" is an in"e!en"ent risk factor for stroke, but it is not correlate" with the !resence of aortic insufficiency. &ulsus !ara"o*us 'choice C) may occur in !atients who have severe asthma or other si$ns of hi$h ri$ht4si"e" !ressures. It is not a feature of aortic insufficiency. 7achycar"ia 'choice %) may occur in !atients with a faile" left ventricle, but it is not a hallmark of aortic insufficiency.

0C) (*!lanation: 7he correct answer is . 7his !atient has !hysical fin"in$s consistent with ri$ht4si"e" 2orner syn"rome, which consists of a tria" of !tosis, miosis, an" anhi"rosis. 7his results from a lun$ cancer in this lon$4time smoker at the a!e* of the ri$ht lun$, which causes com!ression of the cervical sym!athetic !le*us. 7hese !atients may also com!lain of sca!ular !ain an" a ra"iculo!athy in the ulnar nerve "istribution. 7his man has serious "isease, an" woul" not be e*!ecte" to have a normal chest *4ray film 'choice A). Choices C an" % are not locate" at the lun$ a!e* an" woul" not be e*!ecte" to cause the a!!ro!riate nerve com!ression. Althou$h choice ( is locate" at the ri$ht u!!er lobe, a !neumonia "oes not cause nerve com!ression. 0D) (*!lanation: 7he correct answer is A. 7his !atient has the sym!toms of a motility4ty!e "ys!ha$ia in that he has "ifficulty with most soli"s, not liqui"s, from the onset of his sym!toms. 7he nocturnal as!iration of foo" occurs because the eso!ha$us remains fille" for hours or even "ays after com!letin$ a meal. 7he *4ray film reveals the ty!ical "ilate" eso!ha$us of achalasia, which is terme" a bir"As4beak eso!ha$us, with "istal eso!ha$eal ta!erin$. %iffuse eso!ha$eal s!asm 'choice ) will ty!ically !ro"uce Hnon4car"iac chest !ainH in association with a motility4ty!e "ys!ha$ia. (so!ha$eal squamous cancer 'choice C) woul" !ro"uce a mechanical ty!e "ys!ha$ia an" is unlikely in a ?;4year4ol" man without any s!ecific risk factors, i.e. smokin$, "rinkin$, lye in$estion, or &lummer41inson syn"rome. &e!tic stricture 'choice %) is wron$ because it woul" !ro"uce a mechanical, not a motility, ty!e "ys!ha$ia. )clero"erma 'choice () will also !ro"uce a motility4ty!e "ys!ha$ia, but it is very uncommon for men to "evelo! this "isease. -urthermore as!iration in sclero"erma occurs only after a !e!tic stricture has "evelo!e". &rior to this, the lower eso!ha$eal s!hincter is wi"e o!en, an" foo"s "o not accumulate in the eso!ha$us.

0@) (*!lanation: 7he correct answer is %. )ickle cell crisis is the likely cause of this !atientAs unusual combination of sym!toms. &atients with sickle cell anemia are at risk for !ain crises when they are e*!ose" to col", have infection, become "ehy"rate" or are un"er severe stress resultin$ in o*y$en "e!rivation. &ain occurs "ue to occlusion of small vessels by the sickle4sha!e" cells. Althou$h !ain may be felt anywhere, more common sites inclu"e the bones, <oints, an" ab"omen. +en may !resent with !ria!ism 'a !ersistent, !ainful erection unrelate" to se*ual activity or e*citement). 7reatment is lar$ely sym!tomatic an" su!!ortive. It is im!ortant to !erform a thorou$h evaluation in !atients with sickle cell "isease, so that other "ia$noses are not misse" such as: ab"ominal !ain "ue to a sur$ical ab"omen, res!iratory "istress "ue to !ulmonary infarction, or altere" mental status "ue to cerebral infarction. %ehy"ration 'choice A) "oes not ty!ically !ro"uce such !ainful sym!toms. )e!tic arthritis "ue to $onococcal infection 'choice ) is usually unilateral an" associate" with fever. .onococcal infection of the $enitals results in urethritis with sym!toms of frequency, "ysuria, an" urethral "ischar$e, but not !enile erection. A !atient with acute leukemia 'choice C) may also !resent with bone !ain an" !ria!ism, but sym!toms "ue to abnormal bone marrow function are also often !resent, such as feverBinfection "ue to $ranulocyto!enia, an" blee"in$ "ue to thrombocyto!enia. )u!!ortive care with o*y$en an" intravenous flui"s is not sufficient to treat acute leukemiaG chemothera!y is initiate" imme"iately. Choice ( is unlikely, $iven the recurrence of sym!toms an" resolution with su!!ortive care. 0I) (*!lanation: 7he correct answer is (. 7his !atient has two first4"e$ree relatives who have un"er$one total !roctocolectomies for a !oly!osis syn"rome. 7his most likely re!resents the familial !oly!osis coli syn"rome in that both siblin$s ha" hi$h numbers of colonic a"enomas an" un"erwent an a!!ro!riate total !roctocolectomy. If this !atient is foun" to have several "o#en !oly!s "urin$ the fle*ible si$moi"osco!y, it is evi"ence that he is e*!ressin$ the !henoty!ic trait of the familial a"enomatous !oly!osis '-A&) $ene. 6ather than wait for any of these !oly!s to un"er$o mali$nant "e$eneration, he shoul" have a total !roctocolectomy in the imme"iate near future. Once many !oly!s have been "emonstrate", the likelihoo" of one of these becomin$ mali$nant is hi$h.

)che"ulin$ re!eat si$moi"osco!y 'choice A) or colonosco!y 'choice ) in : year is wron$ because this !atient with alrea"y e*tensive !oly!osis has !asse" the !oint of nee"in$ only !erio"ic monitorin$. (valuatin$ the com!lete colon with either a barium enema 'choice C) or full colonosco!y 'choice %) is not nee"e", since there is alrea"y sufficient evi"ence to warrant total !roctocolectomy. ?>) (*!lanation: 7he correct answer is C. Arnol"4Chiari malformations are amon$ the most frequent con$enital anomalies of the CN). 7he small !osterior fossa is a crucial "ia$nostic feature of Arnol"4Chiari ty!e ;. 7his chan$e is !robably res!onsible for "ownwar" "is!lacement of the cerebellar vermis an" me"ulla throu$h the foramen ma$num. 7his lea"s to obstruction of the C)- flow an" hy"roce!halus. Im!ortant associate" abnormalities inclu"e lumbar myelomenin$ocele an" syrin$omyelia. Anence!haly 'choice A) is the most severe form of neural tube "efect, resultin$ from failure of the neural tube to close, either at its rostral en" 'lea"in$ to anence!haly, ence!halocele, or ence!halomenin$ocele) or cau"al en" 'lea"in$ to s!ina bifi"a, menin$ocele, or myelomenin$ocele). Anence!haly is incom!atible with life. Arnol"4Chiari ty!e : malformation 'choice ) is "efinitely more frequent than the ty!e ; malformation, but is usually asym!tomatic. It consists of "ownwar" "is!lacement of the cerebellar tonsils throu$h the foramen ma$num. %an"y4Walker malformation 'choice %) refers to a constellation of anomalies that inclu"e an abnormally lar$e !osterior fossa, absence of cerebellar vermis, an" "evelo!ment of a lar$e e!en"yma4line" cyst that re!resents an e*!an"e" ?th ventricle. 2olo!rosence!haly 'choice () is usually associate" with trisomy :0 an", less often, trisomy :@. Incom!lete se!aration of the cerebral hemis!here alon$ the mi"line lea"s to a sin$le ventricular cavity enclose" within the forebrain. ?:) (*!lanation: 7he correct answer is C. loo" !ressure may be elevate" in times of stress. +ulti!le "eterminations over several visits, an" some form of home or work!lace monitorin$ shoul" be con"ucte" !rior to initiatin$ !harmacolo$ical thera!y in hy!ertensive !atients. A cou!le of weeks of home bloo" !ressure monitorin$ shoul" be a"equate to establish the baseline bloo" !ressure 'choice A).

+enstrual cycles shoul" not have an effect on the bloo" !ressure an" shoul" not $ui"e thera!y 'choice ). 6eturnin$ after ? months of untreate" hy!ertension woul" not be a"visable 'choice %). 2er bloo" !ressure mi$ht be elevate" after e*ercise, an" woul" not ai" in the "ia$nosis or mana$ement "ecision 'choice (). ?;) (*!lanation: 7he correct answer is . 7his !atient has ,e$ionella !neumonia, which is cause" by ,. !neumo!hila, a $ram4 ne$ative bacillus. It is often acquire" from a contaminate" water su!!ly 'air con"itionin$ systems) an" can lea" to outbreaks. 7he ma<or risk factors inclu"e a"vance" a$e, immunosu!!ression, chronic lun$ "isease, an" ci$arette smokin$. Clinical features inclu"e a cou$h, chest !ain, "iarrhea, hi$h fever, malaise, fati$ue, an" anore*ia. A chest *4ray film reveals infiltrates an" later, consoli"ation. .ramAs stain reveals leukocytes, but no or$anisms. Cultures on C9( a$ar $row the or$anism. 7he treatment is erythromycin. Chlamy"ia trachomatis'choice A) is an obli$ate intracellular !arasite that has features similar to $ram4ne$ative bacteria. &neumonia "evelo!s in neonates born to mothers infecte" with the bacteria. +ora*ella catarrhalis'choice C) is a $ram4ne$ative coccus that causes !neumonia in el"erly !atients with CO& %. Clinical features inclu"e low4$ra"e fever, chills, chest !ain, an" malaise. .ramAs stain reveals the or$anism. &enicillinBclavulanic aci" is the treatment. +yco!lasma !neumoniae'choice %) causes a community4acquire" !neumonia that occurs in youn$ a"ults. 7he features are a sore throat, non!ro"uctive cou$h, an" hea"ache. Chest *4ray films show interstitial infiltrates. 7reatment is with erythromycin. &neumocystis carinii'choice () is an o!!ortunistic !atho$en that causes !neumonia in immunocom!romise" hosts. %iffuse interstitial infiltrates are foun" on chest *4ray films. %ia$nosis is ma"e by e*amination of s!utum or bronchial washin$s. 7he treatment is trimetho!rim4sulfametho*a#ole. ?0) (*!lanation: 7he correct answer is A. 7his C?4year4ol" man has a minimal smokin$ history an" yet has very severe em!hysema. 2e also has a history of elevate" transaminases. 7he con"ition "escribe" here, al!ha4:4antitry!sin "eficiency, is the only "isease amon$ the choices that will affect

both or$an systems. 7hey "o not nee" to coe*ist in the same !atient, but there will ty!ically be a family history of either em!hysema in nonsmokers, or liver function test abnormalities without other obvious cause. 7here is also a family history !resent of !recocious em!hysema in his youn$er brother, an" this su$$ests a familial "isease !attern. &rimary hemochromatosis 'choice ) is a $enetically transmitte" "isease with an autosomal recessive !attern that leaves iron "e!osition in the liver, heart, an" !ancreas. It lea"s to a bron#e hy!er!i$mentation of the skin. &rimary sclerosin$ cholan$itis 'choice C) occurs in !atients with ulcerative colitis an" "oes not affect the lun$s. )econ"ary hemochromatosis 'choice %) occurs in !atients who have receive" massive amounts of bloo" transfusion over the years. Wilson "isease 'choice () is a familial $enetic liver "isease that also involves the eye '5ayser4-leischer rin$) an" causes neuro!sychiatric "isor"ers. 7here is no lun$ involvement in Wilson "isease. ??) (*!lanation: 7he correct answer is C. 7he !atient most likely has "evelo!e" he!atic ence!halo!athy, one of the many com!lications that affect cirrhotic !atients. One of the most useful an" essential com!onents of the evaluation of such a !atient is the history. When the history is $iven of a confuse" cirrhotic Hnot takin$ his me"icationsH, an un"erstan"in$ that ence!halo!athy is a !ossible "ia$nosis shoul" imme"iately come to min". Acute hy!onatremia 'choice A) is not su!!orte" by the serum so"ium level of :;/. Acute si$ns of hy!onatremia are seen when the serum so"ium falls more than :; m(qB, in less than ;? hours. 7he si$ns of such an illness involve nausea, vomitin$, confusion an" neurolo$ical fin"in$s relate" to brain e"ema. Ascen"in$ cholan$itis 'choice ) is not su!!orte" by the !hysical e*amination or by the classic CharcotAs tria" 'fever, ri$ht u!!er qua"rant !ain, an" <aun"ice). +etabolic aci"osis 'choice %) is not su!!orte" by the "ata. 7he !atient "oes have a low bicarbonate level, but his !2 is not known. 2e is likely aci"emic, but this is not the most likely cause of his confusion. )ub"ural hematoma 'choice () is not su!!orte" by the !hysical e*amination. 7he most common cause of )2 is traumatic tearin$ of the bri"$in$ veins of the "ura.

?/) (*!lanation: 7he correct answer is C. When varices are seen, the imme"iate a!!ro!riate mana$ement is ban"in$ of these varices, which will usually imme"iately lea" to cessation of blee"in$. (so!ha$eal sclerothera!y is also effective but is associate" with a hi$her instance of !ost4!roce"ure com!lications, such as eso!ha$eal ulceration, stricture, an" bacteremia. &reo!erative interferon 'choice A) is not effective at all in the mana$ement of variceal blee"in$. It is a com!onent of thera!y for he!atitis an" he!atitis C. 1aso!ressin 'choice ) is a vasoconstrictor that re"uces !ortal !ressures, but it is not the first4line of thera!y in control of variceal blee"in$. 7I&) 'trans<u$ular intrahe!atic !ortosystemic shunt) 'choice %) is use" as an intrahe!atic shunt to "ivert bloo" flow from the !ortal venous system to the systemic venous system. ,iver trans!lantation 'choice () is use" only in !atients with variceal blee"in$ who cannot be controlle" by any other means. ?C) (*!lanation: 7he correct answer is (. In this case, the combination of heman$ioblastoma an" an an$iomatous lesion of the retina is "ia$nostic of von 2i!!el4,in"au syn"rome. 7his con"ition is autosomal recessive, cause" by mutations of a $ene in the short arm of chromosome 0. 7he function of the $ene !ro"uct has not yet been clarifie". 2eman$ioblastomas may also "evelo! s!ora"ically an" are most frequently locate" in the cerebellum. 7he cyst4mural no"ule is characteristic of heman$ioblastoma, as well as of a few more brain neo!lasms. 7y!ical brain tumors that often consist of a cyst with a no"ular lesion inclu"e heman$ioblastoma an" !ilocytic astrocytoma in the cerebellum, $an$lio$lioma in the cerebral hemis!here, an" cranio!haryn$ioma in the su!rasellar re$ion. 2eman$ioblastoma has a rich vascular network a"mi*e" with interstitial cells 'the true neo!lastic elements). 7his !eculiar tumor of uncertain ori$in may !ro"uce erythro!oietin an" in"uce e*trame"ullary hemato!oiesis an" erythrocytosis '!resent in this e*am!le). ,i4-raumeni syn"rome 'choice A) is a rare con"ition cause" by mutations in the !/0 tumor su!!ressor $ene. &atients "evelo! a most bewil"erin$ array of mali$nant neo!lasms, inclu"in$ breast cancer, brain tumors '$liomas), an" skin cancer. 7he most characteristic lesions of neurofibromatosis ty!e : 'choice ) inclu"e neurofibromas, which are )chwann4cell tumors of !eri!heral nerves, an" cafV4au4lait s!ots.

Neurofibromatosis ty!e ; 'choice C), i.e., the central form of neurofibromatosis, manifests with multi!le brain tumors, inclu"in$ schwannomas of the ei$hth nerve, menin$iomas, an" $liomas. 7uberous sclerosis 'choice %) lea"s to hamartomatous malformations of several or$ans an" tissues throu$hout the bo"y, inclu"in$ cortical tubers. 7hese consist of no"ules in the corte* that result from "isor$ani#ation of the normal laminar arran$ement of neurons. ?D) (*!lanation: 7he correct answer is A. Ab"ominal aortic aneurysms $reater than / cm have $reater than a 0>= chance of ru!turin$ in 0 years. As most ru!tures result in "eath, an" as the mortality of sur$ical re!air has fallen shar!ly, sur$ical re!air of the aneurysm is in"icate". Aneurysms that are not re!aire" e*!an" on avera$e at about >.? cm !er year. 6e!air of these aneurysms can be accom!lishe" via either tra"itional o!en a!!roaches or new en"ovascular a!!roaches. A liver s!an of I cm 'choice ) is within normal limits for most of the !o!ulation. ,iver cysts 'choice C) are common inci"ental fin"in$s on ima$in$ of the ab"omen. Althou$h there is a lar$e "ifferential "ia$nosis for liver cysts, most etiolo$ies have a beni$n course an" can be followe" conservatively. +ost "isk herniations 'choice %) res!on" to me"ical treatment. 7he !ercenta$e of "isk herniations that require sur$ical intervention is less than :>=. In a""ition, the relationshi! between "isk herniations an" actual sym!tomatic "isease is not linear, an" many !eo!le with one fail to have the other. Frinary stone "isease 'choice () re!resents one of the most common !roblems of the urinary tract. Calcium, uric aci", struvite, or cystine stones within the renal !elvis that are nonobstructive can be followe" me"ically without sur$ical intervention. 7he !atient shoul" be encoura$e" to increase flui" intake. ?@) (*!lanation: 7he correct answer is %. 7his is an 2I1 !atient with si$ns of a !ulmonary infection an" a chest ra"io$ra!h su$$estin$ a !ulmonary !rocess. 7he conce!t un"erlyin$ this question is two4fol". -irst, 2I1 !atients suffer from aty!ical infections an" secon", one of the more common or$anisms, &neumocystis carinii can !resent with AN9 6A%IO.6A&2IC fin"in$. Once these conce!ts are un"erstoo", the course of action is imme"iately clear, be$in the a!!ro!riate intravenous thera!y. .iven that this !atient has a room air o*y$en saturation of less than D/=, steroi"s are in"icate". )teroi"s are only $iven if the question of a"renal insufficiency, a common com!lication of 2I1, is sus!ecte".

Oral IN2, rifam!in, !yra#inami"e, ethambutol 'choice A) woul" be a!!ro!riate for the treatment of tuberculosis. We have yet to see any "ata that this !atient also has 7 . We woul" nee" to see s!utum !ositive for A- stainin$ or a !ositive A- &C6 on s!utum. Intravenous trimetho!rim4sulfametho*a#ole 'choice ) is not the in"icate" thera!y for &C& !neumonia when o*y$en saturation is less than D/= on room air. Intravenous a#ithromycin 'choice C) is the incorrect antibiotic. In an otherwise healthy 0D4year4ol", the likely "ia$nosis woul" be aty!ical community4acquire" !neumonia, for which a#ithromycin mi$ht be useful. In an 2I1 !atient however, this is incorrect. Intravenous am!hotericin 'choice () is not a!!ro!riate since we have no sus!icion of a fun$al infection in this !atient. ?I) (*!lanation: 7he correct answer is A.Clostri"ium "ifficile has been i"entifie" as a ma<or cause of !seu"omembranous colitis, which usually !resents as antibiotic4associate" "iarrhea. 7he "ru$s im!licate" most commonly are am!icillin, clin"amycin, an" ce!halos!orins. +ost !atients !resent with ab"ominal cram!s an" ten"erness, fever, an" leukocytosis. -ecal e*amination reveals leukocytes an" is !ositive for C. "ifficile cytoto*in in I/= of !atients. 7he to*in assay is sensitive an" !ractical, an" shoul" be use" as the first line test when C. "ifficile is sus!ecte". )tool collection for fecal fat quantification 'choice ) is one of the most sensitive tests for steatorrhea. If fecal fat is elevate", the "ifferential "ia$nosis of malabsor!tion shoul" be !ursue". 7he clinical manifestations of malabsor!tion are subtle. &atients usually !resent with a chan$e in bowel habits an" com!lain of bulky, oily, an" foul4smellin$ stools that are "ifficult to flush. Ab"ominal !ain is uncommon. &atients with mo"erate to severe "iarrhea, or in whom an infectious etiolo$y is sus!ecte", shoul" have a stool e*amination for ova an" !arasites an" a stool culture 'choice C) for bacterial !atho$ens. acterial "iarrhea is likely if there is a history of simultaneous illness in in"ivi"uals who have share" contaminate" foo" with the !atient. ,ower ab"ominal cram!s an" ten"erness are !rominent, alon$ with tenesmus an" rectal ur$ency. In some cases, the stool may be $rossly bloo"y. Colonosco!y 'choice %) is useful in establishin$ the "ia$nosis of !seu"omembranous colitis. 2owever, it "oes not establish the etiolo$y an" shoul" be reserve" for !atients with serious or !ersistent "isease to e*clu"e alternative "ia$noses. It is not a first4line test.

/>) (*!lanation: 7he correct answer is A. 7his !atient has the ty!ical sym!toms of $astroeso!ha$eal reflu* "isease '.(6%) an" has now "evelo!e" nocturnal as!iration of aci". 7his is one of the e*tra4eso!ha$eal com!lications of aci" reflu*. It ty!ically occurs at ni$ht, when the !atient is su!ine an" has a relatively !oor $a$ refle* "ue to slee!. 7he other common e*traeso!ha$eal sym!toms of .(6% are laryn$itis an" hoarseness as aci" reflu* inflames the vocal cor"s. Antaci"s taken before be"time 'choice ) shoul" ten" to "ecrease rather than e*acerbate his sym!toms, since they will lower the stomach aci"ity. 7here is no reason for this otherwise healthy man to have a nocturnal as!iration of foo" eaten hours earlier 'choice C). 2; rece!tor anta$onist4in"uce" bronchoconstriction 'choice %) is incorrect, since these "ru$s "o not cause bronchoconstriction. 7racheoeso!ha$eal fistula 'choice () in a"ults in usually the result of an un"erlyin$ broncho$enic carcinoma, which woul" be very unusual in a 0:4year4ol" man, !articularly with no history of smokin$.

USMLE Step 2 Practice Test Block 1 :ame; +nstr#ctions; Ans'er the *#estions $elo' to the $est of yo#r a$ility. =hen yo# finish the test% click the 2heck $#tton at the $ottom to )ie' the res#lts.

1. A 41!year!ol" gastroenterologist atten"s an ann#al national conference in San 5rancisco. >n his ret#rn home% he "e)elops fe)ers% 'ith temperat#res as high as &8.4 2 3142.8 56 o)er the past "ays. /e has also "e)elope" a se)ere% nonpro"#cti)e co#gh 'ith associate" shortness of $reath at rest. +n a""ition% he complains of hea"ache an" fatig#e% an" his 'ife reports that he has $ecome slightly conf#se" at home. /e has also ha" na#sea an" fre*#ent loose stools. >n physical e.amination% his temperat#re is &8.1 2 3142.& 56% $loo" press#re is 11<714 mm /g% p#lse is 8<7min% an" respirations are 217min. >n l#ng e.amination% there are scattere" lo#" rhonchi in $oth l#ng fiel"s. The a$"ominal e.amination is normal. =hich of the follo'ing is the most likely "iagnosisa62ytomegalo)ir#s pne#monia $6Legionnaires "isease c6Myco$acteri#m t#$erc#losis "6Pne#mocystis carinii pne#monia e6Staphylococcal pne#monia :ormal La$s

2.A <(!year!ol" man presents to the clinic complaining of stea"y% "#ll $ack pain o)er the past & 'eeks. /e states that he has recently mo)e" after retiring from a career in $anking an" is searching for a ne' primary care physician. /is past me"ical history is significant for "i)ertic#losis% prior smoking% an" hypertension. /e says that he has r#n o#t of his $loo" press#re me"ication. /e "enies tra#ma to his $ack an" other'ise feels 'ell. >n physical e.amination his $loo" press#re is 1(478& mm /g 'ith a p#lse of 117min. /e has no ten"erness o)er the spinal processes or paraspinal areas. /is a$"omen is o$ese $#t there is a s#ggestion of a non!ten"er% p#lsatile mass in the epigastric region. The remain"er of the physical e.amination is normal. =hich of the follo'ing "iagnoses sho#l" $e consi"ere" at this timea6 A$"ominal aortic ane#rysm 3AAA6 $6Ac#te aortic "issection c62a#"a e*#ina syn"rome "6L#m$osacral "isk herniation e6 Pancreatitis :ormal La$s &. A <(!year!ol" man 'ith emphysema presents for a ro#tine check#p. /e reports that his symptoms ha)e remaine" sta$le% 'ith fre*#ent co#ghing an" "yspnea% "espite compliance to his me"ications% 'hich incl#"e oral pre"nisone% aminophylline% inhale" metaproterenol% an" rotating co#rses of anti$iotics. >n physical e.amination% he has a $arrel!chest an" "istant $reath so#n"s in $oth l#ng fiel"s 'ith soft e.piratory 'heeDes. An arterial $loo" gas re)eals an arterial p/ of (.&2% a p2>2 of 4( mm /g% an" a p>2 of & mm /g. =hich of the follo'ing 'o#l" most likely prolong his s#r)i)ala62ontin#o#s home o.ygen therapy $6/igh!"ose oral pre"nisone c6+nfl#enDa )accine "6Pne#mococcal )accine e6Tracheostomy :ormal La$s 4. A <4!year!ol" man complains of e.tremely se)ere% sharp% shooting pain in his face. /e "escri$es the episo"es as $eing 0like a $olt of electricity0 that are $ro#ght a$o#t $y to#ching a specific area% last

a$o#t <4 secon"s% an" occ#r many times "#ring the "ay. :e#rologic e.amination is completely normal% $#t it is note" that part of his face is #nsha)en $eca#se he fears to to#ch that area. Ca"olini#m!enhance" MA+ sho's no a$normalities of the trigeminal ner)e. =hich of the follo'ing is the most appropriate initial treatmenta6 Anticon)#lsants $6Aspirin c6:onsteroi"al anti!inflammatory "r#gs "6?asoconstrictors e6?aso"ilators :ormal La$s .A 2 !year!ol" man presents 'ith chest pain at rest that is not al'ays relate" to e.ercise. /e reports t'o prior episo"es of fainting "#ring e.ercise $#t has other'ise $een healthy. /e reports a similar history in other family mem$ers an" notes that his father s#""enly collapse" an" "ie" at age 4 after playing tennis. /e "enies #se of cocaine or other recreational "r#gs. Physical e.amination re)eals a systolic e9ection m#rm#r that is lo#"est along the left sternal $or"er. Echocar"iography sho's asymmetric septal hypertrophy 'itho#t o$str#ction. =hich of the follo'ing inter)entions 'o#l" $e most likely to "ecrease this patient,s systolic m#rm#ra6 +nhaling amyl nitrate $6Lying "o'n c6Stan"ing #p "6Use of "igo.in e6 ?alsal)a mane#)er :ormal La$s <. A 41!year!ol" man presents to the clinic complaining of a chronic co#gh o)er the past 4 months% 'hich has no' $een accompanie" $y hemoptysis. /e "enies smoking or any past me"ical history. >n physical e.amination% his hea" an" neck e.amination is normal. /is l#ngs ha)e "iff#se $ilateral rales. 2ar"iac e.amination is normal. La$oratory fin"ings re)eal a so"i#m of 142 mE*7L% a potassi#m of 4.& mE*7L% a chlori"e of 114 mE*7L% a $icar$onate of 24 mE*7L% a BU: of &8 mg7"L% an" a creatinine of 2.8 mg7"L. Urinalysis re)eals microscopic hemat#ria

an" 4E protein#ria. =hich of the follo'ing serologic $loo" tests 'o#l" most help confirm the s#specte" "iagnosisa6Anti!glomer#lar $asement mem$rane anti$o"ies $6Anti!mitochon"rial anti$o"ies c6Anti!ne#trophilic anti$o"ies "6Anti!parietal cell anti$o"ies e6 Anti!smooth m#scle anti$o"ies :ormal La$s (. A <2!year!ol" 'oman 'ith a history of alcoholic cirrhosis is $ro#ght to the emergency "epartment $y her neigh$or $eca#se of increasing lethargy. She has contin#e" to a$#se alcohol "espite a history of m#ltiple esophageal )ariceal $lee"s an" prior a"missions for hepatic encephalopathy. /er past me"ical history is also significant for "#o"enal #lcer% hypertension% an" go#t. >n physical e.amination she is lethargic an" easily falls asleep 'hen not $eing stim#late". /er temperat#re is &(.1 2 381.( 56% $loo" press#re is 121712 mm /g% p#lse is 8<7min% an" respirations are 117min. She has "ry m#co#s mem$ranes. A ne#rologic e.amination is nonfocal% $#t asteri.is is present. =hich of the follo'ing me"ications is most likely to $e contri$#ting to this patient,s clinical con"itiona6 2olchicine $6Enalapril c65#rosemi"e "6>mepraDole e6 Metoprolol :ormal La$s 1. A 21!year!ol" man% 'ho is ( ho#rs stat#s!post an appen"ectomy% complains of se)ere lo'er a$"ominal!s#prap#$ic pain. >n e.amination% his incision is clean% "ry% an" intact an" 'itho#t pain to the to#ch. Palpation of his s#prap#$ic area pro"#ces intense "iscomfort. /e has not #rinate" since the proce"#re. The n#rse notes that he is tachycar"ic% 'ith a reg#lar p#lse of 1147min. =hich of the follo'ing is the most appropriate ne.t step in managementa6A"minister a $eta $locker s#ch as metoprolol $6A"minister an antia"renergic agent s#ch as "o.aDosin c6Begin +? anti$iotics

"6Perform a straight catheteriDation of the $la""er e6 Place a nasogastric t#$e 3:CT6 :ormal La$s 8.A &1!year!ol" 'oman 'ith kno'n /+? presents to her physician 'ith a &!month history of 'atery "iarrhea% se)ere 'eakness% an" a 22!po#n" 'eight loss. M#ltiple stool tests for $acteria% o)a% an" parasites are repeate"ly negati)e. A colonoscopy is normal% as are $iopsies of the colon. =hich of the follo'ing is the most likely e.planation for her "iarrheaa6 2ryptospori"iosis $62ytomegalo)ir#s infection c6Entamoe$a histolytica "6Enteroto.igenic Escherichia coli e6 Shigella "ysenteriae :ormal La$s 14. A patient 'ith long history of sin#s infections cons#lts a physician $eca#se one eye is e.tremely sore. Physical e.amination "emonstrates erythema an" e"ema of the eyeli"s an" con9#ncti)a. The eye$all is protr#"ing an" the patient is #na$le to mo)e his eye in some "irections. A""itionally% the patient is r#nning a high fe)er. =hich of the follo'ing is the most likely "iagnosisa6 Bacterial con9#ncti)itis $6Blepharitis c6@acryocystitis "6/or"eol#m e6>r$ital cell#litis :ormal La$s 11. A 4!year!ol" man is a"mitte" to the hospital after s#staining an ac#te myocar"ial infarction. Eight ho#rs after this e)ent% his $loo" press#re is (47 4 mm /g% an" his p#lse is 4 7min. An E2C re)eals sin#s rhythm. =hich of the follo'ing is the most appropriate inter)entiona6 A"minister atropine intra)eno#sly $6A"minister "o$#tamine c6A"minister a $eta $locker

"6+nsert a trans)eno#s pacemaker e6 Perform car"iac catheteriDation :ormal La$s 12.A &8!year!ol" a#tomo$ile mechanic presents $eca#se of the ne' onset of 'heeDing. /e is other'ise healthy an" r#ns appro.imately & miles per "ay. /e "enies any noct#rnal 'heeDing or co#gh. /e has a history of chronic heart$#rn% for 'hich he takes raniti"ine nightly. /e also has a history of hypertension% 'hich has $een "iffic#lt to control% an" o)er the past se)eral months he has $een taking a com$ination of propranolol% enalapril% an" hy"rochlorothiaDi"e. /is only other me"ication is occasional pse#"oephe"rine for symptoms of allergic rhinitis. >n physical e.amination% he appears comforta$le. /is $loo" press#re is 1&4711 mm /g% p#lse is <17min% an" respirations are 117min. >n l#ng e.amination% soft e.piratory 'heeDes are hear" thro#gho#t $oth l#ng fiel"s. =hich of the follo'ing me"ications is most likely contri$#ting to his 'heeDinga6Enalapril $6/y"rochlorothiaDi"e c6Propranolol "6Pse#"oephe"rine e6Aaniti"ine :ormal La$s 1&.A 2(!year!ol" ski instr#ctor complains of a nonpro"#cti)e co#gh for 14 "ays. /e has also "e)elope" a lo'!gra"e fe)er an" "iff#se m#scle aches. /e has $een a$le to 'ork < ho#rs per "ay $#t has $ecome increasingly "yspneic on long ski r#ns. /e has no prior me"ical history an" "oes not smoke or "rink. /e has a temperat#re of &1.& 2 3144.8 56 an" has scattere" $ilateral crackles. /e has a reg#lar heart rhythm. The remain"er of his physical e.amination is normal. A chest .!ray film re)eals a faint $ilateral interstitial infiltrate. =hich of the follo'ing is the most likely "iagnosisa6Legionella pne#monia $6Myco$acteri#m t#$erc#losis c6Mycoplasma pne#monia "6Pne#mocystis carinii pne#monia e6Staphylococcal pne#monia :ormal La$s

14. A <(!year!ol" 'oman presents to a clinic for re)ie' of her me"ications. She has ha" a history of hypertension for 2 years% type 2 "ia$etes for 24 years an" congesti)e heart fail#re for fi)e years. She is a former smoker of 2 packs of cigarettes per "ay an" her lipi" stat#s is not kno'n at this time. /er c#rrent me"ications incl#"e nife"ipine% hy"ralaDine% isosor$i"e "initrate% gly$#ri"e% a m#lti)itamin% an" con9#gate" estrogens. To"ay in the office% her $loo" press#re is 1<4784 mm /g% her fasting $loo" gl#cose is 214 mg7"L an" her hemoglo$in A1c is (.1B. She reports mo"erate "yspnea on e.ertion% #nchange" from pre)io#s )isits. =hich of the follo'ing is the most appropriate inter)ention at this timea6 +ncrease her gly$#ri"e "osage $6+ncrease her nife"ipine "osage c6@iscontin#e her hy"ralaDine therapy "6@iscontin#e the hy"ralaDine an" a"" captopril e6Begin sim)astatin therapy :ormal La$s 1 .A &1!year!ol" man reports 8 months of "iffic#lty s'allo'ing his meals. The symptoms ha)e ca#se" him to lose 'eight% an" he has $een 'aking in the mi""le of the night 'ith rec#rrent co#ghing. /e has ha" "iffic#lty s'allo'ing soli"s as 'ell as li*#i"s since these symptoms $egan. An esophagram re)eals a "ilate" esophag#s 'ith a smoothly tapere" "istal esophag#s. =hich of the follo'ing manometric fin"ings 'o#l" most likely $e fo#n" in this patientPeristalsis in $o"y Aesting lo'er of esophag#s press#re esophageal sphincter 3LES6 A . @ecrease" B. @ecrease" 2. +ncrease" @. @ecrease" E. +ncrease" :ormal +ncrease" @ecrease" @ecrease" +ncrease"

1<. A &2!year!ol" man is comatose in the intensi)e care #nit. The patient s#staine" a massi)e hea" in9#ry in a hea"!on motor )ehicle acci"ent & months ago an" has $een in a )egetati)e state since that "ate. /e is )entilator!"epen"ent an" recei)es n#tritional s#pport )ia a gastrotomy t#$e. The patient makes no p#rposef#l mo)ements an" "oes not 'ith"ra' from painf#l stim#li. There is "iscor" present 'ithin his family a$o#t 'hether to 'ith"ra' care. /is father "esires care to $e 'ith"ra'n% 'hereas his mother 'ants to contin#e ma.imal s#pporti)e care. =hich of the follo'ing is the most important "etermining factor in helping to "irect f#t#re carea6 The "ecision of a co#rt!appointe" legal g#ar"ian $6The "ecision of the me"ical an" n#rsing team taking care of the patient c6The father,s 'ishes% gi)en that he is the health care pro.y "6The opinion of the hospital ethics committee e6 The patient,s pre)io#sly e.presse" 'ishes regar"ing life s#pport% if kno'n :ormal La$s 1(.A 4!year!ol" man presents to his physician complaining of intermittent palpitations. The patient reports that% a fe' times o)er the past fe' months% he has ha" episo"es of 0po#n"ing in his chest0 that are associate" 'ith shortness of $reath an" occasional chest pain. /e is force" to sit "o'n if he is stan"ing $eca#se of 'eakness an" )ertigo. The patient has a history of hypertension an" mitral )al)e prolapse. /e takes nife"ipine an" thiaDi"e "aily. =hile sitting in the office% the patient $egins to complain of increasing shortness of $reath an" palpitations. /is $loo" press#re is 1447 4 mm /g% an" his p#lse is 114! 1&47min an" irreg#larly irreg#lar. =hich of the follo'ing is the most appropriate management at this timea62all 811 for assistance $6 Ci)e the patient an oral "ose of a $eta $locker c6Ci)e the patient an oral "ose of "igo.in "6Ci)e the patient on oral "ose of calci#m channel $locker e6Make arrangements to ha)e the patient $ro#ght to the local emergency "epartment for electrical car"io)ersion :ormal La$s

11.A &4!year!ol" African American 'oman presents complaining of "yspnea on e.ertion an" a nonpro"#cti)e co#gh. She has pre)io#sly $een in goo" health% e.cept for a history of monon#cleosis 'hile in college. She "oes not smoke nor "rink. She has $een monogamo#s 'ith her h#s$an" since they 'ere marrie" 1< years ago. >n physical e.amination% her temperat#re is &1.& 2 3144.8 56% $loo" press#re is 1&2714 mm /g% p#lse is (47min% an" respirations are 227min. There are p#rple areas of s'elling on her nose an" cheeks. The pharyn. is clear. There are scattere" cer)ical lymph no"es an" cl#$$ing of the peripheral fingernail $e"s. Bilateral inspiratory crackles are hear" on a l#ng e.amination. There is a reg#lar S1 an" S2. The li)er span is 1& cm in the mi"!cla)ic#lar line% an" the spleen tip is palpa$le. A chest .!ray film re)eals $ilateral hilar lympha"enopathy. =hich of the follo'ing 'o#l" $e the most appropriate ne.t step in the e)al#ation of this patienta6/+? anti$o"ies $62er)ical lymph no"e $iopsy c6Transthoracic hilar lymph no"e $iopsy "6Li)er $iopsy e6 Splenectomy :ormal La$s 18. A (4!year!ol" 'oman comes to the physician $eca#se of rec#rrent hea" an" neck pain for the past < months. She ha" a hysterectomy 'ith $ilateral oophorectomy for e.tensi)e en"ometriosis at the age of 44. She "oes not take any me"ications. Physical e.amination re)eals re"#ce" range of motion of the neck. =hen the neck is fle.e" to the right% the patient e.periences pain to the right arm. The patient reports occasional transient )ertigino#s sensations on t#rning her hea". =hich of the follo'ing is the most appropriate ne.t step in "iagnosisa6E2C $6F!ray st#"ies of hea" an" neck c62T of the spinal col#mn "6MA+ of spinal col#mn e6Angiographic st#"ies of cere$ral )essels :ormal La$s 24. A 1!year!ol" African American man is $ro#ght to the emergency "epartment $eca#se he co#l" not get o#t of $e" that morning. /e

complains of 2 'eeks of 'orsening pain in his right sho#l"er an" left arm% an" has ha" "iffic#lty #rinating for the past 14 months. /is $loo" press#re is 1&47<1 mm /g% p#lse is (27min% respiratory rate is 1<7min% an" temperat#re is 81.8 5. There are no sensory or motor "eficits in the #pper e.tremities. /is neck has f#ll range of motion. M#scle strength is "iminishe" in $oth lo'er e.tremities. Straight leg raising "oes not elicit pain% $#t temperat#re an" pain sensation are a$sent in the right foot an" are nearly a$sent in the left foot. Proprioception is spare". Plantar an" "eep ten"on refle.es at the knees an" ankles are a$sent. >n rectal e.am% the prostate is large% firm% an" nonten"er. =hich of the follo'ing is the most likely "iagnosisa6Amyotrophic lateral sclerosis $6Metastatic prostate cancer c6Pott "isease "6Sciatica e6Spinal stenosis :ormal La$s 21.A & !year!ol" clergyman 'ith a 18!year history of type 1 "ia$etes is referre" to a gastroenterologist $y his en"ocrinologist $eca#se of symptoms of early satiety% a$"ominal $loating% na#sea% an" rec#rrent episo"es of )omiting after a large meal. A physical e.amination re)eals mil" retinopathy an" t'o heale" #lcers on the )entral s#rface of his left foot. An a$"ominal e.amination is normal. =hich of the follo'ing is the most likely ca#se of his symptomsa6A#tonomic ne#ropathy $6+nappropriate inhi$ition of gastrin c6+ncrease" resting press#re at the lo'er esophageal sphincter "6Aapi" gastric emptying in the antr#m of the stomach e6 Scarring at the lo'er esophageal sphincter :ormal La$s 22.A 4 !year!ol" 'oman has ha" generaliDe" 'eakness an" a 0pins an" nee"les0 feeling for the past & 'eeks. She e.ercises "aily% rarely "rinks alcohol% an" is a strict )egetarian. /er temperat#re is &( 2 381.< 56% $loo" press#re is 1147(4 mm /g% p#lse is <47min% an" respirations are 117min. E.amination sho's 'eakness of the pro.imal an" "istal m#scles of the lo'er e.tremities. There is impaire" proprioception an"

)i$ratory perception. @eep ten"on refle.es are increase". The gait is ata.ic. =hich of the follo'ing is the most likely "iagnosisa6 C#illain!BarrT syn"rome $6Lam$ert!Eaton syn"rome c6Myasthenia gra)is "6Polymyositis e6S#$ac#te com$ine" "egeneration of the spinal cor" :ormal La$s 2&. A 4 !year!ol" 'oman has a history of asymptomatic )entric#lar co#plets% for 'hich she ha" $een place" on *#ini"ine. A 'eek after this inter)ention% she e.periences a syncopal episo"e an" is $ro#ght to the emergency "epartment. /er electrolytes are 'ithin normal limits. She is place" on a car"iac monitor an" is note" to ha)e a transient 'i"e comple. tachycar"ia 'ith 'a)es of alternating morphology. She appears to $e in torsa"es "e pointes. =hich of the follo'ing is the most appropriate ne.t stepa62heck the le)el of ser#m calci#m $6@iscontin#e *#ini"ine an" o$ser)e c6+ncrease the *#ini"ine "ose "6A"minister +? potassi#m e6 Apply &<4 R of "efi$rillating c#rrent :ormal La$s 24. A 8!year!ol" man presents to his primary care physician 'ith fe)er an" chills. /is past me"ical history is significant for osteoarthritis for many years. /e has a long smoking history of greater than 1 4 pack! years. /e ro#tinely takes only a non!steroi"al anti!inflammatory agent for pain. /e presents 'ith "ays of fe)er an" chills associate" 'ith a pro"#cti)e co#gh. /e has not $een hospitaliDe" recently an" li)es at home 'ith his 'ife an" has no sick contacts. >n physical e.amination he is comforta$le% his temperat#re is &1.8 2 3142 56 an" has $i$asilar crackles hear" $est at the left $ase. =hich of the follo'ing is the most appropriate ne.t step in "iagnosisa6Arterial $loo" gas $62hest ra"iograph c62omplete $loo" co#nt "6>.ygen sat#ration check e6Sp#t#m gram stain :ormal La$s

2 . A <4!year!ol" 'oman initiates chemotherapy for metastatic $reast cancer. She "e)elops se)ere na#sea follo'e" $y ho#rs of )omiting "#ring her first chemotherapy inf#sion% 'hich $ecomes 'orse on s#$se*#ent inf#sions. =hich of the follo'ing 'o#l" $e most effecti)e in controlling her )omitinga6Bism#th $6Morphine c6>mepraDole "6>n"ansetron e6Sertraline :ormal La$s 2<. A 8!year!ol" man presents to his physician for follo'!#p care of his hypertension. The patient has a 14!year history of hypertension that has $een treate" 'ith a )ariety of me"ications. +t is no' 'ell controlle" 'ith a $eta $locker an" thiaDi"e "i#retic com$ination. The patient,s mean $loo" press#re o)er the past < months has $een 14 71 mm /g. /is other me"ical history is remarka$le only for type 2 "ia$etes mellit#s. /e c#rrently takes a lo'!"ose oral s#lfonyl#rea agent an" has ha" reasona$le control of his $loo" gl#cose% 'ith )al#es ranging from 114 to 224 mg7"L. /is most recent hemoglo$in A1c )al#e 'as 1.1B. /e has no allergies to any me"ications an" he *#it smoking cigarettes 2 years ago. =hich of the follo'ing is the most appropriate management at this timea6A"" ins#lin an" an A2E inhi$itor to his regimen $6@iscontin#e the s#lfonyl#rea agent an" $egin ins#lin therapy c6+ncrease the "oses of s#lfonyl#rea $eta $locker "6+ncrease the "oses of s#lfonyl#rea thiaDi"e e6+ncrease the "ose of s#lfonyl#rea an" a"" an A2E inhi$itor to his regimen :ormal La$s 2(. A 4 !year!ol" chronic alcoholic presente" to the emergency "epartment years ago 'ith 24 ho#rs of epigastric pain ra"iating to his $ack% na#sea% an" )omiting. /e gra"#ally reco)ere" from this ac#te episo"e. >)er the ne.t years% he is repeate"ly a"mitte" for similar

symptoms. /e then presents 'ith gra"#al onset of 'eight loss% mi"a$"ominal pain ra"iating to his $ack% an" steatorrhea. =hich is of the follo'ing has most likely occ#rre"a62holangiocarcinoma $6E.ocrine ins#fficiency of the pancreas c6Castric o#tlet o$str#ction "6Pancreatic a"enocarcinoma e6Scarring of the entire length of the common $ile "#ct :ormal La$s 21. A 4!year!ol" man comes to the physician $eca#se of 0ringing in his ears0% a feeling of 0spinning0% an" a progressi)e loss of hearing in his left ear. /e says that this all $egan 0a 'hile ago 0'ith a slight feeling of #nstea"iness an" this 0annoying ringing0. /e ne)er came to the "octor $eca#se he tho#ght he 'as 0going craDy0% $#t no' his 'ife is getting 'orrie" that the tele)ision nee"s to $e m#ch lo#"er an" he constantly says 0'hat-0 'hen she speaks to him on his left si"e. /e has no chronic me"ical con"itions% "oes not take any me"ications% an" "oes not "rink alcohol. E.amination sho's nystagm#s% $#t no other a$normalities. =hich of the follo'ing is the most likely "iagnosisa6Benign positional )ertigo $6Benign rec#rrent )ertigo c6Meniere "isease "6To.ic la$yrinthitis e6?esti$#lar ne#ronitis :ormal La$s 28.A (4!year!ol" 'oman 'ith aortic sclerosis is a"mitte" 'ith chest pain. An infarct is r#le" o#t $y car"iac enDymes% $#t the patient has rec#rrent symptoms 'hen 'eane" off heparin. >n hospitaliDation "ay 2% she has right arm pain% a$sent $rachial p#lse on the right% an" a col" "istal right arm. /er hematocrit is &4B% an" platelets are &4%4447mm&. /er partial throm$oplastin time is <4 sec. =hich of the follo'ing is the most likely ca#se of this patient,s a$sent $rachial p#lsea6Em$oliDation from aortic sclerosis $6/eparin!in"#ce" throm$ocytopenia c6Para"o.ical em$ol#s "6/ypercoag#la$le state from immo$iliDation e6 ?asospasm of the $rachial artery

:ormal La$s &4. A <1!year!ol" man 'ith an 14!pack!year smoking history presents 'ith increasing "yspnea on e.ertion. /e also has "e)elope" a pro"#cti)e co#gh% 'hich has persiste" o)er most of the past 2 years. /e "enies any other me"ical pro$lems an" has ne)er $een hospitaliDe" for e)al#ation of his symptoms. >n physical e.amination% his $loo" press#re is 1&27(4 mm /g% p#lse is (47min% an" respirations are &47min. >n l#ng e.amination% there are lo#" e.piratory rhonchi% 'ith a prolonge" e.piratory phase in $oth l#ng fiel"s. There is an increase" anteroposterior chest "iameter. /eart so#n"s are reg#lar $#t "istant. A chest .!ray film re)eals hyperinflation of $oth l#ng fiel"s an" a normal siDe" heart. =hich of the follo'ing 'o#l" most likely $e e.pecte" on p#lmonary f#nction testsa6 @ecrease" force" e.piratory )ol#me in 1 secon" 35E?16 to force" )ital capacity 35?26 ratio $6@ecrease" total l#ng capacity c6@ecrease" resi"#al )ol#me "6+ncrease" 5E?1 e6+ncrease" 5?2 :ormal La$s &1.A (1!year!ol" 'hite man comes to the emergency "epartment $eca#se of $l#rry )ision an" $loo"!tinge" #rine. /e states that he has a long history of hypertension treate" 'ith a $eta $locker% an A2E inhi$itor an" a calci#m channel $locker. /e reports that he has ha" & "ays of $l#rry )ision an" #rine that is 0cola0 colore". The symptoms $egan 'hen he ran o#t of his me"ications & "ays ago an" he has 0not ha" the time0 to get the prescriptions refille". /is physical e.amination is remarka$le for a $loo" press#re of 2447114 mm /g in $oth arms% an" f#n"#scopic e.amination sho'ing flame hemorrhages an" papille"ema. Urinalysis sho's re" $loo" cells an" some "ysmorphic re" $loo" cell casts. /e has an a$"ominal $r#it. =hich of the follo'ing is the most appropriate ne.t step in managementa6>r"er an E2C an" o$ser)e the patient $6Ci)e the patient his #s#al "aily "oses of BP me"ications an" o$ser)e him c6Ci)e the patient intra)eno#s me"ication to re"#ce his BP

"6>r"er an a$"ominal 2T scan to r#le o#t aortic ane#rysm e6Perform a $e"si"e cystoscopy to e)al#ate the hemat#ria :ormal La$s &2.A <4!year!ol" man is $ro#ght to the emergency "epartment $y his 'ife $eca#se of se)ere epigastric "istress an" na#sea that $egan soon after "inner. The patient says that it is pro$a$ly 09#st in"igestion%0 $#t he appears pale% s'eaty% an" restless. /is $loo" press#re is 1&47(4 mm /g% p#lse is 1147min 'ith occasional premat#re $eats% an" respirations are 247min. The l#ngs are clear to a#sc#ltation. /eart so#n"s are slightly softer than normal% $#t there are no m#rm#rs. =hich of the follo'ing is the most appropriate initial "iagnostic stepa62hest .!ray $62omplete $loo" co#nt c6Echocar"iography "6E2C e6Upper en"oscopy :ormal La$s &&. A (4!year!ol" man 'ith a history of hepatitis 2 presents to the emergency "epartment 'ith hematemesis. The patient "e)elope" hepatitis 2 after m#ltiple transf#sions 'ith a coronary artery $ypass graft 1< years earlier. Li)er f#nction test a$normalities 'ere note" 4 years ago% an" a li)er $iopsy 'as consistent 'ith hepatitis 2 'ith "iff#se fi$rosis an" no"#lar regeneration consistent 'ith mo"erately se)ere cirrhosis. /e has not ha" any e)i"ence of portal hypertension. =hen he a'oke this morning% he $egan )omiting large )ol#mes of $right re" $loo". This 'as follo'e" $y rectal $lee"ing of fresh $loo". >n e.amination in the emergency "epartment% his temperat#re is &(.2 381.8 56% $loo" press#re is 8<7 < mm /g% p#lse is 1127min% an" respirations are 2<7min. /e is anicteric an" has "ry m#co#s mem$ranes. A heart an" l#ng e.amination is normal. There is no e)i"ence of hepatosplenomegaly or ascites% an" no peripheral e"ema is present. A ne#rologic e.amination is nonfocal. =hich of the follo'ing is the most appropriate ne.t step in managementa6 E2C $6Placement of t'o large $ore +? catheters c6Upper gastrointestinal en"oscopy "6E)al#ation for emergency li)er transplant

e62olonoscopy :ormal La$s &4.A (!year!ol" 'oman 'ith a history of hypertension is $ro#ght to the emergency "epartment $eca#se of ac#te onset of hea"ache an" loss of $alance for 2 ho#rs. She is conscio#s an" oriente" to person% space% an" time. She feels na#seate" an" cannot 'alk or stan" 'itho#t help. :e#rologic e.amination re)eals paralysis of con9#gate" lateral gaDe to the right si"e. There is no "ecrease in m#scle strength or sensory loss. A 2T scan of the hea" re)eals an intracere$ellar hematoma. =hich of the follo'ing is the most appropriate ne.t step in managementa6 L#m$ar p#nct#re $6MA+ of the hea" c6S#pporti)e me"ical treatment "6Anticoag#lant treatment e6+mme"iate s#rgical e)ac#ation :ormal La$s & .A 4(!year!ol" man presents to the hospital complaining of palpitations. The patient reports that 'hile cooking $reakfast this morning% he felt his heart 0racing in his chest0 an" 'as #na$le to catch his $reath. /e states that sitting "o'n $ro#ght no relief. /e calle" for an am$#lance an" he 'as $ro#ght to the emergency "epartment. The man has no significant past me"ical history an" takes no me"ications reg#larly% other than raniti"ine for occasional heart$#rn. >n e.amination% the patient is *#ite thin% $#t 'ell "e)elope" an" in mil" "istress. /is glo$es appear e.ophthalmic. /is p#lse is 1447min an" irreg#larly irreg#lar. There are no m#rm#rs% an" the l#ng e.amination is clear. A non!ten"er mi"line thyroi" mass is palpa$le. =hich of the follo'ing fin"ings on his echocar"iogram 'o#l" s#ggest a "iagnosis of long!stan"ing atrial fi$rillationa6@ilate" left )entricle $6@ilate" right )entricle c6Enlarge" left atri#m "6/ypertrophie" )entric#lar sept#m e6 Pericar"ial thickening :ormal La$s

&<.A 4!year!ol" 'oman 'ith a long history of emphysema presents $eca#se of an e.acer$ation of her co#gh an" "yspnea. >n physical e.amination% her $loo" press#re is 12<7<4 mm /g% p#lse is 127min% an" respirations are 247min. >n l#ng e.amination% there are lo#" e.piratory 'heeDes an" rhonchi. The car"iac e.amination is normal. An arterial $loo" gas is performe". =hich of the follo'ing res#lts 'o#l" most likely $e e.pecte"a6 p/ of (.24% p2>2 of <4% an" p>2 of 4< $6p/ of (.&4% p2>2 of 4% an" p>2 of 84 c6p/ of (.& % p2>2 of 4 % an" p>2 of <4 "6p/ of (.4<% p2>2 of 2 % an" p>2 of (< e6p/ of (. 2% p2>2 of &4% an" p>2 of 12 :ormal La$s &(.A &4!year!ol" perf#me sales'oman presents complaining of lo'er a$"ominal cramps an" "iarrhea. She has no prior history of gastrointestinal illnesses an" $egan noting fre*#ent loose stools & "ays earlier. The stools ha)e s#$se*#ently $ecome $loo"y an" associate" 'ith #rgency an" noct#rnal $o'el mo)ements. She has also "e)elope" an increase" temperat#re of &1.4 2 3141.1 56. >n e.amination% she has mil" peri#m$ilical an" left lo'er *#a"rant ten"erness. >n rectal e.amination% the stool is $loo"y. =hich of the follo'ing organisms is most likely ca#sing her symptomsa62ampylo$acter 9e9#ni $62ryptospori"i#m c6Ciar"ia lam$lia "6Staphylococc#s a#re#s e6 To.igenic Escherichia coli :ormal La$s &1.A 24!year!ol" pregnant 'oman is e.periencing rapi"ly progressi)e hearing loss. Se)eral relati)es in her family "e)elope" "eafness prior to age 2 . >toscopic e.amination sho's normal tympanic mem$ranes $ilaterally. /earing testing sho's a con"#cti)e hearing loss% 'ith one ear $eing effecte" more se)erely than the other. =hich of the follo'ing "iagnoses is most likelya6Aco#stic ne#roma

$62hronic otitis me"ia c6>tosclerosis "6Meniere "isease e6 Pres$yc#sis :ormal La$s &8.A 2(!year!ol" pharmacist presents for her ann#al physical e.amination. She "escri$es that o)er the past se)eral months she has a'oken from sleep on se)eral occasions 'ith co#gh% 'heeDing% an" shortness of $reath% $#t "enies ha)ing these symptoms "#ring the "ay. The symptoms generally last for #p to 2 ho#rs $efore gra"#ally s#$si"ing. She has no pre)io#s history of asthma an" e.ercises reg#larly 'itho#t "iffic#lty. She is on no me"ications% e.cept for the fre*#ent #se of o)er!the!co#nter histamine!2 receptor antagonists for "aily episo"es of heart$#rn. She is afe$rile an" has normal )ital signs. /er l#ngs are clear to a#sc#ltation an" perc#ssion. =hich of the follo'ing is the most likely e.planation for her noct#rnal symptomsa6Aci" refl#. $6Laryngospasm c6Mast cell release "6M#c#s pl#gs e6 Upper air'ay o$str#ction :ormal La$s 44. A 22!year!ol" college st#"ent is $ro#ght to the emergency "epartment after $eing fo#n" #naro#sa$le $y his roommates in the morning. /e ha" complaine" of a se)ere hea"ache the night $efore. /is temperat#re is &8. 2 314& 56. Physical e.amination re)eals n#chal rigi"ity an" petechiae o)er $oth legs. 2hest .!ray films are #nremarka$le. After f#n"#scopic e.amination% a l#m$ar p#nct#re is performe". The 2S5 appears clo#"y. La$oratory st#"ies on a 2S5 sample sho'; 2ells....................................14447mm& mostly ne#trophils Protein.................................<.4 g7L Cl#cose................................4.& g7L Plasma gl#cose72S5 gl#cose Q 4.4 >n microscopic e.amination% no $acterial or f#ngal organisms are "etecte" in the 2S5. =hich of the follo'ing is the most likely pathogen-

a6 Escherichia coli $6Cro#p B streptococci c6/emophil#s infl#enDae "6Listeria monocytogenes e6 Meningococc#s f6Pne#mococc#s :ormal La$s 41.A (4!year!ol" man is fo#n" #nresponsi)e at home. The emergency me"ical technicians note his $loo" press#re to $e (4 mm /g $y palpation. /is p#lse is 1247min. /e is $ro#ght to the emergency "epartment% 'here his temperat#re is &8. 2 314& 56% an" respirations are &47min. /e has rales half'ay #p his chest. /eart so#n"s are ina#"i$le. /is #rine o#tp#t is 14 mL7hr. A chest .!ray film re)eals Merley B lines% an" an E2C sho's sin#s tachycar"ia. /e is gi)en anti$iotics an" is taken to the intensi)e care #nit% 'here a right!si"e" catheteriDation sho's an ele)ate" 'e"ge press#re an" "iminishe" car"iac o#tp#t. /is right atrial press#re is not ele)ate". =hich of the follo'ing is the most likely ca#se of this man,s hypotensiona6Castrointestinal $lee"ing $6Cram!negati)e sepsis c6Left )entric#lar "ysf#nction "6Pericar"ial tampona"e e6P#lmonary em$ol#s :ormal La$s 42.An 14!year!ol" 'oman presents 'ith "yspnea on e.ertion. >n physical e.amination% her $loo" press#re is 1447(4 mm /g% an" her p#lse is ( . She has no p#ls#s para"o.#s. /er 9#g#lar )eins are "isten"e"% an" she has "istant heart so#n"s. +n a""ition% she has e.tra thir" an" fo#rth heart so#n"s. /er li)er is enlarge"% an" she has pe"al e"ema. She has occasional premat#re )entric#lar contractions on her E2C. A chest .!ray re)eals clear l#ng fiel"s 'ith a "ilate" car"iac silho#ette. /er echocar"iogram re)eals )entric#lar 'alls 'ith a 0speckle" pattern. 0 =hich of the follo'ing is the most likely "iagnosisa6Alcoholic car"iomyopathy $6Amyloi"osis

c6/emochromatosis "6T#$erc#losis e6?iral myocar"itis :ormal La$s 4&.A 24!year!ol" man presents for e)al#ation of a 4!month history of postpran"ial "iarrhea% 'eight loss of 8 po#n"s% an" lo'er a$"ominal pain. /e "enies recent tra)el or anti$iotic #se. >n physical e.amination% his temperat#re is &1.4 2 3144.4 6% an" he has se)eral oral aphtho#s #lcers. >n a$"ominal e.amination% there is ten"erness an" mil" )ol#ntary g#ar"ing in the right lo'er *#a"rant. A rectal e.amination re)eals $ro'n stool that is strongly g#aiac positi)e. =hich of the follo'ing is most likely ca#sing this patient,s symptomsa6A gram!negati)e organism $65olate "eficiency c6M#cosal #lceration 'ith no transm#ral in)ol)ement in the ascen"ing colon "6To.in!pro"#cing organism e6 Transm#ral inflammation in the region of the terminal ile#m :ormal La$s 44. A 8!year!ol" 'oman comes to the physician $eca#se of rec#rrent episo"es of lighthea"e"ness #pon getting #p in the morning an" occasionally #pon stan"ing #p from a chair. She reports that on t'o occasions she has 0passe" o#t0 soon after getting #p from $e". /er temperat#re is &( 2 381.< 56% $loo" press#re is 1&4714 mm /g% p#lse is (47min an" reg#lar% an" respirations are 147min. She takes a $eta! $locker an" a thiaDi"e "i#retic for mo"erate hypertension "iagnose" si. months ago. Ceneral e.amination an" chest a#sc#ltation are #nremarka$le. =hich of the follo'ing is the most likely ca#se of this patient,s symptomsa6Antihypertensi)e treatment $6Aortic stenosis c6@rop attacks "6Aefle. syncope e6Transient ischemic attacks f6?aso)agal syncope

:ormal La$s 4 .A <&!year!ol" 'oman is a"mitte" to the hospital after an ac#te inferior myocar"ial infarction. She is note" to $e olig#ric% an" has a $loo" press#re of 147 mm /g. A S'an!CanD catheter is place"% re)ealing a "iminishe" p#lmonary capillary 'e"ge of 4 mm /g% normal p#lmonary artery press#re of 2274 mm /g% an" an increase" mean right atrial press#re of 11 mm /g. =hich of the follo'ing is the most appropriate ne.t step in managementa6 Balloon angioplasty $6@igo.in c65l#i"s "6+ntraaortic $alloon co#nterp#lsation e6?asopressors :ormal La$s 4<. A 8!year!ol" man% 'ho is sche"#le" for an a$"ominal aortic ane#rysm 3AAA6 repair in & 'eeks% presents to the physician,s office. The patient,s AAA 'as "iagnose" last 'eek )ia #ltraso#n". +maging at that time re)eale" a . !cm ane#rysm of his a$"ominal aorta e.ten"ing $ilaterally into his iliac arteries. The patient also has mo"erate hypertension% 'ith a mean "aily $loo" press#re of 1 478 mm /g. >n physical e.amination% the patient appears in no "istress. /e 'eighs 2(4 po#n"s an" is feet 8 inches tall. /is l#ngs are clear% an" he has a lo#" S4. /is AAA is palpa$le as a p#lsatile mass in his a$"omen. =hich of the follo'ing is the most appropriate inter)ention to prepare this patient for s#rgerya6Arrange long!term physical therapy $6+mpro)e $loo" press#re control c6+nitiate a 'eight loss program "6Prescri$e a nonsteroi"al anti!inflammatory "r#g 3:SA+@6 e6Prescri$e a regimen of reg#lar aero$ic e.ercise :ormal La$s 4(.A (2!year!ol" retire" attorney presents for his ann#al physical e.amination. /e has not ha" any me"ical illnesses o)er the past year. /e formerly "rank se)eral cocktails $efore "inner for many years an"

has a history of a )ariceal $lee" 2 years ago% $#t has not re!$le" since. /is only me"ications are aspirin an" metoprolol. >n physical e.amination% he is afe$rile. /is $loo" press#re is 1227<1 mm /g% p#lse is <47min% an" respirations are 147min. /e is anicteric an" has no stigmata of chronic li)er "isease. /is li)er e"ge is 8 cm in the mi"cla)ic#lar line $y perc#ssion% an" no spleen tip is palpa$le. There is no e)i"ence of ascites or peripheral e"ema. =hich of the follo'ing me"ications sho#l" the patient most likely a)oi"a62iproflo.acin $6+$#profen c6LoraDepam "6Losartan e6 Pre"nisone :ormal La$s 41. The 2T scan sho's the fin"ings in a 21!year!ol" man 'ho s#staine" a close" hea" in9#ry in a motor )ehicle acci"ent. The hyper"ense areas anterior to the temporal poles posterior to the left occipital lo$e represent ac#te hematomas. This patient is at risk of "e)eloping 'hich of the follo'ing forms of cere$ral herniationa6 2ere$ellar tonsillar herniation $6S#$falcine 3cing#late6 herniation c6Ae)erse cere$ellar herniation "6Transcal)arial herniation e6 Transtentorial 3#ncal6 herniation :ormal La$s 48.A <1!year!ol" 'oman presents for her first physical e.amination in more than 14 years. She reports that she has $een in e.cellent health% "oes not smoke or "rink% an" e.ercises $y r#nning & miles "aily. She is a retire" acco#ntant an" has three healthy gro'n chil"ren. She has $een taking estrogen replacement therapy since menopa#se% an" has $een taking 11 mg "aily after rea"ing a$o#t its car"ioprotecti)e effects in the ne'spaper. >n physical e.amination% she appears 'ell. /er $loo" press#re is 1227(< mm /g% p#lse is (47min% an" respirations are 147min. She is afe$rile. A hea" an" neck e.amination is normal. There is no

9#g#lo)eno#s "istention. /er l#ngs are clear. >n car"iac e.amination% she has a reg#lar S1 an" S2% an" a +7+? crescen"o $lo'ing "iastolic m#rm#r is hear" at the aortic area. An a$"ominal e.amination is normal. A rectal e.amination note" no masses% an" her stool is $ro'n an" g#aiac negati)e. =hich of the follo'ing is the most likely e.planation of the car"iac fin"ings on physical e.aminationa6Prior Bacteroi"es infection $6Prior Escherichia coli infection c6Prior gonococcal infection "6Prior /aemophil#s "#creyi infection e6Prior streptococcal infection :ormal La$s 4. A ((!year!ol" 'oman presents 'ith 12 ho#rs of passing $right re" $loo" from her rect#m in increasingly large amo#nts. She $ecomes "iDDy on stan"ing an" appears pale. /er $loo" press#re is 1127<4 mm /g 'hile stan"ing an" 847 < mm /g on sitting #pright 'ith her legs "angling o)er the si"e of the stretcher. /er a$"ominal e.amination is normal. =hich of the follo'ing is the most likely ca#se of her $lee"inga6A $lee"ing "i)ertic#l#m in the appen"i. $6A comm#nication $et'een an arteriole an" )en#le in the cec#m c6A laceration at the gastroesophageal 9#nction "6Perforation of a sigmoi" "i)ertic#l#m e6 A rectal polyp :ormal La$s

Block 1 E.planations

:) (*!lanation: 7he correct answer is . 7his !atient has the multisystemic fin"in$s consistent with ,e$ionnaires "isease. 7hese inclu"e the CN) fin"in$s of hea"ache an" confusion, !ulmonary fin"in$s consistent with bilateral interstitial infiltrate, an" the nons!ecific $astrointestinal sym!toms of nausea an" "iarrhea. 2e !resumably acquire" the infection while at the conference, where there were !robably infecte" water "ro!lets in the ventilation system. 7here are no risk factors for 2I1 "escribe", so cytome$alovirus !neumonia 'choice A) or &neumocystis carinii !neumonia 'choice %) woul" be unlikely. 7he multisystemic sym!toms with an acute onset, in association with bilateral interstitial infiltrates, are not ty!ical of the !resentation of tuberculosis 'choice C). )ta!hylococcal !neumonia 'choice () may cause severe to*icity an" si$nificant !ulmonary im!airment but woul" not e*!lain the CN) fin"in$s an" $astrointestinal sym!toms.

;) (*!lanation: 7he correct answer is A. It is im!erative to reco$ni#e the !otential !resence of an ab"ominal aortic aneurysm 'AAA). 7he combination of the history of hy!ertension an" smokin$, the new back !ain, an" a !ulsatile mass on e*amination is hi$hly su$$estive for ab"ominal aneurysm. 7he back !ain occurs as the e*!an"in$ mass com!resses structures in the retro!eritoneum. It is !articularly im!ortant to make the "ia$nosis because lar$e aneurysms '$reater than / cm in "iameter) are associate" with a very hi$h risk of ru!ture an" subsequent mortality. Acute aortic "issection 'choice ) is the most common aortic catastro!he requirin$ a"mission to the hos!ital, resultin$ from an e*tension of an intimal tear in the wall of the artery. 7he hallmark of acute "issection !ain is su""en onset, severe chest an" ab"ominal !ain that often ra"iates from an anterior to !osterior "irection. 7he cau"a equina syn"rome 'choice C) is also im!ortant to reco$ni#e, as it is a neurolo$ical emer$ency. 2owever, it is "ue to com!ression of the lower lumbar an" sacral nerve roots, an" !ro"uces sensory loss in a sa""le "istribution, "ecrease" refle*es, urinary incontinence, an" flacci" an" weak le$s, none of which are !resent in this case.

While lumbosacral "isk herniation 'choice %) is one of the commonest etiolo$ies of low back !ain, the !ain is usually "escribe" as shar! or lancinatin$, with ra"iation from the back "own to the le$s. A!!ro*imately I>= of "isk herniations res!on" to conservative me"ical mana$ement. &ancreatitis 'choice () may also cause "ull !ain ra"iatin$ to the back, but the !ain is usually e*cruciatin$. 7he fin"in$s in this case that makes !ancreatitis less likely are the non4ten"er ab"omen an" lack of any constitutional sym!toms. &atients with !ancreatitis a!!ear acutely ill. 0) (*!lanation: 7he correct answer is A. 7his !atient has restin$ hy!o*ia an" therefore woul" benefit in terms of survival with the use of continuous, low4flow home o*y$en thera!y. Care must be taken as to not "eliver hi$h concentrations of o*y$en, as that woul" su!!ress his res!iratory "rive. O*y$en saturation in these !atients shoul" be maintaine" <ust above I>=. Continuous home o*y$en thera!y in this !atient will not only im!rove sym!toms, but also !rolon$ survival. 2i$h4"ose oral !re"nisone 'choice ) may im!rove sym!toms in the short4term "urin$ an acute e*acerbation, but it has no role in the lon$4term mana$ement of !atients with chronic obstructive !ulmonary "isease 'CO&%) an" it "oes not !rolon$ survival. Choices C an" % are a!!ro!riate measures to take in every CO&% !atient to !revent !neumococcal an" influen#a !neumonias res!ectively, but they "o not !rolon$ survival. A tracheostomy 'choice () !lays no role in the mana$ement of CO&%, unless the !atient is in an intensive care unit settin$ an" cannot be weane" from a res!irator. ?) (*!lanation: 7he correct answer is A. 7he clinical "escri!tion is that of tri$eminal neural$ia 'Htic "ouloureu*H), which is treate" with anticonvulsants. Carbama#e!ine is usually the first choice, but !henytoin has also been use". Anti"e!ressants, such as amitri!tyline, have also been trie". )ur$ical "ecom!ression of the nerve or stereotactic ablation are use" in recalcitrant cases. As!irin 'choice ), nonsteroi"al anti4inflammatory "ru$s 'choice C), vasoconstrictors 'choice %), an" vaso"ilators 'choice () "o not have a role in the treatment of tri$eminal neural$ia.

/) (*!lanation: 7he correct answer is . 7his !atient has hy!ertro!hic obstructive car"iomyo!athy. 7his is a car"iac con"ition in which the interventricular se!tum hy!ertro!hies e*cessively. 7he hy!ertro!hie" se!tum an" the anterior leaflet of the mitral valve !ro"uce left ventricular outflow obstruction. +ost cases are inherite" throu$h an autosomal "ominant mo"e of transmission. 7he systolic e<ection murmur is "iminishe" when the !atient lies "own. 7his increases car"iac si#e by increasin$ venous return an" ten"s to "iminish the intensity of the murmur. Afterloa" is increase", an" venous return is increase" to the heart. 7his increases the ventricular si#e an" "iminishes the murmur. Inhalin$ amyl nitrate 'choice A) "iminishes the ventricular si#e an" increases the intensity of the murmur. )tan"in$ u! 'choice C) "ecreases venous return to the heart, a$ain effectively re"ucin$ car"iac si#e, an" lea"s to effective obstruction of the outflow tract, thus increasin$ the murmur. %i$o*in 'choice %) increases contractility, which ten"s to worsen the outflow obstruction an" lea"s to an increase in the murmur. +aneuvers that "iminish left ventricular si#e, such as the 1alsalva maneuver 'choice (), brin$ the anterior leaflet of the mitral valve closer to the se!tum an" increase the amount of obstruction. 7his increases the intensity of the murmur. C) (*!lanation: 7he correct answer is A. 7he combination of hematuria an" hemo!tysis shoul" always raise the !ossibility of .oo"!asture syn"rome. Anti4$lomerular basement membrane antibo"ies are !atho$nomonic for this "ia$nosis. Anti4mitochon"rial antibo"ies 'choice ) are foun" in !atients with !rimary biliary cirrhosis. 7he anti4neutro!hilic cyto!lasmic antibo"ies 'choice C) are foun" in !atients with We$ener $ranulomatosis. We$ener $ranulomatosis may also !resent with !ulmonary an" renal involvement but will have associate" u!!er res!iratory tract fin"in$s, e.$, sinusitis an" sinus abscesses. Anti4!arietal cell antibo"ies 'choice %) are foun" in !atients with the autoimmune "isease known as !ernicious anemia.

Anti4smooth muscle antibo"ies 'choice () are foun" in !atients with autoimmune he!atitis. D) (*!lanation: 7he correct answer is C. &atients with known !ortal hy!ertension have multi!le risk factors for he!atic ence!halo!athy, inclu"in$ "ehy"ration, infection, electrolyte abnormalities 'hy!okalemia an" metabolic alkalosis), se"ative a"ministration, an" $astrointestinal blee"in$. +any of these !atients are on "iuretics, such as furosemi"e, to control ascites relate" to their liver "isease an" hy!oalbuminemia. 7he resultin$ "ehy"ration from e*cessive "iuretic use may !reci!itate an e!iso"e or e*acerbate un"erlyin$ he!atic ence!halo!athy. 7he uricosuric a$ent colchicine 'choice A) is use" for $out. It can cause $astrointestinal sym!toms, marrow "e!ression, an" !eri!heral neuritis, but woul" not be e*!ecte" to cause a chan$e in mental status or !reci!itate he!atic ence!halo!athy. 7he AC( inhibitor enala!ril 'choice ) is use" to control hy!ertension. It can cause an$ioe"ema, ana!hyla*is, hy!otension, neutro!enia, an" fetal morbi"ity. It very rarely, as an i"iosyncratic reaction, will cause a 'new) fulminant he!atic necrosis, but woul" not be e*!ecte" to be a s!ecific !roblem in a !atient with un"erlyin$ cirrhosis. In any event, the "iuretic furosemi"e is a much more common cause of worsenin$ he!atic ence!halo!athy. 7he beta blocker meto!rolol 'choice %) is use" to control either systemic hy!ertension or !ortal hy!ertension. It can cause "e!ression, but only rarely causes mental confusion an" woul" not be e*!ecte" to !reci!itate he!atic ence!halo!athy. 7he !roton !um! inhibitor ome!ra#ole 'choice () is use" for !e!tic ulcer "isease an" is $enerally well tolerate". It woul" not be e*!ecte" to cause a chan$e in mental status or !reci!itate he!atic ence!halo!athy. @) (*!lanation: 7he correct answer is %. &ost4sur$ical !atients often have urinary retention as !art of their normal, imme"iate !osto!erative courseG therefore, in"wellin$ catheters are often !lace". -or a youn$ !atient such as this, however, these catheters are not always !lace". 7he reasons for this are variable, but mostly revolve aroun" the e*!ectation of a s!ee"y an" uneventful !osto!erative course an" "ischar$e home. In !atients without in"wellin$ catheters, urinary retention an" associate" !ain are frequent concerns. 7he !atientAs tachycar"ia is most likely a sinus tachycar"ia relate" to severe !ain. 7he favore" treatment for sinus tachycar"ia is reversal of the un"erlyin$ cause. 7herefore, the a"ministration of a beta4blockers 'choice A) has no role in this settin$.

A"ministration of antia"rener$ic a$ents 'choice ) !lays a role in chronic urinary retention for beni$n !rostatic hy!ertro!hy, which this !atient may have. 7here is no role for such an a$ent in this case, however, where ra!i" "ecom!ression of the bla""er is in"icate". e$innin$ I1 antibiotics 'choice C) woul" be a!!ro!riate if acute infection, such as !yelone!hritis or "iverticulitis, were sus!ecte". As "etaile" in the e*!lanation of the above question, there is no evi"ence for acute infection in this case. &lacin$ a naso$astric tube 'N.7) 'choice () has a role in the treatment of intestinal obstruction an" !ancreatitis, not urinary obstruction. I) (*!lanation: 7he correct answer is A. 7his !atient has a ty!ical small bowel4ty!e "iarrhea, seen in 2I1 with watery "iarrhea, weakness, an" wei$ht loss. 7he most common etiolo$y for this syn"rome is cry!tos!ori"iosis infection of the small intestine, where the s!ores can be seen on the ti!s of the villi on bio!sy. 7his or$anism can be "emonstrate" with s!ecial culture me"ia. Other or$anisms in the same family, such as micros!ori"ia an" Isos!ora belli, !ro"uce i"entical syn"romes. Cytome$alovirus 'choice ) can cause $astrointestinal "isease in AI%) !atients, but is more likely to involve the eso!ha$us, stomach, or colon. )ym!toms of C+1 involvement of the $astrointestinal tract in these !atients inclu"e e!i$astric !ain, nausea an" recurrent vomitin$, "iarrhea, an" u!!er an" lower $astrointestinal blee"in$. (ntamoeba histolytica'choice C) "oes not !ro"uce a small intestinal "iarrheal syn"rome, as in this !atient. It more ty!ically causes an ulcerative ileocolitis. It can also cause he!atic abscesses. 7here is no history in this !atient to su$$est an amebic infection, such as travel to en"emic areas. (nteroto*i$enic (scherichia coli'choice %) !ro"uces a watery illness in !atients with a history of recent travel. It is usually short4live" an" infrequently requires antibiotics. )hi$ella "ysenteriae'choice () !ro"uces a colitis4ty!e "iarrhea, with bloo"y stools in association with lower ab"ominal cram!in$ an" tenesmus. :>) (*!lanation: 7he correct answer is

(. 7his is orbital cellulitis, which is a "an$erous infection of the orbital tissues. Orbital cellulitis can be seen as a com!lication of !aranasal sinusitis, eyeli" trauma, or "entalBoral infections. 7he !resentation "escribe" in the question stem is ty!ical. +a<or com!lications inclu"e !otentially !ermanent visual loss secon"ary to o!tic neuritisG menin$itis secon"ary to s!rea" of the infection to the brainG an" cavernous sinus thrombosis secon"ary to e*tension of clots in orbital veins. 7reatment is with antibiotics in either oral or intravenous form, "e!en"in$ u!on the severity of the infection. )ur$ical "raina$e can be hel!ful if su!!uration is !resent. acteria con<unctivitis 'choice A) can be a com!onent of orbital cellulitis, but "oes not by itself "oes not !ro"uce e*o!hthalmos 'eyeball !rotrusion), limitation of eye movement, or e*tremely severe orbital !ain. le!haritis 'choice ) is an inflammation of the li" mar$ins, an" can be a com!onent of orbital cellulitis, but "oes not by itself !ro"uce the other orbital si$ns an" sym!toms seen in this !atient. %acryocystitis 'choice C) is inflammation of the tear "uct, locate" at the nasal corner of the eye. 2or"eolum 'choice %) is a locali#e" infection of an eyelash follicle or small $lan" in the eyeli". ::) (*!lanation: 7he correct answer is A. 7he !atient is hy!otensive an" bra"ycar"ic. 7his su$$ests a va$al res!onse, an" a"ministerin$ an anticholiner$ic a$ent, such as atro!ine, is the correct treatment. Inotro!ic a$ents, such as "obutamine 'choice ), are not in"icate" at this time, since the !atient is hy!otensive. If other metho"s of resuscitation, such as I1 hy"ration, fail, then a !ressor is in"icate". A beta blocker, such as meto!rolol 'choice C), is in"icate" in the settin$ of a myocar"ial infarction, $iven its car"io!rotective effect. In this acute settin$, however, its effect on the !ulse an" bloo" !ressure will be counter4!ro"uctive. If the bra"yarrhythmia an" hy!otension !ersist after the a"ministration of atro!ine, the insertion of a tem!orary !acemaker is in"icate" 'choice %). If the !atient "evelo!s a sick sinus syn"rome as a result of the infarct, an" is sym!tomatic in terms of hy!otension an" synco!e, then a !acemaker may be nee"e". Car"iac catheteri#ation is in"icate" in the acute settin$ 'choice (). In this !atient, @ hours have ela!se", an" his )7 se$ments have resolve". If he "evelo!s another )7 se$ment elevation myocar"ial infarction, then he will nee" an emer$ent catheteri#ation.

:;) A. (nala!ril . 2y"rochlorothia#i"e C. &ro!ranolol %. &seu"oe!he"rine (. 6aniti"ine (*!lanation: 7he correct answer is C. &ro!ranolol, like other nons!ecific beta blockers, may cause bronchos!asm by blockin$ the beta rece!tors in the bronchial tree. eta stimulation in the lun$s !ro"uces broncho"ilation, an" its blocka"e lea"s to bronchoconstriction. In fact, !ro!ranolol is contrain"icate" in !atients with known asthma or chronic obstructive !ulmonary "isease 'CO&%). (nala!ril 'choice A) may cause !ulmonary sym!toms, in that it may cause a non!ro"uctive cou$h, but is not usually associate" with whee#in$. 2y"rochlorothia#i"e 'choice ) an" raniti"ine 'choice () have no effect on airway res!onsiveness, althou$h the !atientAs un"erlyin$ $astroeso!ha$eal reflu* "isease may lea" to aci" in"uce" bronchoconstriction. &seu"oe!he"rine 'choice %) "oes not cause bronchoconstriction. :0) (*!lanation: 7he correct answer is C. 7his healthy youn$ man has a community4acquire" Haty!ical !neumonia.H It is aty!ical in its $ra"ual onset, absence of hi$her fevers an" ri$ors, an" non!ro"uctive cou$h. 7he results of the lun$ e*amination an" the chest *4ray film reveal the ty!ical fin"in$s of a bilateral interstitial infiltrate. 7his clinical !attern can be cause" by +yco!lasma !neumoniae or alternatively by viral a$ents. Althou$h the +yco!lasma can be culture", this takes D4:> "ays an" requires s!ecial culture techniques that are not commonly available in hos!ital laboratories. .ramAs stain of s!utum usually shows s!arse bacteria, clum!s of "esquamate" res!iratory e!ithelial cells, an" a mi*ture of neutro!hils an" mononuclear cells 'most hel!ful in !ractice in "istin$uishin$ from the more usual bacterial a$ents that ten" to have a "ense neutro!hilic infiltrate). A sin$le col" a$$lutinin reaction with titer $reater than ::C? is a hel!ful 'but a little nons!ecific) confirmatory testG sequential s!ecimens with a fourfol" increase in titer are also consi"ere" confirmatory. Common antibacterial a$ents use" to treat +. !neumoniae inclu"e tetracycline 'not use" for chil"ren) an" erythromycinG clarithromycin an" a#ithromycin are also effective.

,e$ionella !neumonia 'choice A) $enerally !resents with more severe sym!toms, as well as with si$ns of systemic to*icity, hea"ache, nausea, vomitin$, he!atitis, an" "iarrhea. None of these are !resent in this !atient, an" there is no known e*!osure to ,e$ionnaires !neumonia "escribe" here. 7uberculosis 'choice ) $enerally "oes not !resent as a bilateral interstitial infiltrate, an" there are no s!ecific risk factors for tuberculosis "escribe" here. 7he !atient has no "escribe" risk factors for 2I1, so &neumocystis carinii !neumonia 'choice %) is unlikely. )ta!hylococcal !neumonia 'choice () !resents in in"ivi"uals at risk for an invasive )ta!hylococcus aureus infection, e.$., I1 "ru$ users, ma<or trauma victims, or !atients with in"wellin$ I1 catheters. 2e has none of these risk factors. :?) (*!lanation: 7he correct answer is %. 7his !atient has !oorly controlle" hy!ertension an" !oorly controlle" "iabetes. )he nee"s im!rove" thera!y for both, the issue is, how best to "o that. 7he conce!t un"erlyin$ this question is the absolute im!ortance of be$innin$ AC( inhibitor thera!y on both 7y!e : an" 7y!e ; "iabetics. +any clinical trials have shown the beneficial effects of AC( inhibitors on !reventin$ ne!hro!athy an" slowin$ the !ro$ression of establishe" ne!hro!athy in "iabetics. It is the stan"ar" of care that all "iabetics be $iven an AC( inhibitor if they are able to tolerate its bloo" !ressure effects. .iven that she has con$estive heart failure an" hy!ertension, the AC( inhibitor also will be efficacious in their treatment as well. In fact, AC( inhibitors have been shown to be su!erior to hy"rala#ine an" isosorbi"e "initrate in terms of morbi"ity an" mortality in treatment of C2-G all !atients with sym!tomatic C2-, re$ar"less of e<ection fraction, shoul" be !lace" on one. Increasin$ her $lyburi"e "osa$e 'choice A) may be a!!ro!riate $iven her $lucose level an" hemo$lobin A:c, but not as im!ortant as be$innin$ thera!y with an AC( inhibitor. Increasin$ her nife"i!ine "osa$e 'choice ) mi$ht hel! control her bloo" !ressure, but fails to a""ress the su!eriority of a sin$le a$ent, an AC( inhibitor for both bloo" !ressure control an" !revention of "iabetic ne!hro!athy. %iscontinuin$ her hy"rala#ine thera!y 'choice C) is incorrect because her & is alrea"y !oorly controlle" an" this woul" not assist in im!rove" control. e$innin$ simvastatin thera!y 'choice () is not a!!ro!riate until her li!i" status in known. 7hese "ru$s have been shown to be beneficial for both !rimary an" secon"ary !revention of myocar"ial infarction, but shoul" not be !rescribe" in the absence of li!i" "ata.

:/) (*!lanation: 7he correct answer is . 7his !atient has achalasia, which is a neuro$enic eso!ha$eal "isor"er thou$ht to be relate" to a malfunction of the myenteric !le*us of the eso!ha$us. 7he result is that the eso!ha$us behaves as if it has lost the normal !eristaltic mechanism. 7here is also an accom!anyin$ failure to rela* the lower eso!ha$eal s!hincter ',()) when foo" reaches the "istal eso!ha$us. ecause of these !roblems, !atients e*!erience "ifficulty with swallowin$ both soli"s an" liqui"s. 7his is in contrast to masses of the eso!ha$us that cause lumenal narrowin$, in which the swallowin$ of liqui"s is at least initially relatively !reserve". +anometry of !atients with achalasia ty!ically shows "ecrease" !eristalsis in the bo"y of the eso!ha$us with re"uce" restin$ ,() !ressure. 7hese !atients often have nocturnal cou$hin$ because of as!iration of retaine" foo" contents within the eso!ha$us. Choices A an" C are not ty!ical of any con"ition you nee" to remember. Choice % is ty!ical of severe eso!ha$eal "isease in sclero"erma. Choice ( is ty!ical of sym!tomatic "iffuse eso!ha$eal s!asm. :C) (*!lanation: 7he correct answer is (. In these "ifficult situations, in which the !atient is not able to make his or her own "ecisions an" there are "is!arate views amon$ family members, the most im!ortant "irectin$ factor shoul" be what the !atient woul" have "eci"e" were he able to s!eak for himself. 7herefore, !reviously e*!resse" views on what the !atient woul" want if face" with a similar situation coul" !rove invaluable in hel!in$ to "irect care in the "irection that the !atient woul" have wante". Court4a!!ointe" le$al $uar"ians 'choice A) are use" when a !atient has no family or has family members who are not com!etent to hel! make me"ical "ecisions on behalf of the inca!acitate" !atient. If the !atient has ma"e his or her wishes known in any !rior circumstance 'in writin$ or verbally), any sentiments by the me"ical team 'choice ), the !arent 'choice C), or a hos!ital ethics committee 'choice %) woul" be minimi#e". :D) (*!lanation: 7he correct answer is

A. 7his !atient now has atrial fibrillation 'A-) with ra!i" ventricular res!onse '616) an" is consequently hy!otensive. 7his is a me"ical emer$ency. (ven the !hysician carin$ for this !atient is ill equi!!e" to "eal with a !otentially life4threatenin$ e!iso"e of A- with 616. Activatin$ the system is always a!!ro!riate an" ensures that traine" !ersons with a""itional equi!ment an" me"ications a!!ro!riate to an emer$ency situation will be on han" as soon as !ossible. .ivin$ the !atient an oral "ose of a beta blocker 'choice ), "i$o*in 'choice C) or a calcium channel blocker 'choice %) is not a!!ro!riate for an emer$ent settin$. 7hese me"ications will take too lon$ to be effective. +akin$ arran$ements to have the !atient brou$ht to the local emer$ency "e!artment for electrical car"ioversion 'choice () is a!!ro!riate ON,9 A-7(6 I:: has been calle" in the case that the !atient nee"s I++(%IA7( assistance in the office. Without callin$ I::, the !atient may have a seriously a"verse event en route to the hos!ital. :@) (*!lanation: 7he correct answer is . An African American woman with bilateral hilar a"eno!athy shoul" always raise the sus!icion of sarcoi"osis. )he has the facial fin"in$s of lu!us !ernio, an" lym!ha"eno!athy an" he!atos!lenome$aly are commonly seen with this con"ition. In evaluation of the "ifferential "ia$nosis of hilar lym!ha"eno!athy, a tissue "ia$nosis is essential to rule out other !ossibilities in the "ifferential "ia$nosis, e.$., carcinoma, lym!homa, or tuberculosis. Amon$ the "ia$nostic tests "escribe", a cervical lym!h no"e bio!sy woul" be the least invasive metho" of obtainin$ tissue for "ia$nosis. In a !atient with sarcoi"osis, a bio!sy will reveal noncaseatin$ $ranulomas. 7his "isease is ty!ically foun" in African Americans, a!!ro*imately :> times more often than in whites. 7he "isease usually !resents between the a$es of ;> an" ?> an" will most commonly involve the lun$s, skin, eye 'uveitis), an" lym!h no"es. A ell !alsy may also been seen with this con"ition. 2y!ercalcemia is an electrolyte abnormality that may be foun" in a"vance" cases. Althou$h 2I1 'choice A) an" is in the "ifferential "ia$nosis of a !atient with "iffuse lym!ha"eno!athy an" he!atos!lenome$aly, there are no s!ecific risk factors for 2I1.

7ransthoracic hilar lym!h no"e bio!sy 'choice C), liver bio!sy 'choice %), an" s!lenectomy 'choice () mi$ht also obtain tissue for evaluation, but they are much more invasive than cervical no"e bio!sy. :I) (*!lanation:

7he correct answer is . 7he clinical !icture is hi$hly su$$estive of "e$enerative <oint "isease involvin$ the cervical column 'cervical s!on"ylosis). 7his con"ition may cause neck !ain an" hea"aches. Cervical s!on"ylosis may in"ee" re!resent an un"er"ia$nose" cause of hea"ache. -urthermore, osteo!hytes may im!in$e on cervical roots an" cause !ain ra"iatin$ alon$ the u!!er e*tremities an" sensoryBmotor "eficits in a "ermatomal "istribution. 6e"uce" ran$e of motion of the neck is the most frequent ob<ective fin"in$ on !hysical e*amination. E4ray investi$ations will confirm the !resence of ty!ical chan$es associate" with s!inal "e$enerative <oint "isease, i.e., thinnin$ of cartila$e, subchon"ral osteo!orosis, an" !rominent osteo!hytes. It is often associate" with osteo!orosis, es!ecially followin$ meno!ause. (C. 'choice A) may be necessary when the nature an" ra"iatin$ !attern of cervicobrachial !ain su$$ests the !ossibility of myocar"ial ischemia. C7 'choice C) an" +6I 'choice %) of the s!inal column are not in"icate" as initial "ia$nostic investi$ations of an otherwise uncom!licate" cervical s!on"ylosis. +6I is the most sensitive ima$in$ mo"ality to assess the !resence of herniation of nucleus !ul!osus. 7his woul" manifest with an acute sym!tomatolo$y, inclu"in$ !ain that ra"iates "own the arm, muscle s!asm, an" re"uce" "ee! ten"on refle*es of bice!s an" trice!s. An$io$ra!hic stu"ies of cerebral vessels 'choice () are not useful in this case. &aro*ysmal verti$o results from com!ression of the vertebral artery, lea"in$ to transient cerebellar ischemia. ;>) (*!lanation: 7he correct answer is . Carcinoma of the !rostate is the most common mali$nancy in men in the F.)., an" is the secon" most common cause of cancer "eath in men ol"er than //. 7he "isease is more common amon$ blacks than whites. +ost carcinomas of the !rostate are slow4$rowin$, an" $reater than @>= are sta$e C or % at "ia$nosis. )ym!tomatic !atients usually com!lain of "ifficulty voi"in$ an", with a"vance" "isease, have sym!toms of s!inal com!ression. 2emato$enous metastases occur to the bone more frequently than to the viscera, an" an enlar$e", nonten"er !rostate is !al!able on rectal e*am. Amyotro!hic lateral sclerosis 'choice A) is a !ro$ressive motor neuron "isease that affects both u!!er an" lower motor neurons. &atients !resent with a $ra"ual onset of asymmetric weakness of the "istal limb. (ven in a"vance" "isease, sensory an" bla""er function are !reserve". On e*am, there is hy!eractivity of muscle stretch refle*es. &ott "isease 'choice C) is a vertebral osteomyelitis !rimarily involvin$ the thoracic s!ine. &atients usually com!lain of neck or back !ain. &al!ation of the involve" area results in !ain, an" !hysical e*am reveals s!asm of !aras!inal muscles an" limitation of motion.

)ciatica 'choice %) is cause" by irritation of the sciatic nerve ',?, ,/, ):). &ain ori$inatin$ in this nerve ra"iates "own the !osterior as!ect of the thi$h an" the !osterior an" lateral as!ect of the le$. )ciatica $enerally sto!s at the ankle an" is associate" with !araesthesias 'tin$lin$ an" numbness) that e*ten" "istally into the foot. 6efle* loss, muscle atro!hy an" weakness, an" fascicular twitchin$ may be !resent on !hysical e*am. )!inal stenosis 'choice (), which ty!ically affects el"erly !atients, is a narrowin$ of the lateral recess an" intervertebral foramen with resultant encroachment on the roots of the cau"a equina. 7he !ain is bilateral, worsens with stan"in$ an" walkin$, an" is relieve" by rest. ;:) (*!lanation: 7he correct answer is A. 7his !atient has $astro!aresis as a result of autonomic neuro!athy "ue to lon$stan"in$ "iabetes. Autonomic neuro!athy is usually seen in "iabetic !atients with !olyneuro!athy, but it can also be associate" with !ostural hy!otension, "isor"ere" sweatin$, im!otence in men, bla""er "ysfunction, eso!ha$eal "ysfunction, an" consti!ation or "iarrhea. As with other forms of "iabetic !eri!heral neuro!athy, it has been s!eculate" that "ama$e to the tiny bloo" vessels that accom!any an" fee" nerves may contribute to the nerve "ysfunction. If autonomic "ysfunction is sus!ecte" in a "iabetic !atient, a useful maneuver on !hysical e*amination is to look for a "ecrease in heart rate in res!onse to the 1alsalva maneuver 'force" e*!iratory effort a$ainst a close" airway). %iabetic $astro!aresis is "ifficult to mana$eG !atients who cannot tolerate meals of or"inary si#e an" ty!e may tolerate many tiny meals with the use of flui" nutritional su!!lements to ensure a"equate nutrition. Ina!!ro!riate inhibition of $astrin 'choice ) is unrelate" to the $astro!aresis in a !atient with "iabetes. Increase" restin$ !ressure at the lower eso!ha$eal s!hincter ',()) 'choice C) is seen in !atients with achalasia an" !ro"uces a motility4ty!e "ys!ha$ia. .astric em!tyin$ 'choice %) in "iabetic !atients ten"s to be slowe" rather than ra!i", secon"ary to the autonomic "isease. )carrin$ at the ,() 'choice () !ro"uces a mechanical4ty!e "ys!ha$ia. ;;) (*!lanation: 7he correct answer is (. 7his !atient has subacute combine" "e$eneration of the s!inal cor", which is "ue to a vitamin :; "eficiency. It is most often cause" by !ernicious anemia, but it may be acquire" by !atients with strict ve$etarian "iets or small bowel "isease. 7he clinical manifestations inclu"e weakness, !aresthesias, loss of vibratory sensation, increase" "ee! ten"on refle*es, an" e*tensor !lantar res!onses. 7he $ait is ata*ic. +ental chan$es may

also occur. 7he "ia$nosis is ma"e by measurin$ serum vitamin :; levels. 7he treatment is vitamin :; re!lacement. .uillain4 arrV syn"rome 'choice A) is an acquire" "emyelinatin$ neuro!athy that usually follows a viral res!iratory infection or immuni#ations. It is characteri#e" by ascen"in$ weakness, while sensation is lar$ely intact. A main "ia$nostic clue is absent "ee! ten"on refle*es. ,ambert4(aton syn"rome 'choice ) is a neuromuscular "isor"er that causes !ro*imal muscle weakness, !tosis, an" "i!lo!ia. %ee! ten"on refle*es are "e!resse" or absent. 6e!etitive nerve stimulation shows increase" res!onses. )mall cell carcinoma of the lun$ has been associate" with this "isor"er. +yasthenia $ravis 'choice C) is a neuromuscular "isor"er "ue to autoantibo"ies to acetylcholine rece!tors. It is characteri#e" by weakness an" easy muscle fati$ability. 7he e*traocular an" eyeli" muscles are often affecte", lea"in$ to "i!lo!ia an" !tosis, but "ee! ten"on refle*es are !reserve". &olymyositis 'choice %) is a skeletal muscle "isor"er that is characteri#e" by !ro$ressive !ro*imal muscle weakness. &atients often com!lain of "ifficulty climbin$ stairs an" brushin$ hair. Ocular muscles are $enerally not affecte". Creatine kinase is elevate", but muscle bio!sy an" electromyo$ra!hy confirm the "ia$nosis. ;0) (*!lanation: 7he correct answer is . 7he rhythm stri! shows a !olymor!hic ventricular tachycar"ia calle" torsa"es "e !ointes. 7his can be a fatal rhythm an" is associate" with a !rolon$ation of the 87 interval, which coul" have occurre" as a result of the quini"ine a"ministration. 7hus, the a!!ro!riate ste! is to "iscontinue the quini"ine an" observe. 2y!ocalcemia can cause 87 !rolon$ation 'choice A). 2y!ercalcemia can cause 87 shortenin$. 2owever, this !atientAs electrolytes are within normal limits, an" a"ministration of calcium is not nee"e". Increasin$ the quini"ine "ose 'choice C) will lea" to a further !rolon$ation of the 87 interval, further !re"is!osin$ the !atient to torsa"es that may now be !ersistent an" hemo"ynamically si$nificant. 2y!okalemia can lea" to a !rolon$ation of the 87 interval as well an" !re"is!ose to torsa"es. 2owever, the !atient ha" normal electrolytes an" "oes not nee" I1 !otassium 'choice %). 2emo"ynamically unstable ventricular tachyarrhythmia will require "efibrillation 'choice (). 2owever, the event note" in this !atient is transient, an" "efibrillation shoul" not be

nee"e". If the rhythm were !ersistent an" the !atient were unstable, "efibrillation woul" be in"icate". ;?) (*!lanation: 7he correct answer is . 7he sus!icion, base" u!on the clinical e*amination, is that this !atient has !neumonia. 7he only way to "efinitively "ia$nose !neumonia is with an infiltrate !resent on chest ra"io$ra!h. All further "ecision makin$ about this !atient will "e!en" on whether the sus!icion of a !ulmonary infection is confirme". An arterial bloo" $as 'choice A) is not necessary in this situation. 7hese tests are routinely !erforme" on asthmatic an" CO&% !atients when the results are alrea"y known em!irically. -or e*am!le, a CO&% !atient who has a room air saturation of @>= an" is tachy!neic with labore" breathin$ will almost certainly be hy!o*ic an" hy!ercarbic. (ven if this !atient "i" have a !neumonia, the arterial bloo" $as will reveal no useful information that an o*y$en saturation an" thorou$h history woul" not. A com!lete bloo" count 'choice C) is im!ortant, but not before you "etermine whether this !atient has a !neumonia. An elevate" leukocyte count can only be inter!rete" after such information is obtaine". 7his conce!t is a $eneral one in clinical me"icine, never or"er laboratory tests unless there is s!ecific information sou$ht that may hel! to confirm or ne$ate a "ia$nosis. An o*y$en saturation check 'choice %) is not a!!ro!riate at this !oint $iven that the !atientAs a!!earance is beni$n. 2y!o*emia woul" be evi"ent in the !atientAs res!iratory rate, a!!earance, or his ins!iratory effort. A s!utum sam!le 'choice () is often obtaine" when a bacterial !neumonia is sus!ecte" an" selective antibiotic thera!y is "esire", but the !resence of an infiltrate on ra"io$ra!h woul" "ictate antibiotic thera!y base" on his a$e an" !lace of resi"ence 'home, hos!ital, ICF be"). 7hese em!iric $ui"elines are a"equate for most thera!y an" can be tailore" to selective thera!y base" on evolvin$ clinical fin"in$s. ;/) (*!lanation: 7he correct answer is %. )evere nausea an" vomitin$ can accom!any many forms of chemothera!y, an" many !atients e*!erience these si"e effects as the worst !roblem relate" to their treatment. +any a$ents use" to control nausea an" vomitin$ in other settin$s are nearly useless for the very severe nausea an" vomitin$ that accom!anies chemothera!y. It has been !ostulate" that the severe nausea is relate" to release of neurotransmitters by "ama$e" enterochromaffin 'neuroen"ocrine) cells in the small intestine. On"ansetron, which is available in both I1 an" oral forms, is the most effective a$ent for emesis in"uce" by

chemothera!y. 7his "ru$ is a selective inhibitor of one ty!e of the /4hy"ro*ytry!tamine 'synonym serotonin) rece!tor, which is !resent both !eri!herally on va$al nerves an" centrally in the chemorece!tor tri$$er #one of the area !ostrema of the brain. ismuth 'choice A) !re!arations can be use" to control "iarrhea an" mil" nausea of the verti$oBmotion sickness ty!e, but are nearly useless with chemothera!y4in"uce" vomitin$. +or!hine 'choice ) is commonly $iven to cancer !atients to control !ain sym!toms but actually increases vomitin$ by stimulatin$ the chemorece!tor tri$$er #one. Ome!ra#ole 'choice C) re"uces aci" secretion but "oes not affect chemothera!y4in"uce" vomitin$. It is use" as a com!onent of an anti42elicobacter re$imen. )ertraline 'choice () is an anti"e!ressant of the selective serotonin reu!take inhibitor '))6I) family.. ;C) (*!lanation: 7he correct answer is (. 7his !atient has "iabetes an" hy!ertension. 7he mana$ement strate$y for such !atients inclu"es a""ressin$ the !rimary bloo" su$ar !roblem, the !rimary bloo" !ressure !roblem, an" the secon"ary consequences of both the "iabetes an" the bloo" !ressure. 7his man has very $oo" bloo" !ressure control on his current re$imen, an" both of his anti4hy!ertensive me"ications have !roven survival benefit. 2owever, his bloo" !ressure coul" be lowere" even further. .iven the co4morbi"ity of his "iabetes, an a!!ro!riate intervention woul" be to a"" an AC( inhibitor to "eal with "iabetic com!lications, such as renal "isease, as well as to a""ress further lowerin$ of his bloo" !ressure. Increasin$ his sulfonylurea "ose to better control his su$ar is also a!!ro!riate. A""in$ insulin an" an AC( inhibitor to his re$imen 'choice A) is !artially correct. Insulin may eventually be in"icate" in this !atient, but at !resent, he requires a""itional "osin$ of his oral a$ent to conclu"e that such a line of thera!y has faile". %iscontinuin$ his sulfonylurea a$ent an" be$innin$ insulin thera!y 'choice ), as mentione" above, is incorrect since both of these actions are !remature. A $eneral !rinci!le of thera!y is that, unless a com!ellin$ reason e*ists not to ma*imi#e a !articular a$ent, all current me"ications shoul" be o!timi#e" !rior to a""in$ any new !harmacolo$ic interventions. Increasin$ his sulfonylurea an" beta blocker "ose 'choice C) or increasin$ his sulfonylurea an" thia#i"e "ose 'choice %) are !artially correct. Althou$h his beta blocker an" "iuretic have been effective in controllin$ his bloo" !ressure, room for im!rovement still e*ists. ecause of the "iabetes, however, a com!ellin$ reason e*ists not to ma*imi#e

e*istin$ thera!y but to a"" a thir" anti4hy!ertensive a$ent to further lower his bloo" !ressure AN% a""ress his "iabetes. ;D) (*!lanation: 7he correct answer is . )evere e!i$astric !ain ra"iatin$ to the back an" accom!anie" by nausea an" vomitin$ su$$ests !ancreatitis. &ancreatitis is most likely to be encountere" in alcoholics 'such as this man) an" !atients with biliary tract "isease. 7his !atient has a history of recurrent alcoholic !ancreatitis. 7he "evelo!ment of $ra"ual wei$ht loss, chronic !ain ra"iatin$ to the back, an" steatorrhea su$$ests that he has now "evelo!e" chronic !ancreatitis. 7his con"ition is often com!licate" by both en"ocrine an" e*ocrine insufficiency of the !ancreas, secon"ary to loss of much of the tissue of the !ancreas to the "isease !rocess. Cholan$iocarcinoma 'choice A) !ro"uces <aun"ice, has a very !oor !ro$nosis, an" usually occurs in !atients with a !rior history of !rimary sclerosin$ cholan$itis. .astric outlet obstruction 'choice C) woul" !resent with !ost!ran"ial vomitin$ an" early satiety. &ancreatic a"enocarcinoma 'choice %) may also occur in this !atient but is $enerally !resent in ol"er in"ivi"uals. 7here is no evi"ence of scarrin$ of the common bile "uct 'choice (), which woul" !ro"uce <aun"ice. ;@) (*!lanation: 7he correct answer is C. 7his !atient has +eniere "isease, which is characteri#e" by tinnitus, verti$o, an" !ro$ressive hearin$ loss. It is thou$ht to be relate" to a "e$eneration of the vestibular an" cochlear hair cells. 7he treatment inclu"es be" rest, a low4salt "iet, "imenhy"rinate, cycli#ine or mecli#ine. eni$n !ositional verti$o 'choice A) is characteri#e" by !aro*ysmal verti$o an" nysta$mus. It is brou$ht on by certain chan$es in !osition. 2earin$ loss is not !resent. 7he cause is i"io!athic. eni$n recurrent verti$o 'choice ) an" vestibular neuronitis 'choice () are two names for the same syn"rome, which is characteri#e" by the su""en onset of verti$o, nausea, an" vomitin$. 7here is no chan$e in hearin$. 7o*ic labyrinthitis 'choice %) !resents with verti$o. It is "ue to me"ications an" alcohol. Amino$lycosi"es are amon$ the most common cul!rits.

;I) (*!lanation: 7he correct answer is . 2e!arin4in"uce" thrombocyto!enia '2I7) is the result of !latelet a$$re$ation cause" by he!arin4in"uce" antibo"ies. It is seen in := to /= of !atients on he!arin. 7hera!y is "iscontinuation of the he!arin an" use of another anticoa$ulant, such as le!iru"in. When the !latelet count falls below />,>>>Bmm0 the he!arin shoul" be sto!!e". 2I7 can lea" to limb4threatenin$ thromboses, as in this !atient, an" constitutes a me"ical emer$ency. Arterial thrombosis is a manifestation of the 2I7 syn"rome. )e!tic emboli or marantic emboli 'choice A) may cause an ischemic limb as a result of occlusion of an artery. 7reatment involves treatment of the un"erlyin$ con"itionG valve re!lacement may be nee"e". Aortic sclerosis is the result of calcification or sclerosis of the aortic valve an" is not associate" with embolic events. &ara"o*ical emboli 'choice C) may be the result of a "ee! venous thrombosis !assin$ throu$h a !atent foramen ovale an" may be a cause of arterial occlusion as the clot moves from the venous to the arterial system. 7reatment requires closure of the !atent foramen ovale, either sur$ically or throu$h catheteri#ation. 2y!ercoa$ulability from immobili#ation 'choice %) ty!ically causes "ee! venous thromboses. In such a situation, !lacement of a vena cava filter must be consi"ere". Con"itions !re"is!osin$ a !atient to hy!ercoa$ulability inclu"e trauma, stasis, an" con"itions such as cancer. rachial artery vasos!asm 'choice () is a rare occurrence an" shoul" be transient. 7he !atient may com!lain of han" "iscomfort, but, $iven the transient nature of the con"ition, a !ulse shoul" still be !al!ate" an" the e*tremity shoul" not be cool. A !atient with 6aynau"As !henomenon may e*hibit such sym!toms. 0>) (*!lanation: 7he correct answer is A. 7his !atient has a classic !resentation of chronic obstructive !ulmonary "isease 'CO&%), as "emonstrate" by chronic bronchitic sym!toms an" !ro$ressive "ys!nea. 2is !ulmonary e*amination "emonstrates obstruction to airway outflow '!rolon$e" e*!iratory !hase) an" airway obstruction as "emonstrate" by the lou" bilateral rhonchi. 7he increase" A& "iameter is "emonstrate" by the lar$e Hbarrel chest.H 7his is further confirme" on a chest *4ray film, which reveals hy!erinflation in both lun$s. 7he classic !ulmonary function test !attern in these !atients is of a "ecrease" force" e*!iratory volume in : secon" '-(1:) to force" vital ca!acity '-1C) ratio.

7he total lun$ ca!acity 'choice ) is increase" because of chronic air tra!!in$, resultin$ in the ty!ical barrel chest an" hy!erinflation of the lun$ fiel"s. 6esi"ual volumes in these !atients are increase", rather than "ecrease" 'choice C), because of air tra!!in$. 7he -(1:'choice %) an" -1C 'choice () are both "ecrease". 0:) (*!lanation: 7he correct answer is C. 7his !atient has a hy!ertensive emer$ency, as evi"ence" by both the !resence of elevate" systolic an" "iastolic bloo" !ressure, as well as evi"ence of en"4or$an "ama$e. 7he bloo" !ressure numbers themselves are a $oo" reason for ur$ency, but the !resence of increase" intracranial !ressure an" renal failure require that this !atient be tria$e" as an emer$ency. (levate" bloo" !ressure alone, in the absence of sym!toms or en" or$an "ama$e, rarely requires emer$ency thera!y. 7he thera!y for a hy!ertensive emer$ency requires I++(%IA7( ,OW(6IN. of the bloo" !ressure by a!!ro*imately ;>40> mm 2$ by intravenous a"ministration of me"ication. 7he bloo" !ressure shoul" not be re"uce" to normotensive levels because of the risk of watershe" cerebral infarcts with such a "ramatic re"uction. Or"erin$ an (C. an" observin$ the !atient 'choice A) is incorrect. Observation is certainly warrante", but not without intervention. .ivin$ the !atient his usual "aily "oses of bloo" !ressure me"ications an" observin$ him 'choice ) woul" not achieve a ra!i" enou$h re"uction of the bloo" !ressure, $iven the !resence of en"4or$an "ama$e on !hysical e*amination. Or"erin$ an ab"ominal C7 scan to rule out aortic aneurysm 'choice %) is not an a!!ro!riate intervention in the case of an active hy!ertensive emer$ency. &erformin$ a be"si"e cystosco!y to evaluate the hematuria 'choice () is not in"icate" as the almost certain cause for his hematuria is his elevate" bloo" !ressure. 0;) (*!lanation: 7he correct answer is %. Not infrequently, the sym!toms of myocar"ial ischemia are inter!rete" by !atients as of $astrointestinal ori$in '$astric u!set, heartburn, in"i$estion). 2owever, myocar"ial infarction '+I) is usually accom!anie" by ob<ective si$ns of sym!athetic activation, inclu"in$ sweatin$, an*iety, tachy!nea, an" tachycar"ia. A""itional sym!toms may inclu"e li$ht4hea"e"ness, "ys!nea, ortho!nea, cou$h, nausea, an" synco!e. An (C.

e*amination is im!erative in any !atient !resentin$ with this sym!tomatolo$y. A normal tracin$ is rare with acute +I. Chest *4ray 'choice A) woul" not be useful in this case, since there are no !hysical si$ns !ointin$ to the lun$s as a !ossible source of sym!toms. A com!lete bloo" count 'choice ) woul" likely show leukocytosis on the secon" "ay followin$ +I, but it is of little "ia$nostic value in the initial a!!roach. On the other han", serum C54+ isoen#yme levels are elevate" within C hours after the onset of sym!toms. (chocar"io$ra!hy 'choice C) is a hel!ful a"<unctive tool in "emonstratin$ abnormal motility of ischemic se$ments of the ventricular wall followin$ an +I. 2owever, it is not use" as an initial "ia$nostic test !rior to obtainin$ an (C.. F!!er en"osco!y 'choice () with bio!sy is the stan"ar" "ia$nostic tool in the stu"y of $astroeso!ha$eal reflu* "isease, which manifests with !ost!ran"ial burnin$ e!i$astric or substernal !ain relieve" by antaci"s. 00) (*!lanation: 7he correct answer is . 7he initial mana$ement ste! in any !atient with a lar$e volume of $astrointestinal blee" is imme"iate !lacement of I1 access. 7his is true in $eneral with any !atient who is havin$ a si$nificant blee"in$ e!iso"e, since such line !lacement facilitates ra!i" transfusion of whole bloo" or other flui"s. 7wo, rather than one, I1 catheters are recommen"e" to !rovi"e increase" safety 'if, for e*am!le, somethin$ $oes wron$ with one line) an" fle*ibility 'either the same or "ifferent flui"s can be a"ministere" throu$h the two lines, "e!en"in$ on the s!ecifics of the clinical settin$). Althou$h an (C. 'choice A) woul" be useful to rule out ischemia in this ol"er !atient, it "oes not take !rece"ence over establishin$ secure I1 access to allow flui" resuscitation imme"iately. F!!er $astrointestinal en"osco!y 'choice C) will be necessary to "ia$nose an" treat the u!!er $astrointestinal blee", but flui" resuscitation shoul" be "one first. &atients with $astrointestinal blee"in$, even with known variceal hemorrha$e, "o not un"er$o emer$ency liver trans!lantation an" must un"er$o resuscitation an" a systematic evaluation for trans!lant 'choice %). Colonosco!y 'choice () is not a!!ro!riate in this !atient who is havin$ an u!!er $astrointestinal blee", as "emonstrate" by the hematemesis. &atients with brisk u!!er $astrointestinal blee"in$ may !resent with hematoche#ia, as in this !atient. 2owever, since the source is clearly in the u!!er $astrointestinal tract, en"osco!ic evaluation shoul" be$in with an u!!er $astrointestinal en"osco!y.

0?) (*!lanation: 7he correct answer is (. 2y!ertension is a frequent cause of intracerebral hemorrha$es. Cerebellar blee"in$ "iffers from intracerebral blee"in$ 'i.e., that occurrin$ within the cerebral hemis!here) with res!ect to clinical manifestations an" mana$ement. Whereas intracerebral blee"in$ usually "evelo!s slowly, cerebellar hematomas manifest with abru!t onset. Intracerebral blee"in$ usually lea"s to early loss of consciousness, but !atients with cerebellar hematomas remain luci" until the increase" !ressure within the !osterior fossa results in cerebellar tonsillar herniation. Cerebellar hematomas shoul" be evacuate" as soon as !ossible before coma ensues. In contrast to intracerebral blee"in$, !rom!t sur$ical intervention may be life4savin$ an" followe" by com!lete or nearly com!lete neurolo$ic recovery. It is thus essential to reco$ni#e this clinical syn"rome !rom!tly. ,umbar !uncture 'choice A) is "efinitely contrain"icate" in case of sus!ecte" intracerebellar hematoma, as it may !reci!itate fatal herniation of the tonsils throu$h the foramen ma$num. +6I of the hea" 'choice ) woul" not a"" any more information com!are" with the C7 scan in the acute sta$e of intracranial blee"in$. )u!!ortive me"ical treatment 'choice C) is a!!ro!riate for most cases of hy!ertensive intracerebral blee"in$, which "evelo!s in "ee! structures of the cerebral hemis!here or brainstem, i.e., basal $an$lia, centrum semiovale, or !ons. +easures aime" at controllin$ bloo" !ressure an" re"ucin$ e"ema are the mainstay of treatment. Anticoa$ulant treatment 'choice %) is contrain"icate" when there is evi"ence of intra!arenchymal blee"in$. 0/) (*!lanation: 7he correct answer is C. (chocar"io$ra!hy is an invaluable tool for assessin$ car"iovascular function in both normal an" "isease states. In the case of atrial fibrillation 'A-), a chronic course versus an acute an" self4limite" course !orten" com!letely "ifferent treatment strate$ies an" lon$4term !ro$noses. 7he most common cause of chronic A- is valvular "isease, followe" by con$estive heart failure 'C2-). 7he most common anatomic correlate seen in !atients with A- '-ramin$ham 2eart )tu"y) is an enlar$e" left atrium. A "ilate" left ventricle 'choice A) is commonly seen with both C2- an" A

-. 2owever, an enlar$e" left atrium is more closely correlate" with chronic A- than is left ventricular "ilation. 7he causal relationshi! between an enlar$e" left atrium an" A- is unclear. A "ilate" ri$ht ventricle 'choice ) is commonly seen in severe ri$ht heart failure '62-). 7he most common cause of 62- is left heart failure ',2-). 7here is no consistent relationshi! between ri$ht heart si#e an" the !resence of A -. A hy!ertro!hie" ventricular se!tum 'choice %) is commonly seen in hy!ertensive heart "isease or i"io!athic hy!ertro!ic aortic stenosis. Chronic A- in these !atients, however, is much less common. &ericar"ial thickenin$ 'choice (), as seen with chronic !ericar"ial inflammation, is not routinely associate" with chronic A -. Acute A-, however, is associate" with acute !ericar"itis. 0C) (*!lanation: 7he correct answer is C. 7his !atient has an acute e*acerbation of her chronic obstructive !ulmonary "isease 'CO&%). On the basis of the history an" !hysical e*amination, she woul" be e*!ecte" to have a com!ensate" res!iratory aci"osis because CO; retention. -urthermore, she woul" be e*!ecte" to have evi"ence of mil" hy!o*ia. With res!irations of ;?Bmin, which is hi$h but not e*tremely hi$h, she woul" not be e*!ecte" to show the severe levels of aci"osis an" CO; retention illustrate" in choice A. 7he !atient is sufficiently sym!tomatic that the near normal !O; of I? seen in choice woul" be unlikely. Althou$h asthmatics may !resent "urin$ an acute e*acerbation with a res!iratory alkalosis 'choices % an" (), in a !atient with un"erlyin$ CO&%, there is usually a baseline res!iratory aci"osis. 0D) (*!lanation: 7he correct answer is A.Cam!ylobacter <e<uni is a common cause of community4acquire" bloo"y "iarrhea. It is acquire" via the fecal4oral route an" may !ro"uce a non4bloo"y or bloo"y "iarrhea. If the sym!toms are as severe as in this !atient, antibiotics 'e.$., ci!roflo*acin) woul" be a!!ro!riate.

Cry!tos!ori"ium'choice ) is a !arasite that can infect immunocom!etent in"ivi"uals an" will !ro"uce a self4limite" watery "iarrhea. When it infects AI%) !atients it ty!ically lea"s to a chronic, watery "iarrheal syn"rome that !ro"uces severe wei$ht loss. .iar"ia lamblia'choice C) also causes an u!!er $astrointestinal infection but !resents with u!!er ab"ominal sym!toms an" a non4bloo"y "iarrhea. )ta!hylococcus aureus'choice %) causes a foo"4borne infection that will !ro"uce u!!er ab"ominal !ain an" nausea an" vomitin$, secon"ary to a !reforme" in$este" to*in. 7he characteristic feature of this infection is its !rom!t occurrence, i.e., within a!!ro*imately ?4@ hours of the in$estion of the tainte" foo". 7o*i$enic (scherichia coli'choice () !resents with a watery "iarrhea, acquire" throu$h the fecal4oral route as a travelerAs "iarrhea. 0@) (*!lanation: 7he correct answer is C. 7his is otosclerosis, which is an often here"itary 'autosomal "ominant) "isease in which new, immature bone with abun"ant vascular channels cause ankylosis of the sta!e"ial foot !late. 7his causes !ro$ressive con"uctive 'an" in severe cases sensory) hearin$ loss which may become clinically si$nificant in the late teens to early twenties. 'While otosclerotic lesions occur in u! to :>=of Caucasian a"ults, si$nificant hearin$ loss occurs in only about :>= of !atients with bony lesions. ) &re$nancy may cause the con"ition to !ro$ress more ra!i"ly. +any !atients can be mana$e" with hearin$ ai"sG severe cases may res!on" to removal of the sta!es with im!lantation of a !rosthesis. Acoustic neuroma 'choice A) woul" usually be unilateral, an" woul" cause tinnitus, hearin$ loss, an" sometimes "i##iness an" unstea"iness. Chronic otitis me"ia 'choice ) woul" !ro"uce an obviously scarre" or inflame" tym!anic membrane. +eniere "isease 'choice %) causes recurrent severe verti$o that may be accom!anie" by sensory hearin$ loss, tinnitus, an" a feelin$ of fullness in the ear. &resbycusis 'choice () is the common sensorineural hearin$ loss seen with a$in$. 0I) (*!lanation: 7he correct answer is A. 7his !atient has ty!ical sym!toms of the e*tra4eso!ha$eal manifestations of $astroeso!ha$eal reflu* "isease '.(6%). At ni$ht, while she is su!ine, aci" may reflu* across the u!!er eso!ha$eal s!hincter an" into the u!!er airway, tri$$erin$

bronchoconstriction an" her asthmatic sym!toms. 7he history of .(6% is su$$este" by her baseline sym!toms of heartburn requirin$ histamine4; rece!tor anta$onists. 7he treatment for these !atients is aci" re"uction "irecte" at the un"erlyin$ .(6% !rocess. ,aryn$os!asm 'choice ) "oes not !resent with cou$h an" whee#in$, but with hoarseness an" stri"or. +ast cell release 'choice C) is the mechanism whereby e*ercise4in"uce" asthma occurs. +ucus !lu$s 'choice %) can occur in !atients with co!ious secretions, which may cause transient airway obstruction. F!!er airway obstruction 'choice () may occur "urin$ slee! in !atients with slee! a!nea, but there are no si$ns or sym!toms to su$$est this syn"rome in this !atient. ?>) (*!lanation: 7he correct answer is (. 7he clinical !resentation, with nuchal ri$i"ity, fever, an" obtun"ation, is characteristic of acute menin$itis. 7he C)- fin"in$s are "ia$nostic of this acute bacterial infection. Acute bacterial menin$itis is associate" with C)- !leocytosis 'mostly "ue to neutro!hilia), increase" !rotein, an" "ecrease" $lucose. -urthermore, the !atientAs a$e an" fin"in$ of !etechiae !oint to menin$ococcus as the most likely !atho$en. +enin$ococcus is the most common etiolo$ic a$ent of cases affectin$ youn$ immunocom!etent a"ults. )ometimes, but not always, $ram4ne$ative cocci can be "etecte" on $ram4staine" sam!les of the C) -. C)- cultures, however, allow isolation of menin$ococcus unless the !atient has alrea"y receive" antibiotic treatment '!artially treate" menin$itis). In any case, antibiotic treatment with !enicillin shoul" be imme"iately starte" soon after submittin$ a sam!le of C)- for culture stu"ies. Antibiotic thera!y may then be o!timi#e" accor"in$ to culture an" antibiotic sensitivity results. (scherichia coli'choice A) an" $rou! stre!tococci 'choice ) are the most common etiolo$ic a$ents in infants. In this a$e $rou!, bacterial menin$itis may manifest with nons!ecific sym!toms, such as fever, !oor fee"in$, an" e*cessive cryin$. 2emo!hilus influen#ae'choice C) has become a relatively rare cause of menin$itis since the intro"uction of man"atory immuni#ation a$ainst this bacillus. ,isteria monocyto$enes'choice %) is now a frequent cause of menin$itis in infants youn$er than ; months an" in immunocom!romise" a"ults. &neumococcus 'choice -) is the most common a$ent causin$ !urulent menin$itis in the el"erly.

?:) (*!lanation: 7he correct answer is C. 7his !atient is hy!otensive an" has oli$uria. 2e has shock, fever, an" !ulmonary e"ema. 2is elevate" we"$e !ressure is an in"ication of left ventricular failure. 7his may be the result of a myocar"ial infarction. 7his !atient has car"io$enic shock, severe car"iomyo!athy, or myocar"itis. .astrointestinal blee"in$ 'choice A) woul" !resent with hy!otension, tachycar"ia, an" shock. 2y!ovolemia from a $astrointestinal blee" woul" cause a "ecrease in the we"$e !ressure as well. )imilarly, se!tic shock 'choice ) woul" lea" to hy!otension an" "ecrease" we"$e !ressure. 7he car"iac out!ut woul" be increase", an" the systemic vascular resistance woul" be "ecrease". In the settin$ of fever, however, this "ia$nosis must be consi"ere". 7reatment woul" inclu"e su!!ortive thera!y with vaso!ressors an" flui"s, as well as antibiotics. &ericar"ial tam!ona"e 'choice %) coul" !ro"uce elevate" we"$e !ressures, but the obstruction to the ri$ht ventricular inflow shoul" be associate" with equally abnormal ri$ht atrial mean, ri$ht ventricular en" "iastolic, an" !ulmonary artery en"4"iastolic !ressures. A !ulmonary embolus 'choice () woul" lea" to "ecrease" we"$e !ressure. 7he !atient woul" be tachycar"ic, tachy!neic, an" hy!otensive. &ulmonary e"ema woul" not be seen, however. 7reatment woul" inclu"e a"ministerin$ a lytic a$ent an" he!arin. ?;) (*!lanation: 7he correct answer is . In amyloi"osis, the left ventricular wall a!!ears s!eckle" on the echocar"io$ram, an" there is a restrictive car"iomyo!athy. In such a con"ition, ventricular fillin$ is im!aire", an" the car"iac silhouette may be mil"ly enlar$e". An (C. may reveal a host of nons!ecific arrhythmias. &rimary car"iac amyloi"osis usually "evelo!s into "iastolic "ysfunction. Alcoholic car"iomyo!athy 'choice A) is ty!ically the cause of a biventricular "ilate" car"iomyo!athy, which lea"s to both ri$ht4 an" left4si"e" heart failure. An )0 will be hear". An echocar"io$ram will show enlar$e" left an" ri$ht ventricles. 7he walls of the ventricles may a!!ear very thin an" stretche", consistent with volume overloa". 2emochromatosis 'choice C) also may cause a restrictive car"iomyo!athy, as seen in amyloi"osis. 2owever, the s!eckle" !attern mentione" above woul" be absent. Other noncar"iac features inclu"e bron#in$ of the skin an" "iabetes.

7uberculosis 'choice %) may cause a chronic tuberculous !ericar"itis that can manifest clinical sym!toms similar to those seen in constrictive car"iomyo!athy. 7he !resentation is similar to that seen with restrictive features. 2owever, !atients ten" to have normal ventricular wall thickness on echocar"io$ram, !ericar"ial calcification, an absent )0, an" )?. 1iral myocar"itis 'choice (), like alcohol, can lea" to a "ilate" car"iomyo!athy. Fnfortunately, such con"itions may !ro$ress to com!lete left an" ri$ht ventricular failure, ultimately requirin$ car"iac trans!lantation in refractory cases. ?0) (*!lanation: 7he correct answer is (. 7his !atient, with !ost!ran"ial "iarrhea, wei$ht loss, low4$ra"e fever, an" ri$ht lower qua"rant fin"in$s on !hysical e*amination, has the ty!ical !resentation of Crohn "isease, which most commonly involves the terminal ileum. Inflammation in this "isease is transmural, as o!!ose" to the inflammation in ulcerative colitis that is limite" to the mucosa of the lar$e intestine. Althou$h the etiolo$y of Crohn "isease is not known, $ram4ne$ative or$anisms 'choice A) have not been "emonstrate" to be causative. -olate "eficiency 'choice ) "oes not cause a "iarrheal illness. It can, however, result infrequently from Crohn "isease if there is severe !ro*imal small bowel malabsor!tion. +ucosal ulceration in the ascen"in$ colon 'choice C) may be seen "urin$ a colonosco!y in !atients with Crohn "isease, but the inflammation in fact is transmural. 7o*in4!ro"ucin$ or$anisms 'choice %), such as enteroto*i$enic (scherichia coli an" 1ibrio cholerae !ro"uce a watery "iarrheal syn"rome without any other of the fin"in$s "escribe" in this !atient. ??) (*!lanation: 7he correct answer is A. Clinical history is crucial in the "ia$nostic a!!roach to synco!e. -urthermore, measurement of bloo" !ressure shoul" be !erforme" first with the !atient in a su!ine !osition, an" then checke" a$ain u!on stan"in$. 7his !atientAs e!iso"es of li$hthea"e"ness an" synco!e can be best e*!laine" as an effect of antihy!ertensive thera!y. Orthostatic hy!otension is one of the most frequent si"e effects of antihy!ertensive "ru$s an" shoul" always be consi"ere" in the "ifferential "ia$nosis of synco!e of une*!laine" ori$in.

Aortic stenosis 'choice ) may manifest with recurrent synco!e. On !hysical e*amination, a characteristic H"iamon"4sha!e"H 'crescen"o4"ecrescen"o) murmur is hear" in the !recor"ial re$ion. %ro! attacks 'choice C) refer to su""en loss of tone in the lower e*tremities an" !robably result from e!iso"ic cerebral vascular insufficiency. 7he !atient remains fully conscious. 6efle* synco!e 'choice %) may occur after em!tyin$ a "isten"e" bla""er 'micturition synco!e) or followin$ !rolon$e" cou$hin$ 'tussive synco!e). 7ransient ischemic attacks 'choice () rarely manifest with loss of consciousness. 7his may be cause" by severe atherosclerosis of the vertebrobasilar arteries. 1asova$al synco!e 'choice -) occurs most frequently in a"olescents an" youn$ a"ults. &ain, a!!rehension, emotional shock, or other !reci!itatin$ stimuli are the usual tri$$ers. ?/) (*!lanation: 7he correct answer is C. 7he )wan4.an# tracin$ in"icates that the !atient has an elevate" ri$ht4si"e" !ressure an" a low4fillin$ !ressure. Car"iac out!ut is "ecrease" as a result of insufficient left heart fillin$ !ressures. 7his is "ue to the ri$ht ventricular infarct, which causes backin$ u! of venous bloo" an" "ecrease" forwar" flow, !ro"ucin$ a "ecrease in left ventricular fillin$, as in"icate" by the low we"$e !ressure. 7he treatment for this !atient is a$$ressive flui" a"ministration. 7he !atient ha" a ri$ht ventricular infarct, !resumably from involvement of the ri$ht coronary artery. alloon an$io!lasty 'choice A) may ultimately be nee"e" to correct the un"erlyin$ cause of the infarction. 2owever, the acute event has !asse", an" the !atient must first be stabili#e". )he will ultimately nee" a car"iac catheteri#ation. %i$o*in 'choice ) is not nee"e" in this !atient, since she is not in left si"e" heart failure. &ositive inotro!y is not nee"e" as much as flui" resuscitation. Also, the !atient is not in atrial fibrillation. If the !atient ha" sustaine" a left ventricular infarct, she may have nee"e" afterloa" re"uction in the form of intraaortic balloon counter!ulsation 'choice %). )ince the !atient sustaine" a ri$ht ventricular infarct, however, afterloa" re"uction is not as im!ortant as is !reloa" re!letion. 7he !atient is hy!otensive. If the hy!otension "oes not resolve with flui" re!letion, then vaso!ressors such as nore!ine!hrine may be nee"e" 'choice (). 2owever, flui" resuscitation must be continue".

?C) (*!lanation: 7he correct answer is . 7he most im!ortant interventions involve limitin$ the chance of a ru!ture" ab"ominal aortic aneurysm 'AAA), which is the sus!ecte" "ia$nosis here. 7hese inclu"e counselin$ for smokin$ cessation an" im!rovin$ bloo" !ressure control. &hysical thera!y 'choice A), althou$h often effective for the relief of back !ain, !lays no role in the treatment of AA A. Initiatin$ a wei$ht loss !ro$ram 'choice C) or !rescribin$ an e*ercise !ro$ram 'choice () can both !otentially !lay an im!ortant role in im!rovin$ the !atientAs $eneral health. 2owever, they will unlikely alter his short4term risk from AAA ru!ture. &rescribin$ a nonsteroi"al anti4inflammatory "ru$ 'N)AI%) 'choice %) may "ecrease the back !ain, but may worsen the effect of a ru!ture because of the anti!latelet effects of N)AI%s. ?D) (*!lanation: 7he correct answer is C. &atients with a history of !ortal hy!ertension are at risk for he!atic ence!halo!athy. 2e shoul" therefore strictly restrict or entirely avoi" any me"ications with a se"ative effect, e.$., ben#o"ia#e!ines such as lora#e!am. In a""ition to !rescribe" me"icines, over the counter antihistamines an" cou$h reme"ies also often contain "ru$s with se"ative effects, an" the !atient shoul" be encoura$e" to check with the !harmacist an"Bor !hysicianAs office before usin$ any of them. 7he antibiotic ci!roflo*acin 'choice A) "oes not cause he!atic "ecom!ensation, but it can cause hy!ersensitivity reactions an" "i##iness. Ibu!rofen 'choice ) can be use" in !atients with liver "isease an" is in fact !referre" to acetamino!hen as a mil" anal$esic because of the latterAs !otential harmful effects on the liver. Ibu!rofen, like other nonsteroi"al anti4inflammatory "ru$s, can cause $astrointestinal blee"in$ relate" to tiny ulcerations of the stomach. 2owever, there is no evi"ence that this !atient is currently blee"in$. ,osartan 'choice %) is an an$iotensin II rece!tor blocker an" is not contrain"icate" in !atients with liver "isease. ,osartan can cause hy!otension in volume4"e!lete" !atients an" fetal morbi"ity. 7he $lucocorticoi" !re"nisone 'choice () "oes not ty!ically cause he!atic "ecom!ensation, but it can cause the many other si"e effects associate" with $lucocorticoi" use, such as cushin$oi" features, su!!ression of the inflammatory

res!onse with ten"ency to "evelo! infection, flui" an" electrolyte imbalances, an" increase" intracranial !ressure. ?@) (*!lanation: 7he correct answer is (. loo" accumulatin$ in the su!ratentorial com!artment, as well as any ra!i"ly $rowin$ s!ace4occu!yin$ lesion in this re$ion, may lea" to "is!lacement of the uncus over the e"$e of the incisura of the tentorium. 7his is a $rave com!lication referre" to as uncal or transtentorial herniation. 7he herniate" uncus will com!ress the oculomotor nerve, the !osterior cerebral artery, an" the brainstem. 7he !atho!hysiolo$ic consequences inclu"e oculomotor !aralysis 'manifestin$ with fi*e" an" "ilate" !u!il on the same si"e), i!silateral infarction of the occi!ital lobe, an" hemorrha$es within the mi"brain an" !ons. 7he latter may result in res!iratory !aralysis an" "eath. 7his clinical case outlines the im!ortance of two factors in brain in<uries, i.e., "evelo!ment of e"ema an" the fact that the brain is enclose" within ri$i" walls. Almost any !atholo$ic !rocess in the brain 'e.$., blee"in$, infarction, tumors) is associate" with e"ema an" swellin$ of the !arenchyma, which inevitably lea"s to "is!lacement of structures an" com!ression of a"<acent re$ions. Cerebellar tonsillar herniation 'choice A) refers to "ownwar" "is!lacement of the cerebellar tonsils throu$h the foramen of ma$num. 7his results from s!ace4occu!yin$ lesions in the infratentorial com!artment, such as blee"in$ an" tumors. It lea"s to com!ression of the me"ulla an" "eath by car"iores!iratory arrest. )ubfalcine 'cin$ulate) herniation 'choice ) "escribes the lateral "is!lacement of the cin$ulate $yrus beneath the fal* cerebri. 7his event is cause" by s!ace4occu!yin$ masses in the cerebral hemis!here. It lea"s to com!ression of the anterior cerebral artery an" infarction of "e!en"ent cerebral territories 'mostly the me"ial !ortion of the frontal an" !arietal lobes). 6everse cerebellar herniation 'choice C) is a rare form of herniation "ue to mi"brain lesions 'a$ain, hemorrha$es an" tumors) that !ush the mi"brain u!war" throu$h the incisura of the tentorium. 7ranscalvarial herniation 'choice %) may "evelo! in o!en 'i.e., accom!anie" by calvarial bone fractures) hea" in<uries if brain !arenchyma is "is!lace" outsi"e the cranial cavity throu$h a calvarial "efect. ?I) (*!lanation: 7he correct answer is (. 7his !atient has fin"in$s of asym!tomatic aortic insufficiency on !hysical e*amination. 7his lesion may result from a number of causes, several of which are infectious in etiolo$y. Aortic insufficiency may result as a sequela of rheumatic heart

"isease, which occurs as an immunolo$ic res!onse to a stre!tococcal infection. Acute rheumatic fever is ty!ically characteri#e" by car"iac involvement that may cause !ericar"itis, myocar"itis, or en"ocar"itis. Often, the initial car"iac manifestations are asym!tomatic an" become a!!arent only years later with the "evelo!ment of car"iac valvular "isease. acteroi"es 'choice A) is a $ram4ne$ative anaerobe of intestinal ori$in, which, likewise, "oes not cause car"iac "isease. (scherichia coli'choice ), an enteric $ram4ne$ative or$anism, very rarely causes any form of car"iac involvement. 7re!onema !alli"um, the etiolo$y of sy!hilis, may cause aortic insufficiency in its tertiary sta$es. 2owever, se*ually transmitte" "iseases with this, Neisseria $onorrhoeae 'choice C), an" 2aemo!hilus "ucreyi'choice %) have no car"iac manifestations. />) (*!lanation: 7he correct answer is . A communication between an arteriole an" venule in the cecum is a "escri!tion of a vascular ectasia, also known as an arteriovenous 'A1) malformation. 7his a common cause of !ainless colonic blee"in$ in the el"erly an" may !resent with acute $astrointestinal blee"in$ 'as in this case), chronic $astrointestinal blee"in$, or iron4 "eficiency anemia. 7hese lesions may be "ifficult to "emonstrate, as the blee"in$ may be intermittent or the colon may be so full of bloo" that the site of ori$in is obscure". 7echniques use" to "emonstrate blee"in$ A1 malformations inclu"e colonosco!y, intrao!erative en"osco!y, an" visceral an$io$ra!hy. 7reatment of these lesions is !roblematic because many !atients will subsequently "evelo! new or recurrent blee"in$ vessels. A blee"in$ "iverticulum in the a!!en"i* 'choice A) "oes not occur. %iverticular blee"in$ is very common in the el"erly, but it is usually !resent in the si$moi" colon. A laceration of the $astroeso!ha$eal <unction 'choice C), i.e., a +allory4Weiss tear, may !ro"uce bri$ht re" bloo" !er rectum if the blee"in$ is !rofoun". 2owever, it more often !resents with a ty!ical history of vomitin$ an" retchin$ !rece"in$ the blee"in$, which is not "escribe" in this question. &erforation of the si$moi" "iverticulum 'choice %) "oes not result in blee"in$ but will result in acute "iverticulitis. 6ectal !oly!s 'choice () rarely !ro"uce lar$e volume bloo" loss.

USMLE Step 2 Practice Test Block 8 :ame; +nstr#ctions; Ans'er the *#estions $elo' to the $est of yo#r a$ility. =hen yo# finish the test% click the 2heck $#tton at the $ottom to )ie' the res#lts.

1.A 24!year!ol" "ress "esigner complains of a crampy peri#m$ilical pain. These symptoms ha)e $een present o)er the past 8 months since she $egan her first 9o$ after gra"#ating art school. @#ring that time% she has ha" se)eral episo"es of constipation lasting 4! "ays. These are typically follo'e" $y &!4 "ays of fre*#ent loose $o'el mo)ements. She "enies any $loo"y stools% fe)er% 'eight loss% or change in appetite. /er symptoms are generally mil"er on 'eeken"s. /er physical e.amination is normal. /er 'hite $loo" cell co#nt is <(447mm&% her hematocrit is &1B% an" her erythrocyte se"imentation rate is 4 mm7h. /er ser#m al$#min an" li)er f#nction tests are normal. =hich of the follo'ing is the most likely "iagnosisa6 2rohn "isease $6@i)ertic#losis c6Ciar"ia infection "6+rrita$le $o'el syn"rome e6Ulcerati)e colitis :ormal La$s 2. A &4!year!ol" ta. la'yer presents to his physician complaining of "iffic#lty s'allo'ing. >n se)eral occasions o)er the past fe' months% he 'as a'are of meat $ecoming st#ck in his mi"!chest imme"iately after eating. After each episo"e% he ha" se)eral ho#rs of chest pain% 'hich gra"#ally resol)e". >n t'o occasions% he in"#ce" )omiting to o$tain relief. >)er the past 14 "ays% the s'allo'ing "iffic#lty has $ecome

'orse% an" he no' has tro#$le 'ith e)en soft foo"s an" is limiting himself to p#ree" foo". /e has $een taking raniti"ine% magnesi#m hy"ro.i"e% an" omepraDole for 4 years% $#t has remaine" symptomatic "espite these meas#res. /e has $een smoking one pack of cigarettes "aily for 1 years an" "enies any alcohol #se. The physical e.amination is normal. =hich of the follo'ing is the most likely e.planation for these symptomsa6@iff#se esophageal spasm $6Esophageal s*#amo#s carcinoma c6Lo'er esophageal 'e$ "6Peptic esophageal strict#re e6Sclero"erma :ormal La$s &.A &8!year!ol" man comes to me"ical attention $eca#se of a 1!year history of personality changes% a$normal in)ol#ntary mo)ements% an" memory "ysf#nction. /is father an" gran"father "ie" in their 4s $eca#se of progressi)e mental "eterioration accompanie" $y mo)ement a$normalities. The patient is marrie" $#t has no chil"ren. :e#rologic e.amination an" psychometric testing re)eal "iffic#lty in concentration% mil" "epression% an" marke" restlessness. @#ring the e.amination% grimacing of the face an" intermittent shr#gging of the sho#l"ers are note". MA+ e.amination of the $rain re)eals hyperintensity in the region of the ca#"ate on T2!'eighte" images. =hich of the follo'ing is the most likely "iagnosisa62re#tDfel"t!Rako$ "isease $6Cilles "e la To#rette syn"rome c6/#ntington "isease "6Sy"enham chorea e6Tar"i)e "yskinesia :ormal La$s 4.A <4!year!ol" 'oman presents to her physician for management of her hypertension% 'hich has $een treate" #ns#ccessf#lly for se)eral years. She 'as recently hospitaliDe" for p#lmonary e"ema% an" an echocar"iogram at that time sho'e" a mo"erately "epresse" e9ection fraction. She 'as "iagnose" 'ith congesti)e heart fail#re. /er me"ications incl#"e a thiaDi"e "i#retic an" a calci#m channel $locker.

She has an allergy to f#rosemi"e. /er re)ie' of systems is positi)e for t'o!pillo' orthopnea an" occasional paro.ysmal noct#rnal "yspnea. >n physical e.amination% her $loo" press#re is 1 4714 mm /g% an" her p#lse is 147min an" reg#lar. /er l#ngs are clear% an" there are no e.tra heart so#n"s. /er e.tremities are 'itho#t e"ema. =hich of the follo'ing is the most appropriate management at this timea6A"" an A2E inhi$itor to her regimen $6A"" an angiotensin ++ receptor $locking agent to her regimen c6A"" hy"ralaDine to her regimen "6 +ncrease the "ose of her calci#m channel $locker e6+ncrease the "ose of her thiaDi"e "i#retic :ormal La$s .A malno#rishe" mi""le!age" homeless man is $ro#ght to the emergency "epartment. /e is "isoriente" to person% place% an" time an" #na$le to 'alk 'itho#t assistance. /is temperat#re is &(.4 2 381.4 56% $loo" press#re is 1&4714 mm /g% p#lse is 1<7min% an" respiratory rate is 117min. :e#rologic e.amination re)eals lateral nystagm#s. E)al#ation of strength an" sensation can not $e performe". =hich of the follo'ing is the most appropriate ne.t step in managementa6 A"ministration of "iaDepam $6+ntra)eno#s inf#sion of gl#cose c6+ntra)eno#s inf#sion of thiamin "6To.icological screening e6:e#roimaging st#"ies :ormal La$s <. A 28!year!ol" professional tennis player presents 'ith se)ere "iarrhea an" a$"ominal cramps o)er the past 'eek. She has $een in e.cellent health% an" her only me"ical history is a #rinary tract infection% treate" 2 'eeks ago 'ith amo.icillin. Three "ays ago% she $egan ha)ing left lo'er *#a"rant a$"ominal cramps% follo'e" $y "iarrhea that has $ecome increasingly prof#se. >)er the past 24 ho#rs% her temperat#re has increase" to &1.4 2 3141.1 56. /er physical e.amination is remarka$le for mo"erate peri#m$ilical an" left lo'er *#a"rant ten"erness. /er stool is g#aiac negati)e. =hich of the follo'ing 'o#l" most likely $e seen on sigmoi"oscopy-

a6@eep rectal #lcers 'ith normal sigmoi" m#cosa $6M#ltiple sigmoi" "i)ertic#la c6Aectosigmoi" e"ema an" patchy e.#"ates "6Aectosigmoi" strict#re e6Sessile sigmoi" mass :ormal La$s (.A 4!year!ol" man presents to the clinic 'ith a year long history of ha)ing "iffic#lty hol"ing an" #sing a 'riting instr#ment. /e reports that he "e)elops right han" an" forearm spasms 'hen 'riting% so that he cannot 0$en" his 'rist the right 'ay0. /e has $een healthy all his life an" is on no me"ications. /e "enies any "iDDiness or loss of conscio#sness or any history s#ggesti)e of a seiD#re. =hich of the follo'ing is the most likely "iagnosisa6 Benign essential tremor $62arpal t#nnel syn"rome c62er)ical ra"ic#lopathy "65ocal "ystonia e6Parkinson @isease :ormal La$s 1.A 4 !year!ol" male s#""enly loses conscio#sness an" falls to the gro#n". /e has $een pre)io#sly healthy an" has $een on no me"ications. There is no o$)io#s e)i"ence of tra#ma. An electrocar"iogram re)eals 'i"e comple. tachycar"ia at a rate of &44 $eats per min#te. =hich of the follo'ing is the most appropriate inter)entiona6>$tain )ital signs $6A"minister a $ol#s of intra)eno#s li"ocaine c6A"minister a throm$olytic agent "6 Perform asynchrono#s car"io)ersion e6 Perform synchrono#s car"io)ersion :ormal La$s 8.A < !year!ol" man comes to me"ical attention $eca#se of m#ltiple ne#rologic "eficits% incl#"ing loss of sensation in his right han"% 'eakness of the left lo'er leg% an" a )is#al fiel" "eficit. /e has smoke"

t'o packs of cigarettes "aily for 44 years. /is me"ical history is remarka$le for asthma an" emphysema. E.amination of the f#n"#s is #nremarka$le. /is temperat#re is &( 2 381.< 56% $loo" press#re is 1&(71< mm /g% p#lse is 1<7min% an" respirations are 247min. MA+ of the hea" re)eals fi)e "ifferent intracere$ral lesions% ranging from 1 to & cm in main "iameter an" locate" at the gray!'hite matter 9#nction in $oth cere$ral hemispheres. The lesions are sharply "emarcate"% an" contrast enhancement after ga"olini#m a"ministration is present in all of them. =hich of the follo'ing is the most likely "iagnosisa6A$scesses $6Arterio)eno#s malformations c6Em$olic infarcts "6Metastases e6M#ltifocal glio$lastoma m#ltiforme :ormal La$s 14. A 22!year!ol" man is "iagnose" 'ith 2rohn "isease limite" to the terminal ile#m. /is symptoms of mil" right lo'er *#a"rant pain an" postpran"ial "iarrhea resol)e after the initiation of treatment 'ith mesalamine. T'o years later% he "e)elops rec#rrent episo"es of a$"ominal "istention% na#sea% an" )omiting after large meals. >n t'o occasions% these symptoms are accompanie" $y ina$ility to pass flat#s or $o'el mo)ements. =hich of the follo'ing has this patient most likely "e)elope"a65i$rosis an" a strict#re in the terminal ile#m $6A fist#la from the ile#m to the sigmoi" c6Castric o#tlet o$str#ction "6An o$str#cting cecal carcinoma e6An o$str#cting ileal carcinoi" :ormal La$s 11. >c#lar e.amination is performe" on a patient "#ring a ro#tine me"ical check #p. Aetinal e.amination "emonstrates a generaliDe" retinal arteriolar constriction. The light refle. on the arterioles is $roa" an" "#ll. T'o areas of flame!shape" hemorrhages an" m#ltiple cotton 'ool spots are also seen. These fin"ings are most s#ggesti)e of 'hich of the follo'inga62entral retinal artery occl#sion

$62entral retinal )ein occl#sion c6/ypertensi)e retinopathy "6:on!proliferati)e "ia$etic retinopathy e6 Proliferati)e "ia$etic retinopathy :ormal La$s 12. A &4 year!ol" female 'ith /+? an" acti)e Pne#mocystis pne#monia is a"mitte" to the hospital. /er last 2@4 co#nt 'as 44 cells7mm&. She has $een "oing reasona$ly 'ell since a"mission 'ith a sta$le co#rse on appropriate anti$iotics. T'o "ays into the hospitaliDation% she is fo#n" to $e hypotensi)e an" tachycar"ic. /er $loo" press#re is 14744 mm /g an" her p#lse is 1147min. /er temperat#re is &1.& 2 3141 56 orally. /er e.tremities are cool an" "amp. /er mental stat#s e.amination is normal. The remain"er of her physical e.amination is #nchange". =hich of the follo'ing is the most appropriate therapya6+ntra)eno#s fl#i"s $6+ntra)eno#s pressor s#pport c62entral )eno#s pressor s#pport "6A"" a""itional anti$iotics to treat empiric sepsis e6Bloo" transf#sion :ormal La$s 1&.A 24!year!ol" man has ha" a persistent hea"ache an" a fe)er for a 'eek. /e no' presents 'ith a seiD#re. /e has no prior me"ical history an" is on no me"ications. >n physical e.amination% he has a high!gra"e fe)er. /e has left!si"e" 'eakness. /e has a rooting refle. an" appears "isinhi$ite" in his $eha)ior. A 2T scan of the hea" sho's a ring! enhancing lesion in the right frontal lo$e an" an air!fl#i" le)el in the right frontal sin#s. =hich of the follo'ing 'ill likely $e seen on aspiration of the lesiona6 Alpha hemolytic Streptococc#s an" mi.e" anaero$es $6Bacteroi"es fragilis c6B#""ing yeast organisms 'ith hyphae "6Small monon#clear cells s#ggesti)e of B#rkitt lymphoma e6To.oplasma gon"ii cysts :ormal La$s 14. A &2!year!ol" former heroin a""ict has a history of $eing treate" for Pne#mocystis carinii pne#monia t'ice in the past year. >)er the past 2

'eeks% he has "e)elope" $loo"y "iarrhea% 'hich occ#rre" #p to 12 times "aily an" is associate" 'ith #rgency an" the sensation of incomplete e)ac#ation. /e also complains of left lo'er *#a"rant cramping. /e "enies any history of homose.#al acti)ity. =hich of the follo'ing organisms 'o#l" most likely $e responsi$le for his symptomsa6 2ryptospori"i#m $62ytomegalo)ir#s c6Entamoe$a histolytica "6 Enteroto.igenic Escherichia coli e6Escherichia coli 41 ( :ormal La$s 1 .A <4!year!ol" man is $ro#ght to the emergency room one ho#r after "e)eloping the s#""en onset of hea"ache. /e appears conf#se" to time an" space% an" is only a$le to tell his name an" "ate of $irth. The right p#pil is "ilate" an" poorly reacti)e to light. /is 'ife relates that he has s#ffere" from hypertension since the age of 44% $#t has not taken his antihypertensi)e treatment reg#larly. /is temperat#re is &( 2 381.4 56% $loo" press#re is 1(47144 mm /g% p#lse is 147min an" reg#lar% an" respirations are 247min. =hich of the follo'ing is the most appropriate ne.t step in "iagnosisa6Electroencephalographic e.amination $62T scan of the hea" c6MA+ scan of the hea" "6To.icological analysis of $loo" an" #rine e6L#m$ar p#nct#re for 2S5 e.amination :ormal La$s 1<.A 41!year!ol" man presents 'ith a 2!'eek history of fe)er% anore.ia% 'eight loss% an" fatig#e. /e is other'ise healthy an" has not seen a physician recently% $#t "i" recently ha)e his teeth cleane". /e is on no me"ications an" has no allergies. /e "rinks alcohol only occasionally an" "enies +? "r#g #se. >n physical e.amination% he appears ill% 'ith a temperat#re of &1.8 2 3142 56 an" a fe' petechiae in $oth eyes. There is a +++7?+ systolic e9ection m#rm#r consistent 'ith mitral reg#rgitation an" a pericar"ial r#$. Bloo" is "ra'n an" sent to the la$oratory for c#lt#re. =hich of the follo'ing is most likely to confirm the "iagnosisa6 E2C

$6Transthoracic echocar"iogram 3TTE6 c6Stress test "6Transesophageal echocar"iogram 3TEE6 e62ar"iac catheteriDation :ormal La$s 1(. A 44!year!ol" o$ese 'oman presents to the emergency "epartment complaining of & ho#rs of se)ere a$"ominal pain. She has also ha" m#ltiple episo"es of )omiting "#ring this time. She "escri$es the pain as 0'orse than la$or%0 an" it ra"iates to the interscap#lar region. /er temperat#re is &1.8 2 3142 56% an" she has se)ere ten"erness in her right #pper *#a"rant. She reports that she has ha" m#ltiple similar episo"es in the past that ha)e laste" appro.imately &4 min#tes an" then resol)e" spontaneo#sly. =hich of the follo'ing is most likely $eing o$str#cte" $y a gallstonea62ommon $ile "#ct $62ommon hepatic "#ct c62ystic "#ct "6Pancreatic "#ct e6Aight hepatic "#ct :ormal La$s 11. The 'ife of an el"erly patient 'ith chronic o$str#cti)e p#lmonary "isease 32>P@6 calls the physician $eca#se her h#s$an" is e.tremely short of $reath an" appears 0$l#e.0 The patient has a long history of 2>P@ an" has $een compliant 'ith his me"ications of al$#terol inhalers% rotating anti$iotics% an" theophylline. /is 'ife reports that% o)er the past se)eral "ays% he has "e)elope" an increasingly pro"#cti)e co#gh. The patient is $ro#ght to the hospital $y am$#lance. At the hospital% the patient is $arely responsi)e. /e is $reathing 144B o.ygen )ia a tight!fitting face mask an" no longer appears cyanotic. =hich of the follo'ing is the most likely e.planation for his #nresponsi)enessa6 Aspiration $6/yper)entilation c6/ypo)entilation "6Myocar"ial ischemia e6P#lmonary em$ol#s :ormal La$s

18.The longtime primary care physician of an 1 year!ol" 'oman is aske" to help me"iate care $et'een the family an" the hospital,s me"ical ser)ice. The patient s#staine" an ano.ic $rain in9#ry "#ring an in!hospital car"iac arrest one 'eek ago in 'hich the patient ha" a p#lseless perio" for at least min#tes. >n physical e.amination% her )ital signs are normal an" sta$le 'ith the assistance of contin#o#s mechanical )entilation. P#pillary an" corneal refle.es are present $ilaterally. There is episo"ic "ecorticate rigi"ity% $#t no p#rposef#l mo)ement present. An electroencephalogram 3EEC6 s#ggests se)ere% "iff#se cortical "amage. The patient,s h#s$an" asks the physician if she is $rain "ea". =hich of the follo'ing is the most appropriate responsea6+t is too early to pre"ict $rain "eath $y the legal "efinition $6The "iagnosis of $rain "eath can only legally $e ma"e $y a ne#rologist c6The "ecision on $rain "eath m#st a'ait the completion of a magnetic resonance image 3MA+6 "6The presence of $rain stem f#nction an" post#ring r#les o#t $rain "eath% $#t the e.amination fin"ings an" s#pporti)e "ata s#ggest e.tensi)e $rain "amage e6 The s#ggestion of se)ere cortical "amage $y the EEC implies $rain "eath :ormal La$s 24.A &2!year!ol" man presents complaining of se)ere pr#rit#s o)er the past 2 'eeks. /e has a history of #lcerati)e colitis for the past ( years% 'hich has remaine" 'ell controlle" on s#lfasalaDine an" cortisone enemas. /is physical e.amination is #nremarka$le e.cept for e)i"ence of "iff#se e.coriations on his e.tremities an" tr#nk. La$oratory st#"ies re)eal a mil" iron "eficiency anemia an" normal electrolytes. Li)er f#nction tests are normal% e.cept for an alkaline phosphatase that is &22 U7L 3normal% Q114 U7L6. =hich of the follo'ing is the most likely e.planation for his symptomsa6 Erythema no"os#m $6/epatitis 2 c6Primary $iliary cirrhosis "6Primary sclerosing cholangitis e6 Pyo"erma gangrenos#m :ormal La$s

21. A <8!year!ol" retire" physician is $ro#ght to the emergency "epartment $y his 'ife $eca#se of the onset of se)ere mi"a$"ominal pain. /e a'oke at appro.imately 4;44 AM 'ith se)ere pain% 'hich has gra"#ally $ecome 0#n$eara$le.0 /e has a history of rhe#matic fe)er% ac#te cholecystitis res#lting in cholecystectomy% an" a gastric #lcer. >n physical e.amination% he appears ac#tely #ncomforta$le an" complains of increasing pain 'ith any mo)ements. /is temperat#re is &1.( 2 3141.< 56% $loo" press#re is 1<4784 mm /g% an" p#lse is 1447min. /e is anicteric an" has "ry m#co#s mem$ranes. >n a$"ominal e.amination% there is re"#ction in $o'el so#n"s an" "iff#se ten"erness an" in)ol#ntary g#ar"ing to mil" palpation. A rectal e.amination re)eals $ro'n% g#aiac!negati)e stool. Upright chest .!ray an" plain a$"ominal films re)eal free air #n"erneath the left hemi"iaphragm. =hich of the follo'ing is the most appropriate ne.t step in managementa6 A$"ominal 2T scan $6/igh!"ose oral omepraDole c6/istamine!2 receptor antagonist "6>$ser)ation after placement of a nasogastric t#$e e6 Emergent laparotomy :ormal La$s 22.A (2!year!ol" alcoholic man is $ro#ght to the emergency "epartment $y his chil"ren. They state that he has $ecome increasingly conf#se" o)er the past 'eek an" is #nstea"y on his feet. >n physical e.amination% ata.ic gait% $ilateral paresis of the lateral gaDe% an" )ertical an" horiDontal nystagm#s are note". =hich of the follo'ing is the most appropriate ne.t step in managementa6A"ministration of +? "e.trose $6A"ministration of +? mannitol c6A"ministration of +? thiamine "62T of the $rain 'ith contrast e6 L#m$ar p#nct#re an" e.amination of the 2S5 :ormal La$s 2&.A 24!year!ol" me"ical st#"ent complains of mi"epigastric pain that she "escri$es as a 0"#ll ache0 that is relie)e" $y eating. She has a'akene" from sleep on se)eral occasions at 2 AM $eca#se of se)ere

e.acer$ation of these symptoms% 'hich are relie)e" 'ith magnesi#m hy"ro.i"e. She takes fre*#ent acetaminophen for menstr#al cramping. =hich of the follo'ing is the most likely ca#se of her symptomsa6A#tonomo#s gastrin secretion $6Cram!negati)e organism c6Cram!positi)e organism "6Prostaglan"in inhi$ition e6?agal inhi$ition :ormal La$s 24. A (4!year!ol" man presents to the emergency "epartment complaining of a$"ominal pain. /e "escri$es the pain as crampy an" primarily in his left lo'er *#a"rant. /e has ha" minimal na#sea% $#t complains of constipation. /is past me"ical history is significant for hypertension% hyperlipi"emia% go#t% an" "i)ertic#losis. /is me"ications incl#"e atenolol an" sim)astatin. /e is allergic to penicillin. /is temperat#re is &1.4 2 3144.4 56% $loo" press#re is 1447<4 mm /g% p#lse is 1447min% an" respirations are 247min. /is physical e.amination is significant for ten"erness to palpation at the left lo'er *#a"rant 'itho#t re$o#n" or g#ar"ing. /is rectal e.amination is g#aiac negati)e. /is heart an" l#ng e.aminations are #nremarka$le. =hich of the follo'ing is the most likely "iagnosisa6Appen"icitis $6@i)ertic#litis c6@i)ertic#losis "6+schemic colitisSigmoi" )ol)#l#s e6 Sigmoi" )ol)#l#s :ormal La$s 2 .A !year!ol" man comes to the physician $eca#se of "aily hea"aches a'akening him in the early morning. >ften% the pain is e.acer$ate" $y ?alsal)a mane#)ers% s#ch as co#ghing. /e has rarely ha" hea"aches $efore. 2ommon analgesics% s#ch as aspirin an" nonsteroi"al anti! inflammatory "r#gs 3:SA+@s6% ha)e pro)i"e" no $enefits. The patient "enies 'eight loss. /e takes enalapril for mo"erately se)ere hypertension. Bloo" press#re is 'ithin normal limits at this time. :e#rologic e.amination re)eals mil" hypoesthesia in the right han" an" "iminishe" strength in the right arm. =hich of the follo'ing is the most appropriate ne.t step in management-

a6 Trial 'ith "ifferent :SA+@s $62hange in antihypertensi)e me"ication c6S#matriptan treatment "6:e#roimaging st#"ies e6L#m$ar p#nct#re :ormal La$s 2<. A 14!year!ol" $oy presents to the emergency "epartment 'ith hea"ache% na#sea% an" )omiting for the past & "ays. :e#rologic e.amination re)eals n#chal rigi"ity an" papille"ema. A 2T scan re)eals an infiltrating cere$ellar t#mor% 'hich is locate" in the mi"line 3)ermis6% 'ith pla*#e!like e.tensions onto the cere$ellar s#rface. There is no cystic component. The fo#rth )entricle is compresse"% an" the thir" an" lateral )entricles are "ilate". =hich of the follo'ing is the most likely "iagnosisa6Epen"ymoma $6/emangio$lastoma c6Me"#llo$lastoma "6Meningioma e6>ligo"en"roglioma f6Pilocytic astrocytoma :ormal La$s 2(. A 24!year!ol" man 'ith a kno'n history of asthma presents to the emergency "epartment complaining of ho#rs of se)ere 'heeDing an" shortness of $reath. /e has #se" his $roncho"ilator inhaler si. times "#ring the past ho#rs $#t 'ith only minimal relief. >n physical e.amination% he appears "yspneicG his temperat#re is &(.4 2 388.& 56% $loo" press#re is 1117<4 mm /g% p#lse is 14<7min% an" respirations are &27min. There is )isi$le #se of the sternoclei"omastoi" m#scles 'ith each inspiration. A l#ng e.amination re)eals $ilateral "iff#se inspiratory an" e.piratory 'heeDing 'ith poor air mo)ement an" a prolonge" e.piratory phase. =hich of the follo'ing 'ill most likely $e fo#n" on a chest .!ray filma6 Bilateral interstitial infiltrate $6Lo$ar consoli"ation c6Ple#ral eff#sion

"6Pne#mothora. e6:ormal fin"ings :ormal La$s 21.A 44!year!ol" school $#s "ri)er presents to the emergency "epartment complaining of se)ere a$"ominal pain. She reports that the pain $egan appro.imately 1 ho#rs ago% after eating l#nch at a fast!foo" resta#rant. The pain has $ecome increasingly se)ere an" ra"iates to her $ack. She recalls a similar episo"e% lasting & ho#rs% 2 months earlier an" another episo"e% lasting 12 ho#rs% "#ring her last pregnancy. She is fe$rile% 'ith a temperat#re of &1.1 2 3144. 56% an" has right #pper *#a"rant ten"erness 'ith "eep palpation of this area. A rectal e.amination re)eals $ro'n% g#aiac!negati)e stool. /er =B2 is 12%8447mm&% an" her hematocrit is &8B. /er total $ilir#$in is 2.1 mg7"L. =hich of the follo'ing is the most appropriate "iagnostic testa62T scan of the a$"omen an" pel)is $6En"oscopic retrogra"e cholangiopancreatography 3EA2P c6/+@A scan "6Perc#taneo#s transhepatic cholangiogram 3PT26 e6Upper gastrointestinal $ari#m st#"y :ormal La$s 28.A 2!year!ol" man 'ith a history of emphysema spen"s a 2!'eek )acation on a cr#ise ship. Shortly after ret#rning home% he "e)elops high fe)ers an" $ecomes lethargic an" "isoriente". /is 'ife "escri$es that he has $een co#ghing an" short of $reath since ret#rning home. She also "escri$es that he has )omite" se)eral times o)er the past 41 ho#rs an" has ha" "iarrhea. >n physical e.amination% he appears lethargic $#t aro#sa$le. /e is "isoriente" to the c#rrent "ate. /e has lo#"% coarse% rhonchi in $oth l#ng fiel"s. /is a$"ominal e.amination re)eals mil" ten"erness o)er the li)er e"ge. There is no splenomegaly or ascites present. /is ne#rologic e.amination is nonfocal. La$oratory res#lts are nota$le for an aspartate aminotransferase 3AST6 of 112 U7L an" an alanine aminotransferase 3ALT6 of 1 ( U7L. =hich of the follo'ing is the most appropriate treatment for this patienta6+? ceftaDi"ime $6+? ceftaDi"ime c6+? gentamicin "6+? nafcillin

e6+? )ancomycin :ormal La$s &4.A 4(!year!ol" car salesman presents to the clinic complaining of right lo'er *#a"rant pain that $egan 41 ho#rs earlier. /e has $een pre)io#sly healthy an" 'itho#t any history of gastrointestinal "isease. /e "enies na#sea or )omiting. >)er the past 41 ho#rs% he has "e)elope" a fe)er an" non!$loo"y "iarrhea. /is temperat#re is &1.& 2 3144.8 56% an" there is localiDe" right lo'er *#a"rant ten"erness 'itho#t mass. A rectal e.amination re)eals 'atery $ro'n stool that is g#aiac negati)e. =hich of the follo'ing is the most likely e.planation for this patient,s symptomsa62ampylo$acter 9e9#ni infection $6Salmonella enteriti"is infection c6Shigella "ysenteriae infection "6Staphylococc#s a#re#s infection e6 Uersinia enterocolitica infection :ormal La$s &1. A &4!year!ol" international in)estment $anker presents 'ith a &! month history of fre*#ent episo"es of loose stool prece"e" $y left lo'er a$"ominal cramping. 5or the past < 'eeks% the stools ha)e $ecome increasingly $loo"y. >n a n#m$er of occasions% he has ha" a sensation of rectal f#llness $#t has $een #na$le to pass any fecal matter. /e tra)els e.tensi)ely an" has $een to Asia% +n"ia% Pakistan% Cermany% an" S'e"en in past year 'orking on telecomm#nications infrastr#ct#re "eals. >n physical e.amination% he has mil" ten"erness in the left lo'er *#a"rant. A rectal e.amination re)eals grossly $loo"y stools. A sigmoi"oscopy re)eals inflammation e.ten"ing in a symmetric an" circ#mferential pattern from the anal )erge to the "istal "escen"ing colon. M#ltiple stool tests are negati)e for $acterial an" parasitic infections. =hich of the follo'ing is the most likely ca#se of the patient,s symptomsa6 2rohn "isease $62ytomegalo)ir#s c6+schemic colitis "6Ulcerati)e colitis e6Uersinia enterocolitica

:ormal La$s &2.A (!year!ol" man is referre" for a colonoscopy as a colon cancer screening e.amination. /e has $een in goo" health 'itho#t any recent change in $o'el ha$its% or thin cali$er stools% gastrointestinal $lee"ing% or #ninten"e" 'eight loss. There is no family history of colonic polyps or gastrointestinal malignancy. /is physical e.amination is #nremarka$le. A rectal e.amination re)eals no masses% an" his stool is g#aiac negati)e. A sigmoi"oscopy re)eals a 4!mm polyp in the mi"! rect#m. +t is remo)e" 'ith a forceps% an" the histology re)eals it to $e a t#$#lar a"enoma. =hich of the follo'ing is the most appropriate ne.t step in managementa6Sche"#le colonoscopy in & years $6Sche"#le colonoscopy no' c6Sche"#le colonoscopy in years "6Sche"#le sigmoi"oscopy in 1 year e6 Sche"#le a $ari#m enema in the imme"iate near f#t#re :ormal La$s &&. A 18!year!ol" 'oman presents 'ith a 2!'eek history of fre*#ent episo"es of loose stools. The symptoms are accompanie" $y se)ere fecal #rgency% an" she has a'oken 'ith "iarrhea se)eral nights 'eekly. >)er the past 'eek% the stools ha)e $ecome increasingly $loo"y. A sigmoi"oscopy re)eals contin#o#s% symmetric inflammation from the anal )erge to the pro.imal sigmoi" colon. =hich of the follo'ing infections is most likely ca#sing this patient,s symptomsa62ampylo$acter 9e9#ni $62ryptospori"i#m c6Ciar"ia lam$lia "6/erpes simple. e6 Uersinia enterocolitica :ormal La$s &4.A 21!year!ol" man 'ith a 1 !year history of #lcerati)e colitis an" primary sclerosing cholangitis has recent 'orsening of his 9a#n"ice. /is symptoms of #lcerati)e colitis ha)e $een in remission for the past year. /e no' complains of the onset of steatorrhea appro.imately & months

ago an" a 12!po#n" 'eight loss "#ring that time. =hich of the follo'ing 'o#l" most likely acco#nt for this patient,s recent symptomsa6Bacterial o)ergro'th $6Bile salt "eficiency c62eliac spr#e "6Tropical spr#e e6=hipple "isease :ormal La$s & .A 2&!year!ol" me"ical st#"ent ret#rns from s#mmer )acation in the 2ari$$ean% 'here he 'orke" as a )ol#nteer in a local emergency "epartment. @#ring his last 'eek in Ramaica% he "e)elope" 'atery "iarrhea an" mil" a$"ominal cramping. The symptoms ha)e persiste" since his ret#rn to the U.S. At present% he reports that he is ha)ing #p to 14 'atery% non!$loo"y $o'el mo)ements per "ay an" poorly localiDe" a$"ominal "iscomfort. >n physical e.amination% he is afe$rile an" there is no a$"ominal ten"erness. /is stool is light $ro'n an" g#aiac negati)e. =hich of the follo'ing is the most likely e.planation for his symptomsa6Enteroin)asi)e Escherichia coli $6Enteroto.igenic E. coli c6 E. coli 41 (; /( "6Tropical spr#e e6 ?i$rio cholerae :ormal La$s &<. A patient 'ith a history of hypertension calls his physician,s office for a")ice. /e has ha" longstan"ing heart$#rn an" recently cons#lte" 'ith a gastroenterologist. /e #n"er'ent an en"oscopy an" 'as tol" that 0Barrett,s m#cosa0 'as fo#n" $y $iopsy. The patient has rea" in the ne'spaper that people 'ith this con"ition 'ill pro$a$ly "e)elop esophageal cancer. =hich of the follo'ing is the most appropriate response to this concerna60Uo#r concerns are #ngro#n"e"0 $60+t is foolish to 'orry $eca#se this type of cancer is c60Uo# sho#l" che' foo" )ery caref#lly to pre)ent the possi$ility of a mechanical o$str#ction0

"6 0>nly a small minority of patients 'ith Barrett,s esophag#s 'ill "e)elop cancer% an" yo# sho#l" #n"ergo en"oscopic s#r)eillance0 e60Uo# sho#l" cons#lt 'ith an oncologist regar"ing esophageal cancer pre)ention strategies0 :ormal La$s &(.A &4!year!ol" $#s "ri)er presents 'ith complaints of "iffic#lty s'allo'ing. The symptoms $egan gra"#ally% appro.imately 8 months ago% an" ha)e pre)ente" him from che'ing soli"s or "rinking li*#i"s comforta$ly. /e fin"s himself a'akene" at night 'ith co#gh an" occasional morning reg#rgitation of recogniDa$le foo" from the night $efore. /e has learne" to re"#ce his oral intake an" has lost < po#n"s o)er the past 2 months. /e "oes not smoke or "rink an" has no family history of esophageal or other gastrointestinal illnesses. /is physical e.amination is #nremarka$le. =hich of the follo'ing is the most likely "iagnosisa6Achalasia $6Esophageal a"enocarcinoma c6Lo'er esophageal 'e$ "6Peptic strict#re e6Progressi)e systemic sclerosis :ormal La$s &1.A man of Polynesian "escent is taken to the emergency "epartment $eca#se of se)ere a$"ominal pain. The pain% 'hich $egan a$o#t & ho#rs earlier% is peri#m$ilical. She )omite" shortly after the pain starte"% $#t the na#sea mostly s#$si"e" $y the time she arri)e" at the hospital. >n physical e.amination% she appears ac#tely ill an" is perspiring. /er temperat#re is &1.1 2 3144.< 56% $loo" press#re is 114714 mm /g% p#lse is 1147min an" reg#lar an" respirations are 227min. The chest is clear to a#sc#ltation. >n a$"ominal e.amination% there is ten"erness in the right lo'er *#a"rantG this pain is also elicite" 'hen press#re is applie" to the nonten"er left lo'er *#a"rant. The remain"er of the physical e.amination is #nremarka$le. A stat complete $loo" co#nt "emonstrates an erythrocyte co#nt of 4.& million7mm&% a le#kocyte co#nt of 22%4447mm& 'ith <4B segmente" ne#trophils an" 24B $an" forms% an" a platelet co#nt of &44%4447mm&. =hich of the follo'ing is the most appropriate ne.t step in management-

a6Bari#m enema $62T scan of a$"omen c6Esophagogastro"#o"enoscopy "6Laparotomy e6Ultraso#n" of a$"omen an" pel)is :ormal La$s &8.A <1!year!ol" 'oman reports that she ha" an operation 44 years earlier for intracta$le #lcer symptoms. She recalls that a large portion of her stomach 'as remo)e". Ae)ie' of her prior me"ical recor"s in"icates that she #n"er'ent an antrectomy an" a gastro9e9#nostomy. =hich of the follo'ing is she most at risk for "e)elopinga6 Bacterial o)ergro'th $6Blee"ing from the lo'er gastrointestinal tract c62hronic pancreatitis "62onstipation e6Pancreatic ins#fficiency :ormal La$s 44.A 28!year!ol" carpenter presents to the emergency "epartment 'ith his thir" episo"e of #pper gastrointestinal $lee"ing in the past year. >n $oth prior occasions% he 'as fo#n" to $e $lee"ing from a "#o"enal #lcer% an" the $lee"ing 'as controlle" en"oscopically. /e also reports ha)ing #lcers in the "istal "#o"en#m% 9e9#n#m% an" a""itional #lcers t'ice in the past 4 years. /e "oes not take any nonsteroi"al anti! inflammatory "r#gs 3:SA+@s6 an" has teste" negati)e for /elico$acter pylori. =hich of the follo'ing is the most likely ca#se of his rec#rrent symptomsa6A#toimm#ne "estr#ction of the parietal cells $6A#tonomo#s pro"#ction of gastrin c6A#tonomo#s pro"#ction of somatostatin "6+mpaire" pro"#ction of the m#co#s $icar$onate layer in the stomach e6 +nhi$ition of parietal cells secretion :ormal La$s 41.A <!year!ol" alcoholic presents complaining of < months of 'orsening mi"epigastric pain ra"iating to his $ack. The pain is

e.acer$ate" $y eating an" contin#e" alcohol #se. >ne month ago he 'as tol" that he ha" "e)elope" "ia$etes. /e has also note" that% o)er the past 2 months% he has ha" greasy% fo#l!smelling% large!)ol#me stools. =hich is of the follo'ing is the most likely ca#se of this man,s steatorrheaa6Aci" inacti)ation of pancreatic enDymes $6Bacterial o)ergro'th c6 Biliary ins#fficiency "6E.cess secretion of gl#cagon e6+ns#fficient lipase a)aila$le for normal fat "igestion :ormal La$s 42.A 24!year!ol" gra"#ate st#"ent is fo#n" to ha)e ac#te myelogeno#s le#kemia. Before #n"ergoing a planne" $one marro' transplant% he $egins aggressi)e m#lti!"r#g chemotherapy. /o'e)er% his tolerance of this regimen is se)erely limite" $y intracta$le )omiting 'ith each chemotherapy cycle. =hich of the follo'ing 'ill most likely re"#ce )omiting in this patienta6 @opamine $6Erythromycin c6>mepraDole "6>n"ansetron e6>piates :ormal La$s 4&.A <4!year!ol" man presents to the emergency room 'ith se)ere a$"ominal pain. Physical e.amination "emonstrates ten"er% smooth hepatomegaly an" mil" 9a#n"ice. Ser#m chemistries "emonstrate an aspartate aminotransferase 3AST6 an" alanine aminotransferase 3ALT6 of o)er ten times the #pper limit of normal. The hematocrit is <&B. The patient has no kno'n history of smoking or l#ng "isease. 2omplete $loo" co#nt sho's an increase" n#m$er in all cell lines% 'ith the most marke" ele)ation in the erythrocyte line. The morphology of the cells in the peripheral smear is normal. =hich of the follo'ing is the most likely ca#se of this patient,s li)er f#nction a$normalitiesa6 Alcoholic cirrhosis $6B#""!2hiari syn"rome c62holelithiasis

"62hronic hepatitis B e65#lminant hepatitis A :ormal La$s 44. A <1!year!ol" man is fo#n" to ha)e a malignant polyp in the cec#m. /e #n"ergoes a right hemicolectomy% an" the specimen re)eals a 2!cm a"enocarcinoma e.ten"ing into% $#t not thro#gh% the m#sc#laris propria. Ele)en lymph no"es are negati)e% an" there is no e)i"ence of "istant metastatic sprea". /e reco)ers #ne)entf#lly an" is "ischarge" home on the <th postoperati)e "ay. 5o#r 'eeks later% he "e)elops s#""en onset of a$"ominal "istention 'ith )omiting an" the ina$ility to pass flat#s. An a$"ominal e.amination re)eals "istention% "iff#se mil" ten"erness% an" hyperacti)e r#shing $o'el so#n"s in the mi"!a$"omen. A rectal e.amination re)eals no masses an" $ro'n% g#aiac!negati)e stool. An a$"ominal .!ray film re)eals m#ltiple air!fl#i" le)els in the small intestine. =hich of the follo'ing is the most appropriate ne.t step in managementa6Placement of a nasogastric t#$e $6. Small $o'el series c6Bari#m enema "6A$"omen an" pel)ic 2T scan e6 S#rgical e.ploration :ormal La$s 4 .A 42!year!ol" 'oman presents to a physician after se)eral months of intermittent a$"ominal cramps an" "iarrhea. These episo"es are accompanie" $y skin fl#shing that is most prono#nce" in the hea" an" neck area. Sometimes her lips t#rn slightly $l#e "#ring these episo"es. Physical e.amination is nota$le for a m#rm#r hear" o)er the tric#spi" )al)e. Urine special st#"ies for !hy"ro.yin"oleacetic aci" sho' e.cretion of ( mg7"ay Jreference range 4. !8.4 mg7"ayK. 2T scan of the li)er "emonstrates a 2!cm lesion. :ee"le $iopsy of this lesion 'o#l" most likely sho' 'hich of the follo'inga62arcinoi" t#mor% metastatic $62holangiocarcinoma% primary c6/epatocell#lar carcinoma% primary "6M#cino#s a"enocarcinoma of the colon% metastatic e6S*#amo#s cell carcinoma of the esophag#s% metastatic

:ormal La$s 4<.A 1!year!ol" man #n"ergoes a $ari#m enema as a colon cancer screening e.amination. A &!mm polyp is fo#n" in the ascen"ing colon. +t is ro#n"e" an" smooth an" has met all the ra"iographic criteria of a $enign sessile polyp. :o other lesions are seen 'ithin the colon. =hich of the follo'ing is the most appropriate managementa6Aepeat the $ari#m enema in 1 year $6Sche"#le a colonoscopy in 1 year c6Sche"#le a sigmoi"oscopy no' "6Sche"#le a colonoscopy no' e6 Sche"#le a 2T scan to r#le o#t no"al in)ol)ement :ormal La$s 4(.A 24!year!ol" 2n"!year me"ical st#"ent presents to the st#"ent health clinic 'ith complaints of chronic epigastric pain 1 'eek $efore taking Step 2 of the USMLE. The patient has $een a'akene" from sleep $y epigastric pain at appro.imately & AM on se)eral occasions. Aaniti"ine has impro)e" his symptoms transiently. Eating generally impro)es his symptoms for a$o#t &4 min#tes% $#t is then follo'e" $y increase" pain. =hich of the follo'ing is the most likely ca#se of his symptomsa6@iff#se gastric m#cosal )asoconstriction $6Castrin!pro"#cing t#mor c6+nfection or organism in)a"ing the antral m#cosa "6>rganism coloniDing the gastric antr#m e6Ae"#ce" gastrin release 41.A 22!year!ol" pregnant 'oman "e)elops the gra"#al onset of se)ere lo' $ack pain "#ring her 14th 'eek of pregnancy. She $egins taking i$#profen for control of her symptoms. She has pre)io#sly ha" a $lee"ing #lcer an" 'as prescri$e" misoprostol% 'hich she $egan taking to pre)ent #lcer rec#rrence. 5orty!eight ho#rs later% she has se)ere )aginal $lee"ing% an" it is "etermine" that she has ha" a complete" a$ortion. =hich of the follo'ing is the most likely e.planation for this occ#rrencea6 Antiphospholipi"!in"#ce" throm$osis $62ycloo.ygenase!in"#ce" #terine contractions c65actor ?+++ "eficiency3in"#ce" hemorrhage

"6Lipo.ygenase!in"#ce" #terine contractions e6 Prostaglan"in!in"#ce" #terine contractions


Note: Check your own answers before hittin$ the Check button below. When you click the Check button, a browser win"ow will a!!ear that contains a summary of your results. (*!lanations Block 8 E.planations

:) (*!lanation: 7he correct answer is %. 7his !atient has ty!ical cram!y ab"ominal sym!toms with alternatin$ "iarrhea an" consti!ation. )he is a youn$ in"ivi"ual with chronic sym!toms, which is very ty!ical of irritable bowel syn"rome. &atients with this "ia$nosis will $enerally have sym!toms of !ain relate" to meals or stress an" an alteration in bowel habits. 7his chan$e in bowel !attern may take the form of frequent loose stools versus consti!ation, or a combination of both. 7hese !atients have no !hysical fin"in$s or laboratory results to su$$est an inflammatory !rocess. %ietary chan$es an" stress re"uction are usually the $oals of thera!y, with antis!asmo"ic "ru$ thera!y for sym!tomatic relief. Crohn "isease 'choice A) is associate" with a number of inflammatory fin"in$s, inclu"in$ ten"erness on ab"ominal e*amination, muco!urulent or bloo"y stools, wei$ht loss, an"Bor leukocytosis, an"Bor elevate" erythrocyte se"imentation rate '()6). %iverticulosis 'choice ) rarely ha!!ens in !atients this youn$ an" will ty!ically !resent with consti!ation without "iarrheal sym!toms. .iar"ia infection 'choice C) may cause chronic sym!toms with ab"ominal cram!in$, but ty!ically with sym!toms of u!!er $astrointestinal involvement, such as nausea, vomitin$, an" eructations, since .iar"ia ten"s to live in the u!!er small bowel. Flcerative colitis 'choice () will !resent with bloo"y stools an" evi"ence of an inflammatory con"ition, e.$., elevate" ()6. ;) (*!lanation: 7he correct answer is %. 7his !atient !resents with sym!toms consistent with $ra"ual luminal narrowin$ of the eso!ha$us after many years of $astroeso!ha$eal reflu* "isease '.(6%). 7hese sym!toms su$$est the "evelo!ment of a beni$n !e!tic stricture. 7his may occur even if the !atient is

on me"ical thera!y to re"uce aci" secretion, since many !atients will continue to !ro"uce aci" "es!ite stan"ar" me"ical re$imens an" may require very hi$h "oses of !roton !um! inhibitors. 7reatment consists of en"osco!ic "ilation of this stricture an" continue" a$$ressive anti4reflu* thera!y. %iffuse eso!ha$eal s!asm 'choice A) !resents with Hnoncar"iac chest !ain,H usually "escribe" as a mi"4chest'squee#in$ chest !ain. It ty!ically !resents with a motility4ty!e "ys!ha$ia. Althou$h .(6% is a common un"erlyin$ factor, the ty!e of "ys!ha$ia in eso!ha$eal s!asm woul" be consistent with a motility4ty!e "ys!ha$ia, i.e., !atient has "ifficulty with liqui"s as well as soli"s from the onset of his sym!toms. )quamous carcinoma 'choice ) is not a sequelae of lon$stan"in$ .(6 %. -urthermore, his a$e makes it e*tremely unlikely that he woul" "evelo! eso!ha$eal carcinoma 'even with his history of smokin$ as a risk factor). A lower eso!ha$eal web 'choice C) or )chat#ki rin$ !ro"uces "ys!ha$ia in an e!iso"ic !attern as foo"s that are $reater in si#e than the "iameter of the web, become lo"$e" in the "istal eso!ha$us. )clero"erma 'choice () coul" cause "ys!ha$ia, but this woul" be accom!anie" by 6aynau"As !henomenon an" characteristic chan$es in the skin. A""itionally, sclero"erma is about three times as common in women than in men. 0) (*!lanation: 7he correct answer is C. 7he clinical manifestations an" family history are consistent with 2untin$ton "isease. 7his autosomal "ominant con"ition is cause" by an unstable e*!ansion of a CA. trinucleoti"e re!eat in a $ene enco"in$ a novel !rotein name" huntin$tin. 7he a$e of clinical onset is commonly between 0> an" /> years, but may be as early as / years. ehavioral abnormalities an" !ersonality chan$es often !rece"e the characteristic choreiform movements. Irritability, restlessness, an" "ifficulty in concentration are amon$ the most frequent early clinical manifestations. 7he !atholo$ic substrate of this con"ition is "e$eneration of the striatal neurons, es!ecially those in the cau"ate nucleus. Cau"ate nucleus chan$es may be a!!reciate" on +6I e*amination or &(7 scans. Creut#fel"t4Nacob "isease 'choice A) is characteri#e" by ra!i"ly !ro$ressive "ementia associate" with myoclonic movements. 7he "isor"er is familial in :>= to :/= of cases. It is !robably cause" by s!ontaneous mutations of the $ene co"in$ for !rion !rotein. 7he onset of .illes "e la 7ourette syn"rome 'choice ) is usually between ; an" :/ years of a$e. +otor or !honic tics are the !rinci!al manifestations, inclu"in$ sniffin$, blinkin$, s!ittin$, $runts, cou$hs, an" co!rolalia.

)y"enham chorea 'choice %) is one of the ma<or Nones criteria for the "ia$nosis of rheumatic "isease. 7ar"ive "yskinesia 'choice () is a late com!lication of anti!sychotic "ru$s that block "o!amine %; rece!tors. It most commonly manifests with !ersistent chewin$ movements an" intermittent !rotrusion of the ton$ue. ?) (*!lanation: 7he correct answer is A. 7his !atient has both hy!ertension an" con$estive heart failure 'C2-). An im!ortant conce!t to reco$ni#e in the treatment of me"ical con"itions is that certain me"ications overla! syn"romes an" are efficacious in many areas. 7his Hco4treatmentH o!tion ma*imi#es each "ru$ in a re$imen an" often a""resses two or more issues simultaneously. In this case, AC( inhibitors have been shown to be very beneficial in !rolon$in$ the survival of C2- !atients. A""in$ an an$iotensin II rece!tor blockin$ a$ent to a re$imen 'choice ) has become an alternative for AC( inhibitor thera!y in !atients who cannot tolerate these "ru$s for a variety of reasons. Althou$h their efficacy in lowerin$ bloo" !ressure a!!ears to be equivalent to that of AC( inhibitors, no "ata have shown their survival benefit to be similar to AC( inhibitors. ecause such a clear mortality benefit has been "emonstrate" for AC( inhibitors, a com!ellin$ reason e*ists to use them !referentially in almost all !atients with systolic "ysfunction. A""in$ hy"rala#ine to her re$imen 'choice C) woul" certainly hel! to treat her bloo" !ressure but will "o nothin$ in terms of hel!in$ her C2 -. Increasin$ the "ose of her calcium channel blocker 'choice %) or her thia#i"e "iuretic 'choice () woul" most certainly ai" in lowerin$ her bloo" !ressure, but neither of these me"ications has any si$nificant utility for treatin$ either systolic or "iastolic C2 -. /) (*!lanation: 7he correct answer is C. 7he clinical !icture is consistent with Wernicke45orsakoff syn"rome. Wernicke ence!halo!athy is characteri#e" by nysta$mus !ro$ressin$ to o!hthalmo!le$ia, truncal ata*ia an" confusion. 5orsakoff syn"rome refers to alcohol4relate" amnesia an" confabulation. Wernicke45orsakoff syn"rome is "ue to vitamin : "eficiency, which is often seen in chronic alcoholics. 7his "eficiency results in "e$eneration of !eriaque"uctal $ray matter. An alcoholic !atient !resentin$ with these sym!toms shoul" be treate" with !arenteral thiamin, />4:>> m$B"ay for the first few "ays followe" by oral thiamin.

7hera!eutic "oses of other water4soluble vitamins shoul" also be a"ministere" because of the frequent concomitance of multi!le vitamin "eficiencies. A"ministration of "ia#e!am 'choice A) is useful for treatment of alcohol with"rawal, which manifests with a$itation an" sei#ures. Intravenous $lucose infusion 'choice ) shoul" be avoi"e". 7his may in fact !reci!itate or a$$ravate a clinical !icture of Wernicke ence!halo!athy in !atients with thiamin "eficiency. 7o*icolo$ical screenin$ 'choice %) an" neuroima$in$ stu"ies 'choice () woul" be of no use in this clinical situation. C) (*!lanation: 7he correct answer is C. 7his !atient has an acute colitis after the use of antibiotics. 7his is "ue to over$rowth of Clostri"ium "ifficile an" can cause a colitis with the characteristic !seu"omembranes, which are seen with si$moi"osco!y as !atchy e*u"ates. +icrosco!ically, the e*u"ates are com!ose" of necrotic mucosal cells, inflammatory cells, bacteria, an" fibrin. Althou$h almost any antibiotic can lea" to C. "ifficile infection, commonly im!licate" cul!rits inclu"e clin"amycin, broa" s!ectrum !enicillins such as am!icillin an" amo*icillin 'as in this !atient), an" ce!halos!orins. Initial treatment is with cessation of the ori$inal antibiotic, if it is still bein$ use". Anti!eristaltic "ru$s are contrain"icate" as they ten" to !rolon$ the illness. +ost cases subsi"e s!ontaneously within :>4:; "ays without other s!ecific thera!y. 7he colitis usually seen in a !atient with !seu"omembranous colitis is a "iffuse mucosal !rocess an" "oes not ty!ically reveal locali#e" "ee! ulcers 'choice A). Althou$h left lower qua"rant cram!s may be seen with "iverticula, they are $enerally seen in the ol"er !o!ulation an" "o not !resent with "iarrhea 'choice ). 7here are no obstructive sym!toms in this !atientAs history to su$$est that she woul" have any strictures 'choice %). )imilarly, this acute onset of "iarrhea woul" not be e*!ecte" to be the result of a sessile si$moi" mass 'choice (). D) (*!lanation: 7he correct answer is %. )uch a writin$ cram! woul" be an e*am!le of focal "ystonia of unknown cause. In this con"ition, the !atient "evelo!s cram!s with altere" han" an" arm !osture when

attem!tin$ a s!ecific task such as writin$. 7he other con"itions may cause !roblems with writin$ as well but are usually not seen in isolation. eni$n essential tremor 'choice A) features a "istal u!!er e*tremity tremor "urin$ a task. Car!al tunnel syn"rome 'choice ) is cause" by me"ian nerve com!ression an" lea"s to han" weakness, which may affect writin$. Cervical ra"iculo!athy 'choice C) can lea" to han" numbness an" hy!orefle*ia. &arkinson "isease 'choice () !resents with micro$ra!hia, bra"ykinesia, an" often, a slow, H!ill4rollin$H tremor. @) (*!lanation: 7he correct answer is %. 7he !atient is in ventricular tachycar"ia an" is hemo"ynamically unstable as a!!arent from the loss of consciousness. 2e shoul" be emer$ently car"ioverte" asynchronously with ;>> <oules of ener$y initially. 2e is hemo"ynamically unstable, as a!!arent from his loss of consciousness. &recious time will be lost by obtainin$ vital si$ns 'choice A) if car"ioversion can be "one now. A li"ocaine bolus shoul" be $iven once a !ulse an" sinus rhythm is obtaine" to kee! him out of ventricular tachycar"ia. 7his is es!ecially beneficial if the event is ischemic in ori$in 'choice ). 2e may be havin$ an infarct, an" this may be the cause of his ventricular tachycar"ia. (mer$ently, the !hysician nee"s to restore sinus rhythm an" then the man will be treate" for myocar"ial infarction, if in"icate" 'choice C). If the !atient were stable, synchronous car"ioversion coul" be attem!te" 'choice (). I) (*!lanation: 7he correct answer is %. Awareness of the ty!ical +6IBC7 a!!earance of brain metastases is im!ortant because often !atients !resent with cerebral metastases without !rior history of cancer "isease. 7hus, a ra"iolo$ic "ia$nosis of brain metastatic "isease may !rom!t a search for the un"erlyin$ !rimary tumor, which is often a lun$ carcinoma in men an" lun$ or breast carcinoma in woman. +elanomas also have a !eculiar !ro!ensity to metastasi#e to the brain. 7he ty!ical ra"iolo$ic features of brain metastases are summari#e" in this case: multi!licity of lesions, well4circumscribe" bor"ers, an" location at the $ray4white matter <unction.

+ulti!le abscesses 'choice A) may "evelo! in !atients with se!sis an", !articularly, in association with con"itions lea"in$ to se!tic emboli 'e.$., infective en"ocar"itis). Arteriovenous malformations 'A1+s) 'choice ) are abnormal con$lomerates of "isor$ani#e" bloo" vessels com!ose" of arteries, veins, an" interme"iate vessels with "iscontinuous elastic lamina. Intracerebral hemorrha$e is their most frequent mo"e of clinical !resentation. (mbolic infarcts 'choice C) woul" be associate" with we"$e4sha!e" cortical lesions. -requently, embolic infarcts are hemorrha$ic. +ultifocal $lioblastoma multiforme '. +) 'choice (), the most frequent mali$nant !rimary brain neo!lasm, manifests as an ill4"efine" mass in the white matter. Contrast enhancement is usually !resent. +ultifocal . + is a rare event. (ven so, the lesions are !oorly circumscribe" an" centere" in the white matter. :>) (*!lanation: 7he correct answer is A. 7his !atient with Crohn "isease has "evelo!e" sym!toms of a small bowel obstruction, which is a common an" im!ortant com!lication of this con"ition. 7his occurs in Crohn "isease as a result of chronic transmural inflammation, which both !artially "estroys the normal bowel wall an" constricts it with thick ban"s of fibrosis. Other im!ortant intestinal com!lications can inclu"e fistula formation an" chronic abscesses. In a""ition, a wi"e variety of e*traintestinal com!lications can inclu"e autoimmune "iseases 'arthritis, a!hthous ulcers, erythema no"osum, !yo"erma $an$renosum, eye involvement, ankylosin$ s!on"ylitis !rimary sclerosin$ cholan$itis) an" com!lications relate" to "isru!te" bowel !hysiolo$y 'renal com!lications, inclu"in$ ki"ney stones an" urinary tract obstruction, malabsor!tion, an" amyloi"osis secon"ary to lon$stan"in$ inflammation). A fistula from the ileum to the si$moi" 'choice ) can "evelo! in !atients with Crohn "isease but will !resent with sym!toms of "iarrhea 'because of the by!ass of a lar$e !ortion of the colon) an" not obstruction. .astric outlet obstruction 'choice C) may !ro"uce vomitin$, usually of only !artially "i$este" foo"s, an" "oes not usually result in si$nificant ab"ominal "istension. -urthermore, there is no im!airment of !assa$e of flatus or bowel movements. Cecal carcinoma 'choice %) can cause obstruction, but it woul" be e*traor"inarily rare in a ;;4year4ol" man. Carcinoma is more likely to com!licate ulcerative colitis than Crohn "isease. 7here is nothin$ in his history to su$$est the "evelo!ment of carcinoi" syn"rome 'choice () in this youn$ !atient. 7hese tumors, when they "o occur, rarely !resent with a bowel

obstruction, but may !resent with the carcinoi" syn"rome, i.e., facial flushin$, "iarrhea, whee#in$, an" tricus!i" re$ur$itation. ::) (*!lanation: 7he correct answer is C. 7he chan$es illustrate" are those of hy!ertensive retino!athy, an" may a""itionally inclu"e yellow har" e*u"ates '"ue to li!i" "e!osition in the retina) an" a con$este" an" e"ematous o!tic "isk. asically, what ha!!ens is that the eye tries to !rotect itself from the hy!ertension first with arteriolar constriction, an" then with time, thickenin$ of the arteriolar walls '!ro"ucin$ the broa" li$ht refle*). 7he cotton wool s!ots are actually small, su!erficial foci of retinal ischemia, which occur when the arterioles squee#e "own too har". 7he hemorrha$e an" "e!osits occur because of vessel "ama$e with leaka$e of contents. 2y!ertensive retino!athy can be seen in chronic essential hy!ertension, mali$nant hy!ertension, an" to*emia of !re$nancy. 7reatment of the retino!athy is with control of the hy!ertension. '&ractically, !ro$ression can be sto!!e" an" the hemorrha$es will resolve, but the vessel chan$es remain.) Central retinal artery occlusion 'choice A) usually !resents with su""en, unilateral blin"ness an" !ro"uces a !ale o!aque fun"us with a re" fovea. Central retinal vein occlusion 'choice ) can cause !ainless visual loss an" !ro"uces a con$este" an" e"ematous fun"us with numerous hemorrha$es. 7he arteriolar chan$es of hy!ertensive retino!athy are not !resent. Non!roliferative "iabetic retino!athy 'choice %) also causes hemorrha$e an" e*u"ates in the retina, but a""itionally has "istinctive microaneurysms 'visible as re" "ots). &roliferative "iabetic retino!athy 'choice () has the chan$es of non!roliferative "iabetic retino!athy with the a""ition of neovasculari#ation with vessel $rowth into the vitreous. :;) (*!lanation: 7he correct choice is A. 7his !atient is hy!otensive but has minimal effects from the hy!otension such as altere" mental status or si$ns of shock. 7his bloo" !ressure, however, is still not acce!table an" the cause for it must be "etermine". In the interim, the sym!tom 'low &) must be treate". 7he stan"ar" thera!y in all such situations is volume. 7his is a conce!t that A,, !hysicians must un"erstan". +any times, newly $ra"uate" !hysicians worry about HC2-H or Hwet lun$sH with a$$ressive I1 flui" su!!ort. Althou$h in a non4 emer$ent situation when a !atientsA bloo" !ressure is not "an$erously low, these are a!!ro!riate concerns, they are irrelevant in this situation. 7he key !rinci!al to un"erstan" is volume above all else. )ince she is mentatin$ well, there is no ur$ent in"ication for

!harmacolo$ical bloo" !ressure su!!ort an" I1 flui"s are the absolute essential initial mana$ement tools for this !atient. Intravenous !ressor su!!ort 'choice ) is never $iven, as hi$h "ose vaso!ressors such as !henyle!hrine an" "obutamine are only $iven via central venous access. On occasion, as tem!orary thera!y, some vaso!ressors may be $iven by !eri!heral I1. Central venous !ressor su!!ort 'choice C) is not in"icate" before an attem!t at volume resuscitation has been ma"e. If this !atient were hemo"ynamically unstable, it mi$ht be a!!ro!riate to initiate this thera!y at the same time as massive volume resuscitation is be$un. A""in$ a""itional antibiotics to treat em!iric se!sis 'choice %) is not a!!ro!riate until hemo"ynamic stability has been achieve". 6emember the A, , CAs of emer$ency care. Concern for circulation before concern for antibiotic thera!y. In a""ition, there is no evi"ence that this !atient is sufferin$ from se!sis syn"rome. It may be sim!ly that she is hy!ovolemic, $iven her fever an" her illness. loo" transfusion 'choice () is ina!!ro!riate unless the cause for the hy!otension is bloo" loss or the !atient is severely anemic an" woul" benefit from bloo" com!onent thera!y. :0) (*!lanation: 7he correct answer is A. Neurosur$ical as!iration of the lesion woul" most likely show al!ha hemolytic stre!tococcus an" mi*e" anaerobes. rain abscesses can occur in youn$ !eo!le. -rontal lobe "isease is associate" with sinusitis an" reflects oral flora. acteroi"es fra$ilis'choice ) is more commonly foun" in brain abscesses of otic ori$in. It woul" be a !atho$en to sus!ect in a chil" with an inner ear infection. &atients who are immunocom!etent are rarely afflicte" by a fun$al abscess 'choice C). &atients with "iabetes or on steroi"s may be infecte" by +ucor. In !eo!le with severe aci"osis, 3y$omycetes can cause brain abscess, but these fun$i have no yeast !hase. &rimary brain lym!homa is rarely seen in immunocom!etent !atients. 7he (!stein4 arr virus has been linke" to urkitt lym!homa 'choice %), a "isease usually foun" in Africa. 7o*o!lasmosis'choice () is an infection often seen in an immunocom!romise" !atient. It shoul" be hi$h on the "ifferential "ia$nosis of any !atient with 2I1 !resentin$ with neurolo$ic sym!toms. :?)

(*!lanation: 7he correct answer is . A heroin a""ict treate" twice for &neumocystis carinii !neumonia most !robably has clinical AI%). 7he !atientAs sensation of incom!lete evacuation su$$ests that his "istal colon is involve"G the fact that the "iarrhea is bloo"y su$$ests that the causative a$ent is very "estructive. Of the choices liste", both cytome$alovirus an" (ntamoeba histolytica may !ro"uce this sym!tom !attern, but cytome$alovirus is much more common than (. histolytica in AI%) !atients. Cry!tos!ori"ium'choice A) causes a small bowel "iarrheal syn"rome with wastin$. (. histolytica'choice C) may occur in !atients with 2I1 but is less common than cytome$alovirus infection. (nteroto*i$enic (scherichia coli'choice %) woul" !ro"uce a watery "iarrhea. (. coli >:4:/D 'choice () is usually acquire" by eatin$ contaminate" $roun" beef !ro"ucts an" results in a hemolytic4uremic syn"rome. :/) (*!lanation: 7he correct answer is . 7he history of !oorly treate" hy!ertension hel!s in the "ia$nosis. A hy!ertensive intracerebral blee"in$ shoul" be sus!ecte". C7 stu"ies of the brain are the most sensitive for "etection of acute blee"in$. +6I scan of the hea" 'choice C) is usually more sensitive than C7 scan for stu"ies of intracerebral lesions, but less accurate in the early "ia$nosis of intracranial blee"in$. (lectroence!halo$ra!hic e*amination 'choice A) woul" show non4"ia$nostic chan$es. 7o*icolo$ical analysis of bloo" an" urine 'choice %) are most useful in investi$atin$ coma of unknown ori$in. In !articular, to*icolo$ical screenin$ of urine may !rovi"e useful information in cases of sus!ecte" !oisonin$ or into*ication. ,umbar !uncture for C)- e*amination 'choice () woul" not be a"visable since this !atient has evi"ence of increase" intracranial !ressure with uncal herniation 'i.e. oculomotor !aresis). :C) (*!lanation: 7he correct answer is %. 7he !atientAs history an" !hysical e*amination are consistent with subacute bacterial en"ocar"itis. 7he most effective "ia$nostic mo"ality woul" be a transeso!ha$eal

echocar"io$ram '7(() in or"er to "etermine whether this !atient has a valvular ve$etation. An (C. 'choice A) will be useful to follow this !atient for any evolvin$ car"iac con"uction "elay. In the initial evaluation, it will hel! assess e*tent of con"uction "ama$e from the infection but will be of little hel! in the ori$inal "ia$nosis. A transthoracic echocar"io$ram '77() 'choice ) woul" be a!!ro!riate if a 7(( were not !ossible. 2owever, a 77( is much less sensitive than a 7( (. A stress test 'choice C) woul" be useful for risk4stratifyin$ a !atient with chest !ain an" coronary artery "isease. If this !atient was not febrile an" was com!lainin$ of stable chest !ain, a stress test coul" be con"ucte" to assess the risk of a car"iac event. Car"iac catheteri#ation 'choice () woul" show the ve$etation but is too invasive. 7his o!tion is usually reserve" for !atients with an acute coronary syn"rome arisin$ from occlusion of a coronary artery. :D) (*!lanation: 7he correct answer is C. 7his !atient with acute cholecystitis has multi!le risk factors, inclu"in$ female $en"er, obesity, an" a classic history of !rolon$e" biliary colic in association with fevers. 7he !resentation illustrate" is ty!ical an" results from obstruction of the cystic "uct, which "rains the $allbla""er. Obstruction of the common bile "uct 'choice A) or the !ancreatic "uct 'choice %) will !ro"uce acute bacterial cholan$itis, which woul" be "emonstrate" by CharcotAs tria", i.e., ri$ht u!!er qua"rant !ain, fever, an" <aun"ice. Obstruction of either the common he!atic "uct 'choice ) or the ri$ht he!atic "uct 'choice () may $ive a limite" e!iso"e of cholan$itis but will not cause cholecystitis, since the obstruction occurs in the biliary tree above the level of the entry of the cystic "uct. :@) (*!lanation: 7he correct answer is C. 7his !atient has been ina!!ro!riately treate" with a hi$h concentration of o*y$en. ecause he has a history of chronic obstructive !ulmonary "isease 'CO&%), he is most likely a CO; retainer an" "e!en"s on mil" hy!o*ia to stimulate his res!iratory "rive. 2is wife clearly "escribe" that he was hy!o*ic 'HblueH), which is consistent with an

e*acerbation of his un"erlyin$ CO&% an" hy!o*ia. 7hese !atients cannot be treate" with hi$h concentrations of o*y$en without !rior intubation to ensure ventilation. As!iration 'choice A) may occur "urin$ any !erio" of "iminishe" mental status but woul" not be the !reci!itatin$ cause of that chan$e in mental status. 2y!erventilation 'choice ) woul" not cause this chan$e in mental status. +yocar"ial ischemia 'choice %) may have occurre" "urin$ his !erio" of hy!o*ia but woul" not e*!lain his unres!onsiveness. 7here is nothin$ "escribe" in this vi$nette that woul" su$$est the !atient is at risk for !ulmonary embolus 'choice (). :I) (*!lanation: 7he correct answer is %. As !art of the wi"ely acce!te" Fniversity of &ittsbur$h criteria for brain "eath, the !resence of either !osturin$ or brain stem function 'e.$.,. !u!illary refle*es or corneal refle*es), as are !resent in this case, violates the brain criteria for the formal "efinition of brain "eath. 7hat sai", the fact that the !atient has no !ur!oseful activity one week after an ano*ic brain in<ury bo"es !oorly for a meanin$ful neurolo$ical recovery. 7here are !ublishe" stu"ies that stratify lon$4term !ro$nosis of such !atients base" u!on neurolo$ical e*aminations ma"e in the first ?@ hours after in<ury. While the !assin$ of time often ai"s in the !ro$nosticatin$ of likely neurolo$ical recovery, the "ia$nosis of brain "eath can be ma"e at any time an" is not time4"e!en"ent 'choice A). Neurolo$ists are often aske" to hel! !re"ict neurolo$ic recovery an" "ia$nose brain "eath 'choice ), but any !hysician '$enerally two are require") may "o so within current acce!te" "ia$nostic $ui"elines. +6I 'choice C) may hel! assess the e*tent of brain in<ury but has no role in the formal "ia$nosis of brain "eath. 7he ((., even when su$$estive of minimal or no cortical function 'choice (), "oes not e*clu"e brainstem activity an" therefore can not be use" in isolation to make the "ia$nosis of brain "eath. ;>) (*!lanation: 7he correct answer is

%. 7his !atient has ha" lon$stan"in$ ulcerative colitis an" has now "evelo!e" !ruritus in the settin$ of an elevate" alkaline !hos!hatase. 7his is consistent with a "ia$nosis of !rimary sclerosin$ cholan$itis, whose activity is not relate" to the activity of the associate" ulcerative colitis. 7his sclerosin$ !rocess involves both the intra4 an" e*trahe!atic "ucts an" is "ia$nose" by en"osco!ic retro$ra"e cholan$io!ancreato$ra!hy '(6C&). &rimary sclerosin$ cholan$itis occurs most often in youn$ men an" is commonly associate" with inflammatory bowel "isease, !articularly ulcerative colitis. Classically, !rimary sclerosin$ cholan$itis !ro"uces a tria" of !ro$ressive fati$ue, !ruritus, an" <aun"ice, althou$h some !atients will !resent with u!!er qua"rant !ain, fever, he!atos!lenome$aly, or cirrhosis. 7he con"ition is worrisome because it may eventually !ro$ress to "ecom!ensate" cirrhosis, !ortal hy!ertension, ascites, an" liver failure. 7reatment is $enerally su!!ortive, with more s!ecific measures as nee"e" inclu"in$ antibacterial treatment for su!erim!ose" bacterial cholan$itis, "ilation by en"osco!y or a transhe!atic route of si$nificant strictures, an" urso"eo*ycholic aci" to relieve the !ruritus. A variety of anti4inflammatory thera!ies 'e.$., corticosteroi"s, a#athio!rine, methotre*ate) have been trie" but a!!ear to have more a"verse than beneficial effects. ,iver trans!lantation a!!ears to be the only true cure. Althou$h erythema no"osum an" !yo"erma $an$renosum can be skin con"itions seen in association with ulcerative colitis, they "o not !resent with !ruritus an", furthermore, have characteristic fin"in$s on !hysical e*amination. (rythema no"osum 'choice A) !resents as ten"er, re" no"ules, ty!ically foun" on the lower e*tremities. &yo"erma $an$renosum 'choice () are !ustular, ulceratin$ lesions, also $enerally foun" on the e*tremities, which can be very !ainful. 7here is no evi"ence of he!atocellular "ysfunction or transaminase elevation, nor any history of he!atitis risk factors, so he!atitis C is unlikely 'choice ). &rimary biliary cirrhosis 'choice C) "oes in fact !resent with !ruritus an" an elevate" alkaline !hos!hataseG however, it is ty!ically seen in mi""le4a$e" women an" has no association with ulcerative colitis. ;:) (*!lanation: 7he correct answer is (. 7his !atient has evi"ence of a !erforate" viscus, as "emonstrate" by the free air un"er the left hemi"ia!hra$m. With a history of $astric ulcer, it is !ossible that he has !erforate" a recurrent ulcer. &lans shoul" be ma"e imme"iately for emer$ent e*!loratory la!arotomy to !revent !ro$ression of his !eritonitis. &e!tic ulcers that !erforate, !ro"ucin$ free air in the ab"ominal cavity, are usually locate" in either the anterior wall of the "uo"enum or in the stomach. 7he "escri!tion of the !ain illustrate" in the question stem is ty!ical. 7he ab"ominal fin"in$s followin$ !erforation may be mislea"in$, as "iffuse ab"ominal !ain, sometimes with !rominent ri$ht lower qua"rant involvement or ra"iation to either or both shoul"er, may "ominate the clinical !icture rather than !ain locali#e" to the e!i$astrium. reathin$ may e*acerbate the !ain. &rom!t "ia$nosis with

emer$ent la!arotomy will lessen the risks of shock an" establishment of a "isseminate" chemical 'or su!erinfecte") !eritonitis. An ab"ominal C7 scan 'choice A) woul" merely "elay the "efinitive la!arotomy. Oral ome!ra#ole an" histamine4; rece!tor anta$onists 'choices an" C) are effective me"ical thera!ies for $astric ulcerG however, they are su!erfluous in the mana$ement of a !erforate" ulcer. Observation after !lacement of a naso$astric tube 'choice %) is ina!!ro!riate $iven the obvious fin"in$s of a !erforate" viscus. Althou$h a naso$astric tube may be !lace" !rior to sur$ery, there is no role for conservative mana$ement for this !atient. ;;) (*!lanation: 7he correct answer is C. 7his !atient has Wernicke ence!halo!athy. 7his acute "isor"er occurs most commonly in chronic alcoholics an" consists of a clinical tria" of o!hthalmo!le$ia, ata*ia, an" $lobal confusion. Affecte" !atients may com!lain of "ouble vision or "ifficulty with balance. 7here is almost always hori#ontal nysta$mus on lateral $a#e. 1ertical nysta$mus may be !resent in />= of cases. ilateral, often asymmetric, lateral rectus !alsies are characteristic an" may "evelo! ra!i"ly. ilateral !tosis or an a!!arent internuclear o!hthalmo!le$ia occurs rarely. 7he treatment is imme"iate I1 thiamine. If $iven quickly enou$h, recovery be$ins !rom!tly. Nearly all !atients with Wernicke ence!halo!athy recover from the $lobal confusional state, but many are left with a resi"ual "isor"er of memory '5orsakoff syn"rome). If I1 "e*trose 'choice A) is $iven before thiamine, the ence!halo!athy may worsen. I1 mannitol 'choice ) is hel!ful in con"itions associate" with increase" intracranial !ressure. C7 of the brain with contrast 'choice %) is an a!!ro!riate ste! but not the ne*t a!!ro!riate ste! in mana$ement. I1 thiamine shoul" be $iven first. C7 scan of the hea" is hel!ful to evaluate for structural lesions, such as sub"ural hematoma. ,umbar !uncture an" e*amination of C)- 'choice () is not the most a!!ro!riate ne*t ste! either. In the worku! of this !atient, a lumbar !uncture may hel! evaluate for a chronic infection, such as sy!hilis or tuberculosis or other !rocesses causin$ those sym!toms. ;0) (*!lanation: 7he correct answer is . &e!tic ulcer "isease is stron$ly su$$este" by chronic mi"e!i$astric !ain that is severe enou$h to awaken a !atient at ni$ht an" is relieve" by antaci"s such as ma$nesium hy"ro*i"e. Althou$h the "ifferential "ia$nosis liste" in te*tbooks for !e!tic ulcer "isease is lon$, most !atients with ulcer sym!toms are either takin$ nonsteroi"al anti4 inflammatory a$ents 'N)AI%s) or are coloni#e" by the $ram4ne$ative or$anism

2elicobacter !ylori. 7his !atient is not takin$ N)AI%s, so 2. !ylori coloni#ation is the most likely answer. 7his or$anism coloni#es the mucous layer that lines the stomach an" "isru!ts the inte$rity of the mucus, !re"is!osin$ for both chronic $astritis an" !e!tic ulcer "isease. Autonomous $astrin secretion 'choice A) by a $astrinoma, as occurs in 3ollin$er4(llison syn"rome, woul" cause multi!le lar$e com!licate" ulcers, often in association with a secretory "iarrhea. Choice C is wron$ because 2. !ylori is $ram ne$ative, rather than $ram !ositive. &rosta$lan"in inhibition 'choice %) is the mechanism whereby N)AI%s cause ulcers, but this !atient is not takin$ these "ru$s. 1a$al inhibition 'choice () is actually a treatment for ulcer "isease when !erforme" as a va$otomy. ;?) (*!lanation: 7he correct answer is . 7he !resence of cram!in$, left lower qua"rant !ain with a locali#in$ e*amination in a !atient with known "iverticulosis associate" with fever an" consti!ation is a classic !resentation of "iverticulitis. A!!en"icitis 'choice A) becomes less likely $iven that the !atientAs !ain is locali#e" in the left lower qua"rant. 7he classic !ain of a!!en"icitis is locali#e" to the ri$ht lower qua"rant. %iverticulosis 'choice C) is incorrect since this term refers only to the !resence of "iverticula. 7he inflammation of a "iverticulum'a), as su$$este" by this !atientAs history an" e*amination, is terme" "iverticulitis. Ischemic colitis 'choice %) is unlikely $iven that this con"ition is ty!ically associate" with ab"ominal !ain on e*amination that is out of !ro!ortion to the !resentin$ com!laint, !ost4!ran"ial ab"ominal "iscomfort, an" rectal blee"in$. )i$moi" volvulus 'choice () is unlikely since we have not been informe" that the !atient has a lar$e re$ion of hy!erresonance over his ab"omen, as woul" be e*!ecte" with the bowel "istension seen with volvulus. ;/) (*!lanation: 7he correct answer is %. It is still controversial when to use neuroima$in$ stu"ies 'C7 an"Bor +6I) in the "ia$nostic assessment of hea"ache. )ince the overwhelmin$ ma<ority of cases are "ue to mi$raine an" tension hea"aches, neuroima$in$ is usually unnecessary. 2owever, in the !resence of new4onset hea"ache, hea"ache with aty!ical features, an" es!ecially in the !resence of focal neurolo$ic "eficits 'such as this case), neuroima$in$ stu"ies shoul" be !erforme" to evaluate for intracranial mass4occu!yin$ lesions 'e.$., tumors, blee"in$). 7rial with "ifferent N)AI%s 'choice A) an" a chan$e in antihy!ertensive me"ication 'choice ) woul" likely yiel" no benefit in this case an" may "elay the "iscovery of a serious un"erlyin$ CN) lesion. ,umbar !uncture 'choice () is not in"icate" in this case, since there is no clinical evi"ence of a neurolo$ic !rocess that may lea" to "ia$nostic chan$es in the cerebros!inal flui" 'C)-). Caution is stron$ly a"vise" when there are si$ns of increase" intracranial !ressure, since lumbar !uncture may !reci!itate cerebellar tonsillar herniation.

;C) (*!lanation: 7he correct answer is C. &rimary brain tumors re!resent the secon" most common mali$nancy of chil"hoo". +e"ulloblastoma is one of the most frequent. It $rows from the cerebellar vermis an" !resents with si$ns an" sym!toms of hy"roce!halus owin$ to obliteration of the fourth ventricle. &rimitive neuroecto"ermal elements are thou$ht to re!resent the cellular !recursors of this ana!lastic tumor. +e"ulloblastomas consist of sheets of un"ifferentiate" cells with scanty cyto!lasm an" immunohistochemical features of neuronal or astrocytic "ifferentiation. (*tension to the cerebellar surface, !ro"ucin$ so4 calle" su$ar coatin$ or "ro! metastasis to the s!inal cor" throu$h the C)-, re!resents a characteristic mo"e of s!rea" of this tumor. +e"ulloblastomas are ra!i"ly $rowin$ tumors. &robably because of this feature, they are also hi$hly res!onsive to ra"iation an" chemothera!y. (!en"ymoma 'choice A) is another characteristic tumor of chil"ren an" youn$ a"ults. Its e!en"ymal ori$in e*!lains its usual !ro*imity to the ventricles 'either lateral or, more often in chil"hoo", the fourth ventricle). In contrast to me"ulloblastoma, e!en"ymoma $rows as a mass fillin$ the fourth ventricle. 7he !ro$nosis "e!en"s on the !ossibility of com!lete e*cision an" the "e$ree of "ifferentiation of the tumor. Fsually, e!en"ymomas ten" to recur after sur$ical resection. 2eman$ioblastoma 'choice ) is a beni$n tumor with a !rominent ca!illary network. etween the ca!illaries are the truly neo!lastic cells, which are !robably of mesenchymal ori$in. 7he cerebellar hemis!heres are the most common locations, where the tumor "evelo!s as a cyst with a mural contrast4enhancin$ no"ule. It is associate" with von 2i!!el4,in"au syn"rome. +enin$ioma 'choice %) is the most common beni$n intracranial tumor. It "erives from menin$othelial cells an" a!!ears as a "ural4attache" mass 'e*traa*ial). Oli$o"en"ro$lioma 'choice () re!resents about /= of all brain tumors. It usually arises in the cerebral hemis!heric white matter, an" rarely occurs in chil"ren. As the name im!lies, the tumor is com!ose" of neo!lastic oli$o"en"ro$lial cells, which closely resemble normal oli$o"en"rocytes. Althou$h it is a slowly $rowin$ tumor, its lon$4term !ro$nosis is !oor because of re!eate" recurrence after sur$ery an" inevitable !ro$ression to hi$h4$ra"e tumor. &ilocytic astrocytoma 'choice -) is a beni$n 'W2O $ra"e I), well4 circumscribe" astrocytoma of chil"ren an" youn$ a"ults. 7he two most common locations inclu"e the cerebellum 'most commonly in the cerebellar hemis!here) an" the "ience!halic re$ion. Com!lete sur$ical resection is feasible an" usually curative in cerebellar tumors, but "ifficult in "ience!halic tumors. ;D) (*!lanation: 7he correct answer is (. 7his !atient is !resentin$ with an asthmatic attack an" is not res!on"in$ to his usual broncho"ilator thera!y. Althou$h the fin"in$s are consistent with a severe asthmatic attack, i.e., the use of the sternoclei"omastoi" muscles for ins!iration, these !atients will $enerally have a normal chest *4ray film. 7he !atient may have "evelo!e" a low4$ra"e '!robably viral) infection, as su$$este" by his low4$ra"e fever. 2owever, the infections

that are most likely to cause an e*acerbation of asthma are usually of the u!!er res!iratory tractG they woul" consequently be unlikely to !ro"uce any fin"in$s consistent with !neumonia. A bilateral interstitial infiltrate 'choice A) woul" su$$est !neumonia. )imilarly, there are no si$ns or sym!toms to su$$est a lobar consoli"ation 'choice ), consistent with a bacterial !neumonia, in that there is no mention of !ro"uctive cou$h, fever, or ri$ors. 7here are no !hysical fin"in$s '"ullness to !ercussion, "ecrease" movement of "ia!hra$m, "ecrease" or absent breath soun"s) or sym!toms '!leuritic !ain with breathin$) to su$$est a !leural effusion 'choice C). &atients with asthma may "evelo! a !neumothora* 'choice %) because of the hi$h intrathoracic !ressures that occur "urin$ a severe attack. 2owever, the !hysical fin"in$s woul" inclu"e absent breath soun"s on e*amination, an" this !atient has symmetric bilateral breath soun"s. ;@) (*!lanation: 7he correct answer is C. 7his !atient has the classic !resentation of acute cholecystitis. 7he e!iso"es she ha" several months a$o an" "urin$ !re$nancy su$$est a !rior history of biliary colic. A 2I%A scan is a noninvasive nuclear me"icine test that will reveal obstruction of the cystic "uct, which is cause" by an im!acte" $allstone an" is the cause of acute cholecystitis. A C7 scan 'choice A) may show a "isten"e" $allbla""er, but it is not as accurate as a 2I%A scan for evaluatin$ the cystic "uct. (n"osco!ic retro$ra"e cholan$io!ancreato$ra!hy '(6C&) 'choice ) is useful for evaluation of the common bile "uct but is of a less value in evaluation of the cystic "uct an", furthermore, is a far more invasive test than a 2I%A scan. &ercutaneous transhe!atic cholan$io$ra!hy '&7C) 'choice %) is an e*amination that is !erforme" by the interventional ra"iolo$ist by in<ectin$ the intrahe!atic biliary tree !ercutaneously. 7his is rarely "one since (6C& is a more accurate evaluation of the biliary of tree. &7C is of limite" value in evaluatin$ the cystic "uct an" it is an invasive !roce"ure. An u!!er $astrointestinal barium stu"y 'choice () may be useful for the evaluations of !e!tic ulcer "isease but the sym!toms here are far more su$$estive of acute cholecystitis. -urthermore, if the !atient has been vomitin$ she is unlikely to tolerate this e*amination. ;I) (*!lanation: 7he correct answer is . 7his !atient, who has <ust returne" from a cruise, has "evelo!e" sym!toms of to*icity in association with confusion, !ulmonary fin"in$s, $astrointestinal com!laints, an" liver function test abnormalities. 7his shoul" su$$est the "ia$nosis of ,e$ionnaires !neumonia, which he acquire" throu$h the ventilation system on the shi!. 7hese !atients may a!!ear quite to*ic, an" imme"iate initiation of thera!y is essential since "ia$nosis usin$ "irect fluorescent antibo"y assays may take several "ays. )!utum .ramAs stain in these !atients is usually unrevealin$ of the ,e$ionella !neumo!hila or$anism. Cefta#i"ime 'choice A) an" $entamicin 'choice C) are antibiotics effective a$ainst $ram4 ne$ative or$anisms but are not use" in treatment of ,e$ionella. 1ancomycin 'choice () is

use" in !atients with sta!hylococcal infections that are resistant to the nafcillin 'choice %) family of antibiotics. 7hese "ru$s have no role in the mana$ement of ,e$ionnaires "isease. 0>) (*!lanation: 7he correct answer is (. 7his !atient !resents with sym!toms of an acute ileitis an" "iarrhea. 9ersinia enterocolitica is acquire" throu$h the fecal4oral route an" will !rimarily infect the terminal ileum. It can be "istin$uishe" from a !atient with ri$ht lower qua"rant !ain in a!!en"icitis since 9ersinia will !resent with a "iarrheal illness while !atients with a!!en"icitis will ty!ically have "ifficulty movin$ their bowels. 7he "ia$nosis of 9ersinia enterocolitica is base" on clinical sus!icion an" the fin"in$ of the or$anism in stool cultures. Cam!ylobacter <e<uni'choice A), a common foo"4borne infection, may !resent with a bloo"y or non4bloo"y "iarrhea but "oes not have acute ri$ht lower qua"rant sym!toms. )almonella enteriti"is'choice ) "oes !resent with an acute "iarrheal illness, but if any ten"erness is seen on e*amination, it is usually more "iffuse an" not quite so well locali#e" to the ri$ht lower qua"rant. )hi$ella "ysenteriae'choice C) !ro"uces a bloo"y "iarrheal illness, which "oes not ty!ically locali#e to the ri$ht lower qua"rant. )ta!hylococcus aureus'choice %) !resents with an acute u!!er intestinal illness with nausea an" vomitin$ followe" later by "iarrhea. It is the result of in$estion of a !re4 forme" to*in an" usually occurs within ?@ hours of in$estion of the tainte" foo". 0:) (*!lanation: 7he correct answer is %. 7his !atient has the ty!ical subacute or chronic history of bloo"y "iarrhea in association with left lower qua"rant cram!in$. 2e also "escribes sensations of tenesmus. Althou$h he has travele" e*tensively, multi!le stool tests are ne$ative for infectious etiolo$y. &atholo$ically, ulcerative colitis is characteri#e" by inflammation an" often su!erficial ulceration that occur without ski! lesions, be$innin$ at the anal ver$e an" e*ten"in$ varyin$ "istances !ro*imally. Crohn "isease 'choice A) may !ro"uce a colitis but is more ty!ically associate" with ri$ht lower qua"rant sym!toms an" ileitis. Cytome$alovirus 'choice ) may cause a !icture in"istin$uishable for ulcerative colitis but is usually seen only in immunocom!romise" !atients, e.$., those with 2I1 who have low C%? cell counts. Ischemic colitis 'choice C) is usually a se$mental colitis an" "oes not usually start at the anal ver$e. It is more commonly seen in el"erly !atients or in those with hy!ercoa$ulable "isor"ers. 9ersinia enterocolitica'choice () may !ro"uce "iarrhea, infrequently bloo"y. 2owever, 9ersinia favors invasion of the terminal ileum an" !ro"uces the acute onset of ri$ht lower qua"rant sym!toms. 0;) (*!lanation:

7he correct answer is . Althou$h villous a"enomas have a $reater chance of eventual !ro$ression to frank cancer, tubular a"enomas are also consi"ere" !remali$nant. 7he fin"in$ of even a sin$le a"enoma increases the likelihoo" of synchronous a"enomas more !ro*imal in the colon. It is therefore a!!ro!riate to !rocee" with a colonosco!y as the ne*t ste!. Althou$h there is some "isa$reement amon$ e*!erts in the fiel", most e*!erts recommen" that !atients who have ha" a tubular a"enoma or villous a"enoma remove" have two subsequent annual ins!ections of the entire colon, !referably with colonosco!y an" alternatively with barium enema if total colonosco!y cannot be !erforme". If no a""itional !oly!s are i"entifie" in these e*aminations, the screenin$ can subsequently be !erforme" every ;40 years. Colonosco!y at a lon$er interval 'choices A an" C) woul" not be a!!ro!riate $iven the increase" risk of a"enomas at !resent. A si$moi"osco!y 'choice %) woul" be ina"equate to visuali#e the lower !ro*imal colon, an" a barium enema 'choice () woul" miss a!!ro*imately :/= of small a"enomas an" woul" not allow simultaneous removal of any a"enomas that are foun". 00) (*!lanation: 7he correct answer is A. 7his !atient has an acute onset of bloo"y "iarrhea consistent with colitis, which is confirme" with the si$moi"osco!y. Of the or$anisms liste", Cam!ylobacter <e<uni is the one most likely to cause these sym!toms. 7his or$anism can cause "iarrhea in all a$e $rou!s, althou$h the !eak of inci"ence is in youn$ chil"ren. C. <e<uni can be acquire" throu$h e*!osure to contaminate" foo" 'es!ecially un"ercooke" !oultry) or water, or throu$h e*!osure to infecte" "omestic or wil" animals. 7he "iarrhea can be either watery or bloo"y an" is often accom!anie" by a sometimes hi$h fever. White bloo" cells are commonly seen in the fecal material. 9ou shoul" also be aware that there a!!ears to be an association between summer outbreaks of "iarrheal "isease "ue to this or$anism, an" subsequent "evelo!ment of the !eri!heral nervous system autoimmune "isease .uillain4 arrV syn"rome. Cry!tos!ori"ium'choice ) causes a small bowel rather than colonic infection, as was "emonstrate" by si$moi"osco!y. .iar"ia lamblia'choice C) causes nausea, vomitin$, eructations, an" u!!er $astrointestinal sym!toms more commonly than "iarrhea. 2er!es sim!le* 'choice %) "oes not cause a bowel infection. 9ersinia enterocolitica'choice () ten"s to infect the area of the cecum rather than the "istal colon, causin$ ri$ht lower qua"rant fin"in$s an" "iarrhea that can mimic the sym!toms of Crohn "isease. 0?) (*!lanation: 7he correct answer is . 7his !atient has a history of ulcerative colitis an" !rimary sclerosin$ cholan$itis. 2e has "evelo!e" steatorrhea as the result of !oor "elivery of bile salts into the small intestine. 7his occurs because of intra4 an" e*trahe!atic "uct stricturin$ of the biliary tree, which !revent a"equate "elivery of bile salts to the small bowel. In the absence of

a"equate bile salt, mal"i$estion of fats occurs, since micelle formation "oes not occur. acterial over$rowth 'choice A) "oes not occur s!ecifically in !atients with ulcerative colitis or !rimary sclerosin$ cholan$itis. It ty!ically results from an un"erlyin$ anatomic or motility "isturbance, such as a blin" loo!, hy!omotility, a fistula, or a stricture. Celiac s!rue 'choice C) is not associate" with ulcerative colitis or !rimary sclerosin$ cholan$itis. )imilarly, tro!ical s!rue 'choice %) is not associate" with this !atientAs sym!toms of colitis or !rimary sclerosin$ cholan$itis. -urthermore, there is no travel history to su$$est e*!osure. Whi!!le "isease 'choice () is a multi4systemic "isease that causes characteristic small bowel bio!sy fin"in$s an" results in malabsor!tion. It is also associate" with hy!er!i$mentation, lym!ha"eno!athy, car"iac "isease, an" rheumatolo$ic "isturbances. 0/) (*!lanation: 7he correct answer is . 7his !atient !resents with the ty!ical fin"in$s of a secretory "iarrhea cause" by an in$este" to*in. In this case, enteroto*i$enic (scherichia coli is the most common cause of travelerAs "iarrhea. It rarely requires antibiotics, an" !atients will $enerally res!on" to conservative mana$ement. (nteroinvasive (. coli'choice A) !ro"uces a bloo"y "iarrhea with fevers an" fin"in$s of ab"ominal ten"erness. In these !atients, the (. coli causes mucosal ulceration, resultin$ in the bloo"y "iarrhea an" muco!urulent stools often seen. (. coli >:/D: 2D 'choice C) !resents with the a!!earance of severe to*icity an" sym!toms of hemolysis an" uremia, i.e., the hemolytic uremia syn"rome. It is usually foun" as !art of an e!i"emic after in$estion of contaminate" beef. 7ro!ical s!rue 'choice %) may occur in !atients returnin$ from the Caribbean but will $enerally !resent with si$ns an" sym!toms of malabsor!tion, !articularly :;, which are not "escribe" here. 1ibrio cholerae'choice () can !ro"uce a clinical !icture i"entical to infection with enteroto*i$enic (. coli, but it "oes not occur in the Caribbean. Instea", it is far more common on the In"ian an" African continents. 6are cases are foun" alon$ the .ulf Coast of the F.). 0C) (*!lanation: 7he correct answer is %. arrettAs eso!ha$us may occur in a small number of !atients who have $astroeso!ha$eal reflu* "isease '.(6%). 7his con"ition is a meta!lasia of the normal squamous mucosa of the eso!ha$us to a columnar '$lan"ular) ty!e of e!ithelium, an" is usually seen as a res!onse to re!eate" aci" e*!osure to the "istal eso!ha$us. 7obacco an" alcohol use are also thou$ht to contribute to the !rocess. 7he si$nificance of arrettAs eso!ha$us is that it may lea" to the "evelo!ment of low4$ra"e "ys!lasia, hi$h4$ra"e "ys!lasia, or eso!ha$eal a"enocarcinoma. 2owever, this is a very infrequent occurrence when consi"erin$ the lar$e number of !atients with .(6% an" even those with arrettAs

eso!ha$us. arrettAs eso!ha$us usually "oes not resolve with either me"ical or sur$ical thera!y. (n"osco!ic surveillance 'with multi!le small bio!sies, since "ys!lasia cannot be reliably evaluate" by en"osco!ic a!!earance alone) every :4; years has been often recommen"e", but some stu"ies su$$est that it may not be cost4effective. It is not factually true to inform the !atient that his concerns are Hun$roun"e"H 'choice A), because there is in fact a small risk of a"enocarcinoma. It is never a!!ro!riate to belittle a !atientAs concerns an" inform him that his worries are HfoolishH 'choice ). arrettAs eso!ha$us is a histolo$ic chan$e an", unless accom!anie" by a stricture, "oes not !ro"uce sym!toms of mechanical "ys!ha$ia 'choice C). It is ina!!ro!riate to refer the !atient to a cancer s!ecialist 'choice () for the !revention of a very unlikely "evelo!ment of cancerG furthermore, !reventive strate$ies shoul" inclu"e instructions to avoi" factors that e*acerbate .(6% an" shoul" be "elivere" to the !atient by the !rimary care !hysician. 0D) (*!lanation: 7he correct answer is A. 7he "ia$nosis of a motility "isturbance, rather than a mechanical cause for "ys!ha$ia, is su$$este" by the history of "ifficulty with swallowin$ both soli"s an" liqui"s at the onset of his sym!toms. Achalasia is characteri#e" by marke"ly re"uce" or absent !eristaltic waves in the bo"y of the eso!ha$us an" an increase" restin$ !ressure at the lower eso!ha$eal s!hincter ',()) that fails to rela* with swallows. 7his results in stasis of foo" within the eso!ha$us, which can be re$ur$itate", even as!irate", an" lea" to as!iration !neumonias. 7he un"erlyin$ "isease !rocess in achalasia a!!ears to be a malfunction of the myenteric !le*us of the eso!ha$us, which in turn lea"s to "enervation of the eso!ha$eal musculature. -orceful "ilation of the ,() is often initially hel!ful in relievin$ the obstruction an" !ressure within the eso!ha$us, but may have to be re!eate" !erio"ically as obstruction often recurs. 7he a""ition of sublin$ual nitro$lycerine or calcium channel blockers that ten" to rela* the ,() may increase the time between "ilations. )ome !atients are eventually treate" with either a 2eller myotomy or in<ection of botulinum to*in into the ,(). 6are cases of achalasia are com!licate" by eso!ha$eal ru!ture with secon"ary me"iastinitis. (so!ha$eal carcinoma 'choice ) may occur in !atients with a history of arrettAs eso!ha$us secon"ary to lon$stan"in$ .(6 %. It, too, !ro"uces a mechanical4ty!e "ys!ha$ia. In any case, this !atient is far youn$er than the ty!ical !atient who "evelo!s eso!ha$eal a"enocarcinoma seen later in life. A lower eso!ha$eal web 'choice C), also referre" to as a )chat#ki rin$, !ro"uces e!iso"ic "ys!ha$ia to lar$e soli"s that are $reater in "iameter than the si#e of the rin$. A !e!tic stricture 'choice %) !resents with a mechanical4ty!e "ys!ha$ia. 7he "ys!ha$ia initially affects lar$e soli"s, an" then $ra"ually worsens to smaller, softer foo"s, an" eventually liqui"s. It usually occurs in !atients with a lon$stan"in$ history of .(6 %. &ro$ressive systemic sclerosis '&))) 'choice () is far more likely to occur in women. It, too, causes a motility "isturbance of the eso!ha$us an", like achalasia, has re"uce" or absent motility in the bo"y of the eso!ha$us. In &)), however, there is re"uce" ,() !ressure at rest that !re"is!oses to severe .(6% an" its !ossible sequelae. Fnless there

are sym!toms of a mechanical obstruction secon"ary to a stricture, eso!ha$eal retention of foo" "oes not occur. 0@) (*!lanation: 7he correct answer is %. 7his !atient has a classic !resentation of acute a!!en"icitis. With early sur$ery, the mortality an" morbi"ity rate is very low. %elaye" sur$ery is much more !roblematic because the infection may have s!rea" throu$h the a!!en"iceal wall, or frank ru!ture with resultin$ !eritonitis may have occurre". Consequently, it is im!ortant not to "elay la!arotomy even for what woul" a!!ear to be a reasonable "ia$nostic work4u!. 7he "ia$nosis of acute a!!en"icitis is therefore ma"e usually on clinical $roun"s. Cases such as this one are strai$htforwar", but you shoul" be aware that many variations in sym!toms an" si$ns can occur. 7he ten"erness may be "iffuse, !articularly in infants an" chil"ren. Ab"ominal !ain an" ten"erness may not be !rominent, !articularly in the el"erly an" !re$nant women. arium enema 'choice A) is s!ecifically contrain"icate" in acute a!!en"icitis, since it may facilitate !erforation. C7 of the ab"omen 'choice ) is sometimes use" for "ia$nosis in late sta$es of the illness to "etermine the location an" si#e of abscess cavities, but is not hel!ful in acute a!!en"icitis an" "elays sur$ery in early cases. (so!ha$o$astro"uo"enosco!y 'choice C) mi$ht "emonstrate a !erforate" !e!tic ulcer 'in the "ifferential "ia$nosis of acute a!!en"icitis)G its use still "elays sur$ery. Fltrasoun" 'choice () can be hel!ful in late "isease to "ia$nose abscesses. 2owever, ultrasoun" is not use" in the acute settin$ because it is usually not "ia$nostic an" it "elays sur$ery. 0I) (*!lanation: 7he correct answer is A. 7his !atient ha" a illroth II $astro<e<unostomy, which is a commonly use" o!eration for both intractable !e!tic ulcer "isease an" $astric carcinoma. In this o!eration, the "istal a!!ro*imately two thir"s of the stomach is resecte", an" the stomach is then attache" alon$ a lon$itu"inal !lane to the <e<unum. 7he technique usually leaves a foot or more of "uo"enum in a Hblin" loo!H confi$uration in which foo" "oes not !ass throu$h it. 7he o!eration "oes, however, allow !reservation of the e*trahe!atic "uct system, so that biliary an" !ancreatic secretions can still enter the $ut. &atients are at risk of bacterial over$rowth in the blin" loo!. lin" loo! syn"rome can be a cause of steatorrhea relate" to bacterial "estruction of bile salts, macrocytic anemia relate" to malabsor!tion of vitamin :;, an" "iarrhea an" malnutrition relate" to $enerali#e" malabsor!tion of nutrients secon"ary to bacterial "estruction of !ancreatic en#ymes. &atients can blee" after a illroth II by the formation of an ulcer at the $astro<e<unostomy, i.e., a mar$inal ulcer, but there is no cause for lower $astrointestinal tract blee"in$ 'choice ). Chronic !ancreatitis 'choice C) an" !ancreatic insufficiency 'choice () are not com!lications of $astro<e<unostomy an" illroth II o!eration. Consti!ation 'choice %) is not a feature of

havin$ ha" a $astro<e<unostomy. Instea", some of these !atients are !rone to "um!in$ syn"rome, in which "iarrhea is common. ?>) (*!lanation: 7he correct answer is . 7his !atient has a history of severe !e!tic ulcer "isease, as manifeste" by multi!le ulcers, ulcers in unusual sites such as <e<unum, an" ulcers com!licate" by blee"in$. 2e is not takin$ nonsteroi"al anti4inflammatory "ru$s 'N)AI%s), an" he "oes not have 2elicobacter !ylori coloni#ation in the stomach. 7his su$$ests that he really "oes have very hi$h aci" secretion by the stomach, which shoul" raise the !ossibility that he has a $astrin4secretin$ neuroen"ocrine tumor that is tri$$erin$ the hi$h level of aci" release. 7he tumors are most likely to be in either the !ancreas or the !ro*imal small bowel. 7he combination of a $astrin4secretin$ tumor an" severe !e!tic ulcer "isease is sometimes calle" 3ollin$er4(llison syn"rome, an" sur$ical resection of the tumor 'which may be small an" har" to fin") usually !ermits cure of the !e!tic ulcer "isease. Autoimmune "estruction of the !arietal cells 'choice A) causes the "isease known as !ernicious anemia, which is associate" with atro!hic $astritis. )omatostatin 'choice C) inhibits $astrin !ro"uction an" aci" secretion, an" so e*cess somatostatin woul" not be e*!ecte" to cause !e!tic ulcer "isease. Im!aire" !ro"uction of the mucous bicarbonate layer in the stomach 'choice %) is the mechanism whereby N)AI%s cause $astric an" "uo"enal ulcers, but this !atient is not on these me"ications. Inhibition of !arietal cell secretion 'choice () woul" actually re"uce ulcer ten"encies. ?:) (*!lanation: 7he correct answer is (. +i"e!i$astric !ain ra"iatin$ to the back an" e*acerbate" by eatin$ an" continue" alcohol use shoul" su$$est the "ia$nosis of !ancreatitis. )i$nificant risk factors for !ancreatitis inclu"e alcoholism 'as this !atient has) an" biliary tract "isease 'for which there is no evi"ence in this !atient). It is a little unclear from the history whether the !atient is e*!eriencin$ recurrent e!iso"es of acute !ancreatitis or is "evelo!in$ the onset of chronic !ancreatitis. Whichever is the case, both !rocesses are ca!able of "estroyin$ enou$h e*ocrine !ancreatic tissue to !re"is!ose for e*ocrine !ancreatic insufficiency with resultant insufficient !ro"uction of !ancreatic en#ymes, such as li!ase, which normally facilitates fat usa$e, to allow for normal "i$estion. )teatorrhea, or e*cess fat in stool, is seen clinically as $reasy, foul4smellin$ 'because of bacterial action on the fat), lar$e4volume 'in !art "ue to $as !ro"uction by bacteria an" in !art "ue to failure to "i$est most of the foo") stools. 7here is no evi"ence that there is hy!eraci"ity to account for inactivation of !ancreatic en#ymes 'choice A). 7his !henomenon can occur, but it woul" be more likely to be seen in settin$s such as 3ollin$er4(llison syn"rome, in which a $astrin4!ro"ucin$ tumor of the !ancreas or small intestine stimulates e*cess aci" !ro"uction by the stomach. acterial over$rowth 'choice ) can !ro"uce steatorrhea by "econ<u$atin$ bile salts an" makin$ them unavailable to form micelles. 2owever, this

!atient has no un"erlyin$ etiolo$y for bacterial over$rowth, such as blin" loo! syn"rome or hy!omotility. iliary insufficiency 'choice C) can cause steatorrheaG however, althou$h biliary tract "isease can !re"is!ose for !ancreatitis, !ancreatitis "oes not !re"is!ose for biliary insufficiency. 7he !ain of biliary tract "isease is usually locali#e" to the left u!!er qua"rant an" may ra"iate to the left shoul"er. (*cess $luca$on 'choice %) is rarely note" in the absence of the very rare neuroen"ocrine tumor known as $luca$onoma. It "oes not !ro"uce fat mal"i$estion or malabsor!tion. ?;) (*!lanation: 7he correct answer is %. On"ansetron, a /4hy"ro*ytry!tamine0 anta$onist, is the most !otent antiemetic available for chemothera!y4in"uce" vomitin$. It is safe an" is available as an oral or I1 me"ication. On"ansetron has $reatly im!rove" the !ro$nosis of many !atients who were !reviously unable to tolerate full4"ose chemothera!y because of severe vomitin$. It has si"e effects only infrequently, the most common bein$ consti!ation. %o!amine 'choice A) is actually a !ro4emetic com!oun". 7he common me"iators of vomitin$ in the CN) are /4hy"ro*ytry!tamine, "o!amine, an" acetylcholine. (rythromycin 'choice ) also ten"s to cause, not relieve, nausea an" vomitin$. It has motilin4stimulatin$ !ro!erties in the stomach, lea"in$ to increase" $astric motility. It is of thera!eutic value in !atients with "iabetic $astro!aresis. Ome!ra#ole 'choice C) is of no value in inhibitin$ vomitin$. It is a !roton4!um! inhibitor, which is effective in re"ucin$ or eliminatin$ aci" secretion. O!iates 'choice () are commonly use" in !atients receivin$ chemothera!y but are actually !ro4emetic because they stimulate the chemorece!tor tri$$er #one, which lies within the brain but outsi"e the bloo"4brain barrier. ?0) (*!lanation: 7he correct answer is . 7his !atient has !robable !olycythemia vera 'a below normal erythro!oietin level woul" be a hel!ful confirmatory stu"y) which is !resentin$ with u""4Chiari syn"rome. u""4Chiari syn"rome is a rare "isor"er with hi$h mortality rate usually cause" by the thrombosis of ma<or he!atic veins. It can !resent either acutely, as in this case, or more slowly with chronic va$ue ab"ominal !ain an" !ortal hy!ertension. Fn"erlyin$ !re"is!osin$ causes inclu"e myelo!roliferative "isor"ers 'notably !olycythemia vera, as in this case)G other hematolo$ic "isor"ers 'sickle cell "isease, !aro*ysmal nocturnal hemo$lobinuria)G "efective inhibition of coa$ulation 'anti!hos!holi!i" antibo"ies, low antithrombin III, low !rotein C, low !rotein ), an" low factor 1 ,ei"en)G !ossibly estro$ens 'oral contrace!tives, !re$nancy)G an" local liver "isease 'mali$nancies, trauma, or su!!urative lesions). Alcoholic cirrhosis 'choice A) can cause <aun"ice, but A)7 an" A,7 elevations are usually not very hi$h. -urther, nothin$ in this manAs history su$$ests si$nificant alcohol use. Cholelithiasis 'choice C) can cause obstructive <aun"ice when a small stone occlu"es a bile "uct, but A)7 an" A,7 elevations are usually not very hi$h. No risk factors are note" for either chronic he!atits 'choice %) or fulminant he!atitis A

'choice (). Chronic viral he!atitis "oes not usually have very hi$h elevations of A)7 or A,7, but fulminant he!atitis, which u""4Chiari syn"rome mimics, can. ??) (*!lanation: 7he correct answer is A. 7his !atient has si$ns an" sym!toms, as well as *4ray confirmation, of a small bowel obstruction. .iven the histolo$ic fin"in$s of the sur$ical s!ecimen of an early '%ukeAs :) lesion, it is hi$hly unlikely that there is recurrent tumor, es!ecially $iven the !rom!t occurrence of this bowel obstruction. 7he most likely etiolo$y is therefore a"hesions, which have cause" this obstruction. &lacement of a naso$astric tube woul" be the most a!!ro!riate measure to "ecom!ress the bowel. In most of these !atients, this a!!roach will lea" to $ra"ual an" com!lete resolution of the bowel obstruction. )houl" there be any sym!toms of clinical "eterioration with the !lacement of a naso$astric or small intestinal tube, then sur$ery 'choice () shoul" be consi"ere". Althou$h a small bowel series 'choice ) or a C7 scan 'choice %) may be hel!ful in "elineatin$ the site of the a"hesion an" small bowel obstruction, they woul" be !erforme" after bowel "ecom!ression with a naso$astric tube. )imilarly a barium enema 'choice C), if thou$ht necessary, woul" be !erforme" after bowel "ecom!ression. 2owever, a barium enema is unlikely to "emonstrate an obstructive !oint since the obstruction is unlikely to be "ue to anastomotic tumor recurrence, $iven the early lesion an" recent resection. )ur$ical e*!loration 'choice () woul" not be in"icate" until a trial of conservative thera!y with small bowel "ecom!ression an" a !erio" of N&O have been attem!te". ?/) (*!lanation: 7he correct answer is A. Carcinoi" syn"rome is the term use" for the cluster of cutaneous flushin$, ab"ominal cram!s, an" "iarrhea that is seen in !atients who have e*cess amounts of circulatin$ vasoactive substances, inclu"in$ serotonin. 7he usual cause is an intestinal carcinoi" tumor that has metastasi#e" to the liver. .astrointestinal carcinoi"s that have not yet metastasi#e" are much less likely to !ro"uce carcinoi" syn"rome, since the liver will clear the !ortal bloo" of the vasoactive substances. +uch less commonly, carcinoi"s at non$astrointestinal sites, such as lun$ or ovary 'whose venous bloo" "oes not !ass throu$h the liver before enterin$ the systemic circulation), can cause carcinoi" syn"rome without metastatic "isease. 6i$ht4si"e" en"ocar"ial fibrosis, with !ulmonary stenosis an" tricus!i" re$ur$itation, is common in carcinoi" !atients an" is the result of to*ic "ama$e to the heart. /4hy"ro*yin"oleacetic aci" is a urinary metabolite of serotonin that can be use" to confirm the clinical im!ression of carcinoi" syn"rome. Carcinoi" tumors are often in"olent, an" :>4 to :/4year survivals after liver metastasis can occur. Cholan$iocarcinoma 'choice ), he!atocellular carcinoma 'choice C), mucinous a"enocarcinoma of the colon 'choice %), an" squamous cell carcinoma of the eso!ha$us 'choice () "o not usually secrete hormonally active substances.

?C) (*!lanation: 7he correct answer is %. 7he fin"in$ of a sessile mass in the colon, even thou$h it is small an" has a beni$n a!!earance, man"ates a colonosco!y for its removal. 7his woul" allow a com!lete e*cisional bio!sy an" histolo$ic review. Althou$h frank carcinoma is unlikely with a mass this small, either a tubular a"enoma or a villous a"enoma is conceivably !resent an" shoul" be remove" both to establish the "ia$nosis an" to !revent the lesion from eventually !ro$ressin$ to cancer. In a""ition, the structure of the bowel is sufficiently com!le* that it is not uncommon for other small !oly!s to be !resent that were not !icke" u! ra"iolo$ically. 7hese shoul" also be e*cise", an" their histolo$y reviewe". It woul" be ina!!ro!riate to wait : year with a known !oly!, even with a beni$n a!!earance, so follow4u! barium enema 'choice A) or colonosco!y 'choice ) in : year woul" be ina!!ro!riate. A si$moi"osco!y 'choice C) woul" be an ina"equate e*amination to reach the ascen"in$ colon to remove this !oly!. (*traluminal involvement with such a beni$n4 a!!earin$ !oly! is hi$hly likely, so C7 scan 'choice () is unnecessary. ?D) (*!lanation: 7he correct answer is %. &e!tic ulcer "isease is stron$ly su$$este" by this !atientAs chronic e!i$astric !ain that is severe enou$h to awaken him at ni$ht an" is tem!orarily relieve" with raniti"ine an" eatin$. In the absence of nonsteroi"al anti4inflammatory "ru$ 'N)AI%) use, the most likely etiolo$y is antral coloni#ation with 2elicobacter !ylori. 7his $ram4ne$ative or$anism lives in the mucous layer that lines the stomach an" a!!arently uses the mucus as its foo" source. 7he result is focal "isru!tion of the mucosal barrier, !re"is!osin$ for both chronic $astritis an" $astric !e!tic ulcer formation. In the !ast, me"ical !ersonnel ten"e" to blame ulcer formation on Hstress,H with the im!lication that correctin$ the !roblem involve" teachin$ the !atient to be more calm. Now that we more fully un"erstan" !e!tic ulcer "isease, we treat the bacterial coloni#ation as !art of our !rimary thera!y. %iffuse $astric mucosal vasoconstriction 'choice A) "escribes the !atho$enesis of stress4in"uce" $astritis. .astrin4!ro"ucin$ tumors 'choice ) can cause 3ollin$er (llison syn"rome, in which the increase" circulatin$ $astrin marke"ly increases $astric aci" !ro"uction. 7hese tumors are rare, an" !atients who have them ty!ically have a history of multi!le !e!tic ulcers, often with com!lications such as !erforation or massive blee"in$. 7his !atientAs history of sym!toms that have worsene" "urin$ stress is more su$$estive of the more common form of !e!tic ulcer "isease relate" to stress an" 2. !ylori coloni#ation. 7he 2. !ylori or$anisms that are associate" with !e!tic ulcer "isease "o not actually inva"e the antral mucosa 'choice C). Increase", but not "ecrease" 'choice (), $astrin release, as "escribe" above with res!ect to 3ollin$er4(llison syn"rome, may cause !e!tic ulcers. ?@)

(*!lanation: 7he correct answer is (. 7his !atient has taken the !rosta$lan"in miso!rostol, which is "esi$ne" to !revent ulcer occurrence with the use of N)AI%s. It is strictly !rohibite" "urin$ !re$nancy since it has the !re"ictable effect of in"ucin$ uterine smooth muscle contractions. 7his has actually become !art of an Hat4homeH abortion metho" usin$ the combination of oral methotre*ate an" miso!rostol. Anti!hos!holi!i" antibo"ies 'choice A) may in"uce thrombosis of !lacental vessels an" cause recurrent secon"4trimester abortions. 2owever, there is no history to su$$est this "ia$nosis in this !atient. Cycloo*y$enase an" li!o*y$enase 'choices an" %) are en#ymes that are involve" in arachi"onic aci" metabolism. )imilarly, there is no evi"ence $iven to su$$est an un"erlyin$ factor 1III "eficiency 'choice C).

USMLE Step 2 Practice Test Block 14 :ame; +nstr#ctions; Ans'er the *#estions $elo' to the $est of yo#r a$ility. =hen yo# finish the test% click the 2heck $#tton at the $ottom to )ie' the res#lts.

1.A 1<!year!ol" female presents to the emergency room 'ith se)ere right!si"e" a$"ominal pain. /er last menstr#al perio " 'as 2 'eeks ago an" felt to $e normal. She "isplays painf#l "iffic#lty in positioning herself on the e.amination ta$le. /er temperat#re is &1. 2 3141.& 56% $loo" press#re is 1217( mm /g% p#lse is 147min% an" respirations are 227min. E.amination of the throat re)eals mil" pharyngitis. /er a$"omen is "iff#sely ten"er% especially the lo'er a$"omen. Aectal e.amination re)eals ten"erness anteriorly on the right si"e. Stool g#aiac is negati)e. A pel)ic e.amination is performe"% an" there is e)i"ence of cer)ical ten"erness an" *#estiona$le f#llness in the right a"ne.al area. =hich of the follo'ing is the most likely "iagnosisa6>)arian cyst $6Pyelonephritis c6Pel)ic inflammatory "isease "62onstipation e6 En"ometriosis :ormal La$s 2.A 14!year!ol" $oy is e)al#ate" for short stat#re. /e has no significant past me"ical history an" is consi"ere" other'ise healthy $y his parents. /e eats a normal "iet an" has reg#lar meals. /is height an" 'eight ha)e $een consistently at the th percentile since early chil"hoo". /is physical e.amination is normal% 'ith genitalia at Tanner stage &. =hich of the follo'ing is the most likely la$oratory fin"ing for this $oy-

a6Bone age that is e*#i)alent to chronologic age $6@ecrease" complement 2& le)el c6@ecrease" ser#m al$#min concentration "6@ecrease" thyroi" stim#lating hormone e6+ncrease" ser#m creatinine concentration :ormal La$s &. A 1!month ol" $oy is $ro#ght to the emergency "epartment $y his mother% 'ho states that he has $een ha)ing 'hat she "escri$es as 0pro9ectile )omiting0 for the past se)eral "ays. She states that he )omits e)ery time she fee"s him% an" the sit#ation seems to $e getting 'orse% altho#gh he "oes not seem to $e in pain. She "escri$es the )omit#s as non!$ilio#s% an" he has ha" normal stools 'ith no $loo" in them. >n e.amination% the infant appears to $e mil"ly "ehy"rate"% his a$"omen is soft% an" there is a palpa$le% oli)e!siDe"% firm mo)ea$le mass in the right #pper *#a"rant. =hich of the follo'ing is the most likely "iagnosisa6 @#o"enal atresia $6+nt#ss#sception c6/irschspr#ng "isease "6Mi"g#t )ol)#l#s e6Pyloric stenosis :ormal La$s 4.A <!"ay!ol" girl 'ho 'as $orn at home is $eing e)al#ate" for $r#ising an" gastrointestinal $lee"ing. La$oratory fin"ings incl#"e partial throm$oplastin time an" prothrom$in time% greater than 2 min#tesG ser#m $ilir#$in% 4.( mg7"LG alanine aminotransferase% 11 mg7"LG platelet co#nt% &&4%4447mm&G an" hemoglo$in% 1<.& g7"L. =hich of the follo'ing is the most likely ca#se of her $lee"inga65actor ?+++ "eficiency $65actor +F "eficiency c6+"iopathic throm$ocytopenic p#rp#ra "6Li)er "isease e6 ?itamin M "eficiency

:ormal La$s . A (!year!ol" $oy is $ro#ght to the clinic for a lifetime history of $e"'etting. /e has other'ise $een completely healthy an" has met all "e)elopment milestones. /is parents "eny a history of tra#ma% an" the history is not consistent 'ith a$#se. The patient has $een 'etting e)ery night $#t not "#ring the "aytime. /e has no incontinence. =hich of the follo'ing is the most appropriate ne.t step in his e)al#ationa6 +ntra)eno#s pyelogram $6Aenal Ultraso#n" c624!ho#r #rine collection "6Urinalysis e62T of pel)is :ormal La$s <.A &!month ol" infant is $ro#ght to a pe"iatrician,s office $eca#se of increase" lethargy an" irrita$ility. The parents state that the chil" rolle" off the co#ch an" fell on the floor one "ay prior to presentation. /is parents report that the chil" has $een pre)io#sly healthy an" is #p to "ate on his )accinations. /e has $een meeting his "e)elopment milestones. /is fontanelles are f#ll. =hile in office the patient "e)elops a tonic clonic seiD#re. =hich of the follo'ing is the ne.t appropriate stepa6>$tain a hea" comp#teriDe" tomography scan $6Perform a retinoscopic e.amination c62heck ser#m le)els of ammonia "6A"minister intra)eno#s $enDo"iaDepines e6Perform a l#m$ar p#nct#re :ormal La$s (. A to""ler is $ro#ght to the emergency "epartment 'ith $#rns on $oth of his $#ttocks. The areas are moist% ha)e $listers% an" are e.*#isitely painf#l to to#ch. The parents e.plain that the chil" acci"entally p#lle" a pot of $oiling 'ater o)er himself. =hich of the follo'ing is the most important step in managementa6Application of mafeni"e acetate to the $#rne" areas

$6Early e.cision an" grafting of the $#rne" areas c6E"#cation of the parents on acci"ent pre)ention "6Prompt a"ministration of fl#i" res#scitation e6Aeferral to the proper a#thorities for s#specte" chil" a$#se :ormal La$s 1.A pe"iatrician e.amines a 2!month!ol" infant 'ho ha" $een $orn at term. The pe"iatrician hears a contin#o#s m#rm#r at the #pper left sternal $or"er. The peripheral p#lses in all e.tremities are f#ll an" sho' 'i"ene" p#lse press#re. =hich of the follo'ing is the most likely "iagnosisa62oarctation of the aorta $6Patent "#ct#s arterios#s c6Peripheral p#lmonic stenosis "6Persistent tr#nc#s arterios#s e6 ?entric#lar septal "efect :ormal La$s 8.An 1!month!ol" infant% 'ho is #p!to!"ate 'ith his imm#niDations% is $ro#ght to the clinic $y his mother. The mother states that she o)erhear" other mothers talking a$o#t a )aricella )accination that their chil"ren ha)e recei)e". She "oes not 'ant her son to ha)e the chickenpo. )ir#s an" therefore% 'ants him to recei)e the )accine to"ay. The physician e.plains that the infant has not yet reache" the recommen"e" age for the )accine. +f this )isit is in :o)em$er% 'hen is the earliest that this patient can ret#rn for the )aricella )accinea65e$r#ary $65e$r#ary c6April "6May e6R#ne :ormal La$s 14. An inner city family has $een #sing a neigh$or to care for their &! year!ol" chil" 'hile the parents 'ork. The neigh$or is "iagnose" 'ith p#lmonary t#$erc#losis. PP@ test of the &!year!ol" is negati)e. =hich of the follo'ing is in"icate" for the &!year!ol"a6Etham$#tol chemoprophyla.is

$6+soniaDi" chemoprophyla.is c6Aifampin chemoprophyla.is "6Streptomycin chemoprophyla.is e6:o chemoprophyla.is :ormal La$s 11. A 24!year!ol" female is $ro#ght to the Emergency Aoom $y her college roommate 'ho states that the patient )omite" all night. The patient complains of a sore throat an" says she has not eaten for the last t'o "ays. She a"mits to a 0s#gar pro$lem0 an" *#it taking her me"ication $eca#se she has not $een eating. E.amination re)eals an ill!appearing 'oman. /er temperat#re is &(.8 2 3144.2 56% $loo" press#re is 1117(1 mm /g% p#lse is 1<47min% an" respirations are &47min. The patient,s lips an" m#co#s mem$ranes are "ry. There is a fr#ity o"or note" to the patient,s $reath. The l#ng an" car"iac e.amination are #nremarka$le e.cept for mil" tachypnea an" tachycar"ia. La$oratory analysis sho's; So"i#m......................1&4 mE*7L Potassi#m..................<.1 mE*7L 2hlori"e....................144 mE*7L Bicar$onate...............1 mE*7L Urea nitrogen............14 mg7"L 2reatinine.................1.4 mg7"L Cl#cose....................<14 mg7"L p/...........................(.1 p2>2.......................&4 mm /g p>2.........................1 mm /g Urinalysis is positi)e for ketones. =hich of the follo'ing is the most appropriate initial step in managementa6+mme"iate int#$ation $6+ntra)eno#s ins#lin c6+ntram#sc#lar ceftria.one "6+? fl#i" $ol#s 'ith normal saline an" potassi#m e6 Mannitol :ormal La$s

12. An 1!year!ol" $oy is seen in the pe"iatrician,s office for a ro#tine health s#per)ision )isit. The mother states that he has @#chenne m#sc#lar "ystrophy. >n e.amination% he is fo#n" to ha)e hip 'a""le an" enlargement of $oth cal)es. /e is am$#latory% $#t his m#scle strength is "iminishe" symmetrically. =hich of the follo'ing signs is most consistent 'ith @#chenne m#sc#lar "ystrophya605oot "rop0 gait $6Co'er sign c6+ncrease" "eep ten"on refle. "6Myotonia e6Positi)e Ba$inski sign :ormal La$s 1&. A male infant $orn at term is fo#n" to ha)e $ilateral colo$omas% choanal atresia% ear anomalies% an" cryptorchi"ism. There is no history maternal "r#g or alcohol a$#se "#ring pregnancy. There is no family history of similar congenital "efects. =hich of the follo'ing is the most appropriate initial test to e.cl#"e any associate" a$normalities- Top of 5orm 1 a6Bari#m s'allo' $6Echocar"iography c65i$eroptic $ronchoscopy "6Aenal #ltrasonography e6 Skeletal s#r)ey :ormal La$s 14.A 1 !year!ol" girl presents to the emergency "epartment 'ith the s#""en onset of 'atery "iarrhea tinge" 'ith $loo". The girl 'as pre)io#sly healthy. /er only me"ications are topical $enDoyl pero.i"e an" oral clin"amycin for acne )#lgaris. Physical e.amination re)eals a slightly "isten"e" a$"omen that is "iff#sely ten"er. /er temperat#re is &1.1 2 3144. 56. She has not $een e.pose" to any #ncooke" meat an" has not eaten any #n#s#al foo"s. =hich of the follo'ing is the most likely "iagnosisa6Castroenteritis $6+rrita$le $o'el syn"rome c6Pse#"omem$rano#s enterocolitis "6Salmonella infection

e6 Ulcerati)e colitis :ormal La$s 1 . A !year!ol" $oy is $ro#ght to an emergency room $eca#se of a painf#l% s'ollen knee 9oint. The $oy ha" fallen 'hile playing% an" the 9oint ha" s#$se*#ently $eg#n to s'ell. The mother reports that the $oy 'as kno'n to ha)e hemophilia B. Aeplacement of 'hich of the follo'ing is in"icate"a65actor 2 $65actor S c65actor ?++ "65actor ?+++ e65actor +F :ormal La$s 1<. A !month!ol" male infant has a #rine o#tp#t of less than 4.1 mL7kg7hr shortly after #n"ergoing ma9or s#rgery. >n e.amination% he has generaliDe" e"ema. /is $loo" press#re is 84741 mm /g% p#lse is 1447min% an" respirations are 247min. /is $loo" #rea nitrogen is &1 mg7"L% an" ser#m creatinine is 1.4 mg7"L. +nitial #rinalysis sho's a specific gra)ity of 1.411 an" 2E protein. Microscopic e.amination of the #rine sample re)eals 1 =B2 per high!po'er fiel" 3/P56% 11 AB2s per /P5% an" gran#lar casts per /P5. /is fractional e.cretion of so"i#m is &.2 B. =hich of the follo'ing is the most appropriate ne.t step in "iagnosisa62T of the a$"omen an" pel)is $62ysto#rethrography c6+ntra)eno#s pyelography "6Aenal $iopsy e6Aenal #ltrasonography :ormal La$s 1(.A 12!year!ol" $oy presents 'ith a 2!month history of rec#rrent hea"ache an" "eteriorating school performance. :e#rologic e.amination is #nremarka$le e.cept for $itemporal "eficits in his )is#al fiel". F!ray an" MA+ of the hea" are performe". F!ray films sho' calcifications in the s#prasellar region% 'hile MA+ images re)eal a

m#lticystic t#mor "isplacing the optic chiasm. The patient #n"ergoes ne#ros#rgical operation for e.cision of the t#mor. @#ring the proce"#re% the s#rgeon notices that the cystic ca)ities contain a 0machinery oil0!like fl#i". =hich of the follo'ing is the most likely "iagnosisa62raniopharyngioma $62ysticercosis c6Canglioglioma "6Pilocytic astrocytoma e6Pit#itary a"enoma :ormal La$s 11.A <!year!ol" $oy is $ro#ght to the pe"iatrician $eca#se of a &!"ay history of skin lesions. >n physical e.amination% he has m#ltiple yello'% cr#ste" erosions $elo' the nares an" on the cheeks% chin% an" #pper e.tremities. The rest of the e.amination is normal. =hich of the follo'ing is the most appropriate treatment for this con"itiona6>ral acyclo)ir $6>ral amo.icillin c6>ral cephale.in "6Topical ketoconaDole e6Topical 2B hy"rocortisone :ormal La$s 18. A 2!month!ol" infant is e)al#ate" $y a pe"iatric car"iologist. The infant 'as note" at $irth to ha)e an #pper left sternal $or"er e9ection m#rm#r. The infant at that time 'as not cyanotic% $#t slo'ly "e)elope" cyanosis o)er the ne.t t'o months. At the time of the pe"iatric car"iologist,s e.amination% an E2C sho'e" right a.is "e)iation an" right )entric#lar hypertrophy. A chest .!ray film sho'e" a small heart 'ith a conca)e main p#lmonary artery segment an" "iminishe" p#lmonary $loo" flo'. =hich of the follo'ing is the most likely "iagnosisa6 2omplete atrio)entric#lar canal "efect $6/ypoplastic left )entricle c6+solate" atrial septal "efect "6Tetralogy of 5allot e6 Transposition of the great arteries

:ormal La$s 24. A 2 172!year!ol" chil" is e)al#ate" $y a ne#rologist $eca#se of "iffic#lty 'alking. :e#rological e.amination "oc#ments ata.ia an" mental retar"ation. The ne#rologist notes the presence of m#ltiple telangiectasias in)ol)ing the con9#ncti)a% ears% an" antec#$ital fossae. The chil" also has a history of m#ltiple respiratory tract infections. +mm#noglo$#lin st#"ies on the chil" 'o#l" most likely "emonstrate an a$sence of 'hich of the follo'inga6+gA an" +gE $6+gA an" +gC c6+gE an" +gC "6+gE an" +gM e6+gM an" +gC :ormal La$s 21. A 1 !year!ol" $oy 'ith @#chenne m#sc#lar "ystrophy is $ro#ght to the emergency "epartment 'ith increasing respiratory "istress an" cyanosis. >n e.amination% he is "iaphoretic% 'ith gasping respirations% poor air entry% an" "iminishe" responsi)eness. /e is tachycar"ic at 1<4 $eats7min. /is chest .!ray film sho's a ling#lar pne#monia% an" he is int#$ate". /e impro)es o)er the ne.t 14 "ays 'ith anti$iotics $#t is not e.t#$ate" secon"ary to hypo)entilation on 'eaning $eca#se of m#scle 'eakness. =hich of the follo'ing mo"alities 'ill most likely help 'ean him off the )entilatora6:asopharyngeal t#$e $6Press#re controlle" )entilation c6Press#re controlle" )entilation "6S#pplemental o.ygen e6 Tracheostomy :ormal La$s 22.An 1!year!ol" male presents to the emergency "epartment 'ith "ecrease" mental stat#s. /is mother states that she has notice" he has $een "rinking an" #rinating more fre*#ently o)er the past se)eral 'eeks. /e 'as har" to 'ake #p this morning an" complaine" of a$"ominal pain. Physical e.amination re)eals an afe$rile "ro'sy male

'ith clear air'ays an" mil" tachycar"ia. M#co#s mem$ranes are "ry an" his lips are cracke". /is a$"omen is mil"ly ten"er to palpation "iff#sely% $#t there is no re$o#n" or g#ar"ing. La$oratory e)al#ation re)eals a gl#cose of <4 mg7"L an" potassi#m of .8 mE*7L. An arterial $loo" gas analysis re)eals a p/ of (.11. A #rinalysis is positi)e for ketones an" gl#cose. 2T scan of the a$"omen is normal. A chest .!ray film is clear. T'o ho#rs after initiation of treatment the physician a""s potassi#m to the patient,s +? fl#i"s. =hich of the follo'ing $est e.plains this therape#tic "ecisiona6Aci"osis ca#ses e.tracell#lar "epletion of potassi#m $6/yperglycemia ca#ses potassi#m to shift to the e.tracell#lar space c6/yperkalemia 'ill protect the patient against "ysrhythmias "6 /ypokalemia 'ill res#lt as aci"osis is correcte" e6 Potassi#m sho#l" not ha)e $een a""e" to the +? fl#i"s :ormal La$s 2&. An infant comes to the office for his 1!year check #p. /is father states that he is 'orrie" that his son is smaller than he sho#l" $e. The chil",s 'eight is 1.< kg 318 l$6% an" his length is (1 cm 321 in6. /e appears to $e gro'ing appropriately on his gro'th c#r)e. =hich of the follo'ing is the most appropriate e.planation a$o#t gro'th to $e gi)en to the fathera6 +nfants #s#ally "o#$le their $irth 'eight $y 1 year $6+nfants #s#ally triple their $irth 'eight $y 1 year c6 +nfants #s#ally *#a"r#ple their $irth 'eight $y 1 year "6+nfants #s#ally "o#$le their length $y 1 year e6 +nfants #s#ally triple their length $y 1 year :ormal La$s 24.A (!year!ol" $oy arri)es at the emergency room in ac#te "istress. >)er the past & to 4 "ays he has $ecome progressi)ely ill 'ith generaliDe" fatig#e an" mil"% mi"!a$"ominal pain that ha)e $ecome stea"ily 'orse. >n physical e.amination he has a mac#lopap#lar rash on his thighs an" feet 'ith some sprea" of the rash to his $#ttocks. The rash "oes not $lanch an" the some lesions near the ankles look petechial or $r#ise". /is temperat#re is &8.4 23142.2 56 an" he is "ra'ing his knees to his chest for relief of his stomach pains. /e is na#seate" an" )omite" once $efore coming to the hospital. /e has semi!soft "ark stool%

'hich is g#aiac!positi)e. The $oy has not )oi"e" since early morning an" cannot pro)i"e a #rine sample. The "octor "etermines that he is 14B "ehy"rate" an" asks the n#rse to start intra)eno#s fl#i"s. =hich of the follo'ing is the most likely "iagnosisa6 Pancreatitis $6Aocky Mo#ntain spotte" fe)er c6:ephrotic syn"rome "6/enoch!SchPnlein P#rp#ra e6Appen"icitis :ormal La$s 2 .A ne'$orn girl is note" to $e "rooling sali)a% an" she chokes )iolently 'hen she is first fe". >n physical e.amination% she is fo#n" to ha)e a$"ominal "istention an" an imperforate an#s. There is no fist#la to the perine#m or )agina. E.amination of the #rine re)eals no meconi#m in it. Echocar"iogram an" renal sonogram are reporte" as negati)e for other congenital "efects. F!ray films sho' a$#n"ant gas in the gastrointestinal tract. Pict#res taken 'ith a metal marker tape" to the an#s% an" the $a$y hanging #psi"e "o'n% sho' that there is a significant "istance 32. cm6 $et'een the $lin" en" of the rect#m an" the anal marker. Before a "i)erting colostomy is performe"% 'hich of the follo'ing steps sho#l" $e takena6A soft nasogastric t#$e passe"% an" .!rays taken $6Bari#m in9ecte" thro#gh the anal "imple c6Bari#m s'allo' "6:asogastric s#ction for at least 24 ho#rs e6S#rgical e.ploration of the perine#m to see 'hether a primary :ormal La$s 2<. A 4!year!ol" $oy presents 'ith se)ere pains in $oth of his legs. >n physical e.amination% his temperat#re is &(.( 2 388.1 56% $loo" press#re is 1417<1 mm /g% p#lse is 8<7min% an" respirations are 1(7min. /e is note" to ha)e marke" pallor on his lips an" palpe$ral con9#ncti)a. :#mero#s p#rp#ra an" petechiae are note" on his skin. /is spleen is palpa$le & cm $elo' his left costal margin. La$oratory e)al#ation re)eals a 'hite $loo" cell co#nt of 1<447mm&G hemoglo$in% <.1 g7"LG an" platelets% &<%4447mm&. =hich of the follo'ing "iagnoses is most consistent 'ith these fin"ings-

a6Ac#te lymphocytic le#kemia $6Aplastic anemia c6/enoch!SchPnlein p#rp#ra "6+mm#ne throm$ocytopenic p#rp#ra e6 Throm$otic throm$ocytopenic p#rp#ra :ormal La$s 2(. A 1&!year!ol" healthy $oy presents to the physician for a ro#tine health maintenance )isit. A #rinalysis re)eals 1E protein#ria. There is no hemat#ria or $acter#ria. /is physical e.amination is #nremarka$le% re)ealing no e"ema an" a normal $loo" press#re. =hich of the follo'ing is the most likely "iagnosisa6Ac#te glomer#lonephritis $6+gA nephropathy c6Minimal change "isease "6>rthostatic protein#ria e6 Urinary tract infection :ormal La$s 21.An 1!month!ol" $oy is "iagnose" 'ith o$str#cti)e hy"rocephal#s. /e 'as $orn premat#rely after a 2<!'eek gestation to a 2 !year!ol" primigra)i"a an" "e)elope" seiD#res an" hypo.emia soon after $irth. =hich of the follo'ing is the most likely #n"erlying ca#se of this clinical co#rsea6Arnol"!2hiari malformation $6Cerminal matri. hemorrhage c6Mernicter#s "6Peri)entric#lar le#komalacia e6Ulegyria :ormal La$s 28.A yo#ng /ispanic co#ple $rings their first chil" to their physician for a 'ell!chil" e.amination. The chil" is a &!month!ol" healthy!appearing infant% 'hose 'eight is at the 4 th percentile of the normal gro'th c#r)e. @#ring e.amination% the physician o$ser)es an area of $l#e!$lack pigmentation o)er the $#ttocks. The parents say that it 'as present from the time of $irth. =hich of the follo'ing is the most appropriate ne.t step in management-

a6Tell parents that this is a normal fin"ing $6Tell parents that this is a $r#ise ca#se" $y tra#ma c6>r"er 2T7MA+ scans to r#le o#t "ysraphism "65ile a report of s#specte" chil" a$#se :ormal La$s &4.A 14!year!ol" $oy presents 'ith "ecrease" e.ercise tolerance. /e is note" to ha)e a gra"e +++7?+ systolic e9ection m#rm#r $est hear" at the left #pper sternal $or"er an" a gra"e ++7?+ mi"!"iastolic m#rm#r at the lo'er left sternal $or"er. The first heart so#n" is normal. The secon" heart so#n" is 'i"ely split an" fi.e". A right )entric#lar imp#lse is palpate". >n a chest roentgenogram% the p#lmonary artery segment is enlarge"% an" p#lmonary )asc#lar markings are increase". An E2C sho's right a.is "e)iation. =hich of the follo'ing congenital heart "iseases "oes this $oy most likely ha)ea6Aortic stenosis $6Atrial septal "efect c62oarctation of the aorta "6Patent "#ct#s arterios#s e6?entric#lar septal "efect :ormal La$s &1.A 1(!year!ol" girl presents to the emergency "epartment 'ith a 2! "ay history of a painf#l an" s'ollen right $ig toe. She also has ha" a fe)er% 'ith temperat#res #p to &1.8 2 3142 56% at home for 2 "ays. >n physical e.amination% her temperat#re is 141.1 5 3&1.1 26. /er first metatarsal 9oint of the right foot is marke"ly s'ollen an" )ery painf#l to to#ch. An aspirate" fl#i" from the 9oint re)eals a 'hite $loo" co#nt of & %4447 mm&. =hich of the follo'ing is the most likely "iagnosisa6R#)enile rhe#matoi" arthritis $6Co#t c6Lyme "isease "6Pse#"ogo#t e6 Septic arthritis :ormal La$s

&2.A 4!'eek!ol" $oy is $ro#ght to clinic $y his mother $eca#se of a 1 "ay history of la$ore" $reathing. /is $irth 'as #ne)entf#l an" imm#niDations ha)e $een #p to "ate. /is mother reports that the patient "e)elope" con9#ncti)itis on the fo#rth "ay of life. >n physical e.amination% he is $reathing rapi"ly at 44 $reaths per min#te an" is afe$rile. /is chest re)eals $ilateral inspiratory crackles an" a slight 'heeDe. >n chest .!ray% $ilateral pne#monia is e)i"ent. The le#kocyte co#nt is ele)ate" at 1 %444 'ith 44B eosinophils. =hich of the follo'ing is the most likely pathogen ca#sing the patient,s symptomsa6Ascaris l#m$ricoi"es $62hlamy"ia trachomatis c6Mycoplasma pne#moniae "6Pne#mocystis carinii e6 ?aricella Doster )ir#s :ormal La$s &&.A 1 !year!ol" girl 'ith type 1 "ia$etes mellit#s presents to her primary care "octor for a ro#tine check #p. Per#sal of her $loo" s#gar chart in"icates that her recor"e" $loo" gl#cose le)els are ro#tinely $et'een 124 an" 1 4 mg7"L $efore $reakfast% "inner an" $e"time% 'ith the normal $eing 11< mg7"L. She is on :P/ an" reg#lar ins#lin. =hich of the follo'ing is the ne.t appropriate stepa6 @ecrease the "osage of :P/ $6@ecrease the "osage of Aeg#lar ins#lin c6Make no changes an" o$tain a glycosylate" /$ test "6+ncrease the "osage of :P/ e6@ecrease the "osage of Aeg#lar ins#lin :ormal La$s &4. A ne'$orn infant is note" to ha)e microcephaly after $irth. /is mother is &1 years ol". She also has a !year!ol" son 'ho is mentally retar"e" an" she ha" one pre)io#s secon"!trimester miscarriage. +n a""ition% she has a genetic "isease that re*#ire" a special "iet% $#t she "iscontin#e" the "iet many years ago. >n physical e.amination% the infant,s 'eight an" length are $oth $elo' the 14th percentile for his gestational age. /e is also note" to ha)e a gra"e +++ systolic e9ection m#rm#r $est hear" at the lo'er left sternal $or"er. =hich of the follo'ing con"itions "oes the mother most likely ha)ea65ragile F syn"rome

$6Calactosemia c6/ypothyroi"ism "6Maple syr#p #rine "isease e6Phenylketon#ria :ormal La$s & .A !month!ol" infant presents 'ith "iarrhea an" "ecrease" acti)ities for the past & "ays. >n physical e.amination% his p#lse is 1<27min. /is anterior fontanelle is s#nken% an" he has skin tenting. La$oratory in)estigation re)eals so"i#m% 1< mE*7LG potassi#m% .1 mE*7LG chlori"e% 121 mE*7L% $icar$onate% 14 mE*7LG creatinine% 4.8 mg7"LG $loo" #rea nitrogen% 48 mg7"LG an" gl#cose% 1 4 mg7"L. Eight ho#rs after +? fl#i" therapy is starte"% the infant "e)elops a generaliDe" seiD#re. =hich of the follo'ing is the most likely etiology of the seiD#rea6/yperglycemia $6/yperkalemia c6+"iopathic epilepsy "6Aapi" correction of hypernatremia e6 Aapi" correction of meta$olic aci"osis :ormal La$s &<. A 4!year!ol" girl is fo#n" "rinking a $ottle of li*#i" "rain cleaner an" is imme"iately $ro#ght to the emergency "epartment. She appears to $e )ery irrita$le an" is #n'illing to s'allo' any li*#i". E.amination of her oral ca)ity re)eals no e)i"ence of $#rns or #lcerations. =hich of the follo'ing is the most appropriate management of this patienta6Bari#m s'allo' $62T of the a$"omen c6:P> for 12 ho#rs% then clear li*#i" "iet for & "ays "6+n"irect laryngoscopy e6 Esophagoscopy :ormal La$s &(.A 4!year!ol" girl 'ith sickle cell "isease presents to the emergency "epartment 'ith a temperat#re of &8.< 2 314&.2 56. >ther than irrita$ility% the physical e.amination is #nremarka$le. La$oratory e)al#ations re)eal a 'hite $loo" cell co#nt of 11%2447mm&% 'ith 11B

polymorphon#clear ne#trophils% 14B lymphocytes% an" 2B monocytes% an" a hemoglo$in of (.< g7"L. =hich of the follo'ing is the most appropriate ne.t step in managementa6>$ser)e the chil" pen"ing $loo" c#lt#re res#lts $6A"minister amo.icillin orally c6A"minister ceftaDi"ime an" gentamicin intra)eno#sly "6A"minister ceftria.one intra)eno#sly e6 A"minister )ancomycin an" gentamicin intra)eno#sly :ormal La$s &1.A pre)io#sly healthy 1&!year!ol" girl presents to the emergency "epartment 'ith an ac#te onset of re" #rine after she playe" soccer in the morning. /er physical e.amination is #nremarka$le. Urinalysis sho's a re" colorG p/% <.2G specific gra)ity% 1.424G gl#cose% negati)eG $loo"% E4G protein% traceG nitrite% negati)eG le#kocyte esterase% negati)eG 'hite $loo" cell% 47hpfG re" $loo" cell% 17hpf. =hich of the follo'ing is the most likely e.planation of the re" #rinea6Clomer#lonephritis $6/emat#ria c6+ngestion of foo" coloring "6Myoglo$in#ria e6Presence of #rates f6Tra#ma g6Urinary tract infection :ormal La$s

&8.A 4!year!ol" $oy% 'ho has a )entric#loperitoneal sh#nt for congenital hy"rocephal#s% "e)elops fe)er% hea"ache% irrita$ility% lethargy% photopho$ia% an" )omiting. /is temperat#re is &8.< 2 314&.2 56. /e is note" to ha)e n#chal rigi"ity% 'ith the presence of $oth Mernig,s an" Br#"Dinski,s signs. The sh#nt tract is erythemato#s on the s#rface. A l#m$ar p#nct#re is performe" an" sho's a =B2 of 44%4447mm& 'ith 1 B ne#trophils% a gl#cose concentration of 41 mg7"L% an" a protein concentration of 1<8 mg7"L. =hich of the follo'ing is the most likely pathogena6/aemophil#s infl#enDae

$6:eisseria meningiti"is c6Pse#"omonas aer#ginosa "6Staphylococc#s epi"ermi"is e6Streptococc#s pne#moniae :ormal La$s 44.A female infant is $orn $y )aginal "eli)ery at &8 'eeks, gestational age 'itho#t any significant complications. There is no history of any genetic "iseases in the family. She is note" to ha)e a port!'ine stain on the right si"e of her face that is 4 cm in length an" & cm in 'i"th. =hich of the follo'ing treatment mo"alities offers the $est palliation for cosmetic p#rposesa62ryos#rgery $6P#lse" "ye laser c6Aa"iation therapy 'ith gamma particles "6Skin grafting e6Topical corticosteroi" therapy :ormal La$s 41.A 4!year!ol"% apparently healthy chil" is e.amine" $y a pe"iatrician. The pe"iatrician hears a lo#" systolic e9ection m#rm#r 'ith a prominent systolic e9ection click. /e also hears a soft% early "iastolic m#rm#r. Both m#rm#rs are hear" $est at the #pper right sternal $or"er. E2C sho's left )entric#lar hypertrophy. =hich of the follo'ing is the most likely "iagnosisa6Aortic )al)e stenosis $6Atrial septal "efect c6Tetralogy of 5allot "6Transposition of great arteries e6?entric#lar septal "efect :ormal La$s 42. A <!year!ol" girl 'ith ac#te lymphocytic le#kemia 3ALL6 is a"mitte" to the hospital 'ith a generaliDe" )esic#lar rash an" high fe)er. She has no pre)io#s history of chickenpo. an" has ne)er recei)e" imm#niDation for )aricella. /er 4!year!ol" sister recently reco)ere" from chickenpo. that starte" a$o#t 14 "ays ago. >n a"mission% her

temperat#re is &1.4 2 3141.1 56% $loo" press#re is 847 1 mm /g% p#lse is 147min% an" respirations are 247min. =hich of the follo'ing is the most serio#s complication of )aricella that might occ#r in this patienta6Arthritis $62ell#litis c6En"ocar"itis "6/epatitis e6 Pne#monia :ormal La$s 4&.A !year!ol" $oy s#""enly $egins co#ghing 'hile eating pean#ts. /e is choking an" gagging. =hen he is $ro#ght to the emergency "epartment% $#t he is a'ake an" is a$le to gi)e his name. >n physical e.amination% his )ital signs are sta$le. >n e.amination of the chest% inspiratory stri"or an" intercostal an" s#prasternal retractions are apparent. =hich of the follo'ing is the most appropriate initial step in managementa6 Allo' patient to clear foreign o$9ect $y spontaneo#s co#ghing $62lear oropharyn. 'ith m#ltiple $lin" s'eeps 'ith finger c6Position patient an" perform $ack $lo's "6Stan" $ehin" patient an" perform a$"ominal thr#sts e6Perform emergency tracheostomy an" take to s#rgery :ormal La$s 44. A 8!year!ol" $oy presents 'ith a &!month history of m#ltiple episo"es of s#""en a'akening at night. /is mother states that 'hen he 'akes #p s#""enly% he screams% 0CoV Cet a'ayV CoV0 an" "oes not respon" to the parents. /is eyes are 'i"e open% an" he s'eats hea)ily an" looks scare". The parents ha)e ha" to str#ggle to a'aken him. After the episo"es% he has no memory of 'hat happene". =hich of the follo'ing is the most likely "iagnosisa62onf#sional aro#sals $6:ight terrors c6:ightmares "6>$str#cti)e sleep apnea e6 Panic "isor"er :ormal La$s

4 .A mentally retar"e" 14!year!ol" $oy presents 'ith arthritis% nephrolithiasis% an" progressi)e renal fail#re. Since his first years of life% he manifeste" pec#liar ne#rologic a$normalities consisting of self! m#tilati)e $iting of the lips an" fingers% choreoathetosis% an" spasticity. T'o male relati)es on his mother,s si"e presente" 'ith a similar con"ition an" "ie" in their teens. =hich of the follo'ing is the most likely "iagnosisa6 2hronic lea" into.ication $65ragile!F syn"rome c6Co#t "6/#ntington "isease e6 Lesch!:yhan syn"rome :ormal La$s 4<. A pre)io#sly healthy (!year!ol" girl comes to the office 'ith complaints of episo"ic a$"ominal pain o)er the past se)eral months. The pain is peri#m$ilical an" sharp $#t "oes not 'ake her from sleep or interfere 'ith play. She has no fe)er% 9oint complaints% or constipation or "iarrhea. Cro'th an" "e)elopment ha)e $een normal. The physical e.amination is 'ithin normal limits. =hich of the follo'ing is the most likely "iagnosisa6Ac#te appen"icitis $6Ac#te cholecystitis c62rohn "isease "65#nctional a$"ominal pain e6+rrita$le $o'el syn"rome :ormal La$s 4(.A 1 172 !year!ol" girl is sent to a chil"ren,s hospital for e)al#ation follo'ing a nose$lee" 'hich 'as so se)ere as to re*#ire nasal packing an" transf#sion of platelet concentrates. =hen a $loo" sample ha" $een "ra'n in the emergency room for ser#m chemistry st#"ies% the local hospital la$oratory ha" note" that the clot that forme" 'as #n#s#al in that it faile" to retract. Peripheral $loo" smear o$taine" $y finger p#nct#re sho'e" an appropriate n#m$er of normal!siDe" platelets% all of 'hich 'ere in"i)i"#al% 'itho#t cl#mping. At the chil"ren,s hospital% it 'as note" that the chil",s parents 'ere co#sins. Special platelet

st#"ies sho'e" that the chil",s platelet,s faile" to aggregate 'ith any physiologic aggregating agent% incl#"ing a high concentration of e.ogeno#s A@P. =hich of the follo'ing is the most likely "iagnosisa6Bernar"!So#lier syn"rome $62he"iak!/igashi syn"rome c6May!/egglin anomaly "6Throm$asthenia e6?on =ille$ran" "isease :ormal La$s 41. A 2!year!ol" girl is $ro#ght to the clinic 'ith hea"ache% )omiting% an" pallor. /er $loo" press#re is 1&4714 mm /g. >n physical e.amination% she is note" to ha)e aniri"ia an" a large a$"ominal mass. A$"ominal scanning re)eals a poorly )asc#lariDe" t#mor in the #pper pole of the right ki"ney. =hich of the follo'ing is the most likely ca#se of this presentationa6 @eletion of a gene on chromosome 11 $65ragile F syn"rome c6Translocation of chromosome 8 an" 21 "6Trisomy 1& e6T#rner syn"rome :ormal La$s 48.A <!year!ol" $oy 'ith mental retar"ation has recently $een "iagnose" 'ith 5ragile F syn"rome. /is 8!year!ol" sister appears to $e of normal intelligence $#t has symptoms of attention "eficit hyperacti)ity "isor"er 3A@/@6. =hat is the first test that is in"icate" in her 'ork!#p for A@/@a6EEC $62ytogenetic testing c6MA+ "6+ntelligence *#otient 3+N6 test e6Urine for meta$olic screen :ormal La$s 4. A 1 !year!ol" $oy comes to the physician for a")ice a$o#t his facial acne. >n e.amination% the patient has mil" to mo"erate acne% mostly

consisting of open come"ones% some close" come"ones% an" a fe' p#st#les on the forehea" an" cheeks. =hich of the follo'ing is the $est a")ice to gi)e this patienta6A)oi"ance of chocolate an" spicy foo"s $65re*#ent face 'ashing 'ith strong soap c6Topical application of tretinoin or a"apalene "6Treatment 'ith oral anti$iotic e6>ral treatment 'ith isotretinoin :ormal La$s

(*!lanations lock :> (*!lanations :) (*!lanation: 7he correct answer is C. &elvic inflammatory "isease '&I%) can be a "ifficult "ia$nosis to make in a teena$e $irl. 7here may be a history of abnormal uterine blee"in$ an" "ysmenorrhea. 7he teena$er may not always be forthri$ht with information about se*ual activity. +ost !atients with &I% have si$ns of ab"ominal !ain, lower ab"ominal ten"erness, an" the !elvic e*amination reveals cervical motion ten"erness. In a""ition, the !elvic e*amination may reveal !urulent cervical "ischar$e, an" an a"ne*al mass or ten"erness may be !resent. -ever over 0@.> C is frequently !resent. ,aboratory tests may reveal an elevate" white bloo" cell count an" a se"imentation rate $reater than :/ mmBhr. 7he most common !atho$ens in &I% are the $onococcus an" Chlamy"ia. ,ess frequently, &I% is cause" by a mi*e" infection with anaerobic an" aerobic bacteria.In this case, the emer$ency !hysician was initially concerne" the !atient mi$ht have an ecto!ic !re$nancy, but a serum hC. was !erforme" an" the result was ne$ative. An ecto!ic !re$nancy may have been more likely if there ha" been a"ne*al fullness an" ten"erness on one si"e without !ain on cervical motion. An ovarian cyst 'choice A) woul" not cause cervical motion ten"erness. A"ne*al ten"erness woul" be !resent, but not fullness.Cervical cultures for $onorrhea an" Chlamy"ia woul" be ne$ative. A !elvic ultrasoun" woul" be hel!ful in "ifferentiatin$ an ovarian cyst from a tubo4ovarian abscess. In the absence of back !ain or costovertebral ten"erness, !yelone!hritis 'choice ) woul" be unlikely. 7here is no mention of the results of a urinalysis or urine culture tests. Consti!ation 'choice %) woul" be unlikely to result in ri$ht lower qua"rant !ain. In ol"er chil"ren an" a"olescents, consti!ation may be a cause of left u!!er qua"rant !ain or referre" !ain to the left lower chest area. It is always im!ortant in a !e"iatric cases of ab"ominal !ain to obtain a history of bowel movements. (n"ometriosis 'choice () usually has a much more chronic history of !ain e!iso"es. -requently there may be intermenstrual blee"in$ e!iso"es. A family history of en"ometriosis may be hel!ful. Im!ortantly, the !resence of cervical motion ten"erness favors a "ia$nosis of !elvic inflammatory "isease. ;) (*!lanation: 7he correct answer is

A. 7his boy most likely has familial short stature '-))). Chil"ren with -)) usually have a normal birth wei$ht an" len$th. At the a$e of ;40 years, however, their $rowth be$ins to "ecelerate an" "ro!s to aroun" the /th !ercentile. 7he onset an" !ro$ression of !uberty in chil"ren with -)) are normal. one a$e is ty!ically consistent with the chronolo$ic a$e. A "ecrease" com!lement C0 level 'choice ) may su$$est chronic inflammatory "isor"ers. ut, the lack of any si$ns or sym!toms makes any chronic inflammatory "isor"er unlikely. A "ecrease" serum albumin concentration 'choice C) can be secon"ary to a variety of con"itions, such as ne!hrotic syn"rome an" malnutrition. ut, the lack of su!!ortive history an" !hysical e*amination "ata makes these con"itions unlikely. %ecrease" thyroi" stimulatin$ hormone 'choice %) su$$ests hy!erthyroi"ism as the etiolo$y of the boyAs short stature, but it is hi$hly unlikely in this case. An increase" serum creatinine level 'choice () in"icates renal failure, but this is inconsistent with the chil"As history an" !hysical e*amination. 0) (*!lanation: 7he correct answer is (. &yloric stenosis "evelo!s in the first weeks of life. It is cause" by hy!ertro!hy of the !yloric muscle, which obstructs $astric outflow. 7he inci"ence is hi$her in males an" first4born infants. 7he sym!toms inclu"e !ro$ressively worsenin$ vomitin$, which becomes !ro<ectile an" is non4bilious. On e*amination, !eristaltic waves may be seen, an" an olive4si#e" mass is usually !al!ate" in the ri$ht u!!er qua"rant. %uo"enal atresia 'choice A) is usually associate" with other con$enital anomalies. )ym!toms inclu"e bilious vomitin$, ab"ominal "istention, an" failure to !ass meconium. F!ri$ht ab"ominal *4ray films show the classic H"ouble4bubble si$n.H Intussusce!tion 'choice ) !resents with vomitin$, bloo"y stool, an" colicky ab"ominal !ain. As the obstruction !ro$resses, the vomitus becomes bile4staine". On e*amination, there is usually a sausa$e4sha!e" mass in the "istribution of the colon. 2irschs!run$ "isease 'choice C), or a$an$lionic me$acolon, is associate" with failure to !ass meconium or consti!ation an" ab"ominal "istention. %ia$nosis is ma"e by rectal bio!sy. +i"$ut volvulus 'choice %) can occur at any a$e but is common in infancy. )ym!toms inclu"e bilious vomitin$, ab"ominal "istention, !ain, an" bloo"y stools. An u!!er $astrointestinal series is "ia$nostic for volvulus showin$ a HcorkscrewH narrowin$ of the "istal "uo"enum.

?) (*!lanation: 7he correct answer is (. 7he infant in this clinical vi$nette has hemorrha$ic "isease of the newborn as a result of vitamin 5 "eficiency. It was a ma<or cause of blee"in$ in neonates in the !ast, but it is now uncommon because of the routine a"ministration of vitamin 5 at birth. 2owever, it is still encountere" in situations in which babies are born outsi"e the hos!ital. 7he normal newborn has a mo"erate "eficiency of the vitamin 5Y"e!en"ent coa$ulation factors. 7he !lasma levels of these factors fall even further "urin$ the first ;4/ "ays of life, rise a$ain when the infant is D4:? "ays ol", an" attain normal a"ult levels at about 0 months of a$e. 7his variation usually "oes not !ro"uce any blee"in$ or bruises. 2owever, in hemorrha$ic "isease of the newborn, the initial fall is accentuate", an" the restoration is "elaye" an" incom!lete. As a result, coa$ulation abnormalities become severe an" blee"in$ may occur. All newborns shoul" receive >./4:.> m$ of vitamin 5 intramuscularly within the first hour after birth. &rematurity has been associate" with hemorrha$ic "isease of the newborn. %elaye" fee"in$, breast4fee"in$, vomitin$, severe "iarrhea, an" antibiotics also "elay the coloni#ation of the $ut by bacteria. lee"in$ is usually severe an" occurs most commonly on the ;n" or 0r" "ay of life. 7he most common manifestations are melena, lar$e ce!halohematomas, an" blee"in$ from the umbilical stum! an" after circumcision. .enerali#e" ecchymoses, often without !etechiae, intracranial blee"in$, an" lar$e intramuscular hemorrha$es, also may "evelo! in severe cases. In infants with hemorrha$ic "isease of the newborn, the !rothrombin time '&7) is always !rolon$e". 7he !artial thrombo!lastin time '&77) an" the thrombin time are also !rolon$e". )!ecific factor assays reveal "eficiencies of !rothrombinG factors 1II, IE, an" EG an" !roteins C an" ). 7he blee"in$ time an" the !latelet count usually are within normal limits. In the "ifferential "ia$nosis of hemorrha$ic "isease of the newborn, virtually all causes of blee"in$, !articularly thrombocyto!enia an" "isseminate" intravascular coa$ulation '%IC), must be consi"ere". -actor 1III "eficiency 'choice A), or hemo!hilia A, is cause" by factor 1III "eficiency. -actor 1III is the critical cofactor for $eneration of factor Ea by factor IEa. )i$nificant !rolon$ation of the &7 is not foun" in hemo!hilia A. -actor IE "eficiency 'choice ), or hemo!hilia , is not associate" with si$nificant !rolon$ation of the &7. I"io!athic thrombocyto!enic !ur!ura 'choice C) is usually characteri#e" by a !latelet count of P;>,>>>Bmm0. It often follows an acute infection an" has s!ontaneous resolution within ; months. ,iver "isease 'choice %) woul" cause !rolon$ation of &7 an" &77, as well as liver en#ymes.

/) (*!lanation: 7he correct answer is %. .iven the fact that this !atient has ha" a lifelon$ history of be"wettin$, the initial evaluation will inclu"e a urinalysis to rule out infection or blee"in$. No neurolo$ic "ysfunction e*ists in this case. An intravenous !yelo$ram woul" be nee"e" to evaluate for renal failure or chronic urinary tract infections 'choice A). A renal ultrasoun" woul" hel! evaluate structural "ama$e but is not in"icate" in the initial evaluation 'choice ). ;?4hour urine collection is commonly "one in the evaluation of ne!hro!athy in "iabetics 'choice C). A C7 scan at this time woul" not be in"icate" at this sta$e of evaluation 'choice (). C) (*!lanation: 7he correct answer is . 7he chil"As story is worrisome for shaken baby syn"rome, in which the sym!toms may not correlate with the !hysical fin"in$s. 7his "ia$nosis shoul" be consi"ere" in any infant !resentin$ with a "issonant history sus!icious of chil" abuse. 7he chil"As fontanelles are full, in"icative of increase" intracranial !ressure. A retinosco!ic e*amination will in"icate if this is in"ee" the case, since blurre" fun"i woul" su$$est increase" !ressure. A retinosco!ic e*amination can be "one faster than a hea" C7 'choice A). 7he !atient may ultimately nee" a hea" C7 but the eye e*amination shoul" be "one first. Ammonia levels 'choice C) shoul" be checke" if he!atic ence!halo!athy is a consi"eration. 7his is a !ossibility if 6eye syn"rome is on the "ifferential. 7he increase" fontanelle !ressure lea"s to a "ia$nosis of trauma. en#o"ia#e!ines may be nee"e" 'choice %) in the short term to sto! the sei#ure, but it is im!ortant to "etermine the cause of the sei#ure before intervenin$. ecause the !atient has increase" intracranial !ressure 'su$$este" by the full fontanelles), a lumbar !uncture may cause uncal herniation an" shoul" be avoi"e" 'choice (). D) (*!lanation: 7he correct answer is (. )cal"in$ burns in chil"ren shoul" always raise the !ossibility of chil" abuse, an" the !roblem is virtually certain to have occurre" when the !attern of the burns "oes not match the story $iven by the !arents. In this case, ha" the chil" in"ee" !ulle" a !ot of

boilin$ water over himself, the burns woul" follow the "istribution of water runnin$ from hea" to toe. urns on both buttocks are classic for a chil" who is hel" by arms an" le$s, an" "i!!e" into boilin$ water. +afeni"e acetate 'choice A) is not the to!ical a$ent of choice, unless "ee! !enetration is nee"e". )ilver sulfa"ia#ine woul" be the !ro!er choice for to!ical use. In this case, however, the burn itself is less im!ortant than the future welfare of the chil". (arly e*cision an" $raftin$ 'choice ) is in"icate" only in burns that are clearly thir" "e$ree. In chil"ren, those woul" be "ee! re" an" "ry. 7he "escri!tion here is that of secon"4"e$ree burns 'moist, blisters, !ainful). ("ucation of the !arents 'choice C) assumes the in<ury was acci"ental. We have reason to believe it was intentionally inflicte". -lui" resuscitation 'choice %) woul" be minimal in a burn of this si#e. Assumin$ the entire surface of both buttocks is involve", it woul" still re!resent less than :>= of bo"y surface an" it is all secon" "e$ree. @) (*!lanation: 7he correct answer is . 7his is !atent "uctus arteriosus, which is a failure of closure of the "uct between the !ulmonary artery an" the aorta. As many as @>= of si$nificantly !remature 'P;@ week $estation) infants have !atent "uctus arteriosus. In term infants, "elaye" closure is "ia$nose" if the murmur of the !atent "uctus '"escribe" in the question stem) is still !resent at C4@ weeks of a$e. Infants shoul" be evaluate" for other car"iac "isease, since a !atent "uctus arteriosus may be !artially com!ensatin$ for other car"iac anomalies. Infants with heart failure require !rom!t sur$ical correction. Infants without heart failure or com!licatin$ car"iac "efects ty!ically un"er$o elective sur$ery at C months to 0 years to re"uce the risk of infective en"arteritis later involvin$ the !atent "uctus. Coarctation of the aorta 'choice A) will cause "iminishe" !ulses in the le$s an" sometimes arms. &eri!heral !ulmonic stenosis 'choice C) causes murmurs hear" over the lun$ fiel"s. &ersistent truncus arteriosus 'choice %) is a failure of the aorta to se!arate from the !ulmonary artery, an" causes heart failure within "ays to weeks of birth. 1entricular se!tal "efect 'choice () causes a murmur hear" best at the lower left sternal bor"er. I) (*!lanation: 7he correct answer is

. 7he varicella vaccine is recommen"e" at any visit on or after the first birth"ay ':; months) for susce!tible chil"ren 'those who have not ha" the virus). -ebruary 'choice A) is incorrect because the infant will be :: months. A!ril, +ay, an" Nune 'choices C, %, an" () woul" all be a!!ro!riate times to receive the varicella vaccine e*ce!t that the question asks for the earliest !ossible "ate. A $eneral summary of routine immuni#ations is as follows: 2e! vaccine4 At birth, : month, an" C months %7a&4 At ;, ?, an" C months, :/4:@ months, an" ?4C years 7" booster4 ::4:; years, an" then every :> years 2ib4 At ;, ?, an" C months, an" :;4:/ months I&14 At ; an" ? months, C4:@ months, an" ?4C years ++64 At :;4:/ months an" ?4C years 1aricella4 :; months :>) (*!lanation: 7he correct answer is . 9ou are likely to be teste" about the criteria for !ro!hyla*is of tuberculosis. 7he usual a$ent chosen, unless a resistant strain of tuberculosis is clinically sus!ecte", is isonia#i". 7he chemothera!eutic "ose is $iven for C to I months an" is 0>> m$B"ay for a"ults or :>Bm$Bk$B"ay for chil"ren. Chemo!ro!hyla*is is in"icate" in the followin$ $rou!s: :) !ersons whose tuberculin skin test has converte" from ne$ative to !ositive within the !revious ; yearsG all small chil"ren 'P? years of a$e) who are either e*!ose" by known close contact to a !erson with untreate" tuberculosis or who have a !ositive &&%G all 2I1 !atients with !ositive &&%G el"erly !atients with a "efinite conversion of &&%G &&% !ositive !ersons with a!ical scarsG an" &&% !ositive !ersons with si$nificant risk of recurrence "ue to "iabetes mellitus, !rolon$e" corticosteroi" thera!y, $astrectomy, en"4 sta$e renal "isease, or $astric sta!lin$. In youn$ chil"ren, such as in this case, it is !articularly im!ortant not to "elay chemo!ro!hyla*is until the &&% test becomes !ositive, because these chil"ren sometimes have very ra!i" !ro$ression of tuberculosis, an" may be very ill by the time that the &&% is re!eate". (thambutol 'choice A) is usually use" in treatment re$imens when isonia#i" resistance is sus!ecte". 6ifam!in 'choice C) is a""e" to isonia#i" for initial treatment of tuberculosis an" can also be a com!onent in retreatment re$imens. )tre!tomycin 'choice %) is usually use" as !art of retreatment re$imens. No chemo!ro!hyla*is 'choice () woul" be !otentially "an$erous in this chil", who is in a !o!ulation that ten"s to have ra!i"ly !ro$ressive tuberculosis. ::)

(*!lanation: 7he correct answer is . %iabetic ketoaci"osis '%5A) is a life threatenin$ com!lication of "iabetes mellitus. %5A e*ists if there is hy!er$lycemia '$lucose O0>>), ketonemia, aci"osis '!2 PD.0>, 2CO0 P :/) with clinical sym!toms of "iabetes. 7he mainstay of treatment for %5A is intravenous insulin. Intubation 'choice A) is rarely necessary in %5A !atients. 7he !atient is alert enou$h to $ive some history an" a!!ears to be o*y$enatin$ well. Intramuscular ceftria*one 'choice C) may be necessary in this case to treat an un"erlyin$ infection, which may have !reci!itate" this e!iso"e of %5 A. 2owever, this is certainly not first4line treatment for %5 A. Intravenous flui"s 'choice %) are also vital for %5A !atients, as they are severely "ehy"rate". 2owever, !otassium is not initially a""e" to intravenous flui"s, es!ecially if the !otassium level is $reater than C.> m(qB,. +annitol 'choice () is use" in sus!ecte" cases of cerebral e"ema, a com!lication of %5 A. Clinical si$ns inclu"e "eterioration in mental status, hea"ache, an" unequal !u!ils. :;) (*!lanation: 7he correct answer is . %uchenne muscular "ystro!hy is the most common ty!e of muscular "ystro!hies. It is an E4linke" $enetic "isor"er an" ha!!ens in ::0C>> boys. It is characteri#e" by !ro$ressive muscle weakness cause" by "e$eneration of muscle fibers. 7he "isease is rarely sym!tomatic at birth, but usually "evelo!s to a clinically evi"ent sta$e before the a$e of / years. Weakness is !rominent in !ro*imal muscles. Affecte" chil"ren often !resent with toe4walkin$ an" "ifficulty climbin$ stairs. 7he $ait is wa""lin$, an" !atients often fall. 7he .ower si$n is very characteristic of %uchenne muscular "ystro!hy. It is consi"ere" !ositive if the !atient uses his han"s to HwalkH u! the le$s when $oin$ from a !rone to an u!ri$ht sittin$ !osition because he "oes not have enou$h !ro*imal muscle !ower to $et u! in a normal fashion. &seu"ohy!ertro!hy of the calves is also a !rominent feature. +any !atients "ie in their ;>s because of res!iratory failure. In a Hfoot "ro!H $ait 'choice A), the !atients must lift their le$s hi$h enou$h to avoi" tri!!in$ over their "roo!in$ feet. It is characteristic of !eri!heral neuro!athy, which ty!ically causes "istal muscle weakness rather than !ro*imal muscle weakness. %ee! ten"on refle* 'choice C) is usually not affecte" in muscle "ystro!hy. +yotonia 'choice %) is use" to "escribe slow rela*ation of the muscle after contraction an" is characteristic of myotonic muscular "ystro!hy.

7he abinski si$n 'choice (), also calle" the e*tensor !lantar refle*, is normal before :@ months of a$e, but si$nifies "ama$e of the u!!er motor neurons in !atients ol"er than this. It is also associate" with s!asticity an" increase" refle*es. :0) (*!lanation: 7he correct answer is . 7he infant in this clinical vi$nette likely has the C2A6.( syn"rome, which inclu"es colobomas, heart "efects, choanal atresia, retar"ation, $enitourinary abnormalities, an" ear anomalies. With the !resence of four anomalies, an echocar"io$ram is the lo$ical ne*t ste! to "etect any associate" heart "efect. 7he etiolo$y of the C2A6.( syn"rome is unknown, but it may involve altere" mor!ho$enesis "urin$ the secon" trimester of !re$nancy. It is not $enetically transmitte" an" is not associate" with a terato$enic effect of any substance. :?) answer is C. &seu"omembranous enterocolitis is cause" by the to*ins !ro"uce" by Clostri"ium "ifficile. It occurs in some !atients after treatment with antibiotics 'es!ecially clin"amycin, ce!halos!orins, an" amo*icillin). &atients "evelo! fever an" ab"ominal !ain with "iarrhea containin$ leukocytes an" bloo". .astroenteritis 'choice A) is an acute con"ition, usually cause" by a virus, that !resents with vomitin$ an" "iarrhea. 7here is ty!ically no bloo" in "iarrhea associate" with $astroenteritis. Irritable bowel syn"rome 'choice ) is a chronic con"ition that !ro"uces e!iso"ic "iarrhea with !erio"s of consti!ation. 7here often is bloo" in the "iarrhea but it is not associate" with a fever. )almonella infection 'choice %) is a common cause of foo" !oisonin$. -in"in$s inclu"e nausea, vomitin$, ab"ominal !ain, an" "iarrhea, often with bloo". Flcerative colitis 'choice () is a chronic con"ition that has features of bloo"y "iarrhea, ab"ominal !ain, an" wei$ht loss. :/) (*!lanation: 7he correct answer is (. 2emo!hilia is clinically very similar to hemo!hilia A, but is "ue to E4linke" "eficiency of bloo" clottin$ factor IE rather than 1III. 7his !atient nee"s factor IE re!lacement to sto! his blee"in$. 7he metho" use" for calculatin$ the amount of -actor 1III to $ive in hemo!hiliac A !atients is to multi!ly the !atientAs wei$ht in !oun"s by ;> an" then by the "esire" !lasma level in units. If this al$orithm is use" in hemo!hiliac !atients to calculate factor IE levels, it is foun" that the actual achieve" bloo" levels are

only about half that e*!ecte", !ossibly because -actor IE ten"s to bin" to the en"othelium of the vessel walls. -or this reason, it is im!ortant to either monitor the !atient for cessation of blee"in$ or check clottin$ times before assumin$ that a calculate" "ose of -actor IE ha" its inten"e" effect. As!iration of bloo" out of a <oint 'such as in this case) is usually only attem!te" after the blee"in$ !rocess is un"er control. -actor C 'choice A) "eficiency an" -actor ) 'choice ) "eficiency cause a thrombotic ten"ency. -actor 1II 'choice C) "eficiency is a rare, autosomal recessive cause of serious blee"in$. -actor 1III 'choice %) "eficiency causes hemo!hilia A. :C) (*!lanation: 7he correct answer is (. 7his infant "evelo!e" acute renal failure 'A6-) in the imme"iate !ost4o! !erio", as manifeste" by the increase in bloo" urea nitro$en an" serum creatinine an" the "ecrease in urine out!ut. A6- can be classifie" into !rerenal, renal, an" !ostrenal. &rerenal causes inclu"e hy!ovolemia secon"ary to severe "ehy"ration, hemorrha$e, an" hy!otension secon"ary to shock. 6enal causes inclu"e acute tubular necrosis 'A7N), !arenchymal "isor"ers 'e.$., $lomerulone!hritis), an" vascular "isor"ers 'e.$., renal artery thrombosis or renal vein thrombosis). &ostrenal causes inclu"e ureteral or urethral obstruction. 7his infant most likely has A7N, which is cause" by ischemic or to*ic in<ury to the ne!hrons. Ischemia can be cause" by hy!ovolemia, low car"iac out!ut states, or renal vasoconstriction. 7o*ins inclu"e contrast a$ents, antibiotics, uric aci", an" myo$lobin. A7N is characteri#e" by mil" !roteinuria, microsco!ic hematuria, an" the !resence of coarse $ranular casts in the urine. A fractional e*cretion of so"ium $reater than ;= 'or ;./= in neonates) is consistent with renal causes of A6-. 6enal ultrasono$ra!hy is the ima$in$ stu"y of choice for this !atient because it !rovi"es both anatomic an" structural information about the ki"neys. 7he stu"y is noninvasive an" can be easily "one by the be"si"e. %o!!ler stu"ies can also be "one with ultrasoun" technolo$y to assess bloo" flow in the renal vessels, the aorta, an" the inferior vena cava. C7 of the ab"omen an" !elvis 'choice A) can !rovi"e more anatomic "etails but is not a $oo" initial ima$in$ stu"y, es!ecially in this case. 7he contrast "ye nee"e" for C7 can cause further "ama$e to the ki"neys an" thus worsen renal failure. 7he same ar$ument a!!lies to intravenous !yelo$ra!hy 'choice C). Cystourethro$ra!hy 'choice ) !rovi"es structural "etails of the urinary bla""er an" the urethra, but it is not in"icate" in this case.

6enal bio!sy 'choice %) is the $ol" stan"ar" of "ia$nosin$ renal "isease, but it is not in"icate" as an initial stu"y. It mi$ht be useful in !rolon$e" renal failure with an uni"entifie" cause. :D) (*!lanation: 7he correct answer is A. Cranio!haryn$ioma is a histolo$ically beni$n e!ithelial tumor of o"onto$enic ori$in. It is ty!ically locate" in the su!rasellar com!artment, e*ten"in$ variably onto the sella, hy!othalamus, an" o!tic chiasm. 2eavy calcifications, unilocular or multilocular cysts, an" a viscous yellow flui" content are the classic features that allow the "ia$nosis on ra"iolo$ic an" $ross e*amination. +icrosco!ic e*amination will show a tumor com!ose" of islan"s of keratini#in$ e!ithelium with multifocal calcific "e!osits, a !icture similar to the most common of tooth4relate" tumors, i.e., a"amantinoma 'ameloblastoma). Cranio!haryn$iomas !robably ori$inate from mis!lace" o"onto$enic e!ithelium. ,ocation of a neo!lasm is often more im!ortant than histolo$y with re$ar" to the !ossibility of "efinite cure. Cranio!haryn$ioma hi$hli$hts this rule. Althou$h histolo$ically beni$n, recurrences are frequent, as the tumor cannot be entirely e*cise" because of its location close to vital structures 'hy!othalamus, nerves, an" bloo" vessels). 6a"ical sur$ery is usually accom!anie" by severe neurolo$ic "eficits. Cysticercosis 'choice ) is an infestation acquire" by eatin$ im!ro!erly cooke" !ork. It lea"s to formation of 'usually multi!le) cysticerci, which are cystic structures line" by a characteristic three4layere" e!ithelium an" containin$ a clear flui". It is often calcifie". .an$lio$lioma 'choice C) is a neuro$lial tumor com!ose" of variable a"mi*tures of neo!lastic $an$lionic 'i.e., neuronal) cells an" $lial cells 'either astrocytes or oli$o"en"rocytes). 7he tem!oral lobe is the most common location, an" sei#ure is the most frequent clinical manifestation. .rossly, the tumor has cystic an" soli" com!onents. &ilocytic astrocytoma 'choice %) is a well4circumscribe", frequently cystic, low4$ra"e astrocytoma that affects chil"ren an" youn$ a"ults. 7he two most common locations inclu"e the cerebellum an" the "ience!halicBhy!othalamic re$ion. It is an im!ortant "ifferential to consi"er in any tumor of the su!rasellar re$ion. 2owever, !ilocytic astrocytoma is rarely calcifie", an" the cysts "o not contain the Hmachinery oilH flui" of cranio!haryn$ioma. &ituitary a"enoma 'choice () is mainly an intrasellar tumor. A lar$e !ituitary a"enoma 'macroa"enoma) may e*ten" into the su!rasellar re$ion, but calcifications an" cysts are absent. :@) (*!lanation: 7he correct answer is C. ullous im!eti$o 'sta!hylococcal im!eti$o) is cause" by an e!i"ermolytic to*in !ro"uce" at the site of infection, most commonly by sta!hylococci of !ha$e $rou! II. 7he

to*in causes intrae!i"ermal cleava$e below or within the stratum $ranulosum. ullous im!eti$o is most common in infants an" chil"ren. It ty!ically occurs on the face, but it may infect any bo"y surface. 7here may be a few lesions locali#e" in one area, or the lesions may be numerous an" wi"ely scattere". One or more vesicles enlar$e ra!i"ly to form bullae in which the contents turn from clear to clou"y. 7he center of the thin4roofe" bulla colla!ses, an" a thin, flat, honey4colore" crust may a!!ear in the center with a bri$ht re", inflame", moist base that oo#es serum. In most cases, a tinea4like scalin$ bor"er re!laces the flui"4fille" rim as the roun" lesions enlar$e an" become conti$uous with the others .7he bor"er "ries an" forms a crust. 7he lesions have little or no surroun"in$ erythema. 6e$ional lym!ha"enitis is uncommon with !ure sta!hylococcal im!eti$o. 7here is some evi"ence that the res!onsible sta!hylococci coloni#e the nose an" then s!rea" to normal skin !rior to infection. )erious secon"ary infections 'e.$., osteomyelitis, se!tic arthritis, an" !neumonia) may follow seemin$ly innocuous su!erficial infections in infants. 7he "ru$ of choice for im!eti$o is oral ce!hale*in. Clo*acillin, "iclo*acillin, an" a#ithromycin are $oo" alteratives. ecause some cases of im!eti$o are "ue to a mi*e" sta!hylococcalBstre!tococcal infection !enicillin an" amo*icillin 'choice ) are ina"equate for treatment. Oral acyclovir 'choice A) is use" to treat her!es sim!le* virus infection. 7o!ical ketocona#ole 'choice %) is use" to treat fun$al infection of the skin. 7o!ical ;= hy"rocortisone 'choice () is ineffective a$ainst im!eti$o. :I) (*!lanation: 7he correct answer is %. 7his is tetralo$y of -allot, in which severe obstruction of ri$ht ventricular outflow an" a ventricular se!tal "efect allow uno*y$enate" bloo" to !ass from the ri$ht si"e of the heart to the left. In severe cases, cyanosis !resents at birthG in mil"er cases 'such as this baby has), it "evelo!s more slowly. 7he u!!er left sternal bor"er e<ection murmur is "ue to ri$ht ventricle outflow obstruction. 7he (C. an" chest *4ray fin"in$s "escribe" in the question stem are ty!ical for ol"er babies. (arly sur$ical re!air is now recommen"e" for tetralo$y of -allot. Com!lete atrioventricular canal "efect 'choice A) characteristically has (C. fin"in$s inclu"in$ su!erior left a*is "eviation an" a counterclockwise loo! of electrical im!ulses. 2y!o!lastic left ventricle 'choice ) causes severe heart failure with loss of !eri!heral !ulses at ;40 "ays of life. Isolate" atrial se!tal "efect 'choice C) coul" cause a murmur hear" at the u!!er left bor"er of the sternum, but woul" not show the characteristic *4ray fin"in$s illustrate" in the question stem.

7rans!osition of the $reat arteries 'choice () is characteri#e" by severe cyanosis from birth an" a normal (C.. ;>) (*!lanation: 7he correct answer is A. 7he chil"As con"ition is the autosomal recessive "isease, ata*ia4telan$iectasia, which is a multisystem "isor"er of unknown etiolo$y. 7he ata*ia is notice" in early chil"hoo", an" with time, !ro$resses to severe "isability. Choreoathetoi" movements, slurre" s!eech, o!hthalmo!le$ia, an" !ro$ressive mental retar"ation characteri#e the "isease at it a"vances. 7elan$iectasias, as "escribe" in the question stem, are a hel!ful "ia$nostic clue. 7hese chil"ren also are vulnerable recurrent sino!ulmonary infections. Immunolo$ic evaluation may "emonstrate a lack of I$A an" I$(, cutaneous aner$y, an" a !ro$ressive cellular immune "efect. Other features of the syn"rome inclu"e en"ocrine "isor"ers an" a !re"is!osition for certain cancers 'leukemias, brain cancer, an" $astric cancer). +ost of these !atients "ie of their neurolo$ic "eterioration by a$e 0>. I$. 'choices , C, an" () an" I$+ 'choices % an" () are not s!ecifically affecte" in this con"ition. ;:) (*!lanation: 7he correct answer is (. 7his !atient has irreversible muscle weakness, an" his ti"al volume is "iminishe", causin$ im!aire" alveolar ventilation. 7he tracheostomy will re"uce the "ea" s!ace an" airway resistance. It may allow "ee! suctionin$, hel!in$ effect ultimate weanin$ off the ventilator. 7he naso!haryn$eal tube 'choice A) woul" hel! only if there were an obstruction between the nose an" the !osterior naso!haryn*. &ressure controlle" ventilation is a ventilator mo"e 'choice ) an" not an ai" to s!ontaneous ventilation. &ressure su!!orte" ventilation 'choice C) can im!rove s!ontaneous breathin$, but the !atient woul" nee" to remain intubate". )u!!lemental o*y$en will im!rove o*y$enation, but not ventilation 'choice %). ;;) (*!lanation: 7he correct answer is %. 7otal bo"y !otassium is consi"erably "e!lete" "urin$ "iabetic ketoaci"osis, even when the serum !otassium is normal or elevate". 7his is "ue to renal losses from osmotic "iuresis cause by hy!er$lycemia. 7he intracellular !otassium e*chan$es for e*tracellular

hy"ro$en ions in an attem!t to correct the aci"osis. %urin$ correction of ketoaci"osis, insulin "ecreases !otassium levels. Aci"osis 'choice A) causes intracellular !otassium "e!letion, as "iscusse" above. 2y!er$lycemia 'choice ) "oes not cause !otassium to shift e*tracellularly. 2y!erkalemia 'choice C) "oes not !rotect a$ainst "ysrhythmias. 6ather, both hy!erkalemia an" hy!okalemia can in"uce "ysrhythmias. 2y!erkalemia is evi"ence" by !eake" 74waves an" hy!okalemia by F waves on the (5.. Withhol"in$ of !otassium 'choice () is "efinitely wron$ for this !atient. 2y!okalemia !re"is!oses the heart to ventricular "ysrhythmias. ;0) (*!lanation: 7he correct answer is . +ost infants are e*!ecte" to tri!le their birth wei$ht by the a$e of :; months. Infants usually "ouble their birth wei$ht 'choice A) by C months, not : year. Infants usually qua"ru!le their birth wei$ht 'choice C) by ;? months. Infants usually "ouble their len$th 'choice %) by ? years. Infants "o not usually tri!le their len$th 'choice () until they are out of infancy an" into !uberty. ;?) (*!lanation: 7he correct answer is %. 2enoch4 )chKnlein &ur!ura '2)&) is the most likely "ia$nosis. 7his boy has ab"ominal !ain with $uaiac4!ositive stools, but also has a !rominent rash, mostly on his lower e*tremities. Other characteristic fin"in$s of 2)& inclu"e hematuria an" <oint !ains. 7he illness may follow an u!!er res!iratory infection or stre! throat. 7he rash starts out as an urticarial rash an" !ro$resses to become !etechial an" !ur!uric. 7here may be a history of mi$ratory <oint !ain an" arthritis. Affecte" <oints inclu"e ankles, knees, wrists, an" elbows. If the ab"ominal !ain were "escribe" as e!i$astric with ra"iation to the back, !ancreatitis 'choice A) mi$ht have been the likely "ia$nosis.In chil"ren, !ancreatitis is frequently associate" with viral illnesses 'e.$., mum!s), "ru$s 'e.$., sulfonami"es), or un"erlyin$ systemic "isease 'e.$., lu!us). Althou$h !ancreatitis has been re!orte" in association with 2)&, it is not the most likely "ia$nosis. 6ocky +ountain s!otte" fever 'choice ) is one of the most common tick4borne "iseases. 7he ty!ical rash of 6+)- a!!ears within a week of the tick bite. It be$ins on the !alms,

soles, an" e*tremities an" s!rea"s centrally. )evere hea"ache an" !hoto!hobia are common com!laints. 7his chil" "i" not have the ty!ical fin"in$s of ne!hrotic syn"rome 'choice C) inclu"in$: !roteinuria, e"ema, an" oli$uria. Ne!hrotic syn"rome frequently follows an infectious illness. In the classic case of a!!en"icitis 'choice () !eriumbilical !ain !ro$resses with locali#ation to the ri$ht lower qua"rant.Anore*ia, nausea, vomitin$ an" chan$es in bowel movements may all occur. -ever is ty!ically low4$ra"e an" rash is not !resent. ;/) (*!lanation: 7he correct answer is A. Althou$h the !hysical fin"in$ of the im!erforate anus has $iven you a "ia$nosis, the !resentation with "roolin$ an" chokin$ is classic for eso!ha$eal atresia, which is often an associate" anomaly '!art of the H1AC7(6H constellation). In fact, the abun"ance of $as in the $astrointestinal tract is also ty!ical, as the babies $et it by way of the "istal tracheoeso!ha$eal fistula that accom!anies the ma<ority of eso!ha$eal atresias. 7he naso$astric tube coile" on itself woul" confirm the sus!ecte" "ia$nosis, which woul" si$nificantly alter the sur$ical !lans. arium in<ecte" throu$h the anal "im!le 'choice ) woul" !robably en" u! in the tissues, where it woul" be very irritatin$. In the absence of a fistula, im!erforate anus is not likely to be low. 7he stu"y woul" not hel! "efine the im!erforate anus an" woul" "o nothin$ to "ia$nose the eso!ha$eal !roblem. arium swallow 'choice C) woul" "emonstrate the eso!ha$eal atresia, but at the cost of en"in$ u! in the tracheobronchial tree as the baby vomits an" as!irates it. arium is very irritatin$ in that location. If contrast material is nee"e" to "emonstrate the atresia, water soluble contrast material woul" be !referre", an" it woul" have to be !rom!tly sucke" out before it also en"e" u! in the lun$s. %ecom!ressin$ the $astrointestinal tract 'choice %) woul" not be !ossible by naso$astric suction if the eso!ha$us in"ee" is atretic. If it is !atent, the colostomy can be "one without !rior "ecom!ression. If there is a tracheoeso!ha$eal fistula, an" for some reason it cannot be !rom!tly re!aire", a $astrostomy woul" be require" to !rotect the lun$s from as!iration of $astric aci". Attem!ts to reconnect the anal canal 'choice (), when all evi"ence !oints to a hi$h blin" en", woul" be !ointless an" woul" "o nothin$ to "ia$nose the other !otentially lethal anomaly affectin$ the eso!ha$us an" tracheobronchial tree. ;C) (*!lanation: 7he correct answer is

A. 7his boy most likely has acute lym!hocytic leukemia 'A,,). 7he si$ns an" sym!toms of A,, result from su!!resse" marrow function an" invasion of or$ans by leukemic blasts. Anemia is !resent at "ia$nosis in most !atients an" causes fati$ue, !allor, hea"ache, an$ina, or even heart failure. 7hrombocyto!enia is usually !resent, an" many !atients have clinically evi"ent blee"in$ at "ia$nosis, usually in the form of !etechiae, !ur!ura, ecchymoses, blee"in$ $ums, e!ista*is, or hemorrha$e. +ost !atients with A,, are si$nificantly $ranulocyto!enic. In a""ition to su!!ressin$ normal marrow function, leukemic cells can infiltrate normal or$ans. (nlar$ement of lym!h no"es, liver, an" s!leen is common at "ia$nosis. one !ain, thou$ht to result from leukemic infiltration of the !eriosteum or e*!ansion of the me"ullary cavity, is a common com!laint. A,, is the most common form of cancer an" the secon" lea"in$ cause of "eath in chil"ren youn$er than :/ years. A,, has a ma*imal inci"ence between ; an" :> years of a$e, with a secon", more $ra"ual rise in frequency later in life. Chil"ren with a!lastic anemia 'choice ) "o not usually have bone !ain, s!lenome$aly, an" he!atome$aly. 2enoch4)chKnlein !ur!ura 'choice C) is a vasculitis an" !resents with lower e*tremity !ur!ura without thrombocyto!enia. Immune thrombocyto!enic !ur!ura 'choice %) is a chil"hoo" "isease that often follows an acute infection. It is characteri#e" by a "ecrease in the circulatin$ number of !latelets 'P:>>,>>>Bmm0) in the absence of to*ic e*!osure or a "isease associate" with a low !latelet count. It occurs as a secon"ary effect of !eri!heral !latelet "estruction as well as "ecrease" !latelet !ro"uction. It usually resolves s!ontaneously within ; months. 7hrombotic thrombocyto!enic !ur!ura 'choice () is a systemic "isor"er characteri#e" by occlusion of the microcirculation by !latelet clum!s. 7he com!lete clinical !enta" '!resent in fewer than 0>= of cases) inclu"es consum!tive thrombocyto!enia, microan$io!athic hemolytic anemia, fever, renal "ysfunction, an" fluctuatin$ neurolo$ic "eficits. ;D) (*!lanation: 7he correct answer is %. Orthostatic !roteinuria is very common. It occurs when the total urinary !rotein e*cretion rate is hi$her while the chil" is in an u!ri$ht !osition. It is asym!tomatic, an" there is no associate" increase in renal "isease. %ia$nosis is ma"e by obtainin$ a Hfirst mornin$H urine before the chil" has ha" much time in an u!ri$ht !osition. Acute $lomerulone!hritis 'choice A) usually !resents with $ross hematuria an" may or may not be associate" with e"ema. )evere $lomerular in<ury woul" be associate" with !roteinuria.

I$A ne!hro!athy 'choice ) usually !resents with e!iso"ic, asym!tomatic $ross hematuria, with microhematuria occurrin$ between e!iso"es. &roteinuria occurs in more severe, aty!ical cases. +inimal chan$e "isease 'choice C) is characteri#e" by !roteinuria, hy!erli!i"emia, e"ema, an" hy!o!roteinemia. 7he !roteinuria is usually $reater than ;M. A urinary tract infection 'choice () may !resent with :M !roteinuria but the !atient woul" also more ty!ically have hematuria. 7he fact that there are no bacteria woul" make a urinary tract infection less likely. ;@) (*!lanation: 7he correct answer is . 7he clinical manifestations are com!atible with $erminal matri* hemorrha$e '.+2). 7he $erminal matri* is a hi$hly vasculari#e" layer of neuroecto"ermal !recursors linin$ the ventricular cavities, which is most "evelo!e" between ;; an" 0> weeks of intrauterine life. 7he vessels of the $erminal matri* are es!ecially vulnerable to hy!o*ic insults. abies born !rematurely are thus at hi$h risk of hemorrha$e in the $erminal matri* re$ion, as they often suffer from !oor o*y$enation. 7he !atho!hysiolo$ic consequences "e!en" on the severity of .+2. )evere forms of .+2 e*ten" into the ventricles an" result in "eath, or obstructive hy"roce!halus an" neurolo$ic "eficits later in life. oth ty!es of Arnol"4Chiari malformation 'choice A) 'more frequently ty!e ;) may lea" to obstructive hy"roce!halus "ue to com!ression of the fourth ventricles by the "ownwar" "is!lacement of the cerebellar tonsils. 5ernicterus 'choice C) refers to brain "ama$e "ue to accumulation of uncon<u$ate" bilirubin in the central nervous system. 7his com!lication occurs in babies sufferin$ from severe forms of <aun"ice, e.$., those with erythroblastosis fetalis. &eriventricular leukomalacia 'choice %) results from ischemic "ama$e to the !eriventricular white matter in !remature babies. In !remature infants, the !eriventricular white matter is more vulnerable to hy!o*ic in<ury. Fle$yria 'choice () is a com!lication relate" to intrauterine hy!o*ic in<ury. 7he "e!th of the sulci is more !rone to ischemia than the crests of the $yri "urin$ intrauterine life. A fetus sufferin$ from early cerebral ischemic "ama$e will thus "evelo! atro!hy of the base of the $yri, which then acquire a mushroom4like sha!e 'hence the "esi$nation: ule* U mushroom). .erminal matri* hemorrha$e, ule$yria, an" !eriventricular leukomalacia are amon$ the causes of !erinatal brain in<ury, which lea"s to a com!le* set of neurolo$ic manifestations referre" to as cerebral !alsy. ;I) (*!lanation: 7he correct answer is

A. A +on$olian s!ot is an area of hy!er!i$mentation foun" in CC= of all infants of 2is!anic, Asian, an" Native American ethnic back$roun". It is sometimes mistaken for a bruise cause" by trauma 'choice ), an" erroneously re!orte" to the authorities as evi"ence of sus!ecte" chil" abuse 'choice %). All !hysicians, an" es!ecially !rimary care !rovi"ers, shoul" be aware of this common normal feature in or"er to avoi" such mistakes. An auto!sy fin"in$ that may be mistakenly attribute" to chil" abuse is !ostmortem live"o, which is "ue to bloo" settlin$ to the "e!en"ent !arts of the bo"y once circulation sto!s. C7B+6I stu"ies 'choice C) woul" be useful to rule out the !ossibility of an un"erlyin$ neural tube "efect '"ysra!hism) in the !resence of a heman$ioma in a sacral location. 0>) (*!lanation: 7he correct answer is . One of the most common ty!es of structural con$enital heart "isease to !resent in a"olescence is atrial se!tal "efect 'A)%), an" the most common !resentation is a heart murmur. 2owever, some !atients !resent with arrhythmias, "ecrease" e*ercise tolerance, or a !ara"o*ic embolus. 7he !hysical e*amination can show classic fin"in$s of an A)%G in some cases, however, the fin"in$s may be e*tremely subtle. 7he murmur associate" with the A)% is not cause" by bloo" flow traversin$ the actual "efect but rather by the increase" volume of bloo" flow across the !ulmonary valve an", to a lesser e*tent, across the tricus!i" valve. 7hus, the murmurs of an A)% are a systolic e<ection murmur at the u!!er left sternal bor"er an" a mi"4"iastolic murmur at the lower left sternal bor"er. 7he secon" heart soun" is wi"ely s!lit an" fi*e" with re$ar" to res!iration. On !al!ation, a ri$ht ventricular im!ulse is !resent. 7he chest ra"io$ram shows evi"ence of an enlar$e" !ulmonary artery se$ment in the !osteroanterior !ro<ection. 7he su!erior vena cava sha"ow may not be visible because of the rotation of the heart secon"ary to ri$ht ventricular volume overloa". &ulmonary vascularity is increase", an" the heart may be somewhat enlar$e". 7he lateral !ro<ection shows the ri$ht ventricular enlar$ement with fillin$ of the retrosternal airs!ace. 7he (C. has a normal to ri$htwar" a*is an" a ri$ht ventricular volume overloa" !attern in the !recor"ial lea"s. An echocar"io$ram with color %o!!ler e*amination can "emonstrate the A) %. 2owever, because the atrial se!tum is a !osterior structure, it may not be visuali#e" a"equately with a transthoracic echocar"io$ramG therefore, a transeso!ha$eal echocar"io$ram is frequently necessary for "ia$nosis. Closure of the "efect is recommen"e" for !atients with A)% to "ecrease the risk of !ulmonary vascular obstructive "isease, stroke, an" arrhythmias.

Aortic stenosis 'choice A) is often associate" with bicus!i" aortic valve an" !resents with "ys!nea on e*ertion, chest !ain, an" synco!e. A harsh systolic e<ection murmur is ty!ically hear" at the ri$ht u!!er sternal bor"er. Coarctation of the aorta 'choice C) results in obstruction between the !ro*imal an" the "istal aorta. On e*amination, the femoral !ulses are weak an" "elaye" relative to the brachial !ulses. 7urner syn"rome must be consi"ere" in a $irl with coarctation of the aorta. &atent "uctus arteriosus '&%A) 'choice %) usually !resents with a Hmachinery murmurH that is continuous be$innin$ after ):, !eakin$ at );, an" trailin$ off "urin$ "iastole. In"omethacin is often effective in closin$ the &%A in !remature infants. 1entricular se!tal "efect 'choice () is the most common con$enital "efect of the heart an" usually !resents with a wi"e s!ectrum of sym!toms inclu"in$ $rowth failure, con$estive heart failure, an" chronic lower res!iratory infections. &atients with small "efects mi$ht be asym!tomatic but woul" have a holosystolic murmur. 0:) (*!lanation: 7he correct answer is (. )e!tic arthritis refers to microbial invasion of the synovial s!ace. &atho$ens enter the synovial s!ace by either hemato$enous s!rea", local s!rea" from conti$uous infection, or a traumatic or sur$ical infection of the <oint s!ace. Accumulatin$ flui" an" !us ra!i"ly raise the intraarticular !ressure an" !ermanently in<ure vessels an" articular cartila$e. +ore than I>= of cases of se!tic arthritis affect the <oints of the lower e*tremity, with the knee most commonly involve". Acute se!tic arthritis is bacterial, with )ta!hylococcus aureus the most common or$anism, followe" by $rou! A stre!tococci an" )tre!tococcus !neumoniae. Other or$anisms to consi"er vary by a$e $rou!s. 2aemo!hilus influen#ae is common in chil"ren youn$er than 0 years, but the inci"ence is "ecreasin$ with 2ib vaccine. Neisseria $onorrhoeae may be isolate" in neonates an" se*ually active a"olescents. 7he teena$e $irl in this clinical vi$nette most likely has N. $onorrhoeae. .rou! beta4hemolytic stre!tococci is an im!ortant cause in neonates. )ickle cell !atients are at risk for )almonella infection. +ost chil"ren with se!tic arthritis a!!ear ill an" !resent with fever, <oint !ain, <oint swellin$, an" limite" movement of the affecte" <oint. 7he !ain is often severe an" constant, with increase" !ain when the infecte" <oint is move". On !hysical e*amination, the <oint is ty!ically fle*e" to limit motionG local erythema, warmth, an" swellin$ may be !resent. In se!tic arthritis of the hi!, the <oint is often fle*e", with sli$ht ab"uction an" e*ternal rotation, an" limitation of internal rotation with !assive motion. When se!tic arthritis is sus!ecte", "ia$nostic stu"ies shoul" inclu"e a com!lete bloo" count 'C C), bloo" cultures, an erythrocyte se"imentation rate '()6), an" !lain ra"io$ra!hs. If the !atient is febrile with an elevate" ()6 an" C C, the <oint shoul" be as!irate" an" the flui" sent for analysis. Only one thir" of cases will have a !ositive

.ramAs stain. F! to @>= of <oint flui" cultures may be !ositive, but the yiel" "ecreases with antibiotic use. -in"in$s on !lain ra"io$ra!hs may inclu"e <oint s!ace wi"enin$, increase" o!acity of the <oint s!ace, local soft tissue swellin$, "istortion of !eriarticular fat or muscle sha"ows, an" lateral "is!lacement or "islocation of the femoral hea". 7reatment of se!tic arthritis inclu"es sur$ical "raina$e of the !urulent material from the <oint an" I1 antibiotics. 7he choice of antibiotic shoul" be base" !artly on the a$e of the !atient to ensure covera$e of the most likely causative or$anism. (m!iric covera$e shoul" inclu"e an antista!hylococcal a$ent, either a beta4lactamase4resistant !enicillin or a first4$eneration ce!halos!orin. A!!ro!riate $ram4ne$ative covera$e must be !rovi"e" to neonates an" a"olescents. Nuvenile rheumatoi" arthritis 'choice A) is characteri#e" by chronic synovitis an" systemic inflammatory manifestations. .out 'choice ) is characteri#e" by serum uric aci" elevation an" urate "e!osition in the <oint. It commonly affects the first metatarsal <oint. ,yme "isease 'choice C) is a tick4borne illness cause" by orrelia bur$"orferi. It can !resent with fever, myal$ias, an" arthritis. &seu"o$out 'choice %) refers to the "e!osition of calcium !yro!hos!hate "ihy"rate in the <oint. Neither $out nor !seu"o$out are common "isor"ers in chil"ren. 0;) (*!lanation: 7he correct answer is . 7his !atient !resents with sym!toms consistent with neonatal !neumonia "ue to Chlamy"ia trachomatis. 7his a$ent is transmitte" from the motherAs va$inal secretions to the neonate at birth. 7he con<unctivitis !rece"es the !neumonitis. 7achy!nea, hy!o*emia, crackles, whee#in$ an" eosino!hilia are seen. Ascaris lumbricoi"es'choice A) !ro"uce visceral larva mi$rans an" can cause !neumonia an" eosino!hilia. 2owever, this is cause" by in$estion of Ascaris e$$s !asse" by "o$s an" cats. +yco!lasma !neumoniae'choice C) is the etiolo$ic a$ent in aty!ical !neumonias in youn$ !eo!le, but not in neonates. &neumocystis carinii'choice %) causes !neumonia in !atients with acquire" immune "eficiency syn"rome an" other immunocom!romise" !atients. 1aricella'choice () !neumonia is accom!anie" by skin lesions. -urthermore, eosino!hilia is not seen. 00)

(*!lanation: 7he correct answer is C. 7he !atient has near !erfect $lycemic control. 2owever, she may be e*!eriencin$ hi$her levels with snacks, an" com!liance is often an issue in a"olescents. 2er near !erfect $lucose levels are a bit sus!icious, an" it is a"visable to obtain a $lycosylate" hemo$lobin level. 7his is an in"ication of the avera$e $lucose level over the !ast three months, an" woul" $ive insi$ht into the overall $lycemic control in this !atient. %ecreasin$ the N&2 "ose mi$ht increase $lucose levels ?4C hours later 'choice A) since the time of !eak N&2 action is aroun" that amount of time. %ecreasin$ the re$ular "ose woul" lea" to an increase in $lucose levels 0> minutes later 'choice ). Increasin$ the N&2 level woul" "ecrease $lucose levels ?4C hours later 'choice %). Increasin$ the re$ular "osa$e woul" "ecrease $lucose levels 0> minutes later 'choice (). 0?) (*!lanation: 7he correct answer is (. 7he mother ha" !henylketonuria '&5F) early in her life. +any clinically normal female &5F !atients, who were treate" with "iet early in life, "iscontinue "ietary treatment an" have marke" hy!er!henylalaninemia by the time they reach chil"bearin$ a$e. +ost chil"ren born to such women are mentally retar"e" an" microce!halic, an" :/= have con$enital heart "isease, even thou$h the infants themselves are hetero#y$otes. 7his syn"rome, known as maternal &5F, results from the terato$enic effects of !henylalanine or its metabolites 'which cross the !lacenta), affectin$ s!ecific fetal or$ans "urin$ "evelo!ment. It is very im!ortant that maternal "ietary restriction of !henylalanine is initiate" before conce!tion an" continues throu$hout the !re$nancy. 7he biochemical abnormality in &5F is an inability to convert !henylalanine into tyrosine. With a block in the !henylalanine metabolism secon"ary to lack of !henylalanine hy"ro*ylase, minor shunt !athways come into !lay. 7his !ro"uces metabolites, such as !henyl!yruvic aci", !henyllactic aci", !henylacetic aci", an" O4 hy"ro*y!henylacetic aci", which are e*crete" in lar$e amounts in the urine in &5F. )ome of these abnormal metabolites are e*crete" in the sweat, an" !henylacetic aci", in !articular, im!arts a stron$ musty or mousy o"or to affecte" infants. It is !ro!ose" that e*cess !henylalanine or its metabolites contribute to the brain "ama$e an" mental retar"ation in &5F. 2omo#y$otes with this autosomal recessive "isor"er classically have a severe lack of !henylalanine hy"ro*ylase, lea"in$ to hy!er!henylalaninemia an" &5F. Affecte" infants are normal at birth but, within a few weeks, "evelo! a risin$ !lasma !henylalanine level, which in some way im!airs brain "evelo!ment. Fsually by C months of life, severe mental retar"ation becomes evi"ent. )ei#ures, other neurolo$ic abnormalities, "ecrease" !i$mentation of hair an" skin, an" ec#ema often accom!any the mental retar"ation in

untreate" chil"ren. 2y!er!henylalaninemia an" the resultant mental retar"ation can be avoi"e" by restrictin$ !henylalanine intake early in life. 2ence, a number of screenin$ !roce"ures are routinely use" for "etection of &5F in the imme"iate !ostnatal !erio". -ra$ile E syn"rome 'choice A) is an E4linke" "isor"er that results in mental retar"ation of various "e$rees. .alactosemia 'choice ) is an autosomal recessive "isor"er of $alactose metabolism that causes si$nificant "ama$e to the liver, eyes, an" brain. Con$enital hy!othyroi"ism 'choice C) is associate" with cretinism an" severe mental retar"ation. It occurs in : in D>>> births an" is amenable to in utero "ia$nosis an" treatment. +a!le syru! urine "isease 'choice %) is a familial cerebral "e$enerative "isease cause" by a "efect in branche" chain amino aci" metabolism an" characteri#e" by severe mental an" motor retar"ation an" urine with a ma!le4syru!4like o"or. 0/) (*!lanation: 7he correct answer is %. 7he infant in this clinical vi$nette has "evelo!e" hy!ernatremic 'Na, :C/ m(qB,) "ehy"ration from the "iarrhea. 2y!ovolemic !atients who have hy!ernatremia have a relatively $reater loss of water than of so"ium. Initial thera!y requires a"ministration of normal saline or 6in$erAs lactate to restore an effective circulatin$ !lasma volume. -ive !ercent albumin solution or !lasma also can be use". 7hese !atients require a hy!otonic solution containin$ salt to restore the NaM "eficit ';4/ m(qBk$ of bo"y wei$ht) an" to be$in the NaM maintenance '0 m(qBk$ of NaM ) in a solution containin$ ;>4?> mmolB, of 5Cl an" /= $lucose. -or a serum NaM concentration of :/>4:C> m(qB,, this volume shoul" be $iven over a ;?4hour !erio". ecause e*tracellular flui" osmolarity may fall more ra!i"ly than the brain can "issi!ate the i"io$enic osmoles $enerate" to !rotect intracellular osmolarity, an elevate" serum NaM concentration shoul" be correcte" by no more than :> m(qB, !er "ay. -or a serum NaM concentration O:C> m(qB,, the rehy"ration shoul" be s!rea" out over the number of "ays necessary to lower the NaM concentration to :/> m(qB, at a rate of :> m(qB"ay 'e.$., ; "ays for a NaM of :D> m(qB,). oth the "aily fraction of the "eficit an" the "aily maintenance requirement shoul" be !rovi"e". 7he "e$ree of hy!otonicity of the flui" a"ministere" is less im!ortant than the rate of correction. If hy!ernatremia is correcte" too ra!i"ly, the brain cells can be swollen beyon" the cell volume restore" by the osmoles, resultin$ in sei#ures. 2y!er$lycemia 'choice A) is not a common case of sei#ure. 2y!erkalemia 'choice ) causes arrhythmias. I"io!athic e!ile!sy 'choice C) is much less likely in this clinical vi$nette.

6a!i" correction of metabolic aci"osis 'choice () "oes not cause sei#ures. 0C) (*!lanation: 7he correct answer is (. In$estion of corrosive material is a rather common !roblem in !e"iatrics. Annual inci"ence ran$es from />>> to :/,>>>. 7he most common corrosive substances in$este" by chil"ren inclu"e househol" cleaners, "eter$ents, bleaches, "isk batteries, an" coins. ,iqui" "rain cleaner is a hi$hly alkaline substance, an" in$estion can cause severe eso!ha$eal necrosis of the liquefaction ty!e. -ull4thickness in<ury is common. In severe cases, it can cause eso!ha$eal !erforation an" me"iastinitis. Aci" in$estion causes coa$ulation necrosis an" eschar formation. 7he eschar ten"s to !rotect the eso!ha$us from full4thickness in<ury an" corrosive !erforation. When a chil" has a "efinitive history of in$estion of corrosive substance, he or she nee"s to be evaluate" emer$ently. (s!ecially in this case, "ys!ha$ia further su$$ests the !resence of eso!ha$eal in<ury. (ven thou$h there are no ulcerations or burns in the mouth, eso!ha$eal ulceration is very likely because oral lesions correlate !oorly with eso!ha$eal in<uries. 7he chil" shoul" be stabili#e", an" fle*ible eso!ha$osco!y is in"icate" to "irectly visuali#e the eso!ha$us. It mi$ht be !ru"ent to !erform a chest ra"io$ra!h first to rule out overt !erforation an" me"iastinitis. If eso!ha$osco!y shows no eso!ha$eal in<ury, no treatment is necessary. -or mil"4to4mo"erate burns, thera!y shoul" inclu"e I1 hy"ration, anal$esics, an" antibiotics. Com!lications inclu"e eso!ha$eal stricture. &revention is better than treatment. When there is youn$ chil" at home, corrosive substances shoul" always be !ut in a safe !lace, out of the chil"As reach. &e"iatricians shoul" also a""ress this issue with the !arents an" $ive a!!ro!riate antici!atory $ui"ance on routine office visits. arium swallow 'choice A) is not a!!ro!riate in the settin$ of acute corrosive in<ury of the eso!ha$us. C7 of the ab"omen 'choice ) is not in"icate" unless there are !eritoneal si$ns that su$$est stomach !erforation an" !eritonitis. N&O for :; hours, then liqui" "iet for 0 "ays 'choice C) without further evaluation by eso!ha$osco!y is ina!!ro!riate. In"irect laryn$osco!y 'choice %) is not in"icate" unless there is a burn of the laryn* that mi$ht result in laryn$eal e"ema. 0D) (*!lanation: 7he correct answer is

%. Chil"ren with sickle cell "isease are at risk of serious bacterial infection an" se!sis because they have im!aire" s!lenic function. )e!sis in these chil"ren is often cause" by enca!sulate" or$anisms such as )tre!tococcus !neumoniae an" 2aemo!hilus influen#ae ty!e b. 7he course of the infection, if uncontrolle" by a!!ro!riate antibiotic treatment, can be overwhelmin$ an" results in ra!i" "eterioration an" eventual "eath. loo" cultures, !referably two sets, shoul" be obtaine" !rom!tly before the initiation of antibiotic thera!y. 2owever, antibiotics shoul" not be "elaye" while waitin$ for the culture results 'choice A). Intravenous ceftria*one is the most commonly use" antibiotic in a febrile chil" with sickle cell "isease. It is effective a$ainst both ). !neumoniae an" 2. influen#ae. Orally a"ministere" amo*icillin 'choice ) is not a"equate for bacteremia or se!sis in a chil" with sickle cell "isease an" fever. Combine" thera!y of cefta#i"ime an" $entamicin 'choice C) is use" a$ainst sus!ecte" &seu"omonas aeru$inosa infection in !atients with neutro!enia or cystic fibrosis. Althou$h combination thera!y with vancomycin an" $entamicin 'choice () covers a wi"e ran$e of $ram4!ositive an" $ram4ne$ative or$anisms, it is not in"icate" in a febrile chil" with sickle cell "isease. 0@) (*!lanation: 7he correct answer is %. 6e" urine can be cause" by $ross hematuria, hemo$lobinuria, myo$lobinuria, certain foo"s or me"ications 'e.$., rifam!in, nitrofurantoin, chloroquine, a#o "yes, beets, an" blackberries), an" the !resence of urates. 7he stri!4test for bloo" in a urinalysis "oes not test "irectly on re" bloo" cellsG rather it tests the !resence or absence of hemo$lobin. It is sensitive to /4;> re" bloo" cells !er hi$h !ower fiel" 'h!f) in the urine, but it is even more sensitive to free hemo$lobin. ecause of the structural similarity of myo$lobin an" hemo$lobin, the stri!4test rea$ents also react to myo$lobin. 7herefore, a !ositive test for Hbloo"H in a stri! test of urinalysis can mean re" bloo" cells, hemo$lobin, or myo$lobin. It is im!ortant to !erform microsco!ic analysis of the urine to "istin$uish between these !ossibilities. In this case, the absence of a si$nificant number of re" bloo" cells ': 6 CBh!f) "oes not qualify for hematuria, an" the !resence of ?M bloo" on the stri! test is consistent with myo$lobinuria or hemo$lobinuria. 7he fact that it ha!!ens abru!tly after vi$orous e*ercise further su$$ests that the re" urine mi$ht be secon"ary to myo$lobinuria, which ha!!ens not uncommonly after e*ercise. It is im!ortant to confirm with a quantitative test of urine myo$lobin. 7he serum creatinine !hos!hokinase 'C5) level shoul" also be teste" for !ossible rhab"omyolysis. If rhab"omyolysis is !resent with elevate" creatinine !hos!hokinase, the !atient shoul" be a"mitte" for a$$ressive I1 hy"ration an" treatment with so"ium bicarbonate to alkalini#e the urine to !revent !reci!itation of the myo$lobin in the renal tubules.

7he absence of more than 04/ 6 CBh!f rules out $lomerulone!hritis 'choice A) an" hematuria 'choice ). .lomerulone!hritis woul" also be characteri#e" by re" cell casts u!on urinalysis. In$estion of foo" colorin$ 'choice C) is !ossible but less likely without any su!!ortin$ history, an" the urinalysis shoul" be ne$ative for bloo" in that case. 7he !resence of urates 'choice () in the urine can cause Hre" "ia!er syn"romeH in infants youn$er than C months. It is "ue to !hysiolo$ic hi$h e*cretion of uric aci" in infants. 2owever, the urinalysis shoul" be ne$ative for bloo". 7rauma 'choice -) can cause renal contusion an" various "e$rees of hematuria. Frinary tract infection 'choice .) occasionally causes $ross hematuria. Frinalysis is usually !ositive for nitrite, leukocyte esterase, an" W Cs. 0I) (*!lanation: 7he correct answer is %. Fnlike menin$itis occurrin$ in normal chil"ren, ventriculo!eritoneal shunt infections are most commonly cause" by coa$ulase4ne$ative )ta!hylococcus, such as )ta!hylococcus e!i"ermi"is. ). e!i"ermi"is causes ?>= to C>= of all C)- infections in !ersons with ventriculo!eritoneal shunts. Coa$ulase4ne$ative )ta!hylococcus !resents a si$nificant threat to !eo!le who have in"wellin$ "evices or catheters. +ost ). e!i"ermi"is isolates are resistant to multi!le antibiotics, inclu"in$ nafcillin an" o*acillin. 1ancomycin is the "ru$ of choice to ). e!i"ermi"is infection. 6emoval of the in"wellin$ me"ical "evice an" !arenteral antibiotic treatment are often necessary. 5erni$As si$n is !ositive if when the thi$h is fle*e" at a ri$ht an$le, com!lete e*tension of the le$ is not !ossible, or causes e*quisite !ain. A !ositive ru"#inskiAs si$n is when ben"in$ of the neck causes fle*ion of the hi!s. oth si$ns are use" to test nuchal ri$i"ity in evaluatin$ !atients for menin$itis. 2aemo!hilus influen#ae'choice A), Neisseria menin$iti"is 'choice ), an" )tre!tococcus !neumoniae'choice () are the most common !atho$ens of menin$itis in normal hosts. &seu"omonas aeru$inosa'choice C) is a rare cause of menin$itis in chil"ren, with or without ventriculo!eritoneal shunt. ?>) (*!lanation: 7he correct answer is . &ort4wine stains are vascular malformations on the skin, an" most commonly occur on the face. Over time, !ort4wine stains "arken an" !ose !sycholo$ical stress to the affecte" !erson because of cosmetic reasons. &ort4wine stains rarely "isa!!ear on their own, an" the most effective thera!y is !ulse" "ye laser. &ulse" "ye laser re"uces the si#e of most !ort4wine stains, an" in some cases can era"icate the stain com!letely. -actors affectin$ the res!onse to !ulse" "ye laser inclu"e location, timin$, an" si#e. )maller !ort4wine

stains res!on" better than the bi$$er ones. 7he earlier the treatment takes !lace, the better the res!onse. Cryosur$ery 'choice A), ra"iation thera!y 'choice C), skin $raftin$ 'choice %), an" to!ical corticosteroi"s 'choice () are not a!!ro!riate treatment for !ort4wine stains. ?:) (*!lanation: 7he correct answer is A. 7his is aortic valve stenosis, which accounts for /= of "ia$nose" car"iac "efects, but may actually be the most common con$enital anomaly of the heart since many minor cases are never "ia$nose". +ost cases are "ue to bicus!i" aortic valves, an" characteristically !ro"uce a systolic e<ection murmur. An accom!anyin$ aortic insufficiency may !ro"uce an early "iastolic murmur. 7he timin$ of sur$ical correction "e!en"s on the severity of the in"ivi"ual case. Atrial se!tal "efect 'choice ) causes a murmur hear" best at the left u!!er sternal bor"er. 7etralo$y of -allot 'choice C) an" trans!osition of $reat arteries 'choice %) usually !resent in infancy. 1entricular se!tal "efect 'choice () causes a murmur hear" best at the lower left sternal bor"er. ?;) (*!lanation: 7he correct answer is (. 1aricella 'chicken!o*) usually !resents with cutaneous infection in immunocom!etent hosts. 2owever, in immunocom!romise" !atients, such as this $irl with acute lym!hocytic leukemia 'A,,), varicella can cause serious an" life4threatenin$ com!lications, such as varicella !neumonia. 1aricella !neumonia has a very !oor !ro$nosis in !atients with A,,, reachin$ a mortality of rou$hly ;/=. 7he clinical course usually "eteriorates ra!i"ly an" many !atients "ie within 0 "ays of the "ia$nosis. &neumonia is e*cee"in$ly rare in normal chil"ren, but it can ha!!en in ol"er a"ults. I1 acyclovir thera!y is in"icate" in immunocom!romise" !atients with varicella infection. 7hera!y shoul" be initiate" early in the "isease course to ma*imi#e efficacy of the treatment. 1aricella4#oster immune $lobulin '13I.) may mo"ify the "isease course if it is $iven shortly after the e*!osure. It is not useful, however, once the "isease is establishe", as in this clinical vi$nette. Chil"ren with varicella also shoul" not be $iven salicylates because of a theoretically increase" risk of subsequent 6eye syn"rome. &atients shoul" be both contact4 an" airborne4isolate" "urin$ the entire course of the illness. Other, less common, com!lications inclu"e ence!halitis, he!atitis, $lomerulone!hritis, arthritis, an" thrombocyto!enia.

?0) (*!lanation: 7he correct answer is A. )ince the !atient can cou$h an" breathe, he shoul" be allowe" to clear the forei$n ob<ect s!ontaneously, if !ossible. In the mana$ement of forei$n ob<ect obstruction, if the !atient can cou$h an" breathe, it is best to initially observe an" allow s!ontaneous resolution, since intervention may actually be "ama$in$. Often, blin" fin$er swee!s 'choice ) may remove the forei$n ob<ect an" resolve the sym!toms. Also, this will nee" to be "one if the !atient is to be intubate". 7he ne*t ste! woul" be !erformin$ back blows if the !atient was less than : year of a$e 'choice C). If the !atient were over : year ol", ab"ominal thrusts woul" be the ne*t mana$ement o!tion 'choice %). An emer$ency tracheostomy 'choice () shoul" be the last o!tion an" shoul" be un"ertaken only by a !hysician traine" to !erform the !roce"ure. ??) (*!lanation: 7he correct answer is . Ni$ht terrors are a form of !arasomnias. &arasomnias refer to unusual behaviors that occur in the conte*t of slee!, s!ecific slee! sta$es, or in connection with arousal from slee!. Ni$ht terrors are most common in chil"ren a$e" ?4:; years an" ty!ically occur within the first several hours of slee!. 7hey are characteri#e" by the chil" su""enly cryin$ out, sittin$ u! in be" with a terrifie" look, cryin$ inconsolably, !erha!s thrashin$ about, an" e*hibitin$ evi"ence of increase" autonomic arousal with enlar$e" !u!ils, tachycar"ia, ra!i" breathin$, an" sweatin$. 7hese e!iso"es ty!ically last only a few minutes, an" the chil" then returns to slee!, with no memory of the events the ne*t mornin$. Ni$ht terrors are believe" to be "isor"ers of arousal from non46(+ slee! 'sta$e 0 an" ?), in which motor behavior occurs, but conscious awareness an" memory of the action are not !resent. 7hey are more likely to take !lace "urin$ !erio"s of illness, stress, or slee! "e!rivation, but they can ha!!en without any obvious associate" stress. 7hose sub<ect to ni$ht terrors or somnambulistic events shoul" avoi" slee! "e!rivation, which can increase the likelihoo" of their occurrence. +ost chil"ren with infrequent ni$ht terrors $row out of them with maturity an" usually require no s!ecific treatment. Clinicians shoul" thorou$hly e*!lain the !henomenon to the !arents an" reassure them that the chil" is well. 7he e*!ecte" eventual remission of the !roblem shoul" be em!hasi#e". &arents are encoura$e" not to awaken the chil", but to allow the e!iso"e to run its course. If the chil" is not awakene", he or she will return to normal slee! at the en" of the e!iso"e. If the chil" thrashes about wil"ly "urin$ the e!iso"e, the !arents shoul" !rovi"e !rotection from in<ury at that time.

Confusional arousals 'choice A) start $ra"ually 'unlike a full slee! terror in ol"er in"ivi"uals, which be$ins !reci!itously), with moanin$ !ro$ressin$ to cryin$, sittin$, an" thrashin$. 7he chil"ren are "ifficult to arouse an" "o not res!on" to comfortin$, but when allowe" to return to slee!, they "o not ty!ically remember the event the ne*t mornin$. Ni$htmares 'choice C) are fri$htenin$ "reams that awaken the chil" from 6(+ slee!. 7he chil" becomes fully awake an" is scare". 2e or she usually can recall "etails of the "ream. Obstructive slee! a!nea 'choice %) manifests as a!neic e!iso"es "urin$ slee! in which the !atient awakens su""enly. Affecte" !atients usually are obese. &anic "isor"er 'choice () is characteri#e" by recurrent !anic attacks, which initially may occur s!ontaneously an", over time, may "evelo! in a number of a$ora!hobic situations. 7he !atient may e*!erience a sense of terror or fear associate" with a !anic attack, inclu"in$ concerns about "yin$, $oin$ cra#y, or losin$ control. ?/) (*!lanation: 7he correct answer is (. ,esch4Nyhan syn"rome was "escribe" in :IC? in two brothers who manifeste" self4 mutilative behavior, choreoathetosis, an" mental retar"ation be$innin$ in their first year of life. 7his E4linke" here"itary "isor"er is "ue to com!lete "eficiency of hy!o*anthine !hos!horibosyltransferase '2&67), an en#yme that cataly#es the Hsalva$eH !athway of !urines. A salva$e !athway "eficiency results in increase" H"e novoH synthesis of !urines, with consequent over!ro"uction of uric aci". 7his is why !atients with "eficient 2&67 "evelo! hy!eruricemia, with secon"ary uric aci" stones, renal im!airment, an" $outy arthritis. ,esch4Nyhan syn"rome is also characteri#e" by self4mutilative behavior, choreoathetosis, an" mental retar"ation, which manifestations are still lar$ely une*!laine". 6enal failure is the most frequent cause of "eath. Chronic lea" into*ication 'choice A) affects the nervous, $astrointestinal, an" hemato!oietic systems. ehavioral anomalies, hy!ochromic microcytic anemia, an" !eri!heral neuro!athies are the most common manifestations. Chil"ren are !articularly vulnerable to lea" into*ication. -ra$ile E syn"rome 'choice ) is the most common cause of mental retar"ation if %own syn"rome is e*clu"e". 7he con"ition is "ue to a tri!let re!eat e*!ansion in the E chromosome an" manifests with mental retar"ation an" macroorchi"ism, amon$ other, less common anomalies. .out 'choice C) is a systemic con"ition resultin$ from hy!eruricemia. 7he <oints an" the or$ans are !rimarily affecte", with recurrent attacks of acute arthritis most often involvin$ the first metatarso!halan$eal <oint, ankle, heel, knee, wrist, fin$ers, an" elbow. A !ositive family history is frequently !resent, but most cases are "ue to unknown metabolic "efects resultin$ in over!ro"uction or "ecrease" e*cretion of uric aci". (*tra4

articular manifestations inclu"e formation of urate stones in the urinary tract, uric aci" ne!hro!athy, an" to!hi formation. 2untin$ton "isease 'choice %) is an autosomal "ominant neuro"e$enerative "isor"er cause" by a tri!let re!eat e*!ansion in a $ene enco"in$ a novel !rotein of unknown function. It manifests in the thir" or fourth "eca"e with chorea an" !ro$ressive "ementia. &atients frequently !resent a stron$ !ro!ensity towar"s suici"e. ?C) (*!lanation: 7he correct answer is %. -unctional ab"ominal !ain is !ain that lasts for more than 0 months an" often interferes with normal activity. 7he !ain is !eriumbilical an" often har" to "escribe. 7he !ain ty!ically "oes not awaken !atients from slee! or interfere with !leasant activities. 7he !ain is real an" is the result of the re$ulation of $astrointestinal motility in res!onse to either !sycholo$ical or !hysical stress. Acute a!!en"icitis 'choice A) usually occurs with ri$ht lower qua"rant !ain, fever, an" anore*ia. 7he chronic nature "escribe" in the question woul" rule out acute a!!en"icitis. Acute cholecystitis 'choice ) !resents with ri$ht u!!er qua"rant !ain an" vomitin$, an" a$ain the history of several months of sym!toms woul" rule out this "ia$nosis. Crohn "isease 'choice C) woul" usually !resent with ab"ominal !ain, "iarrhea 'usually loose with bloo"), an" anore*ia. Althou$h the ab"ominal !ain lasts several months, such as in this case, it usually causes wei$ht loss an" "elaye" $rowth. Irritable bowel syn"rome 'choice () woul" cause ab"ominal !ain that coul" last several months. 7his con"ition woul" also ty!ically cause bouts of "iarrhea alternatin$ with consti!ation. ?D) (*!lanation: 7he correct answer is %. 7his !atient has thrombasthenia. 7his rare $enetic "isor"er of !latelet function has autosomal recessive inheritance. 7he biochemical "efect a!!ears to be a lack of the !latelet membrane $lyco!rotein .&IIb4IIIa. 7his !rotein normally can bin" to fibrino$en, an" in its absence, !latelet a$$re$ation 'an" the resultin$ clot retraction) can not occur. 7he laboratory fin"in$s liste" in the question stem are ty!ical. Affecte" in"ivi"uals have a !articular !roblem with severe blee"in$ from mucosal surfaces, which commonly un"er$o minor traumas. ernar"4)oulier syn"rome 'choice A) is characteri#e" by very lar$e !latelets. Che"iak42i$ashi syn"rome 'choice ) is an immuno"eficiency "isease with "efective microbici"al activity after !ha$ocytosis an" unusually lar$e !latelets.

+ay42e$$lin anomaly 'choice C) is a thrombocyto!enic "isease with abnormal neutro!hils. 1on Willebran" "isease 'choice () is "ue to an abnormality of factor 1III relate" von Willebran" factor that secon"arily affects !latelet function. ?@) (*!lanation: 7he correct answer is A. 7he bloo" !ressure of this chil" is si$nificantly elevate" for her a$e. 7he a$e, hy!ertension, ab"ominal mass, an" aniri"ia su$$est Wilms tumor, which arises because of the "eletion involvin$ chromosome ::. -ra$ile E syn"rome 'choice ) is a common cause of mental retar"ation. 7ranslocation of chromosomes I an" ;: 'choice C) is calle" the &hila"el!hia chromosome an" causes leukemia. 7risomy :0 'choice %) causes severe birth "efects, inclu"in$ CN) malformations, cleft li!, !oly"actyly, an" mental retar"ation. 7urner syn"rome 'choice () is a cause of short stature, infertility, an" mil" mental retar"ation. ?I) (*!lanation: 7he correct answer is . Cyto$enetic testin$ shoul" be !erforme" on all sisters of males with -ra$ile E. 2etero#y$ous females frequently have "evelo!mental an" behavioral !roblems such as A%2 %. 7hey may also have bor"erline or mil" mental retar"ation. An ((. 'choice A) is likely to be normal, an" unless there are si$ns or sym!toms su$$estive of a sei#ure "isor"er, it woul" not be in"icate". A%2% is a clinical "ia$nosis an" no neuroima$in$ tests such as +6I 'choice C) will be useful in makin$ the "ia$nosis of A%2%. I8 testin$ 'choice %) may be hel!ful in school !lacement, but it is not the first test to be or"ere". Frine for metabolic screenin$ 'choice () is use" to "etect rare inborn errors of metabolism. It woul" be in"icate" in cases of failure to thrive, sei#ures, an" se!sis. +any inborn errors of metabolism are associate" with severe mental retar"ation.

/>) (*!lanation: 7he correct answer is C. Acne vul$aris affects the ma<ority of a"olescents an" is more !revalent in males. 2ormonal influences, abnormal keratini#ation of !ilosebaceous units an" coloni#ation by bacteria '&ro!ionibacterium acnes) are im!ortant !atho$enetic elements. 7reatment of acne "e!en"s on the severity of the con"ition. 7o!ical a!!lication of come"olytic a$ents such as retinoi"s 'tretinoin, a"a!alene, an" the new yeast4"erive" a$ent a#aleic aci") is effective for mil" to mo"erate forms of non4inflammatory acne, characteri#e" by o!en come"ones. %aily a!!lication of these com!oun"s will result in im!rovement within several weeks after startin$ treatment. +il" skin irritation an" scalin$ may be minimi#e" by startin$ with low4concentration creams, an" then !ro$ressively increasin$ the concentration. Another si"e effect of retinoi"s is increase" !hotosensitivity. 7he !atient must be instructe" to avoi" !rolon$e" e*!osure to the sun an" to use a sunscreen. Avoi"ance of chocolate an" s!icy foo"s 'choice A) woul" have no beneficial effects on acne. It is well establishe" that there is no correlation between acne an" s!ecific ty!es of foo"s. -requent face washin$ with stron$ soa! 'choice ) will !robably cause e*acerbation of acne. 7his skin con"ition is not cause" by "irt. .entle face washin$ once or twice "aily with mil" soa!s is recommen"e". 7reatment with oral antibiotic 'choice %) is aime" at "ecreasin$ bacterial coloni#ation It is use" for !atients who fail to res!on" to to!ical treatments or have severe forms of inflammatory acne. 7he antibiotics of choice inclu"e tetracyclines an" erythromycin. Oral treatment with isotretinoin 'choice () is use" for severe cases of acne not res!on"in$ to to!ical come"olytics an" antibiotics. 7his com!oun" acts by "ecreasin$ sebum !ro"uction. In a""ition to various ty!es of si"e effects, isotretinoin is terato$enic. -emale !atients of chil"bearin$ a$e shoul" be require" to use effective means of contrace!tion be$innin$ one month before treatment to one month after treatment.

USMLE Step 2 Practice Test Block 11 :ame; +nstr#ctions; Ans'er the *#estions $elo' to the $est of yo#r a$ility. =hen yo# finish the test% click the 2heck $#tton at the $ottom to )ie' the res#lts.

1.A 1 !year!ol" girl is $ro#ght to the pe"iatrician,s office $eca#se of s#""en "eterioration of school performance. >)er the past month% her mother has notice" an occasional paint stain on the girl,s han"s. /er mother also notice" si. $ottles of type'riter correction fl#i" in her $e"room a$o#t a 'eek ago. She raise" the concern of inhalant a$#se. =hich of the follo'ing is the most likely conse*#ence of chronic inhalant a$#sea6Arrhythmia $6Bronchial asthma c62ere$ral hemorrhage "6Encephalopathy e6 Aespiratory "epression :ormal La$s 2.A 1<!year!ol" girl 'ith an incomplete )accination recor" recei)e" one "ose of the measles% m#mps% an" r#$ella 3MMA6 )accine "#ring a "octor,s )isit. >ne month later% she learns that she is 8 'eeks pregnant% an" she is concerne" a$o#t potential $irth "efects res#lting from the MMA )accine. =hich of the follo'ing most closely appro.imates the risk of $irth "efects secon"ary to MMA )accine e.pos#re "#ring the first trimestera6 Q1B $6&B c6 B "61B e614B :ormal La$s

&.A <!'eek!ol" male infant 'ho 'as $orn at &2 'eeks, gestation 'ith a $irth 'eight of 1 44 g% has ha" an a)erage 'eight gain of 1 g7"ay since $irth. /e takes an iron!fortifie" form#la that is 24 kcal7oD. /is calorie intake is a$o#t 12 kcal7"ay. +t is note" that his stool is poorly forme" an" $#lky. =hich of the follo'ing "ietary mo"ifications 'ill most likely res#lt in "ecrease" steatorrhea an" impro)e" 'eight gaina6A"" pancreatic enDymes to the form#la $62hange to a lactose!free form#la c6 +ncrease calorie intake to 1( kcal7"ay $y increasing )ol#me per fee" "6 S#$stit#te me"i#m!chain triglyceri"es for long!chain triglyceri"es e6S#pplement 'ith )itamins A an" E :ormal La$s 4. A 1<!month!ol" is taken to the emergency room after falling 'hile learning to 'alk. The to""ler has an enlarging% s'ollen $r#ise on his forehea"% 'hich is no' o)er t'o inches across. The parents say that the $r#ise is noticea$ly larger than it 'as 'hen they entere" the emergency room an ho#r earlier. A $loo" sample is "ra'n% an" the chil" ooDes $loo" at the p#nct#re site for 2 min#tes. 2lotting st#"ies on the $loo" sample sho' a prolonge" PTT an" a normal PT. 5ollo'!#p st#"ies sho' )ery lo' le)els of factor ?+++. =hich of the follo'ing is the most likely "iagnosisa6@isseminate" intra)asc#lar coag#lation $6/emophilia A c6/emophilia B "6/yperhomocysteinemia e6?on =ille$ran" "isease :ormal La$s .A (!"ay!ol" $oy 'ho is the pro"#ct of an #ncomplicate" gestation is $ro#ght to the physician $eca#se of hypospa"ias. The $a$y is other'ise healthy% an" is #rinating 'itho#t any "iffic#lty. >n physical e.amination% )ital signs are sta$le% l#ngs are clear an" the heart is $eating at a reg#lar rate. The only a$normal physical fin"ing is the

hypospa"ias. Urinalysis is negati)e for infection. =hich of the follo'ing is the most appropriate ne.t stepa6Meas#ring ser#m creatinine le)el $6Sche"#le a renal #ltraso#n" c6>$tain an intra)eno#s pyelogram "62ystography e6 Performing a circ#mcision :ormal La$s <.A 4!year!ol" chil" comes in for a health maintenance )isit. /is mother is concerne" that he is not "oing some things that other ki"s in his preschool "o. =hich of the follo'ing skills 'o#l" $e e.pecte" of a 4! year!ol"a6 B#il"ing a 14!c#$e staircase $6@ra'ing a s*#are c6@ra'ing a triangle "6@ra'ing a person 'ith si. parts e6Aepeating fi)e "igits :ormal La$s (.A <!'eek!ol" girl comes to me"ical attention $eca#se of a $right re" pla*#e on her neck. The pla*#e is slightly raise"% meas#res 2 cm in greatest "iameter% an" $lanches partially on press#re. The mother reports that the lesion appeare" in the secon" or thir" 'eek an" has $een gra"#ally increasing in siDe. =hich of the follo'ing is the most appropriate ne.t step in managementa6 :o f#rther "iagnostic proce"#re or treatment is necessary $6 :o f#rther "iagnostic proce"#re or treatment is necessary c6Aeferral to a "ermatologist for $iopsy "6Treatment 'ith corticosteroi"s or interferon!alfa e6 S#rgical e.cision :ormal La$s 1.A 1 !year!ol" girl presents to a pe"iatric car"iology clinic 'ith a complaint of chest pain. She states the pain has come an" gone o)er the past year% $#t has increase" in fre*#ency o)er the past fe' 'eeks. She

"escri$es it as a sharp pain o)er her left chest. Physical e.amination re)eals a healthy!appearing 1 !year!ol" girl. /er temperat#re is &(.2 2 388 56% p#lse is 847min% an" respiratory rate is 247min. L#ng e.amination is normal. 2ar"iac e.amination re)eals a late systolic m#rm#r prece"e" $y a click at the ape.. :o hea)e or r#$ is present. An electrocar"iogram an" chest .!ray film are #nremarka$le. =hich of the follo'ing is the most likely "iagnosisa6Atrial septal "efect $6Mitral reg#rgitation c6Mitral stenosis "6Mitral )al)e prolapse e6Tric#spi" reg#rgitation :ormal La$s 8.A !year!ol" $oy s#ffers from a con"ition characteriDe" $y rec#rrent f#ngal an" )iral infections% thymic hypoplasia% tetany% an" a$normal facies. Ser#m le)els of imm#noglo$#lins are mil"ly "epresse"% an" lymph no"e $iopsy sho's lymphocyte "epletion of T!"epen"ent areas. =hich of the follo'ing is the #n"erlying pathogenetic mechanisma6 @e)elopmental "efect of the thir"7fo#rth pharyngeal po#ches $6 +n #tero infection $y h#man imm#no"eficiency )ir#s 3/+?6 c6M#tations of an a#tosomal gene enco"ing a"enosine "eaminase "6M#tations of an F!linke" gene co"ing for a cytokine receptor s#$#nit e6M#tations of an F!linke" gene co"ing for a tyrosine kinase :ormal La$s 14.A !year!ol" $oy is $ro#ght to the physician $eca#se of a malo"oro#s "ischarge coming from his right nostril. /is parents report that the chil" "e)elope" fre*#ent sneeDing an" m#c#s "ischarge for one 'eek. They tho#ght it 'as a common col". The chil" is afe$rile an" in apparent goo" health. There is no history of allergic "iseases or rec#rrent infections. E.amination re)eals o$str#ction of the right nasal fossa 'ith associate" p#r#lent "ischarge from the right nostril. =hich of the follo'ing is the most likely "iagnosisa6 Allergic rhinitis $6R#)enile angiofi$roma

c6:asal foreign $o"y "6:asal polyp e6Ahinitis me"icamentosa f6?asomotor rhinitis :ormal La$s 11.A 4!year!ol" $oy is $eing e)al#ate" for short stat#re. /e has a history of m#ltiple $one fract#res in the past. /e re*#ires a 'heelchair to am$#late an" has hearing "iffic#lty. >n physical e.amination% his height is $elo' the th percentile. /is sclerae are $l#e in color. There is marke" "eformity of his lo'er e.tremities. =hich of the follo'ing is the most likely "iagnosisa6Achon"roplasia $62onstit#tional "elay of gro'th c6@e)elopmental "ysplasia of the hip "65amilial short stat#re e6>steogenesis imperfecta :ormal La$s 12.A <!month!ol" infant presents to the emergency "epartment 'ith the ne' onset of 'eak cry% "ecrease" acti)ity% an" poor fee"ing. The mother also states that the infant has $een constipate" for the past 2 "ays. >n physical e.amination% the infant has a )ery 'eak cry% poor m#scle tone% an" a$sent "eep ten"on refle.es. =hich of the follo'ing is the most likely "iagnosisa62ongenital hypothyroi"ism $6C#illain!BarrT syn"rome c6+nfant $ot#lism "6Myasthenia gra)is e6?accine!associate" poliomyelitis :ormal La$s 1&.A 12!month!ol" infant presents 'ith $ilio#s )omiting an" a$"ominal "istention for 14 ho#rs. /is mother states that the infant has $een constipate" since $irth an" faile" to pass meconi#m "#ring the first 41 ho#rs of life. >n e.amination% he is )ery irrita$le. /is length an" 'eight are $oth $elo' the th percentile accor"ing to his age. /is a$"omen is mo"erately "isten"e". After a "igital rectal e.amination% a fair amo#nt

of stool e9ects o#t from the an#s. =hich of the follo'ing is the most likely "iagnosisa6@#o"enal atresia $6+nt#ss#sception c6/irschspr#ng "isease "6Malrotation e6Pyloric stenosis :ormal La$s 14.A 8!year!ol" $oy is $ro#ght to the pe"iatrician,s office for $e"! 'etting. /is mother states that he has ne)er $een "ry at night. >ccasionally% he has pro$lems controlling his $la""er "#ring the "ay. >n physical e.amination% his $loo" press#re is 817 < mm /g. Both his 'eight an" height are $elo' the th percentile for his age. /is $la""er is enlarge" an" palpa$le a$o)e the symphysis p#$is. =hich of the follo'ing is the most likely ca#se of his pro$lema6>$str#cti)e #ropathy $6Primary poly"ipsia c6Aefl#. nephropathy "6Sickle cell trait e6Unsta$le $la""er :ormal La$s 1 .A 21!month!ol" female has $een li)ing 'ith her mother in a shelter for homeless 'omen an" chil"ren. She is $ro#ght to the Emergency @epartment in stat#s epileptic#s% 'hich is stoppe" 'ith intra)eno#s loraDepam. She is place" on a car"iac monitor an" a 'i"e comple. tachycar"ia is note". The )entric#lar tachycar"ia re)erts to sin#s tachycar"ia after "efi$rillation is performe". /er temperat#re is &1. 2 3141.& 56% $loo" press#re is 124714 mm /g% p#lse is 18 % an" respirations are 2<. Physical e.amination re)eals a lethargic% pale to""ler 'ith "ilate" an" reacti)e p#pils% "ry m#co#s mem$ranes% shallo' respirations%

"iaphoresis an" $risk "eep ten"on refle.es. A #rine to.icology screen is most likely to "etect 'hich of the follo'ing s#$stancesa6Bar$it#rates $62ocaine c6/eroin "6Mari9#ana e6P2P :ormal La$s 1<.A 4!year!ol" girl is $ro#ght to the physician $eca#se of a cr#ste" honey!colore" erythema res#lting from r#pt#re of tiny )esicles an" p#st#les. /er temperat#re is &(.( 2 3142 56. Skin lesions are "istri$#te" o)er the face an" e.tremities. Physical e.amination re)eals enlargement of lymph no"es in the cer)ical an" a.illary regions. =hich of the follo'ing is the most fre*#ent pathogen of this skin infectiona6/#man herpes)ir#s ( $6Staphylococc#s a#re#s c6Streptococc#s pyogenes "6Propioni$acteri#m acnes e6 Trychophyton f#ngi :ormal La$s 1(. A 1<!year!ol" $oy presents 'ith a temperat#re of &1.4 2 3141 56 an" lo' $ack% 'rist% an" knee pain. /e ha" a sore throat 1 month earlier. /is arthritis is "iff#se. Pea!siDe" s'ellings are note" o)er the skin on his knees. /e has a serpigino#s erythemato#s area on his anterior tr#nk. /is $loo" an" throat c#lt#res are negati)e% an" his 2B2 is #nremarka$le. /is antistreptolysin!> 3AS>6 titer is high. =hich of the follo'ing is the most appropriate therapya6Acetaminophen $6Aspirin c6Penicillin "6Penicillin an" aspirin e6 S#pporti)e care :ormal La$s

11. A term ne'$orn is "eli)ere" )aginally follo'ing a $reech presentation. >n physical e.amination% the Barlo' test is positi)e for $ilateral s#$l#.ation of the hips. There is "ecrease" a$"#ction of $oth hips. Besi"es $reech presentation% 'hich of the follo'ing infants are most at risk for "e)elopmental "ysplasia of the hipa6African American infants $65emale infants c6Secon"!$orn infants "6+nfants of mothers 'ith preeclampsia e6 Premat#re infants :ormal La$s 18. A <!month!ol" $oy presents to the Emergency @epartment 'ith a three!"ay history of co#gh% congestion an" lo' gra"e fe)er. The mother states that the $a$y has not $een fee"ing 'ell an" has #se" only t'o "iapers o)er the past 24 ho#rs. Physical e.amination re)eals a pale infant 'ith a temperat#re of &(.1 2 3144.1 56% p#lse of 1(47min% respirations of <47min an" o.ygen sat#ration of 11B on room air. The patient e.hi$its nasal flaring% s#$costal an" intercostal retractions. L#ng e.amination re)eals "iff#se 'heeDing. 2ar"iac e.amination re)eals a reg#lar $#t tachycar"ic rhythm 'ith no m#rm#r. 2entral capillary refill is fo#r secon"s. The remain"er of the e.amination is normal. =hich of the follo'ing is the most appropriate initial step in managementa62hest .!ray film $6Al$#terol ne$#liDer treatment c6Bol#s of intra)eno#s fl#i"s "6>.ygen therapy e6+ntra)eno#s steroi"s :ormal La$s 24. An infant in $ro#ght to the clinic for a ro#tine healthy )isit an" )accinations. She is the pro"#ct of an #ncomplicate" pregnancy an" has $een meeting "e)elopment mileposts. She is fee"ing 'ell% an" her mother reports that the $a$y seems to $e gro'ing 'ell as 'ell. >n

physical e.amination% the infant is afe$rile 'ith sta$le )ital signs. She can lift her hea" to 84 "egrees% her eyes follo' past the mi"line% she la#ghs% regar"s her o'n han" an" has slight a'areness of her mother. =hich of the follo'ing is the most likely age of this infanta6 2 months $64 months c6< months "612 months e6 11 months :ormal La$s 21.A !year!ol" girl presents 'ith a &!"ay history of fe)er% "yspnea% an" intermittent 9oint pain. She has a history of sore throat a$o#t 1 month ago. >n physical e.amination% her temperat#re is &8.< 2 314&.2 56% $loo" press#re is 847<4 mm /g% p#lse is 1147min% an" respirations are 227min. /er knees an" el$o' 9oints are s'ollen an" ten"er to palpation. There is a gra"e +++7?+ "iastolic m#rm#r $est hear" at the ape.. M#ltiple fine% pink mac#les are note" on her tr#nk. These mac#les are $lanching in the mi""le. =hich of the follo'ing is the most likely "iagnosisa6R#)enile rhe#matoi" arthritis $6Lyme "isease c6Ahe#matic fe)er "6Scarlet fe)er e6Septic arthritis :ormal La$s 22.A !year!ol" $oy presents to the emergency "epartment 'ith 2!"ay history of fe)er% anore.ia% loose stools% an" yello' skin color. /e atten"s a large "aycare center. >n physical e.amination% his temperat#re is &1.1 2 3144.( 56% $loo" press#re is 117 < mm /g% p#lse is (47min% an" respirations are 1 7min. +nitial la$oratory e)al#ation re)eals a total $ilir#$in of 1.1 mg7"L an" alanine aminotransferase of (<4 U7L. =hich of the follo'ing is the most appropriate "iagnostic testa6/epatitis B s#rface antigen in ser#m

$6+gC for hepatitis A in ser#m c6+gC for hepatitis B s#rface antigen in ser#m "6+gM for hepatitis A in ser#m e6Stool c#lt#re for hepatitis A :ormal La$s 2&. A (!year!ol" girl complains of increase" #rinary fre*#ency% "ys#ria an" itching on #rination. /er #rinalysis is consistent 'ith a #rinary tract infection. This is her 24th infection in the past year% "espite a"e*#ate anti$iotic co)erage. 5#rther imaging of her $la""er% ki"neys an" #reter re)eals is consistent 'ith )esico#reteral refl#.. =hich of the follo'ing is the ne.t appropriate stepa6 2T scan of the pel)is. $6+ntra)eno#s anti$iotics for t'o 'eeks c6+ntra)eno#s pyelogram "6Aenal arteriogram e6Antirefl#. s#rgery :ormal La$s 24.A (!year!ol" $oy is referre" $y his school for psychiatric e)al#ation. The teachers ha)e notice" that% in the past year% he has $een #na$le to s#stain attention in class% an" has $een fi"geting an" talking to his peers "#ring class. /e seems #na$le to 'ait for others to finish speaking an" keeps interr#pting an" $l#rting o#t ans'ers $efore *#estions are complete". At home% his parents state that he is forgetf#l an" losses things easily. =hich of the follo'ing is the most likely "iagnosisa6Attention "eficit7hyperacti)ity "isor"er 3A@/@ $6Bipolar "isor"er c62on"#ct "isor"er "6Posttra#matic stress "isor"er e6 Aett syn"rome :ormal La$s 2 .A <!'eek!ol" $oy is $ro#ght to the physician $eca#se of a rash in)ol)ing the "iaper area. >n e.amination% there are erythemato#s% slightly scaly patches co)ering the $#ttocks an" the lo'er a$"omen. Skin creases appear spare". The $a$y is other'ise healthy. Physical e.amination re)eals no lympha"enopathy% fe)er% or other signs of

organic illness. =hich of the follo'ing is the most common ca#se of this con"itiona6 2an"i"iasis $6+rritant contact "ermatitis c6Langerhans cell histiocytosis "6Psoriasis e6Se$orrheic "ermatitis :ormal La$s 2<. A 1!"ay!ol" infant appears "#sky in the ne'$orn n#rsery "#ring fee"ing. >.ygen is imme"iately a"ministere" $y nasal cann#la. Shortly after'ar"% she "e)elops tachypnea. >n physical e.amination% her $loo" press#re from the right #pper arm is 47&4 mm /g% her p#lse is 1147min% an" her respirations are <47min. An echocar"iogram is consistent 'ith hypoplastic left heart syn"rome. =hich of the follo'ing 'o#l" likely $e fo#n" on a#sc#ltationa62ontin#o#s "#ctal m#rm#r% $o#n"ing p#lses $62ontin#o#s "#ctal m#rm#r% poor peripheral p#lses c6/olosystolic m#rm#r% poor peripheral p#lses% *#iet secon" heart so#n" "6:o m#rm#r% precor"ial hyperacti)ity% lo#" secon" heart so#n" e6 :o m#rm#r% precor"ial hyperacti)ity% *#iet secon" heart so#n" :ormal La$s 2(. An 1!year!ol" $oy falls on his right han" 'ith the arm e.ten"e"% an" he $reaks his el$o' $y hypere.tension. F!ray films sho' a s#pracon"ylar fract#re of the h#mer#s. =hich of the follo'ing complications is of greatest concern 'ith this type of in9#rya6+nsta$ility that re*#ires open re"#ction an" internal fi.ation $6+ns#fficient remo"eling c6Cro'th plate "amage "6Mal#nion e6?asc#lar an" ner)e in9#ries :ormal La$s 21.A ne' $orn infant is in respiratory "istress an" re*#ires se)eral attempts at res#scitation in the "eli)ery room $eca#se of "iffic#lty $reathing an" fre*#ent cyanosis. The neonatologist notes that "#ring

crying% her $reathing impro)es an" $reath an" heart so#n"s are normal. @irect laryngoscopy is #nremarka$le as 'ell. @eep inspirations $y the neonate are ineffecti)e. =hich of the follo'ing is the most effecti)e inter)entiona6 >$taining a chest .!ray film $6>$taining an electrocar"iogram c6>$taining an arterial $loo" gas "6A"ministering atropine e6 +nserting an oropharyngeal t#$e :ormal La$s 28.A (!year!ol" $oy 'ho has ha" pain in his right leg for 4 months is $eing e)al#ate". The pain is 'orse at night an" is #nrelenting% $#t it can #s#ally $e relie)e" 'ith i$#profen. >n physical e.amination% there is localiDe" ten"erness o)er the anterior aspect of the right thigh. There is also mil" atrophy of the affecte" lim$. A ra"iograph of the fem#r re)eals a ra"iol#cent ni"#s 'ith s#rro#n"ing reacti)e sclerotic $one. =hich of the follo'ing is the most likely "iagnosisa6Bro"ie,s a$scess $6E'ing sarcoma c6>steosarcoma "6>steoi" osteoma e6Stress fract#re :ormal La$s &4. An 1!year!ol" $oy presents to the physician for a ro#tine health maintenance )isit. /is mother states that he has ha" "iffic#lty rea"ing an" concentrating in his secon"!gra"e class. >n e.amination% se)en cafT!a#!lait spots on his $o"y% as 'ell as t'o small% soft masses a$o)e his or$it% are seen. /e also has a.illary freckling. /is mother also has cafT! a#!lait spots on her arms. =hich of the follo'ing is the most likely "iagnosisa62ongenital hypothyroi"ism $6Marfan syn"rome c6:e#rofi$romatosis "6>steogenesis imperfecta e6 T#$ero#s sclerosis :ormal La$s

&1.A 4!month!ol" infant $oy has gaine" only 14 o#nces since $irth. /e has faile" to gain 'eight 'ith m#ltiple form#la preparations. /is stools ha)e $een loose an" fatty. An ol"er sister ha" similar symptoms an" has $een repeate"ly hospitaliDe" for fail#re to thri)e an" rec#rrent p#lmonary infections. =hich of the follo'ing is the most likely ca#se of this patient,s gastrointestinal symptomsa6 Achlorhy"ria $6Bacterial o)ergro'th c62olonic inertia "6Castric hypersecretion e6Pancreatic e.ocrine ins#fficiency :ormal La$s &2.A 4!year!ol" $oy falls from the 9#ngle gym at preschool. /e s#stains minor a$rasions an" cont#sions% an" is taken care of $y the school n#rse. /is parents take him that same afternoon to his reg#lar pe"iatrician an" "eman" 0a thoro#gh check!#p0 for possi$le internal in9#ries. The pe"iatrician complies% an" a complete physical e.amination is normal. /is hemoglo$in is 14 g7"L% an" a #rinalysis sho's the presence of microhemat#ria. =hich of the follo'ing is the most appropriate ne.t step in managementa62T scan of the a$"omen an" pel)is $6Aeass#re the parents that microhemat#ria from minor tra#ma 'ill resol)e spontaneo#sly c6Serial hemoglo$in an" hematocrit "eterminations "6Urologic 'ork#p% starting 'ith a sonogram e6Aetrogra"e #reterogram an" cystogram :ormal La$s &&. A 8!year!ol" $oy is $ro#ght to the pe"iatric clinic $y his mother% 'ho notice" that the left si"e of his mo#th has starte" to "roop o)er the past se)eral "ays. +n a""ition% he is #na$le to close his left eye completely an" complains of it $#rning. Ae)ie' of systems re)eals a col" appro.imately t'o 'eeks ago an" recent "ecrease" taste sensation. Physical e.amination re)eals a 'ell no#rishe" male 'ith normal )ital signs. There is left eye ptosis an" mil" erythema of the left con9#ncti)a. /is smile is asymmetrical on the left. La$oratory e)al#ation% incl#"ing a complete $loo" co#nt an" chemistry profile% are normal. =hich of the

follo'ing infections is most closely associate" 'ith this patient,s con"itiona6Epstein!Barr ?ir#s $6Cro#p A Streptococc#s c6/#man +mm#no"eficiency ?ir#s "6+nfl#enDa e6Measles :ormal La$s &4. A 4!'eek!ol" infant presents 'ith tachycar"ia% tachypnea% an" poor 'eight gain. /is arterial $loo" gas sho's a p/ of (.&4% a Pa2>2 of 41 mm /g% an" a Pa>2 of (4 mm /g. A chest ra"iograph sho's car"iomegaly. Echocar"iography re)eals a str#ct#rally normal heart% left )entric#lar "ilatation% a left )entric#lar e9ection fraction of 24B% an" mil" mitral an" tric#spi" reg#rgitation. +? a"ministration of 'hich of the follo'ing me"ications is the $est initial step in management of this patienta6Angiotensin!con)erting enDyme inhi$itor $62orticosteroi" c6@igo.in "6Epinephrine e65#rosemi"e :ormal La$s & . A 1 !year!ol" girl presents 'ith a 2!"ay history of pain an" s'elling in her left knee. She plays soccer reg#larly on her school team. There is no history of tra#ma. >n physical e.amination% there is marke" s'elling an" ten"erness o)er her anterior ti$ial t#$erosity. A ra"iograph of her left knee re)eals irreg#larities of the t#$ercle conto#r an" haDiness of the a"9acent metaphyseal $or"er. =hich of the follo'ing is the most likely e.planation for her symptomsa6A)asc#lar necrosis of the hip $6Legg!2al)e!Perthes "isease c6>sgoo"!Schlatter "isease "6Septic arthritis e6Slippe" capital femoral epiphysis :ormal La$s

&<. A 12!year!ol" girl 'ith a history of asthma has $een a"mitte" to intensi)e care #nits t'o times in the past an" has ha" three emergency "epartment )isits "#ring the past 12 months. /er only me"ication is inhale" al$#terol as nee"e"% an" she #ses it t'o to three times a "ay. She has noct#rnal symptoms a$o#t t'o times a 'eek. She is free of symptoms no' $#t reports that she gets short of $reath easily. =hich of the follo'ing pharmacologic inter)entions is most appropriatea6 Anticholinergic agent $62romolyn so"i#m c6+nhale" corticosteroi" "6Long!term $roncho"ilator e6 :e"ocromil so"i#m :ormal La$s &(.A 12!year!ol" chil" is $ro#ght to his pe"iatrician for a ro#tine health maintenance )isit. /e has $een 'ell e.cept for occasional attacks of asthma an" has met all "e)elopment milestones. /is imm#niDations are #p to "ate. /e occasionally #ses theophylline for his asthma. Physical e.amination is remarka$le for a $loo" press#re of 1 4784 mm /g in $oth arms. =hich of the follo'ing is the most likely ca#se of his hypertensiona62hronic l#ng "isease $62oarctation of the aorta c62ongenital heart "isease "6Aenal "isease e6Theophylline to.icity :ormal La$s &1. An 11!year!ol" $oy presents 'ith fe)er an" sore throat. A rapi"! strep test confirms streptococcal pharyngitis. /e is lea)ing for a s#mmer camp in 2 "ays. +n the past% he has ha" pro$lem finishing the 'hole co#rse of anti$iotic treatment. =hich of the follo'ing is the $est treatment for his streptococcal pharyngitisa6A single "ose of $enDathine penicillin C intram#sc#larly $6A single "ose of ceftria.one intram#sc#larly c6 A single "ose of procaine penicillin C intram#sc#larly "6Erythromycin orally for "ays

e6 Penicillin ? orally for :ormal La$s

"ays

&8.A pre)io#sly healthy 11!year ol" $oy presents to the physician 'ith a fe)er an" persistent )omiting for 4! "ays. +nitially% the emesis 'as clear% $#t no' it contains streaks of $right re" $loo". 5in"ings on a physical e.amination% complete $loo" co#nt% an" ser#m electrolytes are 'ithin normal limits. =hich of the follo'ing is the most likely ca#se of the hematemesisa6Esophageal )arices $6Esophagitis c6Castritis "6Mallory!=eiss tear e6 Peptic #lcer "isease :ormal La$s 44.A 12!year $oy is $ro#ght to the emergency room $y his parents after a se)eral "ay history of progressi)e 'eakness. /e has $een a healthy chil" 'ho has met all "e)elopment milestones an" all his imm#niDations are #p to "ate. /e reports that he ha" an #pper respiratory infection 2 'eeks ago. >n physical e.amination% his )ital signs are la$ile 'ith a p#lse range of <4!1447 min an" systolic $loo" press#re ranging from 14! 124 mm /g. /e is alert an" oriente". :e#rologic e.amination is impressi)e for $ilateral 'eakness in the #pper an" lo'er e.tremities. @eep ten"on refle.es are a$sent an" sensation is intact. =hich of the follo'ing is the most likely "iagnosisa6C#illain!BarrT syn"rome $6Myasthenia gra)is c6Polymyositis "6Trans)erse myelitis e6?iral encephalitis :ormal La$s 41. A 1 !year!ol" $oy 'ith tall stat#re presents 'ith "ecrease" e.ercise tolerance o)er the past 2 months. /e states that he #se" to play t'o games of $asket$all 'ith no pro$lem% $#t he no' can play only one

game. >n physical e.amination% his 'eight is at the 2 th percentile% an" his height higher than the 8 th percentile. /e is thin an" has a gangly $o"y ha$it#s. /is fingers are long an" thin. There is pect#s "eformity in his chest. /is 9oints are hypere.tensi$le. An ophthalmologic e.amination re)eals mil" s#$l#.ation of the lens. =hich of the follo'ing is the most likely fin"ing on an echocar"iograma6Aortic root "ilatation $6Aortic stenosis c6Bic#spi" aortic )al)e "6@e.trocar"ia e6 P#lmonary stenosis :ormal La$s 42.An 1!year!ol" $oy 'ith sickle cell "isease presents 'ith left leg pain an" a high fe)er. /e has $een ref#sing to 'alk since yester"ay. >n physical e.amination% his temperat#re is &8.1 2 314&.< 56% $loo" press#re is 1227<1 mm /g% p#lse is 1427min% an" respirations are 247min. /is left fem#r is ten"er to palpation & cm a$o)e the left knee% an" there is marke" soft tiss#e s'elling. A plain film of his left leg is normal. A $one scan sho's increase" #ptake aro#n" the metaphysis of the left fem#r. =hich of the follo'ing is the most likely pathogena6Escherichia coli $6/aemophil#s infl#enDae c6Salmonella "6Staphylococc#s a#re#s e6Streptococc#s pne#moniae :ormal La$s 4&.A 24!month!ol" presents to the office 'ith a 2!"ay history of a harsh% $arking co#gh. /is mother states that the co#gh so#n"s like a seal. She also states he has not ha" any fe)er% altho#gh he ha" a r#nny nose earlier in the 'eek. >n e.amination% he is nota$ly hoarse 'ith inspiratory stri"or. /e is not "rooling an" is sitting on his mother,s lap comforta$ly. The rest of his e.amination is 'ithin normal limits. =hich of the follo'ing is the most likely "iagnosisa6Ac#te laryngotracheo$ronchitis $6Aspiration of foreign $o"y in the #pper respiratory tract c6Epiglottitis

"6Laryngomalacia e6S#$glottic stenosis :ormal La$s 44. A 14!year!ol" girl presents to clinic for an ann#al check #p. /er parents are concerne" that the chil" is not meeting normal milestones of a"olescent gro'th. The patient reports no recent illnesses. She is #p!to! "ate on her )accines% an" is not ha)ing any tro#$le in school. She "enies "r#g #se an" is acti)e in sports. >n e.amination% her )ital signs are sta$le. /er l#ngs are clear an" heart is reg#lar in rate an" rhythm. Breast an" areolar enlargement is note" 'ith no conto#r separation of the areola. P#$ic hair is note" to $e "ark. The patient also reports that she starte" menstr#ating 1 months ago. =hich of the follo'ing is the most likely Tanner stage of "e)elopment in this patienta6 Stage + $6Stage ++ c6Stage +++ "6Stage +? e6Stage ? :ormal La$s 4 . A &!'eek!ol" female infant $orn at &1 'eeks, gestation thro#gh an #ncomplicate" )aginal "eli)ery% presents to the emergency "epartment 'ith 2!"ay history of fe)er. The infant ha" $een healthy since $irth. /er temperat#re in the emergency "epartment is 44.4 2 3144.4 56. She appears to $e *#ite lethargic. A c#lt#re taken from the infant gro's gro#p B Streptococc#s 3CBS6 in 24 ho#rs. =hich of the follo'ing infections is most consistent 'ith this presentationa6 En"ocar"itis $6Castroenteritis c6Meningitis "6Pne#monia e6Pyelonephritis :ormal La$s 4<. A 8!month!ol" infant is $ro#ght to the pe"iatrician,s office $eca#se his 'eight is persistently $elo' the 14th percentile. /is mother states

that the infant seems to $e h#ngry all the time an" #s#ally cons#mes 1! 12 oD of form#la e)ery 2!& ho#rs in a""ition to some ta$le foo". /e also has fre*#ent% $#lky% an" malo"oro#s stools. A mala$sorption syn"rome is s#specte". The res#lts of 'hich of the follo'ing tests 'ill most likely $e a$normala6 A$"ominal ra"iography $65ecal fat *#antification c6Ser#m al$#min "6Stool c#lt#re for 2lostri"i#m "ifficile to.ins e6 Stool smear for le#kocytes an" eosinophils :ormal La$s 4(.A 24!month!ol" chil" is seen in the pe"iatrician,s office for a reg#lar health s#per)ision )isit. /e has no history of "e)elopmental "elay. /e 'as $orn $y an #ncomplicate" normal )aginal "eli)ery at term% an" he has not ha" any significant illness or in9#ry prior to this )isit. =hich of the follo'ing motor milestones is most consistent 'ith his agea6 B#il"ing a to'er of t'o c#$es $62opying a circle c6Scri$$ling "6Thro'ing a $all o)erhea" e6=alking $ack'ar" :ormal La$s 41. A 8!year!ol" $oy is $eing e)al#ate" for a syncopal episo"e. /e 'as playing $asket$all 'hen the syncope occ#rre". The episo"e laste" a$o#t 1!2 min#tes% an" then the patient 'oke #p on his o'n. /is mother also notice" that he has $een ha)ing hearing pro$lems. /is physical e.amination is #nremarka$le. An E2C sho's a prolonge" N!T inter)al% 'ith a NTc of 4.48 sec. =hich of the follo'ing is the most appropriate management if he has long N!T syn"romea6Alpha!a"renergic $locker $6Angiotensin con)erting enDyme inhi$itor c6Beta!a"renergic $locker "62alci#m channel $locker e6@i#retic :ormal La$s

48. The parents of a !year!ol" $oy come to the physician concerne" a$o#t their chil",s rec#rrent leg pains. The $oy has $een complaining for se)eral 'eeks a$o#t pain in $oth legs% #s#ally occ#rring soon after going to $e". /e "eri)es relief from r#$$ing his legs an" knees. /e "oes not limp an" is a$le to participate in sports acti)ities. =hich of the follo'ing is the most likely "iagnosisa6 0Cro'ing0 pains $6R#)enile rhe#matoi" arthritis c6>sgoo"!Schlatter "isease 3osteonecrosis of the ti$ial t#$erosity6 "6>steoi" osteoma e6>steosarcoma :ormal La$s 4.A !'eek!ol" infant is $ro#ght to the clinic for a 4!'eek history of noisy $reathing that has not impro)e". She has other'ise $een healthy e.cept for a c#rrent #pper respiratory infection for the past 4 "ays% 'hich accor"ing to the parents% has 'orsene" the noisy $reathing. >n e.amination% she has inspiratory stri"or. The noisy $reathing impro)es 'hen the infant is asleep. =hich of the follo'ing is the most likely "iagnosisa6Bronchoal)eolar carcinoma $65oreign o$9ect o$str#ction c6Laryngomalacia "6Bacterial pne#monia e6T#$erc#losis :ormal La$s

Note: Check your own answers before hittin$ the Check button below. When you click the Check button, a browser win"ow will a!!ear that contains a summary of your results. (*!lanations lock :: (*!lanations

:) 7he correct answer is %. Inhalant abuse is the intentional inhalation of volatile hy"rocarbons, such as mo"el $lue, correction flui", s!ray !aint, an" $asoline, to achieve an altere" mental state. It is a common health !roblem in a"olescence. 7he effect of inhalin$ a lar$e quantity of hy"rocarbons has been "escribe" as Hquick "runkH because it resembles alcoholic into*ication. Initially, eu!horia "evelo!sG then, li$hthea"e"ness an" a$itation. %isorientation, ata*ia, an" "i##iness mi$ht "evelo! with increasin$ into*ication. In e*treme cases, $enerali#e" weakness, hallucinations, an" nysta$mus can occur. Abusers often show "eterioration in school !erformance, "isturbance of family relationshi!s, an" increase" risk4takin$ behaviors. (nce!halo!athy is the ma<or chronic morbi"ity followin$ chronic inhalant abuse. 2y"rocarbons are hi$hly li!o!hilic an" can easily "istribute to the brain. )tu"ies have shown that chronic abusers have ra"io$ra!hic evi"ence of CN) "ama$e, such as loss of brain mass on C7 an" white matter "e$eneration on +6I. Clinically, chronic abusers often have co$nitive an" cerebellar "ysfunction, inclu"in$ !eri!heral an" cranial neuro!athy, visual loss, an" !arkinsonism. Inhalant abuse !oses a si$nificant health threat to teena$ers. )urveys have shown that about :/= to ;>= of hi$h school seniors have use" inhalants in the !ast. 7hese fi$ures, however, likely un"erestimate the true !revalence because of un"er4re!ortin$ an" school "ro!outs. A hi$h level of sus!icion is nee"e" to "ia$nose inhalant abuse. A $oo" history is essential because there is no "ru$ screen test that can "etect inhalant hy"rocarbons. ;) 7he correct answer is A. 7here are no re!orte" cases of con$enital rubella syn"rome 'C6)) cause" by e*!osure to the rubella vaccine "urin$ !re$nancy. Amon$ ;;C !re$nant women known to have receive" the rubella vaccine from :ID: to :I@C, none of the infants "evelo!e" any con$enital malformation that is com!atible to or similar to C6). It is recommen"e", however, that women of chil"bearin$ a$e shoul" receive the rubella vaccine only if they state that they are not !re$nant an" that they have been counsele" not to become !re$nant for 0 months because of the theoretical risk of e*!osure to the rubella virus throu$h vaccination. )houl" !re$nancy occurs within 0 months after vaccination, the woman shoul" be counsele" about the theoretical risk. ut, this is not an in"ication or a reason to terminate the !re$nancy.

0) (*!lanation: 7he correct answer is %. 7he infant in this clinical vi$nette has steatorrhea, as evi"ence" by !oorly forme" an" bulky stools an" !oor wei$ht $ain. One of the most im!ortant reasons for steatorrhea in newborns is bile aci" "eficiency. ile aci"s are very im!ortant in normal absor!tion of fat, which constitutes a ma<or !ortion of an infantAs calories intake. Fnfortunately, the bile aci" !ool in neonates is very small when com!are" with that in a"ults. In a""ition, neonates often lose an e*cessive amount of bile aci"s in their stools. 7his results in !hysiolo$ic steatorrhea because of !oor absor!tion of fat. &reterm infants, like the one "escribe" in this clinical vi$nette, have an even smaller bile aci" !ool an" are more likely to have steatorrhea because of !oor fat absor!tion. 7his will result in !oor wei$ht $ain. 7he solution to this !roblem is to substitute me"ium4chain tri$lyceri"es '+C7s) in the formula for lon$4chain tri$lyceri"es ',C7s), because, unlike ,C7s, +C7s "o not require bile aci"s for absor!tion. A""in$ !ancreatic en#ymes 'choice A) offers no hel! in this case because the !roblem is not a lack of !ancreatic en#ymes. Chan$in$ to a lactose4free formula 'choice ) will not correct the steatorrhea. Increasin$ volume !er fee"in$ 'choice C) without substitutin$ +C7s for ,C7s will only worsen the steatorrhea. In steatorrhea, fat4soluble vitamins, such as A an" (, are lost in the stool. 2owever, su!!lementin$ vitamins A an" ( 'choice () still cannot correct the steatorrhea. ?) : 7he correct answer is . 2emo!hilia is an E4linke" clottin$ "isor"er that occurs in two forms: hemo!hilia A "ue to "eficient factor 1III an" hemo!hilia "ue to "eficient factor IE. )ome in"ivi"uals with hemo!hilia have levels of these factors that are /= of normal or even hi$her, an" have relatively mil" "isease, only requirin$ re!lacement thera!y "urin$ sur$ical !roce"ures or other situations in which si$nificant blee"in$ mi$ht occur. In contrast, in"ivi"uals with factor levels less than := of normal have severe blee"in$ !roblems throu$hout life that usually become a!!arent 'as in this case) by :@ months of a$e. In these in"ivi"uals, e*cessive blee"in$ into <oints an" tissues may cause cri!!lin$ musculoskeletal "isor"ers. %isseminate" intravascular coa$ulation 'choice A) is associate" with an increase" &7, increase" &77, an" "ecrease" !latelets. 2emo!hilia 'choice C) is "ue to a "eficiency of factor IE. 2y!erhomocysteinemia 'choice %) causes a clottin$ ten"ency. 1on Willebran" "isease 'choice () !rimarily affects !latelet function. /) 7he correct answer is . Chil"ren with hy!os!a"ias are !rone to urinary tract infections an" other urinary tract anomalies. 7hey require careful evaluation. A renal ultrasoun" is a safe way of "ia$nosin$ neonatal urinary tract !atholo$y. )erum creatinine level 'choice A) is a measure of renal function an" is not nee"e" in an otherwise healthy chil". An intravenous

!yelo$ram 'choice C) uses contrast an" may cause ki"ney "ama$e. )imilarly, cysto$ra!hy 'choice %) may cause e*!osure to "ye or ra"iation, an" instrumentation may cause com!lications. Circumcision 'choice () is not in"icate", since in secon" or thir" "e$ree hy!os!a"ias, the !re!uce can be use" to construct an absent "istal se$ment of the urethra. C) (*!lanation: 7he correct answer is . A ?4year4ol" chil" is able to "raw a four4si"e" fi$ure 'i.e., a square), count to ?, i"entify four colors, say a four4to five4wor" sentence, an" "raw a !icture of a !erson with at least four !arts. '(asily remembere" as ?4year4ol"s "o thin$s in ?As.) uil"in$ a staircase with cubes 'choice A) is usually not achieve" until the a$e of C. A C4year4ol" can !erform sim!le a""ition an" "raw a !erson with :;4:/ !arts. %rawin$ a trian$le 'choice C) is not achieve" until the a$e of /. 7he avera$e /4year4ol" can "raw a trian$le, count to :>, re!eat a :>4syllable sentence, an" "raw a !icture of a !erson with @4:> !arts 'choice %). 7he ability to re!eat five "i$its 'choice () is ty!ical of a D4year4ol". 7he avera$e D4year4ol" also can re!eat three "i$its backwar", "raw a "iamon" sha!e, an" "raw a !erson with :@4;; !arts. D) 7he correct answer is A. 7he $ross a!!earance of this re" !laque, combine" with its "evelo!ment an" further $rowth be$innin$ in the first month of life, makes a "ia$nosis of heman$ioma most !robable. 2eman$iomas may involve the skin or "ee! or$ans, an" may be small or rather e*tensive lesions. )kin heman$iomas become a!!arent in the first month of life, bein$ occasionally !rece"e" by an erythematous macule. 7hey $row for C4:> months, reach a stable si#e, an" then un"er$o s!ontaneous re$ression. 7he $reat ma<ority of skin heman$iomas "isa!!ear entirely by a$e I. )ome heman$iomas nee" further "ia$nostic evaluation or treatment either because of their location, or because of the likelihoo" of bein$ associate" with other con$enital anomalies. +6I an"Bor C7 stu"ies 'choice ), for e*am!le, woul" be necessary in cases of e*tensive cervical heman$iomas, !eriorbital heman$iomas, or heman$iomas locate" in the lumbosacral re$ion. 7hese heman$iomas may be associate" with un"erlyin$ abnormalities, such as s!inal "ysra!hism, !osterior fossa malformations, or e*tension into orbital or laryn$eal structures. 6eferral to a "ermatolo$ist for bio!sy 'choice C) is rarely necessary when the $ross features are characteristic of classic heman$iomas. 7reatment with corticosteroi"s or interferon4alfa 'choice %) may be in"icate" in cases of !eriorbital heman$iomas. 7hese lesions may e*ten" into the orbit an" lea" to !ro$ressive visual im!airment. O!hthalmolo$ic evaluation an" follow4u! is warrante". )ur$ical e*cision 'choice () is never recommen"e". -urther $rowth of heman$iomas can be !revente" or slowe" by corticosteroi"s or interferon4alfa. ,aser treatment is useful for ulcerate" heman$iomas.

@) 7he correct answer is %. An a!ical click followe" by a late systolic murmur is classic for mitral valve !rola!se. It can be a source of sub<ective chest !ain in chil"ren. +itral valve !rola!se is more common in females. Antibiotic !ro!hyla*is is recommen"e" !rior to "ental !roce"ures. An atrial se!tal "efect 'choice A) is characteri#e" by a fi*e" an" wi"ely s!lit secon" heart soun". +itral 'choice ) an" tricus!i" re$ur$itation 'choice () !ro"uce holosystolic murmurs with relatively uniform intensity. +itral re$ur$itation is hear" at the a!e* while tricus!i" re$ur$itation is best hear" alon$ the lower left sternal bor"er. +itral stenosis 'choice C) is characteri#e" by a mi"4"iastolic murmur hear" after an o!enin$ sna!. I) 7he correct answer is A. 7he constellation of thymic hy!o!lasia, hy!ocalcemia 'with tetany), abnormal facies, an" con$enital car"iac anomalies "efines the con"ition known as %i.eor$e syn"rome. 7his results from a "evelo!mental failure of thir" an" fourth !haryn$eal !ouches, which $ives rise to con$enital absence or anomalies of the !arathyroi", thymus, lower face, an" car"iac structures. Immune "eficiency results from failure of 74lym!hocytes to mature in the thymus. 7hus, fun$al an" viral or$anisms, which are normally controlle" by 74 me"iate" mechanisms, become frequent causes of o!!ortunistic infections. 7he un"erlyin$ $ene "efect is relate" to ;;q:: "eletion, which results in two !artially overla!!in$ con"itions, i.e. %i.eor$e syn"rome an" velocar"iofacial syn"rome. 7hese con"itions are collectively referre" to as chromosome ;;q:: "eletion syn"rome. In utero infection by human immuno"eficiency virus '2I1) 'choice ) woul" result in 74cell "eficiency but woul" not be associate" with con$enital abnormalities. +utations of the autosomal $ene enco"in$ a"enosine "eaminase 'choice C) re!resent the most common cause of the recessive form of severe combine" immuno"eficiency "isease ')CI%), encom!assin$ a hetero$eneous $rou! of con"itions characteri#e" by "eficiency of both 74 an" 4cell mechanisms. )CI% may be autosomal "ominant, autosomal recessive, or E4linke". +utations of the E4linke" $ene co"in$ for a cytokine rece!tor subunit 'choice %) re!resent the most common cause of the autosomal "ominant form of )CI %. +utation of an E4linke" $ene co"in$ for a tyrosine kinase 'choice () is the un"erlyin$ molecular mechanism lea"in$ to E4linke" a$amma$lobulinemia of ruton, a syn"rome characteri#e" by inability of !re4 cell !recursors to mature into 4lym!hocytes. 2umoral immune "eficiency thus manifests. :>) 7he correct answer is C. 7he clinical history is most consistent with a nasal forei$n bo"y. Chil"ren sometimes insert forei$n ob<ects into their nose, such as !ebbles, !eas, foo" !articles, etc. 7he sym!toms at first are limite" to frequent snee#in$ an" obstruction, which can be misinter!rete" as a common col" or aller$y. ,ater, infection "evelo!s, resultin$ in a !urulent an" malo"orous "ischar$e. 7he unilateral involvement is an im!ortant clue to "ia$nosis. Aller$ic rhinitis 'choice A) is characteri#e" by rhinorrhea, with a clear nasal

"ischar$e. Often, con<unctivitis is also !resent. 7he sym!toms may be seasonal or !erennial 'the whole year roun"), "e!en"in$ on the incitin$ aller$en. Nuvenile an$iofibroma 'choice ) is a hi$hly vascular fibrous tumor that classically affects a"olescent males an" a!!ears to be relate" to an"ro$enic stimulation. It manifests with recurrent e!ista*is. Nasal !oly! 'choice %) results from hy!ertro!hy an" hy!er!lasia of the nasal mucosa in res!onse to chronic inflammatory stimuli, most often "ue to aller$ic reactions. A history of aller$ies 'hay fever) is usually !resent. In chil"ren, nasal !oly!s shoul" su$$est the !ossibility of cystic fibrosis. 6hinitis me"icamentosa 'choice () is "ue to chronic use of nasal "econ$estants. 7hese "ru$s 'namely, neosyne!hrine an" o*ymeta#oline) !ro"uce vasoconstriction an" reboun" vaso"ilatation. Over time, tolerance "evelo!s, an" !ersistent vaso"ilatation with rhinorrhea manifests. 1asomotor rhinitis 'choice -) is a frequent con"ition !robably me"iate" by !atholo$ic reactivity to nasal stimuli. Ci$arette smoke an" col" tem!eratures are the most common incitin$ events, which cause runny nose in in"ivi"uals !re"is!ose" to vasomotor rhinitis. ::) 7he correct answer is (. 7he boy in this clinical vi$nette has osteo$enesis im!erfecta, which is a $rou! of "isor"ers cause" by "eficiencies in the synthesis of ty!e : colla$en. Althou$h the hallmark of the "isease is !rominent skeletal "eformity, other anatomic structures rich in ty!e : colla$en, such as <oints, eyes, ears, skin, an" teeth, are affecte" as well. 7he clinical e*!ression of osteo$enesis im!erfecta constitutes a s!ectrum of "isor"ers all marke" by e*treme skeletal fra$ility. -our ma<or subty!es have been "istin$uishe". 7he ty!e II variant is at one en" of the s!ectrum an" is uniformly fatal in utero or "urin$ the !erinatal !erio". It is characteri#e" by e*traor"inary bone fra$ility with multi!le fractures occurrin$ when the fetus is still within the womb. In contrast, the ty!e I form, which is more often "ue to an acquire" rather than to a here"itary mutation, !ermits a normal life s!an but with an increase" number of fractures "urin$ chil"hoo". 7hese "ecrease in frequency after !uberty. Other fin"in$s inclu"e blue sclerae cause" by a "ecrease in colla$en content, makin$ the sclera translucent an" allowin$ !artial visuali#ation of the un"erlyin$ choroi"G hearin$ loss relate" to both a sensorineural "eficit an" im!e"e" con"uction owin$ to abnormalities in the bones of the mi""le an" inner earG an" "ental im!erfections 'small, missha!en, an" blue4yellow teeth) secon"ary to a "eficiency of "entin. In ty!e III, the bony "eformity is !ronounce" an" not necessarily "ue to fractures. +obility is im!aire", an" most !atients require a wheelchair at an early a$e. )tature may be severely com!romise". ecause of a !ro$ressive vertebral column "eformity an" rib fractures, restrictive lun$ "isease is a common !roblem. A basilar im!ression causin$ com!ression of the brainstem an" the craniocervical <unction can !ro"uce central slee! a!nea, hea"ache, an" u!!er motor neuron si$ns. &atients with ty!e I1 osteo$enesis im!erfecta $enerally have re"uce" stature, some bony "eformity, an" abnormal teeth that are o!alescent an" wear easily. 7he ten"ency to fracture is hi$hest in chil"hoo" an" lessens with a"olescence. A "istin$uishin$ characteristic of ty!e I1 is a normal scleral hue. 7he mana$ement of skeletal com!lications lar$ely "e!en"s on ortho!e"ic, !hysical, an" occu!ational thera!y a!!roaches. 7he lon$4term $oal is to maintain function an" in"e!en"ence as an in"ivi"ual. 7hese $oals can be a"vance" in some by <u"icious use of

intrame"ullary ro"s in the lon$ bones of the le$s. If mobility, es!ecially ambulation, can be maintaine", the "eminerali#ation associate" with inactivity can be avoi"e". Fnaffecte" !arents of a chil" with osteo$enesis im!erfecta, as well as all affecte" in"ivi"uals, shoul" have $enetic counselin$. Achon"ro!lasia 'choice A) is one of the chon"ro"ystro!hies that result in "is!ro!ortionately short stature. )hort, tubular bones form because of abnormal en"ochon"ral ossification in the limbs. Constitutional "elay of $rowth 'choice ) is su$$este" by a chil" who is $rowin$ at a normal or mil"ly "ecrease" rate. 7he !atient is "elaye" in !ubertal "evelo!ment, an" the bone a$e si$nificantly la$s behin" the chronolo$ic a$e. %evelo!mental "ys!lasia of the hi!s 'choice C) results from loss of contact between the acetabulum an" the hea" of the femur. -amilial short stature 'choice %) is characteri#e" by a chil" with short !arents, by a bone a$e consistent with the chronolo$ic a$e, an" by a $rowth curve that follows the normal !attern even thou$h it is si$nificantly below the 0r" !ercentile. :;) 7he correct answer is C. Infant botulism results from the !ro"uction of to*in after coloni#ation of the $astrointestinal tract by Clostri"ium botulinum in youn$ chil"ren a$e" :4I months. 7he most common source of the or$anism is the soil or, less frequently, honey. Nearly all cases are "ue to ty!es A or . 7he incubation !erio" is usually between :@ an" 0C hours. )hort incubation !erio"s are associate" with more severe "isease. 7he "isease s!ectrum varies consi"erably, but the most commonly reco$ni#e" form is the Hflo!!y baby syn"rome.H Initial sym!toms are lethar$y, "iminishe" suck, consti!ation, weakness, feeble cry, an" "iminishe" s!ontaneous activity with loss of hea" control. 7hese sym!toms are followe" by e*tensive flacci" !aralysis. 7he case fatality rate is only :=. 7he bulbar musculature is usually affecte" first. In ol"er chil"ren, it results in "i!lo!ia, "ysarthria, an" "ys!ha$ia. Involvement of the choliner$ic autonomic nervous system may result in "ecrease" salivation, with "ry mouth an" sore throat, ileus, or urinary retention. Neurolo$ic evaluation often shows bilateral !aresis of the Cth cranial nerves, !tosis, "ilate" !u!ils with slu$$ish reaction, "ecrease" $a$ refle*, or me"ial rectus !aresis. 7hese sym!toms are followe" by "escen"in$ involvement of motor neurons to !eri!heral muscles, inclu"in$ the muscles of res!iration. &atients are usually afebrile with clear mentation. 7he most common cause of "eath is res!iratory failure. 7he s!ectrum of "isease is quite variableG some !atients have mil" "isease, whereas others have severe !aralysis requirin$ mechanical ventilation. 6es!iratory failure is the ma<or risk, an" !atients must be monitore" carefully with liberal use of ventilatory su!!ort. 7o*ins can be remove" from the $astrointestinal tract with $astric lava$e, cathartics, an" enemas early in the course of "isease. 7he trivalent antito*in or ty!e4s!ecific antito*in for ty!es A, , an" ( is usually $iven only to a"ults. Infants with botulism shoul" not receive either antibiotics "irecte" a$ainst C. botulinum or antito*in, because most "o e*tremely well with su!!ortive care alone an" it has been su$$este" that antibiotics may cause to*in release. 2oney has been im!licate" as a vehicle for s!ores an" shoul" not be fe" to infants youn$er than : year. Con$enital hy!othyroi"ism 'choice A), or cretinism, manifests as im!aire" "evelo!ment

of the skeletal system an" CN). It is associate" with severe mental retar"ation, short stature, coarse facial features, a !rotru"in$ ton$ue, an" umbilical hernia. .uillain4 arrV syn"rome 'choice ) !resents with weakness that "evelo!s symmetrically over several "ays. 7he weakness ty!ically occurs first in the le$s an" ascen"s with time to involve the muscles of the trunk, intercostals, u!!er e*tremity, an" neck. +uscles innervate" by cranial nerves are also involve". 6es!iratory !aralysis can lea" to "eath within hours to "ays. +yasthenia $ravis 'choice %) is a "isor"er of the neuromuscular <unction resultin$ in a !ure motor syn"rome characteri#e" by weakness an" fati$ue, !articularly of the e*traocular, !haryn$eal, facial, cervical, !ro*imal limb, an" res!iratory musculature. -ifteen !ercent of infants born to myasthenic mothers have neonatal myasthenia $ravis because of the trans!lacental !assa$e of acetylcholine rece!tor antibo"ies. 7he con"ition com!letely resolves in weeks to months. 1accine4associate" !oliomyelitis 'choice () is e*cee"in$ly rareG only ei$ht or nine cases are re!orte" yearly. +ost sym!tomatic cases have nons!ecific manifestations of infection. Illness is bi!hasic, an" !aralysis occurs in the secon" !hase. &aralytic "isease occurs with ra!i" onset, involvin$ the cranial nerves, arms, an" le$s. :0) C. 7his infant has 2irschs!run$ "isease, or con$enital a$an$lionic bowel "isease. It is five times more common in boys than in $irls. It results from con$enital absence of $an$lion cells in either !art of or the entire wall of the colon, resultin$ in a state of chronic contraction. In most cases, the a$an$lionic se$ment is limite" to the rectosi$moi" colon. In very rare cases, !art of or the entire small bowel can be a$an$lionic as well. ilious or feculent vomitin$, ab"ominal "istention, an" consti!ation are the classic clinical si$ns. 7here mi$ht also be a history of failure to !ass meconium in the first ?@ hours of life. If only a short se$ment of the colon is involve", 2irschs!run$ "isease mi$ht not be evi"ent until in chil"hoo" or a"olescence. +e$acolon !ro*imal to the a$an$lionic se$ment mi$ht be visible on barium enema. 7he "ia$nosis is confirme" with the "emonstration of an a$an$lionic se$ment of the bowel on !unch bio!sy. %uo"enal atresia 'choice A) usually !resents with vomitin$. A H"ouble bubbleH si$n is seen on ab"ominal ra"io$ra!hy. 7hirty to forty !ercent of cases are associate" with %own syn"rome. Intussusce!tion 'choice ) is certainly in the "ifferential "ia$nosis of vomitin$ an" ab"ominal "istention. In this case, however, the history of failure to !ass meconium in the newborn !erio" an" failure to thrive is much more su$$estive of 2irschs!run$ "isease. Intussusce!tion occurs when one se$ment of the bowel telesco!es into another se$ment <ust "istal to it. 7he most common site of intussusce!tion is the ileocolic <unction. +alrotation 'choice %) is usually cause" by the !resence of a volvulus, which !resents with su""en onset of bilious vomitin$, ab"ominal "istention, rectal hemorrha$e, !eritonitis, an" shock. It is a sur$ical emer$ency. &yloric stenosis 'choice () usually !resents with !ro<ectile vomitin$ in the first ; or 0 weeks of life. On e*amination, an olive4sha!e" mass is usually !al!able in the e!i$astric area. It is cause" by hy!ertro!hy an" hy!er!lasia of the antrum of the stomach, resultin$ in obstruction. It occurs in : of :/> boys an" in : of D/> $irls.

:?) 7he correct answer is A. 7hese fin"in$s su$$est that this boy has bla""er outlet obstruction. 7he most common cause of urethral obstruction in males is !osterior urethral valves. It ty!ically results in urinary obstruction an" vesicoureteral reflu*. Its inci"ence is : in />>>4@>>> males. 7he "e$ree of obstruction variesG therefore, the "e$ree of renal failure also varies. It can ran$e from mil" hy"rone!hrosis to renal "ys!lasia. 1oi"in$ cystourethro$ram is the "efinitive "ia$nostic test. Frethral strictures an" urethral meatal stenosis can also cause obstruction "istal to the urinary bla""er, but these con"itions are uncommon. &rimary !oly"i!sia 'choice ) can cause enuresis, but $rowth failure is not characteristic. 6eflu* ne!hro!athy 'choice C) occurs in !atients who have vesicoureteral reflu*. 7he bla""er shoul" not be !al!able. )ickle cell trait 'choice %) can cause hy!osthenuria, but it usually ha!!ens in African American boys in late chil"hoo" or a"ulthoo". An unstable bla""er 'choice () is not !al!able, an" $rowth is normal. :/) 7he correct answer is . 7his !atientAs clinical !icture is consistent with acute cocaine in$estion. Cocaine is a CN) stimulant that causes increase" heart rate, hy!ertension an" hy!eractive refle*es. Other substances of abuse can cause car"iac arrhythmias an" sei#ures but of the answer choices, only cocaine causes my"riasis or "ilate" !u!ils. 7reatment is su!!ortive an" may inclu"e anti4arrhythmics, anti4hy!ertensives, coolin$ blankets an" se"atives. arbiturates 'choice A) are CN) "e!ressants an" woul" cause hy!otension, normal or constricte" !u!ils an" "iminishe" refle*es. Other classic features of barbiturate over"ose inclu"e ata*ia, slurre" s!eech an" nysta$mus. 2eroin 'choice C) is an o!iate an" woul" !ro"uce sym!toms of CN) "e!ression 'hy!otension, "iminishe" or absent refle*es, hy!oventilation, hy!othermia) an" constricte" !u!ils. Other sym!toms of o!iate into*ication inclu"e nausea, vomitin$, eu!horia, sei#ures an" coma. +ari<uana 'choice %) in$estion or inhalation causes relatively beni$n sym!toms that inclu"e eu!horia, hun$er, tachycar"ia an" in<ecte" con<unctivae. &u!ils usually remain normal. &C& 'choice () is a hallucino$en that causes CN) stimulation 'hy!ertension, tachycar"ia, brisk refle*es, hy!erthermia). Other hallucino$ens such as ,)% an" mescaline cause "ilate" !u!ils but &C& has an effect of constrictin$ !u!ils. )ei#ures an" coma are more common with &C& an" intracranial hemorrha$es have been re!orte". :C) 7he correct answer is . 7he !resentation is consistent with im!eti$o. Im!eti$o may be of two forms, nonbullous an" bullous. Nonbullous im!eti$o 'more frequent) "evelo!s by coloni#ation of traumati#e" skin by !yo$enic bacteria an" manifests with the characteristic vesiculo!ustular eru!tion that results in honey4colore" crusts. ullous im!eti$o is "ue to a strain of )ta!hylococcus aureus that !ro"uces e*foliatin, a to*in causin$ intrae!i"ermal cleava$e. ullous im!eti$o manifests with lar$e flacci" bullae that ru!ture easily. Currently, )ta!hylococcus aureus is the etiolo$ic a$ent of most cases of nonbullous

im!eti$o, while until two "eca"es a$o )tre!tococcus !yo$enes'choice C) accounte" for the ma<ority of cases. 7his chan$e has im!ortant thera!eutic im!lications. ). !yo$enes is sensitive to !enicillin, while ). aureus is not. 7hus, the most a!!ro!riate treatment of both forms of im!eti$o is base" on antibiotics effective a$ainst ). aureus. 7reatment o!tions inclu"e systemic erythromycin, first4$eneration ce!halos!orins, an" to!ical mu!irocin. +u!irocin is a fermentation !ro"uct "erivin$ from &seu"omonas fluorescens. It acts by inhibitin$ bacterial !rotein synthesis. 2uman her!esvirus D 'choice A) is sus!ecte" to be the etiolo$ic a$ent of !ytiriasis rosea. 7his self4limite" "ermatosis is characteri#e" by scaly !ink macules that affect youn$ a"ults an" chil"ren. &ro!ionibacterium acnes'choice %) is thou$ht to !lay an im!ortant role in the !atho$enesis of acne. Coloni#ation by this bacterium lea"s to release of !ro4inflammatory an" chemotactic a$ents that in"uce inflammation of the !ilosebaceous unit. 7rycho!hyton fun$i 'choice () are the most common !atho$enic "ermato!hytes causin$ tinea cor!oris, "ue to su!erficial invasion of the stratum corneum by fun$al or$anisms. :D) 7he correct answer is %. 7his !atient has acute rheumatic fever from $rou! A stre!tococci. 2e has mi$ratory !olyarthritis, erythema mar$inatum, an" subcutaneous no"ules. Other features absent in this !atient are chorea an" car"itis. 2is A)O titer in"icates recent infection with )tre!tococcus. It is a"visable to a"minister !enicillin for the infection. 7he arthritis can be mana$e" with salicylates. Acetamino!hen 'choice A) may be nee"e" to kee! his tem!erature "own an" !revent a febrile sei#ure. 2owever, this is not a treatment. Also, a nonsteroi"al anti4inflammatory "ru$ is nee"e" for the arthritis. As!irin 'choice ) will combat the arthritis, but the infection will remain untreate". )uch a course will e*!ose the !atient to !otential rheumatic heart "isease, inclu"in$ severe mitral stenosis, in the future. 7he !enicillin alone 'choice C) will cover the infection an" eliminate the risk of rheumatic heart "isease. 2owever, the !atient also has a !ainful arthritis that nee"s to be a""resse". )u!!ortive care 'choice () is not an o!tion for this !atient. (ven if he is aller$ic to !enicillin, an alternative covera$e for $ram4!ositive bacteria is man"atory. &ro!hylactic thera!y for an associate" movement "isor"er is not nee"e". :@) (*!lanation: 7he correct answer is . %evelo!mental "ys!lasia of the hi! $enerally inclu"es sublu*ation '!artial "islocation) of the femoral hea", acetabular "ys!lasia, an" com!lete "islocation of the femoral hea" from the acetabulum. 7he inci"ence of "evelo!mental "ys!lasia of the hi! is about : in :>>> live births. 7he left hi! is more commonly involve" than the ri$ht, an" bilateral involvement is more common than involvement of the ri$ht hi! alone. 7he "isor"er is more common in females than in males4in many series, as much as five times more common. A!!ro*imately 0= to ?= of all "eliveries are breech, an" the inci"ence of "evelo!mental "ys!lasia of the hi! is si$nificantly increase" in these infants. 7he combination of female se* an" breech !resentation results in "evelo!mental "ys!lasia of

the hi! in : in 0/ such births. %evelo!mental "ys!lasia of the hi! is also more common in firstborn chil"ren than in subsequent siblin$s 'com!are with choice C). A family history increases the likelihoo" of this con"ition to a!!ro*imately :>=. (thnic back$roun" !lays some role, in that "evelo!mental "ys!lasia of the hi! is more common in white chil"ren than in black chil"ren 'com!are with choice A). Other re!orte" e*am!les inclu"e the hi$h inci"ence amon$ the Nava<o In"ians an" the relatively low inci"ence amon$ the Chinese. +aternal !reeclam!sia 'com!are with choice %) an" !rematurity 'com!are with choice () "o not seem to have an association with this con"ition. In newborns 'u! to C months), it is es!ecially im!ortant to !erform a careful clinical e*amination, since roent$eno$rams are not absolutely reliable in "ia$nosin$ con$enital "ys!lasia of the hi! in this a$e $rou!. 6outine clinical screenin$ shoul" inclu"e both the Ortolani test an" the !rovocative maneuver of arlow. 7he Ortolani test is !erforme" by $ently ab"uctin$ an" a""uctin$ the fle*e" hi! to "etect any re"uction into or "islocation of the femoral hea" from the true acetabulum. 7he !rovocative maneuver of arlow "etects any !otential sublu*ation or !osterior "islocation of the femoral hea" by "irect !ressure on the lon$itu"inal a*is of the femur while the hi! is in a""uction. :I) 7he correct answer is %. Always remember your A Cs. A saturation of I>= corres!on"s to an o*y$en !artial !ressure of aroun" C> mm 2$. 7herefore, this !atient is hy!o*ic an" nee"s o*y$en to hel! "ecrease his work of breathin$. A chest *4ray film 'choice A) is certainly in"icate", but not a !riority. Albuterol nebuli#er treatments 'choice ) are in"icate" as an initial trial. If benefits are seen, they can be continue". )ome stu"ies have shown racemic e!ine!hrine to be more effective than albuterol in viral res!iratory infections. 7hey too shoul" follow initial o*y$en thera!y. %elaye" ca!illary refill is an in"ication for a bolus of I1 flui" 'choice C). 7his however, is the C in A Cs. Central refill, or the time it takes for the color to return to the skin after !ressure is a!!lie" with oneAs fin$er, is usually less than 0 secon"s. )teroi"s 'choice () are not in"icate" in viral u!!er res!iratory infections. In !atients with sus!ecte" asthma, steroi"s are im!ortant, but are not the !rimary intervention. ;>) 7he correct answer is . 7he ability to lift the hea" to I> "e$rees, eyes crossin$ the mi"line, lau$hin$ an" sli$ht awareness of the care$iver are characteristic chil"hoo" "evelo!ment lan"marks of a ? month ol" infant. A ; month ol" infant 'choice A) can lift its hea" to ?/ "e$rees, eyes follow to the mi"line, vocali#es, smiles an" has a state of half4wakin$ consciousness. A C4month ol" infant 'choice C) can roll over, $ras! a rattle, turn to voice, fee" self an" se!arate the worl" into a H!arentH an" Hnot !arentH worl". A :; month ol" chil" 'choice %) can sit without su!!ort, !ull to stan", use a !incer $ras!, babble, in"icate wants, an" have stran$er an*iety. A :@ month chil" 'choice () can walk well, make a tower of ; blocks, say three wor"s, use a s!oon an" a cu!, have tem!er tantrums, an" bri"$e $a!s by brin$in$ ob<ects to the care$iver.

;:) C. 6heumatic fever is an inflammatory "isease, !ossibly autoimmune in nature. Immune res!onses to $rou! A stre!tococcal anti$ens "urin$ !haryn$itis resultin$ in antibo"y cross4reactions with myocar"ial anti$ens remain central to the !atho$enesis. 6heumatic fever involves many tissues, inclu"in$ the heart, <oints, skin, an" CN). &rece"in$ infection with $rou! A )tre!tococcus is a !rerequisite to the "evelo!ment of acute rheumatic fever. Initially, fever, "ys!nea, chest !ain, an" car"iac murmur "evelo!. 7he Nones criteriaZ for "ia$nosis are liste" below: +a<or: Car"itis (rythema mar$inatum an" subcutaneous no"ules )y"enhamAs chorea Arthritis +inor: -ever &olyarthral$ias 6eversible !rolon$ation of the &6 interval (levate" erythrocyte se"imentation rate 6ecent beta4hemolytic stre!tococcal infection 2istory of rheumatic fever Z7wo ma<or criteria or one ma<or an" one minor criterion establish the "ia$nosis. 7he initial e!iso"e of rheumatic fever can be !revente" by early treatment of stre!tococcal !haryn$itis. O!timal thera!y is intramuscular ben#athine !enicillin . as a sin$le "ose. &enicillin 15 or amo*icillin for :> "ays can also be use". -or the !enicillin4aller$ic !atient, either erythromycin succinate for :> "ays or a#ithromycin for / "ays is effective. Nuvenile rheumatoi" arthritis 'choice A) is characteri#e" by chronic synovitis an" systemic inflammatory manifestations. It "oes not !resent with car"itis. ,yme "isease 'choice ) may be$in with mi$ratory !olyarthral$ias an" !ro$ress to attacks of asymmetric oli$oarthritis in lar$e <oints. (rythema chronica mi$rans, the "ia$nostic annular re" lesion at the site of the tick bite, occurs in />= to @>= of cases "urin$ the early !hase of illness. Car"iac involvement is aty!ical. )e!tic arthritis 'choice () is an infection of the synovial cavity. It "oes not follow a stre! !haryn$itis an" "oes not cause car"itis or valvular "eformities. ;;) 7he correct answer is %. 7his boy most likely has he!atitis A. Althou$h it is more common in "evelo!in$ countries, it is still a common infection in the "evelo!e" worl". It is transmitte" by the fecal4oral route when contaminate" foo" or water is con$este". 7he incubation !erio" is about :/4?> "ays. Ninety !ercent of chil"ren acutely infecte" with he!atitis A are asym!tomatic. 7he virus is she" in the feces ;40 weeks before the onset of <aun"ice until a!!ro*imately : week after onset. +ost infecte" chil"ren are infectious for a lon$ time before they are sym!tomatic. 7herefore, he!atitis A is very "ifficult to control. ,ar$e outbreaks frequently occur in "aycare centers.

%ia$nosis is best ma"e by "etermination of I$+ levels a$ainst he!atitis A virus. 7he !resence of I$+ for he!atitis A su$$ests acute infectionG this antibo"y !eaks at ?4C weeks an" "oes not !ersist beyon" C months. I$. is !ro"uce" in the !rimary infection, but for most viral infections, inclu"in$ he!atitis A, it !ersists for a life time. 7herefore, the !resence of I$. a$ainst he!atitis A 'choice ) coul" mean a !revious infection an" is not "ia$nostic of a current infection. )tool culture for he!atitis A 'choice () cannot be use" to confirm the "ia$nosis. &ro!hyla*is with immuno$lobulin is recommen"e" for the househol" an" close contacts of the infecte" !erson within the first ; weeks of e*!osure. )trict han" washin$ is also very im!ortant. 7here is a kille"4virus vaccine available that !rovi"es immunity to travelers to "evelo!in$ countries if $iven in two "oses C4:; months a!art. 7he !resence of he!atitis surface anti$en in the serum 'choice A) si$nifies !revious infection with he!atitis . I$. a$ainst he!atitis surface anti$en 'choice C) comes from an immunolo$ic res!onse to either the he!atitis virus surface anti$en from an infection, or from the he!atitis virus vaccine. ;0) (*!lanation: 7he correct answer is (. 1esicoureteral reflu* is a common anatomical cause of recurrent urinary tract infections in chil"ren. &atients with reflu* have retro$ra"e flow of urine into the ureter an"Bor ki"ney before voi"in$. 7his occurs because of an incom!etent vesicoureteral valve. +any chil"ren out$row mil" "e$rees of reflu* if they are maintaine" on !ro!hylactic antibiotics. +o"erate to severe cases of reflu* frequently require sur$ery. -ailure of antibiotics is also an in"ication for sur$ery. A C7 scan of the !elvis 'choice A) is not a functional stu"y an" will not necessarily a"" to the "ia$nosis since reflu* will not be "emonstrate". Intravenous antibiotics woul" be nee"e" in the settin$ of !ersistent infection on oral antibiotics 'choice ), however, sur$ical intervention will be "efinitive. An intravenous !yelo$ram 'choice C) woul" hel! in outlinin$ the collectin$ system, but woul" not !rovi"e much a""itional information. 2owever, a ra"ionucli"e scan coul" be very hel!ful in "eterminin$ the "e$ree of reflu* !resent with a minimum of ra"iation e*!osure. A renal arterio$ram 'choice %) on an alrea"y "ia$nose" case woul" not be of much value. ;?) A. Attention "eficitBhy!eractivity "isor"er 'A%2%) is characteri#e" by im!ulsivity, hy!eractivity, an" inattention lastin$ at least C months. 7o make the "ia$nosis, the "isor"er must have starte" before a$e D, an" si* si$ns each of inattention an" im!ulsivityBhy!eractivity nee" to be !resent. In chil"ren, bi!olar "isor"er 'choice ) often !resents as e*tremely irritable an" e*!losive moo", with !oor !sychosocial functionin$, "ecrease" slee!, talkativeness, racin$ thou$hts, an" hi$h ener$y. Con"uct "isor"er 'choice C) is "ia$nose" when the chil" has ha" sym!toms of a$$ression towar" !eo!le an" animals, "estruction of !ro!erty, "eceitfulness or theft, an" serious violation of rules. One sym!tom nee"s to last at least C months, even thou$h the sym!toms may

have been !resent intermittently for a year. &osttraumatic stress "isor"er 'choice %) "evelo!s followin$ trauma, an" the chil" usually has sym!toms of hy!ervi$ilance, increase" arousal avoi"ance, an" autonomic reactivity. 6ett syn"rome 'choice () belon$s to the !ervasive "evelo!mental "isor"ers mostly seen in $irls. 7he chil" has normal "evelo!ment throu$h the first / months of life. %urin$ the secon" year, however, she "evelo!s multi!le "eficits, inclu"in$ "ecrease" hea" $rowth, "ecrease" han" skills, social im!airment, an" im!aire" $ait an" trunk movements. ;/) . 7he clinical !resentation is consistent with "ia!er "ermatitis. 7he most im!ortant clinical clue is the "istribution of the erythematous areas. 7he usual cause of "ia!er "ermatitis is irritant contact "ermatitis, resultin$ from the concomitant action of fecal an" urinary en#ymes, entra!!e" moisture an" e*cessive heat in the "ia!er area. Avoi"in$ ti$ht occlusion by "ia!ers, bathin$ with mil" soa!s an" water, an" frequent "ia!er chan$es constitute the essential a!!roach to treatment. In the most severe cases, to!ical a!!lication of low4stren$th corticosteroi" cream an"Bor #inc o*i"e may be in"icate". Can"i"iasis 'choice A) manifests with !a!ules an" !laques rather than e*tensive shiny areas of erythema. Can"i"a infection, however, "evelo!s su!erim!ose" on "ia!er "ermatitis. If si$ns of can"i"iasis are !resent, a to!ical antifun$al a$ent shoul" be use" in treatment. ,an$erhans cell histiocytosis 'choice C) is a severe "isease "ue to neo!lastic !roliferation of ,an$erhans cells. In infants, the con"ition manifests in its systemic life4 threatenin$ form known as ,etterer4)iwe syn"rome. 7he baby is critically ill, with fever an" a "iffuse scaly rash. &soriasis 'choice %) may be confuse" with "ia!er "ermatitis. &soriasis is characteri#e" by well4"emarcate" silvery !laques that often involve skin fol"s in infants. )eborrheic "ermatitis 'choice () manifests with salmon4colore", scaly, oily !laques that may affect the "ia!er area, but most commonly involve the scal! an" face. ;C) 7he correct answer is %. 2y!o!lastic left heart '2,2) syn"rome is a $rou! of closely relate" car"iac anomalies characteri#e" by un"er"evelo!ment of the left car"iac chambers, atresia or stenosis of the aortic an"Bor the mitral orifices, an" hy!o!lasia of the aorta. 7hese anomalies are an es!ecially common cause of heart failure in the :st week of life. 7he left atrium an" ventricle often e*hibit en"ocar"ial fibroelastosis. &ulmonary venous bloo" traverses a !atent foramen ovale, an" a "ilate" an" hy!ertro!hie" ri$ht ventricle acts as the systemic, as well as !ulmonary, ventricleG the systemic circulation receives bloo" by way of a !atent "uctus arteriosus. Infants who have 2,2 syn"rome "evelo! !oor !erfusion, an" metabolic aci"osis when systemic bloo" flow "ecreases. When the "uctus closes, ina"equate bloo" flow to the bo"y occurs because the "uctus is the only !ath for bloo" to flow from the ri$ht ventricle to the bo"y. (ven if the "uctus remains o!en, when the infant is $iven o*y$en, the o*y$en will "ilate the vasculature of the !ulmonary circulation, an" bloo" will !referentially flow to the lower !ressure !ulmonary system, "e!rivin$ the systemic circulation of a"equate !erfusion. (C. usually shows ri$ht a*is "eviation, ri$ht atrial an" ventricular enlar$ement, an" nons!ecific )7 an" 74wave

abnormalities in the left !recor"ial lea"s. Chest ra"io$ra!hy may show only sli$ht enlar$ement shortly after birth, but with clinical "eterioration there is marke" car"iome$aly with increase" !ulmonary vascular markin$s. (chocar"io$ra!hy is "ia$nostic an" will show a "iminutive aortic root an" left ventricular cavity an" absence or !oor visuali#ation of aortic an" mitral valves. An infant with 2,2 syn"rome has a hy!er"ynamic !recor"ium because the enlar$e" ri$ht ventricle is contractin$ a$ainst systemic !ressure. 7he infant also has a lou", or even !al!able, secon" heart soun" ');) because the !ulmonary artery acts as the aorta by !um!in$ bloo" to the systemic circulation throu$h the "uctus arteriosus. 7he hi$h en"4systolic !ressure marke"ly enhances );. 7he flow from the ri$ht ventricle to the !ulmonary artery is not turbulentG therefore, there is usually no si$nificant murmur hear" on auscultation. When the "uctus closes, or when the !ulmonary vasculature resistance falls, the flow to the systemic circulation will "ecrease, causin$ $reatly "iminishe" !eri!heral !ulses. +ana$ement inclu"es infusion of !rosta$lan"in (: an" a"ministration of room air while on a ventilator. &rosta$lan"in (: may o!en the "uctus arteriosus an" restore systemic bloo" flow. A"ministration of room air or even hy!obaric o*y$en '-iO; less than ;:=) an" the use of muscle rela*ants can !revent hy!erventilation an" subsequent !ulmonary vaso"ilation, thus reversin$ systemic hy!o!erfusion an" metabolic aci"osis. ;D (*!lanation: 7he correct answer is (. In $eneral, fractures in chil"ren !ose fewer !roblems than fractures in a"ults. 2owever, this !articular lesion in chil"ren is !rone to result in vascular or nerve in<uries an" requires close follow4u! imme"iately after it has been re"uce" an" immobili#e". O!en re"uction an" internal fi*ation 'choice A) are not necessarily the rule in these cases. Close" re"uction is usually !ossible. 6emo"elin$ 'choice ) is invariably e*cellent in fractures in chil"ren, an" this one is no e*ce!tion. .rowth !late "ama$e 'choice C) occurs only when the fracture crosses the <oint or when the $rowth !late is crushe", neither of which is the case here. +alunion 'choice %) in the u!!er e*tremity is the bi$ fear with "is!lace" navicular fractures, but not with humeral fractures. ;@) 7he correct answer is (. 7he !atient most likely has choanal atresia, which is the !resence of a con$enital membrane between the nose an" the !haryn*. )ince most newborns are obli$ate nose breathers, s!ells of cryin$ force mouth breathin$, im!rovin$ the ventilation. If unilateral, the atresia may not cause sym!toms. Intubation via the oro!haryn* will !rovi"e imme"iate relief an" sur$ery shoul" then be !erforme" to correct the atresia. An *4ray film 'choice A) woul" waste !recious time in the acute settin$ an" woul" not hel! in the "ia$nosis in any event. An electrocar"io$ram 'choice ) woul" be of no utility since this is not a car"iac issue. An arterial bloo" $as 'choice C) woul" in"icate hy!o*emia an" stren$then the case for intubation. Atro!ine 'choice %) is use" tem!orarily to increase the heart rate in sinus bra"ycar"ia an" com!lete heart block.

;I) 7he correct answer is %. Osteoi" osteoma usually affects !atients between the a$es of / an" ;? years. 7his lesion can occur in almost any bone, althou$h it is most common in the lower e*tremities. Osteoi" osteoma also may occur in the !osterior elements of the s!ine. &atients usually !resent with !ain, which is characteristically worse at ni$ht an" relieve" with as!irin. )ince as!irin is no lon$er commonly use" in chil"ren, the !ain is now mana$e" with na!ro*en or ibu!rofen. 7he !rocess may cause $rowth "isturbances, such as le$ len$th "iscre!ancy or bowin$ of an e*tremity, or scoliosis. 6a"io$ra!hs usually show a sclerotic lesion an" sometimes a locali#e" lytic "efect or ni"us 'P: cm in "iameter) within the sclerotic bone. 7he ni"us is best "emonstrate" with C7. 7he "ifferential "ia$nosis most often inclu"es ro"ieAs abscess or stress fracture. 2istolo$ically, the lesion consists of a ni"us of immature, woven, trabecular bone an" osteoblasts surroun"e" by "ense sclerotic bone. 7reatment usually involves sur$ical e*cision. +e"ical mana$ement with salicylates has been "escribe" an" is es!ecially useful with lesions that are "ifficult or unsafe to reach 'e.$., a lesion at the inner as!ect of the acetabulum). Newer techniques for sur$ical e*cision inclu"e !ercutaneous e*cision with C7 $ui"ance an" o!en sur$ical e*cision with ra"ionucli"e $ui"ance. &ain relief after sur$ery is usually imme"iate. ro"ieAs abscess 'choice A) is a chronic locali#e" bone abscess. )ubacute cases may !resent with fever, !ain, an" !eriosteal elevation, whereas chronic cases are often afebrile an" !resent with lon$4stan"in$ "ull !ain. 7he most common site of involvement is the "istal !art of the tibia. 7he lesion is ty!ically sin$le an" locate" near the meta!hysis. About D/= of !atients are youn$er than ;/. 6a"io$ra!hs show a ra"iolucent, irre$ularly sha!e" area with no a"<acent osteoblastic reaction. )ur$ical "ebri"ement an" culture4"irecte" antibiotics are often curative. (win$ sarcoma 'choice ) is a mali$nant tumor of bone arisin$ in me"ullary tissue. It most often occurs in cylin"rical bones. &rominent sym!toms inclu"e !ain, fever, an" leukocytosis. 6a"io$ra!hs have a ty!ical Honion skin a!!earance.H Osteosarcoma 'choice C) usually !resents with a !ainful mass. 6a"io$ra!hy reveals both bone "estruction an" !ro"uction, as well as !eriosteal elevation. )tress fractures 'choice () may $o un"etecte" in stan"ar" ra"io$ra!hs, but a bone scan will show increase" u!take. 0>) 7he correct answer is C. Neurofibromatosis is a multisystem $enetic "isor"er. 7he features of this con"ition are more than si* cafV4au4lait s!ots, two or more neurofibromas, a*illary frecklin$, o!tic $liomas, iris hamartomas ',isch no"ules), an" osseous lesions. 7here is almost always a first4"e$ree relative with neurofibromatosis. Con$enital hy!othyroi"ism 'choice A) is associate" with !oor fee"in$, an enlar$e" fontanelle, an enlar$e" ton$ue, an" an umbilical hernia in the neonatal !erio". It woul" not $o un"etecte" until @ years of a$e. +arfan syn"rome 'choice ) is a connective tissue "isor"er characteri#e" by lon$ fin$ers, hy!ermobile <oints, sublu*ation of the lenses, !ectus carinatum, an" aortic aneurysms. Osteo$enesis im!erfecta 'choice %) is a rare connective tissue "isease characteri#e" by

recurrent fractures, blue sclera, thin skin, an" hy!ere*tensibility of li$aments. 7uberous sclerosis 'choice () is associate" with facial an$iofibromas 'a"enoma sebaceum), retinal hamartomas, sei#ures, an" mental retar"ation. )ei#ures are the most common !resentin$ sym!tom. It is an autosomal4"ominant inherite" "isease. 0:) 7he correct answer is (. 7he !atient "escribe" here has cystic fibrosis. 7his inherite" "isease !rimarily affects the $astrointestinal an" res!iratory systems. Cystic fibrosis can !resent in early infancy with meconium ileusG in the first year of life with steatorrhea an" failure to thriveG or in ol"er chil"hoo" with recurrent !ulmonary infections. 7he !ancreatic secretions are characteristically very thick an" ten" to occlu"e the !ancreatic "uct system. )teatorrhea has "evelo!e" in this infant because of !ancreatic "uct ins!issation of secretions an" the resultant "ecrease in "elivery of !ancreatic en#ymes to the small bowel. Achlorhy"ria 'choice A) is not a feature of cystic fibrosis, but it can com!licate $astric "isor"ers inclu"in$ !ernicious anemia. acterial over$rowth 'choice ) is seen in con"ition with !oor intestinal motility, an" can cause vitamin "eficiencies an" malabsor!tion secon"ary to "e$ra"ation of biliary an" !ancreatic secretions. Althou$h meconium can cause ileus in newborns with cystic fibrosis, this !rocess usually involves small intestine rather than colon 'choice C). .astric hy!ersecretion 'choice %) is not a feature of cystic fibrosis, but it can be seen in !atients with $astrin4secretin$ !ancreatic tumors '3ollin$er4(llison syn"rome). 0;) (*!lanation: 7he correct answer is %. No one thinks this ki" is $oin$ to blee" to "eath, or fears a ma<or urolo$ic in<ury. 2owever, microhematuria after trivial trauma in chil"ren may be a si$n of a con$enital anomaly that makes the urinary tract unusually vulnerable. 7he warnin$ shoul" be hee"e", an" an anomaly sou$ht. 7he first, noninvasive test shoul" be the sono$ram. C7 of the ab"omen an" !elvis 'choice A) woul" be overreactin$. No ma<or intraab"ominal or !elvic in<uries are sus!ecte". 6eassurance, on the other han" 'choice ), woul" miss the si$nificance of this laboratory fin"in$. )erial hemo$lobins 'choice C) misses the si$nificance of the situation. We are not concerne" with the ma$nitu"e of the blee"in$ 'it is microhematuria, not $ross hematuria). 6etro$ra"e stu"ies 'choice () woul" be too invasive to be use" as the first test. )ono$ram will !robably $ive the "ia$nosis if there is a !roblem, an" intravenous !yelo$ram coul" be a""e" if nee"e". 00) A. 7his !atient has ell !alsy, a !ostinfectious aller$ic or immune "emyelinatin$ facial neuritis. (!stein4 arr virus is the !rece"in$ infection in a!!ro*imately ;>= of cases. @/= of !atients have their sym!toms resolve on their own over a !erio" of several weeks. :>= retain mil" facial weakness an" /= have !ermanent severe facial weakness.

7hera!eutic intervention shoul" inclu"e "aily an" nocturnal eye lubricants to !rotect the cornea from "ryin$. .rou! A )tre!tococcus'choice ), 2I1 'choice C), an" influen#a 'choice %) are not associate" with ell !alsy. 2owever, her!es sim!le* virus, ,yme "isease cause" by orrelia bur$"orferi an" mum!s have been associate" with ell !alsy. +easles 'choice () is associate" with subacute sclerosin$ !anence!halitis, a chronic ence!halitis of the central nervous system manifeste" by !ro$ressively bi#arre behavior an" "ecline in co$nitive function. 0?) : 7he correct answer is (. 7his infant has acute con$estive heart failure 'C2-). C2- may be "efine" as ina"equate contractile heart function for the s!ecific hemo"ynamic nee"s. Clinical manifestations inclu"e res!iratory "istress, tachycar"ia an"Bor hy!er"ynamic !recor"ium, an" car"iac enlar$ement evi"ence" by echocar"io$ram. 7he "ifferential "ia$nosis of C2- inclu"es left4to4ri$ht shunt 'A)%, 1)%, &%A, atrioventricular canal, or A1 fistula), left4si"e" obstruction lea"in$ to myocar"ial "ysfunction 'severe coarctation or A)), or intrinsic myocar"ial "ysfunction 'myocar"itis, car"iomyo!athy, or infarct "ue to anomalous coronary artery). In acute C2-, treatment shoul" be$in imme"iately. I1 furosemi"e is the "ru$ of choice because its onset is very ra!i". It can !rovi"e quick sym!tomatic relief an" im!rove res!iratory "istress. An$iotensin4convertin$ en#yme inhibitors 'choice A) are use" for the lon$4term mana$ement of !atients with C2 -. In a"ults, they have been !roven to re"uce mortality an" im!rove sym!tomatic relief. Althou$h "ata are not available for chil"ren, they have been re!orte" to im!rove hemo"ynamic functions. Corticosteroi"s 'choice ) may be useful in myocar"itis but have no role in C2- !er se. %i$o*in 'choice C) requires a lon$er time to take effect on C2-, an" it has only a mo"est influence on myocar"ial function. (!ine!hrine 'choice %) is not use" in the treatment of C2 -. 0/) 7he correct answer is C. Os$oo"4)chlatter "isease is a common cause of anterior knee !ain in the a"olescent. It is a stress reaction of the insertion of the !atella ten"on into the tibia tubercle. 7he con"ition !reviously was more common in a"olescent boys near the time of their $rowth s!urtG however, $irls are bein$ seen more frequently with this "ia$nosis owin$ to more active s!orts !artici!ation. 7here can be some fra$mentation of the ossicles of the "evelo!in$ tibia tubercle or soft tissue involvement of the !atella ten"on insertion without obvious bony chan$es. Often there is only a ten"onitis of the !atella ten"on insertion an" inflammation of the "ee! infra!atellar bursa. Clinically, a youn$ teena$er who is quite active in s!orts will com!lain of !ain an" swellin$ over the tibial tubercle. 7he con"ition may be unilateral or bilateral. On e*amination, there is an obviously swollen, ten"er, an" occasionally warm area over the tibia tubercle. 7he ra"io$ra!h can be normal or show fra$mentation of the area of !atella ten"on insertion. 7he natural

history of untreate" Os$oo"4)chlatter "isease is relatively beni$n, if allowe" to heal an" com!lications are avoi"e". )ince unusual stress at a time of ra!i" $rowth is occurrin$, limitin$ the offen"in$ s!orts activity is usually sufficient to relieve the sym!toms. 5neelin$ an" squattin$ shoul" be limite". &hysical thera!y is occasionally use" if there is unusual hamstrin$ ti$htness. Avascular necrosis of the hi! 'choice A) can occur with ,e$$4Calve4&erthes "isease an" sickle cell "isease. ,e$$4Calve4&erthes "isease 'choice ) is characteri#e" by avascular 'ischemic) necrosis of the femoral hea". &atients frequently !resent with lim!in$. When !ain is !resent, it is often mil", intermittent, an" referre" to the thi$h or knee. )e!tic arthritis 'choice %) refers to microbial invasion of the synovial s!ace. It !resents with <oint !ain, swellin$, an" fever. )li!!e" ca!ital femoral e!i!hysis 'choice () is a "isor"er of the !ro*imal femoral meta!hysis "is!lacin$ anterolaterally an" su!eriorly. 7he sli! can occur acutely from severe trauma, or $ra"ually from constant force on a weakene" e!i!hyseal !late. &atients usually !resent with com!laints of non4ra"iatin$, "ull, achin$ !ain in the thi$h, $roin, hi!, or occasionally the knee. 0C) C. -or !atients with !ersistent asthma sym!toms, inhale" corticosteroi" is the treatment of choice. Asthma is one of the most common res!iratory "iseases in chil"ren. It causes si$nificant morbi"ity an" mortality. If not well controlle", asthma can tremen"ously affect an in"ivi"ualAs "aily activities an" can be life4threatenin$. Inhale" corticosteroi"s "ecrease the "e$ree of inflammation in the airways an" re"uce airway hy!erres!onsiveness. Inhale" corticosteroi" shoul" be use" on a re$ular "aily basis re$ar"less of sym!toms of the "ay. ("ucation for the !atient an" the !atientAs family is e*tremely im!ortant so that a!!ro!riate treatment can be a"ministere" at home. Anticholiner$ic a$ents 'choice A), such as i!ratro!ium bromi"e, can be use" as an alternative to broncho"ilators if !atients e*!erience si$nificant si"e effects from broncho"ilators. Cromolyn so"ium 'choice ) an" ne"ocromil so"ium 'choice () are noncorticosteroi" anti4inflammatory a$ents that can be use" as an alternative to inhale" corticosteroi"s in mil" !ersistent asthma. 7hey act as mast cell stabili#ers. ,on$4term broncho"ilators 'choice %) are not anti4inflammatory a$ents an" shoul" not be em!loye" as a substitute for inhale" corticosteroi"s. 7hey are use" to im!rove sym!toms. 0D) 7he correct answer is %. 6enovascular "isease is the most frequent cause of hy!ertension in youn$ chil"ren. Ailments such as !olycystic ki"ney "isease, con$enital vascular "isease, tumors an" infections can all lea" to hy!ertension, an" a urolo$ic evaluation is im!erative. Chronic lun$ "isease 'choice A) woul" not elevate the bloo" !ressure. Associate" sym!toms woul" inclu"e "ys!nea, cyanosis, an" sym!toms of ri$ht heart failure. Coarctation of the aorta 'choice ) can cause hy!ertension, but is a less common cause of the "isease in this a$e $rou!. loo" !ressure measurement in all the e*tremities woul" be hel!ful in the "ia$nosis. Con$enital heart "isease 'choice C) such as an atrial se!tal "efect or ventricular se!tal "efect is rare, an" woul" be associate" with "ys!nea, cyanosis,

murmurs an" $eneral ill health. 7heo!hylline to*icity 'choice () woul" be associate" with <itteriness, nausea or tachycar"ia. 0@) 7he correct answer is A. 7he treatment of choice for stre!tococcal !haryn$itis is oral !enicillin 1 for :> "ays. 2owever, the chil" in this clinical vi$nette is leavin$ town for a summer cam! in ; "ays. 7here is no way to monitor his com!liance to the re$imen if he is not with the family. Also, he has ha" a !roblem finishin$ his me"icine in the !ast. A better re$imen for him this time mi$ht be a sin$le "ose of an a!!ro!riate antibiotic a$ent. Amon$ the above three choices, only ben#athine !enicillin . is the a!!ro!riate treatment for stre!tococcal !haryn$itis. en#athine !enicillin . can be $iven as a sin$le "ose intramuscularly. It is a lon$4actin$ antibiotic an" can com!lete the treatment. Intramuscular ceftria*one 'choice ) "oes not effectively era"icate stre!tococcal infection in the throat. Intramuscular !rocaine !enicillin 'choice C) is a short4actin$ !enicillin. One "ose of it "oes not era"icate stre!tococcal !haryn$itis. Oral treatment for stre!tococcal !haryn$itis requires a total re$imen of :> "ays. oth erythromycin 'choice %) an" !enicillin 1 'choice () are a!!ro!riate "ru$s. ut a$ain, a sin$le intramuscular antibiotic is a better treatment for this !atient. 0I) 7he correct answer is %. A +allory4Weiss tear occurs in the lower eso!ha$us after forceful or !rotracte" vomitin$. In most cases, the blee"in$ is !ainless an" resolves once the insultin$ inci"ent 'i.e., vomitin$) resolves. (so!ha$eal varices 'choice A) cause bri$ht re" blee"in$ in vomit, which is the most common !resentin$ sym!tom of !ortal hy!ertension. Other si$ns of !ortal hy!ertension 'e.$., s!lenome$aly, he!atome$aly, ascites) shoul" be !resent. (so!ha$itis 'choice ) in chil"ren is most often associate" with $astroeso!ha$eal reflu* "isease. 7he !atient will usually have a com!laint of chest an" ab"ominal !ain, with a history of frequent re$ur$itation. .astritis 'choice C) in chil"ren is usually associate" with ab"ominal !ain. It rarely is associate" with a fever an" sometimes causes vomitin$ with bri$ht re" bloo". &e!tic ulcer "isease 'choice () has sym!toms of mainly nocturnal ab"ominal !ain. 1omitin$ is !resent in only :>= of cases, an" only /= have hematemesis. ?>) 7he correct answer is A. 7he !atient !resents with a !ro$ressive, symmetric motor weakness as well as autonomic "ysfunction. 7here are no sensory "eficits or mental status chan$es. In this settin$ an" with a history of a recent infection, the "ia$nosis of .uillain4 arrV is most likely. Fsually this is self4limite" an" only su!!ortive thera!y is nee"e". &aralysis of res!iratory muscles may occur, requirin$ mechanical ventilation. Occasionally, steroi"s may be nee"e". +yasthenia $ravis 'choice ) woul" !resent with !ro$ressive weakness

as a result of anta$onism at the acetylcholine rece!tors. ("ro!honium effectively increases the concentration of acetylcholine at the neuromuscular <unction an" !roves the "ia$nosis if sym!toms transiently im!rove. &olymyositis 'choice C) is characteri#e" by !rofoun" inflammatory involvement of skeletal muscle. When accom!anie" by cutaneous manifestations, it is terme" "ermatomyositis. 7he characteristic !ro$ressive symmetric !ro*imal muscle weakness an" atro!hy are thou$ht to be cause" by a chronic inflammation of the muscles. 7ransverse myelitis 'choice %) is seen in neurolo$ic "iseases such as multi!le sclerosis. It may !resent acutely with si$ns an" sym!toms referable to a !articular s!inal cor" level. )!astic !ara!aresis, loss of sensation below the level of cor" involvement, an" incontinence are observe". 1iral ence!halitis 'choice () is an inflammatory !rocess involvin$ the brain. It usually !resents with altere" mentation an" sei#ures. With the e*ce!tion of her!es sim!le* ence!halitis, which is treate" with antivirals, the treatment is su!!ortive. ?:) 7he correct answer is A. 7he boy is this clinical vi$nette most likely has +arfan syn"rome, which is an autosomal4"ominant "isor"er of the connective tissues. 7he inci"ence is estimate" to be at least : in :>,>>> !eo!le in the F. ). , with one fourth of all cases occurrin$ as new mutations. 7he $enetic "efect results in abnormal synthesis an" secretion of fibrillin, which is an essential com!onent in connective tissues. &atients with +arfan syn"rome often have involvement in the skeletal, ocular, an" car"iovascular systems. +ost of the morbi"ity an" mortality with this "isease are relate" to the car"iovascular manifestationsG !atients "evelo! aortic root "ilatation. 7his may be associate" with aortic insufficiency an" aortic arch "issection. 7hey may also have mitral valve !rola!se with si$nificant hemo"ynamic consequences. Aortic root, aortic valve, an" mitral valve re!lacement may be nee"e" "urin$ the lifetime of the !atient. &atients with +arfan syn"rome may frequently com!lain of chest !ainG these !atients must be viewe" as havin$ aortic "issection until !roven otherwise. In this !o!ulation, a s!ontaneous !neumothora* is another !otential cause of chest !ain. No "ata e*ist for recommen"in$ beta4blockers to all !atients with +arfan syn"rome, however, beta4blockers clearly slow the !ro$ression of aortic root "ilatation in those with mil" aortic root "ilatation. (*ercise shoul" be limite" to aerobic activities. &atients must avoi" contact s!orts that cause acceleration4 "eceleration in<ury an" isometric maneuvers that ten" to increase central bloo" !ressure. Aerobic activity is encoura$e", but those with establishe" aortic root "ilatation shoul" have a"equate heart rate control with beta4blockers. Aortic stenosis 'choice ) is not associate" with +arfan syn"rome. +arfan syn"rome can cause aortic re$ur$itation instea". Aortic stenosis, in the conte*t of $enetic "iseases, is associate" with WilliamAs syn"rome, which also manifests as mental retar"ation an" characteristic facies, as well as !ulmonary artery stenosis. icus!i" aortic valve 'choice C) is not associate" with +arfan syn"rome. icus!i" aortic valve can lea" to aortic valvular stenosis. %e*trocar"ia 'choice %) refers to the a!e* of the heart !ointin$ towar" the ri$ht. It is associate" with 5arta$ener syn"rome. &ulmonary stenosis 'choice () can be an isolate" con$enital heart "isease or can be associate" with NoonanAs syn"rome.

?;) %. Osteomyelitis is a !yo$enic infection of the bone. 7he !atho$enesis of the "isease is similar to se!tic arthritis, with the ori$in of infection occurrin$ from hemato$enous s!rea", "irect e*tension of a local infection, or "irect inoculation of bone either from trauma 'e. $. , !uncture woun" or o!en fracture) or sur$ical mani!ulation. In chil"ren, the most frequent !resentation is acute hemato$enous s!rea". 7he most common location of osteomyelitis is the meta!hysis of the "istal femur an" !ro*imal tibia. 7he most !revalent !atho$ens are the same as those seen in se!tic arthritis. )ta!hylococcus aureus is the most common !atho$en, with $rou! A beta4hemolytic stre!tococci a "istant secon". Neonates are at risk for $rou! beta4hemolytic stre!tococci. 2aemo!hilus influen#ae may occur in infants an" youn$ chil"ren, but it is not seen as frequently as in se!tic arthritis. In a""ition, chil"ren with 2. influen#ae osteomyelitis usually have fever an" concomitant <oint infection. &atients with !uncture woun"s of the foot are susce!tible to &seu"omonas aeru$inosa osteomyelitis. &atients with sickle cell "isease are at risk for infection by )almonella an" other $ram4ne$ative bacteria, an" !atients in the :@4 to ?@4month a$e ran$e are at increase" risk for acute recurrent )almonella osteomyelitis. )almonella osteomyelitis frequently involves multi!le sites an" creates !unche"4out "estructive lesions of the meta!hysis an" "ia!hysis. 2owever, even in !atients with sickle cell "isease, )ta!hylococcus aureus is still the most common !atho$en for osteomyelitis. +ost !atients with osteomyelitis will !resent with a chief com!laint of fever an" bone !ain. 7he !ain is usually severe, constant, an" a$$ravate" by motion. 7he ol"er the chil", the more e*quisite the !oint ten"erness, because the bone has a thicker meta!hyseal corte* with a "ense fibrous !eriosteum. ,ocali#e" swellin$, warmth, an" erythema are si$ns seen late in the infection, as the !eriosteum becomes more involve". Neonates can !resent with va$ue sym!toms, consistin$ only of irritability an" !oor fee"in$, or can show si$ns of fulminant se!sis. 7he !eri!heral white bloo" cell count may be normal, or elevate" with a left shift. 7he erythrocyte se"imentation rate '()6) is usually elevate", an" bloo" cultures are !ositive in a!!ro*imately C>= cases. one cultures taken either sur$ically or by nee"le as!iration result in a culture yiel" of @>=. &lain ra"io$ra!hs may be normal for u! to ; weeks from the onset of illness, an" the earliest si$ns on !lain films are soft tissue swellin$ an" "is!lacement of muscle !lane. ony chan$es be$in to a!!ear by D4:> "ays, startin$ with a ha#y a!!earance of the meta!hysis followe" by irre$ular areas of trabecular necrosis an" absor!tion. (ventually, sub!eriosteal new bone formation occurs as the infection s!rea"s throu$h the corte*. A bone scan usually "ia$noses osteomyelitis as early as ;?4?@ hours from onset. 7reatment shoul" be$in with em!iric !arenteral antibiotics. 7he selection of antibiotic shoul" inclu"e covera$e of )ta!hylococcus aureus, as well as other or$anismsG the a$ent can !robably be base" on the !atientAs a$e an" history of illness. )ur$ical "ebri"ement may be necessary if !us is !resent on nee"le as!iration, or if evi"ence of either <oint involvement or abscess is !resent. ?0) A. Acute laryn$otracheobronchitis 'viral crou!) usually affects chil"ren youn$er than 0 years an" has sym!toms of a barkin$ cou$h an" ins!iratory stri"or. 7here is usually no

fever, an" often there are !rece"in$ u!!er res!iratory tract sym!toms. As!iration of forei$n bo"y 'choice ) is seen in youn$ chil"ren. 7his "isor"er is characteri#e" by acute onset of cou$hin$, chokin$, an" whee#in$ if a com!lete obstruction occurs. Often, "roolin$ an" ins!iratory stri"or are note" in a !artial obstruction. (!i$lottitis 'choice C) is rarely seen now, since the 2aemo!hilus influen#ae ty!e vaccine has been wi"ely $iven. It is characteri#e" by su""en onset of fever, "roolin$, muffle" voice, cyanosis, an" soft stri"or. Chil"ren will sit in a tri!o" !osition to $ive them the best airway !ossible. It is a true me"ical emer$ency. ,aryn$omalacia 'choice %), or un"er"evelo!ment of the su!ra$lottic area, is a con$enital "isor"er. 7here is !ersistent stri"or that is first note" early in infancy. )ub$lottic stenosis 'choice () often is con$enital, or results from re!eate" en"otracheal intubation. 7he !resentation is often asym!tomatic but can resemble that of airway obstruction. ??) 7he correct answer is C. &hysical chan$es occur ra!i"ly "urin$ a"olescence. Neuroen"ocrine chan$es alon$ the hy!othalamic4!ituitary a*is alter $ona"otro!in function an" concentrations of steroi" hormones. 6e!ro"uctive $rowth in a"olescents is $enerally classifie" by the 7anner sta$es, which !rovi"e a ran$e of a$es at which normal chan$es may occur. )ta$e I 'choice A) occurs in chil"ren a$e" >4:/. 7his is the sta$e of !rea"olescent breast "evelo!ment with no !ubic hair. )ta$e II 'choice ) is notable for breast bu""in$ or thelarche an" occurs in chil"ren a$e" @4:/. A small amount of !ubic hair near the labia is note" an" a $rowth s!urt often follows sta$e II. )ta$e I1 'choice %) is seen in a$es :>4 :D. -urther breast an" ni!!le enlar$ement with some contour se!aration of the areola is note". 7here is more !ubic hair than in )ta$e III, with a"ult quality, but not "istribution. )ta$e 1 'choice () is seen in a$es :;4:@ an" is characteri#e" by com!lete breast enlar$ement with no contour se!aration of the areola. &ubic hair is of a"ult quality an" "istribution. ?/) : 7he correct answer is C. 7o answer this question correctly, one nee"s to un"erstan" that $rou! )tre!tococcus '. )) infections in neonates have two "istinct !atterns: early4onset an" late4onset. (arly4 onset . ) infections often involve the res!iratory tract, causin$ . ) !neumonia 'choice %). It ty!ically ha!!ens in the first week of life. ,ate4onset . ) infections often affect the CN), causin$ mostly menin$itis. It most commonly ha!!ens between weeks ; an" ? of life. )ince the infection occurre" at 0 weeks of life in this case, the infant most likely has menin$itis. In fact, the culture that $rew . ) was taken from the s!inal flui". +ost bacterial infections in the neonatal !erio" are cause" by either . ) or $ram4ne$ative ro"s, es!ecially (scherichia coli. . ) infections cause si$nificant morbi"ity an" mortality in neonates. All !re$nant women shoul" have screenin$ cultures for . ) at ;@ weeksA $estational a$e. If the !re$nant woman is coloni#e" with . ), she shoul" be $iven antibiotics for chemo!ro!hyla*is. A""itional risk factors inclu"e !reterm birth,

!rolon$e" ru!ture of membranes, maternal fever "urin$ labor, an" a !revious history of another chil" havin$ . ) infection. 7hese women shoul" also be $iven chemo!ro!hyla*is to re"uce the risk of . ) infection in the neonate. (n"ocar"itis 'choice A), $astroenteritis 'choice ), an" !yelone!hritis 'choice () are uncommon manifestations of . ) infections in neonates. ?C) 7he correct answer is . 7he most useful test for the I4month4ol" infant in this clinical vi$nette is fecal fat quantification. -at malabsor!tion is likely, with !oor wei$ht $ain an" frequent malo"orous stools. A "etaile" family history an" !hysical e*amination are also useful in evaluation of fat malabsor!tion syn"rome. In a stool sam!le collecte" over 04? "ays, fat e*cretion shoul" not e*cee" :/= of "ietary fat in an infant, or :>= in an ol"er chil". When a malabsor!tion syn"rome is confirme", a sweat chlori"e test is in"icate" to evaluate for the !ossibility of cystic fibrosis. Ab"ominal ra"io$ra!hy 'choice A) !lays no role when a fat malabsor!tion syn"rome is sus!ecte". )erum albumin 'choice C) is abnormal when there is malabsor!tion or wastin$ of !rotein. It will not be hel!ful in this case of fat malabsor!tion. Infection with Clostri"ium "ifficile can cause !ersistent "iarrhea. It is most commonly associate" with antibiotic use. A stool culture for C. "ifficile to*in 'choice %) is use" to evaluate for infection. In this case, C. "ifficile colitis is unlikely. )tool smear for leukocytes an" eosino!hils 'choice () is useful when an inflammatory or aller$ic "isor"er of the bowel is sus!ecte". ?D) %. 7hrowin$ a ball overhea" is most consistent with the motor "evelo!ment of a ;?4 month4ol" chil". As the cerebral corte* "evelo!s, the chil"As han"s an" arms are more able to !erform more skillful an" "elicate motor functions. uil"in$ a tower 'choice A) is a $oo" way to assess a chil"As motor function. A chil" shoul" be able to buil" a tower of two cubes by :? months of a$e. y ;? months, he or she shoul" be able to buil" a tower of at least C cubes. A ;?4month4ol" chil" will not usually be able to co!y a circle 'choice ). 7his ability emer$es aroun" 0C months of a$e. Imitative scribblin$ 'choice C) a!!ears aroun" :C monthsG s!ontaneous scribblin$ a!!ears aroun" :@ months. A chil" shoul" be able to walk backwar" 'choice () by :@ months. Climbin$ stairs or $oin$ "own stairs with su!!ort are more consistently seen at ;? months. 7wo other milestones that are consistent with ;? months of a$e are <um!in$ u! an" kickin$ a ball forwar". ?@) 7he correct answer is C. 7he boy in this clinical vi$nette has lon$ 847 syn"rome ',87)). About />= of cases are familial: 6omano4War" syn"rome has autosomal4"ominant transmissionG Nervell4 ,an$e4Nielsen syn"rome has autosomal4recessive transmission. ,87) occurs in all races an" ethnic $rou!s. 7he !rinci!al sym!toms are synco!e an" su""en "eath from torsa"es "e !ointes '7%&). +ost often, 7%& is self4terminatin$ an" causes a synco!al e!iso"e

from which the !atient quickly recovers. Car"iac arrest occurs if the 7%& is more !ersistent, an" su""en "eath results if the rhythm "oes not return to normal s!ontaneously an" the !atient is not resuscitate". )ynco!e, cause" by 7%&, is the !rimary sym!tom in inherite" ,87). &atients may have one to hun"re"s of e!iso"es. 7he sym!toms commonly occur within the first few years of life in !atients with Nervell4,an$e4Nielsen syn"rome, an" the mortality rate with this form is hi$her than in !atients with 6omano4 War". In 6omano4War" syn"rome, the me"ian a$es at sym!tom onset an" su""en "eath are in the !re4 to mi"4teena$e years. Of interest, at least one thir" of $ene carriers never "evelo! sym!toms, lea" com!letely normal lifestyles, an" have normal life s!ans. A!!ro*imately one thir" has <ust one to a few synco!al s!ells as chil"ren, then none thereafter. )ynco!e an" su""en "eath occur most often "urin$ e*ercise or intense emotion, with an im!ortant minority occurrin$ "urin$ slee!. (vents are uncommon while !atients are awake an" at rest an" without a!!arent !rovocation. 7he !re"ominant feature on an (C. is 87 !rolon$ation. 7he 87c avera$es >. ?I secon"s. 7he $ol"4stan"ar" thera!y for ,87) remains beta4blocker a"ministration, which is effective in @>= to I>= of !atients, with a si$nificant re"uction in the rate of su""en "eath. Asym!tomatic chil"ren an" youn$ !ersons shoul" be treate" !ro!hylactically with beta4blockers on "ia$nosis. 7he im!lantable car"iac "efibrillator 'IC%) is bein$ use" with increasin$ frequency, es!ecially in a!!arently hi$h4risk !atients, such as those e*!eriencin$ 7%& while on beta4blockers an" those who have ha" car"iac arrest. ?I) 7he correct answer is A. 7he clinical history !rovi"e" by the !arents is consistent with H$rowin$H !ains. Althou$h such !ains are most likely unrelate" to $rowth, they "o affect chil"ren between 0 an" :> years. .rowin$ !ains are most commonly bilateral, involve the lower le$ an" knees, manifest with !ain "urin$ rest 'usually at be"time), an" are relieve" by massa$in$ or rubbin$. Chil"ren awaken the ne*t mornin$ feelin$ fine. &hysical activity is not im!aire". ,imb !ains !ro"uce" by or$anic "isease will usually be unilateral 'e*ce!t for rheumatoi" arthritis) or associate" with !hysical si$ns 'swellin$, warmth, etc.). 7he chil" with !hysical in<uries or "isease cannot bear to have the affecte" area touche". .rowin$ !ains often have a familial !re"is!osition. Nuvenile rheumatoi" arthritis 'choice ) is one of the most common connective tissue "isor"ers of chil"hoo". It affects lar$e <oints 'P / <oints in the oli$oarticular form, O / in the !olyarticular form). Affecte" <oints become warm an" swollen. Antinuclear antibo"ies are often !resent. Os$oo"4)chlatter "isease 'osteonecrosis of the tibial tuberosity) 'choice C) affects chil"ren between ? an" :; years of a$e. It is "ue to osteonecrosis 'ase!tic necrosis) of the tibial tuberosity. 7he !atient lim!s an" e*!eriences !ain "urin$ !hysical activities. 7he con"ition is self4limitin$, but activities such as <um!in$, football !layin$, an" runnin$ shoul" be "iscontinue" for ;40 months. Osteoi" osteoma 'choice %) is a small beni$n tumor of the bone that frequently affects the tibia. 7he tumor is usually unilateral an" manifests with nocturnal !ain, which awakens the chil" an" is !rom!tly relieve" by as!irin or N)AI%s. Osteosarcoma 'choice () is a mali$nant bone tumor affectin$ chil"ren an" a"olescents. Fnilateral bone !ain in the se$ment involve" 'usually !ro*imal tibia or "istal femur) or !atholo$ic fracture is the usual mo"e of !resentation. )tress fractures 'choice -) affect small bones that normally

have a thin cortical bone. When sub<ecte" to re!eate" mechanical stress, such as marchin$, skiin$, ballet "ancin$, etc., the bone accumulates microfractures that eventually result in chronic !ain an" swellin$. +etatarsal bones are the most frequently affecte". />) (*!lanation: 7he correct answer is C. 7he !atient has stri"or on e*amination. 7his is an ins!iratory obstruction that is sensitive to airflow chan$es. In chil"ren, the most common cause of stri"or is laryn$omalacia. ronchoalveolar carcinoma 'choice A) may cause stri"or in an a"ult with an e*tensive smokin$ history but shoul" not be seen in someone this youn$. -orei$n ob<ect obstruction 'choice ) shoul" be acute in onset an" cause severe "istress. &neumonia woul" be notable for a fever an" !ro"uctive s!utum. )tri"or woul" not be !resent 'choice %). 7uberculosis 'choice () woul" be associate" with systemic sym!toms of fever, wei$ht loss, sweats, an" hemo!tysis.

USMLE Step 2 Practice Test Block 12 :ame; +nstr#ctions; Ans'er the *#estions $elo' to the $est of yo#r a$ility. =hen yo# finish the test% click the 2heck $#tton at the $ottom to )ie' the res#lts.

1.A )omiting infant is $ro#ght to the emergency room. The $loo" 'ork res#lts re)eal a normal $loo" co#nt% $#t a hyponatremic% hypochloremic% meta$olic alkalosis. =hich of the follo'ing 'o#l" $e consistent 'ith these fin"ingsa6@ia$etes mellit#s $62ystic fi$rosis c6Ethanol poisoning "6+ron ingestion e6+soniaDi" ingestion :ormal La$s 2.A !year!ol" $oy "e)elops a hea"ache% co#gh% myalgia an" a fe)er. /e has $een a healthy chil" 'ith all imm#niDations #p to "ate. /e is gi)en a "econgestant an" an aspirin for his symptoms 'ith some relief. /o'e)er% 4 "ays later% he is $ro#ght $ack $y his parents $eca#se of persistent )omiting an" irrita$ility. >n physical e.amination% he is fo#n" to $e semicomatose% $ecoming com$ati)e on stim#lation. =hich of the follo'ing le)els sho#l" $e meas#re" to ai" in the "iagnosis of this patienta6 Ser#m ammonia le)el $6Ser#m $loo" #rea nitrogen le)el c6Ser#m calci#m le)el "6Ser#m opiate le)el e6 Ser#m so"i#m le)el :ormal La$s

&.A &!'eek!ol" $oy presents to the physician,s office 'ith a 1!'eek history of forcef#l% pro9ectile )omiting. /e has $een )omiting after almost e)ery fee"ing. The )omit#s contains mostly #n"igeste" form#la an" is non!$ilio#s. >n e.amination% his oral m#cosa is "ry% his anterior fontanel appears to $e "epresse"% an" his capillary refill is &!4 secon"s. An a$"ominal e.amination re)eals an oli)e!siDe" mass in the epigastri#m. =hich of the follo'ing electrolyte fin"ings 'ill most likely $e seena6/ypochloremic meta$olic aci"osis $6/ypochloremic meta$olic alkalosis c6:ormal electrolytes "6Aespiratory aci"osis 'ith meta$olic compensation e6Aespiratory alkalosis :ormal La$s 4.A !year!ol" $oy is $ro#ght to his pe"iatrician,s office after he falls from his $icycle an" strikes his hea" against the si"e'alk. There 'ere no 'itnesses to this inci"ent% 'hich occ#rre" 1 ho#rs ago. The chil" is other'ise healthy% #p!to!"ate on his imm#niDations% an" not taking any me"ications. >n physical e.amination% his )ital signs are sta$le. /e has a L 4 cm a$rasion on his forehea". /e is alert an" oriente" to "ate% place an" self. /is motor an" sensory e.aminations are normal an" refle.es are normal. =hich of the follo'ing constit#tes reasona$le managementa6A"mit o)ernight for o$ser)ation $6+nstr#ct parents to o$ser)e ne#rological stat#s for 24 ho#rs c6>$tain a hea" comp#teriDe" tomography scan "6>$tain a sk#ll .!ray film an" "ischarge if normal e6 >$tain a sk#ll .!ray film% o$ser)e for 24 ho#rs an" "ischarge :ormal La$s .A term male infant is fo#n" to $e cyanotic shortly after $irth an" re*#ires en"otracheal int#$ation. >n physical e.amination% his $loo" press#re is <17&4 mm /g 3e*#al in all fo#r e.tremities6% p#lse is 1147min% an" respirations are &27min. /is precor"i#m is "ynamic% has a gra"e +++ systolic m#rm#r% an" a single S2.

2hest ra"iography sho's a normal heart siDe an" increase" p#lmonary )asc#lar markings. An arterial $loo" gas on an 5i>2 of 144B sho's p/ (.&4G Pa2>2% 4( mm /gG Pa>2% 4< mm /g. =hich of the follo'ing "iagnoses is most consistent 'ith these fin"ingsa6Atrial septal "efect $6/ypoplastic left heart syn"rome c6Patent "#ct#s arterios#s "6Tetralogy of 5allot e6 Total anomalo#s p#lmonary )eno#s ret#rn :ormal La$s <.A <!month!ol" $oy is fo#n" to ha)e $ilateral metaphyseal fract#res of $oth pro.imal an" "istal en"s of the ti$ia. The mother says that her $oyfrien" takes care of the infant 'hile she is at 'ork. /er $oyfrien" e.plains the in9#ries as the res#lt of m#ltiple falls from $e". =hich of the follo'ing is the most likely "iagnosisa6Acci"ental tra#ma $6>steogenesis imperfecta c6>steopetrosis "6Physical a$#se e6 Aickets :ormal La$s (.A 14!year!ol" male fell 'hile ri"ing his scooter "o'n a steep hill. +n the Emergency @epartment% his in9#ries incl#"e" a fract#re" 'rist an" a lacerate" spleen% 'hich re*#ire" s#rgical remo)al. T'o years later he is "iagnose" 'ith $acterial pne#monia. =hich of the follo'ing $acterial agents is the most likely pathogen for this patient,s pne#moniaa6 Escherichia coli $6Mle$siella pne#moniae c6:eisseria meningiti"is "6Streptococc#s pne#moniae e6Staphylococc#s a#re#s :ormal La$s

1. A !year!ol" girl presents to the emergency "epartment 'ith ac#te onset of m#scle 'eakness. The mother carrie" the girl to the e.amination room an" states that the m#scle 'eakness starte" in her "a#ghter,s legs yester"ay. The 'eakness $ecame 'orse to"ay an" progresse" to the tr#nk an" $oth arms. She 'as also complaining "iffic#lty of $reathing earlier to"ay. She has $een other'ise healthy $#t ha" a common col" a$o#t 14 "ays ago. >n e.amination% her respirations are 2<7min an" shallo'. She has profo#n" m#scle 'eakness in her lo'er e.tremities% an" mo"erate 'eakness in her #pper e.tremities. /er "eep ten"on refle.es are a$sent. A l#m$ar p#nct#re is performe"% 'hich sho's increase" protein concentration $#t no pleocytosis. =hich of the follo'ing is the most likely "iagnosisa6Bot#lism $6@ermatomyositis c6C#illain!BarrT syn"rome "6Myasthenia gra)is e6To.ic ne#ropathy :ormal La$s 8.A f#ll!term infant is $ro#ght to the office on her <th "ay of life $eca#se her mother note" that she looke" 0yello'0. The mother states that the infant is strictly $reast!fe" an" has $een eating e)ery 2!& ho#rs. >n e.amination% she is note" to $e 9a#n"ice" o)er her tr#nk an" face. There is no scleral icter#s. She is other'ise healthy. Both the mother an" $a$y are Ah positi)e. =hich of the follo'ing is the most likely ca#se of this infant,s 9a#n"icea6Breast!fee"ing 9a#n"ice $6Cl#cose!<!phosphate "ehy"rogenase 3C<P@6 "eficiency c6/ypothyroi"ism "6Physiologic 9a#n"ice e6Ah incompati$ility :ormal La$s 14.A 4!year!ol" girl 'as "iagnose" of left!si"e otitis me"ia a$o#t 14 "ays ago an" 'as prescri$e" oral amo.icillin% 44 mg7kg7"ay for ( "ays. She has since "e)elope" $loo"y "iarrhea 'ith m#c#s% crampy a$"ominal pain% an" fe)er. >n physical e.amination% her temperat#re is &8.4 2 3142.8 56% p#lse is 117min% an" respirations are 1<7min. She has normal

$o'el so#n"s an" is "iff#sely ten"er to palpation. =hich of the follo'ing is the most appropriate initial step in "iagnosisa6Bari#m enema $6E)al#ation of stool for 2lostri"i#m "ifficile to.ins c6E)al#ation of stool for rota)ir#s "6Stool e.amination for o)a an" parasites e6Stool /emocc#lt test :ormal La$s 11. A 1 !year!ol" girl presents 'ith "iplopia after prolonge" rea"ing an" ptosis that 'orsens in the afternoon. >n e.amination% she is note" to ha)e $ilateral ptosis% impaire" e.traoc#lar m#scle mo)ements% facial 'eakness% an" generaliDe" hypotonia an" 'eakness increasing 'ith repetition. =hich of the follo'ing is the $est "iagnostic test for this "isor"era62T of the $rain $6Electromyography c6L#m$ar p#nct#re "6M#scle $iopsy e6:er)e con"#ction )elocity :ormal La$s 12. A neonate "e)elops se)ere cyanosis that $egins 'ithin min#tes of $irth. Bloo" "ra'n one ho#r after $irth sho's meta$olic aci"osis 'ith respiratory aci"osis. A chest .!ray film sho's a narro' $ase to the great )essels an" the heart resem$les an egg on its si"e. E2C is normal. =hich of the follo'ing is the most likely "iagnosisa6Aortic )al)e stenosis $62omplete atrio)entric#lar canal "efect c6Tetralogy of 5allot "6Transposition of the great arteries e6Un"er"e)elope" 3hypoplastic6 left )entricle syn"rome :ormal La$s 1&.A 1&!year!ol" $oy is seen for right knee pain that has persiste" for the past 2 months "espite the #se of o)er!the!co#nter analgesics. /is mother states that he has $een limping since he starte" to ha)e the pain. The pain is of insi"io#s onset% $#t its intensity sometimes s#""enly increases. >n physical e.amination% the $oy is o$eseG his 'eight is

higher than the 84th percentile. 5le.ion of the right hip ca#ses marke" o#t'ar" rotation an" repro"#ces the pain. +nternal rotation of the right hip is limite"% an" the right leg is slightly shortene" compare" 'ith the left leg. =hich of the follo'ing is the most likely "iagnosisa6R#)enile rhe#matoi" arthritis $6Legg!2al)e!Perthes "isease c6Septic arthritis "6Slippe" capital femoral epiphysis e6 To.ic syno)itis :ormal La$s 14. A & year ol" male has s#""en onset of "ry co#gh 'ith a small amo#nt of $right re" $loo" pro"#ce" 'hen he co#ghs. /e has ha" no fe)ers% r#nny nose% or )omiting. +n the Emergency @epartment a chest .!ray sho's hypere.pansion of the right l#ng an" clear l#ng fiel"s. =hich of the follo'ing is the most likely etiology of this chil",s symptomsa6 Bacterial pne#monia $62ystic fi$rosis c65oreign $o"y aspiration "6P#lmonary arterio)eno#s malformation e6T#$erc#losis :ormal La$s 1 . A 1(!year!ol" girl is fo#n" #nconscio#s in a $athroom after ingesting a large amo#nt of #nkno'n sleeping pills. >n arri)ing at the emergency "epartment% she is somnolentG her temperat#re is &(.2 2 381.8 56% $loo" press#re is 1247(4 mm /g% p#lse is <17min% an" respirations are 127min. She respon"s to painf#l stim#li an" to hearing her name lo#"ly $y opening her eyes $riefly. /o'e)er% she ret#rns to the somnolent state imme"iately after'ar". =hich of the follo'ing is the most appropriate initial inter)entiona6+? a"ministration of caffeine $6+? a"ministration of "roperi"ol c6+? a"ministration of 44 mL of normal saline "6+pecac!in"#ce" emesis

e6Castric la)age :ormal La$s 1<.A &!'eek!ol" African American $oy is $ro#ght to the Emergency @epartment $eca#se of a generaliDe" seiD#re 2 ho#rs ago. The infant is highly irrita$le 'ith incessant high pitche" crying. The infant,s 'eight is 2. kg 32 4 gm $elo' $irth 'eight6% $loo" press#re is (4 744 mm /g% p#lse is 14 7min an" respirations are 47min. La$oratory res#lts sho'; Bloo" gl#cose 124 mg7"L Urea nitrogen 4 mg7"L Ser#m so"i#m 1(4 mE*7L Ser#m calci#m 1. mg7"L Ser#m magnesi#m 1. mg7"L =hich of the follo'ing is the most likely ca#se of this infants seiD#re a6 /ypocalcemia $6/ypoglycemia c6/ypomagnesemia "6+ntracranial hemorrhage e6Meningitis :ormal La$s 1(. A patient has the s#""en onset of fretf#lness an" pain. /e c#rls #p 'ith his legs "ra'n in to his a$"omen. >)er the ne.t fe' ho#rs he contin#e" to ha)e episo"es of pain an" cries 'ith tears% $#t $et'een these times he acts normally. The patient,s mother fears something is terri$ly 'rong an" $rings him to the hospital. /is past me"ical history 'as #nremarka$le. The pre)io#s 'eek he ha" ha" a col" 'ith a r#nny nose. /is stools ha" $een normal that "ay. >n e.amination he is *#iet an" his a$"omen is s#rprisingly soft an" normal. +n the emergency room there is a semi!soft stool 'ith some $loo" mi.e" 'ith m#c#s. This classic presentation of int#ss#sception is most likely to occ#r in 'hich of the follo'ing age gro#psa6 Birth to 4 'eeks of age $6 < to 12 months of age c6& to years of age "6Early a"olescence e6Late a"olescence :ormal La$s

11. A 12!year!ol" $oy is $ro#ght to the clinic $eca#se of a se)eral!month history of strange $eha)ior. Accor"ing to his parents% the $oy occasionally 'ill start staring an" not respon". /e 'ill also ha)e tears in his eyes. These episo"es last se)eral secon"s an" he then ret#rns to his $aseline. /e has not s#staine" any hea" tra#ma an" is on no me"ications. =hich of the follo'ing "r#gs is the most appropriate treatmenta6 @iaDepam $6@iphenhy"ramine c6Ethos#.imi"e "6Pheno$ar$ital e6Phenytoin :ormal La$s 18. A !month!ol" girl is $ro#ght to the office $y her mother% 'ho states that the girl ha" an episo"e follo'ing fee"ing "#ring 'hich she $egan to $reathe "eeply% $ecame $l#e% an" then lost conscio#sness. The mother states that she picke" her #p an" hel" her% an" the infant regaine" her #s#al color an" $ecame alert. Physical e.amination re)eals a harsh systolic m#rm#r. =hich of the follo'ing is the most likely "iagnosisa6Aortic stenosis $62oarctation of the aorta c6Patent "#ct#s arterios#s "6Tetralogy of 5allot e6 ?entric#lar septal "efect :ormal La$s 24. A ne'$orn $a$y is note" to ha)e a$normal facies 'ith lo'!set earsG a small rece"ing 9a'G an" 'i"ely separate" eyes. At &4 ho#rs of age% the $a$y "e)elops m#ltiple m#scle spasms. Ser#m st#"ies are nota$le for calci#m of 4. mg7"L. =hich of the follo'ing is the most likely "iagnosisa6Br#ton,s agammaglo$#linemia $62ommon )aria$le imm#no"eficiency c6@iCeorge syn"rome "6Selecti)e +gA "eficiency

e6Transient hypogammaglo$#linemia of infancy :ormal La$s 21. An other'ise healthy !year!ol" $oy is $ro#ght to the emergency "epartment of a small hospital $eca#se of a simple &!cm laceration in his forehea". The patient is crying an" frightene". The practitioner "eci"es to perform conscio#s se"ation $efore s#t#ring the laceration. S#pport personnel an" e*#ipment are a)aila$le for monitoring the patient,s )ital stat#s an" carrying o#t res#scitation meas#res if nee"e". =hich of the follo'ing is the most appropriate pharmacologic agent to achie)e a safe le)el of conscio#s se"ation in this sit#ationa6>ral or rectal mi"aDolam or "iaDepam $62oncomitant opioi" an" $enDo"iaDepine a"ministration c6+ntra)eno#s propofol "6+ntra)eno#s ketamine e62oncomitant analgesic!se"ati)e agents an" m#scle rela.ants :ormal La$s 22.An 1!year!ol" $oy is $ro#ght to the clinic $y his mother% 'ho states that he has $een complaining of pain in $oth knees. The mother also states that he ha" a rash on his leg & 'eeks ago. She sai" it 'as there for almost 2 'eeks an" then 'ent a'ay. She "escri$es the rash as re""ish an" circ#lar% 'ith a small clear area in the center. She sai" the rash 'as not itchy or painf#l. The chil" has also complaine" of hea"aches an" m#scle aches o)er the past se)eral 'eeks. >n *#estioning% the mother states that they 'ere )acationing in =isconsin a$o#t 1 month ago an" the $oy 'as hiking in the 'oo"s 'hen he 'as $itten $y a tick. =hich of the follo'ing is the most likely "iagnosisa6Ba$esiosis $62olora"o tick fe)er c6Lyme "isease "6Aocky Mo#ntain spotte" fe)er e6 T#laremia :ormal La$s 2&. A 44 year ol" 'oman "eli)ers a &124 g 3< l$ 14 oD6 ne'$orn male. /er pregnancy 'as normal e.cept that she note" "ecrease" fetal mo)ement compare" to her pre)io#s pregnancies. She "ecline" an

amniocentesis offere" $y her o$stetrician. Physical e.amination of the ne'$orn re)eals an infant 'ith facial feat#res s#ggesti)e of @o'n Syn"rome. The infant then has $ilio#s )omiting. An .!ray film sho'ing the ki"neys% #reters% an" $la""er 3MUB6 is performe"% 'hich sho's a 0"o#$le $#$$le0 sign. =hich of the follo'ing is the most likely ca#se of the a$"ominal signs an" symptomsa6@#o"enal atresia $6/irschspr#ng @isease c6Malrotation "6Meconi#m ile#s e6Pyloric stenosis :ormal La$s 24.The parents of a 14!year!ol" girl 'ith ne'ly "iagnose"% generaliDe" tonic!clonic epilepsy come to the physician seeking a")ice regar"ing 'hat they sho#l" "o 'hen the chil" has a seiD#re. =hich of the follo'ing s#ggestions is appropriatea62all an am$#lance imme"iately as soon as seiD#re $egins $6P#t something in the chil",s mo#th at the onset of seiD#re c6Try to place the chil" on her si"e "#ring the seiD#re "6Try to restrain the chil" "#ring the seiD#re e6 @o not allo' the chil" to ret#rn to her acti)ities after reco)ery :ormal La$s 2 .A 14!year!ol" $oy has a long history of rec#rrent infections. These ha)e incl#"e" pne#monia% s#pp#rati)e lympha"enitis% persistent rhinitis% "ermatitis% "iarrhea% an" perianal a$scesses. +n)ol)e" organisms ha)e incl#"e" Staphylococc#s a#re#s% Serratia% Escherichia coli% an" Pse#"omonas. Biopsy of skin an" lymph no"es ha)e "emonstrate" gran#lomato#s lesions% e)en tho#gh the only species isolate" 'ere those note" a$o)e. +mm#noglo$#lin le)els are higher than normal. =hich of the follo'ing fin"ings 'o#l" $e most helpf#l in esta$lishing the "iagnosisa6A$sent B cells an" normal n#m$ers of T cells $6@eficient nitro$l#e tetraDoli#m "ye re"#ction in ne#trophils c6/igh ser#m +gM an" )ery lo' ser#m +gC "6?ery lo' 2@11 on the s#rface of 'hite $loo" cells e6?ery lo' ser#m calci#m le)els :ormal La$s

2<.A < month ol" male is $ro#ght in to the Emergency Aoom $y his mother 'ho states that 'hen she picke" him #p from the $a$y!sitter he 'as not acting right. The $a$y!sitter state" that he 'as sleeping more an" 'as f#ssy. >n e.amination the $a$y is st#poro#s. /is temperat#re is &(.1 2 388.8 56% p#lse is 1447min% an" respirations are &<7min. A 4 cm ecchymosis is note" on his right cheek. The remain"er of the physical e.amination is #nremarka$le. The physician s#spects possi$le physical a$#se. /e or"ers a 2T scan of the hea"% skeletal s#r)ey% chemistry panel an" complete $loo" co#nt. =hich of the follo'ing "iagnostic tests sho#l" also $e or"ere"a6 Ammonia le)el $62oag#lation st#"ies c6Lipi" panel "6Thyroi" st#"ies e6Urine electrolytes :ormal La$s 2(. An infant is "eli)ere" at f#ll term $y a spontaneo#s )aginal "eli)ery to a 28!year!ol" primigra)i"a. At "eli)ery% the infant is note" to ha)e s#$costal retractions an" cyanosis "espite goo" respiratory effort. The a$"omen is scaphoi". >n $ag an" mask )entilation% a#sc#ltation of the l#ngs re)eals "ecrease" $reath so#n"s on the left% 'ith heart so#n"s lo#"er on the right. =hich of the follo'ing is the most likely "iagnosisa6@e.trocar"ia 'ith sit#s in)ers#s $6@iaphragmatic hernia c6Pne#monia "6P#lmonary hypoplasia e6 Spontaneo#s pne#mothora. :ormal La$s 21. A 12!year!ol" girl comes to the physician for an ann#al e.amination. She has $een in goo" health for the past year an" has no complaints. She $egan ha)ing menses this year an"% after a fe' irreg#lar cycles% is no' ha)ing a monthly perio". Past me"ical history is significant for m#ltiple episo"es of otitis me"ia as a chil". Past s#rgical history is #nremarka$le. She takes no me"ications an" has no kno'n "r#g allergies. Physical e.amination is #nremarka$le. +f not c#rrently

imm#ne% 'hich of the follo'ing imm#niDations sho#l" this patient most likely recei)ea6/epatitis B )ir#s imm#niDation $6/+? imm#niDation c6Rapanese encephalitis )ir#s imm#niDation "6Aa$ies )ir#s imm#niDation e6Salmonella typhi imm#niDation :ormal La$s 28. Physical e.amination of an infant "eli)ere" to a 42!year!ol"% gra)i"a &% para 2% 'oman% is remarka$le for slight hypotonia an" a poor Moro refle.. 5#rther e.amination re)eals #pslanting palpe$ral fiss#res% epicanthal fol"s% e.cess n#chal skin% an enlarge" tong#e% clino"actyly of the fifth fingers% an" a single trans)erse palmar crease. =hich of the follo'ing is the most likely "iagnosisa6 @o'n syn"rome $6E"'ar"s syn"rome c65etal alcohol syn"rome "6Marfan syn"rome e6T#rner syn"rome :ormal La$s &4. A <!year!ol" chil"% attempting to pet a neigh$or,s "omestic "og 'hile the "og is eating% is $itten in the han". The "og has $een )accinate" reg#larly. =hich of the follo'ing steps are nee"e" for ra$ies prophyla.is in this casea6The chil" sho#l" recei)e ra$ies imm#noglo$#lin only $6The chil" sho#l" recei)e ra$ies imm#noglo$#lin pl#s )accine c6The chil" sho#l" recei)e ra$ies )accine only "6The "og sho#l" $e kille" an" the $rain e.amine" for signs of ra$ies e6 The "og sho#l" $e o$ser)e" for $eha)ioral changes s#ggesti)e of ra$ies :ormal La$s &1.A 1!month!ol" $a$y $oy has $loo"y "iarrhea. :o infectio#s agent is i"entifie"% $#t the $a$y is fo#n" to $e profo#n"ly throm$ocytopenic. The $a$y is also note" to ha)e a skin rash% an" a "ermatologist

"iagnoses ecDema. By three months of age% the $a$y $egins to "e)elop rec#rrent respiratory infections. +f this chil" s#r)i)es #ntil a"olescence% he is at partic#larly high risk of "e)eloping 'hich of the follo'inga62ongesti)e heart fail#re $62rohn "isease c6Lymphoma "6Ahe#matoi" arthritis e6 =ilms t#mor :ormal La$s &2.A 8!month!ol" girl has ha" one serio#s infection after another since a$o#t & months of age% incl#"ing thr#sh% pne#monias% an" "iarrhea. The $a$y is small for age. An ol"er $rother "ie" at age 2 of pne#monia. +mm#nologic e)al#ation "emonstrates lymphopenia an" )ery lo' gamma glo$#lin le)els. Both T an" B cell n#m$ers are )ery lo'. Aa"iologic st#"ies "emonstrate 0fraye"0 long $ones% a$normally thick gro'th arrest lines% an" "ysplasia of the costochon"ral 9#nctions. =hich of the follo'ing is the most likely "iagnosisa6A"enosine "eaminase "eficiency $6Br#ton agammaglo$#linemia c6@iCeorge syn"rome "6:eDelof syn"rome e6F!linke" se)ere com$ine" imm#no"eficiency :ormal La$s &&.A 14!year!ol" girl presents to the emergency room for se)ere lo'er mi"!a$"ominal pain that has $een increasing o)er the past 12 ho#rs. She "escri$es it as crampy an" sharp. /er last menstr#al perio" 'as a$o#t 2 'eeks ago. +t 'as reg#lar 'ith no pain an" laste" 4 to "ays. She has ha" menstr#al perio"s for nearly 2 years% an" o)er the past < months she has notice" some cramping pain the first "ay or t'o of her menses. She "enies se.#al acti)ity an" has not ha" any )aginal "ischarge. /er $o'el mo)ements ha)e $een normal an" she reports no #rinary fre*#ency% #rgency% or $#rning 'ith #rination. =hich of the follo'ing is the most likely "iagnosisa6 @ysmenorrhea $6Ectopic pregnancy c6>)arian cyst "6Pel)ic inflammatory "isease

e6MittelschmertD :ormal La$s &4.A 8!month!ol" $oy is taken to the emergency room $eca#se of high fe)er. Breath so#n"s are "iminishe" in the l#ngs% an" a chest .!ray film sho's lo$ar pne#monia. Pro$a$le streptococcal pne#monia is "emonstrate" in Cram,s stain of sp#t#m an" then later confirme" $y c#lt#re. The chil" respon"s to anti$iotic therapy. A "etaile" history is taken "#ring the a"mission% 'hich re)eals that this is the thir" episo"e of pne#monia in this yo#ng chil"G the t'o pre)io#s episo"es occ#rre" at < an" (172 months of age. >ne of the mother,s $rothers ha" "ie" of infection at age 8. +mm#noglo$#lin st#"ies "emonstrate the follo'ing; +gC 14 mg7"L Jnormal (2&!1<1 mg7"LK +gA <4 mg7"L Jnormal 11!4<& mg7"LK +gM 24 mg7"L Jnormal 41!2(1 mg7"LK St#"ies of the lymphocyte pop#lation "emonstrate normal n#m$ers of T cells an" marke"ly "ecrease" B cells. =hich of the follo'ing is the most likely "iagnosisa6Br#ton agammaglo$#linemia $62ommon )aria$le imm#no"eficiency c6@iCeorge syn"rome "6Transient hypogammaglo$#linemia of infancy e6 =iskott!Al"rich syn"rome :ormal La$s & .A 1<!year!ol" $oy is $ro#ght into the emergency "epartment $y his frien"s. /e is semicomatose% 'ith a p#lse of <47min an" respirations of <!17min. /is p#pils are constricte". =hich of the follo'ing 'ill most likely $e re)eale" on a #rine to.icology screena6Amphetamine into.ication $62ocaine into.ication c6Ethanol into.ication "6>piate into.ication e6 Tricyclic anti"epressant into.ication :ormal La$s

&<. A <!month!ol" pre)io#sly health chil" is $ro#ght to the "octor,s office $eca#se of s#""en onset of lethargy% constipation% generaliDe" 'eakness an" poor fee"ing. /e has $een meeting all "e)elopment milestones an" his imm#niDation sche"#le is #p to "ate. >n f#rther *#estioning% his parents mention that his "iet 'as recently a")ance" to a homema"e form#la of e)aporate" milk an" honey. >n physical e.amination% he has sta$le )ital signs% clear l#ngs% normal skin t#rgor an" f#ll fontanelles. =hich of the follo'ing is the most likely e.planation for the patient,s presentationa6Bot#lism $6/irschspr#ng "isease c6/ypernatremia "6/yponatremia e6/ypothyroi"ism :ormal La$s &(.A 1(!year!ol" $oy comes to me"ical attention $eca#se of rec#rrent sin#sitis an" pne#monia% an" persistent 'atery "iarrhea "#e to Ciar"ia lam$lia. /is parents an" a sister are in e.cellent health. Physical e.amination re)eals enlarge" lymph no"es in cer)ical% a.illary an" ing#inal regions. A lymph no"e $iopsy sho's hyperplastic follicles 'ith an a$sence of plasma cells. La$oratory in)estigations sho'; /ematocrit.......................44B Le#kocyte co#nt...............8%1447mm& :e#trophils..................... B Lymphocytes..................&4B Monocytes..................... B 2@4 T!cell co#nt............1444 cells7mm& Proteins% ser#m...............<.2 g7"L Al$#min........................ .4 g7"L Clo$#lin........................1.2 g7"L A""itional st#"ies "emonstrate se)erely "epresse" le)els of ser#m +gC% 'ith slightly $elo'!normal le)els of +gM an" +gA. =hich of the follo'ing is the most likely "iagnosisa6 Ac*#ire" imm#no"eficiency syn"rome 3A+@S6 $62ommon )aria$le imm#no"eficiency c6/o"gkin "isease "6+solate" +gA "eficiency

e6F!linke" agammaglo$#linemia of Br#ton :ormal La$s &1.An 1!year!ol" $oy presents 'ith a 2!"ay history of rash. The rash starte" on the hea" an" sprea" "o'n'ar" to his tr#nk an" e.tremities. /e also complains of a fe)er% co#gh% an" a r#nny nose for the past "ays. >n physical e.amination% his temperat#re is &1.2 2 3144.( 56% $loo" press#re is 117 < mm /g% p#lse is (<7min% an" respirations are 1<7min. There is a small% irreg#lar re" spot 'ith a central gray color on his $#ccal m#cosa. The rash on his $o"y is erythemato#s an" mac#lopap#lar in *#ality. =hich of the follo'ing is the most likely "iagnosisa6 Erythema infectios#m $6/an"!foot!mo#th "isease c6Measles "6Aoseola infant#m e6 A#$ella :ormal La$s

&8.A 2 'eek post mat#re $a$y is $orn an" imme"iately e.hi$its se)ere respiratory "istress. Pre)io#sly% green!tinge" meconi#m 'as note" in the amniotic fl#i". =hich of the follo'ing is the most appropriate ne.t stepa6Emergency tracheostomy $6+nt#$ation 'ith mechanical )entilation c62hest .!ray film "6>.ygen s#pplementation $y face mask e6S#ction the mo#th an" nasopharyn. :ormal La$s 44.A fet#s is "eli)ere" at 44 'eeks, gestation. @#ring la$or% the fetal heart monitor sho's late "ecelerations an" loss of short! an" long!term )aria$ility. The mem$ranes are r#pt#re" to e.pe"ite the "eli)ery. The fl#i" is note" to contain meconi#m. The infant is "eli)ere" 4 min#tes later. At "eli)ery% the infant appears to $e cyanotic an" limp. /e has poor tone an" "eep refle.es. Moro,s refle. is a$sent. Ten ho#rs later% he

e.periences a seiD#re. =hich of the follo'ing $est e.plains this infant,s perinatal co#rsea6 Encephalopathy from asphy.ia $6+n$orn error of meta$olism c6Aespiratory "istressc "6S#$arachnoi" hemorrhage e6 =er"nig!/offman "isease :ormal La$s 41. A 418< g 38 l$ 4 oD6 infant is "eli)ere" )ia )aginal "eli)ery to a &1! year!ol" mother 'ith gestational "ia$etes. The "eli)ery 'as complicate" $y sho#l"er "ystocia. /e is taken to the ne'$orn n#rsery 'here his initial plasma gl#cose le)el is 24 mg7"L. The initial sp#n hematocrit is < B. =hich of the follo'ing congenital anomalies is this $a$y most likely to ha)ea6Aniri"ia $62left palate c6Macroglossia "6>mphalocele e6 Single palmar crease f6Small left colon :ormal La$s 42.A 2!'eek!ol" infant is $ro#ght to the office for a ne'$orn )isit. /is mother states that the he has $een a )ery slo' eater an" that he is constipate"% not ha)ing ha" a $o'el mo)ement in & "ays. >n physical e.amination% the infant has poor m#scle tone% an enlarge" tong#e% an #m$ilical hernia% an enlarge" anterior fontanelle% an" hypothermia. /e also looks slightly 9a#n"ice"% 'ith slightly "ry skin an" $rittle hair. =hich of the follo'ing is the most likely "iagnosisa6Beck'ith!=ie"emann Syn"rome $62ongenital hypothyroi"ism c6/#rler syn"rome "6Trisomy 21 e6T#rner syn"rome :ormal La$s

4&.A 14!year!ol" presents 'ith 2 months history of hea)y menstr#al!like $lee"ing. Menarche occ#rre" < months ago an" this first menses consiste" of spotting for &!4 "ays 'itho#t cramps. S#$se*#ent perio"s 'ere light in flo' $#t laste" < to 1 "ays. =hich of the follo'ing is the most likely ca#se of her $lee"inga6?on =ille$ran" "isease $6>)arian t#mor c6Thyroi" "isease "6@ysf#nctional #terine $lee"ing e6Pregnancy :ormal La$s 44.A 4!year!ol" $oy presents 'ith !"ay history of fe)er an" increase" irrita$ility. /is temperat#re is 44. 2 2 3144. & 56% $loo" press#re is 817<1 mm /g% p#lse is 1127min% an" respirations are 247min. >n physical e.amination% he is note" to ha)e $ilateral cer)ical lympha"enopathy% cracke" lips% stra'$erry tong#e% an" $ilateral con9#ncti)al in9ection. /is palms an" soles are erythemato#s. There is a polymorpho#s mac#lar rash generaliDe" on his $o"y. =hich of the follo'ing is the most appropriate pharmacotherapya6Amo.icillin $6Aspirin an" corticosteroi" c6Aspirin an" +? imm#ne glo$#lin "62orticosteroi" e6+? imm#ne glo$#lin :ormal La$s 4 . An 1!month!ol" pre)io#sly preterm infant 'ith $ronchop#lmonary "ysplasia presents to the emergency "epartment 'ith lethargy. /is reg#lar me"ications incl#"e f#rosemi"e an" spironolactone. /is temperat#re is &(.4 2 388.& 56% $loo" press#re is <17&2 mm /g% p#lse is 1147min% an" respirations are 147min. /e has poor skin t#rgor an" "ry m#co#s mem$ranes. La$oratory chemistry e)al#ation re)eals; so"i#m% 1&1 mE*7LG potassi#m% &.4 mE*7LG chlori"e% 14 mE*7LG $icar$onate% &1 mE*7LG $loo" #rea nitrogen% &< mg7"LG an" creatinine% 4.4 mg7"L. An arterial $loo" gas sho's p/% (. 2G Pa2>2% 48 mm /gG an" Pa>2% 82

mm /g. =hich of the follo'ing is the most likely e.planation for these fin"ingsa6 Bartter syn"rome $6Primary hyperal"osteronism c6Primary respiratory aci"osis 'ith meta$olic compensation "6Pse#"ohyperal"osteronism e6?ol#me "epletion :ormal La$s 4<. A 4!year!ol" girl is $ro#ght to the physician $y her mother 'ho is concerne" $eca#se her chil" has a )aginal "ischarge. Starting 2 "ays ago% the chil" $egan scratching her )#l)a an" complaining of $#rning 'ith #rination. The chil" is other'ise healthy an" has ne)er ha" a similar pro$lem. E.amination re)eals normal str#ct#ral anatomy for a 4!year!ol" girl. There is no e)i"ence of atrophy. There is an inflammatory erythema on the me"ial aspects of the la$ia ma9ora an" e.coriations. There is a m#co#s "ischarge 'ith a fe' flecks of $loo" intermi.e". =hich of the follo'ing is the most likely ca#se of a )aginal "ischarge in this patienta6Lichen sclerosis $6Pel)ic inflammatory "isease c6Sarcoma $otyroi"es 3em$ryonal rha$"omyosarcoma6 "6Se.#al a$#se e6?aginal foreign $o"y :ormal La$s 4(.A !month!ol" $oy is $ro#ght to the office for a mass in his left groin area. The infant is on the e.amination ta$le% *#ietly s#cking on his pacifier. >n e.amination% there is a fl#i"!fille" sac that "oes not reach the ing#inal ring an" transill#minates 'ell. =hich of the follo'ing is the most likely "iagnosisa6 /ematoma $6/y"rocele c6+ng#inal hernia "6Testic#lar torsion e6 Testic#lar t#mor :ormal La$s

41. A 8!year!ol" girl is $ro#ght to the clinic 'ith complaints of fatig#e% a$"ominal pain an" lo' gra"e fe)er. 5o#r "ays ago her mother notice" a re" rash on the tops of her "a#ghter,s feet% 'hich has no' sprea" to her thighs an" $#ttocks. The rash 'as initially small re" "ots% $#t has no' $ecome patches. /er "a#ghter s#$se*#ently "e)elope" peri#m$ilical% cramping% a$"ominal pain. Ae)ie' of systems is positi)e only for a respiratory )iral illness three 'eeks ago. /er temperat#re is &(.( 2 388.1 56% p#lse is 8<7min% an" respirations are 117min. A$"ominal e.amination is remarka$le for ten"erness to palpation near the #m$ilic#s% $#t the a$"omen is other'ise soft% 'ith no re$o#n" or g#ar"ing% an" no organomegaly. Aaise"% palpa$le p#rp#ric lesions are present on the $#ttocks an" thighs. The ankles are ten"er an" e"emato#s $ilaterally. 2omplete $loo" co#nt re)eals a le#kocyte co#nt of 8%4447mm&. /er hemoglo$in 12. < g7"L% an" platelets are &2 %4447mm&. Ser#m electrolytes are normal. =hich of the follo'ing is her #rinalysis most likely to re)eala6 Ele)ate" le)els of copper $6Cl#cos#ria c6Ae" $loo" cells "6=hite $loo" cell casts e6Ueast :ormal La$s 48.A &!year!ol" $oy is $ro#ght $y his father to the Emergency @epartment 'ith fe)er% hea"ache an" neck pain that "e)elope" o)er the past se)eral ho#rs. The father states he is not the $irth father% an" that he an" his 'ife a"opte" the $oy at 11 months of age after his $irth mother a$an"one" him. Physical e.amination re)eals a lethargic male 'ith a temperat#re of &8.( 2 314&. 56. There is photopho$ia% an" mil"ly in9ecte" con9#ncti)a are appreciate". P#pils are e*#al an" reacti)e an" f#n"#scopic e.amination is #nremarka$le. The patient has neck stiffness 'ith a positi)e Mernig,s sign. A complete $loo" co#nt re)eals a le#kocyte co#nt of 24%4447mm& 'ith <4 segmente" ne#trophils an" 2 $an"s. A l#m$ar p#nct#re is performe" that re)eals ele)ate" 2S5 press#re% "ecrease" gl#cose% an" ele)ate" protein. A Cram,s stain sho's gram!negati)e pleomorphic ro"s. There is no gro'th on $loo" agar. Cro'th on chocolate agar re)eals 'hite colonies. =hich of the follo'ing is the most likely pathogen-

a6/aemophil#s "#creyi $6/aemophil#s infl#enDae type $ c6:eisseria meningiti"is "6Listeria monocytogenes e6Streptococc#s pne#moniae :ormal La$s 4. A 1(!year!ol" girl presents to the office 'ith a !"ay history of a malo"oro#s )aginal "ischarge. She is se.#ally acti)e an" #ses con"oms for se.#al interco#rse. >n e.amination% a thin% 'hite "ischarge is seen. A 0fishy0 o"or is pro"#ce" 'hen M>/ is a""e" to the "ischarge. The )aginal fl#i" has a p/ of . =hich of the follo'ing is the most likely fin"ing on a microscopic e.amination of the )aginal fl#i"a62l#e cells $6 Cram!negati)e "iplococci c6Lacto$acilli "6Pse#"ohyphae e6 Trichomona"s :ormal La$s

Note: Check your own answers before hittin$ the Check button below. When you click the Check button, a browser win"ow will a!!ear that contains a summary of your results. (*!lanations lock :; (*!lanations

:) (*!lanation: 7he correct answer is . Althou$h metabolic alkalosis is an uncommon !resentation for cystic fibrosis, it is the correct answer. &articularly in the summer time with e*cess sweatin$, infants with cystic fibrosis may !resent with "ehy"ration an" this electrolyte !attern. Althou$h "iabetes 'choice A) mi$ht !resent with a low serum so"ium, it is associate" with ketoaci"osis, not alkalosis. (thanol !oisonin$ 'choice C) !oses !articular !roblems with hy!o$lycemia, res!iratory "e!ression, an" metabolic aci"osis. Acute iron !oisonin$ 'choice %) may result in metabolic aci"osis, not alkalosis. -ever an" leukocytosis may be !resent an" coma may ensue. Also, acute iron in$estion may !resent with hemorrha$ic $astroenteritis. Isonia#i" in$estion 'choice () is associate" with metabolic aci"osis an" an elevate" anion $a!. ;) (*!lanation: 7he correct answer is A. 7he chil" is !resentin$ with sym!toms of 6eye syn"rome, which is an acute ence!halo!athy associate" with hi$h ammonia levels. It most commonly occurs in youn$ chil"ren after a viral illness. A"ministration of as!irin increases the risk of "evelo!in$ this "isor"er. 7he vomitin$ is characteristic. 6enal failure lea"in$ to uremia 'choice ) be characteri#e" by an increase in the urea nitro$en level. 2owever, the !resentation woul" be more chronic an" the !atientAs !resentation woul" be more in"icative of renal failure. 2y!ercalcemia 'choice C) can cause mental status chan$es an" mana$ement woul" inclu"e intravenous hy"ration an" furosemi"e. A$ain, !resentation woul" be more acute, an" a history of a mali$nancy, use of thia#i"es, milk alkali syn"rome, or the like, woul" be elicite". O!iate into*ication 'choice %) woul" cause !in!oint !u!ils, "e!resse" breathin$ an" obtun"ation. Combativeness shoul" not occur. 2y!onatremia or hy!ernatremia 'choice () can cause mental status chan$es. 7he !atientAs !resentation makes this less likely. A history of lun$ cancer, "iuretic use, !rimary !oly"i!sia, or "ehy"ration woul" make this "ia$nosis hi$her on the "ifferential "ia$nosis. 0) (*!lanation:

7he correct answer is . 7his baby most likely has !yloric stenosis. &yloric stenosis occurs in a!!ro*imately ::/>> births. +ale infants are more commonly affecte" than female infants are, an" the inci"ence is far $reater in full4term infants than in !reterm infants. Clinical manifestations inclu"e !ro<ectile non4bilious vomitin$ shortly after fee"in$ an" an olive4 si#e" mass !al!able in the e!i$astrium. )ym!toms ty!ically !resent in the ;n" or 0r" week of life. As the vomitin$ continues, hy"ro$en ions an" chlori"e ions be$in to "ecrease in the bo"y, causin$ hy!ochloremic metabolic alkalosis. In a""ition, the infant mi$ht also be hy!okalemic from re!eate" vomitin$. &lain ra"io$ra!hs may "emonstrate the absence of air "istal to the obstructe" !ylorus. In barium contrast stu"ies, a small amount of barium may !ass throu$h the hy!ertro!hie" !ylorus, causin$ the Hstrin$ si$n.H Fltrasoun" stu"ies are also useful in "emonstratin$ the hy!ertro!hie" !ylorus. 7he "ehy"ration an" electrolyte abnormalities shoul" be correcte". %efinite treatment is !yloromyotomy. ?) (*!lanation: 7he correct answer is . 7he !atient has no fin"in$s on history or e*amination. An intracranial blee" or severe brain in<ury is not likely. If !resent, sym!toms shoul" have become manifest within several hours of the in<ury. 7hus the !arents shoul" be a"vise" to monitor for somnolence, vomitin$, sei#ures an" severe hea"aches, an" to return if these occur. Overni$ht a"mission 'choice A) woul" be of little value since the !atient has been sym!tom4free for @ hours alrea"y. A hea" C7 'choice C) woul" be hel!ful in "etectin$ an acute blee" "ue to in<ury to bloo" vessels, or to the brain tissue itself. )uch trauma woul" likely be accom!anie" by somnolence an" mental status chan$es. Were such si$ns !resent, an emer$ent hea" C7 woul" be require". A hea" C7 woul" be the !referre" stu"y in cases of sym!tomatic hea" in<ury instea" of a skull *4ray film 'choice %), since it is more likely to show the in<ury. A skull *4ray film woul" not necessarily rule out the !atholo$y. Also, since the in<ury was @ hours a$o, a"mittin$ woul" not a"" much value. Observation at home is a"visable 'choice (). /) (*!lanation: 7he correct answer is (. 7otal anomalous !ulmonary venous return is characteri#e" by the !ulmonary veins formin$ a confluence behin" the left atrium, an" "rainin$ into the ri$ht atrium. Com!lete mi*in$ takes !lace in the ri$ht atrium, with a ri$ht4to4left shunt throu$h the foramen ovale to the left si"e of the heart. Often, no murmur is hear" on car"iac e*amination, althou$h a short systolic murmur is sometimes hear". (C. often reveals ri$ht atrial enlar$ement an" ri$ht ventricular hy!ertro!hy. 7he chest roent$eno$ram often shows a normal heart si#e with !ulmonary e"ema. If there is obstruction to !ulmonary venous return, as is almost always !resent with veins "rainin$ inferior to the "ia!hra$m, cyanosis can be very !rominent. %efinitive treatment is sur$ical anastomosis of the !ulmonary

vein to the left atrium. Atrial se!tal "efect 'A)%) 'choice A) is a hole in the se!tum between the ri$ht an" the left atria. It results in a left4to4ri$ht shunt an" causes ri$ht ventricular volume overloa" an" increase" !ulmonary bloo" flow. A!!ro*imately 0= to /= of chil"ren with con$enital heart "isease have an A)%, makin$ it the thir" most common con$enital heart "efect. 2y!o!lastic left heart syn"rome 'choice ) is characteri#e" by un"er"evelo!ment of the left ventricle an" the ascen"in$ aorta. 7y!ically, there is obstruction at the mitral valve, causin$ all !ulmonary venous bloo" to shunt throu$h either an A)% or a !atent "uctus arteriosus '&%A) into the ri$ht atrium. 7otal systemic bloo" flow is channelle" throu$h the "uctus arteriosus from the !ulmonary artery. As the "uctus closes, these infants !resent with shock because systemic bloo" flow is si$nificantly re"uce". &%A 'choice C) causes sym!toms of !ulmonary con$estion, "ys!nea, wi"ene" !ulse !ressure, an" boun"in$ arterial !ulsation because aortic bloo" flow is shunte" from left to ri$ht. 7etralo$y of -allot 'choice %) consists of four car"inal car"iac "efects: ':) !ulmonary stenosis '&)), ';) a lar$e ventricular se!tal "efect '1)%), '0) ri$ht ventricular hy!ertro!hy, an" '?) aorta overri"in$ the 1) %. 7he &) causes a harsh systolic murmur easily au"ible over the u!!er left sternal bor"er. 7here is a si$nificant ri$ht4to4left shunt because the lar$e 1)% allows unrestricte" flow from the ri$ht ventricle to the left ventricle, causin$ cyanosis.

C) (*!lanation: 7he correct answer is %. 7here are s!ecific ty!es of traumatic in<uries that are virtually !atho$nomonic of chil" abuse an" can not be e*!laine" by acci"ental trauma 'choice A). +eta!hyseal fractures are the most ty!ical 'althou$h not the most frequent) in<uries "ue to !hysical abuse. 7hey are !ro"uce" by !ullin$ an" twistin$ the limbs, which results in traction on the meta!hyses transmitte" by the <oint li$aments. Other skeletal in<uries that shoul" su$$est abuse inclu"e multi!le fractures of "ifferent a$es, an" fractures of !osterior ribs, sca!ula, s!inous !rocesses, an" sternum. 7he followin$ is a list of in<ury !atterns ty!ically associate" with chil" abuse: ruises on the buttocks, ab"ominal wall, $enitalia, back, an" "orsal han"s an" feet. Acci"ental bruises in the active to""ler are usually on bony !rominences. 2ea" in<uries: sub"ural hematoma, "iffuse a*onal in<ury, an" retinal !etechiae. Ab"ominal in<uries: intestinal ru!ture or hematoma occurs more frequently than in<uries to soli" or$ans from a har" blow. urns: ci$arette burns an" burns of the buttock area "ue to scal"in$ water. Osteo$enesis im!erfecta 'choice ) refers to a $rou! of hetero$eneous "isor"ers "ue to inherite" "eficiencies in synthesis of ty!e I colla$en. 7his con"ition results in skeletal fra$ility an" numerous fractures startin$ in utero or early life. Osteo!etrosis 'choice C) is a here"itary "isease cause" by "ysfunction in osteoclasts. 7he bone acquires a stone4like consistency but is also very brittle. -ractures, hy"roce!halus, an" anemia manifest soon after birth. On *4rays, absence of me"ullary cavity, sclerosis of bone, an" missha!en "istal en"s of lon$ bones are "ia$nostic features. 6ickets 'choice () results from "eficiency of vitamin %, which is infrequent in in"ustriali#e" countries. 7he most ty!ical manifestation of the "isease in to""lers is bowin$ of the le$s.

D) (*!lanation: 7he correct answer is %. 7his !atient un"erwent a s!lenectomy after his in<ury, an" thus is susce!tible to enca!sulate" or$anisms. ). !neumoniae is the most im!ortant !atho$en in as!lenic chil"ren. ,ess common causes of infection inclu"e 2emo!hilus influen#ae ty!e , (. coli, )ta!h. aureus, an" $ram4ne$ative bacilli such as )almonella, 5lebsiella an" &seu"omonas. As!lenic chil"ren are also at increase" risk for fatal malaria an" babesiosis. 7his chil" shoul" receive a !neumococcal vaccine. (. coli'choice A) is a $ram4ne$ative ro" an" is an unlikely cause of community4acquire" !neumonia. (. coli is !art of the normal flora of the human $astrointestinal tract. It can be seen as a cause of as!iration !neumonia, se!sis, neonatal menin$itis, or urinary infections. (. coli is also a frequent cause of several ty!es of bacterial "iarrhea, inclu"in$ enterohemorrha$ic, enteroto*i$enic an" (. coli O:/D:2D, which causes hemolytic4uremic syn"rome. 5lebsiella !neumoniae'choice ) is a $ram4ne$ative bacillus that is an im!ortant nosocomial !atho$en. It is more !revalent in nursin$ homes an" e*ten"e" care facilities than in the community, an" thus woul" be an unlikely cause of this !atientAs !neumonia. Neisseria menin$iti"is'choice C) is a $ram4ne$ative "i!lococcus with multi!le seroty!es that cause invasive "isease. It is a frequent cause of bacterial menin$itis an" se!ticemia. +any !eo!le are asym!tomatic carriers of this or$anism, since they are coloni#e" in their naso!haryn*. It is an enca!sulate" or$anism, so this !atient woul" be at $reater risk for this or$anism, but his hi$hest risk is that of )tre!. !neumoniae. )ta!h. aureus'choice () is a wi"ely !revalent $ram4!ositive coccus that causes lobar !neumonia most often in chil"ren un"er : year of a$e. %ue to his s!lenectomy, his hi$hest risk is still )tre!. !neumoniae. )ta!h. aureus also frequently causes skin infections such as im!eti$o, cellulitis, to*ic shock syn"rome an" scal"e" skin syn"rome. Osteomyelitis, en"ocar"itis, arthritis an" se!ticemia are also commonly cause" by this or$anism. @) (*!lanation: 7he correct answer is C. 7he $irl in this clinical vi$nette has classic .uillain4 arrV syn"rome, also known as acute inflammatory "emyelinatin$ !olyneuro!athy 'AI%&). It is an acute illness characteri#e" by ra!i" "emyelination of !eri!heral nerves. A symmetric ascen"in$ weakness !ro$resses ra!i"ly. In this case, the involvement of the res!iratory muscles is !otentially life threatenin$. )ensory "eficit, involvement of facial musculature, an" autonomic insufficiency may occasionally ha!!en. Weakness usually follows a nons!ecific viral infection by about D4:? "ays. ,umbar !uncture shows elevation of !rotein concentration in the absence of !leocytosis in the C) -. %ia$nosis is ma"e by nerve con"uction stu"ies, which will show "emyelination. 7reatment involves !lasma!heresis or $amma4$lobulin infusion. 6es!iratory failure must

be a$$ressively mana$e" with a"equate res!iratory su!!ort. Chil"ren with this syn"rome have a very $oo" !ro$nosisG full recovery is usually the rule. A few may be left with some resi"ual weakness. otulism 'choice A) results from in$estion of Clostri"ium botulism s!ores an" manifests as ileus, bilateral !tosis, a!nea, $enerali#e" hy!otonia, an" res!iratory failure. %ermatomyositis 'choice ) !resents with !ro$ressive !ro*imal muscle weakness, muscle ten"erness, an elevate" C5 level, an" a characteristic violaceous heliotro!ic rash. +yasthenia $ravis 'choice %) usually first !resents with !tosis, e*traocular muscle weakness, an" ra!i" muscle fati$ue. It is "ue to "ecrease" numbers of acetylcholine rece!tors in the neuromuscular <unction. 7o*ic neuro!athy 'choice (), cause" by a wi"e ran$e of chemicals such as to*ins an" heavy metals, can result in !eri!heral neuro!athy. It "oes not have the characteristic ascen"in$ muscle weakness an" the history of followin$ a viral illness as in .uillain4 arrV syn"rome. I) (*!lanation: 7he correct answer is A. reast4fee"in$ <aun"ice is common. reast4fe" infants have a hi$her inci"ence of increase" bilirubin than formula4fe" infants. It is relate" to "ecrease" intake an" increase" enterohe!atic circulation. 7he "ecrease" intake is usually relate" to failure to establish an a"equate milk su!!ly. .lucose4C4!hos!hate "ehy"ro$enase '.C&%) "eficiency 'choice ) is a re" cell en#yme "efect resultin$ in hemolysis an" the over!ro"uction of bilirubin. It is E4linke" an" usually occurs in those of +e"iterranean, African, or Asian "escent. .C&% shoul" be sus!ecte" in male infants with late onset of <aun"ice. 2y!othyroi"ism 'choice C) is a cause of !rolon$e" <aun"ice in infants. 2owever, the infant woul" be showin$ other si$ns of hy!othyroi"ism 'such as "ecrease" muscle tone, macro$lossia, an" hy!othermia) an" woul" not look healthy. &hysiolo$ic <aun"ice 'choice %) usually a!!ears after ;? hours of a$e, slowly increasin$ to a !eak between 04/ "ays of a$e. In this case, the infant is C "ays ol" an" <ust startin$ to $et <aun"ice"G thus, she is beyon" the ty!ical ran$e for !hysiolo$ic <aun"ice. 6h incom!atibility 'choice () is not in"icate" in this case, since both the mother an" infant are 6h !ositive. :>) (*!lanation: 7he correct answer is . 7he $irl in this clinical vi$nette has !seu"omembranous colitis, which is cause" by the to*ins !ro"uce" by Clostri"ium "ifficile. 7he colon is coloni#e" by C. "ifficile after era"ication of the normal microflora by a course of antibiotics. 1irtually all antibiotics, inclu"in$ metroni"a#ole an" vancomycin, have been im!licate" in the !atho$enesis of !seu"omembranous colitis. &atients usually !resent with watery or bloo"y "iarrhea, tenesmus, fever, an" ab"ominal !ain. )ym!toms ty!ically occur D4:> "ays after initiation of the antibioticG in rare cases, !seu"omembranous colitis can occur u! to C weeks after antibiotic initiation. 7o*ic me$acolon, resultin$ from !ro$ressive loss of muscular tone in the colon, is a rare but serious com!lication that coul" !ro$ress to

!erforation of the colon an" !eritonitis. On si$moi"osco!y or colonosco!y, the "isease is characteri#e" by an accumulation of an inflammatory e*u"ate, the H!seu"omembraneH over the mucosa. Assay for C. "ifficile to*ins 'enteroto*in A or cytoto*in ) is useful in "ia$nosin$ !seu"omembranous colitis. An alternative is to !erform stool cultures for C. "ifficile. Oral metroni"a#ole is the treatment of choice. An alternative treatment is oral vancomycin. Another interestin$ fact about C. "ifficile is that u! to D>= of infants are asym!tomatic carriers of C. "ifficileG therefore, these stu"ies are not useful in "ia$nosis. arium enema 'choice A) can ima$e $ross structural abnormalities of the colon but is useless in "ia$nosin$ !seu"omembranous colitis. (valuation of stool for rotavirus 'choice C) mi$ht not be the best test with the history of recent antibiotic use. 2owever, rotavirus is a common !atho$en, es!ecially "urin$ wintertime, causin$ viral $astroenteritis. 6otavirus infection usually causes fever, ab"ominal !ain, an" watery "iarrhea. (valuation of stool for ova an" !arasites 'choice %) is not a $oo" initial test for a syn"rome that is most consistent with C. "ifficile infection. 7he 2emoccult test 'choice () is useless because the "iarrhea is $rossly bloo"y. ::) (*!lanation: 7he correct answer is . 7his :/4year4ol" $irl has myasthenia $ravis, which is cause" by a "efect in the neuromuscular <unction transmission. 7he neuromuscular <unction "ysfunction is cause" by circulatin$ anti4acetylcholine 'ACh) rece!tor antibo"ies that result in a "ecrease" number of available ACh rece!tors. It can occur at any a$e an" is more common in females. +yasthenia $ravis is usually immune4me"iate"G however, a rare form of con$enital familial myasthenia $ravis also e*ists. +yasthenia is characteri#e" by fati$able weakness that im!roves after rest. It usually first affects the ocular muscles, with !tosis an" "i!lo!ia as early si$ns. %ys!ha$ia an" facial muscle weakness may occur. ,imb4$ir"le muscles may also be involve". +uscle weakness !ro$resses throu$hout the "ay an" im!roves with rest. 6es!iratory involvement may be life4 threatenin$. (lectromyo$ra!hy '(+.) shows a s!ecific "ecremental res!onse to re!etitive nerve stimulation in affecte" muscles an" is "ia$nostic of myasthenia $ravis. 7reatment involves anticholinesterase thera!y. )teroi"s, cytoto*ic "ru$s, !lasma!heresis, an" thymectomy may also be use" in some cases. +yasthenia $ravis is not associate" with s!ecific !atholo$y that is evi"ent in C7 of brain 'choice A), lumbar !uncture 'choice C), muscle bio!sy 'choice %), or nerve con"uction velocity stu"ies 'choice (). A test that may be hel!ful in "ia$nosin$ myasthenia $ravis is the e"ro!honium '7ensilon) test. ("ro!honium chlori"e is a short4actin$ anticholinesterase inhibitor that can briefly restore !ower to the weakene" muscles in myasthenia. Anti4ACh antibo"ies may also be !resent in the serum. :;)

(*!lanation: 7he correct answer is %. 7his is trans!osition of the $reat arteries, in which the aorta arises from the ri$ht ventricle an" the !ulmonary artery arises from the left ventricle. A!!ro*imately /= of con$enital car"iac anomalies have trans!osition of the $reat arteries. Affecte" babies !resent within minutes of birth with severe cyanosis an" metabolic aci"osis secon"ary to inability to o*y$enate tissues. 7he only e*chan$e of bloo" between !ulmonic an" circulatory systems is ty!ically occurrin$ throu$h a !atent "uctus arteriosus. 7he chest *4 ray chan$es illustrate" in the question stem are ty!ical, an" are "ue to su!er!osition of the $reat vessels 'rather than the normal si"e4to4si"e !osition). )ur$ical re!air is usually !erforme" within D to :> "ays of life. Aortic valve stenosis 'choice A) !ro"uces a lou" e<ection murmur with a !rominent systolic click hear" best at the u!!er ri$ht sternal bor"er. Com!lete atrioventricular canal "efect 'choice ) can also cause cyanosis at birth, but will show marke" (C. chan$es, sometimes inclu"in$ absent 8 waves. 7etralo$y of -allot 'choice C) may !resent at birth , with (C. chan$es showin$ ri$ht ventricular hy!ertro!hy an" ri$ht a*is "eviation. Chest *4ray films usually show a small heart with a concave main !ulmonary artery. Fn"er"evelo!e" left ventricle syn"rome 'choice () causes an abru!t onset of severe heart failure with loss of !eri!heral !ulses at ;40 "ays of life. :0) (*!lanation: 7he correct answer is %. )li!!e" ca!ital femoral e!i!hysis ')C-() is a relatively common hi! !roblem, usually seen in the a"olescent near the time of !eak $rowth velocity. It is characteri#e" by "is!lacement of the femoral e!i!hysis from the femoral neck throu$h the $rowth !late. 7he "is!lacement can be either su""en or $ra"ual. A sli!!e" e!i!hysis can later "evelo! in the o!!osite hi! in 0C= of cases. )C-( occurs more commonly in youn$er chil"ren, an" boys are affecte" more than $irls. +echanical stress "urin$ the $rowth s!urt is an im!ortant cause of )C-(, an" at least one half the chil"ren with the con"ition are obese. )C-( occurs with an increase" frequency in association with en"ocrine "isor"ers, es!ecially hy!othyroi"ism, !anhy!o!ituitarism, an" renal "isease. ecause of its frequency, the risk for "e$enerative arthritis, an" the !ossibility of su""en instability an" vascular "ama$e, early "etection of )C-( is e*tremely im!ortant. Clinical fin"in$s can vary "e!en"in$ on whether the sli! is stable or unstable. In a stable sli!, an obese a"olescent boy may com!lain of activity4relate" hi! !ain that is relieve" by rest. 7here may be a mil" lim! after strenuous activity. 7hese sym!toms may last weeks or months. &hysical e*amination shows !ain with hi! motion, restriction of fle*ion, an", !articularly, internal rotation while the hi! is fle*e" to I> "e$rees. 7he "ia$nosis can be elusive, since referral of the !ain may be to the "istal thi$h, !resentin$ as knee !ain. All chil"ren who com!lain of knee !ain shoul" have their hi! e*amine". )ome chil"ren with an e*tremely e*ternally rotate" $ait have foot !ain as their !resentin$ com!laint. 7he unstable sli! "oes not allow the chil" to bear wei$ht, even with crutches. 7his shoul" be consi"ere" the equivalent of an acute, "is!lace" fracture of the hi! an" shoul" be treate" ur$ently. 7he chil" is in e*treme !ain an" usually $ives a history of a fall from a bicycle or a treeG

however, the fall may also be trivial if the e!i!hysis ha" !ree*istin$ weakness. 6a"io$ra!hs confirm the "ia$nosis, an" the !atient must be ur$ently a"mitte" to the hos!ital for treatment. 7he risk for avascular necrosis, even with imme"iate treatment, is as hi$h as ?D=. 6a"io$ra!hs are obtaine" with the !atient in the su!ine !osition for antero!osterior 'A&) an" fro$ lateral !elvis films. Once the "ia$nosis of a stable sli!!e" e!i!hysis is ma"e in a walkin$ chil", sur$ery shoul" be !erforme" without "elay. 7he most common !roce"ure is !ercutaneous !in or screw fi*ation between the femoral neck an" the center of the e!i!hysis. Nuvenile rheumatoi" arthritis 'choice A) is the most !revalent chronic !e"iatric rheumatolo$ic "isease. 7he arthritis is frequently symmetric an" !re"ominantly affects lar$er <oints. &atients can have a fever an" an elevate" erythrocyte se"imentation rate an" white bloo" cell count. ,e$$4Calve4&erthes "isease 'choice ) is avascular necrosis of the femoral e!i!hysis. It occurs more frequently in males an" youn$ chil"ren a$e" ?4@ years. It usually !resents with lim!in$ an" knee !ain. )e!tic arthritis 'choice C) !resents with a !ainful an" swollen <oint, as well as fever. It is "ia$nose" by as!iration of the synovial flui", which will yiel" a very hi$h white cell count an" a !atho$en on culture. 7o*ic synovitis 'choice () is a beni$n an" common con"ition that often !resents with hi! !ain, knee !ain, an" lim!in$. It is usually !rece"e" by a viral illness. :?) (*!lanation: 7he correct answer is C. -orei$n bo"y as!iration is a un"er"ia$nose" !roblem occurrin$ mainly in to""lers an" !reschool chil"ren. Chil"ren who have a !ersistent or su""en4onset cou$h without other si$ns of an infection 'fever, rhinorrhea, malaise, !oor a!!etite, vomitin$) shoul" be sus!ecte" as havin$ a forei$n bo"y in their airway. )ome forei$n bo"ies are visible on *4 ray 'coins, button batteries, metal toys) but most are not 'nuts, har" can"ies, me"icine tablets). 7heir !resence must be in"irectly inferre" from asymmetric lun$ fiel"s 4 either hy!ere*!ansion an" air tra!!in$ of the affecte" si"e "ue to a ball4valve effect of the forei$n bo"y, or colla!se of the affecte" si"e "ue to com!lete airway obstruction. Irritation of the bronchial mucosa by the forei$n bo"y can lea" to erosion an" bloo"y e*!ectorant, as in this case. acterial !neumonia 'choice A) is unlikely since this !atientAs cou$h be$an su""enly an" he ha" no other si$ns an" sym!toms of infection 'e.$., fever, rhinorrhea). Also, the chest *4ray film woul" show evi"ence of !neumonia with a focal infiltrate or enlar$e" !ulmonary vasculature. Cystic fibrosis 'choice ) causes chronic, recurrent res!iratory illnesses, often with bloo"y s!utum, but woul" also have si$ns an" sym!toms of systemic illness. In a""ition, the chest *4ray film of a !atient with cystic fibrosis woul" show chronic chan$es such as cystic abnormalities an" scarrin$ in the lun$s from re!eate" res!iratory infections. A !ulmonary arteriovenous malformation or A1+ 'choice %) is clinically silent unless it is so lar$e that is causes hi$h out!ut con$estive heart failure or it ru!tures. A ru!ture" !ulmonary A1+ causes si$nificant hemo!tysis that is "ifficult to sto! without sur$ical intervention. 7uberculosis 'choice () !resents similarly to !neumonia, with systemic si$ns of illness, an" fever. 7uberculosis often !ro"uces hemo!tysis an" must be sus!ecte" when a !atient com!lains of bloo"y

s!utum. Chil"ren of this a$e are more likely to have e*tra!ulmonary 7 '7 menin$itis, miliary 7 ) rather than !ulmonary 7 . Chest *4ray fin"in$s either show a .hon com!le* 'a!ical lesions), calcifications or lobar infiltrates. :/) (*!lanation: 7he correct answer is (. .astric lava$e shoul" be use" in this somnolent !atient. It is best "one within the first C> minutes of in$estion of the unknown substance. It is !erforme" by insertin$ a lar$e4 caliber oro$astric tube an" then flushin$ the tube with a lar$e amount of normal saline. 7he saline is then as!irate" from the stomach to a container in an attem!t to evacuate any in$este" substance that !ersists in the stomach cavity. I"entification of the in$este" substance is also very im!ortant. With certain in$este" substances, $astric lava$e is contrain"icate". -or e*am!le, the !roce"ure shoul" not be use" if hy"rocarbon in$estion is sus!ecte" because as!iration is much more likely to ha!!en "urin$ a lava$e or an in"uce" emesis. -urthermore, in$estion of alkaline substances causes eso!ha$eal necrosis. In this case, $astric lava$e may cause eso!ha$eal !erforation. I1 a"ministration of caffeine 'choice ) has not been shown to be effective in reversin$ the mental status chan$es in !atients who in$est a lar$e amount of slee!in$ !ills. %ro!eri"ol 'choice C) is an antiemetic an" is not an a!!ro!riate initial treatment in to*ic in$estion. 2owever, it may be use" in con<unction with activate" charcoal to enhance tolerance. I1 a"ministration of a normal saline bolus 'choice %) !lays no role in this !atient who is normotensive an" "oes not a!!ear to be "ehy"rate" or in shock. I!ecac4in"uce" emesis 'choice () is a !oor choice, because the mental status of the !atient is im!aire" an" there is a risk of as!iration when emesis occurs. :C) (*!lanation: 7he correct answer is %. 7he level of serum so"ium in this !atient is :D> m(qB,. Infants who have hy!ernatremic "ehy"ration are irritable an" lethar$ic, an" have a hi$h4!itche" cry. 7his ty!e of "ehy"ration results from a $reater loss of hy!otonic flui" than so"ium an" accounts for about :/= cases of "ehy"ration. ecause the !atient has no history of "iarrhea or vomitin$, the hy!ernatremia may be "ue to ina"equate su!!ly of motherAs milk that "oes not match the insensible water loss. Another cause can be the hi$h concentration of so"ium in mothers milk. .enerally, after the chil"As birth, so"ium in the colostrum "ecreases from its hi$hest level to its lowest level by the fourth week. 2owever, some mothers continue to e*crete hi$h so"ium in their milk an" can !otentially cause recurrent hy!ernatremia an" in some case intracranial hemorrha$e in the infant. 2y!ocalcemia 'choice A), hy!o$lycemia 'choice ) an" hy!oma$nesemia 'choice C) are all !otentially metabolic causes of sei#ures, however in this vi$nette serum calcium, $lucose an" ma$nesium are within normal limits. In !atients with hy!ernatremic "ehy"ration, hy!er$lycemia can result "ue to e*cess $luca$on stimulation. +enin$itis

'choice () shoul" be consi"ere" in any infant with a sei#ure with or without fever. 2owever, the marke" rise of the serum so"ium makes this "ia$nosis unlikely. :D) (*!lanation: 7he correct answer is . Intussusce!tion usually occurs within the C to :;4month4ol" a$e $rou!. It characteristically !resents with e!iso"es of "istress an" cryin$ inters!erse" with quiet !erio"s of normal behavior an" !layin$. +ore than />= of chil"ren will !ass stool mi*e" with mucus an" bloo", the Hcurrant <ellyH stool. Fsually the ab"omen is soft an" nonten"er, but with a"vance" intussusce!tion there may be si$ns of !eritonitis. )ometimes a sausa$e4like mass may be !al!able in the u!!er ab"omen. A barium enema is !erforme" for the "ia$nosis an" treatment of the intussusce!tion. Ab"ominal con"itions that may occur more frequently in the neonatal !erio", birth to ? weeks of a$e 'choice A), inclu"e "isor"ers such as volvulus, !yloric stenosis, an" necroti#in$ enterocolitis. 1olvulus woul" !resent with bilious vomitin$ an" ab"ominal "istention. &yloric stenosis usually !resents at about ? to C weeks of a$e with nonbilious vomitin$. Necroti#in$ enterocolitis is more common amon$ !remature infants an" may !resent with $rossly bloo"y stools. In the 0 to / year a$e $rou! 'choice C) ab"ominal !ain is frequently "ue to consti!ation. Other common ab"ominal con"itions at this a$e inclu"e $astroenteritis, while ab"ominal tumors such as Wilms tumor an" neuroblastoma are less common. A!!en"icitis is rare before the a$e of / years. Intussusce!tion is less likely to occur "urin$ either early a"olescence 'choice %) or late a"olescence 'choice (). A!!en"icitis usually occurs between :> to :I years of a$e. .ynecolo$ical "isor"ers inclu"in$ !elvic inflammatory "isease, ovarian cysts, an" !re$nancy4relate" con"itions, shoul" be consi"ere" in a"olescent $irls who !resent with acute ab"ominal com!laints. :@) (*!lanation: 7he correct answer is C. 7he !atient is havin$ absence or !etit mal sei#ures, an" the "ru$ of choice is ethosu*imi"e or val!roic aci". %ia#e!am 'choice A) is effective in treatin$ status e!ile!ticus but is not use" in treatin$ !etit mal sei#ures. %i!henhy"ramine 'choice ) is an antihistamine an" woul" not be of benefit in treatin$ this illness. &henobarbital 'choice %) an" &henytoin 'choice () are anticonvulsants, but are not use" for absence sei#ures. :I) (*!lanation: 7he correct answer is %. 7his infant is e*!eriencin$ a hy!o*emic s!ell, as seen in !atients who have 7etralo$y of -allot. 7hese hy!ercyanotic s!ells, or H7et s!ells,H usually are self4limite" an" last less than :>4:/ minutes. 7he s!ells often occur imme"iately after fee"in$ or when the chil" is cryin$ vi$orously. 7etralo$y of -allot has the followin$ com!onents: :) ventricular se!tal "efect, ;) overri"in$ aorta, 0) ri$ht ventricular hy!ertro!hy, an" ?) !ulmonic stenosis. Aortic stenosis 'choice A) is ty!ically not a cyanotic lesion. Chil"ren with aortic stenosis

may "evelo! sym!toms of con$estive heart failure, but they woul" not have cyanotic s!ells. Coarctation of the aorta 'choice ) is not a cyanotic heart lesion. 7y!ically there are sym!toms of con$estive heart failure. -in"in$s inclu"e a systolic murmur that ra"iates to the left a*illa, !ulse la$ in the lower e*tremities, an" bloo" !ressure $reater in the u!!er than in the lower e*tremities. &atent "uctus arteriosus 'choice C) is not a cyanotic heart lesion. It !resents with an active !recor"ium an" continuous HmachineryH murmur. 7he !ulses are boun"in$, an" !ulse !ressure is wi"ene". 1entricular se!tal "efect '1)%) 'choice () is not a cyanotic heart lesion. A 1)% may or may not !resent with any !roblems. 7he most common fin"in$ is "ecrease" fee"in$ an" !oor wei$ht $ain in !atients with lar$e left4to4ri$ht shunts. ;>) (*!lanation: 7he correct answer is C. 7his is %i.eor$e syn"rome, which is a con$enital syn"rome in which abnormal "evelo!ment of the thir" an" fourth !haryn$eal !ouches lea"s to absence or hy!o!lasia of the thymus an" !arathyroi" $lan"s. 7he "ia$nosis is sus!ecte" in an infant, often with a malforme" face, who "evelo!s a "ifficult4to4mana$e hy!ocalcemia, ty!ically at ;? to ?@ hours of life. 7he "ia$nosis can be confirme" with chest *4ray, which will fail to show the normal thymic sha"ow. 7he infants shoul" also be evaluate" for con$enital car"iovascular "isease, as this may be !art of the "evelo!mental abnormality. Infants that survive the initial hy!ocalcemia 'requirin$ e*tremely careful me"ical mana$ement) become vulnerable to recurrent infections shortly after birth. 7he "efect usually involves 7 cells 'with normal or near normal cell function), an", in some cases, may resolve s!ontaneously as the chil" a$es an" a small remnant of thymus hy!ertro!hies. rutonAs a$amma$lobulinemia 'choice A) is a usually *4linke" form of severe immuno$lobulin "eficiency relate" to "ecrease" cell numbers. It ty!ically !resents with onset of infections after C months of a$e. Common variable immuno"eficiency 'choice ) ia a usually acquire" form of immuno$lobulin "eficiency that ten"s to !resent in the secon" or thir" "eca"e of life. )elective I$A "eficiency 'choice %) is an often asym!tomatic or nearly asym!tomatic mil" form of immuno"eficiency, the !rimary clinical si$nificance of which is that affecte" in"ivi"ual may un"er$o ana!hylactic reactions to I$A4containin$ bloo" !ro"ucts. 7ransient hy!o$amma$lobulinemia of infancy 'choice () is a self4limite" !re"is!osition to bacterial infections be$innin$ at 0 to C months that is seen in mor!holo$ically normal babies whose cells fail to mature fast enou$h to re!lace maternal antibo"ies. ;:) (*!lanation: 7he correct answer is A. )uturin$ a laceration is one of the most common situations in which se"ation may be require" in a chil". )e"ation may be classifie" as conscious or "ee!. y "efinition, "urin$ conscious se"ation the !atient is able to maintain airway !atency, !rotective airway refle*es, an" res!onses to !hysical stimuli. 7his level of se"ation is in"icate" for chil"ren

'or a"ult !atients) who have not faste" !rior to the !roce"ure, or !atients who "o not require a "ee! level of se"ation. Nevertheless, conscious se"ation shoul" be !erforme" by a!!ro!riately traine" !ersonnel, an" only when equi!ment for resuscitation measures is rea"ily available, shoul" the nee" arise. -or minor sur$ical !roce"ures such as suturin$ uncom!licate" linear lacerations, a"ministration of a short4actin$ or lon$4actin$ ben#o"ia#e!ine 'mi"a#olam or "ia#e!am, res!ectively) by the oral or rectal route !rovi"es sufficient se"ation. Intravenous access is not require". Intravenous mi"a#olam or "ia#e!am can be use" for !roce"ures that !ro"uce more intense !ain or "iscomfort, such as re!air of com!le* lacerations, bone marrow as!iration, an" re"uction of fractures. Concomitant o!ioi" an" ben#o"ia#e!ine a"ministration 'choice ) is use" to achieve not only se"ation, but also an a"equate level of anal$esia. 7he syner$istic action of o!ioi"s an" ben#o"ia#e!ines increases the risk of res!iratory "e!ression. Intravenous !ro!ofol 'choice C) !rovi"es ra!i" onset of se"ation that resolves quickly once infusion is "iscontinue". 7his "ru$ is use" for !roce"ures requirin$ "ee!er levels of se"ation in a!!ro!riately faste" an" stable chil"ren. Intravenous ketamine 'choice %) is an a!!ro!riate alternative to !ro!ofol. Its most common si"e effect is the !ro"uction of visual an" au"itory hallucinations 'about :>= of cases). Concomitant anal$esic4se"ative a$ents an" muscle rela*ants 'choice () is em!loye" for "ee! se"ation an" when muscle rela*ation is necessary for en"otracheal intubation or other "ia$nosticBthera!eutic !roce"ures. ;;) (*!lanation: 7he correct answer is C. ,yme "isease is a subacute infection cause" by the bite of a "eer tick infecte" with orrelia bur$"orferi. 7he si$ns an" sym!toms inclu"e a characteristic rash, erythema mi$rans, which is erythematous with a central clearin$. 7he rash ty!ically eru!ts 040> "ays after the bite. -evers, hea"aches, myal$ias, an" arthritis may also occur. (n"emic areas inclu"e the Northeast, u!!er +i"west, an" the West Coast. abesiosis 'choice A) is cause" by a malaria4like !roto#oan an" is associate" with sym!toms of cyclic fever Wtem!eratures u! to ?> C ':>? -)X, malaise, myal$ias, an" hea"ache. It has been re!orte" in +assachusetts, 6ho"e Islan", an" New 9ork. Colora"o tick fever 'choice ) is foun" in the 6ocky +ountain re$ion of the F.). It is cause" by the bite of a woo" tick. )ym!toms inclu"e fever, chills, hea"ache, con<unctivitis, an", very rarely, a rash. 6ocky +ountain s!otte" fever 'choice %) is cause" by 6ickettsia rickettsii an" is transmitte" by a tick bite. It occurs !re"ominantly in the eastern an" southeastern states. )ym!toms inclu"e a hi$h fever, myal$ias, severe hea"ache, !hoto!hobia, nausea, an" a maculo!a!ular rash occurrin$ !rominently on the !alms, soles, an" e*tremities. 7he !atient often a!!ears to*ic. 7ularemia 'choice () is cause" by the bite of a tick that has been in contact with an animal infecte" with -rancisella tularensis. )ym!toms inclu"e a rash that be$ins as a !ruritic re" !a!ule, as well as enlar$e", ten"er re$ional lym!h no"es. 7here may be hi$h fever, chills, weakness, an" vomitin$. ;0)

(*!lanation: 7he correct answer is A. 7he H"ouble bubbleH si$n is !atho$nomonic for "uo"enal atresia, which is a con$enital anomaly associate" with %own )yn"rome. 7wo lar$e $as collections, one in the stomach an" one in the !ro*imal "uo"enum are the only ra"io$ra!hic lucencies visible in the .I tract. 7he hallmark of "uo"enal obstruction is bilious vomitin$ without ab"ominal "istention. Chil"ren with %own )yn"rome can also have eso!ha$eal atresia, im!erforate anus, en"ocar"ial cushion "efects an" hy!otonia. 2irschs!run$ "isease 'choice ) can cause ab"ominal obstruction with "elaye" !assa$e of stool in the first few "ays of life. It is a functional obstruction cause" by lack of $an$lion cells in +eissnerAs an" AuerbachAs !le*us of the colon, causin$ a!eristalsis in the affecte" se$ment of colon. E4ray films woul" show stool throu$hout the ab"omen but little or no air in the rectum, an" a "ilate" Hme$acolonH <ust !ro*imal to the affecte" se$ment. +alrotation 'choice C) is the failure of the .I tract to !ro!erly rotate "urin$ embryo$enesis. It can !resent with vomitin$ but !lain *4ray films woul" be nons!ecific. A barium enema woul" "emonstrate an abnormally !lace" cecum. +alrotation can lea" to a mi"$ut volvulus, which is a sur$ical emer$ency since the mi"$ut can twist on its bloo" su!!ly, resultin$ in ischemia. +econium ileus 'choice %) occurs when meconium becomes obstructe" in the terminal ileum. It is the most common !resentation of cystic fibrosis in the neonatal !erio". +il" obstruction can be treate" with enemas, but com!lete obstruction may require o!erative mana$ement. &yloric stenosis 'choice () rarely occurs in the newborn !erio". It !resents in the first month of life with !ro$ressive vomitin$ without ab"ominal "istention. It is cause" by hy!ertro!hy of the !yloric muscle at the <unction of the stomach an" "uo"enum. E4rays may show a lar$e "ilate" $astric bubble, but there woul" be no H"ouble bubbleH since the obstruction is !ro*imal to the "uo"enum. ;?) (*!lanation: 7he correct answer is C. One of the most im!ortant tasks that !hysicians can !erform in re$ar" to the treatment of e!ile!sy is counselin$ caretakers about what to "o when sei#ures occur. -reeman et al. ')ei#ures an" (!ile!sy: A .ui"e for &arents, ;n" e"ition, Nohns 2o!kins Fniversity &ress, :IID) list a number of H"oAsH an" H"onAtAs.H %urin$ the sei#ure e!iso"e, "onAt &ut any ob<ect into the !atientAs mouth Call an ambulance unless sei#in$ lasts more than :> minutes 7ry to restrain the !atient %urin$ a sei#ure e!iso"e, "o &lace the !atient on the si"e &ut a !illow or other soft ob<ect un"er the !atientAs hea" ,oosen ti$ht clothin$ aroun" the neck 6emove shar! ob<ects from the surroun"in$s After the sei#ures, caretakers or !arents shoul" remain with the !atient until heBshe is fully alert an" allow himBher to $o back to the usual activities. Callin$ an ambulance 'choice A) is a!!ro!riate if the sei#ure e!iso"e lasts for more than /4:> minutes. &uttin$ somethin$ in the !atientAs mouth at the onset of sei#ure 'choice ) increases the risk of suffocation. 7ryin$ to restrain the chil" "urin$ the sei#ure 'choice %) is a useless measure. Not allowin$ the chil" to return to his activities 'choice () is a mistake if the !atient has become fully conscious an" alert after recovery.

;/) (*!lanation: 7he correct answer is . 7his chil"As con"ition is chronic $ranulomatous "isease, a usually *4linke" recessive con"ition characteri#e" by ina"equate !ro"uction of hy"ro$en !ero*i"e, su!ero*i"e, an" other activate" o*y$en s!ecies in neutro!hils. 7he nitroblue tetra#olium "ye re"uction test "emonstrates the failure of the "efective neutro!hils to !ro"uce these !ro"ucts. 7he clinical result of the en#ymatic "efect is that the neutro!hils can !ha$ocyti#e but not kill bacteria. Affecte" in"ivi"uals are !la$ue" with multi!le, !oorly healin$ infections to which their bo"ies res!on" with $ranuloma formation to remove or$anisms that woul" normally be controlle" by neutro!hils. 7hese !atients are usually treate" with intermittent or continuous antibioticsG bone marrow trans!lant an" interferon thera!y have also been use". Absent cells an" normal numbers of 7 cells 'choice A) su$$ests E4linke" a$amma$lobulinemia. 2i$h serum I$+ an" very low serum I$. 'choice C) su$$ests hy!er I$+ immuno"eficiency. +onoclonal anti4C%:: is use" to "ia$nose leukocyte a"hesion "eficiency, in which the C%:: anti$en on the surface of white bloo" cells 'choice %) is missin$. 7his "isor"er causes a severe form of immuno"eficiency that usually results in "eath by a$e /. )erum calcium levels 'choice () can be marke"ly "ecrease" in %i.eor$e syn"rome. ;C) (*!lanation: 7he correct answer is . )haken infant syn"rome was "efine" by !e"iatric ra"iolo$ist Nohn Caffey in :ID;. A!!ro*imately :4; million cases of chil" abuse an" ne$lect are re!orte" every year, an" nearly ;>>> chil"ren "ie from abuse an" ne$lect annually. A normal !latelet count an" coa$ulation stu"ies eliminate the mis"ia$nosis of blee"in$ abnormalities. An ammonia level 'choice A), li!i" !anel 'choice C), thyroi" stu"ies 'choice %) or urine electrolytes 'choice () are not in"icate". ;D) (*!lanation: 7he correct answer is . Con$enital "ia!hra$matic hernia is often seen on !renatal ultrasoun". A "ia!hra$matic hernia is a "efect in the hemi"ia!hra$m that allows the ab"ominal contents into the thora*. It has the fin"in$s of res!iratory "istress, cyanosis, an" sca!hoi" ab"omen. Auscultation will show "ecrease" breath soun"s on the affecte" si"e. It occurs more often on the left si"e than the ri$ht. %e*trocar"ia 'choice A) refers to the location of the heart in the ri$ht chest an" woul" cause the heart soun"s to be lou"er on the ri$ht si"e. reath soun"s are not "ecrease", an" the ab"omen is not sca!hoi". &neumonia 'choice C) usually "oes not !resent imme"iately after birth with res!iratory "istress, usually takin$ several hours after "elivery to "evelo! sym!toms. 7he ab"omen is not sca!hoi", an" the heart soun"s are normal. &ulmonary hy!o!lasia 'choice %) can be a result of insults 'i.e.,

oli$ohy"ramnios) to the fetus in utero that "ecrease the lun$ si#e. It can also be a result of "ia!hra$matic hernia, since the lun$ is "is!lace" by the ab"ominal contents. )!ontaneous !neumothora* 'choice () often occurs in the neonatal !erio" after vi$orous resuscitation at birth. A left4si"e" !neumothora* woul" cause the res!iratory "istress, cyanosis, an" me"iastinal shift but woul" not !ro"uce a sca!hoi" ab"omen. ;@) (*!lanation: 7he correct answer is A. 2e!atitis virus can cause si$nificant morbi"ity an" mortality. It is s!rea" throu$h se*ual an" bloo"4borne contact. In fact, it is the only se*ually transmitte" "isease for which there is a wi"ely available an" acce!te" immuni#ation to !revent the "isease. 7he current recommen"ations are for universal vaccination a$ainst he!atitis for a"olescents to be $iven at a$es :: to :;. Ol"er a"olescents an" a"ults shoul" receive the vaccine on the basis of risk status, althou$h several $rou!s recommen" universal vaccination of a"olescents. 7his !atient is :; years ol". 7herefore, she shoul" receive he!atitis vaccination. 7rials are un"er way to test 2I1 immuni#ation 'choice ). At !resent, however, there is no wi"ely available an" acce!te" vaccine to be use" universally. Na!anese ence!halitis virus immuni#ation 'choice C) an" rabies virus immuni#ation 'choice %) are two immuni#ations that can be $iven to travelers $oin$ to en"emic areas or in situations where infection with Na!anese ence!halitis virus or rabies virus may occur. Fniversal vaccination a$ainst these two viruses is not recommen"e". )almonella ty!hi immuni#ation 'choice () is use" to !revent infection with )almonella ty!hi. 7his bacterium is the cause of the infection commonly known as ty!hoi" fever. 7his vaccine is recommen"e" to !eo!le travelin$ to areas in which such infection is likely to occur. Fniversal vaccination is not recommen"e". ;I) (*!lanation: 7he correct answer is A. %own syn"rome, or trisomy ;:, is the most common autosomal chromosome abnormality. 7he inci"ence increases as maternal a$e increases. 7he classic features are hy!otonia, u!slantin$ !al!ebral fissures, e!icanthal fol"s, e*cess nuchal skin, an enlar$e" ton$ue, clino"actyly of the fifth fin$ers, an" a sin$le transverse !almar crease. ("war"s syn"rome 'choice ), or trisomy :@, has features of small !al!ebral fissures, low4set ears, low birth wei$ht, microce!haly, rocker4bottom feet, cleft li!, hy!otonia, an" clenche" han"s. -etal alcohol syn"rome 'choice C) is characteri#e" by $rowth retar"ation, small !al!ebral fissures, smooth !hiltrum, a thin u!!er li!, microce!haly, an" a short nose. -eatures often $o unnotice" in the newborn !erio", althou$h sometimes tremulousness an" irritability occur. +arfan syn"rome 'choice %) is associate" with increase" stature, thin limbs, scoliosis, <oint hy!ermobility, an" ocular manifestations. -eatures often $o unnotice" in the newborn !erio". 7urner syn"rome 'choice () has features of !rominent low4set ears, e*cess nuchal skin, broa" chest, e!icanthal fol"s, lym!he"ema of han"s an" feet, an" short stature.

0>) (*!lanation: 7he correct answer is (. +essin$ with a "o$ while the "o$ is eatin$, an" bein$ bitten in res!onse, is consi"ere" to be a !rovoke" attack, an" thus not in"icative of a$$ressive behavior on the !art of the "o$. 7he bite is in the han"G therefore, there is !lenty of time for the virus to travel in the "irection of the brain, so that thera!eutic measures woul" still be effective shoul" the future behavior of the "o$ su$$est that he is rabi". &assive immuni#ation 'choice A), active immuni#ation 'choice C), or a combination of both 'choice ) is not require" in this low4risk situation. 2a" the bite been in the face, an area where rabies is !revalent, a more a$$ressive a!!roach woul" have been <ustifie". 5illin$ the animal an" e*aminin$ the brain 'choice %) is the norm for wil" animals that are ca!ture" alive, since their behavior cannot be "eeme" to be HnormalH or Habnormal.H It is also the a!!ro!riate ste! if observation of a "omestic animal by a veterinarian su$$ests that si$ns of rabies are "evelo!in$ in the animal. 0:) (*!lanation: 7he correct answer is C. 7he baby has Wiskott4Al"rich syn"rome, which is an E4linke" recessive immuno"eficiency "isease characteri#e" by the tria" of thrombocyto!enia 'hemorrha$e may be the !resentin$ com!laint), ec#ema, an" recurrent infections 'often res!iratory). 7he chil"ren have "efects in both 7 an" cell function, an" are vulnerable to !yo$enic bacteria, viruses, fun$i, an" &neumocystis carinii. 7hese !atients have severe "isease, an" formerly often "ie" by a$e :/. )urvivors !ast a$e :> have a :>= inci"ence of cancer, !articularly lym!homa an" acute lym!hoblastic leukemia. +o"ern treatment consists of s!lenectomy, continuous antibiotic thera!y, I1 immuno$lobulin, an" bone marrow trans!lantation. Con$estive heart failure 'choice A) is not a !articular !roblem in these chil"ren. Crohn "isease 'choice ) an" rheumatoi" arthritis 'choice %) are not increase" in these chil"ren. Wilms tumor 'choice () usually occurs in infancy or very early chil"hoo", an" is not a s!ecific com!lication of Wiskott4Al"rich syn"rome. 0;) (*!lanation: 7he correct answer is A. 7his baby has a form of severe combine" immuno"eficiency with autosomal recessive inheritance an" low antibo"y levels. 7his is most likely to be a"enosine "eaminase "eficiency. 7his en#yme is a !urine salva$e en#yme that converts a"enosine an" "eo*ya"enosine to inosine an" "eo*yinosine. When the en#yme is "eficient, abnormally hi$h levels of "A7& accumulate, an" turn off %NA synthesis. 7his !articularly im!acts the immune system, which 'for both 7 an" cell lines) $ears u! s!ecificity by tri$$erin$ massive re!ro"uction of clones that are immunolo$ically active a$ainst a !articular

anti$en. )ome of these chil"ren !resent as illustrate"G in others the immunolo$ic "efect "evelo!s more slowly an" affects 7 cells before cells. ony abnormalities are common in these chil"ren. one marrow trans!lant, an" very recently, $ene transfer, are use" to treat this otherwise fatal con"ition. ruton a$amma$lobulinemia 'choice ) is characteri#e" by !an hy!o$amma$lobulinemia but intact 7 cell function. %i.eor$e syn"rome 'choice C) is characteri#e" by absent thymus, normal cell function, an" often hy!ocalcemia in infancy. Ne#elof syn"rome 'choice %) is characteri#e" by elevate" immuno$lobulins that function !oorly. E4linke" severe combine" immuno"eficiency 'choice () woul" not be seen in a female. 00) (*!lanation: 7he correct answer is (. +ittelschmert# is the most likely "ia$nosis. It occurs for : to ; "ays in the mi"cycle of menstruatin$ females. At ovulation, an ovarian follicle can ru!ture, releasin$ bloo" into the !eritoneal cavity. 7his results in !eritoneal irritation an" !ain. 7he !ain may be intermittent, an" it usually occurs in the lower ab"omen. In this case the $irl is ; years !ostmenarche an" has a history of several months of "ysmenorrhea, in"icatin$ that she has ovulatory cycles. y "efinition, the !ain of "ysmenorrhea 'choice A) shoul" occur at menstrual time. 7his $irl has been menstruatin$ now for a few years an" is likely to have ovulatory cycles. )he may have "ysmenorrhea or !ain with her !erio"s, but the !ain that has taken her to the emer$ency room is inter4menstrual. (cto!ic !re$nancy 'choice ) an" !elvic inflammatory "isease 'choice %) shoul" be consi"ere" as "ia$nostic !ossibilities in any menstruatin$ a"olescent female with shar! ab"ominal !ain. Althou$h they are unlikely in this case with a ne$ative history of se*ual activity an" the absence of irre$ular va$inal blee"in$, the e*aminin$ "octor must always "etermine the nee" of "oin$ a !elvic e*amination an" a !re$nancy test. &hysiolo$ical ovarian cysts 'choice C) are common "urin$ a"olescence but are less likely to be the cause of this $irlAs acute e!iso"e of !ain. 7here is no !rior history of abnormal va$inal blee"in$ or !rior e!iso"es of ab"ominal !ain. 7his $irl com!lains of lower mi"ab"ominal !ain. -ollicular ovarian cysts are $enerally small in si#e an" may !ro"uce estro$en. 7hey are usually asym!tomatic, but may result in menstrual irre$ularities. Cor!us luteum cysts are more frequently sym!tomatic. 7hey may increase in si#e, causin$ !ressure sym!toms an" ab"ominal !ains, as well as "elaye" or heavy menses. A "ia$nosis of ovarian cyst may be sus!ecte" on !elvic e*amination an" confirme" with !elvic ultrasoun". 0?) (*!lanation: 7he correct answer is A. 7his is ruton a$amma$lobulinemia 'E4linke" a$amma$lobulinemia). It causes low or absent numbers of cells, lea"in$ to a !anhy!o$amma$lobulinemia. Cellular immunity is intact. 7he !atients ty!ically "evelo! infections after about si* months of a$e, when maternal antibo"ies have "ecrease" to low levels. 7here is a life4lon$ !re"is!osition for recurrent !yo$enic infections, !articularly of the lun$s, sinuses, an"

bones. While the con"ition is classically consi"ere" E4linke", this can only be !roven in about ;>= of cases. 7hese !atients require life4lon$ immuno$lobulin thera!y an" a$$ressive antibiotic mana$ement when infections "o "evelo!. Common variable immuno"eficiency 'choice ) can have some clinical overla! with rutonAs a$amma$lobulinemia, but is characteri#e" by normal cell levels an" ty!ically !resents in the secon" or thir" "eca"e of life. %i.eor$e syn"rome 'choice C) ty!ically !resents with hy!ocalcemia in infancy. 7ransient hy!o$amma$lobulinemia of infancy 'choice %) has clinical overla! with rutonAs a$amma$lobulinemia, but can be "istin$uishe" by the !resence of normal cell numbers. Wiskott4Al"rich syn"rome 'choice () is an E4linke" "isor"er with ec#ema, thrombocyto!enia, an" recurrent infection. 0/) (*!lanation: 7he correct answer is %. O!ioi"s 'e.$., heroin, co"eine, metha"one) can cause res!iratory "e!ression, coma, sinus bra"ycar"ia, an" !u!illary constriction. %eath usually results from as!iration of $astric contents, res!iratory arrest, or cerebral e"ema. Am!hetamine into*ication 'choice A) inclu"es an*iety, hy!ertension, urinary retention, an" nausea. )evere cases may inclu"e rhab"omyolysis. Cocaine into*ication 'choice ) is characteri#e" by tachycar"ia, hy!ertension, an" "ilate" !u!ils. Over"ose often lea"s to sei#ures, hy!otension, an" res!iratory "e!ression. %ysrhythmias may be seen. (thanol into*ication 'choice C) causes res!iratory "e!ression, hy!o$lycemia, stu!or, an" convulsions. (thanol is not usually measure" on a urinary to*icolo$y screen. 7ricyclic 'e.$., amitri!tyline, imi!ramine) into*ication 'choice () is characteri#e" by "ysrhythmias, coma, convulsions, an" early hy!ertension, followe" by hy!otension an" hallucinations. 0C) (*!lanation: 7he correct answer is A. 7he clue to this !atientAs con"ition is the recent of a honey4containin$ formula. Clostri"ium botulinum s!ores that are commonly foun" in honey $erminate in the infantAs $astrointestinal tract an" !ro"uce the characteristic to*in. Infants youn$er than : year of a$e shoul" not be !lace" on a "iet with honey. 2irschs!run$ "isease 'choice ) is cause" by lack of $an$lia in the colon an" can le" to a me$acolon. )ym!toms inclu"e ab"ominal "istention an" consti!ation. 7he sym!toms woul" be slower in onset. 2y!ernatremia 'choice C) woul" be associate" with fin"in$s of shrivele" skin, tentin$ an" $eneral "ehy"ration. (*cessive sweatin$ or "iarrhea woul" be common causes of the con"ition. 2y!onatremia 'choice %) may be "ue to so"ium "eficit or flui" e*cess. It may be associate" with "iuretic use, cortisol "eficiency, syn"rome of ina!!ro!riate secretion of anti"iuretic hormone or e*cessive intake of free water. 7y!ical sym!toms inclu"e lethar$y an" weakness an" are usually $ra"ual in onset. 2y!othyroi"ism 'choice () woul" !resent with similar sym!toms but woul" be $ra"ual in onset an" is often associate" with mental retar"ation.

0D) (*!lanation: 7he correct answer is . 7he clinical !icture an" laboratory fin"in$s are consistent with common variable immuno"eficiency syn"rome. Im!ortant clues to the "ia$nosis are onset in late a"olescenceByoun$ a"ulthoo", hy!o$amma$lobulinemia with marke"ly "ecrease" I$+, recurrent !yo$enic infections of the u!!er res!iratory tract an" intestinal $iar"iasis, an" failure of lym!hocytes to "ifferentiate into !lasma cells. 7he latter fin"in$, a!!reciable on lym!h no"e bio!sy, e*!lains "eficient immune $lobulin !ro"uction, but the e*act molecular mechanism is obscure. )ome believe that the un"erlyin$ "efect affects 4 lym!hocytes, others !ro!ose that 74lym!hocytes are unable to !ro"uce s!ecific lym!hokines that !romote 4lym!hocyte maturation. 7hese !atients have an increase" risk for 4cell lym!homas, $astric carcinoma, an" skin cancer. 7he only thera!y available consists of monthly intravenous infusion of immune $lobulin. Acquire" immuno"eficiency syn"rome 'AI%)) 'choice A) is cause" by 2I1 an" manifests with o!!ortunistic infections an" neo!lasms that are e*tremely unusual in immunocom!etent hosts. 7he C%? 74cell count is "ecrease", with a reverse" C%?:C%@ ratio. A normal C%? cell count rules out AI%) in this case. 2o"$kin "isease 'choice C) may in"uce immune "eficiency an" recurrent infections "ue to 74cell "ysfunction. )erum immuno$lobulins are normal, however. 2o"$kin "isease may also be e*clu"e", in this case, by the bio!sy results, which show hy!er!lastic follicles instea" of the characteristic !atholo$ic features of 2o"$kin "isease 'e.$. 6ee"4)ternber$ cells within a !olymor!hic cellular infiltrate rich in lym!hocytes, eosino!hils, !lasma cells an" histiocytes). Isolate" I$A "eficiency 'choice %) is the most common form of con$enital immune "eficiency 'inci"ence about : in C>> Caucasians) "ue to "ecrease" !ro"uction of I$ A. Its clinical manifestations may mimic those of common variable immuno"eficiency syn"rome, but I$. levels are within normal limits, unless there is an associate" I$. abnormality. E4linke" a$amma$lobulinemia of ruton 'choice () is "ue to failure of 4 cell !recursors to un"er$o maturation. Consequently, mature 4lym!hocytes "o not form, an" $erminal centers are absent in lym!h no"es. All classes of immuno$lobulins are "eficient. )ino!ulmonary an" intestinal infections be$in in the first year of life, soon after the mother4"erive" immuno$lobulins become "e!lete". 7he un"erlyin$ molecular "efect is a mutation of the $ene enco"in$ ruton tyrosine kinase 'btk), locate" in the E chromosome. 0@) (*!lanation: 7he correct answer is C. +easles 'rubeola) is a very conta$ious, e*anthematous res!iratory "isease with a !atho$nomonic enanthem. A live attenuate" measles vaccine became available in :IC0 in the F. ). an" elsewhere, an" measles is now an unusual "isease in countries where this vaccine is wi"ely use". 2owever, measles continues to occur s!ora"ically in mini4 e!i"emics. 7he measles virus is transmitte" by res!iratory secretions, !re"ominantly throu$h e*!osure to aerosols but also throu$h "irect contact with lar$er "ro!lets. &atients

are conta$ious for :4; "ays before the onset of sym!toms until, ? "ays after the a!!earance of the rash. +easles be$ins with a few4"ay res!iratory !ro"rome of malaise, cou$h, cory#a, con<unctivitis, nasal "ischar$e, an" increasin$ fever. Nust before the onset of the rash, 5o!likAs s!ots a!!ear as :4 to ;4mm blue4white s!ots on a bri$ht re" back$roun". 7he characteristic erythematous, non4!ruritic, maculo!a!ular rash of measles be$ins at the hairline an" behin" the ears, s!rea"s "own the trunk an" limbs to inclu"e the !alms an" soles, an" often becomes confluent. At this time, the !atient is at the most severe !oint of the illness. y the ?th "ay, the rash be$ins to fa"e in the or"er in which it a!!eare". rownish "iscoloration of the skin an" "esquamation may occur later. -ever usually resolves by the ?th or /th "ay after the onset of rashG !rolon$e" fever su$$ests a com!lication of measles. ,ym!ha"eno!athy, "iarrhea, vomitin$, an" s!lenome$aly are common features. 7hera!y for measles is lar$ely su!!ortive an" sym!tom base". (rythema infectiosum 'choice A), or -ifthAs "isease, is a mil" illness cause" by !arvovirus 4:I. It usually be$ins as a marke" erythema of the cheeks, $ivin$ a Hsla!!e" cheekH a!!earance. 2an"4foot4mouth "isease 'choice ) !resents with a !ro"rome of fever an" anore*ia, followe" by ulcers on the ton$ue an" oral mucosa an" a vesicular rash on the han"s an" feet. 6oseola infantum 'choice %) !resents with an abru!t onset of a hi$h fever, with tem!eratures u! to 0I./4?:.: C ':>04:>C -). A maculo!a!ular rash a!!ears on the trunk on the 0r" or ?th "ay when the fever breaks. 6ubella 'choice () causes a mil" syn"rome, which is characteri#e" by an erythematous, maculo!a!ular, "iscrete rash, $enerali#e" lym!ha"eno!athy, an" fever. It can cause con$enital rubella syn"rome in the infant if the !re$nant mother is infecte" with the virus. 0I) (*!lanation: 7he correct answer is (. +econium is the fetal stool, which is mostly com!ose" of "esquamate" cells from the $astrointestinal tract a"mi*e" with enou$h bile to $ive the soft stool a $reenish color. A "istresse" fetus will !ass meconium into the amniotic flui" an" then may as!irate it. 7he infants often have !lacental insufficiency as a result of con"itions such as maternal !reeclam!sia, hy!ertension, or !ostmaturity. 7he as!irate" meconium is very irritatin$ to the lun$s an" causes a chemical !neumonitis. &ostmature infants are !articularly likely to have severe !roblems, because the meconium is "ilute" much less in their small amniotic flui" volume. 7he most im!ortant initial ste! in thera!y is !rom!t suction of the naso!haryn* an" mouth to remove the meconium before more 'or even any) is as!irate". 7his can be !erforme" even before the infant is fully "elivere", as soon as a hea" coate" with meconium emer$es from the birth canal. (mer$ency tracheostomy 'choice A) is not warrante" at this !oint. Intubation with mechanical ventilation 'choice ) is "eferre" until after the naso!haryn* an" mouth are cleare" of meconium. A chest *4ray film 'choice C) is not warrante" at this !oint. O*y$en su!!lementation by face mask 'choice %) is "eferre" until after the naso!haryn* an" mouth are cleare" of meconium. ?>) (*!lanation:

7he correct answer is A. &erinatal as!hy*ia woul" e*!lain the fetal heart tracin$s. 7he !oor tone an" res!iratory effort in"icate the same. )ei#ures woul" be e*!ecte" several hours after mo"erate hy!o*ia. Inborn errors of metabolism 'choice ) shoul" not com!licate the !re$nancy. 6es!iratory "istress 'choice C) after a term !re$nancy is unlikely $iven a"equate surfactant. )ubarachnoi" hemorrha$e 'choice %) is associate" with no sym!toms or with irritability that resolves over "ays. Infants with Wer"ni$42offman syn"rome 'choice () have hy!otonia but no ence!halo!athy. ?:) (*!lanation: 7he correct answer is -. 7his is a classic !resentation of an infant of a "iabetic mother. (*!osure to hy!er$lycemia in utero causes hy!erinsulinism in the fetus, which lea"s to macrosomia, since insulin is a $rowth factor. 2y!erinsulinism continues after birth, causin$ mil" to severe hy!o$lycemia. Other common features inclu"e !olycythemia, hy!ocalcemia, <aun"ice, an" res!iratory "istress syn"rome. )everal con$enital anomalies are associate" with infants of "iabetic mothers, inclu"in$ small left colon, which can cause meconium !lu$$in$, CN) an" car"iac anomalies, sacral a$enesis an" renal vein thrombosis. Aniri"ia 'choice A) actually refers to a hy!o!lastic iris, althou$h the name su$$ests an absence of the iris. It occurs either throu$h "ominant inheritance or s!ora"ically. &atients with aniri"ia may "evelo! Wilms tumor, $laucoma, nysta$mus an" other vision !roblems. 7here is also an abnormality of chromosome :: that causes aniri"ia, $enital anomalies an" mental retar"ation. 7here is no association with maternal "iabetes. Cleft !alate 'choice ) is a common facial anomaly that occurs s!ora"ically or in association with a $enetic syn"rome 'i.e. &ierre 6obin )yn"rome) or "ue to maternal "ru$ e*!osure. It is more common in Asians an" least common in blacks. Cleft !alate results from the failure of the !alatal shelves to fuse "urin$ fetal "evelo!ment. It is not usually associate" with infants of "iabetic mothers. +acro$lossia 'choice C) is a key feature of eckwith4 Wie"emann )yn"rome, an over$rowth syn"rome involvin$ a lar$e4si#e" !atient, liver an" ki"ney enlar$ement, hy!erinsulinism, om!halocele an" macro$lossia. &eo!le with %own )yn"rome also have a relative macro$lossia, "ue to a small man"ible an" ma*illa. Om!halocele 'choice %) is a herniation or !rotrusion of ab"ominal contents into the base of the umbilical cor". 7he ab"ominal contents are covere" only with !eritoneum an" no overlyin$ skin. It occurs s!ora"ically in about : in />>> births, or as !art of the eckwith Wie"emann )yn"rome. Imme"iate sur$ical correction before the ab"ominal contents "ry out is the treatment of choice. A sin$le !almar crease 'choice () is a common feature of 7risomy ;: '%own )yn"rome) but also can be seen as an isolate" fin"in$ in many healthy !eo!le who have a normal karyoty!e. 7his is not a feature cause" by maternal "iabetes. ?;) (*!lanation: 7he correct answer is

. 7he si$ns an" sym!toms of con$enital hy!othyroi"ism may not be obvious for several "ays or weeks. )ym!toms inclu"e !hysical slu$$ishness, consti!ation, lar$e ton$ue, umbilical hernia, hy!othermia, bra"ycar"ia, enlar$e" fontanelles, an" !ersistent <aun"ice. )kin may be "ry an" scaly, an" the hair may be "ry, coarse, an" brittle. eckwith4 Wie"emann syn"rome 'choice A) is associate" with hy!o$lycemia, macrosomia, om!halocele, an" macro$lossia. 2urler syn"rome 'choice C) is an autosomal4recessive "isease that !resents with coarse facies 'lar$e ton$ue, flat nasal bri"$e, an" short neck), mental retar"ation, he!atos!lenome$aly, umbilical hernia, corneal clou"in$, an" severe heart "isease. 7he characteristic features of trisomy ;: 'choice %), or %own syn"rome, inclu"e e!icanthal fol"s, u!slantin$ !al!ebral fissures, macro$lossia, flattene" facial !rofile, clino"actyly of fifth fin$ers, sin$le !almar crease, an" hy!otonia. 7urner syn"rome 'choice () is characteri#e" by low4set ears, e*cess nuchal skin, broa" chest, lym!he"ema of han"s an" feet, an" con$enital heart "isease. ?0) (*!lanation: 7he correct answer is %. %ysfunctional uterine blee"in$ is a common cause of menorrha$ia in a"olescent $irls an" is a "ia$nosis of e*clusion. +enstrual4like blee"in$ shoul" be consi"ere" abnormal if it occurs for more than :> "ays. 7he heavy va$inal blee"in$ may occur with e*cessive flow at re$ular menstrual cycle intervals or may occur with heavy blee"in$ at irre$ular intervals. lee"in$ is usually !ainless. 1on Willebran" "isease 'choice A) may first !resent as menorrha$ia in a youn$ a"olescent $irl, without !revious history of e*cess bruisin$ or noseblee"s. One woul" e*!ect that with von Willebran" "isease, the first menses woul" be heavy, but this $irl still nee"s a work4u!. In von Willebran" "isease, the blee"in$ time an" &77 may both be abnormal an" im!ortantly, the vW- 'ristocetin cofactor activity) is "ecrease". An ovarian tumor 'choice ) woul" be unlikely without si$ns an" sym!toms of ab"ominal !ain, mass, or other sym!toms. Althou$h one sym!tom of an ovarian tumor coul" be abnormal va$inal blee"in$, one woul" not e*!ect it to be cyclical. 7hyroi" "isease 'choice C), inclu"in$ hy!othyroi" an" hy!erthyroi" states, may !resent with abnormal va$inal blee"in$.7hyroi" function tests shoul" always be !erforme" as !art of a work4u! for "ysfunctional uterine blee"in$. Althou$h it may seem unlikely that this :>4year4ol" $irl coul" be !re$nant 'choice (), a !re$nancy test shoul" be or"ere". If the $irl were !re$nant, one nee"s be certain that this $irl was not se*ually abuse" or ra!e". Cervical cultures for $onorrhea an" chlamy"ia shoul" be obtaine" as well as tests for sy!hilis, he!atitis , an" 2I1. ??) (*!lanation: 7he correct answer is C. 5awasaki "isease is a systemic vasculitis of unknown ori$in that remains a lea"in$ cause of acquire" heart "isease in infants an" chil"ren. It is a multisystemic "isease also known as mucocutaneous lym!h no"e syn"rome. Clinical an" echocar"io$ra!hic features remain the basis for "ia$nosis. An 'uni"entifie") infectious ori$in an" a 74cell immune

activation !lay a !rominent role in "isease !atho$enesis. 7umor necrosis factor al!ha '7N-4al!ha) rece!tor levels correlate with the "e$ree of vascular "ama$e an" likelihoo" of coronary artery aneurysm formation. 7he car"iovascular com!lications account for most of the morbi"ity an" mortality. -ever, bilateral non4e*u"ative con<unctivitis, mucous membrane chan$es 'in<ecte" !haryn*, cracke" li!s, or strawberry ton$ue), e*tremity chan$es 'e"ema, "esquamation, erythema, or rash), an" cervical a"eno!athy are common at !resentation. 7he acute manifestations inclu"e myocar"itis, valvular insufficiency, arrhythmias, !ericar"ial effusion, an" con$estive heart failure with coronary abnormalitiesG these "evelo! in :/= to ;/= of !atients. ,eukocytosis an" an elevate" C4reactive !rotein are associate" with the "evelo!ment of coronary artery aneurysms. 7reatment inclu"es as!irin at @>4:>> m$Bk$B"ay in "ivi"e" "oses an" I1 immune $lobulin in hi$h "oses. Corticosteroi"s shoul" be avoi"e" because of the !otential link with increasin$ the likelihoo" of coronary aneurysm "evelo!ment. ?/) (*!lanation: 7he correct answer is (. 7he fin"in$s of "ry mucous membranes, !oor skin tur$or, an" tachycar"ia su$$est that the infant is volume "e!lete". 2e has been takin$ the loo! "iuretic furosemi"e, which is use" to treat broncho!ulmonary "ys!lasia. -urosemi"e causes increase" e*cretion of so"ium 'therefore water), !otassium, an" chlori"e in the urine. 7he bo"y is therefore hy!ovolemic. (*cessive loss of !otassium causes the hy"ro$en4!otassium !um! across the cell membranes to trans!ort hy"ro$en into the cells in e*chan$e for !otassium out of the cells. 7rans!ortation of hy"ro$en into the cells causes the number of hy"ro$en ions in the !lasma to "ecrease an" results in alkalosis. 7he chemorece!tors in the me"ullary res!iratory center of the brain sense the metabolic alkalosis an" res!on" by lowerin$ the res!iratory rate. 7herefore, more carbon "io*i"e is retaine" in the bloo"stream an" !artially corrects the metabolic alkalosis. artter syn"rome 'choice A) is a rare autosomal4recessive "isor"er that manifests as hy!okalemia, hy!ochloremia, an" hi$h renin an" al"osterone levels. &rimary al"osteronism 'choice ) is characteri#e" by hy!ertension, hy!ernatremia, hy!okalemia, an" a su!!resse" renin4an$iotensin system. &rimary res!iratory aci"osis with metabolic com!ensation 'choice C) re!resents a !rimary !rocess of res!iratory failure an" increase" retention of carbon "io*i"e. 7he ki"neys com!ensate by retention of bicarbonate in an attem!t to correct the aci"osis. 2owever, com!ensatory mechanisms will never overcom!ensate for the !rimary !rocess. 7herefore, the !2 shoul" never be more than D.?>. &seu"ohy!eral"osteronism 'choice %), also known as the ,i""le syn"rome, is a rare "isor"er of renal trans!ort of so"ium an" !otassium that resembles !rimary hy!eral"osteronism. Affecte" !eo!le !resent in infancy or early chil"hoo" with hy!ertension, !olyuria, !oly"i!sia, an" hy!okalemic metabolic alkalosis. 7he serum concentration of al"osterone is low. ?C) (*!lanation: 7he correct answer is

(. -orei$n bo"ies, often stool or toilet !a!er, are the most common cause of va$initis in a ?4year4ol" $irl. 7hese ob<ects can cause a bloo"y "ischar$e that may be associate" with few other sym!toms. 2owever, a va$initis may "evelo! secon"ary to the forei$n ob<ect an" this is often characteri#e" by inflammation an" erythema of the labia ma<ora. 7he inflammation an" erythema may be com!oun"e" by the chil"As scratchin$, which will lea" to e*coriations. %ysuria may also be a !resentin$ com!laint, as the aci"ic urine hittin$ the inflame" va$ina an" vulva causes sym!toms. +ost forei$n bo"ies can be remove" by irri$atin$ the va$ina with normal saline attache" to a narrow catheter. 2owever, if a forei$n bo"y is sus!ecte" an" removal with $entle irri$ation is unsuccessful, then the chil" shoul" be brou$ht to the o!eratin$ room for an e*amination un"er anesthesia. Ob<ects that have been in the va$ina for a lon$ time may actually ero"e into 'an" even throu$h) the va$inal wall. 7hese ob<ects shoul" therefore be remove" with $reat care an" un"er "irect visuali#ation. 7he inflammation of the vulvar an" va$inal mucosa may be treate" with to!ical estro$en cream for : week. ,ichen sclerosis 'choice A) is characteri#e" by white, atro!hic, !archment4like skin on the vulva. If the !erianal area is involve", as well as the labia, the lesion may have an hour$lass confi$uration. &elvic inflammatory "isease 'choice ) is hi$hly unlikely in a ?4year4ol" $irl an" woul" rarely !resent with e*ternal fin"in$s only. )arcoma botyroi"es 'embryonal rhab"omyosarcoma) 'choice C) is an e*tremely rare mali$nancy. 2owever, it is the most common mali$nancy of the lower $enital tract in the very youn$ $irl. It often !resents as a bloo"y va$inal "ischar$e an" a!!ears as friable, $ra!elike masses !rotru"in$ throu$h the urethra or va$ina. 7herefore, any va$inal ta$ in a youn$ $irl shoul" not be !resume" to be beni$n. 9oun$ chil"ren who have been victims of se*ual abuse 'choice %) usually have no si$ns of $enital in<ury when they are first e*amine". 2owever, it is essential to consi"er this "ia$nosis whenever vulvar, va$inal, or anal com!laints arise in youn$ $irls. ?D) (*!lanation: 7he correct answer is . A hy"rocele is a flui"4fille" sac in the scrotum that is non4!ainful an" transilluminates on e*amination. 2ematoma 'choice A) woul" be cause" by trauma to the area. 7his shoul" result in a "iscoloration of the scrotal sac an" ten"erness to !al!ation. In$uinal hernias 'choice C) often !rotru"e throu$h the in$uinal rin$ an" shoul" be felt on e*amination. 7esticular torsion 'choice %) woul" !resent as a !ainful testicular mass an" a "iscolore" scrotal sac. 7esticular tumor 'choice () woul" be rare in a /4month ol" an" woul" most likely be a soli" tumor an" not transilluminate. ?@) (*!lanation: 7he correct answer is C. 7he constellation of Hwaist4"ownH !ur!ura, arthritis, ab"ominal !ain an" nonthrombocyto!enic !ur!ura in a youn$ female is classic for 2enoch4)chKnlein syn"rome or 2)&. 7he rash of 2)& is characteri#e" by !etechiae which become !ur!ura. A Hviral illnessH weeks before, is a common antece"ent to 2)&. Checkin$ the !atientAs

urine for hematuria is im!ortant in follow4u! visits as the !atient can "evelo! en"4sta$e renal failure in := of cases. 7he un"erlyin$ !atho!hysiolo$y is relate" to vasculitis of small bloo" vessels. Frinary co!!er 'choice A) is elevate" in WilsonAs "isease or he!atolenticular "e$eneration. .lucosuria 'choice ) is seen in "iabetic !atients. White bloo" cell casts 'choice %) are clusters of leukocytes in the sha!e of the renal tubules, an" are seen in !yelone!hritis. 9east 'choice () is seen in !atients on antibiotics or who are immunocom!romise". ?I) (*!lanation: 7he correct answer is .2aemo!hilus influen#ae is now a rare cause of menin$itis in chil"ren since "evelo!ment of the 2aemo!hilus influen#ae ty!e b '2ib) vaccine. 7he case reveals a questionable immuni#ation history, thus makin$ this !atient susce!tible to 2. influen#ae ty!e b. 5erni$As si$n is elicite" by !lacin$ the !atient in a su!ine !osition, fle*in$ the le$ at the hi! to I> "e$rees an" then strai$htenin$ the knee to elicit !ain in the back or !osterior thi$h as !re"ictive evi"ence of menin$itis. ,aboratory an" C)- "ata su!!ort a bacterial etiolo$y, an" .ramAs stain with $rowth on chocolate a$ar confirms the "ia$nosis of 2aemo!hilus influen#ae as the causative a$ent. 2aemo!hilus "ucreyi'choice A) is the causative a$ent for chancroi" 'soft chancre). Neisseria menin$iti"is'choice C) is a $ram4 ne$ative "i!lococcus that also can $row on chocolate a$ar, but $rows best on mo"ifie" 7hayer4+artin me"ia. +enin$ococcal menin$itis classically !resents with a !etechial rash. )tre!tococcus !neumoniae'choice () an" ,isteria monocyto$enes'choice %) are both or$anisms that cause menin$itis. oth, however, are $ram4!ositive. ,isteria monocyto$enes is !re"ominantly seen as a cause of neonatal menin$itis. />) (*!lanation: 7he correct answer is A. acterial va$inosis ' 1) is characteri#e" by e*cessive "ischar$e an" o"or. It is the most common cause of va$initis in women of chil"bearin$ a$e. Althou$h 1 is seen !re"ominantly in se*ually active !remeno!ausal women, it "oes not a!!ear to be closely associate" with se*ual activity an" is not consi"ere" a se*ually transmitte" "isease. -or many years, the e*act microbiolo$ic cause of this infection was unknownG however, it is now re$ar"e" as a result of the syner$ism amon$ various bacteria, inclu"in$ .ar"nerella va$inalis, anaerobic $ram4ne$ative ro"s, &e!tostre!tococcus s!ecies, +yco!lasma hominis, Frea!lasma urealyticum, an" +obiluncus s!ecies. .. va$inalis is foun" as !art of the va$inal flora in a!!ro*imately one thir" of se*ually active women. Without a critical concentration of other va$inal bacteria, !articularly anaerobes, this or$anism is not thou$ht to be res!onsible for sym!toms. Or$anic aci" metabolites of anaerobes !re"ominate in these s!ecimens, an" the resolution of sym!toms correlates with the a!!earance of metabolites !ro"uce" by ,actobacillus an" stre!tococcal s!ecies. 7he re!lacement of lactobacilli by .. va$inalis, anaerobes, +obiluncus, an" $enital myco!lasmas is a characteristic feature of bacterial va$inosis. &atients with 1 may have

a variety of sym!toms or none at all. As many as />= may be asym!tomatic, whereas />= to D>= com!lain of an un!leasant, fishy or musty va$inal o"or. 7hey also re!ort increase" va$inal "ischar$e. 7he onset of o"or an" "ischar$e associate" with 1 is evenly "istribute" throu$hout the menstrual cycle, an" local "iscomfort is rarely a !roblem. 7he "ia$nosis of 1 is ma"e by e*amination of the !atient an" the "ischar$e. 7he "ischar$e is ty!ically homo$eneous, $rayish white to yellowish white, an" !artially a"herent to the vulva an" va$inal walls. Fn"erlyin$ e"ema or erythema of the vulva or va$ina is aty!ical. 7he !2 of this va$inal "ischar$e is $reater than ?.D. When one "ro! of 5O2 is a""e" to one "ro! of "ischar$e, an intense amine o"or !ro"uces a !ositive whiff test result. 7y!ically, on normal saline wet !re!aration, this "ischar$e has a !aucity of leukocytes an" a !re"ominance of clue cells, !ro"uce" by the a"herence of .. va$inalis to e!ithelial cells. y wet !re!aration, clue cells are best i"entifie" by a sti!!le" birefrin$ence that so "ensely covers the e!ithelial cell that the normal bor"ers an" nuclei are obscure". y .ramAs stain, clue cells can be i"entifie" as e!ithelial cells almost totally covere" by small $ram4ne$ative ro"s. A !aucity of other or$anisms can be seen in the back$roun". 7he sin$le most reliable si$n is the !resence of clue cells. Culture on a s!ecific me"ium is rarely useful. 14associate" or$anisms may be !art of the normal va$inal flora, althou$h concentrations are ty!ically hi$her when 1 is !resent. 7he most effective treatment for 1 is metroni"a#ole. Increasin$ e*!erience su$$ests that va$inal thera!y with metroni"a#ole $el or clin"amycin cream is also effective. 7he risk factors in recurrence are incom!letely un"erstoo", but 1 may "evelo! more often in users of "ia!hra$ms. .ram4ne$ative "i!lococci 'choice ) are seen in $onorrhea. Women with $onorrhea may com!lain of muco!urulent va$inal "ischar$e, "ys!areunia, an" "ysuria. ,actobacilli 'choice C) are normal va$inal flora. 7hey are not seen in !atients with active 1. &seu"ohy!hae 'choice %) are seen in the !otassium hy"ro*i"e !re!aration of a va$inal "ischar$e of !atients with can"i"iasis. Can"i"iasis !resents with itchin$, irritation, an" "ys!areunia. %ischar$e is white an" clum!y. 7richomona"s 'choice () are seen in trichomoniasis, which !resents with a co!ious, !oolin$, an" frothy $ray4white or yellow4$reen "ischar$e. An itchin$, strawberry cervi* is occasionally seen. )aline !re!aration of the "ischar$e reveals motile trichomona"s.

USMLE Step 2 Practice Test Block 1& :ame; +nstr#ctions; Ans'er the *#estions $elo' to the $est of yo#r a$ility. =hen yo# finish the test% click the 2heck $#tton at the $ottom to )ie' the res#lts.

1.A 1444!g infant% $orn at & 'eeks, gestation% is 42 cm in length an" a has a hea" circ#mference of 21 cm. >ne "ay after $irth% she $ecomes )ery irrita$le% trem#lo#s% an" inconsola$le. /er cry is high!pitche". /er p#lse is 1(47min. There are no "ysmorphic facial feat#res. To 'hich of the follo'ing s#$stances 'as this ne'$orn most likely e.pose" in #teroa6 Alcohol $6Bar$it#rates c62ocaine "6Mari9#ana e6 >piates :ormal La$s 2.A 14!year!ol" $oy is $ro#ght to the emergency room $eca#se of persistent mi"epigastric pain for t'o "ays. The pain is getting no $etter% yet it is no 'orse% an" ra"iates to his $ack. The $oy also has ha" fe)er% as 'ell as na#sea an" )omiting that is 'orse 'hen his temperat#re rises in the afternoon. >n e.amination% his temperat#re is &8.2 2 3142. 56 an" there is marke" #pper a$"ominal ten"erness 'ith g#ar"ing. Mil" a$"ominal "istention is present 'ith no a#"i$le $o'el so#n"s. A complete $loo" co#nt re)eals an ele)ate" le#kocyte co#nt an" a normal ser#m amylase. =hich of the follo'ing is the most likely "iagnosisa65itD!/#gh!2#rtis syn"rome $6+nt#ss#sception c6=ilms t#mor "6Pancreatitis

e6Pyelonephritis :ormal La$s &.A $a$y is $orn at &4 'eeks gestation. The amniotic fl#i" is $ro'n an" m#rky. The $a$y has lo' APCAA scores an" appears to $e septic% 'ith lethargy% apnea% $ra"ycar"ia% an" temperat#re insta$ility. The mother li)es on a farm an" gi)es a history of a fl#!like illness one month $efore "eli)ery. Cram,s stain of a smear from the mother,s cer)i. "emonstrates a$#n"ant% pleomorphic% gram!)aria$le cocco$acillary forms. =hich of the follo'ing is the most likely "iagnosisa62ongenital cytomegalo)ir#s infection $62ongenital r#$ella c62ongenital syphilis "6:eonatal herpes simple. infection e6:eonatal listeriosis :ormal La$s 4.An infant is $orn premat#rely an" is small for gestational age. At $irth% the infant is o$)io#sly ill 'ith 9a#n"ice% fe)er% hepatosplenomegaly% myocar"itis% an" rashes. :e#rologic in)ol)ement is prominent% 'ith hy"rocephal#s% intracranial calcifications% an" seiD#res. The mother has a cat an" contin#e" to clean the cat,s litter $o. "#ring the pregnancy. =hich of the follo'ing is the most likely ca#sati)e agenta6 2ytomegalo)ir#s $6/erpes simple. c6A#$ella )ir#s "6To.oplasma e6Treponema palli"#m :ormal La$s .A 8!month!ol" infant is seen in the pe"iatrician,s office $eca#se of fail#re to gain 'eight. /er length an" 'eight are $oth $elo' the th percentile at her age. The patient,s chart in"icates that at the age of age of < months% her length an" 'eight 'ere at the 4th percentile. A caref#l history re)eals that the mother ret#rne" to 'ork 'hen the infant 'as <

months ol"% an" the gran"mother ass#me" the care of the infant most of the time since then. She is recei)ing <!1 oD of iron!fortifie"% co' protein! $ase" form#la e)ery 4 ho#rs. =hich of the follo'ing is the $est initial step in the management of this infanta6Ask ho' the form#la is mi.e" $6>$tain a "etaile" family history for lactose intolerance c6>$tain a stool specimen "6>$tain a s'eat chlori"e test e6>$tain a #rinalysis :ormal La$s <. A &4!ho#r!ol" infant has not passe" meconi#m since $irth. /e 'as f#ll term 'ith a $irth 'eight of &1 < g 31 l$ 1 oD6. The pregnancy 'as #ncomplicate". The $a$y appears 'ell 'ith no respiratory "istress. Slight a$"ominal "istention is note". Aectal e.amination re)eals a slightly tight rect#m an" res#lts in a greenish g#sh of stool. =hich of the follo'ing tests 'ill pro$a$ly confirm the likely "iagnosisa6A stool c#lt#re $6 A rectal $iopsy c6A $ari#m enema "6An alpha1!antitrypsin le)el e6A ser#m TS/ le)el :ormal La$s (. A (!year!ol" $oy presents to the physician,s office 'ith a &!'eek history of left!si"e" anterior cer)ical lymph no"e enlargement. The enlarge" no"es are not ten"er to palpation. A fe' pap#les "e)elope" on the left forearm at the onset of lympha"enopathy. The $oy has a kitten at home. =hich of the follo'ing is the most likely organism ca#sing lympha"enopathy in this chil"a6Actinomyces israelii $6Bartonella henselae c65rancisella t#larensis "6Myco$acteri#m t#$erc#losis e6Staphylococc#s a#re#s :ormal La$s

1.A 4!year!ol" $oy is $ro#ght to the emergency "epartment for a painf#l an" s'ollen right forearm. /e 'as $itten an" scratche" $y a family cat 2 "ays ago in the affecte" area. /is temperat#re is &8.< 2 314&.2 56. The right forearm is erythemato#s% e"emato#s% an" ten"er to to#ch. =hich of the follo'ing is the most appropriate anti$iotic treatment for this patienta6Ampicillin $6Amo.icillin!cla)#lanate c62lin"amycin "6 Tetracycline e6Trimethoprim!s#lfametho.aDole :ormal La$s 8.A 14!year!ol" male presents 'ith a complaint of soreness% an" 'eakness in his legs for the past "ay that has slo'ly sprea" from his cal)es to his thighs. /e no' complains of 'eakness in his tr#nk an" arms. >n e.amination he appears tire" an" lays on the e.amining ta$le. /is temperat#re is &( 2 381.< 56% p#lse is 417min% an" respirations are 227min. Both of his legs are "iff#sely ten"er. @eep ten"on refle.es are a$sent in the lo'er e.tremities% an" sensation is greatly "iminishe". =hich of the follo'ing st#"ies is essential for this patient,s "iagnosisa62reatinine phosphokinase le)els $6Stool c#lt#re for 2ampylo$acter 9e9#ni c6Motor ner)e con"#ction test "62ere$rospinal fl#i" st#"ies e6M#scle $iopsy :ormal La$s 14.A 14!year!ol" $oy is hit $y an a#tomo$ile 'hile 'alking across the street an" is imme"iately taken to the emergency "epartment. >n arri)al% he is conscio#s an" complains of shortness of $reath an" chest pain. Physical e.amination re)eals an ecchymotic area o)er his right chest an" s#$c#taneo#s emphysema. Breath so#n"s are a$sent on the right si"e. /is trachea is "e)iate" to the left% an" his right hemithora. is tympanic to perc#ssion. =hich of the follo'ing is the most appropriate initial step in management of this patienta612!lea" E2C

$62T of the chest c6Plain ra"iography of the chest "62hest t#$e thoracostomy e6 Pericar"iocentesis :ormal La$s

Note: Check your own answers before hittin$ the Check button below. When you click the Check button, a browser win"ow will a!!ear that contains a summary of your results. (*!lanations lock :0 (*!lanations

:) (*!lanation: 7he correct answer is C. 7he most commonly abuse" "ru$ by !re$nant mothers is cocaine. Infants are usually small for $estational a$e ').A) an" sometimes have microce!haly an" neuro"evelo!mental abnormalities. (*!ose" infants are very irritable an" inconsolable in the with"rawal !erio". 7heir cries are often hi$h4!itche". 7hey are also at increase" risk of su""en infant "eath syn"rome ')I%)). &eriventricular leukomalacia 'a CN) ischemic lesion) is also associate" with cocaine e*!osure. Alcohol 'choice A) abuse "urin$ !re$nancy causes fetal alcohol syn"rome. It is characteri#e" by failure to thrive, car"iac "efects, facial "ysmor!hic features 'such as narrow forehea", micro!hthalmia, short !al!ebral fissures, an" micro$nathia), an" neurolo$ic abnormalities. arbiturate 'choice ) use "urin$ !re$nancy causes the infant to have limb anomalies, mental retar"ation, nail hy!o!lasia, an" some "ysmor!hic features, such as a short nose an" a low nasal bri"$e. It is not usually associate" with low birth wei$ht an" microce!haly. 7here is no clear evi"ence that mari<uana 'choice %) use "urin$ !re$nancy is associate" with any terato$enic effects on the infant in humans. In animal stu"ies, however, it has been associate" with fetal $rowth restriction an" terato$enesis. O!iate 'choice () abuse "urin$ !re$nancy is associate" with a hi$h inci"ence of obstetric com!lications, such as !lacental abru!tion, !reterm labor, an" fetal $rowth restriction. 7remors, irritability, vomitin$, an" "iarrhea !resent in the neonatal !erio". ;) (*!lanation: 7he correct answer is %. &ancreatitis is not always an easy "ia$nosis. 7he history of this boy is !articularly sus!ect, even with a normal serum amylase. Nearly one4thir" of all !atients with acute !ancreatitis have a normal serum amylase. An ab"ominal ultrasoun" may be useful in revealin$ an enlar$e" !ancreas. 7his youn$ man is unlikely to have -it#42u$h4Curtis syn"rome 'choice A). Classically, -it#42u$h4Curtis syn"rome is an e*tra!elvic manifestation of !elvic inflammatory "isease in se*ually active women. It is a !erihe!atitis consistin$ of a"hesions between the liver ca!sule an" the "ia!hra$m or the anterior !eritoneal surface. It was ori$inally thou$ht to be cause" solely by Neisseria $onorrhoeae, but recent stu"ies have shown that Chlamy"ia trachomatis an" other or$anisms may also be etiolo$ies. 7he -it#42u$h4Curtis syn"rome has rarely been

re!orte" in men, so a se*ual history shoul" be obtaine" in this case. Intussusce!tion 'choice ) is not hi$h on the list of "ifferential "ia$noses. 7he ty!ical a$e for intussusce!tion is in infancy, es!ecially C to :; months of a$e. 7his youn$ man has !ersistent ab"ominal !ain with fever, makin$ this an unlikely "ia$nosis. Wilms tumor 'choice C) is also unlikely in this case scenario. Wilms tumor usually occurs in early chil"hoo", a$e ; or 0 years. Althou$h Wilms tumor is more !revalent in males, the usual !resentin$ si$ns inclu"e an asym!tomatic ab"ominal mass an" hematuria. &yelone!hritis 'choice () ty!ically !resents with hi$h fever, chills, an" back !ain. Althou$h a urinalysis shoul" be !erforme" in this youn$ man, the sym!toms of mi"e!i$astric !ain su$$est an u!!er $astrointestinal !roblem, not !yelone!hritis. 0) (*!lanation: 7he correct answer is (.,isteria monocyto$enes, the causative a$ent of listeriosis, can be acquire" by mothers e*!ose" to un!asteuri#e" "airy !ro"ucts or raw ve$etables e*!ose" to cattle or shee! manure. 7he con"ition can cause a febrile, flu4like "isease in the mother. 7he affects on the fetus an" newborn "e!en" on the timin$ of the infection. 7he bacteria have a !re"ilection for causin$ amnionitis, which may then !ro"uce abortion, stillbirth, or neonatal se!sis. 7he !resence of brown, murky amniotic flui" may be a hel!ful "ia$nostic clue 'an" a clue on test questions). %isseminate" "isease in the fetus can cause $ranuloma formation 'with associate" tissue "estruction) in many tissues, inclu"in$ liver, a"renal $lan"s, lym!hatic tissue, lun$s, an" brainG the term $ranulomatosis infantise!tica is sometimes use" in these cases. 7he mortality rate in affecte" infants ran$es from :> to />= 'hi$her in early onset "isease). )ome authors recommen" bloo" an" cervical culture in !re$nant women who have a !otential e*!osure an" "evelo! a flu4like illness. Con$enital cytome$alovirus infection 'choice A) can cause asym!tomatic to mil" to severe "isease in neonatesG look for a reference to flu4like sym!toms "urin$ !re$nancy in the question stem. Another clue with some s!ecificity 'also !resent in to*o!lasmosis) is a reference to !eriventricular calcifications. 'A $oo" ne$ative clue is no reference to cats, which woul" have su$$este" to*o!lasmosis.) Con$enital rubella 'choice ) can cause multi!le severe !roblems in infantsG look for a reference to the mother havin$ ha" a rash "urin$ !re$nancy. Con$enital sy!hilis 'choice C) can also cause "evastatin$ "isease in newbornsG look for references to rash involvin$ !alms an" soles, 2utchinsonAs molars, or bone "eformities, inclu"in$ saber shin. Neonatal her!es sim!le* infection 'choice %) can be a "evastatin$ infection of the neonateG look for skin vesicles an" often !rominent neurolo$ic involvement. ?) (*!lanation: 7he correct answer is %. 7his is con$enital to*o!lasmosis. It can look like other severe con$enital infections, so a very hel!ful clue is that there will usually be a reference to Hcat fecesH in the question stem, which is the usual clinical source of the infection. Affecte" babies can be

asym!tomatic or can have severe, "isseminate" "isease which may manifest as $rowth retar"ation, <aun"ice, he!atos!lenome$aly, myocar"itis, !neumonitis, rash, or neurolo$ic involvement 'hy"roce!halus, microce!haly, intracranial calcifications, sei#ures, chorioretinitis). )everely affecte" babies also have thrombocyto!enia, lym!hocytosis, monocytosis, an" elevate" eosino!hils. 7reatment is with combination antibiotic thera!y, inclu"in$ !yrimethamine, sulfa"ia#ine, an" leucovorin. Cytome$alovirus 'choice A) can cause con$enital an" !erinatal infections that can be asym!tomatic or can cause intrauterine $rowth retar"ation, microce!haly, <aun"ice, he!atos!lenome$aly, !eriventricular calcifications, chorioretinitis, an" !neumonitis. 2er!es sim!le* 'choice ) can cause con$enital an" !erinatal infection, which may cause skin vesicles, "isseminate" "isease, an" neurolo$ic "isease. 6ubella virus 'choice C) can !ro"uce con$enital "isease that can cause fetal "eath or $rowth retar"ation, ence!halitis, cataracts, car"iac "efects, hearin$ "efects, an" he!atos!lenome$aly. 7re!onema !alli"um 'choice (), the causative a$ent of sy!hilis, can cause rash, lym!ha"eno!athy, he!atos!lenome$aly, $ummas of the face, bony lesions, an" neurosy!hilis in infants. /) (*!lanation: 7he correct answer is A. 7his I4month4ol" infant !resents with failure to thrive, which is "etermine" by the "ecline in her len$th an" wei$ht to below the /th !ercentile. -ailure to thrive can be either or$anic or inor$anic 'i.e., social). Althou$h it is im!ortant to i"entify the causes of or$anic failure to thrive, most cases are inor$anic. In this clinical vi$nette, since the re"uction of wei$ht ha!!ens after the mother returne" to work, an inor$anic cause is likely. One of the most common causes of failure to thrive is im!ro!er !re!aration of the formula, resultin$ either from an incorrect water4to4formula ratio or from !oor mi*in$ techniques. In a""ition to obtainin$ a "etaile" history of how the formula is mi*e", it mi$ht be very useful to have the caretaker actually "emonstrate how he or she !re!ares the formula. In this case, the most likely reason that the infant has failure to thrive is that the $ran"mother has im!ro!erly mi*e" the formula. ,actose intolerance 'choice ) is an uncommon cause of failure to thrive. It usually !resents with ab"ominal !ain, bloatin$, an" "iarrhea. Obtainin$ a stool s!ecimen 'choice C), sweat chlori"e test 'choice %), or urinalysis 'choice () mi$ht be hel!ful if no i"entifiable inor$anic causes of failure to thrive are i"entifie". Frinalysis is hel!ful in screenin$ for renal "isease. A sweat chlori"e test is use" to "etect cystic fibrosis. )tool s!ecimens can be useful in a $reat variety of $astrointestinal "isor"ers, such as $astroenteritis, !arasitic infection, an" fat malabsor!tion. C) (*!lanation: 7he correct answer is . 2irschs!run$ "isease or con$enital a$an$lionic me$acolon is cause" by a con$enital absence of the $an$lion cells of both the +eissner an" Auerbach !le*uses. It is the most common cause of lower intestinal obstruction in the neonatal !erio". In early chil"hoo" it

may !resent as chronic consti!ation with intermittent fecal soilin$. It occurs !re"ominantly in males an" there is an increase" family inci"ence. )ur$ical treatment is in"icate", but the "ia$nosis is confirme" by a suction bio!sy that can be easily !erforme" without $eneral anesthesia. 7he bio!sy woul" reveal an absence of $an$lion cells in the submucsal an" myenteric !le*uses. A stool culture 'choice A) woul" be !erforme" if one was entertainin$ a bacterial cause of $astroenteritis, es!ecially in a hos!itali#e" !atient. 2owever vomitin$, "iarrhea, an" ab"ominal "istention in a newborn are unlikely to be cause" by $astroenteritis, es!ecially in this case with a ti$ht s!hincter note" on rectal e*amination. A barium enema 'choice C) may be in"icate" in sus!ecte" cases of 2irschs!run$ "isease, but it is the bio!sy that makes the "ia$nosis. 7he barium enema in this case of 2irschs!run$ "isease reveale" a "ilate" !ro*imal bowel with evi"ence of a contracte" "istal rectum. An al!ha:4antitry!sin level 'choice %) woul" not be in"icate" in this case. It is obtaine" when one sus!ects an al!ha:4antitry!sin "eficiency. Affecte" infants woul" !resent with <aun"ice, acholic stools, an" he!atome$aly. A serum 7)2 'choice () woul" be !erforme" if a newborn infant were thou$ht to have hy!othyroi"ism. In the F.)., most states have man"atory newborn screenin$ for thyroi" "isease. -requently, con$enital hy!othyroi"ism is asym!tomatic, but it may !resent with sym!toms of consti!ation, lethar$y, !oor fee"in$, mottlin$, an" !rolon$e" <aun"ice. 7he ty!ical features in this case are more su$$estive of 2irschs!run$ "isease. D) (*!lanation: 7he correct answer is . artonella henselae is the !atho$en of cat4scratch "isease, which is a very common cause of chronic lym!ha"enitis in chil"ren. It ty!ically !resents as an enlar$in$ non4 ten"er lym!h no"e, often locate" in the cervical, a*illary, or in$uinal re$ions. After bein$ scratche" by a kitten, a !a!ule "evelo!s at the scratch site. Affecte" lym!h no"es "rainin$ the involve" area become enlar$e" in ; weeks. Other sym!toms inclu"e low4 $ra"e fever, malaise, fati$ue an" nons!ecific bo"y aches. 7he "ia$nosis of cat4scratch "isease is best ma"e by a serolo$ical test, such as an in"irect fluorescence antibo"y test for antibo"ies to . henselae. Fsually, treatment is necessary only in severe systemic infection. Actinomyces israelii'choice A) rarely causes anterior cervical a"eno!athy. -rancisella tularensis'choice C) is the !atho$en for tularemia, which is characteri#e" by an ulcerative lesion at the site of inoculation, with re$ional lym!ha"eno!athy that is e*quisitely ten"er. +ycobacterium tuberculosis 'choice %) most often causes !ulmonary "isease. A skin !urifie" !rotein "erivative '&&%) test is usually !ositive. )ta!hylococcus aureus'choice () causes acute lym!ha"enitis that is associate" with fever an" ten"er lym!h no"es. @) (*!lanation: 7he correct answer is . Animal bite is a common !roblem in !e"iatrics. In this case, the !atient was bitten by a cat. 7o initiate a!!ro!riate antibiotic treatment, one nee"s to un"erstan" which or$anisms

are most likely causin$ this infection. In a cat4bite woun", the most common or$anisms isolate" are &asteurella multoci"a an" )ta!hylococcus aureus. Amon$ the choices, amo*icillin4clavulanate is the only antibiotic that is effective a$ainst both or$anisms. &. multoci"a infection usually manifests within ;?4?@ hours followin$ the bite or scratch as locali#e" swellin$, erythema, ten"erness, an" serous or san$uino!urulent "ischar$e. -ever, chills, an" lym!ha"eno!athy can also occur. Com!lications inclu"e tenosynovitis, osteomyelitis, an" se!tic arthritis. &. multoci"a is foun" in the oral flora of D>= to I>= of cats, ;/= to />= of "o$s, an" a variable !ercenta$e in other animals. ). aureus is a common !atho$en for cellulitis. It is often foun" on the skin. In a severe case of animal bite, the !atho$ens are usually !olymicrobial. 7herefore, amo*icillin4clavulanate is the treatment of choiceG D4:> "ays thera!y is usually sufficient. Am!icillin 'choice A) is an effective treatment a$ainst &. multoci"a, but it has no effect on ). aureus. Clin"amycin 'choice C) is effective a$ainst most seroty!es of ). aureus, but is ineffective a$ainst &. multoci"a. 7etracycline 'choice %) is effective a$ain &. multoci"a, but it shoul" not be $iven to chil"ren youn$er than @ years. 7rimetho!rim4sulfametho*a#ole 'choice () is not a"equate on its own. When combine" with clin"amycin, however, it is an alternative treatment if the !atient is aller$ic to !enicillin com!oun"s.

I) (*!lanation: 7he correct answer is . .uillain4 arrV syn"rome '. )) is a !ostinfectious !olyneuro!athy that causes "emyelination of O72 motor an" occasionally, sensory nerves. It is classically an ascen"in$ !aralysis. C)- stu"ies are essential for "ia$nosis an" reveal a !rotein level usually twice normal values but with normal amounts of white bloo" cells, normal $lucose level an" an absence of !leocytosis 'elevate" lym!hocytes). Autonomic nervous system involvement can !ro"uce the bra"ycar"ia seen in this vi$nette. Creatinine !hos!hokinase levels 'choice A) may be mil"ly elevate" an" sometimes are normal, but are not essential for "ia$nosis. +otor nerve con"uction tests 'choice C) woul" show "ecrease" velocities, but are not s!ecific for . ). A muscle bio!sy 'choice %) is not in"icate" an" can be normal in early sta$es. ,ate "isease reveals "enervation atro!hy. )tool culture for Cam!ylobacter <e<uni'choice (), a well reco$ni#e" infection associate" with . ), is a$ain not essential for "ia$nosis. y the time the "isease !resents stool cultures are often ne$ative. :>) (*!lanation: 7he correct answer is . 7he fin"in$s on !hysical e*amination stron$ly su$$est tension !neumothora*. 7his is a life4threatenin$ emer$ency that nee"s to be mana$e" imme"iately with either chest tube thoracostomy or nee"le thoracocentesis to relieve the tension on the affecte" si"e of the thora*. &hysical e*amination reveals tachycar"ia, tachy!nea, "ecrease" or absent breath soun"s over the involve" hemithora*, increase" resonance to !ercussion, subcutaneous em!hysema, an" "eviation of the trachea to the o!!osite si"e. 7ension !neumothora*

"evelo!s when air leakin$ into the chest increases intrathoracic !ressure, com!letely colla!sin$ the lun$ on that si"e. It results in "is!lacement of the me"iastinum an" trachea to the o!!osite si"e of the chest an" im!e"es venous return. :;4lea" (C. 'choice A), C7 of the chest 'choice C), an" !lain ra"io$ra!hy of the chest 'choice () may be in"icate" on a trauma !atient after the !atient is stabili#e". In tension !neumothora*, no ima$in$ stu"y shoul" !rece"e the emer$ent relief of tension insi"e the chest. &ericar"iocentesis 'choice %) is in"icate" when there is car"iac tam!ona"e, cause" by buil"4u! of flui" in the !ericar"ium. Car"iac tam!ona"e si$nificantly affects ventricular rela*ation an" marke"ly "ecreases car"iac out!ut.

USMLE Step 2 Practice Test Block 14 :ame; +nstr#ctions; Ans'er the *#estions $elo' to the $est of yo#r a$ility. =hen yo# finish the test% click the 2heck $#tton at the $ottom to )ie' the res#lts.

1.A &2!year!ol" 'oman is $ro#ght to the emergency "epartment $y police for psychiatric e)al#ation. The officers fo#n" her in the street% nake" an" mast#r$ating. She "enies any recent "r#g #se. She is% ho'e)er% )ery irrita$le an" has press#re" speech. =hen *#estione" f#rther% she states that she has ha" se. 'ith 14 "ifferent men in the past & "ays% $eca#se she felt that she 'as 0too $ea#tif#l to not share0 her se.#ality. =hich of the follo'ing is the most likely "iagnosisa6 @ysthymic "isor"er $6/eroin a$#se c6 Mania "6Partial comple. seiD#res e6SchiDophrenia :ormal La$s 2.A 4!year!ol" man presents to an emergency clinic complaining of the onset% o)er the past "ay% of $ilateral tremor in his han"s% "iaphoresis% an.iety% hea"ache% an" the sensation that 0my skin is cra'ling0. /e "enies other symptoms. /is me"ical history is significant only for hypertension% for 'hich he takes hy"rochlorothiaDi"e. /e states that he sees a psychiatrist for $ipolar "isor"er an" an.iety% an" that he takes three me"ications prescri$e" $y the psychiatrist% the names of 'hich he can not remem$er. /e ran o#t of his me"ications three "ays ago% an" he has an appointment for his primary care physician an" his psychiatrist tomorro'. /is temperat#re is &( 2 381.< 56% $loo" press#re is 1 47144 mm /g%

p#lse is 11 7min% an" respirations are 247min. Physical e.amination is nota$le for "iaphoresis an" trem#lo#sness. A"ministration of 'hich of the follo'ing is the most appropriate initial step in this patient,s carea62loni"ine $6/aloperi"ol c6/y"rochlorothiaDi"e "6LoraDepam e6 ProchlorperaDine :ormal La$s &. A <1!year!ol" 'oman% 'ho has $een treate" 'ith amitriptyline for "epression% is $ro#ght to her psychiatrist $y her family. The 'oman is conf#se"% "isoriente"% an" hall#cinating% an" her skin appears "ry an" 'arm. The family notice" that she took more pills than prescri$e". =hich of the follo'ing is the most likely ca#se of this "e)elopmenta6Anticholinergic "eliri#m $6/ypertensi)e crisis c6:e#roleptic malignant syn"rome "6Para"o.ical reaction e6Serotonin syn"rome :ormal La$s 4. A 21!year!ol" 'oman presents to the clinic for the first time 'ith symptoms of ma9or "epressi)e "isor"er lasting se)eral 'eeks. She re*#ests me"ication $eca#se% apart from feeling lo' an" tire"% she feels "istracte"% forgetf#l% an" #na$le to foc#s on her 'ork. She reports ha)ing $een "iagnose" as a chil" 'ith attention "eficit7hyperacti)ity "isor"er an" "ysle.ia% an" she ha" $een gi)en methylpheni"ate. =hich of the follo'ing therape#tic agents 'o#l" $e most appropriate for treatmenta6 AlpraDolam $6B#propion c6Lithi#m "6>lanDapine e6Paro.etine :ormal La$s

.A 1<!year!ol" $oy is $ro#ght to the clinic $y his father 'ho says that he has $een increasingly aggressi)e an" has $een st#m$ling an" tripping aro#n" the ho#se for se)eral 'eeks. /e is especially 'orrie" a$o#t his #ncharacteristic )iolent $eha)ior. The patient,s temperat#re is &1 2 3144.4 56% $loo" press#re is 144784 mm /g% p#lse is 847 min% an" respirations are 227min. /e has slightly "ilate" p#pils an" nystagm#s. /e $egins to ha)e con)#lsions on the ta$le% making the remain"er of the e.amination impossi$le. =hich of the follo'ing is the most likely ca#se of these fin"ingsa62ocaine 'ith"ra'al $6/eroin c6Mari9#ana "6Morphine e6Phencycli"ine 3P2P6 :ormal La$s <. A 28!year!ol" 'oman is $ro#ght to the hospital $y her h#s$an". She has not slept in se)eral "ays an" cleans the ho#se% "rinks 'ine% an" listens to lo#" m#sic in the mi""le of the night. She spent W2444 on a shopping spree o)er 4 "ays an" "eci"e" to change her career an" start a pri)ate $#siness. +n the inter)ie' room% she talks incessantly% giggles 'ith the n#rse% an" #n$#ttons her $lo#se to sho's her ne'ly $o#ght #n"er'ear. She has al'ays $een cheerf#l% an" has ha" short perio"s of time 'hen she 'as more energetic% $#t ne)er like this. She "enies the #se of street "r#gs% an" her #rine "r#g screen is negati)e. =hich of the follo'ing is the most likely "iagnosisa6Bipolar "isor"er type + $6Bipolar "isor"er type ++ c6Bor"erline personality "62yclothymia e6 SchiDophrenia paranoi" type :ormal La$s

(.A &2!year!ol" man is $ro#ght into the emergency "epartment $y a frien" $eca#se of a heroin o)er"ose. After the a"ministration of nalo.one an" a complete physical e.amination% 'hich of the follo'ing la$oratory st#"ies is most important in the e)al#ation of this patient to pro)i"e long!term follo'!#p carea6 Al$#min le)el $6Echocar"iogram c6Electroencephalogram "6/+? anti$o"y test e6 Plasma li)er enDyme le)els :ormal La$s 1.A 2!year!ol" African American 'oman is atten"ing her 11!year!ol" son,s f#neral% 'hen she cries o#t 0@amn yo#% Lor"% for stealing my $a$y from meV0 @#ring the ne.t se)eral "ays% she has se)eral o#t$#rsts of crying spells an" e.treme anger at Co"% an" *#estions her religio#s faith. After a fe' "ays% the crying spells cease. She has ne)er $een "iagnose" 'ith a psychiatric "isor"er in the past%. She is 'ell s#pporte" $y frien"s an" family% an" she "enies any s#ici"al i"eation. =hich of the follo'ing is the most likely "iagnosisa6Berea)ement $6Brief psychotic "isor"er c6Ma9or "epressi)e "isor"er 'ith psychotic feat#res "6Panic "isor"er e6Paranoi" schiDophrenia :ormal La$s 8.A <&!year!ol" 'hite male 'ho has recently retire" from 'ork as a pl#m$er for o)er &4 years ret#rns to his family physician saying that he has $een feeling )ery "o'n lately% an" has $een ha)ing "ecrease" appetite an" a loss of interest in acti)ities that #se" to gi)e him pleas#re. /e is a smoker% "rinks no alcohol% an" is $eing treate" $y his family physician for mo"erate essential hypertension. =hich of the follo'ing is the most appropriate ne.t step in managementa6 @isc#ss acti)ities that 'ill help him en9oy his retirement $6>r"er a thyroi" stim#lating hormone le)el 3TS/6 c6>r"er electrocon)#lsi)e therapy 3E2T6 "6Prescri$e an anti"epressant e6Ae)ie' the patient,s me"ication history

:ormal La$s 14. A 'oman 'ho atten"s 'eekly psychotherapy sessions $ecomes f#rio#s at her psychiatrist% 'ho is a$o#t to lea)e for )acation an" is not 'illing to re)eal 'here he is going. She angrily says that he is 9#st like her father% 'ho al'ays ha" his o'n 0o'n life0 o#tsi"e the family. =hich of the follo'ing $est "escri$es this patient,s reactiona6Acting o#t $62o#ntertransference c6+"entification "6Pro9ection e6Transference :ormal La$s 11.A 2<!year!ol" me"ical st#"ent is arreste" for threatening a neigh$or 'ith a knife. /e is $ro#ght to the hospital $eca#se he trie" to slit his 'rists on the 'ay to the police station. /is roommate arri)es an" states that the patient has $een 0)ery o""0 o)er the past ( months. /e is socially 'ith"ra'n an" has $een o)erhear" saying that the 5B+. is gi)ing him tips on patient!care an" that 0people from the sky0 are telling him to "o 0$a" things.0 /e stoppe" going to his >$!Cyn rotation $eca#se $elie)es that the $a$ies 'ere 0coming to earth0 to gi)e him messages. /e says he "oes not 'ant to $e a "octor anymore. /e appears apathetic an" has emotional $l#nting. The physical e.amination is other'ise normal. To.icologic screening is negati)e. =hich of the follo'ing is the most likely "iagnosisa6Bipolar + "isor"er $6Brief psychotic "isor"er c6@el#sional "isor"er "6SchiDophrenia e6SchiDophreniform "isor"er :ormal La$s 12.A 44!year!ol"% /+?!positi)e patient has $een treate" for Pne#mocystis carinii pne#monia in the hospital. /e has a prior history of "epression an" 'as treate" 'ith paro.etine% 'hich 'as contin#e" in the hospital. =hile in the hospital% he $ecame "elirio#s an" ha" )is#al

hall#cinations. The psychiatry cons#lt team starte" him on olanDapine. The patient,s "eliri#m gra"#ally resol)e"% $#t prior to "ischarge he complaine" that his #rine stream $ecame 'eak an" his $la""er felt f#ll. >n e.amination% the physician confirme" lo'er a$"ominal "istention. =hich of the follo'ing is the most effecti)e treatmenta6BenDtropine $6 Bethanechol c65#rosemi"e "6LoraDepam e6Physostigmine :ormal La$s 1&. A 24!year!ol" man 'as recently hospitaliDe" in a psychiatric #nit% 'here he 'as starte" on olanDapine. Upon )isiting his ne' primary care physician for a ro#tine physical e.amination prior to participating in a )ocational reha$ilitation program% his physician notices that his tho#ghts are *#ite illogical. 5or e.ample% 'hen she asks the yo#ng man 'hat sort of employment he hopes to 'ork to'ar"% he ans'ers% 0if yo# s#$tract some yello' from the sky% it $ecomes greener.0 After a $rief silence% he then states% 0telephone $ills sho#l" ne)er e.cee" t'enty "ollars ... accor"ing to the Book of :#m$ers.0 =hich mental stat#s e.amination fin"ing most acc#rately "escri$es this patient,s tho#ght processesa62lang associations $62oncrete thinking c6Loose associations "6Tangential tho#ghts e6Tho#ght $locking :ormal La$s 14. An 1!year!ol" $oy is $ro#ght to see a chil" an" a"olescent psychiatrist $eca#se of his mother,s complaint that he repeate"ly states that he 'ants to $e a girl. She also notes that he has al'ays seeme" to prefer girls, clothes an" fre*#ently gets #pset in the mornings 'hen his mother "resses him in typical $oy clothes for school. /is play acti)ities are characteriDe" $y games more fre*#ently en9oye" $y girls% an" he prefers to play 'ith girls. =hen his mother remin"s him that he is a $oy an" sho#l" act like a $oy% he fre*#ently gets #pset an" sometimes has a temper tantr#m. The $oy has no significant past me"ical history% an"

physical e.amination is 'ithin normal limits. +nto 'hich of the follo'ing areas is this $oy,s "iffic#lty $est classifie"a6Cen"er i"entity $6+ntellect#al "e)elopment c6Se.#al a)ersion "6Se.#al i"entity e6 Se.#al orientation :ormal La$s 1 .A 44!year!ol" man is a"mitte" to the hospital for e)al#ation an" treatment of "epression associate" 'ith s#ici"al tho#ghts. /e "enies any past psychiatric history. /is me"ical history is significant for hypertension% for 'hich he c#rrently takes hy"rochlorothiaDi"e. /e also "escri$es a history of once ha)ing a painf#l erection that laste" for eight ho#rs an" ha" to $e treate" 'ith intraca)erno#s in9ections in the emergency room. /e states that this 'as tho#ght to $e relate" to an antihypertensi)e me"ication that he #se" to take. @#ring the treatment of this man,s psychiatric con"ition% 'hich of the follo'ing me"ications sho#l" $e a)oi"e"a6B#propion $62lonaDepam c6Paro.etine "6TraDo"one e6Oolpi"em :ormal La$s 1<.A 21!year!ol" man presents to his primary care physician for his yearly physical e.amination. /e c#rrently smokes appro.imately one pack of cigarettes a "ay an" is intereste" in *#itting. /e has no significant past me"ical history an" has no allergies. /e is intereste" in smoking cessation classes% $#t 'o#l" like a""itional help. =hich of the follo'ing me"ications 'o#l" $e the most appropriate agent for treating nicotine "epen"ence in this patienta6B#propion $6B#spirone c62lonaDepam "65l#o.etine e6:efaDo"one

:ormal La$s 1(. A (4!year!ol" 'oman 'ith a long history of type 2 "ia$etes mellit#s #n"ergoes s#rgery for small $o'el o$str#ction. After s#rgery% she "e)elops ac#te renal fail#re. /o'e)er% she ref#ses to #n"ergo "ialysis on the a")ice of her physician% 'ho then calls for an imme"iate psychiatric cons#ltation. The patient tells the psychiatrist that she has li)e" a long life an" "oes not 'ant to $e kept ali)e $y or attache" to a machine% e)en if it means she 'ill "ie. A mental stat#s e.amination sho's that she is not psychotic% that she is f#lly oriente" an" alert% an" that she has no fl#ct#ations of cognition or le)el of conscio#sness. The patient,s family is insistent that she $e "ialyDe" imme"iately. =hich of the follo'ing is the most appropriate statement the psychiatric cons#ltant co#l" makea6 The patient is a'are of the conse*#ences of her "ecision an" "oes not sho' signs of a ma9or psychiatric illness. $6The patient is competent to "eci"e on treatment% an" her ref#sal to #n"ergo "ialysis m#st $e respecte". c6 The patient is competent to "eci"e on treatment% $#t her ref#sal can $e o)err#le" $eca#se of a me"ical emergency. "6 The patient is operating in a s#ici"al manner an" sho#l" $e committe" for treatment against her 'ill. e6 The patient is temporarily incompetent% so start her on "ialysis. :ormal La$s 11.A (2!year!ol" African American man 'ith a history of renal "ysf#nction% congesti)e heart fail#re% an" pre)io#s myocar"ial infarction is c#rrently #n"ergoing "ialysis. Until the past fe' 'eeks% he has $een in goo" spirits an" has a strong family s#pport system that helps him in getting to an" from "ialysis "aily. >)er the past fe' 'eeks% ho'e)er% he has $een feeling increasingly "epresse" an" has $eg#n to act $iDarrely% 'ith persec#tory "el#sions that the go)ernment is poisoning the chemicals #se" on him for "ialysis. Also in the last fe' 'eeks% "#e to increase" stomach pain% his me"ications ha)e $een a"9#ste" an" no' incl#"e cimeti"ine for stomach #lcers% "igo.in% an" a $a$y aspirin "aily. /e also takes "oc#sate so"i#m as nee"e" for stool softening an" i$#profen as nee"e" for mil" arthritis pain. =hich of the follo'ing me"ications 'o#l" $e most likely to ha)e in"#ce" the symptoms the patient is no' e.periencing-

a6Aspirin $62imeti"ine c6@igo.in "6@oc#sate so"i#m e6+$#profen :ormal La$s 18. A &4!year!ol" man is a"mitte" to the psychiatric temporary o$ser)ation #nit complaining of "epression an" s#ici"al i"eation follo'ing a cocaine $inge. @#ring his stay% he starts feeling as if insects are cra'ling #n"er his skin. This is most typical of 'hich of the follo'ing phenomenaa6 @epersonaliDation $6@yskinesia c65ormication "6+ll#sion e6Synesthesia :ormal La$s 24. A 41!year!ol" man presents to a primary care physician $eca#se of a one!'eek history of symptoms consistent 'ith pne#monia. Since this is the patient,s first )isit to the clinic% the physician gathers a f#ll history for a ne' patient assessment. The patient has no significant past me"ical% s#rgical% or psychiatric history. 5amily history is significant for a $rother an" an #ncle 'ith paranoi" schiDophrenia. Social history re)eals that the patient li)es alone% has minimal contact 'ith family% an" "escri$es no real social acti)ities or frien"s. =hen *#estione" a$o#t this% he states% 0+,)e ne)er $een m#ch intereste" in my family or $eing aro#n" people. 0/e has 'orke" "eli)ering ne'spapers for the past 1 years. /e has not "ate" since ha)ing one girlfrien" in the 11th gra"e. @#ring inter)ie'% tho#gh he seems emotionally "etache"% he "enies "epressi)e symptoms or psychotic symptoms. =hich of the follo'ing is the most appropriate psychiatric "iagnosisa6A)oi"ant personality "isor"er $6Psychotic "isor"er% not other'ise specifie" c6SchiDoi" personality "isor"er "6SchiDophrenia% #n"ifferentiate" type e6Social pho$ia

:ormal La$s 21.A 24!year!ol" 'hite man is $ro#ght to the emergency clinic $y t'o frien"s on a S#n"ay morning after a fraternity party the night $efore. /is frien"s note that since the party% he has $een )ery $elligerent% agitate"% an" lo#". They s#spect that he may ha)e #se" "r#gs 'ith some frien"s at the party. /is temperat#re is &( 2 381.< 56% $loo" press#re is 14 78 mm /g% p#lse is 14 7min% an" respirations are 247min. Physical e.amination re)eals sl#rre" speech% #nstea"y gait% an" nystagm#s. The patient appears to $e respon"ing to a#"itory hall#cinations. =hich of the follo'ing syn"romes most likely acco#nts for this patient,s presentationa6Alcohol 'ith"ra'al $62ocaine into.ication c6>pioi" 'ith"ra'al "6Phencycli"ine into.ication e6 ?ali#m 'ith"ra'al :ormal La$s 22.A psychiatrist on a cons#lt team is aske" to see a &2!year!ol" 'oman for "epression. She 'as a"mitte" to the hospital for a 'ork#p of na#sea% )omiting% an" a$"ominal pain. The me"ical team has so far fo#n" no reason for these symptoms. +n talking to the patient% the psychiatrist learns that she has ha" many me"ical pro$lems an"% o)er the past 14 years% has $een in the hospital more than 14 times. She reports ha)ing ha" e.cessi)e menstr#al $lee"ing an" a miscarriage% as 'ell as "iffic#lty $ecoming aro#se" "#ring se.#al interco#rse. She cannot tolerate high! fi$er foo"s an" has heart$#rn an" $loating. T'o years ago% she ha" a l#mp in her throat an" ha" "iffic#lty s'allo'ing% $#t nothing 'as fo#n" "#ring that a"mission. +n the past co#ple of years% she has ha" fre*#ent migraine hea"aches an" $l#rre" )ision an" has seen se)eral ne#rologists. She also h#rt her $ack an" no' has chronic $ack pain. =hich of the follo'ing is the most likely "iagnosisa62on)ersion "isor"er $6CeneraliDe" an.iety "isor"er c6Maske" "epression "6Pain "isor"er

e6SomatiDation "isor"er :ormal La$s 2&. A 2 !year!ol" 'oman presents to the emergency "epartment complaining that she is ha)ing a#"itory hall#cinations an" tremors% along 'ith associate" na#sea an" )omiting. She feels )ery an.io#s. >n mini!mental stat#s e.amination% she scores 22 of &4% an" she appears to $e o$t#n"e". 5rom 'hich of the follo'ing s#$stances is this patient most likely 'ith"ra'inga6AlpraDolam $62affeine c62ocaine "6/eroin e6 :icotine :ormal La$s 24.A &!year!ol" female la'yer 'ho has $een marrie" for 24 years comes to her physician,s office $eca#se she has not felt 0#p to par0 o)er the past 2 months. She is marrie"% has t'o gro'n sons% an" has a goo" practice. +n the past co#ple of 'eeks% she has stoppe" taking care of her appearance an" has fre*#ently calle" in sick to 'ork% 'hen she act#ally has $een ha)ing "iffic#lty getting o#t of $e". She states that she has lost her appetite recently an" her interest in se. 'ith her h#s$an" has "ecrease" consi"era$ly. She recently tol" her h#s$an" that at times she 'on"ers 'hether she sho#l" go on li)ing. She "enies any history of "r#g or alcohol a$#se% an" a complete physical e.amination & months earlier sho'e" her to $e in goo" health. =hich of the follo'ing is the mostly likely "iagnosisa6Bipolar + "isor"er $6CeneraliDe" an.iety "isor"er c6Ma9or "epressi)e "isor"er "6Panic "isor"er e6SchiDophrenia :ormal La$s 2 . A <1!year!ol" 'oman 'ith hypertension comes to the physician $eca#se of "ecrease" appetite% fatig#e% insomnia% ina$ility to concentrate% an" feelings of 'orthlessness o)er the past t'o months. She states that her gran"chil"ren are coming to stay 'ith her for three

'eeks an" she 0'ants to get to the $ottom of this0 $efore they arri)e. /er antihypertensi)e me"ication 'as change" three months ago. =hich of the follo'ing antihypertensi)e me"ications is the most likely ca#se of the patient,s ne' symptomsa62aptopril $6 5#rosemi"e c6Mino.i"il "6Propranolol e6?erapamil :ormal La$s 2<. A &8!year!ol" 'oman 'ith a history of hypomanic episo"es in the past presents to her physician complaining of se)eral of the symptoms of a ma9or "epressi)e episo"e. Ci)en this history% 'hich of the follo'ing is the most likely "iagnosisa6Bipolar + "isor"er $6Bipolar ++ "isor"er c62yclothymic "isor"er "6Ma9or "epressi)e "isor"er e6 S#$stance a$#se :ormal La$s 2(.An 12!year!ol" 'oman% 'hose h#s$an" passe" a'ay < 'eeks ago after a long illness% is a"mitte" to the hospital for 'orsening of her car"iac pro$lems. @#ring the hospital stay% a psychiatrist is in)ite" to cons#lt at the re*#est of her "a#ghters. The "a#ghters think their mother is "epresse"% $eca#se she is tearf#l% often talks a$o#t her "ea" h#s$an"% an" states that she ha" hear" his )oice se)eral times. She $lames herself for not ha)ing gone 'ith him on his last )isit to his sister% an" she starts crying. She has ne)er seen a psychiatrist $efore an" has $een 0strong0 all her life. =hich of the follo'ing is the most likely "iagnosisa6@epression secon"ary to general me"ical con"ition $6@ysthymia c6Crief reaction "6Ma9or "epressi)e "isor"er e6 Psychosis not other'ise specifie" :ormal La$s

21.A me"icine cons#lt is re*#este" on a &2!year!ol" 'oman 'ith paranoi" schiDophrenia 'ho is a patient in a close" psychiatric #nit. Se)eral "ays after the patient,s a"mission% she "e)elope" poly#ria% )omiting% st#por% "iarrhea% an" restlessness. She is c#rrently taking risperi"one% 14 mg gi)en at $e"time% $#t no other me"ications. =hich of the follo'ing is the most likely "iagnosisa6Anticholinergic crisis $6Ac#te "ystonic reaction c6Serotonin syn"rome "6Tar"i)e "yskinesia e6 =ater into.ication :ormal La$s 28.A &4!year!ol" 'oman in her first trimester of pregnancy is e)al#ate" $y her psychiatrist. She complains of )oices telling her to kill herself $eca#se she "oesn,t "eser)e to li)e% an" she feels e.tremely "epresse"% g#ilty% an" 'orthless. =hich of the follo'ing is the most appropriate treatmenta6 2lonaDepam $6Electrocon)#lsi)e therapy 3E2T6 c65l#o.etine "6Lithi#m e6Psychotherapy :ormal La$s &4.A &4!year!ol" 'oman 'ith "oc#mente" $ipolar "isor"er comes to the hospital $eca#se of feelings of g#ilt an" 'orthlessness. She has amasse" tho#san"s of "ollars of cre"it car" "e$t in the past 2 months. /er parents are going to pay it off for her% $#t she $elie)es that this 'ill $ankr#pt them. >n f#rther *#estioning% she re)eals that she $o#ght a g#n earlier in the "ay $eca#se it 'o#l" $e easier for e)eryone if she 0'asn,t here anymore.0 She plans to go to the roof of her $#il"ing% 'ith her ne' p#rchase% 0an" "o 'hat nee"s to $e "one.0 =hich of the follo'ing is the most appropriate ne.t step in managementa6Sen" home 'ith lithi#m $6Sen" home 'ith )alproic aci" c6Sen" home 'ith car$amaDepine "6>$ser)e in the emergency "epartment

e6A"mit to the hospital :ormal La$s &1. A 28!year!ol"% pre)io#sly s#ccessf#l 'oman 'as clim$ing stairs in her ne' home a$o#t a month ago% 'hen the 'hole ho#se fell apart. She en"e" #p in a hospital 'ith a fract#re" left fem#r. The psychiatry team 'as cons#lte" $eca#se the patient complaine" of nightmares an" flash$acks an" 'as afrai" to go to sleep as a res#lt. @#ring the inter)ie'% she is tearf#l% an" afrai" that her fear of falling is pre)enting her from participating eno#gh in her reha$ilitation% an" that the team 'ill "ischarge her from hospital. =hich of the follo'ing is the most appropriate treatment for this patienta6 +nsight!oriente" psychotherapy $6:o therapy $eca#se the patient nee"s to take responsi$ility for her treatment c6 P#t a sitter to stay in the patient,s room 24 ho#rs a "ay to calm her an.iety "6Start an anti"epressant e6Start $enDo"iaDepines :ormal La$s &2.A yo#ng 'oman 'ho s#staine" minor in9#ries after a motor )ehicle acci"ent is a"mitte" to the hospital. @#ring the o$ser)ation perio"% the intern notices that the patient,s )ital signs are all increase"% an" she seems conf#se"% "isoriente"% an" "elirio#s. >n a"mission% she states she has $een #sing me"ication to help 'ith an.iety for co#ple of months. /o'e)er% she ran o#t of it 2 "ays prior to the acci"ent. 5rom 'hich of the follo'ing me"ications is this patient most likely e.periencing 'ith"ra'ala6AlpraDolam $6B#spirone c6/y"ro.yDine "6:efaDo"one e6 Paro.etine :ormal La$s &&. A 'oman comes to the clinic 'ith her 1&!year!ol" gran"son% stating he has ha" $eha)ioral pro$lems for the past 4 years. /e is li)ing 'ith

her no' $eca#se his mother is in "r#g reha$ilitation center. She "escri$es that he has r#n from home t'ice an" has $een kicke" off the school $#s for threatening other ki"s an" challenging the "ri)er. /e is a$o#t to $e e.pelle" from school% since he 'as ca#ght e.torting money from other chil"ren. =hich of the follo'ing is the most likely "iagnosisa6Attention "eficit7hyperacti)ity "isor"er $62on"#ct "isor"er c6+ntermittent e.plosi)e "isor"er "6>ppositional "efiant "isor"er e6Aett syn"rome :ormal La$s &4. A 2(!year!ol" 'oman is 2 'eeks, postpart#m 'ith her first chil". @#ring her first postpart#m follo'!#p )isit% she complains to her physician that she has ha" se)eral crying spells an" has $een increasingly irrita$leG ho'e)er% she has ha" some spells "#ring 'hich she has felt almost e#phoric. She has ha" these symptoms o)er the past 'eek. She has not ha" any pre)io#s psychiatric "isor"ers. =hich of the follo'ing is the most likely "iagnosisa6A"9#stment "isor"er $6@ysthymic "isor"er c6Maternity $l#es "6Postpart#m "epression e6Postpart#m psychosis :ormal La$s & .A &(!year!ol" man li)es alone an" has no close frien"s. /e 'orks "#ring the night shift at the post office an" has little interaction 'ith others. /e has not engage" in se.#al acti)ity since he 'as 11 years ol"% $#t he "oes not feel m#ch "esire. /e maintains a close relationship 'ith his sister% $#t "oes not seek o#t relationships 'ith others. People ha)e tol" him that he seems 0"etache"0 an" that he has "iffic#lty e.periencing or e.pressing emotions. =hich of the follo'ing is the most likely "iagnosisa6Antisocial personality "isor"er $6A)oi"ant personality "isor"er c6Paranoi" personality "isor"er "6SchiDoi" personality "isor"er

e6SchiDotypal personality "isor"er f6Social pho$ia :ormal La$s &<. A 4&!year!ol" 'oman presents to the emergency "epartment complaining of "iDDiness% tremor% "iaphoresis% an" shortness of $reath. She in"icates to the physician that she has come into the hospital 'ith similar complaints t'ice in the past se)eral 'eeks. >n those occasions% as 'ell as c#rrently% her physical e.amination% ro#tine la$oratory st#"ies% E2C% an" car"iac enDymes ha)e all $een #nremarka$le. The patient states that these episo"es are starting to concern her greatly an" she is 'orrie" a$o#t going o#t of her ho#se alone no'. =hich of the follo'ing is the most likely "iagnosisa65actitio#s "isor"er $6CeneraliDe" an.iety "isor"er c6Panic "isor"er "6SchiDophrenia e6 Social pho$ia :ormal La$s &(.The mother of a 2&!year!ol" man $rings her son to a psychiatrist for a secon" opinion one 'eek after his "ischarge from the hospital. The yo#ng man ha" $een hospitaliDe" for one 'eek "#e to the ne' onset of a#"itory hall#cinations an" tangentiality of tho#ghts. @#ring his hospitaliDation% he 'as "iagnose" 'ith schiDophreniform "isor"er an" 'as treate" 'ith risperi"one 2 mg P> B+@. A me"ical 'ork!#p for this first episo"e of psychosis 'as negati)e. +n "isc#ssing co#rse an" prognosis 'ith the patient an" his mother% 'hich of the follo'ing feat#res 'o#l" $e consi"ere" a goo" prognostic in"icatora65amily history of schiDophrenia $6Lack of any kno'n precipitating factors or ac#te stressors c6>nset of psychotic symptoms 'ithin a month of the first change in $eha)ior "6Poor premor$i" e"#cational an" occ#pational performance e6 =ith"ra'n% a#tistic $eha)ior an" flat affect :ormal La$s

&1. A 24!year!ol" gra"#ate st#"ent an" teaching assistant comes into the st#"ent health center after $eing instr#cte" $y his college "ean to seek co#nseling. Accor"ing to the collateral history o$taine" from the patient,s roommates an" instr#ctors% the man e.hi$its annoyingly gran"iose $eha)ior% is fre*#ently "eman"ing of his peers, a"miration% an" has ha" to $e repriman"e" for treating his #n"ergra"#ate st#"ents 'itho#t empathy or #n"erstan"ing in "eman"ing that their 'ork $e s#$mitte" in t'o "ifferent forms for each assignment. Ci)en this history% 'hich of the follo'ing is the most likely "iagnosisa6Bor"erline personality "isor"er $6:arcissistic personality "isor"er c6Paranoi" personality "isor"er "6Passi)e!aggressi)e personality "isor"er e6 SchiDotypal personality "isor"er :ormal La$s

&8.A mother $rings her 1!year!ol" son to a psychiatrist for ne'!onset en#resis. A prior 'ork#p to "etermine a me"ical ca#se 'as negati)e. +n con)ersation% it seems that the en#resis starte" follo'ing parental arg#ments an" separation. The $oy 'ets himself at least t'ice a 'eek an" feels #pset a$o#t it% ref#sing to go for a sleep!o)er at his frien",s ho#se. =hich of the follo'ing is the most commonly #se" treatment for this con"itiona6Beha)ioral therapy $6+nterpersonal therapy c6Pharmacotherapy "6Psycho"rama e6Psychotherapy :ormal La$s 44.A 21!year!ol" man "rops o#t of college. >n *#estioning% he e.plains that he is so afrai" of ha)ing a panic attack that he is no longer 'illing to sit in class $eca#se he 'o#l" not $e a$le to lea)e. The same fear has also le" him to stop going shopping or to the mo)ies% $eca#se he is afrai" of ha)ing a panic attack in the ticket line or theater. =hich of the follo'ing is the most likely "iagnosis-

a6Agorapho$ia $62on)ersion "isor"er c6>$sessi)e!comp#lsi)e "isor"er "6Social pho$ia e6SomatiDation "isor"er :ormal La$s 41.A & !year!ol" man is $ro#ght to the emergency clinic $y his mother $eca#se of an episo"e of sl#rre" speech associate" 'ith the #ncomforta$le sensation that his tong#e is thick an" c#rling #p. The episo"e starte" s#""enly &4 min#tes ago. The patient is note" to $e hol"ing on to his tong#e 'ith his th#m$ an" forefinger. =hen aske" a$o#t this% the patient respon"s 'ith "ysarthria% saying that his me"ication has ca#se" this once $efore an" that he nee"s a shot to make it go a'ay. /is mother reports that the patient has ha" schiDophrenia for 14 years an" consistently takes t'o me"ications prescri$e" $y his psychiatrist. Se)eral "ays ago he ran o#t of one of his me"ications% $#t has contin#e" to take the other one. =hat is the most appropriate initial step in the management of this patienta6AlpraDolam $6BenDtropine c6/aloperi"ol "6LoraDepam e6>lanDapine :ormal La$s 42. A 4 !year!ol" 'oman presents to her primary care physician $eca#se of $l#rre" )ision. She states that this symptom starte" a$o#t t'o "ays ago. She "enies any past history of significant me"ical or ne#rological pro$lems. She "oes state that se)eral "ays ago she starte" treatment for "epression 'ith a psychiatrist% "#e to a t'o!month perio" "#ring 'hich she ha" se)eral "epressi)e symptoms. >n re)ie' of systems% she a"mits to ha)ing increase" her 'ater cons#mption o)er the last se)eral "ays% "#e to a "ry mo#th. She also complains of "iDDiness 'hen she stan"s #p from lying or sitting. /er temperat#re is &( 2 381.< 56% $loo" press#re lying "o'n is 1& 7( mm /g% $loo" press#re stan"ing is 1147<4 mm /g% p#lse lying "o'n is 14%

p#lse stan"ing is 8 % an" respiratory rate is 1<7min. Physical e.amination is #nremarka$le e.cept for mil" my"riasis. =hich of the follo'ing me"ications most likely acco#nts for this patient,s symptomsa6B#propion $62italopram c6+mipramine "6:efaDo"one e6 Sertraline :ormal La$s 4&.A patient has $een coming to psychotherapy for se)eral 'eeks. The psychiatrist is "ealing 'ith co#ntertransference feelings% incl#"ing pro$lems coping 'ith the patient,s intense "e)al#ation of him as a therapist% the patient,s arrogant attit#"e an" sense of entitlement% an" $ore"om "#ring sessions res#lting from the patient,s o$li)io#sness to'ar" him. =hich of the follo'ing is the most likely "iagnosis in this patienta6Antisocial personality "isor"er $6/istrionic personality "isor"er c6Mania "6:arcissistic personality "isor"er e6Paranoi" schiDophrenia :ormal La$s 44. A <4!year!ol" male 'ith generaliDe" a$"ominal pain an" 9a#n"ice is tol" $y his physician after an e.tensi)e 'ork#p that he has a "iagnosis of pancreatic a"enocarcinoma. +n the "ays follo'ing the "iagnosis% the patient states to his "octors that 0+ "on,t ha)e cancer ! all + nee" is a ?itamin B12 shot an" + 'ill feel $etter fast0. The patient is oriente" at all times% scoring 287&4 on a mini!mental state e.am% $#t ref#ses all chemotherapy me"ications% an" a"amantly ref#ses to $e seen $y the hospital oncologist% $eca#se 0he is a cancer "octor an" + "on,t ha)e cancer.0 =hich of the follo'ing "iagnoses 'o#l" $e most applica$le in this casea6Ac#te stress "isor"er $6A"9#stment "isor"er c6AlDheimer "ementia

"6@el#sional "isor"er e6Ma9or "epressi)e "isor"er :ormal La$s 4 .A &2!year!ol" 'oman presents to a psychiatrist $eca#se she 0'orries too m#ch. 0 She a"mits that% o)er the past ( months% she has e.perience" e.treme fatig#e% m#scle tension% an" irrita$ility. She has "iffic#lties falling asleep% partly $eca#se of 'orrisome tho#ghts a$o#t her h#s$an" an" chil"ren. She keeps 'orrying that something $a" is going to happen to them. +n the past month% she has ha" episo"es of shortness of $reath% "iDDiness% an" restlessness% an" has $een #na$le to go to 'ork or "o anything at home. /er physical e.amination an" la$oratory tests% as 'ell as her E2C% are #nremarka$le. =hich of the follo'ing is the most likely "iagnosisa6 A)oi"ant personality "isor"er $6CeneraliDe" an.iety "isor"er c6/ypochon"riasis "6>$sessi)e!comp#lsi)e "isor"er e6 Panic "isor"er :ormal La$s 4<. A 18!year!ol" st#"ent is a"mitte" to the psychiatry inpatient #nit 'ith the 'orking "iagnosis of first!$reak psychosis. Beca#se of com$ati)eness an" threats to others% he is p#t in secl#sion. At first% he ref#se" to take me"ication $y mo#thG ho'e)er% after attacking a n#rse he is gi)en haloperi"ol intram#sc#larly on t'o occasions. /e has no' "e)elope" ac#te torticollis an" t'itching of the mo#th an" face on that si"e. The family is f#rio#s% stating that the treatment ca#se" the seiD#res. =hich of the follo'ing reactions "i" this patient most likely ha)ea6 Ac#te "ystonia $6Akathisia c6Asteri.is "6Lenno.!Casta#t syn"rome e6 Pse#"oseiD#res :ormal La$s

4(.A patient presents to a psychiatrist for "epressi)e symptoms. /e talks a$o#t his past psychiatric pro$lems. /e also mentions he is an alcoholic an" has $een taking "is#lfiram for some time to keep so$er. /e once trie" to "rink after taking the "r#g an" en"e" #p $eing terri$ly sick. =hich of the follo'ing principles $est "escri$es this treatment of alcoholisma62on"itione" a)oi"ance $6E.tinction c65loo"ing "6Positi)e reinforcement e6 Aeciprocal inhi$ition :ormal La$s 41. A patient is talking to his psychotherapist a$o#t his pro$lems. @#ring the session% the patient in*#ires a$o#t the meaning of their talk an" asks 'hy the therapist hasn,t sai" anything. The therapist respon"s% 0Perhaps it 'as "iffic#lt to get fee"$ack from yo#r tro#$le" parents% so yo# got #se" to t#rning to others for reass#rance that yo# are 'orth'hile!9#st like yo# are "oing no' 'ith me.0 =hich of the follo'ing types of inter)ention "i" the therapist #sea6A")ice $62larification c62onfrontation "65acilitation e6 +nterpretation :ormal La$s 48.A 4 !year!ol" 'oman ret#rns to her psychiatrist for her ro#tine $i'eekly appointment t'o months after $eing hospitaliDe" for an episo"e of ma9or "epressi)e "isor"er% rec#rrent% se)ere 'ith psychotic feat#res. @#ring her hospitaliDation% she 'as starte" on t'o me"ications% an anti"epressant an" an antipsychotic% an" she has contin#e" these me"ications "aily as an o#tpatient. At her appointment% she complains to her physician that she has misse" her menstr#al perio" for t'o months. She also complains of ten"erness in her $reasts% an" an occasional small amo#nt of milky "ischarge from her $reasts onto her $lo#se. =hen *#estione" f#rther% she also a"mits to lo' li$i"o o)er the

past month. =hich of the follo'ing me"ications is most likely responsi$le for this constellation of symptomsa6>lanDapine $6Paro.etine c6N#etiapine "6Aisperi"one e6 Sertraline :ormal La$s 4.A 42!year!ol"% #nemploye" la$oratory technician is a"mitte" to the hospital for na#sea% )omiting% an" a$"ominal pain. She tells the physician that she ha" $een "iagnose" in the past 'ith "isseminate" l#p#s erythematos#s an" that she ha" /o"gkin "isease. She seems 'orrie" that an e.tensi)e me"ical 'ork#p faile" to confirm any of the pre)io#s "iagnoses or fin" a ca#se of her act#al symptoms. She insists on $eing gi)en meperi"ine to relie)e her pain. >n e.amination% the only physical fin"ings are scars an" some a$scesses on her thighs. She e.plains that she ha" $een intentionally in9#re" $y the n#rses in a pre)io#s hospital. =hich of the follo'ing is the most likely "iagnosisa65actitio#s "isor"er $6Canser syn"rome c6/ypochon"riasis "6Malingering e6SomatiDation "isor"er :ormal La$s :ote; 2heck yo#r o'n ans'ers $efore hitting the 2heck $#tton $elo'. =hen yo# click the 2heck $#tton% a $ro'ser 'in"o' 'ill appear that contains a s#mmary of yo#r res#lts.

E.planations Block 14 E.planations

:) (*!lanation: 7he correct answer is C. 7his !atient has evi"ence of hy!erse*uality, which is one of the many sym!toms of mania encountere" in !atients with bi!olar "isor"er. Other sym!toms of mania inclu"e $ran"iosity, im!ulsivity, irritability, insomnia, an" elevate" moo". %ysthymic "isor"er 'choice A) is characteri#e" by "e!resse" moo", more often than not, over the course of at least ; years. 2eroin abuse 'choice ) "oes not manifest with hy!erse*ualityG rather, !atients with heroin abuse ten" to show si$ns of slowe" activity because of o!ioi" action, with a thin, $aunt a!!earance an" associate" lethar$y an" anore*ia. &artial com!le* sei#ures 'choice %) can !resent with bi#arre behaviors an" sym!toms, such as olfactory or $ustatory hallucinations or brief !sychosis, but hy!erse*uality is not ty!ically one of the behaviors in"uce". )chi#o!hrenia 'choice () is a thou$ht "isor"er. 7his "ia$nosis requires the !resence of au"itory hallucinations, flattenin$ of affect, an" social autism over the course of at least C months. ;) (*!lanation: 7he correct answer is %. 7he !atient takes an unknown an*iety me"ication, an" is most likely e*!eriencin$ ben#o"ia#e!ine with"rawal sym!toms. Commonly observe" sym!toms of ben#o"ia#e!ine with"rawal inclu"e: an*iety, "ia!horesis, irritability, insomnia, fati$ue, hea"ache, myal$ias, nausea, !erce!tual "isturbances, tremors, an" sei#ures. 7he most a!!ro!riate mana$ement ste! woul" be !arenteral a"ministration of a short4actin$ ben#o"ia#e!ine, such as lora#e!am. Cloni"ine 'choice A) is often use" for the acute treatment of elevate" bloo" !ressure. 2owever, it woul" not be an a!!ro!riate initial mana$ement choice for this !atient, because the elevate" bloo" !ressure is most likely cause" by or e*acerbate" by the with"rawal syn"rome. 2alo!eri"ol 'choice ) can be use" in the emer$ency mana$ement of e*treme an*iety an" a$itation, but it woul" not be an a!!ro!riate choice for a with"rawal syn"rome. Althou$h halo!eri"ol woul" likely !ro"uce se"ation, it woul" not treat the un"erlyin$ with"rawalG the with"rawal woul" likely continue to !ro$ress. 2y"rochlorothia#i"e 'choice C) woul" not be an a!!ro!riate initial mana$ement choice. Althou$h the !atient has an elevate" bloo" !ressure, this is most likely associate" with the with"rawal syn"rome. &rochlor!era#ine 'choice () coul" be use" to treat the nausea an" hea"ache, but it woul" not be an a!!ro!riate initial mana$ement ste! because these sym!toms are most likely "ue to ben#o"ia#e!ine with"rawal, which shoul" be treate" initially.

0) (*!lanation: 7he correct answer is A. Anticholiner$ic "elirium is cause" by the use of anticholiner$ics or other "ru$s with anticholiner$ic !ro!erties, like tricyclic anti"e!ressants. In el"erly !atients, the sym!toms can be !resent even with usual "oses. Over"osin$ results in clou"in$ of consciousness, as well as consti!ation, urinary retention, "ry mouth, elevate" tem!erature, "ry flushe" skin, worsenin$ of $laucoma, an" tachycar"ia. 7he !rimary criterion of a hy!ertensive crisis 'choice ) is an elevate" value of both systolic an" "iastolic bloo" !ressure. 7ricyclic anti"e!ressants usually lower bloo" !ressure an" cause orthostatic hy!otension. Neurole!tic mali$nant syn"rome 'choice C) is a rare com!lication of neurole!tic thera!y resultin$ in muscle ri$i"ity an" elevate" tem!erature, as well as "ifficulties swallowin$, tremor, incontinence, "ia!horesis, mutism, tachycar"ia, altere" level of consciousness, labile bloo" !ressure, leucocytosis, an" elevate" creatine !hos!hokinase. A !ara"o*ical reaction 'choice %) is an unusual reaction to ben#o"ia#e!ines, consistin$ of !ara"o*ical a$itation an" confusion instea" of rela*ation. It is seen in el"erly !atients, as well as in !atients with or$anic "iseases of the CN). )erotonin syn"rome 'choice () can result from a combination of +AO inhibitors an" serotoner$ic a$ents. (ven thou$h altere" mental status may be !resent, tremor, restlessness, hy!errefle*ia, myoclonus, shiverin$, an" "ia!horesis make it "ifferent from anticholiner$ic "elirium. ?) (*!lanation: 7he correct answer is . u!ro!ion is an anti"e!ressant with both "o!aminer$ic an" nora"rener$ic !ro!erties that woul" essentially hel! this !atient not only im!rove "e!ression but also co$nitive functionin$ relate" to her !rior history of attention "eficitBhy!eractivity "isor"er. Al!ra#olam 'choice A) is a ben#o"ia#e!ine with a ra!i" onset of action an" a relatively short metabolite half4life. It has stron$ an*iolytic !ro!erties. (ven thou$h initially it was claime" to have anti"e!ressant !ro!erties too, its hi$hly a""ictive !otential !revents it from wi"er an" lon$4term use. ,ithium 'choice C) is a moo" stabili#er with !otential use for treatment of a$$ression in attention "eficitBhy!eractivity "isor"er. ,ithium a!!ears to be effective only as an a"<unct in the treatment of "e!ressive "isor"er. Its lon$4term use may cause co$nitive im!airment. Olan#a!ine 'choice %) is an aty!ical anti!sychotic with some moo" stabili#in$ !ro!erties. It has not been use" solely for treatment of "e!ression. &aro*etine 'choice () is a selective serotonin reu!take inhibitor '))6I). Althou$h it is effective in most !atients, its anticholiner$ic !ro!erties mi$ht im!air co$nitive function more in this case. /) (*!lanation: 7he correct answer is (. 7his !atient is "is!layin$ the si$ns an" sym!toms of &C& into*ication. &C& causes a$$ression, "istortion of bo"y ima$e, "isor$ani#e" thou$hts, ata*ia, nysta$mus, mi"4

"ilate" !u!ils, myoclonus, fever, hy!ersalivation, an" hy!eracusis. It can lea" to sei#ures, coma, an" "eath. It can also lea" to an acute !sychosis with a hi$h risk of violent behavior an" suici"e. 2alo!eri"ol is the treatment for the violent behavior. Cocaine with"rawal 'choice A) causes severe "e!ression, hy!ersomnolence, ni$htmares, hea"aches, an" sweatin$. 7he sym!toms !eak ;4? "ays after the last "ose. Ata*ia an" nysta$mus are not !resent. 2eroin 'choice ) an" mor!hine 'choice %) are o!ioi"s that !ro"uce eu!horia, hy!oactivity, an" im!aire" concentration. &hysical si$ns inclu"e nee"le Htrack4marks,H miosis, res!iratory "e!ression, hy!otension, bra"ycar"ia, an" "ecrease" bo"y tem!erature. Consti!ation is very common. Nausea, vomitin$, shock, coma, an" !ulmonary e"ema may occur. Over"oses are treate" with nalo*one. +etha"one maintenance is use" for in"ivi"uals with "emonstrate" !hysiolo$ic "e!en"ence an" re"uces "ru$4seekin$ behavior. +ari<uana 'choice C) !ro"uces tachycar"ia an" con<unctival in<ection. %rivin$ may be im!aire" for u! to @ hours. With"rawal is $enerally mil", but can !ro"uce tremor, nysta$mus, sweatin$, irritability, nausea, vomitin$, an" "iarrhea. C) (*!lanation: 7he correct answer is A. i!olar "isor"er ty!e I inclu"es full4blown e!iso"es of mania, with erratic an" "isinhibite" behavior, $ran"iosity, lo$orrhea, overe*ten"e" activities, !oor frustration tolerance, an" ve$etative si$ns, such as increase" libi"o, e*cessive ener$y, "ecrease" slee!, an" wei$ht loss. It also inclu"es e!iso"es of mi*e" moo" an" ma<or "e!ression. i!olar "isor"er ty!e II 'choice ) is "efine" by e!iso"es of ma<or "e!ression an" hy!omania, but not full4blown mania. or"erline !ersonality 'choice C) requires a !attern of unstable !ersonal relationshi!s, self4ima$e, an" affect that are seen as efforts to avoi" aban"onment, unstable relationshi!s, chronic feelin$s of em!tiness, intense an$er, transient "issociative sym!toms, im!ulsivity, i"entity "isturbance, an" recurrent suici"al behavior. Cyclothymia 'choice %) is a less severe form of bi!olar "isor"er, with alternatin$ !erio"s of hy!omania an" mo"erate "e!ression. )ym!toms must be !resent at least ; years for the "ia$nosis. )chi#o!hrenia !aranoi" ty!e 'choice () requires the !resence of active "elusions an" hallucinations, "isor$ani#e" behavior or s!eech, an" ne$ative sym!toms. 7he above sym!toms must be !resent more than C months an" must inclu"e !ro"romal sym!toms an" cause im!airment in social functionin$. Althou$h $ran"iosity may be !art of the clinical !icture, it is not the only sym!tom. D) (*!lanation: 7he correct answer is %. ecause of the frequent use of nee"les in a !atient with heroin "e!en"ence, it is absolutely necessary to rule out the !ossibility of 2I1 infection. 7his is es!ecially true with a !atient who is unable to !rovi"e a history "urin$ an e!iso"e of heroin over"ose. Concomitant 2I1 infection an" the nee" for "ia$nosis will make this laboratory stu"y the most im!ortant of all the stu"ies liste". Albumin level 'choice A) an" other !lasma liver

en#yme levels 'choice () are not usually affecte" by an e!iso"e of heroin into*ication. An echocar"io$ram 'choice ) may be in"icate" in a chronic I1 "ru$ abuser if there is evi"ence of a murmur on auscultation of the heart, !ointin$ to the !ossibility of bacterial en"ocar"itis. An electroence!halo$ram 'choice C) is not necessarily in"icate" in a !atient with heroin into*ication. @) (*!lanation: 7he correct answer is A. ereavement, or normal $rief, is often characteri#e" by many of the same characteristics of "e!ression, inclu"in$ sa"ness, tearfulness, loss of a!!etite, !oor slee!, an" "iminishe" interest in activities that use" to brin$ !leasure. Of im!ortance in "ifferentiatin$ $rief from ma<or "e!ressive "isor"er 'choice C) is that !eo!le with $rief have sym!toms that are time4limite", whereas !eo!le with ma<or "e!ression cannot ever ima$ine feelin$ better. In this case, the !atient "oes not have any evi"ence of !sychosis, eliminatin$ brief !sychotic "isor"er 'choice ) an" !aranoi" schi#o!hrenia 'choice () from the "ifferential "ia$nosis. &anic "isor"er 'choice %) is a syn"rome that consists of "iscrete !erio"s of intense fear or "iscomfort accom!anie" by somatic com!laints, such as !al!itations, tremblin$, shortness of breath or sweatin$G these sym!toms are not consistent with this !atientAs com!laints. I) (*!lanation: 7he correct answer is (. +any me"ications use" to control hy!ertension, such as !ro!ranolol, an" in the !ast, reser!ine, are known to occasionally lea" to "e!ressive sym!toms. y evaluation of the !atientAs me"ication recor", the !hysician can evaluate which me"ications were starte" at what time an" can make a"<ustments to "osa$e or switchin$ to alternative me"ications to control hy!ertension. Chan$in$ the antihy!ertensive me"ication will !ossibly im!rove the "e!ressive sym!toms without the nee" to start an anti"e!ressant 'choice %). Choice A is an a!!ro!riate intervention, but it is not the most a!!ro!riate ne*t ste!, as the etiolo$y of the !atientAs "e!ression may be overlooke". Or"erin$ a 7)2 level 'choice ) is also a!!ro!riate, as hy!othyroi"ism can be an or$anic cause of "e!ression. 2owever, $iven the !atientAs me"ication history, evaluatin$ !ossible !harmacolo$ic causes of "e!ression takes !rece"ence in mana$ement of !atient "e!ression. (lectroconvulsive thera!y 'choice C) is in"icate" in severe intractable "e!ression when not contrain"icate" by sei#ure "isor"er or other factors, but is not in"icate" in this state. :>) (*!lanation: 7he correct answer is (. 7ransference is "efine" by !sychoanalysts as a !atientAs unconscious feelin$s an" behavior towar" the analyst that are base" on infantile wishes the !atient has towar"

!arental fi$ures. Actin$ out 'choice A) is a "efense mechanism by which a !erson e*!resses an unconscious wish or im!ulse throu$h action in or"er to avoi" bein$ consciously aware of the accom!anyin$ affect. .ivin$ in to an im!ulsive act relieves the tension an" brin$s imme"iate $ratification. Countertransference 'choice ) encom!asses a s!ectrum of analystAs reactions to the !atient that are unconscious an" base" on !ersonal conflicts of which the analyst may not be aware. I"entification 'choice C) is a "efense mechanism an" also !lays a role in normal e$o "evelo!ment. I"entification with a love" ob<ect serves as a "efense a$ainst !ain or an*iety relate" to threatene" or real loss or se!aration from the ob<ect. &ro<ection 'choice %) is a "efense mechanism by which inner unacce!table im!ulses are !erceive" as thou$h they were comin$ from the outsi"e an" are ascribe" to someone else. ::) 7he correct answer is %. 7his !atient has schi#o!hrenia. 2e hears voices, has "elusional irrational thou$hts, has a flattene" affect an" lack of motivation, an" has become socially with"rawn. 7his has been $oin$ on for D months. )chi#o!hrenia is a "isease that is characteri#e" by !sychotic sym!toms '!ositive sym!toms) an" flattenin$ of affect an" motivation 'ne$ative sym!toms) an" an im!airment of social or occu!ational functionin$ for at least C months. 7he sym!toms are not cause" by a moo" "isor"er, schi#oaffective "isor"er, a me"ical, neurolo$ic or a substance4in"uce" "isor"er. It commonly be$ins in the early ;>s an" an abnormal !remorbi" level of functionin$ is common. :>= will commit suici"e. 7he treatment consists of anti!sychotic a$ents an" !sychosocial treatment, inclu"in$ reality4 base" thera!y, !atient an" family su!!ort, skill trainin$, an" assistance with "aily livin$ skills. i!olar I "isor"er 'choice A) is "ia$nose" after at least one manic e!iso"e. It is often characteri#e" by e!iso"es of mania mi*e" with e!iso"es of "e!ression. +anic e!iso"es must be a "istinct !erio" of elevate" or irritable moo" with $ran"iosity, !ressure" s!eech, fli$ht of i"eas, racin$ thou$hts, a "ecrease" nee" for slee!, "istractibility, increase" $oal4"irecte" behavior, involvement in !leasurable activities with known un"esirable consequences 'e*cessive sho!!in$, risky se*ual activity an" unwise business investments). 7he e!iso"es cause functional im!airment an" last for at least : week. 7reatment for bi!olar "isor"er is lithium. rief !sychotic "isor"er 'choice ) is characteri#e" by !sychotic sym!toms that last for more than one "ay, but less than one month. It may be !rece"e" by a stressor, an" is self4limite". %elusional "isor"er 'choice C) is characteri#e" by nonbi#arre "elusions 'such as bein$ !oisone", bein$ followe", or havin$ a "isease) that are !resent for at least : month. 7reatment is !sychothera!y. )chi#o!hreniform "isor"er 'choice () resembles schi#o!hrenia but lasts for more than : month but less than C months. It is self4limite". :;) (*!lanation: 7he correct answer is . ethanechol is "irect4actin$ choliner$ic stimulant effective in re"ucin$ the !eri!heral anticholiner$ic effects of some !sychotro!ic me"ications. It can be a"ministere"

intramuscularly, orally, or subcutaneously to efficiently relieve the sym!toms of urinary retention. en#tro!ine 'choice A) is an anti!arkinsonian "ru$ with anticholiner$ic !ro!erties. It woul" only worsen the sym!toms in the case "escribe". -urosemi"e 'choice C) is a loo! "iuretic with a tar$et action on the ki"neys an" thus has no effect on the choliner$ic system. ,ora#e!am 'choice %) is a short4actin$ ben#o"ia#e!ineG it is not use" to counteract !eri!heral anticholiner$ic effects. &hysosti$mine 'choice () !ro"uces choliner$ic stimulation by inhibitin$ cholinesterase. It is useful for relievin$ CN) sym!toms '"elirium, confusion) !ro"uce" by anticholiner$ic me"ications. ecause this !atientAs sym!toms are !eri!heral in nature rather than central, bethanechol woul" be the "ru$ of choice. :0) (*!lanation: 7he correct answer is C. 7his !atient e*hibits thou$ht !rocesses characteri#e" by loosenin$ of associations. 2is i"eas are "isconnecte" an" seem to <um! from one to!ic to an unconnecte" to!ic. ,oose associations are one of the characteristic si$ns of a !rimary thou$ht "isor"er, such as schi#o!hrenia. Clan$ associations 'choice A) occur when the selection of wor"s or themes is base" on a soun" of the wor"s rather than on thou$ht content. Wor"s that rhyme are frequently associate". Clan$ associations, like most other si$ns of thou$ht "isor"er, can either be "ue to a !rimary thou$ht "isor"er, a !rimary moo" "isor"er 'e.$., "urin$ a manic e!iso"e), or relate" to other !roblems 'such as substance into*ication or "elirium). Concrete thinkin$ 'choice ) is characteri#e" by !oor ability to think in abstract terms, "es!ite normal intelli$ence. 7his case !resentation $ives no evi"ence of im!airment in abstract thinkin$. 7an$ential thou$hts 'choice %) "escribe thou$hts that $o off on a tan$ent, unrelate", or minimally relate" to the ori$inal i"ea. With such tan$entiality, the !atient aban"ons his i"eational ob<ective in !ursuit of thou$hts !eri!heral to the ori$inal $oal. 7he interviewer is commonly left with the sense that a question to the !atient elicite" a lon$ strin$ of thou$hts that en"e" u! havin$ nothin$ to "o with the ori$inal question. 7hou$ht blockin$ 'choice () occurs when thou$hts an" s!eech halt, often in mi"4sentence, as if for$otten. 7he thou$ht may be !icke" u! later, after a !erio" of a!!arent confusion. :?) (*!lanation: 7he correct answer is A. .en"er i"entity is a !ersonAs sense of maleness or femaleness. 7he formation of $en"er i"entity is base" on many cultural influences, !hysical characteristics, an" !arental attitu"es. 7he stan"ar" an" healthy outcome in the "evelo!ment of $en"er i"entity is a relatively secure sense of i"entification with oneAs biolo$ical se*. In"ivi"uals may be "ia$nose" with $en"er i"entity "isor"er when the "isturbance causes clinically si$nificant "istress or im!airment in social, occu!ational, or other im!ortant areas of functionin$. Intellectual "evelo!ment 'choice ) is the !rocess of maturation of normal co$nitive functions. 7here is no evi"ence of an intellectual im!airment in this case.

)e*ual aversion 'choice C) refers to a !ersistent or recurrent e*treme aversion to, an" avoi"ance of all, or almost all, $enital se*ual contact with a se*ual !artner. )e*ual aversion "isor"er is an a"ult se*ual "esire "isor"er, alon$ with hy!oactive se*ual "esire "isor"er. )e*ual i"entity 'choice %) refers to a !ersonAs biolo$ical se*ual characteristics: chromosomes, e*ternal $enitalia, internal $enitalia, hormonal com!osition, $ona"s, an" secon"ary se*ual characteristics. In this case, there is no evi"ence of a "isturbance in se*ual i"entity4it is clear that the !atient has a male !henoty!e, an" !resumably a male $enoty!e 'E9). )e*ual orientation 'choice () "escribes the ob<ect of a !ersonAs se*ual im!ulses 'heterose*uality, homose*uality, bise*uality). In this case, we have no information on the "irection of the boyAs se*ual im!ulses, or that he e*!eriences conscious se*ual im!ulses at this sta$e at all. 7he "evelo!ment of a homose*ual orientation is rarely accom!anie" by a "isturbance in $en"er i"entity, as these two "evelo!mental !rocesses are quite "istinct. :/) (*!lanation: 7he correct answer is %. 7ra#o"one is an ol"er anti"e!ressant me"ication that is commonly use" in lower "oses for the treatment of insomnia associate" with "e!ression. %ue to its si$nificant al!ha4 a"rener$ic blockin$ !ro!erties, it has been associate" with !ria!ism. &ria!ism is a serious a"verse event "efine" as a !ersistent, !ainful !enile erection. (rectile "ysfunction occurs in a!!ro*imately />= of men sufferin$ an e!iso"e of !ria!ism. 7his !atient has a history of a !rior e!iso"e of !ria!ism associate" with an antihy!ertensive 'most likely one with si$nificant al!ha4a"rener$ic anta$onism, such as !ra#osin). .iven his history of !ria!ism 'an" "ue to the fact that he is likely !re"is!ose" to subsequent e!iso"es of !ria!ism) tra#o"one shoul" be avoi"e" in this !atient. u!ro!ion 'choice A) is a newer anti"e!ressant that is not associate" with the "evelo!ment of !ria!ism. It is also markete" for smokin$ cessation treatment. Clona#e!am 'choice ) is a lon$4actin$ ben#o"ia#e!ine that is often use" in the treatment of an*iety associate" with ma<or "e!ression, "urin$ the initiation of anti"e!ressant treatment. It is not associate" with !ria!ism an" woul" not be contrain"icate" in this !atient. &aro*etine 'choice C) is a selective serotonin reu!take inhibitor '))6I) that is use" for the treatment of "e!ression an" a variety of !rimary an*iety "isor"ers 'obsessive4com!ulsive "isor"er, !anic "isor"er, social !hobia). It is not associate" with !ria!ism. 3ol!i"em 'choice () is a non4ben#o"ia#e!ine hy!notic a$ent that acts at the $amma4aminobutyric aci" '.A A)4ben#o"ia#e!ine com!le*. It is use" in the treatment of insomnia. It is not associate" with !ria!ism. :C) (*!lanation: 7he correct answer is A. u!ro!ion was a!!rove" by the F.). -oo" an" %ru$ A"ministration in :IIC as a treatment for nicotine "e!en"ence. 7he s!ecific mechanism of action is unclear, but is thou$ht to relate to re"ucin$ cravin$ for nicotine as well as re"ucin$ with"rawal sym!toms after cessation. us!irone 'choice ) an" fluo*etine 'choice %) are also use"

in con<unction with behavioral mo"ification techniques in smokin$ cessation, but they are not a!!rove" as s!ecific treatments for nicotine "e!en"ence. Clona#e!am 'choice C) is a ben#o"ia#e!ine use" to treat an*iety an" with"rawal sym!toms an" is not in"icate" for use in smokin$ cessation. Nefa#o"one 'choice () is an anti"e!ressant me"ication that "oes not have "emonstrate" efficacy in ai"in$ smokin$ cessation. :D) 7he correct answer is A. 7his !atient raises one of the most "ifficult le$al an" ethical !roblems in !sychiatry. It is im!ortant to un"erstan" that com!etency, or lack of com!etency 'choices , C, an" (), can be "etermine" only by a le$al authority, such as a court of law. 7he role of !sychiatrists is solely a"visory in "eterminin$ com!etency. In this situation, only if the !atient is suici"al by virtue of a ma<or !sychiatric illness, or if the !atient were sub<ect to an imme"iate me"ical emer$ency, coul" treatment be involuntarily a"ministere". 7he !sychiatristAs role is to assess a !ersonAs mental status for evi"ence of co$nitive im!airment, as well as to ascertain that the !atient has a thorou$h un"erstan"in$ of the consequences of treatment "ecisions that are ma"e. 7his !atient "oes not meet criteria for treatment a$ainst her will 'choice %), which requires both a mental "isor"er an" the threat of im!en"in$ imme"iate harm to self or others. :@) : 7he correct answer is . Cimeti"ine is the only "ru$ liste" known to cause !sychiatric effects of clinical si$nificance, inclu"in$ "elusions an" !sychosis. 7he e*act mechanism is unknown, but is thou$ht to be relate" to the effects of cimeti"ine on the 24; histamine rece!tor in the brain. 7he treatment of cimeti"ine4in"uce" !sychosis is to re"uce the "osa$e of the me"ication, an" "iscontinue it if an alternative thera!y is available. As!irin 'choice A) has not been shown to in"uce any clinically si$nificant !sychiatric chan$es. %i$o*in 'choice C) has been known to cause "elirium in to*ic levels, but "oes not cause the lon$ stan"in$ "elusion that the !atient above "emonstrates. %ocusate so"ium 'choice %) an" ibu!rofen 'choice () are me"ications not known to cause "emonstrable !sychiatric effects. :I) (*!lanation: 7he correct answer is C. -ormication is a form of ha!tic hallucinations, associate" with the sensation of touch. It is commonly seen in "elirium tremens, am!hetamine !sychosis, an" cocaine into*ication. %e!ersonali#ation 'choice A) is a term that "escribes the feelin$ of bein$ "ifferent, stran$e, or unreal. 7he !erson feels a loss of i"entity. 7his !henomenon is seen in "issociative "isor"ers. %yskinesia 'choice ) is a term "escribin$ any movement "isor"er. Illusion 'choice %) is an erroneous !erce!tion, or a false res!onse to sensory

stimulus. It is commonly seen in to*ic states as well as in schi#o!hrenia an" severe an*iety "isor"ers. )ynesthesia 'choice () is a state in which a stimulus in one sensory mo"ality !ro"uces a sensation in another mo"ality. It is seen in into*ication with hallucino$ens. ;>) : 7he correct answer is C. )chi#oi" !ersonality "isor"er is a cluster A !ersonality "isor"er '!aranoi", schi#oi", schi#oty!al). 7hese "isor"ers are more common in the biolo$ical relatives of !atients with schi#o!hrenia than amon$ control $rou!s. )chi#oi" in"ivi"uals are characteri#e" by a !ervasive !attern of "etachment from social relationshi!s an" a restricte" ran$e of emotional e*!ression. )uch !eo!le usually neither "esire nor en<oy relationshi!s with family or frien"s, choose solitary activities, have little interest in se*ual e*!eriences with another !erson, take !leasure in few 'if any) activities, a!!ear in"ifferent to !raise or criticism of others, an" show emotional "etachment. Avoi"ant !ersonality "isor"er 'choice A) is a cluster C !ersonality "isor"er 'avoi"ant, "e!en"ent, obsessive com!ulsive) characteri#e" by a hi$h an*iety level. 7hese in"ivi"uals have a !ervasive !attern of social inhibition, feelin$s of ina"equacy, an" hy!ersensitivity to a ne$ative evaluation. 7hey avoi" occu!ational activities that involve si$nificant inter!ersonal contact because of fears of criticism or "isa!!roval. Avoi"ant in"ivi"uals are hesitant in relationshi!s because of fears of bein$ shame" or ri"icule" an" view themselves as socially ine!t, !ersonally una!!ealin$, or inferior to others. Fnlike in schi#oi" !ersonality "isor"er, avoi"ant in"ivi"uals stron$ly "esire closer relationshi!s, but are very an*ious about them. &sychotic "isor"er, not otherwise s!ecifie" 'choice ) is a "ia$nosis that in"icates !sychotic sym!toms '"elusions, hallucinations, "isor$ani#e" s!eech, $rossly "isor$ani#e" or catatonic behavior) about which there is ina"equate information to make a s!ecific "ia$nosis. )chi#o!hrenia, un"ifferentiate" ty!e 'choice %) is a "ia$nosis use" to classify in"ivi"uals who meet criteria for schi#o!hrenia, but "o not clearly fit into one of the other ty!es 'catatonic, "isor$ani#e", !aranoi", resi"ual). 7his !atient "oes not have known !sychotic sym!toms. )ocial !hobia 'choice () is a !rimary an*iety "isor"er that has many features in common with avoi"ant !ersonality "isor"er. It is characteri#e" by a marke" an" !ersistent fear of social situations in which the !erson is e*!ose" to unfamiliar !eo!le or to !ossible scrutiny by others. 7he in"ivi"ual fears that he or she will act in a way 'or show an*iety sym!toms) that will be humiliatin$ or embarrassin$. ;:) 7he correct answer is %. &hencycli"ine '&C&) into*ication is characteri#e" by mala"a!tive behavioral chan$es, an" may be associate" with vertical or hori#ontal nysta$mus, hy!ertension, tachycar"ia, numbness or "ecrease" res!onse to !ain, ata*ia, "ysarthria, muscle ri$i"ity, an" sei#ures. &sychotic !henomena '"elusions or hallucinations) may also be !resent. Alcohol with"rawal 'choice A) can be associate" with autonomic hy!eractivity, but alcohol with"rawal woul" not account for the "ysarthria, ata*ia, an" nysta$mus !resent in this

!atient. Other sym!toms of alcohol with"rawal inclu"e han" tremor, insomnia, nausea or vomitin$, transient hallucinations 'commonly visual or tactile), a$itation, an*iety, an" sei#ures. Cocaine into*ication 'choice ) coul" cause behavioral chan$es, !sychotic !henomena, an" elevations in vital si$ns. 2owever, "ysarthria, ata*ia, an" nysta$mus are not usually associate" with cocaine into*ication. O!ioi" with"rawal 'choice C) consists of "ys!horic moo", nausea or vomitin$, myal$ias, lacrimation, rhinorrhea, !u!illary "ilation, !iloerection, sweatin$, "iarrhea, yawnin$, fever, an" insomnia. 7his !atientAs !resentation is not consistent with o!ioi" with"rawal. 1alium with"rawal 'choice () woul" clinically look very much like alcohol with"rawal. With"rawal from ben#o"ia#e!ines can result in serious me"ical com!lications, such as sei#ures. 7he onset of with"rawal sym!toms usually occurs ;40 "ays after the cessation of use, but with lon$4 actin$ "ru$s, such as "ia#e!am '1alium), the latency before onset may be /4C "ays. ;;) 7he correct answer is (. )omati#ation "isor"er is characteri#e" by multi!le me"ical com!laints, resultin$ in si$nificant "ia$nostic testin$ an" me"ical interventions an" causin$ im!aire" social an" occu!ational functionin$. 7he sym!toms, which cannot be e*!laine" by me"ical fin"in$s, inclu"e !ain in at least four sites, one !seu"oneurolo$ic sym!tom, one se*ual "ysfunction sym!tom, an" at least two or more $astrointestinal sym!toms. Conversion "isor"er 'choice A) is a constellation of one or more neurolo$ic sym!toms associate" with !sycholo$ical conflict . 7he sym!toms inclu"e "eficits affectin$ motor or sensory function un"er voluntary control, "isturbances of consciousness, an" !seu"osei#ures. 7here is a clear tem!oral association of the onset of sym!toms an" a stressor. 7he sym!toms are not consciously !ro"uce" an" are not "ue to any other me"ical or !sychiatric con"ition. .enerali#e" an*iety "isor"er 'choice ) is "efine" by an unrealistic or e*cessive worry about activities or life events lastin$ at least C months. In a""ition, si* of the followin$ ty!es of sym!toms must be !resent: fati$ability, muscle tension, irritability, troubles fallin$ or stayin$ aslee!, "ifficulty concentratin$, an" restlessness. 7he sym!toms cause si$nificant im!airment in every"ay functionin$. +aske" "e!ression 'choice C) can be !resente" throu$h somatic com!laints, which can be car"iovascular, $astrointestinal, urinary, or ortho!e"ic, alon$ with "e!ressive sym!toms. 7he "isor"er meets criteria for "e!ressive "isor"er but usually not for somati#ation "isor"er. &ain "isor"er 'choice %) is characteri#e" by !ain as a !rominent feature, alon$ with !sycholo$ical factors that !reci!itate, e*acerbate, an" contribute to its severity. It results in im!airment of every"ay functionin$ an" is not "ue to any other !sychiatric or me"ical con"ition. If a $eneral me"ical con"ition is !resent, it "oesnAt have a role in its onset or maintenance. ;0) is A. 7his !atient has the sym!toms of with"rawal from ben#o"ia#e!ines, such as al!ra#olam 'Eana*). )ym!toms inclu"e insomnia, tremor, $astrointestinal "istress, hallucinosis, an" an*iety. With"rawal from ben#o"ia#e!ines can also be accom!anie" by

$enerali#e" sei#ures. Caffeine 'choice ) with"rawal "oes not have associate" obtun"ation an" usually manifests as mil" !sychomotor a$itation. Cocaine with"rawal 'choice C) is characteri#e" by "ys!horia, !sychomotor a$itation or retar"ation, an" marke" fati$ue. 2eroin with"rawal 'choice %) is characteri#e" by $enerali#e" !ain, nausea, vomitin$, "iarrhea, an" !iloerection '$oose flesh). Nicotine with"rawal 'choice () is characteri#e" by intense cravin$ activity an" mil" to mo"erate !sychomotor a$itation. ;?) 7he correct answer is C. 7his !atient most likely has ma<or "e!ressive "isor"er. )he has ha" sym!toms for ; months, sur!assin$ the %)+4I1 criteria for a minimal len$th of "e!ression of ; weeks. 2er other sym!toms inclu"e loss of a!!etite, hy!ersomnia, "ecrease" libi"o, loss of ener$y an" interest in !leasurable activities, an" va$ue suici"al i"eation, all of which are criteria for ma<or "e!ression. 7he "ia$nosis of bi!olar I "isor"er 'choice A), requires an e!iso"e of mania, with increase" $ran"iosity, irritability, an" im!ulsiveness, either currently or in the !ast4none of which are seen in this case. .enerali#e" an*iety "isor"er 'choice ) requires frequent intermittent e!iso"es of an*iety over a more !rolon$e" !erio" than ; months. 7his "ia$nosis is rule" out in this !atient by the absence of any !rominent sym!toms of an*iety. 7he "ia$nosis of !anic "isor"er 'choice %) requires "iscrete e!iso"es known as !anic attacks, with tachycar"ia, "ia!horesis, an" a sense of im!en"in$ "oom4none of which this !atient "escribes. )chi#o!hrenia 'choice () is rule" out because of the absence of !sychotic sym!toms, such as "elusions, hallucinations, an" "isor$ani#e" thinkin$ ;/) 7he correct answer is %. 7he sym!toms of chan$e in a!!etite, fati$ue, insomnia, lack of concentration, an" worthlessness for $reater than ; weeks are consistent with "e!ression. &ro!ranolol is one of the most common !harmacolo$ical a$ents to cause "e!ression. Other si"e effects of !ro!ranolol inclu"e "i##iness, bronchos!asm, nausea, vomitin$, "iarrhea, an" consti!ation. It may also !reci!itate asthma, con$estive heart failure, an" hy!o$lycemia in susce!tible !atients. Ca!to!ril 'choice A) most commonly causes a cou$h, rash, fever, an" hy!erkalemia. It is contrain"icate" in bilateral renal artery stenosis an" !re$nancy. -urosemi"e 'choice ) causes !otassium "e!letion, hy!er$lycemia, hy!eruricemia, an" hy!ocalcemia. It is contrain"icate" in !atients with hy!eruricemia an" !rimary al"osteronism. +ino*i"il 'choice C) causes hair $rowth 'which is often "esirable), tachycar"ia, an" flui" retention. It is contrain"icate" in !atients with severe coronary artery "isease. 1era!amil 'choice () most commonly causes consti!ation, heart block, hy!erkalemia, an" liver "ysfunction. ;C) (*!lanation:

7he correct answer is . i!olar II "isor"er is characteri#e" by the !resence or history of one or more "e!ressive e!iso"es an" one or more hy!omanic e!iso"es. 2y!omania is "efine" as a "istinct !erio" of !ersistently elevate", e*!ansive, or irritable moo", lastin$ at least ? "ays, but not severe enou$h to cause marke" im!airment in social or occu!ational functionin$ or to necessitate hos!itali#ation. i!olar I "isor"er 'choice A), requires the !resence or history of at least one e!iso"e of full blown mania, with accom!anyin$ $ran"iosity, irritability, an" im!ulsivity. Cyclothymic "isor"er 'choice C) is a "ia$nosis that requires, for at least ; years, the !resence of numerous !erio"s of hy!omanic sym!toms an" numerous !erio"s with "e!ressive sym!toms that "o not meet the criteria for a ma<or "e!ressive e!iso"e. +a<or "e!ressive "isor"er 'choice %) is e*clu"e" as the most likely "ia$nosis because of the !resence of hy!omania. )ubstance abuse 'choice () is not su!!orte" as a "ia$nosis in this !atient without further information. ;D) 7he correct answer is C. A normal or uncom!licate" $rief reaction after the loss of a belove" !erson may resemble "e!ression in some ways 'e.$., chan$es in slee! or a!!etite, sa"ness, with"rawal). 2owever, as the loss becomes remote, the $rief4stricken !erson is able to re4 e*!erience <oy. )elf4blame is focuse" on what was not "one in relation to the "ecease" !erson. Illusions or hallucinations of the "ecease" !erson are common. 7he uncom!licate" $rief reaction can last several months, or lon$er, "e!en"in$ on the relationshi! to the "ecease". %e!ression secon"ary to a $eneral me"ical con"ition 'choice A) can be seen in association with car"io!ulmonary "isease, amon$ other "isor"ersG however, since the sym!toms are relate" to the !reci!itatin$ event, one woul" have to wait till the normal $rief is resolve" an" then reassess the !resence of sym!toms of "e!ression. 7he main "ia$nostic criterion of "ysthymia 'choice ) involves mil"er sym!toms of "e!ression occurrin$ every "ay for at least ; years. +a<or "e!ressive "isor"er 'choice %) can be !reci!itate" by the loss of a belove" !erson, but it has a "istinct quality even thou$h some sym!toms are the same. )uici"al i"eation, $uilt relate" to the !erson alone an" not to the "ecease" !erson, an" feelin$s of worthlessness are common. )i$nificant functional im!airment is ty!ical. &sychosis not otherwise s!ecifie" 'choice () is reserve" as a "ia$nosis for the !resence of !sychotic sym!toms that "o not meet all the criteria for a s!ecific "isor"er an" there is not enou$h information available to establish a more s!ecific "ia$nosis. ;@) 7he correct answer is (. 7his !atient is showin$ the sym!toms of !sycho$enic !oly"i!sia, which is the e*cessive intake of water as a result of a !sychiatric "isor"er. 7he sym!toms of e*cessive water intake inclu"e !olyuria, vomitin$, an" "iarrhea. As the !atient is on a close" !sychiatric unit, the chances of reactions to e*cessive me"ications are rare, $iven the careful monitorin$ of me"ication intake on most !sychiatric units. )urre!titious water consum!tion woul" be easy to overlook in a !atient without a !revious history.

Anticholiner$ic crisis 'choice A) is not a !ossibility in a !atient not receivin$ anticholiner$ic me"ications, such as "i!henhy"ramine or Co$entin. Acute "ystonic reaction 'choice ) is an a"verse reaction of some anti!sychotics, such as halo!eri"ol or trifluo!era#ine, an" is rare in a !atient takin$ ris!eri"one. )erotonin syn"rome 'choice C) is a reaction of autonomic instability that is accom!anie" by fluctuations in bloo" !ressure an" flushin$. It is associate" with the mi*in$ of monoamine o*i"ase inhibitors an" serotonin s!ecific reu!take inhibitors. 7ar"ive "yskinesia 'choice %) is a syn"rome of abnormal involuntary movements associate" with chronic use of ty!ical neurole!tics, such as halo!eri"ol. ;I) (*!lanation: 7he correct answer is . (lectroconvulsive thera!y '(C7) has been safely use" in !re$nancy. In case of !sychotic "e!ression with increase" risk for suici"e, the situation requires e*!e"itious treatment to !rotect the mother an" fetus, an" (C7 is the treatment of choice. Clona#e!am 'choice A) is not in"icate" as a !rimary treatment of "e!ression with !sychotic features. %urin$ the first trimester of !re$nancy, the use of ben#o"ia#e!ines increases the risk of cleft !alate :>4fol". -luo*etine 'choice C), alon$ with tricyclic anti"e!ressants, is currently consi"ere" the safest "ru$ to use "urin$ !re$nancy. In the case of !sychotic "e!ression, however, monothera!y with fluo*etine woul" not be sufficient an" woul" require lon$er in!atient treatment. ,ithium use 'choice %) "urin$ the first trimester of !re$nancy increases the risk for (bsteinAs anomaly :>4;> fol". In a""ition, lithium has not !roven to be an efficient anti"e!ressant in the treatment of !sychotic "e!ression. It is !rimarily use" as a moo" stabili#er. &sychothera!y 'choice () is im!ortant treatment o!tion for hel!in$ women un"erstan" the conflictual feelin$s that arise "urin$ !re$nancy. 2owever, $iven the !sychotic sym!toms an" severity of "e!ression in this case, !harmacothera!y or (C7 are the treatments of choice. 0>) : 7he correct answer is (. Any !atient with serious suici"al thou$hts, suici"al intent, an" a !lan, must be hos!itali#e", a$ainst her will, if necessary. 7he !atient e*!resse" a "esire to "ie, she bou$ht a wea!on, an" "evelo!e" a believable !lan. )he nee"s to be hos!itali#e" for her own safety. &atients with bi!olar "isor"er have a lifetime suici"e rate of :>4:/=. )en"in$ home with lithium 'choice A), val!roic aci" 'choice ), or carbama#e!ine 'choice C) are ina!!ro!riate choices because this !atient requires hos!itali#ation "ue to her hi$h risk of suici"e. A moo" stabili#er is in"icate", but must be $iven as in!atient thera!y. 0:) 7he correct answer is

%. 7he !atient is havin$ sym!toms of !osttraumatic stress "isor"er '&7)%), as well as "e!ressive sym!toms. )ertraline is an anti"e!ressant a!!rove" for treatment of this "isor"er. Other anti"e!ressants an" anticonvulsants have also been shown to be effective in the treatment of &7) %. Insi$ht4oriente" !sychothera!y 'choice A) is focuse" on $ettin$ insi$ht into the un"erlyin$ unconscious conflicts on the basis of e*!loration of transference feelin$s evoke" "urin$ the !rocess. It is not suitable for the treatment of acute &7) %. No thera!y 'choice ) in a !atient with obvious sym!toms that are interferin$ with his or her treatment an" social functionin$ woul" be unacce!table an" consi"ere" ne$lect. A ;?4hour watch by a sitter 'choice C) woul" be in"icate" if the !atient is actively suici"al an" has !oor im!ulse control. 2avin$ a family member for su!!ort is encoura$e", but enforcin$ a re$resse" an" "e!en"ent !osition by a ;?4hour watch woul" not be a!!ro!riate. en#o"ia#e!ines 'choice () can be use" in for sym!tom relief in cases of acute stress reaction, but they are not in"icate" as lon$4term treatment of &7)% because of their a""ictive !otential. 0;) 7he correct answer is A. Al!ra#olam is a short4actin$ ben#o"ia#e!ine with a ra!i" onset of action an" a relatively short half4life. It is in"icate" for treatment of !anic attacks an" an*iety "isor"ers in $eneral. .iven its !ro!erties, however, it also has a hi$hly a""ictive !otential. 7he abru!t "iscontinuation of its use may result in with"rawal "elirium, sei#ures, or "eath. us!irone 'choice ) is an antian*iety a$ent chemically unrelate" to ben#o"ia#e!ines, barbiturates, or se"ativeBan*iolytic "ru$s. It is in"icate" for the mana$ement of an*iety "isor"ers. In human an" animal stu"ies, it has not shown !otential for abuse or !hysical or !sycholo$ical "e!en"ence. 2y"ro*y#ine 'choice C) belon$s to the !i!era#ine $rou! of antihistamines an" is in"icate" for sym!tomatic relief of an*iety an" tension an" as an a"<unct to treatment of or$anic "iseases with an*iety !resent. ecause of its se"ative !ro!erties, it can be use" in !reanesthesia an" it !otentiates the effects of other CN) se"atives. With"rawal "elirium has not been re!orte". Nefa#o"one 'choice %) is an anti"e!ressant structurally unrelate" to +AOs, ))6Is, or tri4 or tetracyclics It is in"icate" for "e!ression an" not solely for an*iety "isor"ers. )tu"ies have not shown that it has any a""ictive !otential or that its "iscontinuation causes "elirium. &aro*etine 'choice () is a selective serotonin reu!take inhibitor '))6I) an" is in"icate" in treatment of an*iety "isor"ers. &aro*etine is not consi"ere" a controlle" substance, an" clinical trials have not reveale" a ten"ency for "ru$4seekin$ behavior. Abru!t "iscontinuation may lea" to flu4like sym!toms, as with some other ))6Is, but not with"rawal "elirium. 00) (*!lanation: 7he correct answer is . Con"uct "isor"er is a chil"hoo"Ba"olescent "isor"er "efine" as a !attern of behavior in which the basic ri$hts of others are violate" with three or more of the followin$ !resent in

the !ast :; months: "estruction of !ro!erty, cruelty to animals an" !eo!le, "eceitfulness or theft, an" serious violations of rules. It causes clinically si$nificant im!airment in social functionin$ an" it is reserve" for !atients youn$er than :@. Attention "eficitBhy!eractivity "isor"er 'choice A) requires the !resence of si* sym!toms of inattention for at least C months to a "e$ree that is mala"a!tive an" si* sym!toms of hy!eractivityBim!ulsivity that cause social im!airment. )ym!toms are !resent in two or more settin$s 'e.$., home an" school), an" some of the sym!toms are !resent before a$e D. 7he sym!toms are not "ue to a $eneral me"ical con"ition or other mental "isor"er. Intermittent e*!losive "isor"er 'choice C) is "ia$nose" in a"ults only after several e!iso"es of failure to resist a$$ressive im!ulses that lea" to assaults or "estruction of !ro!erty. 7he "e$ree of e!iso"es is not !ro!ortionate to !reci!itatin$ stressor. 7he "isor"er is not "ue to any other mental "isor"er or $eneral me"ical con"ition. O!!ositional "efiant "isor"er 'choice %) is a !attern of ne$ativistic an" "efiant behavior lastin$ at least C months with four or more of the followin$: loss of tem!er, ar$uments with a"ults, "efyin$ rules, "eliberately annoyin$ other !eo!le, blamin$ others for own faults, !resence of vin"ictive behavior, !resence of an$er, an" resentment. 6ett syn"rome 'choice () belon$s to the !ervasive "evelo!mental "isor"ers. After a normal "evelo!ment, the onset of "isor"er is between / an" ?@ months an" is characteri#e" by "eceleration of hea" $rowth, loss of !reviously acquire" !ur!oseful han" skills, loss of social en$a$ement, !oor coor"ination of movements, motor mannerisms, an" !reoccu!ation with !arts or ob<ects. 7here is no "elay in lan$ua$e or co$nitive "evelo!ment. 0?) C. +aternity blues is a normal state of sa"ness, "ys!horia, frequent tearfulness, an" "e!en"ence that about ;>= to ?>= of women e*!erience in the !ost!artum !erio". It is thou$ht to be "erive" from ra!i" chan$es in womenAs hormonal levels an" the stress of chil"birth associate" with maternity. A"<ustment "isor"er 'choice A) requires the "evelo!ment of emotional or behavioral sym!toms in res!onse to a stressor occurrin$ within 0 months of the stressor, which also requires si$nificant im!airment in social an" occu!ational functionin$. It is e*clu"e" as a "ia$nosis when the !resence of another A*is I "ia$nosis, such as !ost!artum blues, can account for the con"ition. %ysthymic "isor"er 'choice ) is a "isor"er of "e!resse" moo", more often than not, over the course of at least ; years. It is not an a!!ro!riate "ia$nosis for such a short !erio". &ost!artum "e!ression 'choice %) is a "ia$nosis that requires sym!toms of ma<or "e!ression lastin$ lon$er than /4D "ays. It occurs more often in the months followin$ chil"birth rather than imme"iately subsequent to it. &ost!artum !sychosis 'choice () is a serious "ia$nosis that requires the !resence of au"itory or visual hallucinations in a""ition to frequent suici"al an" sometimes infantici"al i"eation. 0/) 7he correct answer is %. 7his !atient has schi#oi" !ersonality "isor"er, which is characteri#e" by the inability to form relationshi!s an" "ifficulty e*!eriencin$ an" e*!ressin$ emotions. Affecte"

in"ivi"uals "o not seek intimacy an" a!!roval from others. 7hey !refer to be alone an" may !erform well in socially isolate" <obs. 7he inci"ence is thou$ht to be very hi$h, however it is not known since these in"ivi"uals $enerally "o not seek hel!. Accor"in$ to the %)+, !ersonality "isor"ers are characteri#e" by a stable !attern of behavior that "eviates from cultural e*!ectations, is infle*ible, cause "istress, an" social or work im!airment. &ersonality "isor"ers are not cause" by another me"ical illness or substance abuse. In"ivi"uals with antisocial !ersonality "isor"er 'choice A) show a com!lete "isre$ar" for societal norms, rules, an" the interest of others. 7hey frequently en" u! in !rison. Avoi"ant !ersonality "isor"er 'choice ) is characteri#e" by social with"rawal "ue to the fear of criticism. Affecte" in"ivi"uals are ea$er to !lease an" "esire affection. In"ivi"uals with !aranoi" !ersonality "isor"er 'choice C) are sus!icious an" mistrustful, an" frequently misinter!ret the actions of others. )chi#oty!al !ersonality "isor"er 'choice () is characteri#e" by o"" behavior, co$nition, an" !erce!tion. 7hese !eo!le seem similar to schi#o!hrenics, but "o not have !sychosis. )ocially !hobic 'choice -) in"ivi"uals "esire relationshi!s but their an*iety often interferes with the formation of frien"shi!s. 7hey fear unfamiliar situations an" scrutiny by others. 0C) 7he correct answer is C. 7his !atientAs sym!toms are all su$$estive of !anic attacks. 2er sym!toms "o not a!!ear to be relate" to substance use or a $eneral me"ical con"ition. 7he frequency of her sym!toms an" her a$ora!hobia 'fear of social situations) are also in"icative of a "ia$nosis of !anic "isor"er. -actitious "isor"er 'choice A) is a "isor"er in which !hysical sym!toms are intentionally !ro"uce" or fei$ne". 7he motivation for the behavior in the !atient is to assume the sick role. .enerali#e" an*iety "isor"er 'choice ) is characteri#e" by e*cessive an*iety an" worry occurrin$ more "ays than not for at least C months, an" concernin$ activities such as work or school !erformance, with accom!anyin$ muscle tension, irritability, an" slee! "isturbance. )chi#o!hrenia 'choice %) is a thou$ht "isor"er characteri#e" by au"itory hallucinations of at least C monthsA "uration. )ocial !hobia 'choice () is characteri#e" by fear an" avoi"ance of social situations, even thou$h it is often accom!anie" by a "esire for social contact an" interaction with others. It is often accom!anie" by low self4esteem. 0D) C. In schi#o!hreniform "isor"er or schi#o!hrenia, several features are associate" with better lon$4term !ro$nosis. 7he onset of !sychotic sym!toms within four weeks of the first noticeable chan$e in usual behavior or functionin$ is one such feature. Other $oo" !ro$nostic features inclu"e: confusion or !er!le*ity at the hei$ht of the !sychotic e!iso"e, $oo" !remorbi" social an" occu!ational functionin$, an" the absence of blunte" or flat affect. A family history of schi#o!hrenia 'choice A) is associate" with a !oorer !ro$nosis in schi#o!hrenia, whereas a family history of moo" "isor"ers 'es!ecially "e!ressive "isor"ers) is a $oo" !ro$nostic in"icator. A lack of !reci!itatin$ factors or acute stressors 'choice ) is a !oor !ro$nostic in"icator, whereas obvious !reci!itatin$ factors before an e!iso"e of !sychosis may be associate" with a better !ro$nosis. &oor

!remorbi" e"ucational or occu!ational functionin$ 'choice %) is a !oor !ro$nostic in"icator. With"rawn, autistic behavior an" flat affect 'choice () are !oor !ro$nostic in"icators, whereas the lack of these ne$ative sym!toms may be associate" with a better !ro$nosis. 0@) . Narcissistic !ersonality "isor"er !atients frequently "eman" constant attention an" a"miration, an" they are often in"ifferent to criticism. 7hey frequently e*hibit $ran"iosity in behaviors or fantasies, show lack of em!athy, often e*hibit a sense of entitlement, an" frequently e*!loit whatever inter!ersonal relationshi!s they have "evelo!e". or"erline !ersonality "isor"er 'choice A) is "istin$uishe" by instability in self4ima$e, moo", an" inter!ersonal relationshi!s. &aranoi" !ersonality "isor"er 'choice C) is marke" by ri$i"ity, unwarrante" sus!icion, envy, an" a ten"ency to blame an" attribute evil motives to others. &assive4a$$ressive 'choice %) !ersonality "isor"er requires the manifestation of a$$ressive behavior in !assive ways such as intentional inefficiency or stubbornness. )chi#oty!al !ersonality "isor"er 'choice () is e*em!lifie" by eccentric behavior or communication with associate" "efects in the ability to form social relationshi!s. 0I) A. ehavioral thera!y is the most frequently use" treatment in chil"ren with enuresis. %ry ni$hts are recor"e" on a calen"ar an" rewar"e" with a star as a $ift. 7he bu##er an" !a" a!!aratus are use" less for con"itionin$ nowa"ays. Inter!ersonal thera!y 'choice ) is short4term thera!y "evelo!e" for non!sychotic, mil"er forms of "e!ression. It a""resses current relationshi!s an" roles, an" is use" with a"ults. It is not in"icate" for enuresis treatment. &harmacothera!y 'choice C) is rarely use", $iven the success of behavioral a!!roaches. 7olerance to imi!ramine, which has been use", can "evelo! within C weeks. %esmo!ressin has shown some success. &sycho"rama 'choice %) is a metho" of $rou! thera!y in which conflicts an" inter!ersonal relationshi!s are e*!lore" by means of s!ecial "ramatic metho"s. It is not in"icate" in chil"ren with enuresis. &sychothera!y 'choice () is not recommen"e" unless there is evi"ence of other !sycho!atholo$y. 7he e*!loration of conflicts in enuresis has shown little success. ?>) 7he correct answer is A. 7his !atient has a$ora!hobia. Althou$h this con"ition was ori$inally "efine" as the fear of o!en s!aces or of the market!lace, a more functional, mo"ern "efinition is a fear of !anic attacks in situations from which it woul" be "ifficult to $racefully remove oneself. ,ike the !atient in the question stem, a !atientAs fear may involve multi!le settin$s an" may !ro$ress to the !oint of marke"ly ham!erin$ "aily functionin$. 7he !anic attacks may or may not actually have been e*!erience" in the !ast in the !articular settin$s that are of concern to the !atient. )ome cases resolve s!ontaneouslyG others !ursue a wa*in$ an" wanin$ course. ehavioral thera!y is use" to encoura$e !atients to mo"ify their activities. Anti"e!ressants are hel!ful in !atients with coe*istin$ "e!ression.

Conversion "isor"er 'choice ) refers to the "evelo!ment of !hysical sym!toms in res!onse to !sycholo$ical conflict. Obsessive4com!ulsive "isor"er 'choice C) is characteri#e" by recurrent an" obtrusive i"eas an" ur$es. )ocial !hobia 'choice %) s!ecifically refers to clinically si$nificant an*iety in"uce" by social or !erformance situations. Althou$h the overla! with a$ora!hobia shoul" be obvious, a ti! off that a !erson has a$ora!hobia, rather than social !hobia, on a test question 'real life is fu##ier) is the use of the term H!anic attacks. H )omati#ation "isor"er 'choice () is a severe !sychiatric "isor"er in which a !atient has ha" many !hysical com!laints over a !erio" of years that "o not have an a"equate anatomic !atholo$ic e*!lanation. ?:) 7he correct answer is . 7he !atient is e*!eriencin$ an acute "ystonic reaction, which is a form of e*tra!yrami"al si"e effect '(&)) associate" with anti!sychotic me"ications. 7hese si"e effects are relate" to anta$onism of "o!amine rece!tors in the ni$rostriatal !athway. 7he !atient is likely takin$ a conventional anti!sychotic a$ent !lus a !ro!hylactic anticholiner$ic a$ent 'such as ben#tro!ine, "i!henhy"ramine, or trihe*y!heni"yl). F!on sto!!in$ the anticholiner$ic, the "ystonic reaction was more likely to occur. 7he a!!ro!riate initial mana$ement of this !atient woul" inclu"e imme"iate I+ a"ministration of an anticholiner$ic a$ent, such as ; m$ of ben#tro!ine or /> m$ of "i!henhy"ramine. Al!ra#olam 'choice A) is a short4actin$, hi$h4!otency ben#o"ia#e!ine commonly use" for the short4term mana$ement of an*iety. It woul" not be the a!!ro!riate treatment for an acute "ystonic reaction. 2alo!eri"ol 'choice C) is a conventional hi$h4!otency anti!sychotic a$ent that woul" be likely to cause (&) such as "ystonia. It woul" make the "ystonic reaction worse. ,ora#e!am 'choice %) is, like al!ra#olam, a ben#o"ia#e!ine. It is commonly use" in the acute mana$ement of severe an*iety an" a$itation. It woul" not be a!!ro!riate in the mana$ement of acute "ystonia. Olan#a!ine 'choice () is an aty!ical anti!sychotic a$ent. ,ike the other aty!ical a$ents 'clo#a!ine, ris!eri"one, an" quetia!ine), it is associate" with a lower inci"ence of (&) while bein$ more efficacious for the ne$ative sym!toms of schi#o!hrenia. It woul" not be a treatment for (&). ?;) (*!lanation: 7he correct answer is C. Imi!ramine is a tricyclic anti"e!ressant '7CA) that inhibits nore!ine!hrine an" serotonin reu!take. ,ike most 7CAs, imi!ramine also has anta$onistic effects at muscarinic, histaminic, an" a4a"rener$ic rece!tors. 7his !atientAs com!laint of blurre" vision is most likely "ue to the antimuscarinic effects of the me"ication. 7he blocka"e of muscarinic acetylcholine rece!tors causes my"riasis '!u!illary "ilation) resultin$ in blurre" vision. %ry mouth is also "ue to the anticholiner$ic effect of imi!ramine. Orthostatic hy!otension is cause" by the a:4a"rener$ic rece!tor blocka"e associate" with 7CAs. u!ro!ion 'choice A) is an anti"e!ressant with an unknown mechanism of action, thou$h some evi"ence su$$ests that it is a nore!ine!hrine an" "o!amine reu!take

inhibitor. 7reatment with bu!ro!ion is characteri#e" by the absence of si$nificant "ru$4 in"uce" orthostatic hy!otension or anticholiner$ic effects. Citalo!ram 'choice ) is a selective serotonin reu!take inhibitor '))6I) with no si$nificant effects at choliner$ic, a"rener$ic, or histaminic rece!tors. 7herefore, it woul" not cause the cluster of sym!toms seen in this !atient. Nefa#o"one 'choice %) acts !rimarily as an anta$onist at serotonin4; '/427;) rece!tors, althou$h it is also a weak inhibitor of serotonin reu!take. Althou$h it is relate" to tra#o"one, it lacks si$nificant anta$onistic activity at other rece!tors. )ertraline 'choice () is an ))6I, like citalo!ram, an" woul" not cause the si"e effects !resent in this !atient. ?0) 7he correct answer is %. Narcissistic !ersonality "isor"er is "efine" by $ran"iose self, belief in oneAs own s!ecial value, nee" for a"miration from others, sense of entitlement, lack of em!athy, envy of others, an" arro$ant attitu"e. 7he countertransference feelin$s that most !eo!le have towar" these !atients inclu"e a sense of bore"om, since !atient is focuse" com!letely on his or her own issues an" is oblivious of others. Antisocial !ersonality "isor"er 'choice A) is si$nificant for ma<or violation an" "isre$ar" of the ri$hts of others occurrin$ after a$e :@, as in"icate" by failure to conform to social norms, im!ulsivity, reckless "isre$ar" for safety of self an" others, lack of remorse, an" "eceitfulness. 2istrionic !ersonality "isor"er 'choice ) inclu"es e*cessive emotionality an" attention4 seekin$, shallow e*!ression of emotions, s!eech lackin$ "etail, su$$estibility, an" a sense that relationshi!s are more intimate than they are. At the initiation of treatment, these !atients are seemin$ly the most interestin$. +ania 'choice C) requires a "istinct !erio" of at least : week of abnormally elevate" moo", $ran"iosity, "ecrease" slee!, racin$ thou$hts, !ressure" s!eech, an" involvement in !leasurable activities. Countertransference feelin$s in others "ealin$ with these !atients inclu"e a sense of cheerfulness an" irritability, but certainly not bore"om. &aranoi" schi#o!hrenia 'choice () requires the !resence of at least two of the followin$: "elusions, hallucinations, "isor$ani#e" s!eech or behavior, an" ne$ative sym!toms for at least : month. %elusions of $ran"iosity, arro$ance, an" entitlement can be !art of the clinical !icture. ??) . A"<ustment "isor"er is the "evelo!ment of emotional or behavioral sym!toms in res!onse to an i"entifiable stressor occurrin$ within 0 months of the onset of the stressor. 7hese sym!toms or behaviors are clinically si$nificant as evi"ence" by marke" "istress in e*cess of what woul" be e*!ecte" from e*!osure to the stressor, or by si$nificant im!airment in social or occu!ational functionin$. In this case, the stressor is the "ia$nosis of !ancreatic cancer, which im!airs the !atientAs ability to function in the !hysician4 !atient relationshi! without "enial. Acute stress "isor"er 'choice A) is a !sychiatric "isor"er resultin$ from e*!osure to a traumatic event that involves actual or threatene" imme"iate "eath or serious in<ury. 7he !ossibility of "eath in this instance is not imme"iate. 7he !atient shows no si$ns of "isorientation, an" thus "oes not meet criteria for "ementia, so Al#heimerAs "ementia 'choice C) is incorrect. %elusional "isor"er

'choice %) is incorrect because this "ia$nosis requires the !resence of a nonbi#arre "elusion of at least : months "uration, an" this !atient has ha" his "enial for only a few "ays. +a<or "e!ressive "isor"er 'choice () is incorrect because the !atient has not ha" the insomnia, "e!resse" moo", or thou$hts of suici"e that are ty!ical of a "ia$nosis of ma<or "e!ressive "isor"er. ?/) 7he correct answer is . .enerali#e" an*iety "isor"er is "efine" as unrealistic worry about life events for a !erio" lon$er than C months, "urin$ which time a !erson is worrie" most "ays. It also inclu"es at least si* sym!toms of the followin$ ty!es: easy fati$ability, "ifficulties fallin$ aslee!, restlessness, "ifficulties concentratin$, irritability, an" muscle tension. 7he sym!toms are not "ue to other !sychiatric or me"ical con"itions an" they cause si$nificant im!airment in every"ay functionin$. Avoi"ant !ersonality "isor"er 'choice A) is a !ervasive !attern of social inhibition, feelin$s of ina"equacy, an" hy!ersensitivity to ne$ative evaluation as seen throu$h at least four sym!toms of the followin$ ty!e: self4 ima$e of social ine!titu"e, !reoccu!ation with bein$ "islike", avoi"ance of social activities for fear of bein$ ri"icule", an" feelin$s of ina"equacy in inter!ersonal relationshi!s. 2y!ochon"riasis 'choice C) is "efine" as e*cessive concern with oneAs health or "iseases. It is !resent most of the time an" is un<ustifie" by the amount of !hysical !atholo$y. It must be !resent at least C months an" "oes not res!on" to reassurance. 7he main features inclu"e fear of "isease, bo"ily !reoccu!ation with somatic com!laints, feelin$s of frustration towar" "octors, an" "octor4sho!!in$. It causes si$nificant im!airment in functionin$ an" is not "ue to other !sychiatric or me"ical con"itions. Obsessive4com!ulsive "isor"er 'choice %) is characteri#e" by recurrent intrusive thou$hts or im!ulses that cause stress. 7he !erson unsuccessfully attem!ts to i$nore or neutrali#e throu$h action the recurrent thou$hts or im!ulses, even thou$h he or she is aware that those thou$hts are the !ro"uct of his or her own min". Com!ulsions are re!etitive behaviors that the !erson nee"s to !erform as a result of obsessions or to "ecrease the stress relate" to intrusive thou$hts. 7he "isor"er causes si$nificant im!airment in every"ay functionin$ an" is not "ue to other !sychiatric or me"ical con"itions. &anic "isor"er 'choice () is "efine" by recurrent !anic attacks an" !ersistent concern about havin$ new attacks, worries about the im!lications of attacks, an" si$nificant chan$es in behavior relate" to attacks. 7he "isor"er is not "ue to any other me"ical or !sychiatric con"itions. ?C) 7he correct answer is A. Acute "ystonia is an involuntary s!asm of a !articular $rou! of muscles that can involve the neck, <aw, ton$ue, eyes, or the entire bo"y. It can be an early a"verse effect of anti!sychotics, an" it is more common in youn$er men. It is more common with ty!ical anti!sychotics. 7he treatment of choice is !arenteral a"ministration of anticholiner$ics. Akathisia 'choice ) is a sub<ective feelin$ of muscle "iscomfort an" restlessness that can cause a$itation, !acin$, an*iety, an" "ys!horia. It is relate" to the use of anti!sychotics

an" can a!!ear any time "urin$ treatment. It is treate" by anti!sychotic "ose re"uction, !ro!ranolol, or ben#o"ia#e!ines. Asteri*is 'choice C) consists of coarse arrhythmic la!ses of sustaine" !osture. It is relate" to metabolic "isor"ers an" is most easily seen when the !atientAs arms are outstretche". Asteri*is occurs bilaterally. ,enno*4.astaut syn"rome 'choice %) is a variant of !etit mal e!ile!sy an" consists of intellectual im!airment, "istinctive slow s!ike an" wave !attern, an" atonic !ostural la!ses followe" by minor tonic4clonic s!asms. &seu"osei#ures 'choice () are !art of conversion "isor"er an" are not "irectly relate" to the use of anti!sychotics. 7hey are very much like real sei#ures e*ce!t that there is no aura an" no ((. abnormalities. 7he movements are asynchronous an" non4stereoty!e", an" they occur when the !erson is awake. ?D) 7he correct answer is A. Con"itione" avoi"ance is a term that "escribes the !airin$ of an un!leasant stimulus with the stimulus that causes mala"a!tive behavior. (*tinction 'choice ) requires the removal of the rewar" for ina!!ro!riate behavior so that mala"a!tive behavior "ecreases. It is often use" in chil" !sychiatry with !atients who have behavioral !roblems. -loo"in$ 'choice C) is a thera!eutic technique in which a !atient is e*!ose" to the feare" situation without the !ossibility to esca!e. 7his e*!erience is stressful an" it must be "one in a su!ervise" an" controlle" manner. &ositive reinforcement 'choice %) ha!!ens when a sub<ect is rewar"e" for manifestin$ "esire" behavior. 6eci!rocal inhibition 'choice () ha!!ens when a res!onse that is anta$onistic to the un"esire" behavior is !aire" with the behavioral res!onse 'e.$., rela*in$ alon$ with an*iety !rovokin$ stimuli). ?@) 7he correct answer is (. Inter!retation is a technique use" when a thera!ist states somethin$ about the !atientAs behavior or thou$hts of which the !atient may not be aware. It is use" after $oo" ra!!ort with the !atient has been establishe", in or"er to be timely an" effective. A"vice 'choice A) is, in many instances, "esirable to hel! the !atient. It must be !erceive" as em!athic rather than intrusive. 7hus, it shoul" be $iven after the !atient has s!oken freely about the !roblematic issue. Clarification 'choice ) is a technique in which the thera!ist attem!ts to $et "etails from a !atient about the thin$s alrea"y sai". Confrontation 'choice C) is a technique use" when the thera!ist !oints out a thou$ht or behavior that the !atient is not !ayin$ attention to, or missin$, or "enyin$ in some way. It is meant to hel! the !atient face the !roblem in a "irect but res!ectful way. -acilitation 'choice %) is a technique use" by the thera!ist to hel! the !atient continue the interview by !rovi"in$ verbal an" nonverbal cues to encoura$e the !atientAs talkin$. ?I) (*!lanation: 7he correct answer is

%. Althou$h ris!eri"one is an aty!ical anti!sychotic, it is like conventional anti!sychotics in its ability to cause si$nificant elevations in !lasma !rolactin levels. In the tuberoinfun"ibular "o!amine !athway, "o!amine inhibits the release of !rolactin from the anterior !ituitary. Conventional anti!sychotics an" ris!eri"one can cause hy!er!rolactinemia "ue to their "o!amine anta$onism in this !athway, releasin$ the tonic "o!amine inhibition. Clinical manifestations of hy!er!rolactinemia may inclu"e $alactorrhea, se*ual "ysfunction, menstrual irre$ularities inclu"in$ amenorrhea, infertility, an" wei$ht $ain. Olan#a!ine 'choice A) is an aty!ical anti!sychotic a$ent that causes minimal, if any, elevation in !rolactin concentrations. It woul" be very unlikely that routine "oses of olan#a!ine woul" cause sym!toms of hy!er!rolactinemia. &aro*etine 'choice ) is a selective serotonin reu!take inhibitor '))6I) use" in the treatment of "e!ression an" certain an*iety "isor"ers. It woul" not in"uce hy!er!rolactinemia causin$ the above clinical manifestations. 8uetia!ine 'choice C) is an aty!ical anti!sychotic that causes minimal, if any, elevation in !rolactin concentrations. )ertraline 'choice () is an ))6I an" woul" not cause hy!er!rolactinemia. />) 7he correct answer is A. In factitious "isor"er, the in"ivi"ual "eliberately fei$ns the si$ns of a me"ical or mental "isor"er to assume the HsickH role. 7he sym!toms are un"er voluntary control an" cannot be e*!laine" by other un"erlyin$ con"ition. (*ternal incentives for such behavior are usually absent. 7he "isor"er is severely inca!acitatin$ to the !atients, since they usually have multi!le hos!itali#ations an" submit themselves to invasive !roce"ures an" sur$eries. .anser syn"rome 'choice ) belon$s to the $rou! of "issociative "isor"ers, not otherwise s!ecifie". It has been re!orte" in incarcerate" !o!ulations an" is calle" H!rison !sychosis.H It is characteri#e" by the !rovision of a!!ro*imate answers to questions an" is commonly associate" with con"itions such as amnesia, "isorientation, an" !erce!tual "isturbances. 2y!ochon"riasis 'choice C) involves the e*cessive an" !ervasive !reoccu!ation with the fear of havin$ a serious illness base" on a misinter!retation of bo"ily sym!toms. It must be !resent at least C months an" causes si$nificant im!airment in all areas of life. It "oes not res!on" to reassurance an" it !ersists "es!ite me"ical evaluation. +alin$erin$ 'choice %) is the voluntary !ro"uction of !hysical or !sycholo$ical sym!toms to obtain obvious reco$ni#able secon"ary $ain, such as receivin$ financial com!ensation an" avoi"in$ work or le$al !roblems. )omati#ation "isor"er 'choice () is characteri#e" by multi!le somatic com!laints that be$in before the a$e of 0> an" result in me"ical treatment an" si$nificant im!airment of social an" occu!ational functionin$. 7he !atient has to meet all four criteria: four !ain sym!toms, two $astrointestinal sym!toms, one se*ual sym!tom, an" one !seu"oneurolo$ic sym!tom. 7he sym!toms are not "ue to any me"ical con"ition.

USMLE Step 2 Practice Test Block 1 :ame; +nstr#ctions; Ans'er the *#estions $elo' to the $est of yo#r a$ility. =hen yo# finish the test% click the 2heck $#tton at the $ottom to )ie' the res#lts.

1.A 2 !year!ol" mo"erately o$ese 'oman 'ith a history of $ipolar "isor"er "iagnose" at age 18 complains to her physician that her recent str#ggles 'ith her 'eight an" eating ha)e ca#se" her to feel "epresse". She states that she has recently ha" increase" "iffic#lty sleeping% has felt e.cessi)ely an.io#s an" agitate"% an" has ha" increasing tho#ghts of s#ici"e. She often fin"s herself fi"gety an" #na$le to sit still for e.ten"e" perio"s of time. /er family tells her that she is increasingly irrita$le. She has ne)er pre)io#sly attempte" s#ici"e. /er c#rrent me"ications incl#"e lithi#m car$onate an" Dolpi"em as nee"e" for sleep. =hich of the follo'ing is more consistent 'ith a "iagnosis of ma9or "epressi)e "isor"er than $ipolar "epression in this patient "#ring this episo"ea6Early onset of $ipolar "isor"er $6+ncrease" appetite c6+nsomnia "6+ncrease" tho#ghts of s#ici"e e6Psychomotor agitation :ormal La$s 2.A &!year!ol" girl is $eing e)al#ate" for "e)elopmental "elays. /is parents report that the pregnancy 'as #ne)entf#l an" that the "eli)ery 'as 'itho#t complications. The girl reache" all milestones normally "#ring his first year. Length% 'eight% an" hea" circ#mference 'ere all 'ell 'ithin the normal range "#ring the first year. @#ring her secon" year% the pe"iatrician notice" "eceleration of hea" gro'th% an" her parents notice" a gra"#al "ecline in her fine motor a$ilities% fre*#ent

han"!'ringing mo)ements% an" loss of social engagement. /er mo)ements $ecame poorly coor"inate". =hich of the follo'ing is the most likely "iagnosisa6Asperger "isor"er $6Attention!"eficit7hyperacti)ity "isor"er c6A#tistic "isor"er "6 Per)asi)e "e)elopmental "isor"er% not other'ise specifie" e6Aett syn"rome :ormal La$s &. A &2!year!ol" man is in t'ice!'eekly insight!oriente" psychotherapy 'ith a psychiatrist. Aecently% the patient has $een e.ploring his tho#ghts an" feelings aro#n" his 'ife,s complaint that he is too restricte" an" inhi$ite" in their se.#al acti)ity. The patient a"mits that he 'ishes to $e more se.#ally a)aila$le for his 'ife% $#t fin"s himself maintaining a restricte" stance. =hich of the follo'ing "efense mechanisms 'o#l" $est "escri$e this patient,s ten"ency in his se.#al relationship 'ith his 'ifea6Pro9ection $6Aeaction formation c6Se.#aliDation "6SomatiDation e6S#$limation :ormal La$s 4.A &(!year!ol" +? heroin a$#ser% 'ho is /+?!positi)e an" has many legal an" social pro$lems% has $een a"mitte" to the hospital for treatment of pne#monia. @#ring his stay% he intermittently ref#ses to #n"ergo proce"#res or cooperate 'ith "octors $#t insists on going o#t to smoke an" ha)ing )isitors. >n one occasion% a )isitor 'as ca#ght in9ecting "r#gs into his +? inf#sion. =hen confronte"% the patient thre' a tray at the staff an" acc#se" them of $eing #nprofessional. =hich of the follo'ing is the most likely "iagnosisa6Antisocial personality "isor"er $6Bor"erline personality "isor"er c62on"#ct "isor"er "6 Ma9or "epressi)e "isor"er 'ith an.io#s7agitate" feat#res

e6Paranoi" personality "isor"er :ormal La$s .A &2!year!ol" man presents to his primary care physician stating that he feels )ery lonely an" 'o#l" )ery m#ch like to ha)e a close circle of frien"s. /o'e)er% he feels )ery an.io#s in social sit#ations an" a)oi"s social in)itations from ac*#aintances at 'ork. /e states he 'o#l" like to $e more interacti)e in social e)ents% $#t that he 'orries that other people 'ill notice his ina"e*#acies an" talk a$o#t his pro$lems. =hich of the follo'ing is the most likely "iagnosisa6Antisocial personality "isor"er $6@epen"ent personality "isor"er c6CeneraliDe" an.iety "isor"er "6SchiDoi" personality "isor"er e6Social pho$ia :ormal La$s <. A 12!year!ol" $oy is $ro#ght to the physician $eca#se of a pattern of $eha)ior that has $een 'orsening o)er the past year. /is mother states that he has $een $#llying other $oys at school% staying o#t late 'itho#t permission% setting small fires in a$an"one" lots% an" physically a$#sing neigh$orhoo" cats. @#ring the e.amination% a spec#l#m from the e.amination room "ra'er falls o#t from #n"erneath his shirt. =hich of the follo'ing is the most likely "iagnosisa6Antisocial personality "isor"er $6Attention "eficit hyperacti)ity "isor"er 3A@/@6 c62on"#ct "isor"er "6>ppositional "efiant "isor"er 3>@@6 e6To#rette syn"rome :ormal La$s (. A 2(!year!ol" male prisoner 'ith a self!"escri$e" history of physical a$#se is $ro#ght to the emergency "epartment $y prison staff. The patient states that he has se)ere leg pain after falling o#t of his $#nk $e" an" that he is #na$le to 'alk. :e#rologic e.amination sho's normal "eep ten"on refle.es% $#t the patient has "ecrease" sensation to pain an" pinprick. The follo'ing morning% the patient is seen 'alking% $#t

complains that he is #na$le to pass #rine% an" he is tol" $y the n#rse that he 'ill ha)e to ha)e a catheter inserte". As his physician 'alks in #ne.pecte"ly on ro#n"s% the patient is seen sneaking $ack into $e" from the "irection of the restroom. =hich of the follo'ing is the most likely "iagnosisa6Antisocial personality "isor"er $62on)ersion "isor"er c6@r#g "epen"ence "65actitio#s "isor"er e6SchiDophrenia :ormal La$s 1.A 4th!year me"ical st#"ent is performing a mental stat#s e.amination on a (1!year!ol" man 'ith a history of AlDheimer "isease an" recent cogniti)e "ecline. She asks the patient 'hat he 'o#l" "o if he fo#n" someone else,s social sec#rity check mi.e" in 'ith his o'n mail at home. =hich of the follo'ing is the st#"ent testing in the mental stat#s e.amination $y asking this *#estiona6 A$stract thinking $62ognition c6+nsight "6+ntelligence e6 R#"gment :ormal La$s 8.A 2<!year!ol" man is $ro#ght to the hospital $y his family after sitting in his room 'ith the lights o#t an" the "oor close" for t'o!"ays. /e has not eaten o)er this time. A$o#t a 'eek ago% the family notice" the patient $ecoming increasingly agitate" an" paranoi" a$o#t cars "ri)ing $y on the street in front of their ho#se. /e co)ere" the 'in"o's of his $e"room 'ith ne'spaper an" #npl#gge" his ra"io an" tele)ision. At night% he 'as hear" pacing in his room an" talking to himself. Altho#gh he ha" taken olanDapine after a psychiatric hospitaliDation a$o#t si. months pre)io#sly% the family reporte" that he thre' a'ay the me"ication a$o#t a month ago. >n a"mission% the patient is sitting in a chair 'ith his hea" h#ng lo'. /e is "ishe)ele" an" malo"oro#s% after ha)ing #rinate" on himself se)eral times o)er the past t'o "ays. @#ring physical e.amination% the patient appears to $e a'ake% $#t firmly resists any attempts to $e mo)e". /e "oes not follo' instr#ctions% an" the n#rse 'as #na$le to mo)e his arm to o$tain a $loo" press#re

meas#rement 'itho#t assistance. =hich term 'o#l" $est "escri$e this patient,s resistance to $eing mo)e"a6 Akathisia $62ataple.y c6Echopra.ia "6:egati)ism e6Stereotypy :ormal La$s 14. A &!year!ol" 'oman is a"mitte" for "epression 'ith psychotic feat#res. @#ring the e.amination% the psychiatrist asks her 'hether she has e)er ha" pro$lems 'ith her heart. The patient,s respon"s% 0=hat are yo# saying- + "on,t ha)e a heart any moreG there is nothing left insi"e me.0 =hich of the follo'ing is the patient most likely e.hi$itinga62enesthetic hall#cination $6@epersonaliDation c6:ihilistic "el#sion "6>)er)al#e" i"ea e6?er$igeration :ormal La$s 11. A 21!year!ol" 'oman 'ith a history of paranoi" schiDophrenia is $ro#ght $y a frien" to the hospital. The 'oman ha" $een an inpatient at a psychiatric hospitalG for se)eral months after $eing "ischarge"% she ha" $een maintaine" on haloperi"ol "ecanoate shots. 5or the past co#ple of "ays% after the last in9ection% she has appeare" 0strange.0 She is stiff% cannot s'allo' or talk% an" appears trem#lo#s. The frien" is concerne" that she has some kin" of infection% since she has a fe)er. >n e.amination% her temperat#re is &1.( 2 3141.( 56% $loo" press#re is 1 (7144 mm /g% p#lse is 1227min% an" respirations are 247min. She has increase" tone in her neck an" e.tremities% an" appears trem#lo#s% "iaphoretic% an" conf#se". /er le#kocyte co#nt is 18%<447mm& an" the ser#m creatine phosphokinase is marke"ly ele)ate". A 'ork#p for infection is negati)e. =hich of the follo'ing is the most likely "iagnosisa6 Ac#te "ystonic reaction

$6Lethal catatonia c6Malignant hyperthermia "6:e#roleptic malignant syn"rome e6Serotonin syn"rome :ormal La$s 12. A 48!year!ol" 'oman% 'ho is $eing treate" 'ith chemotherapy after s#rgery for $reast cancer% presents at the emergency "epartment of the local hospital. She ha" 9#st starte" taking prochlorperaDine for na#sea the "ay $efore. She is $ro#ght in $y the her h#s$an" $eca#se she has $een acting $iDarrely o)er the past 24 ho#rs% 'ith 'a.y fle.i$ility in her mo)ements an" m#tism. She has no pre)io#s psychiatric history. /er physical e.amination% la$oratory tests% an" )ital signs are all 'ithin normal limits. =hich of the follo'ing is the most appropriate pharmacotherapya6AlpraDolam $6BenDtropine c6/aloperi"ol "6Methylpheni"ate e6?alproic aci" :ormal La$s 1&. A &<!year!ol" 'oman 'ith a long history of alcoholism presents to the emergency "epartment into.icate". >n physical e.amination% she is "isoriente" an" conf#se"% an" has ata.ia% "ysarthria% an" oc#lomotor paralysis. =hich of the follo'ing intra)eno#s s#$stances sho#l" most likely $e a"ministere" firsta6Cl#cose $6/aloperi"ol c6LoraDepam "6Thiamine e6?alproic aci" :ormal La$s 14.A 42!year!ol" man is "iagnose" $y his primary care physician as ha)ing ma9or "epressi)e "isor"er. The patient tells his physician that he is e.tremely concerne" a$o#t his se.#al performance% as he is 'orrie" that he is getting ol"er an" that he is ha)ing some marital "iffic#lties

'ith his 'ife. =hich of the follo'ing anti"epressants 'o#l" $e the $est choice for initial treatmenta6B#propion $65l#o.etine c6+mipramine "6Paro.etine e6Sertraline :ormal La$s 1 . A patient is seen on the psychiatry cons#ltation liaison ser)ice $y a me"ical st#"ent. To assess the patient,s cogniti)e f#nctions% the st#"ent asks the patient 'hat the pro)er$ 0@on,t cry o)er spille" milk0 means. The patient ans'ers that if yo# spill 'hat yo# cook% yo# ha)e to "o it all o)er again. =hich of the follo'ing types of thinking is this patient most likely e.hi$itinga6A$stract thinking $6Blocking c62oncrete thinking "62onfa$#lation e6Magical thinking :ormal La$s 1<.A 2&!year!ol" 'oman is $ro#ght into the emergency "epartment $y am$#lance at mi"night% after her roommate fo#n" her #sing a paper clip to make lacerations on her 'rists. The patient ha" $een seen in the same emergency "epartment t'ice "#ring the past 2 'eeks 'ith a complaint of feeling )ery "epresse" an"% at times% s#ici"al. She has a history of one "r#g o)er"ose on chil"ren,s )itamins. +n $oth presentations to the emergency "epartment% she 'as prescri$e" $enDo"iaDepines for an.iety an" sent home 'ith an o#tpatient psychiatric clinic referral. >n this presentation% she states that if she is not a"mitte" to the hospital she 'ill go o#t an" fin" a $ri"ge to 9#mp off. =hich of the follo'ing is the most appropriate initial step in managementa6 A"mit the patient to the me"ical 'ar" an" a'ait psychiatric cons#ltation in the morning $6E.plain to the patient that her attempts to m#tilate herself 'ill not res#lt in s#ici"e% an" that she nee"s o#tpatient psychotherapy

c6 >r"er an emergency psychiatric assessment "6Aemo)e the patient from hospital gro#n"s $y sec#rity escort e6Set #p a ne' o#tpatient psychiatric clinic appointment :ormal La$s 1(. A &2!year!ol" man 'ith schiDophrenia is $ro#ght to the hospital $y his sister $eca#se he has $ecome incoherent an" locks himself in his room for "ays at a time. /is family is concerne" $eca#se he has trie" many antipsychotic agents an" none of them seem to ha)e 'orke". @#ring the e.amination% the patient stares at the 'all an" cannot seem to fin" the 'or"s to respon" to *#estions. Physical e.amination is normal. /e is place" on cloDapine an" sche"#le" for a )isit to the clinic in 1 'eek. >ne 'eek later% he arri)es at the clinic for his appointment. =hich of the follo'ing is the most appropriate test at this timea6 2omplete $loo" co#nt 32B26 $6Electroencephalogram 3EEC6 c6Electrocar"iogram 3E2C6 "6Prolactin le)els e6 Thyroi"!stim#lating hormone le)els 3TS/6 :ormal La$s 11.A &4!year!ol" man is $ro#ght to the emergency "epartment $y 2 policemen. The policemen say that they finally apprehen"e" the s#spect after many 'omen complaine" that he 'as 0r#$$ing #p against0 them on the s#$'ay. They $ro#ght him to the emergency "epartment $eca#se he is 0o$)io#sly sick0 to $e "oing something 0so craDy0. /e has a long history of ri"ing the s#$'ay all "ay an" r#$$ing his erect penis against non!consenting 'omen. /e is #na$le to keep a 9o$ $eca#se of his 0constant nee" to to#ch #nkno'n 'omen0. =hich of the follo'ing is the most likely "iagnosisa6E.hi$itionism $65rotte#rism c6Pe"ophilia "6Se.#al masochism e6 ?oye#rism :ormal La$s

18.A &4!year!ol" 'oman presents to her primary care physician 'ith m#ltiple symptoms of "epression. She complains of a$o#t 2 months of feeling sa" an" an.io#s. She "escri$es "iffic#lty falling asleep at night% "ecrease" appetite 'ith a 1 l$ 'eight loss% "iminishe" interest in her ho$$ies% an" poor concentration. The patient "enies any other pro$lems. Physical e.amination is 'ithin normal limits. The patient an" her physician "isc#ss initiating treatment 'ith a selecti)e serotonin re#ptake inhi$itor 3SSA+6. =hich of the follo'ing 'o#l" $e the most important a""itional information to gather $efore starting the SSA+a65amily history of "epressi)e episo"es $65amily history of alcoholism or "r#g "epen"ence c6Past history of hypomanic or manic episo"es "6Past history of ki"ney "iseases e6Past history of o$sessions or comp#lsions :ormal La$s 24.A 44!year!ol" man is $ro#ght in for e)al#ation $y the 2oast C#ar" after the small plane he 'as piloting crashe" into the ocean. The man,s 'ife an" t'o frien"s 'ere also on the plane. The man has s#r)i)e" the crash 'ith c#ts an" a $roken arm% $#t he claims he has no memory of the crash or ho' he escape" the plane. /e is also #na$le to e.plain ho' he got his life 9acket on. /is physical e.amination no' is significant only for minor lacerations an" a fract#re" right h#mer#s% an" he has no alteration in conscio#sness. A 2T scan is normal. /e is )ery #pset that the fate of his 'ife is #nkno'n% an" he has nightmares for the ne.t se)eral nights 'hile trying to sleep. =hich of the follo'ing is the most likely "iagnosisa6 @issociati)e amnesia $6@issociati)e f#g#e c6@issociati)e i"entity "isor"er "65actitio#s "isor"er e6 Transient glo$al hypo.ia :ormal La$s 21. A 21!year!ol" man presents to the emergency "epartment complaining of a &!month history of fatig#e% "epression% insomnia% an" 'eight loss. /e states he also has ha" "iffic#lty 'ith feelings of g#ilt o)er his occasional +? heroin #se. >n a mini!mental stat#s e.amination%

he has some mil" cogniti)e "efects% scoring 24 of &4. =hich of the follo'ing is the most appropriate ne.t step in management of this patienta6>r"er fl#o.etine $6>r"er imipramine c6>r"er an /+? test "6>r"er a #rine "r#g screen e6Aefer the patient for o#tpatient "r#g reha$ilitation :ormal La$s 22. A 24!year!ol" man is $ro#ght to the clinic $y his parents% 'ho are concerne" that he is an alcoholic. They 'ant him to check himself in for treatment. +n a "isc#ssion a$o#t his "rinking ha$its% the yo#ng man $rags that he can no' "rink more $efore he gets "r#nk. =hich of the follo'ing terms $est characteriDes this phenomenona6 Blacko#t $62onfa$#lation c6@ereism "6Aesistance e6Tolerance :ormal La$s 2&.A 2<!year!ol" man presents to his primary care physician complaining that he has $een more an" more afrai" to lea)e his ho#se $eca#se he has $een ha)ing fre*#ent an.iety attacks that occ#r 'itho#t 'arning. /e is 'orrie" that he 'ill ha)e an attack at some time 'hile he is in p#$lic. =hich of the follo'ing is the most appropriate initial me"ication for this patient in an ac#te sit#ationa6B#spirone $6 2hlorpromaDine c62lonaDepam "6MirtaDapine e6Trifl#operaDine :ormal La$s 24.After a minor $#t "istressing a#tomo$ile acci"ent% a patient is #na$le to mo)e one leg. 2aref#l physical e.amination "emonstrates no o$)io#s in9#ry that might ha)e ca#se" the paralysis. /is refle.es are intact. A 2T of the spine "emonstrates no $ack in9#ry. The patient is reass#re"%

an" the paralysis resol)es o)er a 2!'eek perio". =hich of the follo'ing is the most likely "iagnosisa6Bo"y "ysmorphic "isor"er $62on)ersion "isor"er c6M#ncha#sen syn"rome "6Pain "isor"er e6SomatiDation "isor"er :ormal La$s 2 . A 2 !year!ol" man presents to a psychiatrist after $eing referre" $y his primary care physician for e)al#ation of possi$le "epression. @#ring the inter)ie'% the patient states that he li)es alone an" 'orks as the film pro9ectionist at a local mo)ie theater. /e has no o#tsi"e interests% $#t claims that he is comforta$le li)ing alone an" has not ha" any changes in appetite or sleep. /e "escri$es ho' he ignores social in)itations from some of his co!'orkers% an" spen"s most of his leis#re time 'atching tele)ision an" rea"ing comic $ooks. =hich of the follo'ing is the most likely "iagnosisa6Antisocial personality "isor"er $6Bor"erline personality "isor"er c6@epen"ent personality "isor"er "6SchiDoi" personality "isor"er e6SchiDophrenia :ormal La$s 2<.A 24!year!ol" man al'ays 'aits for r#sh ho#r to go home after 'ork% e)en tho#gh he occasionally finishes his 9o$ m#ch earlier. /e #s#ally s*#eeDes onto a cro'"e" train an" presses his $o"y against a 'oman. At these times% he #s#ally has erections an" sometimes e9ac#lates. This $eha)ior is most characteristic of 'hich of the follo'ing "isor"ersa6@yspare#nia $6E.hi$itionism c65etishism "65rotte#rism e6Se.#al masochism :ormal La$s

2(.A 22!year!ol" African American male is a"mitte" to a psychiatric #nit for sta$iliDation of an ac#te episo"e of psychosis. /e has e.perience" se)eral 'eeks of "epressi)e an" psychotic symptoms% an" has ha" perio"s of an.iety an" aggression associate" 'ith psychotic symptoms an" anger a$o#t his in)ol#ntary ci)il commitment. /e has $een starte" on an anti"epressant% an antipsychotic% an" has re*#ire" a""itional me"ications on se)eral occasions for agitation. >n the fifth "ay of his hospitaliDation% he "e)elops a s#staine" m#sc#lar contraction in)ol)ing his left neck m#scles% 'ith associate" pain an" "istress. =hich of the follo'ing me"ications is most likely responsi$le for this ac#te reactiona65l#o.etine $6/aloperi"ol c6LoraDepam "6>lanDapine e6Sertraline :ormal La$s 21.A &4!year!ol" 'oman has ha" a history% since a"olescence% of m#ltiple )ag#e physical complaints% incl#"ing hea"aches% na#sea% $loating% a$"ominal pain% "ysmenorrhea% fatig#e% fainting% an" "ys#ria. She is no' $eing seen in a physician,s office% 'here she "escri$es her c#rrent a$"ominal symptoms as 0the 'orst imagina$le. 0 >n physical e.amination% no a$"ominal stiffness or masses are note". Ten"erness is elicite"% $#t the site )aries an" is not repro"#ci$le% e)en a min#te or t'o later. >n talking 'ith the patient% she "oes not seem concerne" a$o#t any specific life e)ents. She lea)es the physician 'ith the impression that she may ha)e an #n"erlying personality "isor"er% possi$ly $or"erline or antisocial. =hich of the follo'ing is the most likely "iagnosisa62on)ersion "isor"er $6/ypochon"riasis c6Malingering "6M#ncha#sen syn"rome e6SomatiDation "isor"er :ormal La$s 28.A 21!year!ol" man is $ro#ght to the emergency "epartment $y his family $eca#se of a t'o!"ay history of conf#sion. >)er the past eight ho#rs% he has $een sitting in a chair 'ith a stiff post#re% not speaking

'hen *#estione". /e 'as release" from the hospital t'o 'eeks prior after a thir" a"mission for schiDophrenia% catatonic typeG an" he has $een taking haloperi"ol 14 mg N/S. >n e)al#ation% the patient seems conf#se" an" sleepy. /is temperat#re is &8.< 2 314&.2 56% $loo" press#re is 14 7144 mm /g% p#lse is 148% an" respirations are 227min. :e#rological e.amination re)eals rigi"ity in his e.tremities. La$oratory analysis re)eals creatine phosphokinase to $e fi)e times the #pper limit of normal. =hich of the follo'ing con"itions most likely acco#nts for this patient,s presenting con"ition an" clinical fin"ingsa6Lethal catatonia $6:e#roleptic!in"#ce" ac#te "ystonia c6:e#roleptic!in"#ce" parkinsonism "6:e#roleptic malignant syn"rome e6 SchiDophrenia% catatonic type I :ormal La$s &4.A yo#ng 'oman comes to a physician for help. @#ring the inter)ie'% she re)eals that she 'as rape" 'hen she 'as a teenager. She is c#rrently "ating a man 'ith 'hom she 'o#l" like to ha)e se.#al interco#rse. =hen they trie"% ho'e)er% she felt 0#ptight0% an" they co#l" not ha)e interco#rse. =hich of the follo'ing is the most likely "iagnosisa6 @yspare#nia $65etishism c65rotte#rism "6Pse#"ocyesis e6 ?aginism#s :ormal La$s &1.A &8!year!ol" $#sinessman 'ith no prior me"ical pro$lems is r#she" to the emergency "epartment follo'ing the s#""en onset of "iDDiness% shortness of $reath% an" palpitations. /is $loo" press#re on a"mission is 1847114 mm /g% his p#lse is 1247min% an" he is "iaphoretic. /is 'ife says that his $eha)ior has change" o)er the past co#ple month since he $ecame 2E> of his company. /e has $ecome moo"y. At times% he seems energetic% e#phoric% or irrita$leG then he seems 0to $e "o'n0 for no reason. /e 9#st ret#rne" from one of many $#siness meetings an" again spent more money than e)er $efore. The patient is smiling

inappropriately an" "enies any alcohol or "r#g a$#se. =hich of the follo'ing 'ill most likely $e fo#n" on a #rine "r#g screena62ocaine $6/eroin c6:icotine "6>rganic inhalants e6Phencycli"ine :ormal La$s &2.A 44!year!ol" 'oman is $ro#ght to the hospital after o)er"osing on alcohol an" pills. +n talking to a psychiatrist% she "enies any prior psychiatric pro$lems $#t says that a$o#t a 'eek ago her apartment $#rne" "o'n. She 'as trying to get a 9o$ $efore that happene"% $#t 'hen she realiDe" that she ha" lost e)erything she ha"% an" that mo)ing in 'ith her family 'o#l" pro$a$ly not $e possi$le% she "eci"e" to take her o'n life. =hich of the follo'ing is the most likely "iagnosisa6Ac#te stress "isor"er $6A"9#stment "isor"er c6Antisocial personality "isor"er "6Brief psychotic "isor"er e6Ma9or "epressi)e "isor"er :ormal La$s &&.A 2!year!ol" man is rec#perating in a hospital after ha)ing s#staine" a recent cere$ro)asc#lar acci"ent that "amage" part of his right temporal lo$e. >nce the patient has rec#perate" from the imme"iate effects of his stroke% to 'hich of the follo'ing psychiatric "isor"ers 'ill he $e most pre"ispose"a62on)ersion "isor"er $6Mania c6Ma9or "epressi)e "isor"er "6SchiDophrenia e6S#$stance a$#se :ormal La$s &4.A 1(!year!ol" a"olescent is $ro#ght to the hospital $y police after $eing stoppe" for "ri)ing too slo'ly. /e "enies any #se of alcohol% $#t seems slo'. /e is la#ghing inappropriately% complaining of $eing

h#ngry% an" has a "ry mo#th. /is gait is some'hat slo' an" his coor"ination is impaire". There is significant con9#ncti)al in9ection% 'hich he tries to e.plain as $eing "#e to pollen allergy. =hich of the follo'ing "r#gs 'ill most likely appear on a #rine "r#g screena6Amphetamines $6BenDo"iaDepines c62ocaine "6>piates e6Tetrahy"rocanna$inol :ormal La$s & . A 4!year!ol" 'oman 'ith a past history of rec#rrent ma9or "epressi)e "isor"er is c#rrently on a maintenance "ose of an anti"epressant. She has $een 'ell for the past 2 years. She is complaining to her psychiatrist a$o#t her "ecrease" a$ility to reach orgasm. =hich of the follo'ing me"ications has most likely ca#se" her anorgasmiaa6Amitriptyline $6B#propion c6MirtaDapine "6:efaDo"one e6 Paro.etine :ormal La$s &<.A 2 !year!ol" 'oman 'ith a se)eral!year history of $inging an" p#rging presents to a psychiatrist complaining of a lack of energy% poor sleep% an" "ecrease" a$ility to concentrate. She is )ery concerne" a$o#t 'eight gain. =hich of the follo'ing me"ications 'o#l" $e the most appropriate to initiatea6 B#propion $65l#o.etine c6/aloperi"ol "6Lithi#m car$onate e6?alproic aci" :ormal La$s

&(. A &2!year!ol" 'oman 'ith a history of panic "isor"er is $ro#ght to the emergency clinic $y her h#s$an" after he fo#n" her hea)ily se"ate" an" minimally responsi)e #pon his arri)al home from 'ork. The h#s$an" informs the physician that this 'ife ha" $ecome increasingly "epresse" o)er the past fe' 'eeks% since the "eath of her mother. /e states that her psychiatrist ha" recently increase" her "oses of alpraDolam an" sertraline "#e to increasing panic attacks along 'ith 'orsening "epression. /er temperat#re is &( 2 381.< 56% $loo" press#re is 14 7< mm /g% p#lse is < % an" respiratory rate is 127min. She is hea)ily se"ate" an" only a$le to #tter a fe' phrases% 'ith sl#rre" speech. She "i" state that she took a "iaDepam an" se)eral e.tra alpraDolam "#e to panic attacks. @#ring the physical e.amination% her respiratory rate "rops to 17min. =hich of the follo'ing me"ications 'o#l" $e the most appropriate to a"minister at this timea6BenDtropine $65l#maDenil c6/aloperi"ol "6LoraDepam e6 :alo.one :ormal La$s &1.A 4 !year!ol" 'hite 'oman presents to her primary care physician for her ro#tine yearly e.amination. /er physician notices that she has lost appro.imately 2 l$ o)er the past year. =hen aske" a$o#t her 'eight loss% she states that she has not ha" m#ch of an appetite since her "a#ghter left to go to college a$o#t 1 months ago. +n "escri$ing this% she is some'hat tearf#l an" complains of loneliness% poor memory% "ecrease" concentration% an" feeling tire". Altho#gh she appears )ery sa"% she "enies feeling "epresse"% an" states that she simply nee"s to $ecome more socially acti)e to "istract herself from her 'orries. /er physical e.amination is normal e.cept for the noticea$le 'eight loss. She 'eighs 14 l$. =hich of the follo'ing is the most likely "iagnosisa6A"9#stment "isor"er 'ith an.iety $6Berea)ement c6@epen"ent personality "isor"er "6@ysthymic "isor"er e6 Ma9or "epressi)e "isor"er :ormal La$s

&8.A 2<!year!ol" man 'ith schiDophrenia comes to the emergency "epartment 'ith a 2!ho#r history of in)ol#ntary contractions of the m#scles in his neck. /e states that he 'as 'atching tele)ision an" 0all of a s#""en + t#rne" my hea" an" my neck locke"0. /e $egan taking a high!potency antipsychotic agent & "ays earlier. E.amination sho's no a$normalities e.cept torticollis. =hich of the follo'ing is the most appropriate pharmacotherapya6Amanta"ine $6BenDtropine c6Bromocriptine "62loni"ine e6Propranolol :ormal La$s 44.A 42!year!ol" 'oman is a"mitte" for o#tpatient electi)e cosmetic s#rgery. After the s#rgery% she "e)elops ac#te shortness of $reath% an" p#lmonary em$ol#s is "iagnose". T'o "ays later% an emergency psychiatric cons#ltation is calle"% as the patient has "e)elope" a shaking tremor% is p#lling o#t her +? lines% an" appears to $e 'atching snakes cra'l aro#n" the floor of her room an" ants cra'ling on her skin. =hich of the follo'ing aspects of the patient,s history 'o#l" $e most likely to point to the "iagnosisa6Alcohol #se $6@epression c6Past LS@ #se "6SchiDophrenia e6Tra#matic chil"hoo" :ormal La$s 41. A &4!year!ol" man presents to his psychiatrist after missing his last appointment t'o months ago. The patient has a history of $ipolar + "isor"er% for 'hich he takes lithi#m car$onate. /e has not taken his me"ications for the past t'o months. Upon presentation% he is )ery lo#" an" talkati)e% 'ith press#re" speech an" racing tho#ghts. /e states that he has not slept for the past fo#r "ays $eca#se he has $een 'orking to esta$lish a catering ser)ice% for 'hich he has $een $#ying many kitchen

#tensils% $aking s#pplies% an" recipe $ooks. /e has also $een 0sampling 'ines from aro#n" the 'orl"0 so that he can $ecome a 'ine e.pert for his ne' $#siness. +n assessing the patient,s affect an" tho#ghts% 'hich of the follo'ing mental stat#s e.amination fin"ings is most likely presenta6Bl#nte" affect $6+nflate" self!esteem an" gran"iosity c6+nappropriate an" e.cessi)e g#ilt "6:ihilistic "el#sions e6 Aepetiti)e han"!'ashing an" co#nting :ormal La$s 42. A 1!year ol" man is a"mitte" to the tra#ma ser)ice after a motor )ehicle acci"ent that ca#se" a fract#re of his pel)is an" right fem#r. /is family reports that he recently lost his 9o$ "#e to poor 'ork performance relate" to a 'orsening pro$lem 'ith alcohol. >n the thir" "ay of his hospitaliDation 3thir" post!operati)e "ay after internal fi.ation of the femoral fract#re6% the patient is note" to $e "isoriente". /e tells the n#rsing staff a$o#t feeling an" seeing snakes cra'ling in his $e". =hat is the most appropriate initial step in the management of this patient,s altere" mental stat#sa62loni"ine $6/aloperi"ol c6LoraDepam "6:alo.one e6 >lanDapine :ormal La$s 4&. A &4!year!ol" 'oman calls her ne' primary care physician to re*#est a refill of her alpraDolam% 'hich ha" $een prescri$e" $y her former physician for se)ere an.iety. /er ne' physician has only seen her once% a$o#t a month ago% after the patient ma"e se)eral s#perficial lacerations on her 'rists "#ring a fight 'ith her $oyfrien". At that time% she ha" $een referre" for emergency psychiatric e)al#ation. >n the telephone% she "escri$es intense an.iety relate" to an #nsta$le relationship 'ith a ne' $oyfrien"% $inges on alcohol o)er the last fe' "ays% alternating irrita$ility% anger% an" "epression% an" rec#rrent )ag#e s#ici"al tho#ghts% 'itho#t a plan or intent to harm herself. 2ontact 'ith her former physician re)eals that these feelings an"

$eha)iors 'ere #nchange" o)er the fi)e years that he ha" seen her for ro#tine health maintenance e.aminations an" minor illnesses. =hich of the follo'ing is the most likely "iagnosisa6 Bor"erline personality "isor"er $6@epen"ent personality "isor"er c6/istrionic personality "isor"er "6:arcissistic personality "isor"er e6SchiDotypal personality "isor"er :ormal La$s 44. A 22!year!ol" 'hite 'oman comes to a psychiatrist,s office after ha)ing $een referre" $y her family physician. She reports no mental pro$lems $#t tells the psychiatrist that she is concerne" a$o#t hair loss. /er mother says that the patient has $een grooming herself fre*#ently an" checking her image in the mirror. /er family physician has e.amine" her se)eral timesG there is no e)i"ence of hair loss. /e has r#n m#ltiple tests% $#t there is no e)i"ence of any a$normality. The 'oman states that she is em$arrasse" an" tries to a)oi" going o#t $eca#se e)eryone 0'ill notice0 an" ask *#estions. She "enies p#lling her hair. @espite this pro$lem% she is still "oing 'ell on her 9o$. =hich of the follo'ing is the most likely "iagnosisa6Alopecia areata $6Bo"y "ysmorphic "isor"er c6/ypochon"riasis "6SchiDophrenia e6Trichotillomania :ormal La$s 4 . A 4 !year!ol" 'oman presents to her physician $eca#se her h#s$an" complains that she snores )ery lo#"ly 'hile sleeping. She "escri$es ho' she often feels tire" thro#gho#t the co#rse of the "ay% an" ne)er feels )ery reste". /er physical e.amination appears normal e.cept for o$esity. She is 1< cm 3 ft% in6 tall% an" 'eighs appro.imately (( kg 31(46 po#n"s. Accor"ing to her h#s$an"% she seems to stop $reathing at times "#ring the night. =hich of the follo'ing is the most likely "iagnosisa62atalepsy $6Mleine!Le)in syn"rome

c6:arcolepsy "6Primary hypersomnia e6Sleep apnea :ormal La$s 4<. A <8!year!ol" patient is a"mitte" to the ne#rology ser)ice follo'ing a stroke. @#ring the ne.t fe' "ays% the staff o$ser)es that the patient has "e)elope" the clinical pict#re of mania. =hich area of the $rain has most likely $een affecte" $y the strokea6 Left hemispheric lesions incl#"ing Broca,s area $6Left prefrontal corte. c6Mi"$rain lesion "6Aight frontal lo$e e6 Thalam#s :ormal La$s 4(.A (2!year!ol" African American 'oman is $eing rea"ie" for "ischarge from the hospital 2 'eeks after a stroke affecting her right occipital corte.. A psychiatric cons#ltation is calle" to e)al#ate the patient for "epression% as she has ha" "ecrease" appetite% some crying spells% an" insomnia. After "etermining that the patient is not s#ffering from "eliri#m% 'hich of the follo'ing me"ications 'o#l" $e most appropriate to treat her "epressi)e symptomsa6Methylpheni"ate $6:ortriptyline c6PhenelDine "6Sertraline e6 Thiori"aDine :ormal La$s 41.A 14!year!ol" $oy "iagnose" 'ith attention "eficit7hyperacti)ity "isor"er 3A@/@6 comes to a ne' "octor for the first time. /is mother reports that they mo)e" recently to the city% $#t that he 'as starte" on a me"ication for his A@/@ $y his pre)io#s "octor. After learning 'hich me"ication the patient has $een taking% the "octor $ecomes concerne" a$o#t the patient,s li)er f#nction. =hich of the follo'ing me"ications is the patient most likely taking-

a6 2loni"ine $6@esipramine c6@e.troamphetamine "6Methylpheni"ate e6 Pemoline :ormal La$s 48.A 22!year!ol" 'oman 'ith anore.ia ner)osa an" a history of "epression% for 'hich she is c#rrently taking $#propion% mo)es to a ne' city an" presents to a local psychiatrist to esta$lish care in her ne' location. She tells the "octor that she has $een on the same "ose of $#propion since she 'as 11 years ol"% an" that her "epression has $een sta$le% $#t that she has lost 14 po#n"s o)er the last month an" 'eighs only 84 po#n"s 341 kg6 no' 'ith a height of feet inches 31< cm6. A$o#t 'hich of the follo'ing complications sho#l" the physician $e most concerne"a62ar"iac arrhythmia $6@ecrease" renal f#nction c6/ypochon"riasis "6>steoporosis e6SeiD#res :ormal La$s 4. A mother $rings in her 8!year!ol" son 'ho has $een recently "iagnose" 'ith attention "eficit hyperacti)ity "isor"er 3A@/@6. She is concerne" that her 4 year ol" son might also ha)e the "isor"er% as he is ha)ing "iffic#lty interacting 'ith other chil"ren at the local "ay care% is increasingly irrita$le% an" 'as late in toilet training. She an" her h#s$an" are also consi"ering ha)ing another chil"% an" they are 'orrie" a$o#t the possi$ility of ha)ing another chil" 'ith A@/@. =hich of the follo'ing sho#l" the physician a")ise this mother concerning her chil",s A@/@a6A@/@ is a $enign "isor"er an" is self!limite" to chil"hoo" $6/er chil" 'ith A@/@ has a 4B risk of "e)eloping antisocial personality "isor"er c6/er chil" 'ith A@/@ has an increase" risk of "e)eloping schiDophrenia% "isorganiDe" type

"6The le)el of her chil",s inattention 'ill lessen "#ring chil"hoo" an" a"olescence e6The ol"er her chil"ren get% the less likely that they 'ill $e to "e)elop A@/@% as the pre)alence of A@/@ is estimate" to "ecline as people age :ormal La$s

E.planations Block 1 E.planations

:) 7he correct answer is (. +a<or "e!ressive "isor"er is a syn"rome that requires a "e!resse" moo" or loss of interest or !leasure an" causes clinically si$nificant "istress or im!airment in occu!ational, social, or other im!ortant areas of functionin$. 7he "ia$nosis of bi!olar "e!ression requires that the !atient have also ha" a manic e!iso"e of bi!olar "isor"er. i!olar "e!ression is associate" with less !sychomotor a$itation com!are" to uni!olar "e!ression 'ma<or "e!ressive "isor"er), an" thus the !atientAs a$itation is more consistent with a ma<or "e!ressive "isor"er "ia$nosis than with the "e!ression of bi!olar "isor"er. (arly onset of bi!olar "isor"er 'choice A) is a risk factor for the "evelo!ment of bi!olar, rather than uni!olar "e!ression in later life. Increase" a!!etite 'choice ) is more a sym!tom of the "e!ression of bi!olar "isor"er than ma<or "e!ressive "isor"er, as one of the criteria for ma<or "e!ressive "isor"er is loss of a!!etite or anore*ia. Insomnia 'choice C) is foun" more often in bi!olar "e!ression than ma<or "e!ressive "isor"er. Increase" thou$hts of suici"e 'choice %) is incorrect because bi!olar "e!ression carries a $reater risk of suici"e than ma<or "e!ressive "isor"er. ;) 7he correct answer is (. 7he chil" "escribe" in this case "escri!tion has the clinical features characteristic of 6ett syn"rome. %urin$ the first five months after birth, the infant has a$e4a!!ro!riate motor skills, hea" circumference, $rowth, an" social interactions. At si* to 0> months, the chil" has !ro$ressive ence!halo!athy with "ecline in !reviously "evelo!e" motor an" social skills. Associate" features inclu"e sei#ures in u! to D/= of affecte" chil"ren an" irre$ular res!iratory !atterns. ,on$4term rece!tive an" e*!ressive communication an" sociali#ation abilities remain at a "evelo!mental level of less than one year. 6ett syn"rome occurs almost e*clusively in females. As!er$er "isor"er 'choice A) is characteri#e" by at least two of the followin$ in"ications of social im!airment: marke"ly abnormal nonverbal communicative $estures, failure to "evelo! !eer relationshi!s, lack of social or emotional reci!rocity, an" an inability to e*!ress !leasure in other !eo!leAs ha!!iness. 6estricte" interests an" !atterns of behavior are also !resent. Attention "eficitBhy!eractivity "isor"er 'choice ) is characteri#e" "evelo!mentally by an a$e4 ina!!ro!riate !oor attention s!an, a$e4ina!!ro!riate features of hy!eractivity an" im!ulsivity, or both. Autistic "isor"er 'choice C) is characteri#e" by im!airments in social interactions an" communication, an" restricte" re!etitive an" stereoty!e" !atterns of behavior, interests, an" activities. Onset is !rior to the a$e of three years. &ervasive "evelo!mental "isor"er, not otherwise s!ecifie" 'choice %) is a "ia$nostic cate$ory that

woul" be use" when a chil" manifests a qualitative im!airment in the "evelo!ment of reci!rocal social interaction an" communication but "oes not meet the criteria for other !ervasive "evelo!mental "isor"ers. 0) 7he correct answer is . 6eaction formation, often seen in obsessional characters, is the term for the "efense mechanism in which an unacce!table im!ulse is transforme" into its o!!osite. In this case, "urin$ insi$ht4oriente" !sychothera!y, the !atient reali#es his wish to be freer in his se*ual relationshi! with his wife 'an im!ulse which he fin"s unacce!table on some level) but fin"s himself res!on"in$ in the o!!osite way 'maintainin$ a restricte" stance). Inhibition may also !artly account for this manAs "ifficulty, in that a renunciation is use" to eva"e an*iety arisin$ out of im!ulses. &ro<ection 'choice A) occurs when an unacce!table inner im!ulse is !erceive" an" reacte" to as thou$h it was outsi"e oneself. On the !sychotic level, this takes the form of "elusions an" hallucinations. )e*uali#ation 'choice C) occurs when an ob<ect or function is en"owe" with se*ual si$nificance that it "i" not !reviously have in or"er to war" off an*ieties associate" with !rohibite" im!ulses. )omati#ation 'choice %) "escribes the "efense mechanism that occurs when emotional concerns are converte" into bo"ily sym!toms, an" the !erson ten"s to react with somatic manifestations. If the !atient in this case ha" a ten"ency to use somati#ation, he mi$ht unconsciously use !hysical sym!toms to $et ri" of the an*iety aroun" his conflicte" se*ual thou$hts. )ublimation 'choice () is a mature "efense mechanism that occurs when a socially acce!table means of e*!ressin$ an im!ulse re!laces one that woul" be socially unacce!table. )ublimation allows instincts to be channele", rather than blocke" or "iverte". -eelin$s are acknowle"$e", mo"ifie", an" "irecte" towar" a si$nificant ob<ect or $oal, an" mo"est instinctual satisfaction occurs. ?) 7he correct answer is A. Antisocial !ersonality "isor"er is "efine" by a !ervasive !attern of "isre$ar" for an" violation of the ri$hts of others. It occurs after a$e :/ an" is accom!anie" by at least three of the followin$: failure to conform to social norms or laws as in"icate" by re!eate" arrests, "eceitfulness, irritability an" a$$ressiveness, "isre$ar" for safety of others, irres!onsibility, an" a lack of remorse after mistreatin$ others. 7he "isor"er is not "ue to schi#o!hrenia or manic e!iso"e. or"erline !ersonality "isor"er 'choice ) is characteri#e" by a !ervasive !attern of instability in inter!ersonal relationshi!s, self4 ima$e, an" affect, as in"icate" by five or more of the followin$: im!ulsivity, efforts to avoi" real or ima$ine" aban"onment, unstable relationshi!s, affective instability, feelin$s of em!tiness, "ifficulty controllin$ an$er, an" i"entity "isturbance. Con"uct "isor"er 'choice C) involves a re!etitive !attern of behavior in which the ri$hts of others or social norms are violate". It must be$in in chil"hoo", an" at least one criterion has to be !resent for C months. 7hree or more of the followin$ criteria are nee"e" to establish the "ia$nosis: cruelty to animals or !eo!le, "estruction of !ro!erty, "eceitfulness or theft, an" serious violations of rules. +a<or "e!ressive "isor"er 'choice %) requires a ;4week !erio"

of "e!resse" moo" or anhe"onia, alon$ with associate" sym!toms of "ecrease" ener$y, chan$es in slee!, chan$es in a!!etite an" wei$ht, chan$es in !sychomotor activity, !resence of $uilt, or suici"al i"eation. 7hese sym!toms are not secon"ary to another mental "isor"er or $eneral me"ical con"ition. &aranoi" !ersonality "isor"er 'choice () is "ia$nose" when there is a !ervasive !attern of "istrust of others be$innin$ in a"ulthoo" an" in"icate" at least by four of the followin$: rea"s threatenin$ meanin$ into beni$n events, recurrently sus!icious without <ustification, reluctant to confi"e in others, !ersistently bears $ru"$es, is !reoccu!ie" with un<ustifie" "oubts, an" sus!ects others are e*!loitin$ harmin$ or "eceivin$ him. 7he "isor"er is not "ue to schi#o!hrenia, moo" "isor"er, or a $eneral me"ical con"ition. /) 0;4year4ol" man !resents to his !rimary care !hysician statin$ that he feels very lonely an" woul" very much like to have a close circle of frien"s. 2owever, he feels very an*ious in social situations an" avoi"s social invitations from acquaintances at work. 2e states he woul" like to be more interactive in social events, but that he worries that other !eo!le will notice his ina"equacies an" talk about his !roblems. Which of the followin$ is the most likely "ia$nosisJ A. Antisocial !ersonality "isor"er . %e!en"ent !ersonality "isor"er C. .enerali#e" an*iety "isor"er %. )chi#oi" !ersonality "isor"er (. )ocial !hobia (*!lanation: 7he correct answer is (. )ocial !hobia is characteri#e" by fear an" avoi"ance of social situations, even thou$h it is often accom!anie" by a "esire for social contact an" interaction with others. It is often accom!anie" by low self4esteem. Antisocial !ersonality "isor"er 'choice A) is characteri#e" by blatant "isre$ar" for the ri$hts of others an" society an" is often accom!anie" by le$al troubles. Its "ia$nosis in chil"ren is referre" to as con"uct "isor"er. %e!en"ent !ersonality "isor"er 'choice ) is characteri#e" by !oorly controlle" an*iety about many "ifferent !roblems, inclu"in$ social an*iety, an" a !ervasive an" e*cessive nee" to be taken care of, with clin$in$ behavior an" fear of se!aration. .enerali#e" an*iety "isor"er 'choice C) is characteri#e" by e*cessive an*iety an" worry occurrin$ more "ays than not for at least C months, an" concernin$ activities such as work or school !erformance, with accom!anyin$ muscles tension, irritability, an" slee! "isturbance. )chi#oi" !ersonality "isor"er 'choice %) is a "isor"er characteri#e" by social with"rawal, but loneliness is not one of the associate" sym!toms. C) 7he correct answer is C. 7his !atient has con"uct "isor"er. 7he "ia$nosis requires a !attern of behavior that violates societal rules an" the basic ri$hts of others. Common features inclu"e lyin$, stealin$, runnin$ away, stayin$ out without !ermission, settin$ fires, truancy, van"alism, cruelty to animals, bullyin$, !hysical a$$ression, an" se*ual a$$ression. At least three of

these features must be !resent to make the "ia$nosis. 7wenty4five to fifty !ercent of these !atients $o on to have antisocial !ersonality "isor"er as an a"ult. Antisocial !ersonality "isor"er 'choice A) is characteri#e" by a re!etitive "isre$ar" for the rules an" laws of society. 7hese !atients rarely e*!erience remorse. It is more common in men. +ore than half of these men have been arreste". It is the a"ult equivalent of con"uct "isor"er. A%2% 'choice ) is a !attern of hy!eractivity, im!ulsiveness, inattention, an" "istractibility before a$e D. +ethyl!heni"ate is the first4line !harmacolo$ic treatment. )i"e effects inclu"e wei$ht loss, tachycar"ia, insomnia, hy!ertension, an" "ia!horesis. O%% 'choice %) is use" to "escribe chil"ren with behavioral !roblems that "o not meet the criteria for con"uct "isor"er. It is characteri#e" by annoyin$, "ifficult, "isru!tive behavior that is a less severe form of con"uct "isor"er. 7ourette syn"rome 'choice () involves multi!le involuntary vocal an" motor tics. It is associate" with obsessive com!ulsive "isor"er. It is more common in males. D) (*!lanation: 7he correct answer is %. -actitious "isor"er !resents with !hysical sym!toms that are consistent with true illness but are un"er the voluntary control of a !atient. It is often "e!en"ent on a !atientAs nee" to fulfill the Hsick roleH an" be un"er the care of a !hysician. Often, multi!le invasive !roce"ures or e*aminations are "one at the request of the !atient before a "ia$nosis is reache". Antisocial !ersonality "isor"er 'choice A) requires a !ervasive !attern of violatin$ the ri$hts of others, as evi"ence" by failure to a"here to social norms an" lawful behavior. Conversion "isor"er 'choice ) is associate" with !hysical sym!toms that are outsi"e of the voluntary control of the !atient an" are relate" to a si$nificant social event in the life of the !atient. 7here is no evi"ence of "ru$ "e!en"ence 'choice C) in the history of this !atient, an" "e!en"ence woul" not lea" to the !resentation $iven. )chi#o!hrenia 'choice () is a "isor"er of thou$ht that requires !ersistent au"itory hallucinations over the course of at least C months. @) 7he correct answer is (. Nu"$ment is the ability of a !atient to evaluate a !articular course of action in or"er to "etermine if it is the most a!!ro!riate one within the !atientAs !articular value system. If the !atient were to say he woul" !ut the letter back in the mail or $ive it back to the !ostman, this woul" in"icate a!!ro!riate <u"$ment. Abstract thinkin$ 'choice A) is the ability to think or !erform symbolically, or to evaluate various as!ects of a situation an" shift between alternatives. Co$nition 'choice ) is teste" by evaluatin$ the intellectual an" !erce!tual levels of mental functionin$. Insi$ht 'choice C) is the ability to reco$ni#e the ob<ective reality of a situation. Intelli$ence 'choice %) is the ability to learn an" a!!ly learne" information to a $iven situation. I)

(*!lanation: 7he correct answer is %. 7his !atient is e*hibitin$ the catatonic sym!tom known as ne$ativism, which is a motiveless resistance to all attem!ts to be move" or to all instructions. )i$ns of catatonia inclu"e stu!or, ne$ativism, ri$i"ity, !osturin$, mutism, stereoty!ies, mannerisms, wa*y fle*ibility, an" catatonic e*citement. Catatonia may be associate" with schi#o!hrenia 'catatonic ty!e), moo" "isor"ers 'with catatonic features), or $eneral me"ical con"itions. Akathisia 'choice A) is usually classifie" as an e*tra!yrami"al si"e effect of anti!sychotic me"ications. It is characteri#e" by a sub<ective feelin$ of muscular tension 'an inner sense of restlessness) that can cause "istressin$ restlessness, !acin$, or re!eate" movements. It may be mistaken for !sychotic a$itation, an" thus ina!!ro!riately treate". Cata!le*y 'choice ) is a tem!orary loss of muscle tone an" weakness !reci!itate" by a variety of emotional states. It is most characteristically associate" with narcole!sy. (cho!ra*ia 'choice C) is a !atholo$ical imitation of the movements of one !erson by another. It can be seen in catatonia, "elirium, "ementia, an" other "isor"ers. )tereoty!y 'choice () is a re!etitive fi*e" !attern of !hysical action, movement, or s!eech. It may be seen in catatonia. 7his !atient "oes not currently e*hibit stereoty!ies. :>) (*!lanation: 7he correct answer is C. Nihilistic "elusions are false feelin$s that the self or others "o not e*ist or are "estroye". It is ty!ical for "e!ression with !sychotic features. At its e*treme, it is calle" Cotar"As syn"rome. 7he !atient com!lains that he or she has lost !ossessions, status, stren$th, heart, bloo", an" intestines, an" that the worl" has been re"uce" to nothin$ness. Cenesthetic hallucination 'choice A) is a false sensation of thin$s occurrin$ in or to the bo"y, most frequently of visceral ori$in. %e!ersonali#ation 'choice ) is a sub<ective sense of bein$ unreal, stran$e, or unfamiliar with oneself. An overvalue" i"ea 'choice %) is an unreasonable, sustaine", false belief maintaine" less firmly than "elusion. 1erbi$eration 'choice () is a "isor"er of thou$ht !rocess in which a !atient meanin$lessly re!eats certain !hrases or wor"s. ::) 7he correct answer is %. Neurole!tic mali$nant syn"rome 'N+)) is a rare com!lication of neurole!tic thera!y that confers hi$h mortality if not reco$ni#e" an" treate" !rom!tly. It is "efine" by the "evelo!ment of severe muscle ri$i"ity an" elevate" tem!erature in association with at least two or more of the followin$: "ys!ha$ia, tremor, "ia!horesis, tachycar"ia, chan$e in level of consciousness, leucocytosis, elevate" or labile bloo" !ressure, an" elevate" creatine !hos!hokinase as an in"icator of muscle in<ury. 7he !re"is!osin$ factors inclu"e hi$h neurole!tic "oses, intramuscular in<ections, an" lithium treatment. Acute "ystonic reaction 'choice A) is one of the e*tra!yrami"al si"e effects e*!erience" by :>= of neurole!tic4treate" !atients within the first hours or "ays of treatment. It ty!ically lasts a cou!le of hours an" res!on"s to anticholiner$ic "ru$s. ,ethal catatonia 'choice ) is a syn"rome associate" with ma<or "e!ressive "isor"er, mania, mi*e" affective state, or schi#o!hrenia. It requires the !resence of at least two of the followin$: motor immobility,

e*treme motor activity, e*treme ne$ativism, !eculiar voluntary movement, echolalia, or echo!ra*ia. Autonomic instability or hy!erthermia can com!licate it. ,ethal catatonia can be a result of a $eneral me"ical con"ition when it "oes not occur "urin$ the course of "elirium or the above4mentione" mental "isor"ers. +ali$nant hy!erthermia 'choice C) has several features in common with N+), such as muscle ri$i"ity, hy!erthermia, an" elevate" creatine !hos!hokinase, as well as a $oo" res!onse to "antrolene. 2owever, mali$nant hy!erthermia is in"uce" by inhalant anesthetics, an" the susce!tibility is inherite". )erotonin syn"rome 'choice () is $enerally the result of an interaction between serotoner$ic a$ents an" monoamine o*i"ase inhibitors. It is characteri#e" by restlessness, myoclonus, chan$es in mental status, "ia!horesis, hy!errefle*ia, tremor, an" shiverin$. :;) 7he correct answer is . &rochlor!era#ine 'Com!a#ine) is frequently use" to treat nausea an" emesis in some !atients. )i"e effects of this me"ication, inclu"in$ e*tra!yrami"al reactions 'e.$., catatonia), are treate" best by anti!arkinsonian me"ications such as ben#tro!ine. Al!ra#olam 'choice A) is a ben#o"ia#e!ine use" to treat an*iety an" with"rawal sym!toms. 2alo!eri"ol 'choice C) is an anti!sychotic me"ication, which, if $iven to this !atient, woul" !robably e*acerbate her current sym!toms. +ethyl!heni"ate 'choice %) is a stimulant use" to treat attention "eficitBhy!eractivity "isor"er 'A%2%). 1al!roic aci" 'choice () is an anticonvulsant use" as a moo" stabili#er in !sychiatry. :0) (*!lanation: 7he correct answer is %. 7his !atient a!!ears to have Wernicke ence!halo!athy, which is "ue to a "eficiency of thiamine that is quite common in chronic alcoholics. )ym!toms of Wernicke ence!halo!athy inclu"e oculomotor "isturbances, cerebellar ata*ia, an" mental confusion. 7reatment consists of $ivin$ thiamine, :>> m$ I1 or I+, alon$ with ma$nesium sulfate $iven before loa"in$ with $lucose. 7reatin$ with $lucose 'choice A) before $ivin$ thiamine has the !otential to e*acerbate Wernicke ence!halo!athy because of the metabolism of $lucose in the brain. 2alo!eri"ol 'choice ) is an anti!sychotic me"ication that is not $enerally in"icate" for use in non!sychotic alcoholics. ,ora#e!am 'choice C) is a ben#o"ia#e!ine that is of use in !reventin$ the with"rawal sym!toms of "elirium tremens, but treatin$ the !atientAs ence!halo!athy is of $reater !riority. 1al!roic aci" 'choice () is an anticonvulsant use" to control sei#ures. It is not in"icate" for use in an alcoholic !atient whose !re$nancy status is unknown an" whose history of sei#ures from "elirium tremens is unknown. :?) 7he correct answer is A. u!ro!ion is the anti"e!ressant of the choices liste" that has the fewest a"verse se*ual si"e effects. )elective serotonin reu!take inhibitors, such as fluo*etine 'choice ), !aro*etine 'choice %), an" sertraline 'choice () are all known to be associate" with erectile an" or$asmic "isturbances, such as "elaye" e<aculation an" anor$asmia. ecause

imi!ramine 'choice C) also has effect on the serotonin levels in the brain, it too has se*ual si"e effects, althou$h to a somewhat lesser "e$ree than the ))6Is. :/) 7he correct answer is C. Concrete thinkin$ is "escribe" as literal thinkin$ that shows a lack of un"erstan"in$ of the nuances of meanin$. 7hese in"ivi"uals lack the ability to use meta!hors. Abstract thinkin$ 'choice A) refers to the ability to a!!reciate nuances of meanin$ an" the ability to use meta!hors an" hy!otheses a!!ro!riately. lockin$ 'choice ) is a "isturbance in thou$ht form, characteri#e" by an abru!t interru!tion in the train of thou$ht before the thou$ht is finishe". After a !ause, the !erson is unable to recall what was bein$ sai". Confabulation 'choice %) is "efine" as the unconscious fillin$ of $a!s in memory by ima$ine" or untrue e*!eriences that the !atient believes are true, even thou$h they are not base" on facts. It is associate" with or$anic !atholo$y. +a$ical thinkin$ 'choice () is a form of thinkin$ in which thou$hts, wor"s, or actions assume !ower, such as causin$ or !reventin$ events. It is ty!ical for the !reo!erational !hase of thinkin$ in chil"ren. :C) 7he correct answer is C. 7he fact that this !atient has a history of a suici"e $esture in the !ast an" e*!resses !resent suici"al i"eation !uts her at risk for harmin$ herself. )he shoul" be ke!t in the hos!ital, at least until a !sychiatric assessment can be ma"e. A"mittin$ the !atient to a me"ical war" 'choice A) will not allow for a"equate monitorin$ of the !atientAs behavior, $iven her suici"al threat. )he nee"s to be a"mitte" to a !sychiatric war". 6eferral to an out!atient clinic 'choice () was not effective before, an" lon$4term !sychothera!y 'choice ) is not in"icate" in the acutely suici"al !atient. 6emovin$ the !atient 'choice %) will not $uarantee that she will not make another suici"e attem!t. :D) (*!lanation: 7he correct answer is A. Clo#a!ine is an aty!ical anti!sychotic that may !ro"uce a$ranulocytosis, requirin$ weekly monitorin$ of the C C. Clo#a!ine blocks both "o!amine an" serotonin rece!tors an" causes only minimal e*tra!yrami"al si"e effects. It is effective in treatin$ the ne$ative sym!toms of schi#o!hrenia an" is often use" in treatment4resistant cases. In a""ition to causin$ a$ranulocytosis ':= inci"ence), clo#a!ine thera!y has a /4:>= inci"ence of sei#ures, but weekly ((.s are not !erforme". It "oes not increase the !rolactin level. An ((. 'choice ) is not necessary. Clo#a!ine "oes "ecrease the sei#ure threshol", however, ((.s are not ty!ically !erforme". An (C. 'choice C) is ina!!ro!riate because clo#a!ine "oes not commonly cause car"iac si"e effects. &rolactin levels 'choice %) are not necessary because clo#a!ine, unlike other anti!sychotics, "oes not lea" to a rise in !rolactin. 7)2

levels 'choice () are affecte" by lithium, not clo#a!ine. ,ithium interferes with the synthesis an" release of thyroi" hormone an" may lea" to hy!othyroi"ism. :@) 7he correct answer is . 7his !atient has a !ara!hilia calle" frotteurism, which is the se*ual arousal that is "erive" from se*ually touchin$ a unsus!ectin$ stran$er. It occurs !rimarily in men an" often be$ins in late a"olescence an" youn$ a"ulthoo". It causes si$nificant im!airment in the in"ivi"ualAs level of functionin$. (*hibitionism 'choice A) is when an in"ivi"ual is se*ually arouse" by e*!osin$ his 'or her) $enitals to stran$ers. 7he in"ivi"ual may masturbate "urin$ the e*!osure. A !e"o!hile 'choice C) is an in"ivi"ual who is se*ually arouse" by !artici!atin$ in, fantasi#in$ about, or viewin$ se*ual acts with a !re!ubescent chil". A se*ual masochist 'choice %) is an in"ivi"ual who is se*ually arouse" by humiliation, !ain, or bon"a$e a$ainst oneself. 1oyeurism 'choice () is when se*ual e*citement is "erive" from watchin$ unknowin$ victims who are un"ressin$ or en$a$in$ in se*ual behavior. :I) 7he correct answer is C. Althou$h all of the choices are valuable !ieces of further information, a !ast history of hy!omanic or manic e!iso"es is the one choice that mi$ht si$nificantly alter treatment !lannin$. It is likely that all anti"e!ressants, inclu"in$ ))6Is, have a !otential to cause a switch into hy!omania or mania, or to accelerate cyclin$ in !atients with bi!olar "isor"er. If the !atient has a history of hy!omanic or manic e!iso"es, her "e!ression woul" be consi"ere" a bi!olar "e!ression rather than a uni!olar "e!ression. In this case, the !atient woul" likely nee" to be starte" on a moo" stabili#er before be$innin$ treatment with an anti"e!ressant me"ication. A family history of "e!ressive e!iso"es 'choice A) is useful information in su!!ortin$ the "ia$nosis of a !rimary moo" "isor"er. 7his "etail woul" !robably not si$nificantly alter treatment !lannin$, althou$h a clear res!onse to one anti"e!ressant in a family member mi$ht su!!ort the use of that same me"ication in the !atient. A family history of alcoholism or "ru$ "e!en"ence 'choice ) is also useful information, thou$h it woul" !robably not alter the treatment !lan. A family history of a""ictive "isor"ers increases the likelihoo" of the !atient e*!eriencin$ !rimary moo" "isor"ers or substance use "isor"ers. ))6Is are not a""ictive. A !ast history of ki"ney "isease 'choice %) is relevant in any !atient, but is not es!ecially im!ortant in this case. ))6Is are !rimarily metaboli#e" throu$h the liverAs cytochrome &?/> microsomal en#ymes. A !ast history of obsessions or com!ulsions 'choice () is inclu"e" in $atherin$ any !sychiatric history, but is not of !articular im!ortance in this case. ;>) 7he correct answer is A. In this instance, an e*tremely stressful event has been followe" by locali#e" loss of memory or amnesia of circumstances surroun"in$ the event, makin$ the "ia$nosis of

"issociative amnesia the most likely "ia$nosis. %issociative amnesia is often accom!anie" by ni$htmares an" an*iety concernin$ the event, both of which this !atient also has. %issociative fu$ue 'choice ) is a "isturbance of i"entity that requires a su""en, une*!ecte" travel away from home or oneAs !lace of work, with inability to recall oneAs !ast. %issociative i"entity "isor"er 'choice C) is also a "isturbance of i"entity. It requires the !resence of two or more "istinct i"entities or !ersonality states, which recurrently take control of the !ersonAs behavior. 7his is !o!ularly known as multi!le !ersonality "isor"er. -actitious "isor"er 'choice %) is a "ia$nosis requirin$ intentional !ro"uction of sym!toms an" $ratification from assumin$ the sick role. 7ransient $lobal hy!o*ia 'choice () is not a likely "ia$nosis $iven this !atientAs lack of altere" consciousness followin$ this event. ;:) 7he correct answer is C. .iven this !atientAs risk factors for 2I1, this is the most likely "ia$nosis in the conte*t of mil" co$nitive "eficits an" moo" "isturbance. 7o be$in treatment with anti"e!ressant me"ication, moo" "isturbance "ue to a $eneral me"ical con"ition must be rule" out. 7hus, or"erin$ fluo*etine 'choice A) or imi!ramine 'choice ) is ina!!ro!riate at this time. A urine "ru$ screen 'choice %) may !rovi"e hel!ful information, but is of less value at !resent than an 2I1 test in treatin$ the !atient. 6eferral to rehab 'choice () woul" be in"icate" at some !oint, but is of less ur$ency in evaluatin$ an" treatin$ the !atientAs current sym!toms. ;;) 7he correct answer is (. 7olerance "escribes the !henomenon of a "rinker nee"in$ $reater amounts of alcohol to $et the same effect. It "evelo!s over time an" is an in"ication of "e!en"ence. A blackout 'choice A) occurs "urin$ a !erio" of alcohol into*ication for which there is a com!lete antero$ra"e amnesia even thou$h !atient is awake an" alert. Confabulation 'choice ) is associate" with 5orsakoff syn"rome an" is a result of chronic alcohol abuse. ecause of retro$ra"e an" antero$ra"e amnesia, the !erson fills in $a!s in memory by ima$ine" or untrue e*!eriences that he or she believes ha!!ene", even thou$h they have no basis. %ereism 'choice C) is mental activity that is not in accor"ance with reality or lo$ic. It is mostly seen in !sychoses. 6esistance 'choice %) is a !henomenon associate" with the "evelo!ment of transference. It refers to inhibition of free association an" an o!!osition of $oals of analysis. ;0) 7he correct answer is C. 7his !atientAs sym!toms su$$est that he has !anic "isor"er, which is most a!!ro!riately treate" acutely with a ben#o"ia#e!ine with a me"ium len$th half4life an" "uration of action, such as clona#e!am. us!irone 'choice A) is an anti"e!ressant me"ication that has not been shown to be effective in the treatment of !anic attack.

Chlor!roma#ine 'choice ), mirta#a!ine 'choice %), an" trifluo!era#ine 'choice () are all anti!sychotics that are not in"icate" for treatment of !anic attacks. ;?) 7he correct answer is . 7his case illustrates conversion "isor"er. In this con"ition, !hysical sym!toms are cause" by !sycholo$ical conflict. 7he sym!toms "evelo! unconsciously an" are, by "efinition, limite" to those that mimic a neurolo$ic "isor"er, such as im!aire" coor"ination, weakness, !aralysis, loss of sensation, blin"ness, "eafness, or inability to s!eak. 7he onset is usually abru!t an" linke" to a stressful event. In most !atients, sym!toms im!rove within ; weeks, althou$h some !atients will have !ersistent or recurrent !roblems. o"y "ysmor!hic "isor"er 'choice A) refers to an abnormal bo"y ima$e, such as when a youthful !atient believes she is terribly wrinkle" even when everyone aroun" her !erceives her as youn$. &atients with +unchausen syn"rome 'choice C) ten" to have elaborate an" "eliberate fabrications of illness that "onAt usually resolve s!ontaneously. &ain "isor"er 'choice %) is the term use" for !atients with !ersistent !roblems of !sycho$enic !ain. )omati#ation "isor"er 'choice () usually involves lon$4stan"in$, nons!ecific bo"ily com!laints. ;/) 7he correct answer is %. 7his !atient has sym!toms of schi#oi" !ersonality "isor"er, which is characteri#e" by restricte" emotions an" social "etachment. &atients with this "isor"er ty!ically are "istant from other in"ivi"uals, an", in contrast to those with avoi"ant !ersonality "isor"er, "o not "esire contact with other in"ivi"uals in social situations. 7hey usually have a very narrow ran$e of interests. Antisocial !ersonality "isor"er 'choice A) is characteri#e" by a "isre$ar" for the basic ri$hts of other in"ivi"uals an" sometimes violent behavior, lea"in$ to le$al an" financial "ifficulties. or"erline !ersonality "isor"er 'choice ) is characteri#e" by severe emotional an" social instability. It is frequently accom!anie" by suici"e $estures, substance abuse, an" very emotionally labile relationshi!s. %e!en"ent !ersonality "isor"er 'choice C) is characteri#e" by clin$in$ an" submissive behavior of one in"ivi"ual towar" another, usually in the conte*t of a romantic relationshi!. &atients with this "isor"er ty!ically have sym!toms in an effort to avoi" the loss of a relationshi!. )chi#o!hrenia 'choice () is a "isor"er of thou$ht that is frequently accom!anie" by au"itory hallucinations, affective flattenin$, an" social autism, which occur over the course of at least C months. ;C) %. 7he "isor"er belon$s to the !ara!hilias, which are characteri#e" by "eviant or bi#arre se*ual im!ulses or !ractices. &atients re!eate"ly en$a$e in this behavior an" are unable to control their im!ulses. In frotteurism, se*ual arousal is achieve" by rubbin$ the $enitals a$ainst women, usually in crow"e" !laces where a !otential victim cannot easily esca!e.

7he "isor"er is more common in nonassertive, !assive men. %ys!areunia 'choice A) belon$s to the se*ual !ain "isor"ers an" is "efine" by !ersistent $enital !ain before, "urin$, or after se*ual intercourse. 7he criteria for this "isor"er e*clu"e va$inismus. (*hibitionism 'choice ) is a !ara!hilia "efine" by the nee" for !ublic e*!osure of the $enitals to evoke shock or fear in victims. Offen"ers are usually male. -etishism 'choice C) is a !ara!hilia "efine" by the nee" to use certain inanimate ob<ects to accom!lish se*ual arousal. It is usually seen in men. 7he "isor"er is accom!anie" by $uilt, which is ty!ically a""resse" in in"ivi"ual !sychothera!y. )e*ual masochism 'choice () is a !ara!hilia in which se*ual !leasure is "erive" from bein$ humiliate" or !hysically or mentally abuse" by the !artner. ;D) 7he correct answer is . 7his !atient is e*!eriencin$ neurole!tic4in"uce" acute "ystonia. About :>= of !atients e*!erience "ystonia as an a"verse effect of anti!sychotics, usually within the first few "ays of treatment. %ystonia can involve the neck 'torticollis), the <aw, the ton$ue, the eyes, other s!ecific muscle $rou!s, an" the entire bo"y. %ystonias are most common with hi$h !otency conventional anti!sychotics, such as halo!eri"ol. &seu"o!arkinsonism an" akathisia are other forms of acute e*tra!yrami"al si"e effects. -luo*etine 'choice A) is a selective serotonin reu!take inhibitor '))6I) that is use" in the treatment of "e!ression, bulimia nervosa, an" several !rimary an*iety "isor"ers. It is not associate" with acute "ystonic reactions. ,ora#e!am 'choice C) may have been use" to control an*iety an" a$itation in this !atient. 2owever, ben#o"ia#e!ines are not associate" with e*tra!yrami"al si"e effects. Olan#a!ine 'choice %) is an aty!ical anti!sychotic a$ent. While these a$ents 'inclu"in$ clo#a!ine, ris!eri"one, olan#a!ine, an" quetia!ine) may be associate" with e*tra!yrami"al si"e effects, es!ecially at hi$her "osa$es, they carry a much lower risk of such neurolo$ical si"e effects. )ertraline 'choice (), like fluo*etine, is an ))6I use" in the treatment of "e!ression an" several an*iety "isor"ers. It is not associate" with acute "ystonia. ;@) 7he correct answer is (. 7his !atient has somati#ation "isor"er. 7he !hysical com!laints are usually va$ue an" not a"equately e*!laine" by a !hysical "isor"er. -ormal criteria for the "ia$nosis have been "efine" an" inclu"e onset before a$e 0>, sym!toms involvin$ at least four "ifferent bo"y !arts, two or more $astrointestinal sym!toms, at least one re!ro"uctive or se*ual sym!tom, an" at least one neurolo$ic sym!tom other than !ain. &atients may have coe*istent !ersonality "isor"ers, notably histrionic, bor"erline, an" antisocial. 7reatment is "ifficult, an" the con"ition ten"s to wa* an" wane throu$hout life. )uici"e is a "efinite risk with overtly "e!resse" !atients who have lon$4stan"in$ "isease. Conversion "isor"er 'choice A) refers to the "evelo!ment of s!ecific, usually "ramatic, !hysical com!laints in res!onse to !sycholo$ic stress. 2y!ochon"riasis 'choice ) is much more common than somati#ation "isor"er an" is usually not as severe. 'Note: in real life, the "istinction may be "ifficult. ) +alin$erin$ 'choice C) is fei$nin$ illness to avoi" a s!ecific social, school,

or work situation. 7he s!ecific situation involve" can usually be elicite" on questionin$. +unchausen syn"rome 'choice %) involves more elaborate an" "eliberate mimickin$ of me"ical con"itions. ;I) 7he correct answer is %. Neurole!tic mali$nant syn"rome 'N+)) is a life4threatenin$ com!lication of anti!sychotic me"ications. )i$ns an" sym!toms inclu"e: muscle ri$i"ity, elevate" tem!erature, autonomic instability, altere" mental status, "ia!horesis, incontinence, elevate" creatine !hos!hokinase, tremor, mutism, leukocytosis, an" myo$lobinuria. Cessation of the anti!sychotic a$ent an" su!!ortive care 'often in an intensive care settin$) are the mainstays of treatment, thou$h "antrolene, bromocri!tine, amanta"ine, an" lora#e!am are also often use". Catatonia may be a risk factor for N+). ,ethal catatonia 'choice A) is a syn"rome with the same clinical features as N+), but without a recent history of anti!sychotic use. .iven this !atientAs history of catatonia, lethal catatonia must be consi"ere". 2owever, "ue to his use of a hi$h4!otency neurole!tic a$ent, his sym!toms are !resumably relate" to the halo!eri"ol. Neurole!tic4in"uce" acute "ystonia 'choice ) is a contraction of a muscle or muscle $rou! associate" with the use of anti!sychotic a$ents, es!ecially hi$h4!otency conventional anti!sychotics such as halo!eri"ol. (*am!les of this form of e*tra!yrami"al si"e effect inclu"e oculo$yric crisis, ton$ue !rotrusion, trismus, torticollis, an" "ystonic !ostures of the limbs or trunk. 7reatment involves imme"iate a"ministration of an anticholiner$ic a$ent intramuscularly. Neurole!tic4in"uce" !arkinsonism 'choice C) is characteri#e" by restin$ tremor, ri$i"ity, an" bra"ykinesia. &arkinsonism woul" not cause the autonomic instability, hy!erthermia, altere" level of consciousness, or laboratory values seen in this !atient. )chi#o!hrenia, catatonic ty!e 'choice () is associate" with catatonic e!iso"es, but woul" not be associate" with this "e$ree of im!airment, unless lethal catatonia is !resent. 0>) 7he correct answer is (. 1a$inismus is one of the se*ual !ain "isor"ers. It is "efine" as involuntary muscle contraction of the outer thir" of the va$ina that interferes with intercourse. It is !revalent in women who have a history of se*ual trauma, emotional abuse, ri$i" reli$ious u!brin$in$, or !sychose*ual conflicts. %ys!areunia 'choice A) is a recurrent or !ersistent $enital !ain occurrin$ before, "urin$, or after intercourse. It can occur in either men or women, but it is more common in women. Or$anic causes must be rule" out. It causes marke" "istress or inter!ersonal "ifficulty. -etishism 'choice ) is relate" to !ara!hilias an" is "efine" as recurrent se*ually arousin$ fantasies or behaviors involvin$ the use of nonlivin$ ob<ects. 7he fantasies cause si$nificant "istress in functionin$. 7he fetish ob<ects are not use" for cross4"ressin$ !ur!oses. -rotteurism 'choice C) involves behaviors or fantasies that are se*ually arousin$ an" involve touchin$ or rubbin$ $enitals a$ainst a non4consentin$ !erson. 7he "isor"er causes clinically si$nificant im!airment in functionin$ an" must last at least C months to meet the "ia$nostic criteria. &seu"ocyesis 'choice %) is a con"ition in which the !atient has si$ns an" sym!toms of !re$nancy. It is

relate" to conversion sym!toms an" can be viewe" as a !sychosomatic "isor"er. 6eality4 base" su!!ortive !sychothera!y is the treatment of choice. 0:) 7he correct answer is A. Cocaine into*ication is characteri#e" by sym!athic stimulation, inclu"in$ tachycar"ia, hy!ertension, an" sweatin$. 7he moo" is elate" an" eu!horic while into*icate", an" there is restlessness an" !ressure" s!eech. &sychotic sym!toms can occur with !rolon$e" use. 2eroin into*ication 'choice ) causes si$nificant behavioral chan$es an" im!aire" social functionin$, as well as !u!illary constriction, "rowsiness, slurre" s!eech, an" im!airment of attention an" memory. )i$ns of nicotine into*ication 'choice C) are nausea, vomitin$, salivation, !allor, weakness, ab"ominal !ain, "iarrhea, "i##iness, hea"ache, tremor, col" sweats, tachycar"ia, confusion, an" sensory "isturbances. Or$anic inhalant into*ication 'choice %) causes "i##iness, nysta$mus, incoor"ination, lethar$y, unstea"y $ait, slurre" s!eech, muscle weakness, tremor, blurre" vision, !sychomotor retar"ation, an" stu!or. &hencycli"ine into*ication 'choice () causes behavioral chan$es shortly after the use of "ru$, as well as two or more of the followin$ si$ns: ata*ia, nysta$mus, hy!ertension, tachycar"ia, "ysarthria, muscle ri$i"ity, numbness, sei#ures, or coma. 0;) 7he correct answer is . A"<ustment "isor"er is e*em!lifie" by a set of behavioral or emotional sym!toms "evelo!in$ as a res!onse to an i"entifiable stressor within 0 months after e*!osure to the stressor. 7he sym!toms are e*cessive com!are" with what one woul" e*!ect from the e*!osure, an" they cause marke" im!airment in social functionin$. Acute stress "isor"er 'choice A) is an*iety !ro"uce" by e*traor"inary life stress. An event is relive" in "reams an" wakin$ thou$hts. 7he sym!toms inclu"e re4e*!eriencin$, avoi"ance, an" hy!erarousal lastin$ less than a month. Antisocial !ersonality "isor"er 'choice C) involves a !ervasive !attern of violation of the ri$hts of others after the a$e of :/, as in"icate" by a failure to conform to social norms, "eceitfulness, im!ulsivity, irritability, "isre$ar" for the safety of others, an" a lack of remorse. rief !sychotic "isor"er 'choice %) requires the !resence of one or more of the followin$: "elusions, hallucinations, or "isor$ani#e" s!eech. %uration of an e!iso"e is : "ay u! to : month, an" it is not "ue to any other me"ical con"ition or substance abuse. +a<or "e!ressive "isor"er 'choice () involves the !resence of "e!resse" moo" or anhe"onia for at least ; weeks on a "aily basis in the !ast month, as well as the a""itional sym!toms of chan$es in a!!etite, wei$ht, slee!, ener$y, an" concentrationG the !resence of $uilt an" suici"al i"eationG an" chan$es in !sychomotor activity. 7he sym!toms are not "ue to a me"ical con"ition or the use of substances. 00) 7he correct answer is

C. Any event that affects the vasculature, such as a myocar"ial infarction or a cerebrovascular acci"ent 'C1A), has been shown to increase the risk of ma<or "e!ressive "isor"er in the months followin$ such an event. 7he !atho!hysiolo$y of such a "evelo!ment is unclear, but is thou$ht to be relate" to the effects of serotonin on vascular !hysiolo$y. Conversion "isor"er 'choice A) is a "isor"er of !hysiolo$ical com!laints that are relate" to social stressorsG it has not shown to be relate" to vascular events. +ania 'choice ) is a syn"rome of increase" elevate" moo", irritability, an" $ran"iosity associate" with bi!olar affective "isor"er, but not associate" with C1As. )chi#o!hrenia 'choice %) is a thou$ht "isor"er characteri#e" by "isturbances in lan$ua$e, thou$ht content, an" !erce!tion. It is not relate" to C1 A. )ubstance abuse 'choice () is not increase" in inci"ence in in"ivi"uals after cerebrovascular acci"ent. 0?) 7he correct answer is (. Cannabis into*ication is usually characteri#e" by sensitivity to e*ternal stimuli, sub<ective slowin$ "own, im!airment of motor skills 'inclu"in$ !roblems with o!eration of motor vehicles), con<unctival in<ection, "ry mouth, tachycar"ia, an" increase" a!!etite. Am!hetamine into*ication 'choice A) !ro"uces mala"a!tive behavior, as well as two or more of the followin$: !u!illary "ilatation, wei$ht loss, !ers!iration, chan$es in bloo" !ressure, !sychomotor chan$es, chan$es in heart rate, muscular weakness, sei#ures, an" coma. en#o"ia#e!ine into*ication 'choice ) usually results in behavioral "isinhibition, eu!horia, slurre" s!eech, incoor"ination, nysta$mus, memory im!airment, an" stu!or or coma. Cocaine into*ication 'choice C) inclu"es mala"a!tive behavior 'eu!horia, hy!ervi$ilance, an*iety, or an$er), tachycar"ia, elevate" bloo" !ressure, nausea, wei$ht loss, !u!illary "ilatation, !sychomotor chan$es, sei#ure, confusion, muscular weakness, an" coma. O!iate into*ication 'choice %) involves mala"a!tive behavior, !u!illary constriction, slurre" s!eech, im!aire" attention an" memory, an" "rowsiness or coma. 0/) (*!lanation: 7he correct answer is (. &aro*etine, alon$ with other selective serotonin reu!take inhibitors, can cause "ecrease" libi"o an" "ifficulties reachin$ or$asm. In stu"ies, the inci"ence of these si"e effects was := to ;= in !atients on !aro*etine, com!are" with those on !lacebo. Amitri!tyline 'choice A) is use" less frequently in the treatment of "e!ression because of its numerous si"e effects an" !ossible lethal consequences with over"ose. It can cause "ecrease" libi"o, but usually "oesnAt cause anor$asmia. u!ro!ion 'choice ) has ha" no ne$ative effects on se*ual functions in !lacebo4controlle" trials. +irta#a!ine 'choice C) can infrequently 'less than :=) cause "ecrease" libi"o, but has ha" no effects on or$asm in !lacebo4controlle" stu"ies. In about := of !atients stu"ie", nefa#o"one 'choice %) has cause" "ecrease" libi"o but has ha" no effect on or$asm com!are" with !lacebo.

0C) 7he correct answer is . 7his !atient a!!ears to have sym!toms of ma<or "e!ression in the conte*t of bulimia. )he shoul" therefore be treate" with an anti"e!ressant me"ication, an" a selective serotonin reu!take inhibitor '))6I), such as fluo*etine, woul" be most a!!ro!riate. u!ro!ion 'choice A) is an anti"e!ressant, but it is contrain"icate" in !atients with bin$in$ an" !ur$in$ behavior because of its lowerin$ of the sei#ure threshol" in !atients with eatin$ "isor"ers. 2alo!eri"ol 'choice C) is an anti!sychotic me"ication an" is not in"icate" in ma<or "e!ression. ,ithium carbonate 'choice %) is sometimes use" to treat "e!ression, but only in the conte*t of bi!olar "isor"er. Its usual in"ication is for the treatment of mania. 1al!roic aci" 'choice () is an anticonvulsant use" as a moo" stabili#er in the treatment of bi!olar "isor"er. 0D) (*!lanation: 7he correct answer is . -luma#enil is a ben#o"ia#e!ine rece!tor anta$onist, which reverses the !sycho!hysiolo$ic effects of ben#o"ia#e!ine me"ications, such as al!ra#olam, "ia#e!am, an" lora#e!am. After I1 a"ministration, fluma#enil has a half4life of D4:/ minutes. -or the initial mana$ement of a known or sus!ecte" ben#o"ia#e!ine over"ose, the recommen"e" initial "ose of fluma#enil is >.; m$ a"ministere" I1 over 0> secon"s. If the "esire" level of consciousness is not obtaine" after waitin$ 0> secon"s, a further "ose of >.0 m$ can be a"ministere" over 0> secon"s. A""itional "oses can be a"ministere" u! to a cumulative "ose of 0.> m$. A secure airway an" I1 access shoul" be establishe" before a"ministration of the "ru$. en#tro!ine 'choice A) is an anticholiner$ic me"ication that can be use" in the acute settin$ to reverse anti!sychotic4in"uce" acute "ystonic reactions. It is also use" on a lon$4term basis for !ro!hyla*is of e*tra!yrami"al si"e effects. 2alo!eri"ol 'choice C) woul" not be a !ro!er treatment for this !atient. It is often use" in the emer$ency settin$ for treatment of a$itation. ,ora#e!am 'choice %) is a ben#o"ia#e!ine an" shoul" not be a"ministere" to this !atient. It woul" worsen the !atientAs ben#o"ia#e!ine into*ication. Nalo*one 'choice () is an o!iate anta$onist that is use" in the treatment of o!iate an" o!ioi" over"ose because it reverses the effects of narcotics. Nalo*one can be a"ministere" I1 '>.? m$) an" this "ose can be re!eate" ?4/ times within the first 0>4?/ minutes. 7he !atient $enerally becomes res!onsive, but, because of its short "uration of action, the !atient may rela!se into coma, makin$ close observation very im!ortant. 0@) 7he correct answer is (. 7he !robable "ia$nosis is ma<or "e!ressive "isor"er. In or"er to meet criteria for this "isor"er, the !atient must have five or more sym!toms "urin$ the same two4week !erio". 7his !atient has several such sym!toms: she a!!ears sa" '"e!resse" affect), she has si$nificant wei$ht loss, she "escribes feelin$ fati$ue", an" she com!lains of !oor concentration. Other sym!toms that she may be e*!eriencin$ inclu"e: "iminishe"

interests or !leasure 'anhe"onia), chan$es in slee! habits 'insomnia or hy!ersomnia), !sychomotor a$itation or retar"ation, feelin$s of worthlessness or ina!!ro!riate $uilt, an" suici"al i"eation. A"<ustment "isor"er with an*iety 'choice A) is the "ia$nosis when the "evelo!ment of si$nificant an*iety sym!toms is in res!onse to i"entifiable stressors within 0 months of the onset of the stressor. 7he stress4relate" "isturbance "oes not meet criteria for another s!ecific A*is I "isor"er. ereavement 'choice ) refers to the state of bein$ "e!rive" of someone by "eath, an" it refers to bein$ in a state of mournin$. Althou$h the !atientAs "au$hter recently left for colle$e, the !atientAs sym!toms can not be accounte" for by bereavement. %e!en"ent !ersonality "isor"er 'choice C) refers to a chronic an" !ervasive !ersonality structure in which the !atient has an e*cessive nee" to be taken care of. 7his lea"s to submissive an" clin$in$ behavior an" fears of se!aration, be$innin$ by early a"ulthoo" an" !resent in a variety of conte*ts. 7here is no evi"ence from this case that the !atient has such !ersonality features. %ysthymic "isor"er 'choice %) refers to a mil"er form of "e!ression that lasts for at least ; years 'or at least one year in chil"ren an" a"olescents). &atients e*hibit two or more "e!ressive sym!toms. 7here is no evi"ence that this !atientAs sym!toms are chronic. 0I) 7he correct answer is . 7his !atient has acute "ystonia. %ystonia is characteri#e" by involuntary muscle s!asms, which in this !atient, are "ue to a hi$h !otency anti!sychotic a$ent. %ystonia is most common in youn$ men, an" often be$ins within "ays of startin$ the "ru$ thera!y. It usually involves the muscles of the hea" an" neck, lea"in$ to torticollis an" ble!haros!asm. It can !ro"uce a life4threatenin$ laryn$os!asm requirin$ intubation. 7he treatment of acute "ystonia is with anticholiner$ic me"ications, such as ben#tro!ine or "i!henhy"ramine. Amanta"ine 'choice A) enhances "o!aminer$ic transmission an" may im!rove the bra"ykinesia, ri$i"ity, an" tremor of &arkinsonAs "isease. It may also be hel!ful in the treatment of neurole!tic mali$nant syn"rome 'N+)). It is not use" for acute "ystonia. romocri!tine 'choice C) is a "o!amine a$onist that is use" in &arkinson "isease an" N+). It is not use" for acute "ystonia. Cloni"ine 'choice %) is an al!ha a$onist that is use" in hy!ertension an" alleviates the motor an" !honic tics in 7ourette syn"rome. &ro!ranolol 'choice () is a beta4blocker that is use" in hy!ertension, an$ina, arrhythmias, tremor, an" social !hobias. It may be hel!ful for !atients with akathisia. It is not use" for acute "ystonia. ?>) 7he correct answer is A. 7he !atientAs hallucinations, a$itation, an" tremor !oint to the "ia$nosis of "elirium tremens '%7s), which shoul" always be consi"ere" when a !atient "oes not have access to alcohol after a"mission to the hos!ital. %e!ression 'choice ) is not a cause of tremor, althou$h it can be associate" with a$itation in "e!ression with !sychotic features. &ast ,)% use 'choice C) has been re!orte" to lea" to flashback visual hallucinations, but woul" not be likely to cause the %7s this !atient is e*!eriencin$. )chi#o!hrenia 'choice %) is a "isor"er of thou$ht often characteri#e" by au"itory hallucinations. It is not

associate" with formication, i.e., the feelin$ that insects are crawlin$ on oneAs skin. Chil"hoo" trauma 'choice () has no "emonstrate" link to the syn"rome e*hibite" by this !atient. ?:) 7he correct answer is . 7his !atient is most likely e*!eriencin$ a manic e!iso"e. 2e has a history of bi!olar I "isor"er, which is characteri#e" by a sin$le manic e!iso"e or recurrent manic e!iso"es, often alternatin$ with ma<or "e!ressive e!iso"es. )ym!toms of a manic e!iso"e, lastin$ for at least one week, inclu"e: !ersistently elevate", e*!ansive, or irritable moo", inflate" self4esteem or $ran"iosity, "ecrease" nee" for slee!, hy!erverbal an" !ressure" s!eech, fli$ht of i"eas, "istractibility, increase in $oal4"irecte" activity, an" e*cessive involvement in !leasurable activities. lunte" affect 'choice A) "escribes a marke" re"uction in emotional e*!ression, which woul" be unlikely in this !atient. Ina!!ro!riate an" e*cessive $uilt 'choice C) is a sym!tom of a ma<or "e!ressive e!iso"e. Althou$h !atients with bi!olar I "isor"er may e*!erience mi*e" e!iso"es 'the full criteria for a ma<or "e!ressive e!iso"e an" for a manic e!iso"e are met concurrently), we have no evi"ence of "e!ressive sym!toms in this !atient. Nihilistic "elusions 'choice %) are an e*am!le of moo"4con$ruent !sychotic features that may be associate" with a ma<or "e!ressive e!iso"e. While !atients e*!eriencin$ a manic e!iso"e commonly have !sychotic features, nihilistic "elusions woul" be quite uncommon, unless other si$nificant "e!ressive features are !resent. 6e!etitive han"4washin$ an" countin$ 'choice () are sym!toms characteristic of obsessive4com!ulsive "isor"er, in which either si$nificant obsessions '!atholo$ical !ersistence of an irresistible thou$ht or feelin$ that cannot be eliminate" an" which is associate" with an*iety) or com!ulsions '!atholo$ical nee" to act on an im!ulse that, if resiste", !ro"uces an*iety, or a re!etitive behavior in res!onse to an obsession) are !resent. ?;) 7he correct answer is C. 7his !atient is most likely e*!eriencin$ alcohol with"rawal "elirium '"elirium tremens). 7his most severe form of alcohol with"rawal is a me"ical emer$ency that can result in si$nificant morbi"ity an" mortality. Fntreate", "elirium tremens has a mortality rate of a!!ro*imately ;>=. In a""ition to "elirium, !atients in this sta$e of with"rawal e*hibit autonomic hy!eractivity, !erce!tual "istortions 'commonly visual or tactile hallucinations, as in this case), an" fluctuatin$ levels of !sychomotor activity. About /= of all alcohol "e!en"ent hos!itali#e" !atients will "evelo! "elirium tremens. 7he con"ition frequently "evelo!s une*!ecte"ly aroun" the thir" hos!ital "ay. en#o"ia#e!ines, such as lora#e!am, are the treatment of choice. ,ora#e!am can be a"ministere" &O, I+, or I1. )u!!ortive mana$ement, inclu"in$ hy"ration an" very close monitorin$, shoul" also be initiate". Cloni"ine 'choice A) may be useful in controllin$ hy!ertension associate" with "elirium tremens, but it woul" not a"equately treat the con"ition or other sym!toms. 2alo!eri"ol 'choice ) is a hi$h4!otency conventional anti!sychotic that woul" not treat "elirium tremens, thou$h it mi$ht mo"ify the

!erce!tual "isturbances associate" with the con"ition. Nalo*one 'choice %) is an o!ioi" anta$onist use" in the treatment of o!ioi" into*ication or over"ose. It woul" not be a !ro!er treatment of alcohol with"rawal. Olan#a!ine 'choice () is an aty!ical anti!sychotic that mi$ht control !sychotic sym!toms associate" with alcohol with"rawal, but woul" not treat the un"erlyin$ con"ition. ?0) 7he correct answer is A. 7he !robable "ia$nosis is bor"erline !ersonality "isor"er. 7his "isor"er is marke" by a !ervasive !attern of instability of inter!ersonal relationshi!s, self4ima$e, an" affect, as well as marke" im!ulsivity by early a"ulthoo". Criteria for this "ia$nosis evi"ent in this !atientAs history inclu"e: affective instability marke" by severe an*iety, irritability, an" "e!ression, ina!!ro!riate an" intense an$er, im!ulsivity as ma"e evi"ent by cuttin$ behavior an" bin$es on alcohol, a !attern of unstable an" intense relationshi!s, an" recurrent suici"al i"eation an" behaviors. 7he chronicity of these behaviors also su!!orts this "ia$nosis. %e!en"ent !ersonality "isor"er 'choice ) is characteri#e" by an e*cessive nee" to be taken care of, which lea"s to submissive an" clin$in$ behavior an" fears of se!aration. 2istrionic !ersonality "isor"er 'choice C) is a cluster !ersonality "isor"er, like bor"erline !ersonality "isor"er. 2istrionic !atients nee" to be the center of attention, an" e*hibit a !attern of e*cessive emotionality an" attention seekin$, se"uctive or !rovocative behavior, shallow e*!ressions of emotion, self4"ramati#ation, su$$estibility, an" an im!ressionistic style of s!eech. Narcissistic !ersonality "isor"er 'choice %) is also a cluster !ersonality "isor"er. 7his "isor"er is characteri#e" by a !attern of $ran"iosity, nee" for a"miration, an" lack of em!athy. )chi#oty!al !ersonality "isor"er 'choice () is characteri#e" by a !attern of social an" inter!ersonal "eficits marke" by acute "iscomfort with, an" re"uce" ca!acity for, close relationshi!s, as well as by co$nitive or !erce!tual "istortions an" eccentricities of behavior. ??) 7he correct answer is . o"y "ysmor!hic "isor"er, or "ysmor!ho!hobia, is the !reoccu!ation with an ima$ine" "efect in a!!earance or e*cessive !reoccu!ation with a sli$ht anomaly !resent. It causes si$nificant "istress. &atient are often ashame" to !resent for treatment. 7hey may frequently check their ima$e an" try to com!ensate for the ima$ine" "efect. 7he "isor"er lea"s to social isolation secon"ary to ima$ine" mockery. It can cause iatro$enic com!lications, like un"er$oin$ cosmetic sur$ery. 7reatment ty!ically focuses more on !revention of iatro$enesis, but it can also inclu"e selective serotonin reu!take inhibitors '))6Is). Alo!ecia areata 'choice A) is a "ermatolo$ic "isor"er in which there is a focal hair loss tri$$ere" by environmental factors an" !ossibly, autoimmune reactions. &sycholo$ical stressors may contribute to the "isor"er, but are not essential. 2y!ochon"riasis 'choice C) is a syn"rome of e*cessive an" !ervasive !reoccu!ation with an" fears of havin$ a serious illness. 7he belief "oes not res!on" to reassurance or me"ical evaluation. It causes si$nificant im!airment in every"ay life an" must be !resent for at least C months. )chi#o!hrenia 'choice %) is "efine" by "elusions, hallucinations,

an" other evi"ence of a formal thou$ht "isor"er lastin$ more than C months. )ome schi#o!hrenic !atients may have somatic "elusions re$ar"in$ bo"ily "efects as !art of the clinical !icture. )i$nificant im!airment in social an" occu!ational functionin$ must also be !resent to establish the "ia$nosis. 7richotillomania 'choice () is an im!ulse4control "isor"er. 7he term encom!asses the com!ulsive ur$e to !ull out oneAs own hair, resultin$ in hair loss. Increase" tension !rior to hair !ullin$ is !resent, but the e*!erience afterwar" is !leasurable an" tension relievin$. It is more common in females. It may have comorbi"ity with other mental "isor"ers, such as "e!ression an" obsessive4com!ulsive "isor"er. ?/) 7he correct answer is (. )lee! a!nea is characteri#e" by multi!le !erio"s of cessation of breathin$ "urin$ slee! 'a!nea), "aytime slee!iness, an" lou" snorin$. Its !revalence often correlates with obesity. Catale!sy 'choice A) is a "isor"er in which a !erson maintains the !osition of the bo"y into which he or she is !lace". It is most often seen in severe cases of catatonic schi#o!hrenia. 5leine4,evin syn"rome 'choice ) is a "isor"er most commonly seen in a"olescent boys, in which there are alternatin$ !erio"s of bulimia an" hy!ersomnia. Narcole!sy 'choice C) is a "isturbance of slee! known as a "yssomnia, in which normal "aytime activity is interru!te" by brief, recurrent, uncontrollable e!iso"es of slee!. &rimary hy!ersomnia 'choice %), by "efinition, is a com!laint of e*cessive slee!iness for at least a month characteri#e" by either !rolon$e" slee! e!iso"es or "aytime slee! e!iso"es that occur almost "aily. ?C) 7he correct answer is %. &ost4stroke mania is a rare !henomenon usually seen in infarctions of ri$ht frontal lobe an" sometimes other !arts of the ri$ht hemis!here. ,eft hemis!heric lesions inclu"in$ rocaAs area 'choice A), are sometimes associate" with the "evelo!ment of catastro!hic reactions that inclu"e restlessness, hy!eremotionality, irritability, an" su""en outbursts of emotion. )troke in the left !refrontal corte* 'choice ) is associate" with the "evelo!ment of "e!ression in more than ;>= of stroke victims within the first C months. +i"brain lesions 'choice C) are associate" with "reamlike hallucinations calle" !e"uncular hallucinosis. 7halamic strokes 'choice () are associate" with !ain syn"romes an" an altere" level of consciousness. ?D) 7he correct answer is %. In a !atient with com!lications from a stroke, a selective serotonin reu!take inhibitor '))6I), such as sertraline, has been "emonstrate" to be the safest an" most effective me"ication for the treatment of clinical "e!ression. +ethyl!heni"ate 'choice A) is a stimulant me"ication that woul" have the !otential to increase heart rate an" sym!athetic tone, which woul" not be a"vanta$eous in the !ost4cerebrovascular in<ury !erio".

Nortri!tyline 'choice ) is a tricyclic anti"e!ressant that has the !otential for !ro"ucin$ car"iac arrhythmias an" vascular com!romise in over"ose. &henel#ine 'choice C) is a monoamine o*i"ase inhibitor, which woul" be contrain"icate" in a !atient with a history of recent vascular in<ury, as the in$estion of tyramine with an +AO inhibitor coul" in"uce a hy!ertensive crisis. 7hiori"a#ine 'choice () is a low4!otency "o!amine anta$onist anti!sychotic me"ication that is not in"icate" for the treatment of "e!ression. ?@) 7he correct answer is (. &emoline is a sym!athomimetic a$ent a!!rove" for the treatment of attention "eficitBhy!eractivity "isor"er. .iven the rate of re!orte" cases of he!atic failure, which ran$es from ? to :D times hi$her than e*!ecte" in $eneral !o!ulation, !emoline can be use" only if informe" consent, with "iscussion of risks an" benefits has been obtaine" from the !atient or le$al $uar"ians. Cloni"ine 'choice A) is re!orte" to re"uce arousal in chil"ren with attention "eficitBhy!eractivity "isor"er. It is a centrally actin$ al!ha4; a$onist that is !artially metaboli#e" in the liver. 2owever, liver failure has not been frequently re!orte" with the use of cloni"ine. %esi!ramine 'choice ) is a tricyclic anti"e!ressant that can cause su""en "eath in chil"ren secon"ary to its ca!acity to in"uce arrhythmias because of 86) wi"enin$. 7hus, it is not wi"ely use" for treatment of attention "eficitBhy!eractivity "isor"er. %e*troam!hetamine 'choice C) is a!!rove" for the treatment of attention "eficitBhy!eractivity "isor"er. Its main a"verse reactions involve the car"iovascular system, the CN), the $astrointestinal system, an" the en"ocrine system. 2e!atic failure has not been re!orte". +ethyl!heni"ate 'choice %) is a stimulant use" for treatment of attention "eficitBhy!eractivity "isor"er. Isolate" cases of elevate" liver en#ymes have been "escribe", as has he!atic comaG however, the inci"ence is very low. ?I) 7he correct answer is (. u!ro!ion is an anti"e!ressant that can lower the sei#ure threshol" for some !atients, an" as a result shoul" be "iscontinue" or carefully monitore" in a !atient with anore*ia nervosa. &atients with anore*ia are at increase" sei#ure risk relate" to lar$e flui" shifts an" electrolyte "isturbances cause" by bin$ein$ an" !ur$in$ behavior. While car"iac arrhythmia 'choice A) is an im!ortant me"ical com!lication of anore*ia nervosa, bu!ro!ion has not been "emonstrate" to a"versely affect car"iac function. %ecrease" renal function 'choice ) is not ty!ically a me"ical com!lication of anore*ia nervosa, an" is also not attributable to the use of bu!ro!ion. 2y!ochon"riasis 'choice C) is a "isor"er that involves !reoccu!ation with fears of havin$ a serious "isease base" on the !atientAs misinter!retation of bo"ily sym!toms, an" is not relate" to the use of bu!ro!ion in "e!resse" !atients. Osteo!orosis 'choice %) is a chronic lon$ term effect of anore*ia nervosa, but is not relate" to the use of bu!ro!ion. />)

7he correct answer is (. Accor"in$ to follow4u! stu"ies, the !revalence of A%2% is estimate" to "ecline by /> = every / years until the mi"4;>As. 7herefore, it is acce!table to counsel this mother that the ol"er her chil"ren $et without sym!toms of A%2%, the less likely they are to have the "isor"er. Choice A is incorrect, because a"olescents an" a"ults are also ca!able of bein$ "ia$nose" with A%2 %. Choice is incorrect, because chil"ren with A%2% have a ;/= chance of "evelo!in$ antisocial !ersonality "isor"er. Choice C is incorrect, because there is no known association between schi#o!hrenia an" A%2%, an" no increase" risk for schi#o!hrenia beyon" that of the $eneral !o!ulation. Choice % is incorrect because her chil"As im!ulsivity an" hy!eractivity will "ecrease as the chil" $rows ol"er, but not the level of inattention.

USMLE Step 2 Practice Test Block 1<


Name: Instructions: Answer the questions below to the best of your ability. When you finish the test, click the Check button at the bottom to view the results.

1.A & !year!ol" 'oman is referre" $y her "ermatologist to a psychiatrist. 5or the past se)eral years% she has ha" a comp#lsi)e nee" to 'ash her han"s after shaking han"s 'ith someone. Since she is in the p#$lic relations $#siness% hi"ing her pro$lem 'hile still performing her rit#al ha)e $ecome increasingly "iffic#lt. She kno's that if she "oesn,t "o it% she $ecomes ner)o#s an" tense. =hich of the follo'ing is the most appropriate treatmenta6 B#spirone $62lonaDepam c65enfl#ramine "65l#)o.amine e6Aisperi"one :ormal La$s 2.A <8!year!ol" 'oman presents to her primary care physician for her ro#tine yearly e.amination. This year% her "a#ghter "ro)e her to the appointment. =hen aske" a$o#t this% the "a#ghter 'hispers to the physician that her mother has gotten lost 'hile "ri)ing to the s#permarket on t'o occasions o)er the past si. months. +t $ecomes clear "#ring the history that the 'oman "oes not remem$er the physician,s name an" seems to ha)e some 'or"!fin"ing "iffic#lties. The "a#ghter also mentions to the physician that she has recently $eg#n managing her mother,s checking acco#nt an" other finances% "#e to forgetf#lness. The patient has osteoarthritis of the knees% $#t is other'ise healthy. ?ital signs are 'ithin normal limits% an" physical e.amination is #nremarka$le. Mental stat#s e.amination re)eals slo' thinking an" impaire" attention. =hich of the follo'ing is the most appropriate ne.t step for e)al#ating this patient,s memory "eclinea6Mini!Mental Stat#s E.amination

$6La$oratory st#"ies c6Aeferral for ne#ropsychological testing "62omp#te" tomography 32T6 scan of the $rain e6L#m$ar p#nct#re :ormal La$s &.A 4!year!ol" female politician has an 0e.treme% paralyDing fear0 $efore 'alking to a po"i#m to gi)e a speech. She is so an.io#s a$o#t these sit#ations that it is $eginning to impair her professional acti)ities. She is terrifie" of st#ttering an" 0making a fool0 of herself in front of #nfamiliar people. =hich of the follo'ing is the most likely "iagnosisa6Agorapho$ia $6CeneraliDe" an.iety "isor"er 3CA@6 c6Panic "isor"er "6Posttra#matic stress "isor"er 3PTS@6 e6 Social pho$ia :ormal La$s 4. A 14!year!ol" $oy is $ro#ght in $y his mother after the police ca#ght him stealing a neigh$or,s $icycle. +n a""ition% the $oy is "oing poorly in school% an" he is s#specte" of $eing in)ol)e" in a recent arson fire in the a"9acent neigh$orhoo". :eigh$ors report that they ha)e often seen him lighting fires an" $#rning trash in )acant lots% an" they s#spect him in t'o other $#rglaries that occ#rre" in the past year. The $oy has a history of fighting an" tr#ancy at school since the age of 14. =hich of the follo'ing is the most likely "iagnosisa6Antisocial personality "isor"er $6Attention "eficit7hyperacti)ity "isor"er c62on"#ct "isor"er "6Mleptomania e6 Pyromania :ormal La$s .A 1 !year!ol" $oy is referre" to a psychiatrist $y his school co#nselor% an" is $ro#ght in $y his mother for increasing hostility an" arg#mentati)eness 'ith his teachers an" to'ar" his parents o)er the

last school year. /e often loses his temper an" often $lames his classmates 'hen his mis$eha)ior ca#ses him to recei)e p#nishment in class. /e ha" "one 'ell pre)io#sly 'ith his school'ork. /e has no history of theft% "estr#ction of property% or physical )iolence. =hich of the follo'ing is the most likely "iagnosisa6Attention "eficit7hyperacti)ity "isor"er 3A@/@6 $62on"#ct "isor"er c6@ysthymic "isor"er "6Mental retar"ation e6>ppositional "efiant "isor"er :ormal La$s <.The mother of a & !year!ol" man calls her primary care physician for a")ice on a "r#g treatment program for her son. She states that he li)es 'ith her since the loss of his 9o$ an" apartment fo#r months ago. She reports that he goes o#t on 'eeken"s an" #ses "r#gs hea)ily. =hen he ret#rns home% #s#ally on Mon"ay mornings% he sleeps for the entire "ay an" seems )ery "epresse". /e is also )ery irrita$le an" an.io#s o)er the co#rse of the ne.t "ay. She kno's of no other 'ith"ra'al symptoms. She $elie)es that he #ses the "r#g thro#gho#t the 'eek% $#t 'ith more intense $inges on the 'eeken"s. >n se)eral occasions% he has calle" her from the local psychiatric emergency center after a"mitting himself for s#ici"al i"eation after #se of the "r#g. =hich of the follo'ing "r#gs is most likely responsi$le for this man,s 'ith"ra'al statea6Alcohol $6BenDo"iaDepines c62anna$is "62ocaine e6/eroin :ormal La$s (.A 41!year!ol" man is "ischarge" after an #ncomplicate" myocar"ial infarction. Se)eral 'eeks later% he )isits his primary care physician complaining of insomnia% anore.ia% an" "epresse" moo". /e appears to $e clinically "epresse". =hich of the follo'ing 'o#l" $e the most appropriate me"ication to initiate for this patient-

a6Methylpheni"ate $6:ortriptyline c6PhenelDine "6Sertraline e6Thiori"aDine :ormal La$s 1.A <1!year!ol" man is a"mitte" to the hospital for "eliri#m associate" 'ith a #rinary tract infection. Upon a"e*#ate treatment of the infection% the patient,s mental stat#s impro)es significantly% tho#gh he is note" to remain partly "isoriente". /e also has an impairment in short!term memory% "iffic#lties in naming simple o$9ects% an" impaire" concentration. /is family mem$ers confirm an 1!month history of gra"#al progressi)e "ecline in cogniti)e a$ilities% 'hich they attri$#te to ol"!age. /o'e)er% the man is no longer a$le to manage his finances an" has gotten lost 'hile "ri)ing to the grocery store on t'o occasions. Prior to "ischarge from the hospital% the n#rsing staff reports that the patient contin#es to ha)e #rinary incontinence% tho#gh his infection has resol)e". /e is also note" to ha)e a )ery #nstea"y gait% re*#iring assistance 'hen 'alking. :o other o$)io#s signs or symptoms are present. =hich "isor"er most likely acco#nts for this patient,s "ementiaa62re#tDfel"t!Rako$ "isease $6/#ntington "isease c6:ormal press#re hy"rocephal#s "6Parkinson "isease e6Pick "isease :ormal La$s 8.A &(!year!ol" man 'ith chronic paranoi" schiDophrenia is $eing inter)ie'e" $y a 4th!year me"ical st#"ent. +n the mi""le of "escri$ing ho' he feels that other people are o#t to kill him% the patient s#""enly stops talking an" $egins staring into space. Appro.imately 24 secon"s later% he starts talking a$o#t ho' his mother 'as not a )ery goo" parent an" 'as e.cessi)e in her "iscipline "#ring his chil"hoo". =hich of the follo'ing terms $est "escri$es this $eha)iora6Echolalia $6Echopra.ia

c6M#tism "6Satyriasis e6 Tho#ght $locking :ormal La$s 14.A &2 year!ol" male 'ith a history of panic "isor"er 'ho 'orks as a salesman presents to his psychiatrist after se)eral 'eeks of treatment 'ith paro.etine. =hile he has note" significant impro)ement in his symptoms% he still notes resi"#al an.iety 'hen p#t into social sit#ations in 'hich he has to speak in p#$lic or $ecome the center of attention. Ci)en that he ha" some impro)ement on his c#rrent me"ication regimen% 'hich of the follo'ing 'o#l" $e the most appropriate ne.t step in managementa62ogniti)e!$eha)ior therapy $6Electrocon)#lsi)e therapy c6Me"ication change "6Psychoanalysis e6Seeing the patient more often for s#pporti)e psychotherapy :ormal La$s 11. A 28!year!ol" 'oman presents to her ne' primary care physician re*#esting me"ication to help 'ith her recent spells of an.iety an" "epression. She 'ishes to $e starte" on a me"ication that 'ill not ca#se too m#ch se"ation. E.amination of the patient,s ol" recor"s re)eals t'o pre)io#s s#ici"e attempts $y o)er"ose% once 'ith acetaminophen at age 1< an" another% 'ith aspirin% "#ring college. Ci)en this history% 'hich of the follo'ing anti"epressant me"ications 'o#l" most likely $e contrain"icate" for this patienta6 B#spirone $65l#o.etine c6:ortriptyline "6Paro.etine e6Sertraline :ormal La$s 12.A (1!year!ol" 'oman is complaining to the "octor in her n#rsing home a$o#t her ne' pro$lems. She reports that her ol" $oyfrien" from

4 years ago calle" her an" is no' harassing her $y controlling her $loo" press#re% mo)ements% an" tho#ghts. She is con)ince" that he 'as initially "oing this thro#gh the phone $y sen"ing special signals 'ith a "e)ice that the 2+A #ses% $#t no' he is a$le to control her thro#gh his tho#ghts. She state" that she co#l"n,t tell the police $eca#se they pro$a$ly 'o#l" not $elie)e her. This patient most likely has 'hich of the follo'ing tho#ght "isor"ersa6 2lang associations $6+"eas of infl#ence c6+"eas of reference "6:oesis e6>$sessions :ormal La$s 1&.A 'oman in her forties is referre" to a psychiatrist $eca#se of "epression. @#ring the initial e)al#ation sessions% she states that she al'ays ha" to 'ork har"% an" make many sacrifices all her life to achie)e goals. She says she lo)es helping others% $#t people al'ays seem to 0make their pro$lems her pro$lems0. She 'as in an a$#si)e marriage 'ith an alcoholic h#s$an" $#t finally "i)orce" him. She recently ha" another relationship 'ith a man 'ho left her after she helpe" him monetarily. =hich of the follo'ing personality traits "oes this person most likely ha)ea6@epen"ent $6/istrionic c6Masochistic "6:arcissistic e6>$sessi)e :ormal La$s 14. A <4!year!ol" man is #n"ergoing chemotherapy an" has occasional na#sea an" )omiting% 'hich are treate" 'ith +? prochlorperaDine. After se)eral "ays of therapy% the patient complains that he feels )ery restless an" agitate" an" he cannot stop mo)ing his legs. =hich of the follo'ing me"ications% if gi)en "#ring his initial regimen% might ha)e $est pre)ente" this reactiona62hlorpromaDine $6/aloperi"ol c6LoraDepam

"6Paro.etine e6Sertraline :ormal La$s 1 . A 2!year!ol" man is $eing e)al#ate" in the emergency "epartment for a recent fall an" $lo' to his hea". A 2T scan is normal% an" his in9#ries in)ol)e only a small laceration o)er the right eye. >n e)al#ation of his la$oratory )al#es% it is "isco)ere" that he has an ele)ate" mean corp#sc#lar erythrocyte )ol#me. This la$oratory fin"ing most likely s#ggests chronic a$#se of 'hich of the follo'ing s#$stancesa6Alcohol $62ocaine c6/eroin "6Mari9#ana e6:icotine :ormal La$s 1<.A patient 'ith a long history of $ipolar "isor"er% mi.e" type% 'as recently starte" on an anticon)#lsant that he 'as tol" 'o#l" help 'ith his moo" s'ings. /e ha" $een on it for 'eeks 'hen he "e)elope" skin changes that looke" like $#rns. /e 'as r#she" to the hospital an" "iagnose" 'ith Ste)ens!Rohnson syn"rome. =hich of the follo'ing me"ications "i" this patient most likely takea62ar$amaDepine $62lonaDepam c6Ca$apentin "6Lamotrigine e6 ?alproate :ormal La$s 1(. A gran"mother $rings her 1&!year!ol" gran"son% 'ho is in her c#sto"y% to a psychiatrist. /e has $een talking $ack to his teachers an" not complying 'ith school r#les. =hen aske" to stop% he gets angry an" $lames others% #s#ally his classmates. +n a""ition% his gra"es are "eteriorating. The gran"mother has $een str#ggling 'ith him in the past year an" thinks he nee"s 0str#ct#re an" control.0 The $oy says she "oesn,t let him "o anything% an" he resents ha)ing to li)e 'ith her.

Psychological testing sho's no in"ications of a learning "isor"er. =hich of the follo'ing is the most likely "iagnosisa6A"9#stment "isor"er 'ith "ist#r$ance of con"#ct $6Attention "eficit7hyperacti)ity "isor"er c62on"#ct "isor"er "62ornelia "e Lange syn"rome e6>ppositional "efiant "isor"er :ormal La$s 11.A 41!year!ol" man 'ith a long history of schiDophrenia% paranoi" type% has $een on the inpatient #nit for almost 2 'eeks. This is his thir" a"mission in the past < months% an" each time he seems to $e less responsi)e to treatment. +n the past% he has $een on typical antipsychotics an" then trie" on "ifferent com$inations% incl#"ing some atypical ones% 'ith limite" s#ccess. /is family is s#pporti)e an" makes s#re he takes his me"ication. Ci)en his most recent poor response% 'hich of the follo'ing is the most appropriate ne.t step in treatmenta62loDapine $6@roperi"ol c6Electrocon)#lsi)e treatment "6Lamotrigine e6Topiramate :ormal La$s 18.A patient is talking to his psychiatrist a$o#t a conflict he has 'ith his partner. They arg#e $eca#se the partner complains that the patient is inefficient an" procrastinates "oing things that are his responsi$ility. /e ne)er completes a task the 'ay in 'hich he 'as aske" to "o it% $#t in his o'n 'ay an" #s#ally m#ch later. =hich of the follo'ing types of "efense mechanism is this patient e.hi$itinga6Acting o#t $6Blocking c6Passi)e!aggressi)eness "6Aegression e6Splitting :ormal La$s

24.A 12!year!ol" $oy is $ro#ght to his pe"iatrician "#e to failing gra"es in school. The $oy faile" last year an" is repeating the si.th gra"e. /is parents ha)e s#specte" for se)eral years that the $oy may ha)e lo' intelligence "#e to repeate" "iffic#lties in school. The family is referre" to a psychologist for psychological testing% incl#"ing an assessment of intelligence. =hich of the follo'ing tests 'o#l" $e most appropriate for e)al#ating the $oy,s intelligence an" to assist 'ith possi$le placement in special classes in his schoola6Millon 2linical M#ltia.ial +n)entory $6Minnesota M#ltiphasic Personality +n)entory c6Aorschach Test "6Stanfor"!Binet Test e6Thematic Apperception Test :ormal La$s 21. A &<!year!ol" 'oman comes to a psychiatrist for an initial appointment after relocating from her home se)eral states a'ay "#e to a transfer of her 9o$. She is on lithi#m for $ipolar "isor"er% an" has $een sta$le for se)eral years after t'o manic episo"es in her early &4,s. She 'as pre)io#sly seen $y a psychiatrist e)ery t'o months an" ha" $loo" "ra'n for ro#tine monitoring relate" to her lithi#m treatment e)ery si. months. Since it has $een a$o#t eight months since her last appointment% the psychiatrist "eci"es to or"er la$!'ork an" to see the patient $ack in one 'eek to finish gathering history% to re)ie' the la$ res#lts% an" to pro)i"e the patient 'ith another prescription. =hich of the follo'ing la$oratory st#"ies 'o#l" $e most appropriate for the physician to or"era6Li)er transaminases $6Platelet co#nt c6Thyroi"!stim#lating hormone "6Urinalysis e6?itamin B12 le)el :ormal La$s 22. A yo#ng 'oman comes to her primary care physician "escri$ing e.treme irrita$ility an" conflicts 'ith her $oyfrien" a 'eek or so $efore her perio". At times% she also $ecomes tearf#l an" "epresse" 'itho#t a reason an" cannot foc#s on 'hat she is "oing. She notice" that she starts

o)ereating or cra)ing chocolate an" s'eets. This #se" to happen occasionally% $#t it has happene" each month for the past 4 months. The symptoms resol)e 'ith menses. =hich of the follo'ing is the most likely "iagnosisa6 A"9#stment "isor"er 'ith "epresse" moo" $62yclothymic "isor"er c6@epressi)e personality "isor"er "6Ma9or "epressi)e "isor"er 'ith atypical feat#res e6 Premenstr#al "ysphoric "isor"er :ormal La$s 2&.An intern is e.amining a patient 'ho 'as $ro#ght $y his family to the emergency "epartment for co#ghing% fe)er% na#sea% an" generaliDe" 'eakness. The patient has $een "iagnose" an" treate" in the past for schiDophrenia% "escri$e" $y the family as 0the type 'ith negati)e symptoms.0 =hich of the follo'ing symptoms 'o#l" this patient most likely ha)ea6Apathy $6A#"i$le tho#ghts c6@el#sions "6/all#cinations e6Tho#ght $roa"casting :ormal La$s 24. A 2(!year!ol" s#ccessf#l $#siness'oman has "e)elope" a fear of flying after an e.tremely ro#gh lan"ing. She is paralyDe" 'ith fear an" #na$le to tra)el for $#siness. /er physician trie" gi)ing her loraDepam to take "#ring the flight% $#t it "i"n,t help. =hich of the follo'ing is the most commonly #se" treatment for this "isor"era6E.pos#re therapy $6/ypnosis c6+nsight!oriente" psychotherapy "6Me"ication e6S#pporti)e therapy :ormal La$s 2 . An 11!year!ol" 'oman presents to her "octor along 'ith her mother% 'ho states that she "oesn,t kno' 'hat to "o 'ith her "a#ghter. E)er since the "a#ghter starte" mo"eling% she $ecame concerne" a$o#t

$eing fat. >)er the past < months% she has lost 2 po#n"s. The "a#ghter states that she has a fear of gaining 'eight. She ha" not ha" her menstr#al perio" for months. She "enies #se of la.ati)es or "i#retics. >n e.amination% she is thin an" has fine hair all o)er her $o"y. /er 'eight is a$o#t 24B $elo' her e.pecte" $o"y 'eight. /er $loo" press#re is 847(4 mm /g% an" her p#lse is 47min an" reg#lar. =hich of the follo'ing is the most appropriate ne.t step in managementa6 A"mit her to the hospital to reesta$lish 'eight an" correct a$normalities $6Aefer the patient for psycho"ynamic psychotherapy c6Sen" the patient for e)al#ation for electrocon)#lsi)e treatment "6 Start the patient on hormone therapy to in"#ce her menstr#al cycle e6Tell the mother not to 'orry an" sen" them home :ormal La$s 2<. A & !year!ol" 'oman comes to the physician for a perio"ic health maintenance e.amination. She has no physical complaints% ho'e)er she is concerne" that she har"ly e)er feels happy. She says that she has $asically $een 0"o'n0 for at least & years. She rarely goes o#t 'ith frien"s an" $asically keeps to herself at 'ork. She states that her 'ork performance has $een sta$le% $#t she #s#ally feels tire" an" 0$lah0. =hich of the follo'ing is the most likely "iagnosisa6 Antisocial personality "isor"er $62yclothymic "isor"er c6@epression "6@ysthymic "isor"er e6SchiDoaffecti)e "isor"er :ormal La$s 2(.A &1!year!ol" 'oman presents to her primary care physician 'ith complaints of fatig#e an" insomnia. Ae)ie' of systems re)eals no me"ical symptoms% $#t the patient "oes en"orse se)eral symptoms on the psychiatric re)ie' of systems. She "escri$es feeling tire" most of the "ay% ha)ing "iffic#lty falling asleep as 'ell as 'aking #p too early% poor appetite% fre*#ent crying spells% poor concentration% an" a recent loss of interest in her ho$$ies. Physical e.amination an" ro#tine la$oratory st#"ies incl#"ing thyroi" stim#lating hormone% are 'ithin normal

limits. =hich of the follo'ing is the most appropriate initial pharmacological inter)entiona6AlpraDolam $6B#spirone c6 +mipramine "6Paro.etine e6PhenelDine :ormal La$s 21. An emergency room intern is attempting to e)al#ate a &2!year!ol" 2a#casian male patient 'ith a long history of schiDophrenia% "isorganiDe" type. +n o$taining the history% the physician asks the patient to "escri$e any a#"itory hall#cinations he hears an" the patient respon"s $y saying 0killin% chillin% an" thrillin.0 =hich of the follo'ing most acc#rately "escri$es this type of responsea62langing $65light of i"eas c6Looseness of associations "6:eologism e6Tho#ght $locking :ormal La$s 28.A patient is $ro#ght to a psychiatrist after $eing pers#a"e" $y her family to get help. She "escri$es that she tho#ght she 'as "epresse". She feels empty an" g#ilty for not ha)ing any feelings for her chil"ren. She notices that mornings are the 'orst. Later in the "ay% she is a$le to "o some things. +n the mornings% ho'e)er% she stays in $e" late an"% e)en tho#gh she has ne)er $een laDy% she is #na$le to "o anything. She falls asleep *#ickly at night% $#t 'akes #p at 4 AM an" is #na$le to get $ack to sleep. She is s#re there isn,t anything that co#l" cheer her #p% e)en for a short time. =hich of the follo'ing is the most likely "iagnosisa62atatonia $62otar",s syn"rome c6@ysthymia "6Ma9or "epression 'ith atypical feat#res e6 Ma9or "epression 'ith melancholic feat#res :ormal La$s

&4.Three ho#rs after an #ne)entf#l appen"ectomy% a pre)io#sly healthy 3e.cept for appen"icitis6 (1!year!ol" man $ecomes "isoriente" an" conf#se". /e repeate"ly asks the n#rses 'here he is% an" his speech pattern is "isorganiDe" an" ram$ling. /is temperat#re is &( 2 381.< 56% $loo" press#re is 124714 mm /g% p#lse is (47min% an" respirations are 117min. The patient is #ncooperati)e% $#t sho's no physical a$normalities. Mental stat#s e.amination is not possi$le $eca#se of an altere" le)el of conscio#sness. La$oratory st#"ies sho' no a$normalities. =hich of the follo'ing is the most likely "iagnosisa6A"9#stment "isor"er $6 Brief psychotic "isor"er c6@eliri#m "6@el#sional "isor"er e6@ementia :ormal La$s &1. A 4!year!ol" 'oman presents to her primary care physician for her ro#tine yearly health maintenance e.amination. She "enies any ne' pro$lems. /er only significant me"ical history incl#"es a 2 !year history of schiDophrenia% 'hich has $een 'ell!controlle" 'ith antipsychotic agents% an" a !year history of hypertension for 'hich she takes a "i#retic. ?ital signs an" physical e.amination are 'ithin normal limits. /o'e)er% the patient is note" to ha)e occasional irreg#lar p#ckering an" lip!smacking mo)ements. She "enies ha)ing notice" the a$normal mo)ements% an" her speech is normal. =hich of the follo'ing is the most likely "iagnosisa6Ac#te "ystonia $6Akathisia c6Pse#"oparkinsonism "6Tar"i)e "yskinesia e6Tar"i)e "ystonia :ormal La$s &2.A <2!year!ol" 'oman is $ro#ght to the emergency "epartment $y her "a#ghter% 'ho is concerne" her mother ha" a 0stroke.0 Accor"ing to the "a#ghter% the 'oman 'oke #p that morning not kno'ing 'here she 'as. She 'as conf#se" an" "i" not remem$er 'hat ha" happene" the

"ay $efore. She recently sa' her primary care physician% 'ho starte" her on a ne' me"ication. The 'ork#p for stroke is negati)e% an" the physician s#spects that the episo"e is relate" to the ne' me"ication. =hich of the follo'ing me"ications 'o#l" most likely ca#se cogniti)e impairmenta6 AlpraDolam $6B#propion c6Selegiline "6Sertraline e6?itamin E :ormal La$s &&. A &2!year!ol" African American male 'ith a history of chronic paranoi" schiDophrenia is $ro#ght to his psychiatrist after failing co#rses of treatment 'ith $oth haloperi"ol an" olanDapine. /is psychiatrist 'o#l" like to start him on cloDapine. =hich of the follo'ing parameters sho#l" the psychiatrist monitor caref#lly in this patienta6Bloo" press#re $6Platelet co#nt c6P#lse "6Aespiratory rate e6=hite $loo" cell co#nt :ormal La$s &4.A &(!year!ol" female 'ith a history of $ipolar "isor"er presents to her physician for a me"ication check. She has $een taking the same "ose of lithi#m car$onate "aily for o)er & years. She is in generally goo" health% "oes not smoke% an" has not ha" a manic episo"e for se)eral years. Ci)en this patient,s history% the )al#es of 'hich of the follo'ing ro#tine la$oratory tests is most likely to ha)e remaine" the same o)er timea6Ser#m $loo" #rea nitrogen le)el 3BU:6 $6Ser#m creatinine le)el c6Ser#m lithi#m le)el "6Thyroi" stim#lating hormone le)el 3TS/6 e6=hite $loo" cell co#nt :ormal La$s

& .A < !year!ol" 'oman presents to a primary care physician 'ith a complaint of poor memory. /er h#s$an" "ie" si. months ago an" she has recently "ecrease" her ch#rch in)ol)ement an" interest in her ho$$ies. She complains of feeling tire" an" 'orrie"% an" has "iffic#lty 'ith attention an" memory. She has a history of a ma9or "epressi)e episo"e appro.imately ten years ago% 'hich re*#ire" hospitaliDation. She 'as treate" 'ith an anti"epressant for t'o years #ntil it 'as tapere" an" "iscontin#e". She c#rrently "enies "epresse" moo". She states that her mother "ie" of AlDheimer,s "isease% an" that she fin"s her memory loss partic#larly 'orrisome in light of this family history. =hich of the follo'ing factors 'o#l" $e more likely if this patient has a pse#"o"ementia of "epression rather than a tr#e "ementiaa6The patient appears #nconcerne" $6The patient emphasiDes "isa$ility relate" to memory loss c6The patient has more "iffic#lty 'ith recent memory than remote memory "6The patient ten"s to conceal the "iffic#lty e6The patient tries har" to ans'er *#estions a$o#t orientation an" registration :ormal La$s &<. A 41!year!ol" o$ese 'oman 'ith a history of psychotic "epression is $ro#ght to the psychiatric emergency "epartment follo'ing a s#ici"e threat. She is "iffic#lt to inter)ie' $eca#se she feels tire" an" sleepy an" states she is on a contin#o#s positi)e air'ay press#re 32PAP6 machine at home secon"ary to ha)ing o$str#cti)e sleep apnea. She reports that she has $een on ProDac an" other me"ications% 'hich she got for 0the )oices.0 @#ring the inter)ie'% the psychiatrist learns she has gaine" 24 po#n"s in the past se)eral months. =hich of the follo'ing me"ications most likely ca#se" this significant 'eight gaina6Amo.apine $6LoraDepam c6 Molin"one "6>lanDapine e6Aisperi"one :ormal La$s

&(.A 42!year!ol" man 'ho is /+?!positi)e is $eing treate" for his first episo"e of Pne#mocystis carinii pne#monia on an inpatient me"ical 'ar" 'hen he $egins to $ecome increasingly "isoriente"% com$ati)e% an" agitate". P#lse o.imetry sho's that he is o.ygenating at 8&B on room air. /e is c#rrently recei)ing nortriptyline for "epression an" ga$apentin for pain "#e to peripheral ne#ropathy. /e has a history of alcohol a$#se for the past se)eral years $efore hospitaliDation an" a remote history of opioi" a$#se. =hich of the follo'ing is the most likely "iagnosisa6Alcohol 'ith"ra'al $6/+? "ementia c6/ypo.emic encephalopathy "6Ma9or "epressi)e "isor"er e6M#ltifactorial "eliri#m :ormal La$s &1.A & !year!ol" 'oman is in long!term% 'eekly% insight!oriente" psychotherapy. She has $een in psychotherapy for one an" a half years% an" has e.plore" iss#es relate" to her "epressi)e symptoms an" her ten"ency to #se physical attracti)eness to get close to men!e)en those for 'hom she has no romantic interest. She is also e.ploring her past relationships 'ith her parents% an" the past importance to her family of her s#ccess in $ea#ty pageants in high school an" college. @#ring the co#rse of psychotherapy% s#$tle se"#cti)e ten"encies ha)e arisen to'ar" the psychotherapist% 'hich often mask her "esire for him to fin" her accepta$le an" interesting. >ne "ay she comes to her psychotherapy session 'itho#t 'earing any make!#p. Thro#gho#t the session% the patient has a sense of "epression an" )ie's herself as ina"e*#ate an" "isappointing to the therapist. The therapist makes the follo'ing comment% 0+ notice that on a "ay 'itho#t yo#r make!#p% yo# fear that + 'ill fin" yo# ina"e*#ate% pro$a$ly m#ch like yo# percei)e" yo#r father,s "isappointment if yo# "i"n,t 'in the $ea#ty contest. +t seems "iffic#lt for yo# to feel accepte" if yo# "o not feel attracti)e. 0 =hich psychotherape#tic techni*#e $est "escri$es the therapist,s commenta62larification $62onfrontation c6Empathic )ali"ation "6Aesistance interpretation e6 Transference interpretation

:ormal La$s

&8.A 8!year!ol" African American 'oman% 'ho has $een /+? positi)e for the past 12 years% is $ro#ght to the emergency "epartment follo'ing an episo"e of com$ati)eness at home. /er family reports that% o)er the past co#ple of "ays prior to a"mission% she has $een talking 0o#t of her hea"%0 stating that her "a#ghter!in!la' ha" $een p#tting poison into the foo". She ref#ses to eat or take me"ication in the hospital% stating that the physicians are also against her. /er last 2@4 co#nt% "one < months prior to this a"mission% 'as 1 7mm&. Physical an" ne#rologic e.aminations are #nremarka$le an" nonfocal. The c#rrent 'ork#p for /+?!relate" infection is negati)e% an" la$oratory tests are other'ise #nremarka$le. >n 5olstein,s Mini Mental Stat#s E.amination she scores 217&4% missing on "elaye" recall% &!stage comman"% 'riting% an" copying% as 'ell as orientation to year. =hich of the follo'ing is the most likely "iagnosisa6 Amok $6@eliri#m c6@el#sional "isor"er "6@ementia e6SchiDophrenia :ormal La$s 44.A me"ical st#"ent is inter)ie'ing a (1!year!ol" patient $ro#ght from the n#rsing home to the emergency "epartment. /e has $een talking to him almost &4 min#tes an" gotten )ery little information. E)ery time he asks a *#estion% the patient starts talking an" goes into #nnecessary "etails% e)ent#ally ans'ering the *#estion $#t only after he ha" tol" his story to the me"ical st#"ent. This is an e.ample of 'hich of the follo'inga62irc#mstantiality $6Clossolalia c6Logorrhea "6Perse)eration e6Tangentiality :ormal La$s

41.A &4!year!ol" male presents to his physician for a refill of his antipsychotic me"ication. The patient has $een taking his me"ication for years% after se)eral hospitaliDations for $iDarre $eha)iors associate" 'ith psychosis. /e has $een "iagnose" 'ith schiDophrenia% "isorganiDe" type% an" has chronic refractory mil"er psychotic symptoms that are e)i"ent only #pon engaging him in con)ersation. =hich of the follo'ing mental stat#s e.amination fin"ings is most likely presenta6@ecrease" f#n" of kno'le"ge $6@isorientation to month c6+mpaire" short!term memory "6Loosening of associations e6Pho$ias :ormal La$s 42.A &<!year!ol" 'oman comes to her gynecologist $eca#se of a three! month history of amenorrhea. Until this time% her menstr#al perio"s ha" $een reg#lar. She also complains of "ecrease" se. "ri)e% 'orsening o)er the past co#ple of months. The patient "enies any other symptoms. She has no significant me"ical history% altho#gh she starte" seeing a psychiatrist fi)e months ago after a $rief hospitaliDation "#ring 'hich she 'as "iagnose" 'ith ma9or "epressi)e "isor"er% se)ere% 'ith psychotic feat#res. /er "epressi)e symptoms are resol)ing. =hich of the follo'ing me"ications is most likely responsi$le for the patient,s presenting complaints "#ring her )isit to her gynecologista65l#o.etine $6N#etiapine c6Aisperi"one "6TraDo"one e6?enlafa.ine :ormal La$s

E.planations Block 1< E.planations

:) (*!lanation: 7he correct answer is %. -luvo*amine is an ))6I that has been a!!rove" for the treatment of obsessive4 com!ulsive "isor"er 'OC%). Its efficacy has been establishe" in several :>4week trials. us!irone 'choice A) is an nonben#o"ia#e!ine antian*iety a$ent. It is in"icate" for an*iety "isor"ers an" relief of tension associate" with every"ay life. It is not a!!rove" for the treatment of OC %. Clona#e!am 'choice ) is a lon$4actin$ ben#o"ia#e!ine that can be use" to treat an*iety if it is associate" with OC %. 2owever, it is not a!!rove" for the treatment of OC %. -enfluramine 'choice C) is an am!hetamine con$ener that was use" in the !ast for wei$ht loss !ur!oses. It is not a!!rove" for the treatment of OC %. 6is!eri"one 'choice () belon$s to the $rou! of aty!ical anti!sychotics. It can be use" in schi#o!hrenia an" other !sychotic "isor"ers with obsessive4com!ulsive sym!toms. It is not a!!rove" for the treatment of OC %. ;) (*!lanation: 7he correct answer is A. 7he +ini4+ental )tatus (*amination '++)() is a brief instrument "esi$ne" to $rossly assess co$nitive functionin$. It assesses orientation, memory, calculation, rea"in$ an" writin$ ca!acity, visuos!atial ability, an" lan$ua$e. 7he ma*imum score is 0>. 7his tool is wi"ely use" as a sim!le, quick assessment of !ossible co$nitive "eficits. 7he ++)( can $ive the clinician a cru"e measurement of co$nitive "ecline, an" the tool can be re4a"ministere" !erio"ically to follow !ro$ression of "ementia. ,aboratory stu"ies 'choice ) are an im!ortant !art of the work4u! for "ementia, an" shoul" be or"ere" after a full history is $athere" an" co$nition is assesse" with the ++) (. Im!ortant laboratory stu"ies in the work4u! of "ementia inclu"e basic chemistries, com!lete bloo" counts, liver, renal, an" thyroi" function stu"ies, sy!hilis serolo$y, serum folate an" vitamin :; levels, erythrocyte se"imentation rate, an" 2I1 screenin$. Other stu"ies may be inclu"e" if in"icate" by history an" !hysical e*amination. Neuro!sycholo$ical testin$ 'choice C) may be beneficial in "ifferentiatin$ "ementia from other neuro!sychiatric syn"romes, for assessin$ the "e$ree of functional im!airment, an" for assessin$ s!ecific areas of si$nificant co$nitive "ecline. Neuroima$in$ stu"ies, such

as a C7 scan 'choice %) may be obtaine" as !art of the work4u! for "ementia, es!ecially to rule4out reversible causes of "ementia, such as intracranial mass lesions, recurrent cerebrovascular acci"ents, an" normal !ressure hy"roce!halus. A lumbar !uncture 'choice () may be in"icate" if serum sy!hilis testin$ is reactive or in the evaluation of normal !ressure hy"roce!halus. 0) (*!lanation: 7he correct answer is (. 7his !atient has a social !hobia '!erformance an*iety). )ocial !hobia is a fear of social or !erformance situations when the in"ivi"ual may be e*!ose" to unfamiliar in"ivi"uals or may be evaluate" by others. It is a common "isor"er that affects men an" women. It im!airs every"ay activities an" relationshi!s. lushin$, shakin$, hy!ertension, an" tachycar"ia occur when the in"ivi"ual is e*!ose" to the feare" situation. eta4blockers are effective in !reventin$ the sym!toms. A$ora!hobia 'choice A) is an irrational fear of !laces where esca!e may be "ifficult. It lea"s to avoi"ance of these situations an" often causes the in"ivi"ual to be homeboun". )ystematic "esensiti#ation may be hel!ful. .enerali#e" an*iety "isor"er '.A%G choice ) is characteri#e" by !ersistent intense worry over many events an" activities that is !resent for at least C months. 7he in"ivi"ual fin"s it "ifficult to control the worry. 7hree of the followin$ sym!toms must be !resent: restlessness, "ifficulty concentratin$, irritability, muscle tension, slee! "isturbance, an" easy fati$ability. It usually be$ins before a$e ;>. 7reatment inclu"es ben#o"ia#e!ines, bus!irone, an" !sychothera!eutic interventions. &anic "isor"er 'choice C) is characteri#e" by un!re"ictable !anic attacks that inclu"e !al!itations, tremblin$, sweatin$, "i##iness, "ys!nea, chest !ain, an" the fear of losin$ control or "yin$. 7ricyclic anti"e!ressants, selective serotonin reu!take inhibitors, ben#o"ia#e!ines, an" !sychothera!eutic interventions are hel!ful. &osttraumatic stress "isor"er '&7)%G choice %) is an an*iety "isor"er characteri#e" by e*!osure to a traumatic event, !ersistent re4 e*!eriencin$ of the event, avoi"ance of stimuli associate" with the event, an" increase" arousal. It im!airs im!ortant activities an" relationshi!s. &sychothera!y an" me"ications are "irecte" at the in"ivi"ualAs s!ecific sym!toms. ?) (*!lanation: 7he correct answer is C. Con"uct "isor"er is "efine" as a re!etitive !attern of behavior in which the basic ri$hts of others, or ma<or a$e4a!!ro!riate societal rules or norms are violate", often with accom!anyin$ a$$ression to !eo!le an" animals, "estruction of !ro!erty, "eceitfulness, or theft. 7his "ia$nosis is usually $iven to in"ivi"uals youn$er than :@ years. Antisocial !ersonality "isor"er 'choice A) is not "ia$nose" before a"ulthoo" an" consists of many of the same criteria that are use" to "ia$nose con"uct "isor"er. It is also usually accom!anie" by reckless "isre$ar" for the safety of self or others. Attention "eficitBhy!eractivity "isor"er 'choice ) is a "ia$nosis requirin$ inattention an" hy!eractivity before the a$e of D, makin$ it an unlikely "ia$nosis in this case.

5le!tomania 'choice %) is the recurrent failure to resist im!ulses to steal ob<ects that are not nee"e" for !ersonal use or for their monetary value. 7he stealin$ is not committe" to e*!ress an$er or ven$eance an" is not in res!onse to a "elusion or hallucination. &yromania 'choice () is the "eliberate an" !ur!oseful settin$ of fire on more than one occasion. 7his "ia$nosis requires that the fire settin$ is not better accounte" for by con"uct "isor"er, or antisocial !ersonality "isor"er. /) (*!lanation: 7he correct answer is (. O!!ositional "efiant "isor"er is a !attern of ne$ativistic, hostile, or "efiant behavior lastin$ at least C months, characteri#e" by losin$ oneAs tem!er, ar$uin$ with a"ults, an" the "eliberate annoyance of !eo!le or blamin$ of others for mistakes or misbehavior. It occurs in the absence of con"uct "isor"er 'choice ), which is characteri#e" by the breakin$ of a$e4a!!ro!riate social norms, with associate" "estruction of !ro!erty, !hysical violence, an" theft. Attention "eficitBhy!eractivity "isor"er 'A%2%) 'choice A) is characteri#e" by attention "eficit an" restlessness. It is an unlikely new "ia$nosis in a :/4year4ol" who has !reviously "one well in school. %ysthymic "isor"er 'choice C) is a "isor"er of "e!resse" moo", more often than not, over the course of at least ; years. +ental retar"ation 'choice %) is a "isor"er requirin$ !sycholo$ical testin$ to make a "ia$nosis. It is not su!!orte" by this !atientAs !revious aca"emic recor". C) (*!lanation: 7he correct answer is %. 7he use of cocaine, es!ecially crack cocaine 'an e*tremely !otent, freebase form that is sol" in small, rea"y4to4smoke amounts) is associate" with the with"rawal sym!toms "escribe" in this case. After cessation of the use of cocaine, or after acute into*ication, a !ost4into*ication "e!ression 'crash) often occurs. 7his "e!ression is associate" with "ys!horia, anhe"onia, an*iety, irritability, fati$ue, hy!ersomnolence, intense cravin$, an" suici"al i"eation. Alcohol 'choice A) is associate" with a !otentially "an$erous set of with"rawal !henomena. 7hese ran$e from mil" with"rawal 'mil" tremulousness) to a full with"rawal syn"rome 'tremulousness, "ia!horesis, hy!errefle*ia, elevate" vital si$ns, an" an*iety) to the very "an$erous with"rawal state known as "elirium tremens 'altere" mental status, hallucinations, autonomic instability, sei#ures, an" !ossibly "eath). Alcohol with"rawal shoul" be treate" in a me"ical settin$ with close observation an" a "eto*ification re$imen. en#o"ia#e!ines 'choice ) can cause "e!en"ence an" a wi"e ran$e of with"rawal !henomena very similar to those of alcohol with"rawal. en#o"ia#e!ine with"rawal shoul" also be mana$e" with me"ical su!ervision. Cannabis 'choice C) may be associate" with a with"rawal state inclu"in$ irritability, insomnia, increase" or "ecrease" a!!etite, restlessness, tremor, increase" refle*es, an" chan$es in vital si$ns. 7he with"rawal !henomena of cannabis are less well "efine" than many other "ru$s, an" "o not lea" to "an$erous me"ical sequelae. 2eroin 'choice () "e!en"ence lea"s to a characteristic with"rawal syn"rome that is very uncomfortable. )ym!toms

inclu"e myal$ias, nausea, vomitin$, "iarrhea, lacrimation, rhinorrhea, !iloerection, an*iety, insomnia, irritability, an" intense cravin$. D) (*!lanation: 7he correct answer is %. In a !atient with car"iac com!lications, a selective serotonin reu!take inhibitor '))6I), such as sertraline, has been "emonstrate" to be the safest an" most effective me"ication for the treatment of clinical "e!ression. +ethyl!heni"ate 'choice A) is a stimulant me"ication that woul" have the !otential to increase heart rate an" sym!athetic tone, which woul" not be a"vanta$eous in the !ost4myocar"ial infarction !erio". Nortri!tyline 'choice ) is a tricyclic anti"e!ressant that has the !otential for !ro"ucin$ car"iac arrhythmias in over"ose. &henel#ine 'choice C) is a monoamine o*i"ase inhibitor, which woul" be contrain"icate" with a history of recent myocar"ial infarction, as the in$estion of tyramine with an +AO inhibitor coul" in"uce a hy!ertensive crisis. 7hiori"a#ine 'choice () is an low4!otency "o!amine anta$onist anti!sychotic me"ication that is not in"icate" for the treatment of "e!ression. @) (*!lanation: 7he correct answer is C. Normal !ressure hy"roce!halus is a !otentially reversible cause of "ementia that causes $ait "isturbances 'unstea"y or shufflin$ $ait), urinary incontinence, an" "ementia. (nlar$ement of the ventricles with increase" cerebros!inal flui" 'C)-) !ressure is foun", an" thera!eutic lumbar !unctures may si$nificantly im!rove sym!toms. Creut#fel"t4 Nakob "isease 'choice A) is a rare "iffuse "e$enerative "isease that usually affects !eo!le in their />s, an" the usual course is about one year. 7he terminal sta$e is characteri#e" by severe "ementia, $enerali#e" hy!ertonicity, an" !rofoun" s!eech "isturbances. It is one of several "iseases !resumably cause" by !rions. 2untin$ton "isease 'choice ) is a here"itary "isease associate" with !ro$ressive "e$eneration of the basal $an$lia an" the cerebral corte*. It is transmitte" in an autosomal "ominant !attern. 7he onset of 2untin$ton "isease occurs between 0/ an" /> years of a$e, or later in rare cases. 7his "isease is characteri#e" by !ro$ressive "ementia, muscular hy!ertonicity, an" bi#arre choreiform movements. &arkinson "isease 'choice %) is characteri#e" !rimarily by motor "ysfunction, but "ementia may be a !art of the "isor"er. 7he characteristic motor sym!toms 'bra"ykinesia, flat facies, restin$ tremor, shufflin$ $ait, etc) are cause" by "e$eneration of the ni$rostriatal "o!aminer$ic tract. &ick "isease 'choice () causes a slowly !ro$ressive "ementia. It is associate" with focal cortical lesions, !rimarily of the frontal lobe. &atholo$ical e*amination of the brain reveals intraneuronal inclusions calle" &ick bo"ies. I) (*!lanation:

7he correct answer is (. 7hou$ht blockin$ is the su""en re!ression of an*iety4!rovokin$ thou$hts in mi"4 sentence. It is a common fin"in$ in many in"ivi"uals with schi#o!hrenia. Often, when the !atient be$ins s!eakin$ a$ain, it is concernin$ somethin$ unrelate" to the to!ic before the !ause in conversation an" thou$ht. (cholalia 'choice A) is the !sycho!atholo$ic re!etition of wor"s or !hrases of one !erson by another, an" ten"s to be re!etitive an" !ersistent. (cho!ra*ia 'choice ) is the !atholo$ic imitation of movements of one !erson by another. +utism 'choice C) is !sycho!atholo$ic voicelessness of a !atient over an e*ten"e" !erio" of time, in the absence of !honetic structural abnormalities. )atyriasis 'choice %) is the e*cessive an" com!ulsive nee" for coitus in a man. Its corollary in a female is terme" nym!homania. :>) (*!lanation: 7he correct answer is A. In many cases of !anic "isor"er, effective treatment involves the use of co$nitive behavior thera!y, which incor!orates e*!osin$ the !atient to "isturbin$ stimuli in an attem!t to "evelo! co!in$ mechanisms in res!onse to the stimuli. (lectroconvulsive thera!y 'choice ) is not in"icate" for use in !anic "isor"er. +e"ication chan$e 'choice C) is not in"icate" when the current re$imen lea"s to si$nificant sym!tom relief, as the amount of time necessary to achieve a"equate res!onse on a new me"ication "oes not <ustify a me"ication switch. &sychoanalysis 'choice %) is a lon$ term, time consumin$ !rocess that is not in"icate" to treat the acute resi"ual an*iety of this !atientAs !anic "isor"er. )eein$ the !atient more often for su!!ortive !sychothera!y 'choice () woul" not be as effective a treatment as co$nitive behavioral thera!y, accor"in$ to research on !anic "isor"er. ::) (*!lanation: 7he correct answer is C. Nortri!tyline is a tricyclic anti"e!ressant, which is lethal in over"ose an" is the lea"in$ cause of over"ose4relate" "eaths in the !sychiatric !o!ulation. Over"ose is associate" with !rolon$ation of the 87 interval, lea"in$ to car"iac arrhythmia an" "eath. .iven this !atientAs history of !revious over"oses, a tricyclic anti"e!ressant woul" be contrain"icate". us!irone 'choice A) is an anti"e!ressant me"ication whose !harmacolo$ic action is not well un"erstoo", but which is not known to be lethal in hi$h "oses. -luo*etine 'choice ), !aro*etine 'choice %), an" sertraline 'choice () are all selective serotoner$ic reu!take inhibitors '))6Is). 7hese anti"e!ressant me"ications are $enerally safe an" effective an" are not lethal e*ce!t in e*tremely hi$h "oses. :;) (*!lanation:

7he correct answer is . I"eas of influence constitute a ty!e of "elusion in which a !erson believes that he or she is bein$ controlle" by another !erson or e*ternal force. Clan$ associations 'choice A) are "isor"ers of thou$ht in which the associations of wor"s are similar in soun" but not in meanin$. Wor"s have no lo$ical connection, but there may be rhymin$. I"eas of reference 'choice C) are "elusions in which a !erson has a false belief that others 'inclu"in$ !eo!le on 71 or ra"io) are talkin$ about him or her. In a broa"er sense, the behavior of others refers to oneself, other !ersons, or ob<ects that have s!ecial si$nificance an" meanin$. Noesis 'choice %) refers to the feelin$ of revelation in which a !erson e*!eriences illumination associate" with a sense of bein$ chosen as a lea"er. Obsessions 'choice () are !atholo$ically !ersistent intrusive thou$hts or im!ulses that cannot be eliminate" from consciousness by lo$ical effort an" thus cause an*iety. 7he !erson is aware that they are not im!ose" from the outsi"e but are a !ro"uct of his or her own min". :0) (*!lanation: 7he correct answer is C. +asochistic traits are frequently seen in the hos!ital settin$, as well as out!atient treatment !ro$rams. (ven thou$h the sa"omasochistic !ersonality "isor"er has been e*clu"e" from the %)+ I1 classification, it is frequently seen in every"ay !ractice. 7he traits inclu"e lon$ sufferin$, com!lainin$, an" a self4"e!recatory character with an e*cessive "eman" for love. %e!en"ent traits 'choice A) are seen in !ersons who avoi" !ositions of res!onsibility or "ecision makin$ because of self4"oubt. 7hey are "e!en"ent on others an" !refer to have a submissive !osition. 2istrionic traits 'choice ) are seen in !ersons who "is!lay se"uctive, flirtatious, an" attention4seekin$ behavior. 7hese !ersons ten" to be su!erficial, vain, an" self4absorbe". Narcissistic traits 'choice %) are seen in !ersons with a sense of self4$ran"iosity an" self4im!ortance. 7here is a !ronounce" sensitivity to criticism an" low self4esteem. 7he sense of entitlement an" the lack of em!athy for others are the most strikin$ features. Obsessive traits 'choice () are seen in !ersons who are controllin$, ri$i", in"ecisive, stin$y, an" "istrustful. 7he traits themselves "o not necessarily constitute a "isor"er. :?) (*!lanation: 7he correct answer is C. 7his !atient has evi"ence of akathisia, the feelin$ of restlessness that sometimes occurs as a reaction to use of neurole!tic me"ications, such as !rochlor!era#ine an" halo!eri"ol. &revention of akathisia is most reliably maintaine" by use of a ben#o"ia#e!ine me"ication, such as lora#e!am $iven I1 "urin$ a"ministration of the neurole!tic "ru$. 7his is !articularly im!ortant in a !atient whose immune system an" metabolic ca!acity may show si$ns of com!romise. Chlor!roma#ine 'choice A) an" halo!eri"ol 'choice ) are both neurole!tic me"ications that woul" e*acerbate akathisia rather than !revent it. &aro*etine 'choice %) an" sertraline 'choice () are both

anti"e!ressant me"ications that are s!ecific serotoner$ic reu!take inhibitors '))6Is). 7hese a$ents have no effect on the !revention of neurole!tic4in"uce" akathisia. :/) (*!lanation: 7he correct answer is A. (levate" mean cor!uscular erythrocyte volume '+C1) is associate" with the chronic abuse of alcohol, which can lea" to a "eficiency in both folate an" thiamine in the "iet, lea"in$ to a me$aloblastic anemia. Cocaine 'choice ), heroin 'choice C), mari<uana 'choice %), an" nicotine 'choice () are all substances of abuse, but none of them are su$$este" by an elevate" +C1 on a laboratory fin"in$. :C) (*!lanation: 7he correct answer is %. ,amotri$ine is a!!rove" as an a"<unct in the treatment of refractory sei#ures. Its effectiveness in bi!olar "isor"er has been "emonstrate" in o!en trials. 7he most serious si"e effect of lamotri$ine is rash. When the "ose is titrate" u! too quickly, the !otentially fatal )tevens4Nohnson syn"rome can occur. Carbama#e!ine 'choice A) is an anticonvulsant that is also effective in the treatment of acute mania in !atients with ra!i" cyclin$ or mi*e" bi!olar "isor"er. Its most serious si"e effects inclu"e a!lastic anemia an" a$ranulocytosis, as well as liver failure. Clona#e!am 'choice ) is a hi$h4!otency ben#o"ia#e!ine a!!rove" for the treatment of sei#ures. It is use" as an a"<unct in the treatment of manic !atients or those with an*iety "isor"ers. 7he most common si"e effects inclu"e se"ation an" ata*ia, as well as the !otential for "e!en"ency. .aba!entin 'choice C) is a!!rove" as an a"<unct in the treatment of !artial an" $enerali#e" sei#ures. It has been also use" to treat bi!olar "isor"ers, !ain syn"romes, an" an*iety "isor"ers. 7he most common si"e effects are somnolence, "ry mouth, "i##iness, an" ata*ia. 1al!roate 'choice () is an anticonvulsant that is also effective in the treatment of acute mania, mi*e" states, an" ra!i" cyclin$ bi!olar "isor"er. Its most serious si"e effects inclu"e he!atoto*icity an" thrombocyto!enia. In !re$nancy, it can cause neural tube "efects. It can also lea" to !olycystic ovarian "isease. :D) (*!lanation: 7he correct answer is (. O!!ositional "efiant "isor"er belon$s to the $rou! of "isru!tive behaviors characteri#e" by a !attern of ne$ativistic hostile behavior lastin$ at least C months an" inclu"in$ four or more of the followin$: ar$uin$ with others, blamin$ others for own mistakes, bein$ an$ry an" resentful, bein$ vin"ictive, refusin$ to com!ly with a"ultsA requests, an" bein$ easily annoye" by others. 7he "isor"er is not "ue to a moo" or !sychotic "isor"er. It causes si$nificant im!airment in functionin$. A"<ustment "isor"er with "isturbance in con"uct 'choice A) re!resents a mala"a!tive res!onse to a stressor

within 0 months of an e*!osure an" "oes not meet the criteria for o!!ositional "isor"er. Attention "eficitBhy!eractivity "isor"er 'choice ) inclu"es either inattention, !resentin$ with si* or more associate" sym!toms for C months, or si* of the hy!eractivity4 im!ulsivity sym!toms that are inconsistent with "evelo!mental level. Con"uct "isor"er 'choice C) is a re!etitive !attern of behavior in which the basic ri$hts of others or social rules are violate". 7he criteria inclu"e a$$ression to animals, "estruction of !ro!erty, violation of rules, "eceitfulness, an" theft. 7hree of the criteria nee" to be !resent at least for :; months, an" at least one nee"s to be !resent in the !ast C months, to make this "ia$nosis. Cornelia "e ,an$e syn"rome 'choice %) is characteri#e" by mental retar"ation, short stature, continuous eyebrows, thin "ownturnin$ li!, microce!haly, small an" malforme" han"s, an" hirsutism. :@) (*!lanation: 7he correct answer is A. Clo#a!ine is an aty!ical a$ent that has been !roven to be more effective than conventional anti!sychotics in the treatment of !atients with resistant schi#o!hrenia. It e*erts an anta$onistic effect on %: an" %? rece!tors, as well as on the al!ha4a"rener$ic, histaminer$ic, serotoner$ic, an" choliner$ic systems. It is 0>= effective in the treatment of !atients with resistant schi#o!hrenia in the first C weeks. 2owever, it !ro"uces troublesome si"e effects. 7he most serious one is a$ranulocytosis. Clo#a!ine requires re$ular bloo" count monitorin$ because of this serious but rare si"e effect. %ro!eri"ol 'choice ) is a butyro!henone that has been a!!rove" as a"<unctive treatment with anesthetics. It can be use" in emer$ency settin$s because of its stron$ se"ative !otential an" !arenteral form. It is not a!!rove" for the treatment of resistant schi#o!hrenia. (lectroconvulsive treatment 'choice C) is in"icate" in treatment of schi#o!hrenia only after me"ication treatment faile". ,amotri$ine 'choice %) is an anticonvulsant a!!rove" as a"<unctive treatment for refractory !artial sei#ures. It has been use" lately in treatin$ bi!olar "isor"er an" !ain "isor"ers, but these instances have been re!orte" only in o!en trials an" case re!orts. 7o!iramate 'choice () belon$s to the $rou! of anticonvulsants a!!rove" as a"<unctive in the treatment of !artial e!ile!sy in a"ults. It has not been a!!rove" for treatment of schi#o!hrenia, but there have been anec"otal re!orts of its use as a moo" stabili#er. :I) (*!lanation: 7he correct answer is C. With !assive4a$$ressiveness, a$$ression towar" others is e*!resse" in"irectly, usually throu$h !rocrastination, stubbornness, !assivity, an" for$etfulness. 7he manifestations usually affect others more than the !erson who is "oin$ it. Actin$ out 'choice A) is a "efense by which a !erson e*!resses an unconscious wish throu$h action to avoi" bein$ conscious of the stron$ accom!anyin$ affect. lockin$ 'choice ) is a transient inhibition of thinkin$ that usually involves stron$ affects an" im!ulses that are bein$ inhibite". 6e$ression 'choice %) is a "efense mechanism in which a !erson returns to an earlier

sta$e of libi"inal "evelo!ment to avoi" tension at the !resent level of "evelo!ment. )!littin$ 'choice () is a "efense mechanism by which e*ternal ob<ects are "ivi"e" into Hall $oo"H an" Hall ba".H It is accom!anie" by abru!t shiftin$ of an ob<ect from one cate$ory to the other. ;>) (*!lanation: 7he correct answer is %. 7he )tanfor"4 inet 7est is a com!rehensive intelli$ence test that is use" in !sychiatry an" e"ucation. Another wi"ely use" intelli$ence test is the Wechsler A"ult Intelli$ence )cale. )uch tests are routine intelli$ence tests in the assessment of bor"erline intellectual functionin$, mental retar"ation, an" s!ecific learnin$ "isor"ers. 7he +illon Clinical +ultia*ial Inventory 'choice A) is a :D/4item, true4false !ersonality inventory. 7his test woul" not be a!!ro!riate for assessin$ intelli$ence. 7he +innesota +ulti!hasic &ersonality Inventory 'choice ) contains over />> true4false items, with :D scales inclu"in$ most ma<or areas of !sycho!atholo$y. 7his !ersonality test !rovi"es a wi"e ran$e of "ata on numerous !ersonality variables an" is su!!orte" by a stron$ research base. 7he 6orschach test 'choice C) contains a stan"ar" set of :> inkblots that serve as stimuli for associations. -ive of the car"s are in black an" white, an" the other five inclu"e colors. A recor" of the !atientAs verbatim res!onse, alon$ with initial reaction times an" total time s!ent on each car" allows for inter!retation. 7his test is referre" to as a !ro<ective test. 7he 7hematic A!!erce!tion 7est 'choice () is also a !ro<ective test that consists of a series of 0> !ictures an" one blank car". After lookin$ at the car", the !atient is instructe" to create a story. 7his test is more useful as a technique for inferrin$ motivational as!ects of behavior than as a basis for makin$ a "ia$nosis. It woul" not be a!!ro!riate for assessin$ intelli$ence. ;:) (*!lanation: 7he correct answer is C. ,ithium affects thyroi" function an" thyroi"4stimulatin$ hormone '7)2) levels shoul" be monitore" every C4:; months. ,ithium often causes a $enerally beni$n an" often transient "ecrease in the concentration of thyroi" hormones. About 0>= of !atients receivin$ lon$4term treatment with lithium will have elevate" 7)2 levels. If sym!toms of hy!othyroi"ism emer$e, treatment with levothyro*ine may be in"icate". %urin$ routine monitorin$ of lithium maintenance treatment, lithium !lasma concentration an" serum creatinine shoul" also be measure" !erio"ically. ,iver transaminases 'choice A) are not affecte" by lithium treatment. ,ithium is almost entirely eliminate" by the ki"neys. %urin$ treatment with val!roate, another moo" stabili#er, liver transaminases are monitore". A si$nificant !ortion of !atients treate" with val!roate will have an elevation in liver transaminases, which is usually asym!tomatic an" resolves with "iscontinuation of the "ru$. &latelet count 'choice ) is not affecte" by lithium treatment. 2owever, treatment with val!roate may cause thrombocyto!enia an" !latelet "ysfunction. &latelet count is usually monitore" "urin$ treatment with val!roate. Frinalysis 'choice %) woul"

not be hel!ful in the routine monitorin$ of maintenance treatment with lithium. ,ithium may cause !olyuria an" "iabetes insi!i"us, which shoul" be monitore" clinically. When !olyuria is si$nificant, renal function shoul" be evaluate" an" followe" u! with a ;?4hour urine collection for creatinine clearance an" with consultation with a ne!hrolo$ist. 1itamin :;'choice () is often or"ere" in the evaluation of "ementia, !sychosis, an" moo" "isor"ers. It woul" not be a routine !art of lithium maintenance monitorin$. ;;) (*!lanation: 7he correct answer is (. &remenstrual "ys!horic "isor"er is a constellation of !hysical an" emotional sym!toms occurrin$ "urin$ the late luteal !hase of the menstrual cycle. 7he sym!toms must be !resent "urin$ most cycles in the !ast year an" "urin$ at least two subsequent cycles. &atients have at least five of the followin$ sym!toms: "e!resse" moo", marke" an*iety, affective lability, "ecrease" interest, "ecrease" ener$y, slee! "isturbance, cravin$ foo", feelin$ overwhelme", an" "ifficulties concentratin$. 7he sym!toms interfere with social or occu!ational functionin$ an" are not "ue to other !sychiatric or me"ical "isor"ers. A"<ustment "isor"er with "e!resse" moo" 'choice A) involves as a criterion a ma<or stressor to which an in"ivi"ual is reactin$ within 0 months after the e*!osure. It also involves the continuous !resence of sym!toms that "o not resolve with the onset of follicular !hase of the cycle. Cyclothymic "isor"er 'choice ) requires the !resence of numerous e!iso"es of hy!omanic an" "e!ressive sym!toms that "o not meet the criteria for ma<or affective "isor"er. 7he !erson must have at least ; continuous months of these sym!toms in a ;4year !erio". 7he "isor"er is not "ue to another !sychiatric "isor"er or me"ical con"ition. %e!ressive !ersonality "isor"er 'choice C) is characteri#e" by !essimism, anhe"onia, low self4esteem, an" moralistic self4"eni$ratin$ features. +a<or "e!ressive "isor"er with aty!ical features 'choice %) is characteri#e" by moo" reactivity, si$nificant wei$ht $ain an" increase" a!!etite, hy!ersomnia, lea"en !aralysis or heavy feelin$ in arms an" le$s, an" a lon$4stan"in$ !attern of sensitivity to inter!ersonal re<ection. Other criteria for ma<or "e!ressive "isor"er, inclu"in$ a ;4week !erio" of "e!resse" moo" an" anhe"onia in the !ast month an" si$nificant im!airment in functionin$, must be met to establish the "ia$nosis. ;0) (*!lanation: 7he correct answer is A. Ne$ative sym!toms of schi#o!hrenia are "ia$nose" in so4calle" ty!e II schi#o!hrenia, with the !re"ominant !resence of flattene" affect, !overty of s!eech or s!eech content, thou$ht blockin$, !oor $roomin$, lack of motivation, anhe"onia, social with"rawal, co$nitive "eficits, an" attentional "eficits. Au"ible thou$hts 'choice ) belon$ to the core sym!toms of schi#o!hrenia !er (mil 5rae!elinAs, as well as 5urt )chnei"erAs criteria, which inclu"e them in first4rank sym!toms. 7he !atient has a feelin$ that his or her own thou$hts are lou"er or are s!oken by voices insi"e the !atientAs hea". %elusions 'choice C) are !ositive sym!toms an" are false beliefs firmly maintaine" even thou$h

contra"icte" by reality. 7hey are most common in schi#o!hrenia. 2allucinations 'choice %) are e*am!les of !ositive or !ro"uctive sym!toms of schi#o!hrenia. 7hese are sensory !erce!tions for which there is no e*ternal stimulus. 7hey can be of any sensory mo"alityG however, in schi#o!hrenia au"itory hallucinations are the most common ones. 7hou$ht broa"castin$ 'choice () is a sense that other !eo!le can rea" a !atientAs thou$hts, or vice versa, an" is ty!ically seen in schi#o!hrenia. 7his is !art of the loss of e$o boun"aries seen in !atients with schi#o!hrenia. 7hou$ht broa"castin$ is an e*am!le of the )chnei"erian first4rank sym!toms, which are !ro"uctive sym!toms in the other classification. ;?) (*!lanation: 7he correct answer is A. (*!osure thera!y, a ty!e of behavior thera!y, is the most commonly use" treatment of s!ecific !hobia. 7he thera!ist usually "esensiti#es the !atient by a $ra"ual e*!osure to the !hobic stimulus. 6ela*ation an" breathin$ control are im!ortant !arts of the treatment. 2y!nosis 'choice ) is use" to enhance the thera!istAs su$$estions that the !hobic ob<ect is not "an$erous. At times, self4hy!nosis can be tau$ht so that the !atient uses it as a metho" of rela*ation when confronte" with the !hobic stimulus. Insi$ht4oriente" !sychothera!y 'choice C) was initially use" to treat !hobias, but analy#in$ unconscious conflicts "i"nAt resolve !hobic sym!toms. It "oes hel! the !atient un"erstan" the ori$ins of the !hobia an" how to "eal with an*iety4!rovokin$ stimuli. +e"ication 'choice %) is use" in the treatment of a s!ecific !hobia only if it is associate" with !anic attacks an" $enerali#e" an*iety. 7he !harmacolo$ic treatment is then "irecte" towar" the !anic attacks. )u!!ortive thera!y 'choice () may be use" in hel!in$ the !atient actively confront the !hobic stimulus "urin$ treatment. It is usually use" in a""ition to an on$oin$ treatment.

;/) (*!lanation: 7he correct answer is A. 7his !atient has lost a si$nificant amount of wei$ht in a short !erio" of time. As in"icate" by her vital si$ns an" !ercenta$e of wei$ht loss, she may nee" a full evaluation of her !hysical status to "etermine an" correct abnormalities because her refusal to eat has !lace" her in a !otentially life4en"an$erin$ situation. (ven thou$h the !atient may have !sycholo$ical issues, !sycho"ynamic !sychothera!y 'choice ) has not !roven to be the thera!y of choice for anore*ic !atients. (lectroconvulsive thera!y 'choice C) is not in"icate" for the treatment of anore*ia nervosa !er se, an" is certainly not a!!ro!riate in this case before all other thin$s are "one. )tartin$ hormone thera!y to in"uce her menstrual !erio" 'choice %) woul" be ina!!ro!riate since the evaluation an" treatment of

the un"erlyin$ cause of the con"ition has not been a""resse". 7ellin$ the mother not to worry an" sen"in$ the !atient home 'choice () woul" be a com!letely ina!!ro!riate an" ina"equate !rofessional assessment of the situation, as this !atient may be in a life4 threatenin$ con"ition. ;C) (*!lanation: 7he correct answer is %. 7his !atient has "ysthymia, which is a mil"er, chronic form of "e!ression. 7he "ia$nosis requires ; years of a "e!resse" moo". It may be associate" with chan$es in a!!etite, slee!, an" concentration, an" with fati$ue an" ho!elessness. 7reatment is !sychothera!y an" anti"e!ressant thera!y. Antisocial !ersonality "isor"er 'choice A) is a !attern of behavior that is characteri#e" by a com!lete "isre$ar" of the rules of society. 7hese in"ivi"uals lie, steal, an" en"an$er others. 7hey rarely feel remorse an" ty!ically en" u! in !rison. Cyclothymic "isor"er 'choice ) is characteri#e" by recurrent an" chronic e!iso"es of hy!omania an" "ysthymia. It is a less severe form of bi!olar "isease. %e!ression 'choice C) is characteri#e" by a "e!resse" moo", chan$es in slee!, ener$y, concentration, an" a!!etite. Other fin"in$s are $uilt, ho!elessness, anhe"onia, an" !sychomotor a$itation or retar"ation. A ma<or "e!ressive e!iso"e requires five or more of these sym!toms for at least ; weeks. 7here is severe "istress an" functional im!airment. It is more severe than "ysthymia. &atients with schi#oaffective "isor"er 'choice () have !sychotic e!iso"es with a moo" "isturbance. 7his !atient is not e*!eriencin$ !sychotic e!iso"es. ;D) (*!lanation: 7he correct answer is %. &aro*etine is a selective serotonin reu!take inhibitor '))6I). Alon$ with other newer anti"e!ressants, the ))6Is are currently the first4line a$ents for the treatment of "e!ression. 7his !atient has a sym!tom !rofile consistent with ma<or "e!ressive "isor"er, an" treatment shoul" be initiate" with a $oal of com!lete remission of sym!toms. ))6Is are safe in over"ose an" have a relatively mil" si"e effect !rofile. Al!ra#olam 'choice A) is a short4actin$ ben#o"ia#e!ine use" in the short4term treatment of an*iety, such as "urin$ initiation treatment for !anic "isor"er while an ))6I is bein$ starte". %ue to the si$nificant risk of "e!en"ence an" with"rawal, such short4actin$ ben#o"ia#e!ines shoul" be use" very s!arin$ly. us!irone 'choice ) is an an*iolytic a$ent that acts as an a$onist or !artial a$onist on serotonin :A '/427:A) rece!tors. When use" in the treatment of con"itions causin$ an*iety, the full clinical res!onse may take two to four weeks, without imme"iate an*iolytic effects. Imi!ramine 'choice C) is a tricyclic anti"e!ressant '7CA) that is efficacious in the treatment of "e!ression. 2owever, 7CAs are not currently first4 line a$ents "ue to their !otential lethality in over"ose, nee" for !re4treatment electrocar"io$ra!hic monitorin$, an" si$nificant si"e effect !rofile '"ue to antia"rener$ic, antihistaminic, an" antimuscarinic effects). &henel#ine 'choice () is a monoamine o*i"ase inhibitor '+AOI) anti"e!ressant a$ent. 7hese a$ents are rarely use" an" woul"

never be consi"ere" a first4line a$ent. &atients takin$ +AOIs '!henel#ine, tranylcy!romine, isocarbo*a#i") must follow a "iet that restricts tyramine "ue to the !otential for a hy!ertensive crisis when in$este" with tyramine4containin$ foo"s. &otential lethality in over"ose is also a concern. ;@) (*!lanation: 7he correct answer is A. When a !atient associates wor"s similar in soun" but not in meanin$, this is known as a clan$ association. In this case the wor"s have no lo$ical connections but sometimes rhyme or have similar !honetic !hrasin$. -li$ht of i"eas 'choice ) is a ra!i" verbali#ation of a constant shiftin$ of i"eas which ten" to be connecte"G it is often foun" in a !atient in a state of mania. ,ooseness of associations 'choice C) is the flow of thou$hts an" i"eas that shift from one to!ic to another in com!letely unrelate" waysG this is foun" most often in !aranoi" schi#o!hrenics who are not actively thou$ht blockin$. A neolo$ism 'choice %) is a new wor" create" by a !atient by combinin$ the syllables of other wor"s, an" is usually i"iosyncratic in its ori$in. 7hou$ht blockin$ 'choice () is the abru!t interru!tion of thou$ht before a thou$ht or i"ea is finishe" in bein$ e*!resse". Fsually it is in"icate" by the !atientAs not bein$ able to recall what was !reviously sai" or what was $oin$ to be sai", an" usually is accom!anie" by !auses in conversation. ;I) (*!lanation: 7he correct answer is (. +a<or "e!ression with melancholic features is characteri#e" by a "e!resse" moo" most of the time an" a lack of reactivity to !leasurable stimuli "urin$ e!iso"es. In a""ition, three or more of the followin$ criteria must be met: "istinct quality of "e!resse" moo", worse "e!ression in the mornin$, early mornin$ awakenin$, !sychomotor retar"ation or a$itation, wei$ht loss, an" ina!!ro!riate $uilt. Catatonia 'choice A) can be a!!lie" to any of the e!iso"es of ma<or "e!ressive or bi!olar "isor"er. In the clinical !icture, the most "ominant feature is motoric immobility, as evi"ence" by wa*y fle*ibility, e*cessive motor activity, e*treme ne$ativism, !eculiar voluntary movements, an" echolalia or echo!ra*ia. Cotar"As syn"rome 'choice ), or nihilistic "elusional "isor"er, is "ia$nose" when !atients com!lain of not only havin$ lost their !ossessions, status, or stren$th but also their inner or$ans. It can be seen in schi#o!hrenic or "e!ressive e!iso"es. %ysthymia 'choice C) involves "e!resse" moo" for most of the time in the !ast ; years an" the !resence of two or more of the followin$: chan$es in a!!etite, chan$es in slee!, fati$ue, low self4esteem, !oor concentration, an" feelin$s of ho!elessness. 7he sym!toms "o not meet the criteria for ma<or "e!ressive e!iso"e. +a<or "e!ression with aty!ical features 'choice %) can be a!!lie" when, in the most recent ; weeks, there is moo" reactivity 'the !atient bri$htens to !ositive events) an" two or more of the followin$ are !resent: wei$ht $ain or increase" a!!etite, hy!ersomnia, lea"en !aralysis, an" sensitivity to re<ection from others.

0>) (*!lanation: 7he correct answer is C. 7his !atient has "elirium, which is also calle" acute confusional state. It is very common in hos!itali#e" an" institutionali#e" el"erly in"ivi"uals. It is characteri#e" by a ra!i" onset of im!aire" co$nition, altere" level of consciousness, "isturbances in attention an" !sychomotor activity, an" altere" slee!4wake cycles. 7he sym!toms ten" to fluctuate an" it is usually reversible when the un"erlyin$ "isor"er is i"entifie" an" treate". Common causes inclu"e !sycholo$ic an" !hysical stress 'for e*am!le, sur$ery), metabolic "isturbances, neo!lasms, infections, me"ications, cerebral an" car"iovascular "iseases, an" with"rawal from alcohol an" !rescri!tion me"ications. A"<ustment "isor"er 'choice A) is a mala"a!tive res!onse to a stressful event. 7he sym!toms inclu"e moo" "isturbances, behavioral chan$es, an" im!aire" functionin$. It usually resolves within C months. rief !sychotic "isor"er 'choice ) is characteri#e" by the abru!t onset of !sychotic sym!toms, inclu"in$ hallucinations, "elusions, an" "isor$ani#ation with im!aire" functionin$. It is !resent for more than : "ay, but less than : month. It is usually !rece"e" by a stressful life event. %elusional "isor"er 'choice %) is characteri#e" by the !resence of nonbi#arre "elusions that last for more than : month. -unctionin$ is not usually im!aire". Anti!sychotic a$ents an" !sychothera!y may be necessary. %ementia 'choice () refers to the slow an" insi"ious onset of co$nitive an" intellectual "eficits with no chan$es in consciousness. 7he sym!toms are stable an" irreversible. 7he most common causes are Al#heimer "isease an" multi4infarct "ementia. 0:) (*!lanation: 7he correct answer is %. 7ar"ive "yskinesia '7%) is a syn"rome characteri#e" by abnormal choreiform an" athetoi" movements occurrin$ late in onset in relation to initiation of anti!sychotic treatment. 7% usually "evelo!s after months to years of treatment with anti!sychotic a$ents, an" is !resumably much less likely to occur with the use of aty!ical anti!sychotics. 6isk factors for 7% inclu"e ol"er a$e, lon$er "uration of anti!sychotic treatment, an" !resence of an affective "isor"er. 7he abnormal involuntary movements usually involve orofacial muscles but may inclu"e the trunk an" e*tremities. 6e$ular e*aminations shoul" be "one to ensure that !atients treate" with these me"ications "o not "evelo! si$ns of 7 %. Acute "ystonia 'choice A) is an abru!t reaction consistin$ of abnormal !ositionin$ or s!asm of the muscles of the hea", neck, limbs, or trunk. 7his form of e*tra!yrami"al si"e effect '(&)) associate" with anti!sychotic me"ications can be terminate" with a"ministration of an anticholiner$ic a$ent. Akathisia 'choice ) occurs in a!!ro*imately />= of !atients takin$ conventional anti!sychotic a$ents. 7his form of (&) consists of the e*!erience of a sub<ective feelin$ of restlessness. &atients may rock, !ace, ta!, or move restlessly while sittin$. 7his si"e effect may be confuse" with an*iety, a$itation, tar"ive "yskinesia, or worsenin$ of !sychosis. 7reatment inclu"es re"ucin$ the anti!sychotic "ose, switchin$ to another a$ent, an" the use of beta4blockers or

ben#o"ia#e!ines. &seu"o!arkinsonism 'choice C) is a form of (&) consistin$ of some or all of the followin$: bra"ykinesia, masklike facial e*!ression, slow s!eech, tremor, co$wheel ri$i"ity, stoo!e" !osture, an" shufflin$ $ait. 7ar"ive "ystonia 'choice () is a late4occurrin$ "ystonia that is a rare a"verse event associate" with anti!sychotic treatment. 0;) (*!lanation: 7he correct answer is A. Al!ra#olam belon$s to the $rou! of short4actin$ ben#o"ia#e!ines. (ven thou$h it has a short half4life, it can !ro"uce confusion, "isinhibition, an" amnestic !roblems like blackouts in the el"erly !o!ulation. 7he risk is increase" if it is combine" with C9&0A inhibitors. u!ro!ion 'choice ) is an anti"e!ressant with "o!aminer$ic an" nora"rener$ic !ro!erties an" minimal anticholiner$ic !ro!erties. Co$nitive functions are usually im!rove" an" not im!aire" with bu!ro!ion. )ele$iline 'choice C) is a selective +AO inhibitor that, when metaboli#e", has three active metabolites, inclu"in$ am!hetamine an" metham!hetamine, that stimulate the release of nore!ine!hrine an" "o!amine, thus im!rovin$ some co$nitive functions. )ertraline 'choice %) is an ))6I, but it also has some "o!aminer$ic !ro!erties an" is currently bein$ markete" as a !ro"uct to im!rove co$nitive functionin$. ))6Is is usually not associate" with co$nitive im!airments. 1itamin ( 'choice () has been use" in el"erly !o!ulation as a scaven$er of free o*y$en ra"icals to im!rove co$nitive functions. 7here are no re!orts of vitamin ( relate" co$nitive im!airment. 00) (*!lanation: 7he correct answer is (. Of all the in"icators liste", white bloo" cell count shoul" concern the !hysician most, as there is a "ocumente" risk of a$ranulocytosis associate" with the use of the anti!sychotic clo#a!ine. +ost !atients on clo#a!ine require weekly monitorin$ of the C C in or"er to !revent the risk of severe infection associate" with a$ranulocytosis. loo" !ressure 'choice A) shoul" be monitore", as there is some chance for orthostasis while on clo#a!ine, but such orthostatic chan$es are usually transient an" not life4 threatenin$. &latelet count 'choice ), !ulse 'choice C), an" res!iratory rate 'choice %) are not known to be a"versely affecte" by the use of clo#a!ine. 0?) (*!lanation: 7he correct answer is C. In $eneral, lithium reaches a stea"y state level in the bloo" stream once a "osa$e is maintaine" over a $iven !erio" of time, an" "oes not chan$e unless there are con"itions such as "ehy"ration that interfere with this equilibrium. ,ithium has been re!orte" to be associate" with !ossible ki"ney "ama$e with lon$4term use, which necessitates followin$

ki"ney function on a re$ular basis, inclu"in$ FN 'choice A) an" creatinine 'choice ) levels. ,ithium has a thyroi"4su!!ressin$ effect, so 7)2 levels 'choice %) must be checke" on a re$ular basis. ,ithium sometimes causes leukocytosis, so white bloo" cell counts must also be checke" re$ularly 'choice (). 0/) (*!lanation: 7he correct answer is . %istin$uishin$ a !seu"o"ementia of "e!ression from a true "ementia is an im!ortant task requirin$ e*amination of subtle as!ects of the mental status e*amination. Althou$h these in"ivi"ual factors may be use" in the assessment of "e!ression4relate" co$nitive "ysfunction, the entirety of the mental status e*amination must be consi"ere" when makin$ "ia$nostic "ecisions. 7reatment for the two con"itions woul" be very "ifferent, an" memory loss in this woman shoul" not be automatically !resume" to be relate" to a !rimary "ementia. In !seu"o"ementia of "e!ression, the !atient often ten"s to em!hasi#e "isability relate" to memory loss much more than !atients e*!eriencin$ true "ementia. If the !atient a!!ears unconcerne" 'choice A) about the memory loss, this woul" be more characteristic of "ementia. Fsually in !seu"o"ementia of "e!ression, the !atient communicates a stron$ sense of "istress. If the !atient has more "ifficulty with recent memory than remote memory 'choice C) true "ementia shoul" be consi"ere". In !seu"o"ementia of "e!ression, si$nificant memory loss for both recent an" remote events usually occurs. If the !atient ten"s to conceal the "ifficulty 'choice %), this woul" be more characteristic of "ementia. If the !atient tries har" to answer questions about orientation an" re$istration 'choice () "ementia shoul" be consi"ere", since !seu"o"ementia of "e!ression is often accom!anie" by little effort to !erform even sim!le tasks. 0C) (*!lanation: 7he correct answer is %. Olan#a!ine is an aty!ical anti!sychotic in"icate" for the mana$ement of !sychotic "isor"ers. In !lacebo4controlle", C4week stu"ies, ;I= of olan#a!ine4treate" !atients $aine" $reater than D= of their baseline wei$ht com!are" with 0= of !lacebo4treate" !atients. Amon$ !atients on lon$4term treatment with olan#a!ine 'more than ;0@ "ays of e*!osure) /C= $aine" more than D= of baseline wei$ht. Amo*a!ine 'choice A) is a !otent anti!sychotic "ru$ with anti"e!ressant activity. Its si"e effects are relate" to "o!aminer$ic !ostsyna!tic blocka"e, as well as nore!ine!hrine selectivity. Wei$ht $ain is similar to other com!arable anti!sychotics. ecause of newer anti!sychotics an" anti"e!ressants with more tolerable si"e4effect !rofiles, amo*a!ine is now use" less often. ,ora#e!am 'choice ) is a short4actin$ ben#o"ia#e!ine an" is not in"icate" for the treatment of !sychotic sym!toms. It can be use" for se"ation, as well as for an*iolytic !ur!oses. +olin"one 'choice C) is structurally unrelate" to other anti!sychotics an" has a rather favorable !rofile. It is not wi"ely use". )tu"ies su$$est that it is si$nificantly less associate" with wei$ht $ain com!are" with other anti!sychotics. Wei$ht loss is

sometimes rather common with its use. 6is!eri"one 'choice () is an aty!ical anti!sychotic in"icate" for the treatment of !sychotic "isor"ers. In !lacebo4controlle" trials, the !ro!ortion of !atients with an increase of more than D= of wei$ht has been about :@= com!are" with I= of !lacebo4treate" !atients. 0D) (*!lanation: 7he correct answer is (. In a !atient such as this with a me"ical illness, a history of the acute onset of a$itation, "isorientation, an" confusion is consistent with an" most su$$estive of the "ia$nosis of "elirium. Alcohol with"rawal 'choice A) is somethin$ to consi"er, but this !atient has a history of alcohol abuse, not "e!en"ence, in the "istant !ast. 7herefore, this "ia$nosis is less likely. 2I1 "ementia 'choice ) is not likely, as this is the !atientAs first o!!ortunistic infection, an" most !atients with 2I1 "ementia are well into the course of their illness before the "ementia !resents. 2y!o*emia 'choice C) is often a cause of "elirium. In this !atient, however, his alteration in orientation an" level of consciousness make "elirium the more a!!ro!riate "ia$nosis. +a<or "e!ressive "isor"er 'choice %) is e*clu"e" as a "ia$nosis when me"ical com!lications, such as "elirium, limit the ability to accurately assess alterations in moo". 0@) (*!lanation: 7he correct answer is (. An inter!retation involves makin$ somethin$ conscious that was !reviously unconscious, such as the connection between the !atientAs sense of "e!ression an" her lack of make4u! as well as the correlation between her relationshi! styles with her father an" with her thera!ist. An inter!retation is an e*!lanatory statement that links a feelin$, thou$ht, behavior, or sym!tom to its unconscious meanin$. In this case, the inter!retation "irectly "eals with the transference 'the !atientAs feelin$s an" behavior towar" the thera!ist that are base" on earlier wishes with im!ortant fi$ures). Clarification 'choice A) involves a reformulation of what the !atient has e*!resse" in or"er to convey a coherent view of what is bein$ communicate". A clarification can hel! the !atient articulate somethin$ that is "ifficult to verbali#e. Confrontation 'choice ) a""resses somethin$ that the !atient "oes not want to acce!t. 7he !atientAs avoi"ance or minimi#ation of a "enie" or su!!resse" feelin$ is i"entifie". (m!athic vali"ation 'choice C) "emonstrates the thera!istAs attunement with the !atientAs internal state to show the !atient that he or she is un"erstoo" on an emotional level. 7his technique is more often use" in a su!!ortive ty!e of !sychothera!y than in an insi$ht4oriente" a!!roach. 6esistance inter!retation 'choice %) is an inter!retation that "irectly "eals with the !atientAs unconscious use of resistance to avoi" !ainful or conflictual issues. 0I) (*!lanation:

7he correct answer is %. 2I1 "ementia is characteri#e" by affective, co$nitive, behavioral, an" motor sym!toms an" si$ns. It !resents as a subcortical !rocess an" is most likely to occur in !atients with a C%? count below ;>>Bmm0. It usually has a slow onset, an", after a !erio" of stability, there can be a !reci!itous "ecline. 7he "ia$nosis is ma"e when other causes of "elirium are e*clu"e"G the "isease may !resent with !sychosis within 2I1 "ementia. 7he sym!toms are controlle" with low "oses of neurole!tics. Amok 'choice A) is a culture4boun" syn"rome. It is seen amon$ +alayans an" !resents as un!rovoke" outbursts of wil" ra$e, causin$ the !erson to run ma"ly an" attack others or commit suici"e. 7here is a !erio" of amnesia afterwar". %elirium 'choice ) is the most common cause of mental status chan$es in !atients with 2I1BAI%). It may be relate" to the !rimary effects of 2I1 infection in the CN), the secon"ary effects of systemic 2I1 "isease, or si"e effects of me"ications. It may also be a result of other usual causes of "elirium 'e.$., en"ocrine, metabolic, sei#ures, trauma, neo!lasms). 7y!ically, the clinical !icture inclu"es a wa*in$ an" wanin$ level of consciousness an" fluctuations in co$nitive functions, as seen in "elirium associate" with other causes. %elusional "isor"er 'choice C) is "efine" as a !sychotic "isor"er with !ersistent nonbi#arre "elusions that are firmly hel" an" may be of !ersecution, somatic nature, $ran"eur, or <ealousy, for e*am!le. )chi#o!hrenia 'choice () is a !sychosis with the onset of sym!toms ty!ically in youn$ a"ulthoo". &atients with schi#o!hrenia can "evelo! "ementia an", if they are 2I1 !ositive, also "evelo! sym!toms relate" to 2I1. 7y!ical schi#o!hrenic sym!toms inclu"e "elusions an" hallucinations of at least : monthAs "uration an" a si$nificant "uration of illness an" im!airment of more than C months. ?>) (*!lanation: 7he correct answer is A. Circumstantiality refers to s!eech that is "elaye" from reachin$ the !oint, characteri#e" by overinclusion of "etails. (ventually, it "oes $et to the ori$inal $oal. .lossolalia 'choice ), also known as s!eakin$ in ton$ues, refers to the e*!ression of a revelatory messa$e throu$h unintelli$ible wor"s. It may be consi"ere" normal an" !art of some reli$ious !ractices. ,o$orrhea 'choice C) is a "isturbance of s!eech an" is characteri#e" by to co!ious, coherent s!eech. &erseveration 'choice %) is manifeste" as a !ersistin$ res!onse to a !rior stimulus even thou$h a new stimulus has been !resente". It can be verbal or motor, an" is seen in co$nitive "isor"ers. 7an$entiality 'choice () is the inability to !rovi"e a $oal4"irecte" association to the $iven thou$ht. 7he !atient never $ets to the "esire" $oal from the startin$ !oint of "iscussion. ?:) (*!lanation: 7he correct answer is %. &atients with "isor$ani#e" ty!e schi#o!hrenia are likely to e*hibit "isor$ani#e" s!eech, "isor$ani#e" behavior, an" flat or ina!!ro!riate affect. (*am!les of "isor$ani#e" thou$hts an" s!eech inclu"e: loosenin$ of associations or "erailment, fli$ht of i"eas,

tan$entiality, circumstantiality, wor" sala", neolo$isms, an" clan$ associations. )uch "isor"ere" thou$ht !rocesses are not always evi"ent "urin$ close"4en"e" questionin$, but frequently become more obvious with o!en4en"e" questions an" "urin$ unstructure" conversation. %ecrease" fun" of knowle"$e 'choice A) is not a usual characteristic of schi#o!hrenia. Althou$h !atients with schi#o!hrenia may have "ifficulty learnin$ "ue to intrusion of !sychotic sym!toms, an" may have subtle co$nitive im!airments later in the course of the illness, intellectual im!airment is not an inherent sym!tom of the "isease. %isorientation to month 'choice ) is not a usual characteristic of schi#o!hrenia. Any !atient with "isorientation must be further assesse" for "elirium. Chronic "isorientation in ol"er !atients may su$$est "ementia. Im!aire" short4term memory 'choice C) is not a usual feature of schi#o!hrenia. +emory "ifficulties shoul" alert the !hysician to assess for co$nitive "isor"ers inclu"in$ amnesia, "elirium, an" "ementia. &hobias 'choice () are !ersistent, irrational, e*a$$erate", an" !atholo$ical fears of some s!ecific stimulus or situation, resultin$ in a com!ellin$ "esire to avoi" the feare" stimulus. A !hobia is not a feature of schi#o!hrenia. %e!en"in$ on the feare" an" avoi"e" stimulus, !atients with !hobias may be "ia$nose" with a s!ecific !hobia or social !hobia. ?;) (*!lanation: 7he correct answer is C. 6is!eri"one is an aty!ical anti!sychotic a$ent. Conventional anti!sychotics are clearly associate" with elevations in !lasma !rolactin concentrations "ue to blocka"e in the tuberoinfun"ibular "o!aminer$ic !athway. %o!amine bin"s to !ituitary lactotro!hs to inhibit the release of !rolactin. Conventional anti!sychotics block "o!amine rece!tors, which releases this inhibition. 7he newer aty!ical anti!sychotics have minimal, if any effect on !lasma !rolactin concentrations, e*ce!t for ris!eri"one, which is associate" with elevate" !rolactin. Anti!sychotic4in"uce" hy!er!rolactinemia may cause si"e effects inclu"in$ amenorrhea an" infertility, se*ual "ysfunction, $alactorrhea, an" wei$ht $ain. .iven this !atientAs non4contributory me"ical history an" lack of other sym!toms, the onset of her com!laints correlates with the initiation of treatment for !sychotic "e!ression. A mornin$ !lasma !rolactin level shoul" be obtaine". -luo*etine 'choice A) is a selective serotonin reu!take inhibitor '))6I). It "oes not cause si$nificant elevations in serum !rolactin. Althou$h ))6Is are commonly associate" with se*ual si"e effects, inclu"in$ "iminishe" libi"o, but es!ecially "elaye" or$asm, they are not associate" with amenorrhea. 8uetia!ine 'choice ) is an aty!ical anti!sychotic a$ent that is not associate" with si$nificant or !ersistent elevations in !lasma !rolactin concentrations, unlike ris!eri"one. 7ra#o"one 'choice %) is an ol"er anti"e!ressant a$ent that is commonly use" for insomnia associate" with "e!ression. Althou$h it may cause se*ual si"e effects, it "oes not interfere with menstruation. 1enlafa*ine 'choice () is a serotonin an" nore!ine!hrine reu!take inhibitor that may cause se*ual si"e effects, but "oes not cause amenorrhea.

USMLE Step 2 Practice Test Block 1(

1.A 24!year!ol" female comes to the physician $eca#se she has ne)er ha" a perio". She has no me"ical pro$lems% has ne)er ha" s#rgery% an" takes no me"ications. E.amination sho's that she is a tall female 'ith long e.tremities. She has normal siDe $reasts% altho#gh the areolas are pale. She has little a.illary hair. Pel)ic e.amination is significant for scant p#$ic hair an" a short% $lin"!en"e" )aginal po#ch. =hich of the follo'ing is the most appropriate ne.t step in the management of this patienta6 :o inter)ention is necessary $6Bilateral gona"ectomy c6Unilateral gona"ectomy "6Bilateral mastectomy e6Unilateral mastectomy :ormal La$s 2.A 4!year!ol" 'oman comes to the physician for an ann#al e.amination. She has no complaints. 5or the past year% she has $een taking tamo.ifen for the pre)ention of $reast cancer. She 'as starte" on this "r#g after her physician "etermine" her to $e at high risk on the $asis of her strong family history% n#lliparity% an" early age at menarche. She takes no other me"ications. E.amination is 'ithin normal limits. =hich of the follo'ing is this patient most likely to "e)elop 'hile taking tamo.ifena6 Breast cancer $6Ele)ate" L@L cholesterol c6En"ometrial changes "6Myocar"ial infarction e6>steoporosis :ormal La$s &. A 22!year!ol" 'oman comes to the physician for an ann#al e.amination. She has $een se.#ally acti)e since the age of 1 an" has

not ha" reg#lar Pap smears or e.aminations. She is c#rrently se.#ally acti)e 'ith m#ltiple partners an" intermittently #ses con"oms. She has no me"ical pro$lems an" takes no me"ications. /er e.amination is #nremarka$le. /er Pap smear is "escri$e" as satisfactory $#t limite" $y the a$sence of en"ocer)ical cells. +t is other'ise 'ithin normal limits. =hich of the follo'ing is the most appropriate ne.t step in managementa6Aepeat the Pap smear in 1 year $6Aepeat the en"ocer)ical portion of the Pap test as soon as possi$le c6 Perform colposcopy 'ith colposcopically "irecte" $iopsies "6Perform laparoscopy 'ith laparoscopically "irecte" $iopsies e6Perform e.ploratory laparotomy :ormal La$s 4.A 24!year!ol" 'oman comes to the physician $eca#se of right lo'er *#a"rant a$"ominal pain. She has ha" the pain off an" on for the past month% $#t it is no' increasing. She has no other symptoms an" no me"ical pro$lems. E.amination re)eals a mil"ly ten"er% right a"ne.al mass. Pel)ic #ltraso#n" sho's a ( cm right a"ne.al comple. cyst. Urine h2C is negati)e. The patient is taken to the operating room for laparotomy an" right o)arian cystectomy. Microscopically the cyst has cartilage% a"ipose tiss#e% intestinal glan"s% hair% an" a calcification that appears to $e a tooth. There is also a large amo#nt of thyroi" tiss#e. =hich of the follo'ing is the most likely "iagnosisa62orp#s l#te#m $6Ectopic pregnancy c6Castric carcinoma "6Str#ma o)arii e6 Thyroi" carcinoma :ormal La$s .A <4!year!ol" 'oman comes to the physician for an ann#al e.amination. She has no complaints. She ha" her last menstr#al perio" at age an" has ha" no )aginal $lee"ing since. She has no me"ical pro$lems an" has ne)er ha" s#rgery. She takes no me"ications an" has no allergies to me"ications. The physical e.amination is #nremarka$le.

She is concerne" a$o#t cancer an" 'ants to kno' 'hich type is the ma9or ca#se of cancer "eath in 'omen. =hich of the follo'ing is the correct responsea6Breast cancer $62er)ical cancer c6En"ometrial cancer "6L#ng cancer e6 >)arian cancer :ormal La$s <.A 18!year!ol" female comes to the physician $eca#se of left lo'er *#a"rant pain for 2 months. She states that she first notice" the pain 2 months ago $#t no' it seems to $e gro'ing 'orse. She has ha" no changes in $o'el or $la""er f#nction. She has no fe)ers or chills an" no na#sea% )omiting% or "iarrhea. The pain is intermittent an" sometimes feels like a "#ll press#re. Pel)ic e.amination is significant for a left a"ne.al mass that is mil"ly ten"er. Urine h2C is negati)e. Pel)ic #ltraso#n" sho's a < cm comple. left a"ne.al mass 'ith feat#res consistent 'ith a $enign cystic teratoma 3"ermoi"6. =hich of the follo'ing is the most appropriate ne.t step in managementa6Aepeat pel)ic e.amination in 1 year $6Aepeat pel)ic #ltraso#n" in < 'eeks c6Prescri$e the oral contracepti)e pill "6Perform hysteroscopy e6Perform laparotomy :ormal La$s (.A &2!year!ol" 'oman% gra)i"a &% para 2% at &( 'eeks, gestation comes to the physician for a prenatal )isit. She has no c#rrent complaints. /er past me"ical history is significant for hepatitis 2 infection% 'hich she ac*#ire" thro#gh a nee"le stick in9#ry at 'ork as a n#rse. She is hepatitis B an" /+? negati)e. She takes no me"ications an" has no allergies to me"ications. /er prenatal co#rse has $een #ncomplicate". She 'ants to kno' 'hether she can ha)e contact 'ith the $a$y or $reast!fee" gi)en her hepatitis 2 stat#s. =hich of the follo'ing is the correct response-

a6There is no e)i"ence that $reast!fee"ing increases /2? transmission $6There is strong e)i"ence that $reast!fee"ing increases /2? transmission c62omplete isolation is not nee"e" $#t $reast!fee"ing is prohi$ite" "6The patient sho#l" $e completely isolate" from the $a$y e62as#al contact 'ith the $a$y is prohi$ite" :ormal La$s 1.A 2 !year!ol" 'oman% gra)i"a 2% para 1% at 22 'eeks, gestation comes to the physician 'ith complaints of $#rning 'ith #rination an" fre*#ent #rination. /er prenatal co#rse has $een #ncomplicate" e.cept for a #rinary tract infection 3UT+6 'ith E. coli at 12 'eeks, gestation% 'hich 'as treate" at that time. Physical e.amination is #nremarka$le. Urine c#lt#re "emonstrates greater than 144%444 colony!forming #nits per milliliter of E. coli. After treating this patient for her c#rrent infection% 'hich of the follo'ing is the most appropriate ne.t step in managementa6 :o f#rther treatment or "iagnostic st#"y is necessary $6Prophylactic anti$iotics for the remain"er of the pregnancy c6+ntra)eno#s anti$iotics for the remain"er of the pregnancy "6+ntra)eno#s pyelogram e6A$"ominal 2T Scan :ormal La$s 8.A 22!year!ol" 'oman in la$or progresses to ( cm "ilation% an" then has no f#rther progress. She therefore #n"ergoes a primary cesarean section. E.amination 2 "ays after the section sho's a temperat#re of &8.1 2 3142.4 56% $loo" press#re of 1147(4 mm /g% p#lse of 847min% an" respirations of 147min. L#ngs are clear to a#sc#ltation $ilaterally. /er a$"omen is mo"erately ten"er. The incision is clean% "ry% an" intact% 'ith no e)i"ence of erythema. Pel)ic e.amination "emonstrates #terine ten"erness. =hich of the follo'ing is the most appropriate pharmacotherapya6Ampicillin $6Ampicillin!gentamicin c62lin"amycin!gentamicin "62lin"amycin!metroni"aDole e6Metroni"aDole

:ormal La$s 14. A <4!year!ol" 'oman #n"ergoes a total a$"ominal hysterectomy an" $ilateral salpingo!oophorectomy for #terine prolapse. >n postoperati)e "ay 1% a complete $loo" co#nt sho's the follo'ing; Le#kocytes....... 447mm& /ematocrit.......&<B Platelets...........24 %4447mm& By postoperati)e "ay 2% the patient is alert an" a$le to am$#late 'itho#t "iffic#lty. She has no complaints. She has not taken in n#trition orally $#t is recei)ing +? fl#i"s. She is )oi"ing 'itho#t "iffic#lty an" has passe" flat#s. /er temperat#re is &( 2 381.< 56% $loo" press#re is 1247(2 mm /g% p#lse is 1<7min% an" respirations are 127min. E.amination sho's her a$"omen to $e soft% nonten"er% an" non! "isten"e". The incision is clean% "ry% an" intact. The rest of the e.amination is #nremarka$le. =hich of the follo'ing is a reason for keeping this patient hospitaliDe" for a longer perio" of timea6A$sent oral intake $6E)i"ence of infection c6/ematocrit "6Urinary tract f#nction e6?ital signs :ormal La$s 11.A &8!year!ol" 'oman% gra)i"a &% para 2% at 44 'eeks, gestation comes to the la$or an" "eli)ery 'ar" after a g#sh of fl#i" 'ith reg#lar% painf#l contractions e)ery t'o min#tes. She is fo#n" to ha)e r#pt#re of the mem$ranes an" to ha)e a cer)i. that is centimeters "ilate"% a fet#s in )erte. presentation% an" a reass#ring fetal heart rate tracing. She is a"mitte" to the la$or an" "eli)ery 'ar". T'o ho#rs later she states that she feels hot an" s'eaty. Temperat#re is &1.& 2 3141 56. She has mil" #terine ten"erness. /er cer)i. is no' 1 centimeters "ilate" an" the fetal heart tracing is reass#ring. =hich of the follo'ing is the most appropriate management of this patienta6A"minister anti$iotics to the mother after )aginal "eli)ery

$6A"minister anti$iotics to the mother no' an" allo' )aginal "eli)ery c6Perform cesarean "eli)ery "6 Perform cesarean "eli)ery an" then a"minister anti$iotics to the mother e6Perform intra!amniotic in9ection of anti$iotics :ormal La$s 12. A 4&!year!ol" primigra)i" 'oman at 14 'eeks, gestation comes to the physician for a prenatal )isit. She is feeling 'ell e.cept for some occasional na#sea. She has ha" no $lee"ing from the )agina% a$"ominal pain% "ys#ria% fre*#ency% or #rgency. She has asthma for 'hich she occasionally #ses an inhaler. E.amination is normal for a 'oman at 14 'eeks gestation. Urine "ipstick is positi)e for nitrites an" le#kocyte esterase an" a #rine c#lt#re sho's 4%444 colony forming #nits per milliliter of Escherichia coli. =hich of the follo'ing is the most appropriate ne.t step in managementa6=ait to see if symptoms "e)elop $6Aesen" another #rine c#lt#re c6>$tain a renal #ltraso#n" "6 Treat 'ith oral anti$iotics e6A"mit for intra)eno#s anti$iotics :ormal La$s 1&. A 28!year!ol" primigra)i" 'oman at &4 'eeks, gestation comes to the physician for a prenatal )isit. At 21 'eeks% she faile" her 4!g% 1! ho#r oral gl#cose!loa"ing test. She also faile" her follo'!#p 144!g% &! ho#r oral gl#cose tolerance test% 'ith a normal fasting gl#cose% $#t a$normal 1% 2% an" &!ho#r )al#es. >)er the past se)eral 'eeks% she has maintaine" goo" control of her fasting an" 2!ho#r postpran"ial gl#cose le)els $y a"hering to the "iet recommen"ations of her physician. She asks the physician 'hat effect her type of "ia$etes can ha)e on her or her fet#s. =hich of the follo'ing is the most appropriate responsea6 Cestational "ia$etes is associate" 'ith fetal anomalies $6Cestational "ia$etes is associate" 'ith intra#terine gro'th restriction c6Cestational "ia$etes is associate" 'ith macrosomia "6Cestational "ia$etes is not associate" 'ith f#t#re "ia$etes e6Cestational "ia$etes 'ith normal fasting gl#cose is associate" 'ith still$irth :ormal La$s

14.A &<!year!ol" primigra)i" 'oman at &< 'eeks, gestation comes to the physician for a prenatal )isit. She is e.periencing goo" fetal mo)ement an" has ha" no loss of fl#i"% $lee"ing from the )agina% or contractions. She has no complaints. /er past me"ical history is significant for mitral stenosis% 'hich she "e)elope" after an episo"e of rhe#matic fe)er as a chil". She also has asthma for 'hich she #ses an al$#terol inhaler "aily. She has herpes o#t$reaks appro.imately once a year. At her last )isit she 'as fo#n" to $e positi)e for Cro#p B Streptococc#s coloniDation. 5or 'hich of the follo'ing "isease processes 'o#l" this patient $enefit $y ha)ing a forceps!assiste" )aginal "eli)ery at the time of "eli)erya6Asthma $6Cro#p B Streptococc#s 3CBS6 coloniDation c6/erpes "6Mitral stenosis e6This patient 'o#l" not $enefit from a forceps!assiste" )aginal "eli)ery :ormal La$s 1 . A &2!year!ol"% /+?!positi)e% primigra)i" 'oman comes to the physician for a prenatal )isit at &4 'eeks. /er prenatal co#rse has $een nota$le for her #se of Di"o)#"ine 3O@?6 "#ring the pregnancy. /er )iral loa" has remaine" greater than 1444 copies per milliliter of plasma thro#gho#t the pregnancy. She has no other me"ical pro$lems an" has ne)er ha" s#rgery. E.amination is appropriate for a &4!'eek gestation. She 'ishes to "o e)erything possi$le to pre)ent the transmission of /+? to her $a$y. =hich of the follo'ing is the most appropriate ne.t step in managementa6>ffer electi)e cesarean section after amniocentesis to "etermine l#ng mat#rity $6>ffer electi)e cesarean section at &1 'eeks c6>ffer electi)e cesarean section at &4 'eeks "6Aecommen" forceps!assiste" )aginal "eli)ery e6 Aecommen" )aginal "eli)ery :ormal La$s 1<. A 14!year!ol" girl comes to the office for a health maintenance e)al#ation. She is concerne" that she has not yet starte" her menstr#al

cycle. /er height has increase" $y & inches since her last )isit 1 year ago% an" her 'eight is #p $y 14 po#n"s. >n physical e.amination% the physician notes a general enlargement of her $reasts an" areola. E.amination of her genital area re)eals p#$ic hair that is coarse an" "ark an" e.ten"s past the me"ial $or"er of the la$ia. =hich of the follo'ing is the most likely "iagnosisa6 2onstit#tional "elay $6 @ysf#nctional #terine $lee"ing c6@ysmenorrhea "6Primary amenorrhea e6Secon"ary amenorrhea :ormal La$s 1(.A &2!year!ol" 'oman comes to the physician $eca#se of amenorrhea. She ha" menarche at age 1& an" has ha" normal perio"s since then. /o'e)er% her last menstr#al perio" 'as 1 months ago. She also complains of an occasional milky nipple "ischarge. She has no me"ical pro$lems an" takes no me"ications. She is partic#larly concerne" $eca#se she 'o#l" like to $ecome pregnant as soon as possi$le. E.amination sho's a 'hitish nipple "ischarge $ilaterally% $#t the rest of the e.amination is #nremarka$le. Urine h#man chorionic gona"otropin 3h2C6 is negati)e. Thyroi" stim#lating hormone 3TS/6 is normal. Prolactin is ele)ate". /ea" MA+ scan is #nremarka$le. =hich of the follo'ing is the most appropriate pharmacotherapya6Bromocriptine $6@iclo.acillin c6Magnesi#m s#lfate "6>ral contracepti)e pill 3>2P6 e6 Thyro.ine :ormal La$s 11.A &2!year!ol" 'oman% gra)i"a &% para 2% at 14 'eeks, gestation comes to the physician for a prenatal )isit. She has some mil" na#sea% $#t other'ise no complaints. She has no significant me"ical pro$lems an" has ne)er ha" s#rgery. She takes no me"ications an" has no kno'n "r#g allergies. She is concerne" for t'o reasons. 5irst% the 0fl# season0 is coming% an" she seems to get sick e)ery year. Secon"% a chil" at her son,s "aycare center recently $roke o#t 'ith 'elts an" 'as sent home.

=hich of the follo'ing )accinations sho#l" this patient most likely $e gi)ena6 +nfl#enDa $6Measles c6M#mps "6A#$ella e6?aricella :ormal La$s 18.A & !year!ol" 'oman% gra)i"a &% para 2% at &8 'eeks, gestation% comes to the la$or an" "eli)ery 'ar" 'ith contractions. Past o$stetric history is significant for t'o normal spontaneo#s )aginal "eli)eries at term. E.amination sho's the cer)i. to $e 4 centimeters "ilate" an" 4B efface". The patient is contracting e)ery 4 min#tes. >)er the ne.t 2 ho#rs the patient progresses to centimeters "ilation. An epi"#ral is place". Artificial r#pt#re of mem$ranes is performe"% "emonstrating copio#s clear fl#i". 2 ho#rs later the patient is still at centimeters "ilation an" the contractions ha)e space" o#t to e)ery 14 min#tes. =hich of the follo'ing is the most appropriate ne.t step in managementa6E.pectant management $6+ntra)eno#s o.ytocin c62esarean "eli)ery "65orceps!assiste" )aginal "eli)ery e6?ac##m!assiste" )aginal "eli)ery :ormal La$s 24. A 2(!year!ol" 'oman% gra)i"a 2% para 1% at 24 'eeks, gestation comes to the physician for a prenatal )isit. She has no complaints. /er o$stetric history is significant for a primary lo' trans)erse cesarean "eli)ery $eca#se of a non!reass#ring fetal tracing & years ago. She has no me"ical pro$lems. She takes prenatal )itamins an" has no kno'n "r#g allergies. She is "e$ating 'hether to ha)e an electi)e repeat cesarean "eli)ery or to attempt a )aginal $irth after cesarean 3?BA26. She 'ants to kno' her chances for a s#ccessf#l ?BA2. =hich of the follo'ing most acc#rately represents the patient,s likelihoo" of ha)ing a s#ccessf#l )aginal "eli)erya6 4B

$6 2 B c6 4B "6(4B e6 144B :ormal La$s 21. A <2!year!ol" 'oman comes to the physician $eca#se of $lee"ing from the )agina. She states that her last menstr#al perio" came 11 years ago an" that she has ha" no $lee"ing since that time. She has hypertension an" type 2 "ia$etes mellit#s. E.amination sho's a mil"ly o$ese 'oman in no apparent "istress. Pel)ic e.amination is #nremarka$le. An en"ometrial $iopsy is performe" that sho's gra"e + en"ometrial a"enocarcinoma. =hich of the follo'ing is the most appropriate ne.t step in managementa6 2hemotherapy $62one $iopsy c6@ilation an" c#rettage "6/ysteroscopy e6/ysterectomy :ormal La$s 22.A & !year!ol" 'oman% gra)i"a 4% para &% at &1 'eeks, gestation comes to the la$or an" "eli)ery 'ar" after a g#sh of clear fl#i" from the )agina. After the g#sh% she has ha" increasing contractions. Sterile spec#l#m e.amination sho's a pool of clear fl#i" in the )agina that is nitraDine positi)e. 2er)ical e.amination sho's that the patient is cm "ilate"% 'ith the fetal face presenting in a ment#m anterior position. E.ternal #terine monitoring sho's that the patient is contracting e)ery 2 min#tes% an" e.ternal fetal monitoring sho's that the fetal heart rate is in the 144s an" reacti)e. =hich of the follo'ing is the most appropriate ne.t step in managementa6E.pectant management $6>.ytocin a#gmentation c65orceps "eli)ery "6?ac##m "eli)ery e62esarean section :ormal La$s

2&.A &<!year!ol" 'oman% gra)i"a &% para 2% at && 'eeks, gestation comes to the physician for a prenatal )isit. She has some fatig#e $#t no other complaints. /er c#rrent pregnancy has $een complicate" $y a Cro#p B Streptococc#s #rine infection at 1< 'eeks. /er past o$stetric history is significant for a primary% classic cesarean "eli)ery years ago for a non!reass#ring fetal tracing. T'o years ago% she ha" a repeat cesarean "eli)ery. Past s#rgical history is significant for an appen"ectomy 14 years ago. =hich of the follo'ing is the ma9or contrain"ication to a )aginal $irth after cesarean 3?BA26 in this patienta6 2lassic #terine scar $6Cro#p B Streptococc#s #rine infection c6Pre)io#s appen"ectomy "6Prior cesarean "eli)ery for non!reass#ring fetal tracing e6T'o prior cesarean "eli)eries :ormal La$s 24.A patient 'ho has $een taking tamo.ifen to pre)ent $reast cancer for the past < months presents complaining of irreg#lar )aginal $lee"ing. An en"ometrial $iopsy is performe" that "emonstrates atypical hyperplasia. =hich of the follo'ing is the most appropriate ne.t step in managementa6@iscontin#e the tamo.ifen $6+ncrease the tamo.ifen "ose c6Aepeat the en"ometrial $iopsy "6Sche"#le a pel)ic #ltraso#n" e6S'itch the patient to estrogen :ormal La$s 2 .A 11!year!ol" 'oman comes to the physician for an ann#al e.amination. She has no complaints. She has $een se.#ally acti)e for the past 2 years. She #ses the oral contracepti)e pill for contraception. She has "epression for 'hich she takes fl#o.etine. She takes no other me"ications an" has no allergies to me"ications. /er family history is negati)e for cancer an" car"iac "isease. E.amination is #nremarka$le. =hich of the follo'ing screening tests sho#l" this patient most likely ha)ea6 2olonoscopy $6Mammogram

c6Pap smear "6Pel)ic #ltraso#n" e6Sigmoi"oscopy :ormal La$s 2<.A 18!year!ol" primigra)i" 'oman at 42 'eeks, gestation comes the la$or an" "eli)ery 'ar" for in"#ction of la$or. /er prenatal co#rse 'as #ncomplicate". E.amination sho's her cer)i. to $e long% thick% close"% an" posterior. The fetal heart rate is in the 144s an" reacti)e. The fet#s is )erte. on #ltraso#n". Prostaglan"in 3PCE26 gel is place" intra)aginally. >ne ho#r later% the patient $egins ha)ing contractions lasting longer than 2 min#tes. The fetal heart rate falls to the (4s. =hich of the follo'ing is the most appropriate ne.t step in managementa6A"minister general anesthesia $6A"minister ter$#taline c6Perform amnioinf#sion "6Start o.ytocin e6 Perform cesarean "eli)ery :ormal La$s 2(. A 2 !year!ol" primigra)i" 'oman comes to the physician for her first prenatal )isit. /er last menstr#al perio" 'as ( 'eeks ago. She has ha" some na#sea an" )omiting $#t other'ise has no complaints. Past me"ical an" s#rgical history are #nremarka$le. /er family history is significant for cystic fi$rosis 'ith an affecte" a#nt. /er h#s$an" has an affecte" co#sin. Physical e.amination is #nremarka$le. Ci)en her family history% she is concerne" a$o#t the risks of ha)ing a chil" 'ith cystic fi$rosis. She in*#ires a$o#t cystic fi$rosis screening. =hich of the follo'ing is the appropriate responsea6Screening is a)aila$le $6Screening is inappropriate in her case c6Screening is man"atory "6Screening is not a)aila$le e6Screening is #nnecessary; she has a 1 in 4 chance of ha)ing an affecte" chil" :ormal La$s

21. A 2!year!ol" 'oman comes to the physician $eca#se of hot flashes. /er last menstr#al perio" 'as 1 year ago. >)er the past year% she has note" a persistence of her hot flashes% 'hich come se)eral times each "ay an" are associate" 'ith a feeling of heat an" fl#shing. They also a'aken her at night an" interfere 'ith her sleep. She has no me"ical pro$lems% takes no me"ications% an" has no kno'n "r#g allergies. She has a family history of car"io)asc#lar "isease an" she "oes not smoke. Physical e.amination is #nremarka$le. She is starte" on estrogen an" me"ro.yprogesterone acetate 3Pro)era6. The a""ition of a progestin is most likely to "ecrease her risk of 'hich of the follo'inga6Breast cancer $6Breast pain c6En"ometrial cancer "6Moo" changes e6=eight gain :ormal La$s 28.A 21!year!ol" 'oman comes to the physician $eca#se of 0$#mps0 on her )#l)a that she has 9#st recently notice". These $#mps "o not ca#se her symptoms% $#t she 'ants to kno' 'hat they are an" 'ants them remo)e". She has no me"ical pro$lems% takes no me"ications% an" has no allergies to me"ications. She smokes one!half pack of cigarettes per "ay. She is se.#ally acti)e 'ith & partners. E.amination sho's & ca#liflo'er!like lesions on the right la$ia ma9ora. =hich of the follo'ing is the most appropriate ne.t step in managementa6Acyclo)ir $6Penicillin c62one $iopsy "62ryotherapy e6?#l)ectomy :ormal La$s &4.A 28!year!ol" patient comes to the physician for an ann#al e.amination. She has normal menstr#al perio"s e)ery &4 "ays. She 'as 1 years ol" 'hen she first $egan ha)ing interco#rse. She #ses con"oms for contraception. /er past me"ical history is significant for m#ltiple sclerosis. This con"ition has re*#ire" her to #se a 'heelchair for the past 4 years% 'hich makes pel)ic e.amination some'hat "iffic#lt for her. She smokes one pack of cigarettes per "ay. Ci)en her "iffic#lty 'ith the pel)ic e.amination% she in*#ires as to ho' often she nee"s to

ha)e a Pap smear performe". =hich of the follo'ing is the correct ans'era6A Pap smear sho#l" $e performe" e)ery year $6A Pap smear sho#l" $e performe" e)ery & years c6 A Pap smear sho#l" $e performe" e)ery years "6A Pap smear sho#l" $e performe" only if there are symptoms e6 A Pap smear is not necessary :ormal La$s &1.A &&!year!ol" 'oman comes to the physician for her first prenatal )isit. /er last menstr#al perio" 'as ( 'eeks ago. She has ha" no $lee"ing or a$"ominal pain. She has no me"ical pro$lems an" takes no me"ications. She has no family history of congenital anomalies. /er h#s$an" is years ol". /e is in goo" health an" also has no family history of $irth "efects. The patient is concerne" that her h#s$an",s age may place their fet#s at increase" risk of a chromosomal anomaly. She 'ishes to kno' the paternal age a$o)e 'hich amniocentesis or chorionic )ill#s sampling sho#l" $e consi"ere". =hich of the follo'ing is the correct responsea6 A$o)e age &4 $6A$o)e age & c6A$o)e age 44 "6 A$o)e age 4 e6There is no age c#toff for paternal risk :ormal La$s &2.A 14!year!ol" girl comes to the physician for an ann#al e.amination. She has no complaints. She $ecame se.#ally acti)e "#ring the past year an" #ses con"oms occasionally for contraception. She has asthma% for 'hich she occasionally takes an al$#terol inhaler. She ha" an appen"ectomy at age 8. Physical e.amination is #nremarka$le incl#"ing a normal pel)ic e.amination. =hen sho#l" this patient $egin ha)ing Pap testinga6+mme"iately $6Age 1< c6 Age 11 "6 Age 24 e6 Age 21

:ormal La$s &&.A 2 !year!ol" n#lliparo#s 'oman at & 'eeks, gestation comes to the la$or an" "eli)ery 'ar" complaining of contractions% a hea"ache% an" flashes of light in front of her eyes. /er pregnancy has $een #ncomplicate" e.cept for an episo"e of first trimester $lee"ing that completely resol)e". She has no me"ical pro$lems. /er temperat#re is &( 2 381.< 56% $loo" press#re is 1<47114 mm /g% p#lse is 117min#te% an" respirations are 127min#te. E.amination sho's that her cer)i. is 2 centimeters "ilate" an" ( B efface"% an" that she is contracting e)ery 2 min#tes. The fetal heart tracing is in the 144s an" reacti)e. Urinalysis sho's &E protein#ria. La$oratory )al#es are as follo's; le#kocytes 8%4447mm&% hematocrit & B% platelets 141%4447mm&. Aspartate aminotransferase 3AST6 is 244 U7L% an" ALT &44 U7L. =hich of the follo'ing is the most appropriate ne.t step in managementa6A"minister o.ytocin $6@ischarge the patient c6Enco#rage am$#lation "6Start magnesi#m s#lfate e6Start ter$#taline :ormal La$s &4.A &&!year!ol" primigra)i" 'oman at 11 'eeks, gestation comes to the physician for a prenatal )isit. /er prenatal co#rse has $een #ncomplicate" th#s far. She has no complaints. She has ha" no loss of fl#i"% $lee"ing% or contractions. She has hypothyroi"ism% for 'hich she takes thyroi" hormone replacement. The patient states that a frien" of hers recently ha" a preterm "eli)ery. The patient is *#ite concerne" a$o#t preterm "eli)ery an" 'ants to kno' 'hether home #terine acti)ity monitoring 3/UAM6 is recommen"e". =hich of the follo'ing is the most appropriate responsea6 /UAM has $een pro)en to ca#se preterm $irth $6/UAM has $een pro)en to pre)ent preterm $irth

c6/UAM has not $een pro)en to pre)ent preterm $irth "6/UAM sho#l" $e starte" imme"iately e6 /UAM sho#l" $e starte" at & 'eeks :ormal La$s & . A &2!year!ol" n#lliparo#s 'oman at &1 'eeks, gestation comes to the la$or an" "eli)ery 'ar" 'ith reg#lar painf#l contractions after a g#sh of fl#i" t'o ho#rs ago. /er temperat#re is 81.< 5 3&( 26. She is fo#n" to ha)e gross r#pt#re of mem$ranes an" to ha)e a cer)i. that is < centimeters "ilate". The fet#s is in $reech position. The patient is then $ro#ght to the operating room for cesarean "eli)ery. =hich of the follo'ing represents the correct proce"#re for anti$iotic a"ministrationa6 A"minister intra)eno#s anti$iotics &4 min#tes prior to the proce"#re $6A"minister intra)eno#s anti$iotics after the cor" is clampe" c6 A"minister intra)eno#s anti$iotics imme"iately after the proce"#re "6A"minister intra)eno#s anti$iotics for 24 ho#rs after the proce"#re e6A"minister oral anti$iotics for 1 'eek follo'ing the proce"#re. :ormal La$s &<. A pre)io#sly healthy 21!year!ol" 'oman has a prof#se% malo"oro#s )aginal "ischarge. E.amination sho's a greenish gray 0frothy0 "ischarge 'ith a 0fishy0 o"or an" petechial lesions on the cer)i.. There is no cer)ical motion ten"erness. /er temperat#re is &(. 2 388.4 56% $loo" press#re is 124714 mm /g% p#lse is <47min% an" respirations are 1<7min. Microscopic e)al#ation of the "ischarge is most likely to sho' 'hich of the follo'inga6 02l#e cells0 $6Cram!negati)e "iplococci c6Cram!positi)e "iplococci "6Motile% flagellate" organisms e6Pse#"ohyphae or hyphae :ormal La$s &(.A 21!year!ol" primigra)i" 'oman at &8 'eeks, gestation comes to the la$or an" "eli)ery 'ar" 'ith painf#l contractions e)ery three min#tes. /er prenatal co#rse 'as #nremarka$le. E.amination sho's

her cer)i. to $e & centimeters "ilate" an" 84B efface". The fetal heart rate tracing is in the 1 4s an" reacti)e. ho#rs later cer)ical e.amination re)eals that the patient is 8 centimeters "ilate" an" at !1 station. The fetal heart rate tracing sho's mo"erate )aria$le "ecelerations 'ith each contraction an" "ecrease" )aria$ility. 5etal scalp sampling is performe" that yiel"s fetal scalp p/s of (.44% (.4 % an" (.4<. =hich of the follo'ing is the most appropriate ne.t step in managementa6E.pectant management $6Episiotomy c65orceps!assiste" )aginal "eli)ery "6?ac##m!assiste" )aginal "eli)ery e6 2esarean "eli)ery :ormal La$s &1.A &1!year!ol"% /+?!positi)e 'oman% gra)i"a &% para 2% at &2!'eeks, gestation comes to the physician for a prenatal )isit. /er prenatal co#rse is significant for the fact that she has taken Di"o)#"ine thro#gho#t the pregnancy. >ther'ise% her prenatal co#rse has $een #nremarka$le. She has no history of mental illness. She states that she has $een 'eighing the $enefits an" risks of cesarean "eli)ery in pre)enting transmission of the )ir#s to her $a$y. After m#ch "eli$eration% she has "eci"e" that she "oes not 'ant a cesarean "eli)ery an" 'o#l" like to attempt a )aginal "eli)ery. =hich of the follo'ing is the most appropriate ne.t step in managementa62ontact psychiatry to e)al#ate the patient $62ontact the hospital la'yers to get a co#rt or"er for cesarean "eli)ery c6Perform cesarean "eli)ery at &1 'eeks "6 Perform cesarean "eli)ery once the patient is in la$or e6 Aespect the patient,s "ecision an" perform the )aginal "eli)ery :ormal La$s

&8. A 2<!year!ol" n#lligra)i" 'oman comes to the emergency "epartment $eca#se of se)ere right lo'er *#a"rant pain. She states that the pain starte" last night. This morning she 'as a'akene" from sleep 'ith se)ere pain in the same area. @#ring the episo"e of pain she also ha" na#sea% )omiting% an" "iaphoresis. >n a"mission to the emergency

"epartment she re*#ire" mg of morphine to control her pain. E.amination is significant for right lo'er *#a"rant ten"erness an" a ten"er right a"ne.al mass on pel)ic e.amination. Urine h2C is negati)e. Urinalysis is negati)e. Trans)aginal #ltraso#n" re)eals an 1 cm right o)arian mass. =hich of the follo'ing is the most likely "iagnosisa6Appen"icitis $6Ectopic pregnancy c6:ephrolithiasis "6>)arian torsion e6Pel)ic inflammatory "isease :ormal La$s 44.An 11!year!ol" 'oman comes to the physician for a")ice regar"ing $irth control. She has $een se.#ally acti)e since the age of 1 an" has ha" n#mero#s se.#al partners since that time. She has trie" the oral contracepti)e pill t'ice% for appro.imately t'o cycles each time% $#t stoppe" $eca#se of irreg#lar $lee"ing. She has ha" gonorrhea once an" 2hlamy"ia t'ice. She "oes not smoke. Physical e.amination is #nremarka$le. =hich of the follo'ing forms of $irth control sho#l" $e recommen"e" for this patienta62on"oms $6@iaphragm c6+ntra#terine "e)ice "6>ral contracepti)e pill e6T#$al ligation :ormal La$s 41. A 44!year!ol" 'oman% gra)i"a 4% para &% at 1 'eeks, gestation comes to the physician for her first prenatal )isit. She has mil" na#sea an" )omiting $#t no other complaints. /er o$stetric history is significant for three f#ll!term% normal )aginal "eli)eries of normal infants. She has no me"ical or s#rgical history an" takes no me"ications. Physical e.amination re)eals an 1!'eek!siDe" #ter#s% $#t is other'ise #nremarka$le. She 'ishes to ha)e chromosomal testing of the fet#s an" 'ants to ha)e chorionic )ill#s sampling performe"% as she "i" 'ith her last pregnancy. 2ompare" 'ith amniocentesis% chorionic )ill#s sampling may place the patient at greater risk for 'hich of the follo'ing-

a65etal @o'n syn"rome $65etal lim$ "efects c65etal ne#ral t#$e "efects "6Maternal sepsis e6Mi"!secon"!trimester a$ortion :ormal La$s 42.A pharmace#tical company sponsors a physician lect#re concerning throm$otic complications of the oral contracepti)e pill 3>2P6. At the start of the presentation% the company,s representati)e makes a short presentation regar"ing their partic#lar $ran" of >2P. /e then procee"s to anno#nce that his company 'o#l" like to a'ar" a gift to the physician in the gro#p 'ho gi)es the largest n#m$er of prescriptions for this pill. =hich of the follo'ing is the most appropriate actiona6 Acceptance of the gift $6Attempt to get colleag#es to prescri$e the me"ication c6Promise to prescri$e more of the me"ication "6Aef#sal of the gift e6 Ae*#est for money rather than a gift :ormal La$s 4&. A 24!year ol" 'oman comes to the physician $eca#se of $#rning 'ith #rination. She states that e)ery time she #rinates there is pain an" that she has a feeling that she constantly nee"s to #rinate e)en tho#gh only a little comes o#t. She has ne)er ha" any similar symptoms $efore. She has no me"ical pro$lems an" no kno'n "r#g allergies. E.amination is #nremarka$le. Urinalysis "emonstrates that the #rine is positi)e for le#kocyte esterase an" nitrites. =hich of the follo'ing is the most appropriate pharmacotherapya6+ntram#sc#lar ceftria.one $6+ntra)eno#s le)oflo.acin c6>ral le)oflo.acin for ( "ays "6>ral trimethoprim!s#lfametho.aDole for & "ays e6 =ait for the c#lt#re res#lts to instit#te therapy :ormal La$s

44. A 21!year!ol" 'oman comes to the physician $eca#se of a$"ominal pain. She states that the pain is in her right lo'er *#a"rant an" has $een getting 'orse o)er the past & months. She has no other symptoms an" a normal appetite. E.amination "emonstrates mil" right lo'er *#a"rant a$"ominal ten"erness. Pel)ic e.amination re)eals mil" right a"ne.al enlargement an" ten"erness. Urine h#man chorionic gona"otropin 3h2C6 is negati)e. A pel)ic #ltraso#n" is o$taine" that sho's a 4!centimeter% heterogeneo#s hyperechoic lesion in the right a"ne.a 'ith cystic areas. >n trans)aginal #ltraso#n"% hair an" calcifications are "emonstrate" 'ithin the cystic areas. =hich of the follo'ing is the most likely "iagnosisa6Appen"icitis $6Benign cystic teratoma 3"ermoi"6 c62orp#s l#te#m cyst "6Ectopic pregnancy e6T#$o!o)arian a$scess :ormal La$s 4 .A < !year!ol" 'oman comes to the physician $eca#se of $lee"ing from the )agina. She states that her last menstr#al perio" 'as at age 4 an" that she has ha" no $lee"ing since. She has no me"ical pro$lems an" takes no me"ications. She is not se.#ally acti)e. E.amination is #nremarka$le% incl#"ing a normal pel)ic e.amination. After informe" consent is o$taine"% an en"ometrial $iopsy is performe". The patient complains of "iscomfort "#ring an" after the proce"#re $#t feels 'ell eno#gh to go home. Later that night% 'ith her a$"ominal pain 'orsening% the patient comes to the emergency "epartment. An #ltraso#n" is performe" that sho's a normal #ter#s an" a"ne.ae $#t a comple. fl#i" collection posterior to the #ter#s. =hich of the follo'ing is the most likely "iagnosisa6 Bo'el perforation $6En"ometritis c6En"ometrial cancer "6T#$oo)arian a$scess e6Uterine perforation :ormal La$s

4<.A 2&!year!ol" female comes to the physician $eca#se of a s'elling in her )agina. She states that the s'elling starte" a$o#t & "ays ago an" has $een gro'ing larger since. The s'elling is not painf#l% $#t it is #ncomforta$le 'hen she 9ogs. She has asthma for 'hich she #ses an al$#terol inhaler% $#t no other me"ical pro$lems. E.amination sho's a cystic mass 4 cm in "iameter near the hymen $y the patient,s left la$ia minora. The mass is nonten"er an" there is no associate" erythema. The mass is freely mo$ile. The rest of the pel)ic e.amination is #nremarka$le. =hich of the follo'ing is the most likely "iagnosisa6Bartholin,s cyst $62on"yloma lata c6Cran#loma ing#inale "6/ematocolpos e6?#l)ar cancer :ormal La$s 4(. A &(!year!ol" 'oman% gra)i"a &% para 2% comes to her physician for follo'!#p on her ectopic pregnancy. She 'as "iagnose" 'ith an ectopic pregnancy ( "ays ago an" gi)en methotre.ate. She no' presents 'ith a$"ominal pain that starte" this morning. E.amination is significant for mo"erate left lo'er *#a"rant ten"erness. La$oratory analysis sho's that her $eta!h2C )al#e has "o#$le" o)er the past 'eek. Trans)aginal #ltraso#n" sho's that the ectopic pregnancy is ro#ghly the same siDe $#t there is an increase" amo#nt of fl#i" in the pel)is. =hich of the follo'ing is the most appropriate ne.t step in managementa6E.pectant management $6Aepeat methotre.ate c6Laparoscopy "6>ophorectomy e6/ysterectomy :ormal La$s 41. A 2<!year!ol" 'oman comes to the physician $eca#se of a l#mp in her )agina. The l#mp is nonten"er $#t is #ncomforta$le 'hen she 'alks. She states that for the last < years this l#mp has appeare" a$o#t once a year. =hen it occ#rs she goes to the "octor 'ho p#ts a catheter into it% 'hich is taken o#t in a fe' 'eeks. She has no other me"ical pro$lems. She is se.#ally acti)e 'ith t'o partners. E.amination sho's a

cystic mass appro.imately 4 cm in "iameter on the right si"e of the )agina near the hymeneal ring. The mass feels like a "iscrete cyst. The rest of the pel)ic e.amination is #nremarka$le. =hich of the follo'ing is the most appropriate ne.t step in managementa6E.pectant management $6>ral anti$iotics c6+ntra)eno#s anti$iotics "6+ncision an" "rainage e6 Bartholin,s cyst mars#pialiDation :ormal La$s 48.A 2 !year!ol" 'oman% gra)i"a 2% para 2 is 4 "ays stat#s post cesarean section an" "e)elops a temperat#re to 144.( 5 3&1.2 26. She ha" her cesarean section 'hen she 'ent into #nstoppa$le preterm la$or 'ith a $reech fet#s. She ha" an #ncomplicate" postoperati)e co#rse #ntil this temperat#re ele)ation. /er p#lse is 1447min% $loo" press#re is 1147(4 mm /g% an" respirations are 1<7min. There is "iscoloration an" cyanosis aro#n" the incision. The area aro#n" the incision is completely n#m$. There is no #terine ten"erness on $iman#al e.am. =hich of the follo'ing is of the most concern in this patienta6 En"ometritis $6Mastitis c6:ecrotiDing fasciitis "6Preeclampsia e6 =o#n" infection :ormal La$s 4. A &2!year!ol" 'oman% gra)i"a 2% para 2% comes to the physician for follo'!#p of an a$normal Pap test. >ne month ago% her Pap test sho'e" a high!gra"e s*#amo#s intraepithelial lesion 3/CS+L6. 2olposcopy "emonstrate" aceto'hite epitheli#m at 2 o,clock. A $iopsy taken of this area "emonstrate" /CS+L. En"ocer)ical c#rettage 3E226 'as negati)e. The patient has no other me"ical pro$lems% has ne)er ha" cer)ical "ysplasia% an" takes no me"ications. =hich of the follo'ing is the most appropriate ne.t step in managementa6Aepeat Pap test in 1 year $6Aepeat Pap test in < months c6Aepeat colposcopy in < months

"6Loop electro"e e.cision proce"#re 3LEEP6 e6/ysterectomy


Normal ,abs

E.planations Block 1( E.planations

:) (*!lanation: 7he correct answer is . 7his !atient has the fin"in$s that are most consistent with an"ro$en insensitivity syn"rome 'formerly calle" testicular femini#ation syn"rome). 7his syn"rome results from $enetic "efects lea"in$ to abnormal an"ro$en rece!tor function. &atients with an"ro$en insensitivity syn"rome are $enoty!ically males '?C, E9) but !henoty!ically females4with breasts an" no e*ternal male $enitalia. 7he reason that breasts "evelo! is that estro$ens, which are e*!resse" at !uberty an" which also result from !eri!heral conversion of an"ro$ens, act u!on the breast tissues uno!!ose" by an"ro$ens because of the an"ro$en rece!tor "efect. 7his uno!!ose" estro$en lea"s to breast $rowth an" the resultant breasts are normal si#e", althou$h they have un"evelo!e" ni!!les an" !ale areolae. 7here are no internal female or$ans, because mullerian4inhibitin$ substance is !resent "urin$ "evelo!ment. 7here are no e*ternal male or$ans because of the an"ro$en rece!tor "efect. 7esticles "o e*ist, but they are intra4ab"ominal. 7he $ona"s have a hi$h rate of mali$nant "e$eneration in !atients with an"ro$en insensitivity syn"rome an" therefore, after !uberty, they shoul" be remove" via bilateral $ona"ectomy. It is im!ortant to wait until after !uberty so that full "evelo!ment can take !lace. 7o state that no intervention is necessary 'choice A) is incorrect. If the $ona"s are not remove" from a !atient with an"ro$en insensitivity syn"rome there is a si$nificant risk that the !atient will "evelo! a $ona"al mali$nancy. 7o !erform a unilateral $ona"ectomy 'choice C) is incorrect. 7o leave one of the $ona"s in woul" still run the risk of mali$nant "e$eneration in that $ona". Once !uberty has taken !lace, therefore, both $ona"s shoul" be remove". 7o !erform a bilateral mastectomy 'choice %) or a unilateral mastectomy 'choice () woul"

be incorrect. In !atients with an"ro$en insensitivity syn"rome 'testicular femini#ation syn"rome) the !rimary concern is for $ona"al mali$nancy an" not breast mali$nancy. ;) (*!lanation: 7he correct answer is C. 7amo*ifen is a nonsteroi"al a$ent with both !ro4 an" antiestro$enic !ro!erties. It was first a!!rove" in :IDD by the F.). -oo" an" %ru$ A"ministration for use in !ostmeno!ausal women with a"vance" breast cancer. )ince that time, it has been a!!rove" for many other uses relate" to breast cancer: as a"<uvant thera!y in !ostmeno!ausal women with resecte" no"e4!ositive "isease, in !ostmeno!ausal women with metastatic breast cancer, an" as a"<uvant thera!y in women '!re4 an" !ostmeno!ausal) with resecte" no"e4ne$ative "isease. 6ecently, much attention has been focuse" on its use for breast cancer !revention. 7here is evi"ence that women at hi$h risk for the "evelo!ment of breast cancer may re"uce their risk by takin$ tamo*ifen. 2owever, althou$h tamo*ifen a!!ears to be antiestro$enic at the level of the breast, it a!!ears to act in a !roestro$enic fashion at the level of the en"ometrium. +any women on tamo*ifen will "evelo! en"ometrial chan$es, inclu"in$ !oly! formation, hy!er!lasia, an" frank invasive carcinoma. 7hus, women on tamo*ifen nee" to be followe" carefully, an" !rom!t evaluation of abnormal va$inal blee"in$ shoul" be con"ucte". 7amo*ifen is use" to !revent breast cancer 'choice A). 7amo*ifen, like estro$en, has been shown to lower bloo" levels of ,%, cholesterol 'choice ). Women on tamo*ifen a!!ear to be at no $reater risk, an" may be at a lower risk, for the "evelo!ment of myocar"ial infarction 'choice %). 7amo*ifen, like estro$en, has been shown to increase bone "ensity an" to re"uce the likelihoo" of "evelo!ment of osteo!orosis 'choice (). 0) (*!lanation: 7he correct answer is . A &a!anicolaou smear shoul" i"eally be a sam!lin$ of the transformation #one. An a"equate sam!le shoul" show en"ocervical cells. When en"ocervical cells are not !resent, there is some question as to whether the transformation #one was fully sam!le". If a woman has no risk factors for cervical "ys!lasia, has ha" three normal annual &a! smears in a row, an" has a current &a! that shows no abnormality other than the absence of en"ocervical cells, then the &a! smear can be re!eate" in : year. 7his !atient, however, has si$nificant risk factors for cervical "ys!lasia, inclu"in$ early initiation of se*ual activity, multi!le !artners, an" un!rotecte" intercourse. 7herefore, this !atient nee"s the en"ocervical !ortion of the &a! test to be re!eate" as soon as !ossible. 7o re!eat the &a! smear in : year 'choice A) woul" be incorrect mana$ement. As note" above, re!eatin$ the &a! smear in : year is correct only in !atients who have no risk factors for cervical "ys!lasia, three normal annual &a! smears, an" a !resent &a! that is normal e*ce!t for the lack of en"ocervical cells. 7o !erform a col!osco!y with col!osco!ically "irecte" bio!sies 'choice C) woul" not be correct. 7his !atient has a normal &a! smear overall. 7he lack of en"ocervical cells makes the smear incom!lete but

not abnormal. 7o !erform la!arosco!y with la!arosco!ically "irecte" bio!sies 'choice %) woul" not be correct. ,a!arosco!y "oes not allow evaluation of the cervi* an" is not in"icate" for abnormal or incom!lete &a! smears. 7o !erform an e*!loratory la!arotomy 'choice () is not in"icate". A$ain, this !atient has a normal but incom!lete &a! smear, an" ma<or sur$ery woul" not be correct mana$ement. ?) (*!lanation: 7he correct answer is %. Cystic teratomas, also known as "ermoi" cysts, are the most common beni$n ovarian neo!lasm. 7hey account for a!!ro*imately :B0 of all ovarian neo!lasms. 7hey may be com!ose" of a variety of cell ty!es an" have a mi*ture of tissues, as this !atient has. When thyroi" tissue makes u! more than />= of the teratoma, the "ermoi" is then referre" to as struma ovarii. A!!ro*imately 0= of ovarian teratomas fall into this cate$ory an" there is an association of struma ovarii with carcinoi" tumor. )truma ovarii is unilateral in a!!ro*imately I>= of !atients an" most '@>=) are beni$n. 6arely struma ovarii is a cause of hy!erthyroi"ism an" !atients with this manifestation may have sym!toms of hy!erthyroi"ism, as well as elevate" levels of thyroi" hormones an" "ecrease" levels of thyroi" stimulatin$ hormone '7)2). 7reatment of struma ovarii is by sur$ical removal of the tumor. A cor!us luteum 'choice A) is a common cause of com!le* cysts in youn$ women. 2owever, a cor!us luteum "oes not contain thyroi" tissue, hair, teeth, an" other such tissues. (cto!ic !re$nancy 'choice ) can cause an a"ne*al mass, an" a live ecto!ic may have various tissues in it when e*amine" microsco!ically. 2owever, this !atient has a ne$ative hC., which effectively rules out ecto!ic !re$nancy unless there is a laboratory error. Also, this cyst has tissues that are foun" in struma ovarii. .astric carcinoma 'choice C) can metastasi#e to the ovary. In fact, /= of all ovarian mali$nancies are metastases from other sites. 7he cancers that most frequently metastasi#e to the ovary are colon, breast, stomach, an" !ancreas. When a $astric carcinoma metastasi#es to the ovary, it is terme" a 5rukenber$ tumor an" has the !atho$nomonic Hsi$net4rin$H cells. 7hyroi" carcinoma 'choice () rarely metastasi#es to the ovary an" rarely woul" be foun" in combination with the other tissue elements that this !atientAs cyst has. /) (*!lanation: 7he correct answer is %. reast cancer accounts for the $reatest number of new cancer cases in women each year. In :IID, there were :@>,;>> new breast cancer cases. 2owever, lun$ cancer is the ma<or cause of cancer "eath in women. In :IID, lun$ cancer accounte" for CC,>>> cancer "eaths in women, com!are" with the ?0,I>> female "eaths cause" by breast cancer. 7here is currently no test use" to screen for lun$ cancer. )mokin$ cessation is the most effective way to re"uce mortality from lun$ cancer. As state" above, breast cancer 'choice A) accounts for the most number of cancer cases in women each year in the F.)., but not the hi$hest number of cancer "eaths. +ammo$ra!hy is the screenin$ metho" use"

to "etect subclinical breast cancer4the sta$e at which breast cancer is least likely to have s!rea". Cervical cancer 'choice ) is the $ynecolo$ic ty!e that causes the fewest number of cancer "eaths, !artly because of the success of &a! test screenin$. &a! testin$ allows !reinvasive lesions to be i"entifie" an" treate", which !revents the !ro$ression to invasive "isease. (n"ometrial cancer 'choice C) is the most common $ynecolo$ic cancer in women ol"er than ?/. 7here is no !roven screenin$ test available for en"ometrial cancer. Ovarian cancer 'choice () is a ma<or cause of cancer "eath in women. +ore women "ie of ovarian cancer than of cervical or en"ometrial cancer combine". 7here is no !roven screenin$ test available for ovarian cancer. C) (*!lanation: 7he correct answer is (. 7his !atient has a !resentation an" fin"in$s that are most consistent with a beni$n cystic teratoma '"ermoi"). %ermoi"s are a ty!e of ovarian $erm cell tumor. .erm cell tumors are the most common ty!e of ovarian neo!lasm in females un"er the a$e of ;> an" "ermoi"s are the most common beni$n ovarian neo!lasm. %ermoi"s can ran$e in si#e from small masses that are note" inci"entally on ultrasoun" an" cause no sym!toms, to lar$e cysts that cause !ain an" !ressure, as this !atient has. ,a!arotomy is the most a!!ro!riate ne*t ste! in the mana$ement of this !atient because, as a"ne*al masses enlar$e44es!ecially when they become $reater than / cm44the risk of ovarian torsion increases. 7hus, la!arotomy with removal of the "ermoi" is in"icate" to !revent torsion. Also, this !atientAs mass is causin$ her sym!toms of !ain an" !ressure an", on that basis, shoul" be remove". -inally, while the mass most likely is a "ermoi", this is not certain without !atholo$ic "ia$nosis an", therefore, the cyst shoul" be remove" an" evaluate" by a !atholo$ist. At the time of sur$ery, close e*amination shoul" be ma"e of the other ovary because "ermoi"s may be foun" bilaterally in more than :>= of cases. 7o re!eat !elvic e*amination in : year 'choice A) woul" not be correct mana$ement. 7his !atient is sym!tomatic with a C cm ovarian mass that is at risk for torsion. )he shoul", therefore, be mana$e" sur$ically. 7o re!eat !elvic ultrasoun" in C weeks choice ) is a!!ro!riate for some a"ne*al masses. -or e*am!le, in a youn$ woman with a small com!le* cyst that a!!ears consistent with a cor!us luteum, it may be most !ru"ent to recheck an ultrasoun" in C weeks to see if the cyst has resolve". 7his !atient, however, is sym!tomatic with a C cm cyst that a!!ears to be a "ermoi", which will not resolve s!ontaneously. )he, therefore, requires sur$ery. 7o !rescribe the oral contrace!tive !ill 'choice C) may hel! to !revent future ovarian cysts but it will not resolve this cyst, which requires sur$ical mana$ement. 7o !erform hysterosco!y 'choice %) woul" not be in"icate". 2ysterosco!y is use" to evaluate the uterine cavity an" woul" not be use" for mana$ement of an a"ne*al mass. D) (*!lanation: 7he correct answer is

A. In the F.)., he!atitis C virus '2C1) is the most common bloo"4borne infection. 2C1 is a sin$le4stran"e" 6NA virus that is transmitte" by bloo"4borne transmission or throu$h se*ual contact. With the "isease bein$ so !revalent4it affects 0.I million Americans4it is not rare to fin" a !re$nant !atient with he!atitis C. In fact, it a!!ears to infect as much as >.C= of the !re$nant !o!ulation. )tu"ies that have been !erforme" so far show that the rate of infection of infants born to he!atitis C4 !ositive, 2I14ne$ative mothers is about /=. 2e!atitis C transmission throu$h breast milk has not been clearly !roven. reast4fe" an" bottle4fe" infants have a rate of infection that is a!!ro*imately ?=. 7herefore, the !atient shoul" be tol" that casual contact is !ermitte" an" that currently there is no evi"ence that breast4fee"in$ increases 2C1 transmission to the baby. 7o state that there is stron$ evi"ence that breast4fee"in$ increases 2C1 transmission to the baby 'choice ) is incorrect. As e*!laine" above, the available stu"ies "o not "emonstrate that breast4fee"in$ increases 2C1 transmission. 7o state that com!lete isolation is not nee"e" but breast4fee"in$ is !rohibite" 'choice C) is incorrect for the reasons "etaile" above. 7o state that the !atient shoul" be com!letely isolate" from the baby 'choice %), or that casual contact with the baby is !rohibite" 'choice () are both incorrect for the reasons "etaile" above. If !atients with he!atitis C were not allowe" contact with their infants, they woul" have to $ive them u!, because he!atitis C is a chronic "isease. -ifty !ercent of !atients with 2C1 "evelo! biochemical evi"ence of chronic liver "isease. 2e!atitis C is not like varicella4#oster 'chicken!o*), where a neonate can be isolate" from the mother until she is no lon$er infectious. @) (*!lanation: 7he correct answer is . 7he most common me"ical com!lication of !re$nancy is infection of the urinary tract. ecause of the anatomic an" !hysiolo$ic chan$es that occur "urin$ !re$nancy, asym!tomatic bacteriuria is more likely to become sym!tomatic an" there is also an increase" !ro$ression to !yelone!hritis "urin$ !re$nancy. (scherichia coli is the causative or$anism in a!!ro*imately @>= of cases of F7I while other $ram4ne$ative or$anisms 'e.$., 5lebsiella, (nterobacter, an" &roteus s!ecies) an" $ram4!ositive cocci 'e.$. enterococci an" $rou! stre!tococci) are res!onsible for the remain"er. F7I in !re$nancy can be treate" with a 04"ay course of antibiotics inclu"in$ trimetho!rim4 sulfametho*a#ole, nitrofurantoin, an" ce!hale*in. It is essential to "ocument successful treatment with a follow4u! urine culture :> "ays after treatment. All women who are treate" for F7I "urin$ !re$nancy shoul" have !erio"ic rescreenin$ for infection with urine cultures or urine "i!stick for nitrites or leukocyte esterase. If a woman "evelo!s a secon" infection, as this !atient has, she shoul" be retreate" an" then !lace" on chronic su!!ression with !ro!hylactic antibiotics. 7he "ru$ of choice for such !ro!hyla*is is nitrofurantoin once a "ay or sulfiso*a#ole once a "ay. 7o state that no further treatment or "ia$nostic stu"y is necessary 'choice A) is incorrect. Women with bacteriuria "urin$ !re$nancy are at increase" risk of "evelo!in$ !yelone!hritis an" are at hi$her risk for low birth wei$ht an" !reterm "eliveries. 7herefore, this !atient shoul" be !lace" on !ro!hylactic antibiotics for the remain"er of the !re$nancy. 7o !lace the !atient on intravenous antibiotics for the remain"er of the !re$nancy 'choice C) woul" not be

in"icate". Once a "ay oral thera!y is usually sufficient to !revent recurrence of the infection. Intravenous !yelo$ram 'choice %) an" ab"ominal C7 scan 'choice () result in si$nificant fetal e*!osure to ra"iation. 7hey shoul" only be !erforme" when absolutely necessary. 7his !atient has a secon" F7I, which "oes not require that either of these stu"ies be !erforme". I) (*!lanation: 7he correct answer is C. 7his !atient has si$ns an" sym!toms that are most consistent with en"ometritis. &ost!artum en"ometritis is believe" to result from or$anisms ascen"in$ from the va$ina an" causin$ a !olymicrobial infection of the en"ometrium. Infection may also involve the myometrium an" !arametrial tissues. &atients with en"ometritis ty!ically !resent with fever an" chills, lower ab"ominal !ain, a foul4smellin$ va$inal "ischar$e, an" malaise. (*amination is si$nificant for fever, ab"ominal ten"erness, an" uterine ten"erness. Cesarean section is the ma<or risk factor for !ost!artum en"ometritis. &atients un"er$oin$ cesarean section have a several4fol" hi$her risk of "evelo!in$ en"ometritis com!are" with those havin$ a va$inal "elivery. 7he treatment of choice for en"ometritis followin$ a cesarean section must inclu"e anaerobic covera$e, alon$ with $ram4!ositive an" $ram4 ne$ative covera$e. 7herefore, the treatment of choice is clin"amycin an" $entamicin. Am!icillin 'choice A) an" am!icillin4$entamicin 'choice ) fail to cover the anaerobic or$anisms that !lay an im!ortant role in the !atho!hysiolo$y of !ost4cesarean section en"ometritis. Clin"amycin4metroni"a#ole 'choice %) an" metroni"a#ole 'choice () have $oo" activity a$ainst anaerobic or$anisms, but fail to cover $ram4ne$ative or$anisms. :>) (*!lanation: 7he correct answer is A. In the current era of me"ical cost containment, !osto!erative hos!ital stays ten" to be si$nificantly shorter than they were in the !ast. 7herefore, it is more essential than ever to make sure that !atients who are "ischar$e" !osto!eratively are, in fact, rea"y for "ischar$e. %ischar$e criteria $enerally inclu"e that the !atient shoul" be alert, able to ambulate 'if this was her !reo!erative level of function), able to tolerate a"equate oral intake, have stable vital si$ns, an" have satisfactory bowel an" urinary tract function. 7his !atient is ; "ays status !ost total ab"ominal hysterectomy an" bilateral sal!in$o4 oo!horectomy '7A24 )O). Althou$h this is a ma<or !roce"ure, it is not uncommon for !atients to be rea"y for "ischar$e at this time. 2owever, this !atient has not ha" a"equate oral intake. 7herefore, this !atient shoul" not be "ischar$e" until she shows evi"ence of bein$ able to tolerate oral intake. &osto!erative infection is frequently seen followin$ 7A24 )O an" it is often a reason to "elay "ischar$e in or"er to treat the infection with antibiotics an" ensure that there is no abscess formation. 7his !atient, however, shows no evi"ence of infection 'choice ). 7herefore, she "oes not nee" to be ke!t hos!itali#e" lon$er on this basis. ,ow hematocrit 'choice C) is a concern in a !osto!erative !atient because it may make the !atient sym!tomatic an" it may be a si$n of continue" blee"in$.

7his !atient, however, has a hematocrit of 0C=, which is well within the e*!ecte" ran$e after 7A24 )O. It is im!ortant to make sure that a !osto!erative !atient is able to urinate normally. +any !atients have "ifficulty with urination secon"ary to $eneral anesthesia, use of the -oley catheter, or bla""er "enervation. 7his !atientAs urinary tract function 'choice %), however, is normal. Fnstable vital si$ns are a very $oo" reason to kee! a !osto!erative !atient hos!itali#e" for a lon$er !erio" of time. 2owever, this !atientAs vital si$ns 'choice () are normal. ::) (*!lanation: 7he correct answer is . Chorioamnionitis is an infection that can "evelo! at any time before an" "urin$ "elivery. 7he most common fin"in$s in !atients with chorioamnionitis are a fever an" uterine ten"erness. An elevate" fetal heart rate is also often seen. 7his !atient has a tem!erature elevation an" uterine ten"erness, which make the "ia$nosis of chorioamnionitis. It is essential that antibiotics be starte" imme"iately because !rom!t initiation of antibiotics, once the "ia$nosis of chorioamnionitis is ma"e, results in better maternal an" neonatal outcomes than if thera!y is "elaye". It is also essential that broa"4 s!ectrum antibiotic thera!y be chosen because a mi*ture of or$anisms is usually involve" inclu"in$ aerobes an" anaerobes. 7he most frequently use" re$imen is am!icillin or !enicillin with $entamicin. In terms of the mo"e of "elivery, va$inal "elivery is acce!table in !atients with chorioamnionitis. While it is "esirable to have an e*!e"itious "elivery, chorioamnionitis is not an in"ication for cesarean "elivery. 7o wait to a"minister antibiotics to the mother after va$inal "elivery 'choice A) woul" not be correct, as the "elay woul" "e!rive both the mother an" the fetus of the beneficial effects of the antibiotics. 7o !erform cesarean "elivery 'choice C) or to !erform cesarean "elivery an" then a"minister antibiotics to the mother 'choice %) woul" not be in"icate". As e*!laine" above, when a woman has chorioamnionitis, it is "esirable to e*!e"ite "elivery, but cesarean "elivery shoul" be !erforme" only for obstetric in"ications. 7o !erform intra4amniotic in<ection of antibiotics 'choice () woul" not be in"icate". Intra4 amniotic in<ection of antibiotics "urin$ labor is not a thera!y use" to treat chorioamnionitis "urin$ labor. :;) (*!lanation: 7he correct answer is %. Asym!tomatic bacteriuria is !resent in ; to I= of !re$nant women. An association between asym!tomatic bacteriuria an" !reterm "eliveryBlow birth wei$ht has been "emonstrate". 7herefore, all !re$nant women shoul" be screene" for asym!tomatic bacteriuria early in the !re$nancy, an" women who "emonstrate bacteriuria '"efine" as a clean4catch, mi"stream urine s!ecimen with ;/,>>> to :>>,>>> colony formin$ units !er milliliter of a sin$le or$anism) shoul" be treate". (. Coli is the or$anism that is isolate" in rou$hly @>= of cases while other $ram4ne$ative or$anisms 'e.$., 5lebsiella, (nterobacter, an" &roteus s!ecies) an" $ram4!ositive cocci

'e.$. enterococci an" $rou! stre!tococci) are res!onsible for the remain"er. Antibiotic sensitivities are often available at the time of "ia$nosis of the asym!tomatic bacteriuria, which will allow for correct choice of me"ications. A 04"ay course of antibiotics may be $iven. &ossible choices inclu"e trimetho!rim4sulfametho*a#ole, nitrofurantoin, an" ce!hale*in. Am!icillin an" amo*icillin can also be use", but u! to :B0 of (. Coli isolates will be resistant to these "ru$s. 7herefore, these "ru$s shoul" be chosen only if the or$anism is sensitive. :> "ays after com!letin$ the me"ication, the !atient shoul" have a follow4u! urine culture as a test4of4cure. Waitin$ to see if sym!toms "evelo! 'choice A) is not a!!ro!riate. acteriuria, even without sym!toms, has been shown to be associate" with !reterm labor an" low birth wei$ht. Asym!tomatic bacteriuria shoul", therefore, be treate". 6esen"in$ another urine culture 'choice ) woul" not be the most a!!ro!riate ne*t ste!. 7his !atient alrea"y has "emonstrable bacteriuria an" treatment shoul" be institute". A"mittin$ for intravenous antibiotics 'choice () or obtainin$ a renal ultrasoun" 'choice C) woul" not be necessary. 7his !atient has asym!tomatic bacteriuria an" not !yelone!hritisG therefore, a 04"ay course of oral antibiotics followe" by a re!eat culture :> "ays later is all that is necessary. :0) (*!lanation: 7he correct answer is C. .estational "iabetes is "efine" as $lucose intolerance that either has its onset or its first reco$nition "urin$ !re$nancy. .estational "iabetes is usually "ia$nose" by means of oral $lucose tolerance testin$. &atients with $estational "iabetes an" normal fastin$ $lucose levels have two ma<or risks. 7he first is fetal macrosomia. Women with $estational "iabetes are known to have lar$er babies, an" this creates an increase" risk of com!lications of "elivery inclu"in$ shoul"er "ystocia an" cesarean "elivery. 7he secon" risk is of the eventual "evelo!ment of overt "iabetes. -ifty !ercent of women with $estational "iabetes will $o on to "evelo! overt "iabetes within the ne*t ;> years. &atients with $estational "iabetes an" abnormal fastin$ $lucose levels "o have an increase" risk of stillbirth. 7o state that $estational "iabetes is associate" with fetal anomalies 'choice A) is incorrect. 2owever, !atients with overt "iabetes "o have an increase" risk of fetal anomalies. 7o state that $estational "iabetes is associate" with intrauterine $rowth restriction 'choice ) is not correct. .estational "iabetes is associate" with macrosomia. 7o state that $estational "iabetes is not associate" with future "iabetes is incorrect 'choice %), as e*!laine" above. 7o state that $estational "iabetes with normal fastin$ $lucose is associate" with stillbirth 'choice () is incorrect. 2owever, overt "iabetes an" $estational "iabetes with abnormal fastin$ $lucose levels 'class A;) are associate" with stillbirth. :?) (*!lanation: 7he correct answer is %. +itral valve stenosis is one of the more common valvular lesions seen in !re$nancy. 7he most common cause of mitral stenosis is rheumatic en"ocar"itis. %urin$ normal !re$nancy there is an increase in the car"iac out!ut an" an increase in !reloa" an"

circulatin$ volume. &atients with mitral stenosis have a fi*e", "ecrease" valve area, which !laces them at risk for the "evelo!ment of !ulmonary hy!ertension an" !ulmonary e"ema. Control of arrhythmias is absolutely essential in these !atients because they are at increase" risk, $iven the left atrial enlar$ement that often $oes alon$ with their mitral stenosis. ,abor an" "elivery can be a !articularly "an$erous time for these !atients. 7herefore, !atients with si$nificant mitral stenosis shoul" be monitore" invasively usin$ a )wan4.an# catheter. It is recommen"e" that the secon" sta$e of labor be shortene" usin$ force!s or vacuum to !revent e*cess maternal 1alsalva efforts an" maternal tachycar"ia. Asthma 'choice A) is not an in"ication for force!s4assiste" va$inal "elivery. In terms of mo"e of "elivery, asthmatic !atients may be mana$e" like any other !atient in the secon" sta$e of labor. .rou! )tre!tococcus coloni#ation 'choice ) is an in"ication for intravenous !enicillin or clin"amycin 'if the !atient has an aller$y to !enicillin). 7hese antibiotics are $iven to !revent . ) se!sis in the neonate. . ) coloni#ation is not an in"ication for force!s4assiste" va$inal "elivery. 2er!es 'choice C) can be transmitte" to the fetus at the time of "elivery. 7herefore, when lesions are !resent in the birth canal, most obstetricians recommen" cesarean "elivery. A history of her!es outbreaks, as this !atient has, is not an in"ication for force!s. 7o state that this !atient woul" not benefit from a force!s4assiste" va$inal "elivery 'choice () is incorrect. As e*!laine" above, $iven this !atientAs mitral stenosis, force!s4assiste" va$inal "elivery woul" be recommen"e". :/) (*!lanation: 7he correct answer is . A si$nificant bo"y of evi"ence has "evelo!e" that transmission rates of 2I1 from mother to infant can be "ecrease" throu$h the use of me"ications an" cesarean "elivery. 7he &e"iatric AI%) Clinical 7rials .rou! '&AC7.) >DC 3i"ovu"ine 6e$imen was shown to "ecrease the rate of transmission from ;/= to @=. 7his re$imen consiste" of 3%1 bein$ $iven ante!artum an" intra!artum to the mother an" !ost!artum to the infant. +ore recent evi"ence is accumulatin$ that the mo"e of "elivery also affects transmission rates. 7he combination of 3%1 thera!y an" cesarean "elivery "ecreases the risk of transmission to a!!ro*imately ;=. ut, the "ecrease in transmission with cesarean "elivery occurs re$ar"less of whether the !atient is receivin$ antiretroviral thera!y. 7hus, cesarean "elivery shoul" be offere" to 2I14!ositive women to !revent transmission. %elivery at 0@ weeks is recommen"e" to re"uce the chances that the !atient will $o into labor or ru!ture her membranes. Once these occur, the benefit of cesarean "elivery is re"uce". 7o offer elective c4section after amniocentesis to "etermine lun$ maturity 'choice A) is incorrect. Amniocentesis shoul" be avoi"e", if !ossible, in the 2I14!ositive woman. 7o offer elective c4section at 0? weeks 'choice C) is incorrect. 7o !erform a cesarean "elivery at 0? weeks risks iatro$enic !rematurity in the neonate. Cesarean "elivery !rior to the onset of labor or ru!ture of membranes is the !reference, an" this can be accom!lishe" at 0@ weeks with a lower risk of iatro$enic !rematurity. 7o recommen" force!s4assiste" va$inal "elivery 'choice %) or va$inal "elivery 'choice () is incorrect. 7he "ecision of which mo"e of "elivery to choose ultimately belon$s to the

!atient. ut, va$inal "elivery woul" not be recommen"e", as cesarean "elivery has been shown to "ecrease transmission rates. :C) (*!lanation: 7he correct answer is A. Constitutional "elay is normal !ubertal !ro$ression at a "elaye" rate or onset. 7he avera$e a$e at menarche is :; :B; years, but it may be "elaye" until :C or may be$in as early as a$e :>. %ysfunctional uterine blee"in$ 'choice ) results when the en"ometrium has !roliferate" un"er estro$en stimulation, an" then be$ins to slou$h an" causes irre$ular !ainless blee"in$. 7his is common in youn$er a"olescents who have not been menstruatin$ lon$. %ysmenorrhea 'choice C) is !ain associate" with menstrual cycles, an" this a"olescent is not menstruatin$ yet. &rimary amenorrhea 'choice %) is a "elay in menarche with no menstrual cycles or secon"ary se* characteristics by :? years of a$e or no menses with secon"ary se* characteristics by :C years of a$e. 7his a"olescent has secon"ary characteristics but is not yet :C years of a$e. )econ"ary amenorrhea 'choice () is the absence of menses for at least three cycles after re$ular cycles have been !resent. :D) (*!lanation: 7he correct answer is A. 2y!er!rolactinemia is the cause in a!!ro*imately :> to ;>= of cases of amenorrhea. It is known that elevate" !rolactin levels alter the hy!othalamic4!ituitary4ovarian a*is such that ovulation is su!!resse" an" menses "o not occur. 7his !atient has amenorrhea, $alactorrhea 'i.e., a milky "ischar$e from the breasts), an" an elevate" !rolactin level. All of these fin"in$s are consistent with hy!er!rolactinemia, likely comin$ from a !ituitary microa"enoma. 7he fact that no mass is seen on the hea" +6I is also consistent with a !ituitary microa"enoma, as small microa"enomas may not be visuali#e". 7he treatment of choice for this !atient is with bromocri!tine. romocri!tine is a "o!amine a$onist that has been shown to "ecrease !rolactin levels an" brin$ about a return of ovulation an" menses. 7he re4establishment of ovulation is es!ecially im!ortant for this !atient who wishes to conceive. %iclo*acillin 'choice ) is often use" to treat a breast infection, which can occur in a nursin$ mother. 7his !atient, however, "oes not have fin"in$s consistent with breast infection. 6ather, the ni!!le "ischar$e is secon"ary to the !atientAs elevate" !rolactin levels. +a$nesium sulfate 'choice C) is use" in obstetrics to !revent sei#ures in !atients with !re4eclam!sia an" to sto! the uterus from contractin$ in !atients with !reterm labor. It is not in"icate" for the treatment of hy!er!rolactinemia. 7he oral contrace!tive !ill 'choice %) woul" not be a!!ro!riate as this is a youn$ woman who wishes to become !re$nant. If she "i" not "esire !re$nancy, the oral contrace!tive !ill woul" be a!!ro!riate thera!y. One of the ma<or concerns in youn$ women with microa"enomas is that "ecrease" levels of estro$en will lea" to bone loss an" the eventual "evelo!ment of osteo!orosis. 7he oral contrace!tive !ill, by !rovi"in$ "aily estro$en an" !ro$estin, will hel! to !revent bone loss. 7hyro*ine 'choice () is use" in

!atients with hy!othyroi"ism. 7his !atient has a normal 7)2 an" no evi"ence of hy!othyroi"ism, an" woul", therefore, not nee" thyro*ine. :@) (*!lanation: 7he correct answer is A. Influen#a !neumonia "urin$ !re$nancy can be a severe illness. Normally Hthe fluH is a self4limite" illness that lasts 04? "ays an" !ro"uces few ma<or sequelae. 2owever, !atients with influen#a !neumonia "urin$ !re$nancy can "evelo! hi$h fever, malaise, cou$h, an" hea"ache. In some cases a bacterial su!erinfection will occur 'often with )ta!hylococcus aureus), which can lea" to !eribronchial infiltrates, cavitation, an" a !leural effusion. Current recommen"ations are that !re$nant women who will be in the secon" or thir" trimester "urin$ the flu e!i"emic season shoul" be $iven the influen#a vaccination. Also, !re$nant women with si$nificant me"ical !roblems shoul" be $iven the vaccination before the influen#a season, re$ar"less of trimester. 7he measles 'choice ), mum!s 'choice C), an" rubella 'choice %) vaccines are live attenuate" vaccines. 7heir use "urin$ !re$nancy is contrain"icate". 7he varicella 'choice () vaccination is use" to !revent chicken!o*. It is a live4virus vaccineG therefore, its use "urin$ !re$nancy is also contrain"icate". :I) (*!lanation: 7he correct answer is . 7his !atient is "emonstratin$ an abnormal labor !attern with arrest of "ilation. 7he normal !attern of labor is one of continue" !ro$ression. Whether a !atient is in the latent !hase or the active !hase, there shoul" be a $ra"ual !ro$ression with an increase in the amount of cervical "ilation. 7his !atient, however, has sto!!e" "ilatin$ an" has ha" her contractions s!ace out consi"erably. An arrest of labor like this can be cause" by several reasons: contractions may not be a"equateG the fetus may have a mal!resentationG or the maternal !elvis may not be able to accommo"ate the fetus. In this case it a!!ears that the contractions are not a"equate, so at this !oint, it woul" be reasonable to $ive intravenous o*ytocin in an effort to re4establish a contraction !attern that can effect a va$inal "elivery. (*!ectant mana$ement 'choice A) woul" not be the most a!!ro!riate ne*t ste!. 7he !atient is clearly "emonstratin$ a "ysfunctional labor !attern at this !oint. 7o Hwatch an" waitH in the face of insufficient uterine contractions is to !lace the !atient at risk of an even lon$er labor an" the corres!on"in$ly hi$her risk of infection. Cesarean "elivery 'choice C) woul" not be the most a!!ro!riate ne*t ste! in mana$ement. 7his !atient may very well nee" a cesarean "elivery if she is truly unable to !ro$ress in labor. 2owever, it is worth attem!tin$ a va$inal "elivery in this multi!arous !atient who has alrea"y ha" two va$inal "eliveries. 7o attem!t a force!s4assiste" va$inal "elivery 'choice %) or a vacuum4assiste" va$inal "elivery 'choice () woul" be contrain"icate". 7his !atientAs cervi* is only / centimeters "ilate". -orce!s an" vacuum cannot be attem!te" in !atients unless they are fully "ilate" an" at M; station or lower.

;>) (*!lanation: 7he correct answer is %. 7he cesarean "elivery rate in the F.). is rou$hly ;/=. +uch effort has been !ut into tryin$ to lower this rate. One thir" of all cesarean births are a result of elective re!eat cesarean "elivery. 7herefore, much attention has been focuse" on va$inal birth after cesarean '1 AC). A few "eca"es a$o, there was an obstetric "ictum that Honce a cesarean, always a cesarean.H 7his is no lon$er the case. )ome women are allowe" to attem!t va$inal birth after a !rior cesarean "elivery. An", in fact, the attem!t is often successful. (stimates are that a!!ro*imately D>= of all women that attem!t 1 AC will be successful. 7his !atient has ha" one !rior cesarean "elivery, an" it was throu$h a low transverse uterine hysterotomy. 6i$ht now, she has no contrain"ications to 1 A C. 7herefore, a 1 AC attem!t may be trie". If the success rate of 1 AC were >= 'choice A) or even ;/= 'choice ), the to!ic woul" be a non4issue. 7he fact that the success rate of 1 AC is so hi$h is what makes the choice between re!eat cesarean an" 1 AC more com!licate". />= 'choice C) is a!!ro*imately the success rate in women who attem!t 1 AC who ha" a !rior cesarean for "ystocia. Women with a !rior cesarean "elivery for "ystocia have a 1 AC success rate of a!!ro*imately />= to D>=. Althou$h this rate is still $oo", it is consistently lower than the rate for women with non4recurrin$ in"ications, such as a non4reassurin$ fetal tracin$. Attem!ts at va$inal "elivery are not :>>= 'choice () successful even in women who have never ha" a cesarean "elivery. In fact, the success rate for va$inal "elivery in women who have not un"er$one !revious cesarean "elivery is about D>=44the same success rate as women attem!tin$ 1 AC with a non4recurrin$ in"ication. ;:) (*!lanation: 7he correct answer is (. (n"ometrial cancer is the most common $ynecolo$ic cancer in women a$es ?/ an" ol"er. 7he main factor that !re"is!oses a woman to the "evelo!ment of en"ometrial cancer is e*!osure to uno!!ose" estro$en, whether en"o$enous or e*o$enous. (n"o$enous factors inclu"e, early menarche, late meno!ause, chronic anovulation, estro$en4secretin$ ovarian tumors, an" obesity. (*o$enous factors inclu"e the in$estion of uno!!ose" estro$en 'as with estro$en re!lacement thera!y). 2y!ertension an" "iabetes have also been associate" with en"ometrial cancer, thou$h this relationshi! may likely be relate" to obesity. 7his !atient has en"ometrial cancer on the basis of her en"ometrial bio!sy result. 7he correct mana$ement for this !atient is with total ab"ominal hysterectomy, bilateral a"ne*ectomy, an" !ossible lym!h no"e sam!lin$. Chemothera!y 'choice A) woul" not be the most a!!ro!riate ne*t ste! in mana$ement. If the !atient were not a sur$ical can"i"ate, because of her obesity, for e*am!le, then ra"iation thera!y coul" be a"ministere". Cone bio!sy 'choice ) is use" in the "ia$nosis an" mana$ement of cervical cancer. It woul" not be use" for this !atient with an en"ometrial bio!sy showin$ en"ometrial cancer. %ilation an" curetta$e 'choice C) or hysterosco!y 'choice %) woul" not be the most a!!ro!riate ne*t ste! in mana$ement. 7he "ia$nosis of

en"ometrial cancer has been ma"e on the basis of the en"ometrial bio!sy. 7herefore, the most a!!ro!riate ne*t ste! in mana$ement is to treat the !atient throu$h hysterectomy or, if hysterectomy is not !ossible because of obesity or me"ical "isease, ra"iation. ;;) (*!lanation: 7he correct answer is A. 7his !atient has a face !resentation. 7y!ically, a fetus in labor is as an occi!ut !resentation. In certain rare instances 'rou$hly : in />> "eliveries), however, the fetus is in a face !resentation. Causes of face !resentation inclu"e an anence!halic fetus, !elvic contraction, an" hi$h !arity. A va$inal "elivery is !ossible when the fetus is in a mentum anterior !osition 'i.e., the fetal chin is oriente" towar" the maternal !ubic sym!hysis.) 7he fetus can fle* its hea", thereby allowin$ "elivery. 7his !atient is in active labor with contractions every ; minutes an" / cm of cervical "ilation. 7he fetus is in mentum anterior !osition. 7herefore, e*!ectant mana$ement is the most a!!ro!riate ne*t ste!. O*ytocin au$mentation 'choice ) is not in"icate". 7his !atient is in active labor on her own an" therefore "oes not nee" o*ytocin to au$ment it. -orce!s "elivery 'choice C) woul" not be in"icate". -orce!s are not use" !rior to full "ilation of the cervi*. Also, with a non4verte* !resentation, force!s woul" be contrain"icate". 1acuum "elivery 'choice %) is not in"icate". As with force!s, vacuum "elivery is not !erforme" !rior to full "ilation of the cervi*. With a face !resentation, vacuum "elivery woul" be contrain"icate". Cesarean section 'choice () woul" not be in"icate". 1a$inal "elivery is !ossible with face !resentation. ;0) (*!lanation: 7he correct answer is A. 7he !resence of a classic uterine scar is an absolute contrain"ication to a va$inal birth after cesarean '1 AC). A classic uterine scar is a vertical incision into the uterus that e*ten"s from the lower uterine se$ment u! into the active myometrial !ortion towar" the fun"us of the uterus. &atients with a !revious classic cesarean "elivery have rou$hly a :>= risk of uterine ru!ture. 7herefore, these !atients shoul" have an elective re!eat cesarean "elivery when the fetus is mature. .rou! )tre!tococcus '. )) urine infection 'choice ) is not a contrain"ication to va$inal "elivery. &atients with . ) urine infection are allowe" to have a va$inal "elivery but must receive I1 antibiotics "urin$ labor to !revent . ) invasive "isease of the newborn. &revious a!!en"ectomy 'choice C), or other intra4ab"ominal sur$ery, is not a contrain"ication to va$inal "elivery. &rior cesarean "elivery for non4reassurin$ fetal tracin$ 'choice %) is not a contrain"ication to va$inal "elivery. &atients with this in"ication for !rimary cesarean "elivery have a!!ro*imately a D>= rate of success with 1 A C. Women with two !rior cesarean "eliveries 'choice () may un"er$o a trial of labor '1 AC). 7his is the case if the two !rior cesarean "eliveries were low4transverse hysterotomies. 2owever, the !atient shoul" be cautione" that the risk of ru!ture "oes increase with the number of !revious cesarean "eliveries.

;?) (*!lanation: 7he correct answer is A. 7amo*ifen is known to act as an estro$en a$onist at the level of the en"ometrium. Numerous stu"ies have shown that women on tamo*ifen "evelo! chan$es in the en"ometrium inclu"in$ !oly!s, hy!er!lasia, an" cancer. 2y!er!lasia runs a continuum from cystic $lan"ular hy!er!lasia to aty!ical hy!er!lasia. &atients with aty!ical hy!er!lasia are at si$nificantly increase" risk for the eventual "evelo!ment of en"ometrial cancer. 7hus, in a !atient who is takin$ tamo*ifen for breast cancer !revention an" "evelo!s aty!ical en"ometrial hy!er!lasia, the tamo*ifen shoul" be sto!!e". If there is a nee" to continue the tamo*ifen, then hysterectomy shoul" be consi"ere". 7o increase the tamo*ifen "ose 'choice ) woul" be contrain"icate". 7his !atient has aty!ical hy!er!lasia, likely cause" by the tamo*ifen. Increasin$ the "ose will only e*acerbate the !roblem. 7o re!eat the en"ometrial bio!sy 'choice C) woul" not be the most a!!ro!riate ne*t ste! in mana$ement. 7he ne*t ste! shoul" be to "iscontinue the tamo*ifen. 7he !atient shoul" then have a re!eat en"ometrial bio!sy in several months to ensure that there is no !ro$ression of the hy!er!lasia. 7o sche"ule a !elvic ultrasoun" 'choice %) woul" not be the most a!!ro!riate ne*t ste! in mana$ement. 7his !atient has known aty!ical hy!er!lasiaG thus, the tamo*ifen shoul" be sto!!e" first. &elvic ultrasoun" can be use" to evaluate the en"ometriumG however, in this case, re$ar"less of what the ultrasoun" shows, the !atholo$y reveals aty!ical hy!er!lasia. 7o switch the !atient to estro$en 'choice () woul" be absolutely contrain"icate". Fno!!ose" estro$en woul" worsen the en"ometrial chan$es. ;/) (*!lanation: 7he correct answer is C. 7he &a! smear has been shown to be a hi$hly effective screenin$ test for cervical cancer. 7he &a! test was intro"uce" in the F.). rou$hly /> years a$o, an" since that time the mortality rate from cervical cancer has "ecrease" by D>=. 7he main "rawbacks to &a! testin$ are that many women "o not $et a re$ular 'or any) &a! smear an" that the test has a hi$h false4ne$ative rate. 7hat is, a $iven &a! smear may be rea" as ne$ative when, in fact, the woman has abnormal cytolo$y. 7he reason for this false ne$ative rate is that there may be errors in sam!lin$, !re!aration, screenin$, an" inter!retation, such that abnormal cells are misse". 9et, if a woman has a yearly &a! test, it is assume" that these abnormal cells will eventually be "iscovere". ecause the natural history of most cervical cancers is believe" to be a $ra"ual !ro$ression over many years, then annual screenin$ 'even with a hi$h false4ne$ative rate) will lea" to lesions eventually bein$ "iscovere" an" a!!ro!riate treatment bein$ $iven. Women shoul" have an annual &a! test when they be$in havin$ se*ual intercourse or at the a$e of :@, whichever comes first. Colonosco!y 'choice A) is use" to screen for colon cancer in some at4risk !atients. 7his !atient is not hi$h4risk an" therefore, at a$e :@, "oes not nee" to have a colonosco!y. 7he mammo$ram 'choice ) is use" to screen for breast cancer. Women shoul" be$in havin$

re$ular mammo$rams at a$e ?>. &elvic ultrasoun" 'choice %) is not use" as a screenin$ test. Certain stu"ies have been "one to evaluate whether !elvic ultrasoun" is a $oo" screenin$ test for ovarian cancer. On the basis of these stu"ies, however, !elvic ultrasoun" is not recommen"e" for this !ur!ose. )i$moi"osco!y 'choice () is also use" to screen for colon cancer. As e*!laine" above, this !atient is not hi$h4risk an" therefore "oes not nee" a si$moi"osco!y. ;C) (*!lanation: 7he correct answer is . Once !atients reach ?; com!lete" weeks of $estation, many !hysicians will in"uce labor for !ost4term !re$nancy. 7his is "one to avoi" the uncommon but catastro!hic outcome of fetal "emise an" the hi$her rates of !lacental insufficiency that "evelo! as !atients $et further !ost4term. &rosta$lan"in '&.(;) $el is an effective a$ent to use for labor in"uction. It has been shown to im!rove the isho!As score, to shorten the len$th of labor an" "elivery, to "ecrease the amount of o*ytocin nee"e", an" to "ecrease the cesarean "elivery rate. 7he main com!lication from its use is uterine hy!erstimulation. 7his hy!erstimulation is "efine" as an increase" frequency of contractions '$reater than / every :> minutes) or an increase" len$th of each contraction '$reater than ; minutes) with evi"ence of fetal "istress. When this hy!erstimulation occurs, the !atient may be treate" with I1 or subcutaneous terbutaline. 7his me"ication usually has a ra!i" onset of action in resolvin$ hy!erstimulation. I1 ma$nesium sulfate can also be use". 7o a"minister $eneral anesthesia 'choice A) woul" be incorrect. 7here are occasions in which the fetal heart rate tracin$ ra!i"ly "eteriorates an" emer$ency cesarean "elivery is nee"e". On these occasions, it may be necessary to a"minister $eneral anesthesia to the mother "urin$ the cesarean. In this case, however, more conservative measures shoul" be trie" !rior to cesarean "elivery. 7o !erform amnioinfusion 'choice C) woul" be incorrect. Amnioinfusion can be use" when a !atient has ru!ture" membranes an" "ecelerations of the fetal heart rate or thickene" meconium. It is not use" with intact membranes. 7o start o*ytocin 'choice %) woul" be contrain"icate". O*ytocin is known to cause uterine hy!erstimulation, as is !rosta$lan"in '&.(;) $el. O*ytocin woul" not be $iven to a !atient in the mi"st of uterine hy!erstimulation. 7o !erform cesarean "elivery 'choice () woul" be incorrect for the reasons "etaile" above. ;D) (*!lanation: 7he correct answer is A. Cystic fibrosis 'C-) is an autosomal4recessive "isease that is common in North American Caucasians of (uro!ean ancestry. In this !o!ulation, the frequency of the "isease is : in ;/>> live births. 7he carrier rate is a!!ro*imately : in ;/ in"ivi"uals. 7he outcome of !atients with C- is hi$hly variable. )ome will "ie in infancy from com!lications of meconium ileus, whereas others will live beyon" the a$e of />. 7he usual clinic manifestations inclu"e !ulmonary "isease with bronchiectasis, !ancreatic

insufficiency, an" failure to thrive. 7he $ene for the "isease is known. 2owever, there are more than />> mutations that can cause C -. 7he most common mutation, which causes D/= of cases in Caucasians, is referre" to as "elta4-/>@. 7he C- $ene has been clone", an" it is !ossible to !erform screenin$ on cou!les. .enetic techniques can also be use" to "etermine whether the fetus has the relevant mutations. In this !atient, with her an" her husban"As family histories, screenin$ woul" be available an" a!!ro!riate. 7o state that screenin$ is ina!!ro!riate in her case 'choice ) is incorrect. 7his !atient has relatives with C- an" so "oes her husban". )he is concerne" about the !ossibility of havin$ a chil" with this "isease. )creenin$ is available an" a!!ro!riate in her case. 7o state that screenin$ is man"atory 'choice C) is ina!!ro!riate. Whether to un"er$o screenin$ for a $enetic "isease is a very !ersonal choice. 7his !atient may not want to know whether she is a carrier or whether her fetus is affecte". +any !atients with $enetic "iseases or with family histories of $enetic "isease feel this way, an" screenin$ is certainly not man"atory. 7o state that screenin$ is not available 'choice %) is incorrect. As state" above, C- screenin$ is available. 7o state that screenin$ is unnecessary because she has a : in ? chance of havin$ an affecte" chil" 'choice () is incorrect. If she an" her husban" were both carriers with the same mutation, the risk woul" be : in ?. 2owever, we "o not know this. Althou$h they both have !ositive family histories, neither may be a carrier. ;@) (*!lanation: 7he correct answer is C. Fno!!ose" estro$en is known to cause en"ometrial hy!er!lasia an" cancer. (stro$en has "irect effects on the $rowth an" "evelo!ment of the en"ometrium. )tu"ies have shown that the a""ition of a !ro$estin can !rotect a woman from the "evelo!ment of en"ometrial hy!er!lasia an" that the a""ition of a !ro$estin to women with en"ometrial hy!er!lasia can lea" the en"ometrium to revert to normal. 7hus, any woman with a uterus who is on estro$en thera!y shoul" also be on a !ro$estin to !rotect her en"ometrium. 7his is usually "one by !lacin$ the !atient on "aily estro$en an" !ro$esterone or on cyclic !ro$esterone. &ro$estins "o not !rotect a$ainst the "evelo!ment of breast cancer 'choice A). In fact, there is evi"ence that !ro$estins may stimulate the $rowth of breast tumors. reast !ain 'choice ) is often a result of !ro$estin thera!y. +oo" chan$es 'choice %) an" wei$ht $ain 'choice () are well4known si"e effects of !ro$estins. ;I) (*!lanation: 7he correct answer is %. 7his !atient has fin"in$s that are most consistent with con"yloma acuminata, or $enital warts. Con"yloma acuminata is cause" by the human !a!illomavirus. 7his virus, of which there are many "ifferent subty!es, infects e!i"ermal cells an" can cause warty $rowths. When the virus affects skin cells on the han"s, the result is the common warts that are often seen in chil"ren. When the virus affects cells on the !erineum, the result is con"yloma acuminata. %ia$nosis is ma"e on the basis of the classic, verrucous

'cauliflower4like) a!!earance of the lesions. 7reatment is with local "estruction. 7his local "estruction can be achieve" in a variety of ways inclu"in$ with cryothera!y 'i.e. free#in$ of the skin), laser thera!y, trichloroacetic aci" 'i.e. chemical "estruction of the skin), or imiquimo". 2owever, while the lesions themselves are often successfully treate" with these locally "estructive a$ents, the virus is not usually com!letely era"icate" an" recurrences of the lesions may occur. Acyclovir 'choice A) is use" to treat her!es viruses. Con"yloma acuminata is cause" by the human !a!illomavirus an", therefore, acyclovir is not use". &enicillin 'choice ) is an antibiotic effective a$ainst bacteria, an" not the human !a!illomavirus. Cone bio!sy 'choice C) is !erforme" on the cervi* when a !atient has hi$h4$ra"e "ys!lasia or cancer. While there is an association between human !a!illomavirus infection an" cervical "ys!lasia, cone bio!sy woul" not be in"icate" for a !atient on the basis of the !resence of con"yloma. 1ulvectomy 'choice () is !erforme" on !atients for vulvar "ys!lasia or cancer. It is not in"icate" for !atients with con"yloma. 0>) (*!lanation: 7he correct answer is A. It is essential that !hysicians who treat women with "isabilities "o not $ive them substan"ar" care because of their "isability. 7his woman has two risk factors that !lace her at $reater risk of havin$ cervical "ys!lasia: early a$e at first intercourse an" smokin$. oth of these characteristics have been shown to be associate" with !atients who "evelo! cervical "ys!lasia. Current recommen"ations are that women be$in havin$ annual &a! smears at the onset of intercourse or a$e :@, whichever comes first. )ome !hysicians believe that in certain low4risk women with three annual normal &a! smears, the interval of screenin$ may be increase" to every 0 years. Others ar$ue that annual &a! smears shoul" be !erforme" in all women, re$ar"less of risk status. -or this !atient, $iven her risk factors, annual &a! smears shoul" be !erforme". 7he fact that !elvic e*amination is "ifficult for her because of her multi!le sclerosis nee"s to be a""resse" by takin$ a!!ro!riate measures so that the e*amination can be ma"e easier for her. )he shoul" not receive ina"equate screenin$ for cervical cancer because she has a "isability. 7o state that a &a! smear shoul" be !erforme" every 0 years 'choice ) is incorrect. 7his !atient, with her early onset of first intercourse an" current ci$arette smokin$, has risk factors for cervical "ys!lasia an" nee"s annual screenin$. 7o state that a &a! smear shoul" be !erforme" every / years 'choice C) is not correct. ecause of the false4ne$ative rate of the &a! smear, screenin$ every / years woul" risk missin$ many cases of "ys!lasia or cancer. 7o state that a &a! smear shoul" be !erforme" only if there are sym!toms 'choice %) is incorrect. &ro$ressin$ cervical "ys!lasia is ty!ically an asym!tomatic !rocess, an" awaitin$ sym!toms !rior to !erformin$ the &a! smear woul" miss most cases. 7o state that a &a! smear is not necessary 'choice () is not correct. All se*ually active women or women ol"er than :@ nee" to have screenin$ with the &a! smear. Women with "isabilities shoul" not receive substan"ar" care because of their "isability. 0:) (*!lanation:

7he correct answer is (. Increasin$ maternal a$e lea"s to an increase" risk of chromosomal anomalies in the fetus. 7hese anomalies inclu"e trisomy :0, :@, an" ;:. A"vance" maternal a$e also lea"s to increase" rates of the se* chromosome aneu!loi"ies ?D EE9 an" ?D EEE. ecause of this relationshi! between a"vance" maternal a$e an" chromosomal anomalies, many e*!erts su$$est amniocentesis or chorionic villus sam!lin$ in women who will be 0/ years of a$e or ol"er at the time of their "elivery. &aternal a$e has not been shown to be relate" to chromosomal anomalies. 7here is evi"ence that a"vance" !aternal a$e is linke" to an increase" risk of autosomal "ominant mutations, which lea" to "iseases such as neurofibromatosis, achon"ro!lasia, A!ert syn"rome, an" +arfan syn"rome. Increasin$ !aternal a$e also may be associate" with E chromosome mutations that are transmitte" throu$h carrier "au$hters to affecte" $ran"sons. 2owever, these risks are e*cee"in$ly small, an" it is currently not !ossible to screen !renatally for all the autosomal "ominant or E4linke" "iseases that a"vance" !aternal a$e may be associate" with. 7herefore, unlike with women, in whom the a$e of 0/ is usually $iven as the cutoff for chromosomal evaluation of the fetus, there is no a$e cutoff for !aternal risk. 7o state that amniocentesis or chorionic villus sam!lin$ shoul" be consi"ere" for a !aternal a$e above a$e 0> 'choice A), 0/ 'choice ), ?> 'choice C), or ?/ 'choice %) is incorrect. As e*!laine" above, a"vance" !aternal a$e is associate" with autosomal "ominant mutations an" E4linke" mutations. 7hese mutations are very rare, an" we are currently unable to screen for all of these mutations !renatally. 0;) (*!lanation: 7he correct answer is A. &a! testin$ is use" to screen women for cervical cancer. 7he "evelo!ment of cervical cancer is believe" to be a $ra"ual !rocess in which the cervical cells $ra"ually !ro$ress from "ys!lasia to carcinoma in situ to invasive cancer. Cervical cancer is certainly linke" to se*ual activity, as the human !a!illomavirus, which is transmitte" throu$h se*ual contact, is believe" to !lay a causative role. )e*ual intercourse also allows e*!osure to other infectious "iseases an" carcino$ens that may !lay a role in the !rocess. 7herefore, a !atient shoul" be$in havin$ &a! testin$ once she be$ins to en$a$e in se*ual intercourse. If a !atient has not ha" se*ual intercourse by the a$e of :@, &a! testin$ shoul" be$in then. &a! testin$ shoul" be !erforme" yearly, !rimarily because a sin$le &a! test has a hi$h false4ne$ative rate 'i.e., the &a! test has a low sensitivity). 7he sensitivity of &a! testin$ is often quote" as @>=. 7herefore, ; of :> women with abnormal cervical cells will be misse" with &a! testin$. 2owever, if the e*amination is re!eate" every year, as it shoul" be, then the likelihoo" of missin$ the lesion over time is much lower. 7o start at a$e :C 'choice ), :@ 'choice C), ;> 'choice %), or ;: 'choice () is too late for this !atient. Althou$h the !ro$ression to cervical cancer is believe" to be a $ra"ual one, there are more a$$ressive forms that are more ra!i"ly !ro$ressive. Also, if one waits until a$e :C, :@, ;>, or ;:, an" the !atient misses that ne*t a!!ointment or has a false ne$ative on the &a! test, then the "isease will be $iven even further time to !ro$ress. Also, to wait until later to "o &a! testin$ with this !atient is to miss an o!!ortunity for cervical cancer

screenin$. 7he !atient may not return for follow4u!. 7herefore, screenin$ shoul" be !erforme" now. 00) (*!lanation: 7he correct answer is %. 7his !atient has severe !reeclam!sia. &reeclam!sia is "ia$nose" on the basis of hy!ertension, e"ema, an" !roteinuria. )evere !reeclam!sia may be "ia$nose" when the !atient has one of the followin$: a hea"ache that "oes not res!on" to anal$esics, visual chan$es, sei#ure, very elevate" bloo" !ressures, !ulmonary e"ema, elevate" liver function tests, severe !roteinuria, oli$uria, an elevate" creatinine, thrombocyto!enia, hemolysis, intrauterine $rowth restriction, or oli$ohy"ramnios. 7he mana$ement of severe !reeclam!sia after 0; weeks is with "elivery. &rior to 0; weeks, consi"eration may be $iven to e*!ectant mana$ement of the !atient "e!en"in$ on the clinical circumstances. 7his !atient is at 0/ weeksA with hea"ache, visual chan$es, elevate" bloo" !ressures, thrombocyto!enia, an" elevate" liver function tests. )he, therefore, nee"s to be "elivere". )he a!!ears to alrea"y be in labor as she is contractin$ every two minutes an" her cervi* is "ilate" an" efface". At this !oint, ma$nesium sulfate shoul" be starte". +a$nesium sulfate has consistently been "emonstrate" to be the most effective me"ication for sei#ure !ro!hyla*is in women with !reeclam!sia. 7o a"minister o*ytocin 'choice A) woul" not be necessary for this !atient. )he a!!ears to alrea"y be in labor with contractions every two minutes. 7o "ischar$e the !atient 'choice ) woul" absolutely be incorrect. )evere !reeclam!tics nee" to be "elivere" or, at the very least, a"mitte" to the hos!ital. 7here is no role for "ischar$in$ a !atient home in the mana$ement of severe !reeclam!sia. 7o encoura$e ambulation 'choice C) woul" also be incorrect. )evere !reeclam!tics shoul" be ke!t on be" rest. 7o start terbutaline 'choice () woul" not be a!!ro!riate. 7erbutaline is use" in obstetrics as a tocolytic a$ent to treat !reterm labor. In this !atient, contractions an" labor are "esirable an" no attem!t shoul" be ma"e to sto! them, as she requires "elivery. 0?) (*!lanation: 7he correct answer is C. 2ome uterine activity monitorin$ '2FA+) became a controversial area of obstetrics "urin$ the :II>s. Of all liveborn neonates, a!!ro*imately D= will be less than ;/>> $ 'low birth wei$ht). A!!ro*imately := will be less than :/>> $ 'very low birth wei$ht). +ost of the infant mortality rate comes from these low birth wei$ht an" very low birth wei$ht neonates. &reterm labor an" "elivery is the cause of many of these cases. 7hus, strate$ies to !revent !reterm "elivery are very much sou$ht after. One such strate$y is 2FA+. With this technique, women are monitore" at home with a toco"ynamometer 'a way to measure uterine contractions). 7he theory is that this home monitorin$ will allow for !reterm labor to be reco$ni#e" an" treate" in its earliest sta$es, which may hel! to !revent !reterm births. In !ractice, however, this has not been !roven to be the case. Numerous stu"ies have been !erforme", an" 2FA+ has not been !roven to !revent

!reterm birth. A !ossible benefit of 2FA+ may be the early reco$nition of !reterm labor, which woul" allow for the a"ministration of corticosteroi"s to brin$ about fetal !ulmonary maturity, even if a !reterm "elivery coul" not be !revente". 7his question has not been fully answere". At !resent, therefore, 2FA+ has not been !roven to !revent !reterm birth, an" its use is not recommen"e". 7o state that 2FA+ has been !roven to cause !reterm birth 'choice A) is not correct. 2FA+ is a noninvasive technique for monitorin$ uterine activity, an" it is use" to try to !revent !reterm birth. )i"e effects an" com!lications are rare. 7o state that 2FA+ has been !roven to !revent !reterm birth 'choice ) is incorrect. 7his is the central area of controversy for this technique, namely that it has not been !roven to !revent !reterm birth. 7o state that 2FA+ shoul" be starte" imme"iately 'choice %), or that 2FA+ shoul" be starte" at 0/ weeks 'choice () is incorrect. As e*!laine" above, 2FA+ has not been !roven to !revent !reterm "eliveryG therefore, its use is not currently recommen"e". 0/) (*!lanation: 7he correct answer is . One of the ma<or risk factors for "evelo!in$ !ost!artum en"ometritis is cesarean "elivery. 7herefore, !ro!hylactic antibiotics are recommen"e" in all cases of nonsche"ule" cesarean "elivery 'i.e., a cesarean "elivery that is not antici!ate"). 7his !atient is havin$ a cesarean "elivery because she is a nulli!arous woman in labor with a fetus in the breech !osition. 7he fact that her membranes broke a few hours a$o, an" that she has "ilate" to C centimeters !uts her at even hi$her risk for !ost!artum en"ometritis because of the !ossible e*!osure that has occurre" to the va$inal flora. Intravenous antibiotics will hel! to !revent infection of the mother if they are $iven before or after the umbilical cor" is clam!e". 7herefore, to minimi#e fetal e*!osure to the antibiotics, the me"ication shoul" be $iven after the umbilical cor" is clam!e". 7his !atient has no evi"ence of chorioamnionitisG therefore there is no in"ication to $ive the antibiotics imme"iately. 7o a"minister intravenous antibiotics 0> minutes !rior to the !roce"ure 'choice A) is !ro!er mana$ement in a non4!re$nant !atient un"er$oin$, for e*am!le, a hysterectomy. In the !re$nant !atient, a"ministerin$ the antibiotic !rior to clam!in$ the umbilical cor" results in unnecessary fetal e*!osure. 7o a"minister intravenous antibiotics imme"iately after the !roce"ure 'choice C) is incorrect because there is a nee"less "elay. Once the umbilical cor" is clam!e", there is no further concern re$ar"in$ unnecessary fetal e*!osure an" the antibiotic may be $iven imme"iately. 7o a"minister intravenous antibiotics for ;? hours after the !roce"ure 'choice %) is in"icate" when the !atient has chorioamnionitis. 7his !atient, however, has no evi"ence of bein$ infecte" an" the antibiotics are bein$ $iven for !ro!hyla*is. 7o a"minister oral antibiotics for : week followin$ the !roce"ure 'choice () is unnecessary. Once the sin$le "ose is $iven at the time of cor" clam!in$, there is no nee" for further treatment. 0C) (*!lanation: 7he correct answer is

%. 7his !atient has trichomoniasis. 7richomoniasis is cause" by a motile, fla$ellate" !roto#oan, 7richomonas va$inalis. 7he sym!toms inclu"e a co!ious, malo"orous 'HfishyH), $reenish4$ray, HfrothyH "ischar$e. 7he vulvar an" va$inal e!ithelium may be erythematous an" e"ematous. Col!osco!y may reveal !etechial cervical lesions 'Hstrawberry cervi*H). A wet mount of the "ischar$e often reveals motile trichomona"s an" !olymor!honuclear leukocytes '&+Ns). 7he treatment is metroni"a#ole. )imultaneous treatment of the se*ual !artner re"uces the risk of reinfection. HClue cellsH 'choice A), va$inal squamous e!ithelial cells coate" with coccobacillary or$anisms, are seen in bacterial va$inosis. 7he sym!toms inclu"e a mo"erate amount of malo"orous 'HfishyH), white to $ray, homo$eneous va$inal "ischar$e. An amine 'HfishyH) o"or is !resent after mi*in$ va$inal secretions with 5O2. 7his is often calle" a !ositive whiff test. )aline !re!arations of the "ischar$e reveal the Hclue cellsH. 7he treatment is metroni"a#ole.)imultaneous treatment of the se*ual !artner has not been shown to re"uce recurrence. .ram4ne$ative "i!lococci 'choice ) are an in"ication of Neisseria $onorrhoeae. N. $onorrhoeae causes a muco!urulent cervical "ischar$e in acute cervicitis an" can lea" to !elvic inflammatory "isease '&I%). &I% is characteri#e" by lower ab"ominal !ain, fever, an" cervical motion ten"erness. %ia$nosis is often ma"e by .ramAs stain of cervical secretions revealin$ $ram4ne$ative "i!lococci an" !olymor!honuclear leukocytes. 7reatment is ceftria*one I+ once an" "o*ycycline or a#ithromycin. 7he ; latter "ru$s are $iven since concomitant chlamy"ial infection is common. )e*ual !artners must be treate". .ram4!ositive "i!lococci 'choice C) are not a common cause of cervical "ischar$e. &seu"ohy!hae or hy!hae 'choice () is an in"ication of can"i"iasis. 1ulvar !ruritus, irritation, an" a thick, white, cotta$e cheese4like "ischar$e are the !re"ominant sym!toms. %ia$nosis is ma"e by 5O2, saline, or .ramAs stain evaluation of the va$inal flui" revealin$ fun$i. 7reatment is flucona#ole &O or imi"a#ole cream. 6outine treatment of se*ual !artners is usually not in"icate". 0D) (*!lanation: 7he correct answer is (. -etal scal! sam!lin$ '-))) is a metho" of fetal assessment that is use" "urin$ labor an" "elivery to obtain fetal bloo" for !2 assessment. Normal labor an" "elivery is characteri#e" by a lowerin$ of the fetal !2 as the labor !ro$resses. 2owever, most fetuses tolerate labor an" "elivery without a "an$erous "ro! in !2 'i.e. an aci"osis that will result in or$an "ama$e). When the fetal heart rate tracin$ is not reassurin$, -)) can be use" to "etermine the aci"4base status of the fetus, which will hel! with mana$ement of the labor. If the !2 is O D.;/ then the !atient may be mana$e" e*!ectantly with continue" observation of the labor an" the fetal heart rate. If the !2 is between D.;> an" D.;/, the -)) shoul" be re!eate" in :/ to 0> minutes. If the !2 is P D.;>, ste!s shoul" be taken to brin$ about "elivery. Aci"emia likely to cause "ama$e to the fetus a!!ears to occur at values P D.>>. 2owever, by usin$ a cutoff of D.;>, there is a mar$in for error to !rotect the fetus. 7his fetus has fetal scal! bloo" !2s of D.>?, D.>/, an" D.>C. 7his level of aci"emia is consi"ere" an in"ication for imme"iate "elivery. ecause the fetus is at 4: station, far too hi$h for force!s or vacuum4assiste" va$inal "elivery, a cesarean "elivery shoul" be !erforme". (*!ectant mana$ement 'choice A) woul" be ina!!ro!riate. 7his

!atient has a fetus at 4: station with scal! !2s consistently less than D.;>. 7herefore, cesarean "elivery shoul" be !erforme". (!isiotomy 'choice ) woul" not be in"icate". (!isiotomy is sometimes use" when force!s or vacuum4assiste" va$inal "elivery will be !erforme". It may also be !erforme" to facilitate an uninstrumente" va$inal "elivery. In this case, however, the !atient requires a cesarean "elivery. -orce!s4assiste" va$inal "elivery 'choice C) or vacuum4assiste" va$inal "elivery 'choice %) woul" be incorrect. -orce!s an" vacuum4assiste" va$inal "elivery shoul" not be !erforme" at stations hi$her than M;. Certainly, at 4: station, this fetus is Htoo hi$hH to attem!t an assiste" va$inal "elivery. 0@) (*!lanation: 7he correct answer is (. Cesarean "elivery has been shown to "ecrease the rate of transmission of 2I1 from an infecte" mother to her fetus. )ome re!orts have shown that the transmission rate can be "ecrease" to as low as ;= with the combination of antiretroviral me"ication an" elective cesarean "elivery !rior to labor or ru!ture of membranes. 2owever, althou$h cesarean "elivery benefits the infant by "ecreasin$ the risk of transmission, the risks of the sur$ery accrue to the mother. 6isks of cesarean "elivery inclu"e blee"in$, infection, an" in<ury to internal or$ans. 2I14infecte" women with low C%? cell counts are known to have hi$her rates of !osto!erative com!lications. 7hus, the "ecision of whether to have a cesarean ultimately belon$s to the mother. 7his !atientAs autonomy must be res!ecte" an" she shoul" have a va$inal "elivery if she so chooses. 7o contact either !sychiatry to evaluate the !atient 'choice A) or the hos!ital lawyers to $et a court or"er for cesarean "elivery 'choice ) woul" be incorrect. &atient autonomy must be res!ecte" when it comes to the "ecision of whether to have a cesarean "elivery. 7his !atient has wei$he" the benefits an" risks an" has !ut a $reat "eal of "eliberation into her "ecision. 7here is no nee" to involve the !sychiatry "e!artment or the hos!ital lawyers in this "ecision. 7o !erform cesarean "elivery at 0@ weeks 'choice C) or once the !atient is in labor 'choice %) woul" not be correct. In 2I14!ositive women who "o want an elective cesarean "elivery, the "elivery shoul" be !erforme" at 0@ weeks to avoi" the risk of labor or ru!ture of membranes. Once labor starts or the membranes are ru!ture", the risk of 2I1 transmission increases. 2owever, this !atient "oes not want a cesarean "elivery, so that o!eration shoul" not be !erforme". 0I) (*!lanation: 7he correct answer is %. 7his !atientAs !resentation is most consistent with ovarian torsion. Ovarian torsion ty!ically occurs in the settin$ of an a"ne*al mass. A mass chan$es the motion H"ynamicsH of the a"ne*ae such that a twistin$ of the a"ne*a becomes !ossible. 7his mass can be a functional ovarian cyst, a "ermoi", a !aratubal cyst, or any number of other beni$n or mali$nant neo!lasms. Once a com!lete torsion has occurre", the arterial su!!ly to the ovary is occlu"e" an" necrosis can result. &atients with a"ne*al torsion can !resent

with a history of intermittent !ain that comes an" $oes as the a"ne*a twists. 7he !ain is usually severe an" often accom!anie" by e!iso"es of nausea, vomitin$, an" "ia!horesis, as this !atient ha". 7hey may nee" narcotics to control the severe !ain. A !elvic mass will almost always be foun" on !hysical e*amination or by ultrasoun". If there is no a"ne*al mass, the "ia$nosis of ovarian torsion is hi$hly unlikely. 7his is true because most normal ovaries "o not have the motion H"ynamicsH that will allow them to twist. A!!en"icitis 'choice A) shoul" always be a consi"eration when a !atient !resents with ri$ht lower qua"rant !ain. 2owever, in this case, the combination of the !ain with the ovarian mass makes ovarian torsion, an" not a!!en"icitis, the most likely "ia$nosis. (cto!ic !re$nancy 'choice ) shoul" also be an im!ortant consi"eration when a youn$ woman !resents with ab"ominal !ain. )ome emer$ency "e!artments have si$ns rea"in$ H7hink (cto!icH to kee! staff aware of this !ossibility. In this case, however, the !atient is not !re$nant 'ne$ative urine hC.) which e*clu"es ecto!ic from the "ifferential. Ne!hrolithiasis 'choice C) can also cause e*cruciatin$ !ain, as "oes ovarian torsion. With ne!hrolithiasis, hematuria will often be !resent. In this !atient, the absence of hematuria an" the !resence of the ri$ht a"ne*al mass make ne!hrolithiasis less likely. &elvic inflammatory "isease 'choice () is a "ia$nosis that merits consi"eration in a woman with ab"ominal !ain with a ne$ative hC. 'it is far less common "urin$ !re$nancy). 2owever, the ovarian mass in this case makes torsion a more likely "ia$nosis than &I %. ?>) (*!lanation: 7he correct answer is A. All of the above o!tions will !rovi"e birth control for this !atient. 2owever, another ma<or factor for this !atient is the !revention of se*ually transmitte" "isease. Other than abstinence, con"oms !rovi"e the best !rotection a$ainst the acquisition of se*ually transmitte" "iseases. 7his !atient, with her early onset of intercourse at the a$e of :/ an" her numerous se*ual !artners, is at hi$h risk for 2I1, he!atitis, her!es, chlamy"ia, $onorrhea, sy!hilis, human !a!illomavirus, an" the eventual "evelo!ment of cervical cancer. It is absolutely essential that she be counsele" re$ar"in$ con"om use an" the im!ortance of her !rotectin$ herself from se*ually transmitte" "iseases as well as !re$nancy. 7he "ia!hra$m 'choice ) is an effective metho" of birth control for motivate" women who are able to use this metho" with each e!iso"e of intercourse. ecause it covers the cervi*, it !rovi"es some !rotection a$ainst "isease. 2owever, it "oes not !rovi"e as much !rotection a$ainst se*ually transmitte" "iseases as con"oms "o. 7he intrauterine "evice 'choice C) is absolutely contrain"icate" in a woman with numerous se*ual !artners an" a recent history of se*ually transmitte" "isease. -urthermore, it is hi$hly subo!timal for youn$ women, in whom a !elvic infection coul" lea" to re"uce" or absent future fertility. 7he oral contrace!tive !ill 'OC&) 'choice %) woul" !rovi"e this !atient with !rotection a$ainst !re$nancyG however, it woul" not !rotect her from se*ually transmitte" "iseases. An i"eal a!!roach may be to have her use both the OC& an" con"oms. 2owever, consistent use of both can be "ifficult. 7ubal li$ation 'choice () woul" !rovi"e this !atient with no !rotection a$ainst se*ually

transmitte" "isease. -urthermore, e*ce!t in very rare circumstances, it is contrain"icate" for an :@4year4ol". ?:) (*!lanation: 7he correct answer is . Chorionic villus sam!lin$ 'C1)) is a !roce"ure in which the chorionic villi are sam!le" throu$h either a transab"ominal or transcervical a!!roach. Amniocentesis is a !roce"ure in which amniotic flui" is remove". In both !roce"ures, cells are remove" an" can be analy#e" for chromosomal abnormalities. A ma<or a"vanta$e of chorionic villus sam!lin$ is that it can be !erforme" at :>4:; weeks, as o!!ose" to amniocentesis, which is !erforme" in the secon" trimester. C1) thus allows a woman to un"er$o an earlier termination than amniocentesis allows for. In the early :II>s, there were several re!orts linkin$ chorionic villus sam!lin$ to limb re"uction "efects in the infants. +ost of the lar$e stu"ies have shown no overall increase" risk of limb "eficiencies amon$ infants whose mothers un"erwent C1). 2owever, there is some evi"ence that one subty!e of limb "efect, calle" transverse "i$ital "eficiency, is more common with C1). 7he risk of fetal %own syn"rome 'choice A) is not increase" by the !roce"ure use" to "etect it. Whether C1) or amniocentesis is use", the fetus has a set risk of %own syn"rome. 7he risk of fetal neural tube "efects 'choice C) is also not increase" by the !roce"ure use" to "etect them. 2owever, C1) "oes not allow for the !renatal "ia$nosis of neural tube "efects. Only amniocentesis allows for the evaluation of amniotic flui" al!ha4feto!rotein, which is often necessary to make the "ia$nosis. +aternal se!sis 'choice %) is not more likely with C1) com!are" with amniocentesis. 7he risk for either !roce"ure is very low. +i"4secon"4trimester abortion 'choice () is more likely with amniocentesis than with C1). Amniocentesis is !erforme" later in !re$nancyG therefore, termination base" on the amniocentesis result is likely to occur later than with C1) '!erforme" at :>4:; weeks). ?;) (*!lanation: 7he correct answer is %. &harmaceutical com!anies often !rovi"e fun"in$ for e"ucational o!!ortunities for !hysicians. 7his involvement is consi"ere" acce!table by some an" unacce!table by others. &harmaceutical com!any involvement, however, shoul" never !lace the !hysician in a situation whereby the interests of the !atient are not !lace" in a !rimary !osition. In the above scenario, the !hysician will be rewar"e" for $ivin$ the most !rescri!tions for this !articular oral contrace!tive !ill 'OC&). 7his rewar" system may !lace the !hysicianAs interest a$ainst that of the !atient. -or e*am!le, a $iven !atient may benefit more from another OC&, but the !hysician will feel !ressure to !rescribe the !ill that will brin$ him the $ift. 7hus, most s!ecialty societies "eclare that !hysicians shoul" not acce!t $ifts if they are $iven secon"ary to the !hysician !rescribin$ certain me"ications. Acce!tance of the $ift 'choice A) woul" le$itimi#e the a!!roach of the com!any of !rovi"in$ $ift incentives to !hysicians that !rescribe their me"ication. 7his is consi"ere" unethical. An attem!t to $et collea$ues to !rescribe the me"ication 'choice ) or a

!romise to !rescribe more of the me"ication 'choice C) to !lease the !harmaceutical com!any re!resentative woul" also !lace the !atientAs interests in a secon"ary !osition. 7his is consi"ere" unethical. A request for money rather than a $ift 'choice () is an e*am!le of "irect conflict of interest behavior. 7he interest of the !atient in this case is not !rimary. 7herefore, schemes of rewar" such as that !resente" above are consi"ere" unethical. ?0) (*!lanation: 7he correct answer is %. 7his !atient likely has an uncom!licate" urinary tract infection 'F7I). &atients with F7Is often !resent with "ysuria, frequency, an" ur$ency. &hysical e*amination is often unremarkable, althou$h there may be some su!ra!ubic ten"erness if a cystitis is the !re"ominant infection rather than a urethritis. Frine H"i!H will often be !ositive for leukocyte esterase an" nitrites. +icrosco!ic urinalysis will often show the !resence of white bloo" cells an" re" bloo" cells. (scherichia coli is the offen"in$ or$anism in about @>= of cases with )ta!hylococcus sa!ro!hyticus bein$ the ne*t most likely causative or$anism. 7reatment of an uncom!licate" urinary tract infection is with a 04"ay course of oral antibiotics. 7rimetho!rim4sulfametho*a#ole ' actrim) has been shown to be safe, effective, an" cost4effective in the treatment of uncom!licate" F7Is. Intramuscular ceftria*one 'choice A) is use" for the treatment of $onorrhea. 7his !atient has fin"in$s consistent with urinary tract infection an" not $onorrhea, an" therefore, a 04"ay course of oral antibiotics is in"icate", rather than intramuscular ceftria*one. Intravenous levoflo*acin 'choice ) can be use" in cases of com!licate" urinary tract infections in !atients that cannot take oral me"ications an" oral levoflo*acin 'choice C) can be use" when the !atient is toleratin$ oral intake. ,evoflo*acin has rou$hly the same bioavailability when taken orally versus intravenously, so the route "e!en"s on the !atientAs status. 2owever, this me"ication is use" when !atients have com!licate" urinary tract infections 'i.e. with &seu"omonas or &roteus s!ecies) or when the !atient has un"erlyin$ me"ical illness. 7o wait for the culture results to institute thera!y 'choice () woul" not be a!!ro!riate. 7he !atient has sym!toms an" fin"in$s consistent with a F7I ri$ht now an" therefore shoul" be treate" now. 7o wait ; or more "ays for the culture results to come back woul" not be a!!ro!riate.

??) (*!lanation: 7he correct answer is . 7his !atient has a !resentation an" fin"in$s that are most consistent with a beni$n cystic teratoma '"ermoi"). Cystic teratomas are, by far, the most common ty!e of ovarian neo!lasm: cystic teratomas account for ;/ to ?>= of all ovarian neo!lasms. 7hey are a ty!e of ovarian $erm cell tumor, which can ran$e in si#e from small masses that are note" inci"entally on ultrasoun" an" cause no sym!toms to lar$er cysts that cause !ain an"

!ressure, as this !atient has. A sin$le $erm cell $ives rise to a teratoma. ecause the $erm cell is toti!otential, the "ermoi" is characteri#e" by all three $erm cell layers: ecto"erm, meso"erm, an" en"o"erm. .ross e*amination of a "ermoi" will often reveal skin, bones, hair, an" teeth, which can often be seen on ultrasoun". 7he !art of the "ermoi" that contains the lar$est number of "ifferent tissues is calle" 6okitanskyAs !rotuberance. ,a!arotomy is usually the most a!!ro!riate mana$ement of a !atient with a "ermoi" because, as a"ne*al masses enlar$e, the risk of ovarian torsion increases. Also, "ermoi"s may cause sym!toms of !ain an" !ressure an", on that basis, shoul" be remove". At the time of sur$ery, close e*amination shoul" be ma"e of the other ovary because "ermoi"s may be foun" bilaterally in more than :>= of cases. A!!en"icitis 'choice A) is usually not a chronic !rocess slowly "evelo!in$ over 0 months. Also, !atients with a!!en"icitis ty!ically have anore*ia an" a!!ear ill. A cor!us luteum cyst 'choice C) is a common cause of a com!le* a"ne*al mass in a youn$ woman. 2owever, this !atient has a !resentation an" a mass with ultrasoun" characteristics that are classic for "ermoi". (cto!ic !re$nancy 'choice %) shoul" always be consi"ere" when a woman of chil"bearin$ a$e !resents with ab"ominal !ain. A ne$ative urine hC. effectively rules out ecto!ic !re$nancy. &atients with a tubo4ovarian abscess 'choice () usually have fevers, si$nificant ab"ominal an" !elvic ten"erness, an" a!!ear ill. ?/) (*!lanation: 7he correct answer is (. 7his !atient !resents with !ostmeno!ausal blee"in$. 7he ma<ority of !atients who have !ostmeno!ausal blee"in$ will not have en"ometrial hy!er!lasia or cancer. 2owever, because !ostmeno!ausal blee"in$ is the most common !resentin$ com!laint of women with en"ometrial cancer, it is im!ortant to rule this out. A common way to evaluate the en"ometrium is with an en"ometrial bio!sy. 7his can be !erforme" with a small suction cannula that is intro"uce" throu$h the cervical os an" into the uterine cavity to $et a sam!le of the en"ometrium. 7he !roce"ure is stan"ar" in the !ractice of $ynecolo$y but is not without risks. One of the risks of en"ometrial bio!sy is uterine !erforation 'i.e. a"vancin$ the cannula too far such that it !enetrates an" !erforates throu$h the wall of the uterus). 7his !atient has evi"ence of uterine !erforation. -irst, she e*!erience" si$nificant !ain "urin$ the !roce"ure an" continuin$ afterwar"s. While en"ometrial bio!sy can cause consi"erable "iscomfort, it is usually of a cram!y nature that shoul" resolve shortly after the !roce"ure. )econ", her !elvic ultrasoun" now shows a com!le* flui" collection !osterior to the uterus, which likely re!resents a collection of bloo" in the !osterior cul4"e4sac. If the !atient has stable vital si$ns an" an acce!table hematocrit, uterine !erforation can be mana$e" e*!ectantly. If, however, the !atient has evi"ence of hemo"ynamically si$nificant blee"in$, then she will require o!erative intervention. owel !erforation 'choice A) is a very unlikely com!lication with an en"ometrial bio!sy. ItAs rare for the cannula to be a"vance" far enou$h to "ama$e the uterus 'uterine !erforation), let alone "ama$e the bowel. (n"ometritis 'choice ) can be a com!lication of an en"ometrial bio!sy. &atients un"er$oin$ en"ometrial bio!sy shoul" be counsele" that infection is one of the risks of the !roce"ure. 2owever, this !atient is afebrile an" the !elvic flui" collection is more su$$estive of a !erforation than an en"ometritis. While it

is !ossible that this !atient has en"ometrial cancer 'choice C), it is not likely that en"ometrial cancer is causin$ her acute !roblem. A$ain, most women with !ostmeno!ausal blee"in$ "o not have en"ometrial cancer. An", this !atientAs su""en onset of !ain an" !elvic flui" collection after en"ometrial bio!sy is most su$$estive of en"ometrial cancer. A !atient with a tuboovarian abscess 'choice %) usually !resents with ab"ominal !ain an" fevers, an" ultrasoun" will reveal a !elvic mass. In a non4 se*ually active !atient with no a"ne*al mass, tuboovarian abscess can be effectively rule" out. ?C) (*!lanation: 7he correct answer is A. 7his !atient has a !resentation an" fin"in$s that are most consistent with a artholinAs cyst. artholinAs cysts "evelo! when a artholinAs $lan" becomes obstructe". 7he artholinAs $lan"s are bilateral structures that are !resent near the !osterior fourchette of the va$ina at the / an" D oAclock !ositions. 7hey secrete mucus, !articularly "urin$ se*ual stimulation, which "rains into the !osterior va$ina.7hey un"er$o ra!i" $rowth "urin$ the !rocess of !uberty an" they shrink after the meno!ause. When the "uct of the artholinAs $lan" becomes obstructe", a artholinAs cyst results. If the cyst becomes infecte", the result is a artholinAs abscess. 7hese abscesses are usually !olymicrobial in nature, althou$h the $onococcus is im!licate" in about ;/= of cases. 7reatment of a sym!tomatic artholinAs cyst is with !lacement of a Wor" catheter. 7his is a small balloon4ti!!e" catheter "evice that is !lace" into a small hole that is !unche" into the cyst itself. 7his catheter allows "raina$e of the cyst an" the formation of an e!itheliali#e" tract that will allow continue" "raina$e once the catheter is remove". 7his tract shoul" !revent the cyst from reformin$. If artholinAs cysts continue to form in s!ite of the use of the Wor" catheter, a marsu!iali#ation !roce"ure may be trie". In this !roce"ure, the cyst walls are suture" o!en to the surroun"in$ skin to !revent re4closure an" re4formation of the cyst. Con"yloma lata 'choice ) is a manifestation of secon"ary sy!hilis. 7hey a!!ear as coalesce", lar$e, !ale, flat4to!!e" !a!ules an" not as a cystic mass. .ranuloma in$uinale 'choice C) is also known as %onovanosis an" is a se*ually transmitte" "isease associate" with the $ram4ne$ative bacillus Calymmatobacterium $ranulomatis. 7he "isease is characteri#e" by !a!ules !ro$ressin$ to ulcers an" not by a vulvar cyst. 2ematocol!os 'choice %) "escribes the con"ition in which there is bloo" fillin$ the va$ina. 7his is often seen with an im!erforate hymen. 1ulvar cancer 'choice () "oes not usually !resent as a sin$le cystic mass at the introitus an", in youn$ women, is far less common than artholinAs cysts ?D) (*!lanation: 7he correct answer is C. An ecto!ic !re$nancy is a !re$nancy that is locate" outsi"e of the normal intrauterine location, most often in the fallo!ian tube. In a stable !atient, an ecto!ic !re$nancy may be treate" me"ically or sur$ically. +e"ical mana$ement is with methotre*ate. When

methotre*ate is $iven, it is essential to have the !atient return for follow4u! to ensure that the beta4hC. value is fallin$, the in"ication that the methotre*ate is workin$. 7his !atient not only has a beta4hC. value that is risin$, but also has other si$ns an" sym!toms consistent with a ru!turin$ ecto!ic !re$nancy. 7he worsenin$ ab"ominal !ain an" left lower qua"rant ten"erness are concernin$ for ru!ture as is the increase" amount of flui" in the !elvis on the ultrasoun". 7his flui" likely re!resents bloo". 7he mana$ement for a ru!ture" ecto!ic is sur$ical. In this case la!arosco!y coul" be !erforme" to i"entify the ecto!ic !re$nancy an" either a sal!in$ostomy 'i.e. makin$ a hole in the tube to remove the ecto!ic !re$nancy) or a sal!in$ectomy 'i.e. removin$ the entire tube) coul" be !erforme". (*!ectant mana$ement 'choice A) woul" be absolutely ina!!ro!riate, as this !atient has a "oublin$ beta4hC. value in s!ite of the methotre*ate thera!y an" has fin"in$s consistent with a ru!ture" ecto!ic. 6e!eat methotre*ate 'choice ) can be $iven to women with an ecto!ic !re$nancy that show !ersistently hi$h levels of serum beta4 hC. on a "ay D evaluation 'i.e. D "ays after the first "ose of methotre*ate). 2owever, this !atient woul" not be a can"i"ate, as she a!!ears to be actively ru!turin$ her ecto!ic !re$nancy. Oo!horectomy 'choice %) an" hysterectomy 'choice () are not the treatments of choice for women with an ecto!ic !re$nancy. All reasonable ste!s shoul" be taken to !reserve the !atientAs uterus an" ovaries "urin$ a sur$ery for ecto!ic !re$nancy. &referably, only the tube itself shoul" be o!erate" u!on, with either a sal!in$ectomy or sal!in$ostomy bein$ !erforme". ?@) (*!lanation: 7he correct answer is (. 7his !atient has a !resentation that is most consistent with recurrent artholinAs cysts. 7he artholinAs $lan"s are !aire" $lan"s that are locate" in the vulvar tissue on both si"es of the fourchette. 7hey secrete mucus into the va$ina. artholinAs cysts occur when the o!enin$ to the "uct becomes occlu"e" an" the $lan" swells from a buil"u! of mucus secretions. If the cyst becomes infecte", the result is a artholinAs abscess. 7hese artholinAs cysts an" abscesses are the most common vulvar cysts. )ome of these cysts are small an" asym!tomatic an" "o not require treatment. 7his !atient, however, is uncomfortable when she walks an" she also has continue" recurrences. 7herefore, sur$ical intervention is in"icate". Often a Wor" catheter is use" to "rain the cyst an" create an ostium to allow future "raina$e. 2owever, as in this !atientAs case, once the catheter is remove" the cyst may form once a$ain. Consequently, this !atient woul" be best serve" with a artholinAs cyst marsu!iali#ation !roce"ure. 7his is a sur$ical !roce"ure in which the artholinAs cyst wall is o!ene" an" the cyst itself is sewn o!en to the va$inal mucosa me"ially an" the skin of the introitus laterally. 7his shoul" allow free e$ress of the cyst flui" an" !revent re4formation of the cyst. A !ortion of the cyst wall can be remove" at the time of sur$ery an" sent for !atholo$ic evaluation to rule out mali$nancy, which, while hi$hly unlikely in a youn$ woman, is still a small !ossibility. (*!ectant mana$ement 'choice A) woul" not be the best choice for this !atient. )he has ha" multi!le recurrent artholinAs cysts over the !ast several years an" e*!ectant mana$ement is unlikely to lea" to resolution of the !resent cyst or lon$4term resolution of the !roblem. Oral antibiotics 'choice ) or intravenous antibiotics 'choice C) are not the

most a!!ro!riate ne*t ste! in the mana$ement of this !atient. 7here is no evi"ence that this is a artholinAs abscess, which is often characteri#e" by ten"erness an" erythema of the mass. 7his a!!ears to be sim!ly a recurrent artholinAs cyst. An", even if this were an abscess, the "efinitive mana$ement is with incision an" "raina$e. Incision an" "raina$e 'choice %) is a!!ro!riate mana$ement of a artholinAs abscess, as state" above. 2owever, this !atient has a artholinAs cyst. 7o sim!ly !erform an incision an" "raina$e of this cyst woul" likely lea" to a recurrence of the cyst, as this !atient has ha" several times in the !ast. 7he more "efinitive thera!y woul" therefore be to "o a marsu!iali#ation !roce"ure. ?I) (*!lanation: 7he correct answer is C. Necroti#in$ fasciitis is a rare but !otentially fatal com!lication of ab"ominal woun" infection. It ty!ically occurs in !atients who are immunocom!romise" or who have "iabetes or cancer. It is a clinical "ia$nosis that is characteri#e" by "iscoloration an" cyanosis aroun" the incision with numbness of the area. It can be !olymicrobial in nature, but anaerobes are frequently involve". It is consi"ere" to be a !otentially fatal con"ition an" a$$ressive treatment with broa"4s!ectrum antibiotics an" sur$ical "ebri"ement is essential. (n"ometritis 'choice A) is characteri#e" by ab"ominal !ain, malaise, foul4 smellin$ lochia, tem!erature elevation, an" uterine ten"erness on bimanual e*amination. 7his !atient "oes not have uterine ten"erness on bimanual e*amination an" her "isease !rocess a!!ears focuse" aroun" the incision site. 7herefore, en"ometritis woul" not be the !rocess of most concern in this !atient. +astitis 'choice ) is an infection of the breast that is characteri#e" by breast !ain, elevate" tem!erature, erythema an" e"ema of the breast. 7his !atientAs !rocess is not involvin$ the breastG therefore, mastitis woul" not be of concern here. &reeclam!sia 'choice %) is characteri#e" by hy!ertension, e"ema, an" !roteinuria. 7he cure for !reeclam!sia is "elivery of the fetus. 7his !atient has no fin"in$s concernin$ for !reeclam!sia an" is !ost!artum, which makes the "evelo!ment of !reeclam!sia much less likely. Woun" infection 'choice () is certainly of concern here. 2owever, this !atient has features to her !resentation that su$$est a !rocess that $oes beyon" sim!le woun" infection. 7he "iscoloration of the woun" e"$es an" cyanosis, alon$ with the loss of sensation aroun" the woun" !oint towar" the more worrisome !rocess of necroti#in$ fasciitis. />) (*!lanation: 7he correct answer is %. 7here is a si$nificant risk that a hi$h4$ra"e squamous intrae!ithelial lesion '2.)I,) will !ro$ress to invasive cervical cancer. Althou$h only rou$hly :/= of !atients with a low4$ra"e squamous intrae!ithelial lesion ',.)I,) will have !ro$ression of the lesion, a si$nificantly hi$her !ro!ortion of !atients with 2.)I, will !ro$ress to cervical cancer. 7herefore, once col!osco!ically "irecte" bio!sy is !erforme", the "ia$nosis of 2.)I, is confirme", an" the "istribution of the lesion is known, removal or "estruction of the entire transformation #one shoul" be !erforme". 7his can be "one with a loo! electro"e

e*cision !roce"ure ',((&). In this !roce"ure, a thin4wire loo! electro"e is use" to e*cise the entire transformation #one. 7he remove" tissue may then be sent to !atholo$y. ,((& can thus be use" as both a "ia$nostic an" thera!eutic !roce"ure. A re!eat &a! in : year 'choice A) is the recommen"e" follow4u! for !atients with a normal &a! test. 7his !atient has 2.)I,G therefore, follow4u! in : year with a re!eat &a! test woul" not be a!!ro!riate. A re!eat &a! test in C months 'choice ) may be a!!ro!riate follow4u! for !atients with aty!ical cells of un"etermine" si$nificance 'A)CF)) or ,.)I, on their &a! test. It is not a!!ro!riate mana$ement for !atients with 2.)I,. 6e!eat col!osco!y in C months 'choice C) woul" not be correct. 7his !atient has known, bio!sy4!roven 2.)I, an" therefore requires treatment of the lesion. 2ysterectomy 'choice () woul" not be a!!ro!riate for this !atient. 2.)I, can usually be treate" a"equately without nee"in$ to !erform hysterectomy. 2ysterectomy may be a!!ro!riate in some !atients with recurrent 2.)I, or in those with lesions that cannot be a"equately treate" with local thera!ies. 2owever, this !atient is a youn$ woman havin$ her first e!iso"e of cervical "ys!lasia.

USMLE Step 2 Practice Test Block 11


Name: Instructions: Answer the questions below to the best of your ability. When you finish the test, click the Check button at the bottom to view the results.

1.A &1!year!ol" 'oman comes to the physician for follo'!#p after an a$normal Pap test an" cer)ical $iopsy. The patient,s Pap test sho'e" a high!gra"e s*#amo#s intraepithelial lesion 3/CS+L6. This 'as follo'e" $y colposcopy an" $iopsy of the cer)i.. The $iopsy specimen also "emonstrate" /CS+L. The patient 'as co#nsele" to #n"ergo a loop electros#rgical e.cision proce"#re 3LEEP6. =hich of the follo'ing represents the potential long!term complications from this proce"#rea6A$scess an" chronic pel)ic inflammatory "isease $62er)ical incompetence an" cer)ical stenosis c62onstipation an" fecal incontinence "6 /ernia an" intraperitoneal a"hesions e6 Urinary incontinence an" #rinary retention :ormal La$s

2.A 2(!year!ol" 'oman% gra)i"a 2% para 2% comes to the physician to ha)e her staples remo)e" after an electi)e repeat cesarean "eli)ery. /er pregnancy co#rse 'as #ncomplicate". She states that she is "oing 'ell e.cept that since the "eli)ery she has notice" some episo"es of sa"ness an" tearf#lness. She is eating an" sleeping normally an" has no strange tho#ghts or tho#ghts of h#rting herself or others. Physical e.amination is 'ithin normal limits for a patient 'ho is stat#s post cesarean "eli)ery. =hich of the follo'ing is the most likely "iagnosisa6 Maternity $l#es $6Postpart#m "epression c6Postpart#m mania "6Postpart#m psychosis e6 PoststeriliDation "epression :ormal La$s &.A 22!year!ol" primigra)i" 'oman comes to the la$or an" "eli)ery 'ar" at term 'ith reg#lar% painf#l contractions. /er prenatal co#rse 'as #nremarka$le. She has a past me"ical history significant for mitral )al)e prolapse 'ith reg#rgitation "emonstrate" on echocar"iography. She takes no me"ications an" has no allergies to me"ications. E.amination sho's that her cer)i. is 4 centimeters "ilate" an" the fet#s is in )erte. presentation. The fetal heart rate is reass#ring. =hich of the follo'ing is the most appropriate management of this patienta6 A"minister intra)eno#s anti$iotics thro#gho#t la$or. $6A"minister intra)eno#s anti$iotics &4 min#tes prior to the "eli)ery. c6A"minister intra)eno#s anti$iotics after the cor" is clampe". "6A"minister intra)eno#s anti$iotics si. ho#rs after the "eli)ery. e6Anti$iotic prophyla.is is not necessary :ormal La$s 4.A 2<!year!ol" primigra)i" 'oman at 42 'eeks, gestation comes to the la$or an" "eli)ery 'ar" for in"#ction of la$or. The prenatal co#rse 'as significant for a positi)e gro#p B Streptococc#s c#lt#re performe" at & 'eeks. Antenatal testing o)er the past 2 'eeks has $een #nremarka$le. The patient is starte" on lactate" Ainger,s +? sol#tion. Sterile )aginal e.amination sho's that the patient,s cer)i. is long% thick% an" close". Prostaglan"in 3PCE26 gel is place" into the )agina% an" electronic fetal heart rate monitoring is contin#e". +n appro.imately <4 min#tes% the

fetal heart rate falls to the 84s% as the toco"ynamometer sho's the #ter#s to $e contracting e)ery 1 min#te 'ith essentially no rest in $et'een contractions. =hich of the follo'ing 'as most likely the ca#se of the #terine hyperstim#lationa6 +nfection $6+? fl#i"s c6Post"ates pregnancy "6Prostaglan"in 3PCE26 gel e6?aginal e.amination :ormal La$s .A 1<!year!ol" female comes to the physician $eca#se of an increase" )aginal "ischarge. She "e)elope" this symptom 2 "ays ago. She also complains of "ys#ria. She is se.#ally acti)e 'ith one partner an" #ses con"oms intermittently. E.amination re)eals some erythema of the cer)i. $#t is other'ise #nremarka$le. A #rine c#lt#re is sent 'hich comes $ack negati)e. Se.#ally transmitte" "isease testing is performe" an" the patient is fo#n" to ha)e gonorrhea. =hile treating this patient,s gonorrhea infection% treatment m#st also $e gi)en for 'hich of the follo'inga6 Bacterial )aginosis $62hlamy"ia c6/erpes "6Syphilis e6Trichomoniasis :ormal La$s <. A 1<!year!ol" n#lligra)i" 'oman comes to the emergency "epartment $eca#se of hea)y )aginal $lee"ing. She states that she normally has hea)y perio"s e)ery month $#t misse" a perio" last month an" this perio" has $een #n#s#ally hea)y 'ith the passage of large clots. She has no me"ical pro$lems% has no history of $lee"ing "iffic#lties% an" takes no me"ications. /er temperat#re is &( 2 381.< 56% $loo" press#re is 1147(4 mm /g% p#lse is 8<7min#te an" respirations are 127min#te. Pel)ic e.amination sho's a mo"erate amo#nt of $loo" in the )agina% a close" cer)i.% an" a normal #ter#s an" a"ne.ae. /ematocrit is &4B. Urine

h2C is negati)e. =hich of the follo'ing is the most appropriate managementa6E.pectant management $6/ysteroscopy c6>ral contracepti)e pills "6Laparoscopy e6 Laparotomy :ormal La$s (.A 12!year!ol" female comes to the physician $eca#se of a )aginal "ischarge. The "ischarge starte" a$o#t 2 months ago an" is 'hitish in color. There is no o"or. The patient has no complaints of itching% $#rning% or pain. The patient starte" $reast "e)elopment at 8 years of age an" her p#$ertal "e)elopment has procee"e" normally to this point. She has not ha" her first menses an" she is not se.#ally acti)e. She has no me"ical pro$lems. E.amination is normal for a 12!year!ol" female. Microscopic e.amination of the "ischarge sho's no e)i"ence of pse#"ohyphae% cl#e cells% or trichomona"s. =hich of the follo'ing is the most likely "iagnosisa6 Bacterial )aginosis $62an"i"a )#l)o)aginitis c6Physiologic le#korrhea "6Syphilis e6Trichomoniasis :ormal La$s 1.A &4!year!ol" 'oman comes the physician $eca#se of lo'er a$"ominal cramping. The cramping starte" 2 "ays ago. E.amination is #nremarka$le e.cept for a pel)ic e.amination that re)eals a 14!'eek siDe" #ter#s. Urine h2C is positi)e% an" pel)ic #ltraso#n" re)eals a 14! 'eek intra#terine pregnancy 'ith a fetal heart rate of 1<4. The patient states that she is not s#re 'hether to keep the pregnancy. =hich of the follo'ing is the most appropriate ne.t step in managementa62o#nsel the patient or refer to an appropriate co#nselor $6:otify the patient,s parents c6:otify the patient,s partner "6Sche"#le a termination of pregnancy e6 Tell the patient that she is likely to ha)e a miscarriage :ormal La$s

8. A 28!year!ol" 'oman comes to the physician for follo'!#p of a right $reast l#mp. The patient first notice" the l#mp 4 months ago. +t 'as aspirate" at that time% an" cytology 'as negati)e% $#t the cyst rec#rre" a$o#t 1 month later. The cyst 'as re!aspirate" 2 months ago an"% again% the cytology 'as negati)e. The l#mp has rec#rre". E.amination re)eals a mass at 14 o,clock% appro.imately 4 cm from the areola. Ultraso#n" "emonstrates a cystic lesion. =hich of the follo'ing is the most appropriate ne.t step in managementa6Mammography in 1 year $6Ultraso#n" in 1 year c6Tamo.ifen therapy "6>pen $iopsy e6Mastectomy :ormal La$s 14.A 2(!year!ol" primigra)i" 'oman at &8 'eeks, gestation comes to the la$or an" "eli)ery 'ar" 'ith a g#sh of fl#i" an" reg#lar contractions. E.amination sho's that she is grossly r#pt#re"% contracting e)ery 2 min#tes% an" that her cer)i. is "ilate" to 4 cm. The fetal heart rate tracing is in the 144s an" reacti)e. She is a"mitte" to la$or an" "eli)ery% an" o)er the follo'ing 4 ho#rs she progresses to 8 cm "ilation. >)er the past ho#r% the fetal heart rate has increase" from a $aseline of 144 to a $aseline of 1<4. 5#rthermore% mo"erate to se)ere )aria$le "ecelerations are seen 'ith each contraction. The fetal heart rate "oes not respon" to scalp stim#lation. The "ecision is ma"e to procee" 'ith cesarean "eli)ery. =hich of the follo'ing is the reason for the cesarean "eli)ery an" the preoperati)e "iagnosisa6 5etal aci"emia $65etal "istress c65etal hypo.ic encephalopathy "6Lo' neonatal APCAA scores e6:on!reass#ring fetal heart rate tracing :ormal La$s 11.A 28!year!ol" 'oman% gra)i"a 2% para 1% at &1 'eeks, gestation comes to the la$or an" "eli)ery 'ar" 'ith fre*#ent painf#l

contractions. /er prenatal co#rse 'as significant for a #rine c#lt#re that sho'e" 144%444 colony!forming #nits7milliliter of Cro#p!B streptococci an" asthma% for 'hich she #ses an al$#terol inhaler. E.amination sho's that she is contracting e)ery 2 min#tes an" her cer)i. is centimeters "ilate" an" 144B efface". =hich of the follo'ing me"ications sho#l" this patient $e treate" 'ith "#ring la$or an" "eli)erya6Betamethasone $65olic aci" c6Magnesi#m s#lfate "6>.ytocin e6Penicillin :ormal La$s 12.A &1!year!ol" primigra)i" 'oman comes to the physician for a prenatal )isit. She is kno'n to $e /+? positi)e. She also has asthma% for 'hich she #ses an inhaler. She ha" a "iagnostic laparoscopy at age 24 for pel)ic pain an" has ha" no other s#rgeries. She has no kno'n "r#g allergies. E.tensi)e co#nseling is gi)en to the patient regar"ing )ertical transmission of /+? to the fet#s. +t is recommen"e" to her that she take antiretro)iral therapy "#ring the pregnancy to "ecrease the )ertical transmission rate. +t is also recommen"e" to her that she ha)e a sche"#le" cesarean "eli)ery. After consi"eration of these options% the patient chooses not to take the antiretro)irals an" opts for a )aginal "eli)ery. =hich of the follo'ing represents the appro.imate risk of )ertical transmission 3from the mother to the fet#s6 for this patienta62B $61B c62 B "6 4B e6144B :ormal La$s 1&. A 22!year!ol" 'oman% gra)i"a 4% para &% at &1 'eeks, gestation comes to the la$or an" "eli)ery 'ar" 'ith a g#sh of fl#i". Sterile spec#l#m e.amination re)eals a pool of fl#i" that is nitraDine positi)e an" forms ferns 'hen )ie'e" #n"er the microscope. The fetal heart rate is in the 1 4s an" reacti)e. An #ltraso#n" "emonstrates that the fet#s is in the $reech position. A cesarean "eli)ery is performe". @#ring the operation% the physician% 'ho has recei)e" no recent imm#niDations% is

st#ck 'ith a nee"le that ha" $een #se" on the patient. =hich of the follo'ing is this physician at greatest risk of contractinga6/+? $6/epatitis B c6/epatitis 2 "6Sca$ies e6Syphilis :ormal La$s 14.A 4&!year!ol" African American 'oman comes to the physician $eca#se of her concern regar"ing $reast cancer. She has no complaints at present. +n past years% she ha" note" $ilateral $reast ten"erness prior to her menses% $#t this has since a$ate". She has no me"ical pro$lems. She ha" t'o cesarean "eli)eries% $#t no other s#rgeries. She takes a lo'! "ose oral contracepti)e pill an" has no kno'n "r#g allergies. She "oes not smoke% an" her family history is negati)e. Physical e.amination is normal. All mammograms 3yearly since age 446 ha)e $een negati)e to "ate. She 'ants to kno' 'hether BA2A1 an" BA2A2 screening 'o#l" $e appropriate for her. =hich of the follo'ing is the correct responsea6 BA2A1 an" 2 screening is not recommen"e" $6BA2A1 an" 2 screening sho#l" $e performe" after age 4 c6BA2A1 an" 2 screening sho#l" $e performe" if $reast pain rec#rs "6BA2A1 screening is recommen"e" e6BA2A2 screening is recommen"e" :ormal La$s 1 . A &2!year!ol" 'oman comes to the hospital for an electi)e repeat cesarean "eli)ery. 5o#r years ago she ha" a primary cesarean "eli)ery for a nonreass#ring fetal heart rate tracing. T'o years ago she chose to ha)e an electi)e repeat cesarean "eli)ery rather than attempt a )aginal $irth after cesarean 3?BA26. /er prenatal co#rse 'as #ncomplicate" e.cept that she has mitral )al)e prolapse. An echocar"iograph "emonstrate" the mitral )al)e prolapse% $#t no other str#ct#ral car"iac "isease. =hich of the follo'ing is the correct management of this patienta6 A"minister intra)eno#s anti$iotics &4 min#tes prior to the proce"#re $6A"minister intra)eno#s anti$iotics imme"iately after the proce"#re c6A"minister intra)eno#s anti$iotics for 24 ho#rs after the proce"#re

"6A"minister oral anti$iotics < ho#rs after the proce"#re e6:o anti$iotics are nee"e" :ormal La$s 1<. A &1!year!ol" 'oman% gra)i"a 4% para 4% comes to the physician 1 "ays after a cesarean "eli)ery complaining of re"ness an" pain at the leftmost aspect of her incision. /er cesarean "eli)ery 'as performe" secon"ary to a non!reass#ring fetal heart rate tracing. She 'as feeling 'ell after the operation #ntil 4 "ays ago% 'hen she "e)elope" pain an" re"ness aro#n" her incision. /er temperat#re is &( 2 381.< 56% $loo" press#re is 1117(1 mm /g% p#lse is 117min% an" respirations are 127min. There is marke" erythema an" in"#ration aro#n" the incision. At the left margin of the incision there is a fl#ct#ant mass. =hich of the follo'ing is most appropriate ne.t step in managementa6 E.pectant management $6>ral anti$iotics only c6 +? anti$iotics only "6+ncision an" "rainage e6 Laparotomy :ormal La$s 1(.A &8!year!ol" 'oman% gra)i"a &% para 2% at term comes to the la$or an" "eli)ery 'ar" complaining of a g#sh of fl#i". E.amination sho's her to $e grossly r#pt#re"% an" #ltraso#n" re)eals that the fet#s is in )erte. presentation. The fetal heart rate is in the 124s an" reacti)e. After a fe' ho#rs% 'ith no contractions present% o.ytocin is starte". Three ho#rs later% the toco"ynamometer sho's the patient to $e ha)ing contractions e)ery min#te an" lasting for appro.imately 1 min#te 'ith almost no rest in $et'een contractions. The fetal heart rate changes from 124s an" reacti)e to a $ra"ycar"ia to the 14s. Sterile )aginal e.amination sho's that the cer)i. is < cm "ilate". =hich of the follo'ing is the most appropriate ne.t step in managementa6@iscontin#e o.ytocin $6Start magnesi#m s#lfate c6Perform forceps assiste" )aginal "eli)ery "6Perform )ac##m assiste" )aginal "eli)ery e6 Perform cesarean "eli)ery :ormal La$s

11.A 21!year!ol" primigra)i" 'oman at term comes to the la$or an" "eli)ery 'ar" 'ith a g#sh of fl#i" an" reg#lar contractions. /er prenatal co#rse 'as remarka$le for her $eing Ah negati)e an" anti$o"y negati)e. /er h#s$an" is Ah positi)e. >)er the follo'ing 14 ho#rs% she progresses in la$or an" "eli)ers a &<44!g $oy )ia a normal spontaneo#s )aginal "eli)ery. The placenta "oes not "eli)er spontaneo#sly% an" a man#al remo)al is re*#ire". To "etermine the correct amo#nt of AhoCAM 3anti!@ imm#ne glo$#lin6 that sho#l" $e gi)en% 'hich of the follo'ing is the most appropriate la$oratory test to sen"a62omplete $loo" co#nt $6Mleiha#er!Betke c6Li)er f#nction tests "6Prothrom$in time e6Ser#m potassi#m :ormal La$s 18.A 22!year!ol" primigra)i" 'oman at term comes to the la$or an" "eli)ery 'ar" $eca#se of painf#l contractions e)ery 2 min#tes. She has ha" no g#sh of fl#i" an" no $lee"ing from the )agina. /er prenatal co#rse 'as #nremarka$le. She takes no me"ications an" has no allergies to me"ications. E.amination sho's that her cer)i. is < cm "ilate" an" 144B efface"G the fet#s is at 4 station. The fetal heart rate has a $aseline in the 1 4s an" is reacti)e. The patient "esires an epi"#ral for pain relief. =hich of the follo'ing sho#l" $e gi)en orally shortly $efore the epi"#ral is place"a6Antaci" $6Anti$iotic c6Aspirin "62lear li*#i" meal e6Aeg#lar 0ho#se0 meal :ormal La$s 24. A &8!year!ol" 'oman% gra)i"a 4% para &% comes to the physician for a prenatal )isit. /er last menstr#al perio" 'as 1 'eeks ago. She has ha" no a$"ominal pain or )aginal $lee"ing. She has no me"ical pro$lems. E.amination is #nremarka$le e.cept for an 1!'eek siDe"% nonten"er

#ter#s. Prenatal la$s are sent. The rapi" plasma reagin 3APA6 test comes $ack as positi)e an" a confirmatory microhemaggl#tination assay for anti$o"ies to Treponema palli"#m 3M/A!TP6 test also comes $ack as positi)e. =hich of the follo'ing is the most appropriate pharmacotherapya6Erythromycin $6Le)oflo.acin c6Metroni"aDole "6Penicillin e6Tetracycline :ormal La$s 21. A <(!year!ol" 'oman comes to the physician $eca#se of pain 'ith #rination an" fre*#ent #rination. She has hypertension for 'hich she takes a $eta!$locker% $#t no other me"ical pro$lems. She states that she is not se.#ally acti)e. She "oes not smoke an" "rinks cran$erry 9#ice "aily. E.amination sho's mil" s#prap#$ic ten"erness an" genital atrophy $#t is other'ise #nremarka$le. Urinalysis sho's 4 to 144 le#kocytes7high po'ere" fiel" 3hpf6 an" to 14 erythrocytes7hpf. =hich of the follo'ing is the most likely ca#se of the infectiona62ar"iac "isease $62ran$erry 9#ice ingestion c6/ypoestrogenism "6:ephrolithiasis e6Se.#al interco#rse :ormal La$s 22.A &8!year!ol" 'oman% gra)i"a 2% para 1% at &4!'eeks gestation comes to the physician for a prenatal )isit. The patient,s "#e "ate 'as "etermine" $y a (!'eek #ltraso#n". /er prenatal co#rse has $een #nremarka$le. She has no complaints of contractions% loss of fl#i"% or $lee"ing from the )agina% an" her $a$y is mo)ing 'ell. E.amination "emonstrates a fetal heart rate of 1 4 an" a f#n"al height of 2( centimeters% 'hich is the same meas#rement as that "etermine" 4 'eeks ago. This patient,s f#n"al height meas#rement is most s#ggesti)e of 'hich of the follo'inga6 +nacc#rate estimate" "ate of "eli)ery 3"#e "ate6 $6+ntra#terine gro'th restriction

c6Premat#re la$or "6T'in gestation e6 Uterine cancer :ormal La$s 2&.A &&!year!ol" 'oman% gra)i"a &% para &% comes to the physician for an ann#al e.amination. She has no complaints. Past me"ical history is significant for t'o episo"es of 2hlamy"ia an" one episo"e of gonorrhea. >$stetric history is significant for three normal spontaneo#s )aginal "eli)eries 'ith gestational "ia$etes "#ring the last t'o pregnancies. She takes no me"ications. 5amily history is significant for paternal coronary artery "isease. Physical e.amination is #nremarka$le. =hich of the follo'ing inter)entions sho#l" this patient most likely ha)ea62hest .!ray e)ery & years $62oronary angiography e)ery & years c65asting gl#cose testing e)ery & years "6Mammography e)ery & years e6Pap testing e)ery & years :ormal La$s 24.A 44!year!ol" 'oman comes to the physician for an ann#al e.amination. She has no complaints. She has menses e)ery 21!&4 "ays that last for & "ays. She has no intermenstr#al $lee"ing. She has asthma% for 'hich she #ses an occasional inhaler. She ha" a t#$al ligation 14 years ago. She has no kno'n "r#g allergies. E.amination is #nremarka$le% incl#"ing a normal pel)ic e.amination. >ne of her frien"s 'as recently "iagnose" 'ith en"ometrial cancer% an" the patient 'ants to kno' 'hen an" if she nee"s to $e screene" for this. =hich of the follo'ing is the most appropriate responsea6Screening for en"ometrial cancer is not cost effecti)e or 'arrante" $6Screening is 'ith en"ometrial $iopsy an" starts at age 44 c6 Screening is 'ith en"ometrial $iopsy an" starts at age 4 "6Screening is 'ith #ltraso#n" an" starts at age 44 e6Screening is 'ith #ltraso#n" an" starts at age 4 :ormal La$s 2 . A &!year!ol" 'oman comes to the physician for an ann#al e.amination. She has no complaints. She has hypertension% for 'hich she takes a thiaDi"e "i#retic% $#t no other me"ical pro$lems. /er past

gynecologic history is significant for normal ann#al Pap tests for many years% her last $eing 2 months ago. A recent mammogram 'as negati)e. /eart% l#ng% $reast% a$"omen% an" pel)ic e.amination are #nremarka$le. =hich of the follo'ing proce"#res or tests sho#l" most likely $e performe" on this patienta62hest .!ray $6Pap test c6Pel)ic #ltraso#n" "6Prostate!specific antigen 3PSA6 e6Aectal e.amination :ormal La$s 2<. A 21!year!ol" 'oman% gra)i"a 2% para 1% at 22 'eeks, gestation comes to the physician $eca#se of a malo"oro#s )aginal "ischarge. She states that she first notice" the "ischarge 2 "ays ago an" since then it has $ecome more prof#se an" malo"oro#s. /er prenatal co#rse has $een #nremarka$le "#ring this pregnancy. /er prior pregnancy 'as complicate" $y preterm la$or an" "eli)ery at &1 'eeks, gestation. E.amination sho's a grayish )aginal "ischarge. A strong amine o"or is release" 'hen M>/ is applie" to a sample of the "ischarge. E.amination of a normal saline 30'et06 preparation re)eals n#mero#s 0cl#e0 cells. =hich of the follo'ing is the most appropriate pharmacotherapya6 :o treatment is nee"e" $6>ral metroni"aDole c6+ntram#sc#lar penicillin "6+? penicillin e6 >ral penicillin :ormal La$s 2(. A 2&!year!ol" 'oman comes to the physician $eca#se she thinks that she may $e pregnant. She misse" her last t'o perio"s an" feels 0"ifferent.0 A #rine pregnancy test is positi)e an" an #ltraso#n" re)eals a 12!'eek fet#s. The patient is )ery concerne" $eca#se she recei)e" the measles!m#mps!r#$ella 3MMA6 )accine fo#r months ago an" 'as tol" to 'ait & months $efore attempting conception. The pregnancy is "esire". The patient asks if she sho#l" ha)e a termination

of pregnancy $eca#se she 'as )accinate" shortly $efore $ecoming pregnant. =hich of the follo'ing is the most appropriate responsea6 There is no )accine risk an" termination is completely inappropriate $6The )accine risk is lo' an" is not in itself a reason to terminate c6The )accine risk is mo"erate an" termination sho#l" $e consi"ere" "6The )accine risk is high an" termination sho#l" $e strongly consi"ere" e6The )accine risk is high an" termination is man"ate" :ormal La$s 21.A 22!year!ol" 'oman comes to the physician $eca#se of a misse" menstr#al perio". She has a comple. past me"ical history. She has hypothyroi"ism% for 'hich she takes thyro.ine% she has an artificial heart )al)e% for 'hich she takes 2o#ma"in% an" she recently starte" tetracycline for acne. She "oes not think that she is pregnant $eca#se she is c#rrently on the oral contracepti)e pill% $#t% if pregnant% she 'o#l" keep the pregnancy. Physical e.amination% incl#"ing pel)ic e.amination% is #nremarka$le. Urine h#man chorionic gona"otropin 3h2C6 is positi)e. =hich of the follo'ing me"ications sho#l" the patient contin#e to take "#ring the pregnancya62o#ma"in $6>ral contracepti)e pill 3>2P6 c6Tetracycline "6Thyro.ine e6 @iscontin#e all me"ications :ormal La$s 28.A 18!year!ol" n#lligra)i" 'oman comes to the emergency "epartment $eca#se of se)ere left lo'er *#a"rant pain. She has $een noticing this pain intermittently for the past & "ays% $#t this afternoon it $ecame persistent an" se)ere an" 'as accompanie" $y na#sea an" )omiting. E.amination sho's left lo'er *#a"rant ten"erness an" a ten"er left a"ne.al mass. Urine h2C is negati)e. Pel)ic #ltraso#n" sho's a ( cm left o)arian comple. mass. =hich of the follo'ing is the most appropriate ne.t step in managementa6E.pectant management $65ollo'!#p #ltraso#n" in < 'eeks c6+ntra)eno#s anti$iotics "6Laparoscopy e6>ophorectomy

:ormal La$s &4. A 2<!year!ol" primigra)i" 'oman at 12 'eeks, gestation comes to the physician $eca#se of pain an" s'elling in her right thigh. She first note" the onset of the pain 2 "ays ago% an" since then it has gro'n 'orse. An #ltraso#n" st#"y performe" on her lo'er!e.tremity )eno#s system re)eals e)i"ence of a pro.imal throm$#s in the right leg. She is starte" on lo'!molec#lar!'eight heparin in9ections. =hich of the follo'ing is an a")antage of lo'!molec#lar!'eight heparin compare" 'ith #nfractionate" heparina6 Lo'!molec#lar!'eight heparin has a shorter half!life $6Lo'!molec#lar!'eight heparin is cheaper c6Lo'!molec#lar!'eight heparin is less likely to ca#se $irth "efects "6 Lo'!molec#lar!'eight heparin is less likely to ca#se throm$ocytopenia e6Lo'!molec#lar!'eight heparin is less likely to cross the placenta :ormal La$s &1. A 28!year!ol" female comes to the physician $eca#se of fe)ers an" $ack pain. She is other'ise healthy 'ith no significant past me"ical history. E.amination is significant for a temperat#re of &1.& 2 3141 56% mo"erate costo)erte$ral angle ten"erness% le#kocytosis% an" 'hite $loo" cells an" re" $loo" cells in the #rine. The patients is "iagnose" 'ith pyelonephritis an" starte" on intra)eno#s anti$iotics. >)er the ne.t t'o "ays% she rapi"ly impro)es% an" $y hospital "ay &% she is tolerating oral intake% )oi"ing 'itho#t "iffic#lty% feeling no pain% an" she has not ha" a fe)er for 41 ho#rs. =hich of the follo'ing is the most appropriate ne.t step in managementa62ontin#e intra)eno#s anti$iotics for 2 'eeks $6 @ischarge home an" recommen" post!coital prophyla.is c6@ischarge home off all anti$iotics "6 @ischarge home to complete a 2!'eek co#rse of oral anti$iotics e6>$tain s#rgical e)al#ation :ormal La$s &2.A &<!year!ol" 'oman% gra)i"a % para 4% at &4 'eeks, gestation comes to the physician for a prenatal )isit. She feels the $a$y mo)ing

an" has not ha" $lee"ing per )agina% contractions% or loss of fl#i". The prenatal co#rse has $een #ncomplicate" th#s far. The patient is intereste" in ha)ing a postpart#m t#$al ligation. She has many *#estions regar"ing the proce"#re% incl#"ing 'hether there is a risk of fail#re. =hich of the follo'ing represents the closest estimate for the likelihoo" of fail#re of a postpart#m t#$al ligationa61 in 14 $61 in 144 c61 in 1444 "61 in 1%444%444 e6There are no reporte" fail#res of postpart#m t#$al ligation. :ormal La$s &&.A 22!year!ol" 'oman% gra)i"a 2% para 4% at 1 'eeks, gestation comes to the physician for a prenatal )isit. She has no complaints. /er first pregnancy res#lte" in a 22!'eek loss 'hen she presente" to her physician 'ith $lee"ing from the )agina% 'as fo#n" to $e f#lly "ilate"% an" "eli)ere" the fet#s. E.amination of the patient to"ay is #nremarka$le. She "eclines to ha)e a cerclage place". =hen sho#l" this patient $egin ha)ing reg#lar cer)ical e.aminationsa614 'eeks $61< 'eeks c622 'eeks "621 'eeks e6&( 'eeks :ormal La$s &4. A 18!year!ol" female comes to the physician $eca#se she has not ha" a menstr#al perio". She e.perience" normal $reast "e)elopment thro#gh p#$erty $#t has yet to ha)e a perio". She has no other complaints. She has no me"ical pro$lems. E.amination sho's the patient to $e tall 'ith long arms an" $ig han"s. The $reasts are normal! appearing e.cept that the nipples are immat#re an" the areolae are pale. Pel)ic e.amination sho's scant p#$ic hair 'ith a $lin"!en"e" )aginal po#ch. =hich of the follo'ing is the most likely "iagnosisa6 Asherman syn"rome $6Mallmann syn"rome c6Polycystic o)arian syn"rome "6Testic#lar feminiDation syn"rome

e6T#rner syn"rome :ormal La$s & . A &!year!ol" 'oman comes to the physician $eca#se of concerns regar"ing menopa#se. She has a perio" almost e)ery month% $#t her cycle is lengthening. She is 'orrie" $eca#se her mother% her t'o ol"er sisters% an" practically all her a#nts ha)e osteoporosis. She "oes not 'ant to $e on estrogen $eca#se she is concerne" a$o#t cancer an" throm$osis. Physical e.amination is 'ithin normal limits. The patient is starte" on ralo.ifene. >n this me"ication% 'hich of the follo'ing is this patient most likely to "e)elopa6Breast cancer $6Ele)ate" cholesterol c6En"ometrial hyperplasia "6/ot flashes e6>steoporosis :ormal La$s &<.A 4(!year!ol" 'oman comes to the physician for an ann#al e.amination. >ne year ago% she 'as "iagnose" 'ith en"ometrial carcinoma an" #n"er'ent a total a$"ominal hysterectomy an" $ilateral salpingo!oophorectomy. She 'as fo#n" to ha)e gra"e +% stage +% "isease at that time. >)er the past year% she has "e)elope" se)ere hot flashes that occ#r thro#gho#t the "ay an" night an" are 'orsening. She is also concerne" $eca#se her mother an" se)eral of her a#nts ha)e se)ere osteoporosis. She 'on"ers 'hether she can take estrogen replacement therapy. =hich of the follo'ing is the most appropriate responsea6 Estrogen replacement therapy is a$sol#tely contrain"icate" $6Estrogen replacement therapy may $e #se"% an" there are no risks c6Estrogen replacement therapy may $e #se"% $#t there are risks "6 Estrogen replacement therapy 'ill lea" to $reast cancer e6 Estrogen replacement therapy 'ill lea" to cancer rec#rrence :ormal La$s &(. A &2!year!ol" 'oman% gra)i"a &% para 4% at 28 'eeks, gestation comes to the physician for a prenatal )isit. She has no complaints. She ha" a prophylactic cerclage place" at 12 'eeks, gestation $eca#se of her

history of t'o consec#ti)e 24!'eek losses. These spontaneo#s a$ortions 'ere $oth characteriDe" $y painless cer)ical "ilation% 'ith the mem$ranes fo#n" $#lging into the )agina on e.amination . Ultraso#n" no' "emonstrates her cer)i. to $e long an" close" 'ith no e)i"ence of f#nneling. =hich of the follo'ing is the most appropriate time to remo)e the cerclage from this patienta6&4!&2 'eeks $6&2!&4 'eeks c6&4!&< 'eeks "6&<!&1 'eeks e6&1!44 'eeks :ormal La$s &1.A !year!ol" 'oman comes to the physician $eca#se of hot flashes. She first note" them a$o#t 8 months ago% an" since then they ha)e $een 'orsening. She states that the flashes come on at )ario#s times thro#gho#t the "ay% $#t that they are especially intense at night. She ha" her last menstr#al perio" appro.imately months ago. /er me"ical history is significant for a p#lmonary em$ol#s at the age of &< an" se)ere "epression. She takes fl#o.etine for "epression an" has no allergies to me"ications $#t smokes one pack of cigarettes per "ay. Physical e.amination is #nremarka$le% incl#"ing a normal pel)ic e.amination. =hich of the follo'ing is the most appropriate pharmacotherapy for this patienta62loni"ine $6Estrogen an" progesterone c6Estrogen only "6Cl#cophage e6Tamo.ifen :ormal La$s

&8.A 24!year!ol" 'oman comes to the physician for an initial prenatal )isit. /er last menstr#al perio" 'as ( 'eeks ago an" a home #rine pregnancy test 'as positi)e. She has ha" no $lee"ing or a$"ominal pain. She "oes complain of increase" fatig#e lately an" some mil" na#sea an" )omiting. E.amination is significant for $oth a systolic an" a "iastolic car"iac m#rm#r. The #ter#s is 1 'eeks, siDe" an" nonten"er. =hich of

the follo'ing fin"ings is most s#ggesti)e of str#ct#ral heart "isease in this 'omana6@iastolic m#rm#r $6Enlarge" #ter#s c65atig#e "6:a#sea an" )omiting e6 Systolic m#rm#r :ormal La$s 44.A 42!year!ol" 'oman% gra)i"a 2% para 1% at 14 'eeks, gestation comes to the physician for her first prenatal )isit. She has no complaints. She has a history of Trichomonas infection% $#t no other me"ical pro$lems. E.amination is significant for a 14!'eek siDe"% nonten"er #ter#s. @#ring the spec#l#m e.amination% a Pap smear is performe" an" gonorrhea an" 2hlamy"ia screening tests are taken. The ne.t "ay% the gonorrhea test ret#rns as positi)e. =hich of the follo'ing is the most appropriate pharmacotherapya62eftria.one $62lin"amycin c6@o.ycycline "6Le)oflo.acin e6Metroni"aDole :ormal La$s 41.A 4!year!ol" 'oman comes to the physician $eca#se of hot flashes. She states that her hot flashes ha)e $een stea"ily 'orsening o)er the past year since she ha" a total a$"ominal hysterectomy an" $ilateral salpingo!oophorectomy for menometrorrhagia. Pathology from the s#rgery sho'e" lo' gra"e en"ometrial hyperplasia. She has no me"ical pro$lems an" takes no me"ications. /er family history is #nremarka$le e.cept for a strong family history of osteoporosis. She states that the hot flashes ha)e $ecome a$sol#tely "e$ilitating for her an" she 'ants to take something that 'ill gi)e her the $est chance of stopping them. =hich of the follo'ing is the most appropriate pharmacotherapya6 AlpraDolam $62loni"ine c6Estrogen "6>ral contracepti)e pill

e6Aalo.ifene :ormal La$s 42. A 22!year!ol" primigra)i" 'oman at 1 'eeks, gestation comes to the physician for her first prenatal )isit. She has ha" some na#sea $#t no other complaints. She has ha" no $lee"ing per )agina or a$"ominal pain. She ha" an o)arian cystectomy at age 11 $#t no other me"ical or s#rgical pro$lems. She takes no me"ications an" has no kno'n "r#g allergies. E.amination is #nremarka$le e.cept for an 1!'eek!siDe" non! ten"er #ter#s. The patient 'ants information on )itamin s#pplementation "#ring pregnancy. =hich of the follo'ing represents the correct amo#nt of )itamin A s#pplementation this patient sho#l" take "ailya614%444 +U $62 %444 +U c6 4%444 +U "6144%444 +U e6?itamin A s#pplementation "#ring pregnancy is not recommen"e" :ormal La$s 4&.A 28!year!ol" 'oman comes to the emergency "epartment $eca#se of a$"ominal "istension an" shortness of $reath. Appro.imately 1 'eek ago% she #n"er'ent fertility treatment 'ith o)#lation in"#ction an" oocyte retrie)al. She has a history of polycystic o)arian syn"rome $#t no other me"ical pro$lems. She ha" laparoscopy 1 year ago as part of a fertility e)al#ation. She has no kno'n "r#g allergies. /er temperat#re is &( 2 381.< 56% $loo" press#re is 14744 mm /g% p#lse is 1&47min% an" respirations are 217min. Physical e.amination is remarka$le for crackles at the l#ng $ases $ilaterally an" a "isten"e"% nonten"er a$"omen 'ith a fl#i" 'a)e. Ultraso#n" "emonstrates $ilaterally enlarge" o)aries 3each I14 cm6 an" free fl#i" in the a$"omen. Urine h2C is negati)e. =hich of the follo'ing is the most likely "iagnosisa6 Ectopic pregnancy $6/emorrhagic o)arian cyst c6>)arian hyperstim#lation syn"rome "6>)arian torsion e6T#$o!o)arian a$scess :ormal La$s

44. A 24!year!ol" 'oman% gra)i"a &% para 2% comes to the physician for her first prenatal )isit. /er last menstr#al perio" 'as 1 'eeks ago% an" a home pregnancy test 'as positi)e. She states that this pregnancy% like her last t'o pregnancies% 'as #ninten"e". She ha" $een #sing con"oms for $irth control in all three instances. She ha" normal )aginal "eli)eries 2 an" 4 years ago. =hich of the follo'ing is the most likely reason for con"om fail#rea6Allergic reaction $6Breakage c6+mproper an" inconsistent #se "6Man#fact#ring "efects e6?aginal infection :ormal La$s 4 . A ( !year!ol" 'oman comes to the physician $eca#se of a$"ominal "istension. She states that she al'ays feels $loate" an" that she gets f#ll *#ickly 'hen eating. She has hypertension% for 'hich she takes an angiotensin con)erting enDyme 3A2E6 inhi$itor% an" no other me"ical pro$lems. E.amination sho's a$"ominal "istension an" a positi)e fl#i" 'a)e. Pel)ic e.amination re)eals a large% nonten"er right a"ne.al mass. A$"ominal 2T scan "emonstrates masses on $oth o)aries% ascites% an" omental caking. 2A!12 le)el is significantly ele)ate". Ser#m alpha! fetoprotein 3A5P6 an" h#man chorionic gona"otropin 3h2C6 are negati)e. =hich of the follo'ing is the most likely "iagnosisa62horiocarcinoma $62ystic teratoma 3"ermoi"6 c6Em$ryonal carcinoma "6Epithelial o)arian cancer e6 Sertoli stromal cell t#mor :ormal La$s 4<. A &1!year!ol" 'oman% gra)i"a 1% para 4% at 1 'eeks, gestation comes to the physician for a prenatal )isit. She has ha" no $lee"ing from the )agina or a$"ominal pain an" no complaints. She has a long history of migraine hea"ache an" recently "e)elope" peptic #lcer "isease 3PU@6. E.amination sho's a nonten"er 1!'eek siDe" #ter#s $#t is other'ise

#nremarka$le. The patient is )ery concerne" that her migraine hea"aches an" peptic #lcer "isease 'ill make her pregnancy intolera$le. =hich of the follo'ing is the most appropriate responsea6Pregnancy is associate" 'ith impro)ement of migraines an" PU@. $6Pregnancy is associate" 'ith 'orsening of migraines an" PU@. c6 Pregnancy is associate" 'ith 'orsening migraines an" impro)e" PU@. "6Pregnancy is associate" 'ith impro)e" migraines an" 'orsene" PU@. e6 Pregnancy has no effect on migraines or PU@. :ormal La$s 4(.A 2&!year!ol" primigra)i" 'oman at 28!'eeks, gestation comes to the physician $eca#se of contractions. She states that they ha)e $een occ#rring e)ery &! min#tes for the past fe' ho#rs an" that they are 'orsening in intensity. E.amination re)eals that the patient is afe$rile an" her a$"omen is nonten"er. /er cer)i. is & cm "ilate"% an" the fet#s is in )erte. position. The patient is starte" on +? magnesi#m s#lfate an" penicillin an" gi)en an intram#sc#lar in9ection of $etamethasone. =hich of the follo'ing represents the most significant conse*#ence of this patient,s preterm la$ora6 2esarean "eli)ery $65orceps assiste" )aginal "eli)ery c6Maternal infection "6:eonatal premat#rity e6Sho#l"er "ystocia :ormal La$s 41. A 22!year!ol" 'oman comes to the physician for an ann#al e.amination. She has normal perio"s e)ery month an" has no complaints. She has no me"ical pro$lems $#t "oes smoke one pack of cigarettes per "ay. She has interco#rse 'ith more than one partner. E.amination is #nremarka$le% incl#"ing a normal pel)ic e.amination. A Papanicolao# smear sho's a high!gra"e s*#amo#s intraepithelial lesion. =hich of the follo'ing is the most appropriate ne.t step in managementa6Aepeat Pap smear in 1 year $6Aepeat Pap smear in < months c6Perform colposcopy

"6Perform a cone $iopsy e6Perform a hysterectomy :ormal La$s 48.A 4!year!ol" girl is $ro#ght to the physician $y her mother $eca#se of a $loo"y% greenish% malo"oro#s )aginal "ischarge. The "ischarge 'as first note" & "ays ago an" has 'orsene" since then. The girl has no other symptoms. The mother reports no concerns regar"ing a$#se of the chil". E.amination is attempte" $#t impossi$le $eca#se of the chil",s a$sol#te ref#sal to $e e.amine". Se)eral efforts at pers#asion are ma"e $#t are #ns#ccessf#l. =hich of the follo'ing is the most appropriate ne.t step in managementa6Aeass#rance an" e.pectant management $6Anti$iotic a"ministration c6Police notification "6E.amination #n"er anesthesia e6Pel)ic e.amination 'ith physical restraint :ormal La$s 4.A &1!year!ol" 'oman% gra)i"a &% para 2% at &2 'eeks, gestation comes to the physician $eca#se of $lee"ing from the )agina. She states that this morning she passe" 2 *#arter!siDe" clots of $loo" from her )agina. >ther'ise% she states that she is feeling 'ell. The $a$y has $een mo)ing normally an" she has ha" no contractions or g#sh of fl#i" from the )agina. /er o$stetrical history is significant for 2 lo'!trans)erse cesarean "eli)eries for non!reass#ring fetal heart rate tracings. An #ltraso#n" is performe" that "emonstrates a complete placenta pre)ia. 5or 'hich of the follo'ing con"itions is this patient at highest riska6 @ystocia $6+ntra#terine fetal "emise 3+U5@6 c6Placenta accreta "6Preeclampsia e6Uterine r#pt#re :ormal La$s

:ote; 2heck yo#r o'n ans'ers $efore hitting the 2heck $#tton $elo'. =hen yo# click the 2heck $#tton% a $ro'ser 'in"o' 'ill appear that contains a s#mmary of yo#r res#lts.

E.planations Block 11 E.planations

:) (*!lanation: 7he correct answer is . 7he loo! electrosur$ical e*cision !roce"ure ',((&) is relatively sim!le an" can be !erforme" in the out!atient settin$ with local anesthesia. 7he !roce"ure involves usin$ a wire loo! to e*cise lesions of the transformation #one. A benefit of ,((&, alon$ with its ease of !erformance, is that it !rovi"es tissue that can be e*amine" histolo$ically. 7he most a!!ro!riate can"i"ates for ,((& are women with hi$h4$ra"e squamous intrae!ithelial lesions '2.)I,). 7he imme"iate risks of ,((& are blee"in$ an" infection. 7he !ossible lon$4term risks inclu"e cervical incom!etence an" cervical stenosis. 7hese may seem like e*act o!!osites, but ,((& can lea" to both of them because, to a certain e*tent, it in<ures the cervi*. If the bo"yAs res!onse to this in<ury is with Htoo muchH scarrin$, then cervical stenosis can result. If too much of the cervi* is in<ure", the cervi* may be too weakene" to carry a !re$nancy to term, an" cervical incom!etence may result. Abscess an" chronic !elvic inflammatory "isease 'choice A) are not known to be lon$4term com!lications of the !roce"ure. Consti!ation an" fecal incontinence 'choice C) shoul" not be cause" by ,((&. ,((& involves the "istal !ortion of the cervi* an" shoul" not involve the intestines or rectum at all. 2ernia an" intra!eritoneal a"hesions 'choice %) shoul" not result from ,((&. 7he !roce"ure "oes not involve entry into the !eritoneal cavityG therefore, there shoul" be no risk of hernia or intra!eritoneal a"hesions. Frinary incontinence an" urinary retention 'choice () are not known to be lon$4term com!lications from ,((&, as the !roce"ure "oes not involve the bla""er. ;)

(*!lanation: 7he correct answer is A. +aternity blues is the term use" to "escribe a common !ost!artum reaction that occurs in /> to D>= of !ost!artum !atients. It is characteri#e" by tearfulness, restlessness, an" an*iety. )ym!toms ty!ically start in the first few "ays !ost!artum an" resolve within ; weeks. 2owever, certain !atients continue to have the sym!toms for several weeks. +any sym!toms may be seen in association with this "isor"er inclu"in$ hea"ache, backache, fati$ue, for$etfulness, insomnia, wee!in$, "e!ression, an*iety, an" ne$ative feelin$s towar" the newborn infant. Interestin$ly, another com!onent of the syn"rome may be e!iso"es of elation, an" such moo" lability can be es!ecially "istressin$ for the new mother. It is unclear what the etiolo$y of these sym!toms is. Certainly, the !ost!artum !erio" with a newborn can be stressful an" life chan$in$, which can certainly lea" to moo" chan$es an" a number of emotional res!onses. )ome researchers have ar$ue" that chan$es in hormone levels are at the root of the maternity blues, but this has never been "efinitively !roven. 7his !atient "oes not have evi"ence of a true !ost!artum "e!ression 'e.$., insomnia, lack of a!!etite, or anhe"onia) or !ost!artum !sychosis 'e.$., bi#arre thou$hts) an" she "oes not have any thou$hts of hurtin$ herself or her baby. 7herefore, the most likely "ia$nosis is maternity blues an" she shoul" be $iven su!!ort an" reassurance. 7he !atient must also be cautione", however, that if her sym!toms "o not resolve, or if they worsen, then she must call or return. &ost!artum "e!ression 'choice ) is a "e!ression that occurs in about :>= of !ost!artum women an" it is more serious than the maternity blues. )ym!toms may inclu"e slee! "isturbances an" chan$es in a!!etite. &ost!artum mania 'choice C) or !ost!artum !sychosis 'choice %) is a !sychiatric "isor"er that occurs in about : !er :,>>> "eliveries. It is characteri#e" by severe an*iety, a$itation, "isor"ere" thou$hts, an" confusion. 2os!itali#ation is require". &oststerili#ation "e!ression 'choice () is a "e!ression that is seen in women followin$ a tubal li$ation or other form of !ermanent sterili#ation. 7his !atient "i" not have a sterili#ation !roce"ure. 0) (*!lanation: 7he correct answer is (. acterial en"ocar"itis is a !otentially life4threatenin$ infection that can "evelo! in !atients with structural car"iac "isease who are e*!ose" to bacteremia. 7he risk of "evelo!in$ en"ocar"itis "e!en"s u!on both the car"iac con"ition an" the nature of the !roce"ure. 7he American 2eart Association !erio"ically !ublishes $ui"elines for the !revention of bacterial en"ocar"itis. Accor"in$ to the American 2eart Association $ui"elines, antibiotic !ro!hyla*is is not necessary for cesarean "elivery or normal va$inal "elivery. 7he !ossible e*ce!tion to this is for !atients with Hhi$h riskH car"iac con"itions, which inclu"e women with a history of en"ocar"itis, or who have !rosthetic heart valves, com!le* cyanotic con$enital heart "isease, or sur$ically correcte" systemic !ulmonary shunts. +itral valve !rola!se if associate" with mitral re$ur$itation '"emonstrate" by %o!!ler or a murmur) is consi"ere" a mo"erate risk con"ition an" therefore antibiotic !ro!hyla*is is not necessary. 7o a"minister intravenous antibiotics throu$hout labor 'choice A), to a"minister intravenous antibiotics 0> minutes !rior to the "elivery 'choice

), to a"minister intravenous antibiotics after the cor" is clam!e" 'choice C), or to a"minister intravenous antibiotics si* hours after the "elivery 'choice %) woul" not be necessary. As e*!laine" above, mitral valve !rola!se with re$ur$itation is consi"ere" to be a mo"erate risk con"ition an", for these con"itions, antibiotic !ro!hyla*is to !revent bacterial en"ocar"itis is not necessary. ?) (*!lanation: 7he correct answer is %. &rosta$lan"in '&.(;) $el is wi"ely use" for labor in"uction. In sim!le terms, it is use" Hto softenH an unfavorable cervi*, to make the cervi* more favorable for in"uction. It has been shown to lea" to an im!rovement in the isho!As score, a shorter "uration of labor, a nee" for lower ma*imal "oses of o*ytocin, an" a re"uce" inci"ence of cesarean "eliveries. &.(; $el can also cause uterine contractions. One of the ma<or si"e effects with &.(; $el is uterine hy!erstimulation. 7his occurs when uterine contractions come one ri$ht after the other, or when there is a tetanic contraction 'a !rolon$e" uterine contraction with no rest !erio"). In this settin$, the fetus can become hy!o*ic with a resultant bra"ycar"ia. 7his !atient ha" the $el !lace" an" C> minutes later ha" uterine hy!erstimulation. Infection 'choice A) has not been shown to cause uterine hy!erstimulation. 7his !atientAs $rou! )tre!tococcus coloni#ation is likely noncontributory. I1 flui"s 'choice ), unless o*ytocin is !resent, "o not cause uterine hy!erstimulation. &ost"ates !re$nancy 'choice C) is the reason for this !atientAs in"uction an" not likely the "irect cause of her uterine hy!erstimulation. 1a$inal e*amination 'choice () "oes not usually cause uterine hy!erstimulation. 1a$inal e*amination with a cervical e*amination can be use" for fetal scal! stimulation4rubbin$ the babyAs hea" to !rovoke an acceleration of the fetal heart rate. 2owever, this "oes not usually !rovoke uterine hy!erstimulation. /) (*!lanation: 7he correct answer is . 7his !atient has a $onorrhea infection. .onorrhea is one of the most !revalent se*ually transmitte" "iseases ')7%s) in the Fnite" )tates. It is more common in !atients of lower socioeconomic status, !atients with multi!le se*ual !artners, an" in urban settin$s. 7he causative or$anism is N. $onorrhoeae, a $ram4ne$ative aerobic "i!lococcus. F! to @>= of women that are infecte" with the or$anism will have no sym!toms at all or only va$ue sym!toms. )ym!toms that are frequently note" are va$inal "ischar$e, !ostcoital s!ottin$, an" urinary sym!toms if the urethra is involve". (*amination may reveal a cervicitis, althou$h this is not always !resent. A !atient foun" to have $onorrhea shoul" be treate" with intramuscular ceftria*one or oral cefi*ime, oflo*acin, or ci!roflo*acin. 7hese me"ications will effectively era"icate the $onococcus. 2owever, because Chlamy"ia trachomatis can be isolate" in u! to />= of women with $onorrhea an" because women treate" for $onorrhea only may soon $o on to "evelo! Chlamy"ia or !elvic inflammatory "isease '&I%), any woman receivin$ treatment for $onorrhea shoul" also be treate" for

Chlamy"ia. 7reatment of Chlamy"ia is with a#ithromycin or "o*ycycline. It is also essential that this !atientAs !artner be treate" as well. When treatin$ a !atient for $onorrhea, there is no nee" to treat the !atient with metroni"a#ole to treat bacterial va$inosis 'choice A) as well, unless there is evi"ence of a bacterial va$inosis . 2er!es 'choice C) often !resents as !ainful vesicles an" ulcers. &atients with $onorrhea "o not nee" to be treate" for her!es as well, unless there is evi"ence for her!es infection. &atients with $onorrhea are at increase" risk of havin$ other se*ually transmitte" "iseases, inclu"in$ sy!hilis 'choice %). It woul" be !ru"ent to check this !atient for sy!hilis with a bloo" test. 2owever, in the absence of a !ositive sy!hilis test, !atients with $onorrhea "o not nee" to be treate" for sy!hilis. 7richomoniasis 'choice () is treate" with metroni"a#ole. A$ain, as with bacterial va$inosis, her!es, an" sy!hilis, unless there is evi"ence of 7richomonas infection, the !atient "oes not nee"e" to be treate" for trichomoniasis. C) (*!lanation: 7he correct answer is C. 7his !atient has menorrha$ia, likely "ue to an anovulatory cycle. %urin$ the first few years after menarche, it is common for women to have some anovulatory cycles an" irre$ular menses. %urin$ an anovulatory cycle, because no e$$ is release" an" no cor!us luteum is forme", there is no !ro$esterone !ro"uction. 7his lack of !ro$esterone means that the en"ometrium is stimulate" by uno!!ose" estro$en. 7his lea"s to a buil"u! of the en"ometrial linin$ an" often, when the !erio" "oes come, menorrha$ia. 7he treatment for this ty!e of blee"in$ is with oral contrace!tive !ills. 7he !ills, by !rovi"in$ estro$en an" !ro$esterone, can hel! to stabili#e the en"ometrium an" halt the blee"in$. ecause this !atient is blee"in$ heavily an" now has a si$nificant hematocrit "ro! '0>=), it is reasonable to !rovi"e hi$h "oses of hormones. A common metho" of "oin$ this is to have the !atient take three !ills !er "ay for three "ays, followe" by ; !ills !er "ay for three "ays, followe" by one !ill !er "ay until the !ack is finishe". It is im!ortant in this case to note that !re$nancy was rule" out with a ne$ative urine hC. test. It is essential to rule out !re$nancy in a youn$ woman who !resents with blee"in$ from the va$ina. (*!ectant mana$ement 'choice A) woul" not be a!!ro!riate. 7his !atient is losin$ enou$h bloo" to have "ro!!e" her hematocrit to 0>=. If one "oes not intervene, there is the risk that the !atient will continue to blee" an" to "ro! her hematocrit even further. &atients with "ysfunctional uterine blee"in$ such as this can lose enou$h bloo" to require a bloo" transfusion with the corres!on"in$ risks 'e.$. infection an" transfusion reaction.) 2ysterosco!y 'choice ) woul" not be the most a!!ro!riate o!tion. With such severe va$inal blee"in$, hysterosco!y will likely not !rovi"e sufficient visuali#ation of the en"ometrium. Also, hysterosco!y e*!oses the !atient to the risks of sur$ery 'e.$. !erforation of the uterus, "ama$e to internal or$ans) for a !roblem that can be mana$e" effectively me"ically. ,a!arosco!y 'choice %) an" la!arotomy 'choice () woul" not be a!!ro!riate. 7his !atient is havin$ uterine blee"in$ that is most likely comin$ from insi"e the uterus 'i.e. the en"ometrial linin$). ,a!arosco!y an" la!arotomy will !rovi"e a view of only the e*terior of the uterus 'the serosal surface) an" thus will not be an effective a!!roach to this !roblem.

D) (*!lanation: 7he correct answer is C. &hysiolo$ic leukorrhea can be seen "urin$ ; "ifferent !erio"s of chil"hoo". )ome female neonates "evelo! a !hysiolo$ic leukorrhea shortly after birth as maternal circulatin$ estro$ens stimulate the newbornAs en"ocervical $lan"s an" va$inal e!ithelium. 7he "ischar$e in these neonates is often $ray an" $elatinous. &hysiolo$ic leukorrhea can also be seen "urin$ the months !rece"in$ menarche. %urin$ this time, risin$ estro$en levels lea" to a whitish "ischar$e not associate" with any sym!toms of irritation. 7his !atient has a whitish "ischar$e, no other sym!toms, an" she has ha" normal !ubertal "evelo!ment u! to this !oint. 7he "ischar$e itself has no characteristics of infection. 7herefore, !hysiolo$ic leukorrhea is the most likely "ia$nosis. acterial va$inosis 'choice A) is not the most likely "ia$nosis in this !atient because the "ischar$e is not malo"orous an" there are no clue cells seen on microsco!ic e*amination of the "ischar$e. Can"i"a vulvova$initis 'choice ) is not the most likely "ia$nosis because the "ischar$e is not thick an" white 'or Hcotta$e4cheeseH4like) an" the !atient has no irritative sym!tomatolo$y. )y!hilis 'choice %) most often !resents with a !ainless ulcer 'calle" a chancre) or is foun" with serolo$ic testin$. A nonmalo"orous, whitish va$inal "ischar$e in a :;4year4ol" female who is not se*ually active is almost certainly not evi"ence of sy!hilis. 7richomoniasis 'choice () is also hi$hly unlikely in this !atient an" the lack of trichomona"s on the microsco!ic e*amination effectively rules out this "ia$nosis. @) (*!lanation: 7he correct answer is A. 7he "ecision of whether to have a termination of !re$nancy is a "ee!ly !ersonal one. 7his !atient has <ust been notifie" that she is !re$nant with a :>4week fetus. )he is unsure whether she wants to kee! her !re$nancy or terminate it. In this settin$, the most a!!ro!riate ne*t ste! is to counsel the !atient re$ar"in$ her o!tions or refer the !atient for counselin$. In a balance" way, the !atient shoul" be fully informe" of all of her o!tions inclu"in$ raisin$ the chil" herself, !lacin$ the chil" u! for a"o!tion, an" abortion. 7o notify the !atientAs !arents 'choice ) is not a!!ro!riate. )uch an act woul" violate the !atientAs confi"entiality. A 0?4year4ol" woman is an a"ult an" issues of !arental notification "o not a!!ly. 7o notify the !atientAs !artner 'choice C) is not a!!ro!riate. 7his notification woul" also violate confi"entiality. 7o sche"ule a termination of !re$nancy 'choice %) woul" not be a!!ro!riate. 7his !atient has <ust informe" the !hysician that she is unsure what she wants to "o. 7o <ust $o ahea" an" sche"ule the termination without !ro!er counselin$ of the !atient woul" not be a balance" or !ro!er a!!roach for the !atient. 7o tell the !atient that she is likely to have a miscarria$e 'choice () is ina!!ro!riate. 7his !atient may have a miscarria$e, as mi$ht any !atient with a first4 trimester !re$nancy. 2owever, once an intrauterine !re$nancy with fetal car"iac activity is i"entifie", the risk of miscarria$e is a!!ro*imately :>=. 7herefore, she is most likely not to have a miscarria$e.

I) (*!lanation: 7he correct answer is %. reast lum!s are a common com!laint in women. +any of these masses are beni$n !rocesses. eni$n con"itions of the breast inclu"e fibrocystic "isease, fibroa"enomas, $alactoceles, abscesses, an" necrosis. It is a!!ro!riate to as!irate a !al!able macrocyst in the breastG the flui" shoul" be !lace" on a sli"e an" sent for cytolo$ic evaluation. If the cytolo$y is ne$ative, no further treatment is nee"e". )ome woul" ar$ue that if the cyst recurs, it may be as!irate" a$ain. 2owever, when a lesion recurs twice, as has occurre" in this !atient, o!en bio!sy is warrante". 7o wait to !erform mammo$ra!hy in : year 'choice A) or ultrasoun" in : year 'choice ) woul" be incorrect mana$ement. -irst, if a mali$nancy is !resent, waitin$ another year will allow !ro$ression of the cancer. )econ", the mammo$ram is not "efinitive. Ima$in$ can contribute information to the worku! of a breast mass, but the "efinitive "ia$nosis rests on histolo$ic evaluation. 7amo*ifen thera!y 'choice C) is use" to both !revent an" treat breast cancer. 2owever, this !atient "oes not yet have a "ia$nosis. )he has a cystic mass that has been as!irate" twice an" has recurre" twice. )he therefore requires a bio!sy to establish a "ia$nosis !rior to the institution of any treatment. +astectomy 'choice () woul" not be in"icate" for this !atient. A$ain, this !atient "oes not have a "ia$nosis, an" to !erform a mastectomy for a recurrent cyst woul" be ina!!ro!riate. :>) (*!lanation: 7he correct answer is (. ,abor an" "elivery re!resents a !rocess of stress for the fetus. With each uterine contraction, bloo" flow to the !lacenta "ecreases, an" the fetus is e*!ose" to transient hy!o*ia. As the labor !ro$resses an" more an" more contractions occur, this hy!o*ia can eventually lea" to a chan$e from aerobic to anaerobic metabolism. 7his chan$e can lea" to a buil"u! of aci" in the fetus, or fetal aci"emia. 2owever, most fetuses tolerate the stress of labor an" "elivery <ust fine. 7he fetus has a variety of !rotective mechanisms, inclu"in$ a bloo" bufferin$ system an" the "ivin$ refle* 'a lowerin$ of the heart rate in times of hy!o*ic stress), to !rotect it from becomin$ "an$erously aci"emic. (lectronic fetal monitorin$ is use" to "etermine whether the fetus is becomin$ "an$erously aci"emic or Hstresse"H "urin$ labor so that "elivery can occur !rior to hy!o*ic "ama$e to or$ans. Fnfortunately, electronic fetal monitorin$ is not a very s!ecific tool for i"entifyin$ fetal aci"emia. +any fetuses with a non4reassurin$ fetal heart rate tracin$ "o not have aci"emia an" are not in "istress. 2owever, it can be very "ifficult to "istin$uish non4 aci"emic fetuses with non4reassurin$ fetal heart rate tracin$s from aci"emic fetuses with non4reassurin$ fetal heart rate tracin$s. 7hus, the "elivery of many fetuses is e*!e"ite" because of the concern for fetal aci"emia when, in fact, the fetus is not aci"emic at all. 7hus, it is most accurate to state, as is in this case, that the fetus was "elivere" because of the non4reassurin$ fetal heart rate tracin$. -etal aci"emia 'choice A) is not the reason for "elivery. In fact, there is a stron$ likelihoo" that this fetus is not aci"emic at all. -etal

"istress 'choice ) is not the reason for "elivery. 7here is a stron$ likelihoo" that this fetus is !erfectly healthy an" will have hi$h neonatal A&.A6 scores an" no "istress at all. -etal hy!o*ic ence!halo!athy 'choice C) is not the reason for "elivery. 7he "esire to !revent hy!o*icBaci"emic "ama$e to or$ans, inclu"in$ the brain, is the reason for e*!e"itin$ "elivery. 2owever, the non4reassurin$ fetal tracin$ "oes not in"icate that hy!o*ic ence!halo!athy is necessarily occurrin$. ,ow neonatal A&.A6 scores 'choice %) can be a marker of fetal aci"emia. 2owever, many fetuses with non4reassurin$ fetal heart rate tracin$s "o not have low neonatal A&.A6 scores. ::) (*!lanation: 7he correct answer is (. 7he .rou! )tre!tococcus '. )) is a bacterium that is a !art of the normal bacterial coloni#ation of many women. %urin$ !re$nancy, as many as ;>4?>= of women will be coloni#e" with . ). +ost babies born to coloni#e" mothers will not "evelo! infection with . ). 2owever, a!!ro*imately : to ? = of neonates will "evelo! infection. 7he likelihoo" of infection is increase" if the mother has !reterm labor an" "elivery 'P 0D weeks), !rolon$e" ru!ture of the membranes 'O:@ hours), or intra!artum tem!erature $reater than 0@.> C ':>>.? -). 7wo !rimary metho"s are use" to "etermine which women will receive antibiotics "urin$ labor. 7he first metho" is base" u!on risk factors. 7he five risk factors are: :. 2istory of a . )4affecte" neonate. ;. Frine culture with . ). 0. &reterm labor 'P0D weeks). ?. +embranes ru!ture" for $reater than ei$hteen hours in labor. /. 7em!erature $reater than 0@.> C ':>>.? -) in labor. A woman with any one of these five risk factors shoul" receive antibiotics in labor. 7he secon" metho" is base" on screenin$, with !re$nant women bein$ screene" for . ) at 0/ to 0D weeks with a culture of the va$ina, !erineum, an" anus. Women shoul" be screene" only if they "o not have a history of a . )4affecte" neonate or . ) bacteriuria. 7his !atient has . ) bacteriuriaG therefore, she "i" not un"er$o screenin$. )he shoul" be treate" with !enicillin "urin$ labor an" "elivery. etamethasone 'choice A) is a corticosteroi" that is $iven to women to accelerate fetal maturity to hel! !revent neonatal res!iratory "istress syn"rome an" other sequelae of !rematurity. 7his !atient is at 0@ weeksA $estation an", therefore, "oes not require betamethasone. -olic aci" 'choice ) is a su!!lement that women shoul" take !reconce!tionally an" "urin$ !re$nancy 'not "urin$ labor an" "elivery) to hel! !revent neural tube "efects. +a$nesium sulfate 'choice C) is use" in obstetrics to !revent !reterm labor an" for sei#ure !ro!hyla*is. 7his !atient "oes not have !reterm labor an" "oes not have !reeclam!sia. O*ytocin 'choice %) is $iven to women to in"uce or to au$ment labor. 7his !atient, however, a!!ears not to nee" o*ytocin as she is contractin$ every / minutes an" !ro$ressin$ in labor. :;) (*!lanation: 7he correct answer is C. )tu"ies have "emonstrate" that in the absence of maternal treatment with antiretroviral thera!y or sche"ule" cesarean "elivery, the rate of vertical transmission is a!!ro*imately

;/=. 7hus, all !re$nant women shoul" be offere" 2I1 testin$ to i"entify those !atients who are infecte" so that they may receive antiretroviral thera!y an" be offere" sche"ule" cesarean "elivery to "ecrease the rate of vertical transmission. ;= 'choice A) re!resents the a!!ro*imate rate of vertical transmission in women who receive antiretroviral thera!y "urin$ the !re$nancy an" a sche"ule" cesarean "elivery 'i.e., a cesarean "elivery !rior to the onset of labor or ru!ture of membranes.) @= 'choice ) re!resents the a!!ro*imate rate of vertical transmission when women are treate" with antiretroviral thera!y "urin$ !re$nancy an" the neonate is treate" !ost!artum. 7his rate was i"entifie" in the lan"mark stu"y from the &e"iatric AI%) Clinical 7rials .rou! >DC stu"y. 7his stu"y showe" that ante!artum, intra!artum, an" !ost!artum #i"ovu"ine '3%1) use woul" re"uce the vertical transmission rate from ;/= to @=. />= 'choice %) an" :>>= 'choice () are incorrect. :0) (*!lanation: 7he correct answer is . )tu"ies have shown that sur$eons can rea"ily acquire he!atitis virus from !atients. 7he risk of acquirin$ he!atitis is si$nificantly hi$her than the risk for 2I1, an" somewhat hi$her than the risk for he!atitis C. 7hus, it is essential that health care workers be immuni#e" a$ainst the he!atitis virus. 7he immuni#ation sche"ule is for a"ministration of the vaccine at :, ;, an" C months. 7he Centers for %isease Control an" &revention recommen"s that !ostvaccination testin$ for antibo"ies be !erforme" to i"entify an a"equate res!onse to the immuni#ation. In"ivi"uals who "o not "emonstrate the formation of antibo"ies after the immuni#ations are $iven shoul" be teste" for he!atitis surface anti$en to ensure that they havenAt alrea"y been infecte". With immuni#ation, the risk of acquirin$ he!atitis from a nee"le stick in<ury is si$nificantly lessene". 2I1 'choice A) can be transmitte" throu$h nee"le4stick in<ury. 2owever, the risk of this transmission is less than that of he!atitis in in"ivi"uals who have not been immuni#e". 2e!atitis C 'choice C) a!!ears to be more transmissible throu$h nee"le4stick in<ury than 2I1, but less transmissible than he!atitis . 2owever, because there is no immuni#ation for he!atitis C available yet, an" because the infection is so wi"es!rea" in the !o!ulation, the risk of transmission is of $rave concern. )cabies 'choice %) is a skin !arasite that is transmitte" throu$h !hysical contact. )y!hilis 'choice () is a se*ually transmitte" "isease that is most often transmitte" throu$h se*ual contact. 7ransmission throu$h nee"le4stick in<ury is not a !rimary route. :?) (*!lanation: 7he correct answer is A. Of the cases of breast cancer that are heritable, a!!ro*imately @>= are "ue to mutations in 6CA: an" 6CA;. 6CA: is associate" with hi$h risk for breast an" ovarian cancer. 6CA; is associate" with a hi$h risk of female an" male breast cancer. On the basis of our current un"erstan"in$, however, less than :>= of all breast cancer cases can be consi"ere" to be heritable. 7herefore, the total number of breast cancer cases

associate" with 6CA: an" 6CA; mutations is a small !ercenta$e of the total number of breast cancer cases. -urthermore, there are numerous mutations that can occur in the 6CA: an" 6CA; $enes an" can be relate" to an increase" cancer risk. )ome !atients who have a mutation associate" with cancer will not $o on to "evelo! cancer. Other !atients may have a stron$ family history of breast cancer but no i"entifiable mutation. At !resent, therefore, screenin$ of the $eneral !o!ulation is not recommen"e". 7his !atient has no family history an" is not in a hi$h4risk $rou!. 2er !rior breast ten"erness was likely mastal$ia relate" to the !remenstrual !hase. 7herefore, 6CA: an" ; screenin$ woul" not be recommen"e" for this !atient. 7o state that 6CA: an" ; screenin$ shoul" be !erforme" after a$e /> 'choice ) is incorrect. As note" above, $iven the limitations of the testin$ for 6CA: an" ; mutations, screenin$ of the $eneral !o!ulation is not recommen"e". 7o state that 6CA: an" ; screenin$ shoul" be !erforme" if breast !ain recurs 'choice C) is incorrect. 7his !atient "oes not nee" screenin$, not because her breast !ain has resolve", but rather because 6CA: an" ; screenin$ is not a!!ro!riate for the $eneral !o!ulation at this time. As note" above, her breast !ain was likely cyclic mastal$ia secon"ary to hormonal chan$es !rior to menses. 7o state that either 6CA: screenin$ 'choice %) or 6CA; screenin$ 'choice () is recommen"e" is not correct. As e*!laine" above, screenin$ for neither of these is recommen"e". :/) (*!lanation: 7he correct answer is (. +itral valve !rola!se affects a!!ro*imately /= of women of chil"bearin$ a$e. Consequently, the issue of mitral valve !rola!se an" the nee" for antibiotics comes u! quite often in obstetrics, !articularly with "elivery 'either va$inal "elivery or cesarean "elivery). acterial en"ocar"itis is a life4threatenin$ infection that can "evelo! in !atients with structural car"iac "isease who are e*!ose" to bacteremia. 7he risk for any $iven !roce"ure "e!en"s u!on the nature of the !roce"ure itself an" on the nature of the car"iac lesion. &erio"ically, the American 2eart Association !ublishes $ui"elines for the !revention of bacterial en"ocar"itis. Accor"in$ to the American 2eart Association $ui"elines, antibiotic !ro!hyla*is is not necessary for cesarean "elivery or normal va$inal "elivery. 7he !ossible e*ce!tion to this is for !atients with Hhi$h riskH car"iac con"itions, which inclu"es women with a history of en"ocar"itis or who have !rosthetic heart valves, com!le* cyanotic con$enital heart "isease, or sur$ically correcte" systemic !ulmonary shunts. +itral valve !rola!se, if associate" with mitral re$ur$itation '"emonstrate" by %o!!ler or a murmur), is consi"ere" a mo"erate risk con"ition an", therefore, antibiotic !ro!hyla*is is not necessary. 7his !atient, therefore, "oes not require antibiotics !rior to, "urin$, or after her cesarean "elivery. 7o a"minister intravenous antibiotics 0> minutes !rior to the !roce"ure 'choice A), imme"iately after the !roce"ure 'choice ), ;? hours after the !roce"ure 'choice C), or to a"minister oral antibiotics C hours after the !roce"ure 'choice %) woul" all be unnecessary. As e*!laine" above, the reason for a"ministerin$ antibiotics to women with structural car"iac "isease is to !revent bacterial en"ocar"itis. acterial en"ocar"itis is a !otentially fatal con"ition. 2owever, there are "ifferent "e$rees of structural car"iac "isease. +itral valve !rola!se with re$ur$itation is

consi"ere" to be a mo"erate risk con"ition. 7he American 2eart Association "oes not recommen" en"ocar"itis !ro!hyla*is for women with mo"erate risk con"itions un"er$oin$ va$inal or cesarean "elivery. :C) (*!lanation: 7he correct answer is %. 7his !atient most likely has a woun" abscess. When antibiotic !ro!hyla*is is use", woun" infections occur at a rate of a!!ro*imately := after cesarean "eliveries. 2owever, this !atient a!!ears to have more than a cellulitis. 7he fluctuant mass at the leftmost as!ect of the incision is hi$hly likely to be an abscess. 7he !ro!er treatment for a woun" abscess is with incision an" "raina$e. 7his !atient is unlikely to im!rove with e*!ectant mana$ement 'choice A). An abscess almost always requires incision an" "raina$e for cure. (*!ectant mana$ement may lea" to worsenin$ of the infection, with the !ossibility of s!rea" to a"<acent structures 'e.$., fascia) or to bacteremia an" se!sis. Oral antibiotics only 'choice ) or I1 antibiotics only 'choice C) may not resolve the abscess. Antibiotics often "o not !enetrate the abscess cavity. ,a!arotomy 'choice () is !robably not necessary for this !atient. )he has a woun" abscess that shoul" be a""resse" with incision an" "raina$e. In the !rocess of the incision an" "raina$e, the fascia shoul" be checke" to ensure that it is intact. As lon$ as the fascia is intact an" there is no intra4ab"ominal !rocess, there is no nee" for la!arotomy. :D) (*!lanation: 7he correct answer is A. 7his !atient has the fin"in$s most consistent with uterine hy!erstimulation4more than / contractions in :> minutes, contractions lastin$ ; minutes or more, or contractions of normal "uration occurrin$ within : minute of each other an" a non4reassurin$ fetal heart rate tracin$. O*ytocin is one of the most frequently use" me"ications in the F.). It is very effective at !ro"ucin$ contractions an" use" very often for in"uction of labor. 7he most common a"verse effect with o*ytocin is a non4reassurin$ fetal heart rate !attern brou$ht about by uterine hy!erstimulation. ecause it has a very short half4life '04/ minutes), "iscontinuin$ the o*ytocin often resolves the hy!erstimulation quickly. In this !atient, with a bra"ycar"ia to the @>s, this ste! is most a!!ro!riate. In situations where the fetal heart rate tracin$ is not as non4reassurin$, the o*ytocin "osa$e may be re"uce" rather than "iscontinue" com!letely. If uterine hy!erstimulation in"uce" by o*ytocin "oes not res!on" to shuttin$ the o*ytocin off, one can start ma$nesium sulfate 'choice ) or $ive terbutaline. oth of these may be $iven intravenously to treat uterine hy!erstimulation that "oes not res!on" to other measures. 7o !erform force!s4assiste" 'choice C) or vacuum4assiste" 'choice %) va$inal "elivery woul" be contrain"icate". 7his !atientAs cervi* is only C cm "ilate". -orce!s an" vacuum are not use" unless the cervi* is fully "ilate". 7o !erform a cesarean "elivery 'choice () woul" not be a!!ro!riate !rior to tryin$ other ste!s. 7his fetus most likely is not sufferin$ a metabolic aci"emia, base" on the fact that its reassurin$ heart rate tracin$ is in the :;>s an" reactive. Its bra"ycar"ia is

"irectly relate" to the hy!erstimulation, which is cause" by the o*ytocin. 7hus, efforts shoul" be ma"e to mana$e the fetal "istress with conservative measures !rior to resortin$ to cesarean "elivery. :@) (*!lanation: 7he correct answer is . Women who are 6h ne$ative are at risk for "evelo!in$ 6h isoimmuni#ation. 6h isoimmuni#ation occurs when an 6h4ne$ative mother becomes e*!ose" to the 6h anti$en on the re" bloo" cells of an 6h4!ositive fetus. 7his e*!osure may lea" the motherAs immune system to become sensiti#e" to the 6h anti$en such that in a future !re$nancy with an 6h4!ositive fetus, the motherAs immune system may HattackH the 6h anti$en on the fetal re" bloo" cells. 7his immune res!onse may lea" to the "evelo!ment of fetal anemia, hy"ro!s, an" "eath. 7o !revent 6h isoimmuni#ation from occurrin$, 6h4ne$ative women who are not 6h alloimmuni#e" shoul" receive 6ho.A+ 'anti4% immune $lobulin) at ;@ weeksA $estation, within D; hours after the birth of an 6h4!ositive infant, after a s!ontaneous abortion, or after invasive !roce"ures such as amniocentesis. 6ho.A+ shoul" also be stron$ly consi"ere" in cases of threatene" abortion, antenatal blee"in$, e*ternal ce!halic version, or ab"ominal trauma. 7he amount that is usually $iven after the "elivery of an 6h4!ositive fetus is 0>> R$. 7his amount is sufficient to cover a fetal to maternal hemorrha$e of 0> m, 'or :/ m, of fetal cells). 2owever, some women will have a fetal to maternal hemorrha$e that is in e*cess of this 0> m,4es!ecially in cases such as manual removal of the !lacenta 'like this !atient ha") or !lacental abru!tion. 7o "etermine the amount of fetal to maternal hemorrha$e that occurre", it is necessary to !erform a 5leihauer4 etke test. 7his aci"4"ilution !roce"ure allows fetal re" bloo" cells to be i"entifie" an" counte". 5nowin$ the amount of fetal to maternal hemorrha$e that took !lace allows the correct amount of 6ho.A+ to be $iven. A com!lete bloo" count 'choice A) will "emonstrate the amount of maternal hemorrha$e, but not the amount of fetal to maternal hemorrha$e. ,iver function tests 'choice C), !rothrombin time 'choice %), an" serum !otassium 'choice () "o not allow for the "etermination of the amount of fetal to maternal hemorrha$e. :I) (*!lanation: 7he correct answer is A. As!iration !neumonitis is a ma<or cause of anesthesia4relate" "eath in obstetrics. +ost often, these as!iration events occur with the use of $eneral anesthesia. &re$nant !atients are at $reater risk for as!iration because of the "elaye" $astric em!tyin$ that occurs "urin$ !re$nancy an" labor. &re$nancy is associate" with increase" levels of !ro$esterone an" "is!lacement of the !ylorus by the !re$nant uterus. ,abor is associate" with !ain an" stress. All of these factors lea" to "elaye" $astric em!tyin$. As!iration !neumonitis is cause" by aci"ic $astric <uices enterin$ the lun$s an" in"ucin$ a sometimes4lethal chemical !neumonitis. When e!i"ural anesthesia is a"ministere", there is a risk of com!lications, inclu"in$ the "evelo!ment of total s!inal anesthesia. 7he

treatment for this com!lication is !ositive4!ressure ventilation with :>>= o*y$en a"ministere" throu$h an en"otracheal tube. 7herefore, when an e!i"ural is $oin$ to be !lace", the !atient shoul" be $iven an antaci" 'often 0> m, of >.0 m,B, so"ium citrate with citric aci", calle" icitra) to increase the stomach !2. 7his will hel! to !revent as!iration !neumonitis shoul" an as!iration event take !lace "urin$ the a"ministration of $eneral anesthesia. It is not necessary to $ive an antibiotic 'choice ) !rior to the a"ministration of an e!i"ural. Antibiotics are use" "urin$ labor for the !revention of $rou! )tre!tococcus se!sis, for !atients with chorioamnionitis, for !atients in nee" of valve or en"ocar"itis !ro!hyla*is, or "urin$ cesarean "elivery for the !revention of infection. As!irin 'choice C) is not $iven !rior to the !lacement of an e!i"ural. A clear liqui" meal 'choice %) or a re$ular HhouseH meal 'choice () shoul" not be $iven to !atients !rior to the !lacement of an e!i"ural. Intake of foo" or liqui"s "urin$ labor !laces the !atient at $reater risk of as!iration !neumonitis. &atients in labor shoul" be allowe" small si!s of water or ice chi!s. ;>) (*!lanation: 7he correct answer is %. 7his !atient has sy!hilis. )y!hilis is a "isease cause" by 7re!onema !alli"um, a s!irochete. A !ainless ulcer, calle" a chancre, ty!ically foun" on the va$ina or cervi*, characteri#es !rimary sy!hilis. If !rimary sy!hilis is untreate" it can !ro$ress to secon"ary sy!hilis, which is characteri#e" by Hmoth4eatenH alo!ecia, a maculo!a!ular skin rash involvin$ the !alms an" soles, an" white !atches on the ton$ue. .umma formation, car"iac lesions, an" central nervous system abnormalities characteri#e tertiary sy!hilis. )y!hilis in !re$nancy is associate" with increase" rates of !reterm "elivery, intrauterine $rowth retar"ation, an" fetal "emise. 2owever, the most "evastatin$ com!lication of sy!hilis in !re$nancy is con$enital infection. Con$enital infection of the fetus can lea" to severe fetal morbi"ity an" mortality. 7he key to !reventin$ con$enital infection of the fetus is a"equate treatment of the mother. 7herefore, every woman shoul" be teste" for sy!hilis "urin$ routine !renatal care. 7he 6&6 test an" 1enereal %isease 6esearch ,aboratory '1%6,) are screenin$ tests for sy!hilis an" are not entirely s!ecific for 7re!onema !alli"um infection. Certain other con"itions, such as autoimmune syn"romes an" !re$nancy itself, can $ive a falsely !ositive 6&6 test. 7herefore, the 6&6 test shoul" be followe" u! with a test that is s!ecific for sy!hilis, such as the +2A47& test. When both of these tests are !ositive an" there is no history of sy!hilis infection an" treatment, then the !atient shoul" be treate" with intramuscular !enicillin. (rythromycin 'choice A) is recommen"e" by some as the first4line treatment for chlamy"ia in !re$nancy 'others recommen" a#ithromycin). (rythromycin is not the "ru$ of choice for sy!hilis in !re$nancy. ,evoflo*acin 'choice ) an" the other fluoroquinolones are consi"ere" contrain"icate" in !re$nancy because of an association with musculoskeletal con$enital anomalies. +etroni"a#ole 'choice C) is use" "urin$ !re$nancy for the treatment of bacterial va$inosis an" trichomoniasis. It is not use" for treatment of sy!hilis. 7etracycline 'choice () is contrain"icate" "urin$ !re$nancy because of effects on fetal bones an" teeth.

;:) (*!lanation: 7he correct answer is C. 7his !atient has a !resentation that is most consistent with urinary tract infection 'F7I). 7wo of the ma<or risk factors for uncom!licate" F7I are se*ual intercourse an" hy!oestro$enism. )e*ual intercourse is believe" to lea" to urinary tract infection by intro"ucin$ coloni#in$ bacteria into the bla""er. )e*ual intercourse has been shown to increase the number of bacteria in the urine u! to ten times. 2y!oestro$enism is believe" to be a risk factor for F7I because it is known that !ostmeno!ausal women not receivin$ estro$en re!lacement thera!y '(67) are at $reater risk for "evelo!in$ a F7I com!are" with those women who "o use (67. -urthermore, estro$en a"ministration has been shown to !revent recurrent infection. Car"iac "isease 'choice A) is a ma<or risk factor for a number of con"itions. 2owever, car"iac "isease is not a known risk factor for F7I. Cranberry <uice in$estion 'choice ) has, for many years, been believe" to hel! !revent F7Is. +any in the me"ical establishment viewe" this as an Hol" wives tale.H 2owever, there have been many stu"ies that have shown that cranberry <uice contains substances that inhibit bacterial a"herence. +oreover, a recent stu"y showe" that el"erly women that "rank cranberry <uice have lower rates of !yuria an" bacteriuria an" a "ecrease" nee" for antibiotics. Ne!hrolithiasis 'choice %) can be a risk factor for the "evelo!ment of an eventual infection, but it is not as common a risk factor as is hy!oestro$enism or se*ual intercourse. -urthermore, this !atient has no evi"ence of ne!hrolithiasis, which ty!ically causes severe to e*cruciatin$ e!iso"es of !ain. )e*ual intercourse 'choice (), as note" above, is a well4known risk factor for the "evelo!ment of a F7I. )e*ually active women with recurrent F7Is may be treate" with a sin$le "ose of antibiotic !ro!hylactically after intercourse. 7his !atient, however, has state" that she is not se*ually active. ;;) (*!lanation: 7he correct answer is . -un"al hei$ht measurement is a !ortion of the !hysical e*amination that shoul" be !erforme" routinely "urin$ !renatal care. It is !erforme" by !lacin$ a measurin$ ta!e on the !ubic sym!hysis an" measurin$ to the to! of the fun"us. etween the $estational a$es of :@ to 0? weeks, there is a rou$h correlation between weeks of $estation an" fun"al hei$ht in centimeters. -or e*am!le, a woman at ;C weeksA $estation shoul" have a fun"al hei$ht that is rou$hly ;C centimeters. 7his !atient is at 0> weeksA $estation an" has a fun"al hei$ht of ;D centimeters. -urthermore, an" !erha!s more im!ortantly, there has been no chan$e in the fun"al hei$ht over the !ast four weeks. 7hese fin"in$s are concernin$ for intrauterine $rowth restriction 'IF.6). IF.6 is a "isor"er in which the fetus is not $rowin$ a!!ro!riately. It is most commonly "efine" as an estimate" fetal wei$ht less than the :>th !ercentile for a $iven $estational a$e. .iven that this !atientAs fun"al hei$ht "oes not a!!ear to have increase" over the !ast ? weeks an" that it is 0 centimeters less than e*!ecte", IF.6 is of concern an" this !atient shoul" be sent for an ultrasoun" to evaluate fetal si#e. 7his !atient is unlikely to have an inaccurate estimate" "ate of "elivery '"ue "ate) 'choice A) because her "ue "ate was "etermine" by a D4week

ultrasoun". Fltrasoun" "atin$ of a !re$nancy is more accurate the earlier in !re$nancy that it is !erforme" an" a D4week ultrasoun" is consi"ere" e*cellent for establishin$ a "ue "ate. &remature labor 'choice C) woul" not be a concern in this !atient with no contractions an" no other sym!toms. A twin $estation 'choice %) shoul" have been seen on the D4week ultrasoun". -urthermore, a fun"al hei$ht that is less than the $estational a$e woul" !re"ict makes twins less likely. Fterine cancer 'choice () is very uncommon "urin$ !re$nancy an" woul" not be e*!ecte" to !resent as "ecrease" fun"al hei$ht. ;0) (*!lanation: 7he correct answer is C. &atients with a history of $estational "iabetes have a hi$h likelihoo" for eventually "evelo!in$ overt "iabetes. 7hese women shoul" therefore be e*tensively counsele" re$ar"in$ the im!ortance of "iet an" e*ercise. Alon$ with counselin$, testin$ is necessary to "etermine which !atients actually "o "evelo! overt "iabetes. 7estin$ shoul" be !erforme" in the first few months followin$ the "elivery. 7his testin$ may be a D/4$, ;4 hour, oral $lucose tolerance test. %iabetes is "ia$nose" if the fastin$ $lucose level e*cee"s :?> m$B",, or two !ost4$lucose measurements e*cee" ;>> m$B",. &atients shoul" then un"er$o fastin$ $lucose testin$ every 0 years. 7his !atient, $iven her history of $estational "iabetes, nee"s to have re$ular testin$. Chest *4ray every 0 years 'choice A) is not recommen"e" as a screenin$ test for this !atient. Althou$h the number of "eaths from lun$ cancer sur!asses that of breast cancer, an" lun$ cancer is the lea"in$ cause of cancer "eath in women, routine chest *4ray is not use" as a re$ular screenin$ test. Coronary an$io$ra!hy every 0 years 'choice ) woul" not be recommen"e" for this !atient. 7his is an invasive !roce"ure that currently is not use" as a re$ular screenin$ test in the $eneral !o!ulation. +ammo$ra!hy every 0 years 'choice %) woul" not be recommen"e" for this !atient. At 00 years of a$e, she "oes not yet require routine mammo$ra!hy. )he shoul" have a mammo$ram every :4; years startin$ at a$e ?>, an" then annually startin$ at a$e />. &a! testin$ every 0 years 'choice () woul" not be recommen"e" for this !atient. &a! testin$ shoul" be !erforme" annually startin$ at a$e :@, or with the initiation of se*ual intercourse. )ome recommen" that the interval can be increase" at the !hysicianAs "iscretion in a low4risk !atient with three normal &a! tests in a row. Others "is!ute this, ar$uin$ that annual &a! tests shoul" be !erforme" on all women. In any event, this !atient, with her history of Chlamy"ia an" $onorrhea, is not low risk an" therefore nee"s annual &a! testin$. ;?) (*!lanation: 7he correct answer is A. (n"ometrial cancer is the most common $ynecolo$ic cancer in women ol"er than ?/. 7here are tens of thousan"s of new cases every year in the F.)., an" thousan"s of "eaths from it yearly. 2owever, there is no effective screenin$ test for en"ometrial cancer at this !oint. It is not cost4effective to screen asym!tomatic women for en"ometrial cancer. Occasionally, a &a! test will "etect abnormal en"ometrial cells, but it is not a !ro!er

screenin$ tool for en"ometrial cancer. &atients with en"ometrial hy!er!lasia or cancer often !resent with irre$ular uterine blee"in$. 7herefore, !atients with irre$ular uterine blee"in$ shoul" be consi"ere" for en"ometrial bio!sy or ultrasonic evaluation of the en"ometrial cavity. 7his strate$y may be mo"ifie" for youn$ !atients, in whom the risk of en"ometrial hy!er!lasia or cancer is limite". 7o state that screenin$ is with en"ometrial bio!sy an" starts at a$e ?> 'choice ) or a$e /> 'choice C) is incorrect. (n"ometrial bio!sy can an" shoul" be use" in certain circumstances. -or e*am!le, a woman with !ostmeno!ausal blee"in$ shoul" un"er$o the !roce"ure. 2owever, en"ometrial bio!sy shoul" not be use" as a screenin$ tool. 7o !erform en"ometrial bio!sies on women with no in"ication other than screenin$ woul" !lace these women at risk for blee"in$, infection, an" uterine !erforation, an" woul" not be cost4effective. 7o state that screenin$ is with ultrasoun" an" starts at a$e ?> 'choice %) or a$e /> 'choice () is incorrect. &elvic ultrasoun" can be use" to hel! "ia$nose en"ometrial hy!er!lasia an" en"ometrial cancer. -or e*am!le, stu"ies have shown that in !ostmeno!ausal women, measurement of the en"ometrial stri!e can be useful in hel!in$ to rule out these con"itions. 2owever, !elvic ultrasoun" has not been shown to be cost4effective or warrante" for screenin$ for en"ometrial cancer. ;/) (*!lanation: 7he correct answer is (. Cancer screenin$ shoul" be an essential !art of an annual e*amination. Colorectal cancer is a ma<or cause of serious morbi"ity an" mortality for women in the F.).: $reater than />,>>> new cases are "ia$nose" each year, an" there are more than ;/,>>> "eaths from colorectal cancer. )creenin$ shoul" be$in at a$e /> in asym!tomatic women with no si$nificant family history. )creenin$ consists of a "i$ital rectal e*amination with fecal occult bloo" testin$. 7his can be !erforme" at the same time that !elvic e*amination is !erforme". )i$moi"osco!y shoul" be !erforme" every 04/ years. 7his !atient has no in"ication for a chest *4ray 'choice A). ,un$ cancer has sur!asse" breast cancer as the ma<or cause of cancer "eath in women. 2owever, chest *4ray films are not use" for lun$ cancer screenin$. 7here is currently no technique available for routine screenin$ for lun$ cancer. 7he &a! test 'choice ) is an im!ortant metho" of screenin$ for cervical cancer an" shoul" be !erforme" annually. 2owever, this !atient ha" a normal &a! test ; months a$o. &elvic ultrasoun" 'choice C) is not in"icate" in this !atient. )he has nothin$ abnormal in her history or !hysical that woul" warrant !elvic ultrasoun". &elvic ultrasoun" is not use" for cancer screenin$. &rostate4s!ecific anti$en '&)A) 'choice %) is an a!!ro!riate test in men. &)A testin$ woul" not be in"icate" in this 'female) !atient. ;C) (*!lanation: 7he correct answer is . 7his !atient has si$ns an" sym!toms that are most consistent with bacterial va$inosis. acterial va$inosis re!resents a chan$e in milieu of the va$ina such that there is a "ecrease in the number of lactobacilli an" an increase in anaerobic or$anisms. &atients

with bacterial va$inosis usually com!lain of a malo"orous va$inal "ischar$e. 7he e*amination is si$nificant for a $rayish "ischar$e that releases a stron$ amine 'or fishy) o"or when 5O2 is a!!lie" to it. 7his is known as a !ositive HwhiffH test. +icrosco!ic e*amination reveals clue cells, which are va$inal e!ithelial cells that are covere" with bacteria. )tu"ies have "emonstrate" an association between bacterial va$inosis an" !reterm "elivery. 7here is some evi"ence that treatment of bacterial va$inosis with oral metroni"a#ole in women at hi$h risk for !reterm "elivery Wi.e., women with a !rior !reterm "elivery or a low !re!re$nancy wei$ht 'P/>k$)X will hel! to !revent !reterm "elivery. Oral metroni"a#ole a!!ears to be better than the to!ical, va$inal formulation for the !revention of !reterm "elivery. 7o state that no treatment is nee"e" 'choice A) is incorrect. -irst, this !atient is sym!tomaticG therefore, treatment is reasonable. )econ", treatment of bacterial va$inosis in the secon" trimester in women at hi$h risk for !reterm "elivery may !revent a !reterm "elivery. Intramuscular !enicillin 'choice C) is use" "urin$ !re$nancy for women with sy!hilis. It is not use" to treat bacterial va$inosis. I1 !enicillin 'choice %) is use" "urin$ labor to !revent neonatal $rou! )tre!tococcus infection. It is not use" for bacterial va$inosis. Oral !enicillin 'choice (), like intramuscular an" I1 !enicillin, is use" for $ram4!ositive infections. &atients with bacterial va$inosis require treatment with a "ru$ that will cover anaerobic infections 'e.$., metroni"a#ole.) ;D) (*!lanation: 7he correct answer is . Women that acquire rubella "urin$ !re$nancy are at risk of "evelo!in$ fetal infection an" con$enital rubella syn"rome. If the mother is infecte" "urin$ the first trimester the malformation rate in the fetus is a!!ro*imately />=. 7hese malformations inclu"e microce!haly, mental retar"ation, cataracts, "eafness, an" con$enital heart "isease4 althou$h all or$ans may be affecte". 7herefore, all women nee" to be vaccinate" with the rubella vaccine4most often $iven as !art of the ++6 series of vaccines. 2owever, because the ++6 vaccine is a live4virus vaccine, there is a concern that a"ministration of the vaccine within 0 months of conce!tion, or "urin$ the !re$nancy, coul" result in birth "efects or illness. 9et, this concern is more theoretical than real as stu"ies !erforme" on women $iven the rubella vaccine shortly before becomin$ !re$nant or "urin$ !re$nancy have faile" to show any increase in the risk of malformations com!are" to the $eneral back$roun" risk in the !o!ulation. 7he current F) immuni#ation !olicy is that the risk of vaccine4associate" "efects is virtually ne$li$ible an" shoul" not be a reason in itself to consi"er termination. 7o state that there is no vaccine risk an" termination is com!letely ina!!ro!riate 'choice A) is incorrect. Althou$h the risk of the ++6 vaccine a!!ears more theoretical than real, one cannot state that there is no risk. 7o state that the vaccine risk is mo"erate an" termination shoul" be consi"ere" 'choice C) or that the vaccine risk is hi$h an" termination shoul" be stron$ly consi"ere" 'choice %) is incorrect. As state" above, the risk of the ++6 vaccine4associate" "efects is so small as to be consi"ere" ne$li$ible. 7o state that the vaccine risk is hi$h an" termination is man"ate" 'choice () is absolutely incorrect. -irst, the vaccine risk is not hi$h. )econ",

even if the fetus were foun" to have severe "efects, it is the !atientAs choice whether or not to kee! the !re$nancy. ;@) (*!lanation: 7he correct answer is %. 2y!othyroi"ism is associate" with several com!lications re$ar"in$ fertility an" !re$nancy. Women with overt hy!othyroi"ism have increase" rates of infertility. Women with uncorrecte" hy!othyroi"ism that "o become !re$nant are at increase" risk of havin$ stillborn an" low4birth4wei$ht infants. 1arious stu"ies have also shown that rates of !reeclam!sia, !lacental abru!tion, an" heart failure may be increase" in !re$nant !atients with hy!othyroi"ism. &re$nancy often lea"s to an increase" requirement for thyroi" hormone re!lacement 'thyro*ine) as the !re$nancy !ro$resses. &re$nant women with hy!othyroi"ism on thyro*ine shoul" have their thyroi" stimulatin$ hormone '7)2) level checke" !erio"ically to "etermine if the "ru$ "osa$e is a"equate. 7his !atient, with her history of hy!othyroi"ism, shoul" continue her thyro*ine "urin$ the !re$nancy. Couma"in 'choice A) is contrain"icate" "urin$ !re$nancy, as it is a known cause of birth "efects. 7his !atient nee"s anticoa$ulation, however, an" shoul" be !lace" on he!arin, which "oes not cross the !lacenta. It is !ossible to become !re$nant while takin$ the oral contrace!tive !ill 'choice ), as the !ill has a small rate of failure. 7here is no known association between first trimester e*!osure an" birth "efects. Now that the !atient has become !re$nant, however, she shoul" sto! takin$ the OC&. 7etracycline 'choice C) is use" to treat some forms of acne an", therefore, some women will become !re$nant while on the me"ication. Its use is contrain"icate" "urin$ !re$nancy, however, because it is associate" with fetal teeth an" bone malformations. 7o state that the !atient shoul" "iscontinue all me"ications 'choice () is absolutely incorrect. While some me"ications are contrain"icate" "urin$ !re$nancy, many are necessary an" shoul" be continue". ;I) (*!lanation: 7he correct answer is %. Ovarian torsion is a sur$ical emer$ency. Ovarian torsion occurs when the ovary com!letely twists an" thus, occlu"es its bloo" su!!ly. &atients often !resent with intermittent !ain as the ovary twists an" untwists an" then constant, severe !ain when the torsion becomes com!lete an" the ovary becomes ischemic. 7ime is of the essence an" can mean the "ifference between savin$, versus losin$, an ovary. 7his is im!ortant for any !atient, but is !articularly im!ortant for a youn$ female of chil"bearin$ a$e, es!ecially one who is nulli$ravi". 7he reason that time is so essential is that the lon$er the ovary stays torse", the more likely it is to become necrotic. +ost sur$eons woul" !erform la!arosco!y on this !atient if they felt it was safe to "o so. 7he !elvis can be fully evaluate" throu$h the la!arosco!e an" a torsion can often be untwiste" usin$ la!arosco!ic instruments. 2owever, with lar$e cysts, some sur$eons !refer to !erform a la!arotomy. (*!ectant mana$ement 'choice A) woul" not be a!!ro!riate for this !atient. When ovarian torsion is consi"ere" to be likely in a !atient, that !atient must have

sur$ery. 7o e*!ectantly mana$e these !atients is to risk further "ama$e to, an" !ossible loss of, the ovary. A follow4u! ultrasoun" in C weeks 'choice ) is a!!ro!riate mana$ement for some ovarian cysts. -or e*am!le, if this !atient were asym!tomatic an" the cyst "i" not have features sus!icious for mali$nancy, one coul" follow4u! with an ultrasoun" in C weeks, as lon$ as the !atient was $iven strict instructions an" !recautions re$ar"in$ the risk of torsion. 2owever, this !atient has severe !ain an" may be infarctin$ her ovary an" therefore nee"s sur$ery. Intravenous antibiotics 'choice C) woul" be a!!ro!riate if the !atient ha" !elvic inflammatory "isease or another infectious !rocess, however, the likely "ia$nosis is torsion, an" sur$ery, rather than intravenous antibiotics, is nee"e". In the !ast, oo!horectomy 'choice () was recommen"e" for any !atient with ovarian torsion. 7he concern was that the torsion woul" lea" to thrombus formation in the ovarian vessels an" that "etorsin$ the ovary coul" lea" to thromboembolism to the !ulmonary vasculature. Current thinkin$ is that the ovary may be "etorse" an" then evaluate". If the ovary a!!ears to be viable, it may be left in4situ. If the ovary a!!ears to be com!letely non4viable an" necrotic, it will be remove". 0>) (*!lanation: 7he correct answer is %. 7his !atient has a "ee! venous thrombosis '%17) in her ri$ht lower e*tremity. &re$nancy is a risk factor for the "evelo!ment of %17s because of alterations in coa$ulation factors, venous stasis, an", often, "ecrease" !hysical activity. It is essential that %17 "urin$ !re$nancy be treate" so that the thrombus "oes not !roliferate or emboli#e an" so that new thrombi "o not form. Couma"in is contrain"icate" "urin$ the first trimester because of the risk of birth "efects in fetuses e*!ose" to this "ru$. Couma"in embryo!athy is a syn"rome consistin$ of nasal hy!o!lasia an" sti!!le" vertebral an" femoral e!i!hyses. )econ"4 an" thir"4trimester e*!osure to Couma"in can lea" to hy"roce!haly, microce!haly, o!hthalmolo$ic abnormalities, fetal $rowth retar"ation, an" "evelo!mental "elay. ,ow4molecular4wei$ht he!arin has been shown to be an e*cellent anticoa$ulant because it has a lon$er half4life an" a more !re"ictable "ose4res!onse relationshi! com!are" with unfractionate" he!arin, which allows once4 or twice4"aily "osin$ without the nee" for frequent laboratory monitorin$ of the !rothrombin time an" activate" !artial thrombo!lastin time. ,ow4molecular4wei$ht he!arin is also less likely to cause thrombocyto!enia an" hemorrha$ic com!lications than unfractionate" he!arin. ,ow4molecular4wei$ht he!arin "oes not have a shorter half4life 'choice A) than unfractionate" he!arin. In fact, low4molecular4wei$ht he!arin has a lon$er half4life, an" it is this quality that allows for once4 or twice4"aily "osin$. ,ow4 molecular4wei$ht he!arin is not chea!er 'choice ) than unfractionate" he!arin. ,ow4 molecular4wei$ht he!arin itself is more e*!ensive, but there is a cost a"vanta$e in that less frequent laboratory monitorin$ is nee"e". Neither low4molecular4wei$ht he!arin nor unfractionate" he!arin is likely to cause birth "efects 'choice C). Neither crosses the !lacenta 'choice () an" neither is associate" with terato$enesis. 0:)

(*!lanation: 7he correct answer is %. 7his !atient has ha" an uncom!licate" course of !yelone!hritis thus far. &yelone!hritis is an infection of the ki"ney. &atients with !yelone!hritis ty!ically !resent with some combination of back !ain, "ysuria, hematuria, frequency, ur$ency, fevers, chills, nausea, an" vomitin$. (*amination often shows an elevate" tem!erature, costovertebral an$le ten"erness, leukocytosis, an" white cells an" re" cells in the urine. Com!letely uncom!licate" cases of !yelone!hritis can be treate" on an out!atient basis. When there are any com!licatin$ factors 'e.$., concern for se!sis, !re$nancy, ol" a$e, or other me"ical illnesses), the !atient shoul" be a"mitte" to the hos!ital for intravenous antibiotics. 2owever, once the !atientAs con"ition has im!rove" an" she is toleratin$ oral intake, she may be "ischar$e" home to com!lete a ;4week course of antibiotics. When "ischar$e", however, she shoul" be $iven strict instructions an" !recautions re$ar"in$ the nee" to return for recurrence of the sym!toms or worsenin$ con"ition. 7o continue intravenous antibiotics for ; weeks 'choice A) woul" not be necessary. Once a !atient with !yelone!hritis is afebrile, "oin$ better, an" able to tolerate oral intake, she may be converte" to oral antibiotics an" be "ischar$e" to home. 7o kee! the !atient hos!itali#e" for a full ; weeks woul" not be necessary. 7o "ischar$e home an" recommen" !ost4coital !ro!hyla*is 'choice ) or to "ischar$e home off all antibiotics 'choice C) woul" not be correct. (ven thou$h the !atient is feelin$ better, she must still com!lete a ;4week course of oral antibiotics an" not <ust use antibiotics for !ost4coital !ro!hyla*is. 7o obtain sur$ical evaluation 'choice () woul" not be necessary. If a !atient with !yelone!hritis is not im!rovin$, then sur$ical evaluation may be require" to "etermine if another etiolo$y is res!onsible or to "etermine if sur$ical intervention is require". 7his !atient, however, is im!rovin$ an" sur$ical evaluation woul" not be necessary. 0;) (*!lanation: 7he correct answer is . &ost!artum tubal li$ation is a hi$hly effective metho" for $ivin$ a woman !ermanent sterili#ation. +any metho"s have been "evelo!e", but the most common metho"s involve "oubly li$atin$ a !ortion of each tube an" e*cisin$ an intervenin$ se$ment. A !ost!artum tubal li$ation can be !erforme" at the time of cesarean "elivery or after a va$inal "elivery. If the !roce"ure is !erforme" after a va$inal "elivery, a relatively small skin incision is usually ma"e in or near the umbilicus. &atients un"er$oin$ !ost!artum tubal li$ation shoul" be warne", however, that the !roce"ure coul" fail. -ailure may result from many factors inclu"in$ recannali#ation of the tube an" !oor sur$ical technique. 7he most commonly quote" failure rate is about : in :>>, althou$h a more accurate fi$ure may be closer to : in 0>>. 2owever, it is im!ossible to $ive one e*act rate, because the risk of failure "e!en"s on the !atientAs a$e. A ;/4year4ol" woman un"er$oin$ tubal li$ation is more likely to e*!erience failure than a ?>4year4ol" woman, because the ;/4 year4ol" has so many more years of fertility ahea" of her. If the failure rate were : in :> 'choice A) few "octors woul" recommen" the !roce"ure. -or a birth control metho" to be useful, it must have a low overall failure rate. A failure rate of :>= woul" be too $reat to <ustify the risk of the !roce"ure. : in :>>> 'choice C) or : in :,>>>,>>> 'choice %) are the

failure rates that obstetricians woul" like to see from tubal li$ation. &erha!s with time an" chan$es in metho"olo$y, the failure rates will continue to fall. At !resent, however, the most commonly quote" failure rate is : in :>>. 7o state that there are no re!orte" failures of !ost!artum tubal li$ation 'choice () is absolutely incorrect. &atients nee" to be cautione" that the !roce"ure can fail an" that if !re$nancy is sus!ecte", they shoul" notify their "octor imme"iately, as the risk of ecto!ic !re$nancy after tubal li$ation is si$nificant. 00) (*!lanation: 7he correct answer is . 7his !atient has an obstetrical history that is consistent with abnormal cervical com!etence. 7his "ia$nosis may be ma"e when the !atient has a history of !ainless cervical "ilation in the secon" trimester. Cervical incom!etence is a cause of secon"4 trimester !re$nancy loss an" !reterm "elivery. Cervical incom!etence may be con$enital an"Bor acquire". Women who have ha" !revious trauma to the cervi* 'e.$. "ilation of the cervi*, cervical coni#ation, or obstetric trauma) an" women with mullerian anomalies, or a history of in4utero e*!osure to "iethylstilbestrol may be at increase" risk. 7his !atient, $iven her history, was offere" a cercla$e. Cercla$e is a !roce"ure in which a suture is !lace" at the level of the internal os after bla""er "issection ')hiro"kar) or as hi$h u! on the cervi* as !ossible '+c%onal"). A !ro!hylactic cercla$e is !lace" between :; an" :C weeksA $estation. Once the cercla$e is !lace", the !atient shoul" not en$a$e in se*ual intercourse, !rolon$e" stan"in$, or heavy liftin$. 7his !atient, however, refuse" to have a cercla$e !lace". .iven her history, however, she nee"s to be followe" closely to ensure that any si$ns of cervical incom!etence are "etecte" as soon as !ossible. 6e$ular e*aminations of the cervi*, either "i$itally or with ultrasoun", shoul" be$in at :C weeks because cervical incom!etence becomes a concern "urin$ the secon" trimester. )tartin$ re$ular e*aminations at :> weeks 'choice A) is unlikely to be hel!ful. Cervical incom!etence most often manifests itself in the secon" or thir" trimester. )tartin$ re$ular e*aminations at ;; weeks 'choice C) or ;@ weeks 'choice %) woul" not be correct, as these $estational a$es may be too late to "etect cervical chan$es. 7his !atient lost her last !re$nancy at ;; weeks, which means that her cervi* may have starte" chan$in$ several weeks earlier. 7o wait until ;; or ;@ weeks woul" risk missin$ cervical chan$es an" the !ossibility of institutin$ chan$es 'e.$., be" rest, hos!itali#ation, or cercla$e !lacement) to hel! !revent !re$nancy loss. 0D weeks 'choice () is the time at which a cercla$e shoul" be remove". In a woman with a history of a ;;4week loss, waitin$ until 0D weeks to start checkin$ the cervi* re$ularly woul" not be a!!ro!riate. 0?) (*!lanation: 7he correct answer is %. 7his !atient has a !resentation an" fin"in$s that are most consistent with an"ro$en insensitivity syn"rome 'also calle" testicular femini#ation syn"rome). 7hese !atients are $enoty!ically male '?C, E9) but !henoty!ically female because they have a "efect that

!revents normal an"ro$en rece!tor function. 7he an"ro$en rece!tor $ene is locate" on the E chromosome an" various "efects in the $ene 'e.$., absence of the $ene or abnormalities in the an"ro$en bin"in$ "omain of the rece!tor) can lea" to this syn"rome. &atients with an"ro$en insensitivity are amenorrheic an" have no internal female structures. 7estes rather than ovaries are !resent. 7hese !atients also have minimal a*illary an" !ubic hair. 7hey "o e*!erience abun"ant breast "evelo!ment at !uberty, as testosterone is unable to su!!ress the formation of breast tissues. 7hese !atients also ten" to be very tall with bi$ han"s an" feet an" lon$ arms. 7estes shoul" be remove" after !ubertal "evelo!ment is com!lete", as many of these !atients will "evelo! $ona"al mali$nancies after !uberty. Asherman syn"rome 'choice A) is amenorrhea cause" by intrauterine a"hesions. 7hese a"hesions ty!ically "evelo! after curetta$e an" infection of the uterus. 5allmann syn"rome 'choice ) is amenorrhea cause" by hy!o$ona"otro!ic hy!o$ona"ism. It is associate" with anosmia, color blin"ness, an" facial "eformities. &atients have normal female structures. &atients with !olycystic ovarian syn"rome 'choice C) usually have the characteristics of oli$omenorrhea, hirsutism, infertility, an" obesity. 7his !atient has none of these characteristics. &atients with 7urner syn"rome 'choice () have a ?/, E $enoty!e. 7hey are !henoty!ically females, often with small stature, short necks, an" wi"e chests. 7his !atient has a eunuchoi" !henoty!e.

0/) (*!lanation: 7he correct answer is %. 6alo*ifene is a me"ication that belon$s to the class of "ru$s calle" selective estro$en rece!tor mo"ulators ')(6+s). 7hese "ru$s, of which the most wi"ely known are ralo*ifene an" tamo*ifen, have !ro4estro$enic effects in some tissues an" anti4estro$enic effects in other tissues. 6alo*ifene has been a!!rove" by the F.). -oo" an" %ru$ A"ministration for the !revention of osteo!orosis. 7his !atient, with her stron$ family history of osteo!orosis, is a $oo" can"i"ate for !revention. 2owever, althou$h ralo*ifene acts as an estro$en a$onist in the bone, it a!!ears to have no effect on hot flashes or to actually cause hot flashes. 7herefore, this !erimeno!ausal !atient is most likely to "evelo! hot flashes while on ralo*ifene. Althou$h "efinitive !roof is not available, it a!!ears that ralo*ifene acts as an estro$en anta$onist in the breast. 7herefore, this !atient woul" not be most likely to "evelo! breast cancer 'choice A) while on ralo*ifene. )he woul" be more likely to "evelo! hot flashes. 6alo*ifene a!!ears to lower cholesterol, es!ecially ,%, cholesterol, in !atients. 7herefore, elevate" cholesterol 'choice ) woul" be less likely while on this me"ication. 6alo*ifene a!!ears to act as an estro$en anta$onist at the level of the en"ometriumG therefore, en"ometrial hy!er!lasia 'choice C) woul" be less likely than hot flashes. 6alo*ifene is use" in the !revention of osteo!orosis 'choice (). 0C) (*!lanation: 7he correct answer is

C. 7he issue of whether a !atient who ha" en"ometrial carcinoma can be !lace" on estro$en re!lacement thera!y '(67) is somewhat controversial. If the !atient is com!letely free of tumor, estro$en re!lacement thera!y shoul" not result in recurrence. An", in this !atientAs case, it woul" be hel!ful for her hot flashes an" osteo!orosis. 2owever, if an estro$en4"e!en"ent neo!lasm is still !resent somewhere in her bo"y, (67 may result in an earlier recurrence. )ta$e I, $ra"e I, en"ometrial cancer is the lowest $ra"e an" lowest sta$e en"ometrial cancer. 7he risk of !ersistent "isease is less than /=. +any $ynecolo$ic oncolo$ists woul" feel comfortable $ivin$ (67 to this !atient. 2owever, the !atient must be fully informe" re$ar"in$ the benefits an" risks of (67. 7hese risks inclu"e not only earlier recurrence, but also the stan"ar" risks such as venous thrombosis. 7o state that estro$en re!lacement thera!y is absolutely contrain"icate" 'choice A) is incorrect. As e*!laine" above, (67 may be $iven to certain !atients with a history of $ra"e I, sta$e I, en"ometrial carcinoma. 7o state that estro$en re!lacement thera!y may be use" an" there are no risks 'choice ) is not a!!ro!riate. (ven in women with no history of en"ometrial carcinoma, there are risks to (67. 7he history of en"ometrial carcinoma a""s a further risk for this !atient. 7o state that estro$en re!lacement thera!y will lea" to breast cancer 'choice %) is incorrect. 7here are a number of stu"ies that show that (67 lea"s to increase" rates of breast cancer. 7here are also a number of stu"ies that show no increase" risk. 7herefore, to make the "efinitive statement that (67 will lea" to breast cancer is incorrect. As e*!laine" above, to state that (67 will lea" to cancer recurrence 'choice () is not correct. 0D) (*!lanation: 7he correct answer is %. 7his !atient has a history that is classic for cervical incom!etence. Cervical incom!etence is characteri#e" by !ainless cervical "ilation, ty!ically in the secon" or early thir" trimester. &atients will often have membranes bul$in$ into the va$ina. In reality, many !atients will !resent with cervical "ilation, but they will also have some cram!in$ or contractions. 7his can make "istin$uishin$ !reterm labor from cervical incom!etence "ifficult. Also, cervical incom!etence often lea"s to bul$in$ membranes that then ru!ture. 7his ru!ture of the membranes can also cause contractions an" labor such that when the !atient !resents, the "ia$nosis of cervical incom!etence versus !reterm labor is clou"e". 2owever, when the "ia$nosis of cervical incom!etence is clear, as it is in this !atient, many !ractitioners favor !lacin$ a cercla$e 'a stitch aroun" the cervi* inten"e" to su!!ort the !re$nancy). 7his cercla$e shoul" be left in !lace throu$hout the !re$nancy. Only when the !atient is at term '0C40@ weeks) shoul" the cercla$e be remove". 7o remove the cercla$e at 0>40; weeks 'choice A), 0;40? weeks 'choice ), or 0?40C weeks 'choice C) !laces the !atient at risk of iatro$enic !rematurity. )ay, for e*am!le, that the cercla$e is remove" at 00 weeks, an" the !atient $oes into labor imme"iately thereafter an" "elivers. 7his woul" result in a 004week newborn, with risks of intraventricular hemorrha$e, res!iratory "istress syn"rome, an" necroti#in$ enterocolitis. 7his outcome woul" have been avoi"e" by leavin$ the stitch in until 0C40@ weeks. 7o remove the cercla$e at 0@4?> weeks 'choice () runs the risk that the !atient may $o into labor !rior to removal of the stitch. 7he concern here is that with labor, the

stitch will cause a cervical laceration. 7herefore, the stitch shoul" be remove" !rior to the likely onset of labor, but not so early so as to result in a !remature newborn if the !atient $oes into labor with removal of the stitch. 7hus, 0C40@ weeks is an i"eal time for removal of a cercla$e. 0@) (*!lanation: 7he correct answer is A. 7his !atient has a !resentation that is most consistent with !erimeno!ausal hot flashes 'or hot flushes as they are sometimes calle"). 7he e*act !atho!hysiolo$y that un"erlies the hot flash is not known. 2owever, it is known that women at the meno!ause an" men that un"er$o orchiectomies e*!erience these sym!toms. 7herefore, it is assume" that it is the removal of normal levels of se* steroi"s from the circulation that results in the hot flash. 7hese hot feelin$s are e*!erience" as a flushin$ that can last from several secon"s to many minutes. 7he first4line treatment for most women is with hormone re!lacement thera!y. 2owever, estro$en is contrain"icate" in this !atient $iven her history of !ulmonary embolus. 7he fact that she is a current smoker also !laces her at $reater risk of "evelo!in$ a thrombus if she were to take hormones. &ro$estins alone have also been shown to relieve hot flashesG however, they may worsen "e!ression an" cause other moo" chan$es in !atients. 7herefore an alternative treatment is nee"e" for her. Cloni"ine has been use" with some success by many women for relief from hot flashes. It is a bloo" !ressure me"ication, but it has been shown to be effective a$ainst hot flashes when use" in low "oses. (stro$en an" !ro$esterone 'choice ) shoul" not be use" in this !atient because of her history of a !ulmonary embolus. Combine" hormone re!lacement thera!y has been shown to increase the risk of clot formation in !atients. With her history an" current smokin$, this !atient woul" be at a !articularly increase" risk. (stro$en only 'choice C) woul" be contrain"icate" in this !atient for two reasons. -irst, her uterus is still in !lace, an" uno!!ose" estro$en woul" !lace her at $reater risk for en"ometrial hy!er!lasia an" cancer. )econ", estro$en woul" increase this !atientAs risk of thrombus formation. .luco!ha$e 'choice %) is an oral hy!o$lycemic me"ication use" in !atients with "iabetes. It is not known to be effective for the treatment of hot flashes. 7amo*ifen 'choice () actually causes hot flashes in many !atients an" is not use" to treat them. 0I) (*!lanation: 7he correct answer is A. &re$nancy brin$s about numerous, normal !hysiolo$ic chan$es in the !re$nant woman. )ome of the most obvious chan$es are those foun" in the car"iovascular system. -or e*am!le, car"iac out!ut rises marke"ly in !re$nancy with increases u! to />= over non!re$nant levels. Car"iac murmurs are common in !re$nancy with as many as I>= of all !re$nant women havin$ some "e$ree of a systolic murmur. %iastolic murmurs are "ifferent, however. 7he fin"in$ of a "iastolic murmur in a !re$nant woman must be thorou$hly evaluate" as this ty!e of murmur is often relate" to im!ortant car"iac "isease. -or e*am!le, mitral stenosis, the most common rheumatic valvular lesion in !re$nancy,

is characteri#e" by a rumblin$ "iastolic murmur. 7herefore, !atients with "iastolic murmurs shoul" have an echocar"io$ra!h an" !ossible referral to a car"iolo$ist for further evaluation. An enlar$e" uterus 'choice ) is a normal fin"in$ in a !re$nant woman. It is im!ortant to e*amine the uterus for si#e at the first !renatal visit to ensure that the si#e correlates to the !atientAs "atin$ by last menstrual !erio". If there is a "iscre!ancy, then the !atient shoul" be sent for an ultrasoun" to obtain correct "atin$, which is essential for the mana$ement of the !re$nancy. -ati$ue 'choice C) an" nausea an" vomitin$ 'choice %) are very common fin"in$s in the first trimester of !re$nancy. While fati$ue can sometimes be a sym!tom of structural heart "isease, it is not nearly as concernin$ as the "iastolic murmur in this !atient. Nausea an" vomitin$ is !resent in anywhere from /> to I>= of all !re$nant women. As note" above, a systolic murmur 'choice () is a very common fin"in$ "urin$ !re$nancy. F! to I>= of all !re$nant women will have such a murmur "urin$ !re$nancy. As lon$ as the murmur is systolic, no lou"er than IIIB1I an" there is no other sym!tomatolo$y, the murmur can be consi"ere" to be beni$n. ?>) (*!lanation: 7he correct answer is A.Neisseria $onorrhoeae is a known cause of cervicitis an" can also !lay a role in the !atho!hysiolo$y of !elvic inflammatory "isease '&I%). In !re$nant women, it is im!licate" as a cause of !reterm birth an" chorioamnionitis. In !ast "eca"es, transmission of the $onococcus at birth from the mother to her infant le" to eye infection '$onococcal o!hthalmia neonatorum) an" blin"ness in many neonates. Fniversal neonatal eye !ro!hyla*is with an antibiotic ointment has re"uce" the rates of $onococcal o!hthalmia neonatorum consi"erably. Any woman who is foun" to be infecte" with the $onococcus "urin$ !re$nancy shoul" be treate". 7he treatment of choice is ceftria*one, which is $iven as a one4time intramuscular in<ection. Cefi*ime can also be use" as an oral, one4 time "ose me"ication that is better tolerate" by some. &atients who cannot tolerate ce!halos!orins can be treate" with s!ectinomycin as a sin$le intramuscular "ose. Any !atient with $onorrhea shoul" also be $iven antibiotics that will cover Chlamy"ia as well. 7hus, this !atient shoul" be $iven not only ceftria*one, but a#ithromycin 'or erythromycin or amo*icillin) as well. It is also essential that the !atientAs se*ual !artner or !artners be treate" an" that a test of cure be !erforme" a!!ro*imately ? weeks later to ensure that the or$anism has been era"icate". Clin"amycin 'choice ) "oes not !rovi"e a"equate covera$e for $onorrhea an" therefore woul" not be the most a!!ro!riate !harmacothera!y. %o*ycycline 'choice C) is often use" to treat Chlamy"ia in non!re$nant !atients. It shoul" not be use" "urin$ !re$nancy because of the effects on fetal teeth an" bones. ,evoflo*acin 'choice %) is contrain"icate" "urin$ !re$nancy because of a !ossible link with arthro!athies in the offs!rin$ of women e*!ose" to the "ru$. +etroni"a#ole 'choice () is use" "urin$ !re$nancy to treat bacterial va$inosis an" 7richomonas. Current recommen"ations are that it shoul" not be use" "urin$ the first trimester. It is not use" to treat $onorrhea.

?:) (*!lanation: 7he correct answer is C. +eno!ause can brin$ about a number of bothersome sym!toms for !atients. 7hese inclu"e an*iety, fati$ue, "e!ression, hea"aches, insomnia, an" "ys!areunia. &erha!s the most common sym!tom is the hot flash, which is an uncomfortable sensation of heat, es!ecially in the face an" chest. 7hese flashes can occur once in a while or several times each "ay. 7here are many thera!ies available for hot flashes, but the most effective a!!ears to be estro$en. Numerous stu"ies have shown estro$en re!lacement to be hi$hly effective in re"ucin$ central nervous system sym!toms such as hot flushes, insomnia, irritability, an*iety, an" hea"aches. 7his !atient may also benefit from estro$en $iven her stron$ family history of osteo!orosis, as estro$en re!lacement has been shown to be beneficial in re"ucin$ bone loss in !ostmeno!ausal women. 7he fact that this !atient ha" en"ometrial hy!er!lasia "oes not !revent her from takin$ estro$en re!lacement thera!y. )he ha" hy!er!lasia, not en"ometrial cancer. An", even in some cases of en"ometrial cancer, some $ynecolo$ic oncolo$ists woul" ar$ue that once thera!y has been $iven 'i.e., hysterectomy an" bilateral oo!horectomy) an" there is no evi"ence of resi"ual "isease, then estro$en re!lacement may be $iven. 7his !atient, with only low $ra"e en"ometrial hy!er!lasia on !atholo$ic evaluation, has no contrain"ication to estro$en an" woul" likely benefit si$nificantly from estro$en re!lacement thera!y. Al!ra#olam 'choice A) is a ben#o"ia#e!ine an" woul" not be the first line treatment for this !atientAs hot flashes. Cloni"ine 'choice ) is an antihy!ertensive that has been shown, in some stu"ies, to be effective in the treatment of hot flashes. 2owever, the most effective treatment is estro$en. 7he oral contrace!tive !ill 'choice %) woul" not be in"icate" for this !atient as the levels of hormones are in e*cess of those nee"e" "urin$ the !ostmeno!ausal !erio". 6alo*ifene 'choice () is a selective estro$en rece!tor mo"ulator. It has been a!!rove" for the !revention of !ostmeno!ausal osteo!orosis. 2owever, some !atients e*!erience an increase in hot flashes while takin$ ralo*ifene. 7hus, estro$en woul" be !referable to ralo*ifene for this !atient whose !rimary com!laint is hot flashes. ?;) (*!lanation: 7he correct answer is (. 1itamin A is an im!ortant vitamin for human re!ro"uction an" normal bo"ily functionin$, an" vitamin A "eficiency is a !roblem throu$hout much of the worl". In the F.). an" other "evelo!e" nations, however, the overwhelmin$ ma<ority of women have sufficient stores of vitamin A in the liver. 7hus, vitamin A su!!lementation "urin$ !re$nancy is not nee"e" or recommen"e" for most women. In fact, vitamin A su!!lementation has been associate" with birth "efects, inclu"in$ cranial neural crest malformations. +ost commonly use" !renatal vitamins contain />>> IF or less, an" this is consi"ere" acce!table. Women shoul" be instructe" not to take any further su!!lementation than this. 7he only e*ce!tion to this rule is for women who may be vitamin A "eficient because of strict ve$etarianism or because they are recent emi$rants from countries in which vitamin A "eficiency is en"emic. 6ecent stu"ies have su$$este" that vitamin A su!!lementation with as little as :>,>>> IF 'choice A) !er "ay may cause

birth "efects. 7herefore, this amount of su!!lementation shoul" be avoi"e". 7he !robable terato$enic "ose of vitamin A, notwithstan"in$ the abovementione" stu"y, is ;/,>>> IF 'choice ) to />,>>> IF 'choice C). &atients shoul" therefore be instructe" to avoi" these levels. :>>,>>> IF 'choice %) woul" certainly not be recommen"e". ?0) (*!lanation: 7he correct answer is C. 7his !atient has a !resentation that is most consistent with ovarian hy!erstimulation syn"rome 'O2))). O2)) most often occurs in !atients un"er$oin$ ovulation in"uction with $ona"otro!ins, althou$h it can also occur with use of clomi!hene citrate. 7he si$ns an" sym!toms of O2)) run a s!ectrum "e!en"in$ on whether the "isease is mil", mo"erate, or severe. In mil" O2)), the ovaries are less than / cm, an" the !atient has mil" wei$ht $ain an" !elvic "iscomfort. In mo"erate O2)), the ovaries can be u! to :> cm in "iameter, an" the !atient has at least a :>4!oun" wei$ht $ain, nausea, an" vomitin$. In severe O2)), the ovaries are $reater than :> cm, with ascites, hy"rothora*, hemoconcentration, an" oli$uria. +ana$ement "e!en"s on the severity of the syn"rome, with mil" cases bein$ mana$e" conservatively an" more severe cases bein$ mana$e" more a$$ressively with the !ossible nee" for !aracentesis, thoracentesis, or sur$ery. &elvic or ab"ominal e*aminations shoul" not be !erforme" if O2)) is on the "ifferential "ia$nosis because e*amination can lea" to ru!ture of the ovarian ca!sule. (valuation shoul" be "one with a careful ultrasoun" e*amination. (cto!ic !re$nancy 'choice A) can cause ab"ominal "istension. 2owever, this !atient has a ne$ative hC., which effectively rules out !re$nancy. A hemorrha$ic ovarian cyst 'choice ) can also cause ab"ominal "istension. 2owever, with a hemorrha$ic cyst, !ain is most often the !resentin$ com!laint. 7he "istension is usually cause" by intra!eritoneal blee"in$, which causes si$nificant !ain an" ten"erness on e*amination. 7his !atient has no ten"erness on e*amination. Ovarian torsion 'choice %) causes si$nificant !ain. Ab"ominal !ain an" ten"erness are not the !re"ominant features of this !atientAs !resentation. 7ubo4ovarian abscess 'choice () causes si$nificant !ain an" often a fever. 7his !atient is afebrile, with no ab"ominal ten"erness. ??) (*!lanation: 7he correct answer is C. Con"oms !rovi"e an e*cellent metho" both of birth control an" of !revention of se*ually transmitte" "iseases ')7%s). When they are !ro!erly an" consistently use", they are a!!ro*imately I@= effective. 2owever, actual use or ty!ical use of the con"om varies, an", in !ractice, they are !robably aroun" @>= effective. Im!ro!er an" inconsistent use accounts for almost all of the "iscre!ancy between the effectiveness with H!erfectH versus Hty!icalH use. &ro!er con"om use requires that a new con"om is use" for each act of intercourse, that some room is left at the ti!, that the !enis is with"rawn while still erect, that the con"om is hel" on firmly to kee! it from sli!!in$ off as the !enis is with"rawn, an" that the con"om is use" with water4base", not oil4base", lubricants.

Aller$ic reaction 'choice A) can be a !roblem with late* con"oms in men or women with late* aller$ies. 2owever, the reaction itself shoul" not cause con"om failure. reaka$e 'choice ) is rare with con"oms. +ost re!orts !ut con"om breaka$e rates at less than ;= an" most of these are "ue to incorrect use 'e.$., not leavin$ room at the ti!). +anufacturin$ "efects 'choice %) are also rare. Con"oms are re$ulate" by the F.). -oo" an" %ru$ A"ministration, which tests numerous batches. )am!les that fail testin$ lea" the entire batch to be "iscar"e". 1a$inal infection 'choice () is an unlikely cause of con"om failure. ?/) (*!lanation: 7he correct answer is %. 7he lifetime inci"ence of ovarian cancer is :.?= ': in D> women). Fnfortunately, there are no early sym!toms of ovarian cancer: !resentin$ sym!toms have to "o with increasin$ tumor mass. 7his !atient has ab"ominal "iscomfort an" early satiety, which are often associate" with ovarian cancer. Other sym!toms that may be seen are fati$ue, urinary frequency, an" shortness of breath. 7he most common fin"in$ on e*amination is a !elvic mass, as this !atient has. +asses, ascites, an" evi"ence of tumor s!rea" may be seen on C7 scan. 6ou$hly @>= of all ovarian cancers are "erive" from ovarian e!ithelium. 7he other ma<or cate$ories of ovarian tumors are $erm cell tumors, se* cor" stromal tumors, an" metastatic tumors. 7he fact that this !atient is D/ years ol", has what a!!ears to be ovarian cancer, an" has an elevate" serum CA4:;/ level 'seen in a!!ro*imately @>= of women with e!ithelial cancers), makes e!ithelial ovarian cancer most likely. Non$estational choriocarcinoma 'choice A) of the ovary is e*tremely rare. -urthermore, in a !atient with choriocarcinoma, the serum hC. shoul" be elevate". Cystic teratoma '"ermoi") 'choice ) accounts for ;/ to ?>= of all ovarian neo!lasms. 2owever, most teratomas are "ia$nose" in !remeno!ausal women an" they "o not usually !resent as bilateral masses, ascites, an" evi"ence of tumor s!rea" with an elevate" serum CA4:;/ level. (mbryonal carcinoma 'choice C) is a rare $erm cell tumor. )erum A-& an" hC. are often elevate" with this tumor. )ertoli stromal cell tumor 'choice () is a rare se* cor" stromal tumor that e*hibits a male or testicular "irection of "ifferentiation. ?C) (*!lanation: 7he correct answer is A. Within the last "eca"e it has been reco$ni#e" that 2elicobacter !ylori !lays a central role in the !atho$enesis of chronic $astritis an" !e!tic ulcer "isease '&F%). Aci" secretion is also known to !lay a role. %urin$ !re$nancy, $astric aci" secretion is re"uce" an" there is also a "ecrease in $astric motility. &re$nancy is also associate" with increase" mucus secretion, which is felt to have a !rotective effect on the $astrointestinal tract. ecause of these !hysiolo$ic chan$es "urin$ !re$nancy, active !e!tic ulcer "isease "urin$ !re$nancy is e*tremely uncommon "urin$ !re$nancy. Women rarely "evelo! &F% in !re$nancy an" women with &F% note consi"erable im!rovement. (stimates are

that I>= of !atients with active &F% will e*!erience remission "urin$ !re$nancy. 2owever, once the !re$nancy is com!lete", almost all women will e*!erience recurrence in the ne*t few years. A si$nificant !ortion 'u! to ;>=) of women e*!erience mi$raine hea"aches "urin$ their lives, so issues re$ar"in$ mi$raines an" !re$nancy are not uncommon. As with &F%, there is usually a "ramatic im!rovement of mi$raines "urin$ !re$nancy. (stimates are that D>= of women with mi$raines will have im!rovement. 2owever, it is interestin$ to note that some women will have their first e*!erience with mi$raine "urin$ !re$nancy an" may only e*!erience mi$raine with !re$nancy. +i$raine hea"ache "urin$ !re$nancy shoul" be treate" with acetamino!hen an" antiemetics. Co"eine or me!eri"ine may be $iven for severe hea"aches. (r$otamine !re!arations shoul" be avoi"e" in !re$nancy. 7he safety of sumatri!tan "urin$ !re$nancy has not been establishe", so !re$nant !atients, at !resent, shoul" seek alternative me"ications. 7o state that !re$nancy is associate" with worsenin$ of mi$raines an" &F% 'choice ), worsenin$ mi$raines an" im!rove" &F% 'choice C), or im!rove" mi$raines an" worsene" &F% 'choice %) is incorrect. As e*!laine" above, !re$nancy is associate" with both im!rove" mi$raines an" &F %. 7o state that !re$nancy has no effect on mi$raines or &F% 'choice () is also incorrect. 7hese two illnesses are e*am!les of the !rofoun" effect that !re$nancy can have on certain con"itions. ?D) (*!lanation: 7he correct answer is %. &reterm labor is a ma<or !roblem in the F.). (stimates are that it affects somewhere between /= an" :>= of all !re$nancies. 7he e*act etiolo$y of the !reterm labor is usually "ifficult to "etermine. 7heories aboun" as to why some women "evelo! contractions an" cervical "ilation !rior to term whereas others "o not. &ossible etiolo$ies inclu"e infection, "ehy"ration, cervical weakness, multi!le $estation, an" uterine anomalies. 7he most si$nificant consequence of !reterm labor is that it often results in !remature "elivery of a !remature neonate. &remature neonates are at hi$h risk for !ulmonary immaturity, intraventricular hemorrha$e, necroti#in$ enterocolitis, a!nea, bra"ycar"ia, an" other com!lications. Cesarean "elivery 'choice A) is not necessarily a consequence of !reterm labor. 7his fetus is in the verte* !osition, an" this !atient, shoul" she have unsto!!able !reterm labor, coul" have a va$inal "elivery. -orce!s assiste" va$inal "elivery 'choice ) is not necessarily a consequence of !reterm labor. If this !atient is in unsto!!able !reterm labor, she may have a va$inal "elivery without the nee" of force!s. +aternal infection 'choice C) may be the cause of this !atientAs !reterm labor, but it is unlikely to be the most si$nificant consequence of the !reterm labor. )houl"er "ystocia 'choice () has been re!orte" to occur even in a !reterm "elivery, althou$h this is rare. 7he most si$nificant consequence of !reterm labor is neonatal !rematurity, not shoul"er "ystocia. ?@) (*!lanation:

7he correct answer is C. 7he &a!anicolaou smear is an e*cellent screenin$ technique for cervical cancer because it is easy to !erform, has a relatively low4cost, an" is noninvasive. All women who are either se*ually active or ol"er than :@ shoul" have annual &a! smears. 7he "ownsi"e of the &a! smear is that it has a low sensitivity an" hi$h false4ne$ative rate. 7hat is, many women with abnormal cervical cells will have a ne$ative &a! smear. 2owever, if a woman receives an annual &a! smear, it is likely that the lesion will be "iscovere". ecause the !ro$ression of cervical "ys!lasia to cancer takes time, it is believe" that these lesions will be "iscovere" early enou$h to cure them so lon$ as annual screenin$ occurs. 7his !atient has what is calle" a hi$h4$ra"e squamous intrae!ithelial lesion '2.)I,). 7hese lesions have a si$nificant risk of eventually !ro$ressin$ to invasive cervical cancer if they are not treate". 7herefore, any !atient with 2.)I, on a &a! smear must un"er$o col!osco!y with "irecte" bio!sies so that the lesion can be remove". 7o re!eat the &a! smear in : year 'choice A) is incorrect. A :4year follow4u! is a!!ro!riate for a !atient with normal &a! smears. 7his !atient has 2.)I, an" therefore nee"s much closer follow4u!. 7o re!eat the &a! smear in C months 'choice ) is incorrect. A C4month follow4u! is a!!ro!riate for !atients with a &a! smear showin$ aty!ical squamous cells of un"etermine" si$nificance 'A)CF)), or in some !atients with a low4$ra"e squamous intrae!ithelial lesion ',.)I,). 7o !erform a cone bio!sy 'choice %) woul" be incorrect. &rior to sur$ical mana$ement of an abnormal &a! smear result, a tissue "ia$nosis shoul" be ma"e '&a! smear !rovi"es only a cytolo$ic "ia$nosis). 7his requires col!osco!y with "irecte" bio!sies. 7o !erform a hysterectomy 'choice () woul" not be !ro!er mana$ement. 2ysterectomy as treatment for 2.)I, is $enerally not in"icate". )ome !atients with recurrent 2.)I,, or those with lesions that cannot be !ro!erly treate" with local thera!y, may be can"i"ates for hysterectomy. 2owever, this !atient is ;; years ol" an" likely "esires future fertility. Also, this is her first 2.)I, &a! smear. 7herefore, the !ro!er mana$ement is to !erform a col!osco!y. ?I) (*!lanation: 7he correct answer is %. A "ischar$e such as that "escribe" above requires a full evaluation. 7he "ifferential "ia$nosis inclu"es infection, tumor, trauma, an" va$inal forei$n bo"y. In a ?4year4ol" $irl, a va$inal forei$n bo"y is a common cause of a $reenish, bloo"y, foul4smellin$ va$inal "ischar$e. 7he most common forei$n bo"y foun" is toilet !a!er or stool. It can often be "ifficult to !erform any ty!e of e*amination on a youn$ chil". 1a$inal forei$n bo"ies are ty!ically seen in $irls between 0 an" D years of a$e, an", at these a$es, many will refuse e*amination. 2owever, it is absolutely essential to "etermine the etiolo$y of the "ischar$e. -ailure to remove a va$inal forei$n bo"y can lea" to "ama$e to the va$ina, severe !elvic inflammatory "isease, or !eritonitis. 7hus, this chil" nee"s to have an e*amination. ecause she refuses to be voluntarily e*amine", the e*amination shoul" be !erforme" un"er anesthesia, at which time va$inosco!y can be !erforme". 6eassurance an" e*!ectant mana$ement 'choice A) woul" not be a!!ro!riate for this chil". 7here is nothin$ reassurin$ about havin$ a bloo"y, malo"orous va$inal "ischar$e. 7his "ischar$e coul", for e*am!le, re!resent a mali$nancy or lea" to severe infection. 7herefore,

e*!ectant mana$ement woul" not be a!!ro!riate. Antibiotic a"ministration 'choice ) woul" not be the correct ne*t ste!. It is unclear what is causin$ the "ischar$e an" to sim!ly assume that the etiolo$y is infectious, an" to sen" the !atient home with antibiotics woul" not be correct. &olice notification 'choice C) woul" not be the most a!!ro!riate ne*t ste!. ?4year4ol" chil"ren can "evelo! va$inal forei$n bo"ies without abuse takin$ !lace. A thorou$h evaluation shoul" be con"ucte", inclu"in$ cultures for se*ually transmitte" "iseases, but !olice notification woul" not be the ne*t ste!. &elvic e*amination with !hysical restraint 'choice () shoul" not be !erforme" on a chil". %oin$ so risks !sycholo$ical "istress an" !ossible future se*ual "ysfunction. />) (*!lanation: 7he correct answer is C. &lacenta !revia is "efine" as a !lacenta locate" over the cervical os. 7here are 0 ma<or ty!es. Com!lete !revia "escribes a !lacenta that com!letely covers the cervical os. &artial !revia is a !lacenta that covers some of the cervical os, with the remain"er of the os uncovere" by the !lacenta. +ar$inal !revia "escribes a !lacenta that is locate" at the e"$e of the cervical os. 7hree ma<or risk factors for !lacenta !revia are maternal a$e, minority race, an" !revious cesarean "elivery. &lacenta accreta "escribes the con"ition in which there is abnormal attachment of the !lacenta to the uterine wall. In this con"ition, the "eci"ua basalis is absent an" the !lacenta is attache" to the myometrium 'accreta) or inva"es into the myometrium 'increta), or !erforates throu$h the myometrium '!ercreta). &atients with a !lacenta !revia an" no !rior cesarean "eliveries have a /= risk of havin$ a !lacenta accreta. &atients with a !revia an" one !rior cesarean "elivery have a ;/= risk of havin$ an accreta. An" !atients with a !lacenta !revia an" ; or more !rior cesarean "eliveries have a $reater than />= risk of havin$ a !lacenta accreta. +any !atients with a !revia an" accreta will require a hysterectomy at the time of "elivery. 7his !atient, with a history of ; !rior cesarean "eliveries an" a !lacenta !revia is at hi$hest risk for !lacenta accreta. 7his !atient woul" not be consi"ere" to be at hi$hest risk of "ystocia 'choice A) because, with a !lacenta !revia, she woul" not be allowe" to labor an", therefore, woul" not be at risk of "ystocia. 7here a!!ears to be no stron$ association between !lacenta !revia an" intrauterine fetal "emise 'choice ) or !reeclam!sia 'choice %). Fterine ru!ture 'choice () is a concern in women who have ha" !rior cesarean "eliveries, an" the risk of ru!ture "oes rise with the number of !revious cesarean "eliveries. It is of !articular concern if the woman is in labor. 7his !atient, with a !lacenta !revia, however, will not be allowe" to labor.

USMLE Step 2 Practice Test Block 18 Name: Instructions: Answer the questions below to the best of your ability. When you finish the test, click the Check button at the bottom to view the results.

1.A &1!year!ol" 'oman comes to the physician $eca#se she has not ha" a menstr#al perio" for ( months. She pre)io#sly ha" normal cycles. She also states that o)er the past year she has felt increasingly 'eak an" tire". She notes that she al'ays feels col" an" that her hair has $een thinning o)er the co#rse of the year. She also complains of constipation% 'eight gain% an" "epression. /er temperat#re is &<.( 2 381 56% $loo" press#re is 1447<4 mm /g% p#lse is <7min#te% an" respirations are 147min#te. E.amination is significant for $rittle hair an" "elaye" "eep ten"on refle.es. Urine h#man chorionic gona"otropin 3h2C6 is negati)e. Thyroi" stim#lating hormone 3TS/6 is 24 HU7mL. Prolactin is normal. =hich of the follo'ing is the most likely ca#se of this patient,s amenorrheaa6/yperprolactinemia $6/ypothyroi"ism c6Mallmann syn"rome

"6Polycystic o)arian syn"rome e6Pregnancy :ormal La$s 2.A ( !year!ol" 'oman comes to the physician $eca#se of irreg#lar )aginal $lee"ing. She has $een menopa#sal for the past 2 years% $#t has note" on!an"!off spotting for the past 2 years% 'hich she fin"s intolera$le. She has a complicate" past me"ical history incl#"ing hypertension% "ia$etes% an" se)ere chronic o$str#cti)e p#lmonary "isease. E.amination is #nremarka$le. An en"ometrial $iopsy is performe" that "emonstrates an en"ometrial polyp 'ith atypical cells that are "iffic#lt to gra"e. =hich of the follo'ing is the most appropriate ne.t step in managementa6/ormone replacement therapy $6>ral contracepti)e pill c6/ysteroscopy "6Laparoscopy e6/ysterectomy :ormal La$s &. A &1!year!ol" 'oman% gra)i"a 1% para 4% at &<!'eeks, gestation 'ith t'ins comes to the physician for a prenatal )isit. The patient has ha" no contractions% $lee"ing from the )agina% or loss of fl#i"% an" the $a$ies are mo)ing 'ell. An #ltraso#n" that 'as performe" to"ay sho's that the presenting fet#s is )erte. an" the non!presenting fet#s is $reech. Both fet#ses are appropriately gro'n an" greater than 2444 g. The patient 'ants to kno' if she sho#l" ha)e a )aginal or cesarean "eli)ery. =hich of the follo'ing is the proper co#nseling for this patienta6 Both )aginal "eli)ery an" cesarean "eli)ery are accepta$le. $62esarean "eli)ery is man"ate" $eca#se the fet#ses are I 2444g. c62esarean "eli)ery is man"ate" $eca#se the secon" t'in is $reech. "6?aginal "eli)ery is man"ate" $eca#se the fet#ses are I 2444g. e6?aginal "eli)ery is man"ate" $eca#se the first t'in is )erte.. :ormal La$s 4. A 2(!year!ol" 'oman comes to the physician $eca#se of fe)ers an" $ack pain. She states that a fe' "ays ago she ha" $#rning 'ith

#rination. >)er the ne.t fe' "ays she "e)elope" fe)ers an" chills an" a pain on the right si"e of her $ack. She has no me"ical pro$lems an" takes no me"ications. /er temperat#re is &1.8 2 3142 56% $loo" press#re is 1147(4 mm /g% p#lse is 1427min#te% an" respirations are 1<7min#te. E.amination sho's a patient in mil" "istress 'ith shaking chills an" right costo)erte$ral angle ten"erness. Le#kocyte co#nt is 11%4447mm&. Urinalysis sho's 144 le#kocytes7high po'ere" fiel". =hich of the follo'ing is the most appropriate ne.t step in managementa6>$ser)ation only $6Spinal magnetic resonance imaging 3MA+6 scan c6>#tpatient management 'ith oral trimethoprim!s#lfametho.aDole "6/ospital a"mission an" initiation of +? trimethoprim! s#lfametho.aDole e6 /ospital a"mission an" a"ministration of a 2!'eek co#rse of +? tetracycline :ormal La$s .A 24!year!ol" patient comes to the "octor $eca#se she has concerns regar"ing her se.#ality. She states that for as long as she can remem$er she has $een se.#ally attracte" to other 'omen. She 'as raise" in a family 'here homose.#ality is consi"ere" 0#naccepta$le%0 so she has ne)er "isc#sse" these feelings $efore. :o'% ho'e)er% she feels that she can no longer hi"e her feelings% $#t she is concerne" that she 'ill ca#se "eep an" irrepara$le harm to her relationship 'ith her parents if she tells them. =hich of the follo'ing is the most appropriate ne.t step in the management of this patienta6Prescri$e a $enDo"iaDepine $6Prescri$e estrogen c6Prescri$e haloperi"ol "6Aeass#re her that time 'ill change her feelings e6Aefer her for psychological co#nseling :ormal La$s <. A <1!year!ol" 'oman comes to the physician $eca#se of a painf#l l#mp in her )agina. She states that the l#mp has $een there for a fe' months% $#t has recently $eg#n to ca#se her pain. She has hypertension% for 'hich she takes a "i#retic% $#t no other me"ical pro$lems.

E.amination sho's a 4 cm cystic mass near the patient,s introit#s $y the right la$ia. The mass is mil"ly ten"er. The remain"er of the pel)ic e.amination is normal. =hich of the follo'ing is the most appropriate ne.t step in managementa6E.pectant management $6SitD $aths c6>ral anti$iotics "6Biopsy of the mass e6=or" catheter placement :ormal La$s (.A 21!year!ol" primigra)i" 'oman at 1 'eeks, gestation comes to the physician for her first prenatal )isit. A home pregnancy test 'as positi)e. She has no complaints. She is concerne"% ho'e)er% $eca#se she is a carrier of the fragile F m#tation. /er h#s$an" is also kno'n to $e a carrier. This is a highly "esire" pregnancy. She 'ants to kno' 'hether there is a 'ay to "etermine 'hether the fet#s is affecte". =hich of the follo'ing is the most appropriate ne.t step in managementa6There is nothing to offer this co#ple $6>ffer testing of the parents c6>ffer MA+ of the fet#s "6>ffer 2n" trimester amniocentesis e6>ffer termination of the pregnancy :ormal La$s 1.A 1(!year!ol" female comes to the physician $eca#se she has not yet ha" a menstr#al perio". She also complains of a lack of $reast "e)elopment. Past me"ical history is significant for anosmia an" color $lin"ness. Past s#rgical history is significant for a cleft palate that 'as repaire" in chil"hoo". She takes no me"ications an" has no allergies to me"ications. E.amination is significant for a$sent $reast "e)elopment% an" a hypoestrogenic )#l)a an" )agina. Urine h2C is negati)e. =hich of the follo'ing is the most likely "iagnosisa6 Anore.ia ner)osa $6Mallmann syn"rome c6Polycystic o)arian syn"rome "6Pregnancy e6 Testic#lar feminiDation syn"rome

:ormal La$s 8. A 21!year!ol" 'oman% gra)i"a 2% para 1% at 21 'eeks, gestation comes to the physician $eca#se of spotting after interco#rse an" a fo#l! smelling )aginal "ischarge. /er prenatal co#rse has% #p to no'% $een #ncomplicate"% an" she has no me"ical pro$lems. Spec#l#m e.amination sho's inflammation of the cer)i. 'ith a m#cop#r#lent cer)ical "ischarge. A gonorrhea an" 2hlamy"ia test is performe" 'hich comes $ack positi)e for chlamy"ia. =hich of the follo'ing is the most appropriate pharmacotherapya6 ADithromycin $6@o.ycycline c6Le)oflo.acin "6Penicillin e6 Streptomycin :ormal La$s 14.A &1!year!ol" 'oman comes to the physician $eca#se of $#rning 'ith #rination. She states that the $#rning starte" a$o#t 2 "ays ago an" has $een gro'ing 'orse since. She has no fre*#ency or #rgency. She ha" one episo"e of pyelonephritis in the past $#t no other me"ical pro$lems. >n e.amination there is no costo)erte$ral angle or a$"ominal ten"erness. The e.amination is significant for a thick% 'hite )aginal "ischarge 'ith erythema an" e.coriations of the la$ia. Urinalysis is negati)e. M>/7:ormal saline smear "emonstrates pse#"ohyphae. =hich of the follo'ing is the most likely "iagnosisa6 2an"i"a )aginitis $6/emorrhagic o)arian cyst c6Pel)ic inflammatory "isease "6Pyelonephritis e6Urinary tract infection :ormal La$s 11.A 2&!year!ol" 'oman calls her physician for the res#lts of her Pap test. She has a history of 2hlamy"ia. She has ne)er ha" an a$normal Pap. She occasionally has #nprotecte" interco#rse. The physician

informs her that the Pap 'as normal. The patient is relie)e"% $#t 'ants to kno' 'hether this res#lt co#l" $e 'rong. The physician e.plains that a Pap test "etects a$normal cells in ro#ghly 4 of e)ery 'omen 'ho ha)e a$normal cer)ical cells. =hich of the follo'ing represents the sensiti)ity of the Papanicolao# testa6 4B $61B c624B "614B e6144B :ormal La$s 12.A &4!year!ol" 'oman% gra)i"a &% para 2 at term comes to the la$or an" "eli)ery 'ar" 'ith a g#sh of $loo"% a$"ominal pain% an" irreg#lar% painf#l contractions. /er prenatal co#rse 'as significant for her $eing Ah negati)e an" anti$o"y negati)e. /er temperat#re is &( 2 381.< 56% p#lse is 1147min#te% $loo" press#re is 1147(4 mm /g% an" respirations are 127min#te. A$"ominal e.amination sho's a ten"er a$"omen an" cer)ical e.amination sho's the cer)i. to $e close" an" long 'ith a significant amo#nt of $loo" in the )agina. The fetal heart rate is in the 1(4s 'ith mo"erate to se)ere )aria$le "ecelerations 'ith contractions. The "iagnosis of placental a$r#ption is ma"e an" an emergent cesarean "eli)ery is performe". To "etermine the correct amo#nt of AhoCAM 3anti!@ imm#ne glo$#lin6 that sho#l" $e gi)en% 'hich of the follo'ing is the most appropriate la$oratory test to sen"a6Apt test $62omplete $loo" co#nt c6Mleiha#er!Betke "6Partial throm$oplastin time e6Ser#m potassi#m :ormal La$s 1&. A &2!year!ol" 'oman% gra)i"a 4% para &% at &8 'eeks, gestation comes the la$or an" "eli)ery 'ar" 'ith painf#l contractions. /er prenatal co#rse 'as #nremarka$le. E.amination sho's that her cer)i. is cm "ilate"% 144B efface" an" the fetal heart rate is in the 1&4s an" reacti)e. She is gi)en meperi"ine for pain control. She progresses rapi"ly an" less than 2 ho#rs later she "eli)ers a (!po#n"% <!o#nce 3&%&4 g6 male fet#s. The one!min#te APCAA score is 1 an" the infant is

making little respiratory effort. =hich of the follo'ing is the most appropriate ne.t step in managementa6Bloo" transf#sion $6Cl#cose c6:alo.one "6Penicillin e6So"i#m $icar$onate :ormal La$s 14. A &1!year!ol" 'oman comes to the physician for an ann#al e.amination an" Pap smear. She has no complaints. She has a reg#lar perio" e)ery month. She is se.#ally acti)e 'ith her h#s$an". She has migraine hea"aches an" is stat#s post a t#$al ligation. She states that she #ses n#mero#s alternati)e me"ications for moo"% sleep% an" "isease pre)ention. E.amination% incl#"ing pel)ic an" $reast e.amination% is #nremarka$le. =hich of the follo'ing is an appropriate *#estion to ask this patienta6 @oes yo#r h#s$an" kno' yo# are #sing these alternati)e me"ications$6@o yo# realiDe ho' "angero#s alternati)e me"icines arec6=hich alternati)e me"ications "o yo# #se"6=hy "on,t yo# stick 'ith tra"itional me"icinese6=hy ha)en,t yo# re)eale" yo#r #se of alternati)e me"ications $efore:ormal La$s 1 .A mother $rings her 12!year!ol" "a#ghter to the physician $eca#se the mother is concerne" that her chil" has "elaye" physical "e)elopment. +n partic#lar% the mother is concerne" $eca#se her "a#ghter has not yet ha" a menstr#al perio". The "a#ghter $egan "e)eloping $reasts at age 14% $#t has not ha" her first perio". The "a#ghter has no me"ical pro$lems an" takes no me"ications. E.amination sho's "e)eloping $reasts an" normal e.ternal female genitalia. =hich of the follo'ing is the most appropriate response to the mothera6Breast "e)elopment at age 14 is a$normally early. $6Breast "e)elopment at age 14 is a$normally late. c6E)al#ation for late menses sho#l" $e starte" imme"iately. "6E)al#ation for late menses sho#l" $e starte" at age 1 .

e6 /er chil",s se.#al "e)elopment is none of her $#siness. :ormal La$s 1<.A 2&!year!ol" 'oman% gra)i"a 2% para 1% at < 'eeks, gestation comes to the emergency "epartment $eca#se of lo'er a$"ominal pain an" fe)ers. She states that her symptoms $egan 2 "ays ago an" ha)e stea"ily 'orsene" since. Past me"ical history is significant for 2 episo"es of gonorrhea an" 1 episo"e of chlamy"ia. Temperat#re is &1.8 2 3142.1 56% $loo" press#re is 1147(< mm /g% p#lse is 1427min% an" respirations are 127min#te. A$"ominal e.amination "emonstrates significant lo'er a$"ominal ten"erness. Pel)ic e.amination sho's a m#cop#r#lent cer)ical "ischarge an" $iman#al e.amination re)eals cer)ical motion ten"erness an" a"ne.al ten"erness. 2omplete $loo" co#nt sho's le#kocytes 11%4447mm&. Pel)ic #ltraso#n" sho's a <!'eek intra#terine gestation 'ith no a"ne.al fin"ings. =hich of the follo'ing is the most appropriate managementa6:o treatment is necessary $6+ntram#sc#lar ceftria.one% oral "o.ycycline% an" "ischarge home c6+ntra)eno#s cefotetan an" "o.ycycline an" hospital a"mission "6 +ntra)eno#s clin"amycin an" gentamicin an" hospital a"mission e6Laparoscopy :ormal La$s 1(.A 2 !year!ol" 'oman% gra)i"a 2% para 2% comes to the physician to "isc#ss $irth control options. She an" her partner ha)e trie" to #se con"omsG ho'e)er% they fin" it "iffic#lt to #se them consistently an" she 'o#l" like to try another form of contraception. She has no me"ical pro$lems% takes no me"ications% an" has no family history of cancer. /er e.amination is 'ithin normal limits. After a "isc#ssion 'ith the physician% she chooses to take the oral contracepti)e pill 3>2P6. She stays on the pill for the ne.t three years. She no' has most significantly "ecrease" her risk of "e)eloping 'hich of the follo'ing malignanciesa6Bone cancer $6Breast cancer c62er)ical cancer "6En"ometrial cancer e6Li)er cancer :ormal La$s

11. A 2<!year!ol" primigra)i" 'oman at & 'eeks, gestation comes to the la$or an" "eli)ery 'ar" $eca#se of painf#l #terine contractions an" a g#sh of fl#i". Sterile spec#l#m e.amination re)eals a pool of clear fl#i" in the )agina that is nitraDine positi)e. =hen the fl#i" is e.amine" #n"er the microscope% a 0ferning0 pattern is seen. 2er)ical e.amination sho's the patient to $e 4 cm "ilate"% 144B efface"% an" at 4 station. 5etal fingers can $e felt along si"e the fetal hea". E.ternal #terine monitoring sho's contractions e)ery 2 min#tes. E.ternal fetal monitoring sho's the fetal heart rate to $e in the 1&4s an" reacti)e. =hich of the follo'ing is the most appropriate ne.t step in managementa6 E.pectant management $6>.ytocin a#gmentation c65orceps "eli)ery "6?ac##m "eli)ery e62esarean section :ormal La$s 18. A 2 !year!ol" 'oman comes to the physician for an ann#al e.amination. She has $een feeling 'ell o)er the past year. /er past me"ical an" s#rgical histories are #nremarka$le. Past o$stetrical history is significant for the term )aginal "eli)ery t'o years ago of a male infant 'ith spina $ifi"a. E.amination is 'ithin normal limits. The patient states that she 'o#l" like to try to $ecome pregnant 'ithin the ne.t fe' months an" 'ants to kno' if she nee"s to start taking any )itamins or me"ications. =hich of the follo'ing s#pplements sho#l" this patient takea6 5olic aci"% 4 mg7"ay starting preconceptionally $6 5olic aci"% 4 mg7"ay starting in the first trimester c6 ?itamin A% 14%444 +U7"ay starting preconceptionally "6?itamin A% 14%444 +U7"ay starting in the first trimester e6 :o s#pplements are nee"e" :ormal La$s 24. A 2 !year!ol" 'oman comes to the physician $eca#se of pain an" $#rning 'ith #rination. She states that the symptoms starte" t'o "ays

ago an" ha)e 'orsene" since. She has no fe)er or chills an" has ne)er ha" these symptoms $efore. She has hypothyroi"ism for 'hich she takes thyroi" hormone replacement. >ther'ise she has no me"ical pro$lems. /er temperat#re is &( 2 381.< 56. E.amination is #nremarka$le incl#"ing a normal pel)ic e.amination. A M>/ an" normal saline 0'et prep0 is performe" on her )aginal "ischarge an" is negati)e. Urinalysis re)eals n#mero#s 'hite $loo" cells. =hich of the follo'ing is the most likely pathogena6Escherichia coli $6:eisseria gonorrhoeae c6Pse#"omonas species "6Staphylococc#s saprophytic#s e6 Trichomonas )aginalis :ormal La$s 21. A &&!year!ol" 'oman comes to the physician $eca#se she has not ha" a menstr#al perio" for 1 months. She ha" menarche at the age of 12 an"% after a fe' years of irreg#lar menses% has since ha" normal monthly menses. She has no me"ical pro$lems an" takes no me"ications. E.amination re)eals a normal!appearing female 'ith no a$normalities note". Urine h#man chorionic gona"otropin 3h2C6 is negati)e. Ser#m thyroi" stim#lating hormone 3TS/6 an" prolactin are also normal. The patient is gi)en a 14!"ay co#rse of me"ro.yprogesterone acetate. Upon completing the 14 "ays% she has a hea)y menstr#al perio". This patient,s 'ith"ra'al $lee"ing in response to the progesterone pro)i"es goo" e)i"ence for 'hich of the follo'inga6 Asherman syn"rome $6En"ogeno#s estrogen pro"#ction c6En"ometrial carcinoma "6Menopa#se e6Pregnancy :ormal La$s 22. A 41!year!ol" 'oman% gra)i"a 4% para &% at term is a"mitte" to the la$or an" "eli)ery 'ar" 'ith reg#lar contractions e)ery 2 min#tes. E.amination sho's that her mem$ranes are grossly r#pt#re" an" that her cer)i. is cm "ilate". >)er the follo'ing & ho#rs% she progresses to f#ll "ilation an" E2 station. A fetal $ra"ycar"ia "e)elops% an" the

"ecision is ma"e to procee" 'ith )ac##m!assiste" )aginal "eli)ery. A ( po#n"% 1 o#nce $oy is "eli)ere". APCAA scores are 1 at 1 min#te an" 8 at min#tes. =hich of the follo'ing $est represents an a")antage of )ac##m e.traction o)er the forceps for e.pe"iting "eli)erya6The )ac##m can $e #se" at higher stations $6The )ac##m can $e #se" for fet#ses in $reech presentation c6The )ac##m can $e #se" in face presentations "6The )ac##m can $e #se" 'ith intact mem$ranes e6The )ac##m "oes not occ#py space ne.t to the fetal hea" :ormal La$s 2&.A 2&!year!ol" primigra)i" 'oman comes to the physician $eca#se of )aginal $lee"ing. /er last menstr#al perio" 'as < 'eeks ago. She has no other symptoms. E.amination sho's a 14!'eek siDe" #ter#s% $#t is other'ise #nremarka$le. Pel)ic #ltraso#n" re)eals a sno'storm pattern consistent 'ith a complete mole. Ser#m $eta!h2C is marke"ly ele)ate" o)er normal pregnant )al#es. A chest .!ray film is negati)e. A "ilation an" e)ac#ation is performe" an" the pathologic "iagnosis is complete hy"ati"iform mole. =hich of the follo'ing is the most appropriate ne.t step in managementa6 E)al#ation in one year $65ollo' $eta!h2C le)els to 4 c6@actinomycin "6Methotre.ate e6/ysterectomy :ormal La$s 24. A 22!year!ol" 'oman% gra)i"a &% para 2% at 22 'eeks, gestation comes to the physician $eca#se of an #lcer near her )agina. She note" this a fe' "ays ago an" it has not impro)e". The #lcer is painless. The patient has no history of me"ical pro$lems an" takes no me"ications. She is allergic to penicillin. E.amination is significant for a 22 'eek! siDe" #ter#s an" a 1 cm% raise"% nonten"er lesion on the "istal portion of the )agina. A rapi" plasma reagin 3APA6 test is sentG the res#lt is positi)e. A microhemaggl#tination assay for Treponema palli"#m 3M/A!TP6 is also rea" as positi)e. =hich of the follo'ing is the most appropriate management for this patienta6A"minister erythromycin $6A"minister le)oflo.acin c6A"minister metroni"aDole

"6A"minister tetracycline e6@esensitiDe the patient an" then a"minister penicillin :ormal La$s 2 .A &2!year!ol" 'oman comes to the physician $eca#se of rec#rrent painf#l o#t$reaks on her la$ia an" )agina. /er first o#t$reak 'as si. years ago. At that time she "e)elope" 'hat she tho#ght 'as a $a" 0fl#0 'ith malaise an" a fe)er% along 'ith a painf#l rash on her la$ia. This initial o#t$reak resol)e"% $#t since then she has ha" appro.imately 1 !14 o#t$reaks each year. Each o#t$reak is prece"e" $y $#rning in her perineal area. A fe' "ays later she "e)elops )esicles% then shallo'% painf#l #lcers that resol)e in a$o#t 14 "ays. =hich of the follo'ing is the most appropriate pharmacotherapya6@aily oral acyclo)ir $6@aily oral estrogen c6@aily topical estrogen "6 @aily oral ferro#s s#lfate e6@aily oral penicillin :ormal La$s 2<. A &4!year!ol" primigra)i" 'oman at &4 'eeks, gestation comes to the physician 'ith reg#lar contractions e)ery < min#tes. /er prenatal co#rse 'as significant for type 1 "ia$etes% 'hich she has ha" for 14 years. >)er the co#rse of 1 ho#r% she contin#es to contract% an" her cer)i. a")ances from close" an" long to a fingertip of "ilation 'ith some effacement. The patient is starte" on magnesi#m s#lfate% penicillin% an" $etamethasone. =hich of the follo'ing is the most likely si"e effect from the a"ministration of corticosteroi"s to this patienta6@ecrease" chil"hoo" intelligence $6+ncrease" maternal ins#lin re*#irement c6Maternal infection "6:eonatal a"renal s#ppression e6 :eonatal infection
:ormal La$s

:ote; 2heck yo#r o'n ans'ers $efore hitting the 2heck $#tton $elo'. =hen yo# click the 2heck $#tton% a $ro'ser 'in"o' 'ill appear that contains a s#mmary of yo#r res#lts.

E.planations Block 18 E.planations

16 (. &re$nancy (*!lanation: 7he correct answer is . )econ"ary amenorrhea is "efine" as the absence of menses for C cycle intervals or :; months in a woman who !reviously ha" re$ular cycles. 7his !atient, therefore, has secon"ary amenorrhea. )he also has a constellation of si$ns an" sym!toms that are hi$hly su$$estive of hy!othyroi"ism. &atients with hy!othyroi"ism often com!lain of some combination of weakness, fati$ue, col" intolerance, consti!ation, wei$ht $ain, "e!ression, or thinnin$ of the hair. &hysical e*amination can reveal bra"ycar"ia an" low bloo" !ressure. ,aboratory evaluation often shows an elevate" 7)2 as the !ituitary attem!ts to stimulate the un"erfunctionin$ thyroi". 2owever, many !atients with hy!othyroi"ism will be asym!tomatic an" the thyroi" abnormality is foun" by thyroi" function tests. 2y!othyroi"ism likely lea"s to amenorrhea throu$h chan$es in .n62 !ro"uction. 7reatment with thyroi" re!lacement will often return these !atients to re$ular menses. 2y!er!rolactinemia 'choice A) is the cause of secon"ary amenorrhea in a!!ro*imately ;>= of cases. 7his !atient, however, has a normal !rolactin level. 5allmann syn"rome 'choice C) is a rare cause of !rimary amenorrhea. 7his syn"rome is characteri#e" by $ona"otro!in "eficiency, anosmia or hy!osmia, cleft li! or !alate, an" minimal se*ual "evelo!ment. 7his !atient "oes not have !rimary amenorrhea. &olycystic ovarian syn"rome 'choice %) is often characteri#e" by obesity, hirsutism, infertility, an" oli$omenorrhea. 7hyroi" "ysfunction is not !art of this syn"rome. &re$nancy 'choice ()

is, by far, the most common cause of secon"ary amenorrhea. 7his !atient has a ne$ative urine hC.. ;) (*!lanation: 7he correct answer is C. 7his !atient is likely havin$ irre$ular s!ottin$ secon"ary to the !oly!. (n"ometrial !oly!s are !ro<ections of en"ometrial tissue that !rotru"e into the en"ometrial cavity. 7hey can be seen in women of any a$e, but are most commonly seen in !erimeno!ausal women. 7his !roblem shoul" be a""resse" for ; reasons: :. 7he blee"in$ !er va$ina is "istressin$ to the !atient. ;. 7here are some aty!ical cells from the bio!sy that may re!resent cancer an" !oly!s can contain mali$nant cells within them. 7herefore, the !oly! shoul" be remove". 7he question then becomes how best to remove it. A hysterosco!y can be !erforme" un"er monitore" anesthesia care '+AC), an a!!roach that !rovi"es a"equate anesthesia without requirin$ the !atient to have $eneral anesthesia. It woul" be !referable to avoi" $eneral anesthesia in a !atient with so many me"ical con"itions. 2ysterosco!y woul" allow visuali#ation of the entire uterine cavity an" removal of the !oly!. A curetta$e shoul" be !erforme" afterwar" to fully sam!le the cavity. 2ormone re!lacement thera!y 'choice A) woul" not be the most a!!ro!riate ne*t ste!. -irst, the !oly! must be remove" an" histolo$ic evaluation of the !oly! an" en"ometrial tissues !erforme" to rule out mali$nancy !rior to institutin$ hormone re!lacement thera!y. 7he oral contrace!tive !ill 'choice ) woul" not be a!!ro!riate mana$ement for a D/4year4ol" woman, as the "ose of hormones is hi$her than necessary. ,a!arosco!y 'choice %) woul" not be in"icate". 7his !atient is havin$ s!ottin$, which is an intrauterine !rocess. ,a!arosco!y allows visuali#ation of only the e*ternal, serosal uterine surface. 2ysterectomy 'choice () woul" not be the most a!!ro!riate mana$ement. 2ysterectomy woul" take care of the !atientAs s!ottin$ an" woul" !rovi"e tissue for !atholo$ic "ia$nosis. 2owever, in this !atient with multi!le me"ical !roblems, the same $oals can be achieve" with the less invasive !roce"ure of hysterosco!y. 0) (*!lanation: 7he correct answer is A. +o"e of "elivery with twin $estations is an area that has $enerate" controversy over time. &atients with verte*4verte* twins are $enerally allowe" to have a va$inal "elivery. &atients with a !resentin$ twin that is non4verte* are $enerally a"vise" to have a cesarean "elivery. &atients with the !resentin$ twin verte* an" the non4!resentin$ twin non4verte* may "eci"e which mo"e of "elivery they woul" !refer. Once the !resentin$ 'verte*) twin has "elivere", there are essentially ; o!tions for "elivery of the secon" 'non4verte*) twin. 7he first o!tion is an e*ternal ce!halic version, in which the hea" of the secon" twin is $ui"e" into the !elvis so that it becomes a verte* !resentation. 7he secon" o!tion is a breech e*traction of the secon" twin. reech e*traction may be !erforme" so lon$ as there is an a"equate !elvis, a fetal wei$ht $reater than ;,>>>$, an e*!erience" !hysician, a fle*e" fetal hea", an" available $eneral anesthesia. 7o state that cesarean "elivery is

man"ate" because the fetuses are O ;>>>$ 'choice ) is incorrect. 7he fact that the fetuses are O ;>>>$ makes a va$inal "elivery with a non4verte* secon" twin !ossible. 7o state that cesarean "elivery is man"ate" because the secon" twin is breech 'choice C) is incorrect. As e*!laine" above, verte*4nonverte* twins may be "elivere" va$inally so lon$ as certain criteria are met. 7o state that va$inal "elivery is man"ate" because the fetuses are O ;>>>$ 'choice %) is incorrect. 1a$inal "elivery is !ossible because the fetuses are O ;>>>$, but the mother may still choose to have a cesarean "elivery. 7o state that va$inal "elivery is man"ate" because the first twin is verte* 'choice () is incorrect. With the first twin verte*, va$inal "elivery is !ossible, but with a non4verte* secon" twin, cesarean "elivery woul" also be entirely a!!ro!riate. ?) (*!lanation: 7he correct answer is %. 7his !atient has a !resentation that is most consistent with !yelone!hritis. &atients with !yelone!hritis ty!ically com!lain of some combination of back !ain, fevers, chills, "ysuria, nausea, an" vomitin$. (*amination will often show an elevate" tem!erature, costovertebral an$le ten"erness, an" an elevate" leukocyte count. Frinalysis may "emonstrate !ositive nitrite an" leukocyte esterase testin$. Frine se"iment often reveals white bloo" cells, re" bloo" cells, an" white cell casts. &yelone!hritis can be mana$e" on an out!atient basis if the !atient is otherwise healthy, has no com!licatin$ factors, an" is reliable to return if her con"ition worsens. A !atient cannot be mana$e" as an out!atient if there is any evi"ence of se!sis. 7his !atient, with her hi$h fevers, shakin$ chills, an" elevate" leukocyte count may have se!sis an" shoul" therefore be a"mitte" to the hos!ital for intravenous antibiotics. 7reatment is with I1 trimetho!rim4sulfametho*a#ole, I1 ceftria*one, I1 $entamicin with or without am!icillin, or an I1 fluoroquinolone. Once the !atient is afebrile, her con"ition is im!rovin$, an" she is able to tolerate oral intake, she may be converte" to an oral antibiotic re$imen to com!lete a :?4"ay course. Observation only 'choice A) woul" not be correct for this !atient. 7his !atient has !yelone!hritis, which is unlikely to resolve without antibiotic thera!y. )!inal +6I 'choice ) is often use" to evaluate !atients with back !ain. 7his !atient, however, has back !ain that is almost certainly relate" to a renal infection, therefore s!inal +6I woul" not be necessary. Out!atient mana$ement with oral trimetho!rim4sulfametho*a#ole 'choice C) is a!!ro!riate in some cases of uncom!licate" !yelone!hritis, as e*!laine" above. 7his !atient, however, is quite ill an" !ossibly se!tic. )he, therefore, requires hos!ital a"mission. 2os!ital a"mission an" a"ministration of a ;4week course of I1 tetracycline 'choice () woul" not be a!!ro!riate. 7etracycline is not a "ru$4of4choice in the treatment of !yelone!hritis. /) (*!lanation: 7he correct answer is (. )e*ual orientation is an issue of tremen"ous im!ortance in the life of an in"ivi"ual. &atients often come to !hysicians for a"vice an" in!ut re$ar"in$ questions of se*ual

orientation. It is essential that the !hysician try to a""ress these issues in an unbiase" manner that will allow the !atient to e*!ress her feelin$s. 2owever, it is also essential that a !hysician reali#e his or her limitations in bein$ able to "eal with com!le* questions re$ar"in$ se*uality, self4e*!ression, an" family "ynamics. 7his !atient has issues not only re$ar"in$ her se*ual orientation, but also her family an" the im!act her se*ual orientation may have u!on them. )he woul", therefore, be most likely to benefit from !sycholo$ical counselin$, where a full evaluation coul" be con"ucte" encom!assin$ the !ersonal an" family issues. 7o !rescribe a ben#o"ia#e!ine 'choice A) woul" not be the most a!!ro!riate ne*t ste! in mana$ement. 7his !atient is not com!lainin$ of acute an*iety or slee! "isturbance. 6ather, she has com!le* issues re$ar"in$ her se*uality an" family "ynamics. 7o sim!ly !rescribe an an*iolytic me"ication rather than a""ress the "ee!er !sycholo$ical issues woul" not be a!!ro!riate. 7o !rescribe estro$en 'choice ) woul" not be the most a!!ro!riate ne*t ste! in mana$ement. 7his !atient "oes not have a "isease !rocess that woul" require estro$en for thera!y. 7o !rescribe halo!eri"ol 'choice C) woul" not be a!!ro!riate. 7his !atient "oes not require an anti4!sychotic me"ication as she has no evi"ence of !sychosis or !sychotic behavior. 7o attem!t to reassure her that time will chan$e her feelin$s 'choice %) woul" not be a!!ro!riate. -irst, her issues re$ar"in$ se*ual orientation may not chan$e with time. )econ", there is no in"ication that a chan$e of feelin$s is what she nee"s. 2er !resent feelin$s an" issues must be a""resse" an" this coul" be "one a!!ro!riately with !sycholo$ical counselin$. C) (*!lanation: 7he correct answer is %. 7his !atient has fin"in$s that initially seem to be consistent with a artholinAs $lan" cyst or abscess. 7he artholinAs $lan"s are !aire" $lan"s foun" on the !osterolateral as!ect of the va$ina at the introitus. 7hese $lan"s normally secrete mucus into the va$ina, !articularly with se*ual stimulation. 7hey $row ra!i"ly "urin$ !uberty an" shrink after the meno!ause. In a youn$ woman it woul" be reasonable to assume that this cystic mass re!resents a artholinAs cyst or abscess. 2owever, one cannot make this assum!tion in a !ostmeno!ausal !atient. A cystic mass on the vulva in a !ostmeno!ausal woman must be bio!sie" as there is a hi$her likelihoo" that this lesion re!resents a artholinAs $lan" carcinoma. &rimary carcinoma of the artholinAs $lan" accounts for about /= of vulvar mali$nancies. %elay in "ia$nosis is common because many clinicians an" !atients assume the mass is a beni$n cyst. Any !ersistent mass in this re$ion, es!ecially in women $reater than ?> years of a$e, shoul" be bio!sie". (*!ectant mana$ement 'choice A) woul" not be a!!ro!riate. -irst, the !atient is sym!tomatic an" therefore requires somethin$ for relief. )econ", there is the !ossibility that this mass re!resents a mali$nancy an" it, therefore, shoul" be bio!sie". )it# baths 'choice ) can be recommen"e" to !atients with certain vulvar lesions. 2owever, this mass must first be bio!sie" to rule out mali$nancy. Oral antibiotics 'choice C) woul" not be the most a!!ro!riate ne*t ste! in mana$ement. 7o sim!ly assume that this mass re!resents an infectious !rocess without obtainin$ tissue for !atholo$ic "ia$nosis woul" not be correct. Wor" catheter !lacement 'choice () woul" be acce!table in a youn$ woman with this lesion. In a woman ol"er than ?>, however, the lesion must be bio!sie" first.

D) (*!lanation: 7he correct answer is %. -ra$ile E syn"rome is the most common inherite" form of mental retar"ation. %own syn"rome may cause more absolute cases of mental retar"ation, but it results from a s!ontaneously occurrin$ trisomy an" most cases are not consi"ere" inherite". 7he $ene for fra$ile E syn"rome is locate" on the lon$ arm of the E4chromosome. It has a com!le* inheritance !attern that is relate" to the number of re!eatin$ cytosine4$uanine4$uanine tri!lets. When $reater than ;>> re!eats are !resent, a !erson will have the full mutation an" have !henoty!ic fra$ile E syn"rome. A !atient with />4;>> re!eats is !henoty!ically normal an" is sai" to have a !remutation. Alon$ with mental retar"ation, relate" features of the fra$ile E syn"rome inclu"e autistic behaviors, s!eech an" lan$ua$e !roblems, facial anomalies, an" macroorchi"ism in a"ult males. %NA4base" molecular analysis can be use" to "ia$nose fra$ile E syn"rome. 7his can be !erforme" on culture" amniocytes obtaine" at amniocentesis. Chorionic villus sam!lin$ is not consi"ere" to be reliable for the "ia$nosis of fra$ile E syn"rome because of "ifferent methylation !atterns in the tro!hoblast com!are" with the fetus. 7o state that there is nothin$ to offer this cou!le 'choice A) is incorrect. As "etaile" above, amniocentesis can be use" to allow for !renatal "ia$nosis of fra$ile E syn"rome. 7o offer testin$ of the !arents 'choice ) woul" not be correct. ase" on the history !rovi"e", the !arents have alrea"y been teste" an" are known to be carriers. 7heir concern at this !oint is whether the fetus will have fra$ile E syn"rome, an" that can only be "etermine" by $enetic analysis of the fetus. 7o offer +6I of the fetus 'choice C) woul" be incorrect. 7he "ia$nosis is ma"e base" on %NA4 base" molecular analysis an" not an ima$in$ stu"y. 7o offer termination of the !re$nancy 'choice () woul" be incorrect. 7his is a "esire" !re$nancy. 7his !atient is seekin$ !renatal "ia$nosis at this !oint, not termination. @) (*!lanation: 7he correct answer is . &atients with 5allmann syn"rome 'i.e., isolate" $ona"otro!in "eficiency or familial hy!o$ona"otro!ic hy!o$ona"ism) can !resent with !rimary amenorrhea. &rimary amenorrhea is "efine" as the absence of menses in a female by the a$e of :C. Associate" fin"in$s in 5allmann syn"rome may inclu"e anosmia or hy!osmia, color blin"ness, an" cleft li! or cleft !alate. 7hese fin"in$s are attributable to the fact that "urin$ embryo$enesis the .n62 neurons ori$inally "evelo! in the e!ithelium of the olfactory !laco"e an" normally mi$rate into the hy!othalamus. 7hus e*ists the link between the mi"line "efects an" the amenorrhea. &hysical e*amination may reveal absent to minimal breast "evelo!ment. 7reatment of the !atient with 5allmann syn"rome is with e*o$enous estro$en an" !ro$estin re!lacement thera!y. If !re$nancy is "esire", ovulation in"uction can be brou$ht about with the !ulsatile a"ministration of e*o$enous .n62. Anore*ia nervosa 'choice A) can cause amenorrhea an" a re"uction in breast si#e, but it is not associate" with anosmia, color blin"ness, an" cleft !alate. 7hese features are associate"

with 5allmann syn"rome. &olycystic ovarian syn"rome 'choice C) is characteri#e" by oli$omenorrhea, hirsutism, infertility, an" obesity. 7his !atient "oes not have a !resentation consistent with !olycystic ovarian syn"rome. &re$nancy 'choice %) shoul" always be the first thou$ht when a !otentially fertile woman !resents with amenorrhea. 2owever, this !atient has a ne$ative urine !re$nancy test an" no fin"in$s consistent with !re$nancy. 7esticular femini#ation syn"rome 'choice () re!resents com!lete an"ro$en insensitivity. 7his syn"rome occurs in in"ivi"uals with a ?C, E9 karyoty!e. Affecte" males have a female a!!earance with breast "evelo!ment. I) (*!lanation: 7he correct answer is A. 7his !atient has Chlamy"ia cervicitis. Chlamy"ia is the most common se*ually transmitte" bacterial or$anism in the Fnite" )tates. It is essential to "etect an" treat chlamy"ial infection "urin$ !re$nancy because maternal chlamy"ial infection is associate" with several com!lications of !re$nancy inclu"in$ !reterm !remature ru!ture of the membranes '&&6O+) an" !reterm labor. Chlamy"ial infection is also associate" with neonatal con<unctivitis, which results from the fetus !assin$ throu$h an infecte" birth canal. 7he !artner of the !atient must be treate" as well as the !atient herself in or"er to !revent reinfection. A test of cure '7OC) shoul" be !erforme" ? to C weeks after treatment is $iven to ensure that the or$anism has been com!letely era"icate" from the !atient an" her !artner or !artners. A#ithromycin has a !rolon$e" tissue half4life an" therefore it can treat chlamy"ia in a sin$le "ose. 7his sin$le "ose treatment allows far $reater com!liance than the multi!le "oses that are require" if erythromycin or amo*icillin is use". 7he sin$le "ose treatment with a#ithromycin also allows the treatment to be Hobserve"H 'i.e., the !atient can be watche" takin$ the me"ication). While the safety an" effectiveness of a#ithromycin "urin$ !re$nancy has not been as well !roven as that of erythromycin or amo*icillin, it is believe" to be safe an" its sin$le "ose quality makes it the "ru$ of choice. %o*ycycline 'choice ) is contrain"icate" "urin$ !re$nancy because of its effects on fetal teeth an" bone. ,evoflo*acin 'choice C) an" the other fluoroquinolones are contrain"icate" "urin$ !re$nancy because of their association with arthro!athies. &enicillin 'choice %) is safe "urin$ !re$nancy but is not consi"ere" to be as effective a$ainst Chlamy"ia as are a#ithromycin, erythromycin, an" amo*icillin. )tre!tomycin 'choice () is contrain"icate" "urin$ !re$nancy because of its relationshi! to ei$hth cranial nerve "ama$e. :>) (*!lanation: 7he correct answer is A. A !atient with can"i"iasis classically !resents with com!laints of a thick, white, Hcotta$e cheese4likeH "ischar$e. )uch !atient may also com!lain of vulvar !ruritus an" burnin$. %ysuria is often seen in cases of can"i"iasis because there is !ain when the aci"ic urine comes in contact with the inflame" va$inal mucosa. 7his "ysuria is often confuse" for a urinary tract infection. 7he keys to "istin$uishin$ between the two are the

e*amination an" laboratory stu"ies. (*amination on a !atient with can"i"iasis often shows a thick, white, "ischar$e as well as erythema of the va$ina an" vulva, as this !atient has. 7he e*coriations that this !atient has are likely !resent because the !atient has been scratchin$ the area. In a urinary tract infection, e*amination of the va$ina an" vulva will most often be unremarkable. 7he 5O2 !re!aration will "emonstrate !seu"o4 hy!hae in cases of can"i"iasis. 7he urinalysis shoul" be ne$ative in cases of can"i"iasis, althou$h if there is contamination of the sam!le, abnormalities may be seen. A hemorrha$ic ovarian cyst 'choice ) ty!ically causes ab"ominal !ain an" ten"erness. It usually "oes not cause burnin$ with urination. 7he "ia$nosis of !elvic inflammatory "isease 'choice C) is ma"e when a se*ually active female has ab"ominal ten"erness, cervical motion ten"erness, an" a"ne*al ten"erness alon$ with a fever, an elevate" white bloo" cell count, a !ositive $onorrhea or Chlamy"ia test, or a muco!urulent cervical "ischar$e. 7his !atient "oes not have these fin"in$s. 7his !atient "oes have a history of !yelone!hritis 'choice %) an" therefore, !yelone!hritis an" urinary tract infection 'choice () woul" be consi"erations. 2owever, the !atient has no fever, costovertebral an$le ten"erness, or abnormal urinalysis, the three fin"in$s most hel!ful for the "ia$nosis of !yelone!hritis. Also, while "ysuria can often be a sym!tom of a urinary tract infection, this !atientAs ne$ative urinalysis an" fin"in$s consistent with another !rocess 'namely can"i"iasis) make F7I less likely. ::) (*!lanation: 7he correct answer is %. 7he &a! test is an e*cellent metho" of screenin$ for cervical cancer. It has a relatively low4cost an" is noninvasive an" effective. Fse of the &a! for screenin$ over the !ast /> years has resulte" in a D>= "ecrease in the mortality from cervical cancer. 2owever, the test is not without its flaws. 7he !rimary "rawback of the test is its hi$h false4ne$ative rate. In the case of the &a!, a false4ne$ative is a woman who has abnormal cervical cells but is "eclare" to have a normal &a! smear. 7hese false4ne$ative results can be cause" by any of the ste!s in the !rocess, inclu"in$ errors in sam!lin$, !re!aration, screenin$, an" inter!retation. 7he lar$er the number of false4ne$ative results, the lower is the sensitivity of a test. )ensitivity of a test is calculate" by "ivi"in$ the number of !atients who have the "isease an" test !ositive for the "isease by the total number of !atients that have the "isease. In the above e*am!le, four women who have abnormal cervical cells will test !ositive for abnormal cervical cells. 7his number '?) shoul" then be "ivi"e" by the total number of women with truly abnormal cells '/). 7his $ives a result of ?B/ or >.@ or @>=. 7o state that the sensitivity of the &a! test is >= 'choice A) or := 'choice ) is incorrect. If this were the case, it woul" mean that the &a! test woul" correctly i"entify none or only : of every :>> women with truly abnormal cervical cytolo$y. 7his woul" make the &a! test a very !oor or com!letely meanin$less screenin$ test. A screenin$ test that is ;>= 'choice C) sensitive is also a very !oor screenin$ test. A screenin$ test shoul" i"eally have hi$h sensitivity an" s!ecificity. A test that is has only ;>= sensitivity woul" i"entify only ;> of every :>> women with a $iven "isease. 7his woul" make it a very !oor screenin$ test. A screenin$ test that has :>>= 'choice () sensitivity for a "isease woul" be i"eal. If the &a! test were :>>= sensitive, it woul" mean that every woman

with abnormal cervical cells woul" be correctly i"entifie". 2owever, this is not the case because of the !ossible errors that were "escribe" above. Also, efforts to increase the sensitivity of a screenin$ test often lea" to a loss of s!ecificity an" an increases in the !ercenta$e of false !ositives. :;) (*!lanation: 7he correct answer is C. Women that are 6h ne$ative are at risk for "evelo!in$ 6h isoimmuni#ation. 6h isoimmuni#ation occurs when an 6h4ne$ative mother becomes e*!ose" to the 6h anti$en on the re" bloo" cells of an 6h4!ositive fetus. 7his e*!osure may lea" the motherAs immune system to become sensiti#e" to the 6h anti$en such that in a future !re$nancy with an 6h4!ositive fetus, the motherAs immune system may HattackH the 6h anti$en on the fetal re" bloo" cells. 7his immune res!onse may lea" to the "evelo!ment of fetal anemia, hy"ro!s, an" "eath. 7o !revent 6h isoimmuni#ation from occurrin$, 6h4ne$ative women who are not 6h alloimmuni#e" shoul" receive 6ho.A+ 'anti4% immune $lobulin) at ;@ weeks of $estation, within D; hours after the birth of an 6h4!ositive infant, after a s!ontaneous abortion, or after invasive !roce"ures such as amniocentesis. 6ho.A+ shoul" also be stron$ly consi"ere" in cases of threatene" abortion, antenatal blee"in$, e*ternal ce!halic version, or ab"ominal trauma. 7he amount that is usually $iven after the "elivery of an 6h4!ositive fetus is 0>> R$. 7his amount is sufficient to cover a fetal to maternal hemorrha$e of 0> m, 'or :/ m, of fetal cells). 2owever, some women will have a fetal to maternal hemorrha$e that is in e*cess of this 0> m,4es!ecially in cases such as manual removal of the !lacenta or !lacental abru!tion 'as this !atient ha"). 7o "etermine the amount of fetal to maternal hemorrha$e that occurre", it is necessary to !erform a 5leihauer4 etke test which is an aci"4"ilution !roce"ure that allows fetal re" bloo" cells to be i"entifie" an" counte". 5nowin$ the amount of fetal to maternal hemorrha$e that took !lace allows the correct amount of 6ho.A+ to be $iven. An a!t test 'choice A) is use" to "ifferentiate fetal from maternal bloo". It can be use" in the "ia$nosis of vasa !revia or with neonatal melena. A com!lete bloo" count 'choice ) will "emonstrate the amount of maternal hemorrha$e, but not the amount of fetal to maternal hemorrha$e. &artial thrombo!lastin time 'choice %), an" serum !otassium 'choice () "o not allow for the "etermination of the amount of fetal to maternal hemorrha$e. :0) (*!lanation: 7he correct answer is C. +e!eri"ine can be use" as a systemic anal$esic "urin$ labor. It is an o!ioi" an" rea"ily crosses the !lacentaG therefore, the fetus is e*!ose" to the me"ication. As an o!ioi", it causes res!iratory "e!ression. Neonates are at $reatest risk for res!iratory "e!ression when "elivery occurs a!!ro*imately ; to 0 hours after me!eri"ine is a"ministere" to the mother. 7his neonate was born a!!ro*imately ; hours after maternal a"ministration of me!eri"ine, which makes neonatal res!iratory "e!ression likely.

Nalo*one is a !ure o!ioi" anta$onist that "is!laces the o!ioi" from its rece!tor sites an" can hel! to reverse the o!ioi"4in"uce" res!iratory "e!ression. It has a short "uration of action so re!eat "oses may be necessary. loo" transfusion 'choice A) woul" not be in"icate". loo" transfusions are use" when there is evi"ence that the neonate is anemic. 7his neonate a!!ears to have res!iratory "e!ression an" not anemia. 7herefore, nalo*one, an" not bloo" transfusion, woul" be in"icate". .lucose 'choice ) shoul" be $iven when there is evi"ence that the neonate is severely hy!o$lycemic. 7his neonate, $iven that its mother receive" an o!ioi" ; hours a$o, is most likely to have res!iratory "e!ression from the o!ioi" an" not hy!o$lycemia. &enicillin 'choice %) is an antibiotic that may be $iven when there is evi"ence of infection. 7his neonate has a !resentation that is most consistent with o!ioi"4in"uce" res!iratory "e!ression an" not infection. )o"ium bicarbonate 'choice () shoul" be $iven to a neonate for "ocumente" metabolic aci"osis. It is often use" "urin$ a !rolon$e" resuscitation. 7he first ste! for this neonate, however, woul" be to try to reverse the res!iratory "e!ression with nalo*one. :?) (*!lanation: 7he correct answer is C. )ome estimates in"icate that rou$hly />= of Americans use some forms of com!lementary an" alternative me"icine 'CA+). 7he cate$ories of these inclu"e min"4 bo"y interventions, such as yo$a, alternative systems of me"ical !ractice such as Chinese me"icine, !harmacolo$ic treatments such as me"icinal !lants, herbal me"icine such as )t. NohnAs wort, "iet thera!ies such as ve$etarianism, manual healin$ metho"s such as massa$e, an" bioelectroma$netic a!!lications such as ma$nets for musculoskeletal !ain. It is essential for the !hysician to work with the !atient re$ar"in$ the use of CA+. 7he first ste! is to fin" out which metho"s the !atient uses. 7his !atient has tol" the !hysician that she uses alternative me"ications. +any !atients "o not offer this information, assumin$ that the usual !hysician will not su!!ort CA+. It is therefore im!ortant to ask the !atient whether she is usin$, or consi"erin$ usin$, CA+. ecause the fiel" of CA+ is so broa", it is essential to ask which ty!es of CA+ the !atient uses. One cannot assume that all alternative thera!ies are equivalent. 7hus, the most a!!ro!riate question to ask this !atient is HWhich alternative me"ications "o you useJH 7his is a non4threatenin$ question that will allow her to further "etail her use. 7o ask, H%oes your husban" know you are usin$ these alternative me"icationsJH 'choice A) is ina!!ro!riate. 7he !hysicianAs role is to care for the !atient. Whether the !atient reveals her use of alternative me"icines to her husban" is not the !rime concern to the !hysician. 7his question is more likely to create conflict than reveal nee"e" information for the !hysician. 7o ask, H%o you reali#e how "an$erous alternative me"icines areJH 'choice ) is incorrect. 7his question is confrontational an" <u"$mental. +any alternative thera!ies are safe an" effective. 7o ask, HWhy "onAt you stick with tra"itional me"icinesJH 'choice %) is ina!!ro!riate. If a !atient has a con"ition an" there is a reme"y from the conventional me"ical system 'known as allo!athy in North America) available, then it is reasonable to offer this reme"y as a !ossibility for the !atient. 2owever, inquirin$ as to why the !atient "oesnAt HstickH with tra"itional me"icine is likely to cause confrontation an" a worsenin$ of the !atient4"octor relationshi!. 7o ask, HWhy havenAt you reveale" your use of alternative me"ications

beforeJH 'choice () is also somewhat challen$in$ an" confrontational. &erha!s the !atient "i" not think a conventional !hysician woul" be acce!tin$ of CA+. 7he im!ortant ste! at this !oint is to i"entify the me"ications an" "iscuss their risks, benefits, an" si"e effects with the !atient, as one woul" with tra"itional me"ications. :/) (*!lanation: 7he correct answer is %. )e*ual "evelo!ment is variable from woman to woman, althou$h there are certain ran$es of normal. 7helarche, also known as breast4bu""in$, is usually the first si$n of secon"ary se*ual "evelo!ment an" this occurs on avera$e between the a$es of @ an" :> years of a$e. 7he $rowth s!urt usually follows after breast bu""in$ an" menarche is one of the last sta$es, occurrin$, on avera$e, between the a$es of :; an" :0. 7his youn$ woman be$an "evelo!in$ breasts at a$e :>, which is entirely a!!ro!riate. 7hat she has not ha" her first menstrual !erio" yet is not abnormal. 7he $eneral rule is that evaluation for "elaye" se*ual "evelo!ment shoul" be starte" if there is no breast "evelo!ment by the a$e of :0 or menses by the a$e of :/. 7his $eneral rule can be a"a!te" to fit the circumstances if there are tem!o or sequence abnormalities. In this case, with breast "evelo!ment occurrin$ normally an" "evelo!ment a!!earin$ to !rocee" in a stan"ar" fashion, the mother can be reassure" an" evaluation for late menses "elaye" until a$e :/, if it has not come before that time. 7o state that breast "evelo!ment at a$e :> is abnormally early 'choice A) or that breast "evelo!ment at a$e :> is abnormally late 'choice ) is incorrect. In North America, thelarche occurs, on avera$e, between a$es @ to :>. 7herefore, this youn$ woman, who be$an breast "evelo!ment at a$e :>, falls into the normal ran$e. 7o state that evaluation for late menses shoul" be starte" imme"iately 'choice C) is incorrect. 7he $eneral rule for evaluation of "elaye" !uberty is that evaluation shoul" take !lace if thelarche has not occurre" by a$e :0 or menarche by a$e :/. 7hese a$es re!resent rou$hly a ;./ stan"ar" "eviation from the mean an" therefore warrant evaluation. 7o tell this mother that her chil"As se*ual "evelo!ment is none of her business 'choice () woul" not be a!!ro!riate. While there are certain se*ual issues where confi"ential "iscussion with an a"olescent is a!!ro!riate, concerns re$ar"in$ "elaye" se*ual "evelo!ment are a!!ro!riate issues for a !arent to be concerne" with. :C) (*!lanation: 7he correct answer is %. &elvic inflammatory "isease rarely occurs "urin$ !re$nancy. 7he inci"ence of &I% "urin$ !re$nancy is far lower than in the non!re$nant state, likely because of the relative !rotection that the !re$nancy !rovi"es a$ainst ascen"in$ infection from the cervi*. 2owever, while &I% "urin$ !re$nancy is rare, it is not im!ossible. 7his !atient has the fin"in$s that are most consistent with a "ia$nosis of &I %. )he has ab"ominal ten"erness, cervical motion ten"erness, an" a"ne*al ten"erness. )he also has a fever, a muco!urulent cervical "ischar$e, an" an elevate" white bloo" cell count. )he also has a history of $onorrhea an" chlamy"ia. 7he correct mana$ement of a

!re$nant woman with &I% is hos!ital a"mission an" treatment with intravenous me"ications. Clin"amycin an" $entamicin shoul" be use". 7o state that no treatment is necessary 'choice A) is absolutely incorrect. A !atient with &I% certainly nee"s treatment. An" a !re$nant !atient with &I% requires hos!itali#ation an" intravenous antibiotics. 7o $ive intramuscular ceftria*one, oral "o*ycycline, an" "ischar$e home 'choice ) woul" not be a!!ro!riate. Non!re$nant !atients that "evelo! &I% may be treate" with intramuscular ceftria*one an" an e*ten"e" course of "o*ycycline ':? "ays). 7his is a stan"ar" out!atient treatment for &I %. A !re$nant !atient, however, must be a"mitte" to the hos!ital for intravenous antibiotics. 7o !rovi"e intravenous cefotetan an" "o*ycycline an" hos!ital a"mission 'choice C) is incorrect. %o*ycycline is a class % "ru$ that shoul" not be use" "urin$ !re$nancy. ,a!arosco!y 'choice () woul" not be the most a!!ro!riate ne*t ste! in mana$ement. 7he "ia$nosis of &I% in this !atientAs case is reasonably certain $iven the !resentation. 7he ne*t ste!, therefore, is treatment with intravenous antibiotics. If these fail, sur$ical alternatives may be consi"ere". :D) (*!lanation: 7he correct answer is %. Numerous stu"ies have "emonstrate" that use of the oral contrace!tive !ill si$nificantly "ecreases a womanAs likelihoo" of "evelo!in$ en"ometrial cancer. Overall, use of the oral contrace!tive !ill a!!ears to "ecrease the risk by a!!ro*imately />=, with $reatest effects in those usin$ the !ill for more than 0 years. One theory to e*!lain the "ecrease" en"ometrial cancer risk in oral contrace!tive users is that the oral contrace!tive !ill !rovi"es almost continuous e*!osure of the en"ometrium to !ro$estins. 7he ma<or factor in the "evelo!ment of en"ometrial cancer is estro$en e*!osure, whether en"o$enously 'e.$., "ue to obesity or chronic anovulation) or e*o$enously 'e.$., from uno!!ose" estro$en re!lacement thera!y). y !rovi"in$ almost "aily e*!osure to !ro$estins, the oral contrace!tive !ill works to counteract the effects of estro$ens. Over time, women on the OC& "evelo! thinner en"ometrial linin$s an" have a lower risk of "evelo!in$ en"ometrial cancer. 7here is no clear relationshi! between bone cancer 'choice A) an" OC& use. 7he relationshi! between breast cancer 'choice ) an" oral contrace!tives remains unclear at this time. 7here is some evi"ence that current users an" those who have recently sto!!e" may be at some increase" risk of breast cancer. 2owever, there is also evi"ence that when breast cancer is "ia$nose" in an oral contrace!tive user, it ten"s to be more locali#e" than in a nonuser. 7he relationshi! between cervical cancer 'choice C) an" the OC& also remains unclear at this time. Overall the results have been inconclusive. All se*ually active !atients shoul" have re$ular screenin$ for cervical "ys!lasia with a &a! smear startin$ at a$e :@ or with the onset of se*ual intercourse. 7he OC& "oes not !rotect a$ainst liver cancer 'choice (). 7he OC& is believe" to increase the risk of certain beni$n liver tumors. :@) (*!lanation:

7he correct answer is A. 7his !atient has a com!oun" !resentation, which ha!!ens when an e*tremity !rola!ses alon$si"e the fetal !resentin$ !art. In this case, the com!oun" !resentation is the fetal verte* alon$ with a fetal arm. Com!oun" !resentation occurs in a!!ro*imately : in :>>> "eliveries an" is brou$ht about when the !elvic inlet is not com!letely occlu"e" by the fetal hea". +ost often this occurs with !remature fetuses. A com!oun" !resentation can be allowe" to un"er$o a normal labor an" "elivery. 7he !rola!se" arm shoul" be left alone, as it will not interfere with the labor an" "elivery in most cases. Often the arm will rise out the way as the verte* "escen"s further. O*ytocin au$mentation 'choice ) woul" not be a!!ro!riate mana$ement. 7his !atient is in active labor, with !ainful contractions every ; minutes an" ? cm of cervical "ilation. O*ytocin is use" in cases in which there is a nee" to au$ment labor 'e.$., when contractions are not a"equate) or to in"uce labor 'e.$., when there are no contractions !resent.) 7his !atient has a"equate contractions. -orce!s "elivery 'choice C) is not in"icate" at this !oint. 7he !atientAs cervi* is not fully "ilate", an" the !resence of the fetal arm, if it !ersists, woul" !revent !ro!er a!!lication of the force!s. 1acuum "elivery 'choice %) woul" not be a!!ro!riate. As with force!s, vacuum is not use" unless the cervi* is fully "ilate" an" the verte* is at M; to M0 station. 7his !atient is only ? cm "ilate", an" the verte* is at > station. 7here is no fetal or maternal in"ication at this !oint for vacuum "elivery. Cesarean "elivery 'choice () is not in"icate". As note" above, most women with a com!oun" !resentation, with a han" !resentin$ by the fetal hea", can un"er$o a normal labor an" "elivery. :I) (*!lanation: 7he correct answer is A. )everal stu"ies have establishe" a relationshi! between folic aci" an" the !revention of neural tube "efects. 7he !resence of a"equate levels of maternal folate a!!ears to !lay a role in the correct "evelo!ment an" closure of the neural tube. ase" on these stu"ies, in :II;, the Fnite" )tates Centers for %isease Control recommen"e" that all women of chil"4bearin$ a$e shoul" consume >.? m$B"ay of folic aci" startin$ !reconce!tionally an" continuin$ for the first 0 months of !re$nancy. Women who have alrea"y ha" a chil" with a neural tube "efect, however, fall into a "ifferent cate$ory. 7his !atient ha" a chil" with s!ina bifi"a ; years a$o. -or a woman such as this, the recommen"ation is that ?.> m$ of folic aci" be taken "aily, startin$ one month before the !lanne" time of conce!tion an" continuin$ on for the first 0 months of !re$nancy. It is believe" that this level of su!!lementation will "ecrease the risk of havin$ another chil" with a neural tube "efect by C> to D>=. 7o recommen" folic aci", ? m$B"ay startin$ in the first trimester 'choice ) woul" be incorrect. 7his !atient, because she has ha" a !rior chil" with a neural tube "efect, shoul" in"ee" be takin$ ? m$B"ay "urin$ the first 0 months of !re$nancy. 2owever, she shoul"nAt start when she is !re$nant, rather, she shoul" be takin$ this level of folic aci" su!!lementation startin$ !reconce!tionally. It is im!ortant that the !re$nant womanAs folate stores are bein$ su!!lemente" !rior to the time of conce!tion. 7o recommen" vitamin A, :>,>>> IFB"ay startin$ !reconce!tionally 'choice C) or vitamin A, :>,>>> IFB"ay startin$ in the first trimester 'choice %) woul" be incorrect. -irst, vitamin

A "eficiency is very rare in the Fnite" )tates. )econ", vitamin A su!!lementation with levels of :>,>>> IFB"ay an" above has been associate" with birth "efects. )u!!lements taken by !re$nant women shoul" contain /,>>> IFB"ay or less. 7o state that no su!!lements are nee"e" 'choice () is incorrect. 7his !atient has a !revious chil" with a neural tube "efect. )he shoul" therefore take ?.> m$ of folic aci"B"ay startin$ one month before conce!tion an" continuin$ throu$h the first 0 months of !re$nancy to hel! !revent havin$ another chil" with a neural tube "efect. ;>) (*!lanation: 7he correct answer is A. 7his !atient has fin"in$s that are most consistent with a lower urinary tract infection. A lower urinary tract infection refers to infection of the bla""er 'cystitis) or urethra 'urethritis). 7he !rinci!al com!laints for women with lower urinary tract infections are "ysuria, ur$ency, an" frequency. +ost often e*amination will be unremarkable. Occasionally, su!ra!ubic ten"erness may be !resent. A urinalysis will often reveal a !ositive leukocyte esterase or nitrite test. 7he microsco!ic analysis will show white bloo" cells. 7he most si$nificant risk factors are relate" to se*ual activity an" hy!oestro$enism. 7hese factors lea" to invasion by !atho$enic or$anisms. (. coli is by far the most common causative or$anism in cases of acute uncom!licate" cystitis. It is res!onsible for a!!ro*imately @>= of these cases. N. $onorrhoeae'choice ) is often associate" with cervicitis an" !elvic inflammatory "isease. 9et, it can also cause urethritis. 2owever, N. $onorrhoeae is a far less frequent cause of acute uncom!licate" cystitis than (. coli. &seu"omonas s!ecies 'choice C) can cause urinary tract infections. It is often seen in !atients with metabolic or anatomic abnormalities. In a routine case of F7I, however, it is not the most common !atho$en. )ta!hylococcus sa!ro!hyticus'choice %) is a somewhat common cause of acute, uncom!licate" F7Is. It accounts for a!!ro*imately :>= of cases. 7richomonas va$inalis'choice () is an or$anism that is most often associate" with va$initis, but can also cause a urethritis. 7his !atient, however, has a ne$ative normal saline Hwet !re!.H &atients with trichomoniasis usually have visible or$anisms on the Hwet !re!.H Also, while 7richomonas va$inalis can cause urethritis, it is not nearly as common a cause as is (. coli. ;:) (*!lanation: 7he correct answer is . &rimary amenorrhea is "efine" as the lack of s!ontaneous uterine blee"in$ by the a$e of :C. )econ"ary amenorrhea is "efine" as the absence of a menstrual !erio" for C months or more in a woman who !reviously ha" normal !erio"s or the absence of menses for :; months or more in women with !reviously irre$ular menstrual !erio"s. 7his !atient, $iven that she !reviously ha" normal menstrual !erio"s, has secon"ary amenorrhea. 7he most common cause of misse" menses in !reviously cyclin$ women is

!re$nancy. 7herefore, it is absolutely essential that a !re$nancy test be !erforme" on any woman with this com!laint. 2y!er!rolactinemia is the cause of amenorrhea in :> to ;>= of cases, so it is also im!ortant that a !rolactin level be checke". An", because thyroi" "ysfunction can also cause a loss of menses, a 7)2 shoul" also be checke". 7his !atient, however, is not !re$nant an" has normal 7)2 an" !rolactin levels. At this !oint, some !hysicians woul" !erform a !ro$esterone with"rawal test. 7his consists of $ivin$ a woman an intramuscular in<ection of !ro$esterone or oral !ro$esterone for / to :> "ays an" then checkin$ to see if the !atient has with"rawal menstrual blee"in$. If with"rawal blee"in$ occurs within D "ays, then !atients are assume" to have a"equate levels of en"o$enous estro$en !ro"uction. +ost !atients with amenorrhea, a"equate en"o$enous estro$en !ro"uction, an" with"rawal blee"in$ after the a"ministration of !ro$estins will have some form of !olycystic ovarian syn"rome '&CO)). Asherman syn"rome 'choice A) "escribes the con"ition in which menstrual !erio"s "o not occur because the uterine cavity has become obliterate" with a"hesions. 7hese a"hesions result from trauma to the basal level of the en"ometrium, most often occurrin$ at the time of "ilation an" curetta$e. &atients with this syn"rome woul" not be e*!ecte" to have menses in res!onse to !ro$esterone. (n"ometrial carcinoma 'choice C) ty!ically !resents with heavy, irre$ular blee"in$ or as !ostmeno!ausal blee"in$. +eno!ause 'choice %) re!resents the loss of menstrual !erio"s as ovarian function "ecreases. &ostmeno!ausal !atients woul" not be e*!ecte" to have with"rawal menses after !ro$esterone e*!osure. 7his !atientAs blee"in$ "oes not !rovi"e $oo" evi"ence of !re$nancy 'choice (). 2er ne$ative urine hC. an" with"rawal blee"in$ after !ro$esterone make it e*tremely unlikely that she is !re$nant. ;;) (*!lanation: 7he correct answer is (. oth force!s an" the vacuum e*tractor can be use" to e*!e"ite the "elivery of a fetus. 7hese instruments are most often use" when there are fetal in"ications, such as a non4 reassurin$ fetal heart rate tracin$, or maternal in"ications, such as maternal e*haustion or maternal contrain"ications to !ushin$ 'such as maternal car"iac "isease.) 7he choice of force!s or vacuum "e!en"s most on the e*!erience an" !reference of the !hysician. In certain cases, one instrument is favore" or man"atory. -or e*am!le, force!s may be use" in face !resentation with a mentum anterior !resentationG in such a case, vacuum is contrain"icate". 7hose who favor vacuum "elivery make several ar$uments. -or e*am!le, as o!!ose" to force!s, the vacuum e*tractor "oes not occu!y s!ace ne*t to the fetal hea"G this shoul" lea" to less trauma to maternal tissues. Also, attem!te" "elivery with the vacuum in a situation of true ce!halo!elvic "is!ro!ortion 'i.e., the fetus cannot be "elivere" throu$h the maternal !elvis) will lea" to a loss of suction an" failure of the !roce"ureG force!s "o not necessarily "islo"$e an" this coul" lea" to continue" efforts bein$ ma"e with increase" likelihoo" of maternal or fetal morbi"ity or mortality. 7o state that the vacuum can be use" at hi$her stations 'choice A) is incorrect. oth the vacuum an" force!s shoul" !referably be use" only in low4 or outlet4 situations 'i.e., with the fetal verte* at M; station or lower.) 7o state that the vacuum can be use" for fetuses in breech !resentation 'choice ) is incorrect. Neither the vacuum nor force!s shoul" be use" when the fetus is !resentin$ as a breech. 7o state that vacuum can be use" in face !resentations

'choice C) is not correct. 1acuum cannot be use" when the fetus is !resentin$ face first. -orce!s may be use" as lon$ as the fetus is in mentum4anterior !osition 'i.e., with the chin facin$ towar" the maternal !ubic sym!hysis.) 7o state that the vacuum can be use" with intact membranes 'choice %) is incorrect. Neither force!s nor vacuum shoul" be use" with intact membranes. ;0) (*!lanation: 7he correct answer is . 7he term $estational tro!hoblastic "isease encom!asses a number of relate" "iseases ori$inatin$ from the !lacenta. 7hese "iseases inclu"e com!lete an" !artial hy"ati"iform moles, invasive moles, !lacental site tro!hoblastic tumors, an" choriocarcinomas. 7his !atient !resents with fin"in$s consistent with a com!lete mole. 7he most common sym!tom is va$inal blee"in$ an" e*amination often "emonstrates a uterus that is lar$er than e*!ecte" for $estational "ates. ,aboratory evaluation often shows a si$nificantly elevate" beta4hC. an" ultrasoun" reveals the absence of a fetus an" the !resence of a HsnowstormH !attern with multi!le echo$enic areas of villi an" clots. 7reatment is with "ilation an" evacuation of the mole. Once there is !atholo$ic confirmation of the "ia$nosis, it is essential that the !atient continue" to be followe" weekly until the beta4 hC. value returns to >. 7he !atient shoul" then be followe" monthly for an a""itional year to ensure that the values stay at > an" that there is no evi"ence of !ersistent or metastatic "isease. (valuation in one year 'choice A) woul" not be a!!ro!riate. 7his !atient may have mali$nant $estational tro!hoblastic "isease, in which case the beta4hC. values will remain elevate" an" not return to > after the evacuation. 7o !ost!one further evaluation for one year risks a si$nificant "elay in "ia$nosis an" mana$ement of !ersistent or mali$nant "isease. %actinomycin 'choice C) is often use" as an alternative thera!y to methotre*ate in !atients with mali$nant $estational tro!hoblastic "isease. As lon$ as this !atientAs beta4hC. values fall to > a!!ro!riately an" stay at >, there is no nee" to treat with %actinomycin. +ethotre*ate 'choice %) is use" as the first4line a$ent in !atients with mali$nant tro!hoblastic "isease. A$ain, there will be no nee" for chemothera!y in this !atient as lon$ as the beta4hC. values fall to > an" stay at >. 2ysterectomy 'choice () woul" not be in"icate" in a ;04year4ol" !atient with beni$n $estational tro!hoblastic "isease who "esires future fertility. ;?) (*!lanation: 7he correct answer is (. 7his !atient has a !resentation that is consistent with !rimary sy!hilis. )y!hilis is cause" by the or$anism 7re!onema !alli"um, which is a hi$hly conta$ious s!irochete. 7he incubation !erio" for the or$anism is anywhere from :> to I> "ays, after which a chancre, which is a raise", !ainless ulcer, will a!!ear. 7. !alli"um cannot be culture", but it can be i"entifie" with "arkfiel" microsco!y or fluorescent antibo"y stainin$ from obvious lesions. )erolo$ic tests can also be use", such as the 6&6 an" 1%6, tests, which are not s!ecific for 7. !alli"um infection an" may be !ositive in !atients with colla$en

vascular "isease, intravenous "ru$ abuse, bacterial an" viral infections, a history of bloo" transfusions, an" even !re$nancy. ecause the 6&6 an" 1%6, are not s!ecific, a tre!onemal s!ecific assay such as the -7A4A ) or +2A47& shoul" also be use" for confirmation. When these are !ositive an" the !atient has no history of treatment, it is absolutely essential that treatment be $iven because sy!hilis in !re$nancy is associate" with a number of com!lications inclu"in$ fetal "emise, IF.6, !reterm "elivery, an" con$enital infection. 7reatment "urin$ !re$nancy must be with !enicillin as no other "ru$ !ermits safe an" effective treatment of the fetus as well as the mother. In a !atient who is aller$ic to !enicillin, oral "esensiti#ation must be !erforme" first in a hos!ital settin$ with a!!ro!riate facilities. 7o a"minister erythromycin 'choice A), levoflo*acin 'choice ), or metroni"a#ole 'choice C) woul" not be !ro!er mana$ement. 7hese are not "ru$s that will effectively treat sy!hilis in !re$nancy. -urthermore, levoflo*acin is contrain"icate" "urin$ !re$nancy, as are all fluoroquinolones, because of the !ossible relationshi! between maternal use an" arthro!athies in the offs!rin$. 7o a"minister tetracycline 'choice %) woul" be a!!ro!riate in the non4!re$nant !atient with sy!hilis who is aller$ic to !enicillin. 7etracycline is consi"ere" a reasonable alternative in that situation. 2owever, in the !re$nant !atient, tetracycline cannot be use" because of effects on the fetal teeth an" bones. Only !enicillin is consi"ere" a"equate for the treatment of sy!hilis in !re$nancy. ;/) (*!lanation: 7he correct answer is A. 7his !atient has a classic !resentation of her!es $enitalis, a venereal "isease cause" by her!es sim!le* virus ty!e II 'I>= of cases) or ty!e I ':>=). Initial infection usually results in $enerali#e" illness inclu"in$ malaise, myal$ias, an" low4$ra"e fever alon$ with the !erineal lesions. 7hese lesions start out as clear vesicles that !ro$ress to ulcers over the followin$ "ays. 7he ulcers may then coalesce to form a lar$er, shallow, !ainful ulcer. After the initial infection, the virus resi"es in the "orsal root sacral $an$lia. -rom there it is !erio"ically reactivate". 6ecurrent e!iso"es are characteri#e" by a !ro"rome of tin$lin$, burnin$, or itchin$ !rior to the a!!earance of the lesions. 7here is no HcureH for her!es $enitalis. Acyclovir can be use" to shorten the "uration of sym!toms. In !atients who have more than C outbreaks !er year, "aily oral acyclovir is recommen"e" to !revent these frequent outbreaks. %aily oral estro$en 'choice ) or "aily to!ical estro$en 'choice C) woul" not be a!!ro!riate !harmacothera!y for these outbreaks. (stro$en 'oral an" to!ical) is use" for !atients with atro!hic va$initis. Atro!hic va$initis is characteri#e" by !ale va$inal mucosa with a loss of ru$ae. It is associate" with estro$en "eficient states such as meno!ause. 7his !atient has no evi"ence of estro$en "eficiency an" therefore estro$en woul" not be recommen"e". %aily oral ferrous sulfate 'choice %) is a!!ro!riate !harmacothera!y for !atients with iron4"eficiency anemia. )ufficient iron stores are necessary for effective erythro!oiesis. 7here is no evi"ence that this !atient is iron "eficient an" the most a!!ro!riate !harmacothera!y to !revent recurrent her!es outbreaks is acyclovir, not ferrous sulfate. %aily oral !enicillin 'choice () woul" not be a!!ro!riate !harmacothera!y for this !atient. 7his !atient has her!es $enitalis an" not a bacterial infection. 7hus, acyclovir, an" not !enicillin, woul" be in"icate".

;C) (*!lanation: 7he correct answer is . Corticosteroi"s are known to lea" to more "ifficult $lucose control in "iabetic women. 7o ensure that these !atients "o not "evelo! "iabetic ketoaci"osis, bloo" $lucose levels shoul" be checke" re$ularly, an" elevate" values treate" with insulin. 7his will often require hos!itali#ation, which is usually require" by the con"ition for which they receive" the corticosteroi"s in the first !lace 'e.$., !reterm labor or !reterm !remature ru!ture of membranes). In !atients who "o not have "iabetes, the hy!er$lycemic effect will last ;40 "ays. )tu"ies have been !erforme" to "etermine whether antenatal treatment with corticosteroi"s lea"s to "ecrease" chil"hoo" intelli$ence 'choice A). 7here is no evi"ence that this relationshi! e*ists. ecause of the immunosu!!ressive !ro!erties of corticosteroi"s, there has been concern that their use may increase rates of maternal infection 'choice C) or neonatal infection 'choice (). 7here is no "efinitive !roof that corticosteroi" use lea"s to hi$her rates of infection in either the mother or fetus. An", althou$h there may be some instances of maternal or neonatal infection in some cases of corticosteroi" a"ministration, the increase" maternal insulin requirement occurs almost without e*ce!tion. Neonatal a"renal su!!ression 'choice %) has not been !roven to result from antenatal corticosteroi" a"ministration.

USMLE Step 2 Practice Test Block 24 :ame;

+nstr#ctions; Ans'er the *#estions $elo' to the $est of yo#r a$ility. =hen yo# finish the test% click the 2heck $#tton at the $ottom to )ie' the res#lts.

1.Ten "ays after #n"ergoing li)er transplantation% a patient,s le)els of gamma!gl#tamyl transferase 3CCT6% alkaline phosphatase% an" $ilir#$in $egin to rise. =hich of the follo'ing is the most appropriate ne.t step in "iagnosisa6Meas#rement of preforme" anti$o"y le)els $6Ultraso#n" of $iliary tract an" @oppler st#"ies of the anastomose" )essels c6Li)er $iopsy an" "etermination of portal press#res "6 Li)er $iopsy an" more "etaile" li)er f#nction tests e6Li)er $iopsy an" trial of steroi" $ol#ses :ormal La$s 2.A pre)io#sly healthy% into.icate"% 18!year!ol" man is "ri)ing a car 'itho#t #sing a seat $elt. /e crashes the car into the $ack of a parke" tr#ck. +n the process he slams his a$"omen into the steering 'heel an" r#pt#res his spleen. =hich of the follo'ing is the most important pro$lem associate" 'ith this type of in9#rya6 Bacteremia $6Electrolyte a$normalities c6E.ternal $loo" loss "6+nternal $loo" loss e6 Peritonitis :ormal La$s &. After s#ita$le calc#lations ha)e $een ma"e #sing the mo"ifie" Parklan" form#la% a (4!kg man 'ith e.tensi)e thir"!"egree $#rns is recei)ing Ainger,s lactate at the calc#late" rate% 'hich happens to $e ( 4 mL7hr. The inf#sion 'as starte" 'ithin &4 min#tes of the time 'hen the $#rn occ#rre". >)er the ne.t & ho#rs% his #rinary o#tp#t is recor"e" as 1 mL% 22 mL% an" 11 mL. +t is )erifie" that the 5oley catheter is open an" "raining freely. The #rine is "ark yello'% 'itho#t $loo"% an" has a specific gra)ity of 1444 an" a so"i#m concentration of 14 mE*7L. The patient,s $loo" press#re is 1447(4 mm /g% his p#lse is

817min% an" his central )eno#s press#re is 2 cm /2>. >n the $asis of these fin"ings% 'hich of the follo'ing is the most appropriate ne.t step in managementa6@i#retics sho#l" $e gi)en $65l#i" a"ministration sho#l" contin#e at the present rate c6The rate of fl#i" a"ministration sho#l" $e "ecrease" "6The rate of fl#i" a"ministration sho#l" $e increase" e6 Treatment is nee"e" for renal fail#re :ormal La$s 4.A 2(!year!ol" immigrant from El Sal)a"or has a 14 L 12 L 8 cm mass in her left $reast. +t has $een present for ( years an" has slo'ly gro'n to its present siDe. The mass is firm% nonten"er% r#$$ery% an" completely mo)a$le% an" it is not attache" to the o)erlying skin or the chest 'all. There are no palpa$le a.illary no"es or skin #lceration. =hich of the follo'ing is the most likely "iagnosisa6Breast cancer $62hronic cystic mastitis c62ystosarcoma phyllo"es "6+ntra"#ctal papilloma e6Mammary "ysplasia :ormal La$s .A &2!year!ol" 'oman in the 2n" month of pregnancy is fo#n" to ha)e a !cm mass in the #pper o#ter *#a"rant of her left $reast. Mammogram sho's no other lesions% an" core $iopsy re)eals infiltrating "#ctal carcinoma. =hich of the follo'ing 'o#l" $e the $est co#rse of action at this timea62hemotherapy no'% "eferring s#rgery #ntil after "eli)ery $6Aa"iation therapy no'% "eferring s#rgery #ntil after "eli)ery c6L#mpectomy an" a.illary sampling% follo'e" in < 'eeks $y ra"iotherapy "6Mo"ifie" ra"ical mastectomy no'% "eferring systemic therapy #ntil later e6 +mme"iate therape#tic a$ortion an" palliati)e $reast s#rgery :ormal La$s

<.+n the co#rse of a ro$$ery% a yo#ng 'oman is sta$$e" repeate"ly. >n arri)al at the emergency "epartment% she is shi)ering an" asks for a $lanket an" a "rink of 'aterG she is note" to $e pale an" perspiring. /er $loo" press#re is (27 4 mm /g an" her p#lse is 1&47min. /er neck an" forehea" )eins are large an" "isten"e". A *#ick initial s#r)ey re)eals entry 'o#n"s in her left chest an" #pper a$"omen. She has $ilateral $reath so#n"s an" a scaphoi"% nonten"er a$"omen. As +? inf#sions of Ainger,s lactate are starte"% her systolic $loo" press#re "rops f#rther to 44 mm /g% no "istal p#lses can $e felt% an" she loses conscio#sness. /er central )eno#s press#re at that time is 21 cm /2>. =hich of the follo'ing is the most appropriate ne.t step in managementa62hest .!ray to "irect f#rther therapy $6Bilateral chest t#$es c6@iagnostic peritoneal la)age "6E)ac#ation of the pericar"ial sac e62rash laparotomy in the emergency "epartment to clamp the aorta :ormal La$s (.A <2!year!ol" man reports an episo"e of gross% painless hemat#ria. There is no history of tra#ma% an" f#rther *#estioning "etermines that he ha" total hemat#ria% rather than initial or terminal hemat#ria. The man "oes not smoke an" has ha" no other symptoms refera$le to the #rinary tract. /e has no kno'n allergies. Physical e.amination% incl#"ing rectal e.amination% is #nremarka$le. /is ser#m creatinine is 4.1 mg7"L% an"% e.cept for the presence of many re" cells% his #rinalysis is normal an" sho's no re" cell casts. /is hematocrit is 4<B. =hich of the follo'ing is the most appropriate initial step in the 'ork#pa62oag#lation st#"ies an" #rinary c#lt#res $6+ntra)eno#s pyelogram 3+?P6 an" cystoscopy c6PSA "etermination an" prostatic $iopsies "6Sonogram an" 2T scan of $oth ki"neys e6Aetrogra"e cystogram an" pyelograms :ormal La$s 1. A <&!year!ol" man% 'ho 'eighs < kg% is in his 2n" postoperati)e "ay after an a$"ominoperineal resection for cancer of the rect#m. An

in"'elling 5oley catheter 'as left in place after s#rgery. The n#rses are concerne" $eca#se% e)en tho#gh his )ital signs ha)e $een sta$le% his #rinary o#tp#t in the past 2 ho#rs has $een Dero. +n the prece"ing & ho#rs% they ha" collecte" < mL% (& mL% an" <1 mL. =hich of the follo'ing is the most likely "iagnosisa6 Ac#te renal fail#re $6@amage to the $la""er "#ring the operation c6@amage to the #reters "#ring the operation "6@ehy"ration e6 Pl#gge" or kinke" catheter :ormal La$s 8.A &(!year!ol" 'oman #n"ergoes a l#mpectomy an" a.illary "issection for a &!cm infiltrating "#ctal carcinoma% "iagnose" $y core $iopsies% locate" on the #pper o#ter *#a"rant of her left $reast. The pathology report of the s#rgical specimen is recei)e" & "ays after the operation. +t in"icates that all margins aro#n" the t#mor are clear% an" that 4 of 1( a.illary lymph no"es ha)e metastatic t#mor. The t#mor is reporte" to $e estrogen an" progesterone receptor negati)e. =hich of the follo'ing sho#l" f#rther therapy most likely incl#"ea6Antiestrogen me"ication 3tamo.ifen6 $62on)ersion to mo"ifie" ra"ical mastectomy c6Aa"iation to the remaining left $reast "6Aa"iation to the remaining left $reast an" systemic chemotherapy e6 Aa"iation to $oth $reasts an" tamo.ifen :ormal La$s 14. A 4 !year!ol" man 'ith alcoholic cirrhosis is $lee"ing from a "#o"enal #lcer. /e has re*#ire" < #nits of $loo" o)er the past 1 ho#rs% an" all conser)ati)e meas#res to stop the $lee"ing% incl#"ing irrigation 'ith col" saline% +? )asopressin% an" en"oscopic #se of the laser ha)e faile". /e is $eing consi"ere" for s#rgical inter)ention. La$oratory st#"ies "one at the time of a"mission% 'hen he ha" recei)e" only one #nit of $loo"% sho'e" a $ilir#$in of 4. mg7"L% a prothrom$in time of 22 secon"s% an" a ser#m al$#min of 1.1 g7"L. /e 'as mentally clear 'hen he came in% $#t has since then "e)elope" encephalopathy an" is no' in a coma. =hich of the follo'ing $est "escri$es his operati)e riska6Accepta$le as he no' is

$6Amena$le to impro)ement if he recei)es )itamin M c6Amena$le to impro)ement if he is gi)en al$#min "6Prohi$iti)e #nless he is "ialyDe" to normaliDe his $ilir#$in e6Prohi$iti)e regar"less of attempts to impro)e his con"ition :ormal La$s 11. A !year!ol" 'oman falls in the sho'er an" h#rts her right sho#l"er. She sho's #p in the emergency "epartment 'ith her arm hel" close to her $o"y% $#t the forearm rotate" o#t'ar" as if she 'ere going to shake han"s. She is in pain an" 'ill not mo)e the arm from that position. /er sho#l"er looks 0s*#are0 in comparison 'ith the ro#n"e" #nh#rt opposite si"e% an" there is n#m$ness in a small area of her sho#l"er o)er the "eltoi" m#scle. =hich of the follo'ing is the most likely "iagnosisa6Acromiocla)ic#lar separation $6Anterior "islocation of the sho#l"er c65ract#re of the #pper en" of the h#meral shaft "6Posterior "islocation of the sho#l"er e6 Scap#lar fract#re :ormal La$s 12. A 22!year!ol" con)enience store clerk is shot once 'ith a .&1 cali$er re)ol)er. The entry 'o#n" is in the left mi"cla)ic#lar line% 2 inches $elo' the nipple. There is no e.it 'o#n". /e is hemo"ynamically sta$le. A chest .!ray film sho's a small pne#mothora. on the left% an" "emonstrates the $#llet to $e lo"ge" in the left paraspinal m#scles. +n a""ition to the appropriate treatment for the pne#mothora.% 'hich of the follo'ing 'ill this patient most likely nee"a6 Bari#m s'allo' $6Bronchoscopy c6E.traction of the $#llet )ia local $ack e.ploration "6E.traction of the $#llet )ia left thoracotomy e6 E.ploratory laparotomy :ormal La$s 1&.A <1!year!ol" man is $ro#ght to the emergency "epartment 'ith e.cr#ciating $ack pain that $egan s#""enly 4 min#tes ago. The pain is constant an" is not e.acer$ate" $y sneeDing or co#ghing. /e is "iaphoretic an" has a systolic $loo" press#re of 84 mm /g. There is an

1!cm p#lsatile mass "eep in his epigastri#m% a$o)e the #m$ilic#s. A chest .!ray film is #nremarka$le. T'o years ago% he 'as "iagnose" 'ith prostatic cancer an" 'as treate" 'ith orchiectomy an" ra"iation. At that time% his $loo" press#re 'as normal% an" he ha" a <!cm% asymptomatic a$"ominal aortic ane#rysm for 'hich he "ecline" treatment. =hich of the follo'ing is the most likely "iagnosisa6@issecting thoracic aortic ane#rysm $65ract#re of l#m$ar pe"icles 'ith cor" compression c6/erniate" "isc "6Metastatic t#mor to the l#m$ar spine e6 A#pt#ring a$"ominal aortic ane#rysm :ormal La$s 14. A mi""le!age" homeless man is $ro#ght to the emergency "epartment $eca#se of )ery se)ere pain in his forearm. /e ha" passe" o#t after "rinking a $ottle of cheap 'ine% an" then slept on a park $ench for an in"eterminate time% pro$a$ly more than 12 ho#rs. Shortly after he 'oke #p an" $egan to 'alk% the pain $egan. There are no signs of tra#ma% $#t the m#scles in his forearm are )ery firm an" ten"er to palpation% an" passi)e motion of his fingers an" 'rist elicits e.cr#ciating pain. P#lses at the 'rist are normal. =hich of the follo'ing is the most appropriate ne.t step in managementa6Analgesics an" o$ser)ation $6+mmo$iliDation in a sling c6+mmo$iliDation in a plaster cast "6Emergency em$olectomy e6Emergency fasciotomy :ormal La$s 1 . A 4!year!ol" o$ese man gi)es a history of $#rning retrosternal pain an" heart$#rn that is $ro#ght a$o#t $y $en"ing o)er% 'earing a tight $elt% or lying flat in $e" at night. /e gets symptomatic relief from o)er! the!co#nter antiaci"s or /2 $lockers% $#t has ne)er $een formally st#"ie" or treate". The pro$lem has $een present for many years an" seems to $e progressing. =hich of the follo'ing is the most appropriate ne.t step in managementa6 Bari#m s'allo' $62ar"iac enDymes an" E2C

c6Proton p#mp inhi$itors "6En"oscopy an" $iopsies e6Laparoscopic :issen f#n"oplication :ormal La$s 1<.A pe"estrian is hit $y a car an" knocke" #nconscio#s. =ithin a fe' min#tes% he starts to mo)e aro#n" an" moan. =hen the am$#lance arri)es% he is mo)ing all fo#r e.tremities an" m#m$ling that his neck h#rts. Shortly thereafter% he lapses again into a "eep coma. +n the emergency "epartment% it is note" that his left p#pil is fi.e" an" "ilate"% an" he has clear fl#i" "ripping from the left ear. The tra#ma team int#$ates him nasally o)er a fi$eroptic $ronchoscope an" "oes a *#ick initial s#r)ey that re)eals no other o$)io#s in9#ries. /e is hemo"ynamically sta$le. =hich of the follo'ing is the most appropriate ne.t step in managementa6 Anti$iotics an" high "ose corticosteroi"s $62er)ical spine an" sk#ll .!ray films c62T scan of the hea"% e.ten"e" to incl#"e the cer)ical spine "6>toscopic e.amination an" la$oratory st#"ies of the fl#i" e6Emergency ear s#rgery to stop the leak of cere$rospinal fl#i" :ormal La$s 1(.=hile 'orking at a $ook$in"ing shop% a yo#ng man s#ffers a tra#matic amp#tation of his in"e. finger. The finger 'as cleanly se)ere" at its $ase. The patient an" the finger are $ro#ght to a first!ai" station% from 'hich $oth are to $e transporte" to a highly specialiDe" me"ical center for replantation to $e "one. =hich of the follo'ing is the correct 'ay to prepare an" transport the se)ere" fingera6 @ry the finger of any traces of $loo" an" place it in a cooler fille" 'ith cr#she" ice $65reeDe it as *#ickly as possi$le% an" transport it immerse" in li*#i" nitrogen c6+mmerse it in col" alcohol for the entire trip "6Paint it 'ith antiseptic sol#tion an" place it on a $e" of "ry ice e6 =rap it in a moist ga#De% place it on a plastic $ag% an" place the $ag on a $e" of ice :ormal La$s

11. An o#t!of!shape% recently "i)orce"% 42!year!ol" man is trying to impress a yo#ng 'oman $y challenging her to a game of tennis. +n the mi""le of the game% a lo#" 0pop0 3like a g#nshot6 is hear"% an" the man falls to the gro#n" cl#tching his ankle. /e limps off the co#rt 'ith pain an" s'elling in the $ack of the lo'er leg. Altho#gh he can still 'eakly plantar!fle. his foot% he seeks me"ical help the ne.t "ay $eca#se of persistent pain% s'elling% an" limping. /e can p#t 'eight on that foot 'ith no e.acer$ation of the pain% $#t the motion of taking a step is painf#l. =hich of the follo'ing 'o#l" $e the most likely fin"ing on physical e.aminationa6 Tapping on the calcane#s is e.tremely painf#l $6The ankle 9oint can $e a$"#cte" farther o#t than the normal contralateral si"e c6 The ankle 9oint can $e a""#cte" farther in than the normal contralateral si"e "6There is a gap in the Achilles ten"on easily felt $y palpation e6 There is crepitation an" grating $y "irect palpation o)er either malleoli :ormal La$s 18. A <<!year!ol" man 'ith "ia$etes an" generaliDe" arteriosclerotic occl#si)e "isease notices a gra"#al loss of erectile f#nction o)er se)eral years. +nitially% he can get erections% $#t they "o not last long eno#gh. Later% he notices a "ecrease in the *#ality of his erections% an" more recently he $ecomes% $y his o'n criteria% completely impotent. /e has occasional% $rief noct#rnal erections% $#t 0he can ne)er get an erection 'hen he nee"s one.0 =hich of the follo'ing is the most appropriate initial step in managementa6Psychotherapy $6Pharmacologic therapy c6Erectile ner)e reconstr#ction "6+mplanta$le penile prosthesis e6P#"en"al artery re)asc#lariDation :ormal La$s 24.A 4!year!ol" 'oman has a se)ere #reteral colic. An intra)eno#s pyelogram sho's a (!mm #reteral stone at the #reteropel)ic 9#nction.

She has a normal coag#lation profile. =hich of the follo'ing 'o#l" most likely $e the $est therapy in this casea6 Plenty of fl#i"s an" analgesics an" a'ait spontaneo#s passage $6E.tracorporeal shock 'a)e lithotripsy 3ES=L6 c6En"oscopic retrogra"e $asket e.traction "6En"oscopic retrogra"e laser )aporiDation of the stone e6>pen s#rgical remo)al :ormal La$s 21.A <1!year!ol" 'oman presents 'ith an o$)io#sly incarcerate" #m$ilical hernia. She has gross a$"ominal "istention% is clinically "ehy"rate"% an" reports persistent fecaloi" )omiting for the past & "ays. Altho#gh tire"% 'eak% an" thirsty% she is a'ake an" alert an" her sensori#m is not partic#larly affecte". La$oratory analysis re)eals a ser#m so"i#m concentration of 111 mE*7L. =hich of the follo'ing is the most likely physiologic e.planation for the ser#m so"i#ma6 She has ac#te 'ater into.ication $6She has $een )omiting an" trapping hypertonic fl#i"s in the $o'el l#men c6She has )omite" an" se*#estere" so"i#m!containing fl#i"s% an" has retaine" en"ogeno#s an" ingeste" 'ater "6There m#st $e a la$oratory error% $eca#se s#ch a ser#m so"i#m le)el 'o#l" ha)e pro"#ce" coma e6 ?ol#me "epri)ation lea"s to renal 'asting of so"i#m :ormal La$s 22.A &1!year!ol" 'oman smashes her car against a $ri"ge a$#tment. She s#stains m#ltiple in9#ries% incl#"ing #pper an" lo'er e.tremity fract#res. She is f#lly a'ake an" alert% an" she reports that she 'as not 'earing a seat $elt an" "istinctly remem$ers hitting her a$"omen against the steering 'heel. /er $loo" press#re is 1& 7( mm /g% an" her p#lse is 117min. Physical e.amination sho's that she has a rigi"% ten"er a$"omen% 'ith g#ar"ing an" re$o#n" in all fo#r *#a"rants. She has no $o'el so#n"s. =hich of the follo'ing 'o#l" $e the most appropriate step in e)al#ating potential intraa$"ominal in9#riesa62ontin#e" clinical o$ser)ation $62T scan of the a$"omen c6Sonogram of the a$"omen

"6@iagnostic peritoneal la)age e6E.ploratory laparotomy :ormal La$s 2&.A 2(!year!ol" man s#staine" penetrating in9#ries of the chest an" a$"omen 'hen he 'as repeate"ly sta$$e" 'ith a long ice!pick. At the time of a"mission% he ha" a right pne#mothora.% for 'hich a chest t#$e 'as place" prior to #n"ergoing a general anesthetic for e.ploratory laparotomy. The operation re)eale" no intraa$"ominal in9#ries an" 'as terminate" sooner than ha" $een anticipate". The patient remaine" int#$ate"% 'aiting for the anesthetic to 'ear off. Beca#se he 'as not mo)ing eno#gh air% he 'as place" on a respirator. Then% he s#""enly 'ent into car"iac arrest an" "ie". All thro#gh this time he ha" $een hemo"ynamically sta$le% an" ne)er ha" any signs of hypotension or arrhythmias. =hich of the follo'ing 'as the most likely ca#se of the car"iac arresta6Air em$olism $65at em$olism c6Myocar"ial infarction "6P#lmonary em$ol#s e6 Tension pne#mothora. :ormal La$s 24. A 1&!year!ol"% o$ese $oy complains of persistent knee pain for se)eral 'eeks. The family $rings him in $eca#se he has $een limping. /e sits in the e.amining ta$le 'ith the sole of the foot on the affecte" si"e pointing to the other leg. Physical e.amination is normal for the knee% $#t sho's limite" hip motion. As the hip is fle.e"% the leg goes into e.ternal rotation an" cannot $e rotate" internally. =hich of the follo'ing is the most likely "iagnosisa6 A)asc#lar necrosis of the femoral hea" $6@e)elopmental "ysplasia of the hip c6 >steogenic sarcoma of the lo'er fem#r "6Slippe" capital femoral epiphysis e6 Ti$ial torsion 'ith foot in)ersion :ormal La$s 2 .A (2!year!ol" man has a &!mm #reteral stone impacte" at the #retero)esical 9#nction. /e has $een ha)ing mil" #reteral colicky pain

for a$o#t 12 ho#rs% an" he has $een gi)en fl#i"s an" analgesics in the e.pectation that he 'ill spontaneo#sly pass the stone. /e then has shaking chills% an" spikes a temperat#re of 44 2 3144 56. =hen seen shortly thereafter% he has flank pain an" looks *#ite ill. =hich of the follo'ing is the most appropriate ne.t step in managementa6A""ition of +? anti$iotics to the c#rrent therape#tic regimen $62r#shing an" e.traction of the stone )ia cystoscopy c6 E.tracorporeal shock 'a)e lithotripsy an" parenteral anti$iotics "6+mme"iate insertion of a s#prap#$ic catheter into the $la""er e6 +? anti$iotics an" imme"iate "ecompression of the #rinary tract a$o)e the stone :ormal La$s 2<.A 14!year!ol" $oy "i)es into the shallo' en" of a s'imming pool an" hits his hea" against the $ottom. =hen he is resc#e"% he sho's a complete lack of ne#rologic f#nction $elo' the neck. /e is still $reathing on his o'n% $#t he cannot mo)e or feel his arms an" legs. The parame"ics caref#lly immo$iliDe his neck for transportation to the hospital% an" they alert the emergency "epartment to his impen"ing arri)al. >nce there% 'hich of the follo'ing 'o#l" most likely ha)e an imme"iate $enefit for this patienta6/yper$aric o.ygenation $6+? anti$iotics c6+? high!"ose corticosteroi"s "6Massi)e "i#resis in"#ce" $y loop "i#retics. e6 S#rgical "ecompression of the cor" :ormal La$s 2(.A (2!year!ol" man is sche"#le" to ha)e electi)e sigmoi" resection for "i)ertic#lar "isease. /e has a history of heart "isease% an" ha" a "oc#mente" myocar"ial infarction 2 years ago. /e c#rrently "oes not ha)e angina% $#t he li)es a se"entary life $eca#se 0he gets o#t of $reath0 if he e.erts himself. @#ring the physical e.amination% it is note" that he has 9#g#lar )eno#s "istention. /e has a hemoglo$in of 12 g7"L. +f s#rgery is in"ee" nee"e"% 'hich of the follo'ing sho#l" most likely $e "one prior to the operationa6E)al#ate the patient as a can"i"ate for coronary re)asc#lariDation $6 Place the patient on intensi)e respiratory therapy

c6>r"er a transf#sion to increase the patient,s hemoglo$in to 14 g7"L "6 Treat the patient for congesti)e heart fail#re e6+f at all possi$le% 'ait < months $efore performing s#rgery :ormal La$s 21. A gro#p of illegal immigrants is sm#ggle" across the $or"er in a close" metal tr#ck in the mi""le of s#mmer. =hen apprise" $y ra"io that the $or"er patrol is on their trail% the sm#gglers a$an"on their charges in the mi""le of the "esert% in the locke" tr#ck% 'ith little 'ater to "rink. The )ictims are fo#n" an" resc#e" "ays later. >ne of them is $ro#ght to the emergency "epartment% a'ake an" alert% 'ith o$)io#s clinical signs of se)ere "ehy"ration an" a ser#m so"i#m concentration of 1 mE*7L. =hich of the follo'ing 'o#l" $e the $est choice an" rate of +? fl#i" a"ministrationa6 L of B "e.trose in 'ater 3@ =6 o)er 2!& "ays $6 L of @ = o)er !14 ho#rs c6 L of B "e.trose in half normal saline 3@ 172 :S6 o)er !14 ho#rs "614 L of @ 172 :S o)er !14 ho#rs e614 L of normal saline o)er 2!& "ays :ormal La$s 28.A (2!year!ol" man comes in complaining of persistent an" nagging lo' $ack pain that he has ha" for se)eral 'eeks. The pain seems to $e increasing in intensity% is 'orse at night% is #nrelie)e" $y rest or positional changes% an" is not e.acer$ate" $y co#ghing% sneeDing% or straining to ha)e a $o'el mo)ement. /e is a chronic smoker% an" for the past & months has ha" persistent co#gh 'ith occasional $loo"y streake" sp#t#m% as 'ell as a 24!po#n" 'eight loss. >n physical e.amination% he is "istinctly ten"er to palpation at a partic#lar spot o)er his lo'er thoracic spine. =hich of the follo'ing is the most likely "iagnosisa6Ankylosing spon"ylitis $6 /erniate" "isk c6Metastatic t#mor to the thoracic spine "6M#ltiple myeloma e6Primary malignant $one t#mor :ormal La$s

&4. The #nrestraine" front!seat passenger in a car that crashes at high spee" is $ro#ght to the emergency "epartment $y parame"ics. At the site of in9#ry% the patient 'as #nconscio#s an" ha" g#rgly respiratory so#n"s% an" the EMTs s#ccessf#lly accomplishe" $lin" nasotracheal int#$ation. The initial s#r)ey in the emergency "epartment sho's normal )ital signs% m#ltiple facial lacerations% an" an #nresponsi)e% comatose patient 'ith fi.e" "ilate" p#pils. Preparations are ma"e to "o a 2T scan of the hea". +t is imperati)e that 'hich of the follo'ing sho#l" $e o$taine" as 'ella6Base of the sk#ll .!ray films $6 E.tension of the 2T to incl#"e the entire cer)ical spine c6Aa"iographs of all the teeth "6Separate 2T scan of the a$"omen e6 Special )ie's of the ma.illary sin#ses :ormal La$s &1.A !year!ol" 'oman has $een kno'n for years to ha)e mitral )al)e prolapse. She has no' "e)elope" e.ertional "yspnea% orthopnea% an" atrial fi$rillation. She has an apical% high!pitche"% holosystolic heart m#rm#r that ra"iates to the a.illa an" $ack. Beca#se of her "eterioration% s#rgery has $een recommen"e". =hich of the follo'ing is the most appropriate proce"#rea6Aortic )al)e replacement $6Mitral commiss#rotomy c6Mitral )al)e ann#loplasty "6Mitral )al)e replacement e6Both aortic an" mitral )al)e replacement :ormal La$s &2.A 2&!year!ol" 'oman seeks help for e.*#isite pain 'ith "efecation an" $loo" streaks on the o#tsi"e of her stools% 'hich she has $een ha)ing for se)eral 'eeks. Beca#se of the pain% she has a)oi"e" ha)ing $o'el mo)ements% an" 'hen she finally "i" the stools 'ere har" an" e)en more painf#l. =hen seen% she has no fe)er or le#kocytosis. Physical e.amination has to $e "one #n"er spinal anesthesia% $eca#se the patient 'as so afrai" of the pain that she initially ref#se" e)en inspection of the area. The e.amination confirms the s#specte" "iagnosis% an" she is place" on stool softeners an" appropriate topical agents% $#t 'itho#t

s#ccess. She is 'illing to #n"ergo more aggressi)e treatment. =hich of the follo'ing is the most appropriate ne.t stepa6 E.cision of the lesion $65ist#lotomy c6+ncision an" "rainage "6Lateral internal sphincterotomy e6A#$$er $an" ligation :ormal La$s &&.An e.ploratory laparotomy for m#ltiple intra!a$"ominal in9#ries has laste" & an" a half ho#rs. M#ltiple $loo" transf#sions ha)e $een gi)en% an" se)eral liters of Ainger,s lactate ha)e $een inf#se". =hen the s#rgeons are rea"y to close the a$"omen% they fin" that the a$"ominal 'all e"ges cannot $e p#lle" together 'itho#t #n"#e tension. Both the $elly 'all an" the a$"ominal contents seem to $e s'ollen. =hich of the follo'ing is the most appropriate management in this sit#ationa6Appro.imate the skin only% #sing to'el clips $62lose the a$"omen 'ith hea)y retention s#t#res c6Ci)e "i#retics an" close the a$"omen in the #s#al 'ay "6Lea)e the a$"omen an" its contents open to the air e6Pro)i"e temporary $o'el co)erage 'ith an a$sor$a$le mesh :ormal La$s &4. A <1!year!ol" man comes in $eca#se of colicky a$"ominal pain an" )omiting of & "ays, "#ration. >n physical e.amination% he is mo"erately "isten"e" an" has high pitche" hyperacti)e $o'el so#n"s an" a !cm ten"er groin mass. >n "irect *#estioning% he e.plains that he has ha" that $#lge for many years% $#t has al'ays $een a$le to 0p#sh it $ack in0 'hen he lies "o'n. 5or the past & "ays% ho'e)er% he has $een #na$le to "o so. /e has a temperat#re of &1.8 2 3142 56 an" a 'hite $loo" cell co#nt of 12% 447mm&. =hich of the follo'ing is the most appropriate management at this timea6A sonogram of the mass $6A trial of nasogastric s#ction an" +? fl#i"s for a fe' "ays c6+nsertion of a long rectal t#$e )ia sigmoi"oscopy "6Man#al re"#ction of the hernia% follo'e" $y a perio" of o$ser)ation e6Urgent s#rgical inter)ention :ormal La$s

& .+n a rollo)er car acci"ent% a 42!year!ol" 'oman is thro'n from the car. The car s#$se*#ently lan"s on her an" cr#shes her. >n physical e.amination in the emergency "epartment% it is "etermine" that she has a pel)ic fract#re% 'hich is confirme" $y porta$le .!rays "one as she is $eing res#scitate". /er initial $loo" press#re is 47&4 mm /g% an" her p#lse is 1<47min an" $arely percepti$le. Thirty min#tes later% after 2 L Ainger,s lactate an" 2 U packe" cells ha)e $een inf#se"% her press#re is only (47 4 mm /g% an" her p#lse is 1&47min. A sonogram "one in the emergency "epartment sho's no intra!a$"ominal $lee"ing% an" a "iagnostic peritoneal la)age confirms that there is no $loo" in the a$"omen 3the reco)ere" fl#i" is pink% $#t not grossly $loo"y6. Aectal an" )aginal e.ams sho' no in9#ries to those organs. There is no $loo" in her #rine. =hich of the follo'ing is the most appropriate ne.t step in managementa6Packing of the )agina an" rect#m $6Angiographic em$oliDation of torn )eins c6E.ternal fi.ation of the pel)is "6>pen re"#ction an" internal fi.ation of the pel)is e6 E.ploratory laparotomy 'ith pel)ic "issection an" hemostasis :ormal La$s &<.A 2!year!ol" n#rse seeks me"ical retirement $eca#se of a 0heart con"ition.0 She complains of "isa$ling attacks of tachycar"ia an" palpitations. The physical e.amination an" E2C st#"ies confirm that in"ee" her p#lse is $et'een 144 an" 14 7min at all times% an" she is in an" o#t of atrial fi$rillation. +t is also note" that she is fi"gety an" constantly mo)ing% an" )ario#s e.aminers remark that she arri)es for tests lightly "resse" 'hen it is rather col" o#tsi"e. Thyroi" f#nction st#"ies sho' ele)ate" free thyro.ine 3T46 an" #n"etecta$le le)els of thyroi" stim#lating hormone 3TS/6. /er thyroi" glan" is not clinically enlarge" or ten"er. =hich of the follo'ing is the most appropriate ne.t step in "iagnosisa65ine nee"le aspiration cytology of the thyroi" glan" $6MA+ of the pit#itary area c6Aa"ioacti)e io"ine #ptake "6Ser#m le)els of 2 pepti"e e6 Ser#m le)els of triio"othyronine 3T&6 :ormal La$s

&(. A 8!year!ol" man is referre" for e)al#ation $eca#se he has $een fainting at his 9o$% 'here he operates hea)y machinery. /e is pale an" ga#nt% $#t other'ise his physical e.amination is remarka$le only for 4E occ#lt $loo" in the stool. La$oratory st#"ies sho' a hemoglo$in of gm7"L 'ith microcytosis% as 'ell as "ecrease" le)els of ser#m iron an" increase" iron $in"ing capacity. =hich of the follo'ing 'ill most likely esta$lish the "iagnosisa6 Upper gastrointestinal series 3s'allo'e" $ari#m st#"ies6 $62olonoscopy c65le.i$le sigmoi"oscopy to 4 cm "6Upper gastrointestinal en"oscopy e6?isceral angiogram :ormal La$s &1.A 24!year!ol" 'oman is a"mitte" to the hospital for a $roken fem#r. The patient 'as in a motor )ehicle acci"ent 24 ho#rs ago an" 'as $ro#ght to the hospital $y EMS. >n the scene% she 'as fo#n" $elte" in her car in the "ri)ers seat% an" her only "oc#mente" in9#ry 'as the leg fract#re. She ha" no loss of conscio#sness or altere" mental stat#s. >n arri)al to the hospital% ra"iographs confirme" a fract#re of her fem#r. She 'as sta$iliDe" o)er night an" sche"#le" for s#rgery the ne.t "ay. =hich of the follo'ing is the ma9or s#rgical risk for this patienta6Air em$olism $62ere$ro)asc#lar acci"ent c65at em$olism "6>steomyelitis e6 Permanent "isa$ility :ormal La$s

&8.A 42!year!ol" 'oman hit her $reast 'ith a $room han"le 'hile "oing ho#se'ork. She notice" a l#mp in that area at the time% an" 1 'eek later the l#mp 'as still present. She then so#ght me"ical a")ice. >n physical e.amination% she has a &!cm% har" mass "eep insi"e the affecte" $reast% an" some s#perficial ecchymosis o)er the area. =hich of the follo'ing is the most appropriate ne.t step% or steps% in management-

a6Aeassess in a$o#t 2 months% 'ith no specific therapy $6/ot packs% analgesics% an" s#rgical e)ac#ation of the hematoma c6 Mammogram% an" no f#rther therapy if the report "oes not i"entify cancer "6Mammogram an" $iopsy of the mass e6 Mastectomy :ormal La$s 44.The #nrestraine" front!seat passenger in a car that crashe" at high spee" arri)es at the emergency "epartment 'ith signs of mo"erate respiratory "istress. Physical e.amination sho's no $reath so#n"s at all on the left hemithora.. Perc#ssion is #nremarka$le% an" his )ital signs are normal. A chest .!ray film sho's a collapse" left l#ng an" m#ltiple air!fl#i" le)els filling the left ple#ral ca)ity. A nasogastric t#$e that ha" $een place" prior to taking the film sho's the t#$e reaching the #pper a$"omen an" then c#rling #p into the left chest. =hich of the follo'ing is the most likely "iagnosisa6 Blo' o#t of p#lmonary $le$s $6Esophageal r#pt#re or perforation c6Left "iaphragmatic r#pt#re "6Left hemopne#mothora. e6 Ma9or in9#ry to the tracheo$ronchial tree :ormal La$s 41.>n the (th postoperati)e "ay after the pinning of a $roken hip% a (<! year!ol" man s#""enly "e)elops se)ere ple#ritic chest pain an" shortness of $reath. =hen e.amine"% he is fo#n" to $e an.io#s% "iaphoretic% an" tachycar"ic% 'ith a $loo" press#re of 14471 mm /g. /e has prominent "isten"e" )eins in his neck an" forehea". Bloo" gases sho' hypo.emia an" hypocapnia. /is chest .!ray film is #nremarka$le. The n#rses ha)e place" him on s#pplemental o.ygen $y face mask. =hich of the follo'ing is the most appropriate ne.t step in managementa6 Aortogram an" emergency s#rgical repair $6E2C an" car"iac enDymes c6+nt#$ation an" respirator% 'ith hyper)entilation an" PEEP "6Aetinal e.amination looking for fat "roplets e6 ?entilation!perf#sion l#ng scan% or spiral 2T scan of the chest

:ormal La$s 42. A yo#ng man s#stains a g#nshot 'o#n" to the $ase of his neck. /e 'as shot point $lank 'ith a .&1 cali$er re)ol)er. The entrance 'o#n" is a$o)e the left cla)icle% $elo' the le)el of the cricoi" cartilage% an" 9#st lateral to the sternomastoi" m#scle. The e.it 'o#n" is 9#st a$o)e the spino#s process of the right scap#la. /e has normal $reath so#n"s on $oth si"es% is a'ake an" alert% is talking 'ith a normal tone of )oice% is ne#rologically intact% an" is hemo"ynamically sta$le. Porta$le .!ray films of the neck an" chest taken in the emergency "epartment sho' some air in the tiss#es of the lo'er neck% $#t are other'ise non! "iagnostic. =hich of the follo'ing is the most appropriate ne.t step in managementa6>$ser)ation for se)eral ho#rs $62T scan of the lo'er neck an" #pper chest c6 Angiogram% esophagogram% esophagoscopy% an" $ronchoscopy prior to s#rgical e.ploration "6+mme"iate s#rgical e.ploration of the lo'er neck thro#gh a collar incision e6+mme"iate s#rgical e.ploration of the #pper chest thro#gh a me"ian sternotomy :ormal La$s 4&.A <(!year!ol" man sho's #p in the emergency "epartment $eca#se he has not $een a$le to )oi" for the past 12 ho#rs. /e feels the nee" to% $#t he cannot "o it. /e gi)es a history that% for se)eral years no'% he has $een getting #p fo#r or fi)e times a night to #rinate. +t 'o#l" take him a consi"era$le time to get the #rinary stream going% an" the stream lacke" force an" often en"e" in a "ri$$le. Beca#se of a col"% 2 "ays ago he $egan taking an antihistamine% taking a "econgestant% an" "rinking plenty of fl#i"s. Physical e.amination sho's a palpa$le% smooth% ro#n" mass arising from the p#$is an" reaching a$o#t half 'ay to'ar" the #m$ilic#s. The mass is "#ll to perc#ssion% an" p#shing on it accent#ates the feeling of nee"ing to )oi". Aectal e.amination re)eals a large% $oggy% non!ten"er prostate glan" 'itho#t no"#les. This a classic presentation for 'hich of the follo'ing ac#te con"itionsa6 Bacterial prostatitis $62ystitis in a patient 'ith $la""er cancer

c6Aenal fail#re "6 Urinary retention in a patient 'ith $enign prostatic hypertrophy e6Urinary retention in a patient 'ith prostatic cancer :ormal La$s 44. Se)eral months after s#staining a cr#shing in9#ry to his arm% a patient complains $itterly a$o#t constant% $#rning% agoniDing pain in that arm% that "oes not respon" to the #s#al analgesic me"ications. The pain in his arm is aggra)ate" $y the slightest stim#lation of the area% s#ch as r#$$ing from the shirt slee)es. The arm is col"% cyanotic% an" moist% $#t it is not s'ollen. P#lses at the 'rist are normal% an" ne#rologic f#nction of the three ma9or ner)es is intact. =hich of the follo'ing is most appropriate to pro)i"e "iagnostic confirmation of the nat#re of the pro$lem an" e)ent#al therapya6Angiogram an" s#$cla)ian )ein $ypass $62er)ical spine .!rays an" cer)ical ri$ resection c6@oppler st#"ies an" arterial reconstr#ction "6@oppler st#"ies an" fasciotomy e6 Sympathetic $lock an" s#rgical sympathectomy :ormal La$s 4 . A (!year!ol" alcoholic man is $eing treate" for ac#te hemorrhagic pancreatitis. /e 'as in the intensi)e care #nit for 1 'eek% 'here he re*#ire" chest t#$es for ple#ral eff#sions an" 'as on a respirator for se)eral "ays. E)ent#ally% he impro)e" s#fficiently to $e transferre" to the floor. Three "ays after lea)ing the #nit% an" a$o#t 2 'eeks after the onset of the "isease% he spikes a fe)er an" "e)elops le#kocytosis. =hich of the follo'ing "e)elopments "o these recent fin"ings most likely s#ggesta62hronic pancreatitis $6Pancreatic a$scess c6Pancreatic pse#"ocyst "6Pel)ic a$scess e6S#$phrenic a$scess :ormal La$s

4<. A &1!year!ol" male immigrant from +n"ia is fo#n" on a ro#tine physical e.amination to ha)e a single% 2!cm no"#le in the right lo$e of his thyroi" glan". The mass is firm% mo)es #p an" "o'n 'ith s'allo'ing% an" is not ten"er. The skin of his face an" neck is pitte" 'ith m#ltiple scars% 'hich s#ggest smallpo.G ho'e)er% he e.plains that the scars are "#e to )ery se)ere acne that he ha" as a yo#ngster% for 'hich he e)ent#ally recei)e" e.ternal $eam ra"iation therapy at the age of 14. /is thyroi" f#nction tests are normal% an" a fine nee"le aspiration 35:A6 cytology of the mass is rea" $y the pathologist as 0in"eterminate.0 =hich of the follo'ing is the most appropriate ne.t step in managementa6:o f#rther care is nee"e" $6Thyroi" f#nction tests sho#l" $e repeate" yearly c6Thyroi" scan an" sonogram are nee"e" "65:A sho#l" $e repeate" #ntil it can $e rea" as $enign or malignant e6Thyroi" lo$ectomy :ormal La$s 4(.A &&!year!ol" 'oman is in)ol)e" in a high!spee" a#tomo$ile collision. She arri)es at the emergency "epartment gasping for $reath. /er lips are cyanotic an" she has flaring nostrils. There are $r#ises o)er $oth si"es of the chest% an" ten"erness s#ggesti)e of m#ltiple ri$ fract#res. /er $loo" press#re is <474 mm /g% p#lse is 1<47min an" fee$le% an" central )eno#s press#re is 2 cm /2>. /er neck an" forehea" )eins are "isten"e". She is "iaphoretic an" has a hint of s#$c#taneo#s emphysema in the lo'er neck an" #pper chest. /er left hemithora. has no $reath so#n"s an" is hyperresonant to perc#ssion. The trachea is "e)iate" to the right% as are the heart so#n"s. =hich of the follo'ing is the most likely "iagnosisa6Air em$olism from tracheo$ronchial in9#ries $6 5lail chest "#e to m#ltiple ri$ fract#res c6 Massi)e intraple#ral $lee"ing from torn intercostal )essels "6Massi)e me"iastinal $lee"ing from r#pt#re" aorta e6 Tension pne#mothora. ca#se" $y l#ng p#nct#re" $y $roken ri$s :ormal La$s 41.A 44!year!ol" 'oman complains $itterly of se)ere hea"ache that has $een present for se)eral 'eeks an" has not respon"e" to the #s#al o)er!

the!co#nter hea"ache reme"ies. She locates the hea"ache to the center of her hea". +t is pretty m#ch constant $#t is 'orse in the mornings. She has no other ne#rologic signs or symptoms. She has ha" 0tension hea"aches0 in the past% $#t she says that those 'ere locate" in the $ack of her hea" an" felt "ifferent from the present pain. She is c#rrently #n"er consi"era$le stress $eca#se she has $een #nemploye" since #n"ergoing mo"ifie" ra"ical mastectomy for T&% :1% M4 $reast cancer 2 years ago. She ha" se)eral co#rses of post!operati)e chemotherapy% 'hich she e)ent#ally "iscontin#e" $eca#se of the si"e effects. =hich of the follo'ing is the most appropriate ne.t step in "iagnosisa62T scan of the hea" $6Psychiatric e)al#ation c6Sk#ll .!rays "6Aortic arch arteriogram e6L#m$ar p#nct#re :ormal La$s 48. A 8!year!ol" $lack man has a rock!har"% "iscrete% 1. !cm no"#le that is felt in his prostate "#ring a ro#tine physical e.amination. /e is completely asymptomatic% an" his prostatic specific antigen 3PSA6 "one & months ago 'as normal for his age. /is last rectal e.amination 'as performe" a year earlier an" 'as #nremarka$le. =hich of the follo'ing 'ill $est esta$lish the "iagnosisa62linical follo'!#p "#ring the ens#ing year $6 Aepeat "etermination of PSA c6Transrectal nee"le $iopsy of the mass "6Transrectal sonogram of the prostate e6Trans#rethral resection of the prostate :ormal La$s 4.A 8!month!ol" infant is $ro#ght in $y her parents $eca#se she has an #m$ilical hernia. Physical e.amination sho's an #m$ilical "efect a$o#t 1 cm in "iameter% 'ith a small $#lge 'hen the girl cries. The hernial contents can $e easily re"#ce". The hernia is not painf#l% an" the girl is other'ise asymptomatic. =hich of the follo'ing is the most appropriate ne.t step in managementa6:o therapy #nless the hernia persists $eyon" the age of 2 years $6Aepeate" in9ections of sclerosing agents

c6Electi)e laparoscopic s#rgical repair "6Electi)e open s#rgical repair e6 Urgent s#rgical repair
Normal ,abs

Note: Check your own answers before hittin$ the Check button below. When you click the Check button, a browser win"ow will a!!ear that contains a summary of your results. (*!lanations Block 24 E.planations

:) (*!lanation: 7he correct answer is . In all other soli" or$an trans!lants, "eterioration of function :> "ays out woul" su$$est an acute re<ection e!iso"e, an" a!!ro!riate bio!sies woul" be "one to confirm the "ia$nosis. In the case of the liver, however, anti$enic reactions are less common, whereas technical !roblems with the biliary an" vascular anastomosis are the most common cause of early functional "eterioration. 7hey are, therefore, the first anomalies to be sou$ht. &reforme" antibo"ies 'choice A) are res!onsible for hy!eracute re<ection, which woul" be evi"ent within minutes of establishin$ bloo" flow to the $raft. Choices C, %, an" ( are centere" on liver bio!sy, which woul" be "one only after technical !roblems have been rule" out. ;) (*!lanation: 7he correct answer is %. 7he s!leen is a hi$hly vasculari#e" or$an, an" is vulnerable to traumatic ru!ture. 7his can occur Hs!ontaneouslyH 'i.e. with minimal trauma such as fallin$ a$ainst a table or even overly vi$orous !al!ation "urin$ a !hysical e*amination) in !atients with an enlar$e" s!leen "ue to "isease 'e.$., leukemias, autoimmune "iseases with re" cell sequestration in the s!leen, or as a com!lication of !ortal hy!ertension). Alternatively, s!lenic ru!ture can occur in !reviously normal in"ivi"uals who have severe trauma to the ab"omen. In either case, the heavily vasculari#e" s!leen is usually unable to sto! 'often massively) blee"in$ internally. (mer$ency s!lenectomy is in"icate" to control the blee"in$. acteremia 'choice A) an" !eritonitis 'choice () are much less of a risk in s!lenic ru!ture than in ru!ture of a hollow viscus such as the colon, since the s!leen is usually sterile. (lectrolyte abnormalities 'choice ) can "evelo! secon"arily to the ischemia !ro"uce" by severe bloo" lossG these are much less critical than the bloo" loss itself an" will often correct s!ontaneously with a"equate re!lacement of bloo". (*ternal bloo" loss 'choice C) is often insi$nificant in in<uries such as this. 0) (*!lanation: 7he correct answer is %. 7he calculations ma"e by stan"ar" formulas are only an e"ucate" $uess. Once flui" a"ministration be$ins, we <u"$e its a"equacy by the information !rovi"e" by urinary out!ut an" central venous !ressure, aimin$ for an out!ut of :4; m,Bk$Bhr, while not e*cee"in$ a venous !ressure of :> or :/. In this case, our calculations fell short of the mark, an" the !atient nee"s more flui"s at a faster rate. %iuretics 'choice A) are not the answer when all in"icators show flui" nee": his venous !ressure is low, his bloo" !ressure an" !ulse rate are mar$inal, an" he has very concentrate" urine. 2e nee"s flui"s[ 7he !resent rate 'choice ) may follow the Hformula,H but it is clearly ina"equate. %ecreasin$ the rate 'choice C) is the very o!!osite of what is nee"e". An" as for renal

failure 'choice (), it is in"ee" !art of the "ifferential "ia$nosis whenever urinary out!ut is not as hi$h as it shoul" be. 2owever, the vi$nette tol" you that his urine is hi$hly concentrate" an" has well less than ;> m(qB, of so"ium: evi"ence of su!erb ki"neys tryin$ to conserve flui" to the best of their ability. %onAt blame them. ?) (*!lanation: 7he correct answer is C. Cystosarcoma !hyllo"es occurs in youn$ women, $rows to hu$e si#e over many years, an" yet s!ares the skin, the no"es, an" the un"erlyin$ chest wall. 7here is no !articular connection with Central America, but often these are seen in immi$rants of limite" financial circumstances, who have ha" no access to me"ical care in their own countries. reast cancer 'choice A) this bi$ an" for these many years, woul" have ulcerate" the skin, woul" be fi*e" to the chest wall, an" woul" have !ro"uce" massive a*illary metastasis. Chronic cystic mastitis 'choice ), also known as mammary "ys!lasia 'choice (), is seen in women of re!ro"uctive a$e, who com!lain of ten"er an" lum!y breasts relate" to the menstrual cycle. ,ar$e cysts can "evelo! in this "isease, but not to the hu$e si#e "escribe" in the vi$nette. Intra"uctal !a!illoma 'choice %) is the most common source of blee"in$ from the ni!!le. 7hese tumors are tiny, <ust a few millimeters in "iameter. /) (*!lanation: 7he correct answer is %. 7he treatment of breast cancer in a !re$nant woman shoul" be the same as that in a non!re$nant woman, e*ce!t for two restrictions: no chemothera!y "urin$ the first trimester, an" no ra"iation thera!y "urin$ the !re$nancy. It is not necessary to terminate the !re$nancy. 7he !referre" treatment for a /4cm tumor woul" be mastectomy 'too bi$ for lum!ectomy). )houl" a*illary no"es be !ositive, systemic thera!y shoul" be "one later. Althou$h the a!!ro!riate sur$ery can be "one "urin$ the !re$nancy, neither chemothera!y 'choice A) nor ra"iation thera!y 'choice ) woul" be acce!table at this time. ,um!ectomy 'choice C) is not a $oo" i"ea for a /4cm tumor. -urthermore, the ra"iation thera!y that must follow lum!ectomy coul" not be $iven in C weeks, while she is still !re$nant. -inally, letAs not terminate a !re$nancy that is "oin$ no harm. 7hera!eutic abortion 'choice () "oes not hel! with the treatment of breast cancer. Neither is the !re$nant woman who $ets cancer of the breast automatically incurable, an" thus only fit for a !alliative !roce"ure. C) (*!lanation: 7he correct answer is %. 7he "ia$nosis of !ericar"ial tam!ona"e shoul" be obvious. 7he !atient has the ty!e of chest woun" that can !ro"uce it, an" the very hi$h central venous !ressure to !rove it.

(vacuation of the bloo" that is !reventin$ normal ventricular fillin$ will !ro"uce instant im!rovement. ,ater, she will nee" re!air of the heart woun" that is !robably the source of the !ericar"ial bloo" an" may also nee" e*!loratory la!arotomy. A chest *4ray 'choice A) woul" never be or"ere" in a "yin$ !atient. 7his !atient is in trouble, an" she nee"s instant action base" on a clinical "ia$nosis. )he woul" "ie while waitin$ for an *4ray. Chest tubes 'choice ) have nothin$ to offer when there are bilateral breath soun"s. In this case, the !atient !robably "oes not have a tension !neumothora* to account for the shock an" the hi$h venous !ressure. %ia$nostic !eritoneal lava$e 'choice C) assumes that the reason for the "eterioration is intra4ab"ominal blee"in$. With a very hi$h central venous !ressure, it is not a reasonable conclusion. Clam!in$ the aorta 'choice () assumes that she is blee"in$ to "eath. )he may be blee"in$, but if that were the cause of her !resent !re"icament, her central venous !ressure woul" be #ero, or near #ero. D) (*!lanation: 7he correct answer is . Althou$h most !atients with hematuria have beni$n "isease, silent hematuria can be "ue to renal, ureteral, or bla""er cancer, an" these mali$nant !rocesses must be effectively rule" out. Intravenous !yelo$ram 'I1&) will visuali#e ki"ney an" ureteral tumors, but is not reliable enou$h to rule out bla""er cancer. %irect visuali#ation of the bla""er mucosa by cystosco!y is the only way to rule out bla""er cancer. 7hus, both !roce"ures are nee"e". Assumin$ hematuria to be a manifestation of clottin$ !roblems or infection 'choice A) is unwarrante" as the first "ia$nostic consi"eration in the absence of a history su$$estive of such !roblems. &rostatic cancer can !ro"uce hematuria when it is a"vance", but ty!ically it "oes not show u! that way in a !reviously asym!tomatic !atient. At a$e C;, this man nee"s a &)A, but this test, alon$ with !rostatic bio!sies 'choice C), woul" "o nothin$ to fin" the source of the hematuria. In !atients with aller$y to the I1& "ye, or with a creatinine above ; m$B", 'neither of which are !resent here), the I1& cannot be "one. In those cases, sono$ram or C7 scan 'choice %) woul" !rovi"e an alternative way to look at the ki"neys. 7he bla""er woul" still remain as a !otential site of un"ia$nose" cancer. 6etro$ra"e stu"ies 'choice () are invasive an" unwarrante" here. A bla""er full of "ye will not necessarily reveal the !resence of a shallow bla""er cancer. 7he collectin$ system outline" by ra"io!aque material woul" not show the renal !arenchyma. @) (*!lanation: 7he correct answer is (. In the !resence of normal !erfusion !ressure, biolo$ical !roblems "o not su""enly "rive the urinary out!ut from normal to #ero. )uch a chan$e is invariably "ue to a mechanical !roblem. Acute renal failure 'choice A) "oes not result in a urinary out!ut of #ero. )ome urine is still !ro"uce", althou$h it is a small volume, on the or"er of / or :> m, !er hour. Intrao!erative "ama$e to the bla""er 'choice ) or the ureters 'choice C) woul" have become obvious imme"iately after the o!eration. %ehy"ration 'choice %)

woul" have !ro"uce" a $ra"ual "ecline in the urinary volume. 7he 0 hours !rece"in$ the onset of the !roblem ha" shown normal values 'about : m, !er k$ of bo"y wei$ht !er hour), with no "ownwar" tren". I) (*!lanation: 7he correct answer is %. ,um!ectomy alone has an unacce!tably hi$h inci"ence of local recurrence, which can be si$nificantly re"uce" by ra"iation thera!y. In a""ition, the !resence of metastatic "isease in the a*illary no"es requires systemic thera!y. As a rule, chemothera!y is !referre" for !remeno!ausal women, which this woman is, but it is also in"icate" here because she is not rece!tor !ositive. Antiestro$ens alone 'choice A) woul" not re"uce the likelihoo" of local recurrence, an" it woul" not hel! much with systemic "isease because she is !remeno!ausal an" rece!tor ne$ative. Conversion to mastectomy 'choice ) is not nee"e" because her sur$ical mar$ins are clear of tumor. 6a"iation alone 'choice C) woul" not suffice because her !ositive a*illary no"es require the a""ition of systemic thera!y. 6a"iation to the o!!osite breast 'choice () is not require" in any event, an" tamo*ifen is the wron$ "ru$ for a !remeno!ausal woman who ha" a rece!tor ne$ative tumor. :>) (*!lanation: 7he correct answer is (. 7he stu"ies show e*tremely mar$inal liver function, which woul" be ti!!e" into overt liver failure by an anesthetic an" an o!eration. 2e is not a sur$ical can"i"ate. Choice A obviously misses the $ravity of his situation. 1itamin 5 'choice ) works only when there is a functionin$ liver that can use it. In the absence of a"equate liver function, it will not correct the !rothrombin time. Albumin 'choice C) can be $iven, but it will have a short life s!an an" will not correct the liver "ysfunction. 7he low albumin is not the main !roblem !er se, it is a sym!tom of how ba" his liver is. 7he same is true of bilirubin 'choice %). It is a sym!tom, not the !roblem. We can o!erate on !atients with much hi$her bilirubin if it is not "ue to intrinsic liver "isease. ::) (*!lanation: 7he correct answer is . Anterior "islocation of the shoul"er is the most common "islocation of that <oint. 7he !osition is classic, as is the lack of the roun"e" contour of the humeral hea". 7he area of numbness re!resents in<ury to the a*illary nerve, a common com!lication of anterior "islocation of the shoul"er. Acromioclavicular se!aration 'choice A) woul" be characteri#e" by very locali#e" !ain at that !articular s!ot an" none of the other features "escribe" here. -racture of the humeral shaft 'choice C) woul" likewise lack the s!ecific "eformity, inasmuch as the humeral hea" woul" still be in !lace to !rovi"e the normal

roun"e" contour. &osterior "islocation 'choice %) ty!ically occurs followin$ massive uncoor"inate" muscle contractions 'electrical in<uries, e!ile!tic sei#ures), an" the arm an" forearm are hel" in a more HnormalH !rotective !osition, close to the bo"y. )ca!ular fracture 'choice () ha!!ens only with e*tremely severe chest traumaG it woul" not ha!!en by fallin$ in the shower. Alon$ with two other in<uries 'fracture of the sternum or fracture of the first rib), sca!ular fracture, when !resent, in"icates that very severe trauma has occurre", an" it is a useful clinical clue to look for hi""en internal in<uries. :;) (*!lanation: 7he correct answer is (. Althou$h this vi$nette "escribes a $unshot woun" of the chest, we must remember that the chest an" the ab"omen are not stacke" u! like !ancakes. 7here is a "ome 4 the "ia!hram 4 that se!arates them, an" thus an area where chest an" ab"omen overla!. Any $unshot woun" below the ni!!les involves the ab"omen, an" such is the case here. 7he mana$ement of all $unshot woun"s of the ab"omen requires e*!loratory la!arotomy. arium swallow an" bronchosco!y 'choices A an" ) are in"icate" if there are si$ns su$$estive of in<ury to those or$ans 'cou$hin$ u! bloo", s!ittin$ u! bloo"), or if the anatomic tra<ectory of the bullet !uts the track in their vicinity. 2ere, we have an entry woun" on the left an" a bullet lo"$e" on the left: the mi"line has not been crosse". 7akin$ out the bullet 'choices C an" %) is unnecessary if the missile is not !ressin$ on some vital structure. A bullet embe""e" in a muscle can be left there. :0) (*!lanation: 7he correct answer is (. Ab"ominal aortic aneurysms have a hi$h inci"ence of ru!ture once they reach or e*cee" a si#e of C cm. Often, the first manifestation is e*cruciatin$ back !ain, as the bloo" leaks into the retro!eritoneal s!ace before the aneurysm blows out into the !eritoneal cavity. 7he combination of a bi$ aneurysm an" su""en severe back !ain shoul" always lea" to this !resum!tive "ia$nosis. ,ookin$ for ortho!e"ic or neurolo$ic e*!lanations can be a "ea"ly mistake. %issectin$ thoracic aortic aneurysm 'choice A) coul" also cause e*cruciatin$ back !ain, but the !ain usually starts as retrosternal an" later mi$rates "own. 7he absence of hy!ertension miti$ates a$ainst this "ia$nosis, an" one woul" e*!ect to see a wi"e me"iastinum on the chest *4ray film. -racture of the s!ine with cor" com!ression 'choice ) coul" in"ee" ha!!en to someone who recently ha" !rostatic cancer, but the sym!toms woul" be !rimarily neurolo$ic "eficits from cor" com!ression. 7he !ain from a herniate" "isc 'choice C) runs "own the le$ an" is e*acerbate" by snee#in$ an" cou$hin$. +etastatic tumor 'choice %) is a $oo" bet in someone with !rostatic cancer. 2owever, the !ain of bony metastasis is !resent for weeks or months, an" is constant, "ull, low $ra"e, an" worse at ni$ht4not the su""en e*cruciatin$ !ain of this vi$nette.

:?) (*!lanation: 7he correct answer is (. 7he !resentation is classic for com!artment syn"rome, tri$$ere" by !rolon$e" ischemia followe" by re!erfusion 'the arm !resse" a$ainst the !ark bench until he woke u! an" chan$e" !osition), an" locate" in one of the two most common sites 'forearm an" lower le$). 2e has the most reliable !hysical fin"in$ '!ain on !assive e*tension), an" the "ia$nosis is not rule" out by normal !ulses. Only a fasciotomy will solve his !roblem. Anal$esics an" observation 'choice A) will result in !ermanent "ama$e to the com!artment muscles. Immobili#ation, by slin$ 'choice ) or cast 'choice C), will allow the hi$h !ressure within the com!artment to continue to "estroy the muscles. (mbolectomy 'choice %) assumes an arterial occlusion, which his normal !ulses rule out. :/) (*!lanation: 7he correct answer is %. 7he clinical !icture is fairly convincin$ for lon$4stan"in$ $astroeso!ha$eal reflu*. 7he main concern is the "e$ree of !e!tic eso!ha$itis that he may have "evelo!e", an" the !ossibility of arrettAs eso!ha$us an" !remali$nant chan$es. (n"osco!y an" bio!sies will !rovi"e the answer. arium swallow 'choice A) woul" !rovi"e anatomic evi"ence of hiatal hernia an" evi"ence of reflu*, but woul" not tell us whether arrettAs eso!ha$us has "evelo!e". Car"iac en#ymes an" (C. 'choice ) woul" be !art of the work4u! 'alon$ with !2 monitorin$) if we were uncertain as to the $enesis of ill4"efine" low retrosternal an" u!!er e!i$astric !ain. 7his man $ives a classic !resentation for reflu*. &roton !um! inhibitors 'choice C) mi$ht likewise be in"icate" for this man, but not until we know the severity an" !otential !remali$nant sta$e of his "isease. Nissen fun"o!lication 'choice () may some "ay be nee"e" here, but one woul" not <um! to a sur$ical solution base" only on a clinical !resentation. :C) (*!lanation: 7he correct answer is C. Clinically, this man has a life4threatenin$ hea" in<ury, with a hi$h !robability that he may have an intracranial hematoma that has to be "raine". C7 scan is the only stu"y that will show such a hematoma. -urthermore, we know that the hea" trauma was severe enou$h to !ro"uce a fracture of the base of the skull 'cerebros!inal flui" "ri!!in$ from the ear)G thus, it may well have !ro"uce" in<ury of the cervical s!ine as well. 7his is likely since he was com!lainin$ of neck !ain, an" it is im!erative that it be "ia$nose" to !rotect his cor", which is !robably still intact 'when he was last awake he still was movin$ all four e*tremities). 7he most e*!e"ient way to "o it is to e*ten" the C7 scan to inclu"e the neck. Antibiotics an" steroi"s 'choice A) are not in"icate". 7he former use" to be $iven for cerebros!inal flui" leaks, but is no lon$er consi"ere" a!!ro!riate. )teroi"s are use" if the cor" is in<ure", but we have reason to believe that it is still intact. Cervical s!ine *4ray films 'choice ) are a $oo" i"ea, but skull *4ray films are not. If his only

!roblem were the cervical s!ine, we woul" in"ee" $o for the *4rays. ut we also have to check his hea", for which we nee" the C7. ,et the C7 take care of both issues. 7he same is true of choice %. We "o not nee" to look into the ear or to stu"y the flui". 7he C7 will show the fracture to the base of the skull, at the same time that it will tell us if an intracranial hematoma has to be "raine". Cerebros!inal flui" leaks cause" by fractures to the base of the skull ty!ically sto! s!ontaneously. )ur$ery is rarely nee"e" for them. When it is nee"e", it is not an emer$ency an" woul" not be "one throu$h the ear. 7hus, choice ( is wron$ on all counts. :D) (*!lanation: 7he correct answer is (. 7he "i$it must be ke!t from "ryin$ out, must not be in<ure" with any chemical a$ents, an" must not be !lace" in "irect contact with ice or allowe" to free#e. %irect contact with ice 'choice A) is one of the "ama$in$ events to be avoi"e". -ree#in$ 'choice ) is absolutely contrain"icate". Alcohol 'choice C) woul" "ama$e the tissues. Antise!tic solutions an" "irect contact with "ry ice 'choice %) woul" "ama$e the fin$er both chemically an" !hysically. :@) malleoli (*!lanation: 7he correct answer is %. 7he clinical "escri!tion is that of a ru!ture of the Achilles ten"on. 7he in<ure" structure is so close to the skin that "irect !al!ation of the $a! in the ten"on is usually !ossible. A fracture of the calcaneus 'im!lie" in choice A) woul" ha!!en with a fall from a hei$ht, lan"in$ on oneAs feet. 7he ability to ben" a <oint beyon" the normal boun"aries 'choices an" C) im!lies "ama$e to the li$aments that kee! that <oint ti$ht. 2owever, such "ama$e woul" not !ro"uce the lou" !o!!in$ noise so characteristic of ru!ture of the Achilles ten"on. .ratin$ an" cre!itation 'choice () are fin"in$s that in"icate bony fractureG if these were !resent, the !atient woul" not be able to !ut wei$ht on the in<ure" si"e. :I) (*!lanation: 7he correct answer is . 7his !atient has or$anic im!otence, but it is not relate" to trauma for which sur$ical reconstruction woul" be in"icate". 2is remainin$ function can be au$mente" with sil"enafil '1ia$ra\). &sychothera!y 'choice A) is the thin$ to "o for !sycho$enic im!otence, which has a su""en onset rather than the $ra"ual "evelo!ment "escribe" in this case. Nerve "ama$e 'as su$$este" in choice C) is the cul!rit in im!otence followin$ !elvic sur$ery 'not the case here). As of now, there is no effective way to reanastomose

those invisible little nerve fibers. &enile !rosthesis 'choice %) is always the last o!tion, never the first one. Once a !rosthesis is inserte", the normal erectile mechanism is "estroye" forever. 2a" the history been that of a youn$ man becomin$ im!otent after a motorcycle acci"ent, a vascular lesion woul" have been the likely !roblem, an" a reconstruction 'choice () woul" be the thin$ to "o. ;>) (*!lanation: 7he correct answer is . (*tracor!oreal shock wave lithotri!sy '()W,) is the most commonly use" metho" to fra$ment urinary stones an" allow their !assa$e. &re$nancy an" coa$ulation !roblems are contrain"ications. 7he first one is rule" out by her a$e, the secon" one we have been tol" she "oes not have. Waitin$ for s!ontaneous !assa$e 'choice A) woul" have been !erfect for a much smaller stone '0 mm) that ha" alrea"y ne$otiate" most of the ureter. A D4mm stone way u! at the uretero!elvic <unction has a very small chance of s!ontaneous !assa$e. 6etro$ra"e en"osco!ic a!!roaches 'choices C an" %) are more invasive than ()W,. 7hey woul" not be the first choice for this scenario. O!en sur$ical removal 'choice () woul" have been $oo" for a much bi$$er stone. A hu$e tar$et 'a stone 0 cm or lar$er) coul" in"ee" be fra$mente" by ()W,, but then we woul" be conten"in$ with "o#ens of still very lar$e stones. In those cases, a "irect a!!roach to e*tract the hu$e intact stone woul" work better. ;:) (*!lanation: 7he correct answer is C. .astrointestinal tract flui"s have a so"ium concentration very close to that of !lasmaG as they are lost 'internally or e*ternally), they shoul" be re!lace" with isotonic, so"ium4 containin$ flui"s. ut that is not what !atients ty!ically "o at home. 7hirsty an" unable to eat soli" 'so"ium4containin$) foo"s, they "rink water, Coke, an" tea, flui"s without si$nificant amounts of so"ium, which the bo"y avi"ly retains because of the severe volume "e!letion. (n"o$enous water from catabolic activity is also retaine". %ilutional hy!onatremia eventually "evelo!s. )he "oes not have Hwater into*icationH 'choice A). 7his term "enotes abnormal water retention "ue to e*cessive water infusion at a time when there is a hi$h level of A%2 in the bloo". 7his !atient is retainin$ water because she is "es!erately volume4"e!lete", not because hi$h volumes of water are bein$ force" into her. 7he hy!onatremia is not "ue to the loss of hy!ertonic flui" 'choice ). 7here are no hy!ertonic flui"s in the $ut, or anywhere else for that matter. 7he only hy!ertonic flui" that we can lose is hi$hly concentrate" urine, but we usually "o so as a !hysiolo$ic res!onse to save water. 9es, we often see comatose an" convulsin$ !atients when they have this much hy!onatremia 'choice %), but that ha!!ens when water retention is massive an" fast. )low water retention allows the brain to a"a!t. One can see even lower serum so"ium concentrations in !atients with a clear sensorium. 1olume "e!rivation lea"s to renal wastin$ of so"ium 'choice () is !lain wron$. What the ki"ney "oes when there is volume "e!letion is to increase reabsor!tion of so"ium, not to "um! it.

;;) (*!lanation: 7he correct answer is (. 7he !resence of an Hacute ab"omen,H which this woman has, is an in"ication for e*!loratory sur$ery an" !rom!t re!air of the in<uries '!robably affectin$ hollow viscera) that have !ro"uce" the si$ns of !eritoneal irritation. Continue" clinical observation 'choice A) woul" be irres!onsible when it is clinically obvious that she alrea"y has an acute ab"omen. What woul" one observe forJ %evelo!ment of se!tic shockJ %eathJ C7 scan 'choice ) is i"eal when the issue is !otential intraab"ominal blee"in$ in a hemo"ynamically stable !atient who can be safely sent to the ra"iolo$y "e!artment. C7 scan mi$ht even be a $oo" i"ea if the !icture of acute ab"omen were equivocal. ut it is not nee"e" here. %ia$nostic !eritoneal lava$e 'choice %) or sono$ram "one in the emer$ency "e!artment 'choice C) are our o!tions when we sus!ect intraab"ominal blee"in$ an" the !atient is too unstable to be sent anywhere. As !ointe" out above, however, when an acute ab"omen has clearly "evelo!e", it is time to o!erate. ;0) (*!lanation: 7he correct answer is A. 7ruly su""en "eath, with no warnin$s whatsoever, brin$s to min" the !ossibility of air embolism. 7he mechanism in this case is su$$este" by the circumstances. 7he !atient ha" "ee! !enetratin$ in<uries that may have involve" a ma<or vein an" an a"<acent bronchus. When he was !lace" on the res!irator, the air was force" throu$h from the tracheobronchial tree into the vein, an" thus into the heart. -at embolism 'choice ) is seen with multi!le lon$ bone fractures, an" the sym!tomatolo$y is res!iratory failure. +yocar"ial infarction 'choice C) woul" be e*tremely unlikely in a youn$ man who was never hy!otensive, an" never showe" arrhythmias. &ulmonary embolus 'choice %) is seen late in the !osto!erative !erio" after several "ays of re"uce" mobility. 7his man woul" have ha" no o!!ortunity to "evelo! clots in ma<or veins in such short clinical course. 7ension !neumothora* 'choice () woul" be unlikely to "evelo! with a chest tube in !lace. 2owever, even if we assume the tube was clo$$e" or kinke" an" thus not functionin$ !ro!erly, a tension !neumothora* "oes not cause su""en "eath: it causes !ro$ressive hemo"ynamic shock an" res!iratory "istress. ;?) (*!lanation: 7he correct answer is %. )li!!e" ca!ital femoral e!i!hysis is an ortho!e"ic emer$ency. 7he clinical !icture is classic: a chubby male in his early teens who is lim!in$ an" cannot rotate his le$ internally. &art of the classic !resentation is also the fact that often hi! !atholo$y !ro"uces knee !ain, but the knee is normal on !hysical e*amination. Avascular necrosis 'choice A) is seen in youn$er chil"ren, aroun" the a$e of C years. %evelo!mental

"ys!lasia 'choice ) is ty!ically "ia$nose" at birth. If it is misse", the affecte" chil" "evelo!s si$nificant sequelae early in life. Osteo$enic sarcoma 'choice C) ha!!ens in the late teens, but the clinical fin"in$s are "ifferent, with a bony mass at the area of ten"erness an" no limitation on the motion of the hi!. 7ibial torsion an" foot inversion 'choice () woul" not limit hi! motion. ;/) (*!lanation: 7he correct answer is (. 7he combination of obstruction an" infection in the urinary tract constitutes a "ire emer$ency that requires, in a""ition to I1 antibiotics, the imme"iate "ecom!ression of the urinary tract above the !oint of obstruction. A""in$ antibiotics without "ecom!ressin$ the urinary tract 'choice A) is not enou$h. 6a!i" "estruction of the ki"ney, an" even "eath from se!tic shock, will ensue if "ecom!ression is not "one. It is too late to crush an" remove the stone 'choice ) once the infection has occurre". Com!licate" instrumentation shoul" not be "one in these circumstances. )uch ste!s shoul" await resolution of the lethal infection4obstruction combination. 7he same can be sai" for the use of e*tracor!oreal shock wave lithotri!sy 'choice C). If it is chosen as the way to mana$e the stone, it shoul" be "one when infection an" obstruction are no lon$er !resent. &uttin$ a catheter into the bla""er 'choice %) woul" !rovi"e "ecom!ression below the level of obstruction. 7he "raina$e of infecte" urine is nee"e" above the obstructin$ !oint. ;C) (*!lanation: 7he correct answer is C. 7here is some evi"ence that hi$h4"ose corticosteroi"s a"ministere" as soon as !ossible after the in<ury will result in a better ultimate outcome. Althou$h the true me"ical value of this observation may be "ebatable, there is a le$al im!erative to use the treatment, which offers some ho!e an" has not been shown to be "etrimental. 2y!erbaric o*y$enation 'choice A) has no role in the acute mana$ement of neurolo$ic in<uries. Antibiotics 'choice ) are likewise unlikely to affect the course of events in a case like this. Althou$h "iuresis 'choice %) is !art of the thera!y use" to "ecrease intracranial !ressure, the a$ent of choice is mannitol, an" the in"ications "o not inclu"e s!inal cor" in<ury. )ur$ical "ecom!ression 'choice () mi$ht be "one, but the "ecision is in"ivi"uali#e" "e!en"in$ on the fin"in$s on +6I. Not all !atients are automatically an" imme"iately taken to the o!eratin$ room. ;D) (*!lanation: 7he correct answer is %. Nu$ular venous "istention in this settin$ is in"icative of con$estive heart failure, a con"ition that woul" make elective sur$ery very risky. +e"ical treatment for con$estive

heart failure can re"uce the risk. Coronary revasculari#ation 'choice A) is a !reo!erative consi"eration for !atients with !ro$ressive an$ina. &reo!erative res!iratory thera!y 'choice ) is nee"e" for !atients with chronic obstructive !ulmonary "isease 'CO&%) who have com!romise" ventilation. A hemo$lobin of :; $B", shoul" be sufficient, an" brin$in$ it u! to :? $B", 'choice C) by transfusions woul" a$$ravate the e*istin$ con$estive failure. Waitin$ C months 'choice () is im!erative before noncar"iac sur$ery is "one after a myocar"ial infarction. In this case, the infarction ha!!ene" ; years a$oG thus, the waitin$ !erio" has alrea"y taken !lace. ;@) "ays (*!lanation: 7he correct answer is C. A rou$h $ui"eline to quantify water loss is that every 0 m(qB, that the serum so"ium concentration is above normal, re!resents about : , of water "eficit. With a value of ://, we can assume a water "eficit of about / ,. 7here is no a"vanta$e to the !atient in remainin$ severely volume contracte" for several "ays, thus the re!lacement shoul" aim for correction in a matter of /4:> hours rather than ; or 0 "ays. 2owever, because his loses were incurre" slowly 'over / "ays), his brain has ha" a chance to a"a!t to the tonicity chan$e 'he is in"ee" awake an" alert). 7hus, the tonicity correction shoul" not ha!!en with the same s!ee" with which the volume is $oin$ to be correcte". 7hat "elay is achieve" by choosin$ a flui" that is not !ure water, but one that has some so"ium in it to "am!en the effect on tonicity. 2alf normal saline is a $oo" choice. / , of %/W over ; or 0 "ays 'choice A) woul" be safe from the view!oint of slowly correctin$ the tonicity, but it woul" unnecessarily !rolon$ the state of volume "e!letion. / , of %/W over /4:> hours 'choice ) coul" well be "ea"ly, because it woul" revert the tonicity to normal at a rate too fast for the brain to follow. Choices % an" ( bu"$et a volume re!lacement well beyon" what is nee"e". Neither woul" be lethal, because %/W is not use", but neither of them is the best answer. ;I) (*!lanation: 7he correct answer is C. 7he a$e, nature of the !ain, !hysical fin"in$s, an" associate" sym!toms are all hi$hly su$$estive of metastatic tumor, an" the source is !robably the lun$. Ankylosin$ s!on"ylitis 'choice A) ha!!ens to youn$er !atients 'in their early 0>s) who have !ain an" stiffness in the mornin$s, an" relief as they become active "urin$ the "ay. 2erniate" "isc 'choice ) can virtually be e*clu"e" by the fact that the !ain is not e*acerbate" by cou$hin$, snee#in$, or strainin$. +ulti!le myeloma 'choice %) is also a "isease of ol" men, but they $et anemia an" multi!le lytic lesions throu$hout the skeleton. &rimary mali$nant bone tumors 'choice () occur in much youn$er !eo!le, an" the e*tremities are a more likely location.

0>) (*!lanation: 7he correct answer is . %eceleration in<ury resultin$ in hea" trauma of sufficient ma$nitu"e to !ro"uce coma an" multi!le facial lacerations may very well have also !ro"uce" in<uries to the cervical s!ine. Althou$h intubation in the fiel" was very a!!ro!riate before the cervical s!ine was visuali#e" 'the !atient was comatose an" ha" si$ns of com!romise" airway), we nee" to know the status of the cervical s!ine before much more is "one to the !atient. )ince C7 is nee"e" to evaluate his hea" in<ury, the most e*!e"itious way to check his cervical s!ine is by e*ten"in$ the C7 to inclu"e the neck. 7he base of the skull 'choice A) will be very well visuali#e" in the C7. No se!arate *4ray films are nee"e". Checkin$ the status of the teeth 'choice C) or the sinuses 'choice () are har"ly the sort of emer$encies that nee" to be a""resse" now. A se!arate C7 scan of the ab"omen 'choice %) woul" have been in"icate" if he ha" been hy!otensive. )o far we have no in"ication of internal blee"in$, an" thus "o not nee" to look for a source. 0:) re!lacement (*!lanation: 7he correct answer is C. 7he !hysical fin"in$s are classic for mitral valve insufficiency. Whenever !ossible, re!air of the native mitral valve is !referable to re!lacement. 7he way to re!air an insufficient valve is to ti$hten the annulus, brin$in$ the leaflets closer to one another. 7here are no !hysical fin"in$s in"icatin$ involvement of the aortic valveG therefore, choices A an" ( are wron$. +itral commissurotomy 'choice ) is the !referre" o!eration for mitral stenosis. 7his !atient has no si$ns of stenosis, an" no history of rheumatic heart "isease to su$$est that she mi$ht have it. +itral valve re!lacement 'choice %) is the choice when re!air of the native valve cannot be "one. 0;) (*!lanation: 7he correct answer is %. 7he clinical !icture is classic for anal fissure, which is !er!etuate" by the fact that the anal s!hincter is Htoo ti$ht.H -orceful "ilatation un"er anesthesia, lateral s!hincterotomy, or botulinum to*in in<ections are acce!table o!tions to Hbreak the cycle.H 7he only one of those choices $iven is the s!hincterotomy. (*cision 'choice A) use" to be "one for this con"ition, before the role of the Htoo ti$ht s!hincterH was eluci"ate". -istulotomy 'choice ) is not the answer. )he has a fissure, not a fistula. Incision an" "raina$e 'choice C) is another o!tion that a""resses a wron$ "ia$nosis. We "o that for !erirectal abscess, which !ro"uces severe !ain with fever an" leukocytosis, but without bloo" streaks, an" "rains s!ontaneously after several "ays if not "ia$nose" an" treate". 6ubber ban" li$ation 'choice () is the answer for internal hemorrhoi"s. Internal hemorrhoi"s can blee", but ty!ically "o not hurt. 7hrombose" e*ternal hemorrhoi"s can hurt tremen"ously, but those are not amenable to rubber ban" li$ation.

00) (*!lanation: 7he correct answer is (. 7his is a new entity known as the ab"ominal com!artment syn"rome, unknown in the "ays when severely traumati#e" !atients "ie" !rior to, or "urin$, sur$ery. 7he life4savin$ massive flui" infusions !ro"uce severe e"ema in the sur$ical fiel". -orce" closure woul" com!romise ventilation an" venous return. A tem!orary !lastic covera$e, or a mesh, allows the bowel to be !rotecte" without un"ue !ressure. Closin$ the skin only 'choice A) can be life4savin$ when hy!othermia "evelo!s "urin$ sur$ery. In this settin$, however, the skin will not come to$ether without un"ue tension. -orce" closure 'choice ) woul" com!romise ventilation an" !ro"uce acute renal failure "ue to !ressure on the inferior vena cava. %iuretics 'choice C) cannot selectively remove the flui" from the swollen tissues. ,eavin$ the bowel e*!ose" to the air 'choice %) is not an o!tion. In the short term, the !atient woul" suffer severe heat lossG later, the bowel woul" "ry out an" !erforate. 0?) (*!lanation: 7he correct answer is (. 7he clinical !icture is that of a stran$ulate" in$uinal hernia. If he only ha" the ten"er mass without si$ns of intestinal obstruction, he mi$ht have omentum tra!!e". If he ha" the intestinal obstruction without fever, leukocytosis, an" the ten"er mass, he coul" be obstructe" but not stran$ulate". ut, the combination that he has is clearly that of obstruction with stran$ulation. 2e nee"s ur$ent sur$ery. A sono$ram to make a "ia$nosis 'choice A) mi$ht be a!!ro!riate for a mass without si$ns of obstruction, if we coul" not clinically be sure that it was a hernia. Naso$astric suction an" I1 flui"s 'choice ) is the stan"ar" a!!roach for obstruction "ue to a"hesions, when there are no si$ns su$$estive of stran$ulation. We "o not o!erate for a"hesions 'they form a$ain), but "o so only to rescue the bowel that is tra!!e". In hernias, on the other han", we want not only to rescue the bowel but also to re!air the hernia. A lon$ rectal tube 'choice C) is use" in O$ilvieAs syn"rome or volvulus, but not in stran$ulate" hernias. +anual re"uction 'choice %) woul" actually be "an$erous in this case, as it mi$ht force a "ea" se$ment of bowel into the ab"omen, increasin$ morbi"ity an" "elayin$ "efinitive treatment. If he ha" no fever, no leukocytosis, an" no ten"erness, such an a!!roach mi$ht be <ustifie" to $ain time for an elective, non4rushe" hernia re!air. 0/) (*!lanation: 7he correct answer is C. 7his is actually a terrible situation, with no easy way out. &elvic fractures can blee" massively, an" often the source is torn veins that are not easily controlle". +inimi#in$ the motion of the bone fra$ments by e*ternal fi*ation can be hel!ful, an" it will not make

the situation worse. &ackin$ the va$ina or rectum 'choice A) woul" hel! if blee"in$ ori$inate" in those or$ans, but they cannot reach the source of blee"in$ in this case. An$io$ra!hy 'choice ) can be very hel!ful when arteries are torn. It cannot "o the same for veins. O!enin$ the fracture" area 'choice %) woul" lose the tam!ona"e effect an" woul" not hel! control the blee"in$. An" as for the sur$eons comin$ to the rescue 'choice (), this is one !lace in which the hi$h an" mi$hty are routinely humble". O!enin$ the !elvic hematoma loses the tam!ona"e effect, an" once into the thick of thin$s, !elvic veins blee" massively an" are not easily controlle". It is best to stay out of these situations. 0C) (*!lanation: 7he correct answer is C. It has been establishe" clinically an" by the laboratory that this !atient is hy!erthyroi", but the thyroi" $lan" "oes not seem to be abnormal. 7he circumstances su$$est that self4 a"ministration of thyroi" hormone for secon"ary $ain 'e.$., wei$ht loss) is a !ossibility. 6a"ioactive io"ine u!take shoul" be hi$h if her $lan" is in"ee" hy!erfunctionin$, but it will be near #ero if it is su!!resse" by the e*o$enous hormone. +ali$nancy is not an issueG thus, fine nee"le as!iration 'choice A) "oes not have a role. 2i$h thyroi" function with un"etectable levels of 7)2 e*clu"es the !ituitary as the source of the !roblem. 7hus, there is no reason to investi$ate it as su$$este" in choice . C !e!ti"e 'choice %) is in"ee" use" to ferret out hormonal self4a"ministration, but it "istin$uishes en"o$enous from e*o$enous insulin, not thyroi" hormone. 70 'choice () nee"s to be "etermine" when clinical si$ns of hy!erthyroi"ism coe*ist with normal levels of 7?. 0D) (*!lanation: 7he correct answer is . Iron "eficiency anemia in the a"ult is always "ue to chronic bloo" loss, an" the source is obvious in this vi$nette: the $astrointestinal tract. In turn, the most likely site, in the absence of other sym!toms, is a cancer of the ri$ht si"e of the colon, which is best seen by colonosco!y. F!!er $astrointestinal series 'choice A) woul" not be likely to reveal the source of this manAs anemia, since the cecum or ascen"in$ colon is the number one tar$et. -le*ible si$moi"osco!y 'choice C) woul" not reach the likely site of the cancer. If the cancer were locate" in the left colon, he woul" likely have visible bloo" in his stools an" a chan$e in bowel habits. F!!er $astrointestinal en"osco!y 'choice %) is the first test when someone vomits bloo". It often will also reveal the source of occult bloo" loss '!e!tic ulcer "isease or as!irin4relate" $astritis) when the colon is foun" to be normal. In this case, however, the cecum or ascen"in$ colon is the number one tar$et. As for visceral an$io$ram 'choice (), it woul" be $reat at the time of massive $astrointestinal blee"in$ 'more than ; m,Bmin), but in this e*am!le it woul" be a very e*!ensive, invasive, an" roun"about way to "emonstrate the !resence of a tumor 'by tumor blush).

0@) (*!lanation: 7he correct answer is C. If a transeso!ha$eal echo !robe is !lace" in every !atient un"er$oin$ femoral reamin$ for fracture re!air, the inci"ence of fat an" !article "ebris in the ri$ht atrium a!!roaches D>=. In fact, a ma<or risk of lower e*tremity ortho!e"ic !roce"ures is !ulmonary embolism "ue to fat or clots. 7he intrame"ullary !ressures $enerate" "urin$ the re!air are $reater than />> !si an" are enou$h to cause venous e*trusion of fat an" other !articulate matter into the circulation. Air embolism 'choice A) is common with neurosur$ical !roce"ures but is not often seen with ortho!e"ic !roce"ures of the lower e*tremity. Althou$h cerebrovascular acci"ent 'choice ) can occur in the absence of a !atent foramen ovale, it is rare. Osteomyelitis 'choice %) is a feare" com!lication of ortho!e"ic sur$ery, an" !recautions such as sterile !re!arations an" antibiotics are taken to $uar" a$ainst it. ecause of this, the com!lication of !ulmonary embolism "ue to fat is much $reater than that of bone infection. &ermanent "isability 'choice (), althou$h a va$ue term, woul" rarely be the result of a lower e*tremity fracture re!air. 0I) (*!lanation: 7he correct answer is %. Althou$h the history of trauma mi$ht su$$est a hematoma or fat necrosis, it is well known that trivial trauma sometimes brin$s to the attention of the !atient an area of the bo"y that ha" not been e*amine" before. A breast mass in a ?;4year4ol" woman requires a mammo$ram an" bio!sy. Waitin$ ; months 'choice A) woul" be unacce!table for a !otential cancer. 2ot !acks an" anal$esics 'choice ) on the assum!tion that this is a hematoma woul" also "elay the "ia$nosis if a cancer is !resent. -urthermore, if this is in"ee" a hematoma one woul" not necessarily want to "rain it. Choice C is incorrect because the mammo$ram is an a"<unct to the bio!sy of a breast mass, not a substitute for it. 7he two stu"ies are com!lementary. +astectomy 'choice () is too ra"ical a ste! before a "ia$nosis has been establishe". ?>) (*!lanation: 7he correct answer is C. 7he left "ia!hra$m can blow out with blunt in<uries, allowin$ the bowel to move u! into the chest. 7he multi!le air4flui" levels su$$est that bowel is in"ee" there, an" the tra<ectory of the naso$astric tube confirms that the ab"ominal viscera 'inclu"in$ the stomach) have been sucke" u! into the thoracic cavity. &ulmonary blebs 'choice A) !ro"uce a !neumothora* when they ru!ture. 7he eso!ha$us 'choice ) virtually never ru!tures with blunt ab"ominal trauma. 9ou nee" a !enetratin$ in<ury, or better yet an en"osco!y, to !erforate it. When that ha!!ens, the outcome is me"iastinitis. A hemo!neumothora* 'choice %) can in"ee" ha!!en in thoracic in<uries, but the *4ray films woul" show one sin$le lar$e air4flui" level, an" the naso$astric tube woul" be in the

stomach, without curlin$ u! into the chest. 7he tracheobronchial tree 'choice () can in"ee" break as a consequence of "eceleration in<uries, but the outcome woul" be a !neumothora* an" air in the me"iastinum an" the subcutaneous tissues. ?:) (*!lanation: 7he correct answer is (. 7he clinical !icture is that of a !ulmonary embolus. Althou$h !ulmonary an$io$ram is the H$ol" stan"ar"H "ia$nostic test, confirmation is usually obtaine" with the less invasive ventilation4!erfusion scan. In some centers, the hi$h4"efinition s!iral C7 scan has been foun" to be a better "ia$nostic mo"ality, an" it is noninvasive. Aorto$ram an" sur$ical re!air 'choice A) assumes the !ain is "ue to a "issectin$ aortic aneurysm. 2a" that been the case, the !ain woul" have been more likely to ra"iate "own towar" the back, an" the !atient woul" have been severely hy!ertensive. (C. an" car"iac en#ymes 'choice ) woul" !robably be "one in this !atient, but only to rule out myocar"ial infarction with $reater certainty. Clinically, we shoul" be sus!ectin$ a !ulmonary embolus, an" ne$ative (C. an" ne$ative en#ymes woul" not s!ecifically confirm the sus!ecte" "ia$nosis. 2y!erventilation 'choice C) is not nee"e" on someone who alrea"y has hy!oca!nia. ,ookin$ for fat "ro!lets 'choice %) is not !articularly useful, even when the clinical "ia$nosis su$$ests fat embolism. -at embolism is more likely to be seen with multi!le comminute" fractures of lon$ bones 'not <ust a broken hi!), an" the clinical manifestations are those of res!iratory failure. 7here woul" be no chest !ain. ?;) (*!lanation: 7he correct answer is C. .unshot woun"s to the base of the neck nee" e*!loratory sur$ery, but the e*act a!!roach an" incision are "etermine" by a more accurate knowle"$e of the location an" e*tent of the in<uries. 7hus, if time !ermits, "ia$nostic stu"ies shoul" !rece"e sur$ical intervention. 7he ma<or vessels, the tracheobronchial tree, an" the eso!ha$us are the !otential tar$ets that have to be investi$ate". Observation 'choice A) mi$ht be a!!ro!riate for a stab woun" in a com!letely asym!tomatic !atient. In $unshot woun"s, we have to e*!ect that in<uries will e*ist, an" they shoul" not be ne$lecte" waitin$ for overt clinical si$ns. C7 scan 'choice ) has "one won"ers for our assessment of close" hea" in<uries an" blunt ab"ominal trauma, but it is not the stu"y that woul" tell us what has ha!!ene" to the ma<or vessels, the eso!ha$us, or the tracheobronchial tree in a $unshot woun". Imme"iate sur$ical e*!loration, either throu$h the neck or the chest, or in combination, mi$ht be force" by a ra!i"ly "eterioratin$ situation. In the absence of such im!erative, a "ecision to o!en the neck 'choice %) or the chest 'choice () is !remature at this !oint. ?0)

(*!lanation: 7he correct answer is %. 7he history an" rectal e*amination fin"in$s are classic for beni$n !rostatic hy!ertro!hy. 7he use of a "econ$estant has le" to stimulation of al!ha a"rener$ic rece!tors, which have further close" the bla""er neck. A bi$, !al!able bla""er has resulte". acterial !rostatitis 'choice A) woul" !ro"uce !ain, fever, an" a very ten"er !rostate on rectal e*amination. Cystitis an" bla""er cancer 'choice ) coul" be e*!ecte" to !ro"uce irritative sym!toms an" hematuria, but not !ainless retention. 6enal failure 'choice C) !ro"uces oli$uria, not anuria. 7he bla""er woul" be em!ty an" thus not !al!able. 7he !atient woul" urinate small amounts an" feel no nee" to urinate more. &rostatic cancer 'choice () is usually first felt as a stony har" no"ule. It woul" be unusual for it to $row to a si#e at which com!lete obstruction is the first manifestation. If it "i", thou$h, the !rostate woul" feel stony har". ??) (*!lanation: 7he correct answer is (. 7he "escri!tion is that of causal$ia, also known as refle* sym!athetic "ystro!hy. If sym!athetic block relieves the sym!toms, !ermanent cure will be obtaine" with sur$ical sym!athectomy. 1enous occlusion 'choice A) woul" !ro"uce swellin$ but not this kin" of !ain. Cervical ribs 'choice ) can !ro"uce neurolo$ic an" vascular sym!toms in the arm, but they are relate" to activity an" !osition an" "o not have the nature "escribe" here. Normal !ulses make arterial insufficiency 'choice C) unlikely. -urthermore, there is no "escri!tion of intermittent clau"ication. Com!artment syn"rome 'choice %) mi$ht have ha!!ene" at the time of in<ury, but if that were the case, it woul" be too late to "o a fasciotomy. ?/) (*!lanation: 7he correct answer is . A very common com!lication of hemorrha$ic !ancreatitis, an" often the reason for the "emise of the !atient, is the "evelo!ment of a !ancreatic abscess. 7he timetable is usually about :>4:? "ays from the onset of the "isease, an" the initial manifestations are fever an" leukocytosis. Chronic !ancreatitis 'choice A) "evelo!s after several years of recurrent attacks of !ancreatitis, an" is characteri#e" by steatorrhea, "iabetes, an" constant !ain. &ancreatic !seu"ocyst 'choice C) is another !otential com!lication of !ancreatitis, but the manifestations are relate" to !ressure sym!toms from the flui" collection, there is no fever or leukocytosis, an" the timetable for "evelo!ment is about C weeks from the onset of the "isease. &elvic abscess 'choice %) an" sub!hrenic abscess 'choice () are in"ee" in the "ifferential "ia$nosis, as they also show u! with fever an" leukocytosis some :>4:? "ays from the ori$inal !roblem. ut, the ori$inal !roblem for these !atients is usually an infectious !rocess in the ab"omen, e.$., a ru!ture" a!!en"i* or a !erforate" viscus. If the !roblem be$an with !ancreatitis, an" then there are si$ns of se!sis, the !ancreas is the lo$ical !lace to harbor the !us.

?C) (*!lanation: 7he correct answer is (. 7he !atient is at hi$h risk for thyroi" cancer 'youn$, male, with a sin$le no"ule an" a history of ra"iation), an" a rea"in$ of Hin"eterminateH in an -NA is a sur$ical in"ication. No further care 'choice A) is totally wron$. It assumes that normal thyroi" function means there is nothin$ wron$ with the thyroi", when in fact thyroi" cancer almost never alters thyroi" function. 7his choice also assumes that if an -NA is not rea" as cancer, the !atient "oes not have that "isease. -ocusin$ on function 'choice ) as the criterion to "o somethin$ is wron$ for the same reasons. 7hyroi" scan an" sono$ram 'choice C) were formerly valuable criteria to select sur$ical can"i"ates 'col" soli" no"ules meant a hi$h risk of cancer), but the -NA !rovi"es a hi$her yiel" of mali$nancy in resecte" s!ecimens, an" thus has ren"ere" the other tests obsolete for this !ur!ose. 6e!eatin$ the -NA 'choice %) assumes that, $iven more cells, the !atholo$ist shoul" be able to "istin$uish beni$n from mali$nant. 7he !atholo$ist has no trouble reco$ni#in$ mali$nant features in !a!illary, me"ullary, or ana!lastic cancers of the thyroi", but cannot "o so with follicular neo!lasms. -ollicular a"enoma an" follicular carcinoma require a look at the entire s!ecimen to tell them a!art. ?D) (*!lanation: 7he correct answer is (. Althou$h we ty!ically associate tension !neumothora* with !enetratin$ chest woun"s, a blunt in<ury can !ro"uce lun$ !uncture, as the <a$$e" e"$es of broken ribs are "riven in at the time of im!act. All the classic fin"in$s of a tension !neumothora* are $iven in the vi$nette. 7racheobronchial in<uries 'choice A) can in"ee" !ro"uce subcutaneous em!hysema. 7hey can also !ro"uce air embolism if ma<or vessels an" ma<or elements of the airway are lacerate" ne*t to one another. When that ha!!ens, thou$h, the clinical manifestation is su""en "eath, ty!ically when the !atient is !lace" on a res!irator. -lail chest 'choice ) is also likely to occur with multi!le rib fractures, but the clinical clue is !ara"o*ical breathin$, an" the eventual !roblem is res!iratory "istress but no hemo"ynamic "ecom!ensation. +assive blee"in$, whether from torn intercostals or ru!ture" aorta 'choices C an" %) woul" in"ee" lea" to hy!ovolemic shock, but the central venous !ressure woul" be #ero an" breathin$ woul" not be !articularly affecte". ?@) (*!lanation: 7he correct answer is A. &ersistent hea"ache in a !atient with recent history of breast cancer '!articularly no"e4 !ositive) is brain metastasis until !roven otherwise. 7he only acce!table course of action is to take a look 'ra"iolo$ically, of course), an" the chea!est an" most reliable way woul" be a C7 scan. -or !rimary brain tumors, the +6I is favore"G however, to show the

!resence of metastasis, an +6I is not nee"e". %es!ite the history of tension hea"aches an" current <ob worries, !sychiatric causes 'choice ) are far "own the line in the "ifferential "ia$nosis. )kull *4rays 'choice C) are almost a vanishin$ test. 7hey may still show linear skull fractures when you are lookin$ for them, but they woul" certainly not show intracranial masses. 1ascular stu"ies 'choice %) were, at one time, the only reliable way to rule out intracranial tumor, but the C7 scan "is!lace" them. We still use them to "efine vascular lesions, which this vi$nette "oes not su$$est. ,umbar !uncture 'choice () woul" not "ia$nose a tumor an" woul" sub<ect the !atient to the risk of herniation of the brainstem. )ave that test for !atients with menin$itis. ?I) (*!lanation: 7he correct answer is C. Cancer of the !rostate will be "iscovere" early by either the "iscovery of a har" no"ule 'as in this case), or the i"entification of elevate" &) A. 7hese are com!lementary e*aminations. One may be normal, while the other may be revealin$. In this case, the recent normal &)A "oes not e*clu"e the nee" to bio!sy this mass. Clinical follow u! 'choice A) is ina!!ro!riate at this a$e, but it is not a com!letely stu!i" o!tion: it woul" be the thin$ to "o if the man ha" been D/. As !ointe" out above, re$ar"less of &)A levels 'choice ) we nee" to bio!sy this mass. A sono$ram 'choice %) mi$ht be nee"e" to i"entify a tumor that is not !al!able, but has been "iscovere" by the &) A. In this case the tumor has been felt. It can be bio!sie", $ui"e" by the fin$er or by a sono$ram if you !refer. ut, the sono$ram will not establish the "ia$nosis, it will only hel! "o the bio!sy. ,et us not resect the !rostate 'choice () before we have a "ia$nosis. %e!en"in$ on the results of the com!lete worku!, one mi$ht elect a "ifferent sur$ical a!!roach or a "ifferent treatment 'ra"iation, for instance). />) (*!lanation: 7he correct answer is A. )mall umbilical hernias can close s!ontaneously u! to the a$e of ; years. 7herefore, if they are asym!tomatic an" not !osin$ an imme"iate risk of stran$ulation, they shoul" be left alone. Obviously, every other answer offere" in this question is wron$, because they all a"vocate a$$ressive thera!y. 2owever, we shall also review other ways in which they mi$ht be incorrect. )clerosin$ a$ents 'choice ) are not a!!ro!riate to mana$e a hernial sac that communicates with the rest of the !eritoneal cavity. Occasionally, if a sur$ical hernia re!air is "one when a lar$e "istal sac has to be left in !lace, it mi$ht be a"visable to "estroy the !eritoneal linin$ with sclerosin$ a$ents so that it "oes not secrete flui". ,a!arosco!ic hernia re!air 'choice C) makes sense when the si#e of the incision or incisions can be si$nificantly re"uce" by the use of la!arosco!y 'for instance a bilateral in$uinal hernia re!air). In this case, however, one woul" nee" bi$$er incisions to intro"uce a 71 camera an" o!eratin$ instruments than one woul" nee" to "irectly close a :4cm su!erficial "efect. (lective o!en sur$ical re!air 'choice %) is what this little $irl

will nee" if she still has the hernia !ast her secon" birth"ay. Fr$ent re!air 'choice () woul" have been in"icate" if the hernia were ten"er, or if the $irl ha" been vomitin$ or $ettin$ "isten"e" at the same time that the hernia became irre"ucible.

USMLE Step 2 Practice Test Block 21 :ame;

+nstr#ctions; Ans'er the *#estions $elo' to the $est of yo#r a$ility. =hen yo# finish the test% click the 2heck $#tton at the $ottom to )ie' the res#lts.

1.A <2!year!ol" man complains of perineal "iscomfort an" reports that there are streaks of fecal soiling in his #n"er'ear. 5o#r months ago% he ha" a perirectal a$scess "raine" s#rgically. Physical e.amination sho's a perineal opening in the skin lateral to the an#s% an" a cor"!like tract can $e palpate" going from the opening to'ar" the insi"e of the anal canal. Bro'nish p#r#lent "ischarge can $e e.presse" from the tract. =hich of the follo'ing is the most likely "iagnosisa6 Anal fiss#re $6Anorectal carcinoma c65ist#la!in!ano "6Piloni"al cyst e6Throm$ose" hemorrhoi"s :ormal La$s 2.A (2!year!ol" man of :or'egian ancestry has a contracte" han" that can no longer $e e.ten"e" an" place" flat on a ta$le. The pro$lem "e)elope" gra"#ally% o)er many years. /e complains of no pain or ne#rologic a$normalities an"% to the e.tent that the "eformity allo's% can mo)e his fingers at 'ill. Physical e.amination "emonstrates the "eformity "escri$e" an" in a""ition sho's the presence of palpa$le fascial no"#les. =hich of the follo'ing is the most likely "iagnosisa62arpal t#nnel syn"rome $6@e N#er)ain tenosyno)itis c6@#p#ytren contract#re "6Palmar tenosyno)itis e6Ahe#matoi" arthritis :ormal La$s &.A 44!year!ol" man complains of )ag#e right #pper a$"ominal "iscomfort that he has ha" for a$o#t 1 month. /e "escri$es no other symptoms% an"% e.cept for en#cleation of one eye at age 21 0for a t#mor%0 he has $een in e.cellent health all his life. /e e.ercises reg#larly an" neither smokes nor "rinks. The only fin"ings on physical

e.amination incl#"e the artificial eye an" a ten"er% enlarge"% an" no"#lar li)er. 2T scan of the #pper a$"omen "emonstrates m#ltiple masses 'ithin the li)er. =hich of the follo'ing 'ill most likely $e fo#n" on $iopsy of these massesa6Metastatic malignant melanoma $6Metastatic prostatic cancer c6Metastatic retino$lastoma "6Metastatic sarcoma e6Primary hepatocell#lar carcinoma :ormal La$s 4. A (1!year!ol" =est Te.as farmer of +rish ancestry has a nonhealing% in"olent% p#nche" o#t% clean!looking 2!cm #lcer o)er the left temple. The #lcer has $een slo'ly gro'ing o)er the past & years. There are no enlarge" lymph no"es in the hea" an" neck. =hich of the follo'ing 'o#l" $est "ictate proper managementa6 5#ll thickness $iopsy of the center of the lesion $65#ll thickness $iopsy of the e"ge of the lesion c6 Pathologic st#"ies after the entire lesion is resecte" 'ith a margin of 1 cm of normal skin all aro#n" "6Aesponse to a trial of ra"iation therapy e6Scrapings an" c#lt#re of the #lcer $ase :ormal La$s . A pe"iatrician,s office gets a phone call from a frantic mother. /er (! year!ol" "a#ghter 'as playing #n"er the sink an" acci"entally spille" Li*#i" Pl#m$er 3a strong% corrosi)e alkaline "rain cleaner6 all o)er her arms an" legs. The n#rse on the phone can hear the girl screaming in the $ackgro#n". =hich of the follo'ing are the most appropriate instr#ctions to gi)e the mothera6 2o)er the $#rne" areas 'ith triple anti$iotic ointment #ntil the girl can $e seen at the office $6Cet the girl into the sho'er right a'ay an" keep the 'ater r#nning o)er her for &4 min#tes $efore $ringing her to the emergency "epartment c6Cet the girl to the emergency "epartment as soon as possi$le

"6 =ash the $#rne" areas 'ith "il#te" )inegar an" $ring the girl to the office e6=rap the $#rne" areas in sterile "ressings $efore $ringing the girl to the emergency "epartment :ormal La$s <.A 8!year!ol" man s#stains $l#nt tra#ma in an a#tomo$ile acci"ent% res#lting in m#ltiple intra!a$"ominal in9#ries. S#rgery for the repair of these in9#ries takes se)eral ho#rs an" re*#ires m#ltiple $loo" transf#sions an" inf#sions of Ainger,s lactate. Before all the operati)e steps are complete"% the patient "e)elops a significant coag#lopathy% a core $o"y temperat#re less than &4 2 38&.2 56% an" refractory aci"osis. The anesthesiologists are a"ministering fresh froDen plasma an" platelet packs. =hich of the follo'ing is the most appropriate ne.t step in management for the s#rgeona6Pro)i"e hemostasis $y li$eral #se of electrocoag#lation $6 =ash the a$"omen 'ith 'arm saline an" contin#e to operate c62omplete the operation as soon as possi$le an" "o a formal a$"ominal clos#re "6Pack the $lee"ing s#rfaces an" close the a$"omen temporarily 'ith to'el clips e6 A$ort the operation an" lea)e the a$"omen open% co)ering the $o'el 'ith mesh :ormal La$s (.A 42!year!ol" man "escri$es intermittent episo"es of se)ere% cr#shing chest pain that e.ten"s to the $ack an" the 9a' an" last any'here from a fe' secon"s to se)eral min#tes. Many times the pain is accompanie" $y "ysphagia an" triggere" $y the ingestion of )ery col" or )ery hot li*#i"s. /o'e)er% sometimes the pain occ#rs for no apparent reason. There is no history of reg#rgitation% an"% altho#gh the pro$lem has $een present for many years% there has $een no progression of the symptoms. Aepeate" E2Cs an" car"iac enDymes ha)e al'ays $een negati)e. Bari#m s'allo' sho's an area of 0corkscre'0 appearance. Manometry sho's that a$o#t one half of 'et s'allo's pro"#ce repetiti)e sim#ltaneo#s esophageal contractions of the esophageal $o"y% an" that

the lo'er esophageal sphincter has normal press#res an" e.hi$its normal rela.ation. =hich of the follo'ing is the most likely "iagnosisa6Achalasia of the esophag#s $62ancer of the lo'er esophag#s c6@iff#se esophageal spasm "6:#tcracker esophag#s e6Oenker,s "i)ertic#l#m :ormal La$s 1.A 2!year!ol" 'oman has terminal cer)ical cancer 'ith e.tensi)e pel)ic in)asion% an" re*#ires strong opiate analgesia to control se)ere pain. +ntermittent na#sea an" )omiting ha)e precl#"e" the #se of oral agents. She 'as on fentanyl patches $#t she "e)elope" allergic skin reactions to the a"hesi)e an" no' re*#ests to $e s'itche" to parenteral me"ication. Beca#se of prior chemotherapy treatments she has no a)aila$le )eno#s access% $#t she is 'illing to ha)e her family a"minister intram#sc#lar in9ections. Ass#ming e*#ianalgesic "osages% 'hich of the follo'ing 'o#l" $e the most appropriate pharmacotherapya62o"eine $6/y"romorphone 3@ila#"i"6 c6Meperi"ine 3@emerol6 "6Metha"one e6Morphine s#lfate :ormal La$s 8.A 4&!year!ol" 'oman comes in $eca#se of a $reast mass. T'o "ays ago% she notice" a l#mp on self!e.amination. She has a 2!cm% firm% nonten"er mass in the left $reast% 'hich is mo)a$le from the chest 'all% $#t not mo)a$le 'ithin the $reast. She has no prior history of $reast "isease% $#t she is 'ell rea" an" 'ell informe"% an" she specifically re*#ests that a $iopsy $e "one 'ith a mammotome. Before procee"ing% 'hich of the follo'ing is the most appropriate initial stepa6@isc#ss the s#rgical options in case cancer is fo#n" $6@o a mammogram to ascertain 'hether $iopsy is nee"e" c6 @o a mammogram to fin" any other lesions that might also nee" to $e a""resse" "6 5irst 'ait for t'o menstr#al cycles to see 'hether there is spontaneo#s resol#tion.

e6 >$tain a fine!nee"le aspirate an" go no f#rther if no malignant cells are fo#n" :ormal La$s 14. A & !year!ol" man falls on an o#tstretche" han" an" comes in complaining of 'rist pain. /e relates that he 'as not a$le to $reak the fall% an" that the heel of his han" took the $r#nt of his f#ll 'eight as it hit the pa)ement. >n physical e.amination% he is "istinctly ten"er to palpation o)er the anatomic sn#ff $o.. Anteroposterior an" lateral .! rays are negati)e. =hich of the follo'ing are the most likely "iagnosis an" most appropriate ne.t step in managementa62arpal na)ic#lar fract#reG th#m$ spica cast $6@e N#er)ain tenosyno)itisG steroi" in9ections c6 @isplace" scaphoi" fract#reG open re"#ction an" internal fi.ation "6Ligamento#s in9#ryG Ace $an"age an" analgesics e6:o fract#reG reass#rance :ormal La$s 11. A (2!year!ol" 'oman has a re"% s'ollen $reast. She states that the con"ition has $een present for at least se)eral 'eeks% perhaps a month or t'o. She has no pain or fe)er. The skin o)er the area looks like orange peel. The area is not 'arm to the to#ch% $#t on physical e.amination there is a f#llness to the entire $reast% 'ith no "iscrete mass. =hich of the follo'ing is the most likely "iagnosisa62hronic cystic mastitis $6+nflammatory cancer of the $reast c6:ormal menopa#sal in)ol#tionary changes "6Pyogenic $reast a$scess e6 T#$erc#lo#s or f#ngal $reast a$scess :ormal La$s 12. A 4<!year!ol" 'oman% 'ho ha" al'ays $een in goo" health% comes in $eca#se of the s#""en onset of )ery se)ere $ack an" leg pain that she e.perience" 2 ho#rs ago 'hen attempting to lift a hea)y o$9ect. She says that she felt 0a $olt of lightening0 r#nning "o'n the $ack of her leg% an" she still has )ery se)ere pain that pre)ents her from 'alking or mo)ing. The pain is e.acer$ate" $y co#ghing% sneeDing% or straining.

She keeps the affecte" leg fle.e"G straight leg raising gi)es her e.cr#ciating pain. She has goo" sphincteric tone an" intact sensation in the perine#m. >nce the "iagnosis is confirme" 'ith the appropriate st#"ies% 'hich of the follo'ing 'ill $e the most appropriate treatmenta6Analgesics an" $e" rest for a$o#t & 'eeks $6Appropriate anti$iotics c6 Bo"y cast for &!< months "6Aa"iotherapy to the affecte" area e6 S#rgical "ecompression :ormal La$s 1&. A <!year!ol" man "e)elops slo'% progressi)e paralysis of the facial ner)e on one si"e. +t took se)eral 'eeks for the f#ll!$lo'n paralysis to $ecome o$)io#s% an" it has $een present no' for & months. +t affects $oth the forehea" an" the lo'er face. /e has no pain any'here% an" no palpa$le masses $y physical e.amination. =hich of the follo'ing is the most likely "iagnosisa6 Bell,s palsy $65acial ner)e t#mor c6/emorrhagic stroke "6Paroti" glan" cancer e6 Pleomorphic a"enoma of the paroti" glan" :ormal La$s 14. A yo#ng mother complains of pain along the ra"ial si"e of the 'rist an" the first "orsal compartment. She relates that the pain is often ca#se" $y the position of 'rist fle.ion an" sim#ltaneo#s th#m$ e.tension that she ass#mes to carry the hea" of her $a$y. >n physical e.amination% the pain is repro"#ce" $y asking her to hol" her th#m$ insi"e her close" fist% an" then forcing the 'rist into #lnar "e)iation. =hich of the follo'ing is the most likely "iagnosisa6Ac#te an" chronic $#rsitis $62arpal t#nnel syn"rome c6/airline #nrecogniDe" fract#re of the carpal na)ic#lar 3scaphoi"6 $one "6 Palmar fascial contract#re 3@#p#ytren,s contract#re6 e6 Tenosyno)itis of the a$"#ctor or e.tensor ten"ons of the th#m$ 3@e N#er)ain,s tenosyno)itis6 :ormal La$s

1 . A 44!year!ol" homeless 'oman presents to the emergency "epartment $eca#se she is 0$lee"ing from the $reast.0 Physical e.amination sho's a h#ge% f#ngating% #lcerate" mass that occ#pies the entire right $reast an" is firmly attache" to the chest 'all. The right a.illa is f#ll of har" masses that are not mo)a$le either. 2ore $iopsies of the $reast are rea" as highly #n"ifferentiate" infiltrating "#ctal carcinoma% an" assay for estrogen an" progesterone receptors are negati)e. =hich of the follo'ing is the most appropriate ne.t step in managementa6 A 44!year!ol" homeless 'oman presents to the emergency "epartment $eca#se she is 0$lee"ing from the $reast.0 Physical e.amination sho's a h#ge% f#ngating% #lcerate" mass that occ#pies the entire right $reast an" is firmly attache" to the chest 'all. The right a.illa is f#ll of har" masses that are not mo)a$le either. 2ore $iopsies of the $reast are rea" as highly #n"ifferentiate" infiltrating "#ctal carcinoma% an" assay for estrogen an" progesterone receptors are negati)e. =hich of the follo'ing is the most appropriate ne.t step in management$6Tamo.ifen therapy c6Aa"iation an" chemotherapy "6Palliati)e mastectomy e6 Aa"ical mastectomy 'ith e.ten"e" lymph no"e "issection :ormal La$s 1<.A $lon"% $l#e!eye"% <8!year!ol" sailor has a non!healing% in"olent% 1. !cm #lcer on the lo'er lip% arising from the )ermilion $or"er. The #lcer has $een present an" gro'ing for the past 1 months. /e is a pipe smoker% $#t has no history of alcohol or "r#g a$#se. Physical e.amination sho's 0'eather!$eaten0 facial skin% $#t no other #lcers. There are no enlarge" lymph no"es in his neck. =hich of the follo'ing is the most likely "iagnosisa6A"enocarcinoma $6Basal cell carcinoma c6Benign #lceration "#e to chronic tra#ma "6+n)asi)e malignant melanoma e6S*#amo#s cell carcinoma :ormal La$s

1(.A 4!year!ol" African American man% 'ith a history of smoking an" "rinking% "escri$es progressi)e "ysphagia that $egan & months ago. /e first notice" "iffic#lty s'allo'ing meatG it then progresse" to other soli" foo"s% then to soft foo"s% an" no' to li*#i"s as 'ell. /e locates the place 'here the foo" 0sticks0 at the lo'er en" of the stern#m. /e has lost &4 po#n"s. =hich of the follo'ing is the most appropriate first step in "iagnosisa6 Bari#m s'allo' $6Castrografin s'allo' c6Esophageal manometry "6Esophageal p/ monitoring e6Esophagoscopy :ormal La$s 11.A &8!year!ol" 'oman complete" her last co#rse of postoperati)e a"9#)ant chemotherapy for $reast cancer < months ago. She no' comes to the clinic complaining of constant $ack pain for a$o#t & 'eeks. She is ten"er to palpation o)er t'o 'ell!circ#mscri$e" areas in the thoracic an" l#m$ar spine. =hich of the follo'ing is the most appropriate ne.t step in managementa62T scan of the tr#nk $6:ee"le $iopsy of the ten"er spots c6Aa"ion#cli"e $one scan "6Sonogram of the affecte" areas e6F!ray films of the affecte" areas :ormal La$s 18.+n preparation for an ing#inal hernia repair% a 22!year!ol" man has a spinal anesthetic place". The le)el of sensory $lock t#rns o#t to $e m#ch higher than ha" $een planne"% an" shortly thereafter his $loo" press#re "rops to ( 724 mm /g. /e looks 'arm an" fl#she"% an" his central )eno#s press#re is near Dero. =hich of the follo'ing sho#l" $e incl#"e" in his therapya6@i#retics an" fl#i" restriction $6=hole $loo" an" clotting factors c6+notropic agents an" car"iac assist p#mp

"6?asoconstrictors an" +? fl#i"s e6?aso"ilators an" +? fl#i"s :ormal La$s 24.A &&!year!ol" 'oman is fo#n" to ha)e a palpa$le thyroi" no"#le "#ring a ro#tine me"ical check#p. A sonogram confirms the presence of a soli"% 1. !cm no"#le in the right lo$e of the thyroi" glan". 5ine nee"le aspirate cytology 35:A6 is reporte" as 0follic#lar t#mor% other'ise #nspecifie".0 At s#rgery% a froDen section is rea" as follic#lar carcinoma. =ith the neck open% the s#rgeon can feel for enlarge" 9#g#lar an" peritracheal lymph no"es% an" fin"s none. =hich of the follo'ing is the most appropriate treatmenta6 En#cleation of the t#mor $6Aight thyroi" lo$ectomy c6Total thyroi"ectomy "6 Total thyroi"ectomy pl#s postoperati)e ra"ioacti)e io"ine e6Total thyroi"ectomy% ra"ical neck "issection% an" postoperati)e ra"ioacti)e io"ine :ormal La$s 21. A 4 !year!ol" 'oman% 'ho 'ears high!heele"% pointe" shoes% complains of pain in the forefoot after prolonge" stan"ing or 'alking. >ccasionally% she also e.periences n#m$ness% a $#rning sensation% an" tingling in the area. Physical e.amination sho's no o$)io#s "eformities an" a )ery ten"er spot in the thir" interspace% $et'een the thir" an" fo#rth toes. There is no re"ness% limitation of motion% or signs of inflammation. =hich of the follo'ing is the most likely "iagnosisa6Co#t $6/all#. rigi"#s c6Metatarsophalangeal artic#lation pain "6Morton,s ne#roma e6Plantar fasciitis :ormal La$s 22. A 22!year!ol" gang mem$er arri)es in the emergency "epartment 'ith m#ltiple g#nshot 'o#n"s to the chest an" a$"omen. /e has la$ore" $reathing an" is cyanotic% "iaphoretic% col"% an" shi)ering. /e

is 'i"e a'ake% an" in a normal tone of )oice he tells e)eryone that he is going to "ie. An initial s#r)ey re)eals a $loo" press#re of <4744 mm /g. /is p#lse is 1 47min an" $arely percepti$le. /e is in o$)io#s respiratory "istress an" has $ig "isten"e" )eins in his neck an" forehea". /is trachea is "e)iate" to the left% an" the right si"e of his chest is hyperresonant to perc#ssion% 'ith no $reath so#n"s. =hich of the follo'ing is the most appropriate initial step in managementa6Emergency $loo" gases $6+mme"iate chest .!ray films c6A'ake en"otracheal int#$ation "6A 1<!ga#ge nee"le inserte" in the secon" right intercostal space e6Pericar"iocentesis :ormal La$s 2&. A <<!year!ol" 'oman picks #p a $ag of groceries o#t of the s#permarket cart to place it in the tr#nk of her car. As she "oes so% she feels sharp% s#""en pain in the mi""le of her arm% an" her h#mer#s s#""enly $reaks. She arri)es at the emergency "epartment cra"ling her armG the "eformity lea)es no "o#$t that the $one is $roken. =hich of the follo'ing is the most likely reason for the fract#rea6Bony metastasis to the h#mer#s from $reast cancer $6>steitis fi$rosa cystica from parathyroi" "isease c6>steomalacia from n#tritional "eficiency "6>steoporosis e6Primary malignant $one t#mor :ormal La$s 24.A <2!year!ol" man has ha" gastroesophageal refl#. "isease "iagnose" $y p/ monitoring% an" present for se)eral years. /e has $een less than totally compliant 'ith me"ical management% 'hich he follo's 'hen the pain is $a"% $#t "iscontin#es 'hen he feels $etter. En"oscopy an" $iopsies sho' se)ere peptic esophagitis% 'ith Barrett,s esophag#s an" early "ysplastic changes% $#t no o)ert carcinoma. A""itional tests sho' goo" esophageal motility% 'ith lo' press#re in the lo'er esophageal sphincter an" normal gastric emptying. =hich of the follo'ing is the most appropriate treatment at this timea6/eller myotomy of the lo'er esophageal sphincter $6Laparoscopic :issen f#n"oplication c6Transhiatal total esophagectomy "6Transthoracic resection of the lo'er esophag#s

e6 ?agotomy% pyloroplasty% an" f#n"ic gastric 'rap :ormal La$s 2 . A 14!year!ol" $oy sli"es "o'n a $anister an" crashes into a large ornamental kno$ at its $ase% there$y in9#ring his scrotal contents. /e presents in the emergency "epartment 'ith ac#te testic#lar pain an" a scrotal hematoma the siDe of a grapefr#it. /e is a$le to )oi" normally% an" his #rine "oes not contain $loo". A rectal e.amination is #nremarka$le. 5in"ings from 'hich of the follo'ing tests 'ill most likely "etermine f#rther therapya6 Aspiration of scrotal contents $6Aetrogra"e cystogram c6Aetrogra"e #rethrogram "6Scrotal sonogram e6Scrotal s#rgical e.ploration :ormal La$s 2<.A 44!year!ol"% o$ese% 'hite 'oman% mother of fi)e chil"ren% gi)es a history of repeate" episo"es of right #pper *#a"rant a$"ominal pain. The pain is $ro#ght a$o#t $y the ingestion of fatty foo"s an" is relie)e" $y the a"ministration of anticholinergic me"ications. The pain is colicky% ra"iates to the right sho#l"er an" aro#n" to the $ack% an" is accompanie" $y na#sea an" occasional )omiting. The patient has no pain at this time% $#t is an.io#s to a)oi" f#rther episo"es. She is afe$rile% an" physical e.amination is #nremarka$le. =hich of the follo'ing is the most appropriate ne.t step in managementa6Sonogram of the $iliary tract an" gall$la""er $6Upper gastrointestinal series 'ith $ari#m c6Anti$iotics% +? fl#i"s% an" nothing $y mo#th "6En"oscopic retrogra"e cholangiopancreatogram 3EA2P6 e6E.ploratory s#rgery :ormal La$s 2(. A 28!year!ol" man presents 'ith a 2!"ay history of se)ere left!si"e" scrotal pain an" s'elling. /e is se.#ally acti)e an" has ha" 0many0 se.#al partners. /is temperat#re is &1.2 2 3144.1 56% $loo" press#re is 1247(4 mm /g% an" p#lse is 147min. E.amination sho's #nilateral

intrascrotal ten"erness an" s'elling. Testic#lar s#pport makes the pain less intense. =hich of the follo'ing is the most likely "iagnosisa6 Epi"i"ymitis $6Prostatitis c6 Testic#lar torsion "6 Urethritis e6?aricocele :ormal La$s 21. A pe"estrian is hit $y a car. The parame"ics report that he 'as #nconscio#s at the site% an" he arri)es at the emergency "epartment in coma% strappe" to a hea" $oar" 'ith san"$ags on either si"e of his hea". +nitial s#r)ey sho's sta$le )ital signs% an" his p#pils are of e*#al siDe an" reacti)e to light. /e is rapi"ly int#$ate" $y the nasotracheal ro#te o)er a fle.i$le $ronchoscope an" then sent for 2T scans of the hea". As he is $eing positione" on the ta$le% it is note" that there is a siDa$le hematoma $ehin" his right ear an" that clear fl#i" is "ripping from the ear canal. =hich of the follo'ing is most a")isa$le% consi"ering this ne' fin"inga6 E.ten" the 2T scan to incl#"e his neck $6@o an MA+ instea" of a 2T scan c6Start anti$iotics "6+n9ect high!"ose corticosteroi"s e6Plan an emergency craniotomy :ormal La$s 28. @#ring a campaign appearance% a political can"i"ate is shot point $lank in the right chest 'ith a .22 cali$er re)ol)er. The entrance 'o#n" is 'ell a$o)e the nipple line% 9#st #n"er the thir" ri$% at the le)el of the anterior a.illary line. /is motorca"e $rings him to the emergency "epartment% $#t he makes it a point to 'alk in% hol"ing his right chest 'ith a $loo"y han" an" 'a)ing for the ne's me"ia. A chest .!ray sho's a hemothora. on the right% an" the $#llet is seen to $e em$e""e" in the right paraspino#s m#scles. A chest t#$e is place" in the right ple#ral ca)ity% an" < 4 mL $loo" is reco)ere". >)er the ens#ing 4 ho#rs% he contin#es to "rain $et'een 2 4 an" & 4 mL $loo" per ho#r. =hich of the follo'ing is the most appropriate ne.t step in managementa62ontin#e" o$ser)ation an" appropriate $loo" replacement $6 A secon" chest t#$e in a $etter position to "rain the $loo"

c6Thoracotomy an" ligation of $lee"ing )essels "6Thoracotomy% ligation of $lee"ing )essels% an" remo)al of the $#llet e6Thoracotomy an" pne#monectomy :ormal La$s &4. @#ring the performance of a s#pracla)ic#lar no"e $iopsy #n"er local anesthesia% a hissing so#n" is s#""enly hear"% an" the patient s#""enly "ies. At the time of the catastrophic e)ent% the target no"e 'as #n"er traction% an" the final c#t 'as $eing ma"e $lin"ly $ehin" it to free it #p completely. The patient% an other'ise healthy 24!year!ol" man% 'as inhaling at that moment. =hich of the follo'ing most likely ca#se" this patient,s "eatha6 Arterial in9#ry 'ith air em$oliDation $6Ma9or )ein in9#ry 'ith air em$olism c6S#""en pne#mothora. 'ith l#ng collapse "6Sympathetic "ischarge e6Tracheal in9#ry :ormal La$s &1.An el"erly 'oman 'ith osteoporosis falls on her o#tstretche" han". She comes in 'ith a "eforme" an" painf#l 'rist that looks like a "inner fork. F!ray films sho' a "orsally "isplace"% "orsally ang#late" fract#re of the "istal ra"i#s. There is also an associate" fract#re of the #lnar styloi". A ne#rologic e.amination is normal. =hich of the follo'ing is the most appropriate managementa62lose" re"#ction an" short arm cast $62lose" re"#ction an" long arm cast c6Skeletal traction "6+ntrame"#llary ro" e6>pen re"#ction an" internal fi.ation :ormal La$s &2.A 4&!year!ol" man "e)elops e.cr#ciating a$"ominal pain at 1;2& PM 3he looke" at his 'atch 'hen the pain 0hit him06. =hen seen in the emergency "epartment a$o#t &4 min#tes later% he has a rigi" a$"omen% lies motionless on the e.amination ta$le% has no $o'el so#n"s% an" is o$)io#sly in great pain% 'hich he "escri$es as constant an" encompassing the entire a$"omen. There is )ery se)ere pain 'hen "eep

palpation of the a$"omen is attempte" in any of the fo#r *#a"rants. /o'e)er% the e.amining han" cannot make m#ch of an in"entation $eca#se of the impressi)e m#scle g#ar"ing. =hen the attempt is a$orte"% he manifests se)ere re$o#n" ten"erness. F!ray films sho' free air #n"er $oth "iaphragms. =hich of the follo'ing "oes this man most likely ha)ea6Ac#te a$"omen% the nat#re of 'hich cannot yet $e "efine" $6Ac#te inflammatory process affecting an intra!a$"ominal )iscera c6Ac#te o$str#ction of an intra!a$"ominal )iscera "6+schemic process affecting intra!a$"ominal organs e6Perforation of the gastrointestinal tract :ormal La$s &&. >n the th postoperati)e "ay after a$"ominal s#rgery% a patient has $een "raining copio#s amo#nts of clear pink fl#i" from his mi"line laparotomy 'o#n". A me"ical st#"ent remo)es the "ressing% confirms that it is soake"% an" sees a normal!appearing fresh 'o#n" 'ith a ro' of skin staples in place. The st#"ent asks the patient to sit #p so he can get o#t of $e" an" $e helpe" to the treatment room for a more thoro#gh e.amination. =hen the patient complies% the 'o#n" opens 'i"ely% an" a han"f#l of small $o'el s#""enly r#shes o#t. =hich of the follo'ing is the most appropriate management at this timea62o)er the $o'el 'ith "ry sterile "ressings an" sche"#le #rgent s#rgical clos#re $6 2o)er the $o'el 'ith sterile "ressings soake" in 'arm saline an" r#sh the patient to the operating room c6+rrigate the $o'el 'ith col" antiseptic sol#tions 'hile a'aiting #rgent s#rgical clos#re "6 Take the patient to the treatment room an" s#t#re the skin e"ges together e6 =earing sterile glo)es% p#sh the $o'el $ack in an" tape the 'o#n" sec#rely :ormal La$s &4.A ((!year!ol" man $ecomes 0senile0 o)er a perio" of & or 4 'eeks. /e #se" to $e acti)e an" manage" all of his financial affairs. :o'% he stares at the 'all% $arely talks% an" sleeps most of the "ay. /is "a#ghter recalls that he fell from a horse a$o#t a 'eek $efore the mental changes

$egan. =hich of the follo'ing 'o#l" a 2T scan of his hea" most likely sho'a6 2hronic epi"#ral hematoma $62hronic s#$"#ral hematoma c6@iff#se intracere$ral $lee"ing "65rontal lo$e infarction e6CeneraliDe"% se)ere $rain atrophy :ormal La$s & . A man 'ho 'eighs < kg s#stains secon" an" thir" "egree $#rns o)er $oth of his lo'er e.tremities 'hen his pants catch on fire. =hen e.amine" shortly thereafter% it is ascertaine" that )irt#ally all of the skin from $oth groins to the tip of the toes% front an" $ack% has $een $#rne". Accor"ing to the mo"ifie" Parklan" form#la% 'hich of the follo'ing is the appro.imate total amo#nt of +? fl#i" that he can $e e.pecte" to re*#ire "#ring the first 24 ho#rs post!$#rna6&4<4 mL $648<4 mL c6<(<4 mL "611<4 mL e611%&<4 mL :ormal La$s &<.A 48!year!ol" 'oman has a firm% 2!cm mass in the right $reast that has $een present for & months. Mammogram has $een rea" as 0cannot r#le o#t cancer%0 $#t it cannot "iagnose cancer either. A fine!nee"le aspiration of the mass 35:A6 an" cytology "o not i"entify any malignant cells. =hich of the follo'ing is the most appropriate ne.t step in managementa6Aeass#rance an" reappointment in a year $6Aepeat mammogram an" 5:A in 1 month c62ore or incisional $iopsies "6L#mpectomy an" a.illary "issection e6Mo"ifie" ra"ical mastectomy :ormal La$s

&(. A yo#ng mother is at the pe"iatrician,s office for a ro#tine 'ell!$a$y )isit for her 11!month!ol" son. +t is imme"iately notice" that one of the $a$y,s p#pils is 'hite% 'hile the other one is $lack. =hen aske" a$o#t it% the mother relates that she sa' that c#rio#s sit#ation for the first time 1 'eek ago% $#t since the $a$y 'as other'ise asymptomatic% she "i" not think it merite" special attention. =hich of the follo'ing is the most appropriate co#rse of actiona6@o nothing% this is a normal anatomic )ariant $6 +n*#ire if the father is an al$ino% an" "o appropriate genetic co#nseling c6Seek an ophthalmologic cons#ltation for s#specte" congenital cataract "6Seek an emergency ophthalmologic cons#ltation for possi$le retino$lastoma e6 Treat the chil" 'ith anti$acterial eye "rops an" re!check in 2 'eeks :ormal La$s &1. A 2&!year!ol" man kno'n to ha)e ne#rofi$romatosis% type 1 3)on Aecklingha#sen,s "isease6% presents 'ith a left lo'er *#a"rant a$"ominal mass an" signs of ne#rologic "eficits in his left leg. +n the ens#ing 'ork#p% it is "etermine" that he has higher than normal )al#es of cata$olites of epinephrine an" norepinephrine in a 24!ho#r #rinary collection. /e is c#rrently normotensi)e. Before in)asi)e steps are taken to $iopsy an" e)ent#ally remo)e his left lo'er *#a"rant a$"ominal mass% 'hich of the follo'ing is the most appropriate ne.t step in managementa62T scan of the hea" looking for meningiomas $6MA+ of his a"renal glan"s c6MA+ of the aco#stic ner)es "6Aa"ion#cli"e scans from the neck to the pel)is looking for e.tra! a"renal pheochromocytomas e6Aa"iation therapy to the left lo'er *#a"rant a$"ominal mass :ormal La$s

&8. A car is in)ol)e" in a hea"!on collision. The "ri)er% 'ho is so$er an" 'earing his seat $elt% e.plains that he clearly sa' his "r#nk% #nrestraine" front seat passenger hit the 'in"shiel" 'ith his face an" the "ash$oar" 'ith his knees. E.amination of the passenger in"ee"

sho's m#ltiple facial lacerations% $#t $eca#se of his into.ication he cannot e.plain 'here else he might $e h#rting. /e is ne#rologically intact% an" his cer)ical spine .!ray films are normal. A""itional in9#ry% representing a potential orthope"ic emergency% is not o$)io#s $#t is s#specte". Therefore% an .!ray film of 'hich of the follo'ing areas sho#l" most likely $e o$taine"a6Both patellas $6Both hips c6The 9a' "6The l#m$ar spine e6 The sk#ll :ormal La$s 44. @#ring a h#nting trip% a yo#ng man is $itten $y a coyote. The animal is capt#re" an" $ro#ght to the a#thorities ali)e. =hich of the follo'ing is the most important criterion to "etermine the patient,s nee" for ra$ies prophyla.isa6 The patient,s history of pre)io#s imm#niDations $6The patient,s clinical co#rse o)er the ne.t fe' 'eeks c6>$ser)ing the animal,s $eha)ior o)er the ne.t fe' "ays "6Milling the animal an" e.amining the $rain e6 The e)ents that took place ha)e alrea"y esta$lishe" the nee" to procee" 'ith ra$ies imm#niDation :ormal La$s 41. A <2!year!ol" 'oman ha" an a$"ominal hysterectomy an" salpingo! oophorectomy & "ays ago. She ha" an in"'elling $la""er catheter "#ring the proce"#re% 'hich 'as remo)e" in the reco)ery room. She has $een )oi"ing at 'ill since then. She also ha" compression pne#matic stockings on $oth lo'er e.tremities "#ring the operation. She $egan am$#lation on the 1st postoperati)e "ay% an" has $een as acti)e as possi$le #n"er the circ#mstances% incl#"ing faithf#l a"herence to a prescri$e" program of incenti)e spirometry. >n the e)ening of the &r" postoperati)e "ay% she spikes a fe)er% 'ith a temperat#re to &8.4 2 314& 56. =hich of the follo'ing is the most likely so#rce of the fe)era6 Atelectasis $6@eep throm$ophle$itis c6+ntra!a$"ominal a$scess

"6Urinary tract infection e6=o#n" infection :ormal La$s 42. A (<!year!ol" man is #n"ergoing an a$"ominoperineal resection for rectal cancer. @#ring the s#rgery% #ne.pecte" se)ere $lee"ing is enco#ntere"% an" the patient is hypotensi)e on an" off for almost an ho#r. The anesthesiologist notes ST "epression an" T!'a)e flattening on the E2C monitor. =hich of the follo'ing is the most likely "iagnosis an" the e.pecte" mortalitya6+ntraoperati)e air em$ol#s% 144B $6Myocar"ial infarction% B to 14B c6Myocar"ial infarction% 4B to 84B "6P#lmonary em$ol#s% B to 14B e6P#lmonary em$ol#s% 4B to 84B :ormal La$s 4&. A 44!year!ol" 'oman has a palpa$le no"#le in the right lo$e of her thyroi" glan". The no"#le meas#res 2 cm an" is firm. The rest of the thyroi" glan" cannot $e felt an" is not ten"er. She also "escri$es losing 'eight in spite of a ra)eno#s appetite% palpitations% an" heat intolerance. She is thin% fi"gety% an" constantly mo)ing% 'ith moist skin an" a p#lse of 14 7min. She has no e.ophthalmos or preti$ial e"ema. /er TS/ is reporte" as m#ch lo'er than normal% an" she has ele)ate" le)els of free T4. =hich of the follo'ing is the most appropriate ne.t step in "iagnosisa6E.ploratory neck s#rgery $6MA+ of the pit#itary glan" c6:ee"le core $iopsy of the thyroi" mass "6Aa"ion#cli"e thyroi" scan e6Ser#m le)els of T& :ormal La$s 44. A <<!year!ol" man presents 'ith progressi)e 9a#n"ice% 'hich he first notice" < 'eeks ago. /e has a total $ilir#$in of 22 mg7"L% 'ith a "irect 3con9#gate"6 $ilir#$in of 1< mg7"L. /is transaminases are minimally ele)ate"% 'hereas his alkaline phosphatase is a$o#t si. times the #pper

limit of normal. A sonogram sho's "ilate" intrahepatic "#cts% "ilate" e.trahepatic "#cts% an" a )ery "isten"e"% thin!'alle" gall$la""er 'itho#t stones. =hich of the follo'ing is the most appropriate ne.t step in "iagnosisa62T scan of the #pper a$"omen $6En"oscopic retrogra"e cholangiopancreatography 3EA2P6 c6E.ploratory laparotomy "6Perc#taneo#s transhepatic cholangiogram 3PT26 e6Serologies to "efine the type of hepatitis :ormal La$s 4 .An ol"er% o)er'eight man complains of "isa$ling% sharp heel pain e)ery time his foot strikes the gro#n". The pain is 'orse in the mornings% pre)enting him from p#tting any 'eight on the heel. F!ray films sho' a $ony sp#r matching the location of his pain% an" physical e.amination sho's e.*#isite ten"erness to "irect palpation right o)er that heel sp#r. 5#rthermore% 'hen the ankle is "orsifle.e"% the entire inner $or"er of the fascia is ten"er to palpation. =hich of the follo'ing is the most likely "iagnosisa6Epiphysitis of the calcane#s $65ract#re of the posterolateral talar t#$ercle c6Plantar fasciitis "6Posterior Achilles ten"on $#rsitis e6 Posterior ti$ial ner)e ne#ralgia :ormal La$s 4<. An el"erly man is in)ol)e" in a rear en" a#tomo$ile collision in 'hich he hypere.ten"s his neck. /e "e)elops paralysis an" $#rning pain of $oth #pper e.tremities% 'hile maintaining goo" motor f#nction in his legs. =hich of the follo'ing is the most likely "iagnosisa6 Anterior cor" syn"rome $62entral cor" syn"rome c6Posterior cor" syn"rome "6Aefle. sympathetic "ystrophy e6Spinal cor" hemisection :ormal La$s

4(. A <(!year!ol" man has ha" an in"olent% #nhealing #lcer at the heel of the right foot for se)eral 'eeks. The patient $egan 'earing a ne' pair of shoes shortly $efore the #lcer starte" an" notice" a $lister as the first anomaly at the site 'here the #lcer e)ent#ally "e)elope". /e in"icates that neither the $lister nor the #lcer e)er ga)e him any pain. The #lcer is &. cm in "iameter% the #lcer $ase looks "irty% an" there is har"ly any gran#lation tiss#e. The skin aro#n" the #lcer looks normal. The patient has no sensation to pin prick any'here in that foot. Peripheral p#lses are 'eak $#t palpa$le. /e is o$ese an" has )aricose )eins% high cholesterol% an" poorly controlle" type 2 "ia$etes mellit#s. =hich of the follo'ing most acc#rately characteriDes the #lcera6@ia$etic #lcer "#e to tra#ma% ne#ropathy% an" micro)asc#lar "isease $6+schemic #lcer "#e to arteriosclerosis c6 +schemic #lcer "#e to em$oliDation "6:eoplastic in nat#re% pro$a$ly s*#amo#s cell carcinoma e6 Stasis #lcer "#e to )eno#s ins#fficiency :ormal La$s 41. A 2 !year!ol" man is sta$$e" once in the right chest. The entrance 'o#n" is on the mi"a.illary line% at the le)el of the fifth intercostal space. /e arri)es at the emergency "epartment mo"erately short of $reath% $#t he is f#lly a'ake an" alert% is talking 'ith a normal tone of )oice% an" has no "isten"e" )eins )isi$le in his neck or forehea". /is $loo" press#re is 1&47( mm /g% an" his p#lse is 127min. Physical e.amination of the chest sho's the 'o#n"% 'hich is not )isi$ly 0s#cking air%0 an" "emonstrates no $reath so#n"s at all on the right si"e% 'hich is tympanitic to perc#ssion. There is no e)i"ence of me"iastinal "isplacement. =hich of the follo'ing 'o#l" $e the most appropriate ne.t step in managementa6 2o)er the 'o#n" 'ith a reg#lar "ressing an" get a chest .!ray $6 2o)er the 'o#n" 'ith ?aseline ga#De% tape" on three si"es c6 En"otracheal int#$ation "6+nsert a chest t#$e at the right ple#ral $ase e6+nsert an 11!ga#ge nee"le into the right ple#ral space at the secon" intercostal space :ormal La$s

48.The #nrestraine" front!seat passenger in a car that crashes s#stains close" commin#te" fract#res of $oth femoral shafts. Shortly after a"mission% he "e)elops a $loo" press#re of 147 4 mm /g% a p#lse rate of 1147min% an" a )eno#s press#re of Dero. /e $ecomes pale% col"% an" clammy% $#t the rest of his physical e.amination an" .!ray films of the chest an" pel)is are #nremarka$le. A sonogram of the a$"omen "one in the emergency "epartment is like'ise negati)e. =hich of the follo'ing is the most likely reason for the lo' $loo" press#rea6Bloo" loss at the fract#re sites $65at em$olism c6:e#rogenic shock from pain "6UnrecogniDe" intracranial $lee"ing e6UnrecogniDe" pericar"ial tampona"e :ormal La$s 4. A 2 !year!ol" man is sta$$e" in the right chest. /e comes in f#lly a'ake an" alert% an"% in a normal tone of )oice% he states that he feels short of $reath. /is )ital signs are normal an" sta$le. >n physical e.amination% he has no $reath so#n"s at the right $ase% an" only faint $reath so#n"s at the ape.. /e is "#ll to perc#ssion o)er the right $ase. A chest .!ray film confirms that he has a hemothora. on that si"e. =hich of the follo'ing is the most appropriate ne.t step in managementa6 >.ygen $y mask% analgesics% an" no specific inter)ention $6 +nt#$ation an" #se of a respirator c6+nsertion of a chest t#$e in the right secon" intercostal space "6 +nsertion of a chest t#$e at the right $ase e6E.ploratory thoracotomy
Normal ,abs Note: Check your own answers before hittin$ the Check button below. When you click the Check button, a browser win"ow will a!!ear that contains a summary of your results. (*!lanations

Block 21 E.planations

:) (*!lanation: 7he correct answer is C. 7he recent history of a "raine" !erirectal abscess, alon$ with the !hysical "escri!tion of the current lesion, clearly i"entify this as a fistula4in4ano. Anal fissure 'choice A) ty!ically occurs in youn$ women, who have e*quisite !ain with "efecation an" bloo" streaks in the stool. 7he lesion is in the anal mucosa, not in the nearby !erineal skin. Anorectal carcinoma 'choice ) always has to be rule" out in any !atient with anorectal com!laints, !articularly those "escribin$ bloo" in the stool. In a"vance" cases, it can show u! as an ulcerate", "rainin$ mass. ut, it woul" not be a "iscrete o!enin$ with no obvious tumor mass, such as "escribe" here. &iloni"al cyst 'choice %) is a $oo" "istracter, because they $et infecte", hurt, "rain !us, soil the un"erwear, an" have a skin o!enin$. 2owever, the o!enin$ is always ce!hala" to the anus, near the mi"lineG the "raina$e is !us, not fecesG an" there is no connection with recent !erirectal abscess. 7hrombose" hemorrhoi"s 'choice () woul" !ro"uce e*cruciatin$ !ain, an" a!!ear as a small. re", an$ry, mass !rotru"in$ out of the anus. If they "rain s!ontaneously, bloo" an" clot, not feces, come out. ;) (*!lanation: 7he correct answer is

C. Ol"er men of )can"inavian "escent are the victims of this otherwise mysterious contracture. 7he !al!able no"ules are classic, as is the "eformity itself. Car!al tunnel syn"rome 'choice A) affects mostly youn$ women, an" the sym!toms are tin$lin$ an" numbness, without a "eformity. %e 8uervain tenosynovitis 'choice ) is seen in youn$ women who com!lain of !ain on the ra"ial si"e of the wrist. 7here is no "eformity. 7enosynovitis of any kin" 'choice %) woul" !ro"uce !ain an" limitation to the movement of the fin$ers 6heumatoi" arthritis 'choice () is also a !ainful con"ition, an" the "eformity is classic alon$ the <oints, rather than in the !alm of the han". 0) (*!lanation: 7he correct answer is A. 7here are two mali$nant tumors of the eye for which enucleation woul" be "one: retinoblastoma an" melanoma. Only melanoma woul" have this weir" timetable, in which ;>4some years may ela!se between !rimary tumor an" metastatic manifestations. In fact, the !atient with a $lass eye an" a liver full of tumor is one of the two classic e*am!les that are $iven 'the other one has a missin$ toe) to illustrate the un!re"ictable behavior of melanoma. &rostatic cancer 'choice ) can show u! with metastases before we know the !rimary is there, but the metastatic site woul" usually be in bone. 6etinoblastoma 'choice C) can in"ee" lea" to eye enucleation, but if it $oes on to kill the !atient 'which it often "oes), it "oes not wait ;>4o"" years to "o it. )arcomas 'choice %) metastasi#e via the bloo" stream but are rare in or$ans that "rain via the !ortal system. 7hus, they favor the lun$s as the metastatic site. -urthermore, the !rimaries are sel"om hi""en. &rimary he!atocellular carcinoma 'choice () is rare in the F.)., where it is outnumbere" ;> to : by metastatic tumors. When it ha!!ens it is usually in a cirrhotic !atient, not in an otherwise healthy !erson. ?) (*!lanation: 7he correct answer is . 7he history 'a fair skinne" !erson who is out in the sun all "ay) su$$ests a skin cancer, an" the location 'the u!!er !art of the face) favors a basal cell cancer but "oes not e*clu"e a squamous cell carcinoma, or even a melanoma. 7hus, "ia$nosis is nee"e" before !ro!er thera!y is institute". 7he e"$e of the lesion offers the best information for the !atholo$ist. A bio!sy of the center of the lesion 'choice A) "e!rives the !atholo$ist of all the clues that are foun" at the interface between the tumor an" the normal skin, an" in lar$e lesions it runs the risk of sam!lin$ necrotic tumor that has out$rown its bloo" su!!ly. A wi"e e*cision before !atholo$ic "ia$nosis 'choice C) risks "oin$ too much 'a basal cell cancer nee"s only : or ; mm of mar$ins) or too little 'a melanoma shoul" have at least ; cm). 6a"iation thera!y 'choice %) is a viable thera!eutic choice for squamous cell carcinoma, but not before a "ia$nosis has been establishe". 2ere, we are e*!ectin$ a basal cell carcinoma, thus this course of action woul" be even less a!!ro!riate. )cra!in$s an" cultures 'choice () assume an infectious !rocess, i$norin$ the stron$ clinical sus!icion of a tumor in this case.

/) "e!artment (*!lanation: 7he correct answer is . y far the most im!ortant thin$ that can be "one for caustic chemical burns is to wash away the caustic a$ent as soon as !ossible, an" the best way to "o that is with massive irri$ation. Any answer that allows the chemical a$ent to stay in touch with the skin, whether mi*e" with antibiotic ointment 'choice A), wra!!e" in ban"a$es 'choice (), or with no s!ecific a""itional instructions 'choice C) will result in continue" burnin$ for the time that it will take to $et to the emer$ency "e!artment or !hysiciansA office. Washin$ with an aci" 'choice %) is !articularly ina"visable. (*ce!t for chemical burns of the eso!ha$us, for which massive irri$ation cannot be "one, one shoul" never H!lay chemistH when "ealin$ with alkaline or aci" burns. 7he chemical reaction will $enerate heat an" com!oun" the !roblem. C) (*!lanation: 7he correct answer is %. In"ee", the a!!ro!riate course of action when coa$ulo!athy "evelo!s in these circumstances is to em!irically !rovi"e fresh fro#en !lasma an" !latelet !acks. 2owever, when hy!othermia com!licates the !icture, the ab"omen also has to be close" imme"iately albeit tem!orarily in the most e*!e"itious manner. 7he blee"in$ surfaces have to be !acke", waitin$ for a more !ro!itious time in which to attem!t hemostasis, once coa$ulation function an" bo"y tem!erature have im!rove". (lectrocoa$ulation 'choice A) is not ma$ic, it requires clottin$ on the !art of the !atient. If the ab"omen is ke!t o!en to "o it, the hy!othermia $ets worse. Warmin$ the ab"omen with saline 'choice ) is not sufficient if the ab"omen is ke!t o!en while sur$ery continues. Althou$h movin$ alon$ quickly 'choice C) woul" be better than leisurely continuation of sur$ery, it is not fast enou$h un"er the circumstances. esi"es, with no clottin$ one can never truly com!lete the o!eration. Abortin$ the o!eration 'choice () is the ri$ht thin$ to "o, but leavin$ the belly o!en is not. Closure with mesh is in"icate" when the ab"ominal com!artment syn"rome !revents normal closure. D) (*!lanation: 7he correct answer is C. 7he clinical, ra"iolo$ic, an" manometric criteria of "iffuse eso!ha$eal s!asm are all "escribe" in the vi$nette. Achalasia 'choice A) ty!ically has clinical !ro$ression, as well as re$ur$itation of un"i$este" foo". In the manometric stu"ies, there are no effective !eristaltic contractions an" there is increase" restin$ !ressure in the lower s!hincter, which fails to rela* "urin$ swallowin$. 7he "ys!ha$ia of eso!ha$eal cancer 'choice ) is ty!ically !ro$ressive, from soli"s to liqui"s. 7he barium swallow woul" be "ia$nostic.

Nutcracker eso!ha$us 'choice %) is very similar to "iffuse eso!ha$eal s!asm. 2owever, on manometry there is a mean "istal eso!ha$eal !eristaltic am!litu"e of more than :@> mm 2$, inclu"in$ an elevate" baseline !ressure in the lower s!hincter. 3enkerAs "iverticulum 'choice () !ro"uces re$ur$itation of un"i$este" foo" an" sym!toms referable to the u!!er eso!ha$us. 7he barium swallow woul" be "ia$nostic. @) A. Co"eine . 2y"romor!hone '%ilau"i") C. +e!eri"ine '%emerol) %. +etha"one (. +or!hine sulfate (*!lanation: 7he correct answer is %. .iven the circumstances, the lon$est actin$ a$ent woul" be the best choice. +etha"one lasts about :; hours, com!are" with aroun" ? hours for all the others o!iates liste" 'choices A, , C, (). I) (*!lanation: 7he correct answer is C. A breast lum! in a ?04year4ol" nee"s to be bio!sie", but before we un"ertake that a$$ressive ste! we must ascertain the full e*tent of her !otential "isease. -in$ers can feel bi$ lum!sG *4ray films are nee"e" to i"entify smaller ones. %iscussin$ cancer thera!y 'choice A) is !remature if we have not yet "ia$nose" cancer. We no lon$er live in the era of bio!sy in the o!eratin$ room, fro#en section "ia$nosis, an" sur$ery un"er the same anesthetic. 7he "ia$nosis shoul" be establishe" first, an" then the o!tions "iscusse". We nee" the mammo$ram to know what else may nee" to be bio!sie", but not to tell us whether a bio!sy is nee"e" 'choice ). We alrea"y have the in"ications for bio!sy of the e*istin$ lesion, re$ar"less of its a!!earance in a mammo$ram. Waitin$ for resolution 'choice %) is okay for a woman with fibrocystic "isease who $ets lum!s every month, but not for this woman who has never ha" breast "isease before. -ine4nee"le as!iration '-NA) before a more invasive bio!sy 'choice () is an acce!table choice, !rovi"e" we un"erstan" that we will res!on" to a !ositive "ia$nosis establishe" by cytolo$y, but will not be "issua"e" from $ettin$ a more com!lete sam!le if the -NA is ne$ative. 7his o!tion is unacce!table as wor"e". :>) (*!lanation: 7he correct answer is A. Non"is!lace" fractures of the car!al navicular are notorious for not showin$ u! on *4 ray films at the time of in<ury. 7he mechanism of in<ury !lus the !hysical fin"in$s "escribe" in this vi$nette are sufficient to make a !resum!tive "ia$nosis an" to in"icate

the use of a cast. 7enosynovitis 'choice ) is not the result of a "irect blow, but it is seen in youn$ mothers who carry the hea" of their babies with a hy!ere*ten"e" thumb an" a fle*e" wrist. %is!lace" sca!hoi" fracture 'choice C) woul" show u! on *4ray films. 7he treatment for those fractures is in"ee" o!en re"uction an" internal fi*ation 'they are notorious for non4healin$), but the "ia$nosis is not correct in this o!tion. ,i$amentous in<ury 'choice %) is often the assum!tion of those who are not aware of the !eculiar nature of this in<ury. A similar mistake is ma"e by those who assume that if an *4ray film a!!eare" ne$ative, there cannot be a fracture 'choice (). ::) (*!lanation: 7he correct answer is . A$e is your first ti!4off: the ol"er the !atient is with any kin" of breast !roblem, the more likely it is to be cancer. 7he "escri!tion is that of inflammatory cancer, where the !ermeation of skin lym!hatics has $iven the e"ema, re"ness, fullness, an" oran$e !eel a!!earance. 7he thick, tumor4la"en skin actually masks the un"erlyin$ mass, which is felt as HfullnessH rather than a "iscreet lum!. Chronic cystic mastitis 'choice A) ha!!ens to youn$er women 'a$e" ;>4?>) with recurrent !ain linke" to the menstrual cycle. +eno!ause 'choice C) shrinks the breast an" makes it more fat than stroma, but it "oes not make it re" an" swollen. &yo$enic abscess 'choice %) ha!!ens almost e*clusively to lactatin$ women. At a$e D;, we know that is not ha!!enin$. 7uberculosis or fun$us 'choice () is our usual $uess when an abscess is not hot an" ten"er. 2owever, a!art from their rarity in the F.)., such thou$hts here woul" "etract from the com!ellin$ "ictum that a re", swollen breast in an ol" woman is cancer until !roven otherwise. :;) (*!lanation: 7he correct answer is A. 7he clinical features are those of a herniate" lumbar "isc. 7he "ia$nosis shoul" be confirme" with an +6I, an" then the !atient shoul" be treate" conservatively with be" rest. +ost !atients $et better with this sim!le a!!roach. .ivin$ antibiotics 'choice ) assumes an infectious !rocess. Infections can occur in the lumbar s!ine or the "iscs, but their sym!toms "o not start su""enly, like this vi$nette "escribes. A bo"y cast 'choice C) mi$ht be nee"e" for fractures, scoliosis, or other s!inal !atholo$y, but castin$ is not nee"e" for an e*tru"e" "isc. 6a"iothera!y 'choice %) assumes a neo!lastic !rocess. Althou$h a weakene" bone may in"ee" ru!ture su""enly, such !atients are usually known to have ha" the kin" of tumor that is likely to metastasi#e to bone 'in women, breast cancer woul" lea" the list), an" woul" have been com!lainin$ of locali#e" bony !ain before the !rocess $ets to the !oint of fracture. )ur$ical "ecom!ression 'choice () woul" have been require" if she ha" s!hincteric "eficits or !erineal anesthesia. :0) (*!lanation:

7he correct answer is . )lowly "evelo!in$ !aralysis on one si"e is su$$estive of a tumor. )ince there are no !hysical fin"in$s to !lace the tumor in the !aroti" $lan", it must be im!in$in$ on the nerve itself at a more !ro*imal location. ellAs !alsy 'choice A) has su""en onset, rather than $ra"ual "evelo!ment. 2emorrha$ic stroke 'choice C) woul" have occurre" su""enly, with an e*cruciatin$ hea"ache. A !aroti" cancer 'choice %) woul" have been !al!able by !hysical e*amination an" woul" have !ro"uce" !ain. &leomor!hic a"enoma 'choice () woul" also have been !al!able, an" such tumors almost never !ro"uce facial nerve !aralysis. :?) (*!lanation: 7he correct answer is (. 7he clinical !resentation is classic for %e 8uervainAs tenosynovitis, inclu"in$ the !ositive -inkelstein si$n 'the !ain re!ro"uce" by ulnar "eviation to stretch the affecte" ten"ons). ursitis 'choice A) occurs where there are bursaG thus, the classic locations are !laces in which ten"ons or muscles !ass over bony !rominences. Car!al tunnel syn"rome 'choice ) affects youn$ women, such as this !atient, but the !resentation is one of numbness alon$ the innervation !attern of the entra!!e" me"ian nerve. -ractures of the car!al navicular bone 'choice C) often $o unreco$ni#e", but they occur when someone falls on an outstretche" han". 7he classic !hysical fin"in$ is !ain with !al!ation over the anatomic snuff bo*. %u!uytrenAs contracture 'choice %) !ro"uces inability to fully o!en an" e*ten" the han", an" it ty!ically ha!!ens to ol"er men of )can"inavian "escent. :/) (*!lanation: 7he correct answer is C. Althou$h this is an im!ressive, very a"vance" cancer with a !oor !ro$nosis, it can be e*!ecte" to shrink si$nificantly with local ra"iation !lus systemic chemothera!y. It may "o so to the !oint at which a !alliative mastectomy becomes technically feasible, somethin$ that cannot be "one at this time. 7akin$ care of the woun", an" acce!tin$ "efeat from the cancer 'choice A), was the only available choice before chemothera!eutic a$ents an" ra"iation thera!y were "evelo!e". It woul" be entirely ina!!ro!riate at the !resent time. 7amo*ifen 'choice ) woul" be the weakest systemic wea!on in this case. )he is !remeno!ausal an" rece!tor ne$ative. 7amo*ifen alone woul" not shrink this tumor to any a!!reciable e*tent. +astectomy, either sim!le 'choice %) or ra"ical 'choice (), is not !ossible at this time. 7he "escri!tion clearly "e!icts an ino!erable tumor. We first nee" to make it o!erable. :C) (*!lanation: 7he correct answer is

(. 7he location an" history are classic for squamous cell carcinoma of the lower li!. 7he absence of metastatic no"es "oes not invali"ate the "ia$nosis, as most cancers in this location "o not metastasi#e until quite late. A"enocarcinoma 'choice A) woul" be very rare in the lower li!. asal cell carcinoma 'choice ) favors the u!!er !art of the face, above a hori#ontal line "rawn across the mouth. eni$n ulceration 'choice C) is always a !ossibility, but it woul" be a terrible mistake to make such assum!tion. As !ointe" out before, this vi$nette is a Hte*tbook caseH for squamous cell carcinoma. +elanoma 'choice %) is a$ain very rare in this location. A history of a !i$mente" lesion that un"erwent chan$es in color, a!!earance, or "iameter woul" have been su$$estive. :D) (*!lanation: 7he correct answer is A. 7he clinical !icture is that of a cancer of the eso!ha$us, an" $iven his race an" history of smokin$ an" "rinkin$, it is !robably a squamous cell carcinoma. 7he "escri!tion of where the "ys!ha$ia is felt su$$ests a low location, but such sub<ective feelin$s lack !recision. 7he tumor will eventually be seen an" bio!sie" by en"osco!y, but the en"osco!ist will first want to know the e*act location of the tumor an" the "e$ree to which the lumen is occlu"e". Otherwise, there is a hi$h risk of instrumental !erforation of the eso!ha$us. 7he best way to obtain that information is to "o a barium swallow. .astro$rafin 'choice ) $ives less "etaile" !ictures an" woul" be calle" for only if !erforation was sus!ecte". +anometry 'choice C) woul" be calle" for if the history su$$este" a motility !roblem. (so!ha$eal !2 monitorin$ 'choice %) woul" be the !erfect test to "ocument $astroeso!ha$eal reflu*. (so!ha$osco!y 'choice () will in"ee" be "one, but not until after the barium swallow. :@) (*!lanation: 7he correct answer is C. 7he most sensitive test to "etect early bone metastasis is the ra"ionucli"e scan. In a woman who recently ha" cancer of the breast, we have to sus!ect bony metastasis when bone !ain "evelo!s. C7 scan 'choice A) woul" be more e*!ensive an" less sensitive. Nee"le bio!sy 'choice ) is invasive an" not the first thin$ to "o. )ono$ram 'choice %) is su!erb for many other thin$s, but not to "etect early bone metastasis. E4ray films 'choice () will be "one after the scan if the ra"ionucli"e Hli$hts u!.H 7he ra"ionucli"e scan is very sensitive, but not terribly s!ecific. Once it li$hts u!, we have to rule out other ra"iolo$ically obvious bony !roblems that mi$ht have tri$$ere" the !ositive scan. :I) (*!lanation: 7he correct answer is %. A hi$h s!inal anesthetic can !ro"uce vasomotor shock by in"ucin$ wi"es!rea" vaso"ilation. 1asoconstrictors are the a!!ro!riate thera!y, but since the ca!acity of the

vascular tree is also increase" un"er these circumstances, fillin$ it u! with a""itional volume is also hel!ful. %iuretics an" flui" restriction 'choice A) woul" com!oun" the !roblem. 7he !atient nee"s more flui", not less. Whole bloo" an" clottin$ factors 'choice ) are not nee"e". 7he volume can be increase" with chea!er an" safer I1 flui"s until !ro!er vascular tone is restore". Coa$ulation factors have not been lost. Inotro!ic a$ents an" mechanical assistance to the circulation 'choice C) are in"icate" in car"io$enic shock, which woul" be rare in a ;;4year4ol" an" woul" be i"entifie", amon$ other thin$s, by a hi$h central venous !ressure. 1aso"ilators an" flui"s 'choice () are sometimes a $oo" combination when hy!ovolemia !lus hi$h !eri!heral resistance "e!rive tissues of !ro!er !erfusion. In this case, however, vaso"ilation alrea"y e*ists 'an" is the $enesis of the !roblem). ;>) (*!lanation: 7he correct answer is %. -ollicular cancers can metastasi#e by way of the bloo" stream to the liver, lun$, brain, or bones. ecause the tumor has ru"imentary functional ca!ability, it can be trace" with an" ablate" by ra"ioactive io"ineG however, the tumor cannot com!ete successfully with normal thyroi" tissue for the ca!ture of io"ine. After removal of the entire $lan", the tumor becomes the most effective io"ine tra!!er in the bo"y. (nucleation 'choice A) is never a $oo" answer when "ealin$ with cancer. ,ocal recurrence woul" virtually be $uarantee". ,obectomy alone 'choice ) woul" leave normal thyroi" in !lace an" thus !revent future use of ra"ioactive io"ine. 7otal thyroi"ectomy 'choice C) is a correct, but incom!lete, answer. 6a"ical neck "issection 'choice () is not nee"e" if there are no !al!able no"es. )houl" they "evelo! later, the !roce"ure coul" be "one then, or the no"es coul" be "ealt with by means of ra"ioactive io"ine. ;:) (*!lanation: 7he correct answer is %. 7he location an" circumstances are classic for +ortonAs neuroma, a beni$n neuroma of the thir" !lantar inter"i$ital nerve. .out 'choice A) ha!!ens to obese, el"erly males, an" re"ness an" si$ns of inflammation in the affecte" <oint are evi"ent. 2allu* ri$i"us 'choice ) is osteoarthritis of the first metatarso!halan$eal <oint. 7here is "eformity an" limitation of motion. 7he <oint is ten"er on !hysical e*amination. +etatarso!halan$eal articulation !ain 'choice C) is likewise associate" with misali$nment of <oint surfaces. 7here is !ain when e*aminin$ the <oint, an" there is no history of numbness, burnin$, or tin$lin$. &lantar fasciitis 'choice () !ro"uces shar! !ain on !hysical e*amination when !ressin$ the !lantar surface of the heel. ;;) (*!lanation: 7he correct answer is

%. 7his !atient obviously has a tension !neumothora* on the ri$ht. 7he !ressure nee"s to be relieve" imme"iately, which insertion of a nee"le will "o. 7hen, a formal chest tube shoul" be inserte". loo" $ases 'choice A) or chest *4ray films 'choice ) are not nee"e" to reco$ni#e the !resence of a tension !neumothora*. 7hese two stu"ies will soon be "one in this !atient, but not before action is taken to save his life by !rom!t "ecom!ression of the tension !neumothora*. A !atient who is awake an" alert an" s!eakin$ with a normal tone of voice has a !atent airway. At this moment, he "oes not nee" en"otracheal intubation 'choice C), althou$h $iven his multi!le in<uries, he will !robably en" u! havin$ sur$ery an" bein$ intubate" for that anesthetic. &ericar"iocentesis 'choice () assumes our first clinical "ia$nosis is !ericar"ial tam!ona"e. If he were still in shock an" still ha" bi$ "isten"e" veins after his !leural s!ace ha" been "ecom!resse", we mi$ht think that he also has a !ericar"ial tam!ona"e in a""ition to the tension !neumothora*. In fact, as the most !ressin$ !roblems are resolve", we mi$ht uncover other reasons for his state of shock, such as internal blee"in$. 6i$ht now, however, what is cryin$ out for hel! is his ri$ht !leural s!ace. ;0) (*!lanation: 7he correct answer is A. A fracture from such trivial strain si$nifies a very weakene" bone. In this a$e an" $en"er, the most likely cause woul" be a lytic lesion from metastatic breast cancer. In a man, we woul" have sus!ecte" metastatic lun$ cancer 'not !rostate, because !rostatic metastases are blastic rather than lytic). 7he bony reabsor!tion of !arathyroi" "isease 'choice ) is mostly seen as cystic lesions in the bones of the han". In very a"vance" cases, bones may be "eformable, but !arathyroi" hormone "oes not weaken ma<or bones to the !oint at which they break. Osteomalacia 'choice C), an" its counter!art in chil"ren, rickets, "eforms bones, but they woul" not break as "escribe" here. Osteo!orosis 'choice %) is in"ee" likely to be !resent in this woman. 2owever, even with a"vance" osteo!orosis, the only s!ontaneous fractures seen are com!ression fractures of vertebral bo"ies. Osteo!orotic bones break easily, but there has to be more trauma than liftin$ a ba$ of $roceries. &rimary mali$nant bone tumors 'choice () occur in youn$ !eo!le. 7hey "o not occur at this a$e. ;?) (*!lanation: 7he correct answer is . 7his man has in"ications for sur$ical intervention, but all he nee"s is an antireflu* o!eration. y far the most commonly use" !roce"ure is a la!arosco!ic Nissen fun"o!lication. 2eller myotomy 'choice A) is use" for achalasia. 2ere it woul" "o the e*act o!!osite of what is nee"e". 7ranshiatal total eso!ha$ectomy 'choice C) is the most commonly use" o!eration for eso!ha$eal cancer. It is only !alliative, since it cannot !rovi"e a true Hcancer o!erationH with wi"e "issection, but eso!ha$eal cancer is rarely amenable to true curative resection when it becomes sym!tomatic. Althou$h this man coul" "evelo! cancer if he is not treate", he "oes not have a"vance" cancer at this time.

7ransthoracic resection of the lower eso!ha$us 'choice %) woul" be the !roce"ure if a very early cancer were to "evelo! at the eso!ha$o$astric <unction. If this man "i" not choose to have the antireflu* sur$ery now, but he remaine" un"er close surveillance, he mi$ht $et to the !oint where he became a can"i"ate for this o!tion. Aci" re"uction 'choice () is not !art of the stan"ar" sur$ical treatment for $astroeso!ha$eal reflu*. We can control aci" me"ically. If we have to o!erate, we "o it to !rovi"e a $oo" one4way valve when the native s!hincter no lon$er works. ;/) (*!lanation: 7he correct answer is %. 7he clinical fin"in$s "o not su$$est urethral in<ury, but testicular fracture is a !otential in<ury that woul" require sur$ical intervention. )ono$ram woul" be "ia$nostic. As!iration 'choice A) is not a $oo" i"ea. We alrea"y know that bloo" is !resent, an" !uttin$ nee"les into it woul" invite bacterial contamination. If the testicle is intact, the hematoma will resolve s!ontaneously. Neither the bla""er 'choice ) nor the urethra 'choice C) nee" to be checke" when the urine has no bloo", the !atient can voi" normally, an" the rectal e*amination is unremarkable. )ur$ical e*!loration 'choice () is not in"icate" unless a "ia$nosis of testicular ru!ture has been ma"e. ;C) (*!lanation: 7he correct answer is A. 7he clinical "escri!tion is classic for biliary colic, "ue to $allstones that are intermittently im!acte" at the cystic "uct. 7he "ia$nostic stu"y of choice to confirm the !resence of $allstones is a sono$ram. An u!!er $astrointestinal series 'choice ) will miss the "ia$nosis. 7his woman "oes not have a !roblem in her stomach or "uo"enum. )he has to be sus!ecte" of havin$ $allstones, an" the stu"y has to tar$et that area. Antibiotics an" I1 flui"s 'choice C) are require" to Hcool "ownH an e!iso"e of acute cholecystitis. 2owever, this woman "oes not have fever, leukocytosis, an" a ten"er ri$ht u!!er qua"rant. )he "oes not have acute cholecystitis. (n"osco!ic retro$ra"e cholan$io!ancreato$ram '(6C&) 'choice %) woul" be an e*!ensive, invasive, an" totally un<ustifiable way to take a look at the $allbla""er. (*!loratory sur$ery 'choice () woul" be even worse. 7his woman will nee" sur$ery, but it shoul" be "irecte" at the $allbla""er an" "one la!arosco!ically as an elective !roce"ure 'i.e., at a convenient time) after a "ia$nosis has been confirme". ;D) (*!lanation: 7he correct answer is A. 7his !atient has e!i"i"ymitis, most likely "ue to Chlamy"ia trachomatis. (!i"i"ymitis refers to inflammation of the e!i"i"ymitis, which lea"s to unilateral intrascrotal !ain,

swellin$, an" fever. 7esticular su!!ort usually relieves the !ain to some e*tent. Asym!tomatic urethritis can be associate" with e!i"i"ymitis "ue to C. trachomatis or Neisseria $onorrhoeae. Frinalysis may reveal !yuria. Frethral culture, urine !olymerase chain reaction '&C6) or li$ase chain reaction ',C6) can confirm the "ia$nosis. 7reatment is with a#ithromycin, "o*ycycline or tetracycline. 7he se*ual !artner shoul" be treate" to avoi" reinfection. &rostatitis 'choice ) is the inflammation of the !rostate, which often !resents with !erianal an" low back !ain, urinary frequency, ur$ency, an" !ain "urin$ urination. 7reatment is with antibiotics. 7esticular torsion 'choice C) is a sur$ical emer$ency, which is cause" by twistin$ of the s!ermatic cor" an" vascular com!romise. It ty!ically occurs in youn$ a"ult males, who !resent with acute unilateral testicular !ain an" normal urinalysis results. 7esticular su!!ort "oes not relieve the !ain. Frethritis 'choice %) is an infection most often cause" by Chlamy"ia or N. $onorrhoeae, an" !resents with a urethral "ischar$e. 7he "ischar$e is mucoi" in chlamy"ial infection an" !urulent in $onorrhea. 7he treatment is ceftria*one an" "o*ycycline. )e*ual !artners shoul" be treate". 1aricoceles 'choice () are a collection of veins that feel like a Hba$ of wormsH. )ur$ery is in"icate" if there is infertility or !ain. 7hey are more common on the left si"e. ;@) (*!lanation: 7he correct answer is A. 7he clinical fin"in$s are in"icative of a fracture of the base of the skull, an" thus he has sustaine" very si$nificant trauma to the hea". 7he inte$rity of the cervical s!ine has to be ascertaine", an" the C7 that he is alrea"y $oin$ to have can be e*ten"e" to inclu"e that area. +6I 'choice ) has no role in the acute trauma situation. If we were lookin$ for a brain tumor, at leisure, an +6I woul" in"ee" be better than a C74but not in this settin$. Antibiotics 'choice C) have !roven to be of no value in base of the skull fractures. Corticosteroi"s 'choice %) are bein$ use" in !atients with s!inal cor" in<ury, but we have not yet "ia$nose" the !resence of such an in<ury. (mer$ency craniotomy 'choice () is not nee"e" to "eal with a basilar skull fracture. 2e woul" nee" one if his C7 scan showe" an intracranial hematoma "is!lacin$ the mi"line structures. 2e mi$ht nee" one later if the leak of C)- !ersists, but he "oes not nee" one now. ;I) (*!lanation: 7he correct answer is C. Althou$h most !enetratin$ woun"s of the chest that !ro"uce hemothora* require no sur$ery, there are cases in which systemic vessels 'almost invariably intercostals), rather than the in<ure" lun$ 'which has low !ressure circulation) are the source of blee"in$. In those cases, sur$ical hemostasis is require". 7hey are i"entifie" by the ma$nitu"e of the bloo" loss. 7y!ical criteria su$$est the nee" for thoracotomy if the initial bloo" recovery e*cee"s :>>> m,, or if subsequent "raina$e a""s u! to C>> m, or more, over the ensuin$ C hours. Continue" observation 'choice A) woul" be ina!!ro!riate $iven the rate of ;/>4 0/> m, of bloo" loss !er hour. 7he !roblem is not lack of "raina$eG thus, another tube

'choice ) woul" not resolve the !roblem. Althou$h choice % is almost i"entical to choice C, it a""s a me"ically unnecessary ste!: the bullet "oes not have to be remove". A""itional o!erative time an" o!erative "issection are not <ustifie" if the bullet is not !ressin$ on some vital structure. If it ha!!ens to be lyin$ there, <ust waitin$ to be !lucke", we woul" of course take it. &neumonectomy 'choice () tar$ets the wron$ or$an. lee"in$ from the lun$ is usually self4limitin$. lee"in$ that requires sur$ery is usually from systemic vessels, most commonly the intercostals. 0>) (*!lanation: 7he correct answer is . +a<or veins at the base of the neck have ne$ative !ressure "urin$ ins!iration an", if in<ure" at that moment, will suck air rather than blee". 7he air embolism then lea"s to su""en "eath. Arterial in<ury 'choice A) woul" have le" to massive blee"in$ but not to su""en "eath. &neumothora* 'choice C) can in"ee" ha!!en when sur$ery is bein$ "one in the su!raclavicular area, an" a suckin$ soun" mi$ht even be hear". 2owever, su""en lun$ colla!se in a youn$, healthy !erson lea"s to "ys!nea, not to su""en "eath. )ym!athetic "ischar$e 'choice %) woul" be har" to !ro"uce while !ullin$ an" "issectin$ a no"e. If it were "one, however, there woul" be vasoconstriction, tachycar"ia, !ers!iration, an" hy!ertension, rather than su""en "eath. 2a" the trachea been in<ure" 'choice (), essentially nothin$ woul" have ha!!ene" at the time. 0:) (*!lanation: 7he correct answer is . 7his is the famous Colles fracture, which ty!ically can be re"uce" well with close" mani!ulation. Castin$ shoul" immobili#e both the wrist an" the elbow, thus a lon$ arm cast is nee"e". A short arm cast 'choice A) woul" not immobili#e the elbow. )keletal traction 'choice C) coul" correct the "eformity, but at the cost of limitin$ future function of the han". An intrame"ullary ro" 'choice %) is usually reserve" for fractures of the shaft of lon$ bones, like the femur. O!en re"uction an" internal fi*ation 'choice () coul" in"ee" !rovi"e a very nice result, but it woul" be an unnecessarily e*!ensive an" intrusive way to "o it. 0;) (*!lanation: 7he correct answer is (. 7here is no "oubt that this !atient has an acute ab"omen, but we can tell more than that. 7he su""en onset, $enerali#e" e*tent, an" silent ab"omen in a man who "oes not want to move su$$ests a !erforation. In a""ition, the !resence of free air in the !eritoneal cavity !in!oints the $astrointestinal tract as the source. We cannot tell whether he !erforate" a !e!tic ulcer, blew out a si$moi" "iverticulum, or ha" his bowel !erforate" by a chicken bone, but there is a hole in his $astrointestinal tract. Choice A un"erestimates

our "ia$nostic ability. An inflammatory !rocess 'choice ) woul" have $ra"ual onset an" woul" be locali#e" to the area of the inflame" viscera. Obstruction 'choice C) has su""en onset, but it lea"s to colicky !ain. 7he !atient woul" be thrashin$ about lookin$ for a !osition of comfort, an" woul" still have bowel soun"s an" not have free air. Ischemia 'choice %) is a better bet in an ol"er !erson. If it were to lea" to necrosis an" !erforation, the latter woul" not be evi"ent 0> minutes after the trouble be$an. 00) (*!lanation: 7he correct answer is . Fntil the !atient attem!te" to $et out of be", he ha" a woun" "ehiscence that coul" have been han"le" by ta!in$ the woun" securely. Once the bowel came out, the !roblem became an evisceration. Imme"iate sur$ical re!air is man"atory. While settin$ it u!, the bowel must be !rotecte" from "ryin$ out, an" the !atient must be !rotecte" from si$nificant heat loss. 7hus, the key is warm an" moist "ressin$s. %ry "ressin$s 'choice A) woul" !revent further contamination but woul" fail in the key elements of Hwarm an" moist.H Col" antise!tic solutions 'choice C) woul" irritate the bowel an" contribute to hy!othermia. Once an evisceration has occurre", the entire ab"ominal wall has to be sur$ically close". )uturin$ the skin e"$es in an ina"equate facility 'choice %) or resortin$ to ta!e 'choice () woul" not suffice. 0?) (*!lanation: 7he correct answer is . &eo!le who are very ol" or alcoholic have smaller brains in a skull that has not chan$e" in si#eG thus, very minimal trauma can make the brain Hrattle aroun"H an" tear a venous sinus, from which a sub"ural hematoma very slowly forms. )enility "oes not occur in a 04week !erio". )uch marke" chan$es in someone with recent trauma shoul" tri$$er a search for chronic sub"ural hematoma. (!i"ural hematomas 'choice A) are ty!ically acute, from a tear of the mi""le menin$eal artery followin$ trauma that fractures the skull. %iffuse intracerebral blee"in$ 'choice C) woul" occur with very severe trauma an" woul" $ive more acute sym!toms. 7he frontal lobe 'choice %) is res!onsible for <u"$ment an" social $races, but not for financial acumen an" level of activity4the functions that this man use" to have an" lost over a short !erio" of time. rain atro!hy 'choice () is in"ee" !resent in the very ol" or the alcoholic. 7hat is what makes them !rone to "evelo! chronic sub"ural hematomas. ut, brain atro!hy alone woul" not e*!lain the mental chan$es that this man "evelo!e" over a few weeks. 0/) (*!lanation: 7he correct answer is (. 7he mo"ifie" &arklan" formula calls for ? m, of 6in$erAs lactate !er kilo$ram of bo"y wei$ht, times the !ercenta$e of the bo"y surface that has been burne"G !lus an a""itional

;>>> m, of "e*trose /= in water to cover maintenance flui" nee"s. In the Hrule of nines,H each lower e*tremity re!resents :@= of the bo"y surface. 7hus, this !atient has sustaine" a 0C= bo"y burn: ? ] C/ ] 0C U I0C>, !lus ;>>> U ::,0C> None of the other o!tions !rovi"e enou$h flui", althou$h in the real worl" the formula calculations are use" only to hel! "etermine a Hball!ark fi$ureH an" a rate of initial infusion. Once flui" is $oin$ in, the fine tunin$ is "one on the basis of hourly urinary out!ut an" central venous !ressure. 0C) (*!lanation: 7he correct answer is C. Ne$ative fin"in$s "o not have the same "ia$nostic value that !ositive fin"in$s have. If this ha" been a :I4year4ol" woman sus!ecte" of havin$ a fibroa"enoma, one woul" have been satisfie" with ne$ative ima$in$ stu"ies 'in that a$e, a sono$ram) or the ne$ative -N A. ut, at a$e ?I, the risk of cancer is much hi$her. .iven ne$ative fin"in$s in the least invasive stu"ies, one woul" feel com!elle" to move to more a$$ressive ways to obtain better tissue sam!lin$. Obviously, reassurance 'choice A) is not <ustifie" yet, an" waitin$ a whole year with what may be a cancer woul" be mal!ractice. 6e!eatin$ the same stu"ies in a month 'choice ) leaves you with the quan"ary of what to "o if they are ne$ative a$ain. No, you nee" more tissue for the !atholo$ist ri$ht now. ,um!ectomy an" a*illary "issection 'choice %) is too much to "o before the "ia$nosis has been establishe". ,um!ectomy alone mi$ht have been okay. An e*cisional bio!sy coul" in"ee" be <ustifie" un"er the circumstances, an" a lum!ectomy is not much more than a bi$ e*cisional bio!sy. ut, messin$ with the a*illa shoul" not ha!!en before we know it is cancer. +astectomy 'choice () is even less acce!table. &atients are $rateful when a cancer is rule" out by !roce"ures that they "o not !erceive as mutilatin$. ut, when sur$ery leaves them "eforme", the H$oo" newsH that there was no cancer may lea" them to call their lawyer. 0D) (*!lanation: 7he correct answer is %. A newly "evelo!e" white !u!il in a chil" raises the !ossibility of retinoblastoma. 7his tumor is so "ea"ly that imme"iate "ia$nosis an" treatment are im!erative. I$norin$ the fin"in$ 'choice A) coul" !rove to be lethal, an" the same can be sai" for any "elays cause" by !ursuin$ bi#arre consi"erations, such as lookin$ for albinos in the family 'choice ) or treatin$ for an eye infection that is not there 'choice (). It coul" be ar$ue" that if an o!hthalmolo$ic consultation is obtaine", even if it is for a wron$ "ia$nosis 'choice C), the true nature of the !roblem will eventually be reco$ni#e". ut an a!!ointment to check for cataracts 'which woul" have been !resent since birth) will not be ma"e with the same ur$ency that the situation requires. 0@)

(*!lanation: 7he correct answer is . 7he concern is that even thou$h he is now normotensive, invasive ste!s mi$ht tri$$er a hy!ertensive crisis from the !reviously un"ia$nose" !heochromocytoma that he !robably has. 7he !resence of catabolites from e!ine!hrine in"icates that the tumor is in the a"renal $lan"s, an" not at an e*tra4a"renal site. 7hus, the "ia$nosis of the !heochromocytoma can best be confirme" by +6I of the a"renals. +enin$iomas 'choice A) an" acoustic nerve tumors 'choice C) occur in ty!e ; neurofibromatosis, not in ty!e :. ,ookin$ for !heochromocytomas outsi"e of the a"renal $lan"s 'choice %) woul" have been a $oo" i"ea if only elevate" catabolites of nore!ine!hrine ha" been "etecte". 7he !resence of hi$h levels of e!ine!hrine catabolites im!licates the a"renal $lan"s. 6a"iation thera!y 'choice () is a ba" i"ea. eni$n neurofibromas can be stimulate" by ra"iation to un"er$o mali$nant transformation. 0I) 7he correct answer is . When hittin$ the knees a$ainst the "ashboar", the femurs can be "riven backwar" an" out of the acetabulum, resultin$ in !osterior "islocation of the hi!s. ecause of the tenuous bloo" su!!ly of the femoral hea"s, such in<ury must be !rom!tly reco$ni#e" an" treate". oth !atellas 'choice A) an" the <aw 'choice C) coul" in"ee" be fracture", but such fractures woul" be easily reco$ni#e" clinically. If they were not i"entifie" until the ne*t "ay, no "ama$e woul" be incurre". 7he lumbar s!ine 'choice %) shoul" always be thou$ht of when someone falls from a hei$ht an" lan"s on his feet, but it is not a likely hi""en in<ury in this settin$. )kull *4ray films 'choice () have $one out of favor as a way to assess hea" in<ury. 7he main issue in hea" in<uries is intracranial blee"in$, an" the stu"y to show it is the C7 scan. ?>) (*!lanation: 7he correct answer is %. (*amination of the animalAs brain for si$ns of rabies will "etermine whether the !ainful an" risky !rocess of rabies !assive an" active immuni#ation is require". 2istory of !revious immuni#ations 'choice A) is use" to "etermine what to "o for tetanus !ro!hyla*is, but it has no a!!lication for rabies, because virtually no one has ever receive" such immuni#ation in the !ast. Waitin$ for si$ns of rabies to a!!ear in the !atient 'choice ) woul" be a "eath sentence. We can !revent rabies, but once establishe" we cannot cure it. Observation of the animalAs behavior 'choice C) is a!!licable when "ealin$ with !rovoke" bites by "omestic !ets. 7he behavior of a wil" animal $ives no clues to the !resence or absence of rabies. If the animal ha" esca!e", choice ( woul" have been correct. ?:) (*!lanation:

7he correct answer is %. 7he timin$ is our ma<or clue. -ever on !osto!erative "ay 0 is usually from the urinary tract. 7he circumstances are also there: she ha" instrumentation of her urinary tract "urin$ the !roce"ure. Atelectasis 'choice A) is usually seen on "ay :, an" she is "oin$ everythin$ !ossible to avoi" this com!lication. %ee! thrombo!hlebitis 'choice ) coul" show u! this early, but is more likely to "o so /4D "ays after sur$ery. -urthermore, the !atient ha" a"equate !rotection "urin$ sur$ery an" has been movin$ aroun" since early on. Intra4ab"ominal abscess 'choice C) woul" nee" at least D4:> "ays to "evelo!. 7his is too early for that. Woun" infection 'choice () is likewise a later com!lication, ty!ically seen about a week after the o!eration. ?;) (*!lanation: 7he correct answer is C. Intrao!erative myocar"ial infarction is mostly seen in el"erly men, an" the most common tri$$erin$ event is !rolon$e" hy!otension. -urthermore, the mortality $reatly sur!asses that of a myocar"ial infarction "e novo 'ie, unrelate" to sur$ery), reachin$ the levels quote". Air embolism 'choice A) can ha!!en when bi$ veins are o!en, allowin$ air to be sucke" inG however, the location of the o!en veins is ty!ically the u!!er chest or lower neck. 7he !atient "ies while un"er$oin$ a !roce"ure un"er local anesthesia, breathin$ s!ontaneously rather than havin$ air blown into his lun$s. Choice correctly i"entifies the !roblem, but assi$ns it a low mortality more ty!ical of infarcts that "o not ha!!en "urin$ sur$ery. &ulmonary emboli 'choices % an" () are not usually seen "urin$ sur$eryG they ty!ically occur /4D "ays later. ?0) (*!lanation: 7he correct answer is %. 7here is no question, both clinically an" by laboratory, that the !atient is hy!erthyroi". )he has no clinical si$ns of acute thyroi"itis, an" none of the other fin"in$s seen in .raves "iseaseG however, she has a thyroi" no"ule, which raises the !ossibility of a hy!erfunctionin$ a"enoma 'a HhotH a"enoma). If in"ee" she "oes, the scan will show that the no"ule tra!s all the io"ine, with su!!ression of the rest of the $lan". (*!loratory neck sur$ery 'choice A) woul" be !remature without first "efinin$ the source of the hy!erfunction. 7he !ituitary 'choice ) is not at fault if the 7)2 is low. 2y!erthyroi"ism an" thyroi" cancer rarely coe*ist. )houl" one wish to e*clu"e the latter, fine4nee"le as!iration woul" be the first test. &ercutaneous core bio!sy of thyroi" no"ules 'choice C) is not favore" in this country. ,evels of 70 'choice () are nee"e" only when clinical hy!erthyroi"ism an" low 7)2 are foun" to e*ist in the !resence of normal levels of free 7?. ??)

(*!lanation: 7he correct answer is A. Obstructive <aun"ice is evi"ent by the hi$h alkaline !hos!hatase an" the "ilate" biliary "ucts. +ali$nancy is su$$este" by the "ilate", thin4walle" $allbla""er without stones. If there is a cancer of the hea" of the !ancreas, C7 has a $oo" chance of showin$ it in a noninvasive manner. (n"osco!ic retro$ra"e cholan$io!ancreato$ra!hy '(6C&) 'choice ) or !ercutaneous transhe!atic cholan$io$ram '&7C) 'choice %) woul" be the ne*t ste! if the C7 scan were ne$ative. A smaller tumor at the am!ulla, the common "uct itself, or even the hea" of the !ancreas, coul" esca!e "etection by the C7 an" necessitate a more invasive test to show u!. (ither of these coul" be use", althou$h (6C& is favore" by most. (*!loratory la!arotomy 'choice C) woul" be !remature at this !oint. )erolo$ies 'choice () woul" have been calle" for if he ha" very hi$h transaminases, normal or near normal alkaline !hos!hatase, an" an unremarkable sono$ram. ?/) (*!lanation: 7he correct answer is C. All the "etails are in the vi$nette, inclu"in$ the association with a heel s!ur that in the !ast le" many of these !atients to un"er$o unnecessary sur$ery to remove the s!ur. 7he s!ur is cause" by the !ull of the fascia an" is not the cause of the !lantar fasciitis. (!i!hysitis of the calcaneus 'choice A) affects chil"ren, an" the !ain occurs alon$ the si"es of the heel where the heel $rowth centers are locate". -racture of the !osterolateral talar tubercle 'choice ) occurs from a su""en <um! on the ball of the foot, an" the !ain an" swellin$ are behin" the ankle. &osterior Achilles ten"on bursitis 'choice %) occurs mostly in youn$ women, an" an erythematous, in"urate", ten"er area is !resent at the !osterosu!erior as!ect of the heel. &osterior tibial nerve neural$ia 'choice () is the footAs equivalent of the car!al tunnel syn"rome, with the !ain often e*ten"in$ to the toes, an" tin$lin$ bein$ !ro"uce" by ta!!in$ the nerve. ?C) (*!lanation: 7he correct answer is . 7he mechanism of in<ury 'hy!ere*tension) an" the relative s!arin$ of the lower e*tremities in the !resence of u!!er e*tremity "eficits are classic for central cor" syn"rome. In anterior cor" syn"rome 'choice A), all functions are lost, e*ce!t for !ositional an" vibratory sense. 7hose in<uries occur with blowout of the vertebral bo"ies. &osterior cor" syn"rome 'choice C) is quite rare, an" it woul" show loss of !ositional an" vibratory sense. 6efle* sym!athetic "ystro!hy 'choice %) !ro"uces a$oni#in$ burnin$ !ain 'thus, it is a $oo" "istracter for this vi$nette), but it ty!ically follows crushin$ in<uries of the affecte" e*tremity an" "oes not affect motion. 2emisection of the s!inal cor" 'choice () !ro"uces loss of one set of functions on one si"e, an" a "ifferent set of functions on the other si"e. In a""ition, it follows a clear4cut !enetratin$ in<ury, rather than hy!ere*tension of the neck.

?D) (*!lanation: 7he correct answer is A. %iabetic ulcers ty!ically "evelo! at !ressure !oints, an" the heel is a favorite location. 7he !atient has evi"ence of neuro!athy, an" the correlation with the trauma inflicte" by the new shoes is classic. Ischemic ulcers, whether "ue to arteriosclerosis 'choice ) or emboli#ation 'choice C) are ty!ically seen at the ti! of the toes, as far away from the heart as one can $et. Neo!lasms 'choice %) can in"ee" "evelo! in lon$4stan"in$ ulcers, but the history woul" have been one of :> or ;> years of healin$ an" breakin$ "own, before hea!e" u! e"$es of cancer be$in to "evelo!. )tasis ulcers 'choice () are seen above the malleolus, surroun"e" by e"ematous, hy!er!i$mente" skin. ?@) (*!lanation: 7he correct answer is A. 7he clinical !icture is that of a !neumothora*, but there is no life4threatenin$ situation that woul" !revent obtainin$ a ra"iolo$ic "ia$nosis of the e*act nature an" e*tent of the !roblem. 7hen the a!!ro!riate thera!y can be institute". Coverin$ the woun" with 1aseline $au#e 'choice ) is the stan"ar" a"vice for suckin$ chest woun"s to !revent further inflow of air into the !leural s!ace. 7his is not ha!!enin$ here. (n"otracheal intubation 'choice C) is not nee"e" if he has a $oo" airway. A !erson who is fully awake an" has a normal tone of voice has a normal airway. A chest tube 'choice %) shoul" not be inserte" blin"ly, not knowin$ yet what is $oin$ on. 2e may very well nee" a tube at the base if the *4ray shows a hemothora*, but we mi$ht !refer to !ut it at the to! if all he has is air. Insertion of a nee"le 'choice () is the correct answer when there is a life4 threatenin$ tension !neumothora*. In that case, he woul" have been in shock, with "isten"e" veins an" me"iastinal "is!lacement. ?I) (*!lanation: 7he correct answer is A. Comminute" fractures of the femurs are known to be one of the few !laces in the bo"y where enou$h occult bloo" loss may occur to lea" to hy!ovolemic shock. -at embolism 'choice ) is also associate" with lon$ bone fractures, but the manifestations are those of res!iratory failure, rather than hy!ovolemic shock. Neuro$enic shock 'choice C) woul" rarely occur from !ain alone, bein$ more common as a sequela of hi$h s!inal cor" transection. When it ha!!ens, the !atient is hy!otensive but looks warm an" flushe" rather than col" an" !ale. Intracranial blee"in$ 'choice %) can lea" to neurolo$ic sym!toms, but not to hy!ovolemic shock. 7here is not enou$h room within the hea" to accumulate the si#able bloo" loss require" to $o into shock. &ericar"ial tam!ona"e 'choice () woul" !ro"uce hi$h central venous !ressure.

/>) (*!lanation: 7he correct answer is %. Althou$h he is hemo"ynamically stable, an" thus !resumably not Hblee"in$ to "eath,H contaminate" bloo" shoul" not be left in the !leural s!ace, where it coul" lea" to the "evelo!ment of an em!yema. A chest tube !lace" at the base shoul" evacuate it. In "oin$ so, we will also learn whether the amount of bloo" recovere" <ustifies a more a$$ressive ste! to sto! the blee"in$. No s!ecific intervention 'choice A) is incorrect because that contaminate" bloo" nee"s to come out. Intubation an" res!irator 'choice ) are not nee"e". A !atient who is awake an" alert an" s!eakin$ in a normal tone of voice has a $oo" airway. 2e "oes not nee" intubation. Neither "oes he nee" a machine to breathe for him, when he is "oin$ it s!ontaneously. A chest tube hi$h in the !leural s!ace 'choice C) is the correct !rescri!tion for a !neumothora*, but not for a hemothora*. Air $oes to the to!, an" bloo" $oes to the bottom. 7o retrieve the bloo", the tube has to be at the bottom. 7horacotomy 'choice () is sel"om nee"e" for a hemothora*. lee"in$ is usually from the lun$, an" it sto!s by itself. When a systemic vessel is in<ure" 'ty!ically an intercostal), we fin" a lot of bloo" when the tube is !lace" 'more than :>>> or :/>> m,), or a substantial amount "rains out in the ensuin$ few hours 'more than C>> m, in C hours). Only in those cases is a thoracotomy in"icate".

USMLE Step 2 Practice Test Block 22 Name: Instructions: Answer the questions below to the best of your ability. When you finish the test, click the Check button at the bottom to view the results.

:.A C@4year4ol" man is brou$ht to the emer$ency "e!artment followin$ a hi$h4s!ee" automobile acci"ent. 2e is alert an" com!lains of chest !ain an" mil" back !ain. 2is bloo" !ressure is @>BC> mm 2$. Chest *4ray shows a wi"ene" me"iastinum, tracheal "eviation, bronchial "is!lacement, an" loss of the aortic knob. Which of the followin$ is the most likely "ia$nosisJ a)Car"iac tam!ona"e b)+yocar"ial contusion c)&ulmonary contusion ")7ension !neumothora* e)7raumatic aortic ru!ture Normal ,abs ;. A football !layer is tackle", an" he "evelo!s severe knee swellin$ an" !ain. On !hysical e*amination with the knee fle*e" at I> "e$rees, the le$ can be !ulle" anteriorly, like a "rawer bein$ o!ene". A similar fin"in$ can be elicite" with the knee fle*e" at ;> "e$rees by $ras!in$ the thi$h with one han", an" !ullin$ the le$ with the other. Which of the followin$ is the most likely in<ure" structureJ a)Anterior cruciate li$ament b),ateral collateral li$ament c) +e"ial collateral li$ament ")+e"ial meniscus e) &osterior cruciate li$ament Normal ,abs 0. A 004year4ol" woman is un"er$oin$ a "ia$nostic work4u! because she a!!ears to have Cushin$ syn"rome. )he has elevate" levels of cortisol, which are not su!!resse" when she is $iven hi$h4"ose "e*amethasone. AC72 levels are $reater than ;>> !$B m,. A chest *4ray film shows a central, 04cm roun" mass on the hilum of the ri$ht lun$. ronchosco!y an" bio!sies confirm a "ia$nosis of small cell carcinoma of the lun$. Which of the followin$ is the !referre" treatment for this womanJ a) ilateral a"renalectomy b).eneral su!!ort only c)&neumonectomy ")6a"iation an" chemothera!y "irecte" at the lun$ cancer e) 7rans4s!henoi"al hy!o!hysectomy an" !ulmonary lobectomy Normal ,abs ?. A /04year4ol" woman comes to the !hysician because of a Hlum!H in her neck. )he says that her masseuse notice" it : month a$o. 7here is no associate" !ain, !ressure, or hoarseness. )he feels fine an" has no other com!laints. )he has no history of ra"iation e*!osure. (*amination reveals a !al!able thyroi" no"ule that is a!!ro*imately 0 cm. Which of the followin$ is the most a!!ro!riate ne*t ste! in "ia$nosisJ

a) Cuttin$ nee"le bio!sy b)-ine nee"le as!iration '-NA) c)Neck ultrasoun" "))ur$ical resection e)7hyroi" hormone re!lacement Normal ,abs /.A /C4year4ol" man has been havin$ bloo"y bowel movements on an" off for the !ast several weeks. 2e re!orts that the bloo" is bri$ht re", it coats the outsi"e of the stools, an" he can see it in the toilet bowl even before he wi!es himself. When he "oes so, there is also bloo" on the toilet !a!er. After further questionin$, it is ascertaine" that he has been consti!ate" for the !ast ; months an" that the caliber of the stools has chan$e". 7hey are now !encil thin, rather the usual "iameter of an inch or so that was customary for him. 2e has no !ain. Which of the followin$ is the most likely "ia$nosisJ a)Anal fissure b)Cancer of the cecum c)Cancer of the rectum ")(*ternal hemorrhoi"s e) Internal hemorrhoi"s Normal ,abs C.A youn$ man is shot with a .?/ caliber revolver, !oint blank in the lower ab"omen, <ust above the !ubis. 7he entrance woun" is at the mi"line, an" there is no e*it woun". E4ray films show the bullet embe""e" in the sacral !romontory, to the ri$ht of the mi"line. %i$ital rectal e*amination an" !roctosco!ic e*amination are ne$ative, but he has $ross hematuria. 2e is hemo"ynamically stable. Which of the followin$ is the most a!!ro!riate ne*t ste! in mana$ementJ a)C7 scan of the ab"omen b)Intravenous !yelo$ram c)6etro$ra"e cysto$ram ")%ia$nostic !eritoneal lava$e e) (*!loratory la!arotomy Normal ,abs D.A front4seat !assen$er in a car involve" in a hea"4on collision relates that he hit the "ashboar" with his knees, however, he is s!ecifically com!lainin$ of severe !ain in his ri$ht hi!, rather than knee !ain. 2e lies in the stretcher in the emer$ency "e!artment with the ri$ht lower e*tremity shortene", a""ucte", an" internally rotate". Which of the followin$ is the most likely in<uryJ a)-emoral neck fracture b)-racture of the shaft of the femur c)Intertrochanteric fracture

")&osterior "islocation of the hi! e) &osterior "islocation of the knee Normal ,abs @.A D:4year4ol" woman is brou$ht to the !hysician by her "istresse" "au$hter. 7he "au$hter relates that, 0 "ays a$o, her mother be$an to com!lain of ri$ht u!!er qua"rant ab"ominal !ain. )he "i" not want to eat an" Htook to her be" sick.H 7he "au$hter recalls that she com!laine" of chills, nausea, an" some vomitin$. &hysical e*amination reveals an obtun"e", hy!otensive, an" obviously very sick el"erly woman. )he has im!ressive !ain to "ee! !al!ation in the ri$ht u!!er qua"rant, alon$ with muscle $uar"in$ an" reboun". 2er tem!erature is ?> C ':>? -), an" laboratory analysis shows a white cell count of ;;,>>>Bmm0 with multi!le immature forms, a bilirubin of / m$B", an" alkaline !hos!hatase of @?> FB,. 7he serum amylase is normal. An emer$ency sono$ram shows multi!le stones in the $allbla""er, normal thickness of the $allbla""er wall without !ericholecystic flui", "ilate" intrahe!atic "ucts, an" common "uct with a "iameter of ;.: cm. 7he sono$ra!her cannot i"entify stones in the common "uct. In a""ition to I1 flui"s an" antibiotics, which of the followin$ is the most a!!ro!riate ne*t ste! in mana$ementJ a) (lective cholecystectomy b)(mer$ency "ecom!ression of the common "uct c)(mer$ency cholecystectomy ")(mer$ency sur$ical e*!loration of the common "uct e) (mer$ency transhe!atic cholecystostomy Normal ,abs I.A ;/4year4ol" man is shot with a .;; caliber revolver. 7he entrance woun" is in the anterome"ial as!ect of his u!!er thi$h, an" the e*it woun" is about 0 inches lower, in the !osterolateral as!ect of the thi$h. 2e has a lar$e, e*!an"in$ hematoma in the u!!er inner thi$h. 7here are no !al!able !ulses in the foot. 7he bone is intact by !hysical e*amination an" *4ray films. Which of the followin$ is the most a!!ro!riate ne*t ste! in mana$ementJ a)%o!!ler stu"ies b) 1eno$ram c)Arterio$ram ")(mbolectomy e) )ur$ical e*!loration Normal ,abs :>.A D4year4ol" boy !asses a lar$e, bloo"y bowel movement. 2e is hemo"ynamically stable, an" he has a hemo$lobin of :? $B",. Naso$astric as!iration yiel"s clear, $reenish

flui". &hysical e*amination, inclu"in$ anosco!y, is unremarkable. Which of the followin$ is the most a!!ro!riate ne*t "ia$nostic testJ a)Celiac arterio$ram b)Colonosco!y c)6a"ioactively labele" technetium scan ")6a"ioactively ta$$e" re" cell stu"y e)F!!er $astrointestinal en"osco!y Normal ,abs ::.An @:4year4ol" man with Al#heimer "isease who lives in a nursin$ home un"er$oes sur$ery for a fracture" femoral neck. On the /th !osto!erative "ay, it is note" that his ab"omen is $rossly "isten"e" an" tense, but not ten"er. 2e has occasional bowel soun"s. 7he rectal vault is em!ty on "i$ital e*amination, an" there is no evi"ence of occult bloo". E4ray films show a few "isten"e" loo!s of small bowel an" a very "isten"e" colon. 7he cecum measures I cm in "iameter, an" the $as !attern of "istention e*ten"s throu$hout the entire lar$e bowel, inclu"in$ the si$moi" an" rectum. No stool is seen in the films. Other than the ab"ominal "istention, an" the rava$es of his mental "isease, he "oes not a!!ear to be ill. 1ital si$ns are normal for his a$e. Which of the followin$ is the most likely "ia$nosisJ a)-ecal im!action b)+echanical intestinal obstruction c)O$ilvie syn"rome ")&aralytic ileus e)1olvulus of the si$moi" colon Normal ,abs :;.A ?C4year4ol" woman was a!!lyin$ her make4u! while also "rinkin$ her mornin$ cu! of coffee. )he notice" in the mirror that a roun", ;4cm mass woul" move u! an" "own in the lower !art of her neck whenever she swallowe". 2er !hysician confirms that she has a sin$le, firm, thyroi" no"ule in the ri$ht lobe. 7here are no other abnormalities in the history or !hysical e*amination. 2er !ulse is @;Bmin an" re$ular. 7hyroi" stimulatin$ hormone '7)2) is within normal limits. Which of the followin$ is the most a!!ro!riate ne*t ste! in mana$ementJ a) Clinical observation, re!eatin$ the 7)2 at least once a year b)%etermination of 70 an" 7? levels c)6a"ionucli"e thyroi" scan ")-ine nee"le as!iration '-NA) cytolo$y of the mass e)6i$ht thyroi" lobectomy Normal ,abs :0. A ??4year4ol" woman is recoverin$ from a mil" e!iso"e of acute ascen"in$ cholan$itis secon"ary to chole"ocholithiasis. When seen initially, she ha" a s!ikin$ fever, leukocytosis, an" a very hi$h alkaline !hos!hataseG however, all these fin"in$s subsi"e" ra!i"ly after she was !lace" on I1 antibiotics. A sono$ram of the ri$ht u!!er qua"rant on the "ay of a"mission showe" the !resence of $allstones in the $allbla""er, but the

"iameter of the biliary "ucts was normal. It was assume" that she ha" !asse" a common "uct stone, an" !lans to "o an en"osco!ic retro$ra"e cholan$io!ancreato$ram '(6C&) were cancele". While awaitin$ elective cholecystectomy, she a$ain "evelo!e" a fever an" leukocytosis, an" her liver function tests showe" minimal elevation of her bilirubin 'to ;./ m$B",) an" alkaline !hos!hatase 'to ::/ FB,). A re!eat sono$ram shows no chan$es in her biliary "ucts, but now there is a C4cm abscess in the ri$ht lobe of the liver. Which of the followin$ is the most a!!ro!riate treatment for this new "evelo!mentJ a) +etroni"a#ole b),on$4term I1 antibiotics c)(6C& an" biliary "raina$e ")&ercutaneous "raina$e of the liver abscess e) O!en sur$ical resection of the ri$ht lobe of the liver Normal ,abs :?.A //4year4ol", 2I14!ositive man has a fun$atin$ mass $rowin$ out of the anus. 2e can feel it when he wi!es himself after havin$ a bowel movement, but it is not !ainful. -or the !ast C months, he has notice" bloo" on the toilet !a!er, an" from time to time there has also been bloo" coatin$ the outsi"e of the stools. 2e has lost wei$ht, an" he looks emaciate" an" ill. On !hysical e*amination, the mass is easily visible. It measures 0./ cm in "iameter, is fi*e" to surroun"in$ tissues, an" a!!ears to $row out of the anal canal. 2e also has rock4har", enlar$e" lym!h no"es on both $roins, some of them as lar$e as ; cm in "iameter. Which of the followin$ is the most likely "ia$nosisJ a) A"enocarcinoma of the rectum b)Con"yloma acuminata of the anus c)(*ternal hemorrhoi"s ") 6ectal !rola!se e))quamous cell carcinoma of the anus Normal ,abs :/. A DI4year4ol" man with atrial fibrillation "evelo!s an acute ab"omen. When seen ; "ays after the onset of the ab"ominal !ain, he has a silent ab"omen, with "iffuse ten"erness an" mil" reboun". 7here is a trace of bloo" on the rectal e*amination. 2e also has aci"osis an" looks quite sick. E4ray films show "isten"e" small bowel an" "isten"e" ri$ht colon, u! to the mi""le of the transverse colon. Which of the followin$ is the most likely "ia$nosisJ a)Acute !ancreatitis b)+esenteric ischemia c)+i"$ut volvulus ")&erforate" viscus e)&rimary !eritonitis Normal ,abs

:C. A ?;4year4ol", ri$ht4han"e" man has ha" a history of !ro$ressive s!eech "ifficulties an" ri$ht hemi!aresis for / months. 2e has ha" !ro$ressively severe hea"aches for the !ast ; months, which are worse in the mornin$s. At the time of a"mission, he is confuse" an" vomitin$, an" has blurre" vision, !a!ille"ema, an" "i!lo!ia. )hortly thereafter, his bloo" !ressure increases to :I>B::> mm 2$, an" he "evelo!s bra"ycar"ia. Which of the followin$ is most likely the si$nificance of the hy!ertension an" the bra"ycar"iaJ a)7he brain tumor has !ro"uce" tentorial herniation b)7he brain tumor is !ressin$ on the hy!othalamus c)7he chronic sub"ural hematoma has ru!ture" ")7he $enesis of his sym!toms is aortic "issection e) 7here is a near4terminal increase in intracranial !ressure Normal ,abs :D.After a $ran" mal sei#ure, a 0;4year4ol" e!ile!tic woman notices !ain in her ri$ht shoul"er, an" she cannot move it. )he $oes to a minor emer$ency clinic, where she has a limite" !hysical e*amination an" antero!osterior 'A&) *4ray films of her shoul"er. 7he films are rea" as ne$ative, an" she is "ia$nose" as havin$ a s!rain an" $iven !ain me"ication. 7he ne*t "ay, she still has the same !ain an" is unable to move her arm. )he comes to the emer$ency "e!artment hol"in$ her arm close to her bo"y, with her han" restin$ on her anterior chest wall. Which of the followin$ is the most likely "ia$nosisJ a)Acromioclavicular se!aration b)Anterior "islocation of the shoul"er c)Articular cartila$e crushin$ ")&osterior "islocation of the shoul"er e)7orn teres ma<or an" minor muscles Normal ,abs :@. A CI4year4ol" man who smokes an" "rinks an" has rotten teeth, has a har", fi*e", ?4 cm mass in his left neck. 7he mass is <ust me"ial to an" in front of the sternomastoi" muscle, at the level of the u!!er notch of the thyroi" cartila$e. It has been there for at least C months, an" it is $rowin$. Which of the followin$ is the most a!!ro!riate ne*t ste! in "ia$nosisJ a)6a"ionucli"e scan of the thyroi" $lan" b))!utum cytolo$y an" C7 scan of the lun$s c)&anen"osco!y 'tri!le en"osco!y) an" mucosal bio!sies ")O!en incisional bio!sy of the mass e)O!en e*cisional bio!sy of the mass Normal ,abs :I. On the /th !osto!erative "ay, it is notice" that lar$e amounts of clear, !ink, salmon4 colore" flui" are soakin$ the woun" "ressin$s of a !atient who ha" a ne$ative e*!loratory la!arotomy for a stab woun" of the ab"omen. 7he la!arotomy was "one throu$h a mi"line su!raumbilical an" infraumbilical incision. When seen by the sur$ical staff, the

!atient is lyin$ in be" in the su!ine !osition, with the "ressin$s remove". In the "im li$ht of his hos!ital room, the incision a!!ears intact an" not !articularly re" or inflame", but there are in"ee" traces of the clear !ink flui" on his skin. 2e has no s!ecific com!laints. 2e is still N&O an" on I1 flui"s, but has alrea"y been !assin$ $as !er rectum, an" !lans ha" been ma"e to fee" him to"ay. 7he ab"omen is not "isten"e", an" he has normal bowel soun"s. 2e is afebrile. Which of the followin$ is the most a!!ro!riate ne*t ste! in mana$ementJ a) Culture the !ink flui" an" start em!iric antibiotic thera!y b) .ently !robe the woun" at several !oints until !us is foun" an" "raine" c)2el! the !atient out of be" an" have him walk to the e*aminin$ room for !ro!er ins!ection of the woun" "))to! !lans for oral fee"in$s an" start total !arenteral nutrition e)7a!e the woun" securely, bin" the ab"omen, an" avoi" events that woul" su""enly increase his intra4ab"ominal !ressure Normal ,abs ;>.A ;4year4ol" chil" has been shot in the arm in a "rive4by shootin$. 2is brachial artery was !artially transecte", an" there was co!ious blee"in$. 7he (+7s control the site of blee"in$ by local !ressure, an" the chil" is no lon$er losin$ bloo"G however, he is hy!otensive an" tachycar"ic. I1 flui" resuscitation is ur$ently nee"e", but several attem!ts at startin$ !eri!heral I1 lines have been unsuccessful. Which of the followin$ woul" be the best alternative route in this situationJ a) Central line via subclavian !uncture b)2y!o"ermoclysis c)Intraosseous cannulation in the !ro*imal tibia ")&ercutaneous femoral vein cannulation e))a!henous vein cut4"own Normal ,abs ;:.A ;?4year4ol" woman sustains multi!le in<uries in a car acci"ent, inclu"in$ a !elvic fracture. )he is hemo"ynamically stable. Initial assessment shows no va$inal or rectal in<uriesG however, when a -oley catheter is inserte", bloo"y urine is recovere". Which of the followin$ woul" be the best way to evaluate her urolo$ic in<uryJ a) )ono$ram of the bla""er b)Intravenous !yelo$ram c)Cystosco!y ")6etro$ra"e cysto$ram inclu"in$ !ost4voi" films e)6etro$ra"e cysto$ram inclu"in$ views of the ureters Normal ,abs ;;.A C;4year4ol" woman has a ?4cm, har" mass un"er the ni!!le an" areola of her rather small left breast. 7he mass occu!ies most of the breast, but the breast is freely movable from the chest wall. 7here is no "im!lin$ or ulceration of the skin over the mass, an"

careful !al!ation of the a*illa is com!letely ne$ative. A core bio!sy of the breast mass has establishe" a "ia$nosis of infiltratin$ "uctal carcinoma, an" the mammo$ram showe" no other lesions in that breast or the other one. A chest *4ray film an" liver function tests are normal. )he has no sym!toms su$$estive of brain or bone metastasis. Which of the followin$ shoul" be offere" to this woman a),um!ectomy only b),um!ectomy with a*illary sam!lin$ an" !ost4o! ra"iation c) 7otal mastectomy only ")+o"ifie" ra"ical mastectomy 'inclu"in$ a*illary sam!lin$) e) 6a"ical mastectomy 'inclu"in$ com!lete a*illary "issection) Normal ,abs ;0. A /?4year4ol" man, who / years a$o un"erwent a la!arotomy for a $unshot woun" to the ab"omen, is a"mitte" to the hos!ital because of !rotracte" vomitin$ an" !ro$ressive ab"ominal "istention. 7he sym!toms be$an / "ays earlier, an" since then he has not ha" a bowel movement or !asse" any $as. At the time of hos!itali#ation, he has hy!eractive bowel soun"s an" some ab"ominal "iscomfort, but "oes not have an acute ab"omen. 2is ab"ominal *4ray films show "ilate" loo!s of small bowel, multi!le air4flui" levels, an" no free air un"er the "ia!hra$ms. 2e is !lace" on naso$astric suction an" I1 flui"s. After C hours, he "evelo!s fever, leukocytosis, ab"ominal ten"erness, an" reboun" ten"erness, an" his ab"omen is silent. Which of the followin$ is the most a!!ro!riate ne*t ste! in mana$ementJ a) A"" antibiotics b) arium ta$ an" serial ab"ominal *4ray films c)C7 scan of the ab"omen ")F!!er $astrointestinal en"osco!y an" intro"uction of a lon$ intestinal tube e)(mer$ency e*!loratory la!arotomy Normal ,abs ;?. While runnin$ to catch a bus, an" ol" man twists his ankle an" falls on his inverte" foot. Antero!osterior 'A&), lateral, an" mortise *4ray films show "is!lace" fractures of both malleoli. Which of the followin$ woul" be the !referre" form of treatmentJ a)Close" re"uction an" castin$ b))keletal traction c)O!en re"uction an" internal fi*ation ")6e!lacement with a metal !rosthesis e)-usion of the ankle <oint Normal ,abs ;/. A :I4year4ol" man sustains multi!le in<uries in a hi$h4s!ee" automobile collision. 7here is a !neumothora* on the left, for which he has a chest tube !lace". Over the ne*t several "ays, a lar$e amount of air "rains continuously throu$h the tube 'a lar$e Hair leakH), an" "aily chest *4rays show that his colla!se" left lun$ is not e*!an"in$. 7he !atient is not on a res!irator. Which of the followin$ is the most likely cause of these fin"in$sJ

a) Air embolism b)In<ury to the lun$ !arenchyma c)In<ury to a ma<or bronchus ")Insufficient suction bein$ a!!lie" to the chest tube e)7ension !neumothora* Normal ,abs

Block 22 E.planations

:) (*!lanation: 7he correct answer is (. 7his !atient has a traumatic aortic ru!ture, which is the most common cause of imme"iate "eath followin$ a motor vehicle acci"ent. %eceleration causes twistin$ of the aorta an" may lea" to ru!ture. F! to I>= of the in"ivi"uals "ie at the scene, however a hi$h in"e* of sus!icion an" early intervention may save the remainin$ few. 7he sym!toms inclu"e chest !ain, back !ain, an" hy!otension. An$io$ra!hy is the $ol" stan"ar" an" imme"iate sur$ery is man"atory. E4ray fin"in$s inclu"e wi"enin$ of the me"iastinum, alteration of the aortic knob, !leural ca!, tracheal "eviation, bronchial "is!lacement, an" eso!ha$eal "eviation. Car"iac tam!ona"e 'choice A) often results from !enetratin$ trauma near the heart an" is cause" by a flui" collection within the !ericar"ium. 7he clinical features inclu"e hy!otension, "isten"e" neck veins, !ulsus !ara"o*us, an" "istant heart soun"s. &ericar"iocentesis is the treatment. +yocar"ial contusion 'choice ) is most commonly seen when there is a "irect blow to the chest wall. -in"in$s inclu"e ri$ht ventricular "ysfunction, arrhythmias, an" an elevation of creatine kinase. An echocar"io$ram may "ocument ventricular wall motion abnormalities. &ulmonary contusion 'choice C) is a hemorrha$e into the !ulmonary !arenchyma. It is characteri#e" by "ys!nea an" hy!o*ia. 7ension !neumothora* 'choice %) occurs when air is tra!!e" within the !leural s!ace an" lea"s to an increase in the intrathoracic !ressure. -in"in$s inclu"e <u$ular venous "istention, hy!otension, tracheal "eviation, an" a me"iastinal shift. 7ension !neumothora* often occurs followin$ blunt trauma. 7reatment is with nee"le thoracocentesis. ;) (*!lanation: 7he correct answer is A. )wellin$ of the knee after trauma usually "enotes the !resence of a si$nificant in<ury. 7he tests "escribe" 'anterior "rawer an" ,achman test) are classic for an in<ury to the anterior cruciate li$ament. 7he lateral collateral li$ament 'choice ), if "isru!te", woul" allow the le$ to be bent inwar" to a $reater e*tent than normally !ossible 'varus test). 7he

me"ial collateral li$ament 'choice C), when in<ure", woul" !ro"uce the o!!osite fin"in$s: the le$ coul" be bent outwar" more than the normal le$ 'val$us test). 7he me"ial meniscus 'choice %), when in<ure", !ro"uces loose intraarticular bo"ies an" lockin$ of the knee. 7he !osterior cruciate li$ament 'choice () is much less commonly in<ure" than the anterior cruciate. When it is in<ure", it !ro"uces the very o!!osite fin"in$s to those "escribe" in the vi$nette: the le$ coul" be !ushe" backwar", as if a "rawer was bein$ close" rather than o!ene". 0) (*!lanation: 7he correct answer is %. 7he en"ocrine worku! is in"icative of ecto!ic AC72 !ro"uction, an" the obvious site is small cell carcinoma of the lun$. 7he lun$ cancer is what is $oin$ to kill this woman, not the en"ocrine manifestations of the tumor. Althou$h small cell carcinoma of the lun$ is rarely cure", lon$er survival can be obtaine" with ra"iation an" chemothera!y. A"renalectomy 'choice A) woul" a""ress the en"ocrine !roblem by "e!rivin$ the ecto!ic AC72 of its tar$et $lan". ut, as !ointe" out above, the lethal "isease here is the lun$ cancer. .eneral su!!ort only 'choice ) woul" lea" to "eath in about ; months. 6a"iation an" chemothera!y can !rolon$ survival by a!!ro*imately ; years. &neumonectomy 'choice C) is not the treatment for small cell carcinoma of the lun$. It is the a!!ro!riate treatment for resectable an" !otentially curable nonJsmall cell cancers of the lun$. 7he combination of hy!o!hysectomy an" lobectomy 'choice () is wron$ for several reasons. &ituitary microa"enomas su!!ress with hi$h4"ose "e*amethasone, an" their !ro"uction of AC72 is ty!ically much lower than that seen in ecto!ic "isease 'i.e., P;>> !$Bm,). 7hus, this woman "oes not nee" !ituitary sur$ery. As far as the lun$ is concerne", sur$ery is not the treatment for small cell carcinoma, an" lobectomy woul" not be a!!licable for a central, hilar tumor. ?) (*!lanation: 7he correct answer is C. An ultrasoun" is the first ste! in the evaluation of a !al!able thyroi" no"ule. An ultrasoun" is a noninvasive technique that can "etermine if the no"ule is cystic or soli", the e*act si#e of the lesion, an" whether there are any a""itional masses. If the no"ule is cystic, a fine nee"le as!iration '-NAG choice ) is !erforme". If the cyst "isa!!ears an" the cytolo$y is beni$n, no a""itional treatment is necessary. If the cyst remains, further evaluation is necessary. If the no"ule is soli" an" P 0 cm, an -NA is !erforme". If the cytolo$y is beni$n, thyroi" hormone re!lacement 'choice () is $iven to su!!ress $rowth. If the no"ule is soli" an" O 0 cm., a nee"le bio!sy 'choice A) is !erforme". If the !atholo$y is in"eterminate or mali$nant, sur$ical resection 'choice %) is the treatment. If the no"ule is beni$n, thyroi" hormone is $iven. /)

(*!lanation: 7he correct answer is C. 7he combination of re" bloo" coatin$ the stools an" a chan$e in bowel habit an" stool caliber s!ells out cancer of the rectum in someone in this a$e $rou!. Anal fissure 'choice A) is ty!ically seen in youn$ women who have very !ainful bowel movements with streaks of bloo". &ain is the "ominant sym!tom in this con"ition. Cancer of the cecum 'choice ) lea"s to anemia an" occult bloo" in the stools, but the bloo" is rarely seen. If it is, the entire stool is bloo"y. -urthermore, there is no chan$e in bowel habit or stool caliber when the tumor is so !ro*imal in the colon. (*ternal hemorrhoi"s 'choice %) hurt an" itch, but they rarely blee". Internal hemorrhoi"s 'choice () "o in"ee" blee", but they "o so without chan$in$ the !attern of bowel movements or the caliber of the stools. C) (*!lanation: 7he correct answer is (. 2e has an obvious in"ication for e*!loratory la!arotomy: a $unshot woun" to the ab"omen. 2e also has evi"ence of in<ury to the urinary bla""er, but that will be "ealt with at the same time that other intraab"ominal in<uries are foun" an" re!aire". C7 scan 'choice A) woul" not chan$e the sur$ical a!!roach an" the sur$ical in"ication. C7 scan is calle" for in cases of blunt trauma to "ia$nose intraab"ominal blee"in$ an" to i"entify intraab"ominal in<uries. Intravenous !yelo$ram 'choice ) woul" in"ee" show the bla""er in<ury, as woul" a retro$ra"e cysto$ram 'choice C). 2owever, we alrea"y know clinically that there is a bla""er in<ury: we know the tra<ectory of the bullet an" we have bloo" in the urine. %ia$nostic !eritoneal lava$e 'choice %) is use" to "ia$nose intraab"ominal blee"in$ in blunt trauma, when the !atient is not stable enou$h to be taken to the C7 scanner. In many centers the "ia$nostic !eritoneal lava$e has been re!lace" by sono$ram "one in the emer$ency "e!artment by the trauma team. D) (*!lanation: 7he correct answer is %. 7he mechanism of in<ury is classic. As the knee hits the "ashboar" in the sittin$ !osition, the femoral hea" is "riven backwar" an" out of the socket. 7he !osition of the in<ure" e*tremity is also ty!ical, with the internal rotation !ro"uce" by the !osteriorly "islocate" femoral hea". 7his in<ury is an ortho!e"ic emer$ency because of the tenuous bloo" su!!ly of the femoral hea". -emoral neck 'choice A) an" intertrochanteric 'choice C) fractures are seen in el"erly !atients who fall an" Hhurt their hi!.H 7hey !resent with a shortene" e*tremity that is e*ternally rotate". A fracture" shaft of the femur 'choice ) woul" !ro"uce !ain ri$ht there 'not in the hi!) an" woul" have an obvious "eformity where the thi$h has an an$ulation that is clearly abnormal. &osterior "islocation of the knee 'choice () is also an ortho!e"ic emer$ency because of the !otential "isru!tion of the !o!liteal artery, but the !ain an" the "eformity woul" be at the knee.

@) (*!lanation: 7he correct answer is . 7he "ia$nosis is acute ascen"in$ cholan$itis. 7his "ea"ly "isease is seen in !atients with lon$4stan"in$ $allstones who $et one or more stones in the common "uct, where they !ro"uce !artial obstruction that allows ascen"in$ infection. 7he fact that the sono$ra!her cannot see the offen"in$ stones is irrelevant, as stones in the common "uct are often not seen in sono$rams. 7he stu"y shows the in"irect evi"ence of obstruction: "ilate" "ucts. 7he chills, very hi$h fever, an" e*tremely elevate" alkaline !hos!hatase are "ia$nostic. An a"vance" clinical form is !resent here, with obtun"ation, ri$ht u!!er qua"rant ten"erness, an" hy!otension. 7he key com!onent of thera!y is imme"iate "ecom!ression of the common "uct, which is full of !us. 2ow it is achieve" is less im!ortant. (mer$ency en"osco!ic retro$ra"e cholan$io!ancreato$ra!hy '(6C&) is usually the first choice, but it can be "one by !ercutaneous transhe!atic cholan$io$ra!hy '&7C) or by o!en sur$ery. (lective cholecystectomy 'choice A) will in"ee" be nee"e" once the acute !roblem is resolve". ut, if our !lannin$ inclu"e" only such elective sur$ery, the !atient woul" never $et it: she woul" be "ea". (mer$ency cholecystectomy 'choice C) woul" not a""ress the issue of !us in the common "uct. )he "oes not have acute cholecystitis, as evi"ence" by the normal thickness of the $allbla""er wall an" the absence of !ericholecystic flui". -urthermore, acute cholecystitis woul" not have !ro"uce" the im!ressive levels of alkaline !hos!hatase. )ur$ical e*!loration of the common "uct 'choice %) is more than she can tolerate at this time. )he "oes not nee" all the stones remove" with a lon$ o!erative !roce"ure. )he nee"s the !us out. 7he rest will come later. Cholecystostomy 'choice () is another choice for very sick !eo!le with acute cholecystitis, which is not the "ia$nosis. I) (*!lanation: 7he correct answer is (. 7his one shoul" be obvious. 7here is no question that the femoral vessels are in<ure", an" the e*!an"in$ hematoma !lus absent !ulses in"icate that the femoral artery is involve" 'the vein may or may not be in<ure"). )ur$ical e*!loration starts with !ro*imal an" "istal control. Once the hematoma is safely entere", the e*tent of the in<uries can be ascertaine" an" the !ro!er re!air "one. %o!!ler stu"ies 'choice A) are won"erful when we are tryin$ to ascertain whether bloo" is flowin$ throu$h a vessel. 7he clinical fin"in$s here are obvious. No fancy technolo$y is nee"e". 1eno$rams 'choice ) are not nee"e" !rior to sur$ery. We can look at the vein "urin$ the o!eration. Arterio$rams 'choice C) are very often use" in vascular trauma, but are not nee"e" here. We woul" use an arterio$ram if the anatomic location of the in<ury su$$este" vascular involvement, but the clinical si$ns "i" not confirm such sus!icion. Arterio$rams are also use" when the s!ecific sur$ical a!!roach is "ictate" by !recise knowle"$e of the site of e*travasation, a situation that "oes not a!!ly here. An embolectomy 'choice %) will !robably be "one at the en" of the sur$ical !roce"ure, as a routine !art of removin$ "ebris from the lumen of a vessel that has been re!aire" or re4anastomose". 2owever, embolectomy alone is the

wron$ answer for this vi$nette. 7he absent !ulses are "ue to "isru!tion of the artery, not to emboli#ation from a "istant source. :>) (*!lanation: 7he correct answer is C. In this a$e $rou!, with no obvious anal !atholo$y an" ne$ative $astric as!irate, the lea"in$ cause of $astrointestinal blee"in$ is +eckelAs "iverticulum. 7he s!ecific source is ulceration of the normal ileal mucosa by aci" !ro"uce" by $astric mucosa in the "iverticulum. 7he technetium scan i"entifies that ecto!ic $astric mucosa. Arterio$ram 'choice A) as a "ia$nostic ste! is a very invasive stu"y that is a!!ro!riate only for very lar$e hemorrha$e 'O; m,Bmin) with clear $astric as!irate. An arterio$ram may also be in"icate" for thera!y 'emboli#ation) in severe $astrointestinal blee"in$. Colonosco!y 'choice ) woul" not i"entify the +eckelAs "iverticulum. Colonosco!y is often nee"e" in the ol"er !atient with lower $astrointestinal blee"in$, in whom the source of the hemorrha$e is likely to be !oly!s, cancer, "iverticula, or an$io"ys!lasia. A ta$$e" re" cell stu"y 'choice %) is often use" as a !relu"e to an arterio$ram in !atients with substantial lower $astrointestinal blee"in$. F!!er $astrointestinal en"osco!y 'choice () woul" have been a!!ro!riate if the $astric as!irate ha" !ro"uce" bloo". ::) (*!lanation: 7he correct answer is C. O$ilvie syn"rome is a ty!e of colonic "ysfunction often seen in el"erly !atients who are not too active to be$in with an" are then further immobili#e" by e*tra4ab"ominal sur$ery. Colonosco!y rules out obstructin$ cancer 'always a consi"eration in this a$e $rou!) an" allows the $as to be sucke" out as the instrument a"vances. A lon$ tube is then left in !lace. -ecal im!action 'choice A) is always a $oo" thin$ to look for in ol" immobili#e" !eo!le with ab"ominal "istention. 2owever, the rectal vault woul" have been full of feces, an" the *4ray films mi$ht have shown the fecal column e*ten"in$ u! into the si$moi" an" "escen"in$ colon. Neither mechanical intestinal obstruction 'choice ) nor !aralytic ileus 'choice %) woul" "evelo! from hi! sur$ery. When these com!lications occur after ab"ominal sur$ery, they affect !rimarily the small bowel, not the colon. 1olvulus of the si$moi" 'choice () is another $oo" thou$ht in the "isten"e" ol" !atient, but the ra"iolo$ic !icture woul" have been "ifferent, with a hu$e "isten"e" si$moi" way u! into the ri$ht u!!er qua"rant an" ta!erin$ towar" the left lower qua"rant with the classic ima$e of a H!arrotAs beak.H :;) (*!lanation: 7he correct answer is %. +ost thyroi" no"ules are beni$n, an" sur$ery must be reserve" for selecte" can"i"ates with the hi$hest likelihoo" of mali$nancy. -ine nee"le as!iration '-NA) is the best way

to make the selection. If rea" by an e*!erience" !atholo$ist as ne$ative for cancer, !atients can be safely followe". If rea" as either in"eterminate or !ositive for cancer, sur$ery woul" be require". Choosin$ sur$ical can"i"ates this way, cancer is foun" at sur$ery in ;>= to ?>= of !atients o!erate", a vast im!rovement over earlier selection metho"s, where the yiel" was aroun" :>= to :/=. Clinical observation alone 'choice A) woul" not be a!!ro!riate. 7hyroi" cancers $row slowly, but they still nee" to be "ia$nose" an" treate". Inci"entally, they ty!ically "o not affect thyroi" function, so that followin$ the 7)2 woul" not alert you to the !resence of mali$nancy. 7hyroi" no"ules can be beni$n but hy!erfunctionin$ 'to*ic a"enoma), an" therefore thyroi" function must be "etermine". ut, that has alrea"y been "one here with the normal 7)2. -urther !ursuit of 70 an" 7? 'choice ) is unnecessary in someone with no clinical evi"ence of hy!erfunction 'normal !ulse). 7hyroi" scan 'choice C) woul" have been the answer :> or ;> years a$o, before -NA "is!lace" it as the best way to select sur$ical can"i"ates. In the ol" "ays, a col" no"ule raise" sus!icions of mali$nancy but $ave low yiel"s at sur$ery. Without a "ia$nosis of cancer, or an in"eterminate -NA, one cannot <ustify the e*tremely a$$ressive a!!roach of sur$ery as the ne*t ste! in mana$ement. 7hus, choice ( is clearly wron$. :0) (*!lanation: 7he correct answer is %. ,iver abscess com!licatin$ biliary tract "isease is "escribe" as H!yo$enicH abscess 'to contrast it with amebic abscess), an" it requires "raina$e like any abscess anywhere else in the bo"y. 7he !ercutaneous route is favore". +etroni"a#ole 'choice A) is the thera!y of choice for amebic abscesses of the liver, an" that con"ition re!resents the only e*ce!tion to the rule that all abscesses have to be "raine". 2owever, this is not an amebic abscess. Amebic abscesses are seen in men '? to : ratio com!are" with women) who come from +e*ico, where the "isease is very common. ,on$4term antibiotics 'choice ) will not reach an" sterili#e an abscess. Abscesses have to be "raine". (n"osco!ic retro$ra"e cholan$io!ancreato$ram '(6C&) 'choice C) is often ur$ently nee"e" to treat acute ascen"in$ cholan$itis, but it will not "o anythin$ for a liver abscess. 6esection 'choice () is not nee"e" for a liver abscess. %raina$e is enou$h. :?) (*!lanation: 7he correct answer is (. 7he entire "escri!tion is classic for anal cancer, but the clincher is the !resence of metastasis in the in$uinal no"es. A"enocarcinoma of the rectum 'choice A) coul" look like this if it arose very low in the rectum, but it woul" not metastasi#e to in$uinal no"es. Con"yloma acuminata 'choice ) coul" $ive fun$atin$ masses, but it woul" not lea" to cache*ia an" woul" not !ro"uce the rock4har" in$uinal no"es. 1iral infections may !rece"e the "evelo!ment of this kin" of tumor 'an" coul" coe*ist with it), but it woul" be wron$ to assume that all the !atient has is the beni$n viral !rocess. (*ternal hemorrhoi"s 'choice C) are not fun$atin$ masses, they "o not blee", an" they "o not lea" to in$uinal

a"eno!athy or cache*ia. 6ectal !rola!se 'choice %) woul" !ro"uce a !rotru"in$ mass with concentric mucosal fol"s, woul" come in an" out with strainin$, an" woul" be a nuisance44but it woul" not !ro"uce a"eno!athy an" cache*ia. :/) (*!lanation: 7he correct answer is . 7he settin$ of an ol" !atient with atrial fibrillation 'or a recent myocar"ial infarction) who "evelo!s an acute ab"omen, stron$ly su$$ests embolus to the mesenteric vessels. 7he combination of ab"ominal !ain an" a trace of bloo" in the lumen is also ty!ical, as is the *4ray film outlinin$ the territory su!!lie" by the su!erior mesenteric. 2e !robably has a "ea" bowel by now, as evi"ence" by his aci"osis an" severe illness. Any one of the other o!tions coul" e*ist, but none are the most likely. Acute !ancreatitis 'choice A) woul" be e*!ecte" in an alcoholic or a !atient with biliary tract "isease. 7here woul" have been no bloo" in the lumen. +i"$ut volvulus 'choice C) woul" be far more likely to ha!!en to an infant with malrotation. A !erforate" viscus 'choice %) is in"ee" !ossible, but the *4ray films woul" have shown free air rather than "isten"e" bowel. &rimary !eritonitis 'choice () woul" have been a consi"eration in a !atient with !ree*istin$ ascites. :C) (*!lanation: 7he correct answer is (. 7he clinical !icture is that of a brain tumor with increase" intracranial !ressure. 2owever, the "evelo!ment of hy!ertension an" bra"ycar"ia 'Cushin$As refle*) si$nifies that the brain has run out of com!ensatory mechanisms to minimi#e the intracranial !ressure elevation $enerate" by increase" intracranial volume. When that !oint is reache", brain !erfusion suffers an" "eath is imminent. Choices A an" correctly i"entify the "isease as a brain tumor, but assi$n an incorrect meanin$ to the hemo"ynamic chan$es. 7entorial herniation woul" lea" to res!iratory arrest. &ressure on the hy!othalamus is not the reason for the Cushin$As refle*. Choice C makes a wron$ "ia$nosis. Chronic sub"ural hematomas are seen in very ol" or alcoholic !atients, where they !ress on the corte*4but "o not Hru!tureH an" create a su""en catastro!he in that fashion. Aortic "issection 'choice %) ha!!ens to hy!ertensive !atients, but what they $et is chest an" back !ain, not a lon$4stan"in$ neurolo$ic !icture as "e!icte" here. :D) (*!lanation: 7he correct answer is %. 7he mechanism of in<ury 'massive contraction of all muscles in the area) an" the misse" "ia$nosis on a sin$le view A& *4ray film are classic for !osterior "islocation of the shoul"er. A*illary view *4ray films are nee"e" to make the "ia$nosis. Acromioclavicular se!aration 'choice A) woul" have been obvious on !hysical

e*amination an" on the *4ray film taken Anterior "islocation 'choice ) is far more common than !osterior "islocation. 2owever, it ha!!ens with re$ular trauma, has a very ty!ical !osture where the arm is hel" close to the bo"y but the forearm an" han" are rotate" out as if rea"y to shake han"s, an" is easily seen on *4ray films. Crushin$ of the articular cartila$e 'choice C) an" tearin$ of shoul"er $ir"le muscles 'choice () are not common in<uries followin$ sei#ures. :@) (*!lanation: 7he correct answer is C. In this settin$ 'ol" man who smokes an" "rinks an" has rotten teeth), the clinical "ia$nosis is metastatic squamous cell carcinoma to a cervical lym!h no"e, from a !rimary 'or multi!le !rimaries) somewhere in the mucosa of the aero"i$estive tract. (n"osco!y an" bio!sies shoul" establish the "ia$nosis. 7hyroi" cancer coul" in"ee" metastasi#e to neck no"es, sometimes before the !rimary tumor is !al!able. In this !articular settin$, however, a ra"ionucli"e scan 'choice A) woul" be a very "istant secon" choice. )!utum cytolo$y an" C7 scan of the lun$s 'choice ) is another tem!tin$ thou$ht for a smoker. ut, when lun$ cancer metastasi#es to the cervical no"es, it affects the su!raclavicular no"es, not the no"es hi$her u! in the neck. O!en bio!sy of the neck mass is an absolute no4no, whether it is e*cisional 'choice () or incisional 'choice %). %oin$ so will in fact confirm the "ia$nosis, but at the cost of contaminatin$ the tissues an" interferin$ with the !lacement of incisions for the "efinitive sur$ery. -urthermore, the location of the !rimary 'or !rimaries) woul" not be establishe". :I) (*!lanation: 7he correct answer is (. 7he situation "escribe" is that of a woun" "ehiscence that has not yet !ro$resse" to a woun" evisceration. 7he former can be "ealt with at leisure, if the latter is avoi"e". 2e will eventually require re4closure, but it can be "one whenever it is most convenient. Cultures an" antibiotics 'choice A) assume the !ink flui" to be a si$n of infection. It is not. It is normal !eritoneal flui" 'with a trace of bloo" still in it from the recent sur$ery) that is see!in$ out throu$h the unheale" woun". &robin$ 'choice ) will not !ro"uce !us, but it mi$ht hasten the "rea"e" evisceration. 2e is afebrile, an" the woun" is not re". Clear !ink salmon4colore" flui" means "ehiscence, not infection. .ettin$ the !atient out of be" 'choice C) is the last thin$ you want to "o. If this a"vice were followe", the !erson hel!in$ the !atient woul" soon be lookin$ at a han"ful of small bowel as it comes rushin$ out of the belly. 7he flui" "escribe" is not bowel contentsG this is not a fistula. 7here is no nee" to $o to !arenteral nutrition 'choice %). ;>) (*!lanation: 7he correct answer is

C. In very small chil"ren, the !referre" alternate route, if !eri!heral veins cannot be cannulate", is the !lacement of a trocar in the bone marrow of a lon$ bone. 7he u!!er tibia is usually chosen. )ubclavian !uncture 'choice A) was at one time routinely use" in the resuscitation of !atients in shock, but it is now reco$ni#e" that it is invasive an" has many !ossible com!lications. -urthermore, frequently in the trauma settin$, the hea" an" neck an" u!!er thora* are not accessible, as many other life4savin$ an" "ia$nostic e*aminations are bein$ "one. 7he e*tremities, on the other han", are free to be use". 7he subcutaneous tissue 'choice ) cannot take flui" fast enou$h to meet the nee"s here, 7he femoral vein by !ercutaneous !uncture 'choice %) or a sa!henous vein cut4"own 'choice () are the !referre" alternate routes in an a"ult when !eri!heral veins cannot be ra!i"ly cannulate". ;:) (*!lanation: 7he correct answer is %. loo"y urine !lus !elvic fracture equals bla""er in<ury in either $en"er, or bla""er or urethral in<ury in the male. In this case, with the very short an" well4!rotecte" female urethra not bein$ sus!ecte", only the bla""er is the obvious can"i"ate. In<ectin$ "ye an" takin$ *4ray films will show the e*travasation, but it is im!ortant to inclu"e !ost4voi" films because e*travasation at the bla""er neck can be obscure" by the "ye that is fillin$ the bla""er. )ono$ram 'choice A) is a $oo", noninvasive way to look at thin$s, but here we can $et far better "etail with a stu"y 'the retro$ra"e cysto$ram) that is not !articularly invasive. Intravenous !yelo$ram 'choice ) woul" show "ye e*travasation, but with far less "etail than that !rovi"e" by "irect in<ection. Cystosco!y 'choice C) woul" be invasive an" not easy to "o. When a cystosco!y is "one, flui" is in<ecte" into the bla""er to e*!an" it an" see the walls. In this case, the in<ecte" flui" woul" $o out into the !eritoneal cavity or the !re!eritoneal s!ace. ,ookin$ at the ureters 'choice () is not necessary when bla""er in<ury is sus!ecte". 7he ureters are rarely in<ure" in blunt trauma. ;;) (*!lanation: 7he correct answer is %. 7he mass is too lar$e, in a breast that is too small, to allow an a"equate lum!ectomy. ,ocal control of the tumor requires mastectomy. 2owever, mastectomy alone is not sufficient. We nee" to know what is ha!!enin$ in the a*illa to make a "ecision re$ar"in$ !osto!erative systemic thera!y. 7he !hysical e*amination was ne$ative, but the reliability of that fin"in$ is not much better than fli!!in$ a coin. ,um!ectomy alone 'choice A) cannot be "one here, an" it woul" not be the a!!ro!riate mana$ement even if the cancer ha" been smaller. ,um!ectomy, a*illary sam!lin$, an" !ost4o! ra"iation 'choice ) woul" have been the correct answer for a smaller tumor in a lar$er breast. 7otal mastectomy alone 'choice C) woul" have been a wise !alliative choice if she ha" ha" evi"ence of "istant metastasis. )he "oes not. We have to $o for cure. We nee" the stan"ar" !otentially curative o!eration: mo"ifie" ra"ical mastectomy, which inclu"es

a*illary sam!lin$. 6a"ical mastectomy 'choice () is no lon$er use" for the treatment of breast cancer. It offers no survival a"vanta$e over the less mutilatin$ mo"ifie" ra"ical. ;0) (*!lanation: 7he correct answer is (. 2e came in with mechanical intestinal obstruction "ue to a"hesions, an" has now "evelo!e" si$ns of bowel stran$ulation. If the stran$ulate" loo! is still viable, it has to be free" imme"iately. If it is necrotic, it has to be resecte" with equal ur$ency to !revent continue" !eritoneal soilin$. Antibiotics 'choice A) will not !rovi"e viability to a com!romise" loo! of bowel, nor !revent !eritonitis if the loo! is "ea". arium ta$ 'choice ) is what we "o in the !osto!erative !erio" after ab"ominal sur$ery when we cannot "eci"e whether a slu$$ish bowel has !aralytic ileus or early mechanical obstruction. 7his is not the situation here. C7 scan 'choice C) is our universal answer when we "o not know what is ha!!enin$ insi"e the belly. 2ere we "o. 2a" he shown u! with obstruction, an" no reasonable etiolo$y for it 'no !rior sur$ery, no hernias), we mi$ht have "one a C7. (n"osco!y an" a lon$ tube 'choice %) will not take care of "ea" or "yin$ bowel. ;?) (*!lanation: 7he correct answer is C. &recise ali$nment of the "is!lace" fra$ments is nee"e" to ensure that the ti$ht mortise of the ankle <oint is restore". Close" re"uction an" castin$ 'choice A) is unlikely to !rovi"e the necessary reali$nment. )keletal traction 'choice ), in $eneral, is in"icate" only in areas of the bo"y where stron$ muscle $rou!s !ull broken bones into unacce!table !ositions. Artificial <oints are usually use" for a"vance" articular "isease. In the trauma settin$, re!lacement with a !rosthesis 'choice %) is as a rule reserve" for fractures where avascular necrosis is !re"ictable. -usion of a <oint 'choice () is the ultimate ste! when everythin$ else has faile". It woul" not be the first choice for a relatively common fracture. ;/) (*!lanation: 7he correct answer is C. 7his !atient most likely has an in<ury to a ma<or bronchus. In a""ition to the wrenchin$ effect of a su""en "eceleration, these can ha!!en when a ma<or blow to the chest occurs at a time when the $lottis is close". If not reco$ni#e" ri$ht away by the !resence of subcutaneous em!hysema, they become evi"ent once the air leak !ersists an" the lun$ "oes not re4e*!an". Air embolism 'choice A) is manifeste" by su""en "eath shortly after a !atient with unreco$ni#e" in<uries to the tracheobronchial tree in !ro*imity to ma<or intrathoracic vessels is !lace" on a res!irator. In<ure" lun$ !arenchyma 'choice ) can in"ee" leak air an" !ro"uce a !neumothora*, but it ty!ically heals ra!i"ly. It is the

"elaye" resolution of the !neumothora* that su$$ests that a ma<or bronchus, rather than lun$ !arenchyma, has been "ama$e". )uction a!!lie" to a chest tube 'choice %) is use" to accelerate the rate of resolution of a !neumothora*, but the lar$e amount of air "rainin$ in this case in"icates that the !leural s!ace fills in as quickly as it is bein$ "raine" out. No amount of suction can kee! u! with what literally is a situation in which the chest tube is suckin$ out the air in the room, by way of the bronchial tear. 7ension !neumothora* 'choice () occurs when air cannot leave the !leural s!ace an" !ressure buil"s u! within. 7he manifestations are res!iratory "istress an" e*trinsic car"io$enic shock. ,aboratory 1alues Z inclu"es in the iochemical &rofile ')+A4:;) ,OO%, &,A)+A, )(6F+ 6(-(6(NC( 6AN.( )I 6(-(6(NC( IN7(61A,) Z Alanine aminotransfer'A,7,.&7 at 0>c)............@44;>FB,.............................@44;>FB, Amylase,)erum......................;/44:;/FB, Z As!artate aminotransferase'A)7,.O7 at 0>c).......................@44 ;>FB, ...............@^^;>FB, ilirubin,)erum'a"ult) totalBB%irect.............>.:44:.>m$B",BB>.>44 >.0m$B",.............;.:D+molB,BB>./+molB, Calcium,)erum'ca;M) .............@.?44:>.; m$B",.............;.:44;.@mmmolB, Cholesterol,serum.................6ec:P;>>m$B",.........P/.;mmolB, Cortisol,)erum.............>@>>h:/44;0+$B",.................:0@44C0/nmolB, :C>>h:04:/+$B",.................@;44?:0nmolB, Creatine 5inase,)erum.....+ale: ;/44I>FB,......................;/44I>FB, -emale::>44D>FB,...D>FB, ZCreatinine )erum...........>.C44:.;m$B",.................../044:>C+molB, (lectrolytes,)erum )o"ium'NaM)..........:0C44:?/m(qB,................:0C44:?/mmolB, Chlori"e'cl4)..............I/44:>/m(qB,...................I/44:>/mmolB, Z&otassium'kM)................0./44/.>m(qB,.................0./44/.>mmolB, icarbonate'2CO0).....;;44;@m(qB,................;;44;@m(qB, +a$nesium'm$;M)............:./44;.>m(qB,............:./44;.>mmolB, (striol,7otal,)erum'in &rre$anay) ;?44;@wksBB0;440Cwks.................0>44:D>n$Bm,BBC>44;@>n$Bm,.........:>?4/I>BB;@>4 ID>nmolB, ;@440;wksBB0C44?>wks.................?>44;;>n$Bm,BB@>440/>n$Bm,.........:?>4DC>BB;@>4 :;:>nmolB, -erritin,)erum.............+ale::/44:/>n$Bm,....................:/44;>>+$B, -emale::;44:/>n$Bm,....................:;44:/>+$B, -ollicle44stimulatin$ hermone,)erumB&lasma..+ale:?44;/mIBFBm,.............?44;/FB,

-emale:&remeno!ause ?440>mIFBm,....?440>FB, +i"Cycle &eak :>44I>mIFBm,....:>44I>FB, &remeno!ause ?>44;/>mIFBm,....?>44;/>FB, .ases,Arterial loo"'room air) !2...........D.0/44D.?/.................W2MX0C44??nmolB, &CO; ...............004?/mm2$.........?.?44/.I5&a &O; ...............D/4:>/mm2$.........:>.>44:?.>5&a Z.lucose,)erum......-astin$:D>44::>m$B",.............0.@44C.:mmolB, ;4h &ost!ran"ial: P:;>m$B",.....................PC.CmmolB, .rowth 2armone....ar$inine )timulation...-astin$:P/n$Bm,.......P/+$B, &rovocative )timuli:ODn$Bm,....O+$B, Immuno$lobulins,)erum I$A..............................DC440I>m$B",...............>.DC440.I>$B, I$(..>4............................40@>IFBm,...............>44 0@>5IFB, I$...............C/>44:/>>m$B",....................C./44:/$B, I$+.............?>440?/m$B",.......................>.?440.?/$B, Iron............../>44:D>+$B",....................I440>+molB, ,actate "ehy"ro$enase,)erum...................?/44I>FB,...?/44I>FB, ,utein$in$ harmone,)erumB&lasma....+ale:C44;0mIFBm,.........................C44;0FB, -emale:-ollicular &hase /440>mlFBm,................./440>FB, +i" Cycle D/44:/>miFBm,.....D/44:/>FB, &ostmeno!ause 0>44;>>mIFBm,.....0>44;>>FB, Osmolality )erum..................;D/44;I/mOsmolB5$...............;D/44;I/mOsmolB5$ &arathyroi" hormone,)erum,N4terminal............;0>44C0>!$Bm,.............;0>4C0>n$B, Z&hos!hate'alkaline),)erum'!4N&& at 0>_c).....;>44D>FB,............;>44D>FB, Z&hos!lorous'inOr$anic),)erum......................0.>44?./m$B",.................:.>44:./mmolB, &rolactin,)erum'h&6,).........P;>n$Bm,...................P;>+$B, Z&roteins,)erum 7otal'recumbent)...................C.>44D.@$B",.................C>44D@$B, Albumin................0./44/./$B",.....................0/44//$B, .lobulin............... ;.044/./$B",....................;0440/$B, 7hyroi"4)timulatin$ harmone,)erum or&lasma.............>./44/.>+FBm,......>./44/.>mFB, 7hyroi"al4io"ine' :;0BF!"ate.................@440>= of A"ministere" "oseB ;? h...................>.>@44>.0> B ;? B h 7hyro*ine'7?),)erum........................../44:;+$B",.....C?44://nmolB, 7ri$lyceri"es,)erum.............................0/44:C>m$B",......................C?44://nmolB, 7riio"othyronine'70),)erum'6IA)......::/44:I>n$B",.............:.@44;.InmolB, 7riio"othyronine'70 ) resin F!take.........................;/440/=.............>.;/44>.0/ ZFric nitro$en,)erum' FN).....................D44:@m$B",.....................:.;440.ommol FreaB, ZFric aci",)erum.........................0.>44@.;m$B",.......................>.:@44>.?@mmolB, C(6( 6O)&INA, -,FI% Cellcount.....................>./cellsBmm0........................>./Z:>C, Chlori"e...................::@44:0;m(qB,...............::@44:0;mmolB,

.amma .lobulin................044:;= total &r!teins...................>.>044>.:; .lucose............................ ?>44D>m$B",..................................;.;440.ImmolB, &ressure.....................D>44:@>mm h;O...........................D>44:@>mm 2 ;O &roteins,7otal...........................................P?>m$B",..................................P>.?>$B, 2ematolo$ic lee"in$ 7ime'7em!late).......................;.D+inutes.......................;.D+inutes (rythrocyte Count..............................+ale: ?.044/.ImillionBmm0...................?.044 /.IZ:>:;B, -emale: 0./44/./millionBmm0...................0./44/./Z:>:;B, (rythrocyte )e"imentation rate'Wester$ern)...............+ale:>44 :/mmBh..................................>44:/mmBh .-emale:>44;>mmBh..................................>44;>mmBh 2ematocrit ....................................+ale:?:44/0=.................................>.?:44>./0 -emale:0C44?C=.........................>.0C44>.?C. 2emo$loblin A:c............................PUC=...............>.>C= 2emo$lobin, loo".......................... +ale ::0./44:D./$B",. .......................;.>I44 ;.D:mmolB, -emale ::;.>44:C.>$B",. .......................:.@C44;.?@:mmolB, 2emo$loblin,&lasma.....................:44?m$B",............................>.:C44>.C;mmolB, ,eokocyte Count an" %ifferential ,eokocyte Count.....................?/>>44::,>>>Bmm0..............................?./44::.>Z:>IB, )e$mente" Neutro!hils.................. /?44C;=...............................>./?44>.C; an"s............................044/=.....................>.>044>.>/ (osino!hils.....................:440=............................>.>:44>.>0 aso!hils........................................>.>44D/=........................................>44>.>>D/ ,ym!hocytes......................;/4400=..................>.;/44>.00 +onocytes..................044D=......................>.>044>.>D +ean Cor!uscular 2ema$lobin......................;/.?440?.C!$Bcell..................>.0I44 >./?fmolBcell +ean Cor!uscular 2ema$lobin Concentration............0:44 0C=2bBcell............................?.@:44/./@mmol 2bB, +ean Cor!uscular 1olume.......................@>44:>>+m0.................@>44:>>fl &artial 7hrombo!lastin time'Activate")......................;/44?>secon"s..................;/44 ?>secon"s &latelet Count..................:/>,>>>44?>>,>>>Bmm0......................:/>44?>>Z:>IB, &rothrombin 7ime................::44:/ secon"s........................::44:/secon"s 6eticulocyent count.................>./44:./= of re" cells .......................>.>>/44>.>:/ 7hrombin treet.........................P; secon"s %eviation from Control...............P ;secon"s %eviation from Control 1olume &lasma..........................+ale :;/44?0m,B5$..............>.>;/44>.>?0,B5$ .-emale :;@44?/m,B5$..............>.>;@44>.>?/,B5$ 6e" Cell.........................+ale :;>440Cm,B5$..............>.>;>44>.>0C,B5$. .-emale ::I440:m,B5$..............>.>:I44>.>0:,B5$

)weat . Chlori"e...............>440/mmolB,.........................>440/mmolB, F6IN( Calcium...................:>>440>>m$B;? h..........;./44D./mmolB;? h Chlori"e...................varies with intake.......................varies with intake Creatinine clearance..................+ale:ID44:0Dm,Bmin. -emale:@@44:;@m,Bmin (striol,7otal'in &re$ancy) 0>wks...................C44:@m$B;? h....................;:44C;+molB;? h 0/wks..................I44;@m$B;? h....................0:44ID+molB;? h ?>wks..................:044?;m$B;? h..................?/44:?C+molB;? h :D42y"ro*ycorticsteroi"s......................+ale:0.>44:>.>m$B;? h....................@.;44 ;D.C+molB;? h -emale:;.>44@.>m$B;? h...................././44;;.>+molB;? h :D45etosteroi"s,7otal...........................+ale:0.>44:>.>m$B;? h....................;@44 D>+molB;? h -emale:;.>44@.>m$B;? h....................;:44/;+molB;? h Osmolality................./>44:?>>mOsmolB5$ O*alate........................@44?>+$Bml..................I>44??/+molB, &otassium.....................1aries with %iet.....................1aries with %iet &roteins,7otal................P :/> m$B;? h..........................P>.:/$B ;? h )o"ium..........................1aries with %iet................1aries with %iet Fric Aci"............1aries with %iet..............................1aries with %iet ,aboratory 1alues Z inclu"es in the iochemical &rofile ')+A4:;) ,OO%, &,A)+A, )(6F+ 6(-(6(NC( 6AN.( )I 6(-(6(NC( IN7(61A,) Z Alanine aminotransfer'A,7,.&7 at 0>c)............@44;>FB,.............................@44;>FB, Amylase,)erum.........................................................;/44:;/FB,...........................;/44 :;/FB, ZAs!artate aminotransferase'A)7,.O7 at 0>c)............@44;>FB, ............................@44 ;>FB, ilirubin,)erum'a"ult) totalBB%irect...................>.:44:.>m$B",BB>.>44 >.0m$B",.........;.:D+molB,BB>./+molB, Calcium,)erum'ca;M) ........................@.?44:>.; m$B",......................;.:44;.@mmmolB, Cholesterol,serum..............................................6ec:P;>>m$B",.......................P /.;mmolB, Cortisol,)erum.........................................>@>>h:/44;0+$B",....................:0@44C0/nmolB, :C>>h:04:/+$B",......................@;44?:0nmolB, Creatine 5inase,)erum.............................+ale: ;/44I>FB,...........................;/44I>FB,

-emale::>44D>FB,............................D>FB, ZCreatinine )erum............................>.C44:.;m$B",...................../044:>C+molB, (lectrolytes,)erum )o"ium'NaM)..................................:0C44:?/m(qB,......................:0C44:?/mmolB, Chlori"e'cl4)....................I/44:>/m(qB,.......................I/44:>/mmolB, Z&otassium'kM)..................0./44/.>m(qB,.......................0./44/.>mmolB, icarbonate'2CO0)................;;44;@m(qB,.........................;;44;@m(qB, +a$nesium'm$;M)..................:./44;.>m(qB,.......................:./44;.>mmolB, (striol,7otal,)erum'in &rre$anay) ;?44;@wksBB0;440Cwks.....................0>44:D>n$Bm,BBC>44;@>n$Bm,...........:>?4/I>BB;@>4 ID>nmolB, ;@440;wksBB0C44?>wks.....................?>44;;>n$Bm,BB@>440/>n$Bm,............:?>4DC>BB;@>4 :;:>nmolB, -erritin,)erum............................+ale::/44:/>n$Bm,....................:/44;>>+$B, -emale::;44:/>n$Bm,.......................:;44 :/>+$B, -ollicle4stimulatin$ hermone,)erumB&lasma..+ale:?44;/mIBFBm,.....................?44;/FB, -emale:&remeno!ause ?440>mIFBm,....?44 0>FB, +i"Cycle &eak :>44I>mIFBm,....:>44I>FB, &remeno!ause ?>44;/>mIFBm,....?>44;/>FB, .ases,Arterial loo"'room air) !2................................D.0/44D.?/..........................W2MX0C44??nmolB, &CO; ..................004?/mm2$............................?.?44/.I5&a &O; ....................D/4:>/mm2$.........................:>.>44:?.>5&a Z.lucose,)erum.......................-astin$:D>44::>m$B",.......................0.@44C.:mmolB, ;4h &ost!ran"ial: P:;>m$B",.....................PC.CmmolB, .rowth 2armone44ar$inine )timulation.........-astin$:P/n$Bm,.................P/+$B, &rovocative )timuli:ODn$Bm,....O+$B, Immuno$lobulins,)erum I$A.................................DC440I>m$B",..................>.DC440.I>$B, I$(.................................>40@>IFBm,....................>44 0@>5IFB, I$...................................C/>44:/>>m$B",...............C./44:/$B, I$+..................................?>440?/m$B",..................>.?440.?/$B, Iron.................................../>44:D>+$B",................I440>+molB, ,actate "ehy"ro$enase,)erum............?/44I>FB,...................?/44I>FB, ,utein$in$ harmone,)erumB&lasma...........+ale:C44;0mIFBm,................C44;0FB, -emale:-ollicular &hase /440>mlFBm,............../440>FB, +i" Cycle: D/44:/>miFBm,.........................D/44:/>FB, &ostmeno!ause :0>44;>>mIFBm,.....................0>44;>>FB, Osmolality )erum..................;D/44;I/mOsmolB5$...............;D/44;I/mOsmolB5$ &arathyroi" hormone,)erum,N4terminal............;0>44C0>!$Bm,.............;0>4C0>n$B,

Z&hos!hate'alkaline),)erum'!4N&& at 0>_c).....;>44D>FB,...............;>44D>FB, Z&hos!lorous'inOr$anic),)erum......................0.>44?./m$B",.................:.>44:./mmolB, &rolactin,)erum'h&6,)..............................P;>n$Bm,...................P;>+$B, Z&roteins,)erum 7otal'recumbent).................................C.>44D.@$B",...................C>44D@$B, Albumin...........................................0./44/./$B",...................0/44//$B, .lobulin................ ;.044/./$B",....................;0440/$B, 7hyroi"4)timulatin$ harmone,)erum or&lasma..................>./44/.>+FBm,.........>./44 /.>mFB, 7hyroi"al4io"ine' :;0BF!"ate...................@440>= of A"ministere" "oseB ;? h...................>.>@44>.0> B ;? B h 7hyro*ine'7?),)erum........................../44:;+$B",....................C?44://nmolB, 7ri$lyceri"es,)erum......................................0/44:C>m$B",......................C?44://nmolB, 7riio"othyronine'70),)erum'6IA)................::/44:I>n$B",.............:.@44;.InmolB, 7riio"othyronine'70 ) resin F!take............;/440/=......................>.;/44>.0/ ZFric nitro$en,)erum' FN)................................D44:@m$B",.....................:.;440.ommol FreaB, ZFric aci",)erum.........................................0.>44@.;m$B",.......................>.:@44 >.?@mmolB, C(6( 6O)&INA, -,FI% Cellcount........................................>./cellsBmm0.................>./Z:>C, Chlori"e...........................................::@44:0;m(qB,............................::@44:0;mmolB, .amma .lobulin.....................................044:;= total &r!teins.....................>.>044>.:; .lucose........................................... ?>44D>m$B",...............................;.;440.ImmolB, &ressure..........................................D>44:@>mm h;O...................D>44:@>mm 2 ;O &roteins,7otal....................................P ?>m$B",..................................P>.?>$B, 2ematolo$ic lee"in$ 7ime'7em!late).......................;.D+inutes.......................;.D+inutes (rythrocyte Count..........................+ale: ?.044/.ImillionBmm0...................?.044 /.IZ:>:;B, -emale: 0./44/./millionBmm0...................0./44 /./Z:>:;B, (rythrocyte )e"imentation rate'Wester$ern)...............+ale:>44 :/mmBh..................................>44:/mmBh .-emale:>44;>mmBh..................................>44 ;>mmBh 2ematocrit ....................................+ale:?:44/0=..............................>.?:44>./0 -emale:0C44?C=.........................>.0C44>.?C. 2emo$loblin A:c............................PUC=...............>.>C= 2emo$lobin, loo".......................... +ale ::0./44:D./$B",. .......................;.>I44 ;.D:mmolB, -emale ::;.>44:C.>$B",. .......................:.@C44;.?@:mmolB, 2emo$loblin,&lasma.....................:44?m$B",............................>.:C44>.C;mmolB, ,eokocyte Count an" %ifferential

,eokocyte Count.....................?/>>44::,>>>Bmm0..............................?./44::.>Z:>IB, )e$mente" Neutro!hils.................. /?44C;=...............................>./?44>.C; an"s............................044/=.....................>.>044>.>/ (osino!hils.....................:440=............................>.>:44>.>0 aso!hils........................................>.>44D/=........................................>44>.>>D/ ,ym!hocytes......................;/4400=..................>.;/44>.00 +onocytes..................044D=......................>.>044>.>D +ean Cor!uscular 2ema$lobin......................;/.?440?.C!$Bcell..................>.0I44 >./?fmolBcell +ean Cor!uscular 2ema$lobin Concentration............0:44 0C=2bBcell............................?.@:44/./@mmol 2bB, +ean Cor!uscular 1olume.......................@>44:>>+m0.................@>44:>>fl &artial 7hrombo!lastin time'Activate")......................;/44?>secon"s..................;/44 ?>secon"s &latelet Count..................:/>,>>>44?>>,>>>Bmm0......................:/>44?>>Z:>IB, &rothrombin 7ime................::44:/ secon"s........................::44:/secon"s 6eticulocyent count.................>./44:./= of re" cells .......................>.>>/44>.>:/ 7hrombin treet.........................P; secon"s %eviation from Control...............P ;secon"s %eviation from Control 1olume &lasma..........................+ale :;/44?0m,B5$..............>.>;/44>.>?0,B5$ .-emale :;@44?/m,B5$..............>.>;@44 >.>?/,B5$ 6e" Cell.............................+ale :;>440Cm,B5$................>.>;>44>.>0C,B5$. .-emale ::I440:m,B5$..............>.>:I44 >.>0:,B5$ )weat . Chlori"e..............................>440/mmolB,......................................>440/mmolB, F6IN( Calcium................................:>>440>>m$B;? h...............................;./44D./mmolB;? h Chlori"e.................................varies with intake.......................varies with intake Creatinine clearance...................+ale:ID44:0Dm,Bmin. -emale:@@44:;@m,Bmin (striol,7otal'in &re$ancy) 0>wks......................................C44:@m$B;? h....................;:44C;+molB;? h 0/wks.......................................I44;@m$B;? h....................0:44ID+molB;? h ?>wks.........................................:044?;m$B;? h..................?/44:?C+molB;? h :D42y"ro*ycorticsteroi"s......................+ale:0.>44:>.>m$B;? h....................@.;44 ;D.C+molB;? h -emale:;.>44@.>m$B;? h...................././44 ;;.>+molB;? h :D45etosteroi"s,7otal...........................+ale:0.>44:>.>m$B;? h....................;@44 D>+molB;? h

-emale:;.>44@.>m$B;? h....................;:44 /;+molB;? h Osmolality.............................../>44:?>>mOsmolB5$ O*alate...................................@44?>+$Bml..................I>44??/+molB, &otassium...............................1aries with %iet.....................1aries with %iet &roteins,7otal...........................P :/> m$B;? h..........................P>.:/$B ;? h )o"ium..................................1aries with %iet................1aries with %iet Fric Aci"................................1aries with %iet..............................1aries with %iet

S-ar putea să vă placă și